Sei sulla pagina 1di 1610

SPMM 2015

Index
Title Page Title Page
Index 1 20 Pharmacokinetics 777
1 Basic_Psychology 2 21 Pharmadynamics 810
2 Social_Psychology 55 22 Adverse_effects 834
3 Sociocultural_Psychiatry 79 23 MOCK Paper A(2) Test 1 881
4 Human_Development 117 24 MOCK Paper A(2) Test 2 921
5 EMIs Development 182 25 MOCK Paper A(2) Test 3 960
6 Classification 211 26 MOCK Paper A(2) Test 4 1001
7 Clinical_Examination 288 27 MOCK Paper A(2) Test 5 1041
8 Rating_Scales 341 28 MOCK Paper A(2) Test 6 1081
9 Descriptive_Psychopathology 362 A1 TEST 1 1122
10 Dynamic_Psychopathology 451 A1 TEST 2 1162
11 Neuroanatomy 469 A1 TEST 3 1203
12 Neurochemistry 500 A1 TEST 4 1243
13 Neuropathology 519 A1 TEST 5 1284
14 Neurophysiology 535 A1 TEST 6 1324
15 Applied_Neurosciences 559 exa1 1365
16 EMIs Neuroscience Part 1 583 exa4 1404
17 EMIs Neuroscience Part 2 647 exa5 1466
18 Molecular_Genetics 694 exa6 1493
19 Basic_Pharmacology 751 mock a1 (2) 1557
001
Digit span test is administered to a patient with suspected cognitive impairment. Normal
reverse digit span in a working age adult is

Select one:
10 ± 2 digits
7 ± 1 digits
7 ± 2 digits

5 ± 2 digits

4 ± 3 digits

Normal forward span is 7 ± 2 digits; while the normal reverse span is 5 ± 2 digits.
The correct answer is: 5 ± 2 digits

002
Skinner's operant conditioning is based on which of the following principles?

Select one:
An association is learnt without overt behavioural expression
A response gets paired with a consequence
A response gets paired with its probability
A response gets paired with an inernal stimulus

A stimulus gets paired with a response

2
Skinner believed that the best way to understand behaviour is to look at the causes of an
action and its consequences. He called this approach operant conditioning.
The correct answer is: A response gets paired with a consequence

003
Situation specific amnesia may arise in all of the following situations except

Select one:
Post traumatic stress disorder
Adjustment disorder
Being the victim of an offence
Being the victim of childhood sexual abuse
Committing an offence

The correct answer is: Adjustment disorder

004
All of the following are deficiency needs except

Select one:
Aesthetic needs
Safety needs

Physiological needs
Love and belonging needs
Esteem needs

3
The correct answer is: Aesthetic needs

005
Wechsler Adult intelligence scales (WAIS-R) are for people

Select one:
Aged 15 and over
7 to 16 year olds
Aged 18 and over
3 to 7 year olds
Aged 16 and over

Wechsler Adult intelligence scales-R is for those aged 16 and over. Wechsler Adult
intelligence scales for children is for those aged 7 to 16 years. Wechsler preschool and
primary intelligence is for 3 to 7.5 year old children.
The correct answer is: Aged 16 and over

006
In studies on arousal levels related to different emotions, which emotion produces the
maximum rise in temperature compared to all other emotions;

Select one:
Sadness
Disgust
Anger
Love
Fear

4
Anger- maximum rise in temperature. Fear and Disgust- drop in temperature. Studies on
emotions have shown that the heart rate increase produced by sadness is usually greater
than that produced by happiness.
The correct answer is: Anger

007
In the alcoholic blackouts, the following type of memory loss is seen:

Select one:
Anterograde amnesia due to failure to consolidation
Retrograde amnesia
Dissociative amnesia
Anterograde amnesia due to failure to recall
Both retrograde and anterograde amnesia

A disruption in normal activity of hippocampus under the influence of alcohol can result in
temporary failure of memory consolidation leading to anterograde amnesia for circumscribed
periods. This is called as alcoholic blackout.
The correct answer is: Anterograde amnesia due to failure to consolidation

008
Which of the following is true regarding processes that govern formation of various types of
memory?

Select one:
Attention allows decoding long term memory store
Lack of attention leads to forgetting

Attention allows short-term memory to enter long-term memory store


Attention allows sensory memory to enter short term memory store

Lack of attention leads to delay in recall

5
The correct answer is: Attention allows sensory memory to enter short term memory store

009
Which conditioning is involved in the aetiology of both agoraphobia and of obsessional
rituals?

Select one:
Classical conditioning
Forced conditioning
Backward conditioning
Operant conditioning

Avoidance conditioning

The term aversive conditioning refers to situations in which behavior is motivated by the threat of
an unpleasant stimulus. There are two main categories of behavior under aversive control:
avoidance behavior and escape behavior. Escape conditioning occurs when the animal learns to
perform an operant to terminate an ongoing, aversive stimulus. It is a "get me out of here" or
"shut this off" reaction, aimed at escape from pain. The behavior that produces escape is
negatively reinforced (reinforced by the elimination of the unpleasant stimulus). For example, the
jump of a rat from electrified platform onto a bowl of water is an escape behavior. Escape
conditioning is converted into avoidance conditioning by giving a signal before the aversive
stimulus starts. If the animal receives a cue or signal that an aversive stimulus is coming, then
after one or two occurrences of the punishing stimulus the cue will trigger an avoidance
behavior. This kind of learning occurs quickly and is very durable. For example, if you sounded a
tone before you electrified the platform, after one or two trials the rat would respond to the tone
by jumping into the water. It would not wait for the shock. Avoidance behaviors are very
persistent even when there is no longer anything to avoid. The reason is that as a result of an
avoidance reaction, one never experiences the aversive stimulus. But this serves as a negative
reinforcement, providing a sense of relief. Because of this, avoidance behavior is self-reinforcing.

The correct answer is: Avoidance conditioning

010
A 24 year old woman develops significant fear of driving after having a relatively trivial road
traffic accident. She avoids using cars even as a passenger. She has no symptoms of PTSD.
Which of the following therapies is best suited for her needs?
Select one:

6
Brief focal dynamic psychotherapy
Cognitive analytic therapy
Interpersonal therapy

Behaviour therapy
Psychoanalytic therapy

This is simple phobia for driving and not PTSD. So behaviour therapy is the correct option.
The correct answer is: Behaviour therapy

011
In Pavlov's original classical conditioning experiments on dogs, which of the following served
as the conditioned stimulus?

Select one:
Food
Saliva
Bell
Hunger
Dog

The correct answer is: Bell

012
Tip of the tongue state is a well investigated example of

Select one:
Context dependent memory
Blocking
Encoding failure

7
Failure of prospective memory
State dependent memory

Sometimes people may have the experience that we know the answer we are seeking but
cannot quite find it and this feeling is known as the tip-of-the-tongue phenomenon, also
abbreviated as TOT phenomenon. This is a well investigated example of Blocking, which is one
of the 'retrieval failure' theories of forgetting. Blocking is said to occur when the subjects are
unable to access information that they know exists in their memory despite great efforts at
recalling even in the presence of retrieval cues. TOT states may last for few seconds or for a
few days.
The correct answer is: Blocking

013
People can only attend to one physical channel of information at a time'-Which theory of
attention states this?

Select one:
Broadbent's filter theory of attention
Cocktail party phenomenon
Dichotic listening experiments
Attenuator model of selective attention
Shiffrin and Schneider's divided attention theory

Dichotic listening experiments show that alternative information is simultaneously processed


and can be attended to if required. Here one kind of information is selected for attention and it is
called as selective or focused attention. Broadbent conducted dichotic listening experiments and
he suggested that "our mind can be conceived as a radio receiving many channels at once". He
supposed that in brain there exists a type of audio filter in our brain that selects which channel
we should pay attention to from the many kinds of sounds perceived. Broadbent proposed that
the filter lies between the sensory buffer and short-term store (what is now called working
memory) that prevents overloading memory. This is called Broadbent's filter theory.

The correct answer is: Broadbent's filter theory of attention

014
A clinical psychologist performed digit span test and immediately asked the patient to count a
three digit number backwards. What is this procedure called as?

Select one:

8
Brown Paterson Task
Primacy effect
Recency effect

Baddeley and Hitch Task


Cohen and Squire experiment

Under normal situations, the duration for which information is held in short term memory is
usually 15 to 20 seconds. STM is also very fragile and information is lost quickly. Brown
Paterson demonstrated that by 15 seconds the original material is completely forgotten.
Brown Paterson task involves introducing distraction (such as counting a three digit number
backwards) immediately after the digit span test in order to prevent rehearsal.
The correct answer is: Brown Paterson Task

015
Which of the following is not a component of the Big Five Theory of personality?

Select one:
Capacity
Agreeableness
Openness
Neuroticism
Extraversion

The Big Five are five broad factors (dimensions) of personality traits. They include 1.
Extraversion which encompasses more specific traits as talkative, energetic, and
assertiveness. 2. Agreeableness which includes sympathtic, kind and affectionate approach. 3.
Conscientiousness that includes traits such as being organized, thorough, and planful. 4.
Neuroticism which includes traits like being tense, moody and anxious. 5. Openness to
Experience which includes traits like having wide interests, being imaginative and insightful. (In
short, the Big Five includes five traits, each beginning with the alphabets of the word OCEAN)

The correct answer is: Capacity

016
Biofeedback can mainly modify which of the following functions?

Select one:

9
Gastrointestinal function
Cardiovascular function
Musculoskeletal function

Respiratory function
Central nervous system function

Biofeedback involves the transmission to subjects of information about biological functions. It


can modify cardiovascular function and is a useful method of reducing muscle tension. It is a
technique for controlling physiological responses by receiving information about these responses
as they occur. Monitoring devices track physiological responses such as heart rate, blood
pressure, and muscle tension and provide the person with feedback in the form of light or tone
whenever they change the response in the desired direction. With practice, a person can learn to
control all sorts of bodily functions predominantly through relaxation.
The correct answer is: Cardiovascular function

017
Which of the following is not a part of the Big Five personality traits?

Select one:
Carefulness
Openness
Agreeableness
Neuroticism
Conscientiousness

Big Five traits include OCEAN - Openness, Conscientiousness, Extraversion, Agreeableness


and Neuroticism. This has provided a unified framework for trait research. NEO decreases with
age; AC increases with age.
The correct answer is: Carefulness

018
The principle of combining separate pieces of information into one meaningful combination is
called

Select one:

10
Encoding
Primacy effect
Chunking

Sensory storage
Recency effect

The correct answer is: Chunking

019
In a scientific experiment, a neutral stimulus (a bell) is paired with a stimulus (food) that
produces an autonomic response (salivation). Which of the following learning mechanisms
has been employed?

Select one:
Systematic desensitization
Shaping
chaining
Operant conditioning
Classical conditioning

The correct answer is: Classical conditioning

020
The strategies to improve encoding includes all except

11
Select one:
Using imageries
Mnemonics
Using primacy-Recency effects

Cueing
Chunking

Strategies to improve encoding include - order and sorting info, chunking, mnemonics, using
imageries, adding importance and salience to the info and using primacy-Recency effects.
Retrieval can be helped by cueing and reinstatement of learning context. The more similar the
retrieval situation is to the encoding situation, the better retrieval. This is called encoding
specificity principle.
The correct answer is: Cueing

021
The most important and common bedside test that is used in demonstrating age-associated
memory impairment would be;

Select one:
Delayed recall
Orientation to time and place
Immediate recall
Naming objects
Attention and calculation

The correct answer is: Delayed recall

022
Learned helplessness has been invoked to explain the aetiology of;

Select one:

12
Obsessive compulsive disorders
Psychosomatic disorders
Anxiety disorders

Psychotic disorders
Depressive disorders

The correct answer is: Depressive disorders

023
A 45-year-old man was admitted to the acquired brain injury unit with severe memory
impairment. Which of the common clinical tests can be used to test his immediate memory?

Select one:
Recall of last meal
Recall of distant personal events
Recall of items after 5 minutes
Recall of recent topics in news
Digit span

The correct answer is: Digit span

024
Which is a form of negative reinforcement used in the treatment of alcohol dependence?

Select one:

13
Apomorphine treatment
Aversion therapy using electric shock
Covert sensitization

Disulfiram
Alcoholics anonymous

The correct answer is: Disulfiram

025
All of the following are true with respect to coping process when faced with a life stressor
except

Select one:
In emotion-focused coping the individual alters his emotional self-appraisal to
the stressful situation
During primary appraisal the individual evaluates his resources to cope
In problem-focused coping the individual attempts to change the stressful
situation
Individuals employ a variety of different strategies to cope

There is no universally best strategy for coping

Primary appraisal - the individual evaluates the stressor. Secondary appraisal - the individual
evaluates his resources and options available to manage the stressful situation.
The correct answer is: During primary appraisal the individual evaluates his resources to cope

026
Which of the following theorists proposed six basic emotions?

Select one:
Ekman
Lorenz

14
Cannon and Bard
James and Lange
Ainsworth

According to Paul Ekman, the six basic emotions are sadness, happiness, anger, fear,
disgust, and surprise. These emotions combine in different ways to form other emotions,
including compassion, boredom, embarrassment, rage, hunger, and more.
The correct answer is: Ekman

027
Information is held in the long term memory by the process of

Select one:
Rote rehearsal
Elaborative rehearsal

Maintenance rehearsal
Recency effect
Displacement of older memories

Rehearsal is supposed to be the transient control process that can aid maintenance of STM and
transfer to LTM. Other control processes include encoding, retrieval strategies and decision to
remember. Rehearsal may be maintenance/rote rehearsal or elaborative rehearsal where
encoding is semantically elaborated or changed. Maintenance rehearsal is simple and refers to
the repetition of items in one's mind and occasionally by verbal repetition (e.g., trying to
remember a telephone number). Elaborative rehearsal involves more extensive understanding
of the meaning of the material, a process involving long-term memory. It is proposed that
rehearsal can take place at three levels of processing. Shallow processing where surface
features are only rehearsed, phonemic processing where sound features are rehearsed or
semantic processing where deeper encoding and meaning related associations are made. A
higher level of processing depends on time available and nature of the material processed.

The correct answer is: Elaborative rehearsal

028
Which of the following refers to the formation of initial memory traces?

Select one:
Retention

15
Forgetting
Retrieval
Storage
Encoding

In all operations involving memory, three different processes are thought to occur. Encoding; It
leads to the formation of initial memory traces and receives information from the outside. The
encoding is acoustic in short term memory and semantic in long term memory. Storage:
Retention of information and maintenance. Retrieval; Accessing and recovering information from
memory stores.
The correct answer is: Encoding

029
Amnesic syndrome is characterised by a marked impairment in

Select one:
Procedural memory
Working memory
Episodic memory
Semantic memory
Implicit memory

The term amnesia refers to episodic memory loss.


The correct answer is: Episodic memory

030
Which one of Maslow's hierarchy of needs represents the need to develop a sense of
personal worth and competence and the need for recognition by others?

Select one:
Aesthetic and cognitive needs
Physiological needs
Self actualisation needs
Esteem needs

16
Motivation needs

Esteem needs- the need to develop a sense of personal worth and competence and the need
for recognition by others. Aesthetic and cognitive needs- These are growth needs involving
knowledge, understanding, beauty and symmetry. The need for self-actualisations is the desire
to become more and more what one is, to become everything that one is capable of becoming..
According to Maslow, the following characters are seen in self-actualizing people. They are
Spontaneous in their ideas and actions, Creative. Interested in solving problems, Appreciate life,
Have a system of internalized independent morality and can view all things in an objective
manner.
The correct answer is: Esteem needs

031
A reinforced operant response weakens when the reinforcement is ceased. This is called

Select one:
Stimulus decoupling
Stimulus sensitisation
Extinction
Stimulus discrimination

Stimulus incubation

The correct answer is: Extinction

032
Which one of the following is a way of measuring personality along the dimensions of
extroversion-introversion and neuroticism-stability?

Select one:
Rorschach ink blot test
Minnesota multiphasic personality inventory
Draw a person test

17
Thematic apperception test
Eysenck Personality Inventory

EPI is a way of measuring personality along the dimensions of extroversion-introversion and


neuroticism stability. The inventory also includes a lie scale.
The correct answer is: Eysenck Personality Inventory

033
Mr. X was seen in the psychiatric outpatient clinic. He complains of absent-mindedness and is
failing to take medications and keep appointments. This is due to

Select one:
Failure of conditioning
Failure of encoding
Failure of context dependent memory

Failure of prospective memory


Failure of State dependent memory

Failure of prospective memory is a common cause of absent mindedness. Prospective memory


is the ability to carry out particular actions or perform planned actions and appears to involve
the prefrontal lobes of the brain. This is one of the findings/theories of retrieval failure.
The correct answer is: Failure of prospective memory

034
Which one among the following is not one of the Gestalt principles of perception?

Select one:
Similarity
Continuity
Closure
Familiarity
Proximity

18
The correct answer is: Familiarity

035
According to Ebbinghaus curve, there is a sharp drop in forgetting over

Select one:
First eighteen hours
First seven days
First twenty four hours
First one month
First nine hours

The correct answer is: First nine hours

036
Mr. X mentioned that he has vivid memories of where he was, what he was doing and was
even able to remember other fine details, when he heard the news of suicide bombing of the
twin towers in 2001. These memories have been termed as

Select one:
Selective memory
Flashbulb memories

Autobiographical memory
Procedural memory
Semantic memory

19
Episodic memory is autobiographical, self-focussed spatio-temporal memory. Flashbulb
memory is a special episodic memory where people remember clearly where and what they
were doing at times of specific global events like September 11, 2001, etc.
The correct answer is: Flashbulb memories

037
In a behavioural technique, actual exposure to feared stimulus takes place for a substantial
amount of time, so the accompanying anxiety response fades away. This is called

Select one:
Aversive conditioning
Covert reinforcement
Covert sensitisation
Flooding (in vivo)
Implosion (in vitro)

Implosion is 'in vitro', - imaginary exposure in behavioural therapy.


The correct answer is: Flooding (in vivo)

038
In a Pavlovian conditioning paradigm, the bell is ringed before the food is presented. But the
bell continued till salivation appeared. This is called

Select one:
Backward conditioning
Pseudoconditioning

Forward conditioning
Trace conditioning
Simultaneous conditioning

20
In forward conditioning, the conditioned stimulus is presented before the unconditioned
stimulus, and the combined stimuli persist until the unconditioned response is elicited during
the pairing.
The correct answer is: Forward conditioning

039
The brain region thought to be responsible for majority of working memory functions is

Select one:
Cerebellum
Dorsal thalamus
Frontal lobe
Hippocampus
Temporal lobe

Several event-related functional magnetic resonance imaging (fMRI) studies have found
persistent frontal-lobe activity during retention intervals of delayed response tasks (a WM
task). The parietal lobe is also important for this function.
The correct answer is: Frontal lobe

040
A Lotto player thinks that more he loses, the more chances that he wins later. So he keeps
playing the Lotto. This is best described as

Select one:
Social loafing
Base rate fallacy

Entrapment
Gambler's fallacy
Availability heuristics

Gambler's fallacy: an outcome is due as it has not happened for some time. So a gambler
thinks that more he loses, the more chances that he wins later.
The correct answer is: Gambler's fallacy

21
041
Raven's progressive matrices test is used to assess

Select one:
Premorbid intelligence
Frontal lobe functions
General intellectual ability

Visuospatial ability
Attentional bias

Raven's Progressive Matrices (often referred to simply as Raven's Matrices) are multiple- choice
intelligence tests of abstract reasoning that provide culturally unbiased IQ estimates. Rey
Osterrieth test comprises of copying a complex figure initially and then reproducing it from
immediate and delayed recall that tests both visual memory and constructional ability
The correct answer is: General intellectual ability

042
Which of the following can test abstract reasoning?

Select one:
Digit symbol substitution test
Clock drawing test
Trail making test
n-back test
Goldstein's card sorting test

The Goldstein-Scheerer Color Form Sorting Test was initially designed to test abstract
reasoning (concept formation rather than the simple identification of abstract rules). Wisconsin
card sorting test is a descendant of Goldstein task nd it tests concept identification rather than
formation
The correct answer is: Goldstein's card sorting test

043
Which of the following behavioural treatment is useful in trichotillomania?

Select one:

22
Massed negative practice
Exposure and response prevention
Habit reversal

Biofeedback
Imaginal flooding

Habit reversal training (HRT) is a behavioral treatment package useful in tics and other OCD
spectrum disorders such as trichotillomania, nail biting, thumb sucking, and skin picking, etc.
The correct answer is: Habit reversal

044
A medical student becomes more comfortable in the anatomy laboratory after several weeks
of exposure to it. Which of the following explains this process?

Select one:
Positive reinforcement
Habituation
Negative reinforcement
Modeling
Classical conditioning

The decline in responsiveness or reaction to the repeated presentation of a stimulus is known


as habituation.
The correct answer is: Habituation

045
Which of the following psychological theories is not concerned with the faculties of memory
and forgetting?

Select one:
Trace decay theory
Hierarchy theory
Interference theory

Repression

23
State dependent learning

Maslow's hierarchy of needs theory is a theory of motivation/drives and not a theory of


memory.
The correct answer is: Hierarchy theory

046
Which of the following is a feature of Type A personality?

Select one:
High competitiveness
Being relaxed
Being patient
Evading stress

Easy-going without time pressure

The correct answer is: High competitiveness

047
Mr. Z was admitted to a neuropsychiatric unit with a diagnosis of an amnesic syndrome. He
was referred to psychology for psychometric testing. Which of the following would be
unimpaired in formal testing?

Select one:
Episodic memory of recent past

Anterograde memory
New learning
Immediate memory
Delayed recall

24
Various disorders can give rise to amnesic syndromes (E.g. hypoxia, herpes encephalitis) and
the features would include 1. Unimpaired immediate memory. 2. The presence of anterograde
amnesia- inability to acquire new information (impaired delayed recall) 3. The presence of
retrograde amnesia of variable extent and severity 4. Unimpaired global intellectual abilities 5.
Unimpaired implicit memory
The correct answer is: Immediate memory

048
When carried out in imagination, Immediate exposure to elements at the top of the hierarchy
without any gradation of anxiety-inducing stimuli is termed as;

Select one:
Implosion
Covert sensitisation
Avoidance
Flooding
Discriminative stimulus

Immediate exposure to elements at the top of the hierarchy without any gradation of anxiety-
inducing stimuli is called as 'flooding' when carried out in vivo and called as 'implosion therapy'
when done through imagination.
The correct answer is: Implosion

049
Which one among the following statements is true about classical conditioning?

Select one:
Classical conditioning occurs when a new stimulus is repeatedly paired with a
conditioned stimulus.
Classical conditioning was first demonstrated by skinner

Classical conditioning takes place irrespective of the nature of the


unconditioned stimulus
Classical conditioning is a slow process depending on repeated
presentations of the stimulus
In classical conditioning, the responses are often autonomic

25
The correct answer is: In classical conditioning, the responses are often autonomic

050
Which one among the following statements regarding needs and motivation is incorrect?

Select one:
according to Yerkes-Dodson curve, there is an inverted U-shaped
relationship between arousal (motivation) and performance level
According to Canon's homeostatic theory, changes in homeostatic system
triggers processes aimed at restoration of system
According to Maslow's hierarchy of needs, Needs at the previous level need to
be satisfied before one can progress to the next level
According to Drive reduction theory, motivation of behaviour is to decrease
the arousal associated with basic drives
In Maslow's hierarchy of needs, cognitive needs precede esteem needs

The correct answer is: In Maslow's hierarchy of needs, cognitive needs precede esteem
needs

051

26
Which of the following best describes Ribot's law in memory disturbances?

Select one:
In organic amnesia remote memory content is lost before recent content.
In organic amnesia recent memory content is lost before remote content.
In dissociative amnesia anterograde amnesia is dense.

In organic amnesia anterograde amnesia cannot occur without retrograde


loss.
In dissociative amnesia recent memory content is lost before remote
content.

Ribot's Law refers to progressive amnesia as a temporal gradient going from the most recent
to the oldest memories
The correct answer is: In organic amnesia recent memory content is lost before remote
content.

052
Flynn effect refers to which of the following phenomenon?

Select one:
Increasing IQ with successive generations
Changes of IQ with cultural variations
If an IQ test is repeated on the same individual the score increases
Changes of IQ with language
If IQ of parents is high, then the IQ of the child is high.

Flynn effect refers to the generational increase in IQ scores. Flynn suggested that IQ tests do
not measure intelligence but rather correlate with a weak causal link to intelligence (Flynn,
1987). Based on the presence of the effect on nonverbal tests such as the Raven's Matrices,
Flynn believed that the generational IQ increase is due an improved problem-solving capacity
rather than actual gain in intelligence.
The correct answer is: Increasing IQ with successive generations

053
Increase in the strength of the response following brief but repeated exposure to the stimulus is
called

27
Select one:
Stimulus generalisation
Habituation
Incubation

Extinction
Stimulus preparedness

The correct answer is: Incubation

054
Which of the following is an inaccurate statement concerning sensory memory?

Select one:
Sensory memory is modality specific
Each sense has its own sensory memory e.g. iconic (visual), echoic
(auditory) etc.
Information is processed in sensory memory.
It has a large capacity but gets disrupted by inflow of new information in
same modality
Information that is attended to is transferred to short-term memory store
and information that is not selected for processing is rapidly lost.

Sensory memory: This is modality specific, has a large capacity but gets disrupted by the
inflow of new information in the same modality. Each sense has its own sensory memory e.g.
iconic (visual), echoic (auditory), etc. Iconic mode lasts for 0.5 seconds while echoic mode

28
lasts for 2 seconds. No processing is involved in sensory memory. Information that is attended
to is transferred to short-term memory store, and information that is not selected for processing
is rapidly lost.
The correct answer is: Information is processed in sensory memory.

055
Which one among the following statements is true?

Select one:
Positive reinforcement reduces the likelihood of a behaviour occurring
Negative reinforcement reduces the likelihood of a behaviour occurring
Intermittent reinforcement schedules result in acquired behaviours that are
more resistant to extinction than behaviours induced by continuous
reinforcement
Partial reinforcement leads to rapid acquisition and slower extinction of a
response
In negative reinforcement, presentation of an aversive stimulus weakens
the response

Partial reinforcement leads to a slower acquisition and slower extinction of a response. In


negative reinforcement, removal of an aversive stimulus strengthens the response while in
punishment presentation of an aversive stimulus weakens the response. Food, water-primary
reinforcers, which are thought to act through the reduction of basic drives/needs. Money,
tokens, fame-secondary reinforcers and they derive their reinforcing qualities through
association with primary reinforcers.
The correct answer is: Intermittent reinforcement schedules result in acquired behaviours that
are more resistant to extinction than behaviours induced by continuous reinforcement

056
Cognitive processing during social learning includes all except

Select one:
Motivation to act
Judgement

Semantic encoding of observed behaviour


Attention
Retention and rehearsal

29
Cognitive processing during social learning involves various domains: 1. Attention to observed
behaviour is the basic element in learning. 2. Visual image and semantic encoding of observed
behaviour memory 3. Memory permanence via retention and rehearsal 4. Motor copying of the
behaviour and imitative reproduction 5. Motivation to act.
The correct answer is: Judgement

057
Which of the following would demonstrate declarative memories?

Select one:
Knowledge of how to drive a car
Knowledge of how to ride a bike
Knowledge of capital of different countries
Knowledge of how to swim
Knowledge of how to write a letter

The correct answer is: Knowledge of capital of different countries

058
A 55-year-old heavy alcoholic was admitted to the medical ward after sustaining a major fall.
He has severe Anterograde amnesia and extensive retrograde memory loss. However his
working memory and procedural memory are unimpaired. What is his most likely diagnosis?

Select one:
Psychogenic amnesia
Transient organic amnesia

Korsakoff's syndrome
Wernicke's encephalopathy
Early stages of Alcoholic dementia

30
Korsakoff's syndrome: It is a form of an amnesic syndrome often occurring in patients with a
history of severe alcohol abuse leading to thiamine deficiency. The patient may have severe
Anterograde amnesia and extensive retrograde memory loss. This retrograde memory loss
includes autobiographical memory loss with relative sparing of the most distant memories.
Working memory and procedural memory are unimpaired. The pathological features are
mostly found in paraventricular and periaqueductal grey matter, thalamus and mamillary
bodies.
The correct answer is: Korsakoff's syndrome

059
Which of the following is correct concerning sensory memory?. Sensory memory has

Select one:
Small capacity but memories are unprocessed and of long duration (>0.5s)
Small capacity and memories are unprocessed and of very short duration
(<0.5s)
Large capacity but memories are unprocessed and of very short duration
(<0.5s)
Large capacity but memories are unprocessed and of long duration (>0.5s)
Large capacity but memories are processed and of very short duration
(<0.5s)

The correct answer is: Large capacity but memories are unprocessed and of very short
duration (<0.5s)

060
Strongest learning occurs when the delay is;

Select one:
More than 0.5 seconds
More than 5 seconds

Less than 5 seconds


Less than 0.5 seconds

31
More than 0.05 seconds

Longer the interval, poorer the learning.


The correct answer is: Less than 0.5 seconds

061
Which one among the following is a secondary emotion?

Select one:
Fear
Joy
Love
Sadness
Anger

Primary emotions-anger, disgust, joy, anticipation, fear, acceptance, surprise and sadness
(classified by Plutchik) . Secondary emotions include love, contempt, submission and
disappointment.
The correct answer is: Love

062
Which one among the following statements is false about modelling?

Select one:
Modelling can Lead to aggression

Modelling can also lead to social facilitation and can result in skilled
behaviour
Modelling can increase the pain threshold and anxiety threshold.
Observational learning is more likely to occur if the model has some
characteristics in common with the observer
Modelling refers to the learning that takes place spontaneously by
observing the behaviour of another person

32
Modelling can also lead to social facilitation, can result in skilled behaviour, can lower the pain
threshold and anxiety threshold. Observational learning is more likely to occur if the model has
some characteristics in common with the observer. Other Effective models for observational
learning include competency of the model, perceived high status and possess social influence
or power.
The correct answer is: Modelling can increase the pain threshold and anxiety threshold.

063
Which of the following test can be used to determine pre-morbid intelligence of a 70-year-old
coal miner who shows signs of cognitive impairment?

Select one:
National Adult Reading Test
Rey-Osterrieth Complex figure test
Minnesota Multiphasic inventory
Wechsler's Adult Intelligence Test - block design subtest

Trail making test B

To determine premorbid IQ NART - National Adult Reading Test can be used. Other tests
include Weschler's test for Adult reading, Barona demographic equation method and matrix
reasoning subtest of WAIS-R.
The correct answer is: National Adult Reading Test

064
The socio-psychological approach is adopted by which of the following theories of
motivation?

Select one:
Maslow's hierarchy of needs
Drive reduction theory

Arousal reduction theory


Needs theory
Homeostatic theory

33
The Homeostatic drive theory (Cannon), Drive reduction theory (Hull) and Arousal reduction
theory (Hebb) are all based on biopsychological perspective, which examines the physiological
motives such as hunger, thirst etc in humans and animals. They are also called as extrinsic
motivation theories. The Needs theory and Goal theory, also called intrinsic motivation theories,
are based on sociopsychological approach which addresses the cognitive motives behind
people's complex behaviour in the context of their social situations such as work, family and
society. Maslow's Hierarchy of needs is based on the humanistic approach that tries to explain
why people strive to achieve higher and higher goals in an attempt to attain self-fulfillment.

The correct answer is: Needs theory

065
Compulsions provide short-term relief of obsessional anxiety. The learning principle operating
here is

Select one:
Positive reinforcement
Negative reinforcement
Reciprocal inhibition
Punishment
Reward orientation

Compulsions provide short-term relief of obsessional anxiety via negative reinforcement. When
practicing compulsive rituals, anxiety is acutely reduced. This provides a reinforcement to
practice the compulsions repeatedly. In other words the termination of the aversive anxiety cued
by obsessions, increases or stamps in the compulsive behaviour that removed the anxiety,
without addressing the core of obsessions.
The correct answer is: Negative reinforcement

066
Which of the following describes Eysenck's model of personality traits?

Select one:
Neuroticism, depression, psychoticism
Neuroticism, apathy, psychoticism

Neuroticism, extroversion, introversion


Neuroticism, extraversion, psychoticism
Neuroticism, introversion, psychoticism

34
Eysenck's theory is also called the three-factor theory or the P-E-N theory. The three higher-
order factors are extraversion­introversion; neuroticism­stability and psychoticism­impulse
control. His work asserts that the P-E-N dimensions are biologically based and largely
heritable.
The correct answer is: Neuroticism, extraversion, psychoticism

067
Which one among the following cannot be consciously inspected?

Select one:
Autobiographical memory
Semantic memory
Episodic memory
Declarative memory
Non declarative memory

Non declarative memory; refers to a collection of abilities and skills that has been acquired
The correct answer is: Non declarative memory

068
A rat receives positive reinforcement (a food pellet) for pressing a bar that turns on a red light,
but not an adjacent bar that turns on a green light. The process that best explains this is

Select one:
chaining
Operant conditioning

Containing
Systematic desensitization
Shaping

35
The correct answer is: Operant conditioning

069
A candidate who is attempting his final exam before graduation thinks that he is going to fail
the whole exam as he has got one 'obvious question' wrong. He is exhibiting

Select one:
Overgeneralisation
Rationalisation
Projection
Denial
Splitting

The correct answer is: Overgeneralisation

070
Which one among the following statements regarding perception is incorrect?

Select one:
The perceptual set is determined by factors both in the perceiver and in the
stimulus.
The perceptual set is influenced by Emotions

Perceptual set' is the perceptual bias that predisposes the perceiver to


notice only certain aspects of a stimulus and to ignore the other aspects.
The perceptual set is influenced by past experiences
Perceptual set predisposes the perceiver to notice all aspects of a stimulus

36
simultaneously

The perceptual set is the perceptual bias that predisposes the perceiver to notice only certain
aspects of a stimulus and to ignore the other aspects. The perceptual set is determined by
factors both in the perceiver and in the stimulus. The factors include the threshold for
perception eg cocktail party phenomenon, personality, past experience, emotions (anxious
people biased towards perception of threat-related stimuli), individual values (children
exaggerate the size of coins), current drive state and environment (Eskimos recognise 100
different kinds of snow).
The correct answer is: Perceptual set predisposes the perceiver to notice all aspects of a
stimulus simultaneously

071
Mr. Y is admitted to a medical ward with a diagnosis of delirium. The cognitive deficit that is
characteristic of delirium is

Select one:
Long term memory impairment
Disorientation for time
Poor attention span
Visuospatial impairment
Short term memory impairment

The correct answer is: Poor attention span

072
A child was told to finish homework tasks before he can play outside. This is based on

Select one:
Cognitive mapping
Reciprocal inhibition
Classical conditioning
Premack's principle

37
Operant conditioning

The correct answer is: Premack's principle

073
Mr. Z visited his doctor and following his visit he remembered the first few things they had
been told better than subsequent information. What is the name of this tendency?

Select one:
Referred effect
Primary effect
Recency effect
Primacy effect

Preliminary effect

The correct answer is: Primacy effect

074
Mr. Y has amnesia following traumatic brain injury involving the hippocampus. Which of the
following would be spared?

Select one:
None of the above

Episodic memory
Procedural memory

38
Semantic memory
Autobiographical memory

Non declarative memory cannot be considered as a system and is a collection of memory for a
variety of tasks. One key finding has been that in patients with damage to the hippocampus
and associated structures these skills are spared.
The correct answer is: Procedural memory

075
The traits included in 'big five' theory include all except

Select one:
Conscientiousness
Neuroticism
Psychoticism

Openness
Extraversion

OCEAN stands for Openness, Conscientiousness, Extraversion, Agreeableness, Neuroticism.


The correct answer is: Psychoticism

076
A parrot was given a new cage. When the parrot touched a lever in the cage, it received an
electric shock. It did not touch the lever again. The learning process that has occurred here is;

Select one:
Observational learning
Shaping
Negative reinforcement
Chaining
Punishment

39
In behaviourism, punishment refers to the consequence for undesired behavior. Punishment
can be either negative or positive, depending on the nature of the consequence.
The correct answer is: Punishment

077
Which of the following test is a measure of nonverbal intelligence?

Select one:
Sternberg triarchic abilities test
General aptitude test battery
Standford binet test

Raven's progressive matrices


Weschler's adult intelligence scale

Raven's Progressive Matrices are used in the assessment of general cognitive abilities in
children and adult. The Progressive Matrices usefully provide an assessment of non-verbal
ability, an important feature in ethnically diverse populations
The correct answer is: Raven's progressive matrices

078
A family has got a new dog. A 3-year-old child in the family is afraid of dogs and would stay in
the same room with the dog only if he could sit on his mother's lap and had his favourite ice
cream. Select the learning theory principle employed here

Select one:
Reciprocal inhibition
Avoidance learning
Classical conditioning
Operant conditioning
Extinction

This scenario is an example of reciprocal inhibition (Wolpe). This principle states that two
opposing emotions cannot stay together for long time - one will reciprocally inhibit the other.
Comfort with mother and ice-cream conflicts with discomfort produced by the dog - eventually
the dog stops eliciting discomfort. This principle underlies most psychological therapies for
anxiety disorders.

40
The correct answer is: Reciprocal inhibition

079
A person cannot have two opposing physiological states at the same time. For example, it is not
possible to be relaxed and anxious simultaneously. In learning theory, this notion is called

Select one:
Stimulus preparedness
Learned helplessness
Reciprocal inhibition
Cognitive dissonance
Participant modelling

When anxiety and a relaxed state are co-existent, then anxiety reduces - this is called as
reciprocal inhibition.
The correct answer is: Reciprocal inhibition

080
A group of subjects studying molecular biology were asked to solve Multiple choice questions.
What mode of retrieval is this?

Select one:
Recall
Reintegration

Reconstruction
Re-learning
Recognition

The correct answer is: Recognition

41
081
George has a history of anxiety disorder. On visiting a church, he developed a sense of
familiarity because his stored memories were brought into his consciousness. This
phenomenon is called

Select one:
Jamais vu
Recognition
Retrospective falsification

DeJa vu
False memory syndrome

The correct answer is: Recognition

082
Timeout refers to a technique that is aimed at

Select one:
Generalising a learnt behaviour
Reducing desirable behaviour
Increasing unwanted behaviour
Learning new behaviour
Reducing unwanted behaviour

Time out or loss of privileges are used as negative reinforcement strategies to reduce
unwanted behaviours in children
The correct answer is: Reducing unwanted behaviour

083
Which one among the following refers to the act of bringing past experiences as they
happened into conscious awareness?

42
Select one:
Recall
Relearning
Recognition

Reintegration
Recollection

The process of recalling an entire memory from a partial cue, like remembering a speech
upon hearing the first few words, is called reintegration.
The correct answer is: Reintegration

084
The concept of motivated forgetting is based on the psychoanalytic concept of

Select one:
Denial
Regression
All of the above
Repression
Displacement

The theory of motivated forgetting suggests that we forget things because unconsciously we
want to.
The correct answer is: Repression

085
The commonest cause of long-term forgetting is

Select one:
Failure in encoding at the time of the input
Retrieval failure
Failure of transfer of information from short term to long term memory

Failure of transfer of information from long term to short term memory

43
Failure in encoding at the time of retrieval

The cause of long­term forgetting may be due to; Failure in encoding at the time of the input,
failure of transfer of information from short-term to long term memory and retrieval failure
(more commonly).
The correct answer is: Retrieval failure

086
Mr. X was admitted to an intensive medical care unit following a severe head injury. On formal
cognitive testing, he was unable to recall salient experiences in the days and weeks before the
injury. What type of amnesia is this?

Select one:
Hysterical amnesia
Post traumatic amnesia
Retrograde amnesia
Transient global amnesia

Anterograde amnesia

The loss of episodic memories that were stored before brain damage had occurred is known as
retrograde amnesia. Many patients may have lost the ability to form or retain new episodic
memories, a condition called anterograde amnesia. Transient global amnesia is caused by
transient cerebral ischemia causing a temporary lack of blood supply to the regions of the brain
concerned with memory functions. The main features include sudden onset of severe
anterograde amnesia with a retrograde amnesia for the preceding days or weeks. Sometimes
amnesic episodes may occur in patients who have had no brain injury but who have suffered a
traumatic or emotionally disturbing life event, and it is called hysterical amnesia.
The correct answer is: Retrograde amnesia

087
The modification of memories in terms of one's general attitude is called

Select one:
False memory syndrome

Semantic memory
Repressed memory
Flashbulb memory

44
Retrospective falsification

Unconscious distortion of memory, as dictated by one's present psychological needs, is called


as Retrospective Falsification. People who retrospectively falsify a story also tend to embellish
the positive and delete the negative aspects of the situation. Example: During marital conflicts
argument ensues over the partners' differing recollections of their original verbal exchanges.
The correct answer is: Retrospective falsification

088
The memories closest to a disruptive event are most vulnerable to loss. This is called

Select one:
Senile hypothesis
Dwarf hypothesis
Ribot's hypothesis
Moral hypothesis
Glutamate hypothesis

Consolidation is perhaps most often suggested as an explanation for the gradient in retrograde
amnesia . After damage to the hippocampal memory system, patients tend to lose more of their
recent than of their remote memories suggested that recent memories might be more
vulnerable to brain damage than remote memories . The Ribot gradient can be explained by
assuming that memories are first dependent on a hippocampal memory system for their
retrieval. Through consolidation memories gradually become stored in the neocortex, making
them independent of the hippocampal system . If the hippocampal system is damaged, recent
memories are lost as they still depend on that system. Old memories have already been stored
in the neocortex through consolidation and are thus spared.

The correct answer is: Ribot's hypothesis

089
Choose the most unstructured test from the following list

Select one:
Sentence Completion Test

45
Stanford Binet Test
Rorschach Inkblot test
Thematic Appreception Test

Weschler Memory Scale

The correct answer is: Rorschach Inkblot test

090
Which among the following is defined as the basic building block or unit of intelligent
behaviour?

Select one:
Adaptation

Assimilation
Equilibration
Accommodation
Schema

The correct answer is: Schema

091
The storage of information in pure form without specification of time and place is called

Select one:
Semantic memory
Episodic memory
Absolute memory

46
Schemas
Declarative memory

Semantic memory is an important part of declarative memory and is concerned with


remembering facts, ideas and concepts. It refers to what is known rather than when and how
the knowledge was acquired. Eg in answering a question 'what is the capital of Scotland', one is
using semantic memory. Most abstract knowledge consists of material drawn from semantic
memory
The correct answer is: Semantic memory

092
Which one among the following is not correctly paired?

Select one:
Pavlov-Classical conditioning
Thorndike-Law of effect
Tolman-Latent learning

Skinner-Modelling
Kohler-insight learning

Modelling is a type of observational learning, explained by the social learning theory, and
Bandura is the key name associated with it.
The correct answer is: Skinner-Modelling

093
A dysthymic patient vividly recalled the events that occurred when he was depressed but
poorly recalled the events that occurred during happier times. This is an example of

Select one:
Failure of prospective memory
Context dependent memory
Blocking
Encoding failure
State dependent memory

47
State­dependent memory; It is a phenomenon in which the retrieval cue at the time of encoding
information is one's internal state rather than the external context. These might be psychological
or physiological (being nervous, drunk, etc.) e.g. a depressed patient will normally tend to focus
on events that happened during periods of depression and will have difficulty recalling more
positive moments. The other common observation is seen in alcoholics, who when sober cannot
recall what occurred when drunk, only to remember again next time when he/she drink too
much.
The correct answer is: State dependent memory

094
Intelligent behaviors arise from a balance between analytical, creative and physical abilities.
This notion refers to which of the following?

Select one:
Sternberg's theory
Spearman's theory
Cyril Burt's two factor model
Cattell & Horn intelligence model
Stanford-Binet theory

Sternberg's Triarchic Theory of (Successful) Intelligence contends that intelligent behaviour


arises from a balance between analytical, creative and practical abilities, and that these
abilities function collectively to allow individuals to achieve success within particular
sociocultural contexts .
The correct answer is: Sternberg's theory

095
A fear of spiders extending itself to a fear of other insects is an example of what behavioral
phenomenon?

Select one:
Stimulus discrimination
Stimulus generalization

Stimulus incubation
Stimulus sensitisation
Stimulus decoupling

48
The correct answer is: Stimulus generalization

096
A patient has a phobia of escalators and avoids London tube stations as a result. Deep muscle
relaxation is paired with a series of imagined scenes that are arranged in a hierarchy from the
least to the most anxiety-producing stimuli. This therapy is called

Select one:
secondary reinforcement
cognitive restructuring
cognitive remediation
Systematic desensitization

flooding

The correct answer is: Systematic desensitization

097
Which of the following is a projective test of personality?

Select one:
Cattell's 16 PF test
Eysenck's personality inventory

Minnesota Multiple Personality Inventory


International personality disorder examination
Thematic Apperception test

49
.
The correct answer is: Thematic Apperception test

098
Which of the following correctly describes the exposure and response prevention treatment for a
lady with fear of contamination and repeated washing?

Select one:
Therapist asks her too limit her washing to 4 times per act
Therapist asks her to wash her hands without touching the dirt
Therapist asks her to say 'STOP' aloud whenever the fear of contamination is
experienced
Therapist asks her to touch the dirty linen but prevents her from washing her
hands
Therapist asks her to touch dirty linen before thoroughly washing her hand
with antiseptics

By asking the patient to touch the linen, the therapist is exposing her to obsession-inducing
situations only to prevent the response of hand washing later.
The correct answer is: Therapist asks her to touch the dirty linen but prevents her from
washing her hands

099
Which of the following statements is correct concerning people with Type B personalities?

Select one:
They may be high achievers and workaholics
They cope poorly when under stress

All of the above


They show impatience and are incapable of relaxation

They show high competitiveness, hostility and aggression

50
Type A persons show impatience, excessive time consciousness, insecurity, high
competitiveness, hostility and aggression and are incapable of relaxation. They may be high
achievers and workaholics. Type B persons are relaxed, and easy-going, creative, often self-
analyze and evade stress but cope poorly when under stress.
The correct answer is: They cope poorly when under stress

100
If the conditioned stimulus (CS) ends prior to the application of unconditioned stimulus (UCS) it
is called

Select one:
Forward conditioning
Delayed conditioning
Trace conditioning
Backward conditioning
Higher order Conditioning

.
The correct answer is: Trace conditioning

101
A 59-year-old man played Lotto consistently with the desire to win big one day. He has won
small amounts of money on certain occasions. What is the reinforcement schedule
demonstrated by his behaviour?

Select one:
Fixed ratio
Variable interval

None of the above


Fixed interval
Variable ratio

51
In variable ratio schedule, every nth response is rewarded on average, but the gap between
two rewarded responses may be very small or fairly large; this schedule is found in fishing and
gambling. This schedule is very resistant to extinction.
The correct answer is: Variable ratio

102
A psychologist performed paired associates test on a 71-year-old man complaining of
memory loss. This test is useful to

Select one:
Intelligence
Verbal memory
Premorbid IQ
Attention
Visual memory

Paired associates test is a test of verbal memory. In this test, subjects are asked to remember
a set of related and unrelated pairs of words. Normally, remembering related word pairs is
easier than the unrelated pairs. But patients with Alzheimer's disease or Mild Cognitive
Impairment perform equally poorly on both arms of the test.
The correct answer is: Verbal memory

103
Of all sensory systems, the system least developed at birth is

Select one:
Vision
Smell

Taste
Touch
Balance

52
The correct answer is: Vision

104
Which of the following best describes flash-bulb memories?

Select one:
Memories of events that happened in broad day-light persisting for longer
periods of time
Vivid memories of personal events associated with intense emotional
arousal
Instantaneous recall that cannot be sustained for further processing
Recall of images in memory with extreme accuracy and in abundant volume

Instances of knowing something that cannot immediately be recalled

The term flashbulb memory refers to memories laid down vividly and accurately as a
photograph when emotionally important events have occurred e.g. a personal loss event or
public disaster such as 9/11.
The correct answer is: Vivid memories of personal events associated with intense emotional
arousal

105
A good test for recent memory is to ask patients which of the following?

Select one:
To subtract 7 from 100
Their date of birth
Spell WORLD backwards
What they had to eat for their last meal
Who is the prime minister of UK

Recent memory is the ability to remember what has been experienced within the past few
minutes (Recall of items after five minutes), hours (Recall of last meal), days (Recall of recent
topics in news). Remote memory is the ability to remember events in the distant past (weeks to

53
years). This can be tested by inquiring about important dates in their lives such as date of
birth, date of marriage and how many siblings they have.
The correct answer is: What they had to eat for their last m

54
001
Which of the following statements does not reduce cognitive dissonance in a cocaine user?

Select one:
"I have more chances of dying when crossing a road than injecting cocaine"
"Smoking is worse killer than cocaine use"
"Alcohol affects the whole body while cocaine is not that bad"

"I like cocaine very much"


"Even Freud used cocaine in those days, there must be something good
about it".

'I like cocaine very much' is likely to increase not decrease the dissonance.
The correct answer is: "I like cocaine very much"

002
At what age do children pass the mirror test?

Select one:
6 Months of age
9 Months of age
3 Months of age
18 months of age
12 Months of age

55
Mirror test- A test for self-recognition in which the child using its mirror image to touch a dot on
its nose is achieved at 18 months of age.
The correct answer is: 18 months of age

003
Which of the following increases the effectiveness of a persuasive communication?

Select one:
Mass media communication
High intensity urging
A credible communicator
High receiver intelligence
High cognitive dissonance

Carl Hovland, at Yale University, studied various factors affecting persuasion.The credibility of a
perceived message is a crucial factor in persuasion . a health report is more persuasive if it is
published in a professional medical journal, than in a tabloid.
The correct answer is: A credible communicator

004
Which of the following describes a self-serving bias?

Select one:
A person attributes other people's behaviour to internal sources A
person attributes other people's behaviour to external sources A
person attributes successes to external sources
A person attributes successes to internal sources

A person attributes failures to internal sources

Self-serving bias- people show a strong bias towards attributing their success to internal
causes while attributing failures to situational causes.

56
The correct answer is: A person attributes successes to internal sources

005
Dissonance is decreased by

Select one:
Expectation of unpleasant consequences of behaviour towards others.
Adding new cognitions
Awareness of responsibility for consequences
Low pressure to comply
Increased choice of options

Festinger proposed the cognitive dissonance theory in 1957. Individuals strive for consistency in
their attitudes with discomfort or dissonance arising if two cognitions are held that are
inconsistent. Dissonance is increased by A. Low pressure to comply; B. Wide perceived choice
of options C. Awareness of responsibility for consequences D. Unpleasant consequences for
others. Dissonance is lowered if the behaviour is altered; cognitions are dismissed, and new
cognitions are added.
The correct answer is: Adding new cognitions

006
Which of the following is true concerning primate psychology?

Select one:
Aggressive male baboons have longer life expectancy
Aggression is higher during periods of instability

Young baboons learn appropriate expression of aggression by observing


their peers
Low ranking baboons are not aggressive
Among baboons, females are not aggressive

During periods of instability such as rank transition, baboons show higher levels of
aggression. This leads to high cortisol levels and stress among the animals.
The correct answer is: Aggression is higher during periods of instability

57
007
Which of the following is most likely to influence individuals to conform to the views of the
group?

Select one:
Other individuals being friends
Older individuals
Individuals feel accepted by the group

Large group size


Ambiguous tasks

Ambiguous tasks tend to lead to more conformity as people may feel less certain of their own
ideas.
The correct answer is: Ambiguous tasks

008
Which of the following correctly describes fundamental attribution error?

Select one:
Refusing to accept one's own errors
Denying the fundamental flaws behind one's own negative behaviour

Attributing others mistakes to the context in which the mistakes occur

Attributing one's own mistakes to one's character and personality

Attributing others mistakes to their personal dispositions

Fundamental Attribution Error refers to overestimating dispositional factors and not situational
factors while attributing causes to other's behaviours. This allows a sense of predictability to be
developed about the other person. It is more pronounced if the attributed behaviour is negative
and undesirable.
The correct answer is: Attributing others mistakes to their personal dispositions

009
The theory of mind is not well developed in which of the following disorders?

Select one:

58
All of the above
Anxiety disorders
Psychosomatic disorders

Depressive disorders
Autistic spectrum disorders

Theory of mind: An understanding that other people possess mental states that involve ideas
and views of the world that are different from our own. Children typically develop the theory of
mind around four years of age. This ability has been implicated to be absent in autistic disorder.
Many of the difficulties experienced by children with autism such as communication and
interpersonal difficulties might be explained in terms of an absence of the theory of mind.
The correct answer is: Autistic spectrum disorders

010
The first aspect of self-concept to develop is the

Select one:
Self actualisation
Self efficacy
Self image
Self esteem
Bodily self

Bodily self-refers to the ability to differentiate one's own body from that of others.
The correct answer is: Bodily self

011
The cognitive dissonance is;

Select one:
Is a type of plea
Phenomenon of group think
Caused by inconsistent cognitions

Inhibits problem solving

59
Form of thought disorder

The principle of cognitive consistency is the focus of Festinger's (1957) theory of cognitive
dissonance. This theory starts from the idea that we seek consistency in our beliefs and
attitudes in any situation where two cognitions are inconsistent.
The correct answer is: Caused by inconsistent cognitions

012
The individual behaviour of group members is less important than that of the whole group?.
What is this concept called?

Select one:
Group think
Sociogram
Deindividuation
Risky shift phenomenon

Polarization

The individual behaviour of group members is less important than that of the whole group. This
is called deindividuation. It is a process where people lose their sense of socialised individual
identity and resort to unsocialized and antisocial behaviours. People normally refrain from acting
in an aggressive and selfish manner in part because they are easily identifiable in societies that
have strong norms against such uncivilised behaviour. In certain situations such as in crowds,
these restraints are relaxed, and people may engage in antisocial behaviour (e.g. the Tottenham
violence August 2011). The larger the group, the greater the anonymity and the greater the
difficulty in identifying a single individual. But in polarization, when individuals express their
opinions separately and then group to decide upon the same matter the eventual outcome is
likely to be more extreme than that of the group average. Risky shift phenomenon: people tend
to make riskier decisions when working as members of a group than they would make when
making the same decision as individuals. Groupthink is the desire to achieve consensus and
avoid dissent in group decisions. Sociogram developed by Moreno is a graphical representation
of relationships in a group.
The correct answer is: Deindividuation

013
An individual's tendency of not to intervene in a help-seeking situation where others are
present is known as

Select one:

60
Diffusion of responsibility
Persuasion
Bystander intervention

Obedience
Pluralistic ignorance

Diffusion of responsibility: people feel that the responsibility is not theirs, and someone else will
do something. In pluralistic ignorance, members of a group convince each other that there is no
problem that requires intervention.
The correct answer is: Diffusion of responsibility

014
Which of the following is a behavioural component of Allport's concept of prejudice? (Nov
2009)

Select one:
Circumlocution
Stereotype
Discrimination
Moral attack
Hostility

Prejudice is essentially an attitude. Hence, it has 1. Cognitive component ­ stereotypes; 2.


Affective component ­ hostility; 3. Behavioural component ­ which according to Allport can be
Anti-locution, avoidance, discrimination. physical attack or extermination in terms of increasing
severity.
The correct answer is: Discrimination

015
In a small group situation, departures from norms are most likely to be tolerated if

Select one:
The setting is private
Presence of strong group leadership
The norms were stated frequently during previous meetings

61
Dissent comes from a member with strong credentials
A consensus was reached before the dissent was expressed

Group members may wish to ignore a norm if it is burdensome or a source of punishment. Norm
rejection is more easily achieved in the absence of enforcement and more readily tolerated if the
rejecting member possesses strengths and abilities needed by the group. A norm that has been
frequently and clearly stated is more difficult to avoid because members cannot then claim
ignorance or misinterpretation. The threat of censure or punishment is a strong deter- rent to
violation of group norms, especially if the group is very cohesive and if the setting is one of
privacy.
The correct answer is: Dissent comes from a member with strong credentials

016
Aggression is a consequence of frustration'- Who proposed this hypothesis?

Select one:
The cue hypothesis of aggression
Lazarus and Folkman
Dollard's frustration-aggression hypothesis
Friedman and Rosenman
Lorenz ethological studies

According to Lorenz, aggression in nonhumans is essentially destructive and is characterised by


ritualisation and appeasement. But in humans it is basically destructive and has become
distorted. According to Berkowitz, aggressive-cue hypothesis frustration provokes anger, not
aggression. For this anger to be expressed as aggression, certain environmental cues are
needed.
The correct answer is: Dollard's frustration-aggression hypothesis

017
A patient who feels unhappy with respect to one aspect of his life soon observes that this
affects several other aspects of his life, turning his mood to a miserable one. This cognitive
bias is called

Select one:
Domino effect
Recall bias

62
Stockholm effect
Hawthorne effect
Attributional fallacy

Domino effect is the psychological equivalent of the butterfly effect in chaos theory. According
to this notion, large changes in a later state are dependent on the initial conditions in which a
small change can result in a large catastrophe.
The correct answer is: Domino effect

018
The attributional bias seen in persecutory delusions are

Select one:
Internal attribution for positive events
Internal attribution for negative events

External attribution for negative events


Internal attribution for all events
External attribution for positive events

It has been proposed that individuals with persecutory delusions make excessive external
attributions of the cause for negative events. It remains unclear from empirical research
findings whether the externalizing bias is also common to other psychotic symptom
presentations.
The correct answer is: External attribution for negative events

019
The term 'cognitive dissonance' was coined by

Select one:
McClelland
Schacter
Festinger
Beck
Bard

63
Festinger coined the term 'cognitive dissonance'.
The correct answer is: Festinger

020
The Likert scale consists of

Select one:
Three responses to each statement

Four responses to each statement

Five responses to each statement

Six responses to each statement

Seven responses to each statement

The Likert scale consists of five responses to each statement- Agree/disagree on a five-point
scale. These are quick to design, but they are relatively sensitive. The Thurston scale has
equal intervals between successive points on the scale. A range of statements is presented,
and you tick those you agree with.
The correct answer is: Five responses to each statement

021
In a social psychology study, the subjects were told that writer's attitudes were determined by a
coin toss. Despite this, the subjects rated writers who spoke in favour of a subject as having on
average a more positive attitude towards the subject than those who spoke against it.
Which of the following does this indicate?

Select one:
Pygmalion effect
Actor-observer bias

Just world phenomenon


Fundamental attribution error
Hawthorne effect

64
We tend to explain behavior in terms of internal disposition, such as personality traits, abilities,
motives, etc. as opposed to external situational factors - this is called as Fundamental Attribution
Error.
The correct answer is: Fundamental attribution error

022
Several people at a train station see an elderly man asking for help. None of the onlookers
offer to help. This is explained by

Select one:
Group think
Halo effect
Risky shift phenomenon
Genovese effect
Group polarization

When alone, individuals will typically intervene if another person is in need of help: this is
called bystander intervention. But intervention becomes less likely to an extent that no single
person will intervene from a crowd or group of observers when someone is in need of help.
This is called bystander apathy or Genovese effect.
The correct answer is: Genovese effect

023
In self-rated scales, a patient who has experienced difficulties in one domain may fail to
recognize and rate his significant improvement in other domains. This effect is called

Select one:
Drift effect
Stockholm effect

Domino effect
Hawthorne effect
Halo effect

The halo effect refers to a cognitive bias whereby the perception of a particular trait is
influenced by the perception of the former traits in a sequence of interpretations

65
66
The correct answer is: Halo effect

024
If a person's performance of a task is influenced by test procedures used, this is called

Select one:
Hawthorne effect
Halo effect
Pygmalion effect
Practice effect
Barnham effect

The Hawthorne effect in psychology refers to the tendency of some people to work harder and
perform better when they are a part of an experiment. This is because of the fact that individuals
often change their behavior simply due to the attention they are receiving from researchers.

The correct answer is: Hawthorne effect

025
Which of the following accurately reflects instrumental aggression?

Select one:
It is often impulsive
It is carried out using destructive weapons

It is often planned
It is motivated by negative feelings
It is often purposeless

Instrumental aggression is carried out for the purpose of achieving a particular goal, e.g.,
kidnapping for ransom. Hence, it is often planned and not impulsive. Hostile (also called angry
or affective) aggression is motivated by the need to express negative feelings, such as anger.

The correct answer is: It is often planned

026

67
Task completion is poor in which of the following types of leadership?

Select one:
Democratic leaderships
Autocratic leadership style
Controlling

Rigid
Laissez-faire leadership

Successful leadership is determined both by the characteristics of the leader and the
characteristics of the situation. This is known as contingency theory of leadership. Autocratic,
democratic and laissez-faire are leadership types proposed by Lewin. Productivity is higher in
laissez-faire leadership than in autocratic leadership. Task completion is good in autocratic and
democratic leaderships but poor in groups led by a leader with laissez-faire style.
The correct answer is: Laissez-faire leadership

027
Vulnerability to conform with the group is more in those that are

Select one:
Less intelligent
Self-reliant
Expressive.
Socially able
Intelligent

Conformity refers to the normative social influence that makes an individual to agree with the
group view despite holding a different personal view. Experiments by Asch using a line length
judging task in a group setting explained various features of conformity seen in social settings.
Conformity increases with group number (maximum effect with three) and the perceived high
status of other group members. Less intelligent members conform more than those with higher
IQ. The presence of dissent in the group reduces conformity.
The correct answer is: Less intelligent

028
The vulnerability to conform is greater in individuals with which of the following characteristic
features?

68
Select one:
Socially able
Naive
Very expressive

Highly intelligent
Self reliant

People of high status more likely to induce conformity while people of low status more likely to
conform. Conformity increases with increasing group size, but after five people the effect
diminishes. People who are naive are more likely to conform, especially to members of their own
group and less likely to conform to members of another group.
The correct answer is: Naive

029
Which of the following is considered to be a factor of social influence when a group makes a
polarized decision?

Select one:
Normative influence
Summative influence
Disintegrative influence
Authoritative influence
Formative influence

Normative influence: People have a need not to appear odd or 'stick out' as a sore thumb. So
they say yes to what other say.
The correct answer is: Normative influence

030
The classic studies by Milgram explain the concept of

Select one:
Social norms
Persuasion
Motivation

69
Conformity
Obedience

The classic studies by Milgram showing that people would obey orders under certain conditions
even if these exceeded the bounds of their usual beliefs explain the social psychology of
obedience. Obedience is influenced by various factors including the perceived authority and
powerfulness of the commander.
The correct answer is: Obedience

031
Which of the following can reduce groupthink?

Select one:
Strong chairperson
Financial incentives
Unplanned meetings of the group
Fewer group members
Open debate

Various strategies that can reduce groupthink include encouraging open debate,
acknowledging the presence of groupthink, seeking external opinion, splitting the group into
smaller units for discussion, holding last chance meetings to encourage challenges and
reserving leader's opinions until after the groups discussion has been completed.
The correct answer is: Open debate

032
A team leader is constantly stressing the group to focus on the commitment to the overall
outcomes more than anything else. She is following a

Select one:
Relationship oriented style
Autocratic style

Persuasive style
Coercive style
Task oriented style

70
Persuasion is the ability to motivate and enthuse the group to pursue the goal. It differs from
coercion where power is used to enforce change through rewards and punishment. Modern
leadership is the very antithesis of coercion because it involves a degree of consensus rather
than blind obedience.
The correct answer is: Persuasive style

033
When many lights are turned on and off in rapid succession, the observer perceives a single
light as moving from one position to the next rather than different lights coming on and off at
different positions. What is this called?

Select one:
Visual hallucination
Pseudohallucination
Cock tail party phenomenon
Phi phenomenon
None of the above

The phi phenomenon is an optical illusion (Max Wertheimer, 1912), based on the principle that
the human eye is capable of perceiving movement from pieces of information, such as repetition
(flickering) of images. In other words, from a slideshow of a group of frozen images presented at
a certain speed, we will perceive a constantly moving image. This is the principle behind motion
pictures.
The correct answer is: Phi phenomenon

034
Which one of the following is a secondary drive?

Select one:
Sex
None of the above

Play
Food
Water

71
In theories of motivation, Mowrer distinguished between primary and secondary drives.
Primary drives or survival drives are those, which are guided by a physiological need like
hunger or thirst and the primary reinforcers include food water and sex. Competence or
secondary drives are those, which people seek out, but which do not fulfil a physiological
need Eg play. Primary motives come and go. Secondary drives are continuous motives.
The correct answer is: Play

035
The semantic differential scale may be subject to which of the following problems?

Select one:
All of the above
Positional response bias
Recall Bias
Selection bias
Performance bias

The semantic differential scale is a visual analogue scale (7 points) with two polarized
adjectives at either extremes separated by a line. The subjects mark their attitudes between
the two. It may be subject to positional response bias.
The correct answer is: Positional response bias

036
A candidate who has appeared for a mock exam before the actual exam does well than a
candidate who has not. This is called

Select one:
Practice effect
Hawthorne effect

Observer effect
Regression of mean
Halo effect

72
Practice effects occur when a participant in an experiment performs the same task twice or
more after an interval. Subjects can either have a positive (subjects become better at
performing the task) or negative (subjects become worse at performing the task) effect.
Repeated measures designs are almost always affected by practice effects.
The correct answer is: Practice effect

037
What factor is, in general, most influential in the development of friendship relationships?

Select one:
Complementarity
Personal similarity
Proximity
Reciprocity
Attractiveness

A classic study of beginning friendship was reported by Theodore Newcomb in The


Acquaintance Process (1961). Newcomb identified four factors that affect the probability of
making an acquaintance. 1. Proximity. We are more likely to get to know somebody with whom
we have regular contact. 2. Reciprocity. We like people who like us.3. Similarity. We like people
who share our values and beliefs. 4. Complementarity. We are attracted to people whose skills
and abilities are complementary to our own. Complementary means different but compatible
and mutually beneficial, like people with different skills who work together for a common
purpose.
The correct answer is: Proximity

038
In Milgram's obedience experiments, the factors that increased obedience include all except

Select one:
Administering by proxy
Relieving the subject from responsibility of actions

Subject achieving an agentic state


Proximity to the shocked victim
Authority figure providing instructions

73
Being proximal to shocked victim will reduce shock administering behaviour and so reduce
obedience.
The correct answer is: Proximity to the shocked victim

039
The most common effect of cognitive dissonance is

Select one:
Memory disturbances
Formal thought disorder
Cognitive distortions
Psychological distress
Behavioural modification

'Cognitive dissonance' is an aversive psychological state aroused when there is a


discrepancy between actions and attitudes (Festinger, 1957). In situations when actions
cannot be reversed, or when doing so requires great effort, adjusting attitudes to be in line
with decisions often reduces this discrepancy.
The correct answer is: Psychological distress

040
The power in a person due to his ability to provide positive accomplishments is known as

Select one:
Expert power
Legitimate power

Reward power
Referent power
Coercive power

Reward power is the power that is conveyed through rewarding individuals for compliance with
one's wishes. This may be achieved through giving bonuses, raises, a promotion, and extra
time off from work, etc. Coercive power is the power to punish. Referent power is the power
through identification with the leader. Legitimate power is power bestowed by virtue of social
position. Expert power is power resulting having greater knowledge or skills.
The correct answer is: Reward power

74
041
Which one among the following is defined by how much we approve of ourselves and how
worthy we think of ourselves?

Select one:
Self efficacy
Self esteem
Self image
Self actualisation
Self awareness

This is self-esteem. Self-image may be defined as the way we think we are like and how we
describe ourselves. Self-efficacy is a term referring to the belief that we can perform
adequately in a given situation. Self-awareness is the state of being or ability to be,
consciously aware of oneself. Humans and other great apes are believed to have this
capacity. Self-actualisation: It is the motive to realise one's full potential. According to
Maslow's theory, self-actualised people have an acceptance of who they are despite their
faults and limitations and experience a drive to be creative in all aspects of their lives.
The correct answer is: Self esteem

042
Tom is a student who attributes a good grade on an exam to his intelligence and hard work
but a poor grade to the teacher's poor ability and unfair test questions. He is exhibiting;

Select one:
Negativity bias
Attentional bias

Barnum effect
Self serving bias
Just world hypothesis

A self-serving attributional bias refers to individuals attributing their successes to internal or


personal factors but attributing their failures to external or situational factors. This bias is a
mechanism for individuals to protect or enhance their own self-esteem.
The correct answer is: Self serving bias

75
043
The presence of others reduces task performance. This is called

Select one:
Group polarization
Group think
Social facilitation

Risky shift phenomenon


Social loafing

The presence of others reduces task performance. This has been called social loafing.
Ringelmann demonstrated it in the 1880s by observing tug-of-war teams. He demonstrated
that the more people there were in a team, the less the effort made by each person in the
team.
The correct answer is: Social loafing

044
Attitudes do not always correlate with one's behaviours. The attitudes that predict behaviour
best are those that are

Select one:
Broad and not task specific
Not emotionally valued
Flexible and changeable
Not based on one's own life experiences
Strong and consistent

Attitudes predict behaviour if 1. They are strong and consistent 2.Based on direct experience
and 3. Specifically relate to the behaviour being predicted.
The correct answer is: Strong and consistent

045
Scapegoating refers to

Select one:

76
Tension between racial groups rise when the economic conditions are
similar
Encouraging conflicts with a political motive
Sexual assault on a vulnerable group leading to conflicts

Lack of empathy on perpetrators of abuse


Captives becoming emotionally attached to the captivators

Scapegoats are singled out victims who are blamed and discriminated by virtue of their group
membership (e.g. racial group, work group, etc.). Scapegoating involves political and economic
factors as well. Under the economic and political competition model, when resources are
relatively scarce, the dominant group in society will try to exploit [people from underrepresented
groups] for material gain. Discrimination and prejudice are heightened during times of tension
and increased competition over limited resources. Hovland and Sears showed that between
1882 and 1930, there was a direct relationship between the price of cotton and the number of
lynchings of Blacks. As the price of cotton decreased, the number of lynchings increased. In the
aftermath of the Tokyo Earthquake of 1923, which claimed the lives of 100,000 people, several
thousand Koreans were said to be massacred. The racial hatred allegedly stemmed from the
competition from the influx of Korean labor for scarce jobs that were available.

The correct answer is: Tension between racial groups rise when the economic conditions are
similar

046
A mother of an autistic child is curious to know the reason behind her son's apparent lack of
empathy towards other children of similar age. Which of the following can be explained to her in
order to address her question? (June 2009)

Select one:
Attribution error
Cerebellar dysplasia
Precontemplative stage
Theory of mind
Cognitive dissonance

Perspective taking is explained on the basis of the theory of mind. Lack of ToM may be the
explanation for the apparent lack of empathy among children with autism.
The correct answer is: Theory of mind

77
047
According to the frustration-aggression hypothesis which of the following is false?

Select one:
Aggression is the one of the many possible responses to frustration
Fear of punishment can inhibit aggression
Aggression is driven by a need to nullify frustration

Unavailability of the frustrator can inhibit aggression


Unexpressed frustration cannot be displaced onto an innocent target

The frustration-aggression hypothesis of Dollard considers aggression to be one of the many


possible products of frustration. In a meta-analysis including 49 studies, Marcus-Newhall,
Pedersen, Carlson and Miller (2000) found consistent evidence that frustrated individuals show
displacement of aggression from the source of the frustration onto a less powerful or more
accessible target.
The correct answer is: Unexpressed frustration cannot be displaced onto an innocent target

78
001
Insanity sometimes is the sane response to an insane society. This was proposed by

Select one:
R.D. Laing
Anthony Cleare
Thomas Szasz

Foucault
Phillip Pinel

RD Laing was a prominent anti-psychiatrist who wrote 'The Divided Self'.


The correct answer is: R.D. Laing

002
Which of the following is NOT true concerning research ethics?

Select one:
Subjects can be recruited by contacting medical colleagues
Written approval must be obtained from ethics committees
Advertisements can be used to recruit subjects
Financial incentives can be offered for subjects to participate in research
A subject is ethically bound to complete a study after giving written informed
consent

79
The correct answer is: A subject is ethically bound to complete a study after giving written
informed consent

003
What is the number of women that have been reported being amputated in the genital area?

Select one:
10 millions
1 million
100 millions
10000

100, 000

WHO has estimated that 100 to 140 million girls and women worldwide are currently living
with the consequences of FGM.
The correct answer is: 100 millions

004
In prematurely born infants, after which week of pregnancy are we legally able to resuscitate
an infant irrespective of parental wishes?

Select one:
22 wks
16 wks

20 wks
21 wks
24 wks

80
The correct answer is: 24 wks

005
In clinical psychiatric population which of the following disorders is more common in higher
social classes?

Select one:
Autism
Anorexia Nervosa
Bulimia Nervosa
ADHD
Schizophrenia

For a long time now it is debated whether the social class differences in anorexia reflects help
seeking or referral patterns only or the actual disease prevalence. Reappraisal of
socioeconomic status from clinical data in anorexia shows that the social class distribution is
consistently weighted toward social classes 1/2. Community studies have shown that the social
class, professional status, and education were not associated with an increased risk of reporting
an eating disorder in such community samples.
The correct answer is: Anorexia Nervosa

006
Partial adaptation of a new culture without giving up one's culture of origin completely is
called

Select one:
Acculturation
Sojourning

Enculturation
Accommodation
Assimilation

81
Enculturation: This refers to culture being learnt through contact with family, friends,
classmates, teachers and the media. This happens in everyone irrespective of migration, etc.
Assimilation refers to the partial adaptation of a new culture (seen in immigrants or refugees)
without giving up one's culture of origin completely.
The correct answer is: Assimilation

007
The term refrigerator mother is associated with which of the following mental illness?

Select one:
Autism
Panic disorder
Schizophrenia
ADHD
Depression

The correct answer is: Autism

008
Institutional neurosis was described by

Select one:
Pavlov
Goffman

Barton
Parsons
Pilowsky

Russell Barton (1976) described 'institutional neurosis', characterized by symptoms such as


apathy, lack of initiative, loss of interest and submissiveness. Factors attributed to this state
include loss of contact with the outside world, enforced idleness, brutality and bossiness of

82
staff, loss of friends and personal possessions, poor ward atmosphere and loss of prospects
outside the institution.
The correct answer is: Barton

009
The four prima facie ethical principles were promulgated by

Select one:
Charaka
Beauchamp and Childress
Hippocrates
Galen
Aristotle

The prima facie principles were promulgated by Beauchamp and Childress.


The correct answer is: Beauchamp and Childress

010
Which of the following risk factor is likely to be causative in a young man diagnosed with
schizophrenia?

Select one:
Having lost his mother before the age of 14
Being a migrant

Living alone
Alcohol use
HLADR2 gene

Immigration is one of the strongest known risk factors for schizophrenia.


The correct answer is: Being a migrant

011
Which class of psychotropic medication are the most likely ones to impair driving
performance?

83
Select one:
Antipsychotics
Psychostimulants
Mood stabilizers

Benzodiazepines
Antidepressants

The correct answer is: Benzodiazepines

012
Which one among the following is classified under higher-level principle?

Select one:
Maleficence
Justice
Beneficience
Respect for autonomy
Best interests

Higher level principles include Deontology (Rights and duties or Rules) and Teleology
(practice based on best interests / outcomes). The prima-facie principles (constituting the
ethical principlism) are lower level principles.
The correct answer is: Best interests

013
When a psychiatric patient is unwell, family and friends report their actions to mental health
professionals in an attempt to get them admitted to an institution. What was the term used by
Goffman to describe it?

Select one:
Role stripping
Batch living

84
Institutional neurosis
Mortification
Betrayal funnel

Goffman also described the social/moral career of a mental patient. i.e., the process whereby a
person with social ties, friends, and family in the community is institutionalized and converted
into an inmate whose world is limited to his immediate hospital ambience. (Peele et al. 1977).
The first step is process by which patients pass through a 'betrayal funnel', as the people they
trust most family and friends conspire against them, reporting their actions to doctors and mental
health professionals (called the 'circuit of agents') who run the decision- making process.

The correct answer is: Betrayal funnel

014
An Asian immigrant in England is observed to practice his religion and cultural traditions at
home but adapts well especially at work with a good degree of fluency in both English and his
native language. This type of enculturation is called

Select one:
Accommodation
Assimiliation

Melting Pot
Biculturalism
Bisection

The acculturative process involves acquisition and retention as well as relinquishing


attitudes/values and practices in both majority and minority populations that continue over
several generations. The rate of change and the circumstances that influence it vary greatly,
both between and within groups. For these reasons, studies of groups experiencing
acculturative change often divide the groups by temporal experience into first-, second-, and
third-generation immigrants. Families within such groups have been categorized as traditional,
transitional, or bicultural. Traditional families are characterized as using their native tongues
rather than English, living in ethnic enclaves, avoiding interaction with majority cultural
institutions, and maintaining preimmigration values and behaviors. Transitional families are
characterized by greater fluency in the language of the host culture and by children who are
becoming familiar with the values and social behaviors of the dominant majority population
through attendance at school and school-related activities. Bicultural families are defined as
those with a high degree of language fluency in their native languages as well as English,
economic stability, and residence in multiethnic settings. Biculturalism appears to be more
adaptive and associated with minimal acculturation stress.

85
The correct answer is: Biculturalism

015
In which of the following disorder an overrepresentation of higher social class is seen in
brothers and children of the patients?

Select one:
Anorexia nervosa
Bipolar disorder
Alcoholism
Depressive disorder
Antisocial personality

An overrepresentation is found in the higher occupational class in bipolar probands' brothers and
children. It is consistently noted that the family of origin in bipolar probands belong to a higher
social class thought the patients themselves might be at a lower social class. The results of one
study clearly showed that higher social class of parents together with longer paternal education
history and larger possession of wealth increased the risk of bipolar disorder in the offspring. It is
speculated that bipolar genes may offer some survival benefits such as excessive creativity or
productivity which uplifts the families to higher social status
The correct answer is: Bipolar disorder

016
Which of the following is a semi-structured interview that can be used to gather information
regarding expressed emotions from family members of a patient with psychosis?

Select one:
Camberwell Family Interview
LEDS Inventory

Brown & Harris Family Interview


Cambridge Family Interview
Holmes & Rahe Inventory

Camberwell Family Interview is a semi-structured interview carried out with relatives of


patients with Schizophrenia.

86
The correct answer is: Camberwell Family Interview

017
Torts are wrongs for which a person is liable in

Select one:
Court of Protection
Civil law
Both Civil and criminal Law
Criminal Law
Military Law

Torts are errors for which a person is liable in civil as opposed to criminal law. They include
negligence, libel, slander, trespass and nuisance. If a person of unsound mind commits a
wrong, then any damages awarded in a court of law are usually only nominal.
The correct answer is: Civil law

018
Major recurring themes behind the stigma against mental illness includes all except

Select one:
Blaming
Cost of treatment required

Poor prognosis
Disruption of social interaction
Dangerousness

Hayward and Bright described four major recurring themes or beliefs behind the stigma
against mental illness. These include 1.Dangerousness 2.Attribution of responsibility 3.Poor
prognosis 4. Disruption of social interaction
The correct answer is: Cost of treatment required

019
Melting pot model refers to which of the following types of countries?

87
Select one:
Countries with increasing immigration trend
Countries with no immigration
Countries with zero population growth

Countries with high population attrition due to emigration


Countries with negative population growth

The United States is often quoted as a Melting Pot of various immigrant societies.
The correct answer is: Countries with increasing immigration trend

020
A 12-year-old girl is called by names at her school after her dad's admission at the local
psychiatric hospital. This is called

Select one:
NIMBY opposition
Self stigma
Nosophobia
Courtesy stigma
Stereotyping

Family and friends may endure a stigma by association, the so-called courtesy stigma
(Goffman, 1963). Courtesy stigma refers to the stigmatization an unaffected person
experiences due to his or her relationship with a person who bears a stigma. Parents of
children with psychiatric conditions are particularly vulnerable to courtesy stigma.
The correct answer is: Courtesy stigma

021
A boy whose mother has depression has a high likelihood to develop one of the following
conditions before age 20?

Select one:
Schizophreniform illness
OCD
Borderline personality disorder

88
Alcohol use disorder
Depression

The strongest risk factor for depression is a personal or family history of depression.
The correct answer is: Depression

022
Which of the following increases the likelihood that a patient with a particular disease gets
stigmatized?

Select one:
Disease is well concealable
Disease is thought to be inflicted by an external agent
Disease leads to disruption in social interactions
Disease elicits pity but no disgust in others
Disease is reversible

The correct answer is: Disease leads to disruption in social interactions

023
According to Holmes and Rahe social readjustment scale, which of the following has the
highest life change value?

Select one:
Birth of a child
Personal injury

Marriage
Road traffic offence
Divorce

89
The Holmes-Rahe scale rates 43 life events by units. After the death of one's spouse (100
units worth), divorce tops the rest of the stressors list and generates about 73 units.
The correct answer is: Divorce

024
A patient refuses to have a nasogastric intubation after an overdose, and the AandE
consultant feels this procedure is potentially lifesaving and complies with expected standard of
care. The best course of action is

Select one:
Refer the patient to a psychiatrist
Do not perform the procedure if the patient is determined to have the
capacity to make this decision
Assume that the patient does not have the capacity to decide whether to
have the procedure or not
Try repeatedly and convince the patient that the procedure is necessary
until they eventually consent
Sedate the patient first and then carry out the procedure

Patients have the right to refuse medical care, even when it seems medically necessary to save
their life. The risks and benefits of accepting or refusing the procedure must be explained to and
understood by the patient in order for them to make an informed decision about such a refusal.
If they show good under- standing of the risks and benefits, then they have the capacity to
decide. Bullying them into changing their mind is inappropriate, although they should be made
aware that the procedure will likely still be made available to them at a later date should they
voluntarily change their mind about having it done. A psychiatry consult may be helpful in this
case if there is concern that an underlying mental illness is affecting the patient's judgment but
is not necessary for determining the patient's capacity to make this decision.

The correct answer is: Do not perform the procedure if the patient is determined to have the
capacity to make this decision

025
Which of the following is regarded as 'Father of Sociology'?

Select one:
Talcott Parsons
Jean-Paul Sartre

Soren Kierkegaard
William Tuke

90
Emil Durkheim

Emile Durkheim, a French journalist, formally established the study of sociology. He is often
thought of as the 'Father of Sociology'. Auguste Comte, who devised a system of societal
reform and advanced the study of sociology, is also claimed as the Father of Sociology by
some.
The correct answer is: Emil Durkheim

026
One year after losing her job of 15 years, a depressed woman gets another job but
unfortunately within two weeks she loses the new job. She feels 'stuck in her life'. Which of the
following category does this event belong?

Select one:
Loss
Entrapment
Threat
Danger

Humiliation

Entrapment includes long-term sustained entrapment includes serious difficulties that can only
get worse or persist according to the subject; or a failed positive event where a potential fresh
start went disastrously wrong within 1-2 wk, leaving the person stuck in square one.
The correct answer is: Entrapment

027
A West African student presents with lethargy, insomnia, palpitation and anxiety. Choose one
culture-bound syndrome:

Select one:
Windigo

Dhat
Latah
Piblokto
Fag

91
The correct answer is: Fag

028
A patient admitted at an inpatient unit is making steady progress. But his symptoms worsen
whenever he comes back from home leave. His mother criticises him of being very reluctant
and lazy to get involved in 'normal life'. The next step in management must consider

Select one:
Behavioural therapy
Increase antipsychotic dose
Family therapy
Detention
Antidepressant drugs

Classical studies by Brown et al., and Vaughn and Leff indicated four times higher relapse
rates for patients with schizophrenia who were discharged to parents who were hostile,
critical, or overly involved (high Expressed emotions - EE), compared to patients whose
parents who did not behave this way. A greater degree of patient adjustment and decreased
relapse are associated with higher levels of family tolerance . Family therapy can help reduce
the EE.
The correct answer is: Family therapy

029
The concept of schizophrenogenic-mother was proposed by

Select one:
Lidz
Fromm-Reichmann

Rutter
Bateson
Wynne

92
The concept of schizophrenogenic-mother was coined by Freida Fromm-Reichmann. These
mothers were described to be 'rejecting, impervious to the feelings of others, rigid in moralism
concerning sex and had a significant fear of intimacy'.
The correct answer is: Fromm-Reichmann

030
The term total institution is associated with

Select one:
Szasz
Foucault
Thomas Hardy
Jacques Lacan
Goffmann

A total institution is a place of work and residence where a great number of similarly situated
people, cut off from the wider community for a considerable time, together lead an enclosed,
formally administered round of life. The term was coined and defined by American sociologist
Erving Goffman in his 1961 work Asylums.
The correct answer is: Goffmann

031
Which of the following aspects best describes one's ethnicity?

Select one:
social activities
genetic factors

group identification
their appearance
values and beliefs

93
Race is perceived as a permanent entity determined by genetic ancestry and characterized by
physical appearance. Culture is perceived as a changeable entity determined by upbringing and
choice and characterized by behaviours and attitudes. Ethnicity is perceived as partially
changeable entity determined by social pressure and psychological need characterized by a
sense of group identity and belonging.
The correct answer is: group identification

032
Which of the following is NOT a vulnerability factor for depression in Brown and Harris study?

Select one:
Having a long standing physical illness
Lack of employment outside home
Absence of a close confiding relationship
Loss of mother before age of 11
Having 3 or more children under 15 living at home

Social and economic circumstances associated with the onset of depression in women living in
inner urban London were studied by Brown and Harris in 1978. They identified four vulnerability
factors: 1. absence of a close confiding relationship; 2. loss of the mother before the age of 11;
3. lack of employment outside the home; 4. Having 3 or more children under 15 living at home.

The correct answer is: Having a long standing physical illness

033
Which of the following describes the code of ethical recommendations used currently to guide
clinical research worldwide?

Select one:
Tuskegee code
GMC guidance

Mt Sinai declaration
Nuremburg Code
Helsinki declaration

94
Helsinki declaration contains the current research ethics code in practice. The Helsinki
Declaration adopted by the 18th World Medical Association General Assembly in 1964 and
has been amended five times since, most recently in 2000. Notes of clarification were added in
2002 and 2004. The current (2004) version is the only official one. Currently since 2007
another revision has been initiated and consultation is open as of April 2008.
The correct answer is: Helsinki declaration

034
A depressed 55-year-old man reports a high amount of adverse life events, but no positive
events preceding his first episode of depression. Which of the following explanation is NOT
correct?

Select one:
His depression is probably due to absence of positive life events
Social adversity experienced from an event depends on contextual rating by the
patient
Recurrent episodes have less preceding life events than the first episode
Stressful life events have established association with depressive disorder
He is recalling more stressful life events due to cognitive bias

Depressed patients may recall more stressful life events due to cognitive bias. It is shown that
the frequency of desirable (or entrance life events) was comparable in controls and the
depressed population; so the absence of positive events cannot be the simple explanation for
depression. It is demonstrated that those with a recurrent episode of depression have less
preceding life events than those with a first episode of depression. This may be related to
kindling phenomenon.
The correct answer is: His depression is probably due to absence of positive life events

035
Which of the following type of life events predicts the onset of pure depressive illness?

Select one:
Humiliation
Threat

Danger
Entrapment
Loss

95
Combined loss and humiliation events are more depressogenic than a threat or other individual
types of events. Humiliation events induce defeat and submission responses that may be
directly related to depression. In a study by Kendler et al. (2003), humiliation predicted onsets of
pure major depression but not pure generalised anxiety episodes, and danger predicted pure
generalised anxiety but not pure major depression episodes. But the results had only moderate
strength in prediction.
The correct answer is: Humiliation

036
A 40-year-old man has chest pain and dizziness but his blood tests and ECG are normal. He is
advised to take medications, fat-restricted diet, exercise. This is called

Select one:
Sick role
Illness
Depression
Worried well
Illness behaviour

The concept of illness behaviour was largely defined and adopted during the second half of
the twentieth century. Broadly speaking, it is any behavior undertaken by an individual who
feels ill to relieve that experience or to better define the meaning of the illness experience.
The correct answer is: Illness behaviour

037
Best evidence base to decrease alcohol harm is

Select one:
Education of public about harm of alcohol
Interventions in drinking environments

Increasing the price of alcohol


Extending the times of sales
Media-advocacy

96
When other factors are held constant, such as income and the price of other goods, a rise in
alcohol prices leads to less alcohol consumption and vice versa. Price increases reduce the
harms caused by alcohol and can also indicate that heavier drinking has been reduced.
Policies that increase alcohol prices delay the initiation of drinking, slow young people's
progression towards drinking larger amounts and reduce heavy drinking among them. Setting a
minimum price per gram of alcohol can be as effective as an across- the-board increase in tax,
with both options costing heavy consumers far in excess of the cost to light consumers. Natural
experiments in Europe consequent to economic treaties have shown that as alcohol taxes and
prices have gone down, so sales and alcohol consumption have usually increased. (From
Evidence for the effectiveness and cost-effectiveness of interventions to reduce alcohol- related
harm, WHO 2009).
The correct answer is: Increasing the price of alcohol

038
Which of the following statements about Camberwell family interview is not true?

Select one:
It includes positive comments
The rating is based on content and vocal tone
It is carried out with patient
It is a semi-structured standard interview
It assess expressed emotions

It is a semi-structured interview carried out with relatives of patients with Schizophrenia.


The correct answer is: It is carried out with patient

039
Which of the following is true with regards to Changing Minds campaign?

Select one:
It is not a part of Care Programme Approach
It provides a legal clause against stigma

It is an initiative from World Health Organisation


It is focussed on geriatric mental health issues
It is an ongoing programme against stigma

97
Concern about the stigma of mental illness culminated in the Royal College of Psychiatrists' 5-
year 'Changing Minds' campaign (1998-2003) whose aim was to promote positive images of
mental illness, challenge misrepresentations and discrimination, encourage patient advocacy
and educate the public about the real nature and treatability of mental disorder
The correct answer is: It is not a part of Care Programme Approach

040
The following are true in relation to the biopsychosocial model except

Select one:
It is based on the fact that each system is at the same time a component of
higher system
The hierarchy and continuum of natural systems were discussed to explain
biopsychosocial model
It is based on systems theory
It is similar to biomedical model
It was proposed by George Engel

The biopsychosocial model is a systems theory-based explanation of disease models. It is not


similar to the biomedical model.
The correct answer is: It is similar to biomedical model

041
The phenomenon of relative deprivation is explained by

Select one:
Barthel index
Jarmen index

Brian Index
Jasper Index
Haschinski index

98
A scoring system developed by the British general practitioner Brian Jarman for the level of
social deprivation in a community, using census data on percentages of old people living
alone, single-parent families, children younger than 5 years of age, unskilled and unemployed
persons, ethnic minorities, overcrowded dwellings, changes of address in previous year, etc.
Although a valid indicator, it is not generally accepted outside the United Kingdom.

The correct answer is: Jarmen index

042
Which of the following describes an extreme anxiety symptom where a person thinks that his
penis is shrinking into his abdomen, and he may die as a result?

Select one:
Fag
Koro
Dhat
Amok
Latah

Koro is known as genital retraction syndrome. When affected, patients believe that their
genitals will be completely sucked into the body, causing death. While this condition
occasionally occurs in women, it is much more common in men. Koro is a considered a
culture-bound syndrome, meaning that it only occurs in certain cultures and does not directly
correspond with diseases or conditions recognised by Western medicine. It is most common in
China, Southeast Asia, and Malaysia, although outbreaks have occurred in Africa as well. The
condition tends to have a different name in every region. For example, the Chinese term of
'suo yang' translates as 'shrinking penis'.
The correct answer is: Koro

043
Understanding the impact of a life event in light of one's current social context and self-
perspective is called contextual rating of social adversity. Which of the following employs the
above method?

Select one:
Holmes and Rahe social adjustment scale
Life cycle chart

Impact of Events scale


Life events and difficulties schedule
Sociogram

99
Life events can be measured using standard scores for each type of life event (e.g. Marriage
100, divorce 90 etc). Brown and Harris popularized a different method whereby life events are
graded according to the inherent meaning of the events to the individual concerned i.e.
contextual rating of the social adversity. Accordingly the effect and impact of a life event is
understood in light of one's current social context and self-perspective. LEDS=Life Events and
Difficulties Schedule was devised by Brown and Harris.
The correct answer is: Life events and difficulties schedule

044
Which of the following ethical principles is not associated with Charaka?

Select one:
Compassion towards patients
Updating medical knowledge
Sympathy towards the sick
Maintaining medical records
Confidentiality

Charaka promoted 4 Cs - confidentiality, caring practice, continuous professional


development and compassion.
The correct answer is: Maintaining medical records

045
A 40-year-old man received inpatient detoxification two weeks ago and since then has been
staying away from alcohol use. According to Prochaska's model, which stage is he in?

Select one:
Maintenance
Action

Relapse
Contemplation
Precontemplation stage

100
The maintenance stage is the phase wherein the person strives to maintain gains made and
while attempting to improve those areas of life harmed by drug use.
The correct answer is: Maintenance

046
A clinician refuses to prescribe clozapine to an elderly lady with resistant schizophrenia as she
had developed neutropenia 13 years ago when it was first prescribed to her. Which of the
following ethical principle is maintained here?

Select one:
Non maleficience
Autonomy
Beneficience
Justice
Teleology

American philosophers Tom Beauchamp and James Childress and British doctor and
philosopher Raanon Gillon pioneered the following prima facie principles: Non-maleficence
refers to avoiding harm (primum non nocere).
The correct answer is: Non maleficience

047
Which of the following syndromes refer to the fear of cold seen in Chinese men?

Select one:
Pa-Leng
Koro

Piblokto
Shenkui
Amok

Pa-Leng is a form of frigophobia seen in China. It is more common in men.


The correct answer is: Pa-Leng

101
048
A doctor carries out what is best for the patient, regardless of patient's own wishes. This
approach can be termed as

Select one:
utilitarianism
maternalism
paternalism

deontology
consequentialism

Paternalism is the interference with people's liberties or autonomy with an argument that this is
done "for their own good."
The correct answer is: paternalism

049
Which of the following statements is correct about abuse of children?

Select one:
Children with disabilities were less likely to be maltreated than children
without disabilities
Boys are more likely to be the subject of sexual abuse
Girls are more likely to be the subject of physical abuse and are at a greater
risk of physical injury
Abuse is inversely proportional to family size
People who were subjected to physical abuse in childhood are more likely to
abuse their own children

About one-third of all individuals who were maltreated would subject their children to
maltreatment. Children with disabilities are 1.7 times more likely to be maltreated than children
without disabilities. Girls are sexually abused three times more often than boys. Boys are at a
greater risk of serious injury and of emotional neglect than are girls.
The correct answer is: People who were subjected to physical abuse in childhood are more
likely to abuse their own children

102
050
A 31-year-old woman of Inuit origin started becoming too excited for no apparent reason,
taking her clothes off and throwing her naked body into cold icy water. Which of the following is
the most likely diagnosis?

Select one:
Amok
Tstsumu
Koro

Latah
Piblokto

Piblokto is seen in Arctic Eskimo communities. It is characterised by a dissociative episode


with excitement often followed by seizures and coma lasting up to 12 hours. Patients may be
withdrawn before the attack and usually has amnesia for the episode; they may tear off
clothing, shout obscenities, eat faeces, and jump naked into ice cold water.
The correct answer is: Piblokto

051
A man doesn't think drinking alcohol is a problem and doesn't want to quit. According to
Prochaska's model, which stage is he in?

Select one:
Relapse
Precontemplation stage
Contemplation
Action
Maintenance

In pre-contemplation stage, the user does not recognize that problem use exists, although this
may be increasingly obvious to those around them
The correct answer is: Precontemplation stage

052
The notion that there are inherent distinctions among ethnic groups and that some ethnicities
are biologically superior to others can be described as

103
Select one:
Xenophobia
Scientology
Racism

Determinism
Right-centeredness

Racism refers to attitudes, beliefs, or behaviors that favour one group over another. The
minority group might be seen as biologically (innately) inferior and, therefore, practices
involving their domination and exploitation are justified.
The correct answer is: Racism

053
A 40-year-old man with schizophrenia is treatment resistant. He lives with his family who
apparently has high EE. Which of the following statements about EE is true?

Select one:
High EE may be the cause for his schizophrenia
Reducing the duration of face to face time spent with family may reduce the
relapse
High EE can cause poor insight
EE can be measured using Camberwell Assessment of Needs (CANE)
High EE has no effect on relapse rates in female patients

In the majority of the studies, high expressed emotion was predictive of relapse in symptoms of
schizophrenia nine months later for both genders. A large amount of face-to-face contact (more
than 35 hours per week) with a relative with a high expressed emotion score increased the risk
of relapse, but in households with a low expressed emotion score, high levels of contact
appeared to be protective .
The correct answer is: Reducing the duration of face to face time spent with family may reduce
the relapse

054
A medical student is undertaking a clinical research project and is keen to know the ethical
principles that are widely implemented in medical practice. Choose the correct option:

104
Select one:
Duty of care and good will
Respect for autonomy, justice and confidentiality
Consent, confidentiality and capacity assessment.

Respect, justice and consent


Respect for autonomy, beneficence and justice

American philosophers Tom Beauchamp and James Childress and British doctor &
philosopher Raanon Gillon pioneered the following prima facie principles: a) autonomy-
respecting patients' wishes and freedom of choice b) beneficence-acting in patients' best
interests c) Non-maleficence-avoiding harm d) Justice-treating problems equally, with
equitable distribution of resources to the needy. These four principles are the main guiding
aspects of current medical practice, and most other related ethical discussions relevant to
clinical practice can be brought under these four aspects.
The correct answer is: Respect for autonomy, beneficence and justice

055
In UK which of the following disorders are most stigmatized?

Select one:
Learning difficulties and personality disorders
Schizophrenia and addictions
Schizophrenia and eating disorders
Schizophrenia and dementia
Schizophrenia and depression

The correct answer is: Schizophrenia and addictions

056

105
Before being diagnosed as having schizophrenia, Mark was regarded as a suspicious young
man by his colleagues. After an acute admission, his colleagues avoided him and so he
became more suspicious if any one ever approached him. Which of the following aspect of
social labeling applies to Mark's suspiciousness following diagnostic labeling?

Select one:
Partial deviance
Formal deviance
Primary deviance

Secondary deviance
Spiral deviance

Edwin Lemert developed the idea of primary and secondary deviance to explain the social
process of labeling. Primary deviance is any general deviance before the person is labeled as
deviant. Secondary deviance is any action that takes place after primary deviance as a reaction
to the institutions and the labeling.
The correct answer is: Secondary deviance

057
Which of the following genetic changes have been shown to influence an individual's
response to stressful life events in terms of later depression?

Select one:
Prion protein polymorphisms
Serotonin transporter polymorphisms
CYP450 polymorphisms
COMT polymorphism
GABA B receptor polymorphisms

Caspi demonstrated a link between 5HT polymorphisms and life events in depressive
response.
The correct answer is: Serotonin transporter polymorphisms

058
Immigrants carry a higher risk of schizophrenia than the native population in UK. Which of the
following is correct in this regard?

106
Select one:
Significant social disadvantage in the immigrants may explain the
association
Pre-psychotic segregation explains the higher risk
Incidence rates of schizophrenia in Caribbean countries are higher than UK

Psychiatric hospital admission rates are lesser among ethnic minorities


Excess risk is specific for African and Caribbean immigrants

Though the frequency of most mental illnesses are found to be higher in migrants that the
natives, schizophrenia has been studied the most. Conflicting explanations have been offered to
explain why migrants have more schizophrenia. Copper has revisited and reappraised the data
available and summarised the main findings as below; a.The excess risk is not specific for
African-Caribbean immigrants. It is also present among African-born Black immigrants to the
UK, and to a lesser extent among immigrants from Asian countries. Hence, any explanation
cannot be purely biological and not simply race specific. b.Incidence rates of schizophrenia in
Caribbean countries are similar to those found in the indigenous UK population; this excludes
country of origin theory which proposes that the immigrants carry such higher incidence rates
from where they come from. The rate for schizophrenia in second generation African-Caribbean
people born in the UK appears to be higher than in the first generation, which is strongly
suggestive of an environmental rather than a genetic effect. c.There is no evidence for selective
immigration from the Caribbean as part of a pre-psychotic segregation. According to this theory
individuals who are psychosis prone find it hard to survive in the countries of birth and so
immigrate to other regions. d.A significant social disadvantage is evident in the immigrant
population with a higher risk of schizophrenia. e.The immigrants' pathways to psychiatric care
are characterized by long delays in seeking professional help, a lower probability of medical
referral, frequent involvement of the police and emergency services and high proportions of
compulsory and secure-unit admissions. The long-term outcome tends to be correspondingly
unfavorable for immigrants. Hospital admission rates are consistently noted to be higher among
ethnic minority population as a whole but variations between groups. In UK, highest rates of
hospital admissions were noted among Irish migrants followed by people born in Caribbean.
Rate of mental illness among South Asian population is notably lower than UK-born white
population. It is unclear if these are effects of migration or social disadvantage or organizational
differences in pathways of care. Census of inpatients, 2005 showed that 9% of in-patients were
black or mixed black white ethnicity while black patients were 44% more likely to have been
sectioned and 50% more likely to have been put in seclusion. Black Caribbean men were 29%
more likely to have been subject to control and restraint. It is speculated that an association with
the use of substances may be a confounder

The correct answer is: Significant social disadvantage in the immigrants may explain the
association

059
Which of the following is NOT a measure of expressed emotions?

Select one:
Critical comments

107
Positive comments and warmth
Emotional over involvement
Hostility

Social isolation

Expressed emotions concept was developed by Brown and Rutter in 1966 as a part of the
Camberwell Family Interview (audiotaped interview with a carer), and later modified by Vaughn
and Leff in 1976. The ratings were based on content and prosodic aspects and emphasis of
speech. Five measures are considered (including positive regard and emotional warmth); The
final scores of emotional over-involvement, critical comments and hostility were the most
predictive measures for relapse.
The correct answer is: Social isolation

060
A doctor wants to establish a long-term therapeutic relationship with her patient. Focusing on
which of the following provides the best likelihood for achieving this?

Select one:
Strengths and interests of the patient

Adherence to medication
Patient's childhood experiences
Wishes of the patient's carers
Insight of the patient

The major functions of medical interviews include gathering information, developing and
maintaining a therapeutic relationship and communicating information. The establishment of
long-term therapeutic relationship hinges on understanding the strengths and interests of a
patient.
The correct answer is: Strengths and interests of the patient

061
A patient with acute relapse of schizophrenia was given time off the ward. He came back next
day with his mother who was blaming him for his illness, and 'what he has put himself through'.
Which of the following actions is likely to be most helpful?

Select one:
Suggest Cognitive Behavioural Therapy for schizophrenia

108
Stop all home leave immediately
Suggest family therapy
Provide psychodynamic support to the mother

Use Mental Health Act to detain the patient to the ward

High EE can be reduced by family therapy focussed on addressing communication patterns


within the family unit.
The correct answer is: Suggest family therapy

062
A Chinese man is afraid of going out of his house. He believes that the cold wind will destroy
his soul. What is the cultural bound syndrome described here?

Select one:
Koro

Piblokto
Pa-Leng
Susto
Windigo

Susto (also called as espanto) is known as a "fright sickness". Susto comes from the Spanish
word for "fright" (i.e. Sudden intense fear, as of something immediately threatening). A more
severe and potentially fatal form of susto is called espanto (also from Spanish, meaning terror
or intense fright). People believe that if a person is suffering from susto, his or her soul is
separated from the body
The correct answer is: Susto

063
The Myth of Mental Illness' was written by

Select one:
Szasz
Cleare
Cooper

Laing

109
Foucalt

Antipsychiatry refers to a confederation of psychiatrists, psychologists, nurses, social and


welfare workers, lay people and patients who oppose the traditional mental health practice
and treatment. Three major pioneers are 1. R.D. Laing, 2. Thomas Szasz and 3. Foucault.
R.D.Laing wrote 'The Divided Self' (1959), Sanity, Madness and the Family (1964). Thomas
Szasz wrote 'The Myth of Mental Illness' 1961) and 'The Manufacture of Madness' (1971).
Foucault wrote Madness and Civilization (1965).
The correct answer is: Szasz

064
A Japanese man is afraid of going out with his friends as he is embarrassed about his bad
body odour. Despite repeated reassurances that he does not smell badly, he insists on being
left alone. The most likely diagnosis is

Select one:
Taijin-jikoshu-kyofu

Piblokto
Pa-Leng
Koro
Susto

Taijin refers to a fear of losing good will of others due to imagined shortcomings of oneself.
Social anxiety, tremulousness, self-consciousness and a sense of physical defect or deformity
are seen; these can develop into anthropophobia (fear of people) - a severe form of social
phobia.
The correct answer is: Taijin-jikoshu-kyofu

065
Mr. Brown is not feeling well and calls off sick at work. Which of the following behaviours, if
expressed by Mr.Brown under these circumstances would be regarded as abnormal?

Select one:
Extending absence from work when the illness continues
Taking self-blame for the illness
Complying with treatment as prescribed by the doctor

Not carrying out normal social roles

110
Seeking help from an internet doctor

The sick role is a concept described by the American sociologist Talcott Parsons.He defined
the sick role as having four chief characteristics. 1.The sick person is freed or exempted from
carrying out normal social roles. 2. People who are sick are not directly responsible for their
disease. 3. It is necessary that a sick person tries to get well. 4. The sick person must seek
competent help and cooperate with medical care to get well.
The correct answer is: Taking self-blame for the illness

066
A young male doctor from India gets married and moves to the UK with his wife and his two
kids. The children are attending a primary school. His retired parents also join him soon after.
Which of the following is true? (October 2008)

Select one:
No one can adjust to the change for next 5 years
The young couple will have the maximum difficulty in adjusting to change
The children will have the maximum difficulty in adjusting to change
The grandparents will have the maximum difficulty in adjusting to change

Everyone will adjusts with same degree of ease

The correct answer is: The grandparents will have the maximum difficulty in adjusting to
change

067
Which of the following is least likely to be consistent with an individual assuming the Sick
Role?

Select one:
The individual takes time off work to aid recovery

The individual refuses medical help and does not accept his/her diagnosis
The individual is considered not responsible for his/her sickness
The individual is relieved of his social and vocational responsibilities

The individual wants to get better

111
A patient assumes the sick role when ill, seeks to get well and relinquishes the sick role once
better.
The correct answer is: The individual refuses medical help and does not accept his/her
diagnosis

068
A 20 years old immigrant Jamaican man develops schizophrenia. Which of the following is
most likely to be true?

Select one:
His immigration is likely to be due to psychotic experiences
His parents are likely to be from a lower socioeconomic class compared to
parents of native patients
He is likely to be poorly educated compared to native patients
He is likely to seek professional help early in the course of his illness
The risk of schizophrenia in his parents is likely to be similar to the risk in
the parents of native patients

Theories such as prepsychotic segregation have failed to explain the high rates of
schizophrenia seen in immigrants. The most acceptable notion at present is that this risk is
mediated by social adversity faced by immigrants.
The correct answer is: The risk of schizophrenia in his parents is likely to be similar to the risk
in the parents of native patients

069
Which of the following is false with respect to Nuremburg code?

Select one:
Nuremburg is a place in Germany
Voluntary consent was upheld as a vital aspect of research

The code pertains to human research ethics


Tried Japanese doctors for unfair treatment of patients
It is related to World War 2

112
Nuremburg code is related to the trial received by Nazi doctors after WWII.
The correct answer is: Tried Japanese doctors for unfair treatment of patients

070
The right action is the one that has the best foreseeable consequences. This is called;

Select one:
Engel's model
Utilitarian moral theory
Paternalistic model
Hammurabi code
Oath of Hippocrates

Consequentialist moral theories are teleological: they aim at some goal state and evaluate the
morality of actions in terms of progress toward that state. The best-known version of
consequentialism is utilitarianism. As most clearly stated by Mill a proponent of the utilitarianism
(teleology), the basic principle of utilitarianism is: Actions are right to the degree that they tend to
promote the greatest good for the greatest number. In contrast, deontological principles are duty
based. Kant's theory is an example of a deontological or duty-based ethics: it judges morality by
examining the nature of actions and the will of agents rather than goals achieved. A
deontological theory looks at inputs rather than outcomes.
The correct answer is: Utilitarian moral theory

071
A patient with paranoid schizophrenia was enraged at a shop and killed the shopkeeper. A
forensic psychiatrist who examined him feels that he needs psychiatric treatment. The court
decides that he must be sentenced to prison. Which of the following ethical principle is closely
related to the above scenario?

Select one:
Non-Maleficience
Distributive justice

Confidentiality
Utilitarianism
Paternalism

113
114
The strengths of utilitarianism lie in its practicality and clarity. It approximates the principle of
'beneficence' and fits well with approaches to public policy. Legislated responsibilities of
psychiatrists, particularly in relation to issues of public safety, are invariably utilitarian in nature
and have usually emerged in the context of social and political responses to issues such as
public safety especially in relation to forensic patients.
The correct answer is: Utilitarianism

072
According to Nuremburg code formulated after the Second World War, human
experimentation can be carried out only if which of the following conditions is satisfied?

Select one:
Written consent is given for infliction of more than necessary pain
Voluntary consent is given
A study causing harm can be completed only if carers agree with patients
Research is economical
Subjects are well compensated for any suffering underwent

According to Nuremburg Code human experimentation, can be carried out only if 1.voluntary
consent is given 2.Research is intended for common good of the society 3.avoidance of
unnecessary pain and suffering is guaranteed for the subjects 4.subject has liberty to withdraw
at any point 5.qualified researchers undertake research 6.scientist must terminate a study if
more harm is being caused than expected to the subjects
The correct answer is: Voluntary consent is given

073
Identify the study that is related to deliberately infected individuals with hepatitis

Select one:
ECA study
Willowbrook study

Isle of wight study


Belmont study
Tuskesse study

115
From 1963 through 1966, studies were carried out at the Willowbrook State School, a New York
State institution for "mentally defective persons." These studies were designed to gain an
understanding of the natural history of infectious hepatitis and subsequently to test the effects of
gamma globulin in preventing or ameliorating the disease. The subjects, all children, were
deliberately infected with the hepatitis virus; early subjects were fed extracts of stools from
infected individuals, and later subjects received injections of more purified virus preparations.
Investigators defended the deliberate injection of these children by pointing out that the vast
majority of them acquired the infection anyway while at Willowbrook, and perhaps it would be
better for them to be infected under carefully controlled research conditions. During the course
of these studies, Willowbrook closed its doors to new inmates, claiming overcrowded conditions.
However, the hepatitis program, because it occupied its own space at the institution, was able to
continue to admit new patients. Thus, in some cases, parents found that they were unable to
admit their child to Willowbrook unless they agreed to his or her participation in the studies. This
case caused a public outcry because of the perception that parents and their children were
given little choice about whether or not to participate in research .
The correct answer is: Willowbrook study

116
Home HiYield Paper A(1) Development

HiYield Paper A(1)

1
According to Thomas and Chess study, what percentage of children are considered to be
'difficult'?

Select one:
25%
40%
60%

10%
1%

Temperament refers to a relatively small number of simple, non-cognitive, non-motivational


stylistic features of behaviour. Generally accepted aspects of temperament are emotionality,
activity and sociability. Thomas and Chess differentiated temperament from motivation,
personality and abilities as a response to external stimulus, opportunity, demand or expectation.
The social context may intensify or minimise the expression of temperamental features, the
influence of temperament on the environment is bi-directional. According to 1963 New York
Longitudinal Study - easy children-40%, difficult children-10%, slow to warm up- 15%.

The correct answer is: 10%

2
Precocious puberty is suspected in girls before the age of

Select one:
9

117
8
10
7

11

Precocious puberty is suspected in boys before age 9, and girls before age 8. Social stress is a
puberty accelerator, with familial disruption and father absenteeism, being one of the most
effective stressors. In UK the average age of onset of puberty in males is 11.2 years; for
females it is 11 years. Menarche on average is at 12.5 years for females. There has been a
decline in western countries in the age of menarche over the last century from 16 in the 1860s
to around 13 in the 1960s. This secular trend is thought to be due to the improved nutritional
status of the society in general.
The correct answer is: 8

3
What percentage of patients eventually achieve level 3 post-conventional morality, described
by Kohlberg's theory of moral development?

Select one:
100

15
5
5
50

Level 3 post-conventional moralities cannot be considered a part of the normal or expected


codes of development and instead represents a philosophical ideal. Only 15% eventually
achieve level 3. Formal operational thought is necessary but not sufficient to achieve level 3
morality.
The correct answer is: 15

4
Around 18 months of age, what percentage of infants has multiple attachments?

Select one:
87

118
18
50
3

By their first birthday, children do form attachments with more than one familiar figure. This
includes father, siblings, grandparents and others. But it is shown that multiple attachments are
the rule rather than exception. Around 18m, 87% infants have multiple attachments; 50%
primarily attached to the mother, 18% to the father and the rest to equally both. Attachment
process itself is more important than who the attachment figure is.
The correct answer is: 87

5
Which of the following is a normal stage of grief?

Select one:
Ambivalence

Anger
Bargaining
Mourning
Protest

The classic work on stages of grief came from Erich Lindemann, who studied 101 bereaved
people and published in 1944, an article titled "Symptomology and Management of Acute Grief".
In this article he described a set pattern of reaction to a loss event (grief): After an unexpected
death, there is an initial shock that lasts 10-14 days. After the initial shock comes a period of
intense sadness, and the grieving person may withdraw from social contact. Next comes anger,
as the grieving person seems to 'protest'; the unexpected death. Finally, within a year or so, the
grief is resolved, and the person returns to normal. .
The correct answer is: Protest

6
Which period refers to the autistic phase of Margaret Mahler's stages of child's early
development?

Select one:
6-36 months

119
12-24 months

0-2 months

6-12 months

2-6 Months

Mahler's theory for the developmental Phase is as follows: Autistic Phase: first 1 to 2 months.
The infant is oblivious to everything but him/herself. Symbiotic Phase: Next 4 to 5 months.
He/she begins to recognise others in his/her universe, not as separate beings, but as
extensions of oneself.
The correct answer is: 0-2 months

7
Stranger fear peaks at

Select one:
1 year

0-3 months

5-8 years

3-6 months

2-3 years

Infants develop a fear of strangers when they are around 6 months of age and it peaks at
around 12-18 months of age. Fear of strangers declines after the age of three years. Separation
anxiety starts at 1 year of age and child shows rapproachment (hugs when coming back) by 18
months.
The correct answer is: 1 year

8
The strange situation procedure is designed for use with children between the ages of

Select one:
6 months and 1 year
18 months and 2 years
1 month and 6 months

1 year and 18 months

120
2 years and 3 years

The strange situation procedure (SSP) developed by Mary Ainsworth in 1978 has become the
standard method of assessing the security of an infant's attachment to the caregiver, which is
usually the mother. It is mainly designed for children between 12 and 18 months.
The correct answer is: 1 year and 18 months

9
Average age of puberty in boys is

Select one:
8-9 years

9-10 years
14-15 years
10-11 years
11-12 years

Currently in UK the average age of puberty is 11­12 years in boys; slightly lower in girls.
The correct answer is: 11-12 years

10
Three infants aged 3 months, 5 months and 12 months are present in a room with their
mothers. When all three mothers attempt to leave the room, which of the following might
happen?

Select one:
5-months-old infant will cry
All three infants will cry
Depends on the degree of attachment

12-months-old infant will cry

3-months-old infant will cry

121
Separation anxiety is seen at around age 12 months. This vignette is not about the security of
attachment as children, irrespective of the degree of attachment, will show some separation
anxiety.
The correct answer is: 12-months-old infant will cry

11
Attachment behaviour typically peaks between

Select one:
5 and 7 months of age
6 and 12 months of age
12 and 18 months of age
36 and 48 months of age
7 and 9 months of age

Attachment behaviour is that displayed by the infant towards the attachment figure usually but
not necessarily the mother. In the first few months, an infant orientates without discrimination,
But by 5-7 months, the infant preferentially orientates and signals to discriminated persons.
Separation protest is observable by 7-9 months. By 9 months, the onset of attachment and
stranger anxiety could be seen. Attachment behaviour peaks between 12-18 months, and
lessens by school age where the relationship is based more on abstract considerations such
as affection, approval rather than on proximity maintenance.
The correct answer is: 12 and 18 months of age

12
Fears of rejection by peer group peaks around the age of

Select one:
10 years
15 years

21 years
17 years
7 years

Themes of anxiety and fears of rejection by friends initially appear by 11 to 13, peaking by 15
years and then declining by 17 years (the effect being stronger for girls than boys)

122
The correct answer is: 15 years

13
A child can use meaningful words without connecting words. Identify the possible age of the
child.

Select one:
18-30 months
More than 30 months
12-18 months
More than 60 months

0-12 months

Telegraphic speech is seen where meaningful words are used without connecting words. At
this stage adults interact with children in a motherese - short simple raised pitch paraphrased
language directed at infants. At about 24 months, children can produce grammatically paired
words and achieve vocabulary for more than 240 words.
The correct answer is: 18-30 months

14
Gender identity in a normally growing child develops around

Select one:
2 years of age
4 years of age
Puberty
Early schooling

1 year of age

Gender identity refers to the unshakable self-conviction of being male or female that begins
around 18 months and gets fixed by 24 to 30 months.
The correct answer is: 2 years of age

15
The theory of mind, the ability to conceptualize false beliefs, develops around

123
Select one:
4 years of age
5 years of age
3 years of age

2 years of age
6 years of age

In the early 80s, psychologists H. Wimmer and J. Perner showed that a full-fledged Theory of
Mind doesn't develop before the age of 4 in most children
The correct answer is: 4 years of age

16
According to Margaret Mahler, the separation individuation phase lasts from

Select one:
2 to 5 months
5 months to 5 years
10 to 18 months
0 to 2 months
18 to 24 months

Separation-individuation phase lasts from 5 months to 5 years of age. It is further subdivided


into; a)Differentiation sub­phase: (5 to 10m) slowly appreciates the difference between mother
and self b)Practicing sub-phase: (10 to18m) gradual increase in interest on the environment;
practices exploration. C)Rapprochement sub-phase: (18 to 24m) Alternating drives to be
autonomous and dependent; Able to explore alone but requires comfort and reassurance on
return. D) Object constancy sub-phase: (2 to 5yrs) Understand that the mother will not be lost if
temporarily away; hence able to function independently
The correct answer is: 5 months to 5 years

17
A boy was observed to be speaking in fully formed grammatical sentences similar to adult
speech with his mother. By what age is this milestone generally achieved?

Select one:
5 years

124
1 year
6 months
3 years

2 years

Cooing­ 2months; Babbling (gaga, dada)­ 6 months (4 to10 months); One­word stage
(mamma, doggie)­ 1 year (10 to18 months); Two word stage (mummy go, daddy go)­ 2 years
(18 to 24 months); Basic adult grammatical sentence­ 3 years; Adult speech­ 5 years.
The correct answer is: 5 years

18
In a typically developing child, babbling is seen by what age?

Select one:
6 weeks
9-10 months
6 months
3 months
12 months

By 6 weeks the child starts 'cooing'. By 6 months, babbling is seen. Babbling is nothing but the
production of speech sounds repetitively. Spontaneous babbling refers to the situation when the
child enjoys making these sounds alone. All babies around the same age, irrespective of the
culture, start babbling. Even the deaf babies of the deaf mute parents start marbling, but,
unfortunately, the deaf babies stop babbling at 9-10 months.
The correct answer is: 6 months

19
Visual acuity reaches near adult levels by

Select one:
2 months of age
4 months of age
12 months of age
1 month of age

125
6 months of age

At birth, Vision is the least developed of the senses, presumably because there is little
opportunity for it in utero; the acuity if newborn is around 20/300.Newborns can see faces at a
distance of 10 to 20 inches, the distance at which mother hold their babies. At birth, infants can
track and scan objects, can discriminate levels of brightness, able to fix objects, fixed focus at
about 20 cm, figure­ground discrimination. At 1month­ differentiate faces; preference shown for
complex stimuli. 2 months - possesses depth perception, prefers 3 dimensional rather than 2
dimensional representations of a face. At 4 months - colour vision and accommodation. At 6
months-accurate acuity (6:6). Visual acuity reaches near adult levels by 6 months and is
complete by 3 years.
The correct answer is: 6 months of age

20
The age at which Erikson's stage of industry vs. inferiority is seen:

Select one:
9-19 years

4 - 5 years
30-50 years
6 to 11 years
3 - 4 years

Erikson's fourth stage, Industry vs. Inferiority, occurs between six years and puberty. This is the
period in which the child wants to enter the larger world of knowledge and work e.g. a school.
Erikson said that successful experiences give the child a sense of industry, a feeling of
competence and mastery, while failure gives them a sense of inadequacy and inferiority, a
feeling that one is a good-for-nothing.
The correct answer is: 6 to 11 years

21
What is the most sensitive period for development of attachment behaviour in human beings?

Select one:
6 to 18 months
3 to 6 months
1 to 3 months

126
3 to 5 years
18 months to 3 years

According to Bowlby, attachment develops in the latter part of infancy, in the period from six to
eighteen months. Attachment behaviour peaks between 12-18 months. After this period, the
attachment gets stabilized, and disruption of the bond is likely to lead to deleterious effects.
Although infancy may be the optimal time to develop attachment, it has been shown that
children adopted at four years are also capable of developing last lasting attachment to their
adoptive parents (Tizard and Hodges 1978).
The correct answer is: 6 to 18 months

22
In Piaget's theory of cognitive development, the concrete operational stage lasts from

Select one:
12 years and above
18 years and above
Birth to 2 years
2 to 7 years

7-12 years

Concrete operational stage lasts from about 7 to 12 years. Now the baby can think logically
about objects and events. Baby also achieves conservation of mass (age 7) and weight (age 9).
The child can classify objects according to several features and can order them in series along
a single dimension e.g. size. The child is also able to perform operations such as those
involved in comprehending the laws of conservation.
The correct answer is: 7-12 years

23
A child plays peek-a-boo games with his mother. At what age is this game first understood
and enjoyed by the child?

Select one:
6 months of age
3 years or age
3 months of age

127
18 months of age
9 months of age

Object permanence starts by 9 months. It is an understanding that objects that disappear from
the field of perception has not ceased to exist; if searched for this object can be found. Hence,
peek-a-boo games are understood and enjoyed. Initially, this is limited as hidden objects are
searched at where they were last seen (around 9­12 months); not at where they were hidden.
Around 18 months, invisible displacements are inferred, and object permanence is completed.

The correct answer is: 9 months of age

24
Which of the following is INCORRECT concerning child development?

Select one:
A 2 years old child can speak more than 50 words
A child starts babbling at 3 months of age
A 4 years old child speaks with correct grammar

A 5 years old child can tell time by quarter of an hour accuracy A

12 months old child can speak up to 3 words

Reception class children (ages 4-5) begin to read the time to the hour, year 1 children (age 5- 6)
should be able to read the time to the hour or the half hour on analogue clocks and year 2
children (age 6-7) should be able to read the time to the hour, half hour or quarter hour on
analogue clocks.
The correct answer is: A 5 years old child can tell time by quarter of an hour accuracy

25
Kohlberg's concept of conventional morality is closely related to which of the following?

Select one:
A child believes that the right thing is what satisfies ones needs A
child believes that the right thing is what adults approve of
A child believes that the right thing is to uphold ethical principles A

child believes that the right thing is one that avoids punishments

128
A child believes that the right thing is what the intuition suggests

According to Kohlberg the three levels of morality include Preconventional (aim is to avoid
punishment), Conventional (aim is to get approval) and Ethical principle (aim is to uphold
universal ethical principles).
The correct answer is: A child believes that the right thing is what adults approve of

26
Which one among the following has been used as a non-verbal method of assessing
children's cognitive abilities?

Select one:
Ability to draw human figures
Ability to draw dinosaurs
Ability to draw a fish
Ability to draw a cat

Ability to draw a bird

Children's ability to draw human figures has been used as a non-verbal method of assessing
children's cognitive abilities. A standardised form of this is the good enough draw a man test. In
this test, the children are asked to draw a man as best as they can and scored according to the
guidelines provided in the test. The greater the details in the drawing, the greater the score. For
Eg, a four year old's drawings usually consist of two body parts, the head and arms (also called
tadpole stage) whereas a six year old is able to draw a person with head, neck and hands.

The correct answer is: Ability to draw human figures

27
The Cambridge study by Farrington et al. identified risk factors in children between 8-10 years
that can predict later delinquent behaviour by the age of 32. Which one of the following is not a
risk factor identified by this study?

Select one:
Low intelligence

Aggression in children
Family criminality
Low educational attainment

129
Antisocial behaviour

According to the Cambridge study by Farrington et al, the most important childhood predictors
(during age 8-10) of delinquency were antisocial child behaviour, impulsivity, low intelligence,
low attainment, family criminality, poverty and poor parental child-rearing behaviour.
Aggression is not a risk factor identified in this study.
The correct answer is: Aggression in children

28
Language is slower to develop in

Select one:
In neglected children
Deaf children
In twins
Boys

All of the listed

Language is slower to develop in boys, in twins, in larger families, in those from social classes 4
and 5 and in children with a lack of stimulation e.g. deaf and neglected children.
The correct answer is: All of the listed

29
Imprinting is characterised by

Select one:
Specificity

Irreversibility
All of the listed
Sensitive period
Life long retention

130
Imprinting was first described by Konrad Lorenz following experiments on ducks and geese
and described it as a specialised form of early learning and an example of an innate
predisposition to acquire specific information. Imprinting is characteristic of primate
development and is also observed in birds. Imprinting is characterised by irreversibility,
sensitive period, specificity and life-long retention.
The correct answer is: All of the listed

30
Klein's depressive position is related to the process of learning to cope with which of the
following conflicts?

Select one:
Hunger
Depression
Difficult relationships
Ambivalence
Sexual needs

Melanie Klein described two positions - paranoid-schizoid and depressed position. The
paranoid-schizoid position is associated with the use of splitting and projection as a defence
mechanism. This position concerns an inability to perceive a whole object and splits all objects
into their good and bad parts. But in the depressive position, the infant tolerates the ambiguity
or ambivalence and can realise that an individual can have both good and bad qualities. In the
paranoid-schizoid position, there is an anxiety is about the survival of the self but in depressive
position the anxiety is about the loss of the object.
The correct answer is: Ambivalence

31
When children are deprived of parental care from their mother or a mother substitute during
infancy and in the first few years of life which of the following conditions can develop?

Select one:
ADHD
Autism

Anaclitic depression
Childhood schizophrenia
Anankastic personality disorder

131
Anaclitic depression refers to the complex of symptoms exhibited by young children who are
deprived of their mother or a mother surrogate during the first few years of life.
The correct answer is: Anaclitic depression

32
Which of the following is not a feature of Piaget's sensorimotor stage?

Select one:
Primary circular reflexes
Exercising reflexes
Object permanence
Secondary circular reflexes
Animism

Animism is a feature of Piaget's preoperational stage. Here inanimate objects are treated as
living objects.
The correct answer is: Animism

33
In a strange situation experiment, a 2-year-old continues to play without getting disturbed
when the mother leaves the room and does not show any response when she comes back.
What type of attachment is seen here?

Select one:
Resistant attachment
Disorganised attachment

Absence of attachment
Anxious avoidant attachment
Secure attachment

Anxious Avoidant attachment is characterized by anxiety and fright within the child and signs of
not trusting the mother resulting in an indifferent attitude towards mother leaving the room or
entering the room. The distress noted when left alone can be rather easily pacified by a
stranger.
The correct answer is: Anxious avoidant attachment

132
34
During the strange situation procedure, presence of minimal distress both at separation and
on reunion with the caregiver is suggestive of

Select one:
Disorganised attachment
Disorientated attachment
Ambivalent/resistant attachment

Anxious/avoidant attachment
Secure attachment

Type A: Anxious avoidant: 15%. Indifferent attitude to mother leaving the room or entering the
room; keeps playing indifferent to mother's presence. Distress when alone, not when the
mother is leaving. Stranger can comfort the child easily. Highly environment directed, low
attachment behaviour.
The correct answer is: Anxious/avoidant attachment

35
Which one of the following types of infants poorly interact with the mother and do not appear
particularly upset when left with a stranger?

Select one:
Securely attached infants
Disorganised infants
Anxious/resistant infants
Toddlers in daycare
Anxious/avoidant infants

This also refers to anxious/avoidant infants. These children show an indifferent attitude to
mother leaving the room or entering the room; keep playing indifferent to mother's presence.
They exhibit distress when alone, not when the mother is leaving. A stranger can comfort the
child easily.
The correct answer is: Anxious/avoidant infants

36

133
According to the Social Learning Theory which of the following plays no role in the
development of gender appropriate behaviours?

Select one:
Reinforcement by role models
Archetypal Schema
Differential treatment by adults

Imitative behaviours
Cognitive processes in children

The Social Learning Theory was proposed by Bandura as a way of explaining how children
acquire their gender identity based on the influence of other people (particularly their parents).
http://scienceaid.co.uk/psychology/gender/learning.html
The correct answer is: Archetypal Schema

37
In Piaget's theory of cognitive development, the process of solving new problems using
existing schemata is termed as;

Select one:
Adaptation
Maturation
Schema
Assimilation
Accommodation

During development, the child develops mental structures called schemas, which enables him or
her to solve problems in the environment. Schema is an organised structure of knowledge or
abilities that change with age or experience. The processes of assimilation and accommodation
bring about adaptation. The process of solving new problems using existing schema is termed
as Assimilation. Accommodation is the process whereby existing response patterns or schemata
are modified to take account of new experiences. (Changing existing schemata in order to solve
new experiences).
The correct answer is: Assimilation

38
Which one of the following statements about attachment theory is not true?

134
Select one:
Children can be attached as strongly to the father as the mother
According to attachment theory, multiple attachments can occur-
Attachment behaviour ceases by puberty

Attachment behaviour is most evident when the infant is stressed


Attachment can develop despite frequent punishment from attachment
figure

Attachment is a close emotional bond between two people characterised by mutual involvement
and affection and a desire to maintain proximity. The term is more usually used to refer to the
bond that forms between a caregiver (normally the mother) and the infant, but can be formed
with father or any other adult who provides care. Attachment behaviour endures for majority of
life cycle. It may attenuate or be replaced especially during adolescence but generally persist. In
the adult new secure bases are formed. Attachment behaviour is directed at one, or a few
individuals usually in order of preference. In the majority of children there are multiple
attachment figures at the age of 18 months.
The correct answer is: Attachment behaviour ceases by puberty

39
Identify the temperamental dimension not included originally by Buss and Plomin in their
temperamental survey:

Select one:
Activity
Impulsivity
Attention span and persistence
Sociability
Emotionality

Buss and Plomin in 1984, proposed the EAS model, which is a strongly biological model that
views temperament as an inherited personality trait exhibited in early life and are evident in the
first year of life. Temperament is assessed using the EAS-Temperament survey. Emotionality,
Activity and Sociability are three major dimensions. The original theory also included
impulsivity but it was excluded in later revulsions because of its poor heritability.
The correct answer is: Attention span and persistence

135
40 Conventional stage of morality
includes

Select one:
Ethical orientation

Punishment orientation

Reward orientation

Authority orientation

Obedience orientation

During conventional morality stage initially a "good boy - good girl orientation" is seen. Later
authority orientation is noted where the concern goes beyond one's immediate group(s) to the
larger society- to the maintenance of law and order. One's obligation to the law overrides one's
obligations of loyalty to one's family, friends and groups. In other words, no individual or his/her
group is considered to be above the law.
The correct answer is: Authority orientation

41
Mahler is associated with

Select one:
Anal phase
Conventional morality
Autistic phase
Operational stage
Individuality vs. inferiority

Mahler's theory of development included the following 2 early phases:


Autistic Phase: first 1 to 2 months. The infant is oblivious to everything but himself.
Symbiotic Phase: Next 4 to 5 months. He begins to recognize others in his universe, not
separate beings, but as extensions to himself.
The correct answer is: Autistic phase

42

136
Which one of the following attachment category has not been described in the adult
attachment interview developed by Mary Main?

Select one:
Disorganised
Autonomous
Dismissing

Entangled
Autocratic

Adult attachment interview (AAI) is an approach to assess an adult's attachment to his/her


parents. It was developed by Mary Main and her colleagues in 1986. Main devised a semi
structured interview with s to assess the security of an adult's overall working model of
attachment. This is based on the fact that infantile attachment pattern can be predicted
reasonably accurately using discourse analysis of adults when recollecting their childhood.
Accordingly, 4 patterns are noted- secure/autonomous, dismissing, preoccupied/entangled,
and unresolved/disorganised.
The correct answer is: Autocratic

43
In which psychiatric disorder, up to three quarters of patients report physical or sexual abuse
and prolonged separation form parents in early childhood?

Select one:
Obsessive compulsive disorder
Schizophrenia
Borderline personality disorder
Paranoid personality disorder
Atypical Grief reaction

Poor quality of caregiving in the form of neglect, abuse, separation, rejection and disrupted
parenting is associated with most forms of psychopathology ranging from depression to
personality disorders.
The correct answer is: Borderline personality disorder

44

137
Who described human infants to have an 'innate state of helplessness and would most likely
perish without a caregiver'?

Select one:
Saunders
Bowlby
Klein

Mahler
Anna Freud

Bowlby believed that attachment is innate and adaptive. We are all born with an inherited need
to form attachments and this need helps us to survive. In his terms, the newborn infant is
helpless and relies on its mother/caregiver for food and warmth and hence the attachment
behaviour is essentially adaptive.
The correct answer is: Bowlby

45
Which one of the following terms refers to the bias towards focussing attention on only one
aspect of a situation and the inability to attend to other features?

Select one:
Symbolic thought
Centration
Egocentrism
Circular reactions
Irreversibility

Centration refers to the bias towards focussing attention on only one aspect of a situation and
the inability to attend to other features, which is demonstrated by piaget's famous conservation
tasks. The experiment involves showing two identical glasses with equal amounts of liquid and
pouring the liquid from one glass into a taller thinner glass. The pre-operational child would say
that taller glass has more liquid because the level has risen higher. The child has difficulty
understanding that despite appearances the quantity remains the same because none has been
added or taken away. It shows that the child has failed to grasp the concept of conservation.

The correct answer is: Centration

138
46
In Margaret Mahler's stages of early development, the term symbiosis refers to

Select one:
Child's perception of mother and self as one unit
Child's lack of exploration of outer world
Child's ability to gratify mother's emotional needs

Child's nutritional dependence on breast milk


Child's lack of exploration of inner world

Mahler used the word symbiosis to refer to the 'undifferentiatedness' or 'boundarylessness'


perceived by a growing child with respect to its relation with the mother.
The correct answer is: Child's perception of mother and self as one unit

47
Who proposed that human beings are born with a 'Language Acquisition device' (LAD) that
enables children gather information about the rules of language use?

Select one:
Melanie Klein
Donald Winnicott
Vygotsky
Chomsky
Margaret Mahler

Noam Chomsky: Children are born with an innate language acquisition device. Transformational
grammar is important in understanding language development. All languages have a surface
structure where syntax is accurate and actual words are used to construct language; and a
deep structure where more semantic sense is made without similar syntactical rules. A single
surface structure can have several deep structures. Children are born equipped with the ability
to decipher the transformational grammar of deep to surface structure conversion. Hence years
up to puberty are sensitive though not critical fro language development. Social interaction view
of language development: Adults such as mother act as LASS (language acquisition support
system). This is essential as the function of language is social interaction.

The correct answer is: Chomsky

139
48
A city council provides three different modes of support for abandoned and abused children -
foster care, institutional support or supervised care within the family of origin. Which of the
following outcome can be expected from the care provided?

Select one:
Cognitive recovery occurs when institutionalised children are moved to
foster care
All three groups will have similar IQ profile
Age at placement under social service care has no influence on cognitive
outcome
Never-institutionalised children will show lower than average IQ
Indiscriminate sociability is seen in children reared within a family system

RCT of abandoned children reared in institutions vs. abandoned children placed in institutions
but then moved to foster care showed markedly poor cognitive outcome for children that were
in institutions; but cognitive recovery occurred significantly for younger children placed in foster
care. This suggests a possible sensitive period in cognitive development. O'Connor and Rutter
(2000) compared young children adopted from Romania with children adopted within the
United Kingdom. They found that at both age 4 and again at age 6, the duration of deprivation
was linearly related to the number of signs of attachment disorders. Children exhibiting
indiscriminate sociability at age 6 had experienced deprivation for twice as long as children
exhibiting no attachment disorder signs. Nelson CA III et al. Cognitive recovery in socially
deprived young children: The Bucharest Early Intervention Project. Science 2007 Dec 21;
318:1937. O'Connor, T.G., and Rutter, M. (2000). Attachment disorder behaviour following
early severe deprivation: Extension and longitudinal follow-up. English and Romania Adoptees
Study Team. Journal of the American Academy of Child and Adolescent Psychiatry, 39,
703-712
The correct answer is: Cognitive recovery occurs when institutionalised children are moved to
foster care

49
In which stage of Piaget's model can a boy make out the difference between the amount of
water present in a tall glass and a wide mouthed container?

Select one:
Conventional stage
Formal operational
Sensorimoto
Pre operational
Concrete operational

140
During concrete operational stage, the ability of conservation of liquid develops (around 6
years) followed by conservation of length, count, weight and volume (around 11 to 12 years) in
a vertical decalage fashion (i.e. stepwise, one by one development).
The correct answer is: Concrete operational

50
Which of the following experiments is not correctly matched with Piaget's stages of cognitive
development?

Select one:
Pendulum experiment- Formal operational stage
Hiding objects- Sensorimotor stage
Mountain tasks- Pre-operational stage
Classification of objects according to rules-concrete operational stage

Conservation tasks- sensorimotor stage

Piaget illustrated ego-centric thinking with his famous mountain task and conservation tasks to
demonstrate the deficiencies in pre-operational thinking. Using a series of experiments piaget
demonstrated that during this stage the child has not grasped the concept of conservation and
does not realise that certain properties of objects e.g. Mass, volume, number, length, and weight
remain unchanged even when the objects appearances are altered.
The correct answer is: Conservation tasks- sensorimotor stage

51
The visual cliff is a test of

Select one:
Touch perception
Depth perception

Height perception
Sound perception
Colour perception

141
Visual Cliff is an apparatus used to test an infant's perception of depth. A pane of thick glass
covers a shallow drop and a deep drop. The underlying surfaces of both deep and shallow
sides are covered with the same chequered pattern. Children of six months and older will not
venture over the 'deep side', and this is taken as an indication that the child can perceive depth.
( Visual Cliff, http://www.abahe.co.uk/visual-cliff.html (accessed April 11, 2015).)
The correct answer is: Depth perception

52
In the strange situation test, the child undergoing the examination is rigid and frozen when
mum returns and does not involve in seeking out contact with her. The attachment behaviour
on display is best characterised as

Select one:
Disorganised attachment behaviour
Anxious avoidant behaviour
Anxious resistant behaviour
Absence of attachment behaviour
Securely attached

Frozen and dazed look is associated with disorganized attachment behaviour.


The correct answer is: Disorganised attachment behaviour

53
An 18­month­old girl is a 'blessing to look after'. She has a regular sleep pattern; she calms
down quickly when she cries. She has a friendly social smile with strangers. Her temperament is
best termed as

Select one:
Insecure
Slow to warm up

Easy
Difficult
Secure

Thomas and Chess classified temperaments into easy, difficult and 'slow to warm up'. This girl
belongs to the easy group.

142
The correct answer is: Easy

54
In a simple experiment using cardboard and toys on a desk, a 4 year old boy could not say
what a person would see from the other side of the desk when only one side of an object was
visible from that side of the desk. Which among the following feature is demonstrated in this
'mountain task' experiment?

Select one:
Transductive reasoning
Object permanence
Egocentrism
Hypothetico deductive reasoning
Symbolic thought

Egocentrism is a feature of pre operational stage (2-7 years). This does not refer to self-
centeredness or selfish attitude. It refers to the restrictive ability of viewing the world from a
single point of view at this developmental stage. In the famous mountain task, the child is
presented with a model of mountain scene consisting of three mountains and asked what an
observer would see from a vantage point different from that of the child. This is usually carried
out by placing a doll in different positions and getting the child to describe what the doll would be
able to see (e.g. cows, horses, etc. that are placed on the surface). Typically children under the
age of eight assume that others share their view point and find it impossible to take a
perspective different from their own
The correct answer is: Egocentrism

55
The parallel process in a female child that resembles the Oedipus complex in males is called

Select one:
Euripides complex
Delphi complex

Electra complex
Cleopatra complex
Jocasta complex

143
The Oedipus complex is an intrapsychic and interpersonal event whereby a 3- to 5-year-old
male child develops rivalry with this father and an attraction to his mother. A similar process in a
female child is called the Electra complex.
The correct answer is: Electra complex

56
A 55-year-old married lady is a mother of three girls who were all raised by her. All are now
grown up and her last daughter left home to study recently. Now the mother feels lonely and
despondent. Which one of the following terms describes this phenomenon?

Select one:
Depression
Empty nest distress
Downshifting
Mid life crisis
Separation anxiety

Empty nest distress or empty nest syndrome refers to the feeling of loneliness when children
leave home which usually occurs during latter part of middle age. However, there is no evidence
to support this mythical concept. The parenting role continues to be important, albeit in a
changed form. Midlife transition or crisis occurs around age 40 to 45, when individuals become
acutely aware of their limitations with their previous goals and re-evaluating their life.
Downshifting refers to voluntary opting out of pressurised career and giving up well-paid job for
more fulfilling life (anti-urbanism).
The correct answer is: Empty nest distress

57
During an Adult Attachment Interview, the subject provides an emotional and long discussion of
childhood memories and is often distressed and tearful. Based on this finding, identify the
category of adult attachment:

Select one:
Autonomous
Autocratic

Entangled
Dismissing
Disorganised

144
In Main's adult attachment interview (AAI), those who had secure attachment provide
spontaneous and coherent answers in a non-defensive manner with sufficient elaboration and
able to talk freely about positive or negative experiences in childhood. Those who had an
avoidant (insecure) pattern often minimise their experiences, often gives brief answers 'I don't
remember statements', do not elaborate on them and do not use colourful metaphors during the
discourse (classified as Dismissing of experiences). The entangled/ pre-occupied group
includes those who had insecure but ambivalent (enmeshed) attachment use multiple
emotionally laden responses and ramble excessively. The subject shows an outpouring of
emotion with a lengthy discussion of childhood memories and is often distressed, angry and
tearful. Broken continuity and interrupted logical flow of thoughts are seen in those who had
insecure disorganised attachment pattern. Here the subject becomes incoherent or irrational
during the interview.
The correct answer is: Entangled

58
Which one among the following is said to be achieved when all information properly fits into
the schemas?

Select one:
Assimilation
Accommodation
Equilibration
Adaptation
Object permanence

Equilibration is a term used in the Piaget's cognitive developmental theory to describe child's
movement between a state of equilibrium, when they are mostly assimilating, to a state of
disequilibrium, when they are mostly accommodating. It is said to be achieved when all
information properly fits into the schema via either processes of adaptation (assimilation and
accommodation). During each developmental state, the child will experience cognitive
disequilibrium which gets solved through adaptation and equilibration results. Each time that
equilibrium occurs, the child produces more effective schemata or mental structures.
The correct answer is: Equilibration

59
Which one of the following fears is not characteristic of infancy?

Select one:
Fear of strange people.
Separation anxiety

Fear of the dark

145
Fear of falling
Fear of loud noises

Fears characteristic of infancy include separation anxiety, fear of loud noises, fear of falling,
fear of strange objects and strange people.
The correct answer is: Fear of the dark

60
According to Thomas and Chess study, all of the following temperamental characteristics are
seen in slow to warm up children except

Select one:
Slow changes in mood
Mild intensity of emotions
Negative response to new stimuli

Fiery emotionality
Gradual adaptations

Fiery emotionality is a feature of difficult children. Three behavioural or temperamental styles


were identified i children by Thomas & Chess study
1. Easy - rhythmic pattern of needs, positive mood, adapts well with a positive approach
to novelty, low intensity of emotions and active - seen in 40%
2. Difficult - Irregular biorhythms, negative mood, less predictable, uncomfortable with
new experiences, react intensely to stimuli (Fiery emotionality), difficult to comfort - seen
in 10%
3. slow to warm up children- slow changes in mood, somewhat regular habits, gradual
adaptations, mild intensity of emotions, adapts poorly to change, but less active - seen in
15% (often labelled as 'shy').

The correct answer is: Fiery emotionality

61
The ability to formulate hypothesis and carry out deductive reasoning is a feature of

Select one:
Concrete operational stage

146
Sensorimotor stage
Formal operational stage
None of the above

Preoperational stage

Formal operational stage: At this stage the child can think logically about abstract prepositions
and test hypothesis systematically. The child can consider a hypothetical future and analyse
various ideologies.
The correct answer is: Formal operational stage

62
Children adopted before what age do well in terms of attachment patterns?

Select one:
One or two years

Eight years
Four or five years
Sixteen years
Infancy

Research shows that the earlier the age of adoption, the better is the outcome for the child.
When a child is adopted during early childhood, then the chances of forming new attachments
are better. Therefore, early adoption is recommended as a matter of social policy. Children
adopted before the age of 4 or 5 have been shown to do well generally. Although late adoption
after the age of eight, does not necessarily lead to problems in adjustment, such children are
more vulnerable and are at risk of developing future problems, like behavioural problems at
home and school (Tizard and Hodges 1978).
The correct answer is: Four or five years

63
A 3-year-old boy prefers playing with toy cars and motorbikes. He appears offended when
someone mistakes him for a girl. Which of the following capacity has developed in this child?

Select one:
Gender identity
Gender role

147
Theory of mind
Sexual orientation
Gender awareness

By 21/2 years of age, children can identify themselves as male or female and recognise
others as male or female. This is called gender identity. Theory of the mind refers to the
awareness that others have cognitive processes and mental status similar to their own; with
this awareness a developing child tries to imagine what others might be thinking.
The correct answer is: Gender identity

64
Which one of the following statements about parental loss is incorrect?

Select one:
Parental death has somewhat lesser impact than parental conflict and
separation
Higher rates of later life depression are seen in these children

Providing day care for more than 4 months at less than 1 year age for
>20hrs a week can increase insecure attachment.
Girls are affected more than boys due to parental divorce
Children who did not adapt well to may result in poor academic
achievement and low self esteem

Most children adapt well to parental divorce if financial support, reasonable contact with non-
custodial parent and successful remarriage of single parent take place. If not, poor academic
achievement, low self-esteem, 2-3 times more antisocial behaviour and higher rates of later life
depression are seen. Boys are more affected than girls due to parental divorce; parental death
has a somewhat lesser impact than parental conflict and separation. Providing day care for
more than 4 months at less than 1 year age for and >20hrs a week can increase insecure
attachment. If not, day care does not affect development adversely
The correct answer is: Girls are affected more than boys due to parental divorce

65
According to Kohlberg, which stage is characterised by obedience in order to conform to the
expectations of the society?

Select one:

148
Social contract orientation of post-conventional morality
Good boy/good girl orientation of conventional morality
Preconventional morality

Ethical principle orientation of post-conventional morality


Authority orientation of conventional morality

The interpersonal concordance or 'good boy-nice girl' orientation is seen at conventional stage
of moral development. Good behaviour is what pleases or helps others and is approved by
them. There is much conformity to stereotypical images of what is majority or 'natural' behaviour
approved by adults at this stage. Behaviour is frequently judged by intention -- 'he means well'
becomes important for the first time. One earns approval by being 'nice'.
The correct answer is: Good boy/good girl orientation of conventional morality

66
Which term was used by Thomas and Chess to describe the reciprocal relationship between a
baby's temperament and its social environment?

Select one:
Regularity
Consonance
Rhythmicity
Goodness of fit
Adaptability

Goodness of fit (Thomas and Chess) describes the reciprocal relationship between a baby's
temperament and its social environment whereby a good match between the both results in
positive development later. Chess and Thomas used the term especially to refer to the
harmonious interaction between a mother and a child. Goodness of fit results when the
opportunities, demands and expectations of the parents and others are in consonance with the
child's temperament and other abilities. On the other hand, 'poorness of fit' arises out of a lack of
consistency between environmental demands and the child's temperamental and other
characteristics.
The correct answer is: Goodness of fit

67
Which one among the following emotions arises around three years of age?

149
Select one:
Disgust
Fearfulness
Sadness

Surprise
Guilt

The primary emotions of joy, sadness, anger, fearfulness, disgust, surprise are present within
the first six months of life and develop over the next three years. However, self-conscious
emotions like jealousy, shame, guilt and empathy arise later around three years. These
emotions require a sense of self, a capacity that develops after 24 months
The correct answer is: Guilt

68
Family instability affects the development of children at home. Which of the following is correct
in this regard?

Select one:
Boys are less affected than girls
Hyperactive children are more affected than easy-going children
There is no effect on cognitive development
Younger children are less affected than older children

Psychosis is the most common effect

Family instability can affect children to various extent depending on sex (boys affected more
than girls), age (younger affected more than older children), and temperament hyperactive
affected more than placid). This has a demonstrable effect on a child's cognitive
achievements; the most common psychopathology noted is a behavioural difficulty.
The correct answer is: Hyperactive children are more affected than easy-going children

69
A teenager is worried about joining the 'all-boys band' group in his school. According to Erik
Erikson, he is at what phase of psychosocial development stage?

Select one:
Identity vs Role confusion

150
Industry vs inferiority
Intimacy vs isolation
Generativity vs stagnation

Initiative vs guilt

Identity vs Role confusion- Adolescence. This stage occurs during adolescence between the
ages of approximately 12 to 18. Up to this stage, according to Erikson, development mostly
depends upon what is done to us. From here on out, development depends primarily upon what
we do. Teens need to develop a sense of self and personal identity. During adolescence,
children are exploring their independence and developing a sense of self. As they make the
transition from childhood to adulthood, teens may begin to feel confused or insecure about
themselves and how they fit into society. As they seek to establish a sense of self, teens may
experiment with different roles, activities and behaviours. According to Erikson, this is important
to the process of forming a strong identity and developing a sense of direction in life.

The correct answer is: Identity vs Role confusion

70
An animal behaviour in which an animal will follow the first moving object that it sees shortly
after birth is called

Select one:
Cocooning

Fixating
Sojourning
Bonding
Imprinting

Imprinting was described by Konrad Lorenz.


The correct answer is: Imprinting

71
Which of the following is NOT an effect of parental separation?

Select one:
Changes in bowel habits

151
Physical aggression
Increased motivation for academic performance
Suicidal ideas

Guilt and accepting responsibility for separation

Children of all age groups are prone to behavioural difficulties after parental divorce -this is
evident even in infants who may show changes in eating, sleeping and bowel patterns, with
fearful or anxious responses. Academic and social aptitude suffers due to divorce; asthma,
injuries, headaches and speech defects are more common in divorced families.
The correct answer is: Increased motivation for academic performance

72
A 7-year-old boy is struggling at school for grades. According to Erik Erikson, he's at what
phase of psychosocial development stage?

Select one:
Initiative vs. guilt

Basic trust vs. mistrust


Autonomy vs. shame
Industry vs. inferiority
Identity vs. Role confusion

Industry vs. inferiority (6-12 years). At the school-going stage, the child's world extends
beyond the home to the school. The emphasis is on academic performance. There is a
movement from play to work. Earlier the child could play at activities with little or no attention
given to the quality of results. Now, he needs to perform and produce good results
The correct answer is: Industry vs. inferiority

73
Negative transition and risk taking behavior are generally less common in societies that mark
transition to adulthood by;

Select one:
Community ceremonies
Final ceremonies

152
Victory ceremonies
Initiation ceremonies
Termination ceremonies

Initiation is a rite of passage ceremony marking entrance or acceptance into a group or society.
It could also be a formal admission to adulthood in a community or one of its formal
components. Risk-taking behavior and negative transition are generally less common in such
societies.
The correct answer is: Initiation ceremonies

74
In Ainsworth strange situation experiment, the child shows no preference between mother and a
complete stranger. Which type of attachment behavior is this?

Select one:
Disorganized attachment behaviour
Securely attachment behaviour

Insecurely attached anxious avoidant behaviour


Insecurely attached anxious resistant behaviour
Insecurely attached anxious-ambivalent behaviour

Children with avoidant attachment styles tend to avoid parents and caregivers. This avoidance
often becomes especially pronounced after a period of absence. These children might not
reject attention from a parent, but neither do they seek out comfort or contact. Children with an
avoidant attachment show no preference between a parent and a complete stranger.

The correct answer is: Insecurely attached anxious avoidant behaviour

75
Which one among the following was the last stage of Erik Erikson's model of psychosocial
development?

Select one:
Initiative vs. guilt
Identity vs. identity confusion
Integrity vs. despair

153
Intimacy vs. isolation
Industry vs. inferiority

Erik Erikson developed an alternative model of psychosocial development based on the crisis at
each developmental stage.The different stages of development start with a Trust vs. mistrust
phase at infancy and ends with Integrity vs. Despair phase at the old age.
The correct answer is: Integrity vs. despair

76
Which one among the following concepts is not correctly linked with its proponent?

Select one:
Thanatos - Sigmund Freud.
Malignant Alienation - Morgan and Watts
Splitting -Melanie Klein

Sibling Rivalry -Alfred Adler


Introversion and Extroversion - Winnicott

Carl Gustav Jung coined the concept of Introversion and extroversion. Jung is the founder of
the school of analytical psychology.
The correct answer is: Introversion and Extroversion - Winnicott

77
Which one of the following is not a feature of 'ex-institutional syndrome'?

Select one:
They were less likely to be selective in choosing their friends
They turned to peers less often for emotional support
These young people related better to adults than to their peers It
is defined in adopted children at the age of eight
They were less likely to have a special friend

154
Tizard and Hodges followed up a group of children who had been in institutions from infancy,
adopted at age 4 and had been looked after by a number of carers who often changed . At age
8, most children had formed a reasonably good attachment with their adoptive parents. At age
16, although the adolescents appeared to be functioning rather well, they showed a
constellation of features termed as an Ex-institutional syndrome. These young people related
better to adults than to their peers, were less likely to have a special friend, were less likely to be
selective in choosing their friends and turned to peers less often for emotional support.
The correct answer is: It is defined in adopted children at the age of eight

78
Which one of the following is not considered to be a Neo-Freudian psychologist?

Select one:
Jean Piaget
Karen Horney
Erich Fromm

Alfred Adler
Harry Stack Sullivan.

Neo-Freudian psychologists were thinkers who agreed with the basis of Freud's
psychoanalytic theory but changed and adapted the theory to incorporate their own beliefs,
ideas and theories. Piaget is not usually considered as a Freudian theorist.
The correct answer is: Jean Piaget

79
Superego is formed at which stage of psychosexual development?

Select one:
Phallic stage
Anal stage
Latency stage
Oral stage
Genital stage

155
The latency period is the stage of suspension of psycho-sexual development between the age of
five and six and puberty. During this period, sexual activity and interest tend to decrease, a
consequence of repression, secondary identifications and the establishing of the superego,
resulting in the resolution or the waning of the Oedipus complex. At about five years of age
during latency stage, the Superego appears, following the end of the oedipal stage. (Hall,
1954:35)
The correct answer is: Latency stage

80
A 7-year-old Iranian boy had to flee from Iran as an asylum seeker together with his mother
who was politically threatened. Upon arrival to UK, he starts sucking his thumb and exhibits
intense anxiety that something terrible might happen to his mother. He refuses to go to local
school but plays with his peer group in the proximity of his mother. In the above scenario, the
psychosexual developmental stage of the child is most likely to be

Select one:
Anal stage
Oedipal stage
Genital stage
Oral stage
Latent stage

At age 7, the latent stage is more likely to be present.


The correct answer is: Latent stage

81
Which of the following cognitive functions is unlikely to be affected secondary to working
memory dysfunction?

Select one:
Error detection
Long-term recall

Executive function
Decision-making
Anterograde episodic memory

156
Executive function and its key components such as error correction are dependent on working
memory.
The correct answer is: Long-term recall

82
Who was the most important person in the following list to be associated with the object
relations theory?

Select one:
Alfred Adler
Carl Gustav Jung
Sigmund Freud
Anna Freud
Melanie Klein

Klein, along with Sigmund Freud and W.R.D. Fairbairn, contributed ideas to make up what we
now know as object relations. First Freud introduced the idea of object choice, which referred to
a child's earliest relationships with his caretakers. Such people were objects of his needs and
desires. The relationship with them became internalized mental representations. Subsequently
Melanie Klein coined the term part objects, for example the mother's breast, which played an
important role in early development and later in psychic disturbances, such as excessive
preoccupation with certain body parts or aspects of a person as opposed to the whole person.
Finally, Fairbairn and others developed the so-called object relations theory.
The correct answer is: Melanie Klein

83
A child initially forms an attachment to a single caregiver, usually the mother. What term was
used by Bowlby to describe this tendency?

Select one:
Indiscriminate attachment
Secure base effect

Mothering
Monotropy
Clear-cut attachment

157
According to Bowlby, the strong innate tendency to attach to one adult female is called
monotropy. This attachment is qualitatively different from later attachments made. Even when
several carers are present, the infants appear to show a clear preference for the primary
caregiver. Attachment process itself is more important than who the attachment figure is. The
'mothering' could be performed by a male or a female. The primary caregiver may be the
mother or father or anyone who provided the greatest interaction with the child.
The correct answer is: Monotropy

84
Which of the following factors protects against child abuse in families?

Select one:
Personality disorder in parents
Young age of parents
History of childhood maltreatment in parents
Network of relatives
High expectation of parents

Protective factors against child abuse include having emotionally satisfying relationships with a
network of relatives or friends. Parents who were abused as children are less likely to abuse
their own children if they have resolved internal conflicts and pain related to their history of
abuse and if they have an intact, stable, supportive, and nonabusive relationship with their
partner.
The correct answer is: Network of relatives

85
How many categories of temperament were identified by Thomas and Chess in New York
Longitudinal study?

Select one:
Three
Nine

Eight
Twelve
Six

158
New York Longitudinal Study was conducted by Thomas and Chess. It was a thirty years
(initially six years) longitudinal study of 138 children, observing childhood temperaments during
their infancy and followed up at 5, 18 and 22 years. The authors employed parental interviews
to ascertain temperamental dimensions ­ 9 such dimensions have been used; Activity level,
Rhythmicity, approach/withdrawal, adaptability, Intensity of reactions, Threshold for
responsiveness, quality of mood, distractibility and attention span / persistence.
The correct answer is: Nine

86
All of the following terms are associated with Donald Winnicott except

Select one:
Potential space
Transitional object
Object permanence
Holding environment
Pathological mother

Object permanence was described by Piaget. All other terms are associated with Winnicott.
The correct answer is: Object permanence

87
Insecure attachment during early childhood is associated with the development of which types of
childhood disorders?

Select one:
Childhood schizophrenia
Mental retardation

Heller's syndrome
Tic disorders
Oppositional defiant disorder

Secure attachment appears to be a protective factor for development of childhood disorders


and insecure attachment is best conceptualised as a risk factor for a number of childhood
disorders. It has been demonstrated in various studies that insecure attachment during early
childhood is associated with the development of behavioural problems especially

159
oppositional defiant disorder at school age. Insecure attachment in combination with other
vulnerability factors such as family dysfunction, difficult child temperament, and poor parental
management can give rise to later childhood disorders.
The correct answer is: Oppositional defiant disorder

88
In a family, it is observed that the parents do not set strict limits for their children. They avoid
confrontation with their children whenever possible, and are considered to be extremely
flexible by their children. This parenting style is called

Select one:
Insecure parenting
Authoritative parenting
Permissive parenting
Neglectful parenting
Avoidant parenting

This is called as permissive parenting with low or absent control but high degree of
nurturance and leniency; No demands are placed on growing child. There is a degree of
inadequate communication in these families.
The correct answer is: Permissive parenting

89
According to Freud's psychosexual theory, Oedipus complex develops during;

Select one:
Oral stage
Genital stage

Phallic stage
Anal stage
Latent stage

According to Freud's psychosexual stages, Oedipus complex develops during phallic stage (3-5
years). Oedipus complex is a wish to have a libidinal relationship with opposite sex parent
(Electra complex in girls) with desire to exclude the rival parent. This leads to a fear of retaliation
from the rival parent in the form of castration anxiety in boys. At the resolution of this stage,
identification with the aggressor i.e. boys identifying with dads is seen.

160
The correct answer is: Phallic stage

90
Castration anxiety is associated with which stage of psychosexual development?

Select one:
Latency stage
Oral stage
Genital stage
Anal stage
Phallic stage

Phallic/oedipal stage is seen between 3 and five years of age during which the child
increasingly focuses on genitalia and libido is directed towards others. His stage involves
castration anxiety and Oedipus complex in males and penis envy and Electra complex in
females. The Oedipal complex refers to the intense attachment to the parents of the opposite
sex along with a wish to destroy the parents of the same sex. Resolution of the Oedipal
complex involves identification with the same sex parent (i.e. the son identifying with the
father).
The correct answer is: Phallic stage

91
A boy believes in obeying rules laid down by the teacher as the primary moral principle to
avoid punishments. Which stage of moral development is he exhibiting?

Select one:
Pre-conventional stage
Post-conventional stage

None of the above


Conventional stage
Automatic obedience

Punishment or obedience orientation is the primary stage of moral development according to


Kohlberg. This is seen in the preconventional stage.The correct answer is: Pre-conventional

161
stage

162
92
Egocentrism is a feature of which of the following stages described by Piaget?

Select one:
Formal operational stage
None of the above
Concrete operational stage

Preoperational stage
Sensorimotor stage

During Preoperational stage (2-7years) the baby learns to use language and to represent
objects by images and words. Thinking is egocentric (only self-perspective is taken) and
animistic. Objects are classified by a single feature. The baby also achieves conservation of
numbers (age 6).
The correct answer is: Preoperational stage

93
Fear of animals comes to prominence in which of the following developmental stages?

Select one:
Later childhood
Infancy
Early childhood
Adult life
Preschool children

Fear of animals peaks by age 3; Fear of the dark­age 4 or 5;Fear of imaginary creatures­5
plus;Fear of open spaces arise in later childhood or adult life; Fears that arise in late childhood
or adult life-fear of sex/open spaces. Teenage onwards fears of failure, illness and death
emerge. Fears that show no particular age trend include fear of snakes or storms. These are
normal developmental fears and not necessarily phobias.
The correct answer is: Preschool children

94
Failure to develop attachment can lead to affectionless psychopathy and developmental
retardation according to Rutter. The term used to describe this adverse outcome is

163
Select one:
Privation
Deprivation
Anaclitic depression

Enmeshed attachment
Detachment

Rutter distinguished deprivation from privation. Deprivation the attachment is formed but lost
temporarily. If it is for a short time then protest, despair, detachment phases (similar to grief) are
seen. This is more common in 8m to 3 yr old. Boys show more deprivation features than girls. It
is more noticeable if aggressive care giving e.g. physical abuse was present before separation.
In prolonged deprivation, separation anxiety sets in. Increased clingy behaviour, psychosomatic
complaints, vacillation and aggression are seen in the child. Privation refers to the non-formation
of attachment; this is very rare and can lead to what Rutter termed as affectionless psychopathy
and developmental retardation. Attention seeking, lack of guilt, antisocial behaviour and
indiscriminate attachment patterns are noted. This is reversible only to some extent.

The correct answer is: Privation

95
Following separation from the mother, the first reaction exhibited by a child is most likely to be

Select one:
Despair
Protest
Clinging behaviour
Anaclitic depression
Detachment

Acute separation reaction occurs when a child is separated from their attachment figure and is
sometimes called as Anaclitic depression. It progresses through classic stages of protest,
despair and detachment.
The correct answer is: Protest

96

164
An 18-month-old child is aware of her vulnerability to separation from her mother. She is
constantly concerned about her mother's location. In terms of Mahler's separation-
individuation theory, this infant is in the stage of

Select one:
object constancy
differentiation
practicing

rapprochement
symbiosis

Rapprochement is arguably the most critical stage of the separation individuation process. The
toddler becomes acutely aware of his separation from the mother. As the symbiotic safety net
diminishes, his separation anxiety gains momentum. The toddler is more concerned with
mother's whereabouts and demonstrates his anxiety through active approach behavior
(rapprochement).
The correct answer is: rapprochement

97
Kubler-Ross described stages of coping in those facing impending death. Which of the
following is NOT a stage included in this description?

Select one:
Anger
Bargaining
Acceptance
Depression
Reaction formation

Elizabeth Kubler-Ross from Switzerland wrote a book called 'On Death and Dying' which
included a cycle of emotional states that is often referred to as the Grief Cycle.This description
did not include reaction formation.
The correct answer is: Reaction formation

98

165
A 4-year-old girl is often noted to talk to an imaginary friend that no one else can see. Her
parents are concerned and bring her to the psychiatric clinic. The best possible course of
action is

Select one:
Schedule the child for detailed psychiatric evaluation
Examine for signs of physical abuse
Notify social services

Reassure the parents about age-related behaviour


Explore history of sexual abuse

Having imaginary companions is a normal phase of development. This may lead to self-talk
behaviour in preschool children.
The correct answer is: Reassure the parents about age-related behaviour

99
A 7-year-old Iranian boy had to flee from Iran as an asylum seeker together with his mother who
was politically threatened. Upon arrival to UK, he starts sucking his thumb and exhibits intense
anxiety that something terrible might happen to his mother. He refuses to go to local school but
plays with his peer group in the proximity of his mother. The above reaction can be considered
as

Select one:
Denial
Regression
Displacement
Sublimation
Modelling

Regression is temporary fall back to previous developmental stage


The correct answer is: Regression

100
The basis of object relations theory is that the primary motivational drive of an individual is to
seek

166
Select one:
Pleasure
Satisfaction
Relationships

Money
Self actualisation

Object relations theory describes with how a growing baby develops relationships with others.
Gratification is thought to be obtained through relationships as well as through satisfaction of
desires.
The correct answer is: Relationships

101
'This is an active process in which painful thoughts and memories are pushed to the
inaccessible corners of the mind'. This statement refers to

Select one:
False memory syndrome
Regression
Re-integration
Retrieval failure
Repression

Sigmund Freud described the concept of repression. Here people unconsciously repress
painful or disturbing memories and is normally taken to mean motivated forgetting.
The correct answer is: Repression

102
Which one of the following is a feature of restricted language code?

Select one:
It is used by the middle class and upper-class children
Restricted language code is characterised by short, incomplete sentences,
which focuses on the present

167
I is frequently employed in the restricted language code
It is characterized by longer, complex sentences that are context-
independent
It focuses on the past and future, employs 'I' commonly and allows for
expression of abstract thought.

The elaborated code is characterized by longer, complex sentences that are context-
independent. It focuses on the past and future, employs 'I' commonly and allows for the
expression of abstract thought. Restricted language code is characterised by short,
incomplete sentences, which tend to be context-dependent, frequently uses like 'you know',
focuses on the present, employs pronoun I rarely and has little room for expressing abstract
thinking. Restricted code is used by the poorer class and the elaborated language code is
used by the middle class and upper-class children.
The correct answer is: Restricted language code is characterised by short, incomplete
sentences, which focuses on the present

103
Which category of play starts around five years of age?

Select one:
Rule- governed play
Symbolic play

Sensorimotor play
Co-operative play
Solitary play

Children's play activities are categorised according to the quality of interactions among each
other.
Solitary play- the child plays on its own;
Parallel play­the child plays along with other children (2 years);
Co-operative play-This is usually evident by 3 years and the child interacts with other
children in complimentary ways like sharing, turn taking, etc ;
Rule- governed play - It starts around five years and the children understand the rules of
games.

The correct answer is: Rule- governed play

168
104
The most common type of attachment behaviour in the Western world is

Select one:
Absence of attachment
Secure
Disorganised

Anxious avoidant
Anxious resistant

Most children are securely attached.


The correct answer is: Secure

105
The scoring for adult attachment interview developed by Mary Main is based on all except

Select one:
The subject's ability to give a coherent account
Meaning as well as language used
The contents of the narratives provided by the interview
Security of the attachment of the subject to any specific person
The subject's ability to give an integrated account of the parenting
experience

The scoring for adult attachment interview (AAI) developed by Mary Main is based on the
contents of the narratives provided by the interview, the subject's ability to give a coherent
account of past experiences, the subject's ability to give an integrated account of the parenting
experience and the meaning as well as the language used by the subject. AAI assesses the
individual's state of mind with respect to attachment and dose not measure the security of the
attachment of the subject to any specific person.
The correct answer is: Security of the attachment of the subject to any specific person

106
A 4-year-old child draws a tall matchstick figure representing her dad, and a short matchstick
figure representing herself walking beside him. Which of the following features is she
exhibiting?

169
Select one:
Animism
Conservation
Lack of conservation

Semiotic function
Egocentricity

Semiotic function refers to using objects or drawings to represent something else. These
objects are called signifiers.
The correct answer is: Semiotic function

107
Which one among the following ability has been demonstrated by the Rouge Test?

Select one:
Depth perception
Self-esteem
Height perception
Sense of self and self recognition
Object permanence

In Rouge test, a spot of rouge is placed on the infant's nose, and he/she is then placed in front of
a mirror. Up to the age of 24 months, children do not show self-recognition, and they appear to
view the images as if it was another child. But about the age of two years, a dramatic change
seems to occur. Most 24-month-old children reach for their nose, which has been demonstrated
in different cultures.
The correct answer is: Sense of self and self recognition

108
According to Jean Piaget's cognitive development theory, primitive self-recognition begins
during

Select one:
Pre operational stage
Pre conceptual stage

170
Sensorimotor stage
Concrete operational stage
Formal operational stage

The other features achieved during sensorimotor stage would include goal-directed
behaviour, object permanence, symbolic thought, representational play, and deferred
imitation.
The correct answer is: Sensorimotor stage

109
Identify the incorrect statement about the EAS dimensions of temperament.

Select one:
Predict adult personality traits
It is genetically grounded

It is highly inheritable
Show less stability over developmental periods
They are of evolutionary importance

Buss and Plomin proposed that the dimensions of temperament show high stability over
developmental periods. With increasing age, they become differentiated and also predict
adult personality traits.
The correct answer is: Show less stability over developmental periods

110
The baby exhibits its first social smile at about

Select one:
Four months of age
Six weeks.
Six months of age
Three months of age
Three weeks

171
The baby exhibits its first social smile at about six weeks. The infant starts to differentiate
familiar and unfamiliar faces at three months.
The correct answer is: Six weeks.

111
Which of the following theories attempt to explain gender differences during development?

Select one:
Stimulus preparedness theory
Social learning theory
Attachment theory
Social exchange theory
Kohlberg's theory

Social learning theory regards gender identity and role as a set of behaviours that are learned
from the environment. The main way that gender behaviours are learned is through the process
of observational learning. Children observe the people around them behaving in various ways,
some of which relate to gender. They pay attention to some of these people (models) and
encode their behaviour. At a later time, they may imitate the behaviour they have observed.
They may do this regardless of whether the behaviour is 'gender appropriate' or not but there
are a number of processes that make it more likely that a child will reproduce the behaviour that
its society deems appropriate for its sex. (Retrieved from
http://www.psychlotron.org.uk/newResources/developmental/AS_AQB_gender_SLTBasics.pdf)
The correct answer is: Social learning theory

112
'Ageist society reduces the social interaction that older adults can have; withdrawal is not
mutual, but forced'. This statement refers to which one of the following ageing theories of role
change?

Select one:
Social exchange theory
Mid-life crisis theory

Socio-emotional selectivity theory


Social disengagement theory
Social reengagement theory

172
1.Social disengagement theory: Mutual withdrawal of society and the individual occurs;
increased individuality and shrinking life space are inevitable moves towards death.
2.Social (non) reengagement theory (aka activity loss theory): Ageist society reduces the
social interaction that older adults can have; withdrawal is not mutual, but forced. 3.Social
exchange theory: Age robs people off the ability to engage in reciprocal roles; retirement is
a special social contract where productivity is exchanged for increased leisure and
decreased responsibilities.
4.Socio-emotional selectivity theory: Wise investment of social energy in old age is to limit
social interaction to those who are most familiar

The correct answer is: Social reengagement theory

113
In Harlow's rhesus monkey studies, orphaned baby monkeys show a preference for clinging to
a

Select one:
Wire mesh surrogate mother that does not dispenses milk
Wire mesh surrogate mother irrespective of feeding
Wire mesh surrogate mother that dispenses milk
Soft cloth surrogate mother that provides food
Soft cloth surrogate mother irrespective of feeding

Harlow's experiments: These experiments established the importance of contact comfort as


basic as need for food in developing mother-infant bonding. Harlow separated rhesus
monkeys from their mothers during their first weeks of life. Harlow substituted a surrogate
mother made from wire or cloth for the real mother. The infants preferred the cloth-covered
surrogate mother, which provided contact comfort, to the wire-covered surrogate, which
provided no contact comfort. This preference was observed irrespective of feeding, i.e. the
terry-cloth soft-surrogate mother was preferred even if it did not have a feeding nipple
attached to it.
The correct answer is: Soft cloth surrogate mother irrespective of feeding

114
A woman lost her husband one month ago. She is feeling low in mood with poor sleep. She
also complains of a poor appetite. She hears the voice of her husband at night and feels his
presence at home. She reports 'longing for some more days with him'. She is most likely to be
experiencing;

173
Select one:
Adjustment reaction
Stage 3 of bereavement
Delayed grief

Stage 1 of bereavement
Stage 2 of bereavement

In the few hours or days following the death of a close relative or friend, most people feel simply
stunned, as though they cannot believe it has actually happened. This sense of emotional
numbness can be a help in getting through all the important practical arrangements that have to
be made, such as getting in touch with relatives and organising the funeral. However, this feeling
of unreality may become a problem if it goes on too long. Seeing the body of the dead person
may, for some, be an important way of beginning to overcome this. Soon though, this numbness
disappears and may be replaced by a dreadful sense of agitation, of pining or yearning for the
dead person. There is a feeling of wanting somehow to find them, even though this is clearly
impossible. This makes it difficult to relax or concentrate, and it may be difficult to sleep properly.
Dreams can be very upsetting. This stage is the second stage of bereavement.
(www.rcpsych.ac.uk)
The correct answer is: Stage 2 of bereavement

115
A 4-month-old child is aware of his mother, but has no sense of individuality. This is seen in
which of the following stages described by Mahler?

Select one:
Idealisation phase
Rapprochment phase
Autistic phase
Separation-individuation phase
Symbiotic phase

According to Margaret Mahler's theory of the mother/child relationship, the symbiotic relation is a
very early phase of development that follows the phase of normal primary autism and precedes
the separation/individualization phase. The symbiotic relation is characterized by an omnipotent
sense of the total enmeshing of mother and child, who thus form a 'unity of two.'
The correct answer is: Symbiotic phase

174
116
A child was observed to be speaking to his grandfather over a toy phone. What kind of play is
this?

Select one:
Parallel play
Co-operative play
Symbolic play

Rule governed play


Sensorimotor play

In sensorimotor play, the infant plays with objects like shaking a rattle, which occurs around six
months. Symbolic or pretend play is evident by two years. Here one object is understood to
stand for another (e.g. a piece of pencil stands for a cigarette) or pretending something exists
where it does not (e.g. speaking to grandfather over a toy phone).
The correct answer is: Symbolic play

117
An 18-months-old boy shows a mug to his mother and says 'Mummy cup'. Which stage of
language development is he at?

Select one:
Critical period
Transformational grammar
Telegraphic period
Babbling
Prelinguistic stage

This is called as two words / telegraphic speech stage. In telegraphic speech, meaningful
words are used without connecting words such as propositions or conjunctions.
The correct answer is: Telegraphic period

118
Which one among the following terms refers to basic predispositions that underlie children's
behaviour?

175
Select one:
Temperament
Adaptability
Threshold of responsiveness

Goodness of fit
Attachment

This is an aspect of personality studied in infants. It describes individual differences in


behavioural style. Temperament in childhood is an important psychological attribute distinct from
such characteristics as cognitive abilities and motivation. Thomas and Chess conceptualise
temperament as representing the behavioural style­ the ‘how' of behaviour. Not all but certain
aspects of temperaments remain stable over many years. Infant's negative emotionality (fear,
etc.), and reactions to new situations (inhibition or neophobia) are the two most stable
temperaments.
The correct answer is: Temperament

119
What are the three components of psychic functioning defined in Freud's structural theory?

Select one:
The oral, anal and oedipal
The conscious, superconscious and subconscious
The id, ego, and superego.
The conscious, unconscious and subconscious
The Eros, Thanatos and libido

Freud developed the idea of distinguishing mental events on the basis of their accessibility to
conscious awareness. This early, 'topographic,' theory of the mind was elaborated in 1900 in
his book Interpretation of Dreams. With 'The Ego and the Id', (1923) Freud subsumed the
topographic concepts Conscious, Preconscious, and Unconscious into the 'structural' concepts
Ego, Id, and Superego.
The correct answer is: The id, ego, and superego.

120
Choose the correct statement about Bowlby's attachment theory?

Select one:

176
Attachment behaviour will cease if there is frequent punishment from
attachment figure
Attachment behaviour ceases by puberty
According to attachment theory, multiple attachments cannot occur

The primary attachment figure is not necessarily the mother


Attachment behaviour is least evident when the infant is stressed

Attachment begins in infancy and lasts throughout a lifetime. A newborn baby immediately
needs someone to take care of them. This person may be a parent, a sibling, or a nanny, but
whoever it is, there will be a bond formed between them. Bowlby and Ainsworth both believed
that this primary caregiver is the one that will most shape the child's personality and character.
The primary caregiver is usually the mother (but need not always be) and strong bonds are
formed within minutes of giving birth. It is important for the new parents and baby to be alone
together right after the birth to establish a strong bond. If there are too many individuals in the
room right after birth, the natural process of attachment can be disrupted and this can have
long-term effects on the relationship between the child and parents (Klaus, Kennell, and Klaus,
1995).
The correct answer is: The primary attachment figure is not necessarily the mother

121
A mother of an autistic child wants to know why her son lacks empathy for other children.
Which of the following could be used to explain the fundamental defect?

Select one:
Social Exchange Theory
Attribution Error
Theory of mind
Face Recognition Error
Learning Theory

Theory of Mind (TOM) is a specific cognitive ability to understand others as intentional agents,
that is, to interpret their minds in terms of theoretical concepts of intentional states such as
beliefs and desires.
The correct answer is: Theory of mind

122
Which of the following is correct with respect to transitional objects described by Winnicott?

177
Select one:
They reduce anxiety at times of stress
They are most important to the child during the age 6 to 9 months
Systematic parental interventions are needed to wean from transitional
objects
Children never change their transitional object
They reduce a child's autonomy

A transitional object, as defined by Winnicott is an item (a piece of cloth, a doll or a toy) that
serves a soothing function for children; the use of these objects are normal in the course of
development, and do not imply maladjustment (Lehmane,1995). It gives tangible comfort at
times of stress. The security objects are generally most important when the child is about two-
and-a-half years old. In addition to providing security, the transitional object allows the child to
show his autonomy. Some children never even have a security object, some jump from one
object to another in short periods of time, and others cling onto one object for years
(Miles,1997).
The correct answer is: They reduce anxiety at times of stress

123
Which of the following models divide the mind into unconscious, preconscious and
conscious?

Select one:
Libidinal
Structural
Topographic
Dream analysis
Drive reduction

Freud developed the idea of distinguishing mental events on the basis of their accessibility to
conscious awareness. This early, 'topographic,' theory of the mind was elaborated in the 1900
Interpretation of Dreams.
The correct answer is: Topographic

124

178
A toddler boy when playing with his mother mentions "cats have four legs, dogs have four
legs. So cats and dogs are the same”. Which one among the following terms describes this
cognitive process?

Select one:
Egocentrism
Animism
Transductive reasoning

Hypothetico deductive reasoning


Artificialism

In transductive reasoning, inferences are made about relationships based on a single attribute.
This may influence the development of animistic thinking, in which inanimate objects are treated
as living objects. An example of animistic thinking is 'things that move are alive, the wind moves-
so it is alive.'
The correct answer is: Transductive reasoning

125
When a mother leaves an infant, the infant can become upset. To compensate and comfort for
this sense of loss, an object with the attributes of the mother is required. Winnicott called these
objects

Select one:
Transcultural objects
Make-shift objects
Transitional Objects
Transformable objects
Traditional objects

Winnicott described transitional objects that are inanimate objects that may be a blanket or
teddy bear that the child uses to provide oral/tactile comfort in dealing with the anxieties inherent
in the task of separation and acceptance of reality and allows a child to become more
independent of the mother.
The correct answer is: Transitional Objects

126
Which of the following stages in moral reasoning may not be achieved by everyone?

179
Select one:
Reward orientation

Obedience orientation

Authority orientation

Universal ethical orientation

Concordance orientation

According to Kohlberg, universal ethical principles may not be achieved by everyone.


The correct answer is: Universal ethical orientation

127
Of all sensory systems, the system least developed at birth is

Select one:
Balance
Touch
Taste
Vision
Smell

Full vision develops by 6 months of age; it is the last of all senses to develop in an infant.
The correct answer is: Vision

128
Who developed the theory of good-enough mothering?

Select one:
Rutter
Charcot
Kanner

Baird
Winnicott

180
Winnicott developed the concept of 'good-enough mothering'. This concept is based on the
understanding that the mother plays a vital role in bringing the world to the infant and offering
empathic anticipation of the infant's needs. If she does these things well enough, the baby will
move towards the development of a healthy sense of self.
The correct answer is: Winnicott

129
According to Vygotsky's developmental theory which one among the following refers to
functions that are not yet fully achieved but are in the process "pipeline”?

Select one:
Zone of proximal development
Zone of terminal development
Zone of intermittent development
Zone of accommodation
Zone of assimilation

According to Vygotsky's theory development is not entirely private or individual-based. A child


acts as an apprentice in social surroundings rather than as a scientist. Parents and teachers
carry out the role of scaffolding (a term introduced by Wood et al. 1976) which serves to
introduce familiarity for tasks, even if those tasks are beyond a child's immediate expertise, so
that the child can develop its own comptence later (collaborative learning). Zone of proximal
development refers to functions which are not yet fully achieved but are in the process 'pipeline'.
While actual achievement measures development after completion retrospectively, the zone of
proximal development has a prospective sense as to what the child will be achieving in the near
future.
The correct answer is: Zone of proximal development

181
 Home HiYield Paper A(1) Development

HiYield Paper A(1)

Started on Saturday, 27 June 2015, 2:36 PM
State Finished
Completed on Saturday, 27 June 2015, 2:36 PM
Time taken 17 secs
Marks 0.00/120.00
Grade 0.00 out of 100.00

Question 1 HiY Development EMI001
Not answered Human development
Which different stages of development below correspond to the appropriate age in the list?
Marked out of 5.00

Flag question
2­6 months
Young adulthood
Margaret Mahler's phase of Old age
symbiosis 1­2 years
3­5 years
0­1 years
Middle adulthood
0­10 years
0­1 years
7­12 years
7­12 years
Concrete operational stage of 6­36 months
Young adulthood
Piaget's cognitive model 0­10 years
Old age
1­2 years
Middle adulthood
2­6 months
7­12 years
3­5 years
6­36 months
6­36 months
Young adulthood
0­10 years
1­2 years
Intimacy vs. isolation stage of Erik Middle adulthood
Erikson's psychosocial development 3­5 years
2­6 months
0­1 years
Old age

Kohler's stage of preconventional

182
morality 0­1 years
Old age
Young adulthood
3­5 years
Middle adulthood
0­10 years
6­36 months
2­6 months
6­36 months
1­2 years
Young adulthood
Phallic stage of Freud's 7­12 years
Middle adulthood
developmental model Old age
7­12 years
3­5 years
2­6 months
0­1 years
Check
1­2 years
0­10 years

The symbiosis phases last from 2­6 months (Margaret Mahler). The concrete operational
stage lasts from 7­11 years. (Jean Piaget). Intimacy vs. isolation stage occurs during young
adulthood. (Erik Erikson). The preconventional morality stage lasts from 0 to 10 years.
(Kohlberg'92s theory of moral development).The phallic stage lasts from 3 to 5 years
(Freudian stages of psychosexual development)
The correct answer is: Margaret Mahler's phase of symbiosis – 2­6 months, Concrete
operational stage of Piaget's cognitive model – 7­12 years, Intimacy vs. isolation stage of Erik
Erikson's psychosocial development – Young adulthood, Kohler's stage of preconventional
morality – 0­10 years, Phallic stage of Freud's developmental model – 3­5 years

Question 2 HiY Development EMI002
Not answered Attachment Behaviour 
For each of the following attachment behaviour, choose the most appropriate age of
Marked out of 6.00
presentation from the list provided;
Flag question

5­7 months of age
First few months
6 to 12 months of age 
Infant orientates visually without
discrimination 7­9 months of age
9­36 months
School age
36­48 months of age
12­ 18 months of age
6 to 12 months of age
5­7 months of age
First few months
9­36 months
Infant shows preferential orientation School age
towards selected persons 12­ 18 months of age
36­48 months of age
7­9 months of age

183
6 to 12 months of age
7­9 months of age
Onset of preferential attachment and 36­48 months of age
stranger anxiety First few months
5­7 months of age
12­ 18 months of age
9­36 months
School age
12­ 18 months of age
7­9 months of age
Formation of goal­directed School age
partnership between child and
First few months
caregiver
5­7 months of age
36­48 months of age
6 to 12 months of age
9­36 months
First few months
7­9 months of age
Lessening of attachment behaviour 12­ 18 months of age
and attachment figure can be
6 to 12 months of age
substituted.
5­7 months of age
9­36 months
36­48 months of age
School age
School age
9­36 months
First few months
6 to 12 months of age
12­ 18 months of age
Attachment behaviour typically peaks 5­7 months of age
during this period 7­9 months of age
36­48 months of age

Check

Phases of attachment:Preattachment phase (birth to 8 or 12 weeks)­ babies orient to their
mothers.
Indiscriminate attachment phase (attachment in making ­ 8 to 12 weeks to 6 months)­ Allows
strangers to handle, infants become attached to one or more persons in the environment. 
Clear­cut attachment (6 through 24 months)­ Preferential attachment, separation anxiety,
object permanence, stranger anxiety. At a later part, weakened stranger anxiety; other
attachment figures may also present.
Attachment behaviour peaks between 12­18 months. After 25 months, the mother figure is
seen as independent.

184
The correct answer is: Infant orientates visually without discrimination – First few months,
Infant shows preferential orientation towards selected persons – 5­7 months of age, Onset of
preferential attachment and stranger anxiety – 7­9 months of age, Formation of goal­directed
partnership between child and caregiver – 9­36 months, Lessening of attachment behaviour
and attachment figure can be substituted. – School age, Attachment behaviour typically peaks
during this period – 12­ 18 months of age

Question 3 HiY Development EMI003
Not answered Language development
Identify the age of language development for each of the descriptions given below;
Marked out of 6.00

Flag question
3 months.
48 months
24 months
The baby speaks 18­40 words.
9 months
Six weeks.
7­11 years
60 months
36 months
24 months
18 months
Six weeks.
12­18 years
7­11 years
First social smile 12 months
9 months
36 months
60 months
18 months
3 months.
18 months
12­18 years
36 months
48 months
3 months.
Repetitive babbling 12 months
60 months
12 months
Six weeks.
9 months
48 months
9 months
12­18 years
Six weeks.
24 months
18 months
7­11 years
7­11 years
Correct use of grammar 24 months
48 months
3 months.
36 months
12 months
60 months
36 months
12­18 years
48 months
A child can use the words 'mama' and 7­11 years
'dada' 3 months.
12­18 years
60 months
9 months
12 months
18 months

185
12 months
Starts to differentiate familiar and
7­11 years
unfamiliar faces
9 months
48 months
18 months
12­18 years
60 months
24 months
Check 36 months
3 months.
Six weeks.
At age 4­6 weeks the baby smiles at the mother (social smile ­ 6 weeks); can recognise
mum'92s face amidst other faces and shows a preference for visually attending to human
faces. At 9­10 months, the baby babbles tunefully. At one year of age, the child can speak one
or two words. At 18 months of age, the child can use many intelligible words '96 up to 40 in
some.
The correct answer is: The baby speaks 18­40 words. – 18 months, First social smile – Six
weeks., Repetitive babbling – 9 months, Correct use of grammar – 48 months, A child can use
the words 'mama' and 'dada' – 12 months, Starts to differentiate familiar and unfamiliar faces –
3 months.

Question 4 HiY Development EMI004
Not answered Developmental fears
For each of the following types of fear, identify the age at which they appear?
Marked out of 6.00

Flag question
No particular age trend
Age 3
Infancy
Fear of imaginary creatures
Age 5 and above
Age 4 to 5
Teenage onwards
Later childhood
Age 4 to 5
No particular age trend
Later childhood
Fear of animals
Age 5 and above
Infancy
Teenage onwards
Age 3
Teenage onwards
Age 4 to 5
No particular age trend
Fear of the dark
Later childhood
Infancy
Age 3
Age 5 and above

186
Fear of illness and death
Age 5 and above
No particular age trend
Age 3
Later childhood
Infancy
Age 4 to 5
Teenage onwards
Age 5 and above
Age 3
Teenage onwards
Fear of loud noises
Age 4 to 5
Later childhood
No particular age trend
Infancy
Age 5 and above
Age 4 to 5
Infancy
Fear of falling
Age 3
No particular age trend
Teenage onwards
Later childhood
Check

Developmental fears: Fear characteristic of infancy include fear of loud noises, fear of falling,
fear of strange objects and strange people.Fear of animals­age 3, Fear of the dark­age 4 or 5,
Fear of imaginary creatures ­ 5 plus, Fear of open spaces arise in later childhood or adult life,
Fears that arise in late childhood include fear of sex and fear of open spaces. From teenage
onwards, fears of failure, illness and death are seen. Fears that show no particular age trend
include fear of snakes and natural events such as storms.
The correct answer is: Fear of imaginary creatures – Age 5 and above, Fear of animals – Age
3, Fear of the dark – Age 4 to 5, Fear of illness and death – Teenage onwards, Fear of loud
noises – Infancy, Fear of falling – Infancy

Question 5 HiY Development EMI005
Not answered Features of language acquisition
Identify the age of language development for the situations given below
Marked out of 3.00

Flag question
36 months
7­11 years
Telegraphic speech, grammatically 24 months
pairs words with vocabulary more
12 months
than 240 words
12­18 years
3 months.
Six weeks.
48 months
9 months
60 months
18 months

187
Language akin to adult speech
24 months
36 months
3 months.
48 months
9 months
12­18 years
Six weeks.
60 months
Six weeks.
7­11 years
60 months
18 months
48 months
Early comprehension of syntax 12 months
12­18 years
7­11 years
3 months.
18 months
9 months
Check 12 months
36 months
24 months
A quick guide for age­specific development is given here: 3 months­babbling, 9 months­
repetitive babbling, 12 months­speaks three words, 18 months­speaks up to 40 words, 24
months­telegraphic speech, grammatically pairs words and vocabulary more than 240 words,
36 months­early comprehension of grammar and syntax, 48 months­correct use of grammar,
60 months­language akin to adult speech.
The correct answer is: Telegraphic speech, grammatically pairs words with vocabulary more
than 240 words – 24 months, Language akin to adult speech – 60 months, Early
comprehension of syntax – 36 months

Question 6 HiY Development EMI006
Not answered Perceptual development
Choose the most mature perceptual function achieved by the child at the following ages from
Marked out of 5.00
the list above
Flag question

Possess perceptual constancy
Vision is the most developed sense at birth
Prefers three­dimensional rather than two­dimensional representations of a face
Six months
Able to track and scan objects
Accurate acuity of vision
Able to fix objects and can have a fixed focus at about 2 cm
The acuity if newborn is around 10/300
Can have colour vision and accommodation
Able to fix objects and can have a fixed focus at about 2 cm
Can differentiate faces
Possess perceptual constancy
Accurate acuity of vision
Can differentiate faces
Prefers three­dimensional rather than two­dimensional representations of a face
Two months Can have colour vision and accommodation
The acuity if newborn is around 10/300
Vision is the most developed sense at birth
Able to track and scan objects

188
The acuity if newborn is around 10/300
Able to track and scan objects
Possess perceptual constancy
Four months
Can have colour vision and accommodation
Vision is the most developed sense at birth
Able to fix objects and can have a fixed focus at about 2 cm
Accurate acuity of vision
Prefers three­dimensional rather than two­dimensional representations of a face
Able to track and scan objects
Can differentiate faces
Prefers three­dimensional rather than two­dimensional representations of a face
Possess perceptual constancy
One month
Can differentiate faces
Able to fix objects and can have a fixed focus at about 2 cm
Can have colour vision and accommodation
Vision is the most developed sense at birth
The acuity if newborn is around 10/300
Can differentiate faces
Accurate acuity of vision
Vision is the most developed sense at birth
Accurate acuity of vision
At birth
Prefers three­dimensional rather than two­dimensional representations of a face
Able to fix objects and can have a fixed focus at about 2 cm
The acuity if newborn is around 10/300
Possess perceptual constancy
Able to track and scan objects
Check
Can have colour vision and accommodation

The foetus responds to bright light flashed on the abdominal wall of the 20­week with changes
in heart rate and position. Vision is the least developed sense at birth; the acuity if newborn is
around 20/300. At birth, infants can track and scan objects, can discriminate levels of
brightness, able to fix objects, fixed focus at about 20 cm, figure­ground discrimination. At
1month, the infant differentiates faces from other stimuli with a preference shown for face­like
complex stimuli. At two months, the infant possesses depth perception, prefers three­
dimensional rather than two­dimensional representations of a face. At four months, colour
vision and accommodation are noted to develop and at six months an accurate acuity (6:6) is
achieved in most infants.
The correct answer is: Six months – Accurate acuity of vision, Two months – Prefers three­
dimensional rather than two­dimensional representations of a face, Four months – Can have
colour vision and accommodation, One month – Can differentiate faces, At birth – Able to track
and scan objects

Question 7 HiY Development EMI007
Not answered Phases of Attachment
With the descriptions given below, identify the phases of attachment according to various
Marked out of 3.00
attachment theories;
Flag question

189
A 10 weeks old baby orientates to her
Proximity seeking
mother
Anxious resistant
Insecure attachment
Independent attachment
Pre­attachment phase
Indiscriminate attachment
Clear cut attachment
Re­orientation
Pre­attachment phase
A 5­month­old baby allows strangers Clear cut attachment
to handle and become attached to Re­orientation
mother, sister and grandmother at Proximity seeking
home. Anxious resistant
Independent attachment
Indiscriminate attachment
Insecure attachment
Proximity seeking
Indiscriminate attachment
A 2­year­old child shows preferential
Independent attachment
attachment to mother but also
Re­orientation
attaches to the father
Pre­attachment phase
Clear cut attachment
Insecure attachment
Anxious resistant
Check

Phases of attachment includes preattachment phase (birth to 8 or 12 weeks) where babies
orient to their mothers. The next phase is indiscriminate attachment (attachment in making ­ 8
to 12 weeks to 6 months) where the babies allow strangers to handle; infants become
attached to one or more persons in the environment. The next phase is clear­cut attachment
(6 through 24 months) featured by preferential attachment, separation anxiety, object
permanence, stranger anxiety. At a later part, weakened stranger anxiety; other attachment
figures may also present. After 25 months, the mother figure is seen as independent.
The correct answer is: A 10 weeks old baby orientates to her mother – Pre­attachment phase,
A 5­month­old baby allows strangers to handle and become attached to mother, sister and
grandmother at home. – Indiscriminate attachment, A 2­year­old child shows preferential
attachment to mother but also attaches to the father – Clear cut attachment

Question 8 HiY Development EMI008
Not answered Temperament Clusters
For each of the three behavioural styles given below, identify their characteristic
Marked out of 9.00
temperamental dimensions from list above (Identify three correct answers for each)
Flag question

Learning disability
Positive emotional display
Regular biorhythms
Easy child (Three choices)
Irregular biorhythms
Excessive sleepiness
High degree of physical energy
Readily adapt to changes
Slow changes in mood

190
High degree of physical energy
Learning disability
Slow to warm up child (ONE choice) Slow changes in mood
Readily adapt to changes
Regular biorhythms
Excessive sleepiness
Positive emotional display
Irregular biorhythms
Learning disability
Slow changes in mood
Readily adapt to changes
Difficult child (ONE choice)
High degree of physical energy
Positive emotional display
Excessive sleepiness
Irregular biorhythms
Regular biorhythms
Check

Thomas & Chess conducted the New York Longitudinal Study (1986). It was a thirty­year
(initially six years) longitudinal study of 138 children, observing childhood temperaments. It
employed parental interviews to ascertain temperamental dimensions. Three behavioural
styles were identified. Easy children (40%) have a rhythmic pattern of needs, adapts well, and
active. Difficult children (10%) are less predictable, uncomfortable with new experiences, with
a predominantly negative mood, often react intensely to stimuli, and are difficult to comfort.
Slow to warm up children (15%) adapt poorly to change, but less active and respond at low
intensity. Rest ungrouped category forms 35%
The correct answer is: Easy child (Three choices) – High degree of physical energy, Readily
adapt to changes, Regular biorhythms, Slow to warm up child (ONE choice) – Slow changes
in mood, Difficult child (ONE choice) – Irregular biorhythms

Question 9 HiY Development EMI009
Not answered Strange situation experiment
Based on the behaviour of the child in strange situation experiment, identify the type of
Marked out of 3.00
attachment pattern exhibited.
Flag question

Secure attachment
The child displays strong initial Disorganised attachment
proximity seeking followed by high
Indiscriminate attachment
avoidance. When the mother returns
Preattachment
the child gets frightened, distressed
Deprived attachment
and frozen.
Anaclitic depression
Avoidant/anxious attachment
Resistant/ambivalent attachment
Secure attachment
The child appears normally
interested in caregiver, and explores Preattachment
busily. There was minimal distress at Indiscriminate attachment
separation and on reunion the child Anaclitic depression
ignored the caregiver and focused on Resistant/ambivalent attachment
the environment. Avoidant/anxious attachment
Disorganised attachment
Deprived attachment

191
Preattachment
Resistant/ambivalent attachment
Anaclitic depression
The child shows a high level of
Deprived attachment
distress on separation from the
mother. On re­union, they approach
Avoidant/anxious attachment
their mother angrily, resist comforting, Secure attachment
pushing their mother away and take a Disorganised attachment
long time to settle down. Indiscriminate attachment

Check

Type A is anxious avoidant seen in 15% of children. There is an indifferent attitude to mother
leaving the room or entering the room; keeps playing indifferent to mother's presence.
Distress when alone, not when the mother is leaving. Stranger can comfort the child easily. 
Type B is secure attachment seen in 70% of children. The child plays independently when the
mother is in the vicinity (secure base effect). Distress when the mother is leaving; seeks
contact on the return of the mother and gets quickly comforted by the mother, not a stranger. 
Type C is Anxious resistant seen in 15% of children. The child is fussy and cries a lot and
cannot use the mother as a secure base to explore around. There is a very high level of
distress when the mother is leaving. But not comforted easily even on her return; appears
ambivalent about her return. The child exhibits active resistance to stranger's efforts to pacify. 
In some cases, a fourth type D is seen in disorganised type. This the in maltreated or
maternally deprived children. Insecure, dazed look and acts as if it is frightened of the mother.
May be a precursor to later personality difficulties or dissociative experiences. The mother
may have been herself abused as a child.
The correct answer is: The child displays strong initial proximity seeking followed by high
avoidance. When the mother returns the child gets frightened, distressed and frozen. –
Disorganised attachment, The child appears normally interested in caregiver, and explores
busily. There was minimal distress at separation and on reunion the child ignored the
caregiver and focused on the environment. – Avoidant/anxious attachment, The child shows a
high level of distress on separation from the mother. On re­union, they approach their mother
angrily, resist comforting, pushing their mother away and take a long time to settle down. –
Resistant/ambivalent attachment

Question 10 HiY Development EMI010
Not answered Adult Attachment Interview
With the descriptions given below, identify the type of attachment behaviour described in Adult
Marked out of 3.00
Attachment Interviews
Flag question

The subject provides spontaneous
and coherent answers in a non­
defensive manner with sufficient
elaboration and able to talk freely
about positive or negative
experiences in childhood.

192
Entangled
Anaclitic depression
Autocratic
Autonomous
Dismissing
Deprivation
Disorganised
Rapprochement
Dismissing
The subject often minimises his or Anaclitic depression
her experiences, often gives brief Autonomous
answers or '"I don't remember"
Disorganised
statements and do not use colourful
metaphors during the discourse
Entangled
Deprivation
Autocratic
Rapprochement
Disorganised
The subject who had insecure but
enmeshed attachment shows an Entangled
outpouring of emotion with a lengthy Rapprochement
discussion of childhood memories Dismissing
and is often distressed, angry and Deprivation
tearful. Autonomous
Autocratic
Anaclitic depression
Check

Main devised a semi­structured adult attachment interview with 15 items (AAI). This is based
on the fact that infantile attachment pattern can be predicted reasonably accurately using
discourse analysis of adults when recollecting their childhood. Accordingly four patterns are
noted. 
Dismissing of experiences: Those who had an avoidant (insecure) pattern often minimise their
experiences, do not elaborate on them and do not use colourful metaphors during the
discourse = type A (or avoidant). 
Secure autonomous: Those who had secure attachment provide spontaneous and coherent
answers with an ability to talk freely about negative experiences in childhood = type B. 
Entangled: Those who had insecure but ambivalent (enmeshed) attachment use multiple
emotionally laden responses and ramble excessively = type C or resistant. 
Unresolved, disorganised: Broken continuity and interrupted logical flow of thoughts are seen
in those who had insecure disorganised attachment pattern = type D.
The correct answer is: The subject provides spontaneous and coherent answers in a non­
defensive manner with sufficient elaboration and able to talk freely about positive or negative
experiences in childhood. – Autonomous, The subject often minimises his or her experiences,
often gives brief answers or '"I don't remember" statements and do not use colourful
metaphors during the discourse – Dismissing, The subject who had insecure but enmeshed
attachment shows an outpouring of emotion with a lengthy discussion of childhood memories
and is often distressed, angry and tearful. – Entangled

Question 11 HiY Development EMI011
Not answered

193
Marked out of 3.00 Separation Individuation Theory
With the features given below, identify the phase in Margaret­Mahler's separation­
Flag question
individuation theory

Formal operational phase
Symbiosis phase
A 1­month old newborn baby spends Pre­operational phase
most time in sleep as if the
Practicing sub­phase
intrauterine aloofness continues
Differentiation sub­phase
Object constancy sub­phase
Rapprochement sub­phase
Normal autism
Normal autism
An 18­month old child has alternating Rapprochement sub­phase
drives to be autonomous and Pre­operational phase
dependent; the child can explore
Object constancy sub­phase
alone but requires comfort and
reassurance on the mother's return.
Formal operational phase
Practicing sub­phase
Differentiation sub­phase
Symbiosis phase
Differentiation sub­phase
A three­year­old child understands Rapprochement sub­phase
that the mother will not be lost if Normal autism
temporarily away; hence able to Pre­operational phase
function independently Symbiosis phase
Formal operational phase
Practicing sub­phase
Object constancy sub­phase
Check

Margaret Mahler described the development of a sense of identity in young children,
independent of their mothers. This is called the separation­individuation theory, and the
proposed stages are supposed to be universal in all children. The first phase is called Normal
Autism (0 to 2 months). In this phase, the child spends most of its time in sleep, as if the
intrauterine aloofness continues. The second phase is Symbiotic Phase (2 to 5 months); inner
and outer world are studied via senses, but the child perceives the mother and self as one
unit. The last phase is called as Separation­Individuation phase, which has four sub­stages.
Differentiation sub­phase: (5 to 10m) slowly appreciates the difference between mother and
self. Practicing sub­phase: (10 to18m) gradual increase in interest on the environment;
practices exploration. Rapprochement sub­phase: (18 to 24m): Alternating drives to be
autonomous and dependent; able to explore alone but requires comfort and reassurance on
return. Object Constancy sub­phase: (2 to 5yrs) Understands that the mother will not be lost if
temporarily away; hence able to function independently.
The correct answer is: A 1­month old newborn baby spends most time in sleep as if the
intrauterine aloofness continues – Normal autism, An 18­month old child has alternating
drives to be autonomous and dependent; the child can explore alone but requires comfort and
reassurance on the mother's return. – Rapprochement sub­phase, A three­year­old child
understands that the mother will not be lost if temporarily away; hence able to function
independently – Object constancy sub­phase

Question 12 HiY Development EMI012

194
Not answered Erikson's psychosocial stages 
Marked out of 3.00
For each of the descriptions below choose the most appropriate developmental stage from
Erikson's descriptions using the list above
Flag question

Autonomy vs Despair
A 3­year­old boy insists that he wants Autonomy vs. Shame
to open a door himself despite not Identity vs. Confusion
having the strength to do so. He cries Integrity vs Despair
when his father attempts to help him. Intellectuality vs Deficit
Happiness vs Despondency
Guilt vs Shame
Generativity vs. Stagnation
Autonomy vs. Shame
Intimacy vs Stagnation
Happiness vs Despondency
A 50­year­old woman works hard to Trust vs. Mistrust
Autonomy vs Despair
save up money to help her children
Intimacy vs Stagnation
leave home
Trust vs. Mistrust
Generativity vs. Stagnation
Guilt vs Shame
Identity vs. Confusion
Generativity vs. Stagnation
Integrity vs Despair
Guilt vs Shame
Intellectuality vs Deficit
Happiness vs Despondency
A 72­year­old retired academic
psychiatrist mostly travels and reads
Identity vs. Confusion
but spends some time in limited Intimacy vs Stagnation
private legal work related to Autonomy vs. Shame
psychiatry Trust vs. Mistrust
Intellectuality vs Deficit
Autonomy vs Despair
Integrity vs Despair

Check

This question can be simply answered using the ages described in the vignettes. 3­year­old
children have the developmental conflict of Autonomy vs. Shame. At age 50, the conflict is
Generativity vs. Stagnation while at age 72; it is around Integrity vs. Despair. But it is important
to note that Erikson himself did not define tight age boundaries for his stages. In Erikson's
model autonomy refers to children gaining more control over activities and acquiring new
skills as described in the vignette. This is crucial for building self­esteem, failing which a
sense of shame is felt. A middle aged adult seeks satisfaction through productivity in career
and family / social network. This is referred to as generativity. An older adult reviews/cherishes
life accomplishments, prepares for the end of life by pursuing lifelong interests, etc. This is
referred to as integrity
The correct answer is: A 3­year­old boy insists that he wants to open a door himself despite
not having the strength to do so. He cries when his father attempts to help him. – Autonomy vs.
Shame, A 50­year­old woman works hard to save up money to help her children leave home
– Generativity vs. Stagnation, A 72­year­old retired academic psychiatrist mostly travels and
reads but spends some time in limited private legal work related to psychiatry – Integrity vs
Despair

195
Question 13 HiY Development EMI013
Not answered Piaget's experiments
For each of the following Piaget's stage of cognitive development, identify the types of
Marked out of 4.00
experiments used to demonstrate them.
Flag question

Pendulum experiment
Visual cliff test
Sensorimotor stage of cognitive Touching the dot experiment
development Strange situation test
Heinz Dilemma
Classification of objects according to rules
Rouge Test
One­dollar/20­dollar experiment
Touching the dot experiment
Piaget's experiments
One­dollar/20­dollar experiment
Pre­operational stage of cognitive
Bobo Doll experiment
Bobo Doll experiment
development Peek­a­boo games
Mountain tasks
Mountain tasks
Peek­a­boo games
Rouge Test
Strange situation test
Pendulum experiment
Peek­a­boo games
Heinz Dilemma
Pendulum experiment
Formal operational stage of cognitive
Classification of objects according to rules
Rouge Test
development Visual cliff test
Strange situation test
Piaget's experiments
Classification of objects according to rules
Heinz Dilemma
Visual cliff test
Bobo Doll experiment
Classification of objects according to rules
Mountain tasks
Strange situation test
Concrete operational stage of
One­dollar/20­dollar experiment
Heinz Dilemma
cognitive development Touching the dot experiment
Pendulum experiment
Piaget's experiments
One­dollar/20­dollar experiment
Peek­a­boo games
Piaget's experiments
Mountain tasks
Check Rouge Test
Bobo Doll experiment
Visual cliff test
Touching the dot experiment
Sensorimotor stage: Object permanence starts by nine months. It is an understanding that
object that disappears from the field of perception has not ceased to exist; if searched for this
object can be found, or it will reappear. Thus, peek­a­boo games are understood and
enjoyed. Initially, this is limited as hidden objects are searched at where they were last seen
(around 9­12 months); not at where they were hidden. Around 18 months, invisible
displacements are inferred, and object permanence is completed. 
Pre­operational stage: Egocentrism is a feature of the pre­operational stage (2­7 years). It
refers to the restrictive ability to view the world from a single point of view at this
developmental stage. In the famous three mountains task, the child is presented with a model
consisting of three mountains, where different objects will be visible from the different
positions taken to view the model. The child is then asked what an observer would see from a
vantage point different from that of the child. This is usually carried out by placing a doll in
different positions and getting the child to describe what the doll would see (e.g., the figure of

196
a cow, or the horses, etc.). Typically children under the age of eight assume that others share
their viewpoint and find it impossible to take a perspective different from their own. This
demonstrates egocentrism. 
The pendulum experiment was designed to demonstrate the concept of conservation seen in
the formal operational stage.
The correct answer is: Sensorimotor stage of cognitive development – Peek­a­boo games,
Pre­operational stage of cognitive development – Mountain tasks, Formal operational stage of
cognitive development – Pendulum experiment, Concrete operational stage of cognitive
development – Classification of objects according to rules

Question 14 HiY Development EMI014
Not answered Play and development
Identify the type of play using the descriptions given below
Marked out of 4.00

Flag question
Rule governed play
Solitary play
The child plays with a rattle by Symbolic play
shaking it Parallel play
Co­operative play
Sensorimotor play
Emotional play
Sensorimotor play
Parallel play
The child speaks to his grandpa over Emotional play
a toy phone Solitary play
Rule governed play
Co­operative play
Symbolic play
Co­operative play
Sensorimotor play
A 7­year­old child enjoys playing Symbolic play
board games with children of similar
Solitary play
age
Parallel play
Rule governed play
Emotional play
Emotional play
Sensorimotor play
A child in a doctor's waiting room Solitary play
invites another child to share the
Parallel play
plastic toy given to him by his mother
Symbolic play
Co­operative play
Rule governed play

Check

Sensorimotor play occurs around six months of age, where the infant plays with objects like
shaking a rattle. Symbolic play is also called a pretend play. It is evident by two years, and
here the child pretends that something exists where it does not like speaking to grandma over

197
a toy phone.
The correct answer is: The child plays with a rattle by shaking it – Sensorimotor play, The child
speaks to his grandpa over a toy phone – Symbolic play, A 7­year­old child enjoys playing
board games with children of similar age – Rule governed play, A child in a doctor's waiting
room invites another child to share the plastic toy given to him by his mother – Co­operative
play

Question 15 HiY Development EMI015
Not answered Freud's psychosexual stages
For each of the characteristics given below, identify the phase at which they are attained
Marked out of 6.00
according to Freud's theory of psychosexual development
Flag question

Lateral phase
Latency phase
Establishment of mature object Phallic phase
relationships Anal phase
Oral Phase
Rectal phase
Genital phase
Potency phase
Anal phase
Conventional Phase
Lateral phase
Castration anxiety and resolution of Oral Phase
oedipal complex Genital phase
Potency phase
Latency phase
Conventional Phase
Rectal phase
Lateral phase
Phallic phase
Phallic phase
Oral Phase
Strive for control
Genital phase
Potency phase
Conventional Phase
Rectal phase
Anal phase
Oral Phase
Latency phase
Latency phase
Lateral phase
Establishment of trust Conventional Phase
Rectal phase
Genital phase
Phallic phase
Anal phase
Potency phase
Oral Phase
Genital phase
Anal phase
Socialization and interest in peers.
Potency phase
Lateral phase
Latency phase
Rectal phase
Conventional Phase
Phallic phase

198
Phallic phase
True self­identity develops during this Oral Phase
stage
Lateral phase
Potency phase
Latency phase
Rectal phase
Genital phase
Conventional Phase
Check Anal phase

Sigmund Freud proposed psychosexual stages that includes oral phase (birth to 1­11/2
years), anal phase (11/2­ 3 years), phallic stage (3­5 years), Latency stage (5­6 years to 12
years) and genital stage (12 years onwards).
Oral Phase: Drive discharge is via sucking; oral erotogenic zone. oral erotism (sucking,
licking, etc.) in early stages; oral sadism (biting, chewing) in later stages. Establishment of
trust occurs at this stage. 
Anal phase: drive discharge via sphincter behaviour and strive for control. Anal erotism refers
to the sexual pleasure in anal functioning. Anal sadism refers to the aggressive wishes linked
to fecal expulsion. Anal fixation is characterized by OCD like pattern; also ambivalence and
sadomasochistic tendencies are associated. 
Phallic stage: Genitals become organs of interest; masturbation­like activity noted. Oedipus
complex ­ wish to have a libidinal relationship with opposite sex parent (Electra complex in
girls) with a desire to exclude the rival parent. This leads to a fear of retaliation from the rival
parent in the form of castration anxiety in boys and loss of mother's love in girls. Electra
complex in girls include penis envy, a wish to have penis is accompanied by blaming the
mother for absence of penis; later this becomes a secret wish to displace mother as object of
father's love and bear his baby. At the resolution of Oedipus and Electra complexes,
identification with the aggressor i.e. dad for a boy and mum for a girl occurs; super­ego
develops from introjection of parental values. 
Latency phase: Socialization, interest in peers seen. Genital phase: Biological maturation
occurs; genital sexuality is born and the establishment of mature object relationships. True
self­identity develops during this stage.
The correct answer is: Establishment of mature object relationships – Genital phase,
Castration anxiety and resolution of oedipal complex – Phallic phase, Strive for control – Anal
phase, Establishment of trust – Oral Phase, Socialization and interest in peers. – Latency
phase, True self­identity develops during this stage – Genital phase

Question 16 HiY Development EMI016
Not answered Developmental stages and virtues 
For each of Erik Erikson's psychosocial stages of development, identify the associated virtue
Marked out of 6.00
from the list provided:
Flag question

Love
Experimentation
Hope
Identity vs. Role Confusion
Competence
Will
Fidelity
Care
Wisdom
199
Love
Competence
Autonomy vs. Shame and Doubt
Will
Hope
Wisdom
Care
Fidelity
Anticipation
Competence
Purpose
Will
Experimentation
Love
Integrity vs. Despair
Experimentation
Wisdom
Anticipation
Fidelity
Care
Love
Purpose
Care
Hope
Purpose
Generativity vs. Stagnation
Anticipation
Wisdom
Experimentation
Fidelity
Hope
Care
Will
Fidelity
Competence
Wisdom
Trust vs. Mistrust
Competence
Experimentation
Love
Hope
Will
Wisdom
Anticipation
Hope
Purpose
Experimentation
Intimacy vs. Isolation
Will
Care
Fidelity
Purpose
Competence
Check
Love
Anticipation

Erik Erikson proposed eight developmental, psychosocial stages. The associated virtues in 8
stages are hope, will, purpose, competence, fidelity, love, care and wisdom. The life crisis,
ages and outcomes are outlined below. 
These include the conflict of basic trust vs mistrust (birth to 12­18 months)­ with an outcome of
hope or insecurity, autonomy vs shame (18 months to 3 years)­ with an outcome of self­
certainty or self­doubt, Initiative vs Guilt (3­6 years)­ with an outcome of experimentation or
fear of punishment and guilt, Industry vs. inferiority (6­12 years)­ with an outcome of sense of
achievement and inadequacy, Identity vs. role confusion (adolescence)­ with an outcome of
strong personal identity or identity confusion, Intimacy vs Isolation (young adulthood)­ with an
outcome of love and commitment or superficial relationships, Generativity vs stagnation

200
(middle adulthood)­ with an outcome of concern about public good or lack of growth, Ego
integrity vs. despair (65 plus years)­ with an outcome of satisfaction or regret over omissions
in life and the fear of death.
The correct answer is: Identity vs. Role Confusion – Fidelity, Autonomy vs. Shame and Doubt
– Will, Integrity vs. Despair – Wisdom, Generativity vs. Stagnation – Care, Trust vs. Mistrust –
Hope, Intimacy vs. Isolation – Love

Question 17 HiY Development EMI017
Not answered Theories of development 
Choose two appropriate options from the given list for each of the following developmental
Marked out of 8.00
theorists
Flag question

Autistic stage
Moral relativism
Authority orientation
Kohlberg
Autonomy ­ inferiority
Intimacy ­ stagnation
Genital stage
Symbiotic stage
Integrity ­ despair
Intimacy ­ stagnation
Anal stage
Genital stage
Intellectuality ­ deficit
Integrity ­ despair
Sigmund Freud Good boy ­ Good girl
Authority orientation
Autonomy ­ shame
Good boy ­ Good girl
Moral relativism
Intellectuality ­ deficit
Autonomy ­ shame
Autonomy ­ inferiority
Anal stage
Intellectuality ­ deficit
Autistic stage
Integrity ­ despair
Erik Erikson Symbiotic stage
Symbiotic stage
Autonomy ­ inferiority
Autonomy ­ shame
Anal stage
Genital stage
Good boy ­ Good girl
Symbiotic stage
Authority orientation
Authority orientation
Moral relativism
Genital stage
Autistic stage
Margaret Mahler Autonomy ­ inferiority
Intimacy ­ stagnation
Intimacy ­ stagnation
Anal stage
Integrity ­ despair
Autonomy ­ shame
Intellectuality ­ deficit
Check Autistic stage
Good boy ­ Good girl
Moral relativism

Erik Erikson proposed eight developmental, psychosocial stages. It includes Basic Trust vs.
Mistrust, Autonomy vs. Shame, Initiative vs. Guilt, Industry vs. Inferiority, Identity vs. Role
Confusion, Intimacy vs. Isolation, Generativity vs. Stagnation, and Ego Integrity vs. Despair. 

201
Margaret Mahler proposed separation­ individuation theory and the stages are normal autism,
symbiosis, and separation­individuation phase. 
Sigmund Freud proposed psychosexual stages that include oral stage, anal stage, the phallic
stage, latency stage and genital stage. 
Six stages are identified in Kohlberg's theory of moral development. It includes punishment
and obedience orientation, reward orientation (or instrumental relativism), good boy/good girl
orientation, authority orientation, social contract or legalistic orientation and universal ethical
orientation.
The correct answer is: Kohlberg – Good boy ­ Good girl, Authority orientation, Sigmund Freud
– Anal stage, Genital stage, Erik Erikson – Autonomy ­ shame, Integrity ­ despair, Margaret
Mahler – Symbiotic stage, Autistic stage

Question 18 HiY Development EMI018
Not answered Child development
Choose ONE developmental concept that best fits each of the scenarios below:
Marked out of 3.00

Flag question
Authoritarian parenting
A 20­months­old child cries Polytropy
whenever her mother, father or Disorganised attachment
childminder bid goodbye to her when Rapprochement
leaving. Bonding
Monotropy
Permissive parenting
Avoidant attachment
Authoritarian parenting
A 14­months­old child has an
Normal autism
Normal autism
indifferent attitude to mother leaving
the room or entering the room. He Individuation
Individuation
keeps playing indifferent to the Insecure attachment
Rapprochement
mother'92s presence but shows signs Ambivalent attachment
Bonding
of distress when alone. Imprinting
Monotropy
Authoritative parenting
Polytropy
Authoritative parenting
Authoritarian parenting
A 28­months­old child can stay alone Avoidant attachment
Monotropy
for prolonged periods but still seeks Imprinting
Ambivalent attachment
reassurance, expresses her joy and Ambivalent attachment
Insecure attachment
runs to get a hug on the return of her Disorganised attachment
mother from work.
Avoidant attachment
Insecure attachment
Imprinting
Permissive parenting
Rapprochement
Bonding
Check Normal autism
Disorganised attachment
Individuation
Authoritative parenting
This child has more than one attachment figure ­ this is called as polytropy. In the avoidant
Polytropy
pattern of attachment, the child has an indifferent attitude towards mother's presence and feels
Permissive parenting
similarly comfortable both with the stranger and the mother. Rapprochement refers to the third
subphase during the separation­individuation process described by Mahler. The child,
despite learning new skills, remains anxious regarding the loss of the love of caregiver when
they are away. This is evident when the child runs to hug the mother on her return from
regular absence (e.g. from work).
The correct answer is: A 20­months­old child cries whenever her mother, father or childminder

202
bid goodbye to her when leaving. – Polytropy, A 14­months­old child has an indifferent
attitude to mother leaving the room or entering the room. He keeps playing indifferent to the
mother'92s presence but shows signs of distress when alone. – Avoidant attachment, A 28­
months­old child can stay alone for prolonged periods but still seeks reassurance, expresses
her joy and runs to get a hug on the return of her mother from work. – Rapprochement

Question 19 HiY Development EMI019
Not answered Developmental age
Choose ONE childhood behaviour that best fits each of the given scenarios:
Marked out of 4.00

Flag question
1 month
5 years
A child can localise sound source 2 years
and discriminates smile response. 6 years
6 months
12 months
3 years
7 years
6 months
3 months
4 years
A child builds six cube towers and 4 years
2 years
can speak in two­word sentences 9months
6 years
with no propositions.
3 months
12 months
9months
5 years
4 years
7 years
12 months
A child can copy a cross and a circle,
but cannot copy a triangle or
1 month
5 years
diamond. He is also toilet trained for 3 years
2 years
both bladder and bowel control. 6 months
3 months
3 years
7 years
12 months
9months
1 month
A child can copy a circle, build nine
1 month
6 years
cubes tower and speak in sentences. 6 years
4 years
3 months
6 months
9months
5 years
Check 7 years
2 years
3 years
At three months a child can babble to some extent, can discriminate mother'92s smile and
localize the source of a sound. At two years of age, a child can build six cube towers; can use
words in a meaningful way as if sending a telegraph (two words with no conjunctions).At age
four a cross can be copied, but not a triangle or diamond.At age three a child can go up stairs
1 foot per step and downstairs 2 feet per step. It can copy a circle, and build a tower of 9
cubes.
The correct answer is: A child can localise sound source and discriminates smile response. –

203
3 months, A child builds six cube towers and can speak in two­word sentences with no
propositions. – 2 years, A child can copy a cross and a circle, but cannot copy a triangle or
diamond. He is also toilet trained for both bladder and bowel control. – 4 years, A child can
copy a circle, build nine cubes tower and speak in sentences. – 3 years

Question 20 HiY Development EMI020
Not answered Attachment related behaviours
For each of the descriptions given below, identify the type of attachment behaviour from the
Marked out of 3.00
list
Flag question

A 3­year­old child was taken to a Secure base effect
toddlers group by his mother, to Disenchantment
whom he is primarily attached. With Indiscriminate attachment
his mother being present, he attempts Re­orientation
to move around, explore and interact Separation anxiety
with others in the room Emotionality
Detachment
Proximity seeking
Emotionality
Separation anxiety
Disenchantment
An 18­month­old baby became Indiscriminate attachment
tearful and distressed when one of Re­orientation
the health visitors took her away from Secure base effect
her mum for an examination. Proximity seeking
Detachment

Re­orientation
Separation anxiety
A 12 month old baby tends to follow Disenchantment
and remain in close contact with her
Proximity seeking
mother all the time
Indiscriminate attachment
Secure base effect
Emotionality
Detachment
Check

According to attachment theory, separation anxiety refers to the unrealistic and often
excessive worries experienced by a child when he/she is away from the primary caregiver,
usually the mother. Children with secure attachment behaviour play independently when the
mother is in the vicinity (secure base effect). They show appropriate distress when she is
leaving and seeks contact on her return and gets quickly comforted by the mother but not so
easily by a stranger.Proximity seeking describes the physical closeness sought by infants to
their caregiver. This behaviour intensifies in times of physiological needs such as hunger,
tiredness or when physically unwell.
The correct answer is: A 3­year­old child was taken to a toddlers group by his mother, to
whom he is primarily attached. With his mother being present, he attempts to move around,
explore and interact with others in the room – Secure base effect, An 18­month­old baby

204
became tearful and distressed when one of the health visitors took her away from her mum for
an examination. – Separation anxiety, A 12 month old baby tends to follow and remain in
close contact with her mother all the time – Proximity seeking

Question 21 HiY Development EMI021
Not answered Developmental anxieties
For the given situations below, identify the type of anxiety exhibited.
Marked out of 4.00

Flag question
Separation anxiety
Annihilation anxiety
This normally occurs around the age Phobic anxiety
of 7­9 months during a child's
Stranger anxiety
development
Signal anxiety
Disintegration
Fear of object loss
Superego anxiety
Superego anxiety
Generalised anxiety
Fear of object loss
This occurs when the mother is Disintegration
recognised as an independent object Separation anxiety
Stranger anxiety
Phobic anxiety
Annihilation anxiety
Generalised anxiety
Separation anxiety
Signal anxiety
Signal anxiety
This has been described to occur Superego anxiety
especially in girls during phallic stage Stranger anxiety
Fear of object loss
Generalised anxiety
Disintegration
Phobic anxiety
Signal anxiety
Annihilation anxiety
Superego anxiety
Annihilation anxiety
Phobic anxiety
This is the most mature form of Disintegration
anxiety characterised by prohibitive
Generalised anxiety
conflicts.
Separation anxiety
Stranger anxiety
Fear of object loss

Check

Hierarchy of anxiety: 

Signal anxiety ­ unconscious perception of external or internal threat leads to resource
mobilization and aversion of threat. 
Disintegration / annihilation anxiety are concerns about fusion with an external object. 

205
Stranger anxiety occurs around 7­9 months age. Separation anxiety occurs when mother
is recognised as independent object. 
Fear of object loss / loss of love occurs especially in girls at the phallic stage and also
called as Castration anxiety in boys.
Superego anxiety is the most mature form of anxiety featured by id vs. super­ego conflicts
as opposed to signal anxiety that occurs when id vs. ego conflicts arise.

The correct answer is: This normally occurs around the age of 7­9 months during a child's
development – Stranger anxiety, This occurs when the mother is recognised as an
independent object – Separation anxiety, This has been described to occur especially in girls
during phallic stage – Fear of object loss, This is the most mature form of anxiety
characterised by prohibitive conflicts. – Superego anxiety

Question 22 HiY Development EMI022
Not answered Psychosexual and cognitive development 
For each of the following ages, choose options that correspond to the most appropriate
Marked out of 8.00
psychosexual and cognitive development from the above list
Flag question

Genital stage
Deductive reasoning
Anal phase
A 9­month old child. (Choose ONE)
Concrete operational stage
Pre­operational stage
Trust vs. mistrust
Autonomy vs. shame
Industry vs. inferiority
Trust vs. mistrust
Generativity
Pre­operational stage
Stagnation
Anal phase
A 4­year old child. (Choose ONE)
Concrete operational stage
Deductive reasoning
Industry vs. inferiority
Generativity
Stagnation
Pre­operational stage
Autonomy vs. shame
Stagnation
Genital stage
Generativity
Concrete operational stage
Anal phase
Autonomy vs. shame
A 17­year old child. (Choose TWO) Trust vs. mistrust
Genital stage
Deductive reasoning
Industry vs. inferiority

Check

206
Erik Erikson proposed 8 developmental psychosocial stages. It includes Basic Trust vs
Mistrust (birth to 12­18 months), Autonomy vs Shame (18 months to 3 years), Initiative vs. Guilt
(3­6 years), Industry vs Inferiority (6­12 years), Identity vs Role Confusion (adolescence),
Intimacy vs Isolation (young adulthood), Generativity vs. stagnation (middle adulthood), Ego
integrity vs despair (late adulthood). Jean Piaget proposed four important stages of cognitive
development. It includes sensorimotor stage (0 to 2 years), pre­operational stage (2­7 years),
concrete operational stage (7­11 years) and formal operational stage (more than 11 years).
Hypothetico­deductive reasoning is a feature of the formal operational stage, which develops
in a proportion of children after the age of 12. Sigmund Freud proposed psychosexual stages
that includes oral phase (birth to 1­11/2 years), anal phase (11/2­ 3 years), phallic stage (3­5
years), Latency stage (5­6 years to 12 years) and genital stage (12 years onwards).
The correct answer is: A 9­month old child. (Choose ONE) – Trust vs. mistrust, A 4­year old
child. (Choose ONE) – Pre­operational stage, A 17­year old child. (Choose TWO) – Deductive
reasoning, Genital stage

Question 23 HiY Development EMI023
Not answered Psychoanalytic concepts of development
Choose the most relevant stage related to each clinical scenario described below.
Marked out of 3.00

Flag question A patient has repeated obsessive
thoughts regarding dirt and Pre­operational
contamination. His anxiety is only Concrete operational
partially reduced by repeated Oral stage
washing and checking hands for Sensorimotor stage
germs. According to the Phallic stage
psychoanalytic theory at which stage Paranoid/schizoid position
is he fixated? Formal operational
Anal stage
A patient indulges in excessive
Paranoid/schizoid position
Latent stage
drinking to bring pleasure. He Latent stage
Reward orientation stage
Formal operational
neglects other sources of leisure
Universal ethics stage
activities completely. According to the Universal ethics stage
Depressive position
psychoanalytic theory at which stage Pre­operational
is he fixated? Reward orientation stage
Phallic stage
At this developmental stage, Oral stage
Anal stage
projection of both bad and good Concrete operational
Concrete operational
impulses onto the maternal object is Sensorimotor stage
Universal ethics stage
seen. This is followed by splitting of Depressive position
Reward orientation stage
the external world into good and bad. Anal stage
Depressive position
The child is unable to unify these
elements into one. Oral stage
Latent stage
Paranoid/schizoid position
Check Sensorimotor stage
Formal operational
Pre­operational
Phallic stage
Orderliness, obstinacy, stubbornness, wilfulness, frugality, and parsimony are features of the
anal character derived from a fixation on anal functions, most typically seen in obsessive­
compulsive neuroses. (Kaplan Synopsis 9th ed: Pg: 199). An early psychoanalytic notion was
that children who are fixated at the oral stage would be more likely to abuse alcohol later in

207
life. In the paranoid­schizoid position (Melanie Klein, 1946) the initial experiences of the infant
are divided between entirely good experiences with "good" objects and bad experiences with
"bad" objects. As a defence from the bad objects, the ego splits itself into two.
The correct answer is: A patient has repeated obsessive thoughts regarding dirt and
contamination. His anxiety is only partially reduced by repeated washing and checking hands
for germs. According to the psychoanalytic theory at which stage is he fixated? – Anal stage, A
patient indulges in excessive drinking to bring pleasure. He neglects other sources of leisure
activities completely. According to the psychoanalytic theory at which stage is he fixated? –
Oral stage, At this developmental stage, projection of both bad and good impulses onto the
maternal object is seen. This is followed by splitting of the external world into good and bad.
The child is unable to unify these elements into one. – Paranoid/schizoid position

Question 24 HiY Development EMI024
Not answered Proponents of developmental concepts
From the attached list choose the concepts attributed to various proponents of psychological
Marked out of 7.00
theories:
Flag question

Scaffolding
Isolation
Anal phase
Mahler
Autistic phase
Hypotheticodeduction
Operations
Animism
Social contract
Anal phase
Scaffolding
Autistic phase
Erikson
Social contract
Operations
Animism
Hypotheticodeduction
Isolation
Anal phase
Scaffolding
Social contract
Kohlberg
Animism
Isolation
Hypotheticodeduction
Operations
Autistic phase
Check

Autistic and symbiotic phase is associated with Margaret Mahler. Generativity vs. stagnation
and Intimacy vs Isolation are stages described in Erik Erikson's psychosocial theory. Good
Boy ­ Good Girl orientation, Obedience Orientation and Social contract Orientation are
different levels described in Kohlberg's theory of moral development.
The correct answer is: Mahler
– Autistic phase, Erikson
– Isolation, Kohlberg
– Social contract

208
Question 25 HiY Development EMI025
Not answered Psychosexual and cognitive development 
For each of the following age ranges, choose options that correspond to the most appropriate
Marked out of 5.00
stages of development from the given list
Flag question

Industry vs. inferiority
Deductive reasoning
Oral phase
0­1 year (Choose ONE)
Concrete operational stage
Autonomy vs. shame
Initiative vs. guilt
Sensorimotor stage
Anal phase
Trust vs. mistrust
Trust vs. mistrust
Sensorimotor stage
Anal phase
4­7 years (Choose ONE)
Concrete operational stage
Autonomy vs. shame
Oral phase
Initiative vs. guilt
Deductive reasoning
Deductive reasoning
Industry vs. inferiority
Initiative vs. guilt
Industry vs. inferiority
7­12 years (Choose TWO)
Autonomy vs. shame
Sensorimotor stage
Anal phase
Oral phase
Trust vs. mistrust
Check
Concrete operational stage

Erik Erikson proposed 8 developmental psychosocial stages. It includes basic trust vs.
mistrust (birth to 12­18 months), autonomy vs. shame (18 months to 3 years), initiative vs. guilt
(3­6 years), industry vs. inferiority (6­12 years), identity vs. role confusion (adolescence),
intimacy vs. isolation (young adulthood), generativity vs. stagnation (middle adulthood), and
ego integrity vs. despair (late adulthood). Jean Piaget proposed four important stages of
cognitive development. It includes sensorimotor stage (0 to 2 years), pre­operational stage (2­
7 years), concrete operational stage (7­11 years) and formal operational stage (more than 11
years). Hypothetico­deductive reasoning is a feature of formal operational stage, which
develops in a proportion of children after the age of 12. Sigmund Freud proposed
psychosexual stages that includes oral phase (birth to 18 months), anal phase (18 months ­ 3
years), phallic stage (3­5 years), latency stage (5 to 12 years) and genital stage (12 years
onwards).
The correct answer is: 0­1 year (Choose ONE) – Oral phase, 4­7 years (Choose ONE) –
Initiative vs. guilt, 7­12 years (Choose TWO) – Concrete operational stage, Industry vs.
inferiority

209
Finish review

210
Home HiYield Paper A(1) Assessment

HiYield Paper A(1)

HiY Classification 001


What percentage of non-schizophrenic patients with psychosis exhibit first rank symptoms of
schizophrenia?

Select one:
40
20
5

10
50

Kurt Schneider enumerated a number of symptoms which he believed were of first-rank


importance in differentiating schizophrenia from related illnesses. According to the International
Pilot Study of Schizophrenia, 58% of patients with acute schizophrenia exhibited at least one
first rank symptom. However, at least 20% of schizophrenic never exhibit a first rank symptom
while almost 10% of non-schizophrenic patients exhibit them.
The correct answer is: 10

HiY Classification 002


The year of publication of ICD 10 is

Select one:
1996
1992
1990

211
1948
1994

ICD-10 was published in 1992 and DSM-4 in 1994. The earliest edition of ICD was first
published by WHO in 1948. DSM-3 was published by the American Psychiatric Association in
1980. DSM-2 in 1968, DSM-1 in 1952.
The correct answer is: 1992

HiY Classification 003


In which of the following personality disorders, the patient shows a preoccupation to avoid
criticism and rejection?

Select one:
Dependent
Anxious Avoidant
Borderline

Passive aggressive
Anankastic

Anxious/Avoidant personality disorder is characterized by a constant perception of tension and


apprehension; an unreasonable but pervasive feeling of being inadequate and inferior to others;
persistent preoccupation with rejection in social situations; reluctance to develop relationships
unless certain of being accepted (and not rejected); avoidance of interpersonal contact due of
fear of criticism, disapproval, or abandonment.
The correct answer is: Anxious Avoidant

HiY Classification 004


The term simple schizophrenia was added to the categories of existing schizophrenias by

Select one:
Bleuler
Hubbard
Hecker
Angst
Liddle

212
The diagnostic category called simple schizophrenia has a long history; it was one of the
traditional schizophrenic subtypes identified by Bleuler and was later accepted by Kraepelin.
The term was removed from the official American nosology but has been retained in the ICD
system.
The correct answer is: Bleuler

HiY Classification 005


A school teacher suspects Asperger's disease in a young boy. Which of the following would
support her concerns?

Select one:
Poor learning of grammar
Other teachers report that he has very few friends
He wants to be a gardener
The mum reports that he did not speak at all before the age of 3

No eye contact during the entire interview

Lack of eye contact is often seen in autistic spectrum disorders.


The correct answer is: No eye contact during the entire interview

HiY Classification 006


What is the minimum symptom duration required to make an ICD-10 diagnosis of
schizophrenia?

Select one:
6 months
3 months

1 month
1 week
1 day

213
For an ICD 10 diagnosis symptom duration of at least 1 month is generally required while with
DSM, duration of at least 6 months is needed. If the duration is less than one month then a
diagnosis of acute schizophrenia-like psychotic disorder should be made.
The correct answer is: 1 month

HiY Classification 007


In post schizophrenic depression the most recent episode of relapse should be within the last

Select one:
12 months
3 months
24 months
6 months
9 months

Some schizophrenic patients develop depressive features within 12 months of an acute


episode of schizophrenia. To make a diagnosis of post-schizophrenia depression, the most
recent episode of relapse must not have been more than 12 months ago. If no current
schizophrenic symptoms at all then the depression can be diagnosed. If florid schizophrenia
symptoms with minor affective disturbances are noted, then the possibility of relapse must be
suspected.
The correct answer is: 12 months

HiY Classification 008


What is the minimum duration required to make a diagnosis of alcohol dependence according to
ICD-10 criteria?

Select one:
2 months
3 months

1 month
12 months
6 months

214
ICD10 alcohol dependence requires at least 3 features out of following list satisfied in the last
12 months: 1. intense desire to drink alcohol 2. difficulty in controlling the onset, termination and
the level of drinking 3. experiencing withdrawal symptoms if alcohol is not taken 4. use of
alcohol to relieve from withdrawal symptoms 5. tolerance as evidenced by the need to escalate
dose over time to achieve same effect 6. salience - neglecting alternate forms of leisure or
pleasure in life 7. A narrowing personal repertoire of alcohol use.
The correct answer is: 12 months

HiY Classification 009


A mother wants to know the likelihood that her teenage son who has been smoking cannabis
for 6 months will develop schizophrenia. The correct answer is

Select one:
Nil increase in risk
6 - 7 times
7 - 8 times
5 - 6 times
2 - 3 times

At present, there is a consensus that cannabis increases the risk of psychosis 2-3 times,
especially if the exposure started early in adolescence and persisted to adulthood.
The correct answer is: 2 - 3 times

HiY Classification 010


According to ICD-10, acute polymorphic psychotic disorder is characterised by an acute onset of
a clear psychotic state from a non-psychotic state within a period of:

Select one:
2 days
1 week

4 weeks
2 weeks
1 day

215
According to ICD-10, acute polymorphic psychotic disorder is characterised by acute onset from
a non-psychotic state to a clearly psychotic state within 2 weeks and polymorphic picture
(unstable and markedly variable clinical picture that changes from day to day or even from hour
to hour). There are several types of hallucinations and/or delusions changing in both type and
intensity from day to day or within the same day. A marked emotional turmoil which ranges from
intense feelings of happiness and ecstasy to anxiety and irritability is also frequently present.

The correct answer is: 2 weeks

HiY Classification 011


According to DSM V, caffeinism is defined as recent consumption of caffeine usually in excess
of

Select one:
100 mg
50 mg
250 mg
200 mg
150 mg

According to DSM, caffeinism is defined as "recent consumption of caffeine usually in excess


of 250 MG accompanied by five or more of the following; restlessness, nervousness,
excitement, insomnia, flushed face, diuresis, GI disturbance, muscle twitching, rambling flow of
thought and speech, tachycardia, periods of inexhaustibility, psychomotor agitation", when
these symptoms can cause significant distress or impairment in social, occupational or other
important areas of functioning and are not due to a general medical condition or better
accounted for by another medical disorder (e.g. anxiety disorder).
The correct answer is: 250 mg

HiY Classification 012


According to ICD 10 what is the minimum duration required for a diagnosis of hypomania?

Select one:
2 weeks
4 days

2 months
4 weeks
1 week

216
ICD 10 requires a duration of at least 1 week for mania and at least 4 days for hypomania.
The correct answer is: 4 days

HiY Classification 013


How long does the natural course of an episode of untreated mania last?

Select one:
4 months
9 months
6 weeks
2 weeks
4 weeks

Except in the elderly, the natural course of mania lasts for 4 months while depression for 6
months. This becomes longer in the elderly who show shorter periods of interepisodic
remissions and more frequent episodes which are considerably longer.
The correct answer is: 4 months

HiY Classification 014


What is the time duration before which most cases with acute uncomplicated delirium
recover?

Select one:
4 weeks
6 months

3 days
1 day
1 year

Most cases of delirium recover in 4 weeks; in chronic lung disease, SABE (subacute bacterial
endocarditis) and carcinoma up to 6 months long delirium may be seen

217
The correct answer is: 4 weeks

HiY Classification 015


A 16-year-old boy who has used cannabis for the first time complains that his legs are being
pulled and moved. Which of the following is most likely?

Select one:
Acute cannabis effect
Dyskinesia
Complex seizure
Schizophrenia
Delusional disorder

Kinaesthetic hallucinations and passivity can occur under the acute intoxication-related
effects of cannabis.
The correct answer is: Acute cannabis effect

HiY Classification 016


For four weeks after an automobile accident where her husband died in front of her, a
schoolteacher suffers from insomnia, poor concentration and tearfulness. She has nightmares
and emotional outbursts when awake. Which of the following is the most likely diagnosis?

Select one:
post-traumatic stress disorder
acute stress disorder
dysthymia
adjustment disorder

depressive episode

Acute stress reaction: starts usually in an hour; resolution starts within 8 hours if the stress is
'hit and run' type or 48 hours if it is prolonged. The presence of physical exhaustion, organic
factors or disease states increases the risk. The stressor is usually one that poses a serious
threat to security, integrity and social position. The patient may initially be dazed with narrowed
attention; disorientation is not uncommon as a result. Sometimes agitation and overactivity are
seen. Partial or complete amnesia for the acute stress reaction is not unheard of. Dissociative
symptoms seem to predominate in some.
The correct answer is: acute stress disorder

218
HiY Classification 017
Which one of the following is not categorised as a pervasive developmental disorder?

Select one:
ADHD
Asperger's syndrome
Autism
Childhood disintegrative disorder
Rett syndrome

According to DSM-IV PDDs include autism, Asperger's syndrome, Rett syndrome, Childhood
disintegrative disorder and PDD-Not Otherwise Specified. Note that in DSM-V, Asperger's
syndrome is described as an autism spectrum disorder.
The correct answer is: ADHD

HiY Classification 018


Mr.X is a 27-year-old gentleman with a long standing diagnosis of schizophrenic illness. He
has both positive and negative symptoms of schizophrenia. Which among the following is
classified as a negative symptom of schizophrenia?

Select one:
Hallucinations
Bizarre behaviour

Affective flattening
Delusions
Formal thought disorder

Schizophrenia is characterised by positive symptoms including delusions, hallucinations, formal


thought disorder, bizarre behaviour and inappropriate affect. The negative symptoms include
poverty of speech (alogia), affective flattening/blunting, avolition, apathy, anhedonia, asociality
and attentional impairment. The negative symptoms become more prominent as the illness
becomes chronic.
The correct answer is: Affective flattening

219
HiY Classification 019
Which term refers to 'discrete episodes of anterograde amnesia that occur in association with
alcohol intoxication'?

Select one:
Pathological intoxication
Wernicke's encephalopathy
Korsakoff's syndrome

Alcoholic dementia
Alcoholic blackouts

Alcoholic blackouts: Alcohol-related blackouts are not included in DSM-IV/ ICD10. Blackouts are
discrete episodes of anterograde amnesia that occur in association with alcohol intoxication.
During a blackout (at the time of intoxication), remote memory is intact but patients experience
specific short-term episodic memory deficit (they can talk about their childhood etc., but cannot
remember what topic the conversation is about or how did they come to where they were) They
can even perform complicated tasks but later will not remember these. The memory gap usually
lasts for hours, rarely a day or more. Alcohol blocks the consolidation of new memories into old
memories the at the hippocampus.
The correct answer is: Alcoholic blackouts

HiY Classification 020


A 49-year-old man is highly alcohol dependent. Due to various physical problems he was
strongly advised to abstain. Unfortunately since last episode of intoxication he has developed
recurrent voices that frighten him. In spite of 2 months of abstinence these hallucinations
continue, and he now presents with persecutory delusions. Which of the following diagnosis
best suits his presentation?

Select one:
Depressive psychosis
Delirium tremens
Schizophrenia
Alcoholic hallucinosis
Delusional disorder

The DSM has a diagnostic criteria for alcohol-induced psychotic disorders (popularly called
alcoholic hallucinosis) and further allows the specification of onset (during intoxication or
withdrawal). The most common hallucinations are unstructured sounds or voices that may be

220
characteristically malign and threatening. The hallucinations usually last less than a week,
when patients believe in the hallucinations though afterwards they may realise the untrue
nature.
The correct answer is: Alcoholic hallucinosis

HiY Classification 021


Which of the following is the best test to differentiate delirium from dementia?

Select one:
Altered consciousness
Poor recall
Neurological localising signs
Presence of visual hallucinations
Presence of delusions

Dementia is not usually associated with altered consciousness unless there is a


superimposed delirium, or if the patient is in an advanced state of profound dementia.
The correct answer is: Altered consciousness

HiY Classification 022


A 55-year old man has been suffering from short-term memory problems, and other problems
related to concentration. Otherwise, his daily activities are not affected. Which of the following is
he most likely to develop?

Select one:
Vascular dementia
Lewy body dementia

Huntington's chorea
Creutzfelt Jacob disease
Alzheimer's dementia

Patients with Mild Cognitive Impairment (MCI), if progressing to further deterioration, are most
likely to convert to Alzheimer's-like dementia.
The correct answer is: Alzheimer's dementia

221
HiY Classification 023
People with which of the following personality disorders find it hard to make decisions easily?

Select one:
Schizotypal personality
Narcissistic personality
Schizoid Personality
Anxious avoidant personality
Anankastic Personality

People with OCPD (anankastic PD) find it hard to decide on even seemingly trivial issues,
leading to frustration at times.
The correct answer is: Anankastic Personality

HiY Classification 024


A mother complains that her 20-year-old son is upsetting his employers by being very rigid in
his duties and not able to delegate tasks. He also has difficulties in discarding his materials and
hoards items including old newspapers in his office. What is the most likely diagnosis?

Select one:
Schizoid personality
Obsessive compulsive disorder

Normal presentation
Anankastic personality disorder
Schizophrenia

People with OCPD (anankastic PD) find it hard to dispose of possessions and may hoard
them. They also struggle to decide on even seemingly trivial issues, leading to frustration at
times.
The correct answer is: Anankastic personality disorder

HiY Classification 025


Mrs. Q is a 43-year-old woman who has a long standing history of the mood disorder of over 20
years. Over the last year alone she has suffered from 5 mood episodes and is currently on
treatment with Semisodium Valproate. Which one among the following is associated with her
condition?

222
Select one:
Hyperthyroidism
Anti-depressant use
Alcohol Abuse

Menopause
Stressful life events

Some patients with bipolar mood disorder have more than four episodes per year; they are
called rapid cyclers. 70-80% of rapid cyclers are women. Some of the factors associated with
the rapid cycling include the use of tricyclic anti-depressant, low thyroxin level, being a female
patient, having a bipolar 2 pattern of illness and the presence of neurological disease.
The correct answer is: Anti-depressant use

HiY Classification 026


Which one of the following disorders is characterised by normal language development?

Select one:
Asperger's syndrome
Rett syndrome
High functioning autism
Autism
Childhood disintegrative disorder

Asperger's syndrome is characterised by severe persistent impairment in social interactions,


repetitive behaviour patterns, and restricted interests. But the IQ and language are often
normal. Unlike autism, patients with Asperger's disorder show no delays in acquiring language,
cognitive development, or age-appropriate self-help skills. Both Rett's syndrome and Heller's
syndrome of Childhood Disintegrative Disorder are associated with disturbances in language
development.
The correct answer is: Asperger's syndrome

HiY Classification 027


According to DSM-IV criteria, the core symptoms of ADHD must have their onset

Select one:
Before 12 years of age

223
Before 7 yrs of age
Before 9 years of age
Before 3 years of age

Before 18 years of age

Diagnostic criteria: According to DSM-IV criteria, to meet the diagnosis of ADHD, some
symptoms must be present before the age of 7 years although ADHD is not diagnosed in many
children until they are older than 7 years when their behaviours cause problems in school and
other places. Note that DSM-V has introduced changes in the diagnosis of ADHD: symptoms
can now occur before age 12 rather than before age 7; several symptoms now need to be
present in more than one setting rather than just the impairment criteria being satisfied in more
than one setting; new descriptions are now added to capture ADHD features in later ages; for
individuals aged 17 or older, only 5 symptoms are required for a diagnosis instead of the usual
6 needed in children. Ref: http://www.cdc.gov/ncbddd/adhd/diagnosis.html

The correct answer is: Before 7 yrs of age

HiY Classification 028


A sportsperson is found to use excessive analgesics to help symptoms of pain following intense
training. He abuses analgesics in doses that are much higher than the prescribed amount and
refuses to stop them despite medical advice. In ICD10, this condition is described in which of
the following chapters?

Select one:
Organic mental disorders

Neurotic, stress-related and somatoform disorders


Disorders of personality and behaviour
Mental and behavioural disorders due to use of psychoactive substances
Behavioural syndromes associated with physiological disturbances and
physical factors

Although it is usually clear that the patient has a strong motivation to take the substance,
dependence or withdrawal symptoms do not develop when abusing analgesics, unlike what is
seen in the case of psychoactive substances specified in F10-F19 ('Mental and behavioural
disorders due to psychoactive substance use') of ICD-10. The abuse of analgesics is described
under F50-F59 ('Behavioural syndromes associated with physiological disturbances and
physical factors').
The correct answer is: Behavioural syndromes associated with physiological disturbances
and physical factors

224
HiY Classification 029
A patient with long-standing anxiety being managed by GP was recently seen by her
psychiatrist who made changes in her medications. She presents to the A and E with
hallucinations of her joints being moved and nausea with agitation and anxiety. What is the
diagnosis?

Select one:
Antidepressant withdrawal
Benzodiazepine intoxication
Benzodiazepine withdrawal
Alcohol intoxication

Acute transient psychosis

Symptoms of benzodiazepine withdrawal include autonomic hyperactivity, increased tremor,


insomnia, nausea or vomiting, transient visual, tactile, or auditory hallucinations or
illusions,psychomotor agitation and anxiety. Sometimes grand mal seizures can occur.
Kinaesthetic hallucinations are characteristically related to benzodiazepine withdrawal and
have been reported in some patients. Alcoholism in the Elderly - American Family Physician,
http://www.aafp.org/afp/2000/0315/p1710.html (accessed April 10, 2015).
The correct answer is: Benzodiazepine withdrawal

HiY Classification 030


Miss X is a 26-year-old woman who has been treated recently for hypomania with a mood
stabiliser. She also has a past history of depression 3 years ago. Which one among the
following is her diagnosis according to ICD10 criteria?

Select one:
Recurrent depressive disorder

Mixed affective state


Rapid cycling disorder
Bipolar disorder type 2
Bipolar disorder type 1

Bipolar disorder is divided into 2 main broad types; type 1 is characterised by full blown mania or
mixed mania and depression, type 2 is characterised by recurrent depression and hypomania
without episodes of full-blown mania or mixed states. Mixed states are cases

225
where manic and depressive symptoms occur simultaneously during the same episode. In rapid
cycling disorder, there will be at least 4 episodes of bipolar disorder occurring within the period
of 1 year.
The correct answer is: Bipolar disorder type 2

HiY Classification 031


Mr.X has been treated recently for hypomania. He also has a previous history of depression.
Which one among the following is his correct diagnosis?

Select one:
Recurrent depressive disorder
Bipolar disorder type 1
Bipolar disorder type 2
Rapid cycling disorder
Mixed affective state

Bipolar affective disorder is divided into two main broad types; Type 1 is characterised by full­
blown mania or mixed mania and depression, Type 2 is characterised by recurrent depression
and hypomania without episodes of either mania or mixed states. Mixed states are cases where
manic and depressive symptoms occur simultaneously. Rapid Cycling: When at least four
episodes of bipolar disorder occur within a period of one year, the condition is described as
'rapid cycling'. Ref: Pandarakalam, JP (2008) Diagnostic conundrums of bipolar disorder.
Progress in Neurology and Psychiatry, 12, 6-11.
The correct answer is: Bipolar disorder type 2

HiY Classification 032


A 40-year-old man with poor medication compliance has a long-running history of dysthymia
withepisodes of hypomania on prescribing antidepressants. Which of the following best
describes his diagnosis?

Select one:
bipolar II disorder
double depression

cyclothymia
bipolar I disorder
bipolar III disorder

226
Bipolar 3 is a variant of bipolar disorder with minimal depression complicated by
antidepressant induced hypomania.
The correct answer is: bipolar III disorder

HiY Classification 033


Which of the following clinical phenomenon is associated with the name Angst?

Select one:
Folie a deux
Cycloid psychosis
Mixed affective states
Bipolarity of mood disorders
Unitary psychosis

Angst has been a strong proponent of bipolarity of mood disorders and the concept of bipolar
spectrum disorders.
The correct answer is: Bipolarity of mood disorders

HiY Classification 034


Which of the following delusions is least likely to be associated with a delusional disorder?

Select one:
Erotomanic delusions
Delusions of jealousy

Hypochondriacal delusions
Bizarre delusions
Persecutory delusions

Delusional disorders, unlike schizophrenia, are characterized by non-bizarre delusions. Note


that DSM-V has removed this distinction now as 'bizarreness' is difficult to define clinically.
The correct answer is: Bizarre delusions

HiY Classification 035


The term schizophrenia was coined by

227
Select one:
Kahlbaum
Freud
Kraeplin

Jung
Bleuler

Bleuler coined the term schizophrenia in 1911.


The correct answer is: Bleuler

HiY Classification 036


The most vocationally disabling personality disorder is

Select one:
Schizoid
Borderline
Antisocial
Paranoid
Obsessive Compulsive

People with either borderline or schizotypal personality disorder have significantly more
occupational impairment than patients with obsessive-compulsive personality disorder.
(Skodol 2002)
The correct answer is: Borderline

HiY Classification 037


A 40-year-old patient with a history of long-standing problems with hearing voices and feeling
under threat from the government now presents with waxy flexibility and resistance to all
instructions. He repeats whatever is said during the interview in a stilted voice but does not
speak spontaneously. The most likely diagnosis is which of the following?

Select one:
malingering
drug-induced psychosis
hebephrenic schizophrenia

228
catatonic schizophrenia

elective mutism

Catatonic schizophrenia is dominated by prominent psychomotor disturbances that may


alternate between extremes such as hyperkinesis and stupor, or automatic obedience and
negativism. Constrained attitudes and postures may be maintained for long periods. Episodes of
violent excitement may be a striking feature of the condition. The catatonic phenomena may be
combined with a dream-like (oneiroid) state with vivid scenic hallucinations (ICD-10)
The correct answer is: catatonic schizophrenia

HiY Classification 038


Dream-like (oneiroid) states with visual hallucinations are features of which of the following?

Select one:
Residual schizophrenia
Paranoid schizophrenia
Catatonic schizophrenia
Hebephrenic schizophrenia
Post schizophrenic depression

Catatonic schizophrenia is an uncommon subtype which is characterised by psychomotor


symptoms ranging from violent excitement through posturing, negativism, waxy flexibility,
perseveration to stupor. It is characterised by marked disturbance of motor behaviour and can
present in three clinical forms; (1) excited catatonia (2) stuporus catatonia and (3) catatonia
alternating between excitement and stupor. Oneiroid states are also reported.
The correct answer is: Catatonic schizophrenia

HiY Classification 039


You are seeing an elderly man with a history of low mood. He says he was feeling well until
18 months ago but progressively became low in his mood since then. He also complains of
becoming more forgetful over the last year. However, he has no problem sleeping and
appetite remains reasonably stable. Few months ago, he woke up suddenly with a limp and
has been suffering from it since then. What is the most likely diagnosis?

Select one:
Alzheimer's disease
Multiple sclerosis

229
Cerebro-vascular accident
Lewy body dementia
Depression

Vascular insufficiency in the elderly is often associated with both cognitive impairment and
depression that can be difficult to treat.
The correct answer is: Cerebro-vascular accident

HiY Classification 040


Which one of the following statements about Rett syndrome is true?

Select one:
There is apparent lack of normal development in the first 2 years of life Is
associated with microcephaly at birth
Occurs mainly in boys
Characterised by progressive lack of acquired abilities

Is associated with borderline IQ

Rett syndrome is a neurodevelopmental disorder that affects girls almost exclusively. It is


characterized by normal early growth and development followed by a slowing of development,
loss of purposeful use of the hands, distinctive hand movements, slowed brain and head
growth, problems with walking, seizures, and intellectual disability. From Rett Syndrome Fact
Sheet: National Institute of Neurological Disorders. Retrieved from
http://www.ninds.nih.gov/disorders/rett/detail_rett.htm
The correct answer is: Characterised by progressive lack of acquired abilities

HiY Classification 041


A lady comes to your outpatient unit with a diagnosis of OCD made by her GP. Which of the
following symptoms is most likely in this patient?

Select one:
Counting
Hoarding
ing
Rearranging items on the table

230
Cleaning

ing is one of the most common features of OCD. It is more common than other
symptoms listed in this question.
The correct answer is: ing

HiY Classification 042


A 12 yr old boy is found to be skipping school repeatedly and using drugs with his friends. He is
most likely to have which of the following diagnosable psychiatric conditions?

Select one:
Asperger's syndrome
Oppositional defiant disorder
Autism
ADHD

Conduct disorder

Conduct disorder is an enduring set of antisocial and aggressive behaviours that evolves over
time, usually characterized by aggression and violation of the rights of others.
The correct answer is: Conduct disorder

HiY Classification 043


A man who recently witnessed his colleague's death in an unfortunate road traffic accident
presents with weakness of both legs. He is not able to balance himself when asked to stand and
needs help to support him. On physical examination, no consistent neurological signs are noted.
The most likely diagnosis is

Select one:
Somatoform disorder

Somatisation disorder
Conversion disorder
Malingering
PTSD

231
This condition is best described as astasia-abasia. It is a conversion disorder.
The correct answer is: Conversion disorder

HiY Classification 044


A man brings his mother to stay with his family permanently, following her retirement. Soon
after this, his wife becomes mute and aphonic. Physical examination reveals no neurological
explanation. Most likely diagnosis is

Select one:
Somatisation disorder
Conversion disorder
Somatoform disorder
Malingering

Hypochondriasis

This condition is best described as hysterical or functional aphonia. It is a conversion disorder.


The correct answer is: Conversion disorder

HiY Classification 045


The diagnosis of induced delusional disorder is most commonly found in which of the
following groups?

Select one:
Family blood relations

Couple relationships
Children and adolescence
Groups of men rather than women
Groups of women rather than men

Induced delusional disorder is more common in a couple but is occasionally seen in groups. This
is a rare delusional disorder characterised by sharing of delusions between usually 2 or
occasionally more persons who usually have closely knit emotional bonds. Only one person has
genuine delusions due to underlying psychiatric disorder, most often schizophrenia or delusional
disorder. On separation, the dependent individual may give up his or her delusions and the
patient with the genuine delusions should be treated appropriately.

232
The correct answer is: Couple relationships

HiY Classification 046


A 32-year-old woman was brought by the Police to the A and E Department after she was found
attempting to gain access to a hotel room where a popular football player was staying. She
stated that the football player invited her into his hotel room and also claimed that they were
secretly married. The football player denied this but admitted that the woman had sent him
hundreds of emails over the last year. The woman had not been in trouble with the law so far
and was functioning well in the community. Which of the following is the most likely diagnosis?

Select one:
De-clerambault syndrome
Othello syndrome
Fregoli syndrome
Ekbom syndrome
Capgras syndrome

De-Clerambault syndrome is classified under persistent delusional disorders. It occurs most


often in women, and there is an erotic conviction that a person with higher status is secretly in
love with the patient. The patients make efforts to contact this person, and some cases are
associated with dangerous or assaultive behaviour.
The correct answer is: De-clerambault syndrome

HiY Classification 047


Choose the one feature which is not defined under Edwards and Gross criteria (1976) for
alcohol dependence syndrome.

Select one:
repeated withdrawals
reinstatement after abstinence
narrowed repertoire decreased
tolerance
subjective awareness of compulsion

233
Edwards and Gross criteria (1976) for alcohol dependence: 1. narrowed repertoire 2. salience of
alcohol seeking behaviour 3. increased tolerance 4. repeated withdrawals 5. Drinking to prevent
or relieve withdrawals. 6. subjective awareness of compulsion 7. reinstatement after abstinence

The correct answer is: decreased tolerance

HiY Classification 048


Following the death of her 85 year old mother, a 49-year-old receptionist has become
increasingly convinced that the prime minister is in love with her and that they have an
ongoing affair. She has clear thoughts and no hallucinations. She is doing well at work and
has no issues with her husband. Which of the following is the most likely diagnosis?

Select one:
delusional disorder acute
reactive psychosis
paranoid personality disorder

prodromal schizophrenia

schizophreniform disorder

Delusions without hallucination and thought disorder, along with preserved family and work
functioning is suggestive of a delusional disorder.
The correct answer is: delusional disorder

HiY Classification 049


A 42-year-old man believes that he has contracted a serious infectious condition when he
travelled to Burma, where his wife met with an accident. Since then he has had numerous tests
including the most sophisticated microbiological investigations but he is not convinced of being
healthy. He continues to feel low as a result for the last 1 year. What is the most appropriate
diagnosis?

Select one:
dysthymia
somatoform disorder

hypochondriasis

factitious disorder

delusional disorder

234
Hypochondriasis is an anxiety state. A patient with hypochondriasis may hold a belief as an
overvalued idea but not to a delusional intensity. The patient in question here seems to hold
the belief with absolute conviction; it is more likely to be a delusional disorder.
The correct answer is: delusional disorder

HiY Classification 050


A 33-year-old man has experienced low mood, fatigue and negative cognitions for the last 2
weeks. There is no personal or family history of mood disorders. He has been generally
healthy apart from an episode of flu that occurred 2 weeks ago. Which of the following is an
appropriate diagnosis?

Select one:
Chronic fatigue syndrome
Bipolar disorder
Encephalitis
No mental illness
Depressive episode

Low mood, fatigue and negative cognitions for the last 2 weeks is suggestive of a depressive
episode.
The correct answer is: Depressive episode

HiY Classification 051


A 40-year-old lady had an episode of depression 4 years ago. For last 4 weeks she is low in
mood, socially isolating herself and fears going out of home; she fears experiencing a panic
attack if she goes out. She is quite tearful and has not slept much for the last four weeks. The
most appropriate working diagnosis is

Select one:
Agoraphobia
Depressive relapse

Generalised anxiety
Dysthymia
Panic disorder

235
Low mood, social isolation, tearfulness, lack of sleep along with a past history of depression is
suggestive of a depressive relapse.
The correct answer is: Depressive relapse

HiY Classification 052


Characteristic features of delirium tremens include all except

Select one:
Diarrhoea
Fluctuant blood pressure and pulse
Hallucinations
Clouded consciousness
Agitation

Features of alcohol withdrawal: Usually starts within 12 hours of the last drink. Tremor,
diaphoresis, sleeplessness, GI distress and anxiety are prominent. Increased urge and
craving for alcohol may be seen. The severity of symptoms depends on the degree of pre-
existent drinking. If unattended, symptoms may peak in 48 hours. In these cases, seizures
may occur. These are grand-mal seizures especially common in those who had previous
seizures (epilepsy or withdrawal seizures), head injury and electrolyte imbalance such as
hyponatraemia or hypokalaemia. In around 5% patients, delirium tremens may set in with
disturbed autonomic functions (pulse, temperature and blood pressure changes on either
direction), clouded consciousness, hallucinations (often Lilliputian) and agitation.
The correct answer is: Diarrhoea

HiY Classification 053


Which of the following subtypes of schizophrenia is classified in DSM-IV but not ICD10?

Select one:
Hebephrenic schizophrenia
Undifferentiated schizophrenia

Disorganized schizophrenia
Paranoid schizophrenia
Catatonic schizophrenia

236
Disorganised schizophrenia is a term used in DSM but not ICD. In ICD-10, this subtype is
described as hebephrenic schizophrenia. Note that in DSM-V the subtyping of schizophrenia
has been dropped completely.
The correct answer is: Disorganized schizophrenia

HiY Classification 054


Which of the following is a neurological cause of catatonia?

Select one:
Hypercalcaemia
Adverse drug effect of a neuroleptic medication
Encephaltis
Hepatic encephalopathy
Adverse drug effect of phencyclidine

The most common psychiatric disorders causing catatonia include major depression and
schizophrenia. Catatonia could also appear as an adverse drug effect of a neuroleptic
medication or phencyclidine. Neurological causes of catatonia include parkinsonism,
encephalitis and certain brain tumours. The common medical causes include hypercalcaemia
and hepatic encephalopathy.
The correct answer is: Encephaltis

HiY Classification 055


A 23-year-old gentleman was started on an anti-psychotic medication 3 weeks ago. During a
follow-up appointment he reports no improvement in his symptoms. What is the most sensible
next step?

Select one:
Add another antipsychotic medication
Augmentation with an anticonvulsant medication

Explore potential non-adherence


Increase the dose of the antipsychotic medication
Change to another antipsychotic drug

237
The commonest cause for lack of improvement in psychotic patients is non-compliance with
antipsychotic medication and therefore it should be explored first. Even when adherence is
not an issue, at least 2-4 weeks of treatment with anti psychotics may be needed to see an
initial effect and obtain a full and optimal response to medication. The patient should be
treated for a minimal period of 6 months. The reasons for non-compliance may include
discomfort resulting from treatment (e.g.side effects), expense of treatment, decisions based
on personal value and judgements, religious or cultural beliefs about the advantages and
disadvantages of the proposed treatment, personality traits or adverse coping styles (e.g.,
denial of the illness) or the presence of a serious mental disorder (e.g schizophrenia).
The correct answer is: Explore potential non-adherence

HiY Classification 056


A patient suffers from florid racing of thoughts, decreased sleep, high energy levels and believes
aliens are putting thoughts into his brain. He also believes his nail bed is made soft by 'Bull-bull
bird'. His history suggests that he always had affective and psychotic disturbances occurring
simultaneously. Which of the following chapters will his diagnosis fall under in ICD?

Select one:
F00 organic
F20 schizophrenia
F30 mood disorders
Not coded
F40 neurosis

Schizoaffective disorder is placed with F20 not 30. Both schizophrenic and mood symptoms
are seen simultaneously.
The correct answer is: F20 schizophrenia

HiY Classification 057


A 28-year-old man presents for the seventh time to his GP with abdominal pain. When
confronted, he admits to faking symptoms. He says "I know this is wrong, but I don't know why I
do this". Which of the following is the most likely diagnosis for this patient?

Select one:
conversion disorder
somatization disorder

factitious disorder
hypochondriasis

238
malinger
ing

239
According to ICD-10 "the patient feigns symptoms repeatedly for no obvious reason and may
even inflict self-harm in order to produce symptoms or signs. The motivation is obscure and
presumably internal with the aim of adopting the sick role. The disorder is often combined with
marked disorders of personality and relationships."
The correct answer is: factitious disorder

HiY Classification 058


Choose one best option regarding the distribution of schizophrenia

Select one:
The incidence of schizophrenia is higher in developing countries
Schizophrenia is more common among women than men
Fathers of patients with schizophrenia belong to a higher social class than
the patients
2% of population has schizophrenia at any given time
The prevalence of schizophrenia is higher in developing countries

Social drift is associated with a diagnosis of schizophrenia.


The correct answer is: Fathers of patients with schizophrenia belong to a higher social class
than the patients

HiY Classification 059


Diagnostic features of neurasthenia include all of the following EXCEPT

Select one:
Duration >6 months
Easy fatiguability after minimal effort

Fatigue alleviated by prolonged rest


Muscular aches and pains
Poor (unrefreshing) sleep

240
CFS criteria US-CDCP (1994) suggests that to diagnose CFS (or neurasthenia in ICD10),
patients must have persistent or relapsing unexplained chronic fatigue. The fatigue must last
for at least 6 months and should not appear to be alleviated by rest.
The correct answer is: Fatigue alleviated by prolonged rest

HiY Classification 060


Global assessment of functioning forms which of the following axes in DSM-IV?

Select one:
One
Two
Three
Five
Four

The multi-axial System of DSM-IV used 5 axes. Axis I - Clinical Disorders. Axis II - Personality
Disorders/ Mental Retardation. Axis III - General Medical Conditions. Axis IV - Psychosocial
and Environmental Problems. Axis V - Global Assessment of Functioning. Note that DSM-5 has
moved to a nonaxial system for diagnosis, with separate notations for crucial psychosocial
factors (formerly Axis IV) and disability or imapirment (formerly Axis V).
The correct answer is: Five

HiY Classification 061


Which one of the following eponymous persistent delusional disorders are not correctly
matched?

Select one:
Capgras syndrome - illusion des sosies
Othello syndrome - morbid jealousy

Fregoli syndrome-delusional dysmorphophobia


Ekbom syndrome - delusion of infestation
De-clerambault's syndrome - erotomania

The persistent delusional disorders are characterised by a persistent, often life long, non-
bizarre delusion or a set of related delusions arising insidiously in mid-life or later. Transient
auditory hallucinations may occur, but schizophrenic symptoms are incompatible with a pure

241
diagnosis. Affect, thought and behaviour are globally normal, but patients' attitudes and
actions in response to these delusions are appropriate and may lead to dangerousness in
disorders such as Othello syndrome.
The correct answer is: Fregoli syndrome-delusional dysmorphophobia

HiY Classification 062


Fugue state has been recognised as a type of

Select one:
Situation specific psychogenic amnesia
False memory syndrome
Global psychogenic amnesia
Post traumatic amnesia
Transient global amnesia

Psychogenic amnesia is usually caused by psychological factors. There are two types - global
and situation specific. Fugue state is a type of psychogenic global amnesia in which there is a
sudden loss of all autobiographical memories, knowledge of self and personal identity.
Usually, there is a period of wandering, and there is an amnesic gap upon recovery. It usually
last a matter of hours or days. Memory recovers fully after few hours or days.
The correct answer is: Global psychogenic amnesia

HiY Classification 063


Which one of the following was a key proponent of the view that psychological diseases are
diseases of the brain?

Select one:
Greisinger
Adolf Meyer

Emil Kraepelin
Karl Jaspers
Charcot

Greisinger wrote an influential book and proposed the view that psychological diseases are
diseases of the brain. As a result, the old humoral theories were sidelined with new ideas about
the connection between mental illness and brain function and the pursuit of these links turned
successful for neurosyphilis, cretinism and dementia.

242
The correct answer is: Greisinger

HiY Classification 064


A 25-year-old man has experienced an irrational fear of darkness since childhood. He is not
distressed by this currently and does not take special measures to avoid dark spaces. Which
of the following is true?

Select one:
He has a specific phobia as he has an irrational fear
He has no specific phobia as fear of darkness is common
He has a specific phobia with loss of insight
He has no specific phobia as he does not have an avoidance behaviour

He has a specific phobia as he has it since childhood

Specific phobias are restricted to highly specific situations such as proximity to certain
animals, natural phenomenon such as heights, thunder, darkness, flying, being trapped in
closed spaces, urinating or defecating in public toilets, eating certain foods, dentistry, or the
sight of blood or injury. The trigger is often discrete, but symptoms of anxiety are similar to
panic attacks seen in other anxiety disorders
The correct answer is: He has no specific phobia as he does not have an avoidance
behaviour

HiY Classification 065


Which of the following is true concerning head injury and schizophrenia?

Select one:
Genetic predisposition to schizophrenia has no influence on vulnerability for
psychosis after head injury
Head injury increases risk of non-affective psychosis more than affective
psychosis
Psychosis related to head injury is usually acute in onset
Head injury increases likelihood of developing schizophrenia
Children are more susceptible to schizophrenia following head injury than
adults

243
Head injury, especially in teens, increases the risk of nonaffective psychosis including
schizophrenia.
The correct answer is: Head injury increases risk of non-affective psychosis more than
affective psychosis

HiY Classification 066


Which one of the following has the worst prognosis among the subtypes of schizophrenia?

Select one:
Residual schizophrenia
Catatonic schizophrenia
Post schizophrenic depression
Hebephrenic schizophrenia
Paranoid schizophrenia

The onset of hebephrenic schizophrenia is insidious, usually in the early second decade. The
course is often progressively downhill. There is a rapid development of negative symptoms and
disorganised behaviour. Severe deterioration without remission occurs over time. A recovery
from the first episode rarely occurs in typical cases.
The correct answer is: Hebephrenic schizophrenia

HiY Classification 067


Mr.Y is a 21-year-old gentleman who was admitted to a psychiatric inpatient unit 4 months ago.
His clinical presentation includes inappropriate affect, senseless giggling, mirror-gazing for long
periods of time, poor self-care and hygiene, marked thought disorder and severe loosening of
association. He also exhibits fragmentary and changeable delusions and hallucinations. Which
of the following is the most likely diagnosis?

Select one:
Post schizophrenic depression
Catatonic schizophrenia

Paranoid schizophrenia
Residual schizophrenia
Hebephrenic schizophrenia

244
Hebephrenic schizophrenia is characterised by marked thought disorder and severe loosening
of associations, Emotional disturbances characterised by inappropriate affect, blunted affect or
senseless giggling, abnormal mannerisms like mirror gazing. There is markedly impaired social
and occupational functioning; poor self care, poor hygiene, extreme social behaviour and
disorganised behaviour. ICD10 recommends a period of 2-3 months of continuous observation
for making a confident diagnosis.
The correct answer is: Hebephrenic schizophrenia

HiY Classification 068


A 40 year old lady has had an episode of depression 4 years ago. For last 4 weeks she is low in
mood, socially isolating herself and fears going out of home; she feels she will get a panic
attack if she goes out. She is quiet tearful and has lost much sleep over her symptoms for last 4
weeks. The most appropriate working diagnosis is depressive relapse; other differential
diagnoses are excluded in the above scenario before arriving at the correct diagnosis. This is
guided by which of the following principles of classification?

Select one:
Atheoretical principle
Hierarchical principle
Descriptive principle
Multiaxial principle
Operationalised principle

Hierarchical approach means that certain disorders take precedence over others while making
a diagnosis. This follows Jasperian hierarchy starting from organic disorders through to
substance use, psychoses, affective disorders, neuroses and personality issues. If a disorder
on the top of the hierarchy can explain presenting symptoms, then a diagnosis from the next
rung of the hierarchy should not be entertained even if some of the symptoms are consistent
with such a diagnosis.
The correct answer is: Hierarchical principle

HiY Classification 069


A 23-year-old man is hospitalized for a fever of unknown origin and persistent cough. He also
complains of being increasingly forgetful and having difficulties driving through familiar routes.
Physical examination reveals several needle marks in his forearms. The most likely diagnosis is

Select one:
HIV dementia
Prion disease

Cocaine induced brain damage

245
Endocarditis
Alcoholic dementia

Needle marks, persistent cough and fever of unknown origin are suggestive of a HIV infection.
Cognitive symptoms suggest HIV dementia.
The correct answer is: HIV dementia

HiY Classification 070


Which one of the following is not a feature of Wernicke's encephalopathy?

Select one:
Confusion
Memory disturbances
Ataxia

Nystagmus
Hyperthermia

The classical triad of ophthalmoplegia, ataxia and confusion is rarely present in Wernicke's
encephalopathy. A presumptive diagnosis of Wernicke's encephalopathy should, therefore, be
made in any patient undergoing detoxification who experiences any of the following signs;
ataxia, hypothermia and hypotension, confusion, ophthalmoplegia/ nystagmus, memory
disturbances and coma/unconsciousness.
The correct answer is: Hyperthermia

HiY Classification 071


A 53-year-old lawyer is concerned about a recent change in the nature of his heart rhythm. His
GP has referred him to various cardiologists who were all unable to find any abnormalities in
ECG, stress test, perfusion scan, echocardiogram and a 24 hour Holter monitoring. But the
lawyer feels that something sinister has been missed and continues spending many hours
consulting various books, journals and internet resources to understand his problems. He has
stopped working as a result. The most likely diagnosis is

Select one:
Conversion disorder
PTSD
Hypochondriasis

246
Munchausen disorder
Somatisation disorder

Hypochondriasis is characterised by an irrational and overvalued idea of harbouring a


physical health problem.
The correct answer is: Hypochondriasis

HiY Classification 072


Which one of the following is not known to be associated with an occurrence of psychotic
features during the episode?

Select one:
Bipolar Affective Disorder - current episode mania
Psychotic depression
Acute and transient polymorphic psychotic disorder

Hypomanic episode
Schizo Affective disorder

Psychotic features are not seen during a hypomanic episode. The clinical features of
hypomania include elated mood, increased energy levels, excitability, lack of sleep, over-
optimistic ideation and expansive mood.
The correct answer is: Hypomanic episode

HiY Classification 073


Asperger's syndrome is characterised by all of the following except

Select one:
Lack of friendships
Restrictive, repetitive or stereotyped patterns of behaviour
Impaired reciprocal social interaction
Impaired intellectual ability and syntactical speech

Lack of awareness of the feeling of others

247
Asperger's syndrome is characterised by severe persistent impairment in social interactions,
repetitive behaviour patterns, and restricted interests. IQ and language are normal. Unlike
autistic disorder, in Asperger's disorder no significant delays occur in language, cognitive
development, or age-appropriate self-help skills. According to DSM-IV, clinical features include
at least two of the following indications of qualitative social impairment: Markedly abnormal
nonverbal communicative gestures, the failure to develop peer relationships, the lack of social
or emotional reciprocity, and an impaired ability to express pleasure in other persons'
happiness. Restricted interests and patterns of behaviour are always present, but when they
are subtle, they may not be immediately identified or singled out as different from those of
other children. Mild motor clumsiness and a family history of autism may be present.
The correct answer is: Impaired intellectual ability and syntactical speech

HiY Classification 074


A woman with borderline personality disorder is keen to know which of her symptoms will get
better in the natural course of her life even if she does not take treatment. The correct answer
will be

Select one:
Identity disturbance
Feelings of abandonment
Impulsivity
Intensity of interpersonal relationships
Affective instability

Symptomatic improvement is common even among the most disturbed borderline patients, and
the prognosis for most, but not all, severely ill borderline patients is better than previously
recognized. Impulsive symptoms resolved the most quickly, affective symptoms were the most
chronic, and cognitive and interpersonal symptoms were intermediate when followed up for six
years. (Zanarini et al. 2003; The Longitudinal Course of Borderline Psychopathology,
http://ajp.psychiatryonline.org/doi/abs/10.1176/appi.ajp.160.2.274 (accessed April 19, 2015)).
The correct answer is: Impulsivity

HiY Classification 075


Which of the following traits is seen in paranoid personality disorder?

Select one:
Fear of abandonment and relying on other people
Inability to make decisions

Increased sensitivity to criticism and setbacks


Does not learn from mistakes
Strive for perfection

248
Patients with paranoid traits are often combative and highly suspicious in nature; this may
elicit a hostile response in others. They are often critical of others, but have great difficulty in
handling even a minor criticism from others.
The correct answer is: Increased sensitivity to criticism and setbacks

HiY Classification 076


Which of the following is true concerning cyclothymia?

Select one:
It is commonly associated with endocrine disturbances
The diagnosis can be made if there is a past history of bipolar disorder
Mood stabilisers are of no use in management
Hypomania is the most common reason for seeking help It

often fails to come to medical attention

Cyclothymia is a mild form of episodic affective disorder characterized by the presence of


numerous periods with hypomanic symptoms and numerous periods with depressive
symptoms (not satisfying the criteria for a major depressive episode) for a period of at least 2
years. Cyclothymia often fails to come to medical attention. After the initial 2 years (1 year in
children and adolescents) required for a diagnosis of cyclothymic disorder, there may be
superimposed manic or mixed episodes (in which case both bipolar I disorder and cyclothymic
disorder may be diagnosed) or major depressive episodes (in which case both bipolar II
disorder and cyclothymic disorder may be diagnosed). Cyclothymic disorder often begins early
in life. Lifetime prevalence rates for this disorder range between 0.4% and 1.0% in the general
population, and between 3% and 5% in mood disorder clinics. In community samples, the
cyclothymic disorder is equally common in men and women, but women are more prevalent in
clinical settings (in a ratio of approximately 3 to 2).
The correct answer is: It often fails to come to medical attention

HiY Classification 077


The term schizoaffective psychosis was coined by

Select one:
Kretschmer
Greisinger

Kraeplin

249
Kasanin
Kanner

The term schizoaffective psychosis was introduced by the Russian-born American psychiatrist
Jacob Kasanin in 1933. The earlier concepts of schizoaffective disorder regarded this diagnosis
as a good prognosis schizophrenia, with psychoanalytic explanations.
The correct answer is: Kasanin

HiY Classification 078


A 24-year-old lady suddenly develops bilateral blindness. She has patchy tunnel-like vision on
field testing. She appears not to be bothered by the problem. This is described as

Select one:
La belle indifference
Anosognosia
Loss of insight

Malingering
Anton's syndrome

Conversion / hysterical disorder is called dissociative disorder of motor movement and


sensations. The degree of disability in this disorder is very variable. La belle indifference is not
universal, but common. The patients are distressed or concerned by the disability. Close
friends or relatives might have had the actual organic illness whose symptoms are present in
conversion disorder patient. A milder and transient variety is seen in adolescent girls
The correct answer is: La belle indifference

HiY Classification 079


The increase in recall of the perceived threat of a natural disaster over time is associated with

Select one:
Gender
Social support
Lack of improvement in PTSD over time
Magnitude of traumatic experience
Personality

250
Post-traumatic stress disorder (PTSD) diagnosis often depends on a retrospective, self-report
of exposure to a life-threatening event. Recalled threat intensity generally increases from the
time of trauma. This amplification in recall intensity is associated with lack of PTSD symptom
improvement .Heir et al. The British Journal of Psychiatry (2009) 194: 510-514
The correct answer is: Lack of improvement in PTSD over time

HiY Classification 080


Which one of the following symptoms is most frequently noted in schizophrenic patients?

Select one:
Auditory hallucinations
Ideas of reference
Suspiciousness
Flatness of affect
Lack of insight

The International Pilot Study of Schizophrenia survey determined the commonest symptoms
exhibited by 306 schizophrenic patients in 9 countries as follows: Lack of insight = 97%;
Auditory hallucinations ­ 74%; Ideas of reference ­ 70% .
The correct answer is: Lack of insight

HiY Classification 081


Mr X is a 31-year-old gentleman who has suffered three manic episodes and 1 episode of
depression in the past. He is already on antipsychotics and antidepressants. When reviewed in
the out-patient clinic, he complains of low mood, frequent crying spells, poor concentration and
lack of enjoyment in things he used to do. Which one of the following is the most appropriate
medication for his condition?

Select one:
Amitriptyline
Lamotrigine

Venlafaxine
Sertraline
Mirtazapine

251
This gentleman is suffering from bipolar depression which is a common and debilitating
disorder which differs from unipolar disorder in severity, time, course, recurrence and
response to drug treatment. Episodes of bipolar depression are more rapid in onset, more
severe, but often shorter in nature compared to unipolar depression. Lamotrigine appears to
be effective both as a treatment for bipolar depression and as a prophylaxis against further
episodes. It does not induce switching or rapid cycling. Antidepressant monotherapy is not
recommended due to the risk of precipitating a switch to mania.
The correct answer is: Lamotrigine

HiY Classification 082


Which one among the following is NOT a core diagnostic feature of ADHD?

Select one:
None of the above
Inattention
Hyperactivity
Impulsivity
Language dysfunction

The cardinal features of ADHD are excessive and impairing levels of activity, inattention, and
impulsiveness. Hyperactivity-impulsivity symptoms include: fidgeting, moving, getting up and
running about, talking excessively; unable to play quietly; continually interrupting. Inattention
symptoms include: being easily distracted, poor attention span and not able to sustain
attention; poor task completion; inability to organize, and making mistakes with tasks that
require attention; the child is forgetful, and often loses items of daily use.
The correct answer is: Language dysfunction

HiY Classification 083


Identify the feature seen only in hysterical but not organic amnesia.

Select one:
Intact procedural memory
All of the above

Intact semantic memory


Loss of personal identity
Impaired episodic memory

252
Loss of personal identity is a unique feature seen in hysterical amnesia. Patients are unable to
give their name, address or any personal information. This amnesia essentially cuts them from
the previous life. In most cases, he amnesia is transient and the fugue state clears over a few
days. If not, the patient usually adopts a new name and identity and begins a new life. As in
organic amnesia, those with fugue normally retain their procedural and semantic memories. The
patient may have episodic memory loss which is usually only retrograde memory loss, and no
anterograde memory impairment is reported.
The correct answer is: Loss of personal identity

HiY Classification 084


Which of the following is a poor prognostic factor for schizophrenia in terms of
relapse/recovery from the illness?

Select one:
Late onset
Higher premorbid functioning
Presence of affective symptoms
Male gender
Acute onset

Males have distinctly poorer prognosis than females with schizophrenia. This is thought to be
linked to the early age of onset in males.
The correct answer is: Male gender

HiY Classification 085


A 34-year-old woman appears flamboyant and dramatic in her behaviour, drawing excessive
attention to herself. Which of the following is very likely?

Select one:
Perfectionist behaviour
Marital problems related to emotional intimacy

Rapid mood swings


Anxiety in social situations
Preoccupation with rejection

253
This is histrionic personality disorder. It is associated with problems related to emotional
intimacy.
The correct answer is: Marital problems related to emotional intimacy

HiY Classification 086


Which of the following is not a common clinical feature of mania?

Select one:
Reduced need for sleep
Irritability
Memory impairment
Physical over activity
Elated, expansive mood that is not characteristic of the individual

The clinical features of mania includes elated, expansive mood that is not characteristic of the
individual, irritability, physical over activity/restlessness, pressure of speech, flight of ideas,
disturbed sleep or reduced need for sleep, increased libido, inflated self esteem, over spending,
over-familiarity and psychotic symptoms. Grandiose delusions are typical, auditory
hallucinations and other delusions including first rank symptoms may also be present. It is
equally common in men and women. The mean age of onset is in the early 20's. The earlier the
onset perhaps, the greater the genetic loading. It is more common in urban areas and equally
distributed among the social classes. The lifetime risk of bipolar disorder is about 1%.
The correct answer is: Memory impairment

HiY Classification 087


A 35-year-old man has low mood, anhedonia, loss of hope on the future, loss of appetite and
poor sleep. He has no other symptoms of depression. What is the severity of his depression?

Select one:
Mild
Dysthymia

Severe
No depression
Moderate

254
For mild depressive episodes, two symptoms from Criterion A i.e. 'core symptoms' + at least
two from Criterion B - 'other symptoms', totaling at least 4 overall, must be present. For
moderate depression, the count required is 2 + 3 / 4 and for severe it becomes 3 + 4 / 5. '4
mild -6 moderate -8 severe' is an easy way to remember these criteria!
The correct answer is: Moderate

HiY Classification 088


Which of the following is true regarding persistent delusional disorder according to ICD 10
diagnostic criteria?

Select one:
No hallucinations must be present
Only single theme of delusion must be present.
More than 3 months duration of delusions
Delusions incongruent to mood
Preceding stressor should be identifiable

The persistence in persistent delusional disorder is defined as a minimum period of 3 months.


The correct answer is: More than 3 months duration of delusions

HiY Classification 089


A 32-year-old ex-nurse often presents to A and E with a plethora of complaints. She has
undergone many surgeries in the past. During one such admission, when confronted, she
admits to injecting faeces into her blood in order to produce septic blood culture for no
apparent reason. What is the most likely diagnosis?

Select one:
Malingering
Dissociative disorder

Munchausen syndrome
Conversion disorder
Somatisation disorder

255
Munchausen syndrome is characterised by self-inflicted lesions and feigned illness with the
goal of receiving medical treatment rather than for any other gain.
The correct answer is: Munchausen syndrome

HiY Classification 090


Which of the following personality disorders is included in DSM but not ICD?

Select one:
Schizoid
Paranoid type
Anxious - avoidant
Histrionic
Narcissitic

According to DSM-V, narcissitic personality disorder is characterised by impaired self functioning


(excessive reference to others for self­definition and self­esteem regulation; exaggerated
self-appraisal varying between inflated or deflated extremes), impaired self- direction
(goal-setting based on gaining approval from others; personal standards are unreasonably high
in order to see oneself as exceptional, or too low based on a sense of entitlement; often
unaware of own motivations), impaired empathy (poor ability to recognize or identify with the
feelings and needs of others; excessively attuned to reactions of others, but only if perceived as
relevant to self; over­ or underestimate of own effect on others) and impaired intimacy (
superficial relationships that primarily serve self­esteem regulation; mutuality constrained by little
genuine interest in others; experiences and predominance of a need for personal gain); and
increased grandosity and attention-seeking traits that antagonise others.

The correct answer is: Narcissitic

HiY Classification 091


A 21-year-old woman often falls asleep during the day. She has fallen asleep while speaking to
her manager twice, causing problems at work. Her night time sleep is undisturbed, and other
spheres of functioning are satisfactory. Physical health is unremarkable except for a history of
an episode of fall 2 weeks ago when she was on a train laughing heartily with her boyfriend.
She did not sustain any head injury then. Which of the following is the most appropriate
diagnosis for this case?

Select one:

narcolepsy
hypersomnia not otherwise specified

primary hypersomnia
circadian rhythm sleep disorder

256
no illness

Narcolepsy is characterised by the presence of both cataplexy and daytime sleepiness.


Emotional stimuli such as laughter, startle, excitement, or anger can precipitate catalepsy.They
tend to fall due loss of tone in anti-gravity muscles of the legs. However, most attacks are mild
and last only a few seconds.
The correct answer is: narcolepsy

HiY Classification 092


The duration criteria set out in ICD-10 for diagnosing personality disorders is

Select one:
6 months
2 years
3 years
None of the above

1 year

There is no duration criterion for diagnosing personality disorders, though the term 'enduring' is
applied in descriptions.
The correct answer is: None of the above

HiY Classification 093


Which of the following is correct concerning atypical anorexia nervosa?

Select one:
All symptoms of anorexia are present but with milder severity
Menarche is never attained

Weight is normal
Depression is not present
Normal body image perception

257
According to ICD, in atypical anorexia, most features of anorexia are present except that one of
the key features such as amenorrhea or altered body image may not be seen. (Please note -
DSM defines atypical anorexia differently; According to DSM this term is used for those who
have all features of Anorexia but absolute current weight above or within normal range).
http://apps.who.int/classifications/apps/icd/icd10online2005/fr-icd.htm?gf50.htm+
The correct answer is: Normal body image perception

HiY Classification 094


Classificatory systems can be used to make a diagnosis based on inclusion and exclusion
criteria. Which of the following properties of classificatory systems aid in the above?

Select one:
Operationalised approach
Atheoretical approach
Multiaxial approach
Aetiological approach
Descriptive approach

From DSM-III onwards, operationalized diagnosis was introduced. This means using intensity,
duration of the symptoms and impairment criteria while making a diagnosis in an algorithmic
manner. Using a diagnostic list more or less, some criteria are necessary while some are
optional for a diagnosis. Characteristic symptoms are pertinent to the diagnosis, such as the
symptom of depression that is found in many different disorders. Discriminating symptoms, e.g.
thought insertion are important for diagnosis since they are not found in other disorders. A
hierarchy of symptoms, arranged in order of importance often accompanies each diagnostic
description in operationalised systems. Additionally inclusion and exclusion criteria may be
described to establish the diagnosis
The correct answer is: Operationalised approach

HiY Classification 095


The most common co-morbid psychiatric disorder seen in children with ADHD is

Select one:
Conduct disorder
Oppositional defiant disorder

Bipolar disorder
Substance abuse
Specific learning disorders

258
Co-morbid psychiatric illness is common in ADHD children and is seen in 50-80% of cases.
50% children may meet criteria of 2 comorbid conditions. Oppositional Defiant disorder in 35-
50%, conduct disorder 25%, anxiety disorder 25% and depressive disorder 15%. Learning
disability 15-40%, specific language impairment in 15-75% children.
The correct answer is: Oppositional defiant disorder

HiY Classification 096


Mr.X is a 29-year-old gentleman who has a 6 month history of persecutory delusions about
being spied on at home by his neighbours. There is no evidence of functional impairment. There
is no history of past psychiatric illness or substance misuse. His most likely diagnosis is

Select one:
Paranoid personality disorder
Simple schizophrenia
Schizophrenic form disorder
Persistent delusional disorder
Paranoid schizophrenia

Persistent delusional disorder is a category in ICD 10 that includes all disorders in which
persistent delusions are the prominent and most important clinical features and delusions must
be present for at least 3 months. It includes delusions of persecution, delusions of grandeur,
delusions of jealousy, somatic delusions or other non- bizarre delusions. There should be a clear
absence of significant or persistent hallucination and absence of organic mental disorders,
schizophrenia and mood disorders. Very often, the individuals carry on near normal and social
and occupational life without arousing suspicion regarding the delusional disorder.

The correct answer is: Persistent delusional disorder

HiY Classification 097


A child is suspected to have Asperger's syndrome. Which of the following is not consistent
with the diagnosis of Asperger's syndrome?

Select one:
Restricted interests
Clumsy motor coordination

Abnormalities in reciprocal social interaction


Stereotyped motor activities

259
Poor speech development till age 3

Language development is typically normal in Asperger's syndrome, differentiating this


condition from other autistic spectrum disorders. Note that in DSM-V, Asperger's syndrome is
no longer considered as a separate diagnostic entity.
The correct answer is: Poor speech development till age 3

HiY Classification 098


A patient with schizophrenia experienced his most recent relapse 8 months ago. He currently
has low mood, anhedonia, flat affect, apathy and suspiciousness over religious people. For the
last 2 weeks he has not eaten well, currently avoids eye contact and appears tearful. Which of
the following diagnosis is appropriate?

Select one:
Simple schizophrenia
Residual schizophrenia
Mixed bipolar episode
Post schizophrenic depression
Paranoid schizophrenia

To diagnose post-schizophrenic depression (ICD10), some schizophrenic symptoms (mostly


negative) must be present though not dominating the picture. The most recent episode of
relapse must not be more than 12 months ago. If no current schizophrenic symptoms at all the
depression can be diagnosed. If florid schizophrenia symptoms with minor affective
disturbances noted, then relapse must be suspected
The correct answer is: Post schizophrenic depression

HiY Classification 099


Which of the following subtype of schizophrenia is diagnosed when a psychotic episode is
followed by minimal or reduced symptoms present for at least a year?

Select one:
Paranoid schizophrenia
Residual schizophrenia

Post schizophrenic depression


Catatonic schizophrenia

260
Hebephrenic schizophrenia

According to the ICD 10 diagnosis residual schizophrenia is characterised by the following


features in additional to the general guidelines of schizophrenia which includes prominent
negative schizophrenic symptoms, evidence in the past of at least one clear-cut psychotic
episode meeting the diagnostic criteria for schizophrenia, a period of at least one year during
which the intensity and frequency of florid symptoms such as delusions and hallucinations have
been minimal or substantially reduced and absence of organic brain disease or disorder.

The correct answer is: Residual schizophrenia

HiY Classification 100


Who first described Bulimia Nervosa?

Select one:
Crow
Liddle
Russell
Greisinger

Klerrmann and Weissmann

Russell first described Bulimia in 1979. Greisinger described unitary psychosis in 1868. Crow
published his two syndrome (Type 1 and 2) hypothesis of schizophrenia in 1980. Liddle
described the three-syndrome model of schizophrenia in 1987. Klerrmann and Weissmann
introduced interpersonal psychotherapy.
The correct answer is: Russell

HiY Classification 101


The term that refers to the neglect of all leisure and other alternate forms of pleasure apart
from alcohol is

Select one:
Dependence

Tolerance
Narrowing of repertoire
Reinstatement

261
Salience

Salience refers to the neglect of all leisure and other alternate form of pleasures apart from
alcohol. One's life revolves around getting alcohol, storing it, saving money to buy it, making
opportunities to drink, etc. Narrowing of repertoire: A social drinker may drink in various
situations at various places and may have various drinks of his choice. A dependent drinker will
settle into a fixed setting, sticking to one beverage. This may help him maintain a steady level of
alcohol in the blood without much fluctuation. Note that some widening of repertoire is inevitable
in the career of an alcohol drinker - moving from weekends only to all days of the week, etc.

The correct answer is: Salience

HiY Classification 102


Which of the following disorders is believed to represent a point on a continuum of 'unitary
psychosis' lying quantitatively between schizophrenic and mood symptoms?

Select one:
Organic mood disorders
Schizoaffective disorder
Bipolar affective disorder
Psychotic depression

Post schizophrenic depression

In schizoaffective disorder, the symptoms of affective disorder and schizophrenia are present
in approximately equal proportion. Both schizophrenic and affective symptoms are present
simultaneously present and are equally prominent. The aetiology is assumed to be
intermediate to that of schizophrenia and affective disorder. The depressive symptoms are
more likely to signal a chronic course compared to manic presentations.
The correct answer is: Schizoaffective disorder

HiY Classification 103


Mr.Z was admitted to a psychiatric ward 7 days ago. On mental state examination, he
appeared thought disordered and also had mood-incongruent delusions and hallucinations. His
mood symptoms were less marked at this stage. What is the most appropriate diagnosis?

Select one:
Schizophrenia

Secondary Mania

262
Bi Polar disorder
Unipolar depression
Schizo Affective disorder

In schizophrenia, thought and perceptual disorders are more prominent while mood symptoms
are less marked in the initial stages. Delusions and hallucinations are stable. Mood incongruent
hallucinations and behaviour are the norm. The patient is incapable of behaving fully consistent
with their delusional ideas.
The correct answer is: Schizophrenia

HiY Classification 104


In ICD-10, schizotypal disorder is classified under

Select one:
Disorders of psychological development
Schizophrenia and related disorder

Disorders of adult personality and behaviour


Cluster A personality disorders
Persistent delusional disorder

Schizotypal disorder is classified along with schizophrenia and related disorders in ICD-10
but along with Cluster A personality disorders in DSM-5. Schizotypy shares some of the
clinical features of schizophrenia, but not the delusions or hallucinations. It is thought to be
related to schizophrenia because schizotypy is more common in the other first-degree
relatives of schizophrenic subjects than in the general population and the relatives for
schizotypal subjects have an increased risk of schizophrenia.
The correct answer is: Schizophrenia and related disorder

HiY Classification 105


Which of the following personality types is classified as a Cluster A personality disorder?

Select one:
Anankastic personality disorder
Histrionic personality disorder
Avoidant personality disorder

263
Schizotypal personality disorder
Antisocial personality disorder

Paranoid, schizoid and schizotypal personality disorders are classified under cluster A.
The correct answer is: Schizotypal personality disorder

HiY Classification 106


Which of the following traits in a woman with borderline personality disorder would resolve at
least partially over 2 years time with or without treatment?

Select one:
Affective instability
Anger control
Emptiness

Self-harm
Unstable relationships

Collaborative Longitudinal Personality Disorders Study (CLPS) by Gundersen et al. reported


that self-harm improves longitudinally over a period of more than 2 years in those with
borderline personality disorder.
The correct answer is: Self-harm

HiY Classification 107


In PTSD, which of the following is associated with a poor prognosis?

Select one:
Vivid memory of the trauma
Reduced threat intensity of recalled trauma
Florid emotional response at the time of trauma
Sense of numbing
Unemployment at the time of trauma

264
Dissociative symptoms such as sense of numbing, detachment or absence of emotional
responsiveness, a reduction in awareness of one's surroundings e.g. 'being in a daze',
derealisation, depersonalisation, dissociative amnesia (inability to recall an important aspect of
the trauma) are associated with poor prognosis.
The correct answer is: Sense of numbing

HiY Classification 108


A 42­year­old learning disabled man lives in a sheltered accommodation; he attends day
centre twice a week and gets regular visitors. He cannot write or read but can speak using
restricted phrases. He needs supervision for daily activities and cannot be trained easily; His
social worker helps him with finances. What level of mental retardation does he have?

Select one:
below average
Moderate
Severe
mild

Profound

Speech minimal; Can talk or learn to communicate. Generally unable to profit from training in
self-help. May contribute partially to self-maintenance under complete supervision later in life.
Approximate IQ range of 20 to 34 (in adults, mental age from 3 to under 6 years). Likely to result
in a continuous need of support.
The correct answer is: Severe

HiY Classification 109


A middle-aged woman went with her husband to see her psychiatrist. She hasn't been sleeping
or eating well for the last 4 weeks. She also complains that she feels her bowels are dead and
are no longer working. According to ICD-10 the severity of her depression can be classified as

Select one:
Dysthymia
Moderate
Mild
Severe
She has no depression

265
All depressive episodes with psychotic symptoms are classed as severe according to ICD-10.
The correct answer is: Severe

HiY Classification 110


A 31-year-old woman 6 days postpartum has not been sleeping well. She has been wandering
in the middle of the night. She is ignoring her baby, and reports that "My child is a sign of a
terrible disaster about to happen to the world". Which of the following is the most likely ICD10
diagnosis?

Select one:
Post traumatic stress disorder
Severe puerperal disorder
Major depressive disorder with psychotic features
Delirium
Postpartum blues

This scenario is suggestive of puerperal psychosis, which is classified under F53.1(Severe


mental and behavioural disorders associated with the puerperium, not elsewhere classified)
The correct answer is: Severe puerperal disorder

HiY Classification 111


A 22-year-old woman presents with intense emotions, repeated wrist-cutting, affective
instability, and unstable relationships. Select the most likely risk factor for her diagnosis.

Select one:
Alcohol abuse
Sexual abuse

Bullying at school
Drug abuse
Death of mother before the age of 12

This is a case of borderline personality disorder. Many patients with borderline traits have
suffered physical and sexual abuse and exposure to chaotic parenting during childhood.

266
The correct
answer is:
Sexual abuse

267
HiY Classification 112
Which uncommon subtype of schizophrenia is characterised by an insidious development of
negative symptoms without evidence of positive symptoms?

Select one:
Hebephrenic schizophrenia
Catatonic schizophrenia
Residual Schizophrenia

Paranoid schizophrenia
Simple schizophrenia

Simple schizophrenia is characterised by an early onset (usually in the second decade), very
insidious and progressive course, presence of characteristic negative symptoms like marked
social withdrawal, loss of initiative and drive and shallow emotional response. People with this
condition drift down the social ladder quickly, living shabbily and wandering aimlessly.
Delusions and hallucinations are usually absent; if present, they are short lasting and poorly
systematised. The prognosis is usually very poor.
The correct answer is: Simple schizophrenia

HiY Classification 113


A 13-year-old boy was sent from school by his teacher as he smelt of alcohol. The GP refers
the family to child psychiatric services where his dad reports that his son is less interested in
exploring new things. He never joins in on a conversation when there are visitors at home; he
gets distressed and sweats profusely when attending parties and other occasions. Most likely
diagnosis is

Select one:
Specific phobia
Alcohol dependence
Agoraphobia
Depression
Social phobia

Social phobia is characterised by "an intense, irrational fear of one or more social or
performance situations in which the individual believes that he or she will be scrutinized by
others. Exposure to social situations immediately provokes an anxiety response. In adults, the
social phobia is recognized as excessive or unreasonable." (ICD-10)
The correct answer is: Social phobia

268
HiY Classification 114
Most children with selective mutism also meet diagnostic criteria for which of the following
disorders?

Select one:
Depression
Asperger's syndrome
Generalised anxiety disorder

Autism
Social phobia

Selective mutism is a disorder characterized by persistent failure to speak in specific settings


(school) despite full use of language at home or with family, may be found in younger children
with social phobia. Selective mutism appears to be more common in girls than in boys.
Approximately 68% of children with selective mutism also meet diagnostic criteria for social
phobia.
The correct answer is: Social phobia

HiY Classification 115


Which of the following is true about patients with anorexia nervosa?

Select one:
The illness has less mortality rate than schizophrenia
The most common cause for presentation is comorbid personality disorder
The incidence is 35% in the monozygotic twins of affected patients
The incidence is 35% in the dizygotic twins of affected patients
There is no association between social class and development of anorexia

Though earlier studies showed a bias towards higher social class, it is increasingly
recognised that this is not accurate.
The correct answer is: There is no association between social class and development of
anorexia

269
116 HiY Classification 116

270
Which one of the following can be a discriminating symptom while making a clinical
diagnosis?

Select one:
Memory problems
Poor concentration
Thought withdrawal

Panic attacks
Anhedonia

Thought withdrawal is a first rank symptom for a diagnosis of schizophrenia.This is considered


to be a discriminating symptom while making a clinical diagnosis.
The correct answer is: Thought withdrawal

HiY Classification 117


The multi-axial version of ICD consists of how many axes?

Select one:
Five
Four
Three
Ten
One

The multi-axial version of ICD-10 uses three axes -Axis 1 - the mental disorder (also
personality disorder and mental handicap); Axis 2 ­ the degree of disability; and Axis 3 ­
current psychosocial problems
The correct answer is: Three

HiY Classification 118


The onset of autistic disorder is usually before the age of

Select one:
Three
Eight

271
Four
Five
Two

By definition, the onset of autistic disorder is before the age of 3 years, although in some cases,
it is not recognized until a child is much older. Autistic disorder is four to five times more frequent
in boys than in girls.
The correct answer is: Three

HiY Classification 119


Different types of panic disorder include all except

Select one:
Nocturnal panic attacks
Situationally predisposed attacks

Unilateral panic attack


Situational panic attacks
Out of blue panic attacks

Panic attacks cannot be unilateral. They are associated with a generalised autonomic activity
and not directly related to cortical seizures that could be unilateral.
The correct answer is: Unilateral panic attack

HiY Classification 120


The concept that overwhelming stress could produce mental illness led to the introduction of
which of the following terms in the early 20th century?

Select one:
Somatisation
Alexithymia
Moral insanity
War neurosis
Precocious degeneration

272
War neurosis was the forerunner of the present day concept of Post Traumatic Stress
Disorder.
The correct answer is: War neurosis

HiY Classification 121


Which one among the following symptoms is not classified under Schneiderian first rank
symptoms of schizophrenia?

Select one:
Thought insertion
Delusional perception
Waxy flexibility
Third person singular hallucinations
Thought broadcasting

Waxy flexibility is classified under Catatonia. Here parts of the patients' body can be placed in
positions that will be maintained for long periods of time even if very uncomfortable and the
patient is flexible like a wax.
The correct answer is: Waxy flexibility

HiY Classification 122


Which of the following culture-bound syndromes refer to fear of turning into a cannibal?

Select one:
Windigo
Koro

Susto
Piblokto
Latah

Windigo involves an intense craving for human flesh and the fear that one will turn into a
cannibal.

273
The correct answer is: Windigo

HiY Classification 123


One of your patients has a co-morbid paranoid personality disorder with a persistent
delusional disorder. Identify the correct statement regarding persistent delusional disorders.

Select one:
Depressive episodes invalidate the diagnosis Delusional
disorders are associated with blunted affect.
The delusions must be present for a period of at least 3 months
Passivity phenomenon is an important feature
Persistent hallucinations are notable in most but not all patients

The persistent delusional disorders are characterized by chronic, often life-long, non-bizarre
delusions often arising insidiously in midlife or later. Transient auditory hallucinations may occur
but clear and persistent auditory hallucinations (voices), schizophreniform symptoms such as
delusions of control and blunting of affect, or the presence of an organic brain disease are all
incompatible with this diagnosis. There is no necessity to have an identifiable stressor.
Symptoms should have been present for at least 1 month (DSM-IV). ICD-10 specifies at least 3
months for a diagnosis of delusional disorder.
The correct answer is: The delusions must be present for a period of at least 3 months

HiY Classification 124


Which of the following is a correct statement regarding anorexia?

Select one:
Anorexic women are more likely to have a history of childhood sexual abuse
compared to depressed women
A significant heritability exists for bulimia but not anorexia

The concordance rates for monozygotic twins is higher than dizygotic twins
Persistent preoccupation with eating is a characteristic feature
Social class distribution of prevalence of anorexia is even

The concordance rates of anorexia nervosa for monozygotic twins 55% and 5% for dizygotic
twins. Early studies found a 30% rate of sexual abuse in patients with eating disorders but later
studies showed no excess of childhood physical or sexual abuse (compared to psychiatric
controls). Family studies have shown that there is an increased incidence of eating

274
disorders in first-degree relatives (7-12 times higher). A significant heritability exists for anorexia
nervosa but not for bulimia nervosa. Persistent preoccupation with eating is a characteristic
feature of bulimia. The class distribution is even in bulimia, but anorexia is more frequently seen
in people of higher social class, at least among those who seek help from professionals.

The correct answer is: The concordance rates for monozygotic twins is higher than dizygotic
twins

HiY Classification 125


Which of the following is a characteristic feature of narcolepsy?

Select one:
Catalepsy
Sleep walking
Sleep paralysis
Apnea
Hypnopompic hallucinations

The cardinal symptoms of narcolepsy include excessive daytime drowsiness with sudden onset
of REM sleep, often accompanied by cataplexy (sudden loss of muscle tone, provoked by
strong emotion). Sleep paralysis and hypnagogic hallucinations may also occur albeit less
commonly.
The correct answer is: Sleep paralysis

HiY Classification 126


Which of the following statements regarding PTSD is true?

Select one:
Specific EEG changes occur in chronic PTSD
Previous history of neurotic illness precludes the development of PTSD
features
PTSD is associated with hypersomnia
PTSD symptoms usually occur within 6 months of the perceived trauma
Autonomic hyperarousal symptoms are usually rare

275
PTSD symptoms usually occur within 6 months of the perceived trauma. The essential
features are autonomic hyperarousal, re-experiencing of aspects of the stressful event and
active avoidance of cues that remind of the trauma. Anhedonia may also be seen; dramatic
acute bursts of anxiety may present as exaggerated startle and hyperarousal. Insomnia is
common. Predisposing factors include maladaptive personality traits (e.g. anankastic) or
previous history of neurotic illnesses.
The correct answer is: PTSD symptoms usually occur within 6 months of the perceived trauma

HiY Classification 127


Which of the following is included as a category of schizophrenia in ICD10?

Select one:
Schizotypal disorder
Paranoid personality disorder
Schizoaffective disorder
Post-schizophrenic depression
Psychotic depression

Post-schizophrenic depression (F20.4) is described along with other schizophrenias in


ICD10.
The correct answer is: Post-schizophrenic depression

HiY Classification 128


Ganser syndrome is included in ICD under which of the following chapters?

Select one:
Reaction to stress and adjustment disorder
Anxiety disorders

Somatoform disorders
Other neurotic disorders
Other dissociative disorders

Ganser syndrome is included in ICD under other dissociative disorders - F44.80.

276
The correct
answer is:
Other
dissociative
disorders

277
HiY Classification 129
A 30-year-old man is involved in a motorcycle accident, in which his girlfriend (passenger) dies.
He presents with paralysis of his left arm, stating he cannot ride a motorcycle again. The
doctors find no organic cause. What is the most appropriate diagnosis?

Select one:
Motor dissociative disorder
Acute stress reaction
Adjustment disorder

Hypochondriasis
Neurasthenia

Dissociative disorders are presumed to be "psychogenic" in origin, being associated closely in


time with traumatic events, insoluble and intolerable problems, or disturbed relationships. The
onset and termination of dissociative states are often reported as being sudden, and the
symptoms usually develop in close relationship to psychological stress. The commonest
varieties are loss of ability to move the whole of part of a limb or limbs. Paralysis may be partial,
with movements being weak or slow, or complete. Various forms and variable degrees of
incoordination (ataxia) may be evident, particularly in the legs, resulting in bizarre gait or inability
to stand unaided (astasia-abasia).
The correct answer is: Motor dissociative disorder

HiY Classification 130


A 21-year-old man has been elated for the last 2 weeks with reduced need for sleep. On
mental state examination, he has pressured speech, racing thoughts and grandiose
delusions. The diagnosis is

Select one:
Acute stress reaction
Hypomania with psychosis
Mania with psychosis
Hypomania without psychosis
Mania without psychosis

The symptoms have lasted for more than a week, which is suggestive of mania. In its more
severe form, mania may be associated with psychotic symptoms such as grandiose delusions.
The correct answer is: Mania with psychosis

278
HiY Classification 131
A 55-year-old woman with suspected multiple sclerosis presents with loss of function in one
arm. She does not seem concerned by this defect. This is best described as

Select one:
Echo de la pensee
Moria
Palimpsest

Bouffee delirante
La belle indifference

La belle indifference is a surprising lack of concern for, or denial of, apparently severe
functional disability. It is often seen in patients with hysteria and medical illness e.g. a
cerebrovascular accident. It is a rare and non-specific symptom of no diagnostic value.
The correct answer is: La belle indifference

HiY Classification 132


Which one of the following is an ICD-10 diagnostic feature of frontotemporal dementia?

Select one:
Insidious onset and gradual progression
3-6 months history of cognitive decline
Memory impairment precede behavioural manifestation
Older age of onset
Abrupt onset and rapid progression

The ICD-10 diagnostic features of fronto-temporal dementia include a progressive dementia


commencing in late middle life, with slowly progressing changes of character and social
deterioration, followed by impairment of intellect, memory and language functions. The
behavioural manifestations commonly precede memory impairment.
The correct answer is: Insidious onset and gradual progression

HiY Classification 133


A man who has always had an anxious disposition has been feeling increasingly on the edge,
has symptoms of breathlessness, chest pain and nausea that are worsening over the last 6
months. What is the most likely diagnosis?

279
Select one:
Adjustment disorder
Generalised anxiety disorder
Mixed anxiety and depressive disorder

Agoraphobia with panic disorder


Da Costa syndrome

Generalised anxiety disorder is characterized by prominent tension, excessive worry with


generalised free-floating persistent anxiety and feelings of apprehension about everyday events
leading to significant stress and functional impairment. Physical symptoms and features of
autonomic arousal are usually seen in GAD. The ICD-10 list contains 22 physical symptoms of
anxiety while there are only 6 in the DSM-5 list. Common physical symptoms include breathing
difficulties, choking sensations, chest pain, nausea/abdominal distress. The symptoms of
autonomic arousal include sweating, palpitations, trembling and dry mouth.
The correct answer is: Generalised anxiety disorder

HiY Classification 134


Which of the following classificatory systems require 6 months duration with at least 2 out of 5
described symptoms to be present for a diagnosis of schizophrenia?

Select one:
DSM-5
Research Diagnostic Criteria ICD10
ICD10
DSM-IV
St Louis Criteria

DSM-5 requires, for the diagnosis of schizophrenia at least two of the symptoms of delusions,
hallucinations, disorganized speech, grossly disorganized catatonic behaviour or negative
symptoms producing social-occupational dysfunction. In DSM-IV a diagnosis could be made
with only one symptom in criterion A - bizarre delusions or Schneiderian hallucinations. Both
DSM-IV and DSM-5 require a total duration of at least 6 months, including 1 month of active
symptoms during this time.
The correct answer is: DSM-5

HiY Classification 135

280
When comparing DSM IV with ICD-10, which of the following types of schizophrenia is not
specified in ICD-10?

Select one:
Undifferentiated
Paranoid
Simple

Catatonic
Disorganized

While simple schizophrenia and hebephrenic schizophrenia are described only in ICD-10, the
disorganized subtype is only found in DSM-IV. The description of hebephrenic schizophrenia
matches disorganized subtype very closely. Note that the category 'post-schizophrenic
depression' is listed along with other subtypes of schizophrenia in ICD-10, not in DSM-IV. Also
note that DSM-5 has done away with all subtypes now.
The correct answer is: Disorganized

HiY Classification 136


A 34-year-old woman becomes very tearful when her husband has to travel for work to a
different place temporarily as she thinks she is unable to function without him. This is a feature
of

Select one:
Histrionic personality disorder
Borderline personality disorder
Anankastic personality disorder
Anxious personality disorder
Dependent personality disorder

Features of dependent personality disorder- assumes a position of passivity, allowing others to


assume responsibility for most areas of their daily life, lack self-confidence, feel unable to
function independently of another person, feels own needs are of secondary importance.
The correct answer is: Dependent personality disorder

HiY Classification 137


Which of the following features are noted in emotionally unstable personality- borderline
type?

281
Select one:
Unstable and capricious mood
Liability to outbursts of anger or violence
Chronic feelings of emptiness

Marked tendency to quarrelsome behaviour


Marked tendency to act unexpectedly and without consideration of the
consequences

Emotionally unstable personality disorder (EUPD) is characterised by a marked tendency to


act impulsively without consideration of the consequences, together with affective instability.
Two variants of this personality disorder are specified in ICD10, and both share this general
theme of impulsiveness and lack of self-control. In the impulsive type, the predominant
characteristics are emotional instability and lack of impulse control. Outbursts of violence or
threatening behaviour are common, particularly in response to criticism by others. In the
borderline type, in addition to emotional instability, the patient's own self-image, aims, and
internal preferences (including sexual) are often unclear or disturbed. There are usually
chronic feelings of emptiness. A liability to become involved in intense and unstable
relationships may cause repeated emotional crises and may be associated with excessive
efforts to avoid abandonment and a series of suicidal threats or acts of self-harm (although
these may occur without obvious precipitants).
The correct answer is: Chronic feelings of emptiness

HiY Classification 138


Which of the following statements is inaccurate about ICD-10 and DSM-5 classificatory
systems used in psychiatry?

Select one:
Both classificatory systems are categorical systems of classification
Both ICD-10 and DSM-5 are diagnostic manuals.
Both are based on clinical descriptions
Both use a 5-point axial system
They cannot provide case formulations or treatment plans

Explanation: ICD does not use 5-point axial system. The multi-axial version of ICD-10 uses
three axes. Axis 1 - the mental disorder (also personality disorder and mental handicap) Axis 2
­ the degree of disability; and Axis 3 ­ current psychosocial problems. The DSM­5 is said to be
non-axial but includes dimensional assessment of symptom clusters.
The correct answer is: Both use a 5-point axial system

282
HiY Classification 139
A 27-year-old woman with no previous medical or psychiatric history presents with a two-
week history of insomnia, grandiosity, and increased energy. She is in a highly distractible
state with a somewhat elated mood. You attempt to do an MMSE, and she scores 23/30.
Which is the most likely diagnosis?

Select one:
Herpes simplex encephalitis
Delirium on dementia
Bipolar affective disorder

Evolving delirium
Acute stress reaction

This patient has features of mania, which has lasted for more than a week. DSM allows a
single manic episode to be considered as part of bipolar disorder. People who have mania are
very distractible and may not engage well during cognitive testing. Therefore, they may score
low on MMSE. In delirium, the duration is much shorter (hours to days), and the patient may
present with behavioural disturbances and perceptual abnormalities.
The correct answer is: Bipolar affective disorder

HiY Classification 140


A 26-year-old woman has fears of being judged negatively by other people. This is suggestive of

Select one:
Borderline personality disorder
Avoidant personality disorder
Anankastic personality disorder
Dependent personality disorder
Histrionic personality disorder

Individuals with avoidant personality disorder harbour fears being judged negatively by other
people, feelings of discomfort in group or social settings and may come across as being socially
withdrawn. They also have low self-esteem and may crave affection despite an overwhelming
fear of rejection.
The correct answer is: Avoidant personality disorder

283
HiY Classification 141
A 17-year-old girl has a BMI < 17 and long-standing amenorrhoea. She has a history of
purging and laxative use but no body image distortions. The most suitable diagnosis is

Select one:
Atypical Anorexia
Anorexia Nervosa
Unspecified eating disorder

Atypical bulimia
Bulimia

This is an atypical case of anorexia nervosa. The term atypical anorexia nervosa is used for
those individuals in whom one or more of the key features of anorexia such as amenorrhoea
or significant weight loss is absent but otherwise present a fairly typical clinical feature.
The correct answer is: Atypical Anorexia

HiY Classification 142


A 5-year boy presents to your clinic. He has difficulty relating to others in a social setting and
has poor non-verbal social skills. However, he has a normal development of speech and
language. What is his most likely diagnosis?

Select one:
Rett's syndrome
Kanner's syndrome
Asperger's syndrome
Heller's syndrome
DiGeorges' syndrome

In Asperger's syndrome, there is usually no language delay or IQ impairment. There is also a


notable absence of the early aloofness and stereotypies that are seen in autism. There may be
narrow interests and often marked clumsiness.
The correct answer is: Asperger's syndrome

HiY Classification 143

284
A 21-year-old man moved from London to Dundee, a month ago. He is complaining of
uneasiness, difficult to concentrate and a feeling of inability to cope since the move. The
diagnosis is

Select one:
Generalized anxiety disorder
Neurasthenia
Social phobia

Adjustment disorder
Acute stress reaction

Adjustment disorder is a state of subjective distress and emotional disturbance, usually


interfering with social functioning and performance, arising in the period of adaptation to a
significant life change or to the consequences of a stressful life event. It usually occurs within 1
month of the stressful event, and the duration of symptoms does not usually exceed 6 months.

The correct answer is: Adjustment disorder

HiY Classification 144


The symptoms of adjustment disorder resolve within a period of

Select one:
3 months of termination of the stressor
1 week of termination of the stressor
2 weeks of termination of the stressor
6 months of termination of the stressor
1 month of termination of the stressor

Adjustment disorder requires a stressor and that the symptoms should resolve within 6
months of termination of the stressor or its consequences.
The correct answer is: 6 months of termination of the stressor

HiY Classification 145


What is the 5th character of the diagnostic code in ICD 10 when classifying patients with
schizophrenia?

285
Select one:
Aetiology
Treatment resistance
Prognosis

Subtype
Course

ICD-10 as a whole is designed to be a central ("core") classification system for a family of


disease- and health-related descriptions. Some members of the family of descriptions are
derived by using a fifth character to specify more detail. For example, in the substance use
disorders, the third character indicates the substance used, the fourth and fifth characters the
psychopathological syndrome, e.g. from acute intoxication and residual states. In dementias, a
fifth character may be added to specify dementia with or without additional
psychiatric/behavioural symptoms (e.g. hallucinations). In schizophrenia, the fifth character
specifies the course.
The correct answer is: Prognosis

HiY Classification 146


The degree of Learning Disability is said to be profound if IQ is below

Select one:
30
20
25
40
35

Profound mental retardation (IQ range below 20) constitutes approximately 1 to 2 percent of
those with intellectual disability. Most individuals with profound disability have one or more
identifiable causes for their condition.
The correct answer is: 20

286
287
001
A girl attending your outpatient unit is 165 cm tall and weighs 50kg. What is her body mass
index?

Select one:
20 to 21
16 to 17
17 to 18

18 to 19

19 to 20

The correct answer is: 18 to 19

002
In occasional cannabis user, how long is cannabis detected in urine after last use?

Select one:
3 hours
4 weeks
12 hours
8 days

3 days

288
Occasional use - up to 3 days. High daily use for long time - up to 4 weeks
The correct answer is: 3 days

003
Mr. X has been consuming a high dose of cannabis for a long time. How long will it be present in
his urine?

Select one:
24 hours
10 days
3 days
7 days

4 weeks

The correct answer is: 4 weeks

004
Amphetamines could be detected in urine for up to

Select one:
48 hours
30 days

6 hours
24 hours

7 days

Amphetamines, codeine and morphine could be detected in urine for up to 48 hours.


The correct answer is: 48 hours

289
005
Which of the following will you use to assess orientation in a hospitalised patient?

Select one:
Ability to repeat a phone number immediately after reading out
Tap the table once whenever the examiner reads letter 'E' from a random list
of alphabets
Ability to recognise pictures of famous personalities
Ability to correctly identify the doctor when asked to point out

Ability to remember his/her own address

Orientation to person is a stable domain which is generally lost only late in the course of a
confusional state.
The correct answer is: Ability to correctly identify the doctor when asked to point out

006
In which of the following situations closed questioning is a useful technique?

Select one:
In a highly distracted patient
To interview an overtalkative patient
At the later part of interview
To rule out less likely symptoms
All of the above

Closed questions are better avoided in early parts of interview as they can produce prescribed
answers lacking in detail. Close questions must be avoided in highly suggestible patients. A
good technique is to start with open; move to closed by the end of the interview. Useful to rule
out less likely symptoms. Stating a presumption followed by tags can be very directive.
The correct answer is: All of the above

007
All of the following help localizing a neurological lesion to cerebellum EXCEPT

290
Select one:
Ankle clonus
Hypotonia
Dysdiadochokinesia

Ataxia
Past pointing

The correct answer is: Ankle clonus

008
Which of the following is an advantage of open ended questions over closed questions?

Select one:
Answers to open questions are highly time-saving
Answers to open questions are doctor-directed
Answers to open questions are highly reliable
Answers to open questions are less precise
Answers to open questions are more informative

Open-ended questions yield highly informative answers. They produce spontaneous


formulations, but the answers are of low reliability, and are non-reproducible at a later date, or
by a different doctor. They do not focus on target symptoms and so are not very time efficient.
The correct answer is: Answers to open questions are more informative

009
A 61-year-old right-handed man presented to the emergency department with an acute onset of
right-leg weakness. Soon his weakness progressed to include both arms and legs, and he
stopped speaking. On examination, he was alert but produced no spontaneous speech or
spontaneous movements. Which artery is most likely to be implicated in this presentation?

Select one:
Cerebello Pontine artery
Posterior cerebral artery

291
Anterior cerebral artery
Middle cerebral artery
Basilar artery

The syndrome of akinetic mutism is typically the result of bilateral hemispheric injury, usually
involving the anterior cingulate gyri. Other lesions producing akinetic mutism include lesions of
the thalami, globus pallidus, internal capsule, and frontal white matter. These lesions are thought
to disrupt anterior frontal-subcortical circuits that subserve motivation. This is usually a result of
anterior cerebral artery occlusion, which in some individuals, can have a predominantly
lateralized course due to an anatomical variation on one side.
The correct answer is: Anterior cerebral artery

010
A patient who sustained a head injury reports loss of memory for events that happened
between 13th September 2014 and 20th September 2014, though he retained full
consciousness soon after the injury was sustained on 13th September 2014. Which of the
following cognitive dysfunctions is he describing?

Select one:
Semantic dementia
Retrograde amnesia

Selective amnesia
Transient global amnesia
Anterograde amnesia

Anterograde memory refers to the newly encountered information from the time of a lesion.
Anterograde memory loss is also commonly elicited when examining a dementing patient -
e.g. forgetting appointments, losing items around the home, inability to remember
conversation leading to repeated questions, etc .
The correct answer is: Anterograde amnesia

011
A patient has normal accommodation reflex but no reaction on testing light reflex. What is the
likely diagnosis?

Select one:
Marcus Gunn pupil

292
Argyll-Robertson pupil
Holmes Adie pupil
Hutchison pupil

Horner's pupil

In 1869, Douglas Argyll-Robertson described several patients whose pupils reacted poorly to
light with a normal near response. Later the aetiology of this pupillary anomaly was discovered
to be tertiary syphilis. Currently, diabetes leads the list of causes for the so called
Argyll-Robertson Pupil or ARP.
The correct answer is: Argyll-Robertson pupil

012
A non-invasive perfusion MRI methodology used to quantify cerebral blood flow is

Select one:
Functional MRI

Single photon emission tomography


Resting state connectivity
Diffusion tensor imaging
Arterial spin labelling

Arterial spin labelling is a non-invasive perfusion MRI methodology used to quantify cerebral
blood flow.
The correct answer is: Arterial spin labelling

013
T1 images in MRI are mostly useful for

Select one:
Assessment of normal brain structure
Visualising periventricular structures
Visualising edema
Detecting tumours

Localising ischemic zones

293
The T1 image most closely resembles a CT scan and is most useful for assessing the overall
brain structure. T1 is also the only sequence that allows contrast enhancement with the
contrast agent gadolinium-diethylenetriamine pentaacetic acid (gadolinium-DTPA)
The correct answer is: Assessment of normal brain structure

014
Which of the following signs can be elicited by having the patient extend both arms with the
wrists dorsiflexed and palms facing forward and eyes closed?

Select one:
Chorea
Myoclonus
Asterixis
Athetosis
Pronator drift

Asterixis can be elicited by having the patient extend both arms with the wrists dorsiflexed and
palms facing forward and eyes closed. Brief jerky downward movements of the wrist are
considered a positive sign. Asterixis is commonly seen with metabolic encephalopathies .
The correct answer is: Asterixis

015
Which of the following is NOT a component of clock drawing test?

Select one:
Using numbers 1 to 12 to create the face of a clock
Being aware of the current time when the test is administered

Indicating the hour correctly according to instructions


Indicating the minutes correctly according to instructions

Placing the hands of the clock correctly

294
CDT (Clock Drawing Test) does not assess one's orientation to time.
The correct answer is: Being aware of the current time when the test is administered

016
The wavelet of EEG that is greater than 13Hz in frequency is

Select one:
Delta
Beta
Theta
Gamma
Alpha

Beta is the frequency range from 12 Hz to about 30 Hz


The correct answer is: Beta

017
Which phenomenon is commonly used in a functional MRI scan to make inferences about
brain activity?

Select one:
Echo planar imaging (EPI)
Diffusion imaging

High angle low-frequency pulses.


Blood oxygen level dependent (BOLD) Phenomenon
Fast low angle shot radiofrequency pulses (FLASH)

295
The correct answer is: Blood oxygen level dependent (BOLD) Phenomenon

018
Which of the following is an advantage of CT scans over MRI?

Select one:
Gray vs. white matter demarcation is well delineated
Posterior fossa lesions are well noted
Multiplanar images are obtained
Lesions close to bone are better seen
Calcified lesions are better visualised

The correct answer is: Calcified lesions are better visualised

019
Which of the following questions is useful in evaluating immediate recall?

Select one:
Can you tell me today's date?
Where were you yesterday?
What did you have for dinner?
Where did you go to school?
Can you repeat these six numbers?

Digit span reflects one's ability to retain information in the short-term memory store. Along with
free recall of items, it is an often used test for immediate recall.
The correct answer is: Can you repeat these six numbers?

020

296
All of the following drugs will increase urine porphyrins in a susceptible individual with
disordered haem metabolism except

Select one:
Oestrogens
Diclofenac
Barbiturates

Chlorpromazine
Benzodiazepines

Acute intermittent porphyria is episodic in nature and the episodes are often triggered by
certain medications including estrogens, barbiturates and benzodiazepines. Diclofenac can
precipitate an episode. Psychiatric manifestations include depression, anxiety, delirium and
psychosis.
The correct answer is: Chlorpromazine

021
Which of the following clinical examination for apraxias is abnormal more commonly with a
right than a left cerebral damage?

Select one:
Imitating a toothbrush action
Construction of polygons
Mimicking hand movements
Using tools appropriately
Whistling and coughing on command

Constructional apraxia is inability to construct elements to a meaningful whole. E.g. inability to


draw or copy simple diagrams or figures. This is typically caused by a lesion in the right cerebral
hemisphere.
The correct answer is: Construction of polygons

022
In MMSE, intersecting pentagons test primarily assesses which of the following functions?

Select one:

297
Constructional praxis
Language
Copying skills

Visual retention
Orientation

Visuospatial ability, especially constructional praxis, is tested using intersecting polygons test.
The correct answer is: Constructional praxis

023
Unilateral frontal lobe damage can result in

Select one:
Does not cause spastic paresis
contralateral spastic hemiplegia
contralateral hypotonia
ipsilateral spastic hemiplegia
Paraplegia

Damage to upper motor neurons originating in frontal motor strip can result in contralateral
spastic hemiparesis
The correct answer is: contralateral spastic hemiplegia

024
Which of the following conditions, when suspected, will necessitate performing a neurological
procedure called Hoover's test?

Select one:
Depression
Conversion disorder
Somatisation disorder
Munchausen syndrome
Hypochondriasis

298
Hoover's test is useful to identify patients with conversion disorder, who has paralysis with One
or more limbs or one side of the face or body may be affected. Here the patient is unable to
raise the affected limb from the couch but can raise the unaffected limb against resistance with
demonstrable pressing down of the heel on the affected side. Ref: Oxford handbook of
psychiatry; 746
The correct answer is: Conversion disorder

025
Various components of language that must be tested in order to localize the brain lesion
causing aphasia include all EXCEPT

Select one:
copying
comprehension
naming
Repetition

fluency

The correct answer is: copying

026
Which of the following is not a frontal release sign?

Select one:
Rooting reflex
Glabellar tap

Snout reflex
Cremasteric reflex
Grasp reflex

299
Primitive reflexes include the glabellar tap, rooting, snout, sucking, and palmomental reflexes.
These are generally absent in adults. When present in the adult, these signs signify diffuse
cerebral damage, particularly of the frontal lobes (hence called frontal-lobe release
signs).Cremastric reflex is a superficial reflex .
The correct answer is: Cremasteric reflex

027
An important investigation useful to differentiate Lewy body dementia from Alzheimer's
disease is

Select one:
MRI scan
CT scan
X-ray
EEG
DaT scan

The correct answer is: DaT scan

028
Mrs Y is an elderly lady with cognitive impairment and visual hallucinations. Select the best
imaging technique to diagnose her condition.

Select one:
CT scan
PET scan

DAT scan
f-MRI scan
MRI Scan

300
The correct answer is: DAT scan

029
Which of the following SPECT finding is seen in Alzheimer's disease?

Select one:
Multiple zones of ischemia in left hemisphere
Increase in temporal perfusion
Increase in parietal perfusion
Increase in frontal perfusion
Decrease in temporal perfusion

Numerous studies have suggested that temporoparietal hypoperfusion seen on brain imaging
with SPECT may be useful in diagnosing Alzheimer's disease ( SPECT perfusion imaging in the
diagnosis of Alzheimer's disease,
The correct answer is: Decrease in temporal perfusion

030
Which one of the following techniques can measure the extent of myelination of white matter
fibres at a macroscopic level?

Select one:
Resting state connectivity
Diffusion tensor imaging
Functional MRI
Single photon emission tomography
Arterial spin labelling

301
Diffusion tensor imaging (DTI) measures at the macroscopic level the extent of myelination of
white matter fibres using fractional anisotropy, a measure of the degree and directionality of
diffusion of water molecules. More recent advances in diffusion tensor imaging include
deterministic and global probabilistic tractography, quantitative and more probabilistic measures
respectively, of connectivity between reconstructed white matter fibres.

The correct answer is: Diffusion tensor imaging

031
Which of the following is not a functional neuroimaging technique?

Select one:
fMRI scan
Single positron emission computerised tomography
Positron emission tomography
Magnetoencephalography
Digital subtraction angiography

Digital Subtraction Angiography (DSA) is a powerful technique for the visualization of blood
vessels in the human body. DSA is a fluoroscopy technique to visualize blood vessels in a bony
or dense, soft tissue area. It does not measure brain function as such but identifies
abnormalities in the integrity of cerebral vasculature.
The correct answer is: Digital subtraction angiography

032
A patient presenting with carbon monoxide poisoning is suspected to have bilateral posterior
parietal damage due to anoxia. If this turns out to be a case of Balint's syndrome which of the
following will NOT be a presenting feature?

Select one:
Simultanagnosia
Diplopia

Oculomotor apraxia
Optic ataxia
Intact oculocephalic reflexes

302
Balint's syndrome consists of a triad of simultanagnosia (inability to attend to more than one item
of a complex scene at a time), optic ataxia (inability to guide reaching or pointing despite
adequate vision), and oculomotor apraxia (inability to voluntarily direct saccades to a visual
target). Fields may be full when challenged with gross stimuli, and oculocephalic reflexes are
intact. This syndrome results from bilateral damage including the superior-parieto-occipital
region, which disrupts the dorsal ('where') visual processing stream linking visual with parietal
association areas. Possible causes include carbon monoxide poisoning, watershed infarction,
leucodystrophy, and the posterior cortical variant of Alzheimer's disease.
The correct answer is: Diplopia

033
Which of the following is a compound question?

Select one:
I think you are not well. Am I right?
What can I do for you?
What is the average height of British women?
Do you know how IQ is calculated?
Do you go on holidays and are you able to relax then?

Adding two or more questions in a single statement is referred to as a compound question


(consider MCQs - they are always compound). This confuses the patient and may lead to
either a vague response or no response.
The correct answer is: Do you go on holidays and are you able to relax then?

034
Anomic dysphasia is suggestive of a lesion in which of the following regions of the brain?

Select one:
Dominant fronto-temporal lesion
Non-dominant fronto-temporal lesion

Dominant temporo-parietal lesion


Non-dominant temporo-parietal lesion
Dominant occipital lesion

303
In anomic dysphasia, confrontation naming is affected more than any other function (when an
object is shown, the patient cannot tell its name). It is often due to small left-sided lesions in
language cortex - temporo-parietal junction.
The correct answer is: Dominant temporo-parietal lesion

035
In schizophrenia, the anatomical structure in the brain that is most consistently reduced in size
is

Select one:
striatum
insula
hypothalamus
cerebellum

hippocampus

Most of the listed structures have been observed to have structural abnormalities in
schizophrenia. In fact, the hippocampus has been repeatedly shown to have a reduced
volume though not in all studies. Insula, which forms a part of the paralimbic cotex, has been
unequivocally shown to have volumetric abnormalities in schizophrenia. Other consistently
affected structures include anterior cingulate cortex, dorsolateral prefrontal cortex and
thalamus.
The correct answer is: insula

036
On mini-mental examination, a 74-year-old man loses all points on orientation to time and
recall, while other faculties are intact. How many points has he lost in total?

Select one:
Six
Sixteen
Thirteen
Five
Eight

304
5 points are awarded for orientation in time and 3 points for recall.
The correct answer is: Eight

037
Which of the following is NOT a good practice while using interpreters?

Select one:
Ask the interpreter about patient's guarded nature while answering
Offering to debrief the interpreter if needed after interpretation
Encourage interpretation and then translation of mental status examination
Brief the interpreter before seeing the patient.

Sticking to the same interpreter for a single case

When using an interpreter, explain the need for providing a literal translation of a patient's
account. If the interpreter provides a 'summarised translation' or 'interpretations' of the reported
account, several salient aspects may be missed during a Mental Status Examination.
The correct answer is: Encourage interpretation and then translation of mental status
examination

038
One of the most well established anatomical changes seen in schizophrenia is

Select one:
Enlargement of the lateral ventricle
Enlargement of corpus callosum

Enlargement of the amygdala


Enlargement of the third ventricle
Enlargement of cerebellar peduncles

305
The correct answer is: Enlargement of the lateral ventricle

039
A 29-year-old man presents with multiple abdominal scars from different surgeries, giving an
appearance of er board abdomen. What is the most likely psychiatric cause?

Select one:
Conversion disorder
Delusional disorder
Hypochondriasis
Dissociative disorder
Factitious disorder

The correct answer is: Factitious disorder

040
Select the most appropriate investigation for a patient on antipsychotic medications
presenting with weight gain, excessive thirst and fatigue.

Select one:
Fasting glucose
Liver function tests

Full blood count


Electrocardiogram
Fasting lipids

The symptoms of weight gain, excessive thirst and fatigue are suggestive of impaired glucose
tolerance. A fasting blood glucose is indicated to diagnose antipsychotic related diabetes
mellitus.
The correct answer is: Fasting glucose

041

306
The brain region with most prominent changes observed using neuroimaging in ADHD is

Select one:
Frontal cortex
Occipital cortex
Thalamus

Parietal cortex
Insula

The correct answer is: Frontal cortex

042
A previously reliable and conscientious man has become increasingly profane, irascible, and
irresponsible over the past 3 - 6 months. These symptoms suggest that a tumour, if present, is
most likely to be located in the

Select one:
Medial wall
Temporal Lobe
Parietal Lobe
Cerebellum
Frontal lobe

The correct answer is: Frontal lobe

043
In a motorbike accident, a 21-year-old driver sustains significant head injury. Following recovery
from emergency neurological status, it is found that he has significant problems in error
correction and categorization of objects. Which of the following is a likely area of lesion?

Select one:
Diencephalon

307
Parietal lobes
Frontal lobes
Occipital lobe

Temporal lobe

Categorization and error correction are tests for frontal lobe functioning.
The correct answer is: Frontal lobes

044
Blood oxygen level dependent measurement BOLD is a technique used in which of the
following imaging modalities?

Select one:
MRS spectroscopy

SPECT scan
Functional MRI
Contrast CT
PET scan

Neuronal activity within the brain causes a local increase in blood flow, which in turn increases
the local hemoglobin concentration. Although neuronal metabolism extracts more oxygen in
active areas of the brain, the net effect of neuronal activity is to increase the local amount of
oxygenated hemoglobin. This change can be detected with the T2 sequence, called Blood
Oxygen Level Dependent (BOLD) technique. This process is the basis for the technique of
fMRI
The correct answer is: Functional MRI

045
In SPECT imaging iomazenil (I-123) is used to label

Select one:
GABA-A receptors
D-2 receptors
Alpha-2 receptors

308
5-HT2 receptors
Glycine receptors

Iomazenil is used for GABA-A receptors


The correct answer is: GABA-A receptors

046
Which of the following tests would be required to make a diagnosis of Huntington's disease in a
patient without a family history?

Select one:
EEG
Immunological tests
Sleep studies

Gene testing
Electromyography

Sporadic appearance of new mutations is not uncommon in the Huntington disease gene.
Hence, a genetic test is advised even in those who present without a family history.
The correct answer is: Gene testing

047
A 24-year-old man who is being investigated for jaundice presents with a gradual change in
personality characterised by disinhibition and recklessness. On examination, he has tremors
and dysarthria. Which of the following signs is highly likely in this case?

Select one:
Xanthomas on extensor aspect of forearm
Golden brown pigmentation around cornea
Port wine stain on face
Ash leaf macules
Cafe au lait spots

309
This scenario is suggestive of Wilson disease with features of dysarthria, tremor, jaundice
and a change in personality. Wilson disease is characterised by the presence of a Kayser-
Fleischer ring (brown pigmentation due to copper deposition around cornea)
The correct answer is: Golden brown pigmentation around cornea

048
Which of the following has a white, bright appearance in a CT scan?

Select one:
White matter
Haemorrhage
Brain tumour
Brain edema
CSF

Blood appears bright in CT and T1 MRI.


The correct answer is: Haemorrhage

049
The reason for using confrontation technique during a clinical interview is to

Select one:
Deal with difficult patients
Reveal some absurd notions that patients might hold

Make patients stop lying to the clinician


Help the patient face a difficult aspect of their problems.

Clarify certain associations that patients may not see

The correct answer is: Help the patient face a difficult aspect of their problems.

310
050
A 32-year-old man has left his wife and children without informing them, has traveled a long
distance and has taken up a new name to avoid being found. He is drinking heavily, appears
depressed and suicidal. Physical examination is unremarkable except for generalized lymph
node enlargement and some muscle wasting. Which of the following is a likely diagnosis?

Select one:
Major depression
Alcohol dependence
Dissociative fugue

HIV illness
Lymphoma

Generalised lymph node enlargement with muscle wasting along with social withdrawal
(possibly due to perceived stigma) is suggestive of a HIV illness.
The correct answer is: HIV illness

051
Which of the following is a risk factor for QT prolongation on ECG?

Select one:
Hypokalemia
Age 25 to 45
Male sex
Hypercalcemia
Tachycardia

Risk factors for prolonged QT include organic heart disease, metabolic abnormalities such as
hypokalaemia (by far the most common), hypocalcaemia, hypomagnesaemia, and bradycardia.
A female preponderance may be caused by sex differences in specific cardiac ion densities.

The correct answer is: Hypokalemia

052

311
Basal ganglia is implicated in a type of memory that deals with skills, habits, and
nonassociative learning and does not require conscious awareness and concentration, and
usually remains intact after brain injury. This type of memory is called

Select one:
Implicit memory
Flash-bulb memory
Insight memory

Episodic memory
Declarative memory

The correct answer is: Implicit memory

053
A patient agrees that he has significant problems with the use of alcohol. He takes
responsibility and understands the importance of having a change in his lifestyle. He has not
taken any active steps to change his drinking pattern so far and continues heavy alcohol use.
Which of the following levels of insight does he have?

Select one:
Intellectual insight
Complete loss of insight
Externalization of cause
Emotional insight
Awareness of problems but denial of responsibility

Intellectual insight: admission that the patient is ill and that symptoms or failures in social
adjustment are caused by the patient's own particular irrational feelings or disturbances
without applying this knowledge to future experiences.
The correct answer is: Intellectual insight

054
Inappropriate lack of concern about a functional deficit or disability is known as

312
Select one:
Anosognosia
Prosopagnosia
La belle indifference

Perseveration
Alexithymia

La belle indifference is the inappropriate lack of concern about the deficit or disability. It is
seen in hysteria (conversion disorders), and also in some cases with multiple sclerosis. In
anosognosia, the patient either refuses to acknowledge the deficit or disowns the deficit. In
contrast, patients with la belle indifference appreciate the deficit but show no emotional
concern.
The correct answer is: La belle indifference

055
When copying a complex figure, a patient with brain injury manages to reproduce global
features of the design but misses the finer details of the figure. He is most likely to have

Select one:
Angular gyrus lesion
Bilateral damage
Right hemisphere damage
Fusiform gyrus lesion
Left hemisphere damage

While performing Rey Osterreich Complex figure test, patients with right hemisphere damage
fail to reproduce global pattern but copy isolated details. Those with left sided damage capture
the global figure but not the finer details.
The correct answer is: Left hemisphere damage

056
Confrontation test carried out during physical examination is useful to detect which one of the
following?

Select one:
Astigmatism

313
Myopia
Cerebellar lesions
Lens dislocation

Lesions of optic tract

Confrontation test is used to assess the central visual field by using a red pin.
The correct answer is: Lesions of optic tract

057
The presence of intracytoplasmic inclusion bodies is a microscopic finding in which of the
following conditions?

Select one:
Alcoholic dementia

Lewy body dementia


Alzeimer's disease
Creutzfeldt-Jakob disease
Huntington's disease

The correct answer is: Lewy body dementia

058
In a motorbike accident, a 21-year-old driver sustains significant head injury. Following
recovery from emergency neurological status, it is found that he has significant cognitive
deficits. Which of the following is NOT a test of parietal lobe functioning?

Select one:
Two point discrimination
Right left orientation
Lexical fluency

314
Stereognosis
Finger agnosia

Lexical fluency is a frontal lobe test, whereas Stereognosis, finger agnosia, Right left
orientation, two point discrimination are tests for parietal lobe.
The correct answer is: Lexical fluency

059
Which of the following is not a test for executive function?

Select one:
Wisconsin Card Sorting Test
Line orientation test
Category Test

Trail Making Test


Stroop test

Judging line orientation, Rey Osterrieth test and clock drawing are tests of visuospatial
functioning.
The correct answer is: Line orientation test

060
Which one of the following has been used in vitro as a powerful method of analytical
chemistry and enables the measurement of the concentration of different metabolites in the
brain?

Select one:
Structural MRI scan
CT Scan
Functional MRI scan
X-Ray
Magnetic Resonance Spectroscopy.

315
MRS utilizes the principle of nuclear magnetic resonance and can be used to measure
metabolite concentration without the need for radio isotopes.
The correct answer is: Magnetic Resonance Spectroscopy.

061
The earliest appearance of brain atrophy in Alzheimer's disease is most marked in which of
the following brain regions?

Select one:
Insular lobe
Medial Temporal Lobe
Frontal lobe
Occipital lobe
Posterior Parietal Lobe

The correct answer is: Medial Temporal Lobe

062
An anorexic patient repeatedly vomits. Which of the following is a possible complication?

Select one:
None of the above
Hyponatraemia

Hypokalaemia
Metabolic alkalosis
Metabolic acidosis

Loss of gastric acid can lead to metabolic alkalosis.


The correct answer is: Metabolic alkalosis

063

316
Generalised diffuse delta and theta waves in the EEG are seen in

Select one:
Temporal lobe seizures
Metabolic encephalopathy
Infantile spasms
CJD
Normal Stage 1 sleep

The correct answer is: Metabolic encephalopathy

064
Which of the following arteries on occlusion leads to symptoms suggestive of dorsolateral
prefrontal dysfunction?

Select one:
Middle cerebral
Peduncular
Basilar
Posterior cerebral
Anterior cerebral

DLPFC consists of the lateral portions of Brodmann areas 9 - 12, areas 45, 46, and the
superior part of area 47. These regions receive their blood supply from the middle cerebral
artery.
The correct answer is: Middle cerebral

065
Patients with schizophrenia are least likely to perform poorly on which of the following tests?

Select one:
Category test

317
Trail B of the Halstead-Reitan battery
Wisconsin card sort test
Speech sounds perception test

MMSE

Patients with schizophrenia perform far more poorly than controls on category test (assess
higher-order reasoning), Wisconsin card sort test (tests higher-order reasoning), speech sounds
perception test, Trail B of the Halstead-Reitan battery (tests the ability to shift flexibility between
cognitive sets). Tests of overall cognitive ability such as MMSE are usually intact in
schizophrenia.
The correct answer is: MMSE

066
Clinical features of drug-induced hyponatraemia include

Select one:
Muscle cramps
Constipation
Diarrhoea
Tremors
Sweating

Signs and symptoms of hyponatremia include nausea and vomiting, headache, confusion,
lethargy, fatigue, appetite loss, muscle cramps and restlessness with irritability.
The correct answer is: Muscle cramps

067
All of the following are signs of an upper motor neuron (UMN) lesion except

Select one:
Spasticity
Rigidity
Muscular atrophy
Extensor plantar

318
Weakness

Atrophy is seen in LMN lesions, not in UMN lesions.


The correct answer is: Muscular atrophy

068
A 29-year-old African-American man who uses intravenous heroin presents with a puffy face
and steady increase in weight despite normal appetite. On examination his blood pressure is
150/100mmHg. The most likely diagnosis is

Select one:
Hepatitis B liver disease
Hypothyroidism
Opiate withdrawal
Nephropathy
HIV seroconversion

Heroin-associated nephropathy includes different morphological findings following chronic drug


abuse. The exact cause of renal damage is still ambiguous. Some suspect heroin itself, while
others blame adulterants, other diseases like hepatitis B and C infection or HIV, as the
causative factors for a spectrum of morphologically variable changes in the kidneys.
The correct answer is: Nephropathy

069
Which of the following speech best describes Broca's aphasia?

Select one:
Non-fluent aphasia

Fluent aphasia
Telegraphic speech
Scanning speech
Staccato speech

319
In Broca's aphasia, due to damage to the motor speech zone, the language appears non-
fluent.
The correct answer is: Non-fluent aphasia

070
The symptom or sign that responds earlier than others when thiamine is replaced in a patient
with Wernicke's encephalopathy is

Select one:
Opthalmoplegia
Apathy
Confusion
Ataxia
Amnesia

Ophthalmoplegia responds within hours, but cognitive impairment takes longer.


The correct answer is: Opthalmoplegia

071
A forensic inpatient is asked to undergo narcoanalysis by the Crown Court. Which of the
following statements about narcoanalysis is NOT true?

Select one:
Useful technique when repression and dissociation are suspected.
Organic muteness gets better with amobarbital infusion

Psychogenic muteness gets better with amobarbital infusion


Amobarbital is used

Benzodiazepines can be used

Interviews with amobarbital have both diagnostic and therapeutic indications. These are
sometimes helpful in differentiating nonorganic and organic conditions, particularly in patients
with symptoms of catatonia, stupor, and muteness. Organic conditions tend to worsen with

320
infusions of amobarbital, but nonorganic or psychogenic conditions tend to get better because of
disinhibition, decreased anxiety, or increased relaxation. Therapeutically, amobarbital interviews
are useful in disorders of repression and dissociation such as amnesia and fugue.
Benzodiazepines can be substituted for amobarbital.
The correct answer is: Organic muteness gets better with amobarbital infusion

072
A patient undergoing an assessment for dementia is asked to draw a circle and place numbers
inside the circle to depict the face of a clock. All of the following domains of cognition are crucial
for successful completion of this task except

Select one:
Executive function
Constructional praxis
Comprehension
Orientation to time
Visuospatial ability

CDT (Clock Drawing Test) does not assess one's orientation to time.
The correct answer is: Orientation to time

073
Which of the following statements is true concerning bedside tests of orientation?

Select one:
Most patients do not remember the length of time spent in hospital in spite of
intact orientation.
Orientation to time is often affected in delirium

Disorientation to one's own name indicates hippocampal lesions


Intact orientation excludes memory disorders

Orientation to person is the most sensitive test

Orientation is usually assessed to time, place and person; it is not particularly sensitive, and
intact orientation does not exclude a significant memory disorder, particularly if there is concern
about memory from an informant. Time orientation is the most helpful, and should include the
time of day. Many normal people do not know the exact date, and being out by two

321
days or less is considered normal when scoring this formally. Time intervals are often poorly
monitored by patients with delirium, moderate to severe dementia, and in the amnesic
syndrome, and are easily tested by asking about the length of time spent in hospital. Person
orientation includes name, age, and date of birth. Disorientation to one's own name is usually
only seen in psychogenic amnesia.
The correct answer is: Orientation to time is often affected in delirium

074
In MRS, which nuclei are commonly used for in vivo measure of phosphocreatinine?

Select one:
H-1
Na-23
Li-7
F-19
P-31

Na-23 is employed for intra and extra cellular sodium measurement.


P-31 is employed for quantifying energy handling and cellular metabolism (ATP/ADP),
membrane physiology, phosphomono/diester quantification and phophocreatinine
measurement.
H1 is used for quantifying glutamate, lactate, N-acetylaspartate, choline, creatinine.
F-19 MRS is used to study labelled drugs and deoxyglucose.
The correct answer is: P-31

075
A lactate solution is infused in a 32-year-old woman. This results in acute anxiety, palpitations,
sweating, fear of loss of control and dizziness. What is the most likely diagnosis?

Select one:
Lactic acidosis
Malingering

Generalized anxiety disorder


Panic disorder
Lactose intolerance

322
The correct answer is: Panic disorder

076
Copying intersecting polygons in MMSE tests the functions of

Select one:
Hippocampus
Parietal lobes
Temporal lobes
Cerebellum
Frontal lobes

The correct answer is: Parietal lobes

077
Which of the following tasks is most useful in evaluating a patient's ability to concentrate?

Select one:
Naming five large cities
Reporting current world events

3 items recall
Performing serial 7s
Estimating the distance between Sheffield and London

The correct answer is: Performing serial 7s

323
078
Which of the following investigations involve introducing a synthetic radionucleotide for
obtaining brain images?

Select one:
T2 weighted scan
PET scan
fMRI

T1 weighted scan
Diffusion Imaging

PET involves using radionucleotides to study neuronal activity, neurochemistry and


pharmacology of the brain.
The correct answer is: PET scan

079
Which neuroimaging modality can give information about metabolic changes in the brain?

Select one:
Structural MRI Scan
PET scan
Ultrasonogram brain
SPECT scan
CAT scan

Both PET and SPECT scan give information about regional cerebral blood flow and ligand
binding but PET scan also gives information about metabolic changes in the brain. The most
commonly used isotopes in PET are fluorine-18, nitrogen-13, and oxygen-15. These isotopes
are usually linked to another molecule, except in the case of oxygen-15 (15O). The most
commonly reported ligand has been [18F]fluorodeoxyglucose (FDG). This gives information
directly about neuronal metabolism.
The correct answer is: PET scan

080
Which of the following cognitive domains is assessed by Wisconsin Card Sorting Test?

324
Select one:
Language
Visuospatial skills
Immediate recall

Retrieval
Planning and set shifting

Executive functioning consists of volition, planning, purposive action, execution, error


monitoring and error correction. These are cognitive control mechanisms managed by
prefrontal areas of the brain.
The correct answer is: Planning and set shifting

081
A 18-year-old man presents with the bizarre behaviour of picking his clothes and the carpet. He
appears agitated and mutters unintelligibly. He has a high temperature on examination. When
examining him, it is found that when his knee is extended with flexed hip, he develops a 'catch'
and flexes the other hip as a reflex. This is called

Select one:
Reverse leg raising sign
Positive Romberg's sign
Straight leg raising sign
Positive Brudzinki's sign
Positive Kernig's sign

The correct answer is: Positive Kernig's sign

082
Alexia without agraphia (pure alexia) is seen in lesions of

Select one:
Middle meningeal artery

325
Posterior inferior cerebellar artery
Middle cerebral territory
Posterior cerebral territory

Anterior cerebral territory

In alexia without agraphia or pure word blindness, patients retain the ability to write, but are
unable to read (even words that they have just written) and often have right homonymous
hemianopia. This disorder results from the loss of visual input to the language area [usually the
dominant (left) angular gyrus] without the involvement of the language area or output from the
language area to the motor cortex. The ability to process visual input into language (i.e.
reading) is lost in these patients. The pathways involved in this syndrome are believed to be
localized to the inferior portion of the splenium of the corpus callosum on the side of the
dominant (usually left) cerebral hemisphere. This is associated with posterior cerebral artery
damage.
The correct answer is: Posterior cerebral territory

083
Which region of the brain is often highlighted when performing tasks inside a scanner during a
functional MRI scan?

Select one:
Thalamus

Basal ganglia
Cerebellum
Primary motor cortex
Hypothalamus

The correct answer is: Primary motor cortex

084
In which of the following conditions do you expect to see a pulvinar sign on magnetic
resonance imaging?

Select one:
Viral encephalitis

326
Lewy Body dementia
Amyloid angiopathy
Pick's disease

Prion disease

The presence of regions of high signal intensity in the posterior thalamus on T2-weighted MR
images is known as the 'pulvinar' sign. It is seen in 90% of patients with autopsy proven variant
CJD.
The correct answer is: Prion disease

085
Inability to recognise familiar faces is seen in

Select one:
Anosognosia
Simultanagnosia
Prosopognosia
Somatognosia
Visuospatial apraxia

The correct answer is: Prosopognosia

086
A diabetic patient who is taking antidepressants is noted to have polyuria and polydipsia. His
urine and plasma osmolality are both low. Choose the most likely cause for his presentation.

Select one:
Psychogenic polydipsia
Hyponatraemia
Diabetes insipidus

Hyperglycaemia
SIADH

327
Antidepressant induced SIADH does not appear to be dose related, however it does appear to
be more common with serotonergic agents. When both urine and plasma osmolality is low, this
indicates that excessive water is being consumed and excreted - this is seen in psychogenic
polydipsia. In hyperglycemia, plasma osmolality will not be reduced; in SIADH, urine osmolality
will be high.
The correct answer is: Psychogenic polydipsia

087
Which of the following endocrine changes is noted in anorexia nervosa?

Select one:
Increased luteinising hormone levels
Decreased plasma cortisol levels
Low thyroxine T4 levels
Decreased carotene levels in plasma
Raised growth hormone levels

In adolescents and adults with anorexia nervosa, increased basal growth hormone (GH)
secretion, a higher number of secretory bursts, and higher GH concentration is seen. This can
have adverse effects on bone mass and eventual growth curve.
The correct answer is: Raised growth hormone levels

088
If you perform a routine cognitive examination on a 90-year-old woman with no history
suggestive of dementia, which of the following is least likely to be seen?

Select one:
Reduced new learning efficiency
Reduced working memory
Reduced visuospatial ability
Reduced motor speed
Reduced attentional span

328
Reduced short-term, working memory is a feature of dementia. It is not expected as a part of
healthy aging. But reduced motor speed, visuospatial skills, attentional span, and decreased
new learning ability are seen in healthy aging process.
The correct answer is: Reduced working memory

089
A depressed patient who is having difficulties with alcohol use is attempting to narrate her
problems regarding her job, but stops after initiating the discussion. The examiner praises the
patient for doing well despite having a difficult time. Which of the following interview technique is
he using?

Select one:
Setting traps
Reinforcement
Acknowledgment of affect
Suggestive lead
Validation

Reinforcement aids in further disclosure and continuation of a diagnostic interview.


The correct answer is: Reinforcement

090
A tremor, which decreases with volitional movements and appears primarily when not
engaged in tasks, is most likely to be a

Select one:
postural tremor
cerebellar ataxia

physiologic tremor
senile tremor

resting tremor

A resting tremor occurs with the body part in complete repose and often dampens or subsides
entirely with action
The correct answer is: resting tremor

329
091
A patient is asked to hold out both arms with palms facing upwards and maintain the posture
with eyes closed. His left arm slowly drops down compared to right arm. Which of the following
lesions is most likely to be present?

Select one:
Left hemisphere lesion
Cranial nerve X nuclei lesion
Cerebellar vermis lesion

Right cerebral hemisphere lesion


Thoracic spinal cord lesion at T12

A pronator drift is elicited by having the patient extend both arms with the wrists supinated and
palms facing upwards and eyes closed - slow unequal drift towards pronation indicates
hemiparetic weakness. The left arm drop in this case indicates a right cerebral lesion.
The correct answer is: Right cerebral hemisphere lesion

092
Inability to attend to the parts and the whole of an object at the same time is called

Select one:
Somatognosia
Visuospatial apraxia
Prosopognosia
Simultanagnosia
Anosognosia

Simultanagnosia is characterized by the failure of an individual to perceive more than a single


object at a time.
The correct answer is: Simultanagnosia

093
REM pattern is recorded during daytime in a student who is studying for exams for the last two
weeks. The most likely reason is

Select one:

330
Delirium
Antipsychotic use
Cocaine intoxication

Sleep deprivation
Normal pattern

REM sleep during daytime can be seen in narcolepsy, sleep deprivation or withdrawal from
stimulants.
The correct answer is: Sleep deprivation

094
The investigation that has no helpful role in diagnosing narcolepsy is

Select one:
CSF analysis
Human leucocyte antigen typing
Multiple Sleep Latency test
Sleep polysomnogram
Sleep deprivation test

Sleep deprivation test is not helpful to diagnose narcolepsy. The multiple sleep latency test is
helpful in supporting the diagnosis. Findings in sleep polysomnogram include sleep latency of
less than 10 min and sleep-onset rapid eye movement (REM) periods. There is a strong
association with the HLA-DQB1*0602 marker in almost all individuals regardless of ethnicity. An
abnormally low concentration of hypocretin-1 (orexin-A) is noted in CSF. In some cases, human
leukocyte antigen typing may be helpful in determining whether narcolepsy is likely caused by a
lack of hypocretin or another cause.
The correct answer is: Sleep deprivation test

095
Which of the following is characteristic of a senile pupil?

Select one:
Sluggish light and accommodation reaction
Sluggish accommodation reaction; preserved light reaction

331
Exaggerated accommodation and light reaction
Preserved accommodation reaction; sluggish light reaction

No changes in light and accommodation reaction

The pupil size becomes smaller with aging. This constricting of pupil is called senile miosis;
the pupil becomes sluggish to light and accommodation.
The correct answer is: Sluggish light and accommodation reaction

096
During a clinical interview, a doctor tells her patient 'I am impressed by what you have
achieved'. What type of statement is this?

Select one:
Normalisation
Counter-transference

Validation
Negative therapeutic alliance
Statement of respect

Affirmative statements that are genuine and appropriate indicating respect and dignity offer
positive reinforcement during a psychiatric interview.
The correct answer is: Statement of respect

097
All of the following techniques are useful in facilitating disclosure EXCEPT

Select one:
Silence
Positive reinforcement
Acknowledgement of affect
Suggestive questions
Validation

332
The correct answer is: Suggestive questions

098
During the course of a clinical interview, the psychiatrist says, 'So from what you have been
telling me, your sleep is poor, you have excessive disturbing thoughts and you have lost
weight in last 2 months. Is that right?' What technique is he using?

Select one:
Interpretation
Summation

Facilitation
Clarification
Repetition

The correct answer is: Summation

099
Grey matter reduction is seen in patients with psychosis. The transition of patients from ultra
high risk to psychotic patients involves a volume reduction in which part of the brain region?

Select one:
Prefrontal cortex
Caudate nucleus
Cerebellum
Superior temporal gyrus

Hippocampus

333
The correct answer is: Superior temporal gyrus

100
A patient presents with sensory impairment over the contralateral face, arm, and leg involving
pinprick, touch, vibration, position, two-point discrimination, and stereognosis. This patient is
most likely to have a lesion in which of the following structures?

Select one:
Frontal lobe
Thalamocortical projections
Hippocampal region
Amygdalar nucleus
Occipital lesion

Thalamo-parietal projections, when damaged, can give rise to the above reported sensory
disturbances in addition to disturbances in higher sensory integration resulting in failed 2
point discrimination and stereognosis.
The correct answer is: Thalamocortical projections

101
The most appropriate description for executive function is

Select one:
The capacity to identify stimulus that induces a perception
The capacity of problem solving

The capacity of controlling emotion and impulse


The capacity of purposefully applying one's physical skill

The capacity of accomplishing goal-directed behavior

334
The generally accepted descriptions of executive function is that it is the capacity of
purposefully applying one's mental skills, or the capacity of accomplishing goal-directed
behavior.
The correct answer is: The capacity of accomplishing goal-directed behavior

102
Wernicke's encephalopathy is directly caused by a deficiency of

Select one:
Thiamine
Alcohol dehydrogenase
Riboflavin
Niacin
Aldehyde

The correct answer is: Thiamine

103
Which of the following laboratory tests should be conducted prior to initiating lithium
treatment?

Select one:
Alkaline phosphatase
Thyroid-stimulating hormone (TSH)
Platelet count
Erythrocyte sedimentation rate
Calcium

335
As the presence of anti-TSH antibodies can result in hypothyroidism when overlooked, a test
for TSH can come handy before starting lithium.
The correct answer is: Thyroid-stimulating hormone (TSH)

104
In a secluded patient, behavioural observation is carried out every 5th minute through a glass
window. This technique is called

Select one:
Event sampling
None of the above
Time sampling
Functional analysis
ABC analysis

Behavioural observation methods include observing and recording behavioural events, to study
mental state or plan intervention. They are of 2 types: Event sampling: e.g. every fifth or tenth
event is coded in detail. Time sampling: observations may be made only every 5 or 10 mins.

The correct answer is: Time sampling

105
The most common opportunistic infection in a patient with AIDS is

Select one:
Leprosy
Toxoplasma

Cytomegalo virus
Cryptococcus
Syphilis

Toxoplasmosis is common in AIDS. Brain imaging shows multiple ring-shaped contrast


enhancing lesions. MRI is considered superior to CT scanning with IV contrast material with
either modality improving the diagnostic yield and accuracy.
The correct answer is: Toxoplasma

336
106
A patient can follow commands and repeat phrases, but he has an incomprehensible speech.
He is likely to have

Select one:
Transcortical aphasia
Conduction aphasia
Broca's aphasia

Wernicke's aphasia
Dyscalculia

In transcortical motor aphasia repetition and comprehension are intact. In transcortical


sensory aphasia, speech is fluent, repetition is intact and comprehension is impaired.
The correct answer is: Transcortical aphasia

107
A 32-year-old school teacher is admitted for constipation and acute abdominal pain. She
develops motor weakness of her legs on the administration of hypnotics and diclofenac. She
experiences visual and tactile hallucinations with intense anxiety. Which of the following
laboratory test is indicated?

Select one:
Serum folate
Urine porphyrins
Serum lipid levels
Serum ceruloplasmin
Urine glucose

The scenario depicts acute intermittent porphyria (AIP). It is one of the groups of disorders of
haem metabolism, characterised by neurological and psychiatric manifestations without
obvious cutaneous markers. AIP manifests itself by abdomen pain, neuropathies, and
constipation, but, unlike most types of porphyria, patients with AIP do not have a rash. It is an
autosomal dominant disorder with the presentation starting between ages 18 and 40. It is
episodic in nature and the episodes are often triggered by certain medications including
estrogens, barbiturates and benzodiazepines. Diclofenac can precipitate an episode.
Psychiatric manifestations include depression, anxiety, delirium and psychosis. Most important
lab test is demonstrating increased urinary porphobilinogen during acute attacks. Treatment is
aimed at reducing haem synthesis by administering haemin

337
The correct answer is: Urine porphyrins

108
Which of the following is NOT a good technique to use for a poor historian?

Select one:
Using verbal and nonverbal facilitation.
Collecting information in parts over multiple interviews.
Shifting to the neutral ground when necessary
Using open-ended questions

Using multiple choice questions

Use of open-ended questions and commands to increase the flow of information, use of
continuation techniques to keep the flow coming and shifting to the neutral ground when
necessary are useful techniques for a poor historian.
The correct answer is: Using multiple choice questions

109
Which of the following is a supportive technique not aimed at eliciting information?

Select one:
Clarification
Redirection

Validation
Interpretation
Symptom expectation

Validation helps to decrease a patient's sense of embarrassment about a feeling or


behaviour. Validation is often employed when quoting how it is normal for people to have
different emotions/ reactions/ behaviours, etc.
The correct answer is: Validation

110
Failure of object recognition despite normal sensory input and motor functions is calle

338
Select one:
Visual agnosia
Prosopagnosia
Astereognosis

Anosognosia
Neglect

The correct answer is: Visual agnosia

111
Letter cancellation, star cancellation and line bisection tasks examine which of the following
cognitive dysfunction?

Select one:
Visual neglect
Apraxia
Agraphia
Disorientation
Acalculia

The correct answer is: Visual neglect

339
340
001
Mr X is a 56-year-old gentleman assessed in the outpatient clinic. He is presenting with
features of alcohol withdrawal. The patient is asked to complete (SAWS) Short Alcohol
Withdrawal Scale. The nurse wants to know the total scores, above which he might require
pharmacotherapy for withdrawal. Choose the correct answer from the following options.

Select one:
8
10
15

12

(SAWS) short alcohol withdrawal scale is a self-completion questionnaire. Symptoms cover the
previous 24 hour period. Total scores above 12 require pharmacotherapy. The symptoms
assessed include anxiety, sleep disturbance, problems with memory, nausea, restless, tremor,
feeling confused, sweating, and miserable, heart pounding and each rated from 0 to 4 based on
the severity.
The correct answer is: 12

002
The maximum possible score from General Health Questionnaire (28 items version) is

Select one:
84
28

341
12
42

72

GHQ was developed as a screening tool to detect those likely to have or be at risk of
developing psychiatric disorders. It is available in a variety of versions using 12, 28, 30 or 60
items, the 28-item version is used most widely. Each item is scored as a 4 point Likert (0-3)
allowing a total possible score on the GHQ 28 of 0 to 84.
The correct answer is: 84

003
Which of the following rating scale for depression has more cognitive items than the others?

Select one:
Young's rating scale
Hamilton Rating Scale
Beck's depression inventory
General Health Questionnaire
Montgomery Asberg scale

The Beck Depression Inventory (BDI - II) is a 21-item measure designed to assess the
cognitive, behavioural, affective and somatic symptoms of depression. It gives a relatively
higher emphasis on cognitive items that other scales for measuring clinical depression
The correct answer is: Beck's depression inventory

004
Which of the following rating scales for depression used in adults is well validated in children
aged 14 and older?

Select one:
Clifton Assessment Procedure
Beck Depression Inventory
Cornell Depression Scale
Weschler assessment scale
Edinburgh Depression Scale

342
Reynolds Child Depression Scale and the Children's Depression Inventory were developed
specifically for children and are written at lower reading levels. In addition, BDI is validated for
use in those who are above 14 years of age.
The correct answer is: Beck Depression Inventory

005
You want to screen for depression in adolescents of age between 14 and 18-years. Which is
the best screening tool?

Select one:
Hamilton depression rating scale
Child behavioural list
Child and adolescent functional assessment scale
Beck's depression inventory
SCID (Structured clinical interview for DSM-IV)

BDI can be used to screen depression above 14 years age. Other tools mentioned are not
depression specific (CBCL and functioning assessment scales). HAMD is not used as screening
but to assess severity after a diagnosis is established. It is not popularly used for adolescents.
SCID is not a screening tool; it has been used to assess current or lifetime psychiatric diagnoses
in accordance with DSM criteria in any English-speaking person who can participate in the
interview. The majority of studies have used the SCID on adults though it has been successfully
administered to adolescents.
The correct answer is: Beck's depression inventory

006
Which one among the following rating scales is a self-administered instrument?

Select one:
Abbreviated Mental Test
Glasgow Coma Scale

Hamilton Depression Scale


Beck's depression inventory
Clinical global improvement scale

343
Beck depression inventory is a self-rating scale with 21 items measuring the severity of
depression.All others on this list, Brief psychotic rating scale, clinical global improvement
scale, Hamilton rating scale for depression and GCS are all administered by clinician
The correct answer is: Beck's depression inventory

007
Which of the following refers to the extent to which performance on a test fits with theoretical
schema about the test attributes?

Select one:
Criterion validity
Convergent validity
Construct validity
Face validity
Concurrent validity

Construct validity refers to the degree to which inferences can legitimately be made from the
operationalizations in a study to the theoretical constructs on which those operationalizations
were based.
The correct answer is: Construct validity

008
Concurrent validity is also known as

Select one:
Face validity
Construct validity

Incremental validity
Criterion validity
Predictive validity

344
In criterion-related validity, we examine whether the operationalised criteria used in a scale is
aligned to the theory behind a construct that is being tested.
The correct answer is: Criterion validity

009
In psychometry internal consistency refers to

Select one:
Degree of correlation between one test item with other items
Degree of agreement between two raters
Degree of agreement between results obtained in two different occasions
Degree of correlation between two different versions of the scale

Degree of agreement between two similar scales

In internal consistency, which is a type of reliability estimation, we measure how well the items
that reflect the same construct yield similar results. This can be done using average Inter-item
correlation, split half reliability, average item-total correlations or as it is often the case, using
Cronbach's alpha.
The correct answer is: Degree of correlation between one test item with other items

010
Which of the following is a self-rated questionnaire used as a screening instrument for the
presence of psychiatric illness?

Select one:
Global Assessment of Functioning Scale
Quality of Life Interview

Structured Clinical Interview


General Health Questionnaire
Diagnostic Interview Schedule

General Health Questionnaire (GHQ) is a self-rated questionnaire used as a screening


instrument for the presence of psychiatric illness. The patient is asked to report the presence
of a list of symptoms in the preceding weeks.
The correct answer is: General Health Questionnaire

345
011
A tool designed to test cognitive functions has subtests that measure cognitive abilities well.
This test can be said to have

Select one:
Good concurrent validity
Good construct validity
Good predictive validity

Good face validity


Good incremental validity

Construct validity seeks agreement between a theoretical concept and a specific measuring
device or procedure.
The correct answer is: Good construct validity

012
All of the following are projective tests except

Select one:
Rorschach test
Sentence Completion Test
HAMD
Draw A Person Test
Thematic Apperception Test

TAT, Draw-A-Person test, Rorschach test, and Sentence Completion Test are all projective
test. HAMD is a test for depression - it is not a projective test.
The correct answer is: HAMD

013
The SF-36 scale measures which of the following?

Select one:
Occupational Functioning
Stress Level at Work

346
Developmental psychopathology

Health Related Quality of Life


Social Readjustment

Health-related quality of life (HRQOL) refers to functioning and well-being in physical, mental
and social dimensions of life. The SF-36 is a frequently used multi-item HRQOL instrument. The
SF-36 Health Survey (36 item short form health survey) is a generic outcome measure designed
to examine a person's perceived health status. Outcome studies using the SF-36 can be done at
outpatient clinics, via postal survey or telephone interview. The SF-36 can also be used in a
computerised format. It assesses physical, social and mental well-being.
The correct answer is: Health Related Quality of Life

014
Which one of the following scales is used for monitoring clinical recovery?

Select one:
PSE
CAGE
MMPI
AUDIT
HoNOS

Health of the Nation Outcome Scales (HoNoS)was specifically designed as an outcome


indicator for monitoring clinical change.
The correct answer is: HoNOS

015
Which of the following is not a type of validity?

Select one:
Face validity
Construct validity
Ecological validity
Interrater validity
Content validity

347
Inter-rater reliability is different from validity.
The correct answer is: Interrater validity

016
Considering Beck's Depression Inventory, which of the following is true?

Select one:
Refers to symptoms over the last five days.
It has 21 items
Maximum score is 42
Has no item on suicidal ideas

Strictly clinician-rated scale

Beck Depression Inventory was created by Dr. Aaron T. Beck. It is a 21-question multiple-
choice self-report inventory.
The correct answer is: It has 21 items

017
With respect to Young Mania Rating Scale, which of the following is least likely to be correct?

Select one:
It is sensitive to change
It does not have items on suicidality

Psychotic symptoms are not included in the scale


It has 11 items
It is self-rated

Young Mania Rating Scale is the most often used for mania. It does not rate depression or
psychotic symptoms. For studies in which both mania and depression are investigated the
Bech-Rafaelsen scales for both mania and melancholia, may be most appropriate.

348
The correct answer is: It is self-rated

018
Which of the following is correct concerning the SCID?

Select one:
Based on a dimensional diagnostic scheme
Used in conjunction with ICD-10 classification
Provides the Axis V diagnosis in DSM-IV.
Useful for untrained lay interviewers
Primarily used in research

Structured Clinical Interview for DSM (SCID) is a clinician-administered semi-structured


clinical interview for use with patients in whom a psychiatric diagnosis is suspected. It is
primarily used in research with trained interviewers to inform the operationalized diagnosis
psychiatric disorders.
The correct answer is: Primarily used in research

019
Which of the following rating scales retain the aspects of a clinical examination during
consultation?

Select one:
HoNOS
CATEGO
SANS
PSE
BPRS

PSE retains the aspects of a mental state examination


The correct answer is: PSE

020
The Clifton Assessment Procedure is used in which of the following situations?

349
Select one:
Residential care for the elderly
Schizophrenia rehabilitation units
Children in foster care

Care homes for the learning disabled


Prisons with high mental health burden

CAPE (Pattie and Gilleard, 1979) is intended to assess level of disability and estimate need for
care. It consists of a short cognitive scale and a behavioural rating scale. The latter has four
sub-scales: physical disability, apathy, communication difficulties and social disturbance. It is
quick and easy to administer, and is widely used by professional staff and care workers
especially at elderly residential care settings
The correct answer is: Residential care for the elderly

021
Which of the following interview schedules have replaced the Present Status Examination in
contemporary epidemiological studies?

Select one:
SCAN
PANSS
SSPI
CASH
SANS

PSE is now being replaced by SCAN, which incorporates much of the old PSE.SCAN stands
for Schedules for Clinical Assessment in Neuropsychiatry
The correct answer is: SCAN

022
Brief Psychiatric Rating Scale is a clinician-rated scale used to measure the severity of clinical
symptoms based on observation. It is most commonly used in patients with a diagnosis of

Select one:
Alcohol use disorder

350
Personality disorders
Schizophrenia
Bipolar disorder

Somatisation

Brief Psychiatric Rating Scale (BPRS) measures major psychotic and non-psychotic
symptoms, primarily used for schizophrenia patients. Clinician-rated based on observation.
The correct answer is: Schizophrenia

023
What is the most reliable instrument to make clinical diagnosis according to DSM criteria

Select one:
AIMS

SCID
SCAN
HAMD
MMPI

The structured clinical interview for the DSM (SCID) is a semistructured interview that is
arguably the most reliable instrument for psychiatric diagnoses.
The correct answer is: SCID

024
Which of the following is not an alcohol assessment tool?

Select one:
MAST
AUDIT
CAGE
SCOFF
CIWA

351
SCOFF is an eating disorders questionnaire, devised by researchers at St George's Hospital
Medical School. The questions focus on some key symptoms of anorexia and bulimia.
The correct answer is: SCOFF

025
Which of the following is not a rating scale for mood disorders?

Select one:
Beck's inventory
Hamilton Rating scale
Montgomery-Asberg rating scale
Simpson-Angus scale
Young's rating scale

Simpson Angus Rating scale is used to measure extrapyramidal symptoms.


The correct answer is: Simpson-Angus scale

026
Which of the following technique is best utilised minimally if possible in those with high
suggestibility?

Select one:
Open questions
Structured questionnaire

Validation
Summarising techniques
Facilitating comments

Structured interviews have demonstrated a high degree of reliability, validity, and legal
defensibility. But in highly suggestible patients, rigid interview questions are bound to induce
responses that are in agreement with the tone of the question, leading to inaccurate answers.

352
The correct answer is: Structured questionnaire

027
The term test-retest reliability refers to

Select one:
The correlation between the 1st and 2nd test of a number of subjects.
The correlation between the 1st and 2nd test in the same subject
The correlation between the 1st and 2nd test carried out by different raters.
The agreement between 2 raters.
The correlation between ratings before and after treatment.

Test-Retest Reliability: A measure of the ability of a psychological testing instrument to yield the
same result for a single subject at 2 different test periods, which are closely spaced so that any
variation detected reflects reliability of the instrument rather than changes in the subject's status.

The correct answer is: The correlation between the 1st and 2nd test in the same subject

028
You have designed a study questionnaire and would like to measure the inter-rater reliability.
Which of the following will you be testing?

Select one:
The questionnaire is valid when it gives same results if assessed on two
different occasions by same rater
The scores given by one rater are well concealed and cannot be copied by
another rater
The questionnaire gives same results when used by two different raters on the
same patient
Self-rating and clinician rating gives the same score from the questionnaire
The questionnaire is valid when it gives same results if assessed by two
different assessors on two different patients

Inter-rater reliability refers to the consistency among the different raters using a scale or an
instrument. To test inter-rater reliability, each rater is asked to assign a score to each test item.
The correlation between the two (or more) ratings is then computed to determine the level of

353
inter-rater reliability.
The correct answer is: The questionnaire gives same results when used by two different raters
on the same patient

029
Which of the following is a gold standard test for intelligence?

Select one:
Auditory Verbal Learning Test
National Adult Reading Test
Quick Test
Wechsler Adult Intelligence Scale
Wisconsin Card Sorting Test

WAIS is the gold standard intelligence measurement scale that can be used from ages 16 to
89.
The correct answer is: Wechsler Adult Intelligence Scale

030
A 22-year-old man is admitted with troublesome voices discussing his sexuality. He also
believes they influence his actions and control his thoughts. Which of the following instrument
can be used to monitor his symptoms?

Select one:
Schedule for Assessment of Negative symptoms
Brief Psychiatric Rating Scale

Bristol Assessment Scale


Thought Language Index
General Health Questionnaire

BPRS is useful to monitor changes in positive psychotic symptoms.


The correct answer is: Brief Psychiatric Rating Scale

031

354
Which of the following scales can be used by primary care physicians, general medical
professionals and also in community studies?

Select one:
Signs and Symptoms in Psychotic Illness
Scale of Assessment of Positive Symptoms
General Health Questionnaire

Simpson-Angus Scale
Brief Psychiatric Rating Scale

The original General Health Questionnaire (GHQ) comprised 60 items and versions with fewer
items have been developed from this, e.g. the GHQ - 30, GHQ - 28 and GHQ- 12. The GHQ -12
is a brief, well-validated instrument that has been widely used in studies such as the British
Household Panel Survey (BHPS) to detect 'caseness'.
The correct answer is: General Health Questionnaire

032
Which of the following is the most important aspect of construct validity for a test of cognitive
impairment in dementia?

Select one:
The scores depend on educational attainment of the subjects
It predicts the likelihood of progression to end stage dementia
It includes most of the components of cognition that is commonly affected by
dementia
Scores are not affected by taking medications
Experts agree with the choice of sub items for the global scale

Construct validity refers to the degree to which inferences can legitimately be made from the
items in your scale to the theoretical constructs on which the measurement based. When we
measure cognitive impairment using a scale for 'cognition' programme, is our label an
accurate one? When you measure what you term 'self-esteem' is that what you were really
measuring? If our scale includes most of the components of cognition that is commonly
affected by dementia, we can be assured of a good degree of construct validity.
The correct answer is: It includes most of the components of cognition that is commonly
affected by dementia

355
033
Which of the following scales is used for measuring drug-induced movement abnormalities in
general and tardive dyskinesia in particular?

Select one:
AIMS scale
Barnes Akathisia scale
Simpson Angus scale

Lane scale
LUNSERS scale

Abnormal involuntary movements scale is used to examine for TD. Though it includes
Parkinsonian symptoms to some extent, dystonias are not picked up noticeably by AIMS.
The correct answer is: AIMS scale

034
The assessment tool useful in measuring the severity and change in psychotic symptoms is

Select one:
Neuropsychiatric inventory
CATEGO
Young mania rating scale
Brief psychiatric rating scale
Present state examination

BPRS was designed to assess psychopathology (including positive, negative, and affective
psychopathology) in patients with, or suspected of having, schizophrenia or other psychotic
illnesses.
The correct answer is: Brief psychiatric rating scale

035
You have designed a study questionnaire and would like to measure the construct validity.
Which of the following will you be testing?

Select one:

356
The questionnaire predicts what will happen in the future
Constituent parts of the questionnaire are suitable and related to the
phenomenon being tested
The questionnaire covers most symptoms related to the phenomenon being
measured
The questionnaire gives same results when te7v sted on the same patient
twice by a single rater
The questionnaire gives same results when used by two different raters on the
same patient twice

Construct validity has traditionally been defined as the experimental demonstration that a test is
measuring the construct it claims to be measuring. Regardless of how construct validity is
defined, there is no single best way to study it. In most cases, construct validity should be
demonstrated from a number of perspectives. Hence, the more strategies used to demonstrate
the validity of a test, the more confidence test users have in the construct validity of that test, but
only if the evidence provided by those strategies is convincing
The correct answer is: Constituent parts of the questionnaire are suitable and related to the
phenomenon being tested

036
Which one of the following questionnaires can be used to identify psychiatric 'caseness' in the
general population?

Select one:
BDI Beck's depression inventory
HAM-D
Structured clinical interview DSM-IV
GHQ General Health Questionnaire
PSE Present state examination

Goldberg introduced the General Health Questionnaire (GHQ). GHQ was developed as a
screening tool to detect those likely to have or be at risk of developing psychiatric disorders
The correct answer is: GHQ General Health Questionnaire

037

357
Choose the correct statement about Brief Psychiatric Rating Scale?

Select one:
It is an assessment of global psychiatric symptomatology
Each item is rated on a 6-point scale
It focuses on negative symptoms
It is a 30-item scale

It is self administered

The Brief Psychiatric Rating Scale (BPRS) is a rating scale used for quantifying various
psychiatric symptoms (depression, anxiety, hallucinations and unusual behaviour). Each
symptom is rated 1-7 and depending on the version between a total of 18-24 symptoms are
scored.
The correct answer is: It is an assessment of global psychiatric symptomatology

038
Which of the following scales is NOT used for measuring drug-induced movement
abnormalities?

Select one:
LUNSERS scale
Lane scale
Barnes Akathisia scale
Simpson Angus scale
AIMS scale

Lane scale is used for minor physical anomalies


The correct answer is: Lane scale

039
Mr. Y is a 30-year-old man who presented to the A and E after an overdose. He believes that
there are cameras spying on his movements. He has stopped going out and no longer
socialises with friends. He has been hearing voices discussing his sexuality in an unpleasant
way. Which of the following tools would be helpful to assess and monitor his psychopathology?

358
Select one:
MADRS
PANSS
MMSE

HAS
GHQ

Based on two established psychiatric rating systems, the 30-item PANSS was conceived as an
operationalized, drug-sensitive instrument that provides balanced representation of positive
and negative symptoms and gauges their relationship to one another and to global
psychopathology. It thus constitutes four scales measuring positive and negative syndromes,
their differential, and the general severity of illness.
The correct answer is: PANSS

040
Which of the following is true concerning Brief Psychiatric Rating Scale (BPRS)?

Select one:
Useful for global psychiatric symptom ratings
Cannot be used to detect treatment response
It requires no clinical training to administer
It has 20 items
It has 8 items

The BPRS assesses the level of 18 (or 24 in the full version) symptom constructs across
positive, and to some extent negative domain as well as general psychopathology. It is
particularly useful in gauging the efficacy of treatment in patients who have moderate to severe
psychoses. It is based on the clinician's interview with the patient and observations of the
patient's behaviours over the previous 2-3 days. The patient's family can also provide the
behaviour report. Score for each symptom construct ranges from 1 (not present) to 7
(extremely severe).
The correct answer is: Useful for global psychiatric symptom ratings

041
Which of the following scales of depression is most sensitive to change?

359
Select one:
Visual analogue scale
Beck depression inventory
Zung self-rating scale

Montgomery-Asberg depression rating scale


Hospital anxiety and depression scale

Montgomery-Asberg depression rating scale is most sensitive to change. It has a ten items
version, which requires a clinical interview like Hamilton depression rating scale.
The correct answer is: Montgomery-Asberg depression rating scale

042
Which of the following is a useful screening tool for early detection of an eating disorder?

Select one:
Prochaska questionnaire
AUDIT
CAMCOG questionnaire
SCOFF questionnaire
CAGE questionnaire

In terms of screening patients with eating disorder, Not all patients will volunteer their symptoms
or regard themselves as ill. However, early detection of an eating disorder in patients with
unexplained weight loss improves prognosis. The SCOFF questionnaire (Morgan 1999b) is a
useful screening tool, which uses five simple questions, with two or more positive answers
prompting the need to take a more detailed history. Although it is perhaps more commonly used
in primary care, it has also been validated in secondary care and can serve as a helpful
aide-memoire during the assessment process.
The correct answer is: SCOFF questionnaire

043
Which is the most commonly used screening tool to diagnose dementia in older people?

Select one:
CAMDEX

360
Addenbrooke's cognitive examination
Abbreviated mental test score
Mini-mental state examination

Mini-COG

The correct answer is: Mini-mental state examination

361
001
Depersonalisation is also described as a/an

Select one:
'what if' phenomenon

'why me' phenomenon

'if not' phenomenon

'what next' phenomenon

'as if' phenomenon

The correct answer is: 'as if' phenomenon

002
What does the term paranoia literally mean?

Select one:
Grandiosity'
Self reference'
On suspicion'
Besides mind'

362
inside mind'

Paranoia means 'besides one's mind'.


The correct answer is: Besides mind'

003
A 35 year old man in your inpatient unit keeps his ears blocked with cotton and tape. He is
disturbed by two persons discussing between themselves about every movement that he
makes. They also discuss his emotions and plans. The most likely description is

Select one:
Referential delusions 2nd
person hallucinations 3rd
person hallucinations
Thought broadcast
1st person hallucinations

The correct answer is: 3rd person hallucinations

004
Patients with chronic schizophrenia may lose the track of their age and may state an age
different from their actual age. To be called as age disorientation, the difference in stated age
and actual age must be at least

Select one:
One year

10 years
5 years
20 years
3 years

363
The correct answer is: 5 years

005
All of the following are accepted as neologisms except

Select one:
Personal jargon invented to describe the schizophrenic experience A
phrase constructed with incorrect grammar
A new word incorrectly built up with origins clearly understandable An

accepted word used in an unusual way acquiring a new meaning

Completely new word, the derivation of which cannot be understood

There are multiple definitions for neologisms such as; 1. Completely new word that its
derivation cannot be understood 2. A new word incorrectly built up with origins clearly
understandable also known as word approximation E.g handshoe for glove. 3. An accepted
word used in an unusual way acquiring a new meaning 4. Technical neologisms: Personal
jargon invented to describe the schizophrenic experience For example a patient using the
neologism 'aerotelegony' to describe the process by which his thoughts were influenced from
the outside.
The correct answer is: A phrase constructed with incorrect grammar

006
A 42 yr old man is at a haste to catch the last train for the day to London. In his rush, he reads
'Swindon' as London and boards the wrong train. Which of the following has taken place?

Select one:
Hallucination
Pareidolic illusion

Imagery
Completion illusion
Affect illusion

364
Affect illusions arise in the context of a particular mood state as described in this example.
The correct answer is: Affect illusion

007
A patient who is on antipsychotic drugs is seen pacing around the ward but when questioned
he described no subjective discomfort or restlesness in legs. Which of the following best
describes his condition?

Select one:
Chorea
Agitation
Akathisia
Passivity
Dyskinesia

The correct answer is: Akathisia

008
Inability to verbalize one's strong emotions is termed as

Select one:
Ambivalence
Apathy

Anhedonia
Alexithymia
Avolition

Alexithymia: First described by Sifnoes in 1973. It is characterized by inability to verbalize


affect and elaborate fantasies.

365
The correct answer is: Alexithymia

009 Reduced symbolic thinking is a


feature of

Select one:
Ambivalence
Avolition
Apathy
Alexithymia
Anhedonia

Reduced symbolic thinking is a feature of alexithymia. Patients have difficulty in distinguishing


between somatic and psychological feelings.
The correct answer is: Alexithymia

010
Which one of the following is not a negative symptom of schizophrenia?

Select one:
Anhedonia
Attentional impairment

Affective flattening or blunting


Alexithymia
Avolition

The prominent negative symptoms include affective flattening or blunting, attentional


impairment, avolition-apathy (lack of initiative), anhedonia, asociality, alogia (lack of speech
output).
The correct answer is: Alexithymia

011
Which of the following is not a catatonic sign?

Select one:

366
Catalepsy
Posturing
Negativism
Ambivalence
Ambitendence

Ambivalence is an inability to make decisions; it is not a catatonic sign on its own unless it
manifests as ambitendence.
The correct answer is: Ambivalence

012
Which of the following is associated with Ganser's syndrome?

Select one:
Automatisms
Amnesia for behaviour

Confabulation
Anhedonia
Accurate answers

Ganser, in 1898 described a constellation of symptoms clouding of consciousness with


disorientation, auditory and visual hallucinations (or pseudo-hallucinations), amnesia for the
period during which the symptoms were manifest, conversion symptoms following recent
emotional stress in prisoners. This syndrome is dissociative in origin and goes by his name.
The correct answer is: Amnesia for behaviour

013
A depressed patient on the ward complained of total inability to experience pleasure. What is
the term used to denote this type of pathological change in mood?

Select one:
Apathy
Affective flattening
Affective blunting

367
Anhedonia
Incongruity of affect

The correct answer is: Anhedonia

014
Which of the following statements with respect to pseudohallucinations is true?
Pseudohallucinations

Select one:
Are under the control of subject's volition
Are located in the inner subjective space
Are experienced as real and concrete.

Are typically located in the outer objective space.


Are pathognomonic of personality disorders

The correct answer is: Are located in the inner subjective space

015
In which of the following forms does morbid jealousy NOT occur usually?

Select one:
In dementia
As misidentification syndrome
In depression
As delusion

368
As an overvalued idea

The correct answer is: As misidentification syndrome

016
Which of the following is true with regard to the mental state of ecstasy?

Select one:
Associated with persecutory ideas
Associated with somatoform pain state
Cannot occur in the absence of pathological phenomenon
Associated with altered boundary of self

Some degree of altered consiousness is always seen

The correct answer is: Associated with altered boundary of self

017
The patient would think 'I must put the kettle on', and after a pause of not more than one
second would hear a voice say 'I must put the kettle on'. This is best described as

Select one:
Thought insertion

Thought diffusion
Thought provocation
Audible thoughts
Thought broadcast

369
The correct answer is: Audible thoughts

018 Hallucinations in depressive psychosis are


usually

Select one:
Gustatory
Auditory
Olfactory
Visual
Tactile

In depressive psychosis, the hallucinations are usually either second person derogatory or
first person singular with a strong negative connotation.
The correct answer is: Auditory

019
Ekbom's syndrome is least likely to be related to

Select one:
Cocaine use
Dementia

Autism
Schizophrenia
Alcohol withdrawal

Ekbom's syndrome is a delusional belief that one's skin is infested with multiple tiny mite like
organisms. It is a mono-symptomatic delusional disorder where the core delusion pertains to
parasitic infestation. It may be secondary to tactile hallucinations seen in cocaine intoxication or
alcohol withdrawal.

370
The correct answer is: Autism

020
Choose one of the following statements that is true about autochthonous delusions.

Select one:
Autochthonous delusion is a type of secondary delusion
Autochthonous delusions and wahneinfall are equivalent terms.

Autochthonous delusions may give rise to secondary delusions

Retrospective delusions are the same as autochthonous delusions

Autochthonous delusions arise as a result of auditory hallucinations

Primary delusions such as autochthonous delusions may give rise to many secondary delusions
that may get tightly systematized later. For example, a patient who suddenly gets an
autochthonous idea that Martians are invading his office, may then start connecting normal
behaviours of his colleagues to Martians and may form a systematized belief complex involving
cameras, alien sign languages, UFOs etc.
The correct answer is: Autochthonous delusions may give rise to secondary delusions

021
In spite of being instructed not to pull his shirt up and show his abdomen, a patient does the
same whenever a doctor comes to speak to him. Which of the following symptom is he
exhibiting?

Select one:
Automatism
Echopraxia

Negativism
Mannerism
Automatic obedience

In automatic obedience the patient carries out every instruction regardless of the
consequence. To demonstrate this, Emil Kraepelin would ask the patient to put out their
tongue and he would prick it with a pin; patients with automatic obedience continued to put
their tongue out when asked to, although every time they did so their tongue was pricked
The correct answer is: Automatic obedience

371
023
Echolalia is characterised by

Select one:
Automatic repetition of one's own speech
Automatic repetition of the actions of others
Automatic repetition of one's own actions
Virtual absence of movement and speech but preserved consciousness

Automatic repetition of other person's speech

Echolalia refers to the automatic repetition of other person's speech. Echopraxia refers to the
automatic repetition of visually perceived actions of others. Akinetic mutism is the virtual
absence of movement and speech in the presence of full consciousness.
The correct answer is: Automatic repetition of other person's speech

024
Phantom mirror image is a synonymous term for which one of the following hallucinations?

Select one:
Reflex hallucination
Autoscopic hallucination
Haptic hallucination
Hypnagogic hallucination
Teichoscopic hallucination

Autoscopy, also called phantom mirror-image, is the experience of seeing oneself and knowing
that it is oneself. It is not just a visual hallucination because kinaestethic and somatic sensation
must also be present to give the subject the impression that the hallucinated percept is one's
own self.
The correct answer is: Autoscopic hallucination

025
Kinaesthetic hallucination is reported in cases with

Select one:

372
Amphetamine withdrawal
Benzodiazepine withdrawal
Cocaine withdrawal

Alcohol withdrawal
Cannabis withdrawal

Symptoms of benzodiazepine withdrawal include autonomic hyperactivity, increased tremor,


insomnia, nausea or vomiting, transient visual, tactile, or auditory hallucinations or illusions,
psychomotor agitation. anxiety (most prominent), with grand mal seizures and kinaesthetic
hallucinations reported in a few patients.
The correct answer is: Benzodiazepine withdrawal

026
A patient, who lost his right arm amputated following a crush injury, suffers from recurrent
sensations arising out of the lost limb. This experience is a

Select one:
Delusion
Body image disturbance
Hallucination
Pseudohallucination
Somatisation

The correct answer is: Body image disturbance

027
Circumstantial speech is seen in all EXCEPT

Select one:
Learning disability

373
Mania
Anankastic personality disorder
Dementia

Broca's Aphasia

In Broca's aphasia, fluency is often reduced.


The correct answer is: Broca's Aphasia

028
Which of the following is true with regard to primary delusions?

Select one:
They are more common in chronic rather than in acute schizophrenia.
They are usually well elaborated.

The content is usually aligned to cultural and social norms


Their origin can be traced back to a person's premorbid mental state

By definition, they are not understandable

Jaspers defined primary delusions as un-understandable. Their 'primary' nature also means
that no observable abnormalities in mental state precede these delusions. In contrast,
secondary delusions arise out of other mental phenomenon such as hallucinations or euphoric
mood etc.
The correct answer is: By definition, they are not understandable

029
A patient on an adult psychiatric ward says that Dr. Smith has been his consultant for the past
15 years. Even though he recognises his face, he knows he is not Dr. Smith anymore but an
imposter. What is this phenomenon called?

Select one:
Capgras syndrome
Fregoli's syndrome
Borognosia
Prosopognosia

374
Illusion of doubles

The correct answer is: Capgras syndrome

030
Which of the following is characterised by patients maintaining unusual postures for long time
with no resistance to a passive movement?

Select one:
Cataplexy
Advertence
Spasticity
Mitgehen

Catalepsy

Advertence is a catatonic sign wherein a schizophrenic patient dramatically turns fully towards
the doctor/examiner when he is spoken to. Aversion is another catatonic feature wherein the
patient turns completely away.
The correct answer is: Catalepsy

031
Which one among the following is not a motor symptom of schizophrenia?

Select one:
Cataplexy

Psychological pillow
Negativism
Ambitendency
Catalepsy

375
Recognised motor symptoms of schizophrenia include catatonia, catalepsy, negativism,
ambitendency, psychological pillow, mannerism, stereotypy, echopraxia, mitmachen,
mitgehen and automatic obedience.
The correct answer is: Cataplexy

032
Which of the following is a thought disorder where figure-ground differentiation apparently
fails?

Select one:
Neologism
Concrete thinking
Tangentiality
Circumstantiality
Vorbeireden

The correct answer is: Circumstantiality

033
Choose the incorrect association from the following list

Select one:
Flight of Ideas-mania
Thought blocking-schizophrenia

Circumstantiality-obsessive compulsive disorder


Perseveration- Frontal lobe syndrome
Inhibition of thinking-Depression

The correct answer is: Circumstantiality-obsessive compulsive disorder

376
034
The most common type of seizure with an aura is

Select one:
Pseudoseizures
Grandmal seizure
Absence seizure

Complex partial seizure


Parietal seizure

The correct answer is: Complex partial seizure

035
A patient with OCD has intrusive thoughts about the safety of his family. He fights these
thoughts by often tapping his desk 5 times and also by praying mentally 5 times as fast as he
could. The acts of praying are best termed as

Select one:
Spiritual obsessions
Delusions
Normal behaviour
Coping skill
Compulsions

The correct answer is: Compulsions

036
A man shouts 'Biance' every time he sees a pretty woman. Which of the following best
describes this phenomenon?

377
Select one:
Compulsive act
Obsessive image
Obsessive impulse

Mental compulsion
Rumination

Compulsions occur as motor acts. They may not be preceded by an identifiable obsession
especially in disorders other than OCD such as autism, dementia, Tourette's etc. where
compulsive acts can be seen without preceding obsessions.
The correct answer is: Compulsive act

037
Suggestibility is a prominent feature of

Select one:
Perseveration
Circumstantial thinking
Tangentiality
Over inclusive thinking
Confabulation

The role of suggestibility is important in those who present with confabulation, pseudologia,
retrospective falsification or false memory. Suggestible subjects accept statements from
others, act upon their commands and deny evidence from the senses or from rational
understanding that would contradict these statements.
The correct answer is: Confabulation

038
Astasia-Abasia has been observed in

Select one:
Dissociative fugue
Conversion disorder
Depressive pseudodementia

378
Phobic anxiety disorder
Schizophrenia

Astasia-abasia is also known as Blocq's disease. This is the inability to walk or stand in a
normal manner. The gait appears bizarre, unpredictable and not suggestive of any organic
lesion. It is often characterized by swaying and almost falling, with recovery at the last
moment. It is a conversion symptom (dissociative motor disorder in ICD-10 and conversion
disorder in DSM-IV).
The correct answer is: Conversion disorder

039
La belle indifference is typically seen in

Select one:
Multiple sclerosis
Schizophrenia
Conversion disorder
Dissociative amnesia
Dissociative fugue

The correct answer is: Conversion disorder

040
Which of the following is NOT a delusional misidentification syndrome?

Select one:
Fregoli syndrome
Cotard's syndrome

Capgras syndrome
Reduplicative paramnesia

379
Syndrome of subjective doubles

Fregoli syndrome: The patient believes that strangers have been replaced by familiar people.
So, an unfamiliar person is credited with the identity of a familiar person. Delusions of
misidentification to persons include Capgras, Fregoli, Intermetamorphosis and syndrome of
subjective doubles. Delusions of misidentification to places include reduplicative paramnesia.
The correct answer is: Cotard's syndrome

041
An 82-year-old lady admitted to the psychiatric ward insists that she is already dead and
should be buried, and her viscera have stopped working and are rotting away. What is the
probable diagnosis?

Select one:
Couvade syndrome
Briquet syndrome
De Clerembault syndrome
Cotard's syndrome

Hypochondriacal syndrome

Cotard's syndrome is a presentation of psychotic depressive illness seen particularly in elderly


people where there is a combination of depressed mood with nihilistic delusions and
hypochondriacal delusions.
The correct answer is: Cotard's syndrome

042
Which of the following syndromes and delusional types are not correctly associated?

Select one:
Morbid jealousy-delusions of marital infidelity
De Clerembault syndrome-Delusions of love
Cotard's syndrome-nihilistic delusions
Ekbom's syndrome-delusions of infestation
Couvade syndrome-delusion of pregnancy in postpartum mothers

380
Couvade syndrome is a conversion symptom seen in partners of expectant mothers during their
pregnancy. The symptoms vary but mimic pregnancy symptoms, and so include nausea,
vomiting, and abdominal pain and food cravings.
The correct answer is: Couvade syndrome-delusion of pregnancy in postpartum mothers

043
A woman believes that the tennis player Roger Federer is in love with her. She has never seen
him in person but believes that he is delivering romantic e-mails to her inbox in various
disguised names. She is exhibiting

Select one:
De Clerambault's delusion
Cotard delusion
Morbid jealousy
Capgras delusion
Fregoli delusion

The correct answer is: De Clerambault's delusion

044
Which of the following cannot be considered as a component of descriptive psychopathology?

Select one:
Defence mechanisms
Understanding patient's experience

Using terms to describe behaviour


Establishing rapport with patient
Empathetic approach

381
The correct answer is: Defence mechanisms

045
A patient complains that people come to know what she is thinking by looking into her eyes.
This can be described as

Select one:
Overvalued idea
Thought broadcast
Thought alienation
Delusion of reference
Thought withdrawal

The correct answer is: Delusion of reference

046
A male patient with a diagnosis of schizophrenia says 'I had a hysterectomy at age 3 and
since then I became a man'. This can be described as which of the following symptom?

Select one:
Delusional misinterpretation
Delusional memory

Pseudologia fantastica
Delusional perception
Confabulation

382
The correct answer is: Delusional memory

047
Which one of the following is not a type of primary delusion?

Select one:
Delusional atmosphere
Delusional memory
Delusional misidentification
Delusional perception
Autochthonous delusions

The 4 types of primary delusions include delusional mood, delusional perception, delusional
memory and autochthonous delusions
The correct answer is: Delusional misidentification

048
A sense of perplexity and uncertainty that exists during a prodrome of psychosis may be
linked most appropriately to which of the following experiences?

Select one:
Negative symptoms
Delusional memory

Autochthonous delusions
Depression
Delusional mood

.
The correct answer is: Delusional mood

049
Which one of the following statements about delusional mood is incorrect?

383
Select one:
Delusional mood is same as delusional atmosphere.
The patient feels profoundly apprehensive and uncomfortable

Delusional mood is often followed by a fully formed delusion

Delusional mood is characteristically associated with perplexity

Delusional mood is a first rank symptom of schizophrenia

Delusional mood is a primary delusional state, but not a FRS.


The correct answer is: Delusional mood is a first rank symptom of schizophrenia

050
On seeing a cat crossing the road, a cab driver starts thinking that the MI5 are about to arrest
him soon. This is an example of

Select one:
Delusional perception
Somatic passivity
Delusional misinterpretation
Thought insertion
Delusional memory

The correct answer is: Delusional perception

051
When one of the other residents pushed a salt cellar towards him, a patient knew that he must
return home 'to greet the Pope who is visiting Ireland to see his family and reward them'. This is
called

Select one:
Partial delusion
Delusional misinterpretation
Referential delusion

384
Grandiose delusion
Delusional perception

This is a delusional perception; in grandiose delusions a sense of falsely elevated self esteem is
often seen.
The correct answer is: Delusional perception

052
A patient at your clinic reports, 'When I heard the railway station master's whistle, I knew that
there was a plot to attack me.' This is called

Select one:
Referential delusion
Secondary delusion
Delusional mood

Delusional perception
Partial delusion

A delusional perception must not be confused with delusional misinterpretation. In the former,
there will be no rational link between the judgement arrived at and the original perception; but in
the latter some link via mental state, mood, anxiety or relevance can be easily discerned.
The correct answer is: Delusional perception

053
Which one of the following phenomena can be described as ego-syntonic?

Select one:
Delusions
Depersonalisation
Derealization
Both depersonalisation and derealization
Obsessions

385
Delusions are ego-syntonic. The subject who experiences them does not feel uncomfortable
by the mere presence of the phenomenon. The rest all described above are ego-dystonic,
where the phenomena per se, not merely the consequences, are distressing and
unacceptable.
The correct answer is: Delusions

054
Which of the following is not included in the classification of Delusional Disorders in ICD 10?

Select one:
Delusions of infestations
Delusions of grandiosity
Delusions of control
Delusions of love
Delusions of jealousy

Delusions of control are bizarre delusions seen often in schizophrenia-like disorders, and not in
a persistent delusional disorder.
The correct answer is: Delusions of control

055
Which of the following statements is not true with regard to depersonalisation?

Select one:
Depersonalisation is rare in psychiatric practice
Depersonalisation is often a difficult experience to describe

Depersonalisation is a frequent experience of normal healthy people

Depersonalisation is often accompanied by the symptom of derealization.

Affect is invariably involved in depersonalisation

Depersonalisation is a common occurrence in normal population (around 10-30% reported


prevalence) and in psychiatric clinics (more than 25% in some series).

386
The correct answer is: Depersonalisation is rare in psychiatric practice

056
Erotomania has been reported in association with all EXCEPT

Select one:
Paranoid schizophrenia
Mania
Delusional disorder.
Alcoholism
Depersonalisation syndrome

DeClerambault's syndrome is a form of delusion of love (erotomania). The patient is usually a


woman who believes that a celebrity / someone at a higher social status is in love with her.
The correct answer is: Depersonalisation syndrome

057
Psychomotor retardation is a characteristic feature of

Select one:
Dementia
Schizophrenia

Borderline personality disorder


Depression
Bulimia

Melancholia is a form of depression that is defined as a quality of mood, which is distinct from
grief, occurring in association with significant psychomotor retardation often with somatic
symptoms of depression (as described in ICD-10).
The correct answer is: Depression

058
The commonest psychiatric diagnosis reported in patients with autoscopic hallucinations is

Select one:

387
Depression
Hysterical Dissociative states
Schizophrenia
Dementia

Bipolar disorder

It is also seen in normal people when they have extreme fatigue, where it is usually
accompanied by impaired consciousness. Sims states that depression is the commonest
psychiatric cause of autoscopy.
The correct answer is: Depression

059
Which of the following is the commonest cause of stupor in psychiatric units?

Select one:
Mania
Epilepsy

Dissociation
Depression
Stroke

Stupor is a state of more or less complete loss of activity where there is no reaction to external
stimuli; it can be regarded as an extreme form of hypokinesia and the most common cause in
psychiatry wards is depression.
The correct answer is: Depression

060
Agitated depression is a well-recognized feature of

Select one:
Depression in children
Depression in young men
Depression in young women
Depression in the elderly

388
Depression in middle aged women

This is a mixed affective state described by Emil Kraepelin.


The correct answer is: Depression in the elderly

061
Which of the following is not a mixed affective state?

Select one:
Depressive mania
Inhibited mania
Depression with poverty of thought
Excited/agitated depression

Manic stupor

In mixed states incongruity between motor features and mood state is found: mania with
inhibition, depression with excitation etc.
The correct answer is: Depression with poverty of thought

062
Of various thought disorders classified by Andreasen, which of the following is more common in
schizophrenia than mania?

Select one:
Circumstantiality

Neologisms
Flight of ideas
Clanging
Derailment

389
Of various thought disorders classified by Andreasen, clanging and flight are more common in
mania while derailment (loosening) and thought blocking and to some extent tangentiality and
poverty of content of speech are seen more often in schizophrenia - other items were largely
non specific for the 2 diagnoses.
The correct answer is: Derailment

063
Elderly women who develop Ekbom's delusion most commonly visit which of the following
medical department before being referred?

Select one:
Psychiatric
Rheumatology
Neurology
Ophthalmology
Dermatology

While the term Ekbom syndrome is used in Parkinson's to describe Restless Legs Syndrome,
Ekbom's delusion refers to delusional parasitosis.
The correct answer is: Dermatology

064
When the word 'pink' is considered, linked words such as 'Barbie' and 'girls' get activated in
brain. This is called

Select one:
Syntax
Indirect priming

Secondary priming
Direct priming
Cohesion

Reaction times are decreased in response to words preceded by semantically related words
('cat'-'dog') in comparison with words preceded by semantically unrelated words ('bus'-'dog'),
this is known as semantic priming effect. "When a concept node is activated by corresponding
word stimulus, this activation then spreads through the network to connected nodes, falling off

390
with decreasing relatedness. The degree to which one concept activates another is thus
presumably proportional to their relatedness in the semantic network". Large priming effects for
indirectly related words (those related only through at least one other concept, such as 'cat' and
'cheese,' mediated by 'mouse') have been observed in schizophrenia patients with thought
disorder. This is called indirect priming.
The correct answer is: Direct priming

065
Pseudologia fantastica is most commonly associated with

Select one:
Pick's dementia
Frontal lobe damage
Obsessional personality
Dissocial personality
Borderline personalityNarcissistic perosnality

Pseudologia fantastica or fluent plausible lying (pathological lying) is seen in personality


disorders of antisocial or hysterical type. It often carries a grandiose tinge and can be
triggered by a personal crisis, such as facing legal proceedings. A person with pseudologia
may believe their own stories and often there is a blur between fantasy and reality. When
confronted, these individuals may admit to lying.
The correct answer is: Dissocial personality

066
Which of the following best describes paraschemazia?

Select one:
Mixture of meaningless words
Distortion of body image

Inappropriate familiarity in a new place


Distorted awareness of time
Loss of emotional control following brain injury

391
Paraschemazia or distortion of body image is described as a feeling that parts of the body are
distorted or twisted or separated from the rest of the body and can occur in association with
hallucinogenic use, with an epileptic aura and with migraine on rare occasions.
The correct answer is: Distortion of body image

067
Retardation of thinking seen in depressive disorder is appropriately classified as which one of
the following?

Select one:
Disturbed form of thought
Disturbed content of thought
Disturbed control of speech
Disturbed thought agency
Disturbed stream of thought

Severe depression with psychomotor retardation may be associated with mutism, but more
often there is poverty of speech and the patient replies to questions in a slow and drawn-out
fashion: this is a disturbance in thought stream.
The correct answer is: Disturbed stream of thought

068
Musical hallucinations has been reported in all of the following except

Select one:
Temporal lobe lesions
Cochlear defects

Schizophrenia
Dominant parietal lobe lesions
Sensory deprivation

Hearing loss or deafness is a very important factor associated with musical hallucinations. The
hallucinatory experiences are more intense when the surrounding noise is low. Brain imaging
studies of patients with musical hallucinations reveal a dysfunctional temporal cortex, especially
on the left more than the right hemisphere.
The correct answer is: Dominant parietal lobe lesions

392
069
Epileptic automatism is associated with

Select one:
Preserved consciousness
Loss of postural control
Presence of aura
Loss of muscle tone
Duration lasting few seconds to few minutes

In the presence of disturbed consciousness, but preserved muscle tone and postural ability,
patients with temporal lobe epilepsy may sometimes carry out complex motor acts termed as
epileptic automatisms. These are usually shortlived.
The correct answer is: Duration lasting few seconds to few minutes

070
Changes in the shape of objects especially with a loss of symmetry is called

Select one:
Eidetic imagery
Macropsia

Micropsia
Dysmegalopsia
Asterognosis

Dysmegalopsia refers to a change in the perceived shape of an object. Some authors reserve
the term dysmegalopsia to describe objects that are perceived to be larger (or smaller) on one
side than the other, while others use the term generically to describe any change in perceived
size . Others use the term metamorphosia rather than dysmegalopsia to describe objects that
are irregular in shape.
The correct answer is: Dysmegalopsia

071
Which of the following is not a culture-bound disorder?

393
Select one:
Bouffee delirante
Koro
Frigophobia

Windigo
Dysmorphophobia

The correct answer is: Dysmorphophobia

072
A 7-year-old girl speaks very clearly and fluently with her friends at school but becomes mute
when at home. This is called

Select one:
Akinetic mutism
Poverty of content of speech
Elective mutism
Poverty of speech
Hysterical mutism

The correct answer is: Elective mutism

073
A 64-year-old patient admitted in a stroke ward bursts out to laughter or tears within minutes
with no control over these emotions. What is the psychopathology seen in this case?

Select one:
Depression
Mania
Mixed affective state

394
Grief reaction
Emotional incontinence

The correct answer is: Emotional incontinence

074
Which of the following is NOT a catatonic sign?

Select one:
Posturing
Flexibilitas cerea
Mitgehen

Negativism
Essential tremor

Essential tremor is an autosomal dominant condition with incomplete penetrance (35% no


family history); it is worsened by stress and improved by alcohol.
The correct answer is: Essential tremor

075
Delusions of motor control are thought to be related to

Select one:
Loss of proprioceptive reflexes
A form of motor neglect
Paroxysmal spike activity in premotor area
Failure to predict sensory feedback of one's own movements

Inability to control execution of motor maps

395
According to this sensory-feedback model if something seems to be going wrong with the
action, it is quite possible to correct for it on the basis of concurrent self-awareness. Delusions
of motor control are thought to be caused by a failure to predict sensory feedback of own
movements (One of the models used to explain delusions of control in patients with
schizophrenia).
The correct answer is: Failure to predict sensory feedback of one's own movements

076
Deja vu and jamais vu are disorders of

Select one:
Familiarity
Volition
Perception
Behaviour
Mood

These are best described as pathological states involving familiarity.


The correct answer is: Familiarity

077
A constellation of unchanging facial expression, decreased spontaneous movements, poverty of
expressive gestures, poor eye contact and affective non responsivity is termed as

Select one:
Lability of affect
Incongruity of affect

Flattening of affect
Reactive affect
Blunting of affect

Flattening of affect occurs in schizophrenia, depression and brain lesions.

396
The correct answer is: Flattening of affect

078
A patient says 'I am constantly being followed by people I know, but I can't recognize them
because they are always in disguise'. What symptom is she exhibiting?

Select one:
Capgras delusion
Fregoli delusion
Intermetamorphosis
Cotard delusion
De Clerambault's delusion

The correct answer is: Fregoli delusion

079
The organic states associated with autoscopic hallucinations of seeing oneself in external
space include all except

Select one:
Parietal tumours
Toxic infective states

Temporoparietal lesion
Epilepsy
Frontal lobe dementia

The organic states commonly associated with autoscopy are epilepsy, focal lesions affecting
the parieto-occipital region and toxic infective states affecting the basal regions of the brain.
Occasionally patients with schizophrenia have autoscopic hallucinations but they are more
common in acute and sub-acute delirious states.
The correct answer is: Frontal lobe dementia

397
080
Choose one incorrect statement about fugue states:

Select one:
Consciousness is unimpaired.
It is a dissociative reaction to unbearable stress.
Fugue state generally has an insidious onset
It is associated with amnesia
Spontaneous resolution is a common outcome

Fugue states are rare syndromes with an abrupt onset and a duration of few hours/days.
There will be loss of episodic memory and sense of personal identity. It is associated with a
period of wandering, leaving residual amnesic gaps for the period of the fugue. The factors
predisposing would include stress, depressed mood, head injury, alcohol abuse and
epilepsy.(Kopelman)
The correct answer is: Fugue state generally has an insidious onset

081
A patient can hear voices whenever the noise of water running through a tap is heard. This is
called

Select one:
Reverse hallucination
Synesthesia
Reflex hallucination
Functional hallucination
Extracampine hallucination

In functional hallucination, "an auditory stimulus causes a hallucination but the stimulus is
experienced as well as the hallucination. In other words, the hallucination requires the
presence of another real sensation. For example, a patient with schizophrenia first heard the
voice of God as her clock ticked; later she heard voices coming from the running tap and
voices coming from the chirruping of the birds" .
The correct answer is: Functional hallucination

082

398
Vorbereiden is seen in

Select one:
Couvade syndrome
Tourette's syndrome
Ganser syndrome

Cotard syndrome
Asperger's syndrome

Vorbereiden is approximate answering seen in those with a variant of hysterical


pseudodementia described by Ganser. It is different from the flight of ideas. It is often used
interchangeably with vorbeigehen and is often described as a part of the rare Ganser
syndrome.
The correct answer is: Ganser syndrome

083
Which of the following is a term used to describe the phenomenon of hearing one's own
thoughts aloud?

Select one:
Mitmachen
Mitgehen
Gegenhalten
Gedankenlautwerden
Word salad

The correct answer is: Gedankenlautwerden

084
Which of the following correctly describes a hypnagogic hallucination?

399
Select one:
Hallucinations on waking up from sleep
Hallucinations when falling asleep
Hallucinations that occur only in day time sleep

Hallucinations that form a part of dreaming


Hallucinations that induce a sleep-like state

Hallucinations on waking up from sleep are called hypnopompic hallucinations.


The correct answer is: Hallucinations when falling asleep

085
Flashback phenomenon is commonly reported after ingesting which of the following
substances?

Select one:
Cocaine
Psychotropic medication
Nicotine
Hallucinogen
Psychostimulant

The correct answer is: Hallucinogen

086
Which of the following factors predict a high persistence of delusional ideas?

Select one:

400
Younger patients
Having a diagnosis of schizophrenia
Married patients

Having a delusion of grandiosity


Having single primary delusion

A diagnosis of schizophrenia is the best predictor of persistence of delusional beliefs.


The correct answer is: Having a diagnosis of schizophrenia

087
A 66 yr old patient diagnosed with schizophrenia is observed to have rigidity at rest but carries
out voluntary movements normally. Which of the following is true?

Select one:
He has extrapyramidal rigidity from antipsychotics
He is a malingerer
He has spasticity due to cerebrovascular pathology
He does not have catatonia
He is exhibiting catatonia

Increased resting muscle tone that comes down during voluntary action is characteristic of
catatonia.
The correct answer is: He is exhibiting catatonia

088
A 78-year-old patient is observed to be lying when asked what he had for breakfast or who
visited him the day before. He is not embarrassed by this and does not appear to be jocular
with regard to the lies; the lies are usually trivial bringing no harm to anyone. Which of the
following is true?

Select one:
He is having memory difficulties
He is having Ganser's syndrome
He is playing practical jokes intentionally

401
He is exhibiting pseudologia fantastica
He has an antisocial personality

Confabulation refers to creating stories to fill gaps in memory - an unconscious behavior seen in
patients with memory impairment. It is not intentional lying.
The correct answer is: He is having memory difficulties

089
The Doppleganger phenomenon is best described as a/an

Select one:
Somatoform disorder
Ideational disturbance
Prerceptual disturbance

Malingering phenomenon
Body image disturbance

It refers to the awareness of another human being accompanying the self. Reported in
psychosis and in severe sleep deprivation.
The correct answer is: Ideational disturbance

090
Which one of the following statements about illusions is false?

Select one:
Illusions can be brought on by sensory deprivation
Visual illusions may be influenced by the prevailing emotional tone.

Both affect and complete illusions resolve on closer attention.

Illusions occur when the perception threshold is increased Illusions

can be normal experiences

402
Illusions occur when the sensory perceptual threshold is reduced, as in delirium, leading to
inappropriate perceptions from ambiguous or incorrect stimuli.
The correct answer is: Illusions occur when the perception threshold is increased

091
Which one of the following is not true about Folie a Deux?

Select one:
The person with the primary psychosis is usually the active member. In
folie, a deux the associate usually experiences a primary delusion The
associate is usually in some way disadvantaged and dependent The
commonest relationship among sufferers is that of twin sisters.
The delusions are usually persecutory in nature.

Folie a deux is a term derived from French words meaning 'madness shared by two'. An
individual develops a delusion similar to another individual who already has an established
delusion; usually the two individuals have a close relationship (often sisters). In DSM­5, this is
termed as Shared Psychotic Disorder.
Note: Folie a trios - madness shared by three. Folie a quatre - madness shared by four. Folie a
famille - madness shared by the family. Folie a plusieurs - madness shared by many.
The correct answer is: In folie, a deux the associate usually experiences a primary delusion

092
The phenomenon of deja vu is

Select one:
Inappropriate familiarity to a new place or event
Transient loss of sense of familiarity to a well accustomed place
Inappropriate familiarity with a new thought

Loss of sense of familiarity to a well known person

Mixture of meaningless words

. Deja entendu, the feeling of auditory

403
recognition, and deja pense, a new thought recognised as having previously occurred, are
related to deja vu, being different only in the modality of experience. These can be
experienced by normal subjects as well as among those with temporal lobe epilepsy.
The correct answer is: Inappropriate familiarity to a new place or event

093
A middle-aged gentleman was noticed to be giggling inappropriately on hearing the news of his
father's death. What is the term used to denote this type of pathological change in affect?

Select one:
Affective blunting
Apathy
Affective flattening
Anhedonia
Incongruity of affect

The correct answer is: Incongruity of affect

094
Which of the following is NOT a feature associated with alexithymia?

Select one:
Increase in abstract thinking ability
Reduced symbolic thinking

Difficulties in differentiating body sensations and emotional states.


Dimunition of fantasy
Difficulties in recognizing one's own feelings

Alexithymia is a condition where a person is unable to describe his or her emotions in words.
Frequently, alexithymic individuals are unaware of what their feelings are, and may have
diminished abstract and fantasizing abilities.
The correct answer is: Increase in abstract thinking ability

404
095
Which of the following can differentiate depersonalisation seen in normal persons and those
with a depersonalisation-related psychiatric problem?

Select one:
No difference is notable between the two groups
Shorter duration in the latter
Intense affective change is seen in the latter

Feelings of detachment is not seen in the former


Sense of time is not altered in the former

The correct answer is: Intense affective change is seen in the latter

096
A woman states that she is often woken up when drifting to sleep by a voice saying a sentence
or phrase that has no discoverable meaning. Which one of the following statements about her
experience is true?

Select one:
It does not occur in organic states
It can occur in the absence of a diagnosable mental illness
EEG is likely to show beta rhythm
Such experiences can occur only in the auditory modality
It is a sign of nocturnal epilepsy

The correct answer is: It can occur in the absence of a diagnosable mental illness

405
097
Which one of the following is not a characteristic feature of transsexualism?

Select one:
It is characterized by normal anatomical sex.
The diagnosis is usually made after puberty.
It is a delusional belief of mistaken sexual identity

Distress is due to the inappropriateness of one's anatomical sex to


perceived gender
Subjects show a marked preoccupation with the wish to undergo surgical
correction

Transsexualism occurs as an overvalued idea - not a delusion.


The correct answer is: It is a delusional belief of mistaken sexual identity

098
A patient answers 'five' when asked how many days a week has. Which of the following is true
about the psychopathology exhibited by the patient?

Select one:
It is done intentionally
It is seen only in prison population
It is not seen in schizophrenia
It is a disturbance of thought control
It is a disturbance of thought form

Approximate answers suggest that the patient understands the questions but appears to be
deliberately avoiding the correct answer. It is described under formal thought disturbances
though it is often thought to be dissociative in origin.
The correct answer is: It is a disturbance of thought form

099
Which of the following is true concerning depersonalisation?

Select one:

406
It is experienced as an unpleasant state
The subject feels that the world around him is unreal
Multiple personalities emerge during the experience

It is associated with approximate answers


The subject cannot recall events later

Depersonalisation is a non-specific symptom that is often associated with an unpleasant


subjective state, especially during situations of pathological nature.
The correct answer is: It is experienced as an unpleasant state

100
Which one of the following statements about delusional perception is false?

Select one:
It refers to a type of secondary delusion
A normal perception is interpreted with delusional meaning and has
immense personal significance.
It may be preceded by delusional mood.
It refers to a delusion that follows a normal perception.
It is one of the first rank symptoms of schizophrenia

Delusional perception is a primary delusion and a Schneiderian First Rank Symptom.


The correct answer is: It refers to a type of secondary delusion

101
Which one of the following statements regarding deja vu and jamais vu is false?

Select one:
Deja vu is associated with inappropriate familiarity
Deja vu may occur in normal individuals
Deja vu is unusual in schizophrenia

Jamais vu is the sense that events being experienced for the first time have
been experienced before.

407
Deja vu occurs more often in TLE with a right-sided focus

Jamais vu is described as the loss of sense of familiarity when an event that has been
attended before is experienced once again.
The correct answer is: Jamais vu is the sense that events being experienced for the first time
have been experienced before.

102
Which of the following neurological disturbances closely mimic schizophrenic speech
disturbance?

Select one:
Anomia
Motor aphasia
Jargon aphasia
Non fluent aphasia

Conduction aphasia

There is more overlap between formal thought disorder and Wernicke's aphasia than any other
form of aphasia But compared to Wernicke's aphasia (jargon aphasia), schizophrenic discourse
often has a preferred theme or preoccupation; aphasic discourse rarely does. Speakers with
schizophrenia often jump from one subject to another based on the sounds or associations of
words they have uttered (association chaining or glossomania). This is seldom observed in
jargon aphasia. Schizophrenic discourse often includes rare words, evidence of a large, intact
vocabulary; jargon aphasia, even when very fluent, shows a restricted vocabulary. More
importantly, verbal comprehension remains intact in schizophrenia while it is impaired in
Wernicke's aphasia.
The correct answer is: Jargon aphasia

103
Who first considered the distinction between 'understanding' and 'explaining' as a core skill in
psychiatric practice?

Select one:
Jaspers

Durkheim
Kraepelin

408
Kretschmer
Schneider

Jaspers is often regarded as the father of phenomenology in psychiatry. He distinguished


'understanding' as a component that is different from 'explaining' an observed phenomenon.
The correct answer is: Jaspers

104
In formal thought disorder, asyndesis refers to

Select one:
Lack of genuine causal links in speech
Lack of logical arguments in speech
Lack of wide vocabulary in speech

Lack of information in speech


Lack of adjectives in speech

Cameron (1944), used the term 'asyndesis' to describe the lack of adequate connections or
causal links between successive thoughts
The correct answer is: Lack of genuine causal links in speech

105
Apathy is characterised by

Select one:
Lack of eye contact
Lack of movements
Lack of insight
Lack of feeling
Lack of speech

409
Apathy is characterised by an absence of feelings, in extreme states presenting as a lack of
motivation, desire and will. It is associated with anergia and lack of volition.
The correct answer is: Lack of feeling

106
In logoclonia, the subject keeps repeating which of the following?

Select one:
Last syllable
Last word
Examiner's question
Recently hear word
Last sentence

Logoclonia: Spastic repetition of a terminal syllable. It occurs in Parkinsonism.


Verbigeration: repetition of senseless sounds, syllables or words. It occurs in expressive
aphasia and catatonic schizophrenia.
The correct answer is: Last syllable

107
A patient withdrawing from alcohol sees small Chinese soldiers marching on his carpet. This
phenomenon is called

Select one:
Pseudohallucination
Affective illusion

Macropsia
Micropsia
Lilliputian hallucination

Lilliputian hallucinations are visual hallucinations of diminutive persons or animals. Unlike the
usual organic visual hallucinations, these may be accompanied by pleasure and amusement.
The correct answer is: Lilliputian hallucination

410
108
'It is not me who is unhappy, but they are projecting unhappiness into my brain. They project
upon me laughter for no reason.' This is called

Select one:
Made impulse
Made act
Blunted affect
Somatic passivity
Made affect

This is a passivity symptom described along with other FRS.


The correct answer is: Made affect

109
A 33-year-old patient with psychotic depression spilt all the contents of a boiling sauce pan by
inverting it upside down.When asked about this event, she said ' It came to me from the Polish
Embassy. It was nothing to do with me; they wanted it so I picked up the pan and poured it.' She
is describing

Select one:
Made act
Made affect

Delusional perception
Perseveration
Made impulse

She owns up the action, but not the impulse behind it. Hence, this is made impulse, not made
act
The correct answer is: Made impulse

110
In most cases of dysmorphophobia, the affected mental state is associated with

411
Select one:
Anxiety disorder

Major mood disorder

Personality disorder

Psychotic disorder

Eating disorder

Most cases of dysmorphophobia is associated with a major mood disorder. Most patients with
Dysmorphophobia have other associated conditions like mood disorder, anxiety disorder and
psychosis.
The correct answer is: Major mood disorder

111
A 70-year-old patient with a diagnosis of chronic schizophrenia keeps touching her head as if
she is saluting someone, even when no one is around. She denies 'hearing voices or seeing
things'. Which of the following is the most likely cause?

Select one:
Stereotypy
Mannerism
Echopraxia
Chorea
Hallucinatory behaviour

Unusual repeated performances of a goal-directed motor action or the maintenance of an


unusual modification of an adaptive posture are known as 'mannerisms'. Examples of this sign
are unusual hand movements while shaking hands, when greeting others, and during writing.
Other examples may include peculiarities of dress, hairstyle and writing.
The correct answer is: Mannerism

112
A 45-year-old man in a psychiatric inpatient unit is noted to move his right arm inwards while
throwing his left arm outwards repeatedly while walking on the corridor. These are not noted
when he is sitting in the lounge. He is exhibiting
Select one:

412
Parkinsonism
Akathisia
Dystonia

Mannerism
Stereotypy

This is an unusual repeated performance and maintenance of a modification of an adaptive


posture during walking - this is best termed as a mannerism.
The correct answer is: Mannerism

113
A patient who is almost completely bald was observed to imitate combing his hair repeatedly but
without a comb or brush. He is diagnosed to have schizophrenia. Which of the following is the
most likely phenomenon observed?

Select one:
Perseveration
Stereotypy
Mannerism
Passivity
Akathisia

This is an example of mannerism - a repetitive act that is apparently goal-directed but not
necessary or required.
The correct answer is: Mannerism

114
A patient repeatedly refers to computers as 'mathscreen'. His speech exhibits numerous other
disturbances. This can be termed as

Select one:
Stammering
Parapraxis
Stereotypy

413
Stock words
Metonymy

Metonyms are imprecise expressions that are used in place of more exact words that are
available to describe an object or event.
The correct answer is: Metonymy

115
Teichopsic hallucinations are typical of

Select one:
Migraine
Temporal Lobe epilepsy
Lacunar infarcts

Sleep deprivation
Parietal lobe lesions

Teichopsia is also known as scintillating scotoma; it is the sensation of a luminous


appearance before the eyes, with a zigzag, wall-like outline seen in migraine.
The correct answer is: Migraine

116
A psychiatrist attempts to move a patient's arm. Before doing so, he instructs the patient to
resist moving and not to let him manipulate his arm. But the patient continues to move his arm
in the direction of the force. Once the psychiatrist removes the application of force, the patient's
arm comes back to the original position. This phenomenon is called

Select one:
Grimacing
Posturing
Negativism
Mitgehen
Automatic obedience

414
Mitgehen refers to a form of extreme cooperation in which the patient moves their body in the
direction of the slightest pressure on the part of the examiner. For example, the doctor puts his
forefinger under the patient's arm and presses gently, after that the arm moves upwards in the
direction of the pressure. Once the pressure stops, the arm returns to its former position. Light
pressure on the occiput of the patient, who is standing, leads to bending of the neck, flexing of
the trunk and, if the pressure continues, the patient may fall forward.
The correct answer is: Mitgehen

117
Psychopathology of near death experiences include all of the following except

Select one:
Motor perseveration
Autoscopic experience
Depersonalisation
Transcendental experience
Out of body experience

In motor perseveration, the patient may draw a square when asked to do so, but continues to
draw squares even when asked to draw circles (impaired set-shifting). This is not seen in Out-
of-Body or Near Death Experiences.
The correct answer is: Motor perseveration

118
One-way amnesia is associated with

Select one:
Histrionic personality disorder
Schizotypal personality diso

Multiple personality disorder


Borderline personality disorder
Anankastic personality disorder

415
In multiple personality disorder, one-way amnesia is reported. One-way amnesia refers to the
phenomenon where in the personality A knows B's existence, while B is not aware of A's life.
The correct answer is: Multiple personality disorder

119
Refraining from speech and making no attempt at spoken communication despite being fully
conscious is called

Select one:
Mutism
Stuttering
Stammering
Neologisms
Vorbereiden

Refraining from speech and making no attempt at spoken communication despite adequate
level of consciousness is called as mutism. The cause of mutism includes affective disorder,
schizophrenia, personality disorders, organic cerebral disorders, Dissociative disorders,
neurosis and learning disability.
The correct answer is: Mutism

120
A 34-year-old man with schizophrenia resists moving his limbs during a neurological
examination. Despite repeated reassurances from the examining doctor, he resists yielding
his limbs for passive manipulations. He is exhibiting

Select one:
Mannerism
Automatism

Negativism
Obstruction
Borderline traits

Negativism is an apparently motiveless resistance to all interference. It is found in catatonia,


severe learning disability and dementia. Negativism may be passive when all interference is
resisted and orders are not carried out, or it may manifest as active when the patient does the

416
exact opposite of what they are asked to do, in a reflex manner.
The correct answer is: Negativism

121
Which of the following is a thought disorder that predicts relapse in schizophrenia?

Select one:
Circumstantiality
Tangentiality
None of the listed
Vorbeireden
Concrete thinking

FTDs seen in acute state have no prognostic value in predicting relapses


The correct answer is: None of the listed

122
Which of the following are the two stages in a delusional perception?

Select one:
Normal perception and misinterpretation
Delusional mood and delusional significance
Normal perception and delusional significance
Recall of abnormal memory and delusional significance

Abnormal perception and delusional significance

A delusional perception is the attribution of a new meaning, usually in the sense of self-
reference, to a normally perceived object.
The correct answer is: Normal perception and delusional significance

123
A woman wakes up at 6am every morning, does 1 hour of vacuuming, prepares her list for the
day and arrives at work 30 minutes before the start time. At work, she is always ahead of her
schedule and always plans things well in advance. She expects her junior colleagues to be at

417
her standard. What is the correct term to describe this scenario?

Select one:
Compulsion
Obsessive thinking
Obsessional traits

OCD
Normal phenomena

Unless there is some evidence of psychosocial dysfunction, this must be considered as


normal.
The correct answer is: Normal phenomena

124
Obsessional slowness is common in which of the following type of obsessions?

Select one:
Washing obsessions
Obsession of symmetry
Obsessions of contamination
Cleaning obsessions
Religious obsessions

An obsessive need for symmetry and exactness and a compulsion to order and arrange often
results in slowness
The correct answer is: Obsession of symmetry

125
A patient gets recurrent intrusive thoughts about death. This sometimes involves her own
death or death of her loved ones. She does not believe they are true, but she is afraid she
might yield to them. The form of psychopathology exhibited here is

Select one:
Obsessions
Suicidal ideas

418
Loss of control
Delusions
Homicidal ideas

The form of experience is obsession; the content is related to death.


The correct answer is: Obsessions

126
Common characteristics of schizophrenic auditory hallucinations include all except

Select one:
Speaking in one's mother tongue
Being multiple

Having different accent


Often present throughout the day incessantly
Male voice

Signs pointing to a genuine hallucinatory experience include: Increase in background noise


reduces the severity and gives more control to the sufferer; The voices are perceived in the
same space as the other normal perceptions, and perceived simultaneously with other
environmental objects; The voices have different accent but use plain language (non-
technical) and often affect­laden (angry or sad, etc); The voices are never continuous, almost
always episodic; The voices almost never speak in other languages that the patients do not
know.
The correct answer is: Often present throughout the day incessantly

127
A middle-aged woman suffers a panic attack and believes she is going to die. Which term
could be used to describe this psychopathology?

Select one:
Schemas
Overvalued idea
Obsessional rumination
Thought monitoring

419
Outcome fear

Outcome fear is a diagnostic feature of panic disorder. The bias in information processing that
leads to outcome fear is termed as catastrophic misinterpretation.
The correct answer is: Outcome fear

129
Which of the following is not a type of formal thought disorder described by Carl Schneider?

Select one:
Derailment
Fusion
Substitution
Omission

Overinclusion

Some of the terms described by Carl Schneider include fusion, omission, substitution,
derailment and drivelling.
The correct answer is: Overinclusion

130
A dominating but solitary belief that is neither delusional nor obsessional but influences one's
life to a notable extent is called

Select one:
Compulsion

Stereotype
Rumination
Cognitive Distortion
Overvalued idea

420
Overvalued idea is a non-delusional, non-obsessional abnormal belief. The subject holds a
belief which is in itself acceptable and comprehensible but comes to dominate his/her thinking
and behaviour. The idea will have great significance and value to the subject.
The correct answer is: Overvalued idea

131
Dysmorphophobia is typically a/an

Select one:
Delusional idea
Cognitive distortion
Cultural idea
Obsessional idea
Overvalued idea

Dysmorphophobia: It is a type of overvalued idea where the patient believes one aspect of his
body is abnormal or conspicuously deformed. It is the expressed opinion of others that result in
the belief. The degree of conviction is the main discriminating factor between delusion and
overvalued idea. Though dysmorphophobia is usually considered as an overvalued idea, it can
reach delusional intensity. Dysmorphophobia can remit following surgery in some patients.

The correct answer is: Overvalued idea

132
A patient repeats the last word that she uttered without any apparent purpose. This is called

Select one:
Echolalia
Logoclonia

Perseveration
Palilalia
Stammering

Palilalia is a compulsive involuntary repetition of a semantically acceptable phrase or word


spoken by the patient herself.
The correct answer is: Palilalia

421
133
A hospital manager is presenting data to a trust board. He says 'murder of 2 hospitals' instead
of 'merger of 2 hospitals'. This is an example of

Select one:
stock word
logoclonia
paragrammatism
neologism
parapraxis

A Freudian slip, also called parapraxis, is often an error in speech that is interpreted as
occurring due to the interference of some unconscious wish, conflict, or train of thought. The
concept is thus a part of the psychodynamic theory.
The correct answer is: parapraxis

134
Illusions that are created out of vague sensory percepts by an admixture of imagination and
emotional states are called

Select one:
Pseudohallucinations
Imagery

Pareidolia
Fantasy
True hallucinations

Pareidolic Illusions are meaningful percepts produced when experiencing a poorly defined
stimulus. It occurs more commonly in children than in adults. These illusions are created out of
sensory percepts by an admixture with imagination, and typically images are seen from shapes.
On closer attention, the intensity of pareidolic illusions becomes more intricate and detailed.

The correct answer is: Pareidolia

422
135
Which of the following is false with regard to pseudohallucinations?

Select one:
Patient may have insight about the absence of reality
May occur in a subjective space e.g. within the head
They may accompany true hallucinations
Pathognomonic of borderline personality disorder
Patient does not usually search for the bearer of the voice

Jaspers believed that pseudo-hallucinations are variants of fantasy/mental imagery and, thus
not carrying the same diagnostic implications as true hallucinations. Hare suggested it was
more meaningful to distinguish them in terms of the degree of insight.
The correct answer is: Pathognomonic of borderline personality disorder

136
Which of the following is most likely to be seen in an 18-year-old girl with hysterical aphonia?

Select one:
Vocal cord movements are restricted
Patient loses ability to speak aloud but whispers
Speech is absent
There is difficulty in articulation
Patient loses ability to cough

Dysarthria: There is difficulty in articulation, which occurs in lesions of the brain stem,
schizophrenia and cerebellar disorder.
Dysphonia: Impairment of ability to vocalize with some hoarseness of voice, which occurs in
the paralysis of the 9th cranial nerve and disease of vocal cords.
Aphonia: Patient loses ability to vocalise and whispers, occurs in conversion disorders. Aphonia
is defined as the absence of vocalization or phonation and differs from mutism, which is the
absence of speech. Hysterical aphonia is characterized by a normal whisper and cough.
Examination with a laryngoscope reveals normal vocal cord movement with respiration.

The correct answer is: Patient loses ability to speak aloud but whispers

423
137
In which of the following conditions the stimulus is perceived as corresponding object but the
quality of the percept is altered?

Select one:
Negative hallucinations
Perceptual distortions
Illusions

Hallucinations
Imagery

Distortions are changes in perception resulting from a change in the intensity and quality of
the stimulus or the spatial form of the perception.
The correct answer is: Perceptual distortions

138
You are interviewing a newly admitted patient in your ward. When asked what he had for
breakfast, he says 'cereals'. Then when you asked 'what is your name?' he says 'cereals'.
When you asked 'what is his date of birth?' he continues to say 'cereals'. The most likely
symptom exhibited here is

Select one:
Stuttering
Stereotypy
Automatism
Perseveration
Mannerism

Perseveration occurs when mental operations persist beyond the point at which they are
relevant and thus prevent progress of thinking. Perseveration may be mainly verbal or
ideational.Verbal perseveration can be a perseveration of theme rather than the actual words
and this can be regarded as impairment of switching. In other cases the set or attitude is
perseverated, and the patient cannot solve a new problem because they cannot break free from
their previous set. Verbal perseveration can occur in schizophrenia and organic states.
The correct answer is: Perseveration

424
139
Which one of the following is a type of abnormally induced movement?

Select one:
Tics
Chorea
Stereotypy

Perseveration
Tremors

Signs characterized by abnormal induced movement include perseveration, automatic


obedience and other catatonic signs.
The correct answer is: Perseveration

140
A 32-year-old shift worker presents with a history of headaches occurring in the mornings for a
few months. His GP has been treating him for a diagnosis of tension headache. Which of the
following features suggest a different pathology?

Select one:
Pain relieved by analgesics
The localisation of pain is vague.
Associated with low mood and anxiety
Postural worsening of pain
Pain worse with stress

Postural change may indicate intracranial pathology.


The correct answer is: Postural worsening of pain

141
A 78-year-old man hospitalized after a fall, repeatedly sees small angels flying around his
head. Which of the following is least likely to be true in this case?

Select one:
This phenomenon can occur due to anticholinergic toxicity

425
This phenomenon is reported in delirium tremens
The perception has occurred without appropriate stimulus
This phenomenon is associated with organic disorders of the brain

Presence of intact consciousness suggests a functional disorder than an


organic mental illness

The correct answer is: Presence of intact consciousness suggests a functional disorder than
an organic mental illness

142
Concrete thinking is usually tested by means of

Select one:
Verbal fluency test

Proverb testing
Cognitive estimates test
Luria test
Sorting test

Concretism is considered an important aspect of schizophrenic thought disorder. Traditionally it


is measured using the method of proverb interpretation, in which metaphoric proverbs are
presented with the request that the subject tell its meaning. Interpretations are recorded and
scored on concretistic tendencies.Its reliability is doubtful, and it is rather complicated to
perform.
The correct answer is: Proverb testing

143
A woman reports that she can see her recently deceased husband though she does not
believe he could be alive. This is best described as a

Select one:
Perceptual distortion

426
Visual hallucination
Affect illusion
Pseudohallucination

Pareidolia

So called hallucinations of widowhood are indeed pseudohallucinations in


phenomenological terms.
The correct answer is: Pseudohallucination

144
Which of the following is not a type of perceptual distortion?

Select one:
Hyperacusis

Hyperaesthesia
Macropsia
Dysmegalopsia
Pseudohallucinations

Micropsia, Macropsia, dysmegalopsia, altered sensory perceptions such as


hypoacusis/hyperacusis are classified as sensory distortions.
The correct answer is: Pseudohallucinations

145
Alexithymia is common in those with

Select one:
Psychosomatic disorders
Dementia
Psychotic disorder

Mood disorder
Personality disorder

427
It occurs in psychosomatic disorders, substance abuse, masked depression, PTSD and in
some cases of sexual deviance
The correct answer is: Psychosomatic disorders

146
Overvalued ideas are NOT noted in the core symptoms of which one of the following
disorder?

Select one:
Transsexualism
Anorexia nervosa
Body dysmorphic disorder
PTSD
Morbid jealousy

PTSD is an anxiety disorder; overvalued ideas are not described in the diagnostic criteria of
PTSD
The correct answer is: PTSD

147
In pure word blindness which of the following is true?

Select one:
Speech fluency is diminished
Speech comprehension is lost

Cannot write spontaneously


Reading comprehension is impaired
Writing comprehension is impaired

Also called subcortical visual aphasia, pure word blindness is associated with reduced reading
comprehension but the patient can normally speak and understand the spoken word; he/she
can write spontaneously and to dictation.

428
The correct answer is: Reading comprehension is impaired

148
Stereotypy is characterised by

Select one:
Mimicking examiner's movements and speech

Maintenance of unchanged facial expression


Repetitive, non-goal-directed motor activity

Spontaneous maintenance of limb posture Odd


purposeful movements

Odd purposeful (or quasi-purposive) movements are called mannerisms. Repetitive, non-goal
directed motor activity is called stereotypy. Maintaining unchanged queer facial expression is
described as grimacing.
The correct answer is: Repetitive, non-goal-directed motor activity

149
A 35 yr old woman is on sodium valproate for many years. She is having lack of sleep
following a holiday and tells her husband 'Life is not a game, every idiot has to earn a name,
so work hard for fame, if not you will end up feeling shame'. She is exhibiting

Select one:
Rhyming
Metonymy

Punning
Pressured speech
Neologisms

An example of flight of ideas associated with rhymes comes from a manic patient described by
Fish. She was asked where she lived and she replied: 'Birmingham, Kingstanding; see the king
he's standing, king, king, sing, sing, bird on the wing, wing, wing on the bird, bird, turd, turd.'

The correct answer is: Rhyming

429
150
The term anhedonia was coined by

Select one:
Bleuler
Ribot
Kraeplin

Sifneos
Kretschmer

An inability to enjoy anything in life or even get pleasure from everyday occurrences is called
as anhedonia (for example being hugged by one's children).The term was coined in 1896 by
Ribot, a French psychologist.
The correct answer is: Ribot

151
A voice in a flat monotone describes everything the patient is doing: 'She is peeling potatoes,
she has now got hold of the peeler, she does not want that potato...' This is best described as

Select one:
Running commentary
Second person hallucinations
Imperative hallucinations
First person hallucinations
Autochthonous perceptions

Running commentary hallucinations are a part of Schneider's first rank symptoms of


schizophrenia. Mellor, C. S. First rank symptoms of schizophrenia. Br. J. Psychiatry 117, 15-23
(1970).
The correct answer is: Running commentary

152
The commonest psychiatric diagnosis reported in patients with Capgras syndrome is

Select one:

430
Psychotic depression
Schizophrenia
Alzheimer's dementia

Depression
Bipolar disorder

Here the patient believes that a familiar person has been replaced by an impostor whose
external appearances are identical to the real person. It was first described by Rebout-
Lachaux. Most patients with Capgras syndrome are females and the commonest reported
psychiatric diagnosis is schizophrenia. Psychological Medicine (1991), 21:905-910
The correct answer is: Schizophrenia

153
Crowding of thoughts occurs mainly in

Select one:
Schizophrenia
Dementia
Mania
Learning disability
Depression

Crowding of thought occurs in schizophrenia. Here the patient describes his thoughts as being
passively concentrated and compressed in his head. The associations are experienced as being
excessive in amount, too fast, inexplicable and outside the person's control. Experientially, this
is different from a manic flight of ideas.
The correct answer is: Schizophrenia

154
Schnauzkrampf is associated with

Select one:
Dementia
Schizophrenia
Bipolar disorder

431
Depression
None of the above

Schnauzkrampf: A characteristic facial expression in which the nose and lips are drawn
together.
The correct answer is: Schizophrenia

155
A chronic schizophrenia patient snouts his lips and maintains this posture for a long time even
when he is active doing other work. Which of the following best describes the above sign?

Select one:
Catalepsy
Snout spasm
Cataplexy
Posturing
Mannerism

In catatonia the lips may be thrust forward in a tubular manner known as 'snout spasm' (or
schnauzkrampf) and although this is obviously a disorder of expression it is best regarded as a
stereotyped posture.
The correct answer is: Snout spasm

156
Which of the following is true regarding pseudohallucinations?

Select one:
Cannot be seen in normal people
Seen in Ganser's syndrome

Insight is not retained


Associated with amnesia for the object
Occurs in external outer space

432
Ganser's syndrome is associated with pseudohallucinations, as described originally by
Ganser.
The correct answer is: Seen in Ganser's syndrome

157
A special ability of memory in which visual images are drawn from memory accurately at will
and described as if being perceived currently is noted in some people. Which of the following is
true about this?

Select one:
It is common in depression
Common in the elderly
It is a type of hallucination
Seen in some religious practices
Always part of pathological experience

Eidetic images represent visual memories of almost hallucinatory vividness that are found in
disorders due to substance misuse, especially hallucinogenic agents such as LSD and
mescaline. It is seen more commonly in children and those involved in group religious
practices.
The correct answer is: Seen in some religious practices

158
A partially sighted, 77-year-old lady reports seeing Peter Rabbit images in front of her eye.
She has no psychotic features. Which of the following is true about the above symptom?

Select one:
Her consciousness may be altered
It cannot occur unless she is completely blind in at least one eye
The Rabbit will not disappear on sleeping

She may retain insight


The objects seen are usually static

Charles-Bonnet syndrome comprises of visual hallucinations without any other psychotic


features or any evidence of psychiatric disorder. It is associated with visual impairment. The
content of the hallucinations varies from straight lines to complex pictures of people and

433
buildings. They may be enjoyable with full insight or distressing.
The correct answer is: She may retain insight

159
A patient complaints of odd sensations such as 'electric heating' emanating from her knees.
This can be described as

Select one:
First rank symptom
Somatic delusion
Overvalued idea
Somatic passivity
Somatic hallucination

Fish describes a patient who felt a frictional rub along her ribs that had no physical
explanation and was unreal. This is also a somatic hallucination.
The correct answer is: Somatic hallucination

160
A patient says 'X-rays sent by my neighbour enter the back of my neck, where the skin tingles
and feels warm, they pass down the back in a hot wavy current about six inches to the sides of
my hip bone.' This is best described as

Select one:
Made volition
Made impulse

Synaesthesia
Somatic passivity
Kinaesthetic hallucination

As the sensation is ascribed to the neighbour, an external agent, this is best described as a
somatic passivity rather than a somatic hallucination.
The correct answer is: Somatic passivity

434
161
Overinclusive thinking is tested by using a

Select one:
Verbal fluency test
Proverb testing
Similarities test

Sorting test
Cognitive estimates test

Overinclusion was first described by Cameron in 1944. Goldstein's card sorting test can be used
to test overinclusion. According to Goldstein sorting tasks mainly are designed to assess
conceptual thinking, and he emphasized that the test primarily evaluates the categorical or
abstract attitude.
The correct answer is: Sorting test

162
A patient comes to the clinic and in a formal manner addresses you and the nurse as 'Hello
sir, how are you? How is madam?' This is called

Select one:
Stock word
Neologism
Scanning speech
Stereotypy
Stilted speech

Stilted speech - Speech excessively stilted and formal. e.g. 'The attorney comported himself
indecorously'.
The correct answer is: Stilted speech

163
While assessing the psychopathology of a psychotic patient, you notice that a major thought
gets replaced by a minor thought in his thinking process. This is called

Select one:

435
substitution

derailment

drivelling

fusion

omission

In derailment the thought slides on to a subsidiary thought while in substitution a major


thought is substituted by a subsidiary one. Omission consists of the senseless omission of a
thought or part of it. In fusion, heterogeneous elements of thought are interwoven with each
other while in drivelling there is disordered intermixture of constituent parts of one complex
thought.
The correct answer is: substitution

164
A 9-year-old girl can see the colour white associated with the number 7 and see the colour
purple whenever she comes across the number 9. This is called

Select one:
Functional hallucination
Reflex hallucination
Reverse hallucination
Synaesthesia
Extracampine hallucination

Synaesthesia is the experience of a stimulus in one sense modality producing a sensory


experience in another. Colour-Number synaesthesia (reading or hearing numbers is
associated with seeing colours) is the most common type.
The correct answer is: Synaesthesia

165
Which of the following is a formal thought disturbance?

Select one:
Thought insertion
Obsessions

436
Tangentiality
Stuttering
Delusions of persecution

With tangentiality, the patient can never get to the point; it is a formal thought disorder seen in
schizophrenic speech samples.
The correct answer is: Tangentiality

166
Emotional lability is least likely to be due to

Select one:
Frontal lobe damage
Pseudo bulbar palsy

Bipolar disorder
Temporal lobe atrophy
Delirium

Emotional lability is characterized by rapid exaggerated affective changes, which occurs


commonly in elderly and frontal lobe injury.
The correct answer is: Temporal lobe atrophy

167
Auditory hallucinations with clear consciousness are least likely to be due to

Select one:
Temporal lobe epilepsy
Alzheimer's disease.
Cocaine abuse
Alcohol abuse
Schizophrenia

437
438
Alcoholic hallucinosis, any functional psychiatric disorder, and early dementia can cause
auditory hallucinations with clear consciousness.
The correct answer is: Temporal lobe epilepsy

168
Automatism is a recognised phenomenon seen most commonly in patients with

Select one:
Borderline personality disorder
Hysterical dissociation
Bipolar disorder
Schizophrenia
Temporal lobe epilepsy

Automatism: Clouding of consciousness occurring around an epileptic seizure accompanied by


complex actions without the subject's awareness. It lasts less than 5 minutes but rarely even
up to an hour. Violence is rare.
The correct answer is: Temporal lobe epilepsy

169
Which of the following is true concerning the psychopathology of morbid jealousy?

Select one:
It is not seen as an isolated phenomenon
The belief can be held with delusional fixity

It is more common in women than men


It is an obsessional state
It intensifies on geographical separation of partners

Morbid jealousy can occur as an isolated state but can also occur in association with alcohol or
cocaine misuse, impotence, schizophrenia, mania, depression, organic conditions like
dementia, and terminal stages of alcoholism and punch-drunk syndrome. Male to female ratio is
3:1.
The correct answer is: The belief can be held with delusional fixity

439
170
Which of the following statements about alcoholic hallucinosis is incorrect?

Select one:
Patients with alcoholic hallucinosis have a clear sensorium.
These usually appear in persons abusing alcohol for a long time.
The hallucinations respond poorly to antipsychotics.

The hallucinations usually last less than a week, when patients believe in
the hallucinations though afterwards they may realise the untrue nature.
The most common hallucinations are unstructured sounds or voices that
may be characteristically malign and threatening.

Alcoholic hallucinosis: The most common hallucinations are unstructured sounds or voices that
may be characteristically malign and threatening. The hallucinations usually last only for few
days, during which the patients lack insight and believe in an external source for hallucinations
though afterwards they often realise their untrue nature. These usually appear in persons
abusing alcohol for a long time. Delusions are usually secondary interpretations of the
hallucinations. The patient is usually distressed, anxious, and restless. Unlike those
experiencing delirium tremens, those with alcoholic hallucinosis have a clear sensorium. As an
empirical timescale, hallucinations lasting for more than six months should initiate suspicion of
other psychoses including schizophrenia. The hallucinations respond well and rapidly to
antipsychotics. The prognosis is good provided that the person remains abstinent. But the
evidence base for treating alcoholic hallucinosis is very poor.
The correct answer is: The hallucinations respond poorly to antipsychotics.

171
Regarding pure word deafness, which one of the following statements is false?

Select one:
Here a patient can speak fluently.
The patient cannot understand speech, even though hearing is unimpaired for
other sounds
It is also called as sub cortical auditory dysphasia.
The patient can recognize the meaning of words.
The patient can read and write correctly.

440
Also called as subcortical auditory dysphasia, pure word deafness is characterized by the
patient being able to speak, read and write fluently. But he cannot understand speech, even
though hearing is unimpaired for other sounds; he hears words as sounds, but cannot
recognize the meaning even though he knows that they are words.
The correct answer is: The patient can recognize the meaning of words.

172
According to Jaspers which of the following is true about primary delusions?

Select one:
They occur before other symptoms
They are un-understandable in terms of other mental experiences
They are specific for schizophrenia
They have prognostic significance in schizophrenia

They are resistant to treatment

Jaspers emphasized the importance of un-understandability. Primary delusional experiences


tend to be reported in acute schizophrenia but are less common in chronic schizophrenia,
where they may be buried under a mass of secondary delusions arising from primary delusional
experiences, hallucinations, formal thought disorder and mood disorders.
The correct answer is: They are un-understandable in terms of other mental experiences

173
Which one of the following statements about first rank symptoms of schizophrenia is NOT
true?

Select one:
They carry prognostic significance
They are also seen in Organic Psychosis

These are a group of symptoms which are useful in the diagnosis of


schizophrenia
They are also seen in affective psychoses
They are neither pathognomonic nor specific to schizophrenia

441
Presence or absence of FRS in schizophrenia does not add any prognostic information to a
presentation. We cannot say that someone has a more severe form of schizophrenia if they
have a high number of FRS.
The correct answer is: They carry prognostic significance

174
Which of the following is true about first rank symptoms of schizophrenia?

Select one:
They are essential for diagnosing schizophrenia
They represent disturbance of ego-boundary
They are not seen in other psychiatric disorders
They are specific for schizophrenia
They emphasize on content rather than form

The original psychoanalytic interpretation of FRS is that the boundary between the ego and
the surrounding world breaks down, leading to thought and volition being alienated.
The correct answer is: They represent disturbance of ego-boundary

175
Which one among the following is not a first rank symptom of schizophrenia?

Select one:
Thought insertion
Thought withdrawal

Thought broadcasting
Delusional perception
Thought Block

The correct answer is: Thought Block

176

442
'My thoughts leave my head in a type of mental ticker-tape. Everyone around me has only to
pass the tape through their mind, and they know my thoughts.' This patient is describing

Select one:
Thought broadcast
Thought withdrawal
Thought echo

Thought blocking
Thought insertion

This is the classical feature of thought broadcast: everyone comes to know what we think, as
and when we think. [Excerpt from TABLE 1. Examples of positive symptoms of schizophrenia.
(n.d.).
The correct answer is: Thought broadcast

177
A 35-year-old lady in your inpatient unit covers her head with a veil. She claims that other
people can come to know what she thinks. The most likely description is

Select one:
Thought insertion
Thought broadcast
Thought withdrawal
Thought blocking
Obsessions

The correct answer is: Thought broadcast

178
'If I think of anything, at once those opposite to me know it and it is embarrassing'. This is best
termed as

443
Select one:
Referential delusion
Delusional misinterpretation
Thought diffusion

Thought Block
Thought echo

Thought diffusion is the same as thought broadcast, which is described here. This description
was given by a shopkeeper with schizophrenia and was reported by Schneider himself.
The correct answer is: Thought diffusion

179
A man with psychotic symptoms said: 'When I try to think, my head gets full of noise; it is like as
if my brain were in an uproar with my thoughts'. The most appropriate psychopathology is

Select one:
Thought diffusion
Thought broadcast
Thought provocation
Thought echo
Crowding of thoughts

Schneider described a man with schizophrenia who had the experience of his thoughts being
spoken aloud.
The correct answer is: Thought echo

180
"The thoughts of Elvis come into my mind. He treats my mind like a screen and flashes his
thoughts on to it like you flash a picture." This patient is experiencing

Select one:
Thought blocking
Thought insertion
Thought broadcast

444
Obsessions
Thought withdrawal

The correct answer is: Thought insertion

181
A patient experiences some disturbing thoughts. He feels these thoughts originate elsewhere
and reach him fully formed. He does not feel he owns these thoughts. Which of the following
terms best describes the above?

Select one:
Thought withdrawal
Thought insertion
Obsessions
Thought broadcasting
Thought blocking

The correct answer is: Thought insertion

182
A 48-year-old man who exhibits decreased speech stated that his thoughts were 'taken from
me years ago by a parish council'. The most appropriate psychopathology described here is

Select one:
Crowding of thoughts
Retrospective falsification

Thought diffusion
Delusional memory
Thought withdrawal

445
Schneider gave the description of a man who was experiencing thought withdrawal in the
above format.
The correct answer is: Thought withdrawal

183
"I am thinking about my mother, and suddenly my thoughts are sucked out of my mind by a
phrenological vacuum extractor, and there is nothing in my mind." This is best described as

Select one:
Thought blocking
Thought withdrawal
Thought insertion
Thought broadcast
Thought echo

The correct answer is: Thought withdrawal

184
The ratio between number of different words used during a discourse and total number of
spoken words is reduced in a patient with schizophrenia. What is the above called?

Select one:
Word count
Cloze ratio

Cohesion ratio
Type token ratio
None of the above

446
The type-token ratio (TTR) is a measure of vocabulary variation within a written text or a
person's speech. It is reduced in schizophrenic speech.
The correct answer is: Type token ratio

185
Dereistic thinking is a term used to describe which one of the following?

Select one:
Lateral thinking
Abstract thinking
Logical thinking
Unrealistic thinking
Goal directed thinking

Undirected fantasy thinking in the past, has also been termed autistic or dereistic thinking.
The correct answer is: Unrealistic thinking

186
Coenaesthesia is a type of

Select one:
Visual hallucination
Olfactory hallucination

Eidetic imagery
Visceral hallucination
Delusional belief

Coenaesthetia refers to unfounded bodily sensations related to visceral, somatic hallucinations


seen in schizophrenia. Examples of coenaesthetic hallucinations are a burning sensation in the
brain, a pushing sensation in the blood vessels, a cutting sensation in the bone, etc.

The correct answer is: Visceral hallucination

187

447
Which one of the following is not a core feature of Ganser's syndrome?

Select one:
Visual hallucinations
Somatic conversion features
Clouding of consciousness

Approximate answers or VORBEIGEHEN (to pass by)


Pseudohallucinations

Ganser syndrome refers to the production of approximate answers. e.g. Question: "What is the
capital of France?" - Answer: "London". It is occasionally associated with organic brain illness
but is much more commonly seen as a form of malingering in those attempting to feign mental
illness e.g. in prisoners awaiting trial.
The correct answer is: Visual hallucinations

188
Eidetic imagery is best classified as a

Select one:
Vivid visual memory
Pseudohallucinations
Affect illusion
Hallucination
Complete illusion

Eidetic imagery is a particular type of exceptionally vivid visual memory, more common in
children than adults.
The correct answer is: Vivid visual memory

189
Which of the following is NOT a first rank symptom of schizophrenia?

Select one:
Voices giving repeated feedback as and when the patient does an act
Voices commanding the patient

448
Voices discussing among themselves
Voices echoing patient's own thoughts
Voices giving commentary on the patient

Voices sometimes give instructions to a patient, who may or may not act upon them; these are
termed 'imperative' or 'command hallucinations' and are not a FRS.
The correct answer is: Voices commanding the patient

190
Which one of the following is NOT an elementary hallucination?

Select one:
Seeing flashes of light
Hearing multiple buzzing sounds

Unstructured flakes of images in a delirious patient


Voices repeating the word 'go'
Single tone of unclear sound

Auditory hallucination may be elementary and unformed, and experienced as simple noises,
bells, etc. Elementary auditory hallucinations can occur in organic states and prodromal
psychosis.
The correct answer is: Voices repeating the word 'go'

191
Which one of the following is a type of catatonic phenomenon?

Select one:
Aphonia
Waxy flexibility
Dyskinesia
Oculogyric crisis
Akathisia

449
The correct answer is: Waxy flexibility

450
1
A 45-year-old man who has longstanding suspiciousness towards police becomes anxious and
acts in a strange manner on seeing 2 policemen at a public event. The police become
suspicious of him and arrest him as a precautionary measure. The underlying psychodynamic
mechanism is

Select one:
Repression
Acting out
Projective identification

Suppression
Projection

Projective identification is used to project the bad object into (not onto) another person, so it
becomes a part of that person. The person then identifies with that other person and hence
has means to control them. The person projected into may consequently be pressured to
behave congruently with the projective phantasy, believing and accepting their role.
The correct answer is: Projective identification

2
Following poor feedback from his boss at work, Mark spends all his evening playing tennis
with his friend. This is best described as

Select one:
Displacement

451
Altruism
Turing into self
Aggression

Sublimation

Sublimation refers to a mature type of defence mechanism where socially unacceptable


impulses are transformed into acceptable forms.
The correct answer is: Sublimation

3
In a psychoanalytic group setting, a male group member is noted to make advances towards
women both inside and outside the group whenever his self-esteem was low, but did not show
sustained interest when he started feeling better within himself. This led to many broken
relationships. Which of the following defence mechanism is being used?

Select one:
Reaction formation

Isolation
Sublimation
Displacement
Acting out

Acting out refers to a defence mechanism whereby the patient involves in behaviors that
relieve tension on a temporary basis through gratification of needs instead of verbalization of
emotions.
The correct answer is: Acting out

4
Which of the following is a Kleinian defence?

Select one:
Splitting
All of the listed
Projection
Grandiosity

452
Omnipotence

SIPDOG is a mnemonic for Klein's defences. Splitting, Introjection, Projective Identification,


Denial, Omnipotence, Grandiosity.
The correct answer is: All of the listed

5
Klein's depressive position is related to the process of learning to cope with which of the
following conflicts?

Select one:
Difficult relationships

Ambivalence
Sexual needs
Depression
Hunger

Melanie Klein described two positions - paranoid-schizoid and depressive position. The
paranoid-schizoid position is associated with the use of splitting and projection as a defence
mechanism. This position concerns an inability to perceive a whole object and splits all objects
into their good and bad parts. But in the depressive position, the infant tolerates the ambiguity
or ambivalence and can realise that an individual can have both good and bad qualities. In the
paranoid-schizoid position, the anxiety is about the survival of the self but in depressive
position the anxiety is about the loss of the object.
The correct answer is: Ambivalence

6
Which one of the following refers to the contents of the collective unconscious that describes a
pancultural representation of human experience expressed through images and symbols?

Select one:
Animus
Shadow
Archetypes

Anima
Persona

453
Examples of pancultural representation of human experience (i.e. the Jungian archetypes)
include the mother, the child, the hero, the self, the shadow, anima and animus. Anima refers to
the unconscious feminine aspects of the male. Animus refers to the unconscious masculine
aspects of the female. Shadow refers to the unacknowledged aspects of oneself that includes
both creative impulses and destructive urges. 'Self' refers to the main archetype that gives the
personality a sense of oneness. Persona refers to the outer concealed aspects of oneself.
The correct answer is: Archetypes

7
Who described the unconscious as composed of both the collective unconscious and
personal unconscious?

Select one:
Winnicott
Carl Gustav Jung
Sigmund Freud
Alfred Adler
Anna Freud

Carl Gustav Jung described a holistic theory of psyche that can be applied across varied
cultures. According to him, the unconscious mind includes the collective and personal
unconscious. He postulated that the personal unconscious is unique to each individual, but
the collective unconscious is universal, common to mankind, contains the archetypes that
include anima, animus, shadow or self and Hero.
The correct answer is: Carl Gustav Jung

8
In psychoanalytic terms, condensation refers to

Select one:
Representing abstract concepts by using recognisable images
Combining various unconscious impulses into a single image while
dreaming
Avoidance of unconscious impulses
Enacting dreams as they occur
Transfer of energy from an original object to a symbolic representation of

454
the object

Condensation is a term used in dream analysis. It refers to the process of several unconscious
impulses being combined into a single image in the manifest dream content e.g., one's father
and a horrible teacher may be unified and occur as a single dreadful monster in a child's dream.

The correct answer is: Combining various unconscious impulses into a single image while
dreaming

9
Which of the following defence mechanism is classified as a manic defence?

Select one:
Repression
Sublimation

Reparation
Humor
Denial

Denial, grandiosity, omnipotence are some of the manic defenses described by Klein. Humor
and sublimation are mature defenses.
The correct answer is: Denial

1
A GP, who has an argument with her teenage daughter the previous night, insists with
excessive force to her next patient to stop smoking. This is called

Select one:
Projection
Reaction formation
Denial
Displacement
Acting out

455
Displacement refers to showing emotions at a target different than the one unconsciously
intended.
The correct answer is: Displacement

11
A young man brings his 19-year-old girl friend who vacantly stares at the wall, not responding to
the conversation. He reports that she started behaving in this manner since they had a serious
argument. The most likely defence mechanism operating here is

Select one:
Acting out
Repression
Projection
Dissociation
Displacement

In general, dissociation includes day dreaming, 'zoning out,' or doing things on 'autopilot.' For
example, staring out during lectures and thinking what to do in the evening, driving a car and
not able to describe what you saw on the way are some daily examples of dissociation.
The correct answer is: Dissociation

12
The predominant defence noted in those with amnesic fugue reactions is

Select one:
Dissociation
Splitting

Somatisation
Introjection
Acting out

Some dissociation is helpful in keeping one portion of one's life from interfering with another
(e.g., not bringing problems home from the office). However, dissociation is responsible for
certain somatoform and dissociative disorders including amnesia and fugue.
The correct answer is: Dissociation

456
13
Defence mechanisms are a function of the

Select one:
Ego
Id
Both Id and Ego

Superego
Both Ego and superego

The first systematic account of defence mechanisms was written by Freud's daughter Anna
Freud in the book 'Ego and the Mechanisms of Defence' in 1936.
The correct answer is: Ego

14
Which of the following was employed by Freud as a psychoanalytic technique to uncover the
unconscious?

Select one:
Free association
Empathy
Narco-analysis
Cognitive restructuring
Introspection

Using the technique of free association, Freud asked patients to relate anything that came into
their mind, regardless of how apparently unimportant or potentially embarrassing the memory
threatened to be. This technique assumed that all memories are arranged in a single associative
network and that sooner or later the subject would stumble across the crucial memory.
The correct answer is: Free association

15
According to Freud which of the following is true?

457
Select one:
Transference is not seen in the absence of repressed wishes
Id obeys the reality principle
Phallic phase is the last phase in psychosexual development

Id functions through primary process thinking


Conscious, subconscious and preconscious form structural model of mind

Id functions by primary process thinking which denies the existence of rational boundaries of
thinking, sense of time, space and probability. It does not obey reality principle but instead
serves pleasure principle. Transference occurs irrespective of presence or absence of
repressed wishes. Id, ego and superego form the structural components while conscious is a
part of topographic model. Genital not the phallic stage is the last stage of psychosexual
development.
The correct answer is: Id functions through primary process thinking

16
Which of the following defence mechanism is characterised by changing the shape of one's
self-representation to become more like an ideal and admired person?

Select one:
Introjection
Identification
Modelling
Projection
Displacement

In identification, a person changes himself to be like someone else who is admired. It is not
same as modelling which is a social learning concept. In modelling only behaviour, not self-
representation is modelled.
The correct answer is: Identification

17
Which one of the following concepts is not correctly linked to its proponent?

Select one:
Splitting -Melanie Klein

458
Malignant Alienation - Morgan and Watts
Introversion and Extroversion - Winnicott
Sibling Rivalry -Alfred Adler

Thanatos - Sigmund Freud.

Carl Gustav Jung described the concept of Introversion and extroversion. Jung was the
founder of the school of analytical psychology.
The correct answer is: Introversion and Extroversion - Winnicott

18
Which one of the following is true about the Ego?

Select one:
Unconsciously motivated

It is instinctive
Governed by the pleasure principle
It is governed by the reality principle
It is the most primitive part of the personality

The Id is the most primitive part of the personality and it is instinctive, unconsciously motivated
and is governed by the pleasure principle. The Ego is consciously driven, governed by the
reality principle and contains reality-oriented parts of the self. The ego is largely conscious. The
superego is formed by introjects of parental figures. The superego has both conscious and
unconscious aspects, and can be equated to the conscience of the individual.
The correct answer is: It is governed by the reality principle

19
Which one of the following is not considered to be a Neo-Freudian?

Select one:
Karen Horney
Harry Stack Sullivan.
Jean Piaget
Erich Fromm

459
Alfred Adler

Neo-Freudian psychologists were thinkers who agreed with the basis of Freud's
psychoanalytic theory but changed and adapted the theory to incorporate their own beliefs,
ideas and theories. Piaget is not usually considered as a Freudian theorist.
The correct answer is: Jean Piaget

2
Who coined the term 'collective unconscious'?

Select one:
Freud
Mahler
Adler
Jung

Lacan

Collective unconscious refers to mental symbols and other mental content outside of our
awareness but are perceived cross-culturally. An example of this is the 'Wise Old Man'
concept.
The correct answer is: Jung

21
Superego is formed at which stage of psychosexual development?

Select one:
Phallic stage

Oral stage
Genital stage
Anal stage
Latency stage

460
The latency period is the stage of suspension of psycho-sexual development between the age of
five and six and puberty. During this period, sexual activity and interest tend to decrease, a
consequence of repression, secondary identifications and the establishing of the superego,
resulting in the resolution or the waning of the Oedipus complex. At about five years of age
during latency stage, the Superego appears, following the end of the oedipal stage.
The correct answer is: Latency stage

22
The major source of values in one's super-ego, according to psychoanalytic theory, is

Select one:
Parental ideals
Social models such as celebrities
Innate values in newborn
Knowledge from school education
Peer group

The superego is one of the three agencies making up the psychic apparatus in Freud's second
topography, the structural theory (1923). It results essentially from the internalization of
parental authority.
The correct answer is: Parental ideals

23
Carl Gustav Jung is associated with all of the following terms except

Select one:
Free association
Primal scream

Neurasthenia
Dementia praecox
Moral insanity

Carl Gustav Jung is associated with the following terms- Personal unconscious, collective
unconscious, introvert, extrovert, archetypes, persona, anima and animus.
The correct answer is: Primal scream

461
24
Oedipus complex and castration anxiety are associated with which stages of psychosexual
development?

Select one:
Anal stage
Genital stage
Latency stage
Oral stage
Phallic stage

Phallic/oedipal stage is seen between 3 and five years of age during which the child
increasingly focuses on genitalia and libido is directed towards others. His stage involves
castration anxiety and Oedipus complex in males and penis envy and Electra complex in
females. The Oedipal complex refers to the intense attachment to the parents of the opposite
sex along with a wish to destroy the parents of the same sex. Resolution of the Oedipal
complex involves identification with the same sex parent (i.e. the son identifying with the
father).
The correct answer is: Phallic stage

25
The 'repressive barrier' to censor unacceptable wishes and desires is maintained by

Select one:
Super-ego
Preconscious system

Unconscious system
Id
The conscious system

The preconscious interfaces with both unconscious and conscious - contents of unconscious
become conscious by 'squeezing' through the preconscious. It maintains the 'repressive
barrier' to censor unacceptable wishes and desires (not the repressed contents).
The correct answer is: Preconscious system

462
26
Which of the defence mechanisms is most often used by people with narcissistic personality?

Select one:
Displacement
Projective identification
Sublimation
Reaction formation
Projection

Projection is a narcissistic defense mechanism that refers to the tendency of attributing the
origins of the undesirable impulses and feelings to another person, therefore the unacceptable
inner impulses are perceived as though they were outside of the self . E.g. A cheating husband
blames his wife of being unfaithful
The correct answer is: Projection

27
One person (A) projects undesirable impulses and feelings onto another person (B), then B
unconsciously alters her behavior in a way as if the projected idea was true. The underlying
psychodynamic mechanism is

Select one:
Projective identification
Acting out

Suppression
Repression
Projection

Projective Identification (PI) is an unconscious interaction that occurs between two persons;
this was first described by Melanie Klein. Person A projects undesirable impulses and feelings
to person B, then Person B unconsciously alters the behavior in a way as if the projected idea
was true. Example ­ a customer unconsciously felt irritable; however, criticized a sales person
for being unpleasant; the sales person was annoyed and started to act in a way as if the
original criticism was true.
The correct answer is: Projective identification

463
28
After his father's sudden death, Tom took over his family business, which he used to detest for
a long time. The defence mechanism in operation is

Select one:
Reaction formation
Displacement
Repression

Altruism
Regression

Reaction Formation occurs when a person feels an urge to do or say something and then
actually does or says something that is effectively the opposite of what they really want. A
common pattern in Reaction Formation is where the person uses 'excessive behavior', for
example using exaggerated friendliness when the person is actually feeling unfriendly. e.g., a
person who is angry with a colleague actually ends up being particularly courteous and friendly
towards them; or a mother who has a child she does not want becomes very protective of the
child.
The correct answer is: Reaction formation

29
The basis of object relations theory is that the primary motivational drive of an individual is to
seek

Select one:
Self actualisation
Money
Satisfaction
Relationships
Pleasure

Object relations theory describes with how a growing baby develops relationships with others.
Gratification is thought to be obtained through relationships as well as through satisfaction of
desires.
The correct answer is: Relationships

464
3
In psychodynamic theory, an active process in which painful thoughts and memories are
pushed to the inaccessible corners of the mind is known as

Select one:
Repression
Retrieval failure
Regression

False memory syndrome


Re-integration

The concept of repression was described by Sigmund Freud. Here people unconsciously
repress painful or disturbing memories and is normally taken to mean motivated forgetting.
The correct answer is: Repression

31
Which of the following is correct concerning defence mechanisms employed by the ego?

Select one:
Defenses operate only in pathological situations
All defences lead to complete resolution of anxiety
Repression is considered as the primary defence
Acting out is a mature defence
Immature defences are characteristic of psychosis

Repression (sometimes called motivated forgetting) is a primary ego defense mechanism since
the other ego mechanisms use it in tandem with other methods (Repression - Changing minds.
(n.d.).

The correct answer is: Repression is considered as the primary defence

32
Idealization and denigration are products of which of the following Kleinian processes?

Select one:
Regression

465
Splitting

Repression
Projective identification
Projection

Splitting refers to splitting of the self into good and bad. Idealization and devaluation
(denigration) are the results of splitting and are central in the Kleinian school of thought.
The correct answer is: Splitting

33
Which of the following is NOT a Kleinian defence?

Select one:
Denial
Sublimation
Grandiosity
Introjection
Projective identification

Sublimation is believed to be the healthiest and mature defence mechanism: It refers to


refocusing of psychic energy away from negative outlets to healthy and creative behaviour.)
The correct answer is: Sublimation

34
Which one among the following is considered as a mature defence mechanism?

Select one:
Projection
Sublimation

Denial
Regression
Repression

466
Primitive/immature defenses include splitting, projection, projective identification, idealization,
denigration, introjection, passive aggression and turning against the self. Neurotic defences
include repression, regression, denial, displacement, dissociation, isolation, reaction formation,
intellectualisation, rationalization, externalisation and identification with the aggressor. Humour,
anticipation, sublimation, altruism and suppression (SASHA) are all mature defence
mechanisms.
The correct answer is: Sublimation

35
The most mature level of anxiety according to psychoanalytic theory is

Select one:
Disintegration anxiety
Superego anxiety
Annihilation anxiety
Castration anxiety
Separation anxiety

At the most mature level of the hierarchy of anxiety, superego anxiety is related to guilt
feelings about not living up to internalized standards of moral behaviour derived from the
parents. This is seen after resolution of the oedipal conflict.
The correct answer is: Superego anxiety

36
Which one among the following statements about dream work is true?

Select one:
Primary process thinking does not occur in dreaming
The mental mechanisms involved in the psychoanalytic therapy of dreams
include primary elaboration
The dream work involves a conversion of latent content into manifest
content.
Dream work converts the manifest content of the dream into its latent
content
Dreams are always interpreted in psychoanalytic psychotherapy

467
According to Freud, dreams are the royal road to the unconscious. In dreams, Freud
distinguished between latent content (unconscious feelings that include the wishes, anxieties
and fantasies) and manifest content ( recalled dream). The dream work involves a conversion of
latent content into manifest content.
The correct answer is: The dream work involves a conversion of latent content into manifest
content.

37
In his lifetime, Freud proposed various theories with many revisions of his former theories
being evident in his later works. The terms superego, id and ego are based on

Select one:
Dream work model
Cathartic model
The topographical model
The structural model
Hysterical Model

Freud's structural model includes the Id, Ego and Superego. The topographical model
consists of concepts such as conscious, pre-conscious and the unconscious. The cathartic
model posits that hysterical neurosis is mainly due to dammed up repressed feelings,
especially sexual feelings. Release of these feelings was described by Freud as opening of
the psychic abscess. According to him, through free association or hypnosis these feelings
are released leading to a resolution of hysteria.
The correct answer is: The structural model

468
Home HiYield Paper Neurosciences

HiYield Paper A(2)

001
The part of hypothalamus that acts as a satiety centre is

Select one:
Supraoptic nucleus
Ventromedial hypothalamus
Paraventricular hypothalamus

Suprachiasmatic nucleus
Lateral hypothalamus

Check

Ventromedial hypothalamus acts as a satiety centre; lateral hypothalamus as a feeding


centre. In animals with a lesion of the ventromedial hypothalamus abnormal feeding
behaviours including hyperphagia that leads to obesity are noted.
The correct answer is: Ventromedial hypothalamus

002
What proportion (in percentage) of the brain is occupied by prefrontal cortex?

Select one:
20
70
50
30
10

469
Check

The prefrontal cortex (PFC) is the anterior part of the frontal lobes of the brain, lying in front of
the motor and premotor areas, which occupies nearly 30% of the brain.
The correct answer is: 30

003
In what percentage of left-handed people the left hemisphere is dominant?

Select one:
90
20
16
10
64

Check

The hemisphere contralateral to the dominant hand is the dominant hemisphere, and it
mediates language and speech functions. In right-handed people, the left hemisphere is
dominant. However in 10% of right-handed people the right hemisphere is dominant. In left
handed people only about 20% are right hemisphere dominant as expected, with 64% left
hemisphere dominant and 16% showing bilateral dominance. Generally, apraxia results from
dysfunction in the dominant hemisphere and agnosia results from dysfunction in the non-
dominant hemisphere.
The correct answer is: 64

004
Cranial nerves can carry both sensory and motor impulses. Which one of the following cranial
nerves has only a motor component?

Select one:
Vestibulocochlear
Glossopharyngeal

Olfactory
Optic
Abducent

Check

470
Pure sensory - Olfactory, optic and vestibulocochlear nerves.
Pure motor - Oculomotor, trochlear, abducens, accessory and hypoglossal nerves.
Mixed -Trigeminal, facial, glossopharyngeal, vagus nerves.
Autonomic- Facial, glossopharyngeal and vagus nerves.

The correct answer is: Abducent

005
Which one of the following effects is associated with lesions of the left hemisphere?

Select one:
Prospagnosia
Apraxia
Autotopagnosia
Alexia with Agraphia
Visual spatial Agnosia

Check

Left hemisphere lesions may produce alexia, agraphia, acalculia, colour anomia without
aphasia, Broca's aphasia, Wernicke's aphasia and Gerstmann syndrome. Right, hemisphere
lesions may produce constructional apraxia, prospagnosia, autotopagnosia, visual spatial
agnosia, anosognosia, receptive amusia, and contralateral neglect.
The correct answer is: Alexia with Agraphia

006
Which one of the following types of neurons has no axons?

Select one:
Golgi type 1 neurons
Amacrine neurons

Polysynaptic neurons
Golgi type 2 neurons
Bipolar neurons

Check

Neuronal cells are classified on the basis of their size into three types: Golgi type 1 ­ Long axon;
Golgi type 2 ­ Short axon terminating near the parent cell; Amacrine neurons - No axon.

471
The correct answer is: Amacrine neurons

007
Nominal aphasia can be localised to

Select one:
Angular gyrus Posterior
parietal cortex Inferior
frontal gyrus
Superior temporal gyrus
Broca's area

Check

Posterior lesions involving parietal cortex, particularly of the angular gyrus, can produce quite
pronounced anomia for visually recognized objects and may be associated with alexia.
The correct answer is: Angular gyrus

008
Which area of the brain is responsible for normal arithmetical ability?

Select one:
Fusiform gyrus
Angular Gyrus

Cuneus gyrus
Cingluate gyrus
Supramarginal gyrus

Check

The angular gyrus is situated in the parietal lobe, that lies near the upper boundary of the
temporal lobe, and immediately posterior to the supramarginal gyrus. It is involved in the
cognitive domains of language, number processing, spatial cognition, memory retrieval,
attention, and theory of mind. Damage to the left angular gyrus iresults in computational
difficulties such as acalculia and anarithmetica.
The correct answer is: Angular Gyrus

009
Which artery supplies most of the corpus callosum?

472
Select one:
Middle cerebral artery
Anterior cerebral artery
Posterior inferior cerebral artery

Posterior cerebral artery Basilar

artery

Check

Infarcts of the corpus callosum are not common due likely to a rich blood supply from the main
arterial systems, specifically the anterior cerebral and anterior communicating, but also from
posterior cerebral arteries.
The correct answer is: Anterior cerebral artery

010
Which of the following cells are the largest of all neurons in the human brain?

Select one:
Betz cells
Basket cells
Golgi cells
Purkinje cells
Stellate cells

Check

Large pyramidal cells called Betz cells are seen in the primary motor cortex.Betz cells are
pyramidal cell neurons located within the fifth layer of the grey matter in the primary motor
cortex. These can reach a diameter of 100 µm in some cases, and are known to be the largest
neurons in the nervous system. Betz cells represent about 10% of the total pyramidal cell
population in layer V of the human primary motor cortex.
The correct answer is: Betz cells

011
A lesion affecting which of the following structures can cause dysarthria?

Select one:
Homunculus
Parietal lobe

473
Temporal lobe
Cerebellum
Spinal cord

Check

Dysarthria refers to speech that is characteristically slurred, slow, and difficult to understand. It
is caused by upper motor neurone lesions of the cerebral hemispheres or lower motor neurone
lesions of the brain stem. It also results from disruption to the integrated action of upper motor
neurones, basal ganglia and cerebellum.
The correct answer is: Cerebellum

012
The internal capsule is supplied by which of the following structures?

Select one:
Anterior cerebral artery
Basilar artery

Posterior inferior cerebellar artery


Middle cerebral artery
Circle of Willis

Check

The internal carotid artery enters the circle of Willis and divides to form the anterior cerebral and
middle cerebral arteries. The anterior cerebral artery supplies the medical and superior lateral
aspects of the cerebral cortex to the parietal/occipital border. The middle cerebral artery
supplies the greater part of the lateral aspect of the cerebral cortex. The middle cerebral artery
supplies the Broca's and Wernicke's area in the dominant hemispheres. The posterior cerebral
artery supplies the inferomedial temporal lobe and the occipital lobe. Internal capsule contains
traversing corticospinal tract and receives blood supply from different parts of the circle of Willis.
The superior portion of the anterior limb is supplied by lenticulostriate branches of the middle
cerebral artery while the inferior half is supplied by the recurrent artery of Heubner arising from
the anterior cerebral artery. The genu receives supply from lenticulostriate branches of middle
cerebral artery, while posterior limb is supplied by the lenticulostriate branches of middle
cerebral artery (superior half) and anterior choroidal artery off of the internal carotid artery
(inferior half).
The correct answer is: Circle of Willis

013
Which of the following is affected if there is an occlusion of the posterior cerebral artery?

474
Select one:
Cingulate gyrus
Corpus callosum
Dentate gyrus

Urcinate gyrus
Arcuate fasciculus

Check

The dentate gyrus is a part of the hippocampal formation. It is one of the few areas of the
human brain where new neurons are continuously produced in adulthood. The posterior
cerebral artery provides blood supply of the hippocampal formation.
The correct answer is: Dentate gyrus

014
The thalamus and hypothalamus are constituents of which of the following embryological
structures?

Select one:
Myelencephalon
Metencephalon
Telencephalon
Mesencephalon
Diencephalon

Check

During ontogeny, the midline neural tube differentiates into the following vesicles; 1.
Prosencephalon, which differentiates into the Telencephalon and Diencephalon 2.
Mesencephalon (midbrain structures) and 3. Rhombencephalon which differentiates into the
Metencephalon and Myelencephalon. The hypothalamus and thalamus are constituents of the
diencephalon.
The correct answer is: Diencephalon

015
Mrs. X is an 83-year-old lady who has been diagnosed with early stages of Alzheimer's
disease. The brain region, which could show prominent changes, is;

Select one:
Prefrontal cortex

475
Entorhinal cortex
Superior temporal gyrus
Caudate nucleus

Prefrontal cortex

Check

Profound loss of layer two entorhinal cortex neurons occurs in very mild Alzheimer's disease.
The entorhinal cortex plays a crucial role connecting the neocortex and hippocampal formation
and is affected severely likely contributing to memory impairment.
The correct answer is: Entorhinal cortex

016
Synaptogenesis is at its highest during which of the following phases of life?

Select one:
Preschool years

Immediately preceding puberty


During adolescence
After 65 years of age
First 2 years of life

Check

Synaptogenesis occurs very rapidly from the second trimester through the first ten years or so
of life. The peak of synaptogenesis occurs within the first two postnatal years.
The correct answer is: First 2 years of life

017
Mr. Y developed normal pressure hydrocephalus following trauma in the basal brain. The
obstruction is most likely to have occurred in the;

Select one:
Foramen of Luschka
Foramen of Munro
Cisterna Magna
Arachnoid granulations

Foramen of Magendie

476
Check

The question indicates that there is an obstruction. If there is an obstruction after trauma, this is
likely to be at Foramen of Monro. A number of cases will have non-obstructive form, where
arachnoid villi are implicated. Posttraumatic hydrocephalus (PTH) is a frequent and serious
complication that follows a traumatic brain injury. PTH may present as normal pressure
hydrocephalus (NPH, no increase in intracranial pressure) or as a syndrome of increased
intracranial pressure due to either communicating or noncommunicating hydrocephalus. In
noncommunicating hydrocephalus (obstructive hydrocephalus), CSF accumulates in the
ventricles because of CSF flow blockage. As a result, the ventricles enlarge and the
hemispheres expand. The commonest site of block in non-communicating hydrocephalus is
foramen of Monroe. Other sites include; Third ventricle, Aqueduct of Sylvius, Fourth ventricle,
Foramen of Luschka and Foramen of Magendie. Impaired CSF absorption may cause
communicating hydrocephalus. The apparent mechanism is partial occlusion of the arachnoid
villi in this case. (Posttraumatic Hydrocephalus - Medscape Reference,
http://emedicine.medscape.com/article/326411-overview).
The correct answer is: Foramen of Munro

018
Which one of the following is not a function of astrocytes?

Select one:
Maintaining the blood brain barrier
None of the above
Nutrition of neurons
Breakdown of some neurotransmitters
Formation of CNS myelin sheath

Check

Formation of CNS myelin sheath is a function of oligodendrocytes and not astrocytes.


Oligodendrocytes are seen in CNS (not in peripheral nerves, where Schwann cells replace
them). They produce myelin sheaths, which help in saltatory conduction (pole to pole
jumping), which quicken the process of signal transmission.
The correct answer is: Formation of CNS myelin sheath

019
Obstruction to CSF circulation leads to non-communicating hydrocephalus. The cerebrospinal
fluid passes into the subarachnoid space via recesses in which of the following structures?

Select one:
Lateral ventricle

477
First ventricle
Fourth ventricle
Third ventricle

Aqueduct of sylvius

Check

CSF is secreted by the choroid plexus in the lateral, third and fourth ventricles and at a rate of
300 ml/day, which is almost protein free. It passes into the subarachnoid space via recesses in
the fourth ventricle-Foramina of Magendie and Foramina of Luschka. Obstruction to its
circulation commonly occurs within third or fourth ventricle, leading to non-communicating
hydrocephalus. Obstruction to CSF flow in the subarachnoid space leads to communicating or
normal pressure hydrocephalus.
Circulation of CSF

1. From lateral ventricle to 3rd ventricle- via Interventricular foramina of Monro


2. From 3rd to 4th Ventricle- via cerebral aqueduct of Sylvius
3. From 4th ventricle to subarachnoid space via Foramen of Magendie (single medial
foramen) and Foramen of Luschka (two, lateral foramina)

The correct answer is: Fourth ventricle

020
A 62-year-old gentleman presented to the outpatients department with a history of changes in
personality, socially disinhibited behaviour, poor judgement and irritability. His carer also
reported a recent history of urinary incontinence. Which part of the brain is most likely to be
affected?

Select one:
Occipital lobe

Frontal lobe
Cerebellum
Temporal Lobe
Parietal lobe

Check

Frontal lobe dysfunction would include features such as disinhibition, loss of abstract ability,
errors of judgement, irritability, elevated mood, and childishness, lack of drive, impaired
initiation, attention and concentration.
The correct answer is: Frontal lobe

478
021
Forced utilization behaviour can be a feature of damage to which of the following structures?

Select one:
Cerebellar damage
Parietal lobe damage
Temporal lobe damage
Occipital lobe damage

Frontal lobe damage

Check

If the frontal lobe superior to the eye (orbitofrontal cortex) is damaged, forced utilization can be
seen. When objects are placed in front of a subject with frontal damage, the subject will pick
them and start using them even when the subject is instructed not to use them.
The correct answer is: Frontal lobe damage

022
Which of the following enzymes is exclusively seen in astrocytes?

Select one:
COMT

MAO-B
Glutamate dehydrogenase
Tyrosine hydroxylase
Acetylcholinesterase

Check

Glutamate dehydrogenase is seen only in astrocytes and not in other glial cells.
The correct answer is: Glutamate dehydrogenase

023
Lesions in the subthalamic area are associated with which of the following movement
disorders?

Select one:
Hemiballismus
Huntington's chorea

479
Dystonia
Athetosis
Parkinsons disease

Check

Lesions in the corpus striatum are associated with dystonia, athetosis and chorea. Lesions in
the subthalamic nucleus are associated with Hemiballismus. Lesions in the substantia nigra are
associated with Parkinson's disease.
The correct answer is: Hemiballismus

024
Which of the following is NOT a part of Papez circuit?

Select one:
Mamillary body
Parahippocampal gyrus

Cingulate gyrus
Hippocampus
Heschl's gyrus

Check

The Papez circuit consists of the following: Hippocampal formation, fornix, mammillary bodies,
mammillothalamic tract, anterior thalamic nucleus, cingulate gyrus, Parahippocampal gyrus,
entorhinal cortex and perforant pathway ending in the hippocampus
The correct answer is: Heschl's gyrus

025
The limbic Papez circuit includes which of the following structures?

Select one:
Hippocampus
Temporal pole
Medullary centre
Trapezoid body
Red nucleus

480
Check

Amygdala, septal nuclei, fornix, parahippocampal gyrus and mammillothalamic tract are all
constituents of the limbic system in addition to the hippocampus.
The correct answer is: Hippocampus

026
In which area of the brain is the dentate gyrus located?

Select one:
Hippocampus
Hypothalamus
Dorsolateral prefrontal cortex
Amygdala
Red nucleus

Check

The dentate gyrus is part of the hippocampal formation. It is thought to contribute to the
formation of new episodic memories. It is currently known to have high rates of neurogenesis in
animal experiments using adult rats
The correct answer is: Hippocampus

027
Mirror neurons are found in which part of the brain?

Select one:
Inferior parietal lobe
Lateral frontal cortex

Medial-dorsal frontal cortex


Medial temporal lobe
Inferior frontal cortex

Check

Functional magnetic resonance imaging studies reveal activity of the inferior frontal and
superior parietal cortices both during the eprformance of an action, and the observation of
other person performing the same action. These brain regions supposedly contain mirror

481
neurons, which explain why we imitate activity that we observe (modelling). For example while
watching sports like tennis the sports fans may feel so emotionally involved in the game and
react almost as if they were part of the game themselves. This behavior is explained by the
presence of mirror neurons in our brain.
The correct answer is: Inferior frontal cortex

028
Which of the following statements about microglia is correct?

Select one:
It is mainly responsible for myelination
It constitutes 70% of glial population in brain
It is mesodermal in origin
It is sensitive to sodium channel stimulation
It is present outside brain and spinal cord

Check

Unlike macroglia (astrocytes and oligodendrocytes) and neurons, which are derived from
neuroectoderm, microglial progenitors arise from peripheral mesodermal (myeloid) tissue
The correct answer is: It is mesodermal in origin

029
Which function of the brain is served by angular gyrus?

Select one:
Balance & co-ordination
Motor movements

Language
Memory
Executive functions

Check

The angular gyrus is a region of the inferior parietal lobe of the brain that is involved in the
processing of auditory and visual input and in the comprehension of language. It is Brodmann
area 39 of the human brain. Lesions are associated with anomia.
The correct answer is: Language

482
030
Foramina of Munro are channels that connect

Select one:
Lateral and third ventricles
Third and fourth ventricle
Two third ventricle
4th ventricle and spinal canal

Two lateral ventricles

Check

In the brain, the interventricular foramina of Monro connect the 2 lateral ventricles with the 3rd
ventricle at the midline. These foramen serve as conduits for cerebrospinal fluid (CSF)
produced in the lateral ventricles to reach the third ventricle and then the rest of the brain's
ventricular system.
The correct answer is: Lateral and third ventricles

031
Which one of the following structures is involved in the visual pathway?

Select one:
Medial geniculate body
Heschl's gyrus

Inferior Colliculus
Lateral geniculate body
Tegmentum

Check

The medial fibres of the optic nerve cross in the optic chiasma to join the contralateral optic
tract. The lateral fibres of the optic nerve pass through the ipsilateral optic tract. The fibres
synapse in the lateral geniculate body of the thalamus. From here the optic radiation runs within
the posterior part of the internal capsule and terminates in the visual cortex. Myers loop is part
of the optic radiation, which loops anteriorly from the lateral geniculate ganglion into the
temporal lobe before travelling posteriorly in the occipital cortex
The correct answer is: Lateral geniculate body

032
Which of the following structures divide the frontal lobe from the temporal lobe?

483
Select one:
Central sulcus
Lateral sulcus
Cingulate gyrus

Calcarine fissure
Superior temporal gyrus

Check

The lateral sulcus (also called Sylvian fissure or lateral fissure) is a prominent landmark of the
human brain. It divides the frontal lobe and parietal lobe from the temporal lobe.
The correct answer is: Lateral sulcus

033
The foramen of Monro connects

Select one:
Fourth ventricle to subarachnoid space
Lateral ventricle to third ventricle
Fourth ventricle to central canal of spinal cord
Lateral ventricle to fourth ventricle
Lateral ventricle to second ventricle

Check

The foramen of Monro (also known as the interventricular foramen) is part of the ventricular
system and the connection between the third ventricle and the lateral ventricles. It allows for
the flow of CSF, and, therefore, if blocked or effaced, causes a reduction in CSF flow from the
lateral ventricles despite no associated reduction in CSF production from the choroid plexus.
The correct answer is: Lateral ventricle to third ventricle

034
Which of the following statements regarding amygdala is NOT correct?

Select one:
Damage leads to impaired fear conditioning
Emotional face processing involves amygdalar activity
It is involved in emotional memory

484
Learning does not occur in the absence of memory consolidation mediated by
amygdala
Monkeys with amygdalar damage show impaired maternal behaviours

Check

Amygdala is involved in rating the emotional importance of an experience and activating the
level of hippocampal response accordingly. Amygdalar damage leads to loss of fear
conditioning and in monkeys, loss of maternal behaviour has also been noted. Despite the
amygdalar damage, learning and consolidation of memory can occur, especially in the
absence of emotional valence and arousal.
The correct answer is: Learning does not occur in the absence of memory consolidation
mediated by amygdala

035
OCD is associated with an abnormality in which of the following parts of the brain?

Select one:
Inferior olivary nucleus
Medial temporal lobe
Dendate nucleus

Lentiform nucleus
Red nucleus

Check

Basal ganglia dysfunction is implicated in OCD. The components of basal ganglia are the
caudate nucleus, the putamen and globus pallidus. Putamen and globus pallidus together
form the lenticular/lentiform nuclei. Neuroimaging studies with PET scan suggest the
involvement of both caudate and lentiform nuclei dysfunction in patients with OCD.
The correct answer is: Lentiform nucleus

036
Which of the following is correct with respect to blood brain barrier?

Select one:
Lipophilic substances penetrate the barrier better
Basal forebrain lacks the barrier
Endothelial cells form gap junctions
Microglial foot processes form the blood brain barrier

485
Inflammation can strengthen blood brain barrier

Check

Lipid soluble molecules, such as ethanol and caffeine can penetrate through the barrier
relatively easily via the lipid membranes of the cells. Brain endothelial cells are joined by tight
junctions of high electrical resistance providing an effective barrier against molecules.
The correct answer is: Lipophilic substances penetrate the barrier better

037
Which one of the following cells is considered as descendants of macrophages that clear
neuronal debris following cell death?

Select one:
Microglial cells
Ependymal cells
Schwann cells

Astrocytes
Oligodendrocytes

Check

The microglia are descendants of macrophages. They are scavenger cells, which clear
neuronal debris following cell death.
The correct answer is: Microglial cells

038
The ventral tegmental area is located in which part of the brain?

Select one:
Midbrain
Occipital lobe
Pons
Medulla
Temporal lobe

Check

486
The ventral tegmentum is located in the midbrain and contains dopaminergic cells that give
rise to the mesolimbic dopamine pathway. This pathway is crucial for reward and sensations of
pleasure.
The correct answer is: Midbrain

039
Which of the following is correct with respect to cerebral dominance?

Select one:
Most left handed individuals have bilateral dominance Most
left handed individuals have right cerebral dominance Most
left handed individuals have left cerebral dominance
Right handed individuals always have right cerebral dominance

Dominance cannot be ascertained in left handed individuals

Check

In 10% of right-handed people, the right hemisphere is dominant. In left handed people only
about 20% are right hemisphere dominant as expected, with 64% left hemisphere dominant
and 16% showing bilateral dominance.
The correct answer is: Most left handed individuals have left cerebral dominance

040
Which one of the following is a function of oligodendrocytes?

Select one:
Metabolic clearance of transmitters
Gliosis and scar formation

Structural support of neurons


Contributing to the blood brain barrier
Myelination

Check

The function of astrocytes includes providing structural support of neurones, forming CNS
neuroglial scar tissue (gliosis), phagocytosis and also contributing to the blood brain barrier.
The function of oligodendrocytes includes CNS myelin sheath formation and phagocytosis.
The correct answer is: Myelination

487
041
Neural crest cells originate from

Select one:
Endoderm
Pharyngeal pouch
Mesoderm
Ectoderm
Neural tube

Check

Neural crest refers to a group of embryonic cells that originate from the neural tube (the
precursor of the spinal cord) but later dissociate from the central nervous system. The
embryonic crest cells migrate to numerous locations in the body to form diverse structures
such as (1) the neurons and glial cells of the sensory, sympathetic, and parasympathetic
nervous systems, (2) the adrenergic cells of the adrenal medulla, (3) the pigment-containing
cells of the epidermis, and (4) many of the skeletal and connective tissue components of the
head (http://www.ncbi.nlm.nih.gov/books/NBK10065/).
The correct answer is: Neural tube

042
Which of the following is a purely motor cranial nerve?

Select one:
Facial nerve
Oculomotor nerve
Vagus nerve
Glossopharyngeal nerve
Vestibulocochlear nerve

Check

Pure sensory-Olfactory, optic and Vestibulocochlear nerves.


Pure motor-Oculomotor, trochlear, abducens, accessory and hypoglossal nerve.
Mixed-Trigeminal, facial, glossopharyngeal, vagus nerve.

The correct answer is: Oculomotor nerve

043

488
Which area of the brain is associated with 'social valuation'?

Select one:
Calcarine fissure
Lingual gyrus
Orbitofrontal cortex

Anterior cingulate gyrus


Hippocampal gyrus

Check

Orbitofrontal cortex is often included as the brain region with a prominent role in social
valuation, though many other regions such as the temporoparietal junction, anterior cingulate
cortex and fusiform gyrus also play a part in this function. Damage to OFC can cause a
pseudopsychopathic syndrome.
The correct answer is: Orbitofrontal cortex

044
Which one of the following is seen as a key function of the non-dominant cerebral
hemisphere?

Select one:
Logical reasoning
Pictorial memory
Analytical capacity
Verbal learning
Language functions

Check

Disorders of verbal and pictorial memory are commonly dissociated by brain injury with verbal
memory being affected in left and picture memory predominantly by right brain injury; However,
this is disputed as many subjects use verbal encoding when remembering pictures.
The correct answer is: Pictorial memory

045
Primary sensory cortex is located in which of the following areas?

Select one:
Superior temporal area

489
Inferior parietal lobule
Dorsolateral prefrontal cortex
Precentral gyrus

Postcentral gyrus

Check

Brodmann areas 3, 1 and 2 comprise the primary somatosensory cortex of the human brain
(or S1) - most of which is located on the postcentral gyrus. Lesions affecting the primary
somatosensory cortex produce characteristic symptoms including: agraphesthesia,
astereognosia, loss of vibration, proprioception and fine touch. The primary motor cortex (or
M1) is a brain region that in humans is located in the posterior portion of the frontal lobe. It
works in association with pre-motor areas to plan and execute movements. M1 is located on
precentral gyrus.
The correct answer is: Postcentral gyrus

046
Hippocampus is supplied by

Select one:
Posterior choroidal artery

Posterior cerebral artery


Posterior inferior cerebellar artery
Middle cerebral artery
Anterior cerebral artery

Check

The hippocampal arteries mainly arise from posterior cerebral artery and to a lesser extent
from the anterior choroidal artery.
The correct answer is: Posterior cerebral artery

047
The motor cortex occupies

Select one:
Heteromodal association cortices
Supplementary motor area
Frontal eye fields

490
Precentral gyrus
Postcentral gyrus

Check

Primary Motor Cortex is located in the precentral gyrus, which also encompasses premotor,
supplementary motor and frontal eye fields. Except olfaction, all other sensations are relayed
via the thalamus to sensory cortex. It occupies the postcentral gyrus.
The correct answer is: Precentral gyrus

048
Which part of the brain is last to mature?

Select one:
Amygdala
Prefrontal cortex
Midbrain

Parahippocampal cortex
Thalamus

Check

In phylogeny as in ontogeny, the association cortex of the frontal lobe, also known as the
prefrontal cortex, is a late-developing region of the neocortex. It is also one of the cortical
regions to undergo the greatest expansion in the course of both evolution and individual
maturation. In the human adult, the prefrontal cortex constitutes as much as nearly one-third of
the totality of the neocortex (Excerpt from Frontal lobe and cognitive development,
http://www.ncbi.nlm.nih.gov/pubmed/12815254 (accessed April 13, 2015)).
The correct answer is: Prefrontal cortex

049
Which of the following cells are uniquely found in the cerebellum?

Select one:
Purkinje cells
Betz cells
Pyramidal cells
Granule cells
Stellate cells

491
Check

Purkinje cells are a class of GABAergic neurons located in the cerebellar cortex only. Purkinje
cells send inhibitory projections to the deep cerebellar nuclei and constitute the sole output of all
motor coordination in the cerebellar cortex.
The correct answer is: Purkinje cells

050
The most common type of neuronal cells seen in the cerebral cortex is

Select one:
Stellate cell
Spindles cells
Granule cell
Purkinje cells
Pyramidal cells

Check

The pyramidal neurons with their triangular-shaped cell bodies make up nearly 75% of the
cortical neurons. Stellate cells (25%) are present in all the layers except layer 1.
The correct answer is: Pyramidal cells

051
What is the function of an endosome?

Select one:
Recycling of cell membrane
Production of hydrogen peroxide
Transcribe information from mRNA
Check the synthesised proteins for structural conformity before delivery

Lipid peroxidation

Check

An endosome is a membrane-bound cellular compartment of the endocytic membrane


transport pathway extending from the plasma membrane to the lysosome. Molecules
internalized from the plasma membrane are transported via endosomes to lysosomes for

492
degradation; similarly molecules can also be recycled back to the plasma membrane.
The correct answer is: Recycling of cell membrane

052
Which one among the following structures is a constituent of mesencephalon?

Select one:
Subthalamus
Cerebellum
Red nucleus
Corpus striatum
Medullary centre

Check

Prosencephalon includes 1. Telencephalon, which gives rise to cerebral hemispheres and


contains the pallium, rhinencephalon, and basal ganglia 2. Diencephalon consisting of thalamus,
subthalamus, hypothalamus and epithalamus consisting of the habenular nucleus and pineal
gland. Mesencephalon represents the growing midbrain and consists of 1. Tectum, in turn
consisting of the corpora quadrigemina, made up of the superior and inferior colliculi 2. Basis
pedunculi 3. Tegmentum containing the red nucleus, fibre tracts and grey matter surrounding the
cerebral aqueduct. Finally, the rhombencephalon includes 1. Metencephalon consisting of the
pons, the oral part of the medulla oblongata and cerebellum and 2. Myelencephalon consisting
of the caudal part of the medulla oblongata.
The correct answer is: Red nucleus

053
Electrical synapses are seen in

Select one:
Skin
Olfactory epithelium

Bone marrow
Retina
Lungs

Check

An electrical synapse is formed at a narrow gap between the pre- and postsynaptic neurons
(gap junction) and helps conduct electrical messages across cells. Electrical synapses are
abundant both in the retina and cerebral cortex of animals.

493
The correct answer is: Retina

054
Which one among the following types of neuroglia is present in the peripheral nervous
system?

Select one:
Schwann cells
Astrocytes
Microglia
Ependyma
Oligodendrocytes

Check

Neuroglia, also called as interstitial cells outnumber neurones by a factor of five to ten times.
The main types of neuroglial cells in the central nervous system include Microglia, Ependyma,
Oligodendrocytes and Astrocytes. The main types of neuroglia that is present in the peripheral
nervous system include Schwann cells and satellite cells.
The correct answer is: Schwann cells

055
Which structure separates the two lateral ventricles in the human brain?

Select one:
Septum pellucidum
Pons

Aqueduct of sylvius
Septum pallidum
Corpus callosum

Check

The body of the lateral ventricle lies immediately below the corpus callosum and they are
separated by septum pellucidum. An anomalous splitting of the septum pellucidum may be
more common in schizophrenia than in general population (cavum septum pellucidum). The
third ventricle lies between thalamus and hypothalamus. The fourth ventricle lies above the
pons and just below the cerebellum. The Aqueduct of Sylvius links the third and fourth
ventricles.
The correct answer is: Septum pellucidum

494
056
Which of the following constitute the most common cells of the cerebral cortex?

Select one:
Large pyramidal cells
Stellate cells
Horizontal cells of cajal

Glial cells
Small Pyramidal cells

Check

The cerebral cortex is stratified into six layers. The most common cells are stellate cells, which
are present in all the layers except layer 1, and pyramidal cells. Layer 1 contains glial cells and
dendrites from neurones of deeper layers and the horizontal cells of Cajal. Layers 2 and 3 are
composed of small pyramidal cells, whose axons project out of and within the hemispheres.
Stellate and fusiform cells lie in layer 4 and provide local connections and receive ascending
fibres. Layer 5 consists of large pyramidal cells. The cerebellar cortex is three layered. The
molecular layer consisting of basket cell and stellate cell, Purkinje layer consisting of Purkinje
cell and a granular layer consisting of granule and Golgi cell.
The correct answer is: Stellate cells

057
Huntington's disease is caused due to degeneration of

Select one:
Pons
Prefrontal cortex
Cerebellum
Striatum
Subthalamic nucleus

Check

Huntington's disease is caused due to degeneration of the striatum (mainly caudate nucleus)
and selective loss of GABA-ergic neurones.
The correct answer is: Striatum

058
Which of the following nuclei of basal ganglia contains melanin pigment?

495
Select one:
Globus pallidus
Substantia nigra
Caudate nucleus

Subthalamic nuclei
Putamen

Check

Substantia nigra appears black because of melanin pigment.


The correct answer is: Substantia nigra

059
Which of the following structures give rise to cerebral hemispheres during embryogenesis?

Select one:
Myelencephalon
Diencephalon
Mesencephalon
Metencephalon
Telencephalon

Check

Telencephalon gives rise to cerebral hemispheres and contains the pallium, rhinencephalon,
and basal ganglia
The correct answer is: Telencephalon

060
Which part of the brain is associated with aura in epilepsy?

Select one:
Frontal lobe
Pre central gyrus
Occipital lobe

Temporal lobe
Parietal lobe

496
Check

The early warning symptoms of a seizure are called aura; these are due to the onset of the
seizure activity in a circumscribed part of the temporal lobe. The aura may stop without further
spreading, or go on to spread resulting in generalisation and altered consciousness. The
temporal lobe is involved in consolidating memories and processing emotions; seizures in this
area may begin with feelings of fear, feelings of joy, recall of certain music, smells, and other
unusual symptoms.
The correct answer is: Temporal lobe

061
Which lobe is initially affected in Alzheimer's disease?

Select one:
Prefrontal lobe
Temporal lobe
Frontal lobe
Parinetal lobe
Occipial lobe

Check

The temporal lobes, especially the hippocampal areas in them, are affected early in Alzheimer's
disease. Medial temporal lobe atrophy and loss of hippocampal volume is one of the earliest
findings seen in patients with Alzheimer's disease.
The correct answer is: Temporal lobe

062
A patient presents with rapid onset personality changes and memory difficulties. On
examination, he has homonymous superior quadrantanopia. Where is the lesion likely to be?

Select one:
Optic chiasma
Lateral Geniculate Body

Occipital lobe
Temporal lobe
Retina

Check

497
A lesion anterior to the optic chiasma results in loss of the visual field to ipsilateral eye. Any
lesion of or posterior to the optic chiasma results in visual defects in both eyes. Lesions behind
the chiasma result in homonymous defects. Homonymous hemianopia results from loss of the
optic tract or radiation on one side. Damage to Myers loop results in homonymous superior
quadrantanopia and it may be the first indication of a lesion within the temporal lobe.
The correct answer is: Temporal lobe

063
Regarding the blood supply of language areas which of the following is correct?

Select one:
Wernicke's area is supplied by posterior cerebral artery

Transient ischemia of carotid system produces aphasia

Transient ischemia of carotid system produces dysarthria

Vertebrobasilar system supplies motor speech area Right

sided carotid block produces Broca's aphasia

Check

The carotid system is responsible for supplying language areas. A lesion can produce
transient aphasia.
The correct answer is: Transient ischemia of carotid system produces aphasia

064
Nucleus accumbens forms a part of which of the following structures of the brain?

Select one:
Hypothalamus
Ventral striatum

Pineal gland
Dorsal region of thalamus
Orbitofrontal cortex

Check

The nucleus accumbens is a collection of neurons and forms the main part of the ventral
striatum, located proximal to the head of the caudate nucleus and the anterior portion of the
putamen.
The correct answer is: Ventral striatum

498
Finish review

499
Question 1 001
Not answered Which one of the following is the rate-limiting step in the synthesis of dopamine?
Marked out of 1.00
Select one:
Flag question
Dopamine beta Hydroxylase
Choline acetyltransferase
Catechol o methyl transferase

Phenyl ethanolamine N-Methyltransferase


Tyrosine Hydroxylase

The rate-limiting step in the biosynthesis of dopamine is tyrosine hydroxylase, which converts
tyrosine into L-DOPA. The rate-limiting step in the biosynthesis of serotonin is tryptophan
hydroxylase. The enzyme, dopamine ß-hydroxylase, converts dopamine to noradrenaline.
Phenylethanolamine-N-methyl transferase is involved in the conversion of noradrenaline to
epinephrine. Choline acetyltransferase is involved in the biosynthesis of ACh.
The correct answer is: Tyrosine Hydroxylase

002
Which one of the following receptors is a ligand-gated cation channel?

Select one:
Noradrenaline
5-HT 7
5-HT 1 b
5-HT 1a

500
5-HT 3

There are fourteen 5HT receptors; all are G-protein coupled apart from 5HT-3, which is a
ligand-gated cation channel. 5HT-1 group (5HT-1a, 5HT-1b, 5-HT-1d) are inhibitory and are
negatively coupled to cyclic AMP. 5HT-2 receptors (5HT-2a,2b,2c) are excitatory and act
through the phospholipase C/inositol phosphate pathway. 5HT-4, 5HT-5,5HT-6,5HT-7
receptors are positively coupled to Cyclic AMP and are thus excitatory.
The correct answer is: 5-HT 3

003
The Nucleus of Meynert is a prominent site of localization for which neurotransmitter in the
brain?

Select one:
GABA
Norepinephrine
Serotonin
Acetylcholine

Dopamine

Acetylcholine is localized to an ascending system of cholinergic n originating in the


reticular formation and nucleus basalis of Meynert.

The correct answer is: Acetylcholine

004
The drug 'khat' (Catha edulis) has effects comparable to

Select one:
Cannabis

Benzodiazepines
Alcohol
Amphetamine
LSD

501
Cathinone is an alkaloid present in the leaves of the khat bush that grows in East Africa and
southern Arabia and is often chewed because of its stimulating properties. Cathinone, which is
S(-)-alpha-aminopropiophenone, has a pharmacological profile closely resembling that of
amphetamine; indeed, in a wide variety of in vitro and in vivo experiments it was demonstrated
that cathinone shares the action of amphetamine on CNS as well as its sympathomimetic
effects; thus, for example, drug­conditioned animals will not distinguish between cathinone and
amphetamine. It operates through the same mechanism as amphetamine; i.e. it acts by
releasing catecholamines from presynaptic storage sites. Thus, much experimental evidence
indicates that cathinone is the main psychoactive constituent of the khat leaf and that, in fact,
this alkaloid is a natural amphetamine (Excerpt from Cathinone, a natural amphetamine.
Retrieved from http://www.ncbi.nlm.nih.gov/pubmed/1508843).
The correct answer is: Amphetamine

005
Which of the following receptors increase adenylate cyclase to stimulate cell machinery?

Select one:
Dopamine D2
Dopamine D3
Alpha 1
Dopamine D4

Beta adrenergic

Alpha1 receptors phospholipase C coupled; Alpha 2 are Gi-coupled(inhibitory). Beta


receptors Gs coupled - so they increase adenylate cyclase.
The correct answer is: Beta adrenergic

006
D1 and D2 receptors are mainly located in which of the following structures?

Select one:
Nucleus accumbens

Amygdala
Caudate-putamen
Prefrontal cortex
Hippocampus

502
There are two families of DA receptors. D1 and D5 receptors (D1-like), which are positively
coupled to Cyclic AMP, increase cyclic AMP. D2, D3, D4 (D2-like) that inhibit and decreases
cyclic AMP. D1 and D2are predominant in the caudate and putamen, D3-nucleus accumbens,
D4-- the prefrontal cortex, D5-Hippocampus. Both D1 and D 2 have wide distribution (striatal,
mesolimbic and hypothalamic) while D3 and D4 are more localised (mesolimbic, cortical and
hippocampal).
The correct answer is: Caudate-putamen

007
An enzyme involved in synthesis of acetylcholine is

Select one:
Choline acetyl transferase
Dopa hydroxylase
Acetylcholinesterase
Butyrylcholinesterase
Mono amine oxidase

Cortical choline acetyl transferase (ChAT) is reduced to a greater extent (85%) in patients with
hallucinations in Lewy body dementia than in those without hallucinations (50%). This is more
pronounced in parieto-temporal regions, hippocampus and entorhinal cortex. As a
consequence, brain acetylcholine levels are reduced in DLB similar to Alzheimer's. This may
partially explain the altered sleep-wake patterns seen in DLB and also the response of
hallucinations to acetylcholinesterase inhibitors.
The correct answer is: Choline acetyl transferase

008
The neurochemical changes reported in Alzheimer's disease include

Select one:
Increased DOPA decarboxylase
Increased levels of choline acetyl transferase

Increased levels of acetyl cholinesterase


Increased GABA levels
Decreased levels of acetyl cholinesterase

503
The neurochemical changes reported in Alzheimers disease would include decreased levels of
acetylcholinesterase, decreased levels of choline acetyltransferase, decreased GABA levels
and decreased levels of noradrenaline.
The correct answer is: Decreased levels of acetyl cholinesterase

009
Stimulation of nicotine receptors leads to the release of which of the following
neurotransmitters?

Select one:
Serotonin
Dopamine
Melatonin
Adrenaline
Norepinephrine

Nicotine is highly lipid soluble and rapidly enters the brain after inhalation (smoking). Nicotine
receptors are found on dopaminergic cell bodies, and stimulation of nicotine receptors leads to
release of dopamine
The correct answer is: Dopamine

010
Which one of the following is an incorrect pairing of the disease condition and the primary
neurotransmitter that is abnormal in the diseases?

Select one:
Noradrenalin-depression
Dopamine - Alzheimer's dementia

GABA- epilepsy
Dopamine- Schizophrenia
Serotonin-anxiety

Dopamine is implicated in both schizophrenia and Parkinson's disease. Glutamate is


implicated in many neurodegenerative disorders while GABA is implicated in epilepsy. Both
noradrenaline and serotonin are relevant for depression and anxiety. Acetylcholine is more

504
relevant than dopamine for Alzheimer's dementia. Please note that the explanation here is an
over-simplification - most neuropsychiatric disorders include a generalised disturbance in
multiple neurotransmission systems.
The correct answer is: Dopamine - Alzheimer's dementia

011
Which of the following neurotransmitters can be called catecholamines?

Select one:
Serotonin only
Dopamine only
Noradrenalin only
Dopamine and serotonin
Dopamine and noradrenalin

All the three are monoamines, but dopamine and noradrenaline are catecholamines while
serotonin is an indolamine.
The correct answer is: Dopamine and noradrenalin

012
Which neurotransmitter plays an important role in the neurochemical changes seen in
Huntington's disease?

Select one:
Dopamine
Endorphins

Serotonin
GABA
Noradrenaline

Reduced GABA, Reduced glutamic acid decarboxylase, reduced acetyl choline, reduced
substance P, Raised somatostatin and reduced corticotrophin releasing factor are seen in
Huntington's disease.
The correct answer is: GABA

505
013
Which of the following is an ionotropic receptor?

Select one:
5HT2A receptor

Dopamine D2 receptor

GABA-B receptor

GABA-A receptor
Norepinephrine receptor

GABA­A ­ opens chloride channel; inhibitory ­ leads to hyperpolarization; made of five


subunits and at least 14 subunit subtypes
The correct answer is: GABA-A receptor

014
Which one of the following acts as a fast acting excitatory neurotransmitter?

Select one:
5-hydroxytryptamine
Muscarinic Acetylcholine receptors

Dopamine
Glutamate
Noradrenalin

Receptors for neurotransmitters can be either 1. Directly coupled to an ion channel (ionotropic
receptors) and so concerned with fast transmission Eg GABA-A, Nicotinic type of Ach receptors,
N-Methyl-D-aspartate type of glutamate receptors). Glutamate is an example of a fast acting
excitatory neurotransmitter where the receptors (NMDA) are directly linked to a sodium channel.
Activation of the GABA-A receptor, which is linked to a chloride channel, results in an influx of
chloride ion into the neurone causing hyper polarisation. 2. Coupled to an intracellular effector
system through G-Protein (Metabotropic receptors) and so responsible for slow
neurotransmission (dopamine, nor adrenaline, most 5-HT and muscarinic Ach receptors).

The correct answer is: Glutamate

015
Which of the following substances serve as a precursor for GABA?

506
Select one:
Adrenaline
Dopamine
Glucagon

Butyrylcholinesterase
Glutamate

GABA is one of the most ubiquitous inhibitory neurotransmitters in the CNS, and an estimated
40% of all brain synapses use it. Within the cerebellum, Purkinje cells produce GABA. GABA
projections are found arising in the striatum, terminating in the substantia nigra and Globus
pallidus. Loss of these GABA neurones in the caudate and putamen occurs in Huntington's
chorea. Underactivity of the GABAergic system would theoretically result in epilepsy and
reduction in GABA-BDZ receptors have been demonstrated in seizure foci.
The correct answer is: Glutamate

016
Which of the following is an aminoacid neurotransmitter?

Select one:
Glycine
Acetylcholine
Neurotensin
Dopamine
Endorphins

The important inhibitory amino acids are GABA and glycine. Excitatory amino acids are
glutamic acid, aspartate and homocysteine. Glycine is synthesized primarily from serine by
serine trans-hydroxymethylase and glycerate dehydrogenase, both of which are rate limiting.
Glycine acts as a mandatory adjunctive neurotransmitter for glutamate activity and an
independent inhibitory neurotransmitter at its own receptors.
The correct answer is: Glycine

017
Which of the following best fits GABA-A receptor?

Select one:

507
G protein coupled
Ligand dependent regulator of nuclear transcription
Metabotropic

Receptor with intrinsic enzyme activity


Ionotropic

GABA A receptors are Ionotropic and can mediate postsynaptic inhibition


The correct answer is: Ionotropic

018
Regarding excitatory amino acids which one of the following statements is incorrect?

Select one:
Excitatory amino acid neurotransmitters are essential in learning and
memory
Calcium entry in neurons is influenced by glutamate
Glutamate can be a potent neurotoxin when excessive
Ketamine is a selective agonist at NMDA receptors
Glutamate is the most abundant excitatory neurotransmitter

There are four main types of excitatory amino acid receptors; N-methyl D-aspartate (NMDA),
amino-3 hydroxy 5-methyl 4-isoxazole propionate (AMPA), kainate and Metabotropic (G-
protein coupled receptors). Ketamine is a non-competitive antagonist at NMDA receptors.
Glutamate is the major excitatory neurotransmitter with a wide distribution in the brain. There is
a possible link between the glutamate receptor activation and long-term potentiation in the
hippocampus as the physiological substrate of memory. Glutamate transmission has been
postulated to have a link with psychosis. Excess glutamate via NMDA mediated, calcium-
dependent excitotoxicity can result in neuronal damage
The correct answer is: Ketamine is a selective agonist at NMDA receptors

019
Which of the following best fits glucocorticoid receptor?

Select one:
Ionotropic

508
Ligand dependent regulators of nuclear transcription
Receptors with intrinsic enzyme activity
Metabotropic

G protein coupled

Glucocorticoids act via their binding to the intracellular GR (Glucocorticoid Receptor) which
translocates to the nucleus to modulate gene expression. The GR belongs to a nuclear
receptor superfamily of transcription factors.
The correct answer is: Ligand dependent regulators of nuclear transcription

020
The principal location of noradrenergic neuronal cell bodies in the CNS is

Select one:
Raphae nucleus

Nucleus basalis of meynert


Caudate and putamen nucleus
Locus coereleus
Hippocampus

The principal location of noradrenaline is the locus coereleus. The principal location of
acetylcholine is in the cell bodies in the nucleus basalis of Meynert with axons innervating the
hippocampus. The neurones containing 5-HT are located in the midbrain and brain stem median
raphe nuclei.
The correct answer is: Locus coereleus

021
The effects of magic mushroom are very similar to those of

Select one:
Cannabis
LSD
Cocaine
Methadone

509
Heroin

Many hallucinogens occur naturally in 'magic mushrooms', datura or 'angel's trumpet'


(atropine-like effects), mescaline and DMT (dimethyltryptamine). Magic mushrooms contain two
hallucinogenic substances: psilocin and psilocybin. Psilocybin gets converted to psilocin and
acts similar to LSD, although it is about 100 times less potent than the synthetic LSD itself. The
most popular hallucinogen in current use is the synthetic drug LSD.
The correct answer is: LSD

022
The predominant CNS metabolite of noradrenaline is

Select one:
5HIAA (Hydroxy Indole Acetic Acid)
COMT (Catechol-O-methyl transferase)
Vanillyl mendalic acid (VMA) Coenzyme
A
MHPG (3-methoxy,4-hydroxy phenyl glycol)

The CNS metabolite of noradrenaline is MHPG (3-methoxy,4-hydroxy phenyl glycol). Outside


the brain, the peripheral metabolite is principally VMA (vanillyl mendelic acid: this is why the
levels of VMA increases in peripheral noradrenaline producing tumours such as
phaeochromocytoma). Noradrenaline or its metabolites usually do not cross the BBB with the
exception of MHPG.
The correct answer is: MHPG (3-methoxy,4-hydroxy phenyl glycol)

023
Supraspinal analgesia related to opioid release is mediated by

Select one:
Mu receptors

Nicotine receptor subunits


Kappa receptors
Sigma receptors
Delta receptors

510
The general role of the endogenous opioids includes the regulation of pain (supraspinal and
spinal analgesia), anxiety, and memory.
The correct answer is: Mu receptors

024
Which of the following receptors is stimulated when tobacco is smoked?

Select one:
Adrenergic (alpha)
Nicotinic
Muscarinic
Adenosine
Endocannabinoid

Nicotine acts on nicotinic cholinergic receptors.


The correct answer is: Nicotinic

025
Which of the following type of glutamate receptors is crucial for the normal function of
inhibitory interneurons?

Select one:
AMPA
GABA-B

NMDA
Sigma
Kainate

Inhibitory interneurons comprise only about 20% of cortical neurons but have influential roles in
cortical maturation, function, and plasticity. NMDA receptors are predominantly located on
these GABA interneurons.

511
The correct answer is: NMDA

026
The neural mechanism of memory formation may include changes in physical properties of
neurons and synapses. One such change called LTP or Long Term Potentiation is mediated
by

Select one:
Cannabinoid receptor CB1
NMDA receptor
GABAA receptor
Adenosine A1
Serotonin 5HT2A

NMDAhas a role in memory acquisition, developmental plasticity, epilepsy, and ischemic


brain injury. NMDA receptor mediates long-term potentiation
The correct answer is: NMDA receptor

027
Which of the following is a rosette shaped receptor?

Select one:
NMDA receptors
Beta adrenergic receptors

Thyroid hormone receptors


Alpha adrenergic receptors
Serotonin 5HT2 receptors

The GABA-A receptor's structure is typical of most ligand-gated (ionotropic) receptors


['doughnut with a hole in the centre' or 'rosette' shaped]. It is made up of five protein subunits
arranged in a circular fashion. This channel opens up when GABA binds to the recognition site.
Each protein subunit is made of membrane-spanning aminoacid chains (each traversing the cell
membrane four times). A large N-terminal at the extracellular end mediates GABA- channel
interactions. In the middle of the string is a large intracellular loop of amino acids with four sites
where phosphorylation occurs. NMDA is also an ionotropic receptor with rosette shape.

The correct answer is: NMDA receptors

512
028
Glycine and d-serine both act as co-agonists at

Select one:
Dopamine receptors
NMDA receptors
GABA receptors
Adrenaline receptors
Acetylcholine receptors

Glycine and D-serine both act as co-agonists at NMDA receptors. The excitatory glycine site
on the NMDA receptor is called non-strychnine-sensitive glycine receptor.
The correct answer is: NMDA receptors

029
Which of the following regions in the brain is predominantly cholinergic?

Select one:
Dorsal raphe
Median raphe

Substantia nigra
Locus coeruleus
Nucleus basalis of Meynert

The ascending system of cholinergic neurons originating in the reticular formation and the
cholinergic cells in the nucleus basalis of Meynert are the major locations of cholinergic
neurons in the brain.
The correct answer is: Nucleus basalis of Meynert

030
Which of the following enzymes mediate several catalytic reactions involving aminoacids that
synthesize neurotransmtters such as dopamine, serotonin and tryptamine?

Select one:

513
Hydrolase
Reductase
Oxidase

Decarboxylase
Hydroxylase

Monoamine Oxidases are a family of enzymes that catalyze the oxidation of monoamines.
They are present on the outer membrane of mitochondria.
The correct answer is: Oxidase

031
Which of the following best describes D2 receptor?

Select one:
Presynaptic inhibitory receptor
Intranuclear receptor
Hetero-receptor
Auto-inducing receptor
Rosette shaped

Most neurotransmitters inhibit their own release through auto- receptors. Dopamine inhibits its
own release through D2 autoreceptors.
The correct answer is: Presynaptic inhibitory receptor

032
Which of the following are carrot-shaped eosinophilic inclusions seen in hematoxylin and
eosinophilic stains?

Select one:
Rosenthal fibres
Pick cells
Lewy bodies

Hirano bodies

514
Tau proteins

Rosenthal fibers appear on H&E stained sections as brightly eosinophilic carrot-shaped or


corkscrew-like structures. Rosenthal fibres are alterations in astrocytic processes and provide a
diagnostic signature of Alexander's leukodystrophy. These distinctive structures are present in
association with longstanding gliosis as is seen around cavities in the CNS, in low grade,
discrete astrocytomas such as pilocytic astrocytomas, and in patients suffering from
Alexander's disease. Hirano bodies are oval to elongated rod-shaped, eosinophilic inclusions,
which are few in normal elderly and high in people with Alzheimer's disease.
The correct answer is: Rosenthal fibres

033
The most abundant neurotransmitter in the median raphe nuclei is

Select one:
Noradrenaline
GABA
Acetylcholine
Glycine

Serotonin

Serotonin transmission from the median raphe nuclei provides rich projections to the frontal
cortex
The correct answer is: Serotonin

034
Tryptophan is the precursor of which of the following neurotransmitters?

Select one:
Noradrenalin

Dopamine
Adrenalin
Serotonin
Acetylcholine

515
Tryptophan ? 5 hydroxy l-tryptophan ? serotonin
The correct answer is: Serotonin

035
Which of the following neurotransmitters are not correctly matched with their precursors?

Select one:
Adrenaline-Noradrenaline
Dopamine-Tyrosine
Serotonin-Histidine
Noradrenaline-Dopamine
The Noradrenaline neurones originate in the brain stem, from a series of
nuclei including the locus cereleus

The biosynthesis of catecholamines starts from the conversion of tyrosine into L-


dihydroxyphenylalanine (L-DOPA) by tyrosine hydroxylase. L-DOPA is then decarboxylated by
DOPA decarboxylase to form dopamine. The conversion of dopamine to norepinephrine comes
about by the action of the enzyme dopamine ß-hydroxylase. The rate-limiting enzyme in the
biosynthesis of serotonin is tryptophan hydroxylase. Here, tryptophan is converted to 5-
hydroxytryptophan by tryptophan hydroxylase and by 5-hydroxytryptophan decarboxylase into
serotonin. Histidine is the precursor of histamine not serotonin.
The correct answer is: Serotonin-Histidine

036
Where do phospholipids get produced in a cell?

Select one:
Ribosomes
Lysosomes

Smooth endoplasmic reticulum


Nucleus
Rough Endoplasmic reticulum

516
Throughout the human body, especially in those cells that produce hormones and other
secretory products, a vast network of membrane-bound vesicles and tubules called the
endoplasmic reticulum, or ER is present. Smooth ER plays different functions depending on
the specific cell type including lipid (incl.phospholipid) and steroid hormone synthesis,
breakdown of lipid-soluble toxins in liver cells, and control of calcium release in muscle cell
contraction.
The correct answer is: Smooth endoplasmic reticulum

037
The enzyme involved in catabolism of GABA is

Select one:
Transferase
Transaminase
Decarboxylase
Oxidase
Hydroxylase

The breakdown enzyme involved in GABA metabolism is GABA transaminase.


The correct answer is: Transaminase

038
A 44-year-old woman is on antipsychotic depot injections. She develops a white discharge from
her breasts. Hyperprolactinemia is noted. Which of the following dopamine pathways has a role
in prolactin regulation?

Select one:
Mesolimbic
Incertohypothalamic

Mesocortical
Nigrostriatal
Tubero infundibular

Dopaminergic pathways can be classified as 1. Long: Nigrostriatal (movement), mesocortical


and mesolimbic. 2. Short: Tuberoinfundibular (prolactin inhibiting), incertohypothalamic 3.
Ultrashort: amacrine cells in the retina, olfactory system.

517
The correct answer is: Tubero infundibular

518
001
The macroscopic changes seen in Alzheimer's disease includes

Select one:
Senile plaques
Neuronal loss
Ventricular enlargement
Vacuolisation
Neurofibrillary tangles

The macroscopic changes in Alzheimers disease would include Enlargement of the lateral
and third ventricles, global brain atrophy, reduction in brain weight and sulcal widening.
Neuronal loss, senile plaques, neurofibrillary tangles, reactive astrocytosis and shrinking of
dendritic branching are the microscopic changes seen in the cerebral cortex in Alzheimers
disease. Cerebral atrophy and ventricular enlargement are macroscopic changes.
The correct answer is: Ventricular enlargement

002
Lewy bodies are eosinophilic, haloed, intra neuronal inclusions. They contain which of the
following materials?

Select one:
Glial tissue
Paired helical filaments

Dopamine metabolites

519
Tau protein
Amyloid substance

Lewy bodies contain Tau protein, Ubiquitin, protein neurofilaments, granular material, dense
core vesicles and microtubule assembly protein. In Lewy body dementia, the density of Lewy
bodies is much higher in the cingulate gyrus, parahippocampal gyrus and temporal cortex.
The correct answer is: Tau protein

003
The enzyme involved in the metabolism of amyloid precursor protein that prevents amyloid
formation is

Select one:
Protein kinase
Delta-secretase
Gamma-secretase
Alpha-secretase
Beta-secretase

Alpha secretases are a family of proteolytic enzymes that cleave amyloid precursor protein in
its transmembrane region. In conditions like Alzheimer's disease, when amyloid precursor
protein is processed by beat-secretase and gamma secretase, it gives rise to beta amyloid
peptide, which plays a crucial role in the pathogenesis of Alzheimer's dementia.
The correct answer is: Alpha-secretase

004
Hirano bodies are seen in brain autopsy of a patient who had a history of cognitive
impairment. Which of the following diagnosis is most likely?

Select one:
Vascular dementia
Wilson disease
Crutzfeld Jakob disease
Alzheimer's dementia

Lewy body dementia

520
Hirano bodies are rod-shaped eosinophilic bodies in the cytoplasm of neurons that may be
set free in the extracellular space if the neuron dies. Hirano bodies seen in Alzheimer's are
intracellular aggregates of actin and actin-associated proteins. They are frequently seen in
hippocampal pyramidal cells
The correct answer is: Alzheimer's dementia

005
The immunological staining used in detection of Pick's disease is

Select one:
Glycosaminoglycan.
Amyloid P
Basic fibroblast growth factor antibodies
Anti-tau antibodies
Heparan sulfate

A variety of stains can be used to visualise Pick bodies and Pick cells, but the most efficient
and highly specific method is to use immunohistochemical staining with anti-tau and anti-
ubiquitin antibodies. The neurofibrillary tangles of Alzheimer's can be stained with antibodies to
basic fibroblast growth factor, amyloid P, and heparan sulfate glycosaminoglycan.
The correct answer is: Anti-tau antibodies

006
Which one of the following is a protective factor against Alzheimer's disease?

Select one:
Down's syndrome
Apolipoprotein e2 allele

Head injury
Post menopausal estrogen decline
Age

521
Apolipoprotein e2 allele is a proven protective factor against Alzheimer's disease. Other
possible protective factors are smoking, NSAIDs, oestrogen, premorbid intelligence and
education.
The correct answer is: Apolipoprotein e2 allele

007
Which of the following is correct concerning white matter hyperintensities seen in mood
disorders?

Select one:
Best seen in T1 images of MRI
Seen only in bipolar patients
Associated with good recovery from mood episodes
Associated with vascular risk factors
Seen more often in younger patients

A strong association between mood disorder and the number and severity of focal signal
hyperintensities on T2-weighted images has been established. These white matter
hyperintensities (WMH) occur particularly in the deep subcortical white matter and to a lesser
extent in the basal ganglia and periventricularly. They are seen in excess in bipolar and
unipolar mood disorder, with an odds ratio of 3 to 7. In major depression, WMH are particularly
common in elderly subjects, where they are linked to risk factors for, and the presence of,
vascular disease (Harrison, P.The neuropathology of primary mood disorder | Brain,
http://brain.oxfordjournals.org/content/125/7/1428).
The correct answer is: Associated with vascular risk factors

008
The pathological factor that correlates most with cognitive decline in Alzheimer's disease is

Select one:
Burden of diffuse plaques
Burden of neurofibrillary tangles
Burden of neuritic plaques
Burden of gliosis
Burden of vascular amyloid load

522
A staging scheme devised by Braak and Braak (1995) is widely used to describe the extent of
tangle related abnormalities (distribution from entorhinal cortex to isocortex) in AD and
correlates well with the severity of dementia. Stages V-VI operationally define AD.
The correct answer is: Burden of neurofibrillary tangles

009
Selective cerebellar atrophy as a neuropathological change is likely to be seen in which of the
following dementias?

Select one:
Lewy body dementia
Creutzfeldt-Jakob Disease
Alzheimer's dementia
Normal Pressure Hydrocephalus
Parkinson's disease dementia

In Creutzfeldt - Jakob disease, macroscopic changes would include generalised cerebral


atrophy, sometimes a selective cerebellar atrophy and ventricular enlargement. Histological
changes would include status spongiosus, neuronal degeneration without inflammation and
astrocytic proliferation. There is a nerve cell loss, gliosis and spongiform changes.
The correct answer is: Creutzfeldt-Jakob Disease

010
A 55-year-old lady is a known alcoholic. She drinks in binges and presents with deficits in
new learning though she could retain messages long enough to hold a conversation. The
most prominent pathology in this patient will be seen in

Select one:
Hippocampus
Dorsolateral prefrontal cortex

Hypothalamus
Dorsal medial thalamus
Cerebellum

523
Wernicke's encephalopathy is characterized by degenerative changes including gliosis and
small hemorrhages in structures surrounding the third ventricle and aqueduct (i.e. the
mamillary bodies, hypothalamus, mediodorsal thalamic nucleus, colliculi, and midbrain
tegmentum), as well as cerebellar atrophy.
The correct answer is: Dorsal medial thalamus

011
Which of the following best describes the difference between CJD and vCJD? The variant
CJD has

Select one:
Earlier onset of ataxia
Negative tonsillar biopsy
Absence of florid plaques on autopsy
Earlier age of onset
Typical EEG changes

In 1996, a new type of CJD called vCJD (variant CJD) was reported. Variant CJD appears to
affect younger people than the other forms of the disease, with an average age of death of 29
years. Death occurs on an average within 14 months.
The correct answer is: Earlier age of onset

012
A 33-year-old man presents with anxiety and depression. Soon he develops rapidly
progressive cognitive impairment with myoclonus, ataxia and eventually akinetic mutism.
Characteristic pathological change expected in brain autopsy, in this case, is

Select one:
Lewy bodies
Extensive vacuole formation

Ventricular dilatation
Lack of gliosis
Eosinophilic extraneuronal deposits

The description suggests CJD. Microscopically CJD shows a spongiform encephalopathy


secondary to neuropil vacuolisation.

524
The correct answer is: Extensive vacuole formation

013
Neurofibrillary tangles are most commonly present in which of the following locations in
Alzheimer's?

Select one:
Brain stem
Auditory association cortex
Hippocampus
Prefrontal cortex
Insula

Neurofibrillary tangles occur with ageing and in all the other conditions listed above. NFTs are
found in the hippocampus (Ammons horn) and are also found in the Amygdala,
parahippocampal gyrus, neocortex, locus coereleus, and nucleus of meynert and Raphe nuclei.
Senile Plaques are found in the same sites as those of NFTs. Plaques include extra cellular
Amyloid as a component, and beta A-4 protein is a major biochemical component of plaque
Amyloid.
The correct answer is: Hippocampus

014
Which of the following is a true statement concerning human HIV disease?

Select one:
Psychosis is the most common HIV related psychiatric problem.
CSF viral load is the best predictor of HIV related CNS disease.

HIV cross the blood brain barrier using macrophages


CD4 count is the better indicator for severity than viral load

HIV does not cause programmed cell death of neurons

In order to enter the brain, HIV-1 must cross the BBB using mechanisms that remain unclear.
The generally accepted model is the 'Trojan Horse hypothesis'. HIV enters the CNS as a
passenger in cells trafficking to the brain via CD4 T cells or monocytes (Retrovirology: HIV-1
associated dementia. Retrieved from http://www.retrovirology.com/content/3/1/28).
The correct answer is: HIV cross the blood brain barrier using macrophages

525
015
You have noticed some degree of cognitive decline and weight loss in an intravenous drug
user. The most probable diagnosis is

Select one:
Neurosyphilis
Alzheimer's dementia
CADASIL

Binswanger's disease
HIV dementia

One of the common sources of acquiring HIV infection is intravenous drug use. Subcortical
dementia is common in patients with HIV in late stages of their illness
The correct answer is: HIV dementia

016
Marked atrophy of the caudate nucleus is seen in which of the following disorders?

Select one:
Huntington's disease
Punch drunk syndrome
Alcoholic dementia
Creutzfeldt-Jakob Disease
Lewy Body dementia

Macroscopic findings in Huntington's disease include 1. Small brain with reduced mass 2.
Marked atrophy of the corpus striatum, particularly the caudate nucleus 3. Marked atrophy of
the cerebral cortex, particularly the frontal lobe gyri 4. Dilatation of the lateral and third
ventricles. Histological changes in Huntington's disease would include neuronal loss in the
cerebral cortex and neuronal loss in the corpus striatum and astrocytosis in the affected
regions.
The correct answer is: Huntington's disease

017
Neurofibrillary tangles seen in Alzheimer's disease are made up of

526
Select one:
Hypophosphorylated tau protein
Amyloid fibrils
Actin filaments

Synaptophysin
Hyperphosphorylated tau protein

Neurofibrillary tangles are composed of cytoskeletal elements, primarily abnormally


hyperphosphorylated tau protein. Tau is a peptide required for microtubule assembly.
Microtubules are essential to transport of materials down axons. Hyperphosphorylation of Tau
proteins can result in the self-assembly of tangles of paired helical filaments and straight
filaments, which are involved in the pathogenesis of Alzheimer's disease.
The correct answer is: Hyperphosphorylated tau protein

018
A 10 year old child in a special school has challenging behaviour associated with aggression
and stereotyped motor activity. Which of the following seen in MRI is associated with autism?

Select one:
Cerebral atrophy
Hypoplastic cerebellum
Medial temporal atrophy
Caudate atrophy
White matter hyperintensities

Hypoplasia of cerebellar vermis and to some extent the cerebellar hemispheres is


documented.
The correct answer is: Hypoplastic cerebellum

019
In those with Mild Cognitive Impairment, which of the following biomarkers can detect
worsening cognitive function?

Select one:
Decreased tau protein in CSF

527
Increased Tau-to-Amyloid ratio in CSF
Decreased Ubiquitin-to-Amyloid ratio in CSF
Decreased Tau-to-Amyloid ratio in CSF

Increased beta amyloid in CSF

Individuals with early Alzheimer's dementia have reduced CSF beta-amyloid and increased
levels of CSF tau and phosphorylated tau. The CSF tau/amyloid beta ratio is nearly 5 times
higher in those who later convert to full-blown Alzheimer's dementia compared to non-
converters. Increased CSF tau/A-beta (amyloid) ratios show strong promise as a preclinical
biomarker to predict future dementia in cognitively normal older adults. Fagan et al., Arch
Neurol. 2007 Mar;64(3):343­9.
The correct answer is: Increased Tau-to-Amyloid ratio in CSF

020
Which of the following is correct with respect to planum temporale?

Select one:
It is present only on dominant cerebral hemisphere

It is normally larger on left side


It specialises in visual processing
In schizophrenia left More than right asymmetry is noted

It is situated in anterior temporal surface

The planum temporale, the posterior superior surface of the superior temporal gyrus, is a
highly lateralized brain structure involved with language. In schizophrenic patients, a
consistent reversal of the normal left-larger-than- right asymmetry of planum temporale
surface area is noted ( Planum temporale asymmetry reversal in schizophrenia. Retrieved
from http://www.ncbi.nlm.nih.gov/pubmed/9137122).
The correct answer is: It is normally larger on left side

021
Which of the following is a feature more likely to be seen in Pick's disease than other
dementias?

Select one:
Reactive astrocytosis

528
Gliosis
Tau proteins
Ventricular shrinkage

Knife blade gyri

Pick's disease is associated with selective asymmetrical atrophy of the frontal and anterior
temporal lobes. Patients also show knife blade gyri and ventricular enlargement. The
histological changes in Picks disease would include Pick's bodies, neuronal loss and reactive
astrocytosis, and these changes may be seen in the cerebral cortex, substantia nigra, locus
coereleus and basal ganglia. Pick bodies are round argyrophilic intraneuronal inclusions and
pick cells are swollen cortical pyramidal cells.
The correct answer is: Knife blade gyri

022
Alpha-synuclein is a major constituent of which of the following?

Select one:
Neurofibrillary tangles

Pick's bodies
Prion particles
Neuritic plaques
Lewy bodies

Lewy bodies are weakly eosinophilic, spherical, cytoplasmic inclusions. Lewy bodies in
Parkinson's disease and DLB contain accumulations of alpha-synuclein
The correct answer is: Lewy bodies

023
Autopsy of an elderly man who suffered from marked tremors and a progressive cognitive
decline shows cells negative for tau protein. What is the likely diagnosis?

Select one:
Lewy body dementia
Vascular dementia
Alzheimer's dementia

529
CJD
Frontal lobe degeneration

Lewy bodies, especially those that are seen in diffuse Lewy body disease (dementia), has tau
protein deposits.
The correct answer is: Lewy body dementia

024
A 40-year-old woman was found wandering on streets after the recent funeral of her husband.
She can give her personal demographic details and has no past psychiatric history. She has no
physical injuries. Which of the following deficits is likely? (June 2008)

Select one:
Loss of semantic memory
Loss of new learning capacity
Continuous anterograde amnesia
Loss of procedural memory
Loss of memory of personal events well rehearsed

This is characteristic of dissociative amnesiWell rehearsed personal events are lost (such as
the fact that her husband was ill before death, for example).
The correct answer is: Loss of memory of personal events well rehearsed

025
Arteriosclerotic changes in major arteries are most commonly found in which of the following
types of dementia?

Select one:
Multi-infarct dementia
Lewy body dementia
Alzheimer's dementia
Parkinson's disease dementia
Normal Pressure Hydrocephalus

530
In multi-infarct dementia, brain damage due to multiple cerebral infarcts leads to local or
general brain atrophy and ventricular enlargement. The presence of arteriosclerotic changes is
commonly noted.
The correct answer is: Multi-infarct dementia

026
Which disease can be studied by inducing 'autoimmune encephalomyelitis'?

Select one:
Acquired brain injury
Parkinson's disease
Huntington's disease
Multiple sclerosis
Alzheimer's disease

Experimental autoimmune encephalomyelitis (EAE) is a laboratory model used to study the


efficacy of potential agents treating multiple sclerosis (MS). EAE is also used to investigate the
pathogenesis of autoimmunity, CNS inflammation and demyelination.
The correct answer is: Multiple sclerosis

027
The neuropathological findings in Punch Drunk syndrome include which of the following
changes?

Select one:
Knife blade gyri
Basal ganglia enlargement

Neurofibrillary tangles
Caudate hypertrophy
Pulvinar signs

531
Dementia pugilistica (DP) is a type of neurodegenerative disease that can affect around 15% of
boxers who suffer concussions after 12-16 years of boxing on average. Symptoms and signs of
DP develop progressively over a long latent period.Histological changes in Punch Drunk
syndrome (Dementia Pugilistica) would include neuronal loss and neurofibrillary tangles.
Thinning of the corpus callosum, perforation of the septum pellucidum and ventricular
enlargement is also seen in this condition.
The correct answer is: Neurofibrillary tangles

028
The spongiform appearance of brain tissue in CJD is due to

Select one:
Apoptosis of neuronal cells
Glial proliferation
Demyelination
Neuropil vacuolation
Excessive fibrosis

Microscopically CJD shows a spongiform encephalopathy secondary to neuropil


vacuolisation.
The correct answer is: Neuropil vacuolation

029
Depigmentation of the substantia nigra is seen in which of the following disorders?

Select one:
Creutzfeldt-Jakob Disease
Huntington's disease
Dandy-Walker malformation
Alzheimers disease
Parkinson's disease

The macroscopic changes in idiopathic Parkinsons disease would include depigmentation of


the substantia nigra, particularly the zona compacta, and depigmentation of the locus coereleus
and diffuse cortical atrophy. The histological changes in idiopathic Parkinsons disease would
include Reactive astrocytosis, neuronal loss, Presence of Lewy bodies in

532
different areas such as the substantia nigra, locus cereleus, dorsal motor nucleus of the
vagus, hypothalamus, nucleus basalis of meynert, Edinger-Westphal nucleus and raphe
nuclei.
The correct answer is: Parkinson's disease

030
Knife blade atrophy is seen in

Select one:
Pick's disease
Alzheimer's disease
Lewy body dementia
CJD
Progressive supranuclear palsy

Knife-blade type atrophy refers to the marked atrophy of the frontal and temporal lobes with
relative sparing of parietal and occipital lobes in patients with Pick's disease.
The correct answer is: Pick's disease

031
In which of the following conditions both balloon cells and Hirano bodies are seen?

Select one:
Alzheimer's disease
HSE encephalitis

Neurosyphilis
CJD
Pick's disease

Hirano bodies are fusiform or spheroidal eosinophilic bodies, that are commonly observed in the
hippocampi of the elderly and are especially numerous in patients with various dementias or
degenerative diseases (Ultrastructure of Hirano bodies - Springer. Retrieved from
http://link.springer.com/article/10.1007%2FBF00687999). Both Hirano bodies and Balloon cells
are seen together in Pick's disease.
The correct answer is: Pick's disease

533
032
Which of the following structures is larger on the left side compared to the right hemisphere in
healthy right handed subjects?

Select one:
Cingulate cortex
Planum temporale
Superior colliculus

Thalamus
Amygdala

The planum temporale shows a marked leftward volume asymmetry that is related to the
degree of right­handedness; this is more marked in males (nearly ten times asymmetrical on
the left in males)
The correct answer is: Planum temporale

534
Home HiYield Paper Neurosciences

001
Which one of the following exists as CNS neurotransmitter as well as a hormone?

Select one:
Glucagon
Serotonin
Vasopressin

Dopamine
Thyroxine

Neuropeptides (e.g., endorphins, enkephalins, vasopressin) exist as neurotransmitters and as


hormones. By clinging to the nerve cell membrane for varying periods of time, they can
modulate the flow of information. Thus, they are referred to as neuromodulators.
The correct answer is: Vasopressin

002
EEG changes in Angelman's syndrome are notable by the age of

Select one:
Soon after birth
Two years
Puberty
7 years
21 years

535
In Angelman syndrome, EEG changes are notable by the age of 2. Prolonged runs of high
amplitude 2-3 Hz frontal activity with superimposed interictal epileptiform discharges are seen in
all ages More than2. Occipital high amplitude rhythmic 4-6 Hz activity facilitated by eye closure
is seen under the age of 12 years. There is no difference in EEG findings in AS patients with or
without seizures (Angelman syndrome: is there a characteristic EEG?,
http://www.ncbi.nlm.nih.gov/pubmed/15668045 (accessed April 13, 2015)).
The correct answer is: Two years

003
Peak cortisol level in normal physiological states is seen at

Select one:
0.583333333333333
0.75
8.33333333333333E-02
0.25
0.416666666666667

A diurnal variation in ACTH and cortisol levels occurs in humans, with peak cortisol levels
occurring around 6:00-7:00 AM.
The correct answer is: 0.25

004
The resting membrane potential of a neuron is around

Select one:
170 mv
70 Mv

30 Mv

-30 mv

-70mV

536
There is a negative resting membrane potential of around 70 mv. It is maintained by the
sodium pump, which actively transports sodium ions out of the cell and potassium ions into
the cell. The ATP provides the energy.
The correct answer is: -70mV

005
The duration of normal sleep latency in healthy adults is

Select one:
15-20 minutes
30-40 minutes
0-5 minutes
90-100 minutes
50-60 minutes

Sleep latency is the time from lights out to sleep onset. A sleep onset latency of 15 to 20
minutes is indicative of "little or no" sleep debt. REM latency: Time from sleep onset to first
REM episode. Normally it is about 90 minutes in adults.
The correct answer is: 15-20 minutes

006
Infants have a dominant rhythm in which of the following EEG wave frequency?

Select one:
3 Hz
10 Hz

13 Hz
40 Hz
7 Hz

The infants have a dominant rhythm of 3 Hz, the dominant frequency increases with
maturation. In the newborn, the EEG is dominated by delta and theta waves.
The correct answer is: 3 Hz

537
007
What percentage of epileptic patients will have normal EEG between attacks?

Select one:
70-90%
1-2%
10-15%

30-50%
5-10%

The value of the EEG is that it can suggest an abnormal function in the presence of normal
structure. 10-15% of normal individuals show some abnormality of the EEG. 30-50% of
epileptic patients will have normal EEG between attacks.
The correct answer is: 30-50%

008
Regarding major depression, which of the following statements on neuroendocrine
abnormalities is correct?

Select one:
Large dose of cortisol exacerbates depressive symptoms
50% of patients show raised cortisol levels
Low levels of corticotrophin releasing factor is found in CSF of patients
Dehydroepinandrosterone levels are low
Atrophy of adrenal cortex is seen in 20% of patients

Many depressed patients have a high level of cortisol in plasma and urine. It is now known
that 50% of depressed patients exhibit hypercortisolemia.
The correct answer is: 50% of patients show raised cortisol levels

009
Peak cortisol level in normal individuals is noted around

Select one:
12 AM Mid night

538
9 AM after breakfast
6 AM in the morning
12 PM Mid day

6 PM in the evening

A diurnal variation in ACTH and cortisol levels occurs in humans, with peak cortisol levels
occurring around 6:00-7:00 AM.
The correct answer is: 6 AM in the morning

010
A 42-year-old man is a journalist. He goes to sleep at 10PM every night and wakes up at
530AM every morning. If he volunteers for a sleep study using polysomnogram the time from
sleep onset to first REM episode will be

Select one:
30 minutes

60 minutes
160 minutes
10 minutes
90 minutes

REM latency: Time from sleep onset to first REM episode. Normally it is about 90 minutes in
adults.
The correct answer is: 90 minutes

011
Regular 3 Hz complexes are seen in EEG of patients with which of the following conditions?

Select one:
ADHD
CJD
Antisocial personality disorder
Angleman's syndrome

Absence seizures

539
Regular 3 Hz complexes are commonly seen in EEG of patients with Absence seizures (Petit-
mal type of epilepsy)
The correct answer is: Absence seizures

012
A patient is administered 1g of dexamethasone at 11PM, and his plasma samples are
collected to measure hormone levels on the following morning. Which of the following is
correct concerning the physiological changes overnight?

Select one:
ACTH levels are suppressed
CRH levels are increased
Glucocorticoid receptors are blocked
Prolactin levels are increased
Cortisol levels are increased

The dexamethasone suppression test (DST) is used to demonstrate a failure of feedback


suppression of ACTH/CRH and continuous production of endogenous cortisol despite
administration of exogenous steroid (dexamethasone). Normally administering
dexamethasone must reduce cortisol in plasma. This is because of intact HPA function
leading to reduced ACTH and CRH. In depression and other psychiatric hypercortisolemic
states (also in organic hypercortisolemic states such as Cushing's), this does not occur.
The correct answer is: ACTH levels are suppressed

013
Mr. Y presents to the emergency department with a history of apathy and weight loss. On
further investigations, he has hyponatraemia and hyperkalemia. What is his possible
diagnosis?

Select one:
Acute porphyria
Cushing's syndrome

Cohn's syndrome
Diabetes insipidus
Addison's disease

540
The most common symptoms of Addison's disease are fatigue, lightheadedness, muscle
weakness, apathy, fever, weight loss, anxiety, nausea, vomiting, diarrhea, headache, sweating,
changes in mood and personality, joint and muscle pains. Some have marked cravings for salt
or salty foods due to hyponatraemia. Affected individuals may note increased tanning since
adrenal insufficiency is manifested in the skin primarily by hyperpigmentation. Other features
include hypoglycemia, low blood pressure, hyperkalemia and hypercalcemia
The correct answer is: Addison's disease

014
Which type of EEG trace is expected when eyes are closed during relaxation?

Select one:
Gamma
Beta
Theta
Alpha
Delta

Alpha waves are predominant when eyes are closed, but the patient is not sleeping.
The correct answer is: Alpha

015
Which one of the following is characteristic of REM sleep?

Select one:
Atonia
Reduced recall of dreaming if awoken

Decreased heart rate


Abolition of tendon reflexes
Upward ocular deviation with few or no movements

541
Features of REM sleep include Increased recall of dream if awoken, Increased sympathetic
activity, Increased heart rate, systolic blood pressure, respiratory rate & cerebral blood flow,
Penile erection or vaginal blood flow, Increased protein synthesis, Occasional myoclonic
jerks, Maximal loss of muscle tone and transient runs of conjugate ocular movements.
Nightmares occur in REM sleep - hence they are well recollected
The correct answer is: Atonia

016
Which one of the following classes of drugs increases beta activity?

Select one:
Lithium
Anticonvulsants
Benzodiazepines
Antipsychotics
Antidepressants

Antipsychotics and antidepressants produce slowing of beta activity with an increase in theta
and delta activity. Alcohol, Barbiturates and Benzodiazepines increases beta activity, frequently
mixed with low-amplitude theta activity. Lithium may produce some slowing of the alpha rhythm.
Anticonvulsants, analgesics, beta blockers and narcotics have little effect on the EEG

The correct answer is: Benzodiazepines

017
Which one of the following ions is freely permeable across cell membranes?

Select one:
Chloride ions
All of the listed options

Sodium ions
Potassium ions
Organic ions

Potassium ions-Relatively permeable Sodium ions-relatively impermeable Chloride ions-


freely permeable Organic ions-relatively impermeable

542
The correct answer is: Chloride ions

018
Which of the following receptor on stimulation leads to sleep onset?

Select one:
Noradrenergic - receptors
5HT2 receptors

Glucocorticoid receptors

Histaminergic receptors

Cholinergic receptors

The noradrenergic rapid eye movement (REM)-OFF neurons in locus coeruleus and
cholinergic REM-ON neurons in tegmentum show a reciprocal firing pattern. The REM-ON
neurons fire during REM sleep whereas REM-OFF neurons stop firing during REM sleep.
http://www.ncbi.nlm.nih.gov/pubmed/17704548
The correct answer is: Cholinergic receptors

019
1-2 Hz shaped wave pattern is seen in

Select one:
Hepatic encephalopathy
Huntington's disease

Alzheimer's disease
CJD
Multi-infarct dementia

The classic EEG finding in CJD is generalized 1-2 Hz bi- or triphasic sharp- and slow-wave
complexes. These may not appear until very late in the clinical course, if at all, and may require
repeated testing. The earliest EEG finding is more often rhythmic generalized slowing.
The correct answer is: CJD

020

543
A 3 months old newborn undergoes awake EEG investigation to detect seizure activity, but no
abnormalities were found. Which of the following waveform will be prominent in the recording?

Select one:
Delta and theta
Spindle waves
Alpha and beta

Mu and lambda
K complexes

Newborns have dominant delta and theta waves. Infants have irregular medium- to high-
voltage delta activity in the awake tracing of the infant; alpha range develops in posterior
areas by early childhood; by mid­adolescence EEG essentially has the appearance of an
adult tracing. The normal dominant alpha rhythm is usually achieved by 12-14 years old
The correct answer is: Delta and theta

021
The levels of ACTH and CRF is increased and plays an important role in the
neuroendocrinology of;

Select one:
Autism
Dementia
Schizophrenia
Depression
Biploar disorder

The dexamethasone suppression test (DST) is used to demonstrate a failure of feedback


suppression of ACTH/CRH and continuous production of endogenous cortisol despite
administration of exogenous steroid (dexamethasone). Normally administering
dexamethasone must reduce cortisol in plasma. This is because of intact HPA function
leading to reduced ACTH and CRH. In depression and other psychiatric hypercortisolemic
states (also in organic hypercortisolemic states such as Cushing's), this does not occur, and
ACTH/CRF levels are raised.
The correct answer is: Depression

544
022
Which of the following associated with neuroendocrine changes during sleep is correct?

Select one:
Testosterone decreases when sleep starts
GH decreases when sleeping
Cortisol shoots up in slow wave sleep

During REM sleep melatonin decreases


Prolactin decreases in early morning sleep

Melatonin regulates circadian rhythms. It has both synchronizing and phase-shifting


properties in the regulation of biological rhythms. It reduces during REM sleep.
The correct answer is: During REM sleep melatonin decreases

023
Generalised spike-wave discharges with decreased waves on photic stimulation are seen in
EEG of patients with;

Select one:
Myoclonic spilepsy
Absence seizures
Generalised seizures
CJD
Angelmann syndrome

A decreased wave on photic stimulation followed by generalised spike and wave discharges is
seen in patients with generalised tonic-clonic seizures.
The correct answer is: Generalised seizures

024
Which of the following disorders is characterised by a predominant presence of triphasic
waves in EEG?

Select one:
Head injury

545
Depressive pseudodemntia
Herpes simplex encephalitis
Dementia of Alzheimer's type

Hepatic encephalopathy

Triphasic waves (1.5 to 3.0 per second high-voltage slow-waves) are a distinctive but
nonspecific electroencephalographic (EEG) pattern originally described in a stuporous patient as
'blunted spike and wave.' Since their findings were limited to patients with hepatic failure,
triphasic wave encephalopathy (TWE) became synonymous with hepatic encephalopathy. Since
then, TWE has been associated with a wide range of toxic, metabolic, and structural
abnormalities (Excerpt from EEG Triphasic Waves - Medscape Reference,
http://emedicine.medscape.com/article/1139819-overview (accessed April 13, 2015).)
The correct answer is: Hepatic encephalopathy

025
Flattened trace of EEG is a feature seen in

Select one:
Petitmal epilepsy

Creutzfeldt Jacob disease


Delirium
Partial seizures
Huntington's disease

A significant reduction in voltage with a predominant absence of alpha rhythms, sometimes


leading to flat traces, is seen in Huntington's disease.
The correct answer is: Huntington's disease

026
Which one of the following is not a characteristic feature of Kleine-Levin syndrome?

Select one:
Episodic nature
Hypersomnia
Male predominance

546
Hyper sexuality
Hyperphagia

Kleine-Levin syndrome affects males predominantly, the sufferers are usually under the age of
25, patient sleeps for 20 hours or more per day during an episode and it can last for several
days. Hypersomnia and Hyperphagia are the important characteristic features.
The correct answer is: Hyper sexuality

027
REM sleep includes which of the following physiological features?

Select one:
Abolition of tendon reflexes
Increased parasympathetic activity
Reduced heart rate

Increased recall of dream if awoken


Reduced cerebral blood flow

Features of REM sleep: 1. Increased recall of dream if awoken 2. Increased sympathetic activity
3. Increased heart rate, systolic blood pressure, respiratory rate, cerebral blood flow 4. Penile
erection or vaginal blood flow 5. Increased protein synthesis 6. Occasional myoclonic jerks 7.
Maximal loss of muscle tone 8. Transient runs of conjugate ocular movements. Features of
non-REM sleep 1. Reduced recall of dreaming if awoken 2. Increased Parasympathetic activity
3. Decreased heart rate, systolic blood pressure, respiratory rate, cerebral blood flow 4.
abolition of tendon reflexes 5. An upward ocular deviation with few or no movements.

The correct answer is: Increased recall of dream if awoken

028
Which of the following brain regions when stimulated via magnetic pulse reduces
depression?

Select one:
Right temporal
Left prefrontal
Right occipital

547
Left temporal
Right parietal

Magnetic stimulation may have therapeutic effects in affective disorders similar to ECT. Few
studies have reported a beneficial effect of left prefrontal rTMS in severe depression.
The correct answer is: Left prefrontal

Question 29 029
Not answered The delta waves have a frequency of
Marked out of 1.00
Select one:
Flag question
13-40 Hz
8-13 Hz
3.5-7.5 Hz

7-11 Hz
Less than 4 Hz

EEG-Different wave forms are classified by frequency: Alpha-8-13Hz Beta 13-40 Hz Theta 4-8
Hz Delta-less than 4Hz
The correct answer is: Less than 4 Hz

030
REM rebound on cessation is seen with the use of which of the following?

Select one:
Zolpidem
Modafinil
SSRIs
Methylphenidate
Inhalants

548
Stimulants reduce the total sleep time by decreasing both REM sleep and Slow Wave Sleep.
On cessation of stimulants (except modafinil), REM rebound occurs.
The correct answer is: Methylphenidate

031
The frequency of ß waves seen in normal EEG is

Select one:
8-13 Hz
More than 40Hz
4-8Hz
Less than 7 Hz
More than 13Hz

EEG-Different wave forms are classified by frequency: Alpha: 8-13Hz Beta: 13-40 Hz Theta: 4- 8
Hz Delta: less than 4Hz
The correct answer is: More than 13Hz

032
Which of the following statement concerning REM sleep is INCORRECT?

Select one:
EEG shows activity similar to awake state
Rapid eye movements are noted

Cardiac activity is similar to awake state


Muscle tone is similar to awake state
Penile erection occurs

Criteria for REM sleep includes not only rapid eye movement, but also low muscle tone and an
EEG pattern that is for most part similar to awake stage or stage 1 sleep but with intermittent
appearance of rapid, low-voltage EEG (saw tooth pattern) . (Ref: Vaughn & Bazil, Sleep
Disorders -Chapter 18: Merritt's Neurology)
The correct answer is: Muscle tone is similar to awake state

549
033
Orexin (hypocretin) neurons are dysfunctional in which of the following disorders?

Select one:
Obesity
Catatonia
Chorea

Anorexia
Narcolepsy

The VLPO- ventrolateral preoptic nuclei - induces sleep by putting the brakes on the arousal
nuclei. This switching is stabilised by orexin neurons (also called hypocretin) from the
hypothalamus. Patients with narcolepsy have few orexin neurons in the hypothalamus. Orexin
neurons are mainly active during wakefulness and reinforce the arousal system.
The correct answer is: Narcolepsy

034
Which one of the following statements is true about normal EEG?

Select one:
The normal dominant alpha rhythm is usually achieved by 8-10 years old
Closing the eyes usually abolishes the dominant alpha rhythm.
Ongoing delta activity when awake is almost always abnormal in the adult
Beta activity is almost always abnormal in the adult
The dominant alpha rhythm is seen in the Fronto-temporal region.

The normal dominant alpha rhythm is usually achieved by 12-14 years old. The dominant
alpha rhythm is seen posteriorly in the occipito-parietal region. Opening the eyes,
concentrating, arousal states and anxiety usually abolishes the dominant alpha rhythm.
Ongoing delta activity is almost always abnormal in the adult Some Beta activity is seen in
most adults, predominantly pre-central. Small amounts of theta activity are normal in adults.
The correct answer is: Ongoing delta activity when awake is almost always abnormal in the
adult

035
Which of the following is a posterior pituitary hormone?

550
Select one:
Melatonin
Gonadotrophin
Prolactin

Testosterone
Oxytocin

Vasopressin (ADH - antidiuretic hormone) and oxytocin are peptides differing from each other in
only two amino acids in their sequences. Both are synthesized in the supraoptic nuclei and
paraventricular nuclei of the hypothalamus. (posterior pituitary hormones)
The correct answer is: Oxytocin

036
At resting membrane potential which ion is greatest in concentration inside the cell?

Select one:
Chloride
Calcium
Sodium
Magnesium
Potassium

At resting membrane potential, there are more potassium ions inside and more sodium ions
outside the plasma membrane
The correct answer is: Potassium

037
Mrs. X is a patient with bipolar affective disorder with secondary amenorrhea. The
investigation to be done is

Select one:
Serum testosterone levels
FSH and LH levels
Thyroid levels

551
Pregnancy test
Serum prolactin levels

The most common cause of secondary amenorrhoea is pregnancy. Also, anticonvulsants


used in bipolar disorder can reduce the efficacy of oral contraceptives.
The correct answer is: Pregnancy test

038
Which of the following is associated with low levels of cortisol?

Select one:
PTSD
Depression
Anorexia

Psychosis
Mania

In PTSD hypocortisolemia is seen in a subgroup of patients; this may be due to aberrant


feedback to the pituitary due to excessive glucocorticoid receptors - probably a genetic
vulnerability.
The correct answer is: PTSD

039
Which of the following physiological changes are noted after ECT in clinically depressed
individuals?

Select one:
Reduced noradrenaline turnover
Beta waves predominance in EEG
Reduced ß receptors
Increased a2 receptors
Increase in 5HT2 receptors

552
After ECT treatment, EEG immediately shows delta and theta excess but returns to normal
levels after three months of the end of treatment. Reduced ß noradrenergic receptor and
increased noradrenaline turnover is also noted. Alpha 2 receptors are reduced in a change that
is similar to antidepressants. Brain 5-HT2 receptors were considered potential targets for
therapeutic efficacy of electroconvulsive therapy (ECT), but pre-clinical studies showed that
electroconvulsive shock up-regulates 5-HT2 receptors in contrast to antidepressant
medications, which down-regulate brain 5-HT2 receptors. Using an [18F]setoperone PET scan
Yatham and Liddle (2010) have now demonstrated that unlike in rodents, and similar to
antidepressants, ECT reduces brain 5-HT2 receptors in individuals with depression.
The correct answer is: Reduced ß receptors

040
Which of the following sleep EEG changes is seen when taking benzodiazepines for
insomnia?

Select one:
Increased slow wave sleep
Reduced stage 1 sleep
Reduced stage 2 sleep
Increased REM sleep
Reduced REM latency

Multiple sleep-related changes including a decrease in sleep latency, increased sleep time,
reduced stage 1 sleep, increased stage 2 sleep, reduced REM and Slow Wave Sleep, and
REM rebound on cessation are seen with benzodiazepines.
The correct answer is: Reduced stage 1 sleep

041
Which of the following stages of sleep predominates in a neonate?

Select one:
Stage 2 sleep
Stage 3 sleep

Stage 4 sleep
Stage 1 sleep
REM sleep

553
Newborns sleep about 16 hours a day. They spend More than50% of sleep time in REM
sleep. Sleep-onset REM is also seen in neonates.
The correct answer is: REM sleep

042
Regarding the neuronal resting membrane potential, which of the following is true?

Select one:
Hyperpolarise with moving out of chloride.
Depolarise with rapid moving out of sodium.
Repolarise with moving out of potassium
Repolarise with rapid cellular entry of sodium.
Depolarise with rapid cellular entry of potassium.

An action potential is initiated in the axon hillock when the synaptic signals received by the
dendrites and soma are sufficient to raise the intracellular potential from -70 mV to the threshold
potential of - 55mV. When this potential is reached, the Na+ channels present in the axon initial
segment will open. This Na+ influx causes a rapid reversal of the membrane potential from -90
to +40 mV. When the membrane potential reaches +40mV, the Na+ channels close and the
voltage-gated K+ channels open. K+ ions move out of the axon, and 'repolarizes' the
membrane.
The correct answer is: Repolarise with moving out of potassium

043
A 34-year-old patient who is taking sulpride undergoes EEG investigation for detecting drug-
related EEG changes. What would you expect to see?

Select one:
decrease in delta activity

new onset spike and waves

decrease in alpha activity

slowing of beta activity

decrease in theta activity

Slowing of beta activity with an increase in alpha, theta and delta activity is seen with
antipsychotics.

554
The correct answer is: slowing of beta activity

044
K complexes during sleep EEG are seen during

Select one:
Stage 3 sleep
Stage 4 sleep
Stage 2 sleep
Stage 1 sleep
REM sleep

The K-Complex is a phasic transient EEG activity that occurs around once every 2-3 minutes
during stage 2 non-REM sleep. In REM sleep, the K-Complex cannot be elicited even by an
external stimulus.
The correct answer is: Stage 2 sleep

045
There is a nocturnal surge in the level of growth hormones during which of the following
stages of sleep?

Select one:
After waking up
All stages of sleep

Stage 1 and 2 n-REM sleep


Stage 3 and 4 n-REM sleep
REM sleep

The growth hormone regulates carbohydrate and lipid metabolism. There is a nocturnal surge
during slow wave sleep. Its release is inhibited by somatostatin, and it inhibits the peripheral
production of the Insulin-like growth factor (IGF-1).
The correct answer is: Stage 3 and 4 n-REM sleep

046
Which stages of sleep are considered as slow wave sleep?

555
Select one:
Stage 1 and 2 n-REM sleep
Stage 3 and 4 n-REM sleep
Stage 0 and 1 n-REM sleep

REM sleep
Stage 2 and 3 n-REM sleep

REM sleep: The eyes undergo rapid movements, and there is a high level of brain activity.
Non-REM sleep: There is reduced neuronal activity. Stage 3 and 4 of NREM are grouped as
slow wave sleep because the slow delta waves predominate in these two stages.
The correct answer is: Stage 3 and 4 n-REM sleep

047
An EEG showing high amplitude repetitive bilaterally synchronous symmetrical polyphasic
sharp wave and slow wave complexes, which occur every 4-15 seconds are characteristic of

Select one:
Neurosyphilis
Creutzfeldt Jacob disease
Herpes simplex encephalitis
Sub acute sclerosing pan encephalitis
Absence seizures

CJD-Periodic stereotyped discharges in the EEG at a rate of about 1 per second Herpes
simplex encephalitis-episodic discharges recurring every 1-3 seconds with variable focal waves
over the temporal areas. Neurosyphilis- non-specific increase in slow waves occurring diffusely
over the scalp. Absence seizures-multiple spike and slow wave complexes, Regular 3 Hz
Complexes.
The correct answer is: Sub acute sclerosing pan encephalitis

048
The brain region that acts as monitor of circadian rhythm is

Select one:
Hippocampus

556
Suprachiasmatic nucleus
Locus ceruleus
Nucleus accumbens

Dorsal medial thalamus

The master clock of the brain is the suprachiasmatic nucleus (SCN) located in the anterior
hypothalamus - this orchestrates circadian rhythms and is synchronized by signals from the
retina.
The correct answer is: Suprachiasmatic nucleus

049
A 47-year-old lady presents with low mood and lethargy. She is noted to have a history of
untreated hypothyroidism. Which of the following is inconsistent with a diagnosis of
hypothyroidism?

Select one:
Increased TSH levels in blood

Cold intolerance
Decreased libido
Weight gain
Tachycardia

Features of hypothyroidism are: Fatigue, Depressed mood, Weight gain Decreased libido,
Cold intolerance, Psychomotor retardation, Dry skin and Poor memory.
The correct answer is: Tachycardia

050
The EEG wave with a frequency of 4 to 8 Hz is

Select one:
Theta
Delta
Alpha
Gamma

557
Beta

EEG-Different wave forms are classified by frequency: Alpha: 8-13Hz Beta: 13-40 Hz Theta: 4- 8
Hz Delta: less than 4Hz. Theta and delta are also called slow waves due to their low- frequency
band.
The correct answer is: Theta

051
Which of the following is an innovative treatment with some beneficial effect in dystonia?

Select one:
Vagal nerve stimulation
Transcranial magnetic stimulation (rTMS)
Leucotomy
Behavioural reversal therapy

Nerve ablation

Repetitive transcranial magnetic stimulation (rTMS) is being increasingly explored as a


therapeutic tool for depression, schizophrenia and movement disorders associated with
deficient inhibition throughout the central nervous system. There is some evidence for its use in
dystonia as well though the evidence is very preliminary compared to the convincing evidence
in the treatment of depression and hallucinations.
The correct answer is: Transcranial magnetic stimulation (rTMS)

558
001
A 79-year-old man with a history of progressive cognitive decline for the last year has difficulty in
remembering names but is good at day­to­day functioning. The most likely diagnosis is;

Select one:
CJD
Picks disease
Vascular dementia

Alzheimer's dementia
Binswanger's disease

In the early stages of Alzheimer's disease, people often present with cognitive difficulties such
as forgetting trivial details, names, faces, etc. but will show largely preserved functional abilities.

The correct answer is: Alzheimer's dementia

002
A 66-year-old patient is incidentally diagnosed to have a carotid bruit. Which of the following
can be expected if he develops a thromboembolism?

Select one:
Nystagmus
Amaurosis
Ataxia
Transient global amnesia

559
Diplopia

Amaurosis fugax is loss of vision in one eye due to a temporary lack of blood flow to the retina. It
may be a sign of an impending stroke.
The correct answer is: Amaurosis

003
Pauline was found to be wandering on the streets after the sudden death of her husband.
What type of amnesia is she most likely to exhibit?

Select one:
Post traumatic amnesia
Amnesia for personal information
Amnesia for childhood
Lacunar amnesia

Global amnesia

This is called as fugue state, a form of dissociative amnesia, which involves unplanned travel or
wandering, and is sometimes accompanied by the establishment of a new identity. The episodes
result from trauma or stress. It is a rare functional disorder characterized by profound but
reversible amnesia for personal identity, including changes in one's personality and other
identifying behavioural characteristics of an individual. The state is usually short-lived but in
exceptional cases, it can last for months or longer.
The correct answer is: Amnesia for personal information

004
Lesion of ventromedial frontal cortex is associated with

Select one:
Apathy

Disorganisation
Apraxia
Increased gambling
Memory problems

560
This results in pseudodepressive syndrome
The correct answer is: Apathy

005
On routine examination of eyes, accommodation reflexes were present but pupils were small
and irregular with an absence of reaction to light. This type of pupil is called

Select one:
Marcus Gunn pupil
Senile pupil
Anosognosia
Kayser-Fleischer pupil
Argyll Robertson pupils

Syphilitic or diabetic neural damage can lead to the obliteration of pupillary reaction when
stimulated by light thought the response to near / accommodation reflex is intact. This is called
ARP (Argyll Robertson Pupil) - it is frequently quite miotic with an irregular shape. The Argyll
Robertson pupil is also recalcitrant to pharmacologic dilation.
The correct answer is: Argyll Robertson pupils

006
Spelling the word 'WORLD' backwards tests which of the following?

Select one:
repetition
recall

attention
registration
orientation

Attention can be tested in a number of ways including serial 7s, digit span, spelling "world"
backwards, and recitation of the months of the year in reverse order.

561
The correct answer is: attention

007
Simultanagnosia is associated with which of the following syndromes?

Select one:
Balint's syndrome
Anton's syndrome
Ekbom's syndrome
Gerstmann syndrome
Marchiafava-Bignami disease

Balint's syndrome consists of a triad of simultanagnosia (inability to attend to more than one
item of a complex scene at a time), optic ataxia (inability to guide reaching or pointing despite
adequate vision), and oculomotor apraxia (inability to voluntarily direct saccades to a visual
target). Fields may be full when challenged with gross stimuli, and oculocephalic reflexes are
intact. This syndrome results from bilateral damage including the superior-parieto-occipital
region, which disrupts the dorsal ("where") visual processing stream linking visual with parietal
association areas. Possible causes include carbon monoxide poisoning, watershed infarction,
leucodystrophy, and the posterior cortical variant of Alzheimer's disease
The correct answer is: Balint's syndrome

008
Which one of the following is a test of frontal lobe function?

Select one:
Complex figure of Rey
Speech sound perception test

Benton visual retention test


Benton verbal fluency test

Seashore rhythms test

Frontal lobe function tests include verbal fluency tests, category tests, trail making test and
Wisconsin card sorting test, Stroop colour word interference test etc. Temporal lobe tests:
Benton visual retention test, speech sound perception test, seashore rhythms test and revised
Wechsler memory scale. Rey Osterreith complex figure is not a frontal lobe test.

562
The correct answer is: Benton verbal fluency test

009
Following a traumatic brain injury, a 41-year-old woman loses the ability to recognize her
husband's face. But she is able to recognize him from his 'voice' and his 'preferential routines'.
The brain area most likely to be damaged by the injury is

Select one:
Left inferior temporal
Bilateral medial prefrontal
Right dorsolateral prefrontal
Right inferior temporal
Bilateral occipito-temporal

Acquired prosopagnosia is usually associated with bilateral or right-sided lesions of the


occipital - temporal junction (FUSIFORM GYRUS). In rare cases of prosopagnosia after left-
sided lesions in left-handed subjects, it is attributed to a reversed hemispheric specialization
for face processing
The correct answer is: Bilateral occipito-temporal

010
A construction worker sustains a head injury after a sudden fall. Following this he presents with
a change in behaviour, violent outbursts, impulsivity and reduced speech fluency. Which of the
following could be the most likely MRI finding in this patient?

Select one:
Bilateral occipital contre-coup injury
Diffuse axonal injury

Bilateral parietal contusion


Bilateral orbito-frontal contre-coup injury
Bilateral temporal contre-coup injury

There are four regions of the prefrontal corex that are frequently mentioned in the scientific
literature. These are orbitofrontal, dorsolateral prefrontal, anterior cingulate cortex and
ventromedial cortex (the latter two together form the medial frontal surface).
Pseudopsychopathy is a result of orbitofrontal damage.
The correct answer is: Bilateral orbito-frontal contre-coup injury

563
011
Which of the following subtests of Wechsler Adult Intelligence Scale (WAIS) declines with
aging?

Select one:
Information
Vocabulary
Block design
Picture completion
Object assembly

In WAIS, hold tests are vocabulary, information, object assembly and picture completion. Non-
hold tests are block design, digit span, similarities and digit symbol. A deterioration quotient is
derived from the difference between 'don't hold' and 'hold' test scores. Hold tests are supposed
to be resistant to age­related decline; so they may be sensitive for organic brain damage such
as dementia. But this is disputed.
The correct answer is: Block design

012
Which of the following aphasia is characterised by loss of fluency and repetition ability but
preserved comprehension?

Select one:
Transcortical motor aphasia
Broca's aphasia

Transcortical sensory aphasia


Conduction aphasia
Wernicke's aphasia

In Broca's aphasia the speech is nonfluent; it often appears laboured with any interruptions
and pauses. Abnormal word order and a characteristic agrammatism are noted. Speech is
telegraphic.
The correct answer is: Broca's aphasia

564
013
Which of the following tests can differentiate organic from psychiatric stuporous states?

Select one:
Caloric testing
Electromyography
Wada test

Behavioural memory test


Dichotic listening

Caloric testing in organic stupor will usually reveal tonic deviation whereas in a psychiatric
stupor (catatonia/depression) ocular nystagmus will be present. This is because the fast
phase reflects the correction following tonic deviation, and this requires the patient to be
conscious
The correct answer is: Caloric testing

014
Which of the following is noted when the temporal order of memory is disturbed leading to out-
of-context retrieval?

Select one:
Confabulation
Blackouts
Transient global amnesia
Anomia
Disorientation

Confabulation is seen in Korsakoff's syndrome and also in dementias. Confabulation often


involves the misordering and fusion of real memories that end up being retrieved out of
context.
The correct answer is: Confabulation

015
Which of the following structure is most commonly affected in alexia without agraphia?

Select one:

565
Corpus callosum

Arcuate fasciculus

Wernicke's area

Broca's area Right

visual cortex

Alexia without agraphia (pure word blindness) almost always involves an infarct to the left
posterior cerebral artery affecting splenium of the corpus callosum and left visual cortex. So
the affected person, who is still able to see with right visual cortex, cannot undertake lexical
word processing making him unable to read. But he or she can still normally write.
The correct answer is: Corpus callosum

016
Which of the following is INCORRECT with regard to vegetative states?

Select one:
Language comprehension is absent
Cycles of eye closure and opening are absent
Spontaneous maintenance of respiration
Complete absence of awareness of environment
Noxious stimulus elicits no response

Monti et al. provided a set of criteria for diagnosing vegetative states: "Three main clinical
features define the vegetative state: (a) cycles of eye opening and closing, giving the
appearance of sleep­wake cycles; (b) complete lack of awareness of the self or the
environment; and (c) complete or partial preservation of hypothalamic and brain stem
autonomic functions
The correct answer is: Cycles of eye closure and opening are absent

017
Gerstmann syndrome is associated with lesions of which of the following structures?

Select one:
Dominant parietal lobe

566
Non dominant temporal lobe
Dominant temporal lobe
Occipital lobe

Non dominant parietal lobe

Gerstmann syndrome is associated with lesions of the dominant parietal lobe. It consists of
finger agnosia, right-left disorientation, agraphia and dyscalculia. Lesions in the non-
dominant parietal lobe hemisphere result in anosognosia, hemisomatognosia, dressing
apraxia and prosopagnosia.
The correct answer is: Dominant parietal lobe

018
The type of amnesia seen in Korsakoff's syndrome is

Select one:
Implicit

Procedural
Episodic
Working memory deficits
Infantile memory deficits

Korsakoff's syndrome is characterized by anterograde amnesia, retrograde amnesia with


episodic memory impairment, confabulation, lack of insight and apathy. The retrograde
amnesia extends back at least 25 to 30 years and includes loss of memory for remote
information and autobiographical memory for incidents or events from the patient's past.
The correct answer is: Episodic

019
Which of the following psychological tests is used in research on animal models of
depression?

Select one:
Spitz test
Radial arm maze task
Light-Dark box

567
Morris water maze task
Forced swim test

The behavioural despair test (also called the Porsolt test or forced swimming test) is a test used
to measure the effect of antidepressant drugs on the behaviour of laboratory animals (typically
rats or mice). Animals are initially made to swim in a cylinder filled with water from which they
cannot escape even if they swim vigorously but rescued manually after 15 minutes. After a day,
the forced swimming experiment is repeated but only for 5 minutes. The total time for which the
animal shows no effort to escape or swim out of the cylinder during the second trial is the
immobility time that indexes helplessness. This time is decreased by antidepressants.

The correct answer is: Forced swim test

020
A 59-year-old gentleman is being assessed in the memory clinic. He presents with
progressive loss of memory for 'words' but has good day-to-day memory for events. What is
his most likely diagnosis?

Select one:
Lewy body dementia
Parkinson disease dementia
Alcoholic dementia

Frontotemporal dementia
Alzheimer's dementia

In patients with frontotemporal dementia, there is a relative preservation of day-to-day memory


(episodic). The temporal lobe variant presents more commonly with semantic dementia, a
syndrome of progressive word finding difficulty, loss of language comprehension, depletion of
conceptual knowledge and impairment of object recognition.
The correct answer is: Frontotemporal dementia

021
While testing executive functions of frontal lobe, response inhibition can be specifically tested
using

Select one:
Go-No Go test
Category fluency test

568
Cognitive estimates test
Digit span test
Proverb interpretation

Impulsivity is thought to reflect a failure of response inhibition, and is seen in inferior frontal
pathology. It can be assessed using the Go-No-Go task. The examiner instructs the patient to
tap once in response to a single tap and to withhold a response for two taps. This test can be
made more difficult by changing the initial rule after several trials (for example, "tap once when
I tap twice, and not at all when I tap once").(Kipps and Hodges (2006). Retrieved from
http://jnnp.bmj.com/content/76/suppl_1/i22.full)).
The correct answer is: Go-No Go test

022
Ability to recognise the number or alphabet scratched on one's skin without seeing is called

Select one:
2 point discrimination

Chromatopsis

Stereognosis

Nosognosis

Graphesthesia

Ability to recognise the number or alphabet that is scratched on one's skin without seeing is
called graphesthesia, and it is a parietal lobe function.
The correct answer is: Graphesthesia

023
A patient tells you that he has done poorly in the Hayling's test. What do you infer?

Select one:
He is jumping to conclusions
He has black and white thinking
He has verbal memory deficits
He has visuospatial deficits
He has set-shifting deficits

569
The Hayling Sentence Completion test measures both response initiation and suppression. Two
sets of 15 sentences each having the last word missing is presented to the subject. In the first
part of the test, the subject has to fill the missing word to complete the sentence in a meaningful
manner. The speed of completion of this task is the measure of response initiation. The second
part requires subjects to complete the test sentences with nonsensical finishing word and thus
require an effective suppression of sensible words. This gives a measure of response
suppression. A poor performance suggests frontal deficits.
The correct answer is: He has set-shifting deficits

024
The clinical sign of finger-nose ataxia is seen in lesions of which of the following structures?

Select one:
Inferior colliculus
Superior colliculus
Thalamus

Inferior olivary nucleus


Pyramidal decussation

Inferior olivary nucleus serves motor coordination via projecting climbing fibers to the
cerebellum. Axons arising from cells in the inferior olive cross and enter the inferior cerebellar
peduncle to reach the contralateral cerebellar cortex. Inferior olivary lesions lead to
appendicular ataxia that can be tested using finger-nose test.
The correct answer is: Inferior olivary nucleus

025
Double vision that is present only on horizontal gaze is seen in

Select one:
Myasthenia gravis
Isolated 6th nerve lesion
Exophtholmos of hyperthyroidism
Isolated 3rd nerve lesion
Bilateral 4th nerve lesion

570
The nucleus of the abducens (6th) nerve is located in the paramedian pontine region in the
floor of the fourth ventricle. It innervates the lateral rectus, which abducts the eye. Patients
complain of double vision on horizontal gaze only. This finding is referred to as horizontal
homonymous diplopia
The correct answer is: Isolated 6th nerve lesion

026
An increase in oral and sexual behaviours, placidity and a loss of fear or anger with apathy is a
feature of which of the following conditions?

Select one:
Kluver-Bucy syndrome
Ganser syndrome
Frontal lobe syndrome
Gerstmann syndrome
Cerebellar syndrome

Kluver-Bucy syndrome results from bilateral ablation of temporal lobes and destruction of the
uncus, amygdala and hippocampus in monkeys. A similar syndrome is seen in humans in
association with a number of disorders such as Alzheimer's dementia, Pick's disease, herpes
simplex encephalitis and cerebral tumours when medial temporal lobe is involved.
The correct answer is: Kluver-Bucy syndrome

027
Following a traumatic brain injury, a 41-year-old woman loses the ability to identify her body
parts correctly. She is not able to use numbers for simple calculations. The brain area most
likely to be damaged is

Select one:
Right inferior parietal
Left angular gyrus

Left inferior temporal


Bilateral occipito-temporal
Right supplementary motor

571
Gerstmann syndrome is characterized by four primary symptoms: Dysgraphia/agraphia,
Dyscalculia/acalculia, Finger agnosia and Left-right disorientation. This is rarely seen as full
presentation of tetrad; It is often associated with brain lesions in the dominant (usually left)
side of the angular and supramarginal gyri (parietal lobe).
The correct answer is: Left angular gyrus

028
Loss of verbal fluency is associated with which of the following lesions?

Select one:
Right frontal
Right parietal
Left temporal
Left parietal
Left frontal

Motor speech disorder is a feature of left frontal lesions due to damage to Broca's area located
in this region.
The correct answer is: Left frontal

029
Cerebellar dysfunction is associated with all of the following except

Select one:
Dysdiadochokinesia
Intention tremor

Micrographia
Truncal ataxia
Past pointing in finger-nose test

572
Cerebellar dysfunction is characterised by truncal ataxia, dysequilibrium, reduced muscle tone,
decreased reflexes, incoordination of movements which manifest by poor ability to perform
rapid alternating movements (Dysdiadochokinesia), past pointing in finger-nose test
(dysmetria) and intention tremor. Micrographia is a feature of basal ganglia damage seen in
Parkinson's.
The correct answer is: Micrographia

030
A 47-year-old man presented to the neurology clinic with a diagnosis of bitemporal
hemianopia. A lesion affecting which part of the brain or eye could be attributed to this
presentation?

Select one:
Thalamus
Optic nerve
Optic radiation
Optic chiasma
Geniculate body

A lesion affecting optic chiasma e.g., pineal tumours and craniopharyngioma can cause
bitemporal hemianopia.
The correct answer is: Optic chiasma

031
Which part of the brain is associated with forced utilization behavior?

Select one:
Ventromedial part of frontal lobe
Temporal lobe

Orbitofrontal lobe
Parietal lobe
Dorsolateral frontal lobe

If the frontal lobe region that is superior to the eye is damaged, forced utilization may be seen.
When objects are placed in front of the subject, the subject with forced utilization will pick an
object and make attempts to use it even when he/she is instructed not to do so. This behaviour

573
is attributed to the damage of the orbital frontal structures and the caudate nuclei.
The correct answer is: Orbitofrontal lobe

032
In delirium, which of the following domains of orientation is the first to be affected?

Select one:
Orientation to age
Orientation to language
Orientation to person
Orientation to place
Orientation to time

Orientation is usually assessed for time, place and person; it is not a particularly sensitive
cognitive test. Intact orientation does not exclude a significant memory disorder, particularly if
there is concern about memory in the history from an informant. Time orientation is the most
helpful, affected early in delirium and the test should include a on the time of the day (Kipps
and Hodges (2006).

The correct answer is: Orientation to time

033
Which of the following is noted in dorsolateral prefrontal syndrome?

Select one:
Reduced verbal output
Poor impulse control

Psychopathic dyscontrol
Poor planning ability
Apathy

Cognitive dysfunction, diminished judgment, planning, and insight along with a concrete and
inflexible attitude is seen in DLPFC lesions. Reduced spontaneous behaviours are also
notable. (DYSEXECUTIVE SYNDROME). Poor impulse control, explosive outbursts and
inappropriate behaviour are seen in orbitofrontal lesions. (PSEUDOPSYCHOPATHIC
SYNDROME).

574
The correct answer is: Poor planning ability

034
Alexia without agraphia (pure alexia) is seen in lesions of

Select one:
Middle meningeal artery
Posterior inferior cerebellar artery
Middle cerebral territory
Posterior cerebral territory
Anterior cerebral territory

The posterior cerebral artery arises from basilar artery and supplies the inferomedial temporal
lobe and the occipital lobe, and some parts of the posterior parietal cortex. Lesions in this
arterial supply can cause alexia without agraphia.
The correct answer is: Posterior cerebral territory

035
A 64-year-old lady presents with acute onset vertigo and double vision. On examination, one
sided facial numbness, diplopia and ataxia are noted with absent gag reflex. Which of the
following lesion is most likely?

Select one:
Posterior cerebral artery
Anterior choroidal artery

Middle cerebral artery


Internal carotid artery (left)
Posterior inferior cerebellar artery

The symptoms are consistent with Wallenberg or lateral medullary syndrome which is
common in posterior inferior cerebellar artery lesions.
The correct answer is: Posterior inferior cerebellar artery

036

575
A 78-year-old man fails to eat the food on one side of his plate following recovery from a stroke.
When asked to bisect a straight line, he chooses to bisect it closer to the right end. The brain
area most probably affected is

Select one:
Left parietal lobe
Right parietal lobe
Left occipital lobe

Right occipital lobe


Left frontal lobe

Neglect of personal and extrapersonal space is usually caused by lesions of the right
hemisphere - often the inferior parietal or prefrontal regions.
The correct answer is: Right parietal lobe

037
A reduction in the score on the Seashore Musical Aptitude Test may be seen in which of the
following type of brain lesions?

Select one:
Frontal lobe damage
Left parietal lobe damage
Right parietal lobe damage
Right temporal area damage
Left temporal area damage

Right temporal area damage may result in a reduction of the score on the seashore musical
aptitude test and receptive amusia. In Seashore Rhythms test, thirty pairs of tape-recorded,
non-verbal sounds are presented. For each pair, individuals decide if the two sounds are the
same or different, marking 'S' or 'D' respectively on their answer sheets. The pairs are grouped
into three subtests.This is based on the Seashore Tests of Musical Ability. It evaluates auditory
attention and concentration, and the ability to discriminate between non-verbal sounds
The correct answer is: Right temporal area damage

038
A 53-year-old woman has alexia and agraphia. She is likely to exhibit

576
Select one:
Impaired auditory comprehension
Normal speech
Right visual field defect
Intact writing

Color agnosia

Alexia without agraphia occurs in occipitotemporal lesions (Note: alexia with agraphia often
results from a left angular gyral lesion). Right homonymous hemianopia is commonly
associated with alexia; speech includes anomic disturbances but color recognition and
auditory comprehension are often intact.
The correct answer is: Right visual field defect

039
A better preserved letter fluency but poor category fluency is noted in

Select one:
Korsakoff's syndrome
Alexia with agraphia
Semantic dementia
Frontal lobe damage
Normal young adults

With respect to the memory for word meaning and general knowledge (semantic memory), the
key neural substrate is the anterior temporal lobe. The ability to generate categories based on
semantic memory is tested in category fluency tests; thus these tests tap on semantic memory
as well as fluency. In frontotemporal dementia (semantic dementia) this memory domain is
affected earlier than phonological or letter (verbal) fluency.
The correct answer is: Semantic dementia

040
A 74-year-old man who recently suffered a vascular stroke, refers to apples as 'oranges'. This
is called

Select one:
Word blindness

577
Lingual apraxia
Wernicke's aphasia
Semantic paraphrasia

Cortical blindness

Semantic paraphasia refers to the mis-selection of words due to semantic confusion, where
the actual utterance bears some conceptual relationship to the intended utterance
(e.g.`bread` for `food`). In contrast, phonemic paraphasia refers to the mis-selection due to
phonemic confusion, where the actual utterance bears a sound similarity to the intended
utterance (e.g. `shark` for `sharp`).
The correct answer is: Semantic paraphrasia

041
Which feature is not associated with supranuclear palsy?

Select one:
Loss of balance

Dementia
Spared eye movements
Tau proteins
Brainstem degeneration

The most common early complaint in progressive supranuclear palsy (PSP) is unsteadiness of
gait and unexplained falling (retropulsion). It may take a year or longer for the characteristic
syndrome-comprising supranuclear ophthalmoplegia, pseudobulbar palsy, and axial dystonia to
develop fully. They may have difficulty in voluntary vertical movement of the eyes (vertical gaze
palsy). The principal areas of the brain affected are: the basal ganglia, the brainstem, particularly
the portion of the midbrain where "supranuclear" eye movement resides, the cerebral cortex and
the dentate nucleus of the cerebellum. The neurons display neurofibrillary tangles, which are
clumps of tau protein.
The correct answer is: Spared eye movements

042
Absence of ankle jerks with upgoing plantars is noted in

Select one:
Brown Sequard syndrome at T10

578
Peripheral neuropathy related to medications
Internal capsule lesions of corticospinal tract
Spinothalamic tract lesions

Subacute combined degeneration of spinal cord

UMN lesion of corticospinal tracts is expected to cause exaggerated ankle reflex (i.e. clonus)
with upgoing plantar normally. But in subacute combined degeneration cord, Syphilitic
taboparesis and Friedrich's ataxia and MND we see absence of ankle jerk as spinal reflex
pathway is affected (afferent) while UMN type damage still produces Babinski - upgoing
plantar.
The correct answer is: Subacute combined degeneration of spinal cord

043
Which of the following best describes Ribot's law?

Select one:
Anxiety decreases with increasing task difficulty

Arousal and performance are related in an inverted U shaped correlation


The gradient of episodic memory loss starts from the most recent to more
remote events
Organic memory loss affects remote memories before affecting more
recent ones
Observation increases degree of conformity

Ribot's Law of retrograde amnesia: 'The dissolution of memory is inversely related to the
recency of the event'. Recent memories are more likely to be lost than the more remote
memories in organic amnesia (though there are some exceptions to this rule).
The correct answer is: The gradient of episodic memory loss starts from the most recent to
more remote events

044
An 18-year-old girl presents with sudden blindness. Fundoscopy, visual evoked potentials
and MRI brain are normal and no organic cause could be found. On perimetry, which of the
following findings will support a diagnosis of conversion disorder?
Select one:

579
Symmetrically constricted visual field
Tunnel vision
Paracentral scotoma

Macular sparing
Bitemporal (heteronymous) hemianopia

Tunnel vision refers to an absence of projected disparity between near and far fields of vision.
The patients report having patchy spirals of field loss. This is seen in hysteria and malingering.

The correct answer is: Tunnel vision

045
A 73-year-old man is presenting with one-year history of progressive cognitive decline. He
retains full insight into his cognitive difficulties. The most likely diagnosis is

Select one:
All of the above
Vascular dementia
Frontotemporal dementia
CJD
Huntington's dementia

In people with vascular dementia, the impairment of cognitive function is often patchy or
uneven; there may be memory loss, intellectual impairment, and focal neurological signs.
Insight, judgement and personality may be relatively well preserved.
The correct answer is: Vascular dementia

046
When performing the Mini Mental Status Examination (MMSE), an inability to copy intersecting
polygons is suggestive of

Select one:
Visuospatial deficits
Attentional deficits
Visual agnosia

580
Ideomotor apraxia
Comprehension deficits

Inability to draw shapes or construct geometrical patterns is seen in those with visuospatial
defects. Mostly nondominant hemisphere is damaged.
The correct answer is: Visuospatial deficits

047
In a patient with right-sided brain lesion resulting from a head injury which of the following is
most likely?

Select one:
Finger agnosia
Language comprehension deficits
Limb apraxia
Visuospatial neglect
Motor aphasia

Visuospatial disorders, dressing and constructional apraxias and anosognosia may occur with
lesions of either hemisphere but are observed more frequently and are of greater severity with
lesions of the nondominant one.
The correct answer is: Visuospatial neglect

048
A 25-year-old man sustains a severe head injury in a motor cycle race event. On recovery, he
is noted to have grasp reflex, poor judgement and impaired reasoning with impulsivity. Which of
the following tests is highly likely to be abnormal?

Select one:
Wisconsin card sorting test
Wechsler adult intelligence scale IQ score

Paragraph retention test


Wechsler memory scale
National adult reading test

581
The Wisconsin Card Sorting Test (WCST) contains stimulus cards of different colour, form, and
number. These are presented to patients to sort into groups according to a single principle (e.g.,
to sort by colour, ignoring form and number). Persons with damage to the frontal lobes or the
caudate and some persons with schizophrenia give abnormal responses.
The correct answer is: Wisconsin card sorting test

582
 Home HiYield Paper Neurosciences

HiYield Paper A(2)

Started on Saturday, 9 May 2015, 2:19 AM
State Finished
Completed on Saturday, 9 May 2015, 7:29 PM
Time taken 17 hours 10 mins
Marks 224.48/272.00
Grade 82.53 out of 100.00

Question 1 HiY Neurosciences EMI001
Partially correct Psychometric assessments
With the descriptions given below, identify the type of neuropsychological tests from the list
Mark 3.00 out of
provided;
4.00

Remove flag
Wisconsin card sorting test
Wechsler adult intelligence scale
This consists of ten
Raven's progressive matrices
tests, including the trail
Stanford­Binet intelligence scale
making test and critical
Halstead­Reitan battery of neuropsychological tests
flicker frequency test
Thematic Apperception Test
Rorschach test
Wechsler memory scale
Sentence completion test
Wisconsin card sorting test
This consists of 120 Bender visual motor gestalt test
Wechsler adult intelligence scale
items plus several Luria­Nebraska Neuropsychological Battery
Raven's progressive matrices
alternative tests,
Stanford­Binet intelligence scale
applicable to the ages Wisconsin card sorting test
Halstead­Reitan battery of neuropsychological tests
between 2 years and Wechsler adult intelligence scale
adulthood. Thematic Apperception Test
Raven's progressive matrices
Rorschach test
Stanford­Binet intelligence scale
Wechsler memory scale
Halstead­Reitan battery of neuropsychological tests
Sentence completion test
Thematic Apperception Test
Bender visual motor gestalt test
Rorschach test
Luria­Nebraska Neuropsychological Battery
Wechsler memory scale
Sentence completion test
This test results can Bender visual motor gestalt test
reveal whether a Luria­Nebraska Neuropsychological Battery
subject has amnestic
Korsakoff's syndrome

583
Wisconsin card sorting test
Wechsler adult intelligence scale
This is a test of
intelligence, Raven's progressive matrices
considered to be free of Stanford­Binet intelligence scale
sociocultural bias. Halstead­Reitan battery of neuropsychological tests
Thematic Apperception Test
Rorschach test
Wechsler memory scale
Sentence completion test
Check
Bender visual motor gestalt test
Luria­Nebraska Neuropsychological Battery

Explanation: 
Halstead­Reitan battery of neuropsychological tests consists of ten tests, including the trail
making test and critical flicker frequency test. It was developed in an attempt to improve the
reliability of the criteria used to diagnose brain damage. The trial­making test is a test of
visuomotor perception and motor speed and the critical flicker frequency tests visual
perception. 
Stanford­Binet intelligence scale­ this test is most commonly used in the individual
examination of children. It is helpful to assess intelligence and consists of 120 items plus
several alternative tests, applicable to the ages between 2 years and adulthood. 
Wechsler memory scale­ screens for verbal and visual memory and, therefore, yields a
memory quotient. So the results can reveal whether a patient has amnestic Korsakoff's
syndrome. 
Raven's progressive matrices is a test of performance intelligence, considered to be free of
cultural bias or language bias.
The correct answer is: This consists of ten tests, including the trail making test and critical
flicker frequency test – Halstead­Reitan battery of neuropsychological tests, This consists of
120 items plus several alternative tests, applicable to the ages between 2 years and
adulthood. – Stanford­Binet intelligence scale, This test results can reveal whether a subject
has amnestic Korsakoff's syndrome – Wechsler memory scale, This is a test of intelligence,
considered to be free of sociocultural bias. – Raven's progressive matrices

Question 2 HiY Neurosciences EMI002
Partially correct Neuropsychological evaluation
A 54­year man presents with one year history of progressive change in personality and
Mark 2.00 out of
behaviour profile. He has become gradually apathetic and depressed and experiences
Mini­Mental State Examination
4.00
recurrent frontal headaches. He also experiences occasional word­finding difficulties. EEG
Go ­ No Go test
Remove flag shows unilateral frontal slowing. You are considering an intracranial space­occupying lesion.
Wechsler Memory Scale
National Adult Reading Test
Rivermead Behavioural Memory Test
Stroop test
Which test would give a
Cognitive Estimates Test
fast estimate of his
Digit span
current performance
IQ? Wisconsin Card Sorting Test
Raven's progressive matrices

584
The test indicates that Mini­Mental State Examination
his current performance
Go ­ No Go test
IQ is in the low average
range. How would you Wechsler Memory Scale
estimate the IQ level National Adult Reading Test
before he may have Rivermead Behavioural Memory Test
sustained any brain Stroop test
damage in recent Cognitive Estimates Test
months / years? Digit span
The estimate of his Wisconsin Card Sorting Test
premorbid IQ is 15 Raven's progressive matrices
points higher than his
current performance IQ.
Mini­Mental State Examination
It is recommended that
he has a full WAIS IQ Go ­ No Go test
assessment to measure Wechsler Memory Scale
both performance and National Adult Reading Test
verbal IQ. On the WAIS, Rivermead Behavioural Memory Test
his verbal IQ is found to Stroop test
be impaired over and Cognitive Estimates Test
above his performance Digit span
IQ. Which test is a part
Wisconsin Card Sorting Test
of the WAIS verbal
Raven's progressive matrices
subtests?
An MRI scan shows a
Mini­Mental State Examination
large meningioma
Go ­ No Go test
compressing
Wechsler Memory Scale
dorsolateral prefrontal
cortex on the left. Which National Adult Reading Test
test result is most likely Rivermead Behavioural Memory Test
to be impaired? Stroop test
Cognitive Estimates Test
Digit span
Wisconsin Card Sorting Test
Check Raven's progressive matrices

Explanation: Raven's matrices are used to estimate performance IQ ­ they are useful
especially when cultural or language boundaries exist for neuropsychological testing. It taps
on general intelligence with visuospatial problem­solving. 
NART (National adult reading test) tests premorbid IQ. 
Digit span is a part of verbal subtests of WAIS (Wechsler adult intelligence scale). 
WCST (Wisconsin card sorting test) is a standard test for executive functions affected by
frontal lesions.
The correct answer is: Which test would give a fast estimate of his current performance IQ? –
Raven's progressive matrices, The test indicates that his current performance IQ is in the low
average range. How would you estimate the IQ level before he may have sustained any brain
damage in recent months / years? – National Adult Reading Test, The estimate of his
premorbid IQ is 15 points higher than his current performance IQ. It is recommended that he
has a full WAIS IQ assessment to measure both performance and verbal IQ. On the WAIS, his
verbal IQ is found to be impaired over and above his performance IQ. Which test is a part of
the WAIS verbal subtests? – Digit span, An MRI scan shows a large meningioma compressing
dorsolateral prefrontal cortex on the left. Which test result is most likely to be impaired? –

585
Wisconsin Card Sorting Test

Question 3 HiY Neurosciences EMI003
Partially correct Brain structure and function
For each lesion described below choose possible clinical correlates
Mark 3.33 out of
5.00

Flag question
Cortical blindness
Finger agnosia
A right­handed man Autotopagnosia
with left frontal lobe Receptive dysphasia
lesion  Constructional apraxia
Headache
Expressive dysphasia
Impaired smell sensation
Cortical blindness
Finger agnosia
A right­handed man Autotopagnosia
with right parietal lobe Receptive dysphasia
lesion  Constructional apraxia
Headache
Expressive dysphasia
Impaired smell sensation
Cortical blindness
Finger agnosia
A right­handed man Autotopagnosia
with left parietal lobe Receptive dysphasia
lesion  Constructional apraxia
Headache
Expressive dysphasia
Impaired smell sensation

Check

Explanation: The effects of left frontal disease include right hemiplegia, motor speech disorder
(expressive dysphasia due to Broca's lesion) with agraphia, sympathetic apraxia of left hand,
loss of verbal associative fluency and perseveration. The effects of a unilateral disease of
non­dominant parietal lobe include visuospatial disorders, topographic memory loss,
anosognosia, dressing and constructional apraxia. Gerstmann syndrome (finger agnosia,
right­left confusion, dysgraphia, and dyscalculia) is a feature of dominant parietal lobe lesion.
The correct answer is: A right­handed man with left frontal lobe lesion 
– Expressive dysphasia, A right­handed man with right parietal lobe lesion 
– Constructional apraxia, A right­handed man with left parietal lobe lesion 
– Finger agnosia

Question 4 HiY Neurosciences EMI004
Partially correct Cognitive function tests
Select one best for each of the following situations;
Mark 4.00 out of

586
5.00
Stanford­Binet test
Remove flag Trail making test
National adult reading test
Executive function Halstead­Reitan Battery  
Rey­Osterrieth test
Vocabulary test
Paired associate learning test
Raven's progressive matrices
Digit Span
Stanford­Binet test
Wisconsin card sorting test
Trail making test
National adult reading test
Non­verbal  
Halstead­Reitan Battery
intelligence
Rey­Osterrieth test
Vocabulary test
Paired associate learning test
Raven's progressive matrices
Digit Span
Stanford­Binet test
Wisconsin card sorting test
Trail making test
National adult reading test
Brain damage Halstead­Reitan Battery  
Rey­Osterrieth test
Vocabulary test
Paired associate learning test
Raven's progressive matrices
Digit Span
Stanford­Binet test
Wisconsin card sorting test
Trail making test
National adult reading test
Tests of visual  
Halstead­Reitan Battery
memory
Rey­Osterrieth test
Stanford­Binet test
Vocabulary test
Trail making test
Paired associate learning test
National adult reading test
Raven's progressive matrices
Halstead­Reitan Battery
Digit Span
Rey­Osterrieth test
Wisconsin card sorting test
Vocabulary test
Paired associate learning test
Tests of verbal  
memory Raven's progressive matrices
Digit Span
Wisconsin card sorting test

Check

Explanation: The Wisconsin Card Sorting Test (WCST) is useful to measure executive
functions and is useful to detect prefrontal lobe pathology. It contains stimulus cards of
different colours, shapes, and numbers. Subjects are asked to sort these into groups
according to a single principle (e.g., to sort by colour, ignoring form and number). Persons
with damage to the frontal lobes or to the caudate and some persons with schizophrenia give
abnormal responses. 

587
Raven's progressive matrices test is useful to measure non verbal intelligence. It involves
color pattern matching exercises and is independent of education and cultural influence. 
Halstead & Reitan developed a battery of tests that was used to determine the location and
the effects of specific brain lesions. 
Rey­Osterreith test is a test of visual memory that involves the subject to copy a complex figure
made of basic geometric shapes and then to draw it from memory. 
Paired associates test is a test of verbal memory, available for paired and unpaired words.
The correct answer is: Executive function – Wisconsin card sorting test, Non­verbal
intelligence – Raven's progressive matrices, Brain damage – Halstead­Reitan Battery, Tests
of visual memory – Rey­Osterrieth test, Tests of verbal memory – Paired associate learning
test

Question 5 HiY Neurosciences EMI005 Paired associate learning test


Partially correct Cognitive test domains Trail making test
Select one best answer for each of the following situations;
Mark 3.00 out of Rey­Osterrieth test
5.00 National adult reading test
Remove flag Digit span
Bender Visual Motor Gestalt Test
Vocabulary test
Measures working
memory
Raven's progressive matrices
Stroop test
Paired associate learning test
Wisconsin card sorting test
Trail making test
Rey­Osterrieth test
National adult reading test
Digit span
Bender Visual Motor Gestalt Test
Measures set shifting Vocabulary test
abilities and response Raven's progressive matrices
inhibition Stroop test
Wisconsin card sorting test

Paired associate learning test
Trail making test
Measures visuospatial Rey­Osterrieth test
and perceptuomotor National adult reading test
speed Digit span
Bender Visual Motor Gestalt Test
Vocabulary test
Raven's progressive matrices
Stroop test
Paired associate learning test
Wisconsin card sorting test
Trail making test
Rey­Osterrieth test
National adult reading test
Measures pre­morbid Digit span
intelligence Bender Visual Motor Gestalt Test
Vocabulary test
Raven's progressive matrices
Stroop test
Wisconsin card sorting test

588
Trail making test
Rey­Osterrieth test
National adult reading test
Digit span
Bender Visual Motor Gestalt Test
Screening measure for Vocabulary test
signs of organic Raven's progressive matrices
dysfunction Stroop test
Wisconsin card sorting test

Check

Explanation:
Working memory refers to the very limited capacity, which allows us to retain information for a
few seconds and depends on the intact function of the dorsolateral prefrontal cortex. Digit
span is a relatively pure test of attention and is dependent on working memory, but it is not a
specific test; it is impaired in acute confusional states, focal left frontal damage, aphasia, and
late stages of dementia, but not affected in Korsakoff's syndrome or medial temporal lobe
damage (Kipps and Hodges, 2005). Normal digit span is 7 +/­ 2 depending on age and
general intellectual ability. In the elderly, or intellectually impaired, 5 can be considered
normal. Reverse span is usually one or two less than forward span (5 ± 2 digits. 
Stroop test: It measures set shifting abilities and response inhibition. It is a test of frontal
function and selective attention ability. 
Trail making test: It consists of two parts. In part, A simple number sequence is used to join the
dots. Test B uses alternating numbers and letters and is thought to be more sensitive to frontal
lobe dysfunction. It tests visuomotor tracing, attention and conceptualisation and most
importantly, set shifting (test B). 
National adult reading test taps on previous word knowledge before becoming ill (or
sustaining brain injury) ­ hence it is used to estimate the premorbid IQ. 
The Bender Visual Motor Gestalt Test is a test of visuomotor coordination that is useful for both
children and adults. Children <3 cannot reproduce the test designs meaningfully. It is most
frequently used in adults as a screening measure for organic dysfunction e.g. after head
injury.
Excerpt from Cognitive assessment for clinicians ­­ Kipps and Hodges (2005). Retrieved from
http://jnnp.bmj.com/content/76/suppl_1/i22.full
The correct answer is: Measures working memory – Digit span, Measures set shifting abilities
and response inhibition – Stroop test, Measures visuospatial and perceptuomotor speed –
Trail making test, Measures pre­morbid intelligence – National adult reading test, Screening
measure for signs of organic dysfunction – Bender Visual Motor Gestalt Test

Question 6 HiY Neurosciences EMI006
Correct MMSE
Identify one cognitive process, which is tested by different items of MMSE
Mark 5.00 out of
5.00

Flag question

589
Repetition of 3 objects Writing
(Apple, Table & Penny) Retention & Recall
after a distraction task Language­understanding
Attention & Concentration
Orientation to time
Orientation to place
Constructional Praxis
Naming
Long term memory
Writing
Registration
Retention & Recall
Language­understanding
Spell the word
Attention & Concentration
'WORLD' backwards
Orientation to time
Orientation to place
Constructional Praxis
Naming
Long term memory
Writing
Registration
Retention & Recall
Language­understanding
Months of the year
Attention & Concentration
backwards
Orientation to time
Orientation to place
Constructional Praxis
Naming
Long term memory
Writing
Registration
Retention & Recall
Carrying out a 3 stage
command such as Language­understanding
'Take this paper in your Attention & Concentration
right hand, fold it in half Orientation to time
and put it on the floor.' Orientation to place
Constructional Praxis
Naming
Long term memory
Writing
Registration
Retention & Recall
Language­understanding
What day of the week is
Attention & Concentration
today?
Orientation to time
Orientation to place
Constructional Praxis
Naming
Long term memory
Check
Registration

Explanation: Repetition of 3 objects (Apple, Table & Penny) after a distraction task tests
retention and recall abilities. Attention can be tested in a number of ways including serial 7s,
digit span, spelling "world" backwards, and recitation of the months of the year in reverse
order or continuous performance tests. Although serial 7s is commonly used, it is frequently
performed incorrectly by the elderly, as well as by patients with impaired attention (Kipps and
Hodges, 2005). Serial subtraction by seven is frequently said to be a measure of

590
concentration (intense one­task orientation). It does not test inattention, which is a term often
used in the context of spatial neglect. A reverse­order month of the year is a highly over­
learned sequence and is a preferred measure of sustained attention. 
The 3­stage command tests language comprehension. 
Excerpt from Cognitive assessment for clinicians ­­ Kipps and Hodges (2005). Retrieved from
http://jnnp.bmj.com/content/76/suppl_1/i22.full
The correct answer is: Repetition of 3 objects (Apple, Table & Penny) after a distraction task –
Retention & Recall, Spell the word 'WORLD' backwards – Attention & Concentration, Months
of the year backwards – Attention & Concentration, Carrying out a 3 stage command such as
'Take this paper in your right hand, fold it in half and put it on the floor.' – Language­
understanding, What day of the week is today? – Orientation to time

Question 7 HiY Neurosciences EMI007
Partially correct Behavioural tests
For each of the following functions, choose the most relevant behavioural tests;
Mark 3.00 out of
6.00

Remove flag
Radial arm maze task
Morris water maze task
Visuospatial memory T­maze test
(Choose THREE Depressed plus maze test
answers) Facilitated swim test
Light­dark box
Radial arm maze task
Yoked shock test
Morris water maze task
Latent inhibition test
T­maze test
Spitz test
Depressed plus maze test
Facilitated swim test
Light­dark box
Sensorimotor gating
Yoked shock test
(Choose ONE answer)
Latent inhibition test
Spitz test
Radial arm maze task
Morris water maze task
T­maze test
Depressed plus maze test
Facilitated swim test
Light­dark box
A model of depression
Yoked shock test
(Choose ONE answer)
Latent inhibition test
Spitz test
Radial arm maze task
Morris water maze task
T­maze test
Depressed plus maze test
Facilitated swim test
Light­dark box
Yoked shock test
Anxiety (Choose ONE Latent inhibition test
answer) Spitz test

591
Check

Explanation: 
The radial arm maze was designed by Olton and Samuelson in 1976 to measure spatial
learning and memory in rats. 
The Morris water navigation task is an experimental procedure employed in to study spatial
learning and memory. It was developed by Edinburgh­based neuroscientist Richard G. Morris
in 1981 to show that hippocampal lesions impaired spatial learning. 
The T­maze is a popular test for the investigation of spatial learning and memory in rodents. 
Latent inhibition test is a test for sensorimotor gating.Latent inhibition is defined as an animal's
unconscious capacity to ignore stimuli that experience has shown is irrelevant to its needs.
Yoked shock test is associated with the concept of 'Learned helplessness', which is an
explanatory model for depression. 
The behavioural despair test (also called the Porsolt test or forced swimming test) is a test
used to measure the effect of antidepressant drugs on the behaviour of laboratory animals
(typically rats or mice). 
The light/dark box test measures the anxiety­related behaviour (Crawley, Neurosci Biobehav
Rev. 9:37­44, 1985). The light/dark test may be useful to predict anxiolytic­like or anxiogenic­
like activity in mice. Classic anxiolytics (benzodiazepines), as well as the newer anxiolytic­like
compounds (e.g. serotonergic drugs or drugs acting on neuropeptide receptors), can be
detected using this paradigm. It has the advantages of being quick and easy to use, without
requiring the prior training of animals. 
The elevated plus maze is a test for anxiety­like behavior (fear/anxiety). A mouse is placed in
the center of the maze and the number of entries and amount of time spent in the open and
closed arms are recorded during a brief trial.
(Ref: The mouse light/dark box test. Bourin M, Hascoet M. For pictorial representations see
www. dsm.fujitahu.ac.jp/E/behaviorcore/test_intro/test_intro_e.htm ; Also see
http://www.ncbi.nlm.nih.gov/pubmed/12600702 )
The correct answer is: Visuospatial memory (Choose THREE answers) – Radial arm maze
task, Morris water maze task , T­maze test, Sensorimotor gating (Choose ONE answer) –
Latent inhibition test, A model of depression (Choose ONE answer) – Yoked shock test,
Anxiety (Choose ONE answer) – Light­dark box

Question 8 HiY Neurosciences EMI008
Correct Inheritance patterns in psychiatry
For the following inheritance patterns identify the associated disorder from the given list.
Mark 6.00 out of
6.00

Remove flag
Autism
Catatonic Schizophrenia
Complex non­
Wilson's disease
Mendelian pattern of
Alzheimer's disease
inheritance (Choose
Gerstmann Strauss syndrome
TWO)
Fragile X syndrome
Neurofibromatosis
Normal pressure hydrocephalus
Duchenne's dystrophy

592
Alzheimer's disease
Gerstmann Strauss syndrome
Fragile X syndrome
Autosomal recessive Neurofibromatosis
pattern (Choose ONE) Normal pressure hydrocephalus
Duchenne's dystrophy
Huntington's disease

Autism
Catatonic Schizophrenia
Wilson's disease
Trinucleotide repeat
Alzheimer's disease
CAG (Choose ONE)
Gerstmann Strauss syndrome
Fragile X syndrome
Neurofibromatosis
Normal pressure hydrocephalus
Duchenne's dystrophy
Autism
Huntington's disease
Catatonic Schizophrenia
Wilson's disease
Trinucelotide repeat
Alzheimer's disease
CGG (Choose ONE)
Gerstmann Strauss syndrome
Fragile X syndrome
Neurofibromatosis
Normal pressure hydrocephalus
Duchenne's dystrophy
Check
Huntington's disease

Explanation: Both schizophrenia and autism are complex non­Mendelian disorders where no
clear autosomal or sex­linked pattern could be demonstrated in pedigrees. Wilson disease
shows an autosomal recessive pattern of inheritance. Fragile X syndrome is associated with a
large sequence of triplet repeats of CGG sequence. In Huntington's the genetic defect is an
excessive trinucleotide repeat of CAG sequence on chromosome 4.
The correct answer is: Complex non­Mendelian pattern of inheritance (Choose TWO) –
Autism, Catatonic Schizophrenia, Autosomal recessive pattern (Choose ONE) – Wilson's
disease, Trinucleotide repeat CAG (Choose ONE) – Huntington's disease, Trinucelotide
repeat CGG (Choose ONE)
– Fragile X syndrome

Question 9 HiY Neurosciences EMI009
Partially correct Molecular genetic processes
Identify processes using descriptions given below
Mark 2.67 out of
4.00

Flag question

593
This is a process by Phosphorylation
which one gene give
Glycosylation
rise to more than one
proteins
Alternate Splicing
Epistasis
Translation
Recombination
X­inactivation
Transcription

Phosphorylation
Glycosylation
These are different Alternate Splicing
ways in which proteins
Epistasis
can be modified (two
answers)
Translation
Recombination
X­inactivation
Transcription
Phosphorylation
Glycosylation
This is a distinct Alternate Splicing
process by which
Epistasis
protein is formed from
mRNA
Translation
Recombination
X­inactivation
Transcription
Check

Explanation: 
Alternate or differential splicing is a process by which the exons of transcripted mRNA are
pasted or reconnected in multiple ways during RNA splicing, thus enabling single gene to
code for many proteins. 
Post­Translational Modifications occurring at the peptide terminus of the amino acid chain
play an important role in translocating them across biological membranes. Many proteins,
particularly in eukaryotic cells, are modified by the addition of carbohydrates, a process called
glycosylation. The addition of a phosphate group, phosphorylation, is also a post­translational
modification process. 
Translation refers to production of proteins from RNA. It takes place in cytoplasm on
ribosomes where specific m­RNAs are involved. 

Excerpt from Post­Translational Modifications: An introduction. Retrieved from Protein Folding
and Processing ­ The Cell ­ NCBI Bookshelf. Retrieved from
http://www.ncbi.nlm.nih.gov/books/NBK9843/
The correct answer is: This is a process by which one gene give rise to more than one
proteins – Alternate Splicing, These are different ways in which proteins can be modified (two
answers) – Phosphorylation, Glycosylation, This is a distinct process by which protein is
formed from mRNA – Translation

594
Question 10 HiY Neurosciences EMI010
Partially correct The risk of schizophrenia to relatives 
A lady with schizophrenia is keen to know the risk of schizophrenia to her family members.
Mark 3.00 out of
What is the lifetime risk in percentage to each of the following relatives?
4.00

Flag question
20­25
15­20
35­40
Her brother 10­15
40­45
25­30
5­10
0­5
30­35
20­25
15­20
35­40
Her mother 10­15
40­45
25­30
5­10
0­5
30­35
20­25
15­20
Her child if she marries 35­40
someone without 10­15
schizophrenia 40­45
25­30
5­10
0­5
30­35
20­25
An adopted child of her 15­20
second brother. This 35­40
child has the 10­15
velocardiofacial 40­45
syndrome. 25­30
5­10
0­5
30­35
Check

Explanation: If one parent has schizophrenia, the risk to the child is 13%. If both parents have
schizophrenia, then the risk is 46% ­ close to monozygotic twin risk. The risk to a half sibling is
4%. Please note that being a sibling or being a child of someone with schizophrenia carries
the same risk. Adopted children carry the same risk as the general population unless they
carry genetic influences, which inflate (VCFS) or deflate (Down's) the risk.
The correct answer is: Her brother – 10­15, Her mother – 5­10, Her child if she marries
someone without schizophrenia – 10­15, An adopted child of her second brother. This child
has the velocardiofacial syndrome. – 25­30

595
Question 11 HiY Neurosciences EMI011
Partially correct Classifying mutations
For the following mutations, select the most appropriate term
Mark 3.00 out of
4.00

Flag question
Missense
Transversion
In frame
An increase in the size
Nonsense
of a triplet repeat
Expansion
Frame­shift
Silent
Transition
Missense
Transversion
In frame
Substitution of one
Nonsense
amino acid for another
Expansion
Frame­shift
Silent
Transition
Missense
Transversion
In frame
A 5 base pair deletion
Nonsense
in an axon
Expansion
Frame­shift
Silent
Transition
Missense
Transversion
In frame
A to T point mutation Nonsense
Expansion
Frame­shift
Silent
Transition
Check


Explanation: A silent mutation causes no change in protein product ­ this is possible because
a single amino acid is often coded by more than one triplet sequence. 
In a silent mutation, one triplet sequence is replaced by a different sequence but without
changing amino acid product. 
In missense mutation, the new mutant codon specifies a different amino acid with variable
effects on final protein product. For example, haemophilia, sickle cell anaemia. 
In nonsense mutation, the new codon is UUA UGA or UAG, which signals 'stop' to the amino
acid sequence resulting a in a nonfunctional protein. 

596
In frame shift mutations, the deletion or insertion is not in multiples of three codons e.g. a
segment of 5 bases deletion mutations.
Point mutations are usually substitutions where one base is replaced by another. It could be
termed as transition if a purine is replaced by another purine or a pyrimidine replaced by
another pyrimidine (e.g. A to G). It is called transversion if a purine is replaced by a pyrimidine
or vice versa (e.g. A to T).
The correct answer is: An increase in the size of a triplet repeat – Expansion, Substitution of
one amino acid for another – Missense, A 5 base pair deletion in an axon – Frame­shift, A to T
point mutation – Transversion

Question 12 HiY Neurosciences EMI012
Correct Differences among genetic studies 
Select the type of study for each description given below
Mark 3.00 out of
3.00

Flag question
Pedigree studies
Outcome studies
Adoption studies
Identify a specific
Linkage studies
disease locus
Population studies
Transgenic studies
Treatment studies
Association studies
Family studies
Pedigree studies
Outcome studies
Establish chromosomal Adoption studies
location of a disease Linkage studies
locus Population studies
Transgenic studies
Treatment studies
Association studies
Family studies
Pedigree studies
Outcome studies
Adoption studies
Implicate molecular
Linkage studies
pathways
Population studies
Transgenic studies
Treatment studies
Association studies
Family studies
Check

Explanation: 
Twin and adoption studies are useful to differentiate the effects of genes and environment.
The search for a disease gene typically begins with linkage analysis. This type of study helps
to find out the approximate location of the gene in relation to another DNA sequence (called a
genetic marker) whose position is already known. 

597
Transgenesis defines any process that involves the transfer of a gene from one species to
another. It is often used in the context of inserting modified mouse genes in the mouse
genome to study gene function. Transgenesis and gene targeting represent the two most
direct and powerful approaches for analysing gene function in higher organisms. 
Association studies are more straightforward where a case­control design is often adapted,
and a sample of cases affected by a disorder is compared with controls. The frequency of
alleles at the marker locus is then compared in the two groups. This method, though helpful to
identify the likelihood of a specific locus being associated with a disease, cannot make strong
causal inferences.
The correct answer is: Identify a specific disease locus – Association studies, Establish
chromosomal location of a disease locus – Linkage studies, Implicate molecular pathways –
Transgenic studies

Question 13 HiY Neurosciences EMI013
Correct Genetic syndromes
From the mechanisms and chromosomal loci below choose one right option from given list:
Mark 5.00 out of
5.00

Flag question
Fragile X
Myotonic dystrophy
Uniparental disomy Huntington's disease
and paternal deletion of
Prader­Willi syndrome
a locus at chromosome
15
Klinefelter's syndrome
Edward's syndrome
Down's syndrome
Turner's syndrome
Angelman's syndrome
Fragile X
Patau syndrome
Myotonic dystrophy
Huntington's disease
Robertsonian
Prader­Willi syndrome
translocation 14:21
Klinefelter's syndrome
Edward's syndrome
Down's syndrome
Turner's syndrome
Angelman's syndrome
Fragile X
Patau syndrome
Myotonic dystrophy
Huntington's disease
CTG trinucleotide
Prader­Willi syndrome
repeats
Klinefelter's syndrome
Edward's syndrome
Down's syndrome
Turner's syndrome
Angelman's syndrome
Fragile X
Patau syndrome
Myotonic dystrophy
Huntington's disease
Prader­Willi syndrome
Klinefelter's syndrome
Trisomy 13
Edward's syndrome
Down's syndrome
Turner's syndrome
Angelman's syndrome

598
Fragile X
Myotonic dystrophy
Huntington's disease
Trisomy 18 Prader­Willi syndrome
Klinefelter's syndrome
Edward's syndrome
Down's syndrome
Turner's syndrome
Angelman's syndrome
Check
Patau syndrome

Explanation: 
The Prader­Willi (PWS) and Angelman (AS) syndromes are phenotypically distinct but
genetically overlapping neurodevelopmental disorders associated with chromosome 15q11­
q13 imprinting. Nearly 70% of patients with PWS have a deletion in their paternally derived
15q11­q13. Maternal uniparental disomy (inheriting both copies from the mother when the
embryo is formed) occurs in most of the remaining patients (25%). Most patients with
Angelman's syndrome have a deletion in their maternally derived 15q11­q13. Paternal
uniparental disomy occurs in about 4% of Angelman's syndrome. 
The Robertsonian translocation 14:21 is unbalanced, and the baby has three copies of the
long arm of chromosome 21 instead of two. This causes translocation Down's syndrome. 
The myotonic dystrophy mutation, identified as an unstable deoxyribonucleic acid (DNA)
sequence (CTG) prone to increase the number of trinucleotide repeats, produces clinical
manifestations of the disease in skeletal muscle, the heart and many organ systems (Novelli
et al., 1993). 
Trisomy 13 causes Patau syndrome while 18 causes Edward's syndrome. 

Excerpt from Novelli G, et al., The dynamic genomics of myotonic dystrophy and its clinical
relevance: an overview. Biomed Pharmacother. 1993;47(8):321­30.
The correct answer is: Uniparental disomy and paternal deletion of a locus at chromosome 15
– Prader­Willi syndrome, Robertsonian translocation 14:21 – Down's syndrome, CTG
trinucleotide repeats – Myotonic dystrophy, Trisomy 13 – Patau syndrome, Trisomy 18 –
Edward's syndrome

Question 14 HiY Neurosciences EMI014
Correct Genetic deletion syndromes 
Choose the most appropriate deletion for each description below
Mark 4.00 out of
4.00

Flag question
Chr21p22
Chr15q11­13 paternally inherited
Obesity, short stature, Chr16p11
small limbs, decreased
Chr7q11
IQ with hyperphagia
and skin picking.
Chr1q4
Chr4q12
Chr15q11­13 maternally inherited
Chr 5p12
Chr17p12

599
Congenital Chr21p22
supravalvular aortic Chr15q11­13 paternally inherited
stenosis, elfin facies
Chr16p11
with microcephaly,
sometimes appears
Chr7q11
disinhibited in social Chr1q4
settings. Chr4q12
Chr15q11­13 maternally inherited
Chr 5p12
Chr17p12
Chr21p22
Developmental delay, Chr15q11­13 paternally inherited
low IQ, jerky Chr16p11
movements especially Chr7q11
hand­flapping, frequent Chr1q4
smiling, and seizures. Chr4q12
Chr15q11­13 maternally inherited
Chr 5p12
Chr17p12
Chr21p22
Feeding problems due Chr15q11­13 paternally inherited
to difficulty swallowing
Chr16p11
and sucking, cat­like
Chr7q11
cry with poorly
developed facial
Chr1q4
features. Chr4q12
Chr15q11­13 maternally inherited
Chr 5p12
Chr17p12
Check

Explanation: 
Skin picking associated with Lesch­Nyhan syndrome, Prader­Willi syndrome, or mental
retardation.Hyperphagia/food foraging/obsession with food along with a short stature in case
1 supports a diagnosis of Prader­Willi Syndrome. Paternal 15q11 deletion is associated with
this condition. 
Williams's syndrome (case 2) is a developmental disorder classically characterized by the
triad of typical facial features, infantile hypercalcemia, and supravalvular aortic stenosis.
Deletion of elastin gene at chromosome 7q11 is associated with both the familial autosomal
dominant pattern and the more frequent de novo sporadic cases of this syndrome. 
The earliest description of the Angelman syndrome (case 3) by the physician Harry Angelman
in 1965, included several children having "flat heads, jerky movements, protruding tongues,
and bouts of laughter." 
Cri du chat syndrome is associated with a deletion in chromosome 5.
The correct answer is: Obesity, short stature, small limbs, decreased IQ with hyperphagia and
skin picking. – Chr15q11­13 paternally inherited, Congenital supravalvular aortic stenosis,
elfin facies with microcephaly, sometimes appears disinhibited in social settings. – Chr7q11,
Developmental delay, low IQ, jerky movements especially hand­flapping, frequent smiling,
and seizures. – Chr15q11­13 maternally inherited, Feeding problems due to difficulty
swallowing and sucking, cat­like cry with poorly developed facial features. – Chr 5p12

Question 15 HiY Neurosciences EMI015

600
Partially correct Types of genetic studies 
Mark 3.75 out of
Choose the appropriate methods of answering each of the following questions from the list
provided:
5.00

Flag question
Twin studies
Adoption studies
You are interested in
Association analysis
finding out whether a
Cohort study
condition is familial or
not. (Choose ONE)
Family study
Linkage analysis
Positive eugenics
Population studies
Twin studies
What is the relative Adoption studies
contribution of genetic Association analysis
and environmental Cohort study
factors in this disease? Family study
(TWO options) Linkage analysis
Positive eugenics
Population studies
Twin studies
A researcher wants to Adoption studies
localize the genes Association analysis
responsible for a Cohort study
disease or condition Family study
(Choose ONE) Linkage analysis
Positive eugenics
Population studies
Twin studies
Adoption studies
A researcher wants to
Association analysis
study the life time
Cohort study
incidence of a genetic
disease (Choose ONE)
Family study
Linkage analysis
Positive eugenics
Population studies
Check

Explanation: The family study method is a simple genetic investigation. It involves acquiring
psychiatric history related to the proband's family. A comparison can be then made as to how
many relatives are affected in one group compared to another. Twin and adoption studies are
useful to differentiate the effects of genes and environment. The search for a disease gene
typically begins with linkage analysis. This type of study helps to find out the rough location of
the gene relative to another DNA sequence (called a genetic marker), which has its position
already. Studying the whole population, not a selected group of high­risk pedigrees or
families, is required to establish lifetime incidence of a disease.
http://genome.wellcome.ac.uk/doc_WTD020778.html
The correct answer is: You are interested in finding out whether a condition is familial or not.

601
(Choose ONE) – Family study, What is the relative contribution of genetic and environmental
factors in this disease? (TWO options) – Twin studies, Adoption studies, A researcher wants to
localize the genes responsible for a disease or condition (Choose ONE) – Linkage analysis, A
researcher wants to study the life time incidence of a genetic disease (Choose ONE) –
Population studies

Question 16 HiY Neurosciences EMI016
Partially correct Genes implicated in neuropsychiatric disorders
For each condition below choose the appropriate gene/s from the list provided
Mark 6.00 out of
8.00

Flag question
LRRK2
PINK1
Alpha­synuclein
CADASIL (ONE) MAPT
CYP3A4
CFTR
NOTCH3
5HTT
DISC1
LRRK2
PINK1
Alpha­synuclein
Schizophrenia (ONE) MAPT
CYP3A4
CFTR
NOTCH3
5HTT
DISC1
LRRK2
PINK1
Alpha­synuclein
Parkinson's disease
MAPT
(THREE)
CYP3A4
CFTR
NOTCH3
5HTT
DISC1
LRRK2
PINK1
Alpha­synuclein
FTD (ONE) MAPT
CYP3A4
CFTR
NOTCH3
5HTT
DISC1
Check

Explanation: 

602
Genetic studies of Parkinson's (PD) implicate genes including SNCA (4q21 ­ encodes a­
synuclein), Leucine­rich repeat kinase 2 (LRRK2), parkin and PTEN­induced kinase 1
(PINK1). 
Mutations in the gene encoding the microtubule­associated protein tau gene (MAPT) linked to
chromosome 17q21 have been known to produce familial FTD associated with
Parkinsonism,amyotrophy, disinhibition, and dementia (FTDP­17) (Rosenberg, 2007).
Another mutation at a proximal site on the same chromosome involves progranulin and
results in ubiquitin­positive inclusions rather than tauopathypathology. Arch Neurol. 2008;
65(4):460­464. 
CADASIL (Cerebral Autosomal Dominant Arteriopathy with Sub­cortical Infarcts and
Leukoencephalopathy) is "an inherited form a of cerebrovascular disease that occurs when
the thickening of blood vessel walls blocks the flow of blood to the brain". Through positional
cloning, Notch3 was found to be the gene responsible for the disorder, and mutations have
been demonstrated in more than 90% of CADASIL patients. 
Dysbindin, DISC 1 and neuregulin are associated with schizophrenia.

Excerpt retrieved from http://www.ninds.nih.gov/disorders/cadasil/CADASIL.htm and
http://www.jci.org/cgi/content/full/105/5/561 
Rosenberg RN. Progranulin and Tau Gene Mutations Both as Cause for Dementia: 17q21
Finally Defined. Arch Neurol. 2007;64(1):18­19.
The correct answer is: CADASIL (ONE) – NOTCH3, Schizophrenia (ONE) – DISC1,
Parkinson's disease (THREE) – LRRK2, PINK1, Alpha­synuclein, FTD (ONE) – MAPT

Question 17 HiY Neurosciences EMI017
Correct Inheritance of diseases
Choose one best option for the most common mechanism from the given list for each
Mark 4.00 out of
description below:
4.00

Flag question
X­linked dominant
Tumours on the surface Genomic imprinting
of the brain with patchy Non disjunction of chromosomes
sclerosis of the brain Spontaneous mutations
surface. Visceral Environmental disorder
tumours are also noted X­linked recessive
Mitochondrial inheritance
Autosomal dominant
Autosomal recessive
X­linked dominant
Progressive dementia Genomic imprinting
and movement disorder Non disjunction of chromosomes
beginning in middle
Spontaneous mutations
adulthood; depression
and frank psychosis
Environmental disorder
may also occur X­linked recessive
Mitochondrial inheritance
Autosomal dominant
Autosomal recessive
Hyperuricemia,
movement disorder and
intense self­mutilation
in a stereotyped

603
fashion. Impaired renal X­linked dominant
function and cerebral
Genomic imprinting
palsy are also noted.
Non disjunction of chromosomes
Spontaneous mutations
Environmental disorder
X­linked recessive
Mitochondrial inheritance
Autosomal dominant
Autosomal recessive
X­linked dominant
Male phenotype but Genomic imprinting
infertile with eunuchoid Non disjunction of chromosomes
appearance. Small
Spontaneous mutations
testes with Barr body
on cytological
Environmental disorder
examination X­linked recessive
Mitochondrial inheritance
Autosomal dominant
Autosomal recessive
Check

Explanation: Tuberous sclerosis most commonly occurs as a result of spontaneous mutations,
though an autosomal dominant pattern is noted in familial cases. Huntington disease is
trinucleotide repeat disease with autosomal dominant inheritance. Lesch­Nyhan syndrome is
an X­linked recessive condition. Kleinfelter syndrome results from an aneuploidy due to
nondisjunction.
The correct answer is: Tumours on the surface of the brain with patchy sclerosis of the brain
surface. Visceral tumours are also noted – Spontaneous mutations, Progressive dementia
and movement disorder beginning in middle adulthood; depression and frank psychosis may
also occur – Autosomal dominant, Hyperuricemia, movement disorder and intense self­
mutilation in a stereotyped fashion. Impaired renal function and cerebral palsy are also noted.
– X­linked recessive, Male phenotype but infertile with eunuchoid appearance. Small testes
with Barr body on cytological examination – Non disjunction of chromosomes

Question 18 HiY Neurosciences EMI018
Correct Genetic variations in disease 
For the following genetic changes, identify the associated disorder from the given list.
Mark 4.00 out of
4.00

Flag question
Familial Alzheimer's disease
Prader­Willi syndrome
CJD
Deletion in
Fragile X syndrome
chromosome 15
Downs syndrome
Frontotemporal dementia
Patau syndrome
Edward syndrome
Schizophrenia
Lesch­Nyhan syndrome

604
Trisomy 18 Familial Alzheimer's disease
Prader­Willi syndrome
CJD
Fragile X syndrome
Downs syndrome
Frontotemporal dementia
Patau syndrome
Edward syndrome
Schizophrenia
Familial Alzheimer's disease
Lesch­Nyhan syndrome
Prader­Willi syndrome
Chromosome X CJD
deletion affecting Fragile X syndrome
purine metabolism Downs syndrome
Frontotemporal dementia
Patau syndrome
Edward syndrome
Schizophrenia
Familial Alzheimer's disease
Lesch­Nyhan syndrome
Prader­Willi syndrome
CJD
ZNF804A
Fragile X syndrome
polymorphism
Downs syndrome
Frontotemporal dementia
Patau syndrome
Edward syndrome
Schizophrenia
Check
Lesch­Nyhan syndrome

Explanation: 
Lesch­Nyhan syndrome is an extremely rare condition, due to a mutation in HPRT gene
(hypoxanthine­guanine phosphoribosyl transferase) on the short arm of chromosome Xq26­
27, with a nearly total loss of the enzyme leading to hyperuricaemia. 
ZNF804A is one of the strongest candidate genes for schizophrenia observed in genome­
wide association studies, yet its physiological role in disease pathophysiology is currently
unknown. 
Excerpt retrieved from http://www.ncbi.nlm.nih.gov/pubmed/20664580
The correct answer is: Deletion in chromosome 15 – Prader­Willi syndrome, Trisomy 18 –
Edward syndrome, Chromosome X deletion affecting purine metabolism – Lesch­Nyhan
syndrome, ZNF804A polymorphism – Schizophrenia

Question 19 HiY Neurosciences EMI019
Correct Cell biology
Using descriptions given below, identify the type of enzymes from the list
Mark 3.00 out of
3.00

Flag question

605
Used in experiments to
Aminoacyl t­RNA synthetase
decrease gene
Si­ RNA
expression
r­RNA
DNA Polymerase
t­RNA
RNA Polymerase
m­RNA
Double stranded DNA
pre m­RNA
Aminoacyl t­RNA synthetase
Single­stranded DNA
Si­ RNA
Provides a bridge r­RNA
between m­RNA and DNA Polymerase
amino acids t­RNA
RNA Polymerase
m­RNA
Double stranded DNA
pre m­RNA
Aminoacyl t­RNA synthetase
Single­stranded DNA
Si­ RNA
Enzymes which r­RNA
attaches amino acids to DNA Polymerase
t­RNA t­RNA
RNA Polymerase
m­RNA
Double stranded DNA
pre m­RNA
Check
Single­stranded DNA

Explanation: 
Small interfering RNA (Si­RNA) is a class of double­stranded RNA molecule, involving in RNA
interference pathway, where it interferes with expression of a specific gene.
T­RNA is a small RNA molecule that transfers a specific active aminoacid to growing
polypeptide chain at the ribosomal site of protein synthesis during translation. It also has a
terminal site for amino acid attachment.
Aminoacyl t­RNA synthetase is an enzyme that catalyze the esterification of a specific amino
acid or its precursors to one of all its compatible cognate t­RNA to form an aminoacyl t­RNA.
The correct answer is: Used in experiments to decrease gene expression – Si­ RNA, Provides
a bridge between m­RNA and amino acids – t­RNA, Enzymes which attaches amino acids to
t­RNA – Aminoacyl t­RNA synthetase

Question 20 HiY Neurosciences EMI020
Correct Clinical genetics
Choose one option for each of the following descriptions
Mark 5.00 out of
5.00

Flag question

606
This occurs exclusively Williams syndrome
in females and the
Fragile X syndrome
gene associated is
MECP2 at Xq28
Angelman syndrome
Prader Willi syndrome
Turner's syndrome
Down's syndrome
Lesch­Nyhan syndrome
Rett's syndrome
Neurofibromatosis
Williams syndrome
The penetrance is Velocardiofacial syndrome
Fragile X syndrome
greater in males than
Angelman syndrome
females. The gene
Prader Willi syndrome
sequence has been
cloned and designated
Turner's syndrome
as FMR­1 Down's syndrome
Lesch­Nyhan syndrome
Rett's syndrome
Neurofibromatosis
Williams syndrome
Congenital disorder
caused by a micro
Velocardiofacial syndrome
Fragile X syndrome
deletion of Angelman syndrome
chromosome 22, Prader Willi syndrome
associated with Turner's syndrome
learning disability and Down's syndrome
schizophrenia Lesch­Nyhan syndrome
Rett's syndrome
Neurofibromatosis
Williams syndrome
Autosomal dominant Velocardiofacial syndrome
Fragile X syndrome
condition with NFI gene Angelman syndrome
17q11.2 involved. The
Prader Willi syndrome
gene product is thought
to be a tumour
Turner's syndrome
suppressor gene Down's syndrome
Lesch­Nyhan syndrome
Rett's syndrome
Neurofibromatosis
Williams syndrome
Velocardiofacial syndrome
Fragile X syndrome
X­linked recessive Angelman syndrome
condition due to a
Prader Willi syndrome
mutation in HGPRT
Turner's syndrome
gene
Down's syndrome
Lesch­Nyhan syndrome
Rett's syndrome
Neurofibromatosis
Check Velocardiofacial syndrome

Explanation: Rett syndrome occurs only in females. Although the etiology is unknown it is
though to be associated with MECP2 at Xq28. 
Fragile X syndrome is the most common inherited cause of learning disability. The penetrance
is greater in males than females. The fragile X mental retardation 1 (FMR­1) gene is
responsible for fragile X syndrome caused by abnormal trinucleotide CGG repeat at a fragile

607
site on the X chromosome (Xq27.3). 
Velocardiofacial syndrome is also known as DiGeorge's syndrome. It is a congenital disorder
caused by a microdeletion of chromosome 22. 
Neurofibromatosis: Autosomal dominant condition with 50% new mutations. More than 90%
patients have paternal NF1 allele mutated. NFL gene is located at17q11.2. The gene product
is neurofibromin, thought to be tumour suppressor gene. 
Lesch Nyhan syndrome is an X­linked recessive condition due to a mutation in HGPRT gene
on the short arm of chromosome Xq26­27. There is a defect in hypoxanthine guanine
phosphoribosyl­transferase with an accumulation of uric acid; Note that a partial HGPRT
deficiency results in recurrent gout.
The correct answer is: This occurs exclusively in females and the gene associated is MECP2
at Xq28 – Rett's syndrome, The penetrance is greater in males than females. The gene
sequence has been cloned and designated as FMR­1 – Fragile X syndrome, Congenital
disorder caused by a micro deletion of chromosome 22, associated with learning disability
and schizophrenia – Velocardiofacial syndrome, Autosomal dominant condition with NFI gene
17q11.2 involved. The gene product is thought to be a tumour suppressor gene –
Neurofibromatosis, X­linked recessive condition due to a mutation in HGPRT gene – Lesch­
Nyhan syndrome

Question 21 HiY Neurosciences EMI021
Partially correct Location of genetic deletions
Identify the site of deletion that has occurred in each of the following disorders. The numbers
Mark 4.00 out of
provided in the list correspond to human chromosomal numbers.
5.00

Flag question
7
17
22
Angelman syndrome 1
5
15
13
11
9
7
3
17
22
Criduchat syndrome 1
5
15
13
11
9
7
3
17
22
1
5
15
Velocardiofacial 13
syndrome
11
9
3

608
7
17
22
Smith Magenis
1
syndrome
5
15
13
11
9
7
3
17
22
Williams syndrome 1
5
15
13
11
9
Check
3

Explanation: Angelman syndrome is caused due to a deletion in 15q12 of maternal origin;
80% due to deletion of maternally derived chromosome 15. In Prader­Willi syndrome there is
a deletion in 15q12 of paternal origin; 75% deletion of paternally derived chromosome 15. 
Criduchat syndrome is due to a partial deletion 5p15.2.
The velocardiofacial syndrome is associated with micro deletion of chromosome 22q11.2;
90% arise denovo, with 10% having an affected parent. 
Smith­Magenis syndrome is associated with a complete or partial deletion of 17p11.2.
Williams's syndrome is also known as 'hypercalcemia with supravalvular aortic stenosis'.
Clinical features include short stature, growth retardation, unusual facial features including a
broad forehead, elfin like facies, hoarse voice, and cardiovascular abnormalities
(supravalvular aortic stenosis). Verbal skills are often better than motor and visual spatial
skills.
The correct answer is: Angelman syndrome – 15, Criduchat syndrome – 5, Velocardiofacial
syndrome – 22, Smith Magenis syndrome – 17, Williams syndrome – 7

Question 22 HiY Neurosciences EMI022
Correct Neuroanatomical regions
Identify one brain area that is most important in the following functions;
Mark 4.00 out of
4.00

Flag question

609
Reward processing Dorsolateral prefrontal cortex
Nucleus Accumbens
Cerebellum
Fusiform gyrus  
Amygdala
Inferior olivary nucleus
Hippocampus
Angular gyrus
Medulla
Dorsolateral prefrontal cortex
Left inferior frontal area
Nucleus Accumbens
Cerebellum
 
Emotions Fusiform gyrus
Amygdala
Inferior olivary nucleus
Hippocampus
Angular gyrus
Medulla
Dorsolateral prefrontal cortex
Left inferior frontal area
Nucleus Accumbens
Cerebellum
 
Dysdiadachokinesia Fusiform gyrus
Amygdala
Inferior olivary nucleus
Hippocampus
Angular gyrus
Medulla
Dorsolateral prefrontal cortex
Left inferior frontal area
Nucleus Accumbens
Cerebellum
 
Executive functions Fusiform gyrus
Amygdala
Inferior olivary nucleus
Hippocampus
Angular gyrus
Medulla
Check
Left inferior frontal area

Explanation: 
The area of the brain implicated in reward system regulation is nucleus accumbens. The area
of the brain involved in mediating emotional response such as fear the is amygdala. 
Cerebellum has the important role of preparing a motor plan and predicting balance needed
between muscle groups to carry out the intended action smoothly. Cerebellar lesions produce
ataxia, dysdiadochokinesia, coarse intentional tremors, along with hypotonia, past pointing
and pendular knee jerk. Dysdiadochokinesia refrs to impaired ability to perform rapid,
alternating movements.
The dorsolateral prefrontal cortex is responsible for executive functions such as planning,
judgement, and decision­making.
The correct answer is: Reward processing – Nucleus Accumbens, Emotions – Amygdala,
Dysdiadachokinesia – Cerebellum, Executive functions – Dorsolateral prefrontal cortex

610
Question 23 HiY Neurosciences EMI023
Partially correct Neuropsychology 
A trainee psychologist is keen to test different areas of cognitive domains. Choose the most
Mark 2.00 out of
appropriate test(s) for each of the following brain functions
8.00

Flag question
Tower of London test
Wisconsin card sorting test
Rey complex figure test
Executive functions
Clock drawing test
(Choose TWO)
Continuous performance test
National adult reading test
Rivermead test
Paired associates test
Russell test
Tower of London test
Wisconsin card sorting test
Rey complex figure test
Attention (Choose
Clock drawing test
ONE)
Continuous performance test
National adult reading test
Rivermead test
Paired associates test
Russell test
Tower of London test
Wisconsin card sorting test
Rey complex figure test
Visuospatial function
Clock drawing test
(Choose TWO)
Continuous performance test
National adult reading test
Rivermead test
Paired associates test
Russell test
Tower of London test
Wisconsin card sorting test
Rey complex figure test
Intelligence (Choose
Clock drawing test
ONE)
Continuous performance test
National adult reading test
Rivermead test
Paired associates test
Russell test
Check

Explanation: 
The Tower of London test and Wisconsin Card Sorting Test are well­known tests used in
applied clinical neuropsychology for the assessment of executive functioning specifically to
detect deficits in planning. 
Attention can be tested in a number of ways including serial 7s, digit span, spelling "world"
backwards, and recitation of the months of the year in reverse order or continuous
performance tests. 
Clock drawing task and Rey complex figure test are useful to test visuospatial functions. 

611
The Wechsler Adult Intelligence Scale (WAIS) is the most widely used intelligence test in
clinical practice. National adult reading test taps on previous word knowledge before
becoming ill ­ hence used to estimate premorbid IQ
The correct answer is: Executive functions (Choose TWO) – Tower of London test, Wisconsin
card sorting test, Attention (Choose ONE) – Continuous performance test, Visuospatial
function (Choose TWO) – Rey complex figure test, Clock drawing test, Intelligence (Choose
ONE) – National adult reading test

Question 24 HiY Neurosciences EMI024
Correct Neuroanatomical localization 
Identify one brain area that is most important in the following functions;
Mark 4.00 out of
4.00

Flag question
Left inferior frontal cortex
Inferior olivary nucleus
Amygdala
Fear Nucleus Accumbens
Precentral cortex
Hippocampus
Angular gyrus
Fusiform gyrus
Cerebellum
Left inferior frontal cortex
Posterior superior temporal lobe
Inferior olivary nucleus
Amygdala
Production of speech Nucleus Accumbens
Precentral cortex
Hippocampus
Angular gyrus
Fusiform gyrus
Cerebellum
Left inferior frontal cortex
Posterior superior temporal lobe
Inferior olivary nucleus
Amygdala
Comprehension of
Nucleus Accumbens
language
Precentral cortex
Hippocampus
Angular gyrus
Fusiform gyrus
Cerebellum
Left inferior frontal cortex
Posterior superior temporal lobe
Inferior olivary nucleus
Amygdala
Initiation of physical Nucleus Accumbens
movement Precentral cortex
Hippocampus
Angular gyrus
Fusiform gyrus
Cerebellum
Check Posterior superior temporal lobe

612
Explanation: Amygdala plays an important role in emotional learning, fear conditioning and
modulation of memory consolidation. 
Broca's area, which is linked to the production of speech, is located in the posterior part of the
inferior frontal gyrus in the dominant hemisphere; destruction results in expressive dysphasia. 
Wernicke's area is located in the posterior section of the superior temporal gyrus in the
dominant cerebral hemisphere.
The frontal lobe can be anatomically divided into the precentral cortex, prefrontal cortex and
the orbitofrontal cortex. 
The precentral cortex lodges the primary motor cortex and is involved in motor planning,
initiation and control.
The correct answer is: Fear – Amygdala, Production of speech – Left inferior frontal cortex,
Comprehension of language – Posterior superior temporal lobe, Initiation of physical
movement – Precentral cortex

Question 25 HiY Neurosciences EMI025
Partially correct Neuroanatomy­ Connecting bundles
Identify the tracts, which plays a potential role in each of the following;
Mark 3.33 out of
5.00

Flag question
Superior longitudinal fasciculus
Uncinate fasciculus
The tract of fibres that Anterior commissure
connect frontal and
Habenular commissure
temporal lobes
(Choose TWO)
Inferior colliculi
Corticospinal tract
Inferior longitudinal fasciculus
Ventromedial tract
Superior colliculi
Superior longitudinal fasciculus
Uncinate fasciculus
A dominant structure Anterior commissure
involved in stereotactic
Habenular commissure
brain mapping (Choose
ONE)
Inferior colliculi
Corticospinal tract
Inferior longitudinal fasciculus
Ventromedial tract
Superior colliculi
Superior longitudinal fasciculus
Uncinate fasciculus
This tract plays a Anterior commissure
crucial role in object
Habenular commissure
recognition (Choose
ONE)
Inferior colliculi
Corticospinal tract
Inferior longitudinal fasciculus
Ventromedial tract
Superior colliculi
Check

Explanation: 

613
The uncinate fasciculus (Green in the image below) is a 'U' shaped white matter tract in the
brain that connects part of limbic system such as temporal pole, hippocampus and amygdala
in the medial temporal lobe with prefrontal regions including the ventromedial and
orbitofrontal cortex. It also connects the inferior frontal gyrus with temporal lobe. The superior
longitudinal fasciculus (sky­blue in the image below) is a long anteroposterior association
fibre bundle that is lateral to the centrum ovale in each hemisphere and connects the frontal,
temporal, parietal and occipital lobes. 

(Diffusion Tensor Image from Jang 2010: Frontiers in Human Neuroscience.
http://journal.frontiersin.org/Journal/10.3389/fnhum.2013.00749/full).
The anterior commissure (AC) and posterior commissure (PC) are two dominant anatomic
structures serve as indirect localization of intracerebral anatomy and is crucial in stereotactic
brain mapping. The inter commissural line has also been used as a reliable reference line for
stereotactic surgery. In terms of brain mapping in stereotactic surgery the gold standard
continues to rely on probabilistic and indirect targeting, relative to a stereotactic reference, i.e.,
mostly the anterior (AC) and the posterior (PC) commissures. For a visual representation, see
http://en.wikipedia.org/wiki/File:Gray720.png.
The Inferior longitudinal fasciculus (orange in the image below) plays a potential role in object
recognition. Studies using diffusion tensor imaging suggest an association between integrity
loss and object recognition deficits. Moreover, the severity of clinical impairment is reflected in
the degree of loss of ILF integrity. (Ref: Orbitus et al., Dev Med Child Neurol. 2012
Jan;54(1):38­43.)
The correct answer is: The tract of fibres that connect frontal and temporal lobes (Choose
TWO) – Superior longitudinal fasciculus, Uncinate fasciculus, A dominant structure involved in
stereotactic brain mapping (Choose ONE) – Anterior commissure, This tract plays a crucial
role in object recognition (Choose ONE)
– Inferior longitudinal fasciculus

Question 26 HiY Neurosciences EMI026
Correct EEG changes in neuropsychiatry
For each of the following psychiatric conditions, identify the EEG changes most likely to occur
Mark 5.00 out of
5.00

Flag question

614
Creutzfeldt­Jacob
Periodic generalized 1­2 Hz sharp waves
disease
Episodic discharges recurring every 1­3 seconds
Non­specific increase in slow waves
Focal slow wave activity
Generalized multiple spike­wave discharges
Triphasic waves
Flattened trace
Regular 3 Hz complexes
Focal delta activity
Periodic generalized 1­2 Hz sharp waves
Diffuse giant waves
Episodic discharges recurring every 1­3 seconds
Non­specific increase in slow waves
Absence seizures Focal slow wave activity
Generalized multiple spike­wave discharges
Triphasic waves
Flattened trace
Regular 3 Hz complexes
Focal delta activity
Periodic generalized 1­2 Hz sharp waves
Diffuse giant waves
Episodic discharges recurring every 1­3 seconds
Non­specific increase in slow waves
Myoclonic epilepsy Focal slow wave activity
Generalized multiple spike­wave discharges
Triphasic waves
Flattened trace
Regular 3 Hz complexes
Focal delta activity
Periodic generalized 1­2 Hz sharp waves
Diffuse giant waves
Episodic discharges recurring every 1­3 seconds
Non­specific increase in slow waves
Huntington's dementia Focal slow wave activity
Generalized multiple spike­wave discharges
Triphasic waves
Flattened trace
Regular 3 Hz complexes
Focal delta activity
Periodic generalized 1­2 Hz sharp waves
Diffuse giant waves
Episodic discharges recurring every 1­3 seconds
Non­specific increase in slow waves
Hepatic
Focal slow wave activity
encephalopathy
Generalized multiple spike­wave discharges
Triphasic waves
Flattened trace
Regular 3 Hz complexes
Focal delta activity
Check
Diffuse giant waves

Explanation: 
CJD­ Generalized periodic 1­2 Hz sharp waves are seen in nearly 90% patients with sporadic
CJD. Less often in familial / hormonal transplant­related forms. It is not seen in the variant
form. 
Absence seizures (Petit mal seizures)­ Regular 3 Hz complexes. 

615
Myoclonic epilepsy­ Generalized multiple spike­wave discharges. 
Huntington's disease­ an initial loss of alpha; later flattened trace. 
Diffuse generalized slowing in metabolic and endocrine disorders. 
Triphasic waves: 1.5 to 3.0 per second high­voltage slow­waves especially in hepatic
encephalopathy
The correct answer is: Creutzfeldt­Jacob disease – Periodic generalized 1­2 Hz sharp waves,
Absence seizures – Regular 3 Hz complexes, Myoclonic epilepsy – Generalized multiple
spike­wave discharges, Huntington's dementia – Flattened trace, Hepatic encephalopathy –
Triphasic waves

Question 27 HiY Neurosciences EMI027
Partially correct EEG changes and psychiatric presentations
For each of the following conditions, identify the most likely EEG findings reported
Mark 4.00 out of
5.00

Flag question
Large 8­12 Hz spikes
in groups during clonus
No characteristic pattern
Alzheimer's disease  
Immature posterior temporal slow waves
Some alpha and generalised increased delta waves
Flattened trace
Reduced alpha waves
3 Hz Spike and waves
Large 8­12 Hz spikes
Bilateral spike and slow wave complexes
in groups during clonus
Large localised spikes or sharp waves
No characteristic pattern
Partial seizures Paroxysms of triphasic waves  
Immature posterior temporal slow waves
Some alpha and generalised increased delta waves
Flattened trace
Reduced alpha waves
3 Hz Spike and waves
Large 8­12 Hz spikes
Bilateral spike and slow wave complexes
in groups during clonus
Large localised spikes or sharp waves
No characteristic pattern
Schizophrenia Paroxysms of triphasic waves  
Immature posterior temporal slow waves
Some alpha and generalised increased delta waves
Flattened trace
Reduced alpha waves
3 Hz Spike and waves
Large 8­12 Hz spikes
Bilateral spike and slow wave complexes
in groups during clonus
Large localised spikes or sharp waves
No characteristic pattern
Paroxysms of triphasic waves
Immature posterior temporal slow waves  
Some alpha and generalised increased delta waves
Psychopathy Flattened trace
Reduced alpha waves
3 Hz Spike and waves
Bilateral spike and slow wave complexes
Large localised spikes or sharp waves
Paroxysms of triphasic waves

616
Large 8­12 Hz spikes
Delirium in groups during clonus
No characteristic pattern
Immature posterior temporal slow waves  
Some alpha and generalised increased delta waves
Flattened trace
Reduced alpha waves
3 Hz Spike and waves
Check Bilateral spike and slow wave complexes
Large localised spikes or sharp waves
Paroxysms of triphasic waves
Explanation:
In Alzheimer's disease, there is reduced alpha wave. EEG is rarely normal in advanced
dementia, which may be helpful in differentiating pseudo­dementia from dementia. 
In partial seizures, there are largely localised spike or spike­wave discharges. 
There is no characteristic EEG pattern reported in schizophrenia. 
There is increased incidence of EEG abnormalities in people with aggressive behaviour,
antisocial personality disorder and psychopathic disorders. Immature posterior temporal slow
waves are reported in psychopathy. 
In delirium, there are generalised increased delta waves and some alpha wave activity in
EEG.
The correct answer is: Alzheimer's disease – Reduced alpha waves, Partial seizures – Large
localised spikes or sharp waves, Schizophrenia – No characteristic pattern, Psychopathy –
Immature posterior temporal slow waves, Delirium – Some alpha and generalised increased
delta waves

Question 28 HiY Neurosciences EMI028
Correct EEG waves
Match the frequencies with their EEG wave forms;
Mark 5.00 out of
5.00

Flag question
Delta waves
Theta waves
Alpha waves
<4 Hz Spike waves
Beta waves
Omega waves
Mu waves
Focal activity
Single waves
Delta waves
Lambda waves
Theta waves
Alpha waves
>13 Hz Spike waves
Beta waves
Omega waves
Mu waves
Focal activity
Single waves
Lambda waves

617
Delta waves
8­13 Hz
Theta waves
Alpha waves
Spike waves
Beta waves
Omega waves
Mu waves
Focal activity
Single waves
Delta waves
Lambda waves
Theta waves
Alpha waves
4­8 Hz, occurring
Spike waves
during sleep
Beta waves
Omega waves
Mu waves
Focal activity
Single waves
Delta waves
Lambda waves
Theta waves
Alpha waves
7­11 Hz, occurs over
Spike waves
motor cortex
Beta waves
Omega waves
Mu waves
Focal activity
Single waves
Check
Lambda waves

Explanation: 
Delta waves (<4 Hz): These are not seen in waking EEG. It is common in deeper stages of
sleep; the presence of focal/generalized delta in awake EEG is a sign of pathology. 
Beta waves (> 13 Hz) are seen at a frontocentral position in the normal waking EEG. 
Alpha waves (8­13 Hz) are dominant brain wave frequency, which occurs when eyes closed
and relaxing. It disappears with anxiety, arousal, eye opening or focused attention. The
Dominance of alpha waves reduces with age. 
Theta waves (4­8 Hz) are a small amount of sporadic waves seen in wake EEG at the fronto­
temporal area. It is prominent in drowsy or sleep EEG. Excessive theta in awake EEG is a sign
of pathology. 
Mu waves (7­11 Hz) occurs over the motor cortex. It is related to motor activity, characterised
by arch like waves; gets attenuated by a movement of the contralateral limb.
The correct answer is: <4 Hz – Delta waves, >13 Hz – Beta waves, 8­13 Hz – Alpha waves, 4­
8 Hz, occurring during sleep – Theta waves, 7­11 Hz, occurs over motor cortex – Mu waves

Question 29 HiY Neurosciences EMI029
Correct EEG & Psychotropics
Identify EEG changes caused by each of the following classes of psychotropic medications;
Mark 9.00 out of
9.00

618
Flag question
Decrease in theta and delta activity
Increased beta but decreased alpha activity
Antipsychotics  No effect on awake EEG
Diffuse slowing of all activity  
Increase in alpha and slow wave activity
Generalised spike discharges
Increase in mu and lambda activity
Decrease in theta and delta activity
Increased beta but decreased alpha activity
No effect on awake EEG
Antidepressants   
Diffuse slowing of all activity
Increase in alpha and slow wave activity
Generalised spike discharges
Increase in mu and lambda activity
Decrease in theta and delta activity
Increased beta but decreased alpha activity
No effect on awake EEG
Anticonvulsants  
Diffuse slowing of all activity
Increase in alpha and slow wave activity
Generalised spike discharges
Increase in mu and lambda activity
Decrease in theta and delta activity
Increased beta but decreased alpha activity
No effect on awake EEG
Benzodiazepines   
Diffuse slowing of all activity
Increase in alpha and slow wave activity
Generalised spike discharges
Increase in mu and lambda activity

Check

Explanation: 
With antipsychotics there is a slowing of beta activity with an increase in alpha, theta and delta
activity with antidepressants too there is a slowing of beta activity with an increase in alpha,
theta and delta activity. 
Lithium causes slowing of alpha and paroxysmal activity. 
Anticonvulsants have no effect on awake EEG. 
With benzodiazepines, there is an increase in beta and a decrease in alpha waves. Overdose
leads to diffuse slowing.
The correct answer is: Antipsychotics 
– Increase in alpha and slow wave activity, Antidepressants 
– Increase in alpha and slow wave activity, Anticonvulsants – No effect on awake EEG,
Benzodiazepines 
– Increased beta but decreased alpha activity

Question 30 HiY Neurosciences EMI030
Correct

619
Mark 9.00 out of EEG & Recreational drugs
9.00 Identify EEG changes caused by each of the following classes of psychotropic medications;
Flag question
Increased alpha activity
No effect on awake EEG
Slowing of beta activity
Alcohol  
Increase in theta and delta activity
Generalised spike discharges
Increased beta but decreased alpha activity
Decrease in theta activity
Decrease in delta activity
Increased alpha activity
Increased alpha and theta activity
No effect on awake EEG
Slowing of beta activity
Opioids   
Increase in theta and delta activity
Generalised spike discharges
Increased beta but decreased alpha activity
Decrease in theta activity
Decrease in delta activity
Increased alpha activity
Increased alpha and theta activity
No effect on awake EEG
Slowing of beta activity
Marijuana  
Increase in theta and delta activity
Generalised spike discharges
Increased beta but decreased alpha activity
Decrease in theta activity
Decrease in delta activity
Increased alpha activity
Increased alpha and theta activity
No effect on awake EEG
Slowing of beta activity
Cocaine  
Increase in theta and delta activity
Generalised spike discharges
Increased beta but decreased alpha activity
Decrease in theta activity
Decrease in delta activity
Increased alpha activity
Increased alpha and theta activity
No effect on awake EEG
Slowing of beta activity
Barbiturates  
Increase in theta and delta activity
Generalised spike discharges
Increased beta but decreased alpha activity
Decrease in theta activity
Decrease in delta activity
Check Increased alpha and theta activity

Explanation: 
With primarily sedating drugs such as benzodiazepines, barbiturates and opioids, decreased
alpha wave activity is noted. With primarily recreational drugs such as alcohol, cannabis,
cocaine, nicotine and caffeine there is an increase in alpha wave activity. 
With alcohol, there will be increased alpha activity and increased theta activity. Withdrawal
from alcohol increases beta activity. Note that while delirium tremens is associated with beta
(fast) wave activity ­ other deliria have increased slow waves. 

620
Opioids­ Decreased alpha activity; increased voltage of theta and delta waves; in opioid
overdose, slow waves are predominant. 
Marijuana (cannabis) ­ there is increased alpha activity in the frontal area of the brain. 
Cocaine­ there is increased alpha activity. Barbiturates­ there is decreased alpha activity and
increased beta activity.
The correct answer is: Alcohol
– Increased alpha and theta activity, Opioids 
– Increase in theta and delta activity, Marijuana – Increased alpha activity, Cocaine –
Increased alpha activity, Barbiturates
– Increased beta but decreased alpha activity

Question 31 HiY Neurosciences EMI031
Partially correct Chemistry of Neurotransmitters
Match the following chemical groups with the neurotransmitters in the list
Mark 6.00 out of
9.00

Remove flag
Glutamate
Glycine
GABA
Serotonin  
Monoamines
Thyroxine
Estradiol
Coenzyme A
Cholecystokinin
Glutamate
Glycine
GABA
Aminoacids (3  
Serotonin
answers)
Thyroxine
Estradiol
Coenzyme A
Cholecystokinin
Glutamate
Glycine
GABA
Serotonin  
Peptides
Thyroxine
Estradiol
Coenzyme A
Cholecystokinin

Check

Explanation: 
Neurotransmitters include monoamines, aminoacids and peptides. 
Monoamines include dopamine, serotonin, norepinephrine, epinephrine, acetyl choline and
histamine.
Aminoacids include GABA, glycine and glutamate. 

621
Peptides include endorphins, cholecystokinin, angiotensin­2, neurotensin and corticotrophin­
releasing hormone.
The correct answer is: Monoamines – Serotonin, Aminoacids (3 answers) – Glutamate,
Glycine, GABA, Peptides
– Cholecystokinin

Question 32 HiY Neurosciences EMI032
Correct Enzymes involved in disorders
Choose one option from the above list of enzymes involved in each of the following disorders.
Mark 4.00 out of
4.00

Flag question
Monoamine oxidase B
Tryptophan hydroxylase
Catechol­o­methyl transferase
Alzheimer's disease Tyrosine Decarboxylase  
Acetylcholinesterase
Glycerate dehydrogenase
Tyrosine hydroxylase
Glutamic acid decarboxylase
Monoamine oxidase A
Monoamine oxidase B
Tryptophan hydroxylase
Catechol­o­methyl transferase
Huntington's  
Tyrosine Decarboxylase
disease
Acetylcholinesterase
Glycerate dehydrogenase
Tyrosine hydroxylase
Glutamic acid decarboxylase
Monoamine oxidase A
Monoamine oxidase B
Tryptophan hydroxylase
Catechol­o­methyl transferase
Tyrosine Decarboxylase  
Epilepsy
Acetylcholinesterase
Glycerate dehydrogenase
Tyrosine hydroxylase
Glutamic acid decarboxylase
Monoamine oxidase A
Monoamine oxidase B
Tryptophan hydroxylase
Catechol­o­methyl transferase
Tyrosine Decarboxylase  
Depression
Acetylcholinesterase
Glycerate dehydrogenase
Tyrosine hydroxylase
Glutamic acid decarboxylase
Monoamine oxidase A
Check

Explanation: 
Acetylcholinesterase is the breakdown enzyme involved in the synthesis of acetylcholine, and
reduced cholinergic function is involved in the pathogenesis of Alzheimer's disease. 

622
Glutamic acid decarboxylase is the enzyme involved in the production of GABA, which has an
important role in the etiology of many disorders like epilepsy and Huntington's disease. 
Tryptophan hydroxylase is the synthesis enzyme for serotonin, whose low levels are
associated with increased depression, aggression, suicide and impulsivity.
The correct answer is: Alzheimer's disease – Acetylcholinesterase, Huntington's disease –
Glutamic acid decarboxylase, Epilepsy – Glutamic acid decarboxylase, Depression –
Tryptophan hydroxylase

Question 33 HiY Neurosciences EMI033
Partially correct Transmitters and their receptors 
Choose the correct neurotransmitter involved in the activity of each of the following receptors
Mark 4.80 out of
6.00

Flag question
Glycine
Glutamate
GABA
NMDA receptor
Serotonin
(Choose TWO)
Endocannabinoids
Dopamine
Adrenaline
Opioids
Acetylcholine
Glycine
Glutamate
GABA
Muscarinic receptor Serotonin
Endocannabinoids
Dopamine
Adrenaline
Opioids
Acetylcholine
Glycine
Glutamate
GABA
Anandamide receptors Serotonin
Endocannabinoids
Dopamine
Adrenaline
Opioids
Acetylcholine
Glycine
Glutamate
GABA
Mu receptors Serotonin
Endocannabinoids
Dopamine
Adrenaline
Opioids
Acetylcholine

Nicotinic receptors

623
Glycine
Glutamate
GABA
Serotonin
Endocannabinoids
Dopamine
Adrenaline
Opioids
Acetylcholine
Check

Explanation: NMDA receptors are made up of subunits with distinct binding sites for
glutamate, glycine, and magnesium ion. Acetylcholine activates both muscarinic and nicotinic
receptors. 
Muscarinic receptors are G­protein­coupled, and there are five subtypes (M1, M2, M3, M4,
and M5). 
Nicotinic receptors are ion channels; they are predominantly seen in the peripheral
parasympathetic system and are less common than muscarinic receptors in CNS. Nicotinic
receptors in the CNS mediate attention. 
Anandamide (weak ligand) and 2­arachnidonylglycerol (strong) are formed from arachidonic
acid and ethanolamine. They both belong to the endocannabinoid family. The two types of
cannabinoid receptors, central (CB1) and peripheral (CB2), bind tetrahydrocannabinol (THC),
the active ingredient of marijuana. Anandamide lowers intraocular pressure, decreases
activity level, and relieves pain. 
Opiate receptors include the following subtypes: mu receptor, acted upon by morphine, opiate
alkaloids, b­endorphin, and enkephalins ,delta receptor acted upon by enkephalins, kappa
receptor acted upon by dynorphin and sigma receptor that is different from the other opiate
receptors in that naloxone does not reverse the agonist activity on this subtype.
The correct answer is: NMDA receptor (Choose TWO) – Glycine, Glutamate, Muscarinic
receptor – Acetylcholine, Anandamide receptors – Endocannabinoids, Mu receptors –
Opioids, Nicotinic receptors – Acetylcholine

Question 34 HiY Neurosciences EMI034
Correct Chemical precursors
Identify the precursors for the following neurotransmitters;
Mark 9.00 out of
9.00

Flag question
Mandelic acid
Adrenaline
5 Hydroxy Indole Acetic Acid
Precursors of
Tyrosine
dopamine
Choline
Anandamide
L­tryptophan
Histidine
Acetaldehyde

624
Precursor of serotonin  Mandelic acid
Adrenaline
5 Hydroxy Indole Acetic Acid
Tyrosine
Choline
Anandamide
L­tryptophan
Histidine
Acetaldehyde
Mandelic acid
Adrenaline
5 Hydroxy Indole Acetic Acid
Precursor of
Tyrosine
noradrenaline
Choline
Anandamide
L­tryptophan
Histidine
Acetaldehyde
Mandelic acid
Adrenaline
5 Hydroxy Indole Acetic Acid
Precursor of
Tyrosine
acetylcholine
Choline
Anandamide
L­tryptophan
Histidine
Acetaldehyde
Mandelic acid
Adrenaline
5 Hydroxy Indole Acetic Acid
Precursor of histamine Tyrosine
Choline
Anandamide
L­tryptophan
Histidine
Acetaldehyde
Check

Explanation: L­Dopa & tyrosine are the precursors for both dopamine and noradrenaline. The
source of serotonin is tryptophan from which 5­hydroxytryptophan is synthesized before 5­
hydroxy tryptamine (serotonin) is produced. The sources of acetylcholine are choline and
acetyl­coenzyme A. The precursor material for histamine is histidine.
The correct answer is: Precursors of dopamine – Tyrosine, Precursor of serotonin 
– L­tryptophan, Precursor of noradrenaline
– Tyrosine, Precursor of acetylcholine – Choline, Precursor of histamine – Histidine

Question 35 HiY Neurosciences EMI035
Correct Chemical transmitters
Identify the appropriate enzymes using descriptions below
Mark 3.00 out of
3.00

625
Flag question
Ghrelin
Insulin
Secreted by
hypothalamus and
Angiostatin
enhances food intake Neuropeptide Y
CCK
Melatonin
Gastrin
Leptin
Ghrelin
Insulin
Angiostatin
Secreted by stomach
Neuropeptide Y
and stimulates hunger
CCK
Melatonin
Gastrin
Leptin
Ghrelin
Insulin
Considered as an Angiostatin
internal measure of Neuropeptide Y
energy state CCK
Melatonin
Gastrin
Leptin

Check

Explanation: NPY is involved in the regulation of energy balance, memory and learning, and
epilepsy. NPY mediated activity increases food intake and the proportion of energy stored as
fat and decreases physical activity and nociceptive ability of the brain. It is secreted by the
hypothalamus. 
Ghrelin is produced by the fundus of the stomach and epsilon cells of the pancreas. Ghrelin
stimulates hunger; its levels increase before and decrease after having meals. Ghrelin has a
neurotrophic effect in the hippocampus and aids cognitive adaptation to changing
environments and learning.
Leptin plays a key role in regulating energy intake and expenditure, by regulating appetite
and metabolism. It is considered to be an internal measure of energy state, and is one of the
most important adipose derived hormones in human body.
The correct answer is: Secreted by hypothalamus and enhances food intake – Neuropeptide
Y, Secreted by stomach and stimulates hunger – Ghrelin, Considered as an internal measure
of energy state – Leptin

Question 36 HiY Neurosciences EMI036
Correct Breakdown enzymes for neurotransmitters
From the list above, identify the breakdown enzyme involved in the lifecycle of various
Mark 7.00 out of
neurotransmitters:
7.00

Flag question

626
Acetylcholinesterase
Tryptophan hydroxylase
Dopamine Dopamine beta­hydroxylase
Monoamine oxidase A
Transaminase
Monoamine oxidase B
Glutamic acid decarboxylase
Tyrosine hydroxylase
DOPA decarboxylase
Acetylcholinesterase
Choline acetyltransferase
Tryptophan hydroxylase
Dopamine beta­hydroxylase
Breaks down
Monoamine oxidase A
norepinephrine
Transaminase
Monoamine oxidase B
Glutamic acid decarboxylase
Tyrosine hydroxylase
DOPA decarboxylase
Acetylcholinesterase
Choline acetyltransferase
Tryptophan hydroxylase
Dopamine beta­hydroxylase
Breaks down serotonin Monoamine oxidase A
Transaminase
Monoamine oxidase B
Glutamic acid decarboxylase
Tyrosine hydroxylase
DOPA decarboxylase
Acetylcholinesterase
Choline acetyltransferase
Tryptophan hydroxylase
Located on the Dopamine beta­hydroxylase
postsynaptic Monoamine oxidase A
membrane Transaminase
Monoamine oxidase B
Glutamic acid decarboxylase
Tyrosine hydroxylase
DOPA decarboxylase
Acetylcholinesterase
Choline acetyltransferase
Tryptophan hydroxylase
Dopamine beta­hydroxylase
Breaks down GABA Monoamine oxidase A
Transaminase
Monoamine oxidase B
Glutamic acid decarboxylase
Tyrosine hydroxylase
DOPA decarboxylase
Check
Choline acetyltransferase

Explanation: The breakdown enzymes of dopamine and norepinephrine include monoamine
oxidase (MAO) & Catechol­o­methyl transferase (COMT).There are two types of MAO. MAO­A
more selectively metabolizes norepinephrine and serotonin while MAO­B more selectively
metabolizes dopamine. Acetylcholine is broken down by acetylcholinesterase enzyme. GABA
is broken down into succinic semialdehyde and glutamate by GABA transaminase. Vigabatrin
inhibits this enzyme.

627
The correct answer is: Dopamine – Monoamine oxidase B, Breaks down norepinephrine
– Monoamine oxidase A, Breaks down serotonin – Monoamine oxidase A, Located on the
postsynaptic membrane
– Acetylcholinesterase, Breaks down GABA – Transaminase

Question 37 HiY Neurosciences EMI037
Correct Breakdown products of neurotransmitters
From the list provided, identify the breakdown products involved in the synthesis of various
Mark 6.00 out of
neurotransmitters:
6.00

Flag question
Homovanillic acid
Vanillyl mandelic acid
5­hydroxyindole acetic acid
Dopamine  
Succinic acid
Monoamine oxidase A
Choline
Choline acetyl transferase
Aspartate aminotransferase
Homovanillic acid
Tyrosine hydroxylase
Vanillyl mandelic acid
5­hydroxyindole acetic acid
Norepinephrine  
Succinic acid
Monoamine oxidase A
Choline
Choline acetyl transferase
Aspartate aminotransferase
Homovanillic acid
Tyrosine hydroxylase
Vanillyl mandelic acid
5­hydroxyindole acetic acid
Serotonin  
Succinic acid
Monoamine oxidase A
Choline
Choline acetyl transferase
Aspartate aminotransferase
Homovanillic acid
Tyrosine hydroxylase
Vanillyl mandelic acid
5­hydroxyindole acetic acid
Acetylcholine  
Succinic acid
Monoamine oxidase A
Choline
Choline acetyl transferase
Aspartate aminotransferase
Homovanillic acid
Tyrosine hydroxylase
Vanillyl mandelic acid
5­hydroxyindole acetic acid
GABA  
Succinic acid
Monoamine oxidase A
Choline
Choline acetyl transferase
Aspartate aminotransferase
Check Tyrosine hydroxylase

628
Explanation: The breakdown products of norepinephrine are 3­methoxy­4­
hydroxyphenylglycol (MHPG) & VMA ­ vanillyl mandelic acid. MHPG is the major metabolite in
CNS while VMA is the major metabolite from peripheral nervous system/endocrine system.
GABA is broken down to glutamate and eventually to succinic acid. Dopamine is broken down
to homovanillic acid.
The correct answer is: Dopamine – Homovanillic acid, Norepinephrine
– Vanillyl mandelic acid, Serotonin – 5­hydroxyindole acetic acid, Acetylcholine – Choline,
GABA – Succinic acid

Question 38 HiY Neurosciences EMI038
Partially correct Functions of neurotransmitters
For each of the following neurotransmitters, identify their main functions.
Mark 6.00 out of
10.00

Flag question
Comprehension
Thinking
Intelligence
Acetylcholine  
Learning
Arousal
Anxiety
Motivation
Feeding
Comprehension
Thinking
Intelligence
Dopamine  
Learning
Arousal
Anxiety
Motivation
Feeding
Comprehension
Thinking
Intelligence
Serotonin  
Learning
Arousal
Anxiety
Motivation
Feeding
Comprehension
Thinking
Intelligence
Learning  
Arousal
Anxiety
GABA Motivation
Feeding

629
Comprehension
Noradrenaline
Thinking
Intelligence
Learning  
Arousal
Anxiety
Motivation
Feeding
Check

Explanation: 
Main functions of acetylcholine: Modulates arousal, learning, memory, rapid eye movement
sleep, pain perception, and thirst and parasympathetic mediation. 
Main functions of dopamine: motivation, novelty seeking, reward circuitry and motor
movement gating in basal ganglia. 
Main functions of serotonin: mood, the perception of pain, feeding, sleep­wake cycle, motor
activity, sexual behaviour, and temperature regulation. 
Main functions of GABA: Mediates anxiety, seizure cessation, and actions of
benzodiazepines, barbiturates, and alcohol. 
Main functions of noradrenaline: Arousal, anxiety, mood regulation, autonomic mediation
The correct answer is: Acetylcholine
– Learning, Dopamine
– Motivation, Serotonin – Feeding, GABA
– Anxiety, Noradrenaline
– Arousal

Question 39 HiY Neurosciences EMI039
Partially correct Localisation of innervations 
Identify the predominant localization of innervations for each of the following neurotransmitter
Mark 4.80 out of
systems in the brain
6.00

Flag question
Median raphe nuclei
Dorsal raphe nuclei
Locus coeruleus
Norepinephrine Nucleus basalis of Meynert
No discrete pathways
Basal ganglia
Mesolimbic cortex
Prefrontal cortex
Hippocampus
Median raphe nuclei
Dorsal raphe nuclei
Locus coeruleus
Nucleus basalis of Meynert
No discrete pathways
Basal ganglia
Serotonin (Choose Mesolimbic cortex
TWO) Prefrontal cortex
Hippocampus

630
Median raphe nuclei
Dorsal raphe nuclei
Locus coeruleus
GABA Nucleus basalis of Meynert
No discrete pathways
Basal ganglia
Mesolimbic cortex
Prefrontal cortex
Hippocampus
Median raphe nuclei
Dorsal raphe nuclei
Locus coeruleus
Glutamate Nucleus basalis of Meynert
No discrete pathways
Basal ganglia
Mesolimbic cortex
Prefrontal cortex
Hippocampus
Median raphe nuclei
Dorsal raphe nuclei
Locus coeruleus
Acetylcholine Nucleus basalis of Meynert
No discrete pathways
Basal ganglia
Mesolimbic cortex
Prefrontal cortex
Hippocampus
Check

Explanation: 
The main area of localization of norepinephrine neurons is locus coeruleus with extensive
projection on to neocortex and other brain areas. 
Serotonin is most commonly found in the periphery (gut, platelets) but cannot cross blood
brain barrier. The neurons are localized in median and dorsal raphe nuclei with widespread
cortical projections and in the spinal cord. 
GABA neurons do not appear as discrete pathways but widely distributed throughout the brain
especially as interneurons. GABA is the most common inhibitory amino acid neurotransmitter.
It is seen in 60% of the synapses of the brain. 
Glutamate is the major excitatory neurotransmitter that is widely distributed in neocortical
projections, hippocampus, and cerebral cortex. 
The localization of acetylcholine is in the ascending system of cholinergic neurons originating
in the reticular formation and cholinergic cells in the nucleus basalis of Meynert.
The correct answer is: Norepinephrine – Locus coeruleus, Serotonin (Choose TWO) – Median
raphe nuclei, Dorsal raphe nuclei, GABA – No discrete pathways, Glutamate – Hippocampus,

631
Acetylcholine – Nucleus basalis of Meynert

Question 40 HiY Neurosciences EMI040
Correct Disorders & neurotransmitters 
For each of the disorders, identify the neurotransmitter involved.
Mark 7.00 out of
7.00

Flag question
Cholecystokinin
Histamine
Serotonin
Schizophrenia Neuropeptide Y  
GABA
Somatostatin
Dopamine
Acetylcholine
Substance P
Cholecystokinin
Histamine
Serotonin
Neuropeptide Y  
Depression
GABA
Somatostatin
Dopamine
Acetylcholine
Substance P
Cholecystokinin
Histamine
Serotonin
Neuropeptide Y  
Anxiety disorders 
GABA
Somatostatin
Dopamine
Acetylcholine
Substance P
Cholecystokinin
Histamine
Serotonin
Alzheimer's  
Neuropeptide Y
dementia
GABA
Somatostatin
Dopamine
Acetylcholine
Substance P
Cholecystokinin
Histamine
Serotonin
Neuropeptide Y  
GABA
Parkinson's disease Somatostatin
Dopamine
Acetylcholine
Substance P

632
Check

Explanation: 
In schizophrenia, the dopaminergic activity is thought to be high in the mesolimbic area
leading to positive symptoms, but it may be low in the mesocortical area leading to anhedonia
and negative symptoms. 
In depression, the levels of serotonin and norepinephrine are low. In anxiety disorders, levels
of norepinephrine are affected. 
GABA mediates anxiety and GABA dysregulation is implicated in various anxiety disorders. 
There is reduced cholinergic function in Alzheimer's dementia. The levels of dopamine are
low in Parkinson's disease.
The correct answer is: Schizophrenia – Dopamine, Depression – Serotonin, Anxiety disorders
– GABA, Alzheimer's dementia – Acetylcholine, Parkinson's disease – Dopamine

Question 41 HiY Neurosciences EMI041
Partially correct Serotonin receptor & action
For each of the following actions, identify the serotonin receptor most likely to be responsible
Mark 4.00 out of
5.00

Flag question
5HT1D
5HT6
5HT3
Regulation of circadian
5HT2B
rhythm
5HT1B
5HT1A
5HT2C
5HT7
5HT2A
5HT1D
5HT6
5HT3
Antiemetic effect 5HT2B
5HT1B
5HT1A
5HT2C
5HT7
5HT2A
5HT1D
5HT6
5HT3
Anxiogenic and
5HT2B
anorexic effect
5HT1B
5HT1A
5HT2C
5HT7
5HT2A

Antidepressant action
due to agonistic effect

633
5HT1D
5HT6
5HT3
5HT2B
5HT1B
5HT1A
5HT2C
5HT7
5HT2A
5HT1D
5HT6
5HT3
Antimigraine effect 5HT2B
5HT1B
5HT1A
5HT2C
5HT7
5HT2A
Check

Explanation: 5HT7 is thought to be relevant to the regulation of circadian rhythm with some
suspected effect on biological symptoms of depression. 5HT3­ antiemetic effect (on
antagonism). 5HT2C­ Anxiogenic and anorexic effect (agonists), 5HT1A­ antidepressant
(agonist), anxiolytics (partial agonist), 5HT1D­ Antimigraine (on antagonism)
The correct answer is: Regulation of circadian rhythm – 5HT7, Antiemetic effect – 5HT3,
Anxiogenic and anorexic effect – 5HT2C, Antidepressant action due to agonistic effect –
5HT1A, Antimigraine effect – 5HT1D

Question 42 HiY Neurosciences EMI042
Incorrect Neuropathology 
Identify 3 disorders for each of the following group of neuropathological changes.
Mark 0.00 out of
6.00

Flag question
Alzheimer's disease
Pick's disease
Progressive supranuclear palsy
Tauopathies (Choose
Parkinson's disease
THREE)
Lewy body dementia
Multisystem atrophy
HIV dementia
Vascular dementia
Schizophrenia
Alzheimer's disease
Huntington's disease
Pick's disease
Progressive supranuclear palsy
Synucleinopathies Parkinson's disease
(Choose THREE)
Lewy body dementia
Multisystem atrophy
HIV dementia
Vascular dementia
Schizophrenia
Huntington's disease

634
Check

Explanation: Tauopathies are diseases with tau protein deposits. The disorders include
Alzheimer's disease, Pick's disease, progressive supranuclear palsy, corticobasal
degeneration, frontotemporal dementia with Parkinsonism (FTDP­17 MATP variant).
Synucleopathies are diseases with alpha­synuclein deposits. The disorders include
Parkinson's disease, Dementia of Lewy body type and multisystem atrophy.
The correct answer is: Tauopathies (Choose THREE)
– Alzheimer's disease, Pick's disease, Progressive supranuclear palsy, Synucleinopathies
(Choose THREE)
– Parkinson's disease, Lewy body dementia, Multisystem atrophy

Question 43 HiY Neurosciences EMI043
Correct Neuropathological findings in various disorders 
Identify the most likely neuropathological finding for each of the following conditions:
Mark 5.00 out of
5.00

Flag question
Post synaptic dopamine receptor blockade
Thinning of the corpus callosum
Brick red pigmentation in basal ganglia
Autism  
Dopamine D2 receptor supersentivity in the niagrostriatal pathway
Global brain atrophy
Spino cerebellar degeneration
Atrophy of the corpus striatum
Multiple cerebral infarcts
Depigmentation of cells in basal ganglia
Post synaptic dopamine receptor blockade
Hypoplasia of the cerebellar vermis
Thinning of the corpus callosum
Brick red pigmentation in basal ganglia
Multi­infarct dementia  
Dopamine D2 receptor supersentivity in the niagrostriatal pathway
Global brain atrophy
Spino cerebellar degeneration
Atrophy of the corpus striatum
Multiple cerebral infarcts
Depigmentation of cells in basal ganglia
Post synaptic dopamine receptor blockade
Hypoplasia of the cerebellar vermis
Thinning of the corpus callosum
Brick red pigmentation in basal ganglia
Punch drunk  
Dopamine D2 receptor supersentivity in the niagrostriatal pathway
syndrome
Global brain atrophy
Spino cerebellar degeneration
Atrophy of the corpus striatum
Multiple cerebral infarcts
Depigmentation of cells in basal ganglia
Post synaptic dopamine receptor blockade
Hypoplasia of the cerebellar vermis
Thinning of the corpus callosum
Brick red pigmentation in basal ganglia
 
Dopamine D2 receptor supersentivity in the niagrostriatal pathway
Huntington's disease Global brain atrophy
Spino cerebellar degeneration
Atrophy of the corpus striatum
Multiple cerebral infarcts

635
Post synaptic dopamine receptor blockade
Thinning of the corpus callosum
Brick red pigmentation in basal ganglia
Parkinson's disease  
Dopamine D2 receptor supersentivity in the niagrostriatal pathway
Global brain atrophy
Spino cerebellar degeneration
Atrophy of the corpus striatum
Multiple cerebral infarcts
Depigmentation of cells in basal ganglia
Check
Hypoplasia of the cerebellar vermis

Explanation: 
In autism, hypoplasia of cerebellar vermis and to some extent the cerebellar hemispheres is
documented. Purkinje cell count in the cerebellum is significantly reduced. 
Multi­infarct dementia is a type of vascular dementia characterized by multiple cerebral
infarcts in the brain. Often the dementia is more acute in onset, with step­wise progression,
and the patients may have multiple neurological deficits. 
Punch drunk syndrome is also called as boxer's encephalopathy, often occurring due to
repeated head injury and commonly seen in boxers. Thinning of the corpus callosum is a
characteristic finding seen in boxer's encephalopathy. 
In Huntington's disease, pathologically there is severe loss of small neurons in the caudate
and putamen with subsequent astrocytosis. With the loss of cells, the head of the caudate
becomes shrunken, and there is atrophy of the corpus striatum. 
In Parkinson's disease, there is a depigmentation of the substantia nigra.
The correct answer is: Autism – Hypoplasia of the cerebellar vermis, Multi­infarct dementia –
Multiple cerebral infarcts, Punch drunk syndrome – Thinning of the corpus callosum,
Huntington's disease – Atrophy of the corpus striatum, Parkinson's disease – Depigmentation
of cells in basal ganglia

Question 44 HiY Neurosciences EMI044
Correct Neuropathological findings (2)
Identify the most common neuropathological finding for the following conditions:
Mark 5.00 out of
5.00

Flag question
Atrophy of the corpus striatum
Dopamine D2 receptor supersensitivity in the nigrostriatal pathway
Hypoplasia of the cerebellar vermis
Wilson's disease Brick red pigmentation in basal ganglia
Preponderance of atrophy in frontotemporal regions
No characteristic gross morphological changes
Diffuse global atrophy
Multiple cerebral infarcts
Post synaptic dopamine receptor blockade
Small infarctions of the white matter that spare the cortical regions

636
Tardive dyskinesia Atrophy of the corpus striatum
Dopamine D2 receptor supersensitivity in the nigrostriatal pathway
Hypoplasia of the cerebellar vermis
Brick red pigmentation in basal ganglia
Preponderance of atrophy in frontotemporal regions
No characteristic gross morphological changes
Diffuse global atrophy
Multiple cerebral infarcts
Post synaptic dopamine receptor blockade
Atrophy of the corpus striatum
Small infarctions of the white matter that spare the cortical regions
Dopamine D2 receptor supersensitivity in the nigrostriatal pathway
Hypoplasia of the cerebellar vermis
Alzheimer's disease Brick red pigmentation in basal ganglia
Preponderance of atrophy in frontotemporal regions
No characteristic gross morphological changes
Diffuse global atrophy
Multiple cerebral infarcts
Post synaptic dopamine receptor blockade
Atrophy of the corpus striatum
Small infarctions of the white matter that spare the cortical regions
Dopamine D2 receptor supersensitivity in the nigrostriatal pathway
Hypoplasia of the cerebellar vermis
Binswanger's disease Brick red pigmentation in basal ganglia
Preponderance of atrophy in frontotemporal regions
No characteristic gross morphological changes
Diffuse global atrophy
Multiple cerebral infarcts
Post synaptic dopamine receptor blockade
Atrophy of the corpus striatum
Small infarctions of the white matter that spare the cortical regions
Dopamine D2 receptor supersensitivity in the nigrostriatal pathway
Hypoplasia of the cerebellar vermis
Creutzfeldt Jacob
Brick red pigmentation in basal ganglia
disease
Preponderance of atrophy in frontotemporal regions
No characteristic gross morphological changes
Diffuse global atrophy
Multiple cerebral infarcts
Post synaptic dopamine receptor blockade
Check
Small infarctions of the white matter that spare the cortical regions

Explanation: 
Wilson's disease is due to copper deposits in the lenticular nuclei (pallidum and putamen)
leading to brick red pigmentation in basal ganglia. 
Tardive dyskinesia is due to dopamine D2 receptor supersensitivity in the basal ganglia and
is one of the long term side effects of antipsychotics (esp. conventional antipsychotics). 
The gross changes seen in Alzheimer's disease include diffuse atrophy, flattened cortical sulci
and enlarged cerebral ventricles. 
Binswanger's disease is a type of vascular dementia (subcortical arteriosclerotic
encephalopathy) resulting from multiple small white matter infarctions that do not appear on
the gray matter regions of the cortex. 
In CJD, there are no characteristic gross morphological features of dementia because of the
typical short course of the disease. But pathological changes such as vacuolization can be
noted at the histological level.

637
The correct answer is: Wilson's disease – Brick red pigmentation in basal ganglia, Tardive
dyskinesia – Dopamine D2 receptor supersensitivity in the nigrostriatal pathway, Alzheimer's
disease – Diffuse global atrophy, Binswanger's disease – Small infarctions of the white matter
that spare the cortical regions, Creutzfeldt Jacob disease – No characteristic gross
morphological changes

Question 45 HiY Neurosciences EMI045
Correct Pathophysiology (1)
For the following conditions, identify the pathophysiological mechanism involved;
Mark 5.00 out of
5.00

Flag question
Prion proteins
Subcortical leukomalacia
Dorsal raphe nucleus serotonin
 
Korsakoff's psychosis Loss of hypocretin neurons
Human Leucine Rich Repeat Kinase 2 (LRRK2 mutations)
Mamillary body alpha­ketoglutarate
Ceruloplasmin deficiency
Tau protein
Beta­amyloid precursor protein
Prion proteins
Alpha synucleinopathy
Subcortical leukomalacia
Dorsal raphe nucleus serotonin
Binswanger's  
Loss of hypocretin neurons
disease
Human Leucine Rich Repeat Kinase 2 (LRRK2 mutations)
Mamillary body alpha­ketoglutarate
Ceruloplasmin deficiency
Tau protein
Beta­amyloid precursor protein
Prion proteins
Alpha synucleinopathy
Subcortical leukomalacia
Dorsal raphe nucleus serotonin
Alzheimer's disease Loss of hypocretin neurons  
Human Leucine Rich Repeat Kinase 2 (LRRK2 mutations)
Mamillary body alpha­ketoglutarate
Ceruloplasmin deficiency
Tau protein
Beta­amyloid precursor protein
Prion proteins
Alpha synucleinopathy
Subcortical leukomalacia
Dorsal raphe nucleus serotonin
Narcolepsy Loss of hypocretin neurons  
Human Leucine Rich Repeat Kinase 2 (LRRK2 mutations)
Mamillary body alpha­ketoglutarate
Ceruloplasmin deficiency
Tau protein
Beta­amyloid precursor protein
Prion proteins
Alpha synucleinopathy
Subcortical leukomalacia
Dorsal raphe nucleus serotonin
CJD Loss of hypocretin neurons  
Human Leucine Rich Repeat Kinase 2 (LRRK2 mutations)
Mamillary body alpha­ketoglutarate
Ceruloplasmin deficiency
Tau protein

638
Check

Explanation: Korsakoff's syndrome is a neurological disorder characterized by severe memory
impairment, confusion, anterograde and retrograde amnesia. The primary cause of the
disorder is alcohol abuse, leading to a deficiency in thiamine (vitamin B1). A prolonged
shortage of thiamine can result in neuronal loss and micro­haemorrhages in the
paraventricular and the periaqueductal gray area as well as mamillary bodies. Thiamine is a
cofactor for the enzymes transketolase, pyruvate dehydrogenase (PDH) and alpha­
ketoglutarate dehydrogenase (a­KGDH). Reduced levels of thiamine results in a decrease in
function of the three enzymes, thus disrupting cell function and would ultimately lead to
apoptosis in Korsakoff's syndrome. 
Binswanger's disease causes subcortical dementia and is a severe form of cerebrovascular
disease with periventricular white matter damage that may occur in patients around 50 years
of age. 
Beta­amyloid precursor protein gene found on the long arm of chromosome 21 is implicated
in early­onset Alzheimer's disease. 
Orexins A and B (aka hypocretins) are closely related neuropeptides derived from a single
gene. They act on OX1 and OX2 receptors in the lateral hypothalamus and other brain areas
involved in stress regulation. Loss of hypocretin neurons is seen in patients with narcolepsy. 
CJD is the commonest form of prion dementia affecting 1 in every 3 million people per year
and typically presents in the 6th or 7th decade of like with personality change followed by a
rapidly progressive dementia accompanied by ataxia, choreoathetosis movements and
myoclonus.
The correct answer is: Korsakoff's psychosis – Mamillary body alpha­ketoglutarate,
Binswanger's disease – Subcortical leukomalacia, Alzheimer's disease – Beta­amyloid
precursor protein, Narcolepsy – Loss of hypocretin neurons, CJD – Prion proteins

Question 46 HiY Neurosciences EMI046
Correct Pathophysiology (2)
For the following conditions, identify the pathophysiological mechanism involved;
Mark 5.00 out of
5.00

Flag question
Dorsal raphe nucleus serotonin
Prions
Ceruloplasmin deficiency
Parkinson's disease Subcortical leukomalacia
Mamillary body alpha­ketoglutarate
Tau protein
Alpha synucleinopathy
Loss of hypocretin neurons
Beta­amyloid precursor protein
Dorsal raphe nucleus serotonin
Prions
Ceruloplasmin deficiency
Wilson's disease Subcortical leukomalacia
Mamillary body alpha­ketoglutarate
Tau protein
Alpha synucleinopathy
Loss of hypocretin neurons
Beta­amyloid precursor protein

639
Dorsal raphe nucleus serotonin
Prions
Multisystem atrophy Ceruloplasmin deficiency
Subcortical leukomalacia
Mamillary body alpha­ketoglutarate
Tau protein
Alpha synucleinopathy
Loss of hypocretin neurons
Beta­amyloid precursor protein
Dorsal raphe nucleus serotonin
Prions
Ceruloplasmin deficiency
Corticobasal
Subcortical leukomalacia
degeneration
Mamillary body alpha­ketoglutarate
Tau protein
Alpha synucleinopathy
Loss of hypocretin neurons
Beta­amyloid precursor protein
Check

Explanation: 
Alpha­synuclein deposits are seen in most patients with Parkinson's disease. Autosomal
dominant mutations in Human Leucine Rich Repeat Kinase 2 (LRRK2 mutations) gene
represent the most common monogenetic cause for Parkinson's disease and explain up to 7%
of familial PD cases. The patients' usually show the late onset of disease and are
indistinguishable from idiopathic patients with respect to clinical and pathological features. 
Wilson's disease is a rare recessively inherited defect in a copper transporter protein, which
causes defective incorporation of copper into apo­ceruloplasmin and failure to excrete copper
in bile. Multiple system atrophy (MSA) is a degenerative neurological disorder classified as a
synucleinopathy. Alpha­synuclein deposits are seen in patients with multisystem atrophy. 
MSA is a degenerative disorder involving nerve cells in specific areas of the brain that results
in problems with movement, balance and other autonomic functions, e.g., bladder control or
blood pressure regulation. The cause of MSA is unknown. The typical age of onset is in the
late 50s to early 60s. 
Tau deposits are seen in corticobasal degeneration. It is a progressive neurological disorder
characterized by "nerve cell loss and atrophy (shrinkage) of multiple areas of the brain
including the cerebral cortex and the basal ganglia. Symptoms are similar to those found in
Parkinson disease, such as poor coordination, akinesia (an absence of movements), rigidity
(a resistance to imposed movement), disequilibrium (impaired balance); and limb dystonia
(abnormal muscle postures)" (NINDS).

Corticobasal Degeneration Information Page. Retrieved from
http://www.ninds.nih.gov/disorders/corticobasal_degeneration/corticobasal_degeneration.htm
The correct answer is: Parkinson's disease
– Alpha synucleinopathy, Wilson's disease – Ceruloplasmin deficiency, Multisystem atrophy –
Alpha synucleinopathy, Corticobasal degeneration – Tau protein

Question 47 HiY Neurosciences EMI047
Partially correct

640
Mark 4.00 out of Microscopic features 
5.00 You are working in the neuroscience department, and you have a special interest in
Flag question
Alzheimer's disease. For each of the definitions given below, identify the neuropathological
terms used to describe them.

Neurofibrillary tangles
Fibrils of multimeric Granulovascular degeneration
chains of peptides Hirano bodies
deposited
Senile plaques
extracellularly with a
beta­pleated sheet
Astrocytic gliosis
conformation Cerebral amyloid angiopathy
Pick bodies
Spongiform encephalopathy
Microglial activation
Neurofibrillary tangles
Granulovascular degeneration
Rod shaped Hirano bodies
eosinophilic bodies in
Senile plaques
the cytoplasm of
neurons
Astrocytic gliosis
Cerebral amyloid angiopathy
Pick bodies
Spongiform encephalopathy
Microglial activation
Neurofibrillary tangles
Granulovascular degeneration
Abnormally Hirano bodies
phosphorylated tau Senile plaques
proteins Astrocytic gliosis
Cerebral amyloid angiopathy
Pick bodies
Spongiform encephalopathy
Microglial activation
Neurofibrillary tangles
Small vacuoles with Granulovascular degeneration
central granules in the Hirano bodies
cytoplasm of neurons, Senile plaques
especially in the Astrocytic gliosis
temporal lobes Cerebral amyloid angiopathy
Pick bodies
Spongiform encephalopathy
Microglial activation
Neurofibrillary tangles
Granulovascular degeneration
Hirano bodies
Senile plaques
Accumulation of beta Astrocytic gliosis
A4 peptide Cerebral amyloid angiopathy
predominantly in the Pick bodies
walls of blood vessels
Spongiform encephalopathy
in the cerebral cortex.
Microglial activation

641
Check

Explanation: 
Senile plaques: Amyloids are fibrils of multimeric chains of peptides deposited extracellularly.
They have a beta­pleated sheet structure. The peptide involved is called Aß (beta A4) peptide.
Amyloid is insoluble. Plaques vary in appearance, and two main subtypes are recognized­
diffuse plaques and neuritic plaques. 
Hirano bodies: Rod shaped eosinophilic bodies in the cytoplasm of neurons; may be set free
in the extracellular space if the neuron dies. Hirano bodies are aggregates of actin and
associated proteins occurring intracellularly. They are frequently seen in hippocampal
pyramidal cells. 
Neurofibrillary tangles: Composed of cytoskeletal elements, primarily abnormally
phosphorylated tau protein. Tau is a peptide required for microtubule assembly. Microtubules
are essential to transport of materials down axons. 
Granulovascular degeneration of the neurons: small vacuoles with central granules, in the
cytoplasm of neurons especially in the temporal lobes. 
Cerebral amyloid angiopathy (CAA): This is the accumulation of Aß in the walls of arteries and
arterioles in the brain and overlying leptomeninges. CAA pathology is seen in nearly 30% of
the elderly people but over 90% of patients with AD, in whom the angiopathy is much more
severe.
The correct answer is: Fibrils of multimeric chains of peptides deposited extracellularly with a
beta­pleated sheet conformation – Senile plaques, Rod shaped eosinophilic bodies in the
cytoplasm of neurons – Hirano bodies, Abnormally phosphorylated tau proteins –
Neurofibrillary tangles, Small vacuoles with central granules in the cytoplasm of neurons,
especially in the temporal lobes – Granulovascular degeneration, Accumulation of beta A4
peptide predominantly in the walls of blood vessels in the cerebral cortex. – Cerebral amyloid
angiopathy

Question 48 HiY Neurosciences EMI048
Partially correct Ascending & Descending Tracts
For each of the following spinal column tract, identify the types of sensations that each carries
Mark 2.00 out of
with it.
4.00

Flag question
Muscular activity
Proprioception and pressure and touch sensation
Visceral function control
Anterior spinothalamic
Proprioceptive and cutaneous sensation
tract
Discriminative touch and Proprioceptive sensation
Voluntary movement
Pain and temperature sensation
Light touch and pressure sensations
Vibration sensations
Muscular activity
Proprioception and pressure and touch sensation
Visceral function control
Proprioceptive and cutaneous sensation
Discriminative touch and Proprioceptive sensation
Voluntary movement
Pain and temperature sensation

642
Lateral spinothalamic
tract

Muscular activity
Proprioception and pressure and touch sensation
Visceral function control
Anterior and posterior
Proprioceptive and cutaneous sensation
spinocerebellar tract
Discriminative touch and Proprioceptive sensation
Voluntary movement
Pain and temperature sensation
Light touch and pressure sensations
Vibration sensations
Muscular activity
Proprioception and pressure and touch sensation
Visceral function control
Lateral corticospinal
Proprioceptive and cutaneous sensation
tract
Discriminative touch and Proprioceptive sensation
Voluntary movement
Pain and temperature sensation
Light touch and pressure sensations
Vibration sensations
Check

Explanation: The anterior spinothalamic tract carries light touch and pressure sensations. The
lateral spinothalamic tract carries pain & temperature sensation. The anterior & posterior
spinocerebellar tract carries proprioceptive, pressure and touch sensation. The lateral
corticospinal tract carries fibers responsible for performing voluntary movements in the body.
The correct answer is: Anterior spinothalamic tract – Light touch and pressure sensations,
Lateral spinothalamic tract – Pain and temperature sensation, Anterior and posterior
spinocerebellar tract – Proprioception and pressure and touch sensation, Lateral corticospinal
tract – Voluntary movement

Question 49 HiY Neurosciences EMI049
Correct Diagnosis of Creutzfeldt­Jacob disease
Identify the different methods of diagnosis of CJD.
Mark 7.00 out of
7.00

Flag question
Microcysts
Immunoperoxidase
Mild generalized cerebral atrophy
Macroscopic finding in
MRI scan
advanced CJD
EEG
14­3­3
14­3­1
CT scan

643
Microcysts
Characteristic Immunoperoxidase
microscopic finding in Mild generalized cerebral atrophy
CJD MRI scan
EEG
14­3­3
14­3­1
CT scan
Microcysts
Immunoperoxidase
This staining is helpful Mild generalized cerebral atrophy
to identify abnormal MRI scan
prion proteins in tissues EEG
14­3­3
14­3­1
CT scan
Microcysts
Immunoperoxidase
This abnormal protein Mild generalized cerebral atrophy
can be found in the MRI scan
CSF by immuno­assay EEG
14­3­3
14­3­1
CT scan
Microcysts
Immunoperoxidase
This is the most Mild generalized cerebral atrophy
supportive diagnostic MRI scan
test in variant CJD EEG
14­3­3
14­3­1
CT scan
Check

Explanation: 
There are no characteristic gross pathologic features of CJD because of the typical short
course of the disease. Persons living beyond six months to a year may have some degree of
generalized cerebral atrophy. Microscopically CJD shows a spongiform encephalopathy
secondary to neuropil vacuolisation. Many round or oval vacuoles, either single or
multiloculated, are seen in the neuropil of cortical gray matter. The vacuoles may combine to
form microcysts. Neuronal loss and gliosis are also noted in CJD. Prion protein (PrPc) is a
normal neuronal cell surface encoded by a gene on chromosome 20. This is converted via a
conformational change to an abnormal form designated as PrPSc. This abnormal form is
protease­resistant and can accumulate in the central nervous system of affected persons. This
accumulation triggers a further conversion of normal PrPc to PrPSc and accounts for the
degenerative changes in the cerebral cortex. These abnormal PrPs can be transmitted
person­to­person via pituitary extracts, corneal transplants, dural grafts, and contaminated
electrodes from neurosurgical procedures. The PrP can be identified in tissues with
immunoperoxidase staining. An abnormal protein called 14­3­3 can be found in the CSF by

644
immunoassay, but this protein is non­specific and may be found in association with viral
encephalitis and stroke too. It is less frequent in variant than typical CJD. In familial cases of
CJD, the typical EEG changes are often lacking, and the 14­3­3 protein is absent from CSF
half the time. The MRI (especially the FLAIR sequence) is the most useful supportive
diagnostic test in variant CJD. A particular abnormality in the posterior thalamic region
(pulvinar sign) is seen in 90% cases. This sign is highly sensitive and specific for variant CJD.

Excerpt retrieved from http://path.upmc.edu/cases/case86/dx.html
The correct answer is: Macroscopic finding in advanced CJD
– Mild generalized cerebral atrophy, Characteristic microscopic finding in CJD
– Microcysts, This staining is helpful to identify abnormal prion proteins in tissues –
Immunoperoxidase, This abnormal protein can be found in the CSF by immuno­assay – 14­3­
3, This is the most supportive diagnostic test in variant CJD – MRI scan

Question 50 HiY Neurosciences EMI050
Partially correct Basal ganglia lesions
Identify the pathophysiological changes identified in various disorders
Mark 4.80 out of
6.00

Flag question
Degeneration of the striatum
Selective loss of GABAergic neurons
Striatal dopaminergic dysfunction
Obsessive compulsive
Copper deposits in the lenticular nuclei
disorder
Selective loss of serotonergic neurons
Increased caudate blood flow
Acute bilateral anoxic damage to basal ganglia
Calcium deposition in the basal ganglia
Depigmentation of substantia nigra
Degeneration of the striatum
Subthalamic nucleus lesion
Selective loss of GABAergic neurons
Striatal dopaminergic dysfunction
Huntington's chorea (2
Copper deposits in the lenticular nuclei
answers)
Selective loss of serotonergic neurons
Increased caudate blood flow
Acute bilateral anoxic damage to basal ganglia
Calcium deposition in the basal ganglia
Depigmentation of substantia nigra
Degeneration of the striatum
Subthalamic nucleus lesion
Selective loss of GABAergic neurons
Striatal dopaminergic dysfunction
Tourette's syndrome Copper deposits in the lenticular nuclei
Selective loss of serotonergic neurons
Increased caudate blood flow
Acute bilateral anoxic damage to basal ganglia
Calcium deposition in the basal ganglia
Depigmentation of substantia nigra
Degeneration of the striatum
Subthalamic nucleus lesion
Selective loss of GABAergic neurons
Striatal dopaminergic dysfunction
Copper deposits in the lenticular nuclei
Fahr's disease Selective loss of serotonergic neurons
Increased caudate blood flow
Acute bilateral anoxic damage to basal ganglia
Calcium deposition in the basal ganglia
Depigmentation of substantia nigra
645
Degeneration of the striatum
Selective loss of GABAergic neurons
Striatal dopaminergic dysfunction
Hemiballismus Copper deposits in the lenticular nuclei
Selective loss of serotonergic neurons
Increased caudate blood flow
Acute bilateral anoxic damage to basal ganglia
Calcium deposition in the basal ganglia
Depigmentation of substantia nigra
Check
Subthalamic nucleus lesion

Explanation: 
In OCD, there is both reduced, and increased volumes of caudate nuclei reported.Patients
have higher caudate blood flow. Increased caudate metabolism has been found to reduce
after effective treatment of the OCD. 
Huntington's chorea­ there is degeneration of the striatum (mainly caudate nucleus) &
selective loss of GABAergic neurons. 
Striatal dopaminergic dysfunction is a feature of Tourette's syndrome featured by multiple
motor tics and vocal tics in children. 
Fahr's disease is caused by progressive calcium deposition in the basal ganglia. (early onset
­ schizophreniform psychoses and catatonic symptoms; later onset ­ dementia and
choreoathetosis). 
Subthalamic nucleus damage (especially infarctions) can lead to hemiballismus. 
Ref: Ring, HA & Serra­Mestres, J. Neuropsychiatry of the basal ganglia.
J Neurol Neurosurg Psychiatry 2002;72:1 12­21
The correct answer is: Obsessive compulsive disorder – Increased caudate blood flow,
Huntington's chorea (2 answers) – Degeneration of the striatum, Selective loss of GABAergic
neurons, Tourette's syndrome – Striatal dopaminergic dysfunction, Fahr's disease – Calcium
deposition in the basal ganglia, Hemiballismus – Subthalamic nucleus lesion

Finish review

646
 Home HiYield Paper Neurosciences

HiYield Paper A(2)

Started on Saturday, 9 May 2015, 9:47 PM
State Finished
Completed on Sunday, 10 May 2015, 2:44 PM
Time taken 16 hours 57 mins
Marks 175.57/195.00
Grade 90.03 out of 100.00

Question 1 HiY Neurosciences EMI051
Correct Blood supply to brain
Match the specific areas of the brain with their blood supply from the list provided
Mark 8.00 out of
8.00

Flag question
Lateral aspects of cerebral cortex
Broca's & Wernicke's area
Inferomedial temporal lobe
Anterior cerebral artery Occipital lobe  
Medial aspects of cerebral cortex
Superior lateral aspects of cerebral cortex
Internal capsule
Medulla
Pons
Lateral aspects of cerebral cortex
Broca's & Wernicke's area
Inferomedial temporal lobe
 
Middle cerebral artery Occipital lobe
Medial aspects of cerebral cortex
Superior lateral aspects of cerebral cortex
Internal capsule
Medulla
Pons
Lateral aspects of cerebral cortex
Broca's & Wernicke's area
Inferomedial temporal lobe
Occipital lobe  
Medial aspects of cerebral cortex
Superior lateral aspects of cerebral cortex
Posterior cerebral Internal capsule
artery
Medulla
Pons

647
Lateral aspects of cerebral cortex
Broca's & Wernicke's area
Inferomedial temporal lobe
Occipital lobe  
Vertebral arteries
Medial aspects of cerebral cortex
Superior lateral aspects of cerebral cortex
Internal capsule
Medulla
Pons
Lateral aspects of cerebral cortex
Broca's & Wernicke's area
Inferomedial temporal lobe
Basilar artery Occipital lobe  
Medial aspects of cerebral cortex
Superior lateral aspects of cerebral cortex
Internal capsule
Medulla
Pons
Check

Explanation: 
The internal carotid artery enters the circle of Willis and divides to form the anterior cerebral
and middle cerebral arteries. The anterior cerebral artery supplies the medical and superior
lateral aspects of the cerebral cortex to the parietal/occipital border. 
The middle cerebral artery supplies the greater part of the lateral aspect of the cerebral cortex,

including the Broca's and Wernicke's area in the dominant hemispheres. 
The posterior cerebral artery arises from basilar artery and supplies the inferomedial temporal
lobe and the occipital lobe. The internal capsule is supplied by branches from the circle of
Willis. 
The posterior inferior cerebellar artery and anterior spinal artery, branches of the vertebral
artery, supply medulla.
The basilar artery that runs along the midline of the pons supplies Pons.
The correct answer is: Anterior cerebral artery – Medial aspects of cerebral cortex, Superior
lateral aspects of cerebral cortex, Middle cerebral artery – Lateral aspects of cerebral cortex,
Broca's & Wernicke's area, Posterior cerebral artery – Inferomedial temporal lobe, Occipital
lobe, Vertebral arteries – Medulla, Basilar artery – Pons

Question 2 HiY Neurosciences EMI052
Partially correct Cerebral arteries & lesions 
Using the clinical descriptions below, identify the artery most likely to be involved
Mark 3.00 out of
4.00

Flag question
Left middle cerebral artery
Subclavian artery proximal to ipsilateral vertebral artery
Vertebral artery
Right anterior cerebral artery
A 48­year­old Posterior inferior cerebellar artery
gentleman presenting Right middle cerebral artery
in an akinetic mute Bilateral anterior cerebral artery
Posterior cerebral artery

648
state has quadriparesis
with legs more affected
than the arms.

A 64­year­old Left middle cerebral artery
gentleman develops Subclavian artery proximal to ipsilateral vertebral artery
brain stem ischemia Vertebral artery
during exercise. When
Right anterior cerebral artery
arms are held straight,
Posterior inferior cerebellar artery
there is a drop in more
than 30mm Hg Right middle cerebral artery
between the arms Bilateral anterior cerebral artery
Posterior cerebral artery
A 57­year­old Basilar artery
Left middle cerebral artery
gentleman who is a Left anterior cerebral artery
Subclavian artery proximal to ipsilateral vertebral artery
right handed individual Vertebral artery
presenting with left
Right anterior cerebral artery
hemiparesis and
Posterior inferior cerebellar artery
sensory loss involving
face & arms. He also Right middle cerebral artery
neglects his right side Bilateral anterior cerebral artery
Posterior cerebral artery
A 67­year­old Basilar artery
Left middle cerebral artery
gentleman with Left anterior cerebral artery
Subclavian artery proximal to ipsilateral vertebral artery
presenting with left
Vertebral artery
sided weakness
especially in the legs.
Right anterior cerebral artery
The weakness is milder Posterior inferior cerebellar artery
in the arm, and the face Right middle cerebral artery
is spared. Bilateral anterior cerebral artery
Posterior cerebral artery
Basilar artery
Check Left anterior cerebral artery

Explanation: 
The effects of bilateral anterior cerebral artery infarct include quadriparesis with legs more
affected than arms and akinetic mutism (ventromedial or cingulate syndrome). The lesion, in
this case, would be in the cingulate gyri. The anterior cerebral artery supplies the medial
surface of the brain, including the ventromedial frontal lobe, the cingulum, the premotor cortex,
and the motor strip. 
Subclavian steel syndrome is caused due to lesions involving subclavian artery proximal to
the ipsilateral vertebral artery. The patients may present with brain stem ischaemia following
arm exertion, causing a difference in BP of more than 20 mm Hg between the arms. 
Middle cerebral artery lesions are characterized by hemiplegia, hemianesthesia. If the lesion
involves non­dominant hemisphere, then contralateral neglect is highly likely. 
Anterior cerebral artery lesions involve contralateral hemiplegia/hemiparesis with weakness
involving more of the legs than the arms, and the face will usually be spared. 

649
Excerpt retrieved from Frontal Lobe Syndromes Clinical Presentation ­ Medscape Reference.
(n.d.). Retrieved from http://emedicine.medscape.com/article/1135866­clinical
The correct answer is: A 48­year­old gentleman presenting in an akinetic mute state has
quadriparesis with legs more affected than the arms. – Bilateral anterior cerebral artery, A 64­
year­old gentleman develops brain stem ischemia during exercise. When arms are held
straight, there is a drop in more than 30mm Hg between the arms – Subclavian artery
proximal to ipsilateral vertebral artery, A 57­year­old gentleman who is a right handed
individual presenting with left hemiparesis and sensory loss involving face & arms. He also
neglects his right side – Right middle cerebral artery, A 67­year­old gentleman with
presenting with left sided weakness especially in the legs. The weakness is milder in the arm,
and the face is spared. – Right anterior cerebral artery

Question 3 HiY Neurosciences EMI053
Correct Brain lesions 
For each of the following clinical presentation, identify the most likely brain lesion
Mark 4.00 out of
4.00

Flag question A 77­year­old man has
Non­dominant parietal lobe lesion
suffered a head injury Bilateral posterior parietal lesion
and is now incontinent. Cervical spine lesion
He has limited verbal Dominant parietal lobe lesion
output and lacks Medial frontal lesion
spontaneous Optic tract lesion
movement Non dominant temporal lobe lesion
Pontine lesion
A 63­year­old man Bilateral calcarine lesion
Non­dominant parietal lobe lesion
suffered a car accident Dorsolateral prefrontal cortex lesion
Bilateral posterior parietal lesion
and became Dominant temporal lobe lesion
Cervical spine lesion
quadriplegic. He is
Dominant parietal lobe lesion
conscious but not able
to speak or vocalise.
Medial frontal lesion
He can respond to Optic tract lesion
questions by blinking Non dominant temporal lobe lesion
Pontine lesion
A 69­year­old man has Bilateral calcarine lesion
marked difficulties with Dorsolateral prefrontal cortex lesion
seeing objects
Dominant temporal lobe lesion
properly. The CT
trainee holds a pen,
and he reports 'I see a
pen.....I think'. Then the
Non­dominant parietal lobe lesion
trainee holds up a Bilateral posterior parietal lesion
mobile phone and the Cervical spine lesion
man reports 'I see a Dominant parietal lobe lesion
phone'. When the Medial frontal lesion
trainee now holds the Optic tract lesion
pen and mobile phone Non dominant temporal lobe lesion
overlapping one Pontine lesion
another he says, I think
Bilateral calcarine lesion
I see a pen. When the
trainee asks again, the
Dorsolateral prefrontal cortex lesion
man says 'Wait.....I see Dominant temporal lobe lesion
a phone.'

650
A 56­year­old man has Non­dominant parietal lobe lesion
suffered a head injury
Bilateral posterior parietal lesion
and presented with
memory difficulties. He Cervical spine lesion
is apathetic, planning Dominant parietal lobe lesion
deficits and impairment Medial frontal lesion
of working memory. Optic tract lesion
Non dominant temporal lobe lesion
Pontine lesion
Bilateral calcarine lesion
Check
Dorsolateral prefrontal cortex lesion
Dominant temporal lobe lesion

Explanation: Medial frontal syndrome (ventromedial), also called as apathetic or pseudo
depressive type is featured by a paucity of spontaneous behaviours, sparse verbal output and
reduced social knowledge. Incontinence is also a feature of frontal lobe impairment. 
In severe lesions involving the pons, patients can present with weakness of arms & legs with
dysphagia and loss of speech. However, they can respond to questions by blinking. 
Case 3: This patient could be suffering from simultanagnosia (inability to attend to more than
one item of a complex scene at a time), a feature of Balint's syndrome resulting from lesions of
the bilateral posterior parietal regions. Balint's syndrome consists of a triad of
simultanagnosia, optic ataxia (inability to guide reaching or pointing despite adequate vision),
and oculomotor apraxia (inability to voluntarily direct saccades to a visual target). Visual fields
are unaffected, and oculocephalic reflexes are intact. 
Dorsolateral pre­frontal cortex dysfunction is called as dysexecutive or disorganized type. It is
featured by executive dysfunction, diminished judgment, and planning and poor insight. The
patients are also described to be concrete and inflexible in nature.
The correct answer is: A 77­year­old man has suffered a head injury and is now incontinent.
He has limited verbal output and lacks spontaneous movement – Medial frontal lesion, A 63­
year­old man suffered a car accident and became quadriplegic. He is conscious but not able
to speak or vocalise. He can respond to questions by blinking – Pontine lesion, A 69­year­old
man has marked difficulties with seeing objects properly. The CT trainee holds a pen, and he
reports 'I see a pen.....I think'. Then the trainee holds up a mobile phone and the man reports 'I
see a phone'. When the trainee now holds the pen and mobile phone overlapping one
another he says, I think I see a pen. When the trainee asks again, the man says 'Wait.....I see a
phone.' – Bilateral posterior parietal lesion, A 56­year­old man has suffered a head injury and
presented with memory difficulties. He is apathetic, planning deficits and impairment of
working memory. – Dorsolateral prefrontal cortex lesion

Question 4 HiY Neurosciences EMI054
Correct Neuroanatomical localization (1)
Identify the neuroanatomical location of lesions for the clinical presentations described below:
Mark 5.00 out of
5.00

Flag question
Supplementary motor area
Cerebellum
A 57­year­old woman is Occipito­temporal area
unable to recognize Lateral temporal lobe
faces Lateral prefrontal cortex
Posterior superior temporal lobe
Basal forebrain
Dorsal occipital area
Medial temporal lobe

651
Supplementary motor area
Cerebellum
An 83­year­old woman Occipito­temporal area
has difficulty in learning
Lateral temporal lobe
new information
Lateral prefrontal cortex
Posterior superior temporal lobe
Basal forebrain
Dorsal occipital area
Medial temporal lobe
Supplementary motor area
Bilateral anterior parietal area
Cerebellum
A 57­year­old man is Occipito­temporal area
unable to understand
Lateral temporal lobe
speech but appears
fluent.
Lateral prefrontal cortex
Posterior superior temporal lobe
Basal forebrain
Dorsal occipital area
Medial temporal lobe
Supplementary motor area
A 49­year­old man can
Bilateral anterior parietal area
see clearly in one small Cerebellum
part of the visual field in Occipito­temporal area
the centre while other Lateral temporal lobe
areas appear foggy Lateral prefrontal cortex
since he sustained a Posterior superior temporal lobe
head injury. Basal forebrain
Dorsal occipital area
Medial temporal lobe
Supplementary motor area
Bilateral anterior parietal area
Cerebellum
Occipito­temporal area
A 58­year­old man in a
brain injury unit is
Lateral temporal lobe
described to have an Lateral prefrontal cortex
expressionless face. Posterior superior temporal lobe
He cannot Basal forebrain
communicate and only Dorsal occipital area
has tracking eye Medial temporal lobe
movements preserved. Bilateral anterior parietal area

Check

Explanation: 
The area of the brain involved in recognizing faces is fusiform gyrus, which is situated in the
occipitotemporal region. This patient is unable to learn new information, which is anterograde
amnesia, often seen in patients with Alzheimer's disease. The pathology is mainly in a medial
temporal lobe, which lodges the hippocampus. 
Wernicke's area is situated in the posterior superior temporal lobe and is involved in language
comprehension. 
Case 4 illustrates key­hole vision that is a feature of cortical blindness involving the dorsal
occipital area of the brain. 

652
Anterior cerebral artery supplies most of the medial surface of the brain (structures include
ventromedial frontal lobe, the cingulum, the premotor cortex, supplementary motor area and
the motor strip). In lesions involving supplementary motor areas, the patients can present with
akinetic mutism with only tracking movements of the eyes being preserved.
The correct answer is: A 57­year­old woman is unable to recognize faces – Occipito­temporal
area, An 83­year­old woman has difficulty in learning new information – Medial temporal lobe,
A 57­year­old man is unable to understand speech but appears fluent. – Posterior superior
temporal lobe, A 49­year­old man can see clearly in one small part of the visual field in the
centre while other areas appear foggy since he sustained a head injury. – Dorsal occipital
area, A 58­year­old man in a brain injury unit is described to have an expressionless face. He
cannot communicate and only has tracking eye movements preserved. – Supplementary
motor area

Question 5 HiY Neurosciences EMI055
Partially correct Connecting bundles
Identify the areas of the brain that are connected with each other
Mark 6.00 out of
8.00

Flag question
Medial frontal gyrus
Medial geniculate body
Uncinate fasciculus Superior colliculi
connects temporal lobe
Lateral geniculate body
to which of the listed
structures?
Broca's area
Parietal
Inferior frontal gyrus
Mamillary bodies
Inferior colliculi
Medial frontal gyrus
Which of the listed
Dorsomedial thalamus
Medial geniculate body
options refers to the Superior colliculi
thalamic nuclei of the Lateral geniculate body
auditory system Broca's area
connected to midbrain? Parietal
Inferior frontal gyrus
Mamillary bodies
Inferior colliculi
Medial frontal gyrus
Dorsomedial thalamus
Medial geniculate body
Superior colliculi
Arcuate fasciculus Lateral geniculate body
connects Broca's area Broca's area
with which of the listed Parietal
structures? Inferior frontal gyrus
Mamillary bodies
Inferior colliculi
Dorsomedial thalamus

Which of the listed
options refers to the
midbrain structure of

653
visual tract connected Medial frontal gyrus
to the thalamus?
Medial geniculate body
Superior colliculi
Lateral geniculate body
Broca's area
Parietal
Inferior frontal gyrus
Mamillary bodies
Inferior colliculi
Check
Dorsomedial thalamus

Explanation: 
The uncinate fasciculus is a white matter tract that connects parts of the limbic system such as
hippocampus and amygdala in the temporal lobe with the orbitofrontal region. It mainly
connects the inferior frontal gyrus with temporal lobe. 
Mediate geniculate body and inferior colliculi are two brain stem nuclei of the auditory system.
Note that the lateral geniculate body and superior colliculi are involved in visual processing. 
The arcuate fasciculus is the tract that connects the posterior part of the temper­parietal
junction with the frontal cortex in the brain. It mainly connects Wernicke's and Broca's areas. 
Lateral geniculate body and superior colliculi are two brain stem nuclei of the visual system.
The correct answer is: Uncinate fasciculus connects temporal lobe to which of the listed
structures? – Inferior frontal gyrus, Which of the listed options refers to the thalamic nuclei of
the auditory system connected to midbrain?
– Medial geniculate body, Arcuate fasciculus connects Broca's area with which of the listed
structures?
– Broca's area, Which of the listed options refers to the midbrain structure of visual tract
connected to the thalamus? – Superior colliculi

Question 6 HiY Neurosciences EMI056
Correct Developmental organization
For each of the vesicles below, identify the brain structures that develop from it
Mark 11.00 out of
11.00

Flag question
Cerebellum
Tectum and Tegmentum
Vagus nerve
Telencephalon   
Corpus striatum
Caudal part of the medulla oblongata
Adrenal medulla
Thalamus
Cerebellum
Tectum and Tegmentum
Vagus nerve
 
Corpus striatum
Caudal part of the medulla oblongata
Diencephalon
Adrenal medulla
Thalamus

654
Cerebellum
Tectum and Tegmentum
Vagus nerve
Mesencephalon  
Corpus striatum
Caudal part of the medulla oblongata
Adrenal medulla
Thalamus
Cerebellum
Tectum and Tegmentum
Vagus nerve
Metencephalon  
Corpus striatum
Caudal part of the medulla oblongata
Adrenal medulla
Thalamus
Cerebellum
Tectum and Tegmentum
Vagus nerve
Myelencephalon  
Corpus striatum
Caudal part of the medulla oblongata
Adrenal medulla
Thalamus

Check

Explanation: During embryonic development the midline neural tube differentiates into the
following vesicles. 
Prosencephalon which differentiates into the Telencephalon (cerebrum, striatum and
pallidum) and Diencephalon (thalamus, subthalamus, pineal body, hypothalamus and
epithalamus). 
Mesencephalon (midbrain)has two parts ­ tectum or corpora quadrigeminal, made up of
the superior & inferior colliculi and tegmentum containing the red nucleus and
periaqueductal grey matter. 
Rhombencephalon which differentiates into Metencephalon that forms pons, the rostral
part of the medulla oblongata and cerebellum and Myelencephalon that forms the caudal
part of medulla oblongata.

The correct answer is: Telencephalon 
– Corpus striatum, Diencephalon
– Thalamus, Mesencephalon
– Tectum and Tegmentum, Metencephalon
– Cerebellum, Myelencephalon
– Caudal part of the medulla oblongata

Question 7 HiY Neurosciences EMI057
Correct

655
Mark 3.00 out of Neuroanatomical regions (1)
3.00 Identify one brain area that is most important in the following functions;
Flag question
Amygdala
Fusiform gyrus
Medulla
Reward processing Inferior olivary nucleus
Hippocampus
Angular gyrus
Left inferior frontal area
Superior parietal cortex
Nucleus Accumbens
Amygdala
Cerebellar vermis
Fusiform gyrus
Medulla
Prosopagnosia Inferior olivary nucleus
Hippocampus
Angular gyrus
Left inferior frontal area
Superior parietal cortex
Nucleus Accumbens
Amygdala
Cerebellar vermis
Fusiform gyrus
Medulla
Recalling previously
Inferior olivary nucleus
learned material
Hippocampus
Angular gyrus
Left inferior frontal area
Superior parietal cortex
Nucleus Accumbens
Check
Cerebellar vermis

Explanation: 
The brain regions implicated in reward system regulation include nucleus accumbens and
ventral tegmental area. 
The area of the brain involved in mediating emotional response such as fear is an amygdala.
Bilateral amygdalar damage is associated with a loss of fear conditioning, as evidenced from
the comprehensive investigations in patient SM who has complete, focal bilateral amygdala
damage. 
The brain area involved in speech production is Broca's area, which is situated, in the left
inferior frontal cortex. 
Prosopagnosia is difficulty in recognizing faces, mediated through fusiform gyrus.
Hippocampus is involved in recalling previously learned material 

Excerpt retrieved from Tranel et al. (2006), Altered experience of emotion following bilateral
amygdala damage. http://www.ncbi.nlm.nih.gov/pubmed/17354069
The correct answer is: Reward processing – Nucleus Accumbens, Prosopagnosia – Fusiform
gyrus, Recalling previously learned material – Hippocampus

656
Question 8 HiY Neurosciences EMI058
Correct Neuroanatomical regions (2)
Identify one brain area that is most important in each of the following functions
Mark 5.00 out of
5.00

Flag question
Hypothalamus
Anterior cingulate cortex
Planning and Hippocampus
programming of Cerebellum
movement Amygdala
Corpus callosum
Medulla
Basal ganglia
Left inferior frontal area
Hypothalamus
Dorsolateral prefrontal cortex
Anterior cingulate cortex
Posterior superior temporal lobe
Hippocampus
Motivational circuitry Cerebellum
Amygdala
Corpus callosum
Medulla
Basal ganglia
Left inferior frontal area
Hypothalamus
Dorsolateral prefrontal cortex
Anterior cingulate cortex
Posterior superior temporal lobe
Hippocampus
Preparing a motor plan
Cerebellum
and predicting balance
Amygdala
Corpus callosum
Medulla
Basal ganglia
Left inferior frontal area
Hypothalamus
Dorsolateral prefrontal cortex
Anterior cingulate cortex
Posterior superior temporal lobe
Hippocampus
Executive functions Cerebellum
Amygdala
Corpus callosum
Medulla
Basal ganglia
Left inferior frontal area
Hypothalamus
Dorsolateral prefrontal cortex
Anterior cingulate cortex
Posterior superior temporal lobe
Hippocampus
Comprehension of
Cerebellum
speech
Amygdala
Corpus callosum
Medulla
Basal ganglia
Left inferior frontal area
Check
Dorsolateral prefrontal cortex
Posterior superior temporal lobe

657
Explanation: The basal ganglia are involved in the planning and programming of movement.
They also have a role in the processes by which an abstract thought is converted into
voluntary action. 
The motivation circuitry involves anterior cingulate cortex and its reciprocal connections with
the rest of the prefrontal cortex. 
The cerebellum has the important role of preparing a motor plan and predicting balance
needed between muscle groups to carry out the intended action smoothly. Cerebellar lesions
produce ataxia and coarse intentional tremors, along with hypotonia, past pointing and
pendular knee jerk. 
The dorsolateral prefrontal cortex is responsible for executive functions such as planning,
judgement, and decision­making. 
Wernicke's area, which is situated in the posterior superior temporal lobe, is involved in
speech comprehension.
The correct answer is: Planning and programming of movement – Basal ganglia, Motivational
circuitry – Anterior cingulate cortex, Preparing a motor plan and predicting balance –
Cerebellum, Executive functions – Dorsolateral prefrontal cortex, Comprehension of speech –
Posterior superior temporal lobe

Question 9 HiY Neurosciences EMI059
Correct Neuroanatomical localization (2)
Identify the brain regions where the following structures are situated
Mark 4.00 out of
4.00

Flag question
Midbrain
Diencephalon
Prefrontal cortex
Basal ganglia  
Entorhinal cortex
Parietal lobe
Occipital lobe
Temporal lobe
Medulla oblongata
Cerebellum
Midbrain
Pons
Diencephalon
Prefrontal cortex
Striatum Basal ganglia  
Parietal lobe
Occipital lobe
Temporal lobe
Medulla oblongata
Cerebellum
Midbrain
Pons
Diencephalon
Prefrontal cortex
Basal ganglia  

Ventral tegmental Parietal lobe
area Occipital lobe
Temporal lobe
Medulla oblongata
Cerebellum
Pons

658
Midbrain
Diencephalon
Prefrontal cortex
Basal ganglia  
Hypothalamus
Parietal lobe
Occipital lobe
Temporal lobe
Medulla oblongata
Cerebellum
Check
Pons

Explanation: 
The entorhinal cortex is situated in the temporal lobe. 
The basal ganglia are a group of gray matter nuclei that forms the largest subcortical structure
in the brain. The components of the basal ganglia include the caudate nucleus, the putamen
and the globus pallidus. Putamen and globus pallidus together forms the lentiform nucleus.
Caudate and putamen are often clubbed together by the name striatum. 
The ventral tegmental area is situated in the midbrain. 
The hypothalamus is a part of the diencephalon.
The correct answer is: Entorhinal cortex – Temporal lobe, Striatum – Basal ganglia, Ventral
tegmental area – Midbrain, Hypothalamus – Diencephalon

Question 10 HiY Neurosciences EMI060
Correct Pathophysiology of Schizophrenia
Identify the one important area of the brain involved in each of the following
Mark 4.00 out of
pathophysiological findings in schizophrenia
4.00

Flag question
Heschl's gyrus
Corpus striatum
D2 occupancy in this
Occipital pole
region by
Prefrontal cortex
antipsychotics, predicts
antipsychotic response
Planum temporale
Amygdalar volume
Thalamic volume
Left temporal horn
Lateral ventricles
Heschl's gyrus
Enlargement of this
Dorsal raphe
Corpus striatum
structure can be seen Brain weight & volume
Occipital pole
as a consequence of Prefrontal cortex
long­term treatment Planum temporale
with haloperidol Amygdalar volume
Thalamic volume
Left temporal horn
Lateral ventricles
Dorsal raphe
Brain weight & volume

659
A consistent reversal of Heschl's gyrus
normal left larger than
Corpus striatum
right asymmetry of this
structure is noted
Occipital pole
Prefrontal cortex
Planum temporale
Amygdalar volume
Thalamic volume
Left temporal horn
Lateral ventricles
Heschl's gyrus
Dorsal raphe
Corpus striatum
Neuroimaging studies Brain weight & volume
Occipital pole
suggest decreased
Prefrontal cortex
activity in this part of the
Planum temporale
brain
Amygdalar volume
Thalamic volume
Left temporal horn
Lateral ventricles
Check
Dorsal raphe
Brain weight & volume

Explanation: Dopamine receptor occupancy (D­2) in the corpus striatum predicts response to
antipsychotics. It also plays an important role in causing akathisia and other extrapyramidal
side effects caused by antipsychotics. 
Enlargement of basal ganglia can be seen in schizophrenia as a consequence of treatment
with classical neuroleptics, which can be reversed by the use of atypical substances. 
The planum temporale, the posterior superior surface of the superior temporal gyrus, is a
highly lateralized brain structure involved with language. In schizophrenic patients, a
consistent reversal of the usual left­larger­than­ right asymmetry of planum temporale surface
area is noted. 
Neuroimaging studies suggest decreased activity in the prefrontal cortex of the brain, relating
to negative symptoms of schizophrenia and reduced motivation.
The correct answer is: D2 occupancy in this region by antipsychotics, predicts antipsychotic
response – Corpus striatum, Enlargement of this structure can be seen as a consequence of
long­term treatment with haloperidol – Corpus striatum, A consistent reversal of normal left
larger than right asymmetry of this structure is noted – Planum temporale, Neuroimaging
studies suggest decreased activity in this part of the brain – Prefrontal cortex

Question 11 HiY Neurosciences EMI061
Partially correct Neuroanatomical localization
Identify the most important function or dysfunction resulting from the damage to each of the
Mark 6.00 out of
following structures:
9.00

Flag question
Aphasia
Dyscalculia
Limb apraxia
Dominant hemisphere Personality
(Choose THREE) Constructional apraxia

660
Aphasia
Dyscalculia
Non­dominant Limb apraxia
hemisphere (Choose Personality
ONE) Constructional apraxia

Aphasia
Dyscalculia
Bilateral and not strictly Limb apraxia
localised (Choose Personality
ONE) Constructional apraxia

Check

Explanation: 
Dominant hemisphere functions include aphasia (language), calculation, praxis (limb
apraxia).
Non­dominant hemisphere function includes visuospatial deficits, constructional apraxia,
dressing apraxia, neglect and impaired visual perception. 
Face recognition involves both hemispheres. 
Broader psychological constructs such as personality and memory are not strictly localised to
brain regions.
The correct answer is: Dominant hemisphere (Choose THREE)
– Aphasia, Dyscalculia, Limb apraxia, Non­dominant hemisphere (Choose ONE)
– Constructional apraxia, Bilateral and not strictly localised (Choose ONE)
– Personality

Question 12 HiY Neurosciences EMI062
Partially correct CSF Flow
Identify the connections that facilitate the flow of CSF
Mark 2.67 out of
4.00
Foramen of Magendie
Flag question
Foramen of Luschka
Foramina of Monroe
Lateral ventricles to 3rd
Cerebral aqueduct of Sylvius
ventricle
Foramen of Lucas
Foramen magnum
Jugular foramen
Foramen ovale
Internal auditory meatus
Foramen Rotandum

661
3rd ventricle to 4th Foramen of Magendie
ventricle Foramen of Luschka
Foramina of Monroe
Cerebral aqueduct of Sylvius
Foramen of Lucas
Foramen magnum
Jugular foramen
Foramen ovale
Internal auditory meatus
Foramen of Magendie
Foramen Rotandum
Foramen of Luschka
4th ventricle to Foramina of Monroe
subarachnoid space (2 Cerebral aqueduct of Sylvius
answers) Foramen of Lucas
Foramen magnum
Jugular foramen
Foramen ovale
Internal auditory meatus
Check
Foramen Rotandum

Explanation: CSF is secreted by the choroid plexus in the lateral, third and fourth ventricles
and at a rate of 300 ml/day, which is almost protein free. 
Route: From the lateral ventricle to 3rd ventricle via interventricular foramina of Monroe. From
3rd to 4th Ventricle via cerebral aqueduct of Sylvius. From 4th ventricle to subarachnoid space
via Foramen of Magendie (middle) and Foramen of Luschka (two, lateral).
The correct answer is: Lateral ventricles to 3rd ventricle – Foramina of Monroe, 3rd ventricle to
4th ventricle – Cerebral aqueduct of Sylvius, 4th ventricle to subarachnoid space (2 answers)
– Foramen of Magendie, Foramen of Luschka

Question 13 HiY Neurosciences EMI063
Correct Brain lesions & syndromes
For each of the clinical descriptions given below, identify the terms used to describe them
Mark 5.00 out of
5.00

Remove flag A 57­year­old man with
Kluver­Bucy syndrome
history of blindness Prosopagnosia
following major stroke Marchiafava­ Bignami syndrome
denies any visual Simultanagnosia
deficits and attempts to Achromatopsia
walk in his room Anton syndrome
without any aid Korsakoff syndrome
Alexia without agraphia
Finger Agnosia
Kluver­Bucy syndrome
Horner syndrome
Prosopagnosia
This is one of the Marchiafava­ Bignami syndrome
primary symptoms of Simultanagnosia
Gerstmann syndrome Achromatopsia
Anton syndrome
Korsakoff syndrome
Alexia without agraphia

662
A 67­year­old woman Kluver­Bucy syndrome
has suffered from a
Prosopagnosia
watershed infarction of
Marchiafava­ Bignami syndrome
the brain. Following this
Simultanagnosia
event, she is unable to
attend to more than one Achromatopsia
item of a complex Anton syndrome
scene at a time Korsakoff syndrome
Alexia without agraphia
Finger Agnosia
Kluver­Bucy syndrome
A 77­year­old man with
Horner syndrome
Prosopagnosia
history of recent Marchiafava­ Bignami syndrome
cerebral infarct is Simultanagnosia
unable to discriminate Achromatopsia
colours Anton syndrome
Korsakoff syndrome
Alexia without agraphia
Finger Agnosia
Kluver­Bucy syndrome
Horner syndrome
Prosopagnosia
Marchiafava­ Bignami syndrome
A 67­year­old woman
Simultanagnosia
can normally speak Achromatopsia
and comprehend what Anton syndrome
is spoken. She can Korsakoff syndrome
write spontaneously, Alexia without agraphia
but reading Finger Agnosia
comprehension is Horner syndrome
impaired.

Check

Explanation: 
Anton syndrome is a visual agnosia, in which the patient denies any deficit and may attempt to
negotiate the environment, invariably without success. In the curious phenomenon known as
blindsight, visual stimuli can induce a response despite cortical blindness. 
Gerstmann syndrome is characterized by four symptoms: dysgraphia/agraphia,
dyscalculia/acalculia, finger agnosia and right­left disorientation. It is rarely seen as a full
tetrad. It is often associated with brain lesions in the dominant parietal lobe (angular and
supramarginal gyri). 
Balint syndrome is a triad of simultanagnosia (inability to attend to more than one item of a
visual scene at the same time), optic ataxia (inability to guide one's reach despite intact
vision), and oculomotor apraxia (inability to freely direct visual saccades to a target). Possible
causes include carbon monoxide poisoning, watershed infarction, leukodystrophy, and the
posterior cortical variant of Alzheimer's disease. 

663
Colour processing deficits such as achromatopsia (loss of ability to discriminate colours) are
often associated with pure alexia after medial occipitotemporal damage, following left
posterior cerebral artery infarction (Kipps & Hodges, 2005). 
Pure word blindness (alexia no agraphia): Here the patient can normally speak and
comprehend what is spoken; he can also write spontaneously and to dictation, but reading
comprehension is impaired. 

Kipps and Hodges (2005), Cognitive assessment for clinicians. Excerpt retrieved from
http://jnnp.bmj.com/content/76/suppl_1/i22.full
The correct answer is: A 57­year­old man with history of blindness following major stroke
denies any visual deficits and attempts to walk in his room without any aid – Anton syndrome,
This is one of the primary symptoms of Gerstmann syndrome – Finger Agnosia, A 67­year­old
woman has suffered from a watershed infarction of the brain. Following this event, she is
unable to attend to more than one item of a complex scene at a time – Simultanagnosia, A 77­
year­old man with history of recent cerebral infarct is unable to discriminate colours –
Achromatopsia, A 67­year­old woman can normally speak and comprehend what is spoken.
She can write spontaneously, but reading comprehension is impaired. – Alexia without
agraphia

Question 14 HiY Neurosciences EMI064
Correct Neurohistology 
Match the following neuroglial cells with their specific functions from the list provided:
Mark 7.00 out of
7.00

Flag question
CNS myelin sheath formation.
Peripheral myelin sheath formation
Aids the flow of CSF
Astrocytes   
Breakdown of some neurotransmitters
Apoptotic clearance
Act as scavenger cells at sites of CNS injury
Neuronal support in sensory and autonomic ganglia
Neurilemma formation
CNS myelin sheath formation.
Peripheral myelin sheath formation
Aids the flow of CSF
Oligodendrocytes  
Breakdown of some neurotransmitters
Apoptotic clearance
Act as scavenger cells at sites of CNS injury
Neuronal support in sensory and autonomic ganglia
Neurilemma formation
CNS myelin sheath formation.
Peripheral myelin sheath formation
Aids the flow of CSF
Breakdown of some neurotransmitters  

Microglia
Apoptotic clearance
Act as scavenger cells at sites of CNS injury
Neuronal support in sensory and autonomic ganglia
Neurilemma formation

664
Ependymal cells CNS myelin sheath formation.
Peripheral myelin sheath formation
Aids the flow of CSF
Breakdown of some neurotransmitters  
Apoptotic clearance
Act as scavenger cells at sites of CNS injury
Neuronal support in sensory and autonomic ganglia
Neurilemma formation
CNS myelin sheath formation.
Peripheral myelin sheath formation
Aids the flow of CSF
Schwann cells  
Breakdown of some neurotransmitters
Apoptotic clearance
Act as scavenger cells at sites of CNS injury
Neuronal support in sensory and autonomic ganglia
Neurilemma formation
Check

Explanation: Astrocytes are the most numerous of all three types of glial cells. These are star­
shaped cells that enable nutrition of neurons, breakdown of some neurotransmitters, and
maintaining the blood­brain barrier. 
Oligodendrocytes are seen in CNS (not in peripheral nerves, where Schwann cells replace
them). They produce myelin sheaths that help in the saltatory conduction of an action potential
(pole to pole jumping), which quicken the process of signal transmission. 
The microglia are descendants of macrophages. They are scavenger cells that clear neuronal
debris following cell death. 
Ependymal cells are a particular type of glia that cover the ventricles and enable CSF
circulation. 
Schwann cells are seen in the peripheral nervous system where they produce myelin
sheaths.
The correct answer is: Astrocytes 
– Breakdown of some neurotransmitters, Oligodendrocytes – CNS myelin sheath formation.,
Microglia – Act as scavenger cells at sites of CNS injury, Ependymal cells – Aids the flow of
CSF, Schwann cells – Peripheral myelin sheath formation

Question 15 HiY Neurosciences EMI065
Correct Neuroanatomical lesions in CNS disorders 
For each of the condition below choose the most specific option/s from given list
Mark 6.00 out of
6.00

Flag question
Inferior frontal gyrus
Hypothalamus
Corpus callosum
Korsakoff's  
Angular gyrus
psychosis
Medial Temporal Lobe
Fusiform gyrus
Third ventricle
Dorsolateral prefrontal cortex
Mamillary bodies

665
Inferior frontal gyrus
Gerstmann
Hypothalamus
syndrome
Corpus callosum
Angular gyrus  
Medial Temporal Lobe
Fusiform gyrus
Third ventricle
Dorsolateral prefrontal cortex
Mamillary bodies
Inferior frontal gyrus
Hypothalamus
Corpus callosum
 
Prosopagnosia Angular gyrus
Medial Temporal Lobe
Fusiform gyrus
Third ventricle
Dorsolateral prefrontal cortex
Mamillary bodies
Inferior frontal gyrus
Hypothalamus
Corpus callosum
Alzheimer's disease Angular gyrus  
Medial Temporal Lobe
Fusiform gyrus
Third ventricle
Dorsolateral prefrontal cortex
Mamillary bodies
Check

Explanation: 
About 80 per cent of alcoholic patients recovering from Wernicke's encephalopathy develop
Korsakoff's amnesic syndrome. Diagnosis of Korsakoff's syndrome correlates best with the
presence of lesions in the dorsomedial thalamus. But lesions in the mammillary bodies, the
mammillothalamic tract and the anterior thalamus may be more relevant to memory
dysfunction than lesions in the medial dorsal nucleus of the thalamus (Kopelamn et al., 2009).
Gerstmann syndrome is characterized by four primary symptoms: Dysgraphia/agraphia,
Dyscalculia/acalculia, Finger agnosia and Left­right disorientation. This is often associated
with brain lesions in the dominant (usually left) side of the angular and supramarginal gyri
(parietal lobe). 
Prosopagnosics cannot recognise familiar faces. Face processing is a bilateral function; more
vital areas may be present on the right hemisphere. Acquired prosopagnosia is usually
associated with bilateral or right­sided lesions of the occipital ­ temporal junction (fusiform
gyrus). 
In Alzheimer's disease, the main areas affected are the hippocampus and medial temporal
lobe. Atrophic changes are mainly observed in these two areas in even in very early stages of
Alzheimer's disease.

Barton, JJ., (2008). Prosopagnosia associated with a left occipitotemporal lesion. Excerpt
retrieved from http://www.ncbi.nlm.nih.gov/pubmed/18374372 
Kopeman et al., (2009). The Korsakoff Syndrome: Clinical Aspects, Psychology and
Treatment. Excerpt retrieved from http://alcalc.oxfordjournals.org/content/44/2/148

666
The correct answer is: Korsakoff's psychosis
– Mamillary bodies, Gerstmann syndrome – Angular gyrus, Prosopagnosia – Fusiform gyrus,
Alzheimer's disease
– Medial Temporal Lobe

Question 16 HiY Neurosciences EMI066
Partially correct Neurotransmitters
For the following neurotransmitters choose the associated disorders from the given list
Mark 6.00 out of
8.00

Remove flag
Alzheimer's disease
Obsessive compulsive disorder
Huntington's disease
Dopamine  
Panic disorder
Parkinson's disease
Motor neurone disease
Depression
Alzheimer's disease
Obsessive compulsive disorder
Huntington's disease
Noradrenaline  
Panic disorder
Parkinson's disease
Motor neurone disease
Depression
Alzheimer's disease
Obsessive compulsive disorder
Huntington's disease
Acetylcholine  
Panic disorder
Parkinson's disease
Motor neurone disease
Depression
Alzheimer's disease
Obsessive compulsive disorder
Huntington's disease
Panic disorder  
Parkinson's disease
GABA Motor neurone disease
Depression

Check

Explanation: Dopamine is associated with Parkinson's disease and schizophrenia.
Noradrenalin is associated with both anxiety and depression. Acetylcholine is associated with
Alzheimer's disease. GABA is thought to be associated with schizophrenia, anxiety as well as

667
Huntington's disease.
The correct answer is: Dopamine
– Parkinson's disease, Noradrenaline – Depression, Acetylcholine – Alzheimer's disease,
GABA
– Huntington's disease

Question 17 HiY Neurosciences EMI067
Partially correct Neuropeptides & disorders
Identify the function/disorder associated with each of the neuropeptides listed below
Mark 5.40 out of
9.00

Remove flag
Mood
Panic
Circadian function
Beta­endorphins  
Anxiety
Arousal
Cognition
OCD
Mood
Panic
Circadian function
Vasopressin  
Anxiety
Arousal
Cognition
OCD
Mood
Panic
Circadian function
Cholecystokinin   
Anxiety
Arousal
Cognition
OCD
Mood
Panic
Circadian function
Anxiety  
Arousal
Orexins 
Cognition
OCD

Mood
Panic
Circadian function
Met­enkephalin   
Anxiety
Arousal
Cognition
OCD

668
Check

Explanation: Endorphins are endogenous opioid neurotransmitters produced by the pituitary
and the hypothalamus during exercise, emotional excitement and experience of pain.
Endorphins like their synthetic counterparts ­ opiates, produce analgesia and a feeling of well­
being. Beta­endorphin is the most potent natural opioid. It is associated with pain and mood
regulation. "Although the effects of vasopressin analogs on cognition are inconsistent, overall
(Strupp and Levitsky, 1985), the evolving evidence seems to suggest that vasopressin
regulates various aspects of cognition, including learning, memory, and/or attention"(Jentsch,
2003). 
Cholecystokinin is associated with anxiety and pain. It has been implicated in the
pathophysiologies of schizophrenia, eating disorders and movement disorders. CCK infusion
induces panic attacks in people with panic disorder. CCK antagonists may have anxiolytic
effects. 
Orexins A and B (hypocretins) are closely related neuropeptides derived from a single gene.
They act as OX1 and OX2 receptors, which are highly expressed in the lateral hypothalamus
and other brain areas involved in stress regulation. Orexins were initially identified as
important regulators of feeding but are now seen as involved in circadian function, sleep and
response to stress, including neuroendocrine control. 
The enkephalins (Leu­enkephalin and met­enkephalin) mediate pain perception, mood and
neuroendocrine behaviours; they are found in the brain, spinal cord, nerve plexuses and
exocrine glands of the gastrointestinal tract (Ref: J Neurosci. 2003 Feb 1;23(3):1066­71)
The correct answer is: Beta­endorphins
– Mood, Vasopressin – Cognition, Cholecystokinin 
– Anxiety, Orexins 
– Circadian function, Met­enkephalin 
– Mood

Question 18 HiY Neurosciences EMI068
Correct Agnosias & Apraxias (1)
Find one term to describe each of the following presentations
Mark 5.00 out of
5.00

Remove flag
Simultagnosia
Finger agnosia
A 76­year­old Agraphaesthesia
gentleman is having
Dressing apraxia
difficulty recognizing
complex pictures
Prosopagnosia
Visuospatial agnosia
Ideomotor apraxia
Astereognosia
Hemisomatognosia
Simultagnosia
Anosognosia
Finger agnosia
Agraphaesthesia
Dressing apraxia
An 83­year­old woman
is having difficulty
Prosopagnosia
copying a cube Visuospatial agnosia
diagram on extended Ideomotor apraxia
cognitive examination Astereognosia
Hemisomatognosia

669
A 73­year­old
gentleman with a
Simultagnosia
history of major stroke
Finger agnosia
has a left sided Agraphaesthesia
hemiplegia. However, Dressing apraxia
he denies having any Prosopagnosia
physical problems Visuospatial agnosia
including the difficulty Ideomotor apraxia
he has in walking. Astereognosia
A 78­year­old Hemisomatognosia
gentleman with Anosognosia
Simultagnosia
Alzheimer's dementia Finger agnosia
has difficulty in Agraphaesthesia
recognizing the faces of
Dressing apraxia
his children,
Prosopagnosia
grandchildren and
close friends/ However
Visuospatial agnosia
he recognizes their Ideomotor apraxia
voices Astereognosia
Hemisomatognosia
Simultagnosia
Anosognosia
Finger agnosia
A 76­year­old woman
complains of a feeling
Agraphaesthesia
that the limbs on one Dressing apraxia
side of the body are Prosopagnosia
missing Visuospatial agnosia
Ideomotor apraxia
Astereognosia
Hemisomatognosia
Check Anosognosia

Explanation: Simultanagnosia is the inability to recognize complex pictures. 
Visuospatial agnosia is the inability to construct or copy figures. Copying three­dimensional
shapes such as a wire cube, interlocking pentagons, or constructing a clock­face with
numbers are good tests of constructional ability and may also highlight visuospatial neglect if
it is present. 
Anosognosia is failure to recognize a disability. The patient does not recognise functional
deficits of their body. 
While patients with prosopagnosia cannot recognise familiar faces, they often use other
indicators such as voice, clothes or body movements to aid identification (Kipps and Hodges
2005). The identification deficit may not be limited to faces; identifying objects within other
categories may also be impaired (for example, different models of motor cars and types of
flowers). 
Hemisomatognosia is a term used when a part of the body is felt to be absent or neglected.
The correct answer is: A 76­year­old gentleman is having difficulty recognizing complex
pictures – Simultagnosia, An 83­year­old woman is having difficulty copying a cube diagram
on extended cognitive examination – Visuospatial agnosia, A 73­year­old gentleman with a

670
history of major stroke has a left sided hemiplegia. However, he denies having any physical
problems including the difficulty he has in walking. – Anosognosia, A 78­year­old gentleman
with Alzheimer's dementia has difficulty in recognizing the faces of his children, grandchildren
and close friends/ However he recognizes their voices – Prosopagnosia, A 76­year­old
woman complains of a feeling that the limbs on one side of the body are missing –
Hemisomatognosia

Question 19 HiY Neurosciences EMI069
Partially correct Constituents of brain regions
Choose the components of the following structures. (Each option can be used once, more
Mark 7.50 out of
than once or not at all)
10.00

Remove flag
Septal Nuclei
Cingulate gyrus
In addition to putamen, Hippocampus
what other structure
Globus pallidus
constitutes the corpus
striatum?
Lentiform nucleus
Amygdala
Corpus striatum
Orbitofrontal cortex
Caudate nucleus
Septal Nuclei
Cingulate gyrus
In addition to putamen, Hippocampus
what other structure
Globus pallidus
constitutes the lentiform
nucleus?
Lentiform nucleus
Amygdala
Corpus striatum
Orbitofrontal cortex
Caudate nucleus
Septal Nuclei
Cingulate gyrus
Which frontal structure Hippocampus
is a constituent of the Globus pallidus
Papez circuit? Lentiform nucleus
Amygdala
Corpus striatum
Orbitofrontal cortex
Caudate nucleus
Septal Nuclei
Cingulate gyrus
In addition to
hippocampus, what
Hippocampus
other structure Globus pallidus
constitutes the medial Lentiform nucleus
temporal lobe? Amygdala
Corpus striatum
Orbitofrontal cortex
Caudate nucleus

Check

671
Explanation: 
The basal ganglia are a group of gray matter nuclei that forms the largest subcortical structure
in the brain. The components of the basal ganglia include the caudate nucleus, the putamen
and the globus pallidus. Putamen and globus pallidus together forms the lentiform nucleus.
Caudate and putamen are often clubbed together by the name striatum. Both the subthalamic
nuclei and the substantia nigra are physiologically related to the basal ganglia though not
considered to be a part of it. 
The Papez circuit in its original description consists of the hippocampus, the fornix, the
mamillary bodies, the anterior nucleus of the thalamus, and the cingulate gyrus. The
boundaries of the limbic system were subsequently expanded to include the amygdala,
septum, basal forebrain, nucleus accumbens, and orbitofrontal cortex. 
The hippocampus and amygdala are components of the temporal lobe and play important
roles within the limbic system.
The correct answer is: In addition to putamen, what other structure constitutes the corpus
striatum?
– Caudate nucleus, In addition to putamen, what other structure constitutes the lentiform
nucleus?
– Globus pallidus, Which frontal structure is a constituent of the Papez circuit? – Cingulate
gyrus, In addition to hippocampus, what other structure constitutes the medial temporal lobe?
– Amygdala

Question 20 HiY Neurosciences EMI070
Correct Neuropsychological Tests 
For each clinical description below choose ONE appropriate neuropsychological test.
Mark 4.00 out of
4.00

Remove flag A 55­year­old lady
Beck Depression Inventory
scores a 26/30 on the Wisconsin Card Sorting Test
MMSE. But she Bender Gestalt Test
responds saying, "I Blessed Rating Scale
don't know", "I can't be Rorschach Test
bothered" for many Weschler's Memory Test
items in the test. National Adult Reading Test
Wechsler Adult Intelligence Scale­Revised
Rey­Osterrieth Test
Beck Depression Inventory
The family of a 60­year­
MMSE
Wisconsin Card Sorting Test
old lady with traumatic
brain injury is seeking Bender Gestalt Test
compensation for the Blessed Rating Scale
cognitive impairment Rorschach Test
sustained following Weschler's Memory Test
RTA. You are asked to National Adult Reading Test
provide a report on Wechsler Adult Intelligence Scale­Revised
possible premorbid IQ
Rey­Osterrieth Test
of this lady.
MMSE

A 60­year­old man has
poor attention span
with significant difficulty
organizing and
planning even simple

672
daytime activities; he Beck Depression Inventory
vehemently denies any
Wisconsin Card Sorting Test
impairment in memory.
Bender Gestalt Test
Blessed Rating Scale
Rorschach Test
Weschler's Memory Test
National Adult Reading Test
Wechsler Adult Intelligence Scale­Revised
A 16­year­old girl is Rey­Osterrieth Test
Beck Depression Inventory
disruptive and MMSE
aggressive at school.
Wisconsin Card Sorting Test
She often loses her Bender Gestalt Test
pocket money and Blessed Rating Scale
performs poorly at Rorschach Test
school. Her brother has Weschler's Memory Test
a moderate learning National Adult Reading Test
disability. Wechsler Adult Intelligence Scale­Revised
Rey­Osterrieth Test
Check MMSE

Explanation: 
Case 1: This woman is probably suffering from clinical depression, which could mimic
dementia in the elderly. Mild level of cognitive impairment is possible in depression and 'don't
know' responses are frequently noted during bedside cognitive tests. Becks' depression
inventory could be used to screen her for depression. 
NART ­ National adult reading test: This test taps on previous word knowledge before
becoming ill ­ hence used to estimate premorbid IQ. 
The Wisconsin Card Sorting Test (WCST) is useful to test frontal executive function such as
organizing and planning. It contains stimulus cards of different colour, form, and number.
These are presented to patients to sort into groups according to a single principle (e.g., to sort
by colour, ignoring form and number). Persons with damage to the frontal lobes or the
caudate and some persons with schizophrenia give abnormal responses. 
The Wechsler Adult Intelligence Scale (WAIS­R): Most widely used intelligence test in clinical
practice.The latest revision, the WAIS­III, is designed for persons 16 to 89 years of age.
Wechsler Intelligence Scale for Children­III [WISC­III] is used for <16.The WAIS is composed
of 11 subtests made up of six verbal subtests and five performance subtests, which yield a
verbal IQ, a performance IQ, and a combined or full­scale IQ.
The correct answer is: A 55­year­old lady scores a 26/30 on the MMSE. But she responds
saying, "I don't know", "I can't be bothered" for many items in the test. – Beck Depression
Inventory, The family of a 60­year­old lady with traumatic brain injury is seeking compensation
for the cognitive impairment sustained following RTA. You are asked to provide a report on
possible premorbid IQ of this lady. – National Adult Reading Test, A 60­year­old man has poor
attention span with significant difficulty organizing and planning even simple daytime
activities; he vehemently denies any impairment in memory. – Wisconsin Card Sorting Test, A
16­year­old girl is disruptive and aggressive at school. She often loses her pocket money and
performs poorly at school. Her brother has a moderate learning disability. – Wechsler Adult
Intelligence Scale­Revised

Question 21 HiY Neurosciences EMI071

673
Correct Agnosias & Apraxias 
Mark 5.00 out of
Find one term each to describe the following presentation;
5.00
70­year­old electrician
Remove flag
suffered a left
hemisphere CVA but Simultagnosia
has recovered the right Prosopagnosia
arm and leg function Dressing apraxia
completely. He found
Anosognosia
that although he
Hemisomatognosia
suffered no residual
weakness or loss of
Ideomotor apraxia
sensation, he could not Bucco­facial apraxia
use his tools and Ideational apraxia
instruments. Constructional apraxia
Astereognosia
Simultagnosia
A patient finds it difficult Prosopagnosia
to perform sequential
Dressing apraxia
actions, even though
Anosognosia
he understands and
carries out simple
Hemisomatognosia
imitations Ideomotor apraxia
Bucco­facial apraxia
Ideational apraxia
Constructional apraxia
Simultagnosia
Astereognosia
Prosopagnosia
An inability to copy Dressing apraxia
intersecting pentagons
Anosognosia
on mini­mental state
examination
Hemisomatognosia
Ideomotor apraxia
Bucco­facial apraxia
Ideational apraxia
Constructional apraxia
Simultagnosia
Astereognosia
Prosopagnosia
A 76­year­old man with
Dressing apraxia
stroke has difficulty in
Anosognosia
whistling and coughing
on command
Hemisomatognosia
Ideomotor apraxia
Bucco­facial apraxia
Ideational apraxia
Constructional apraxia
Simultagnosia
A 63­year­old man with Astereognosia
Prosopagnosia
history of younger
Dressing apraxia
onset dementia has
Anosognosia
difficulty to put on
Hemisomatognosia
clothing that has been
turned inside­out Ideomotor apraxia
Bucco­facial apraxia
Ideational apraxia
Constructional apraxia
Check
Astereognosia

674
Explanation: 
Ideomotor apraxia is a disorder of goal­directed movement. The patient knows what to do but
not how to do it. Patients show impairment in the ability to use tools. Abnormalities include (i)
abnormal amplitude of motor actions; (ii) body­part­as­object substitution, e.g. the patient uses
his finger to represent a toothbrush when asked to brush his teeth; (iii) abnormal orientation of
body part performing the action, and (iv) temporal errors. Improves on imitation and with the
use of an actual tool. Tool use more affected than gestures. 
Ideational apraxia is the inability to perform a multiple­step task. The patient does not know
what to do. They exhibit an impairment in carrying out the correct sequences of actions that
require ordered use of various objects and tools (Zadikoff and Lang, 2005). 
Constructional apraxia is inability to construct elements into a meaningful whole. e.g., inability
to draw or copy simple diagrams or figures. A lesion in the right cerebral hemisphere typically
causes it. 
Bucco­facial apraxia is the inability to coordinate and carry out facial and lip movements such
as whistling, winking, coughing, etc., on command. The subject cannot perform skilled
movements involving the lips, mouth, and tongue in the absence of paresis. 
Dressing apraxia is having the patient put on clothing that has been turned inside out.

Zadikoff, C & Lang AE. (2005). Apraxia in movement disorders. Retrieved from
http://brain.oxfordjournals.org/content/brain/128/7/1480.full.pdf
The correct answer is: 70­year­old electrician suffered a left hemisphere CVA but has
recovered the right arm and leg function completely. He found that although he suffered no
residual weakness or loss of sensation, he could not use his tools and instruments. –
Ideomotor apraxia, A patient finds it difficult to perform sequential actions, even though he
understands and carries out simple imitations – Ideational apraxia, An inability to copy
intersecting pentagons on mini­mental state examination – Constructional apraxia, A 76­year­
old man with stroke has difficulty in whistling and coughing on command – Bucco­facial
apraxia, A 63­year­old man with history of younger onset dementia has difficulty to put on
clothing that has been turned inside­out – Dressing apraxia

Question 22 HiY Neurosciences EMI072
Partially correct Psychiatry & Endocrine Disorders 
Using the clinical descriptions given below, identify the endocrine disorder.
Mark 3.00 out of
4.00
A 35­year­old Diabetes insipidus
Remove flag
gentleman is Addison's disease
presenting with fatigue, Hyperthyroidism
muscle pain,
Pheochromocytoma
depression, increased
thirst. His wife reports
Crohn's syndrome
an insidious change in Hyperparathyroidism
his personality Hypothyroidism
Cushing's syndrome
A 42­year­old woman Diabetes mellitus
Diabetes insipidus
with paroxysmal Addison's disease
palpitations and raised Hyperthyroidism
blood pressure. She
Pheochromocytoma
was referred to
psychiatrist for severe
Crohn's syndrome
anxiety and panic Hyperparathyroidism
attacks Hypothyroidism
Cushing's syndrome

675
A 56­year­old Diabetes insipidus
gentleman presented to Addison's disease
GP with a history of Hyperthyroidism
polyuria, polydipsia,
Pheochromocytoma
tiredness and
weakness. He tested
Crohn's syndrome
positive for urinary Hyperparathyroidism
ketones Hypothyroidism
Cushing's syndrome
A 42­year­old woman Diabetes mellitus
Diabetes insipidus
complains of dry skin,
Addison's disease
tiredness & lethargy.
She has bradycardia
Hyperthyroidism
and has some cognitive Pheochromocytoma
difficulties. Crohn's syndrome
Hyperparathyroidism
Hypothyroidism
Cushing's syndrome
Diabetes mellitus
Check

Explanation: 
Hyperparathyroidism is caused by an enlargement of parathyroid glands leading to too much
production of parathyroid hormone and increased levels of calcium in the blood. The clinical
features include back pain, bone pain, blurred vision, depression, and profound change in
personality, fatigue, increased thirst, increased urinary output, itchy skin, and nausea, loss of
appetite and muscle pain. 
Phaeochromocytoma is characterized by increased heart rate, increased blood pressure,
palpitations, and anxiety often resembling a panic attack, headaches & diaphoresis. 
Diabetes mellitus is characterised by polyuria, polydipsia, tiredness, weight loss, and fatigue
with excessive urinary ketones when untreated. 
Hypothyroidism manifests as weight gain, constipation, dislike of cold, menorrhagia, hoarse
voice, depression, cognitive difficulties, dry skin, tiredness & lethargy and bradycardia.
The correct answer is: A 35­year­old gentleman is presenting with fatigue, muscle pain,
depression, increased thirst. His wife reports an insidious change in his personality –
Hyperparathyroidism, A 42­year­old woman with paroxysmal palpitations and raised blood
pressure. She was referred to psychiatrist for severe anxiety and panic attacks –
Pheochromocytoma, A 56­year­old gentleman presented to GP with a history of polyuria,
polydipsia, tiredness and weakness. He tested positive for urinary ketones – Diabetes
mellitus, A 42­year­old woman complains of dry skin, tiredness & lethargy. She has
bradycardia and has some cognitive difficulties. – Hypothyroidism

Question 23 HiY Neurosciences EMI073
Correct Psychiatry & Endocrine Disorders 
Using the clinical descriptions given below, identify the disorder from list above
Mark 4.00 out of
4.00
A 30­year­old woman
Remove flag complains of vomiting,
headaches and
abdominal pain and

676
reddish colouration of
Diabetes mellitus
urine. She is labile in
Acute porphyria
her mood and often
appears confused
Cushing's syndrome
during these attacks. Hyperthyroidism
This usually happens Hypothyroidism
after a night out, having Diabetes insipidus
alcohol with friends Addison's disease
Hyperparathyroidism
Pheochromocytoma

Diabetes mellitus
A 21­year­old Acute porphyria
gentleman with Cushing's syndrome
polyuria, polydipsia, Hyperthyroidism
high plasma osmolality, Hypothyroidism
low urine osmolality Diabetes insipidus
Addison's disease
Hyperparathyroidism
Pheochromocytoma
Diabetes mellitus
A 36­year­old woman Acute porphyria
with hirsutism, weight Cushing's syndrome
gain and hypertension. Hyperthyroidism
She is also clinically Hypothyroidism
depressed. Diabetes insipidus
Addison's disease
Hyperparathyroidism
A 37­year­old Pheochromocytoma
Diabetes mellitus
gentleman with Acute porphyria
palpitations, excessive
Cushing's syndrome
sweating and weight
Hyperthyroidism
loss despite increased
Hypothyroidism
appetite. He also
suffers from anxiety Diabetes insipidus
symptoms. Addison's disease
Hyperparathyroidism
Pheochromocytoma
Check

Explanation: 
Case 1: The scenario depicts acute intermittent porphyria (AIP). It is one of the groups of
disorders of haem metabolism, characterised by neurological and psychiatric manifestations
without visible cutaneous markers. AIP manifests itself by "abdomen pain, neuropathies, and
constipation, but, unlike most types of porphyria, patients with AIP do not have a rash". It is an
autosomal dominant disorder with the presentation starting between ages 18 and 40. It is
episodic in nature, and the episodes are often triggered by certain medications including
estrogens, barbiturates and benzodiazepines. Diclofenac can precipitate an episode.
Psychiatric manifestations include depression, anxiety, delirium and psychosis. The most
important lab test is demonstrating increased urinary porphobilinogen during acute attacks.
Treatment is aimed at reducing haem synthesis by administering haemin.

677
Case 2: Diabetes insipidus results from impaired renal water resorption either due to a
reduction in pituitary ADH secretion (cranial DI) or impaired renal response to ADH
(Nephrogenic DI). This condition is characterised by polyuria, polydipsia, dilute urine,
dehydration if not drinking. Plasma osmolality should be high and urine low. 
Case 3: Cushing's syndrome is characterised by weight gain, menstrual irregularities,
amenorrhoea, hirsutism, depression, muscle weakness, fatigue and fractures. 
Case 4: Hyperthyroidism is characterised by weight loss despite increased appetite, irritability,
heat intolerance, sweating, diarrhoea, tremor, frenetic activity, anxiety, emotional lability,
psychosis, itch, and oligomenorrhoea. 

Excerpts retrieved from: Acute Intermittent Porphyria ­ Medscape;
http://emedicine.medscape.com/article/205220­overview (accessed April 5, 2015).
The correct answer is: A 30­year­old woman complains of vomiting, headaches and
abdominal pain and reddish colouration of urine. She is labile in her mood and often appears
confused during these attacks. This usually happens after a night out, having alcohol with
friends – Acute porphyria, A 21­year­old gentleman with polyuria, polydipsia, high plasma
osmolality, low urine osmolality – Diabetes insipidus, A 36­year­old woman with hirsutism,
weight gain and hypertension. She is also clinically depressed. – Cushing's syndrome, A 37­
year­old gentleman with palpitations, excessive sweating and weight loss despite increased
appetite. He also suffers from anxiety symptoms. – Hyperthyroidism

Question 24 HiY Neurosciences EMI074
Partially correct Neuroimaging techniques
From the listed options choose the best answer for each of the following descriptions:
Mark 4.00 out of
5.00

Remove flag
Functional MRI
Magnetoencephalography
Dynamic measurement Single photon emission computed tomography
of resonance of
MRI angiography
hydrogen and
phosphorus ions
Computed tomography
Magnetic resonance spectroscopy
Positron emission tomography
Contrast CT scan
Magnetic resonance imaging
Functional MRI
MRI venography
Magnetoencephalography
Measurement of static Single photon emission computed tomography
magnetic activity of MRI angiography
brain Computed tomography
Magnetic resonance spectroscopy
Positron emission tomography
Contrast CT scan
Magnetic resonance imaging
Functional MRI
Measurement of MRI venography
Magnetoencephalography
regional brain Single photon emission computed tomography
metabolism by tagging MRI angiography
organic compounds in Computed tomography
metabolic pathway Magnetic resonance spectroscopy
Positron emission tomography
Contrast CT scan
Magnetic resonance imaging
MRI venography

678
Functional MRI
Measurement of Magnetoencephalography
receptor binding and Single photon emission computed tomography
distribution using MRI angiography
radiolabelled receptor Computed tomography
agonists or antagonists
Magnetic resonance spectroscopy
Positron emission tomography
Contrast CT scan
Magnetic resonance imaging
Functional MRI
MRI venography
Magnetoencephalography
Measurement of task­ Single photon emission computed tomography
related changes in
MRI angiography
blood oxygenation
levels
Computed tomography
Magnetic resonance spectroscopy
Positron emission tomography
Contrast CT scan
Magnetic resonance imaging
Check
MRI venography

Explanation: 
Magnetic resonance spectroscopy measures the resonance­induced signal from several
biologically important nuclei with an odd number of protons. 
Changes in the magnetic field potentials due to neuronal activity in the brain can be captured
using magnetoencephalography. 
A PET scan gives direct information about neuronal metabolism. 
SPECT uses radioactive compounds to study regional differences in cerebral blood flow
within the brain. This records the pattern of photon emission from the bloodstream according
to the level of perfusion in different regions of the brain. SPECT uses compounds labeled with
single photon­emitting isotopes: iodine­123, technetium­99m, and xenon­133. 
f­MRI measures task­related changes in blood oxygenation levels. Neuronal activity is
associated with a local increase in blood flow, which can be observed as an increase local
hemoglobin concentration. This change can be detected using a T2 sequence, called Blood
Oxygen Level Dependent (BOLD) technique in fMRI. The method detects tissue perfusion
rather than neuronal metabolism directly. 
Ref: Section 3.3 ­ Neuroimaging ­ Kaplan and Sadocks Synopsis of Psychiatry. Excerpt
retrieved from http://flylib.com/books/en/4.518.1.10/1/ (accessed April 21, 2015).
The correct answer is: Dynamic measurement of resonance of hydrogen and phosphorus ions
– Magnetic resonance spectroscopy, Measurement of static magnetic activity of brain –
Magnetoencephalography, Measurement of regional brain metabolism by tagging organic
compounds in metabolic pathway – Positron emission tomography, Measurement of receptor
binding and distribution using radiolabelled receptor agonists or antagonists – Single photon
emission computed tomography, Measurement of task­related changes in blood oxygenation
levels – Functional MRI

Question 25 HiY Neurosciences EMI075
Correct Frontal lobe tests 
A 55­year­old man presents with cognitive problems, personality changes and disinhibition.
Mark 5.00 out of
You are performing various frontal lobe tests and the medical students on clinical attachment

679
5.00 are keen to know the names of these tests. With the descriptions given below, identify the type
of frontal lobe test:
Remove flag

Error correction
Proverb interpretation test
'What is common Lexical fluency
between an apple and Luria motor test
a banana?' Cognitive estimates testing
Glabellar tap
Inferential reasoning
Trail making test
Go­No Go test
Error correction
Proverb interpretation test
Lexical fluency
How tall is an average
Luria motor test
English woman?
Cognitive estimates testing
Glabellar tap
Inferential reasoning
Trail making test
Go­No Go test
Error correction
Could you tell me as Proverb interpretation test
many as words as Lexical fluency
possible starting with
Luria motor test
the letter F in one
minute but not names
Cognitive estimates testing
of people or places? Glabellar tap
Inferential reasoning
Trail making test
Go­No Go test
Error correction
Proverb interpretation test
This test has two parts.
Lexical fluency
In test A, a simple
Luria motor test
number sequence is
used to join the dots.
Cognitive estimates testing
Test B uses alternating Glabellar tap
numbers and letters. Inferential reasoning
Trail making test
Go­No Go test

Error correction
You instruct the patient Proverb interpretation test
to tap once in response Lexical fluency
to a single tap and to Luria motor test
withhold a response for Cognitive estimates testing
two taps. Glabellar tap
Inferential reasoning
Trail making test
Go­No Go test
Check

Explanation: 

680
To assess the faculty of inferential reasoning, we can use questions about the similarity
between two conceptually similar objects. When testing similarity it is best to first test simple
pairs such as "apples and oranges" or "desk and chair", followed by more abstract pairs such
as "honesty and sincerity" or "music and melody". 
To test phonological (letter) fluency, patients are asked to produce as many words as possible
starting with the alphabets F, A, and S, one after the other. To test semantic (category) fluency,
patients are asked to produce as many words as possible in the category of animals (or
supermarket goods). 
The cognitive estimates test is an attempt to prompt improbable responses that are sometimes
seen in patients with frontal lesions. The questions include checking for facts such as the
height of Big Ben, the population of UK, or the speed of a typical train. 
Trail making test consists of two parts. In part A, a simple number sequence is used to join the
dots. In part B alternating numbers and letters are used as this is more sensitive to frontal lobe
dysfunction. Trail making tests are not specific to frontal functions, but also test visuomotor
tracing, attention, conceptualisation and set shifting. 
Impulsivity reflects a failure of response inhibition, and is seen in inferior frontal pathology. It
can be assessed using the Go­No­Go task. The examiner instructs the patient to tap once in
response to a single tap and to withhold a response for two taps. This test can be made more
difficult by changing the initial rule after several trials (for example, "tap once when I tap twice,
and not at all when I tap once").
The correct answer is: 'What is common between an apple and a banana?' – Inferential
reasoning, How tall is an average English woman? – Cognitive estimates testing, Could you
tell me as many as words as possible starting with the letter F in one minute but not names of
people or places? – Lexical fluency, This test has two parts. In test A, a simple number
sequence is used to join the dots. Test B uses alternating numbers and letters. – Trail making
test, You instruct the patient to tap once in response to a single tap and to withhold a response
for two taps. – Go­No Go test

Question 26 HiY Neurosciences EMI076
Partially correct Neuroimaging tests in psychiatry 
Identify the most likely neuroimaging findings for the following psychiatric conditions
Mark 4.00 out of
6.00
A 22­year­old
Remove flag gentleman with Ventricular enlargement
persecutory delusions Increased blood flow in parietotemporal areas in SPECT
and 3rd person Poor activation of dorsolateral prefrontal cortex in executive tasks during f­M
auditory hallucinations.
Lower prefrontal metabolism
In addition to
Loss of white matter in the dorsolateral prefrontal cortex
ventricular
enlargement, what Increased frontal lobe volumes
other feature can be Progressive increase in brain volume in first few years of diagnosis
seen? Decreased metabolism in prefrontal and anterior cingulate cortex
A 29­year­old woman
with low mood, Ventricular enlargement
anhedonia and poor Increased blood flow in parietotemporal areas in SPECT
sleep. In addition to Poor activation of dorsolateral prefrontal cortex in executive tasks during f­M
white matter Lower prefrontal metabolism
hyperintensities, what Loss of white matter in the dorsolateral prefrontal cortex
other feature can be Increased frontal lobe volumes
seen? Progressive increase in brain volume in first few years of diagnosis
Decreased metabolism in prefrontal and anterior cingulate cortex

681
A 77­year­old man with
Ventricular enlargement
history of short­term
Increased blood flow in parietotemporal areas in SPECT
memory loss and poor
concentration. In Poor activation of dorsolateral prefrontal cortex in executive tasks during f­M
addition to medial Lower prefrontal metabolism
temporal atrophy, what Loss of white matter in the dorsolateral prefrontal cortex
other feature can be Increased frontal lobe volumes
seen? Progressive increase in brain volume in first few years of diagnosis
Decreased metabolism in prefrontal and anterior cingulate cortex
Check

Explanation: 
Case 1: The diagnosis is schizophrenia. Neuroimaging findings in schizophrenia include
ventricular enlargement and loss of grey matter ­ especially insula, anterior cingulate cortex,
medial temporal lobe and dorsolateral pre­frontal cortex with some progressive loss of brain
volume in the first few years of diagnosis. fMRI shows poor dorsolateral pre­frontal cortex
activation in executive tasks, while decreased N­acetyl aspartate in prefrontal cortex is seen in
spectroscopy. 
Case 2: The diagnosis is depressive disorder. Neuroimaging findings in depression include
periventricular and deep WM hyperintensities, subcortical thalamic and striatal
hyperintensities, decreased frontal and basal ganglia volumes, and decreased metabolism in
prefrontal cortex. 
Case 3: The diagnosis is Alzheimer's dementia. Neuroimaging findings in Alzheimer's
disease include ventricular enlargement, reduced medial temporal lobe volume ­ especially
hippocampus and a reduction in parieto­temporal fMRI activation and SPECT blood flow.
The correct answer is: A 22­year­old gentleman with persecutory delusions and 3rd person
auditory hallucinations. In addition to ventricular enlargement, what other feature can be
seen?
– Poor activation of dorsolateral prefrontal cortex in executive tasks during f­MRI, A 29­year­
old woman with low mood, anhedonia and poor sleep. In addition to white matter
hyperintensities, what other feature can be seen? – Decreased metabolism in prefrontal and
anterior cingulate cortex, A 77­year­old man with history of short­term memory loss and poor
concentration. In addition to medial temporal atrophy, what other feature can be seen? –
Ventricular enlargement

Question 27 HiY Neurosciences EMI077
Correct Parietal lobe function
Find one term each to describe the following presentation;
Mark 4.00 out of
4.00
A 65­year­old
Flag question gentleman is unable to
identify familiar objects
Agraphaesthesia
such as key and coins Astereognosis
by touching while his Visuospatial agnosia
eyes are closed. He Finger agnosia
has difficulty in Dressing apraxia
describe their shape Agraphia
and texture. However, Dyscalculia
he identifies them when Right to left disorientation
shown to him.
Two point discrimination
Constructional apraxia
682
Agraphaesthesia
A 67­year­old woman Astereognosis
was unable to Visuospatial agnosia
recognize what number Finger agnosia
is scratched into her Dressing apraxia
skin without seeing Agraphia
Dyscalculia
Right to left disorientation
Two point discrimination
Agraphaesthesia
A 73­year­old man with Constructional apraxia
Astereognosis
advanced Alzheimer's Visuospatial agnosia
disease is unable to Finger agnosia
name fingers in his Dressing apraxia
hands Agraphia
Dyscalculia
Right to left disorientation
Two point discrimination
Agraphaesthesia
Constructional apraxia
Astereognosis
A 66­year­old woman
with vascular dementia Visuospatial agnosia
has difficulty in Finger agnosia
subtracting three from Dressing apraxia
100. Agraphia
Dyscalculia
Right to left disorientation
Two point discrimination
Check
Constructional apraxia

Explanation: Astereognosis is defined as inability to recognize objects by palpation, and
without visual inspection. Agraphaesthesia is defined as inability to recognize what number or
alphabet is scratched on one's skin without seeing. Finger Agnosia is inability to
name/number fingers. Dyscalculia is an inability to calculate.
The correct answer is: A 65­year­old gentleman is unable to identify familiar objects such as
key and coins by touching while his eyes are closed. He has difficulty in describe their shape
and texture. However, he identifies them when shown to him. – Astereognosis, A 67­year­old
woman was unable to recognize what number is scratched into her skin without seeing –
Agraphaesthesia, A 73­year­old man with advanced Alzheimer's disease is unable to name
fingers in his hands – Finger agnosia, A 66­year­old woman with vascular dementia has
difficulty in subtracting three from 100. – Dyscalculia

Question 28 HiY Neurosciences EMI078
Correct Neurocognitive examination findings
Identify characteristic findings for each of the following situations;
Mark 6.00 out of
6.00
A 70­year­old man is
Remove flag diagnosed with frontal
meningioma on the
dominant side. He has
impaired verbal

683
fluency. What other
Ataxic gait
feature can be
Pure Word Deafness
expected?
Dyscalculia
Impaired set shifting
Verbal Memory Deficits
Homonymous hemianopia
Finger Agnosia
Stereognosis
Ataxic gait
A 59­year­old man Pure Word Deafness
suffers from a left Dyscalculia
parietal stroke. He has
Impaired set shifting
dyscalculia. What other
feature can be
Verbal Memory Deficits
expected? Homonymous hemianopia
Finger Agnosia
Stereognosis
A 60­year­old lady
Ataxic gait
sustains a fall and Pure Word Deafness
knocks the back of her Dyscalculia
head against the floor. Impaired set shifting
She is diagnosed with Verbal Memory Deficits
a large haematoma in Homonymous hemianopia
the occipital region.  Finger Agnosia
Stereognosis

Check

Explanation: Impaired verbal fluency and impaired set shifting can be seen in frontal lobe
lesions.A dominant parietal lesion can cause finger agnosia and dyscalculia. Stereognosis is
normal; astereognosis can occur in parietal lesions.Cortical blindness and elementary visual
hallucinations in the form of photopsia are typical in occipital infarcts.
The correct answer is: A 70­year­old man is diagnosed with frontal meningioma on the
dominant side. He has impaired verbal fluency. What other feature can be expected?
– Impaired set shifting, A 59­year­old man suffers from a left parietal stroke. He has
dyscalculia. What other feature can be expected? – Finger Agnosia, A 60­year­old lady
sustains a fall and knocks the back of her head against the floor. She is diagnosed with a
large haematoma in the occipital region. 
– Homonymous hemianopia

Question 29 HiY Neurosciences EMI079
Correct Neuropsychological measurement 
Identify the neuropsychological assessment, which could be used in each of the following
Mark 4.00 out of
situations
4.00

Remove flag

684
You want to use an IQ Digit span
test that has cultural
Stanford­Binet test
and lingual sensitivity
in order to measure IQ National Adult Reading Test
of a refugee who Ravens progressive matrices
speaks very limited Cognitive estimates test
English. Trail making test
Rivermead Behavioural Memory test
Stroop test
Luria­Nebraska battery
You want to use a tool Mini­Mental State Examination
Digit span
to screen for the Stanford­Binet test
presence of a
National Adult Reading Test
significant brain injury
Ravens progressive matrices
before undertaking a
detailed
Cognitive estimates test
neuropsychological Trail making test
examination. Rivermead Behavioural Memory test
Stroop test
You want to measure Luria­Nebraska battery
Digit span
the impairment of Mini­Mental State Examination
Stanford­Binet test
everyday memory
National Adult Reading Test
functioning to facilitate
Ravens progressive matrices
rehabilitation of
individuals with
Cognitive estimates test
acquired brain Trail making test
damage. Rivermead Behavioural Memory test
Stroop test
Luria­Nebraska battery
Check
Mini­Mental State Examination

Explanation: Ravens Progressive Matrices test IQ using non­verbal, performance measures. It
is not as culture­biased as the WAIS and does not require a fluent use of English language.
Both Luria­Nebraska battery and Halstead­Reitan battery are used as screening measures to
detect the presence of brain damage and localise possible areas damaged using
neuropsychological tests before further detailed exploration. The Rivermead Behavioural
Memory Test is designed to measure the demand made on memory by normal daily life to aid
rehabilitation. It is also sensitive to change. It is not based on any theoretical model of
memory.
The correct answer is: You want to use an IQ test that has cultural and lingual sensitivity in
order to measure IQ of a refugee who speaks very limited English.
– Ravens progressive matrices, You want to use a tool to screen for the presence of a
significant brain injury before undertaking a detailed neuropsychological examination.
– Luria­Nebraska battery, You want to measure the impairment of everyday memory
functioning to facilitate rehabilitation of individuals with acquired brain damage.
– Rivermead Behavioural Memory test

Question 30 HiY Neurosciences EMI080
Correct EEG abnormalities
For each of the following scenarios choose the most likely EEG abnormality.
Mark 5.00 out of
5.00

685
Remove flag A 45­year­old
gentleman presents Fast beta activity
with depression and Normal EEG
anxiety with cognitive Reduced & slow alpha waves
impairment. His Flattened trace
grandfather committed
Spike and wave activity
suicide at the age of 65
and his father
Periodic K Complexes
developed 'some Sleep spindles
dementia' when he was Posteriorly reduced alpha.
60. Increased delta waves
Focal frontal slowing of EEG
A 68­year­old man has
been dependent on
Triphasic sharp waves.
alcohol since the death
of his wife three years Fast beta activity
ago. He develops a Normal EEG
chest infection and gets Reduced & slow alpha waves
admitted at a hospital
Flattened trace
where he develops a
Spike and wave activity
sudden onset of visual
hallucinations and Periodic K Complexes
intense fear. He is Sleep spindles
visibly shaking and Posteriorly reduced alpha.
sweating on Increased delta waves
examination. Focal frontal slowing of EEG
A 32­year­old lady has Triphasic sharp waves.
been arrested for
combative behaviour at
Fast beta activity
a public place. She has Normal EEG
a history of episodic Reduced & slow alpha waves
aggression and has Flattened trace
been found to be Spike and wave activity
confused and amnesic Periodic K Complexes
following some Sleep spindles
episodes of bizarre Posteriorly reduced alpha.
behaviour.
Increased delta waves
Focal frontal slowing of EEG
Check Triphasic sharp waves.

Explanation: 
The first scenario is suggestive of Huntington's disease. In a number of patients, it presents
with subtle psychiatric disturbances before neurological symptoms can be seen. A
generalized reduction of alpha activity is an early sign; later the EEG may show a flat trace. 
The second scenario is one of the delirium tremens. In patients with other types of delirium,
usually the EEG shows a diffuse slowing of the background rhythm. In delirium tremens, the
EEG shows fast activity or can be normal.(Ref: Gleason OC. Delirium. Am Fam Physician.
2003 Mar 1;67(5):1027­1034). 
The last scenario is suggestive of complex partial seizures. Spike and wave activity is
classical of epilepsy; in this case it may initially be lateralized.
The correct answer is: A 45­year­old gentleman presents with depression and anxiety with
cognitive impairment. His grandfather committed suicide at the age of 65 and his father
developed 'some dementia' when he was 60. – Reduced & slow alpha waves, Flattened trace,

686
A 68­year­old man has been dependent on alcohol since the death of his wife three years
ago. He develops a chest infection and gets admitted at a hospital where he develops a
sudden onset of visual hallucinations and intense fear. He is visibly shaking and sweating on
examination. – Fast beta activity , Normal EEG, A 32­year­old lady has been arrested for
combative behaviour at a public place. She has a history of episodic aggression and has
been found to be confused and amnesic following some episodes of bizarre behaviour. –
Spike and wave activity

Question 31 HiY Neurosciences EMI081
Correct Behavioural genetics 
Choose the most relevant pattern of inheritance for each scenario described below.
Mark 5.00 out of
5.00
A 32­year­old teacher
Remove flag gives birth to a female
Chromosomal translocation
child. The child was
Meiotic nondisjunction
hypotonic at birth and Uniparental disomy
needed extended Parent of origin effect
resuscitation after birth. Autosomal recessive
Umbilical hernia and Autosomal dominant
Simian crease were X­linked dominant
noted. Polymorphism
An 18 months old X­linked recessive
female child presents Trinucleotide repeat syndrome
with developmental
stagnation, Chromosomal translocation
deceleration of the Meiotic nondisjunction
head circumference, Uniparental disomy
and psychomotor
Parent of origin effect
regression with crying
Autosomal recessive
spells. Odd circular
movements around the
Autosomal dominant
midline of the body are X­linked dominant
also noted. Later, Polymorphism
autistic traits become X­linked recessive
apparent. Trinucleotide repeat syndrome
A 4 years old child
shows severe mental
retardation, seizures,
Chromosomal translocation
and ataxic gait. There is
Meiotic nondisjunction
no family history of Uniparental disomy
similar illness. Facial Parent of origin effect
features reveal blue Autosomal recessive
eyes with the flattened Autosomal dominant
occiput, prominent jaw, X­linked dominant
wide mouth and Polymorphism
pointed chin.
X­linked recessive
Trinucleotide repeat syndrome
Check

687
Explanation: Down Syndrome ­ Trisomy 21 (95% of Down syndrome) is caused by a meiotic
nondisjunction event (in 88% of trisomies)or extra chromosome 21 material (from
chromosome 14) due to a Robertsonian translocation in the karyotype of one of the parents. 
More than 99 percent of classic Rett syndrome cases result from new mutations in the MECP2
gene, but a few families have been described with an X­linked dominant pattern of
inheritance.
Angelman syndrome is caused by the loss of the normal maternal contribution to a region of
chromosome 15, most commonly by deletion, uniparental disomy, translocation, or single
gene mutation in that region.
The correct answer is: A 32­year­old teacher gives birth to a female child. The child was
hypotonic at birth and needed extended resuscitation after birth. Umbilical hernia and Simian
crease were noted. – Chromosomal translocation, Meiotic nondisjunction, An 18 months old
female child presents with developmental stagnation, deceleration of the head circumference,
and psychomotor regression with crying spells. Odd circular movements around the midline of
the body are also noted. Later, autistic traits become apparent. – X­linked dominant, A 4 years
old child shows severe mental retardation, seizures, and ataxic gait. There is no family history
of similar illness. Facial features reveal blue eyes with the flattened occiput, prominent jaw,
wide mouth and pointed chin. – Uniparental disomy, Parent of origin effect

Question 32 HiY Neurosciences EMI082
Correct Neurochemical mediators 
For each of the following description, choose the most appropriate neurotransmitter from the
Mark 4.00 out of
list above.
4.00

Remove flag
Nitric oxide
Acts predominantly as Dopamine
a hormone; the Adrenaline
immediate precursor is GABA
a major Neuropeptide Y
neurotransmitter. Histamine
Acetylcholine
Anandamide
Glutamate
Nitric oxide
Serotonin
Dopamine
The major inhibitory Endorphin
Adrenaline
neurotransmitter of the Glycine
GABA
central nervous system
Noradrenaline
Neuropeptide Y
in humans.
Histamine
Acetylcholine
Anandamide
Glutamate
Nitric oxide
Inhibition of reuptake of Serotonin
Dopamine
this transmitter can Endorphin
Adrenaline
produce intense reward Glycine
GABA
and sense of Noradrenaline
Neuropeptide Y
satisfaction in cocaine
Histamine
users.
Acetylcholine
Anandamide
Glutamate
Serotonin

688
Nitric oxide
Dopamine
Substances present in Adrenaline
tobacco smoke mimic
GABA
the action of this
neurotransmitter.
Neuropeptide Y
Histamine
Acetylcholine
Anandamide
Glutamate
Check
Serotonin
Endorphin
Glycine
Noradrenaline
Epinephrine (also known as adrenaline) is a hormone and a neurotransmitter. Tyrosine is first
oxidized to L­DOPA, which is subsequently decarboxylated to give dopamine, oxidation of
which gives norepinephrine, which is methylated to give epinephrine. [Kaplan & Sadock's
Synopsis of Psychiatry: Behavioral Sciences/Clinical Psychiatry, 10th Edition Pg 103]
The two major amino acid neurotransmitters are GABA and glutamate. GABA is an inhibitory
amino acid, and glutamate is an excitatory amino acid.[Kaplan & Sadock's Synopsis of
Psychiatry: Behavioral Sciences/Clinical Psychiatry, 10th Edition Pg 108]. 
Amphetamines cause the release of dopamine and cocaine blocks the uptake of dopamine in
the nucleus accumbens. It is thought that cocaine and amphetamine exert the hedonistic
effect due to increased dopaminergic neurotransmission in Nucleus accumbens.[Kaplan &
Sadock's Synopsis of Psychiatry: Behavioral Sciences/Clinical Psychiatry, 10th Edition Pg
104]. 
The psychoactive component of tobacco is nicotine, which affects the central nervous system
(CNS) by acting as an agonist at the nicotinic subtype of acetylcholine receptors. [Kaplan &
Sadock's Synopsis of Psychiatry: Behavioral Sciences/Clinical Psychiatry, 10th Edition Pg
440]
The correct answer is: Acts predominantly as a hormone; the immediate precursor is a major
neurotransmitter. – Adrenaline, The major inhibitory neurotransmitter of the central nervous
system in humans. – GABA, Inhibition of reuptake of this transmitter can produce intense
reward and sense of satisfaction in cocaine users. – Dopamine, Substances present in
tobacco smoke mimic the action of this neurotransmitter. – Acetylcholine

Question 33 HiY Neurosciences EMI083
Correct Neural tube derivatives 
Match the brain components with their derivatives of the neural tube
Mark 5.00 out of
5.00

Flag question
Myelencephalon
Mesencephalon
Telencephalon
Caudal part of medulla
Rosterencephalon
oblongata
Paraencephalon
Metencephalon
Rhomboncephalon
Diencephalon

689
Thalamus
Myelencephalon
Mesencephalon
Telencephalon
Rosterencephalon
Paraencephalon
Metencephalon
Rhomboncephalon
Diencephalon
Myelencephalon
Mesencephalon
Telencephalon
Corpus striatum Rosterencephalon
Paraencephalon
Metencephalon
Rhomboncephalon
Diencephalon
Myelencephalon
Mesencephalon
Telencephalon
Hypothalamus Rosterencephalon
Paraencephalon
Metencephalon
Rhomboncephalon
Diencephalon
Myelencephalon
Mesencephalon
Telencephalon
Red nucleus Rosterencephalon
Paraencephalon
Metencephalon
Rhomboncephalon
Diencephalon
Check

Explanation: During embryonic development the midline neural tube differentiates into the
following vesicles. 
Prosencephalon which differentiates into the Telencephalon (cerebrum, striatum and
pallidum) and Diencephalon (thalamus, subthalamus, pineal body, hypothalamus and
epithalamus).
Mesencephalon (midbrain) has two parts ­ tectum or corpora quadrigemina, made up of
the superior & inferior colliculi and tegmentum containing the red nucleus and
periaqueductal grey matter.
Rhombencephalon which differentiates into Metencephalon that forms pons, the rostral
part of the medulla oblongata and cerebellum and Myelencephalon that forms the caudal
part of medulla oblongata.

The correct answer is: Caudal part of medulla oblongata – Myelencephalon, Thalamus –
Diencephalon, Corpus striatum – Telencephalon, Hypothalamus – Diencephalon, Red

690
nucleus – Mesencephalon

Question 34 HiY Neurosciences EMI084
Correct EEG findings 
You are working in the EEG clinic. You are requested to correlate some EEG findings with
Mark 5.00 out of
probable diagnoses associated with them. Identify the disorders for each of the following EEG
5.00
findings
Remove flag

Herpes simplex encephalitis
Infantile spasms
Episodic discharges Structural lesions
every 1­3 seconds with Depressive pseudodementia
variable focal waves. Angelman's syndrome
Neurosyphilis
CJD
Hepatic encephalopathy
Huntington's chorea
Herpes simplex encephalitis
Myoclonic epilepsy
Infantile spasms
Structural lesions
Generalised polyspike
Depressive pseudodementia
and wave pattern
Angelman's syndrome
Neurosyphilis
CJD
Hepatic encephalopathy
Huntington's chorea
Herpes simplex encephalitis
Myoclonic epilepsy
Infantile spasms
Structural lesions
Triphasic wave pattern Depressive pseudodementia
Angelman's syndrome
Neurosyphilis
CJD
Hepatic encephalopathy
Huntington's chorea
Herpes simplex encephalitis
Myoclonic epilepsy
Infantile spasms
Structural lesions
Depressive pseudodementia
Angelman's syndrome
Neurosyphilis
Focal slow wave
CJD
activity
Hepatic encephalopathy
Huntington's chorea
Myoclonic epilepsy

691
1 to 2 Hz periodic Herpes simplex encephalitis
generalised sharp
Infantile spasms
waves against a low
and slow background
Structural lesions
Depressive pseudodementia
Angelman's syndrome
Neurosyphilis
CJD
Hepatic encephalopathy
Huntington's chorea
Check
Myoclonic epilepsy

Explanation: 
CJD­ Generalized periodic 1­2 Hz sharp waves are seen in nearly 90% patients with sporadic
CJD. Less often in familial / hormonal transplant­related forms. It is not seen in the variant
form. 
Myoclonic epilepsy­ Generalized multiple spike­wave discharges. 
Diffuse generalized slowing is seen in metabolic and endocrine disorders. 
Triphasic waves: 1.5 to 3.0 per second high­voltage slow­waves especially in hepatic
encephalopathy. There is focal slowing or focal spike activity in structural lesions of the brain. 
In herpes simplex encephalitis, there are episodic discharges every 1­3 seconds with variable
focal waves over the temporal lobes.
The correct answer is: Episodic discharges every 1­3 seconds with variable focal waves. –
Herpes simplex encephalitis, Generalised polyspike and wave pattern – Myoclonic epilepsy,
Triphasic wave pattern – Hepatic encephalopathy, Focal slow wave activity – Structural
lesions, 1 to 2 Hz periodic generalised sharp waves against a low and slow background –
CJD

Question 35 HiY Neurosciences EMI085
Correct Synthetic enzymes of neurotransmitters
Identify the enzyme involved in the immediate synthesis of each of the following
Mark 5.00 out of
neurotransmitters
5.00

Remove flag
Acetylcholinesterase
5­hydroxyindole acetic acid
Homovanillic acid
Dopamine  
GABA transaminase
Phenylethanolamine­N­methyltransferase
Dopamine beta­hydroxylase
Choline acetyltransferase
Dopa decarboxylase
Acetylcholinesterase
Glutamic acid decarboxylase
5­hydroxyindole acetic acid
Tryptophan hydroxylase
Homovanillic acid
GABA transaminase  
Phenylethanolamine­N­methyltransferase
Norepinephrine
Dopamine beta­hydroxylase
Choline acetyltransferase
Dopa decarboxylase
Glutamic acid decarboxylase

692
Acetylcholinesterase
5­hydroxyindole acetic acid
Homovanillic acid
Serotonin  
GABA transaminase
Phenylethanolamine­N­methyltransferase
Dopamine beta­hydroxylase
Choline acetyltransferase
Dopa decarboxylase
Acetylcholinesterase
Glutamic acid decarboxylase
5­hydroxyindole acetic acid
Tryptophan hydroxylase
Homovanillic acid
Acetylcholine  
GABA transaminase
Phenylethanolamine­N­methyltransferase
Dopamine beta­hydroxylase
Choline acetyltransferase
Dopa decarboxylase
Acetylcholinesterase
Glutamic acid decarboxylase
5­hydroxyindole acetic acid
Tryptophan hydroxylase
Homovanillic acid
GABA  
GABA transaminase
Phenylethanolamine­N­methyltransferase
Dopamine beta­hydroxylase
Choline acetyltransferase
Dopa decarboxylase
Check
Glutamic acid decarboxylase
Tryptophan hydroxylase

Explanation: 

DOPA decarboxylase produces dopamine from dopa. 
Dopamine­b­hydroxylase modulates norepinephrine production from dopamine. 
Phenylethanolamine­N­methyltransferase modulates conversion of norepinephrine to
epinephrine.
The synthetic enzyme for acetylcholine is choline acetyltransferase (CAT) and not
acetylcholinesterase; the latter is the breakdown enzyme. 
Glutamic acid decarboxylase (GAD) produces GABA from glutamate.

The correct answer is: Dopamine – Dopa decarboxylase, Norepinephrine – Dopamine beta­
hydroxylase, Serotonin – Tryptophan hydroxylase, Acetylcholine – Choline acetyltransferase,
GABA – Glutamic acid decarboxylase

Finish review

693
001
In which of the following phases of the cell cycle does cell division occur?

Select one:
G1 phase
G0 phase
G2 phase
S phase
M phase

Each cell undergoes a natural cycle in terms of its replication and nucleic acid synthetic activity.
The cell cycle consists of four separate phases: G1 phase, S phase, G2 phase and M phase. G1
stands for growth phase 1, S for synthetic phase, G2 for growth phase 2 and M phase for
mitosis phase. Cells in the quiescent G0 phase of the cycle are stimulated by the growth factors
(e.g. EGF, epithelial growth factor; PDGF, platelet­derived growth factor; IGF, insulin­like growth
factor) and result in activation of transcription factors and lead to the initiation of DNA synthesis,
followed by mitosis and cell division. Thus from G0 the cell moves on to G1 when the
chromosomes are prepared for replication. This is followed by the synthetic
(S) phase, when the 46 chromosomes are duplicated into chromatids, followed by another
gap phase (G2), which eventually leads to mitosis (M).
The correct answer is: M phase

002
Molecular genetic findings in psychiatric disorders are consistent with which of the following
models?

Select one:

694
Single genes of large effect
None of the listed
Single genes of small effect
Multiple genes of large effect
Multiple genes of small effect

Multiple genes - small effect model proposes that diseases such as schizophrenia result from
the co-action of multiple (ranging in principle from a few to many thousand) common variants
(`polymorphisms'), each of which has a small effect on illness susceptibility. When an individual
inherits several, or many, susceptibility variants together, they have a sizable influence on
disease risk. Hence the mutations or polymorphisms are not sufficient by themselves to cause
disease, but they occur very commonly so they can interact in combinations and produce the
disease. This model is more popular currently and forms the basis of association and linkage
studies being carried out widely .
The correct answer is: Multiple genes of small effect

003
What percentage of the human genome is considered to be active with coding sequences?

Select one:
20
2
30

40

10

The proportion of coding and noncoding DNA within genomes varies widely among organisms.
More than 98% of the human genome does not encode protein sequences, including most
sequences within introns and most intergenic DNA.
The correct answer is: 2

004
If both parents suffer from an autosomal recessive disorder, the probability of each child being
affected is

695
Select one:
1:1
1:2
1:4

1:3

1:5

Autosomal recessive disorders manifest themselves only when an individual is homozygous for
the disease allele; i.e. both chromosomes carry the mutated gene. So if both parents suffer from
a disorder, the probability of each child being affected is 100% or 1:1. If carriers marry, the
offspring has a 1 in 4 chance of being homozygous and affected and a 1 in 2 chance of being a
carrier, and a 1 in 4 chance of being genetically normal. Consanguinity increases the risk.

The correct answer is: 1:1

005
A 24-year-old woman has a 32-year-old brother with schizophrenia. What is the risk
(expressed as percentage) of her developing schizophrenia accor DSM criteria?

Select one:
13
6
10
2

16

The lifetime risk of schizophrenia in the sibling of a patient is 9-10%.


The correct answer is: 10

006
Lucy is Lenny's half sister. The proportion of genes that might be shared between Lucy and
Lenny is

Select one:
50

696
33
25
20

12

First-degree relatives share one-half of their genes. Second-degree relatives share one-
fourth, and third-degree relatives share one-eighth.
The correct answer is: 25

007
What is the likelihood (in percentage) of a man acquiring Huntington's disease if his father has
the illness and carries the mutant chromosome?

Select one:
33
75
50
25

100

Huntington's disease is transmitted by an autosomal dominant inheritance pattern. If one


parent is an affected carrier, the likelihood of transmission to any given child is 50%. The
protein huntingtin is coded on the short arm of chromosome 4.
The correct answer is: 50

008
In a closed islandic population, frequency of genotype HH is 50, hh is 10 while Hh is 40. What is
the gene frequency of H?

Select one:
14
0.7
70
7

697
1.4

Out of 200 gene copies in 100 people, H occurs 50+50+40 = 140 times. Hence gene
frequency of H is 140/200 = 0.7
The correct answer is: 0.7

009
A newly identified psychiatric disorder called 'somatomania' is found to be inherited in a
simple Mendelian autosomal recessive fashion. If two patients with somatomania meet at a
psychiatric ward, marry and give birth to two children, what are the chances (in percentage)
that each child has inherited somatomania?

Select one:
33
25
100
75

50

In autosomal recessive disorders, children born to parents who are both diseased (not just
carriers but diseased and so has homozygous recessive alleles) have 100% risk of
inheritance.
The correct answer is: 100

010
The number of daughter cells at the end of a complete meiosis of one human cell is

Select one:
2
5
4
3

698
Meiosis is divided into two parts: meiosis I and meiosis II. At the end of the meiotic divisions,
there are four daughter cells whereas only two are produced from the mitotic process. Of the
two parts of meiosis, meiosis 1 is the reduction division. The main differences from mitosis are
the occurrence of synapsis (crossing over) in the prolonged prophase phase and non-
separation of sister chromatids during anaphase 1, leading to reduced (half) chromosomal
numbers in daughter cells. Meiosis 2 is same as a normal mitosis (no reduction in chromosomal
numbers).
The correct answer is: 4

011
The number of autosomes in a human somatic cell is

Select one:
2
22
44
23

46

Chromosomes are cellular structures containing one linear molecule of DNA. Human cells are
called diploid as they have 46 chromosomes, 23 inherited from each parent; thus there are 23
'homologous' pairs of chromosomes (22 pairs of 'autosomes' and two 'sex chromosomes').
The sex chromosomes, called X and Y, are not homologous but are different in size and
shape. Males have an X and a Y chromosome; females have two X chromosomes,
The correct answer is: 44

012
The total number of possible different codons that can occur in a human DNA is

Select one:
24

34
18
64

36

699
Some amino acids are encoded by more than one codon, inasmuch as there are 64 possible
base triplets and only 20 amino acids. In fact, 61 of the 64 possible triplets specify particular
amino acids and three triplets (called stop codons) designate the termination of translation.
Thus, for most amino acids, there is more than one code word .
The correct answer is: 64

013
When compared with the general population, the range of full-scale IQ in people with Turner's
syndrome is shifted down by

Select one:
30 points
20 points
0 points
40 points

10 points

Learning disability is rare in Turner's syndrome. But note that verbal-performance dissociation
can be seen.
The correct answer is: 0 points

014
An autosomal recessive disorder shows Hardy-Weinberg equilibrium. It has an incidence of 1 in
4900. The frequency of carriers of the disease is

Select one:
1 in 30
1 in 70

1 in 35
1 in 10

1 in 40

According to Hardy-Weinberg principle, the carrier frequency is twice the square root of the
frequency of a disease.
The correct answer is: 1 in 35

700
015
If both parents are carriers of an autosomal recessive illness, then the probability of
inheritance in children is

Select one:
1 in 4
1 in 2
100%
1 in 3

0%

If both parents carry ONE autosomal recessive gene each (both are carriers), there is a 1 in 4
chance of the child developing the disease. If both parents carry TWO autosomal recessive
genes each (both are diseased), there is a 100% chance of the child developing the disease.
The correct answer is: 1 in 4

016
Which of the following gives the value of one centiMorgan?

Select one:
100 base pairs of DNA
1000 basepairs of DNA
100% recombination frequency between two loci
1% recombination frequency between two loci

10 base pairs of DNA

Genetic distances are often expressed in centiMorgans (cM). One centiMorgan is equal to a
1% recombination frequency between two loci. One cM is approximately equal to 1 million
base pairs of DNA (1 Mb).
The correct answer is: 1% recombination frequency between two loci

017
The risk of unipolar depression in a first degree family member with bipolar disorder is given
by

701
Select one:
6-7 times higher
2-3 times higher
4-5 times higher

8-9 times higher


10 times higher

Lifetime risk rates of unipolar disorder in a member of the general population unrelated to
anyone with affective disorders is 5-10%. In relatives of a bipolar proband, the risk, of the
unipolar disorder is A. first-degree relative 10­20% (relative risk = 2­3times); B. monozygotic
co-twin 15-25% (relative risk = 3-5 times). Also, note that the relatives of bipolar patients have a
risk of 20-30% for 'any affective disorder' in their lifetime. Note that the risk to offspring with
both parents affected by bipolar disorder is 50-65% risk for bipolar and 50-75% risk for any
affective disorder.
The correct answer is: 2-3 times higher

018
Soon after gamete fusion and fertilization, nondisjunction of chromosomes takes place in an
early mitotic cell division in chromosome 21. Which of the following chromosomal makeup will
be seen in the foetus?

Select one:
Triploidy
21 monosomy
Euploidy
21 trisomy mosaic

21 trisomy

These aberrations result from the failure of chromosome or chromatids to separate ('non-
disjunction') in meiosis, with one gamete receiving two copies of that chromosome and one
another with no copies of the chromosome. This can produce (i) an extra chromosome, so
resulting in a fetus that is 'trisomic' and has three instead of two copies of the chromosome; or
(ii) no chromosome, so the fetus is 'monosomic' and has one instead of two copies of the
chromosome. Non-disjunction can occur with autosomes or sex chromosomes. However, only
individuals with trisomy 13, 18 and 21 survive to birth, and most children with trisomy 13 and
trisomy 18 die in early childhood.Occasionally, non-disjunction can occur during mitosis
immediately after two gametes have fused. This leads to the formation of two cell lineages with
different chromosomal makeup. Non-disjunction occurs more often with the sex chromosome,
and sometimes with chromosome 21, resulting in a 'mosaic' individual.

702
The correct answer is: 21 trisomy mosaic

019
A 17-year-old boy's mother and paternal aunt both suffer from alcoholism. Compared to the
general population, the risk of alcoholism in this boy is increased by a factor of

Select one:
8 times
3 times
2 times
16 times

4 times

Family studies show that the relative risk is increased by about 4-6 times for alcoholism if
there is a strong family history. The majority of adoption studies show that the risk of
alcoholism in adopted children is strongly correlated with their biological parents rather than
adoptive parents (3-4 times higher).
The correct answer is: 4 times

020
A distraught mother whose first child is diagnosed recently with autistic disorder wants to
know the relative risk of the same disorder to recur in her next child. The correct answer is

Select one:
3 times
50 times

10 times
4 times

100 times

Autism is thought to be strongly genetic.


The correct answer is: 50 times

021
Heritability of bipolar disorder is

703
Select one:
75-85%
55-65%
65-75%
85-100%

45-55%

The overall heritability of the bipolar disorder varies between 0.75 to 0.80.
The correct answer is: 75-85%

022
If a disease has 75% heritability, then which of the following statement is correct?

Select one:
75% of the diseased patients have inherited the condition
75% of the disease related genes are passed on from one generation to
another
The risk of the disease in first degree relatives of patients is 75%
75% of the features seen in a diseased patient is due to genetic component

75% of the liability for the disease is inherited

Heritability is defined by the proportion of variation of an observed feature that is due to


genetic factors.
The correct answer is: 75% of the liability for the disease is inherited

023
Which of the following disorders is inherited via X-linked dominant inheritance?

Select one:
Cerebellar vermis herniation with hydrocephalus
Brain tumours, epilepsy, learning disability and multiple renal cysts.
A 3 yr old girl with learning disability and stereotyped hand movements
around midline of her body.
A 2 yr old boy with repeated respiratory infections, puppet like face,

704
hypothyroid profile and poor cognitive development born to a mother of
advanced maternal age.
Pseudohypertrophy of calf muscles with normal IQ.

X-linked dominant disorders are rare. Because females have higher gene frequency for X
chromosomes compared to males, females have twice as much chance as males to inherit an
X-linked disease-causing mutation. Male to male transmission of the disease-causing mutation
is not seen. Vitamin D-resistant rickets is the best-known example. An atypical pervasive
developmental disorder called Rett's syndrome is inherited in X-linked dominant fashion. It is
characterised by a deceleration in head circumference, learning disability and stereotyped hand
movements around the midline of her body.
The correct answer is: A 3 yr old girl with learning disability and stereotyped hand movements
around midline of her body.

024
Which of the following correctly describes gene mapping?

Select one:
A strategy by which parental identity is established in case of disputes
A method by which genes are distributed during mitosis
A method by which genes are distributed in meiosis
A strategy that permits finding the chromosomal location of genes

A process by which ethnographic genetic distribution is studied

Gene mapping refers to any strategy that permits finding the chromosomal location of one or
more genes, often related to a disease. Genetic mapping of disease genes is a very useful
method because it does not require any knowledge of a gene's function to find the
chromosomal location initially. Once located then the identity of the disease gene could be
dissected. Not all genetic studies are aimed at gene mapping; certain simpler designs are
primarily aimed at demonstrating the presence or absence of a genetic influence in the
aetiology of a disease or trait. These include family studies, twin studies, and adoption studies.
Gene mapping studies involve linkage analysis, sib-pair analysis and to some extent allelic
association studies.
The correct answer is: A strategy that permits finding the chromosomal location of genes

025
A 77-year-old Vietnam War veteran presents with shooting pain in his back. On neurological
examination, you find light near dissociation of Argyll Robertson Pupil. Which of the following is
true?

705
Select one:
Consensual light reflex will be present
Both light and accommodation reflex will be present
Light reflex will be present

Accommodation reflex will be present


Afferent pupillary defect will be seen

ARP - Argyll Robertson Pupil (note: Accommodation Reflex Present -light reflex absent) is due
to an afferent defect in the pupillary reflex pathway - most probably a pretectal lesion.
The correct answer is: Accommodation reflex will be present

026
Which of the following criteria must be satisfied for a trait to be called an endophenotype?

Select one:
Must be independent of the clinical state
Must be associated with a candidate gene
All of the listed options
Must be co segregating with actual illness
Must have biological plausibility

To be termed as an endophenotype, Gottesman suggested certain criteria to be satisfied by an


identified disease marker. These are as follows: Must be associated with a candidate gene or
region Must be present with a high relative risk in relatives, thus co segregating with actual
illness Must be a parameter associated with disease with biological plausibility Must be
independent of the clinical state (i.e. must not be a state but a trait marker) Must be heritable
Must be present in relatives more often than general population
The correct answer is: All of the listed options

027
The process by which one gene gives rise to more than one protein is

Select one:
Inverse mutation
Transcription

706
Alternate splicing
Epistasis
Translocation

In human DNA, the genetic information that includes the code for making a protein is located in
fragments (exons, red boxes), which are interrupted by non-coding fragments (introns, green
boxes). By the process of alternative splicing, the introns are removed, and the exons spliced
together in different combinations, generating different messenger RNAs (mRNA) that are
decoded (translated) into distinct proteins. In humans, it is estimated that alternative splicing
occurs in more than 60% of genes, thus abolishing the one gene, one protein dogma long held
in medical genetics .
The correct answer is: Alternate splicing

028
In the human DNA, codons code for a specific

Select one:
Sugar
Protein
Phosphate
Nuclei acid
Amino acid

The genetic code is the relation between the sequence of bases in DNA (or its RNA
transcripts) and the sequence of amino acids in proteins. Three nucleotides (codon) encode
an amino acid. Proteins are built from a basic set of 20 amino acids, coded by several
combinations of the four bases (Retrieved from http://www.bioscience-
explained.org/ENvol4_1/pdf/spliceeng.pdf).
The correct answer is: Amino acid

029
Which of the following enzymes mediate the binding of tRNA with amino acids?

Select one:
RNA polymerase
Protein synthetase

707
Amino acid dehydrogenase
Aminoacyl synthetase
Reverse transcriptase

While syntheszing protein chains, the correct amino acid is added to the coding tRNA by a
specific enzyme called aminoacyl-tRNA synthetase. This synthetic process is called
aminoacylation.
The correct answer is: Aminoacyl synthetase

030
Choose the correct statement concerning Prader-Willi syndrome

Select one:
Associated with short stature
Occurs only in males

Social anxiety is a common feature


Affected children have a normal IQ
Average life expectancy of 6 to 8 years

Patients with PWS are often short statured and obese with learning disabilities.
The correct answer is: Associated with short stature

031
Which type of genetic study uses cases and controls or families with internal controls?

Select one:
Segregation studies
Family studies
Association studies
Linkage analysis

Twin studies

708
The simplest scenario of genetic association analysis is to study the genotype frequency of a
polymorphism in a group of independent patients (cases) and a group of normal individuals
(controls). They might be designed as simple case-control studies, or even better if they make
use of family-based internal controls.
The correct answer is: Association studies

032
A patient with schizophrenia is more likely to marry someone with a psychiatric problem or
substance use. This is termed as

Select one:
Consanguineous mating
Natural selection
Genetic drift
Pleiotropy
Assortative mating

Assortative mating is a well established phenomenon noted in psychiatric genetic


epidemiology
The correct answer is: Assortative mating

033
A mutation in the shank3 gene is associated with which of the following features?

Select one:
Parkinsonism
Autism

Bulimia
Alcohol dependence
Anorexia nervosa

SHANK3 or 'SH3 and multiple ankyrin repeat domains 3' is located at 22q13.3. 22q13.3
deletion syndrome (called Phelan-McDermid syndrome) reduces cell-to-cell communication
between neurons caused by a lack of SHANK3 protein contributing to the developmental delay,
intellectual disability, and absent or severely delayed speech. At least seven SHANK3

709
gene mutations have been found in people who have the features of autism or similar
conditions known as autism spectrum disorders .
The correct answer is: Autism

034
In which of the following patterns of inheritance does skipping one generation but
reappearing on the next is commonly seen?

Select one:
Translocations
Mitochondrial inheritance
Autosomal dominant
X linked dominant
Autosomal recessive

The correct answer is: Autosomal recessive

035
In which of the following types of inheritance, 25% of children inherit the disease even when
both parents are apparently healthy but share a similar genotype?

Select one:
X linked recessive
Mitochondrial

Autosomal dominant
Autosomal recessive
X linked dominant

710
Autosomal recessive disorders are only manifest in the homozygous state and normally only
affect individuals in one generation, usually in one sibship in a family. They affect both males
and females equally. Offspring of parents who are heterozygous for the same autosomal
recessive allele have a 1 in 4 chance of being homozygous for that allele. The less common an
autosomal recessive allele, the greater the likelihood that the parents of a homozygote are
consanguineous.
The correct answer is: Autosomal recessive

036
Which of the following phenomenon has the highest heritability?

Select one:
IQ
Neuroticism
Extraversion
Bipolar disorder
Novelty seeking

Bipolar disorder has higher heritability than other listed traits.


The correct answer is: Bipolar disorder

037
In Robertsonian translocation, the fusion usually occurs at the

Select one:
Cytoplasm
Centromere

Non sense mutations


Stop codons
Aneuploidy

A Robertsonian translocation is a type of nonreciprocal translocation involving two


homologous (paired chromosomes) or non-homologous chromosomes (i.e.,. two different
chromosomes, not belonging to a homologous pair). A feature of chromosomes that are
commonly found to undergo such translocations is that they possess an acrocentric
centromere (having a large arm with the majority of genes, and a short arm with sparse

711
number of genes). During a Robertsonian translocation, chromosomes break at their
centromeres and the two long arms fuse to form a metacentric chromosome. The short arms
also join to form a smaller chromosome, but this small piece gets lost over a few cell divisions.
The correct answer is: Centromere

038
In a patient with macroorchidism, large ears, prominent jaw, a high-pitched voice, and mental
retardation, which of the following trinucleotide repeats could be seen?

Select one:
CCT
CTG
CGG
CTT
CAG

The above description suits Fragile X syndrome. This X-linked condition accounts for more
cases of mental retardation in males than any other condition except Down syndrome with a
frequency of 1 in 4000. It can affect females but 50% less frequently than in males. A fragile
site near the tip of the long arm of the X chromosome was initially suspected. Now it is known
that fragile X results from an expansion of a trinucleotide repeat (CGG) near a gene called
FMR1.
The correct answer is: CGG

039
Which of the following genetic loci is associated with late onset senile dementia of Alzheimer's
type?

Select one:
Chromosome 1 Presenilin 2
Chromosome 17 tau protein

Chromosome 21 Beta amyloid


Chromosome 14 Presenilin 1
Chromosome 19 Apo E

Others are associated with presenile - early onset variety.

712
The correct answer is: Chromosome 19 Apo E

040
Which of the following chromosomal loci codes for Catechol-O-methyl transferase (COMT)
enzyme?

Select one:
chromosome 8q11
chromosome 4q11
chromosome 22q11
chromosome 11q22

chromosome Xq11

COMT gene resides in 22q11 locus.


The correct answer is: chromosome 22q11

041
Which of the following phenomena explain most of the epigenetic variations?

Select one:
Mutations and polymorphism
Aneuploidy
DNA methylation and histone modification
Polymorphism and microdeletions
Splicing errors and methylation

DNA methylation and histone modification explain most the of epigenetic variations discovered
to date. Crow has argued for long that epigenetic defects explain most of the concordance
seen in schizophrenia; according to Crow, the hemispheric laterality and language
specialisation unique to human brains is the source of schizophrenic defect and it can be
ascertained only through an epigenetic enquiry.

The correct answer is: DNA methylation and histone modification

042

713
Which of the following conditions is an example of aneuploidy?

Select one:
Thalassemia
Huntington's disease
Absence seizures

Phenylketonuria
Down's syndrome

The most common non-fatal aneuploidy is trisomy 21 or Down syndrome, affecting 1 in 800
births.
The correct answer is: Down's syndrome

043
The commonest cardiac malformation seen in Down's syndrome is

Select one:
Endocardial cushion defect
Pulmonary stenosis
Fallot's tetralogy
Atrial ostium secundum defect
Atrial stenosis

Congenital heart defects are common in Down (40­50%); they are frequently observed in
patients with Down syndrome who are hospitalized (62%) and are a common cause of death in
this aneuploidy in the first 2 years of life. The most common congenital heart defects are
endocardial cushion defect (43%), ventricular septal defect (32%), secundum atrial septal defect
(10%), tetralogy of Fallot (6%), and isolated patent ductus arteriosus (4%). PDA is seen in 16%
while pulmonary stenosis in 9%

The correct answer is: Endocardial cushion defect

044
Which of the following terms refers to the phenomenon by which one genetic alteration
influences another genetic alteration eventually mitigating the effects of each other?

714
Select one:
Imprinting
Transition
Crossing over

Epistasis
Synapsis

Gene-gene interaction between different alleles of different genes is called epistasis. This can
occur at the same step or different stages of the same biochemical pathway. At times, the effect
of one change may mitigate or modify the other.
The correct answer is: Epistasis

045
Which of the following is responsible for dementia seen in adults with Down's syndrome?

Select one:
Loss of genetic material corresponding to presenilin -1
Loss of genetic material in chromosome 21
Genetic material lost from chromosome 14
Loss of genetic material corresponding to Apoe4
Extra genetic material in chromosome 21

The correct answer is: Extra genetic material in chromosome 21

046
In an attempt to find out if trichotillomania has a genetic basis, family history is collected by
interviewing all possible first-degree relatives of 46 consecutive patients attending a hair
disorder clinic. What type of study is this?

Select one:
Family study
Gene mapping
Association study

715
Linkage analysis
Sib pair approach

There are two types of family studies. The family history method is simple but unreliable; here
psychiatric history is taken from the probands himself/herself. A comparison can be then made
as to how many relatives are affected in one group compared to another. A more thorough but
more time-consuming approach is the family study method. Here all available relatives are
directly interviewed.
The correct answer is: Family study

047
In which of the following phases of the cell cycle does a neuron remain for the longest period of
its life?

Select one:
G1 phase
G0 phase
G2 phase
S phase
M phase

While certain cells pause or freeze the cycle temporarily and stay in G0, e.g. liver cells,
neurons remain in G0 indefinitely. Collins, K et al. The cell cycle and cancer.
The correct answer is: G0 phase

048
Which of the following is NOT a step in polymerase chain reactions?

Select one:
Cycling
Annealing the DNA

Extending the primers


Gel electrophoresis
Denaturing the DNA

716
Minute amounts of DNA can be amplified over a million times using an in vitro technique called
polymerase chain reaction. The technique has three steps. Double-stranded genomic DNA is
denatured by heat into single-stranded DNA. The reaction is then cooled to favour DNA
annealing, and the primers bind to their target DNA. DNA polymerase is used to extend the
primers in opposite directions using the target DNA as a template. After one cycle, there are two
copies of double-stranded DNA, after two cycles there are four copies, and this number rises
exponentially with the number of cycles. The cycling is set to produce necessary number of
amplifications. Gel electrophoresis is not an inherent part of PCR.
The correct answer is: Gel electrophoresis

049 Heritability is a
measure of

Select one:
Concordance of disease between twins
Risk of disease in first degree relatives of probands
Genetic contribution of disease in an individual
Degree of penetrance of a disease
Genetic contribution to phenotypic variation in population

The relative influence of genetic factors in determining the variance in a trait is expressed as
heritability.
The correct answer is: Genetic contribution to phenotypic variation in population

050
Following synthesis, proteins inside the cell are stored temporarily in

Select one:
Ribosomes
Nucleus
Nucleolus
Golgi complex
Cilia

717
The Golgi complex is a dynamic system that gives off vesicles and vacuoles for further
processing and transport of synthesized protein molecules.
The correct answer is: Golgi complex

051
A 45-year-old man is diagnosed with cognitive impairment. The family history suggests that
individuals in the most recent generations of the family pedigree have developed a myriad of
psychiatric disturbances at an earlier age and with greater severity than previous generations.
Choose one correct diagnosis.

Select one:
Fragile X syndrome
Huntington syndrome
Motor neuron disease.
Velocardiofacial syndrome
Frontotemporal dementia

Huntington's disease is inherited in an autosomal dominant manner with full penetrance and a
prevalence rate of about 5 per 100,000. The gene responsible is an expanded and unstable
CAG trinucleotide repeat on the short arm of chromosome 4 - 4p16.3. This results in translation
of an extended glutamine sequence in huntingtin, the protein product of the gene. Huntingtin is
expressed throughout the body. Its function is unclear.Though slightly unusual for a genetic
disease; the onset is usually between 30 and 50 years of age. Most adult­onset HD cases have
CAG expansions of 40-55 repeats, while greater expansions (and more than 70 repeats) are
seen in childhood-onset HD. The phenomenon of anticipation, as described in the above
scenario, is well demonstrated in Huntington's.
The correct answer is: Huntington syndrome

052
A patient is referred with a diagnosis of schizophrenia. Chromosome analysis shows a 22q11
deletion confirming a diagnosis of Di George syndrome. Which of the following is NOT
compatible with this diagnosis?

Select one:
pharyngeal insufficiency
supra-valvular aortic stenosis
hypercalcaemia
fish mouth
severe learning difficulties

718
In velo-cardio-facial or DiGeorge syndrome, hypoparathyroidism leading to hypocalcemia
(observed in 60% of patients) usually begins in the neonatal period, occasionally manifesting in
the form of tetany or tonic convulsions.
The correct answer is: hypercalcaemia

053
The mechanism of meiosis fits with which of the following descriptions?

Select one:
Occurs in all tissues
Has a short prophase
Produces daughter cells identical to parent cells
Produces diploid somatic cells
Includes recombination

During the early stages of cell division in meiosis, two chromosomes of a homologous pair
may exchange segments - this is called recombination.
The correct answer is: Includes recombination

054
In premutant carriers of fragile X expansion, which of the following features may be seen?

Select one:
psychotic episodes
increased risk for intention tremor and ataxia

no clinical features
mental retardation

macroorchidism

Men who are not retarded but carry an increased number of CGG repeats in the FMR1 locus
are called premutation carriers. Though premutant carriers were long thought to be free from
clinical features, it is now known that they are at increased risk for developing intention tremor

719
and ataxia especially after middle age. Likewise, women who are premutation carriers (55-
200 CGG repeats) are at increased risk of premature ovarian failure and/or mild cognitive or
behavioral abnormalities.
The correct answer is: increased risk for intention tremor and ataxia

055
Which of the following terms describes deletion or insertion mutations that take place in
multiples of triplet codons?

Select one:
Inframe mutations
Substitutions
Frame shift mutations
Transitions
Inversions

According to the effect on triplet sequence, mutations could be frame shift or in-frame. In frame
shift mutations, the deletion or insertion is not in multiples of three codons e.g. a segment of 5
bases deletion mutations. This leads to a shift in triplet reading frame with variable results. In
frame, mutation refers to changes happening in multiples of 3 bases, with no disturbances in the
actual reading frame.
The correct answer is: Inframe mutations

056
Which of the following nuclear components undergo removal during splicing?

Select one:
Histones
Introns

Trinucleotide repeats
Exons
Transfer-RNA

An intron is any nucleotide sequence within a gene that is removed by RNA splicing to
generate the final mature RNA product of a gene
The correct answer is: Introns

720
057
Which of the following diagnosis is associated with a mutation in KISS1 gene?

Select one:
Wernicke syndrome
Cushing syndrome
Williams syndrome
Kallmann syndrome
Addison syndrome

Kallmann Syndrome (Hypogonadotropic hypogonadism along with anosmia, and occasionally


learning difficulties) is associated with Kiss1 gene mutations. Prevalence of Kallmann syndrome
is ~ 1 in 8000, M:F =5:1. X-linked recessive, autosomal dominant and autosomal recessive
patterns of inheritance are observed; however, many cases are sporadic with no Mendelian
inheritance pattern. A number of genes are implicated in KS, Kiss1 accounting for only a very
small portion of cases.
The correct answer is: Kallmann syndrome

058
The investigation of choice used in the diagnosis of Klinefelter's syndrome is

Select one:
Fetal topography
Fetal Ultrasound

Amniocentesis
Karyotyping
Chorionic villus sampling

A karyotyping is frequently used to confirm the diagnosis of Klinefelter's syndrome. In this


procedure, white blood cells from the infant are separated from the plasma, mixed with a
tissue culture medium, incubated, and ed for chromosomal abnormalities, such as the
presence of an extra X chromosome. Diagnosis can also be made prenatally via chorionic
villus sampling or amniocentesis tests in which fetal tissue is extracted, and the fetal DNA is
examined for genetic abnormalities.
The correct answer is: Karyotyping

721
059
For any particular trait, the pair of alleles of each parent separate and only one allele passes
on from each parent to an offspring. Which of the following Mendelian principles refer to the
above phenomenon?

Select one:
Law of common fate
Law of uniformity
Law of independent assortment

Law of segregation
Law of closure

The second Mendelian law is called the principle of segregation. It states that for any particular
trait, the pair of alleles of each parent separate and only one allele passes from each parent on
to an offspring. Which allele in a parent's pair of alleles is inherited is a matter of pure chance.
For example if there are two alleles both determining black colour in mother and two alleles with
one determining white colour and one determining black colour in the father, then these two
alleles segregate and only one of them could be passed on to the second generation. This was
later proved to be true by studying chromosomes during cell division.

The correct answer is: Law of segregation

060
Right homonymous hemianopia is caused by which of the following lesions?

Select one:
Optic chiasma lesion
Lesions of right optic radiation
Lesions of left optic radiation
Lesions of right optic nerve
Lesions of left optic nerve

Lesions of the left-sided optic tract, lateral geniculate body, optic radiations, and striate cortex
(any retro chiasmatic structure) can cause right homonymous hemianopia.
The correct answer is: Lesions of left optic radiation

722
061
When the same disease phenotype is caused by mutations in different loci, this is called

Select one:
Homogeneity
Epistasis
Allelic heterogeneity

Locus heterogeneity
Pleiotropy

Locus heterogeneity exists when the same disease phenotype can be caused by mutations in
different loci. Locus heterogeneity becomes especially important when genetic testing is
performed by testing for mutations at specific loci. For example, early-onset Alzheimer's could
be caused by mutations in chromosome 1, 14 or 21.
The correct answer is: Locus heterogeneity

062
A common method used to measure recombination fraction in a linkage study is

Select one:
Delta score
Log of odds score
Predicted recombination frequency
Genetic distance
Observed recombination frequency

LOD or logarithm of odds score is a measure of the statistical strength of association used in a
genetic linkage analysis. The value of the LOD score is the ratio between the probability of
obtaining a result (of association) if the two loci are linked to the probability of obtaining the
results if the two loci are not linked. Higher the score, more the likelihood of a true association.
The correct answer is: Log of odds score

063
Serotonin transporter promoter region shows short and long allele variants as a
polymorphism. Which of the following statements about polymorphisms, in general, is true?

723
Select one:
Generally occur at a rate above 6% in population
Maintained by evolutionary processes
Produce a functional defect in the subject

Cannot be used for linkage analysis

Can be seen only in exons

Polymorphism refers to variations in genetic make-up at a particular locus noted in general,


apparently healthy population. To be defined as polymorphism, the variant must occur in at least
1% of the total population and must be associated with normal (not abnormal i.e.
disease-causing) expression of the final phenotype. This excludes spontaneous mutations that
are random and so cannot simultaneously occur in such significant (1%) proportion of total
population. ABO blood groups are good examples of polymorphism expressed in protein
products of genes. It is assumed that evolutionary selection serves to maintain polymorphisms.

The correct answer is: Maintained by evolutionary processes

064
Which of the following is not true concerning Huntington's disease?

Select one:
Age of onset is inversely related to disease severity
Disease chromosome inherited from the father is unstable
Trinucleotide repeat of 34-45 is seen in normal individuals
Characteristic protein deposits form nuclear inclusions
Maternal inheritance results in more severe and earlier onset disease

Spermatogenesis induces more instability in HD


The correct answer is: Maternal inheritance results in more severe and earlier onset disease

065
The origin of single X chromosome in Turner's syndrome is most likely to be of

Select one:
Dizygotic twin origin

724
None of the listed
Paternal origin
Chromosome escaping random inactivation

Maternal origin

As nearly 80% of meiotic error in Turner's is paternal, the remaining single X chromosome is
usually of maternal origin.
The correct answer is: Maternal origin

066
Equatorial alignment of chromosomes is seen in which of the following phases of cell
division?

Select one:
Interphase

Anaphase
Telophase
Prophase
Metaphase

In metaphase, the chromosomes are aligned at the metaphase plate (a plane that is equally
distant from the two spindle poles) - equatorial alignment.
The correct answer is: Metaphase

067
Which of the following aminoacids constitute the initiation sequence of protein translation?

Select one:
Tyrosine
Isoleucine
Threonine
Valine

Methionine

725
Translation includes three steps - initiation, elongation and termination. The ribosome contains
two sites: Peptidyl P site where methionine containing tRNA initially binds and aminoacyl A site
where each new incoming tRNAs with activated amino acids can bind. In elongation step, amino
acids are added one by one in a sting like fashion to produce proteins. Chain termination is
signaled by one of the three codons UAA, UGA or UAG.
The correct answer is: Methionine

068
Which one of the following processes can inactivate a gene?

Select one:
Unwinding of DNA strands
Uncoiling of chromosome
Methylation
Condensation
Crossing over

DNA methylation is a source of epigenetic modifications.


The correct answer is: Methylation

069
Which of the following best describes the commonest type of mutations seen in thalassemia?

Select one:
Nonsense
Inframe

Missense
Frame shift
Silent

726
Beta thalassemia is the commonest single gene disorder in the world. It is generally considered
as an autosomal recessive disorder as the inheritance of two mutant ß-globin genes is
necessary to produce a notable clinical illness. It is characterized by mostly missense or
somewhat less frequently nonsense mutations in the [beta]-globin gene (HBB), which reduce or
completely abolish gene expression. Heterozygous inheritance of a [beta]- thalassemia
mutation results in a trait condition, sometimes termed [beta]-thalassemia minor. Homozygous
inheritance results in [beta]-thalassemia major (absent Hb A production) or [beta]-thalassemia
intermedia (severely reduced Hb A production), which are associated with moderate to severe
hemolytic anemia.
The correct answer is: Missense

070
Which of the following inheritance patterns includes maternal transmission but no paternal
transmission and affects both sexes?

Select one:
Mitochondrial
Autosomal dominant
X liked dominant
Genomic imprinting
X linked recessive

Mitochondrial DNA is wholly inherited from the ovum. The sperm has no mitochondria in its
head; the head is made of nuclear material and acrosomal cap. The body of sperm has many
mitochondria that provide energy in propelling the tail. The body and tail are shed on the entry of
sperm into the ovum. Hence, the mitochondria of an embryo are completely maternal- derived.

The correct answer is: Mitochondrial

071
Which of the following genes is NOT implicated in schizophrenia?

Select one:
D aminoacid oxidase
Dysbindin

Monoamine oxidase
Neuregulin
COMT

727
MAO gene is not implicated; it is implicated in neuroticism and in anxiety disorders.
The correct answer is: Monoamine oxidase

072
Which of the following statements about zygosity in twins is true?

Select one:
Dizygotes share all except few genes
Monozygotes are formed from two sperms fertilizing one ovum
Dizygotes and monozygotes have same amount of environmental sharing
Monozygotes share more uterine environment than dizygotes

Anatomical similiarity is sufficient to establish zygosity

Monozygotic (MZ, or "identical") twins are formed when an embryo is cleaved during early
development. The result is two genetically identical embryos. Dizygotic (DZ, or "fraternal") twins
are the result of the fertilization of two different ova by two different sperm cells. DZ twins are
genetically the same as non-twin siblings, sharing only 50% of their genes. Approximately 70%
of monozygotic twins are monochorionic while dizygotes almost always have separate
chorions/amnions. Hence, monozygotes share more uterine environment than dizygotes.
The correct answer is: Monozygotes share more uterine environment than dizygotes

073
Which of the following values of the log of odds [LOD score] is commonly used as a threshold
level of significance in linkage studies?

Select one:
More than 2
More than 0.05

More than 3
Less than 0.05
More than 1

728
To estimate the likelihood that two loci are truly linked with a specific recombination frequency,
an LOD score is used. An LOD score greater than 3 is usually interpreted as statistical evidence
of linkage (i.e., the numerator is 1,000 times greater than the denominator, indicating that
linkage is 1,000 times more likely than nonlinkage). Conventionally an LOD score of -2 or less is
taken as evidence that two loci are not linked (i.e., nonlinkage is 100 times more likely than
linkage).
The correct answer is: More than 3

074
Which of the following genetic variants of Down's syndrome shows less cognitive impairment
and milder features than other variants?

Select one:
Mosaicism
Full trisomy of chromosome 21
Duplication of a portion of chromosome 21
Both translocation and mosaicism
Robertsonian translocation

Mosaics have a milder spectrum of the malformations as not all cells bear the chromosomal
aberration.
The correct answer is: Mosaicism

075
Which of the following is true concerning a woman who has an autosomal recessive disorder
but her husband is genotypically normal?

Select one:

Only her daughters will receive the mutant gene


All her children would have the disease phenotype

None of her children will have the disease phenotype


All her daughters would be carriers, all her sons would have the disease

All her sons would be carriers, all her daughters would have the disease

As all of the children will be carriers, none will have the expressed phenotype in normal
circumstances

729
The correct answer is: None of her children will have the disease phenotype

076
In which of the following mutations does the mutant codon specify a stop signal?

Select one:
Missense mutation
Silent mutation
Nonsense mutation
Inframe mutation
Frameshift mutation

A silent mutation causes no change in protein product - this is possible because a single
amino acid is often coded by more than one triplet sequence. In a silent mutation, one triplet
sequence is replaced by a different sequence but without changing amino acid product. In
missense mutation, the new mutant codon specifies a different amino acid with variable
effects on final protein product. For example, haemophilia, sickle cell anaemia. In non sense
mutation, the new codon is UUA UGA or UAG, which signals 'stop' to the amino acid
sequence resulting in a nonfunctional protein.
The correct answer is: Nonsense mutation

077
Which of the following is an autosomal dominant disorder?

Select one:
Fredreich's ataxia
Phenylketonuria

Noonan's syndrome
Hurler's syndrome
Tay-sach's disease

Noonan syndrome is caused by a genetic mutation and is acquired in an autosomal dominant


fashion mostly or as a spontaneous mutation rarely. Noonan's syndrome is characterised by
short stature, congenital heart defect and delayed mental development of varying degree.
Patients with Noonan syndrome also have a characteristic appearance: short neck, cervical skin
fold, low set ears, hypertelorism. Additionally lymphatic dysplasia leading to cystic hygroma and
occipital fold enlargement may be seen in the fetus. The incidence is around

730
1:1000-2500. Mutations in the HEXA gene coding the lysosomal enzyme beta- hexosaminidase
produces TaySach's disease that is common in people of Ashkenazi (eastern and central
European) Jewish heritage. It is inherited in an autosomal recessive pattern. Friedreich's ataxia
is an autosomal recessive disease involving the defective FXN gene, which codes for
theproduction of a protein called "frataxin." In the normal version of the gene, a sequence of
DNA (labeled "GAA") is repeated between 7 and 22 times. A higher number of this triplet repeat
expansions produce the disease
Hurler syndrome is an autosomal recessive disease, with genetic loci at 4p16. The disease is
usually caused by multiple missense and frameshift mutations which produce Alpha-L-iduronate
deficiency. PKU is an autosomal recessive disorder. It is the most common genetic disease
involving amino acid metabolism.
The correct answer is: Noonan's syndrome

078
CADASIL is associated with a mutation in which of the following genes?

Select one:
Dysbindin
LRRK2
Parkin gene
Notch3
COMT

An unusual form of subcortical small vessel disease occurs in the rare inherited condition of
CADASIL (cerebral autosomal dominant arteriopathy with subcortical infarcts and
leukoencephalopathy) in which a mutation in the Notch3 gene results in deposition of granular
material in the walls of small arteries and arterioles, narrowing of vessel lumens and destruction
of smooth muscle cells in their walls.
The correct answer is: Notch3

079
Which of the following methods can be used to detect the presence of abnormal trinucleotide
expansions?

Select one:
Northern blotting
Karyotyping

Southern blotting
PCR

731
FISH

The correct answer is: PCR

080
Nucleosides consist of which of the following components?

Select one:
Phosphorylated nucleotide
Phosphate and nitrogenous base
Pentose sugar, phosphate and nitrogenous base
Pentose sugar and phosphate
Pentose sugar and nitrogenous base

The correct answer is: Pentose sugar and nitrogenous base

081

732
A man suffers from an autosomal recessive disorder and has mild learning difficulties and
special dietary requirements. Which of the following enzymes is affected as a result of the
mutated gene?

Select one:
Phenylalanine hydroxylase
Phenylalanine decarboxylase
Alcohol dehydrogenase

Hypoxanthine guanine phosphoribosyl hydroxylase


Hypoxanthine guanine phosphoribosyl transferase

The diagnosis is phenylketonuria, which is an inborn metabolic disorder with autosomal


recessive inheritance. It is a complex metabolic condition caused by mutations of the
phenylalanine hydroxylase gene.
The correct answer is: Phenylalanine hydroxylase

082
Variation of a genetic code across the population without any associated overt phenotypic
change is best described as

Select one:
Mutation
Epigenesis
Polymorphism
Gene silencing
Point deletion

Polymorphisms are natural variations in a gene, DNA sequence, or chromosome that have no
adverse effects on the individual and occur with fairly high frequency in the general population.
Gene silencing refers to the epigenetic processes of gene regulation whereby the "switching off"
of a gene takes place by a mechanism other than genetic modification. This occurs when RNA
is not translated to a protein downstream
The correct answer is: Polymorphism

083

733
Deletion in the paternally derived chromosome 15q11-q13 leads to which of the following
disorders?

Select one:
Tuberous sclerosis
Huntington's disease
Di George syndrome
Angelman's syndrome
Prader Willi syndrome

Though no structural differences exist between maternal and paternally inherited chromosomes
in humans, there are some subtle functional differences that are increasingly being appreciated.
For example, a deletion of part of the long arm of chromosome 15 (15q11- q13) will give rise to
the Prader-Willi syndrome (PWS) if it is paternally inherited. A deletion of a similar region of the
chromosome gives rise to Angelman's syndrome (AS) if it is maternally inherited. This may be
due to differential regional expression of the chromososmes.
The correct answer is: Prader Willi syndrome

084
Which of the following phenomena takes place during interphase?

Select one:
Preparation for cell division
Cell division
Quiescence
Cell union
Nuclear division

Cell division is a process by which cells reproduce " here, a sequence of steps enable the
replicated genetic material in a parent cell to be equally distributed to two daughter cells.
Before a dividing cell enters mitosis, it undergoes a period of growth called interphase.
Interphase is the 'holding' stage between two successive cell divisions. In this stage, the cell
replicates its genetic material and organelles in preparation for division. It is the longest
phase, and all steps in cell cycle except stage (M) together constitute interphase.
The correct answer is: Preparation for cell division

085

734
You are seeing a 55-year-old man in the outpatients' clinic. He has a history of gradual
progressive memory loss, and his MMSE is 20 out of 30. The gene implicated for his condition
codes for

Select one:
Glutamate
Presenilin-1
Neuronal Sortilin related receptors (SORL1)

Dysbindin
Apolipoprotein -E4

The presenilin-1 gene located on chromosome 14 and mutations in this gene can cause early
onset Alzheimer's disease. SORL1 directs trafficking of APP into recycling pathways; when
SORL1 is under-expressed, APP is sorted into Aß- amyloid generating compartments. SORL1 is
associated with late onset dementia, not the presenile type.
The correct answer is: Presenilin-1

086
Which of the following condition results from a genetic abnormality in the locus coding
progranulin?

Select one:
Primary progressive aphasia
Alzheimer's disease
Tay Sach's disease
Prion disease
Parkinson's disease

Progranulin (PGRN) mutations cause frontotemporal lobar degeneration (FTLD). Encoded by a


single gene on chromosome 17q21, the mutations cause a clinical phenotype that is similar to
those with sporadic frontotemporal dementia - primary progressive aphasic type.
The correct answer is: Primary progressive aphasia

087
Which of the following is true with regard to the genetic risk of schizophrenia when moving
from a closer relative to a distant relative?

735
Select one:
Risk initially increases, then falls
Risk increases
Risk depends on the sex of the relative

Risk remains the same


Risk decreases

For both schizophrenia and bipolar disorder there is a very rapid, non-linear decrease of risk
when moving from a genetically identical individual (i.e. monozygotic co-twin where the risk is
50-60%), to an individual who shares half the genes (e.g. sibling, parent, dizygotic co-twin
where risk is around 10%).
The correct answer is: Risk decreases

088
Which of the following type of Down's syndrome could be inherited with equivalent likelihood
from either of the parents?

Select one:
Monosomy 21
Unbalanced translocation
Trisomy 21
Mosaicism
Robertsonian translocation

People with translocation Down syndrome can inherit the condition from an unaffected parent,
either the mother or the father.
The correct answer is: Robertsonian translocation

089
A 4-year-old child presents with developmental delay, growth retardation, generalised dystonia
and an IQ of 60. On investigation, hyperuricaemia is noted. Which of the following is likely to be
seen?

Select one:
Insensitivity to pain

736
Progressive dementia
Self injurious behaviour
Liver failure

Hair loss

Repeated self-injurious behaviour is a common feature of Lesch-Nyhan syndrome in which


hyperuricemia is also noted.
The correct answer is: Self injurious behaviour

090
The most common feature of Prader-Willi syndrome is

Select one:
Epilepsy

Repeated hand wringing


Large testicles
Short stature
Simian palmar crease

The features of Prader-Willi syndrome includes obesity, small hands and feet, microorchidism,
cryptorchidism (hypogonadism), short stature, almond-shaped eyes, flat face, prominent
forehead and bitemporal narrowing and orthopaedic problems such as congenital dislocation of
hip, scoliosis etc are seen as the child develops.
The correct answer is: Short stature

091
Which of the following can be used to study the abnormalities in the candidate genes
associated with schizophrenia in patients with psychosis?

Select one:
Chromosomal count
Amount of methylation
Single nucleotide polymorphisms
Heterochromatin material

737
Translocation

Genome-wide association studies are based on linkage disequilibrium (LD), using several
hundred thousand marker single nucleotide polymorphisms or SNPs.
The correct answer is: Single nucleotide polymorphisms

092
The gene polymorphism involved in serotonin transporter variations is

Select one:
5q31
SL6CA4
XLMR
Xq 26-27

HGPRT

The correct answer is: SL6CA4

093
Which of the following blotting techniques can be used to visualize restriction fragment length
polymorphisms?

Select one:
Southern blot

Northern blot
Dot blot

738
Eastern blot
Western blot

Restriction fragment length polymorphisms are those variations which change the sites at
which restriction enzymes can act on a DNA molecule, rendering differences in the final
'restricted' or cleaved DNA when these enzymes are applied in vitro. These fragments can be
visualized on a Southern blot
The correct answer is: Southern blot

094
The best technique to detect a large sequence of DNA is

Select one:
Electron microscopy
PCR
Southern blotting
Linkage study
Radiolabelling technique

Southern blotting was named after Edward M. Southern who developed this procedure at
Edinburgh University in the 1970s. Southern blotting is designed to locate a particular
sequence of DNA within a complex mixture. Fluorescent in situ hybridization (FISH) is a
technique that exploits the principle of nucleic acid hybridization, in which single-stranded
nucleic acids (usually DNA, sometimes RNA) interact with sufficiently similar, complementary
sequences, to form hybrid complexes. This process helps to identify an unknown sequence
and locate it on a given chromosome.
The correct answer is: Southern blotting

095
Which of the following describes the process by which introns are removed from freshly made
RNA sequences?

Select one:
Post translational modification
Exocytosis
Cytokinesis

739
Splicing
Synapsis

Transcripted RNA initially contains the 'junk' sequences - introns that do not code for
polypeptides. This unprepared RNA is called heterogeneous nuclear RNA or hnRNA. This
hnRNA then undergoes splicing aided by nucleosomes in the nucleus to remove non-coding
sequences and results in messenger RNAs (mRNA).
The correct answer is: Splicing

096
The function of the enzyme RNA polymerase is best described as

Select one:
Synthesis of multiple copies of damaged RNA
Synthesis of DNA from RNA template
Synthesis of DNA from DNA template
Synthesis of RNA from DNA template
Synthesis of RNA from RNA template

RNA Polymerase is an enzyme that catalyzes the synthesis of RNA from ribonucleoside
triphosphates in the presence of a DNA template.
The correct answer is: Synthesis of RNA from DNA template

097
Which of the following pairing among nucleotides does not occur in human DNA sequences?

Select one:
AT pair
GC pair
TG pair
CG pair
TA pair

740
The correct answer is: TG pair

098
Which of the following is least likely regarding diseases that show the phenomenon of
anticipation?

Select one:
Dominant mode of transmission is noted
The age of onset and severity of illness reduces with successive
generations
Trinucleotide expansion is seen
Infants are normal at birth
Most of them are neuro-degenerative disorders

With successive generations, if the onset is earlier and the outcome becomes more severe in a
genetic disease, the term anticipation is employed to describe the inheritance.
The correct answer is: The age of onset and severity of illness reduces with successive
generations

099
In a family, father suffers from an autosomal dominant disease. Mother is a homozygous
normal individual. Which of the following correctly describes the risk of this couple having a
child with the autosomal dominant disorder?

Select one:
It depends on the age of the father at time of birth
It depends on the birthorder

It depends on the sex of the child


The risk is 50% irrespective of sex
The risk increases with incomplete penetrance

741
The correct answer is: The risk is 50% irrespective of sex

100
Which of the following best describes the term genotype?

Select one:
The genetic makeup of a family with a specific disorder
An unobservable marker of familial risk for a condition
The detectable characteristics of an individual organism
A numerical representation of all of the potential combinations of parental
traits.
The sum total of all the genes that is inherited by an organism.

The term genotype describes the actual set of genes carried by an organism. In 1909, the
clarifying distinction between the concept of "genotype" and the "phenotype" was provided by the
Danish botanist Wilhelm Johanssen. Gottesman and Shields described "endophenotypes" as
internal phenotypes discoverable by a "biochemical test or microscopic examination."
The correct answer is: The sum total of all the genes that is inherited by an organism.

101
Mrs. Brown gives birth to twins in her first delivery. The twins look alike with respect to their hair
colour but look different with respect to the colour of iris. Which of the following statement is
most likely with the available information?

Select one:
They are dizygotic twins
They are monozygotic twins

They are concordant for iris colour


They are identical twins

742
They are
concordant
for hair
colour

743
Two individuals are concordant if they share the same trait; if they do not share the trait, they are
discordant. For a trait that is completely genetic, one would expect a concordance of 100% for
MZ twins because they share all of their genes. The concordance rates for DZ twins should be
only about 50%. In contrast, a trait that has no genetic basis should have equivalent
concordance rates for MZ and DZ twins.
The correct answer is: They are concordant for hair colour

102
What is the risk of Alzheimer dementia in a patient heterozygous for Apo E4 compared to
those with no apoE4 allele?

Select one:
Three times higher (OR=3)
Nine times higher (OR=9)
Same risk (OR=1)
Twenty times higher (OR=20)
Fifteen times higher (OR=15)

The correct answer is: Three times higher (OR=3)

103
Which of the following refers to the production of new copies of RNA from DNA?

Select one:
Translation
Synapsis

Replication

744
Transcription
Modification

Using DNA template to form more DNA is replication; Using DNA template to form more RNA
is transcription; Using RNA template to form protein is translation.
The correct answer is: Transcription

104
An aborted foetus was found to have 69 chromosomes in each cell. Which of the following
terms best describes the abnormality?

Select one:
Diploid
Euploid
Haploid
Monoploid
Triploid

When cells possess chromosomal numbers different from normal diploidy, they are called
aneuploid cells. Aneuploidy can occur in single numbers e.g. trisomy 21, trisomy 18, monosomy
of Turner's, etc. Very rarely, the entire chromosome set will be present in more than two copies,
so the individual may be triploid rather than diploid and have a chromosome number of 69.
Triploidy and tetraploidy (four sets) result in spontaneous abortion.
The correct answer is: Triploid

105
Which of the following syndromes is characterised by severe mental retardation, rocker
bottom feet, low-set ears, micrognathia and clenched hands?

Select one:
Trisomy 18
Velo cardio facial syndrome

Down's syndrome
Patau syndrome
William's syndrome

745
Edward's syndrome is characterised by severe mental retardation and rocker bottom feet, low-
set ears, micrognathia (small jaw), congenital heart disease, clenched hands, and a prominent
occiput. It is a result of trisomy 18. It occurs at a frequency of 1:8000 and often death occurs
within1 year of birth.
The correct answer is: Trisomy 18

106
In which of the following conditions chromosomal duplication is seen?

Select one:
Cri-du-chat Syndrome
Foetal Alcohol Syndrome
Turner's Syndrome
Reye's Syndrome
Trisomy 21

Reye's and FAS are not chromosomal disorders. Cri-du-chat is a deletion syndrome while
Turner's is a monosomy with non-disjunction (no duplication).
The correct answer is: Trisomy 21

107
A young girl with learning difficulties is diagnosed to have trisomy X. Her chromosomes were
karyotyped to diagnose the genetic defect. How many Barr bodies would she have in her
somatic cells?

Select one:
Information insufficient to answer the question
One

Two
None
Three

746
X inactivation occurs very early in the development of female embryos. When an X
chromosome is inactivated, it could be visualized under the microscope as a highly
condensed Barr body in the nuclei of interphase cells. All X chromosomes in a cell are
inactivated except one, irrespective of original number of X chromosomes in a cell. Thus
females with trisomy X will have two Barr bodies. X inactivation occurs via DNA methylation.
The correct answer is: Two

108
In a family with the X-linked recessive transmission of a genetic disease, there are two
daughters and one son born to a diseased father and normal non-carrier mother. One of the
two daughters exhibits a moderate degree of the disorder while the other is a silent carrier.
Which of the following can explain the above?

Select one:
The daughter is biologically male
Combined X linked dominant and recessive transmission
Diseased daughter is the unaffected carrier
The disease cannot be X linked recessive

Unfavourable lyonisation of X chromosomes

If a recessive disease-causing mutation occurs on the single X chromosome of a man, this is


sufficient to cause disease as another X chromosome is not available to compensate any
deficiencies. As females have two copies of the X chromosome, they need a double identical
mutation for disease expression which is extremely rare. But during random X inactivation if
most X chromosomes carrying normal alleles are inactivated (called unfavourable Lyonisation),
then these females can manifest the disease phenotype " termed as manifesting heterozygotes.
But nevertheless the severity of expressed disease is mild and can go unnoticed too.

The correct answer is: Unfavourable lyonisation of X chromosomes

109
Which of the following steps is NOT a part of polymerase chain reaction (PCR)?

Select one:
Extending primers in opposite directions
Vector based replication of DNA
Denaturing DNA by heat
Annealing DNA by cooling
Amplifying copies of DNA

747
The PCR technique has three steps. A. Double-stranded genomic DNA is denatured by heat
into single-stranded DNA. The reaction is then cooled to favour DNA annealing, and the primers
bind to their target DNA. B. DNA polymerase is used to extend the primers in opposite directions
using the target DNA as a template. After one cycle, there are two copies of double- stranded
DNA, after two cycles there are four copies, and this number rises exponentially with the number
of cycles. C. The cycling is set to produce necessary number of amplifications. Note that the
vector-based replication is used for DNA cloning, not PCR.
The correct answer is: Vector based replication of DNA

110
Choose the correctly paired techniques and genetic tools.

Select one:
Plasmid - Flourescent in situ hybridisation
Flourescent probes - Polymerase chain reaction
Heat denaturation - DNA cloning
Oligonucleotide primers - Western blotting
Vectors - DNA cloning

A cloning vector is a small piece of DNA into which a foreign DNA fragment can be inserted
The correct answer is: Vectors - DNA cloning

111
Which of the following is a clinical feature seen in Turner's syndrome (45X0)?

Select one:
Webbed neck
Narrow chest

Prognathism
Receding hairline
Tall stature

748
Webbed neck with prominent low hairline, retrognathism, short stature and broad chest are
features of Turner's.
The correct answer is: Webbed neck

112
Neuregulin-1 was identified as a candidate gene for schizophrenia after typing 950
microsatellite markers in 110 Icelandic patients. Which of the following approach was used to
complete the above?

Select one:
Family studies
Sib pair analysis
Whole genome scan
Association studies
Twin studies

Stefansson et al. found that neuregulin-1 is a candidate gene for schizophrenia after typing 950
microsatellite markers covering the whole genome in 110 Icelandic patients for whom they had
reconstructed their genealogical relationships. This was done using whole genome scan, which
is a type of linkage analysis, where markers placed at regular intervals covering the whole
genome, are typed. It is tedious but often the first approach when no genetic information is
available about a particular phenotype
The correct answer is: Whole genome scan

113
Mr X is a tall man with low IQ and behaviour problems. Identify the type of sex chromosome
disorder?

Select one:
Downs syndrome
XYY male

Klinefelter's syndrome
Turner's syndrome
Trisomy X

749
XYY males have a karyotype of 47, XYY. This condition is seen in 1:1000 male births.
Behavioural problems are commonly seen. IQ may be slightly lower than average. Studies on
the population of men in maximum-security psychiatric hospitals in the 1960s seemed to show
an excess of men with this karyotype as well as men with Klinefelter's syndrome.
The correct answer is: XYY male

750
001
Which of the following has at least a preliminary evidence base for managing Treatment-
Resistant Depression?

Select one:
Fluoxetine and clonazepam
Fluoxetine and mianserin
Citalopram and valproate

Venlafaxine and Mirtazapine


Duloxetine and fluoxetine

Venlafaxine-mirtazapine combination is called Californian rocket fuel, which is recommended by


NICE in the treatments of refractory depression. It is usually well tolerated, has reasonable
literature support and is becoming more widely used.
The correct answer is: Venlafaxine and Mirtazapine

002
Buspirone is a

Select one:
GABA agonist
5-HT1A partial agonist
5-HT1A agonist

5-HT1A antagonist

751
GABA antagonist

It is a partial agonist
The correct answer is: 5-HT1A partial agonist

003
Which of the following is linked with intentional nonadherence?

Select one:
Missed doses after long period on treatment
High degree of trust on the treating doctor
A desire for self-efficacy
Receipt of adequate levels of drug information

Higher severity of illness

Patients with a high desire for self-efficacy are found to have higher degree of intentional
nonadherence.
The correct answer is: A desire for self-efficacy

004
A placebo is best described as which one of the following?

Select one:
A preparation of active compound without a label

An active preparation of precursor molecules


A preparation with higher than average propensity to cause side effects
A compound that is active for an unrelated disorder

A dummy preparation of inactive compound

Medication possesses a potential for therapeutic efficacy far beyond its chemical capacity.
The placebo effect is a psychobiological phenomenon that can be attributable to different
mechanisms, including an expectation of clinical improvement and Pavlovian conditioning.

752
The Journal of Neuroscience, 9 November 2005, 25(45): 10390­10402;
The correct answer is: A dummy preparation of inactive compound

005
Ethnic differences in psychiatric drug effects are noted in which of the following
pharmacological actions?

Select one:
Blood levels of haloperidol
Hydroxylation of tricyclics
All of the listed
Prolactin response to antipsychotics
Alcohol metabolism

Maximal haloperidol concentration in plasma after rapid tranquillisation is significantly high for
Asians than Caucasians. Nearly 40% Asians and around 60% South American Native Indians
lack Aldehyde dehydrogenase enzyme in sufficient amounts to metabolise alcohol.
Caucasians appear to have lower plasma levels of tricyclic antidepressants and attain plasma
peaks later when compared with Asians (of Far Eastern ancestry as well as those from the
Indian subcontinent). These differences have been attributed to a greater incidence of slow
hydroxylation among Asians when compared with Caucasians. . On administration of
antipsychotics, Asian subjects were reported to produce greater serum prolactin levels than
Caucasian subjects.
The correct answer is: All of the listed

006
Which of the following best describes lofexidine?

Select one:
Natural opioid
Alpha 1 antagonist

Alpha 2 agonist
Synthetic opioid
Opioid antagonist

753
In opioid detoxification, if short duration of treatment is desirable, a2 adrenergic agonists such
as lofexidine and clonidine are preferred to methadone.
The correct answer is: Alpha 2 agonist

007
Which of the following chemical class of drugs used as antidepressants can increase seizure
risk heavily?

Select one:
Triazolopyridine
Hydrazine derivatives
SSRIs
Cyclopyrrolone
Aminoketone

Aminoketone ( Bupropion) is contraindicated in seizure disorder.


The correct answer is: Aminoketone

008
Which of the following is a substituted benzamide used as an antipsychotic?

Select one:
Amisulpride
Benzhedrine

Paliperidone
Risperidone
Olanzapine

Amisulpride is a substituted benzamide that has atypical antipsychotic activity.


The correct answer is: Amisulpride

009
Choose a secondary amine from the below list:

754
Select one:
Clomipramine
Dothiepin
Amitriptyline

Amoxapine
Imipramine

Secondary amines = desipramine, amoxapine, nortriptyline and protriptyline also duloxetine)


[more potent mg to mg basis; less sedating; more noradrenergic, less antihistaminic or
anticholinergic than tertiary].
The correct answer is: Amoxapine

010
Which of the following best applies to the chemical structure of buspirone?

Select one:
Imidazopyridine
Benzisoxazole
Thienobenzodiazepine
Aminoketone
Azaspirone-decanedione

Buspirone, anxiolytics, is an azaspirone. Bupropion is an aminoketone.


The correct answer is: Azaspirone-decanedione

011
Buspirone belongs to the chemical class of

Select one:
Azaspirones
Cyclopyrrolone
Imidazopyridine

Pyrazopyrimidine

755
Butyrophenone

Buspirone is believed to exert its anxiolytic effect by acting as a partial agonist at the 5-HT1A
autoreceptor. It is an azaspirodecanedione anxiolytic.
The correct answer is: Azaspirones

012
The new antipsychotic drug paliperidone belongs to the class of

Select one:
dibenzothiazepines
benzixosazoles
thienobenzodiazepine
dibenzodiazepines

benzamides

Being a metabolite of risperidone, paliperidone belongs to the same class - benzixosazoles.


The correct answer is: benzixosazoles

013
The drug alprazolam belongs to which of the following class of psychotropics?

Select one:
Stimulant

Antipsychotics
Anticholinesterase
Benzodiazepines
Antidepressant

Alprazolam is a benzodiazepine that is used for panic disorder in the USA.


The correct answer is: Benzodiazepines

756
014
Which of the following is termed as a norepinephrine and dopamine re-uptake inhibitor?

Select one:
Varenicline
Nefazadone
Buspirone
Mirtazapine

Bupropion

Buproprion is quite widely used in the USA as an antidepressant; it appears to inhibit the
reuptake of both dopamine and noradrenaline and, therefore, tends to have a slightly alerting
action. In many European countries, it has been introduced, at a lower than antidepressant
dose, in the treatment of nicotine withdrawal in smoking cessation programmes.
The correct answer is: Bupropion

015
Discovery of psychotropic use of lithium is associated with

Select one:
Sen and Ghose

Bleuler
Delay and Deniker
Cade
Kane

Believing that mania might be caused by intoxication by normal body products, Cade's
experiments led him to focus on lithium urate. He observed that guinea pigs became very
lethargic on the compound, and he subsequently conducted a clinical trial of lithium with ten
manic patients.
The correct answer is: Cade

016
Which of the following can be called a major tranquilliser?

757
Select one:
Risperidone
Chlorpromazine
Propofol

Chlordiazepoxide
Secobarbital

Antipsychotics came to be known as major tranquilizers while barbiturates and


benzodiazepines were called minor tranquilizers. 1950-52, Presurgical antihistamine
chlorpromazine was shown to have antipsychotic effects independently by Delay and
Deniker's team, and Charpentier from France
The correct answer is: Chlorpromazine

017
Which one of the following drugs is not correctly paired with its proponents?

Select one:
Chlorpromazine-Delay and Deniker
Imipramine-Kuhn
Clozapine-Bowden
Lithium- John Cade
Haloperidol-Janssen

Since the publication of the landmark study by Kane et al (1988), clozapine has been
regarded as an effective treatment for people with treatment-resistant schizophrenia.
The correct answer is: Clozapine-Bowden

018
Which one of the following names is associated with the use of valproate in mania?

Select one:
Delay and Deniker
John Cade
Kuhn

758
Janssen
Bowden

Bowden is associated with use of valproate in Mania (1994). A controlled trial by Kane et al in
1987 showed that clozapine might be effective in schizophrenia that had been resistant to all
other types of antipsychotic therapy.
The correct answer is: Bowden

019
Zopiclone belongs to the chemical class of

Select one:
Butyrophenone
Cyclopyrrolones
Imidazopyridine

Pyrazopyrimidine
Azaspirones

Zopiclone- cyclopyrrolone Zolpidem-imidazopyridine Zaleplon-pyrazopyrimidine


The correct answer is: Cyclopyrrolones

020
Discovery of tranquilising properties of chlorpromazine is associated with

Select one:
Delay and Deniker
Bleuler
Cade
Sen and Ghose
Kane

759
Chlorpromazine was synthesized on December 11, 1951 by Paul Charpentier, in the
laboratories of Rhone-Poulenc, a French pharmaceutical company, and released for clinical
investigation in May 1952 as a possible potentiator of general anesthesia. Clinical
investigations with CPZ began at Saint-Anne's hospital - at Pierre Deniker's service in Jean
Delay's department in 1952.
The correct answer is: Delay and Deniker

021
Choose the correct match from the following pairs:

Select one:
Risperidone: dibenzoxapine
Droperidol: butyrophenones
Aripiprazole: Benzisothiazole
Thioridazine: diphenyl butyl piperidine
Flupentixol: dihydroindole

Droperidol and haloperidol are classified as butyrophenones. The correct matching for other
mentioned drugs is given below- Piperidine derivatives- Thioridazine, Thioxanthenes-
Flupentixol; Benzisoxazole derivative­ Risperidone; arylpiperidylindole (quinolone)­
Aripiprazole.
The correct answer is: Droperidol: butyrophenones

022
A potential advancement in the treatment of dementia is

Select one:
Drugs acting on dendritic receptors
Drugs acting on plasma enzymes

Drugs acting on presynaptic receptors


Drugs acting on intracellular mechanisms
Drugs acting on synaptic enzymes

760
Drugs acting on ß-secretase are promising candidates to enter clinical trial for Alzheimer's
disease as of now. ß-secretase activity has a low optimum pH, and this membrane-bound
enzyme is predominantly localized in acidic intracellular compartments (for example,
endosomes and the trans-Golgi network).
The correct answer is: Drugs acting on intracellular mechanisms

023
Which of the following is an inhibitor of norepinephrine and serotonin re-uptake?

Select one:
Reboxetine
Bupropion
Galantamine
Duloxetine
Aripiprazole

SNRIs - serotonin and noradrenaline reuptake inhibitor. This includes venlafaxine, milnacipran,
duloxetine. Reboxetine is an NARI - Noradrenaline reuptake inhibitor. NaSSA - Noradrenergic
and specific serotonergic antagonist - which includes Mirtazapine and mianserin.

The correct answer is: Duloxetine

024
Which of the following antidepressant is an enantiomer of another antidepressant drug?

Select one:
Fluoxetine
Paroxetine

Citalopram
Escitalopram
Sertraline

761
Research evidence suggests that escitalopram may be more effective and have an earlier onset
of action than citalopram. This is attributed to escitalopram's binding to both the re- uptake site
and an allosteric site causing conformational changes in the 5-HT Transporter and enhancing
re-uptake blockade.
The correct answer is: Escitalopram

025
Which one among the following is a butyrophenone derivative?

Select one:
Chlorpromazine
Thioridazine
Haloperidol
Flupenthixol
Zuclopenthixol

First-generation agents are usually classified into three groups: phenothiazines


(chlorpromazine, Thioridazine), butyrophenones (e.g., haloperidol), and several other minor
chemicals (e.g., thiothixene, molindone, and loxapine), based on their structure.
The correct answer is: Haloperidol

026
Which one of the following led a trial that proved Clozapine's effectiveness in treating resistant
schizophrenia?

Select one:
Kretschmer
Cade

Kraepelin
Kane
Bleurer

The most widely cited study on rebirth of clozapine was the one published in Archives of
Psychiatry in 1988 by Kane and colleagues (Arch Gen Psychiatry 1988; 45:789­796). 268
patients with treatment-resistant schizophrenia (failing three different conventional neuroleptic

762
trials, including a six-week trial of haloperidol at maximum (60mg) doses) were randomly
assigned to clozapine up to 900 mg or chlorpromazine up to 1800 mg. 30% of the clozapine
group responded while only 4% in the chlorpromazine group responded.
The correct answer is: Kane

027
Select the reason why clozapine was initially withdrawn before Kane's landmark trial:

Select one:
High costs
High fatality rates
Lack of efficacy
Unclear dosing schedule
Excessive sedation

Clozapine was introduced in Europe in 1975.As a result of reports from Finland, where 16
patients out of 2260 exposed (0.7%) developed agranulocytosis and 8 (50%) of them
subsequently died from secondary infections, the drug was voluntarily withdrawn from use.
Following pressure from psychiatrists to reintroduce clozapine, trials in patients with
treatment-resistant schizophrenia, under close haematological monitoring were devised, which
showed significant improvement in 30% of patients after six months (Kane et al., 1988).
Subsequent studies showed improvement in 61% of patients if treatment was continued for up
to one year .
The correct answer is: High fatality rates

028
Which of the following ethnic differences in pharmacogenetics create natural alcohol
deterrence?

Select one:
Low cholinesterase
Lack of aldehyde dehydrogenase

High aldehyde dehydrogenase


High alcohol dehydrogenase
High CYP 3A4

763
Nearly 40% Asians and around 60% South American Native Indians lack Aldehyde
dehydrogenase enzyme in sufficient amounts to metabolise alcohol - this serves as a natural
deterrent in these communities
The correct answer is: Lack of aldehyde dehydrogenase

029
Which of the following psychotropic agents has the same pharmacokinetic properties in
Asians and White Europeans?

Select one:
Alprazolam
Haloperidol
Diazepam
Chlorpromazine
Lithium

Lithium is completely excreted renally; its excretion does not differ between Asians and
Europeans.
The correct answer is: Lithium

030
A 15-year-old boy has not been to his school for last few weeks as he has been disturbed by
derogatory auditory hallucinations. He has been hearing voices in the third person for two
months and is receiving messages from the television. The most likely treatment option is

Select one:
Parental Training
Family Therapy

Fluoxetine
Cognitive Behavioural Therapy for anxiety
Low dose risperidone

The diagnosis is early onset schizophrenia and treatment should be one of the second-
generation antipsychotics like Risperidone (low dose).
The correct answer is: Low dose risperidone

764
031
Which one of the following is the first antidepressant to be introduced?

Select one:
SNRIs
Tetracyclics
SSRIs

Tricyclics
MAO inhibitors

Iproniazid is a monoamine oxidase inhibitor. This was the first antidepressant developed in
1952. Iproniazid was being used in treating tuberculosis discovery of mood lifting properties.
But hypertensive reactions precluded large-scale use. Imipramine manufactured as
chlorpromazine derivative came to market soon.
The correct answer is: MAO inhibitors

032
The antidementia drug with an NMDA receptor antagonistic property is;

Select one:
Tacrine
Galantamine
Donepezil
Rivastigmine
Memantine

Memantine as an NMDA receptor antagonist can prevent the pathological stimulation of


NMDA receptors.
The correct answer is: Memantine

033
Which drugs among the following is classified as a reversible MAOI?

Select one:
Isocarboxazid

765
Tranylcypromine
Moclobemide
Raclopride

Phenelzine

Reversible MAO inhibitors include moclobemide and brofaromine. These are less likely to
induce hypertensive crisis than first-generation MAOIs.
The correct answer is: Moclobemide

034
A patient is diagnosed with mild cognitive impairment. Which one of the following is
recommended for prevention of dementia? (June 2008)

Select one:
Arachidonic acid (unsaturated fatty acid)

Donepezil
No recommendation
Memantine
NSAIDS

Currently there is no recommended prophylactic intervention is available for preventing the


conversion of MCI to dementia.
The correct answer is: No recommendation

035
Placebo effect is NOT mediated via

Select one:
Nocebo response
Natural remission
classical conditioning
expectancy

operant conditioning

766
A placebo makes patients feel better for reasons unrelated to the specific healing properties of
the treatment. A nocebo makes patients feel worse (or does other harm) in the same way.
The correct answer is: Nocebo response

036
A 45-year-old man with paranoid schizophrenia is started on a thienobenzodiazepine. Which of
the following drugs is prescribed here?

Select one:
Paliperidone
Piperazine
Olanzapine
Quetiapine
Clozapine

Olanzapine is a thienobenzodiazepine.
The correct answer is: Olanzapine

037
Which of the following is true about placebo response in drug trials?

Select one:
No placebo response is seen in personality disorders
A group of ‘placebo reactors' with stereotyped personality can be identified

Degree of placebo response is similar irrespective of the treatment studied


Opioids may have a role to play in placebo response

Placebo response to depression is gradual but persistent

Endogenous opioids (e.g. endorphins) play a significant role in mediating placebo-induced


analgesia. Dopamine reward system is being increasingly implicated in placebo effects in
psychotropic. Placebo response to depression starts abruptly, occurs early in treatment and is

767
less likely to persist. Placebos work better for severe than mild pain, but mildly depressed
patients respond well than severely depressed ones. There are no homogenous placebo
reactors in the population.
The correct answer is: Opioids may have a role to play in placebo response

038
Which of the following is a true statement about the placebo effect?

Select one:
Placebo response is perceived to be better for tablets than capsules
Placebo effects do not operate on active drugs
Placebo response is likely if patient is of lower social class
Severely depressed subjects respond well to placebos than mildly
depressed ones
Oral medications elicit a stringer placebo effect than injections

Placebo reactors are often less educated and belong to somewhat lower social class than
non-reactors.
The correct answer is: Placebo response is likely if patient is of lower social class

039
Before approval of a drug molecule, mutagenicity, carcinogenicity and organ system toxicity
are studied at which phase of clinical trials?

Select one:
Preclinical animal phase
Phase 1

Phase 3
Postmarketing surveillance
Phase 2

Preclinical Animal Studies: The pathway a drug must undergo before approval starts with
animal studies where the molecule is demonstrated to have specific actions. These extensive
preclinical animal studies must be carried out at least on two different animal species.
Mutagenicity, carcinogenicity and organ system toxicity are studies at this phase
The correct answer is: Preclinical animal phase

768
040
Reserpine was originally introduced as an antipsychotic. It is extracted from which of the
following plants?

Select one:
Hypericum alkaloids
Solanacea Henbane
Rauwolfia serpentina
Hellebore
Belladonna

Rauwolfia serpentina was used in India for more than 2000 years ago for insanity. Reserpine
was isolated from it in 1953 and synthesized and introduced as an antipsychotic in 1954. It
provided insights into the role of dopamine, noradrenaline and serotonin in the brain and also
the mechanism of antidepressant/antipsychotic drug actions.
The correct answer is: Rauwolfia serpentina

041
Rivastigmine is a

Select one:
Irreversible inhibitor of AchEs
NMDA receptor antagonist

Reversible non-competitive inhibitor of AchEs


Competitive inhibitor of AchEs
Reversible competitive inhibitor of AchEs

Rivastigmine is a reversible and non inhibitor of acetylcholinesterase. It inhibits both


acetylcholinesterase and butyryl cholinesterase in CNS.
The correct answer is: Reversible non-competitive inhibitor of AchEs

042
Which one among the following is a benzisoxazole derivative?

769
Select one:
Amisulpride
Clozapine
Risperidone

Sulpiride
Ziprasidone

Risperidone is a benzisoxazole derivative while sulpride and amisulpride are substituted


benzamides. Ziprasidone, rather confusingly, is a Benzisothiazole, while clozapine is a
Dibenzodiazepine.
The correct answer is: Risperidone

043
Which of the following drugs is correctly matched with its metabolite?

Select one:
Naloxone - naltrexone
Imipramine - nomifensine
citalopram - escitalopram
risperidone - paliperidone

fluoxetine - duloxetine

Escitalopram is not a metabolite but an enantiomer of citalopram. Paliperidone is the active


phase 1 metabolic product of risperidone.
The correct answer is: risperidone - paliperidone

044
Duloxetine and milnacipran are classified as

Select one:
Monoamine oxidase inhibitors
Tricyclic antidepressants
Selective serotonin reuptake inhibitors

Serotonergic and noradrenergic reuptake inhibitors

770
Adrenergic type 2 antagonists

Both these drugs belong to the same class as venlafaxine.


The correct answer is: Serotonergic and noradrenergic reuptake inhibitors

045
Which drug is useful for the treatment of OCD?

Select one:
Trazadone
Duloxetine
Phenelzine
Venlafaxine

Sertraline

SSRIs remain as the first-line treatments for OCD. Sertraline is SSRI drug, which is also useful
for the treatment of OCD in children. (Refer - POTS study).
The correct answer is: Sertraline

046
Venlafaxine is best described as a

Select one:
SNRI (Serotonin and noradrenaline reuptake inhibitor)

SSRI (Selective serotonin reuptake inhibitors)


NARI (Nor adrenaline reuptake inhibitors)
Tricyclic antidepressant
Alpha 2 adrenoreceptor antagonist

Venlafaxine and duloxetine are drugs showing selectivity in inhibiting the reuptake of both
noradrenaline and serotonin. They are best grouped as SNRIs.

771
The correct answer is: SNRI (Serotonin and noradrenaline reuptake inhibitor)

047
Which drug could be used for the treatment of mixed affective episodes of bipolar disorder?

Select one:
Carbamazepine
Gabapentin
Sodium Valproate
Lithium
Lamotrigine

Mixed affective states are characterised by combinations of manic and depressive symptoms
during one episode and may occur in up to 40% of bipolar patients. Mixed affective states have
been insufficiently researched to arrive at definitive recommendations for treatment.Sixty six
percent of these patients exhibit a poor response to lithium
NICE recommends that mixed affective episodes are treated as manic episodes. Sodium
valproate as a first line option for the treatment of acute manic episode and prophylaxis of
bipolar affective disorder, and thus can be used as first line for mixed affective episodes as well.
The correct answer is: Sodium Valproate

048
Zimeldine caused demyelination disorder on introduction, leading to its withdrawal. Which of
the following class does it belong to?

Select one:
MAO inhibitors
Tricyclics

Benzodiazepines
Antipsychotics
SSRIs

Carlssen synthesized purpose made SSRI Zimeldine - but withdrawn due to the incidence of a
hypersensitivity syndrome and demyelinating disease that followed its use.
The correct answer is: SSRIs

772
049 Acamprosate is a

Select one:
Synthetic taurine analogue
Type of benzodiazepine
Long acting opioid agonist
Alpha-1 agonist
Opiod receptor antagonist

Acamprosate is a synthetic taurine analogue, which appears to act centrally on glutamate and
GABA neurotransmitter systems, although the mechanism has not been fully established.
The correct answer is: Synthetic taurine analogue

050
What is cyproterone acetate?

Select one:
Glucocorticoid analogue
Testosterone antagonist
Mineralocorticoid anatagonist
Progesterone antagonist
Testosterone partial agonist

Cyproterone has a structure similar to testosterone. It works by preventing testosterone from


binding to the receptors (antagonist).
The correct answer is: Testosterone antagonist

051
You are giving a talk to your hospital staff about the importance of clinical trials. How would
you explain the term 'standardised difference in effectiveness'?

Select one:

773
The difference in the effect of treatment in the active treatment group
compared to the placebo group after taking into account the variation in the
treatment effect
The effect of treatment in the active treatment group
The difference in the effect of treatment in the active treatment group
compared to the placebo group
The number of patients that require one treatment compared to the other in
order to detect one positive response
The effect of treatment in the placebo group

The number of patients that require one treatment compared to the other in order to detect one
positive response Is the NNT (Number Needed to Treat). The standardised mean difference is
also called Cohen's d.
The correct answer is: The difference in the effect of treatment in the active treatment group
compared to the placebo group after taking into account the variation in the treatment effect

052
The term active placebo refers to which one of the following?

Select one:
None of the above
The placebo produces side effects
The placebo gains positive effect with repeated use.
The placebo has activity against the treated illness.
The placebo has some activity inherently

An 'active placebo' has some activity inherently, but not against the treated condition. When a
drug administered as placebo produces prominent side-effects, it is known as a 'nocebo'.
Placebo sag is a term used to refer to decrease in placebo effect with repeated or chronic
administration of placebo drugs.
The correct answer is: The placebo has some activity inherently

053
Which of the following psychotropics is well known to cause sudden death?

Select one:

774
Temazepam
Procyclidine
Zopiclone

Lithium
Thioridazine

Antipsychotic related sudden death may be due to cardiac arrhythmias or even seizures
asphyxiation or malignant hyperthermia. Drugs causing QT prolongation are associated with
more sudden deaths (esp. thioridazine, droperidol). Clozapine is also implicated.
The correct answer is: Thioridazine

054
Flupenthixol belongs to the chemical class of

Select one:
Diphenylbutylpiperidine
Piperidine phenothiazine
Thioxanthene
Piperazine phenothiazine
Aliphatic phenothiazine

Flupenthixol and zuclopenthixol are thioxanthenes.


The correct answer is: Thioxanthene

055
Which of the following hormones is most commonly used in the treatment of depression?

Select one:
Cortisol
Vasopressin

Prolactin
Thyroid hormone
Growth hormone

775
The link between thyroid function and depression is well-studied in psycho-endocrinology.
Notably, hypothyroidism can be accompanied by depression. All the hormones of the
hypothalamic- pituitary-thyroid axis have been used in the treatment of depression, alone or in
combination with other agents, although the most commonly used are thyroxine and
levothyroxine.
The correct answer is: Thyroid hormone

056
An 81-year-old woman with dementia presented to the hospital with chest pain. Upon
admission, she becomes acutely confused and agitated. She is combative and attacks a nurse
who tried to pacify her. She refuses to accept a nurse to sit beside her and observe her
continuously and threatens to kick anyone who attempts this. The best option available for the
medical team is

Select one:
Use heavy sedation to prevent physical attacks
Discharge her back to home
Use temporary physical or pharmacological restraints with regular re-
evaluation of her medical status
Continue to try verbal de-escalation to calm her down
Transfer her to a psychiatric ward

Using rapid tranquillisation or physical restraints is sometimes necessary for the elderly to
reduce harm to the patient and/or the health professionals. Only a minimal amount of restraint
should be used, and the need for continued restraint re-evaluated frequently. Transferring to a
psychiatry ward does not seem practical at this time due to the acute level of patient agitation
and his behaviors, and verbal de-escalation has already failed to calm the patient. Involving a
carer may help to calm the patient down, and that individual may be able to assist in making
decisions for his care.
The correct answer is: Use temporary physical or pharmacological restraints with regular re-
evaluation of her medical status

776
Home HiYield Paper Pharmacology

001
A patient works and drives to work at 6:30 AM and wants night sedation. Based on half life,
which of the following is the best? (June 2008)

Select one:
Lorazepam
Zopiclone
Zolpidem
Temazepam
Diazepam

Zolpidem is rapidly absorbed through the gastrointestinal tract, reaching peak concentration
from 30 minutes to 2 hours after administration. It is metabolized in the liver to several inactive
metabolites and has an elimination half-life of approximately 2.5 hours. Hence, a 'hangover'
effect is rare.
The correct answer is: Zolpidem

002
A patient suddenly withdraws from a medication. How many half-lives does it take for the
medication to be completely excreted?

Select one:
5
3
2

777
8

10

From a clinical standpoint, it is common to assume that a drug is effectively eliminated after 4- 5
half-lives, assuming first order kinetics
The correct answer is: 5

003
Zaleplon has a half-life of approximately

Select one:
12-24 hours
More than 24 hours
1-3 Hours

1 hour

3-6 Hours

Zopiclone- 3-6 hours Zolpidem- 1-3 hours Zaleplon- approximately 1 hour


The correct answer is: 1 hour

004
What is the volume of distribution of drug A if the plasma concentration immediately after IV
administration of 10mg is 0.01mg/mL?

Select one:
100mL
1000mL
1 mL
10mL

0.1mL

778
(Volume of distribution) Vd =Q/Cp, where Vd-volume of distribution, Q-quantity of drug and
Cp-plasma concentration at time of administration ('zero time').
The correct answer is: 1000mL

005
A drug X is used to treat acute behavioural disturbances. It can be given orally or
intravenously. 15mg of the oral drug is needed for an acutely distu man of weight 70kgs
while 10mg is sufficient if given intravenously. What is the bioavail of IV administration of
10mg?

Select one:
10.5mg
10mg
5mg
1.5mg
15mg

When a drug is administered intravenously the availability of the drug is 100%.


The correct answer is: 10mg

006
A drug follows first order kinetics and has a t1/2 of 4 hours. In a healthy male how long will it
take for this drug to reach steady state assuming no loading dose is given, and the drug is
administered at equal intervals regularly?

Select one:
10 hours
2 hours

4 hours
20 hours

1 hour

It is estimated that it takes 4-5 t ½ for a drug to reach the steady plasma level.
The correct answer is: 20 hours

779
007
Steady-state plasma concentration is usually achieved after

Select one:
1-2 half-lives
2-3 half-lives
10-20 half lives
4-5 half-lives

9-10 half-lives

Steady state concentration is achieved after repeated doses, which lead to an equilibrium
between absorption and elimination. It is usually achieved after 4-5 half-lives. When the drug is
given at shorter intervals than the half-life, this leads to lesser fluctuations in plasma
concentration. Attempts to reduce plasma fluctuations include using slow release (called MR
for modified release or XL for extended release) preparations, e.g., lithium, venlafaxine or to
allow long intervals between administration, e.g., depot antipsychotics.
The correct answer is: 4-5 half-lives

008
How many minimum weeks of interval should be given between fluoxetine and phenelzine
when switching from the SSRI to the MAOI?

Select one:
5 weeks
1 week
12 weeks
3 weeks

2 weeks

When the switch involves an MAOI, sufficient time must be given for medication clearance.
Fluoxetine requires a much longer period-6 weeks-whereas sertraline and paroxetine require
about two weeks when switching to an MAOI.
The correct answer is: 5 weeks

009
In a patient with severe liver disease, antipsychotic of choice will be

780
Select one:
Amisulpride
Clozapine
Quetiapine

Risperidone
Olanzapine

Amisulpride is predominantly renally excreted, so dosage reduction should not be necessary as


long as renal function is normal, but there are no clinical studies in people with hepatic
impairment and little clinical experience.(Maudsley Pres. Guidelines 9th)
The correct answer is: Amisulpride

010
Urinary acidification can help eliminate which of the following medications on overdose?

Select one:
Amphetamines
Salicylates
Paracetamol
Tricyclics
Barbiturates

Acidification may help in the elimination of amphetamines and phencyclidine, but often
complications of such a procedure override any benefits.
The correct answer is: Amphetamines

011
Which of the following drugs have a half-life of 3 days?

Select one:
Quetiapine
Reboxetine
Aripiprazole

Carbamazepine

781
Fluoxetine

Aripiprazole has a half-life of 75 to 96 hours.


The correct answer is: Aripiprazole

012
A newly developed antipsychotic is said to be highly permeable across lipid membranes and so
achieve very high bioavailability when given orally. Which of the following properties of the drug
enable this?

Select one:
All of the above
Low concentration gradient across membranes
Low protein binding
Exists in highly ionised form predominantly
Balanced hydrophilic and lipophilic properties

If highly hydrophilic; the drug cannot cross the lipid cell membrane. On the other hand, if
highly lipophilic, the drug is not soluble enough to cross the water layer adjacent to the
cell.Balanced hydrophilic and lipophilic property is essential for high permeability.
The correct answer is: Balanced hydrophilic and lipophilic properties

013
The area under the curve after a single dose allows determination of the

Select one:
Plasma half-life (t-1/2)

Metabolism
Elimination
Distribution
Bioavailability of the drug

782
The area under the curve (AUC) is proportional to the amount of drug in plasma and allows
determination of fraction of dose absorbed (i.e. Bioavailability of the drug). Low Bioavailability
usually means high first pass metabolism. T-1/2 is the time taken for the plasma concentration
to fall by a half. It only applies for drugs with first order kinetics.
The correct answer is: Bioavailability of the drug

014
Which of the following should be considered when switching from one brand of medication to
another brand?

Select one:
Rate of absorption
Bioequivalence
Bioavailability
Volume of distribution
Clearance

Bioequivalence is important feature to be considered when changing from one brand to other
brand of the same compound e.g. camcolit vs. priadel for lithium carbonate or Clozaril vs.
zaponex for clozapine. It is a measure of comparability of plasma levels of two different
formulations of the same active compound when given at same dose and same route of
administration
The correct answer is: Bioequivalence

015
Which of the following drugs has the least propensity to cause a withdrawal effect?

Select one:
Temazepam
Zolpidem

Buspirone
Zopiclone
Zaleplon

No evidence of a withdrawal syndrome and no interaction with alcohol have been reported
with Buspirone.

783
The correct answer is: Buspirone

016
Advantages of buspirone over benzodiazepines when used for anxiety includes all of the
following except

Select one:
Buspirone cause no dependence
Buspirone can be given once daily
Buspirone can be withdrawn more easily
Buspirone has no street value
Buspirone causes no tolerance usually

The advantage of buspirone is its lack of withdrawal and tolerance effects. Its main clinical
disadvantage lies in the delay in onset of its therapeutic effect (up to 2 weeks in some cases)
and its limited efficacy in attenuating anxiety in those patients who had previously responded to
benzodiazepines. It is usually given twice or thrice daily.
The correct answer is: Buspirone can be given once daily

017
Which of the following drugs has its plasma levels reduced even after regular administration of
the same dose for a month?

Select one:
Lithium
Clozapine

Chlormethiazole
Valproate
Carbamazepine

Carbamazepine exhibits autoinduction: it induces the expression of the hepatic microsomal


enzyme system CYP3A4, which metabolizes carbamazepine itself.
The correct answer is: Carbamazepine

018

784
Which one of the following is a potent hepatic enzyme inducer?

Select one:
Gabapentin
Carbamazepine
Lamotrigine

Valproate
Lithium

Liver enzyme induction can increase the rate of metabolism of many drugs resulting in lower
plasma levels and so lessened efficacy. Phenytoin, phenobarbitone, Primidone and
carbamazepine are potent enzyme inducers that tend to lower the plasma levels of each other
and other drugs. Carbamazepine especially is a potent hepatic enzyme (cytochrome P450)
inducer that induces its own metabolism as well as other drugs and is metabolised by CYP3A4.

The correct answer is: Carbamazepine

019
Valproate levels are decreased by

Select one:
Lithium
Cimetidine
Erythromycin
Carbamazepine
Fluoxetine

Valproate can increase the plasma levels of some drugs like TCAs (particularly clomipramine),
lamotrigine and phenobarbitone by competitive inhibition of their metabolism. The level of
valproate itself is decreased by carbamazepine.
The correct answer is: Carbamazepine

020
Which of the following is true concerning tricyclic metabolism?

Select one:

785
Elderly clear more than adults
Children clear more tricyclics than adults
None of the drugs need plasma monitoring

Metabolism is not affected by CYP enzyme polymorphisms

They do not interact with chlorpromazine

Tricyclic antidepressants are not effective in prepubertal children but may have marginal
efficacy in adolescents with depression. Due to more extensive metabolism, young people
require higher mg/kg doses than adults.
The correct answer is: Children clear more tricyclics than adults

021
SSRI most selective for serotonin reuptake is

Select one:
Citalopram
Paroxetine
Sertraline
Duloxetine
Fluoxetine

Of the available SSRIs, citalopram and escitalopram are the most selective for serotonin
receptor blockade; citalopram is made up of both cis and R enantiomers. Escitalopram is 100
times more potent than the R-enantiomer.
The correct answer is: Citalopram

022
Which of the following acts on presynaptic receptors as the main mechanism of action?

Select one:
Donepezil
Diazepam
Phenelzine

Propronalol

786
Clonidine

Clonidine, lofexidine act at alpha2 presynaptic receptor.


The correct answer is: Clonidine

023
Which of the following enzymes are responsible for oxidative metabolism of most psychotropic
drugs?

Select one:
Cytochrome P450 family
Glycoprotein P
Aryl Sulfatase
Monoamine Oxidase

G Protein

The cytochrome P450 (CYP450) enzyme superfamily is the primary phase I enzyme system
involved in the oxidative metabolism of drugs and other chemicals. These enzymes also are
responsible for all or part of the metabolism and synthesis of a number of endogenous
compounds, such as steroid hormones and prostaglandins.
(http://pharmafactz.com/index.php/general-principles/pharmacokinetics-pharmacodynamics)
The correct answer is: Cytochrome P450 family

024
The active metabolite of imipramine is

Select one:
lofepramine

mcPP
Desipramine
Nortriptyline
Amitriptyline

787
The active metabolite of amitriptyline is nortriptyline and imipramine is desipramine.
The correct answer is: Desipramine

025
Which one among the following statements is least likely to be correct with respect to the
tissue distribution of drugs?

Select one:
Drugs with high affinity for tissues such as fat have high volume of
distribution
The drug distribution in plasma to various tissues depends on plasma
protein binding
The drug distribution in plasma to various tissues depends on tissue
permeability
Distribution is uniform across the body tissues
Tissue distribution leads to a fall in plasma concentration

Drugs are not evenly distributed in the body. There are three main compartments through which
a drug may be distributed. (1) The Vascular compartment (2) The extra cellular compartment (3)
The intracellular compartment.Drugs that are large molecules and those which bind tightly to
plasma proteins remain in the vascular compartment; drugs with low lipid solubility are
distributed in the vascular plus the extracellular compartment whilst lipid soluble drugs are
distributed through the three compartments. Drugs such as Phenytoin, diazepam,
chlorpromazine and amitriptyline are all highly bound to plasma proteins. They are very
lipophilic and once absorbed pass readily into peripheral fat stores; from which they are
gradually released long after the last dose has been administered.
The correct answer is: Distribution is uniform across the body tissues

026
Drug B is a prodrug whose metabolite is an active hypnotic agent. If drug A induces the
metabolism of drug B, which of the following is true?

Select one:
Effects of drug B are more pronounced
No change occurs in drug effect

Effect of drug A will increase


Hypnotic effect is lost
Level of drug B's metabolite falls in blood

788
Prodrug's metabolism is increased. It will undergo more metabolism increasing the level of the
active metabolite; drug B's effect will be more pronounced.
The correct answer is: Effects of drug B are more pronounced

027
Which of the following drugs has a metabolite that has the longest half-life?

Select one:
Fluoxetine
Duloxetine
Venlafaxine
Paroxetine
Citalopram

There are substantial differences in term of their half-life between fluoxetine and others SSRIs.
The half-life of fluoxetine and its active metabolite norfluoxetine is respectively 2 to 4 days and 7
to 15 days, more extended than other SSRIs (approximately one day).
[http://www.ncbi.nlm.nih.gov/pubmed/10598311]
The correct answer is: Fluoxetine

028
The SSRI with longest total half-life is

Select one:
Citalopram
Fluoxetine

Duloxetine
Sertraline
Paroxetine

789
The only serotonin reuptake inhibitor with an active metabolite is fluoxetine, whose metabolite
norfluoxetine has a half-life of 7 to 15 days. Thus, it may take several months to achieve steady
state with this agent. This is considerably longer than citalopram (half-life 1.5 days) or sertraline
and paroxetine (half-lives 24 hours).
The correct answer is: Fluoxetine

029
Which one among the following statements is true with respect to the interaction of fluoxetine
with other drugs?

Select one:
Fluoxetine reduces the effects of propranalol
Fluoxetine reduces the plasma concentration of carbamazepine
Fluoxetine reduces the concentration of Haloperidol
Fluoxetine enhances the effects of oral anticoagulants

Fluoxetine reduces the plasma concentration of Phenytoin

Fluoxetine increases concentration of haloperidol, carbamazepine and phenytoin. Fluoxetine


enhances the effects of oral anticoagulants and propranolol.
The correct answer is: Fluoxetine enhances the effects of oral anticoagulants

030
Which of the following drugs is excreted unchanged in urine?

Select one:
Quetiapine
Gabapentin

Valproate
Diazepam
Morphine

Gabapentin doses must be reduced for patients with renal insufficiency as it is solely excreted
by kidneys.
The correct answer is: Gabapentin

790
031
Bioavailability is least likely to be influenced by

Select one:
Metabolism
Absorption
Distribution

Excretion
Gender

Bioavailability is the fraction of an ingested dose of a drug that gains access to the systemic
circulation. It may be low because the drug is metabolised in the gut wall or liver before
reaching the systemic circulation.
The correct answer is: Gender

032
Which of the following is not altered much when considering pharmacokinetics in the elderly?

Select one:
Total body fat
Renal clearance
Plasma protein binding
GI absorption
Gastric pH

Following changes are noted- Increase in total body fat, gastric pH is increased as acidity drops,
reduced renal clearance, Decrease in plasma protein binding and decreased number of brain
acetylcholine postsynaptic receptors. However, GI absorption and hepatic metabolism are not
altered.
The correct answer is: GI absorption

033
Most orally administered psychotropic drugs undergo hepatic metabolism followed by renal
clearance. Which of the following statement about clearance is true?

Select one:

791
Clearance can occur only via urine
It helps calculating half life of a drug in the body
It is fixed for each individual with minor variations among drugs

It is the amount of drug eliminated from the body in a given time

It always depends on the dose consumed

Clearance is defined as the volume of blood cleared of a particular drug in unit time. It is
specific for each drug and does not depend on drug concentration in plasma (because if
concentration increases, elimination will also increase under first order kinetics). It represents
the relationship between the rate of drug elimination (t1/2) and plasma level. For drugs with first
order kinetics, clearance is constant irrespective of dose consumed because the rate of
elimination is directly proportional to plasma level. Total body clearance depends on renal and
non-renal clearance such as sweat, bile etc.Renal elimination without significant liver
breakdown is seen in lithium, amisulpride, surprise, gabapentin, acamprosate and amantadine.

The correct answer is: It helps calculating half life of a drug in the body

034
A new molecule investigated for anxiety disorder failed at initial stages of animal testing as it
failed to reach the brain crossing the blood-brain barrier. Which of the following factors decide
the degree of blood-brain barrier permeability?

Select one:
Frequency of administration of drug
Time of drug administration
Half life of drug
Lipid water partition coefficient
Nitrogen content of the drug

The ability of a drug to pass blood brain barrier depends on its molecular size, lipid solubility
and ionic status. Unionized molecules that are freely available and less protein bound are
transported across the barrier easily. In general higher the lipid-water partition coefficient,
greater the ability to cross the barrier
The correct answer is: Lipid water partition coefficient

035
Which of the following drugs is excreted completely via renal route?

792
Select one:
Olanzapine
Lithium
Quetiapine

Amisulpride
Carbamazepine

Lithium is completely renally handled for its clearance. Though largely renal, amisulpride has
some hepatic involvement.
The correct answer is: Lithium

036
Regarding the pharmacokinetics of Lithium which one of the following statements in not true?

Select one:
It passes freely through the glomerular membrane independent of serum
concentration.
Lithium has a wide therapeutic range
It is not bound to serum proteins
Lithium renal clearance is reduced when sodium is depleted.

The half-life of Lithium lies between 10 and 24 hours

Lithium has a narrow therapeutic range, i.e., when compared with other drugs, there is a
narrow gap between the minimum blood level (0.4 m mol/L) required for therapeutic efficacy
and the maximum level (1.5 m mol/L) beyond which toxicity may ensue.
The correct answer is: Lithium has a wide therapeutic range

037
Which one among the following is preferable to be used in the elderly than other tricyclic
antidepressants?

Select one:
Imipramine
Dothiepin

793
Trimipramine
Amitryptyline
Lofepramine

Both Lofepramine and Nortriptyline are less sedative, less anticholinergic. Lofepramine is the
least cardiotoxic of the ones listed here and so preferred in the elderly.
The correct answer is: Lofepramine

038
A psychotropic drug is observed to undergo phase 2 metabolism directly without undergoing
phase 1 reactions. Which one of the following fits with the above description best?

Select one:
Lithium
Fluoxetine

Chlordiazepoxide
Diazepam
Lorazepam

lorazepam, temazepam and oxazepam undergo direct phase 2 reactions.


The correct answer is: Lorazepam

039
Which of the following is best choice to treat alcohol withdrawal in a man with severe liver
damage?

Select one:
Naltrexone
Carbamazepine
Chlordiazepoxide
Lorazepam
Diazepam

794
Irrespective of the indications, the choice of the benzodiazepine should be contingent upon
the circumstances:e.g. shorter-acting benzodiazepines such as lorazepam and oxazepam
may be more appropriate in the presence of liver damage.
The correct answer is: Lorazepam

040
Which of the following is least likely to be a pharmacokinetic property of the tricyclic
antidepressants?

Select one:
Extensive hepatic breakdown
Their activity may delay their own absorption

Lipid solubility
Metabolism being reduced by smoking
Extensive protein binding in the plasma

Tricyclics are rapidly and almost completely absorbed from the gut and are widely distributed in
the tissues of the body. They readily cross lipid barriers such as blood-brain barrier and
placenta. They are extensively bound to plasma proteins. E.g. Imipramine 80-95%. Smoking
and barbiturates facilitate metabolism and neuroleptics inhibit metabolism. Due to
anticholinergic effects, they may reduce their own absorption (delayed gastric emptying esp.
amitryptiline)
The correct answer is: Metabolism being reduced by smoking

041
Which of the following hypnotic drugs acts for the longest duration?

Select one:
Temazepam
Triazolam
Zolpidem
Zaleplon

Nitrazepam

795
Nitrazepam is a long acting benzodiazepine with an elimination half-life of 15-38 (mean
elimination half-life 26 hours)
The correct answer is: Nitrazepam

042
Which of the following factors can improve absorption of orally administered drugs?

Select one:
High hydrophilic capacity of drug
Fast intestinal motility
None of the above
Presence of food in the GI tract
Presence of P glycoprotein

This is mainly due to lack of absorption from the intestine related to presence of inhibitory factors
like food, or gastric acid or due to changes in intestinal motility e.g. having diarrhoea or vomiting
can affect drug absorption. Inherent properties of certain drugs can also affect their absorption
e.g. highly hydrophilic drugs cannot cross the lipid cell membrane, while highly lipophilic drugs
will struggle to cross the water layer in extracellular space Inhibition of P- glycoprotein (e.g. by
grapefruit juice) can increase absorption of certain medications
The correct answer is: None of the above

043
A clear therapeutic window is noted for which of the following?

Select one:
Nortriptyline
Amitriptyline

Carbamazepine
Fluoxetine
Clozapine

Although blood levels are available for many antidepressants; those for imipramine,
desipramine, and nortriptyline have been best established. Imipramine and desipramine appear
to have a curvilinear dose-response curve with an optimal range of 150 to 300 ng/mL.

796
Nortriptyline appears to have a therapeutic window in the range of 50 to 150 ng/mL, usually
reached by doses ranging from 50-100mg/day.
The correct answer is: Nortriptyline

044
Which one among the following drugs can decrease lithium excretion and precipitate toxicity?

Select one:
Diazepam
Valproate
Propranalol
Haloperidol
NSAIDs

Thiazide diuretics, Furosemide, NSAIDs, Haloperidol, carbamazepine, SSRIs and ACE


inhibitors can decrease its clearance, increase serum lithium levels and can precipitate lithium
toxicity. Though haloperidol is associated with increased neurotoxicity in combination with
lithium, it does not have any direct effect on lithium levels.
The correct answer is: NSAIDs

045
The metabolism of which one among the following follow first order kinetics?

Select one:
Alcohol
Oral Antipsychotic

Depot antipsychotic
Absorption of controlled release drugs
Phenytoin

Most drugs show first order kinetics where the rate of absorption or elimination is directly
proportional to the amount of drug remaining. The concentration of a drug in the plasma will
normally follow first order kinetics, decreasing in an exponential manner related to plasma
concentration. However, drugs like ethanol and Phenytoin are metabolised by zero order
kinetics, their concentration decreasing in a linear fashion i.e., at a constant rate irrespective
of plasma concentration. With zero-order kinetics, a fixed amount of drug is absorbed each

797
time independent of drug concentrations and hence a small increase in drug ingestion results in
a large increase in plasma concentration. Absorption of controlled release drugs and depot
antipsychotics also follow zero-order kinetics.
The correct answer is: Oral Antipsychotic

046
Which one among the following statements is incorrect about the oral route of administration?

Select one:
Preparations that are soluble in gastrointestinal fluids are absorbed better
Oral route drugs are subject to renal first pass metabolism
Lipid solubility aids absorption
Oral route leads to variable plasma concentration because the absorption
may be erratic
Acid-resistant drugs are better absorbed

Drugs taken orally are subject to metabolism by the liver (First-Pass Effect). A drug taken
orally is absorbed from the gut into the enterohepatic circulation, where the liver inactivates
some of the drug before even it reaches the systemic circulation, which is known as the first
pass effect. It can be avoided if the drug is injected intravenously whereby the drug directly
enters the systemic circulation.
The correct answer is: Oral route drugs are subject to renal first pass metabolism

047
The drug of choice for alcohol detoxification in a patient with liver disease is

Select one:
Chlordiazepoxide
Diazepam
Carbamazepine
Chlormethiazole
Oxazepam

Oxazepam has a lesser propensity to accumulate and cause adverse reactions in the elderly or
people with hepatic impairment. As it does not require hepatic oxidation, an is entirely
metabolized via glucuronidation, it is a safer choice if benzodiazepines are required in those

798
with liver failure.
The correct answer is: Oxazepam

048
Which of the following drugs has a higher therapeutic index than the others listed?

Select one:
Lithium
Phenytoin
Type 1 antiarrythmics
Paracetamol
Tricyclics

Drugs available over the counter usually have a better therapeutic index (else they won't be
sold without a prescription). This does not mean that they are absolutely safe.
The correct answer is: Paracetamol

049
Enzyme autoinhibition is most likely to be seen with which of the following antidepressants?

Select one:
Sertraline
Duloxetine
Fluoxetine
Paroxetine
Citalopram

Three of the five commonly available SSRI antidepressants (fluoxetine, paroxetine and
sertraline) show autoinhibition of CYP2D6 though paroxetine is the one that is most affected.
The correct answer is: Paroxetine

050
A fatal combination with respect to potentially severe serotonin syndrome is

799
Select one:
Moclobemide and paroxetine
Paroxetine and methylphenidate
MDMA and moclobemide

Pethidine and tranylcypromine


Phenelzine and Amitriptyline

MAOIs potentiate the actions of general anaesthetics, sedatives, including alcohol,


antihistamines, centrally acting analgesics (particularly pethidine due to an enhanced release of
5-HT) and anticholinergic drugs. Thus, sever serotonin syndrome may result with co-
administration of pethidine and MAOIs.
The correct answer is: Pethidine and tranylcypromine

051
Hepatic metabolism may involve various chemical reactions. Which one of the following is not a
common chemical activity seen in hepatic metabolism?

Select one:
oxidation
reduction
dealkylation
hydroxylation

polymerisation

Hepatic metabolism may involve non-synthetic reactions that involve oxidation, reduction,
hydroxylation, dealkylation, demethylation, acetylation; methylation may produce active
compounds (Phase 1). The metabolism also involves synthetic reactions such as conjugation
usually with glucuronic acid and sulphation that produce inactive water-soluble compounds.
(Phase 2)
The correct answer is: polymerisation

052
Which of the following atypical agents have the shortest half life?

Select one:
Quetiapine

800
Aripiprazole
Olanzapine
Clozapine

Risperidone

Half life of quetiapine is around 6 hours, requiring multiple daily dosings.


The correct answer is: Quetiapine

053
Under which of the following conditions interactions with drugs that alter protein binding
becomes important?

Select one:
Obesity

Children
Lung disease
High altitude living
Renal disease

Protein binding interactions become relevant in the presence of a renal disease where
proteinuria can occur.
The correct answer is: Renal disease

054
The enhancement of drug effects following the repeated administration of the same dose of a
drug is called

Select one:
Tolerance
Neuroadaptation
Habituation
Sensitisation

Cross Tolerance

801
Tolerance refers to the diminished response to the administration of a drug after repeated
exposure. It may be behavioural tolerance through learning to cope with the effects, increased
metabolism, reduced receptor sensitivity or number, activation of a homeostatic mechanism.
Cross-tolerance refers to shared tolerance among drugs with similar chemical structure.
Cross-tolerance forms the basis for a number of drug interactions (E.g. alcohol with
barbiturates). Sensitisation refers to the enhancement of drug effects following the repeated
administration of the same dose of the drug. Drug sensitization occurs in drug addiction and is
the physiological opposite of drug tolerance.
The correct answer is: Sensitisation

055
What is least likely to be correct concerning the pharmacology of clozapine?

Select one:
High affinity towards D4 receptors
Slow release from D2 receptors
Has antimuscarinic properties
Acts on D1 receptors
Blocks 5HT2A receptors

the rates of antipsychotic dissociation from the human-cloned D2 receptor. For instance,
radioactive haloperidol, chlorpromazine, and raclopride all dissociate very slowly over a 30-
minute time span, while radioactive quetiapine, clozapine, remoxipride, and amisulpride
dissociate rapidly, in less than 60 seconds. These data also match clinical brain-imaging
findings that show haloperidol remaining constantly bound to D2 in humans undergoing two
positron emission tomography (PET) scans 24 hours apart. Conversely, the occupation of D2
by clozapine or quetiapine has mostly disappeared after 24 hours.(Atypical antipsychotics:
mechanism of action, http://www.ncbi.nlm.nih.gov/pubmed/11873706 (accessed April 11,
2015).)
The correct answer is: Slow release from D2 receptors

056
Which of the following types of drug molecules is most efficiently absorbed?

Select one:
Small, water soluble electrically charged molecules
Small, water soluble electrically neutral molecules

Large, water soluble electrically neutral molecules

802
Small, lipid soluble electrically neutral molecules
Large, lipid soluble electrically neutral molecules

To achieve a therapeutic effect most drugs need to cross at least one lipid membrane. This will
depend on the size and shape of the molecule, lipid solubility and ionic charge. Hence, it is
more efficient for small, water-soluble electrically neutral molecules, which can cross
membranes easily.The blood-brain barrier only allows lipid-soluble molecules in the brain
The correct answer is: Small, lipid soluble electrically neutral molecules

057
Which of the following is an enzyme inducer?

Select one:
Caffeine
Smoking
Grapefruit juice
Paroxetine
Valproate

Smoking induces CYP1A2 via PAH.Caffeine is an inhibitor. Paroxetine, to some extent


fluoxetine, neuroleptics, amitriptyline and clomipramine inhibit
The correct answer is: Smoking

058
Which of the following psychotropic will be affected most by contact with moist air?

Select one:
Olanzapine
Gabapentin

Carbamazepine
Topiramate
Sodium valproate

803
Deliquescent salts such as valproate absorb moisture from the air
The correct answer is: Sodium valproate

059
A 44-year-old man is on warfarin for thromboprophylaxis. He returns from a holiday to
mainland Europe where he obtained a preparation for treating his depression. The GP is
concerned about his INR and organizes an urgent blood test. What is the likely drug taken?

Select one:
St John's Wort
Duloxetine
Carbamazepine
Citalopram
Venlafaxine

St John's Wort, an over-the-counter herbal preparation with antidepressant properties, poses a


high-risk interaction with oral anticoagulants by reducing the anticoagulant effect of coumarins
such as warfarin.
The correct answer is: St John's Wort

060
There is poor oral absorption of most psychotropics in which of the following parts of the
gastrointestinal system?

Select one:
Ileum
Stomach

Jejunum
Colon
Rectum

Absorption by oral administration occurs primarily in the small bowel, though absorption of
many slow or sustained release drugs occurs in the large bowel. There is poor absorption in
the acidic stomach but conversely good absorption in the alkaline jejunum, ileum, colon and
rectum.
The correct answer is: Stomach

804
061
Some drugs such as fluoxetine move from first order to zero order kinetics in supratherapeutic
doses. What happens to their t1/2 in such cases?

Select one:
t½ doubles
t½ becomes independent of dose
t½ becomes dependent on dose
t½ quadruples

t½ halves

In very high supratherapeutic doses, saturation of enzymes can happen for drugs such as
fluoxetine, wherein first order elimination switches to become zero order.. When the system
facilitating first order clearance of drugs gets saturated, drugs follow zero-order kinetics. Here a
constant amount, not a fraction, of the drug is cleared per unit time. This means that
irrespective of the amount of drug in plasma or dose of drug administered, only a fixed unit of
drug is cleared by the body.. Here the concept of half­life does not hold true as ‘half life'
depends on the dose administered.
The correct answer is: t½ becomes dependent on dose

062
Which of the following is incorrect with respect to the pharmacokinetic properties of
diazepam?

Select one:
The peak concentration is achieved within 30-90 minutes after single dose
The protein binding ratio is 30-40%

Bioavailability is almost complete with the oral dose


Desmethyldiazepam is the active phase 1 metabolite.

The rate of absorption is rapid

The protein-binding ratio of diazepam is 95%.


The correct answer is: The protein binding ratio is 30-40%

805
063
Which of the following medications must be used with caution when fluvoxamine is
prescribed?

Select one:
Theophylline
Naproxen
Amlodipine

Salbutamol
Paracetamol

Fluvoxamine decreases the clearance of cytochrome CYP1A2 substrates such as


theophylline, leading to toxic levels when co-administered.
The correct answer is: Theophylline

064
The ratio between minimum plasma level causing toxic effect to minimum plasma level
causing therapeutic effect is called

Select one:
Toxic dose
Therapeutic index
Volume of distribution
Affinity
Potency

Therapeutic index: It is the ratio of minimum plasma concentration causing toxic effects to that
causing a therapeutic effect. This can vary according to the toxic symptom specified for a given
drug
The correct answer is: Therapeutic index

065
Which of the following undergoes metabolism predominantly by first order kinetics?

Select one:

806
Phenytoin
Depot antipsychotics
Venlafaxine XL

Alcohol
Thioridazine

Elimination of a constant quantity per time unit of the drug quantity present in the organism is
called zero order kinetics. 95% of the drugs in use at therapeutic concentrations are eliminated
by first-order elimination kinetics. A few substances are eliminated by zero-order elimination
kinetics because their elimination process is saturated, especially at higher doses. Examples are
Ethanol, Phenytoin, high dose Salicylates, high dose Fluoxetine, high dose Omeprazole.
Because in a saturated process the elimination rate is no longer proportional to the drug
concentration but decreasing at higher concentrations, zero-order kinetics are also called
'non-linear kinetics' in clinical pharmacology. The absorption rate constant is slower than the
elimination rate constant and, therefore, the depot antipsychotics exhibit 'flip-flop' kinetics where
the time to steady-state is a function of the absorption rate, and the concentration at
steady-state is a function of the elimination rate. ( Elimination kinetics - UNIL,
http://sepia.unil.ch/pharmacology/index.php?id=94 (accessed April 11, 2015).)
The correct answer is: Thioridazine

066
Which of the following can be given as a loading dose in acute mania?

Select one:
Lithium
Clozapine
Valproate
Olanzapine
Clonazepam

Both lithium and valproate take at least seven days to produce antimanic effects. But loading
doses of (usually 20mg/kg/day) for valproate can increase the speed of action.
The correct answer is: Valproate

067
Which of the following medications has the least protein-binding?

807
Select one:
venlafaxine
paroxetine
valproate
clomipramine

fluoxetine

Venlafaxine has lower levels of protein binding (25%-30%) than other SSRIs (80%-100%). As a
result, the free fraction of venlafaxine is much greater than SSRIs at the same total plasma drug
level.
The correct answer is: venlafaxine

068
A 30-year-old is on combined oral contraceptive pill. The risk of becoming pregnant is
increased by all except

Select one:
Venlafaxine
Topiramate
St John's wort
Carbamazepine
Phenytoin

Low potency oral contraceptives may be rendered ineffective by enzyme-inducing


medications such as carbamazepine, phenytoin, St John's Wort and topiramate.
The correct answer is: Venlafaxine

069
A drug X, when given at a dose of 300 units, is cleared from the body at a rate of 30 units per
hour. When the dose is increased to 600 units, it is still cleared at a rate of 30 units per hour.
Which of the following is true? The drug follows

Select one:
first order kinetics
zero order at lower doses and first order at higher doses

808
first order at lower doses and zero order at higher doses
no specific kinetics

zero order kinetics

When the system facilitating first order clearance of drugs gets saturated, drugs follow zero-
order kinetics. Here a constant amount, not a fraction, of the drug is cleared per unit time. This
means that irrespective of the amount of drug in plasma or dose of drug administered, only a
fixed unit of the drug is cleared by the body. As such, increasing dose might result in serious
toxicity in this case.
The correct answer is: zero order kinetics

809
Home HiYield Paper Pharmacology

HiYield Paper A(2)

001
Mechanism of action of buprenorphine is via partial agonism at

Select one:
ß-opioid receptors
µ-opioid receptors
Adrenergic receptors

k-opioid receptors
Cholinergic receptors

Buprenorphine. A partial opiate agonist at mu-receptors is available in sublingual form as an


alternative to methadone for agonist maintenance treatment of opioid dependence.
The correct answer is: µ-opioid receptors

002
How much Zuclopenthixol depot is equivalent to 50mg of haloperidol given every four weeks?

Select one:
200mg every 2-4 weeks
20 mg every 2-4 weeks
20mg every 3 weeks
100mg every 2-4 weeks

400mg every 2 weeks

810
Using antipsychotic depot conversion chart from POMH (Prescribing Observatory for Mental
Health), 200mg / 2 weeks or 400mg/4 weeks of zuclopentixol depot is equivalent to 50mg of
haloperidol depot every 4 weeks.
The correct answer is: 200mg every 2-4 weeks

003
A 27-year-old man who is taking mirtazapine for generalized anxiety disorder and major
depression experiences significant sedation at a dosage of 7.5 mg/day. Which
neurotransmitter receptor is most likely being affected?

Select one:
Nictonic receptors
Histamine Type 1 receptors
5HT Type 2 receptors
Muscarinic type 1 receptors

5HT type 3 receptors

Blockade of 5HT2 receptors by mirtazapine helps improving the quality of sleep in depressed
patients.
The correct answer is: 5HT Type 2 receptors

004
Buspirone acts on

Select one:
GABA-b receptor complex
5HT2a receptor

5HT1a and 5HT2a receptor


5HT1a receptor
GABA-a receptor complex

811
Buspirone acts on 5HT1-a receptors. It is a partial agonist at post-synaptic 5HT1-a receptors
and a full agonist at pre-synaptic 5HT1-a receptors.
The correct answer is: 5HT1a receptor

005
Weight gain caused by clozapine is possibly related to

Select one:
Alpha-2 antagonistic activity
5HT1-a receptor antagonistic activity
5HT2-a receptor antagonistic activity

5HT2-c receptor antagonistic activity


5HT3 receptor antagonistic activity

The mechanism by which weight gain occurs during treatment with antipsychotics is poorly
understood, but the broader receptor affinities of the agents and their antagonism of histamine
H1 and serotonin 5-HT2C receptors have been implicated.
The correct answer is: 5HT2-c receptor antagonistic activity

006
A 85-year-old lady with a history of cerebrovascular disease, atrial fibrillation, and prior
myocardial infarction presents with moderately severe agitated depression with features of
self-neglect. She is taking warfarin, lisinopril, and has been on a maximum dose of
venlafaxine for last four weeks with moderate effect. The most appropriate intervention to
manage the depression is

Select one:
Add a benzodiazepine
Prescribe trazodone

Add mirtazapine
Trial of sertraline after stopping venlafaxine
Electro convulsive therapy

Mirtazapine works well in combination with venlafaxine; it will be the ideal choice in the above
scenario.
The correct answer is: Add mirtazapine

812
007
Which of the following types of drug molecules mimic endogenous chemical transmission?

Select one:
Antagonists
Agonists
Partial agonists
Non-competitive antagonists
Irreversible enzyme inducers

Most antagonist drugs are competitive in nature, are displaced from their binding site by
agonists so that at high doses the agonist can still exert maximum effect and cause a shift to
the right in the dose-response curve for the agonist. Agonists are drugs that mimic endogenous
transmission.A partial agonist can partially antagonise the effect of full agonists.
Buprenorphine, buspirone and aripiprazole are all partial agonists.
The correct answer is: Agonists

008
Disulfiram acts on

Select one:
Sigma receptors
Alcohol dehydrogenase

Glutamate dehydrogenase
Aldehyde dehydrogenase
Nicotinic cholinergic system

In usual doses, disulfiram acts via inhibition of acetaldehyde dehydrogenase that produces
deterrent effects when alcohol is consumed (flush reaction).
The correct answer is: Aldehyde dehydrogenase

009
Which of the following actions can cause anorgasmia?

813
Select one:
Beta agonism
GABA-A agonism
Alpha 1 agonism

Antihistaminergic property
Alpha 1 antagonism

Alpha 1 antagonism, 5HT2A/2C stimulation (delayed ejaculation in SSRIs) can cause


anorgasmia. Retrograde ejaculation due to Alpha 1 block, anticholinergic and antihistaminic
effects
The correct answer is: Alpha 1 antagonism

010
Which of the following has atypical antipsychotic activity?

Select one:
Fluphenazine
Promazine
Chlorpromazine
Amoxapine
Amisulpiride

Amisulpride has atypical antipsychotic activity as it produces very low EPSEs and is more
D2/3 selective compared to other typicals.But it does not have 5HT2A blockade like other
newer atypicals
The correct answer is: Amisulpiride

011
Drug that is comparatively safe as an adjunctive to MAOI in treating depression is

Select one:
Amitriptyline
Clomipramine
Fluoxetine

814
Lithium
Paroxetine

MAOIs potentiate the actions of TCAs, which may provide an explanation for the use of such a
combination in the treatment of therapy-resistant depression.There is evidence to suggest that a
combination of an MAOI with clomipramine is more likely to produce serious adverse effects
than occurs with other TCAs.
The correct answer is: Amitriptyline

012
Which of the following is a partial D2 agonist?

Select one:
Risperidone
Amisulpride
Pramipexole
Aripiprazole
Buspirone

Aripiprazole is a dopamine D2 receptor partial agonist with partial agonist activity at serotonin
5HT1A receptors and antagonist activity at 5HT2A receptors
The correct answer is: Aripiprazole

013
The therapeutic effects of conventional antipsychotics is mainly due to

Select one:
Blockade of dopamine-2 receptors in the mesocortical pathway
Blockade of alpha-1 receptors
Blockade of dopamine-2 receptors in the nigrostriatal pathway
Blockade of dopamine-2 receptors in the tuberoinfundibular pathway

Blockade of dopamine-2 receptors in the mesolimbic pathway

815
Blockade of dopamine-2 receptors in the mesolimbic pathway leads to the therapeutic effects.
Blockade of dopamine-2 receptors in the mesocortical pathway leads to production or
worsening of negative symptoms. Blockade of dopamine-2 receptors in the nigrostriatal pathway
leads to EPSEs. Blockade of dopamine-2 receptors in the tuberoinfundibular pathway leads
features of hyperprolactinaemia such as galactorrhoea, sexual dysfunction, gynaecomastia,
infertility, amenorrhoea, and possibly accelerated osteoporosis. Blockade of alpha-1 receptors
leads to sedation, dizziness, orthostatic hypotension.
The correct answer is: Blockade of dopamine-2 receptors in the mesolimbic pathway

014
Which of the following is a 5HT1A partial agonist?

Select one:
Buspirone
Pramipexole
Amisulpride
Risperidone
Aripiprazole

Buspirone is an azaspirodecanedione, a non-benzodiazepine anxiolytic. Buspirone's


mechanism of action is related to 5HT partial agonism.
The correct answer is: Buspirone

Q
015
Tyramine gains neuronal access and releases stored catecholamines via which of the
following routes?

Select one:
GABA
NMDA channel

Calcium channels
Sodium channels
Catecholamines reuptake channels

As with most indirect sympathomimetics, tyramine enters the presynaptic neuron through
amine uptake pumps - the channels that are blocked by antidepressants such as TCAs.

816
The correct answer is: Catecholamines reuptake channels

016
Which one among the following is a weak inhibitor of cytochrome P450?

Select one:
Both 1 and 2
Fluvoxamine
Citalopram
Paroxetine
Fluoxetine

Citalopram and Escitalopram do not produce cytochrome P 450 inhibition. Fluoxetine,


Fluvoxamine and Paroxetine are potent inhibitors of several hepatic cytochrome enzymes.
Sertraline is safer, while citalopram is the safest. Citalopram is a weak inhibitor of 2D6.
The correct answer is: Citalopram

017
Which one among the following has a stimulant effect?

Select one:
Dothiepin
Lofepramine

Imipramine
Amitryptyline
Desipramine

Desipramine has relative selectivity for noradrenaline re-uptake and has a stimulant effect
The correct answer is: Desipramine

018
A patient on mirtazapine treatment develops rapidly spreading bacterial infection. Which of
the following must be immediately ed?

Select one:

817
Liver function
Differential count
Renal function
CSF profile
Blood glucose

Severe neutropenia, defined as less than 0.5 x 10^9/L mature neutrophil cells, is a potentially
life-threatening side effect of mirtazapine, occurring with a risk of up to 1 in 1,000 instances
The correct answer is: Differential count

019
The chronic administration of tricyclics results in

Select one:
Upregulation of beta-adrenergic receptors
Down regulation of beta and alpha adrenergic receptors

Up-regulation of alpha-adrenergic receptors


Downregulation of alpha-adrenergic receptors

Downregulation of beta-adrenergic receptors

It has been consistently reported that chronic administration (2 to 4 weeks of treatment) of


antidepressant drugs causes a downregulation of beta receptors in rats. However, such effects
are not observed after acute or single administration of these drugs. Since most of the
antidepressant drugs tested thus far produce such effects, it has been suggested that
downregulation or decreased responsiveness of beta-adrenergic receptors may be related to
the therapeutic effects manifested by antidepressant drugs. This is yet to be clarified.
(Retrieved from http://onlinelibrary.wiley.com/doi/10.1002/ddr.430030502/abstract)
The correct answer is: Downregulation of beta-adrenergic receptors

020
A new drug A is a non-competitive antagonist at NMDA receptors. If availability of glutamate at
the receptor sites is increased experimentally after administering drug A, which of the following
can happen?

Select one:
Effect of drug A is partially reversed

818
Glutamate levels decrease
No change takes place
Effect of drug A is reversed fully

Effect of drug A increases

Noncompetitive antagonists alter the receptor site in some way so the effects can be reversed
only partially by increasing the dose of the agonist drug. Non-competitive antagonism reduces
both the potency and the efficacy of agonists.
The correct answer is: Effect of drug A is partially reversed

021
Endogenous substrates for monoamine oxidase (MAO) isoenzymes include

Select one:
Tyramine, acetylcholine, and dopamine.
Acetylcholine,dopamine,and epinephrine.
Epinephrine, dopamine, and serotonin.
Tyramine, serotonin, and histamine.
Histamine, acetylcholine, and serotonin.

Endogenous monoamines serotonin, dopamine and norepinephrine, are broken down by MAO
enzymes. MAO-A is more selective for serotonin while MAO-B is more so for dopamine.
The correct answer is: Epinephrine, dopamine, and serotonin.

022
Which of the following medications has been found to be effective in patients with premature
ejaculation?

Select one:
Flunarizine
Fluoxetine
Flupentixol
Fluphenazine

Trifluoperazine

819
Due to the inhibitory effects of serotonin on the central ejaculatory reflex, selective serotonin
reuptake inhibitors (SSRIs) are useful in the treatment of Premature Ejaculation. These
inhibitors have been shown to delay ejaculation in multiple placebo-controlled randomized
studies (Biri, et al., 1998; Haensel, Klem, Hop, Slob, 1998; Kara et al.,, 1996; Kim and Soo.
1998; Waldinger , Hengeveld, Zwinderman, Olivier, 1994, 1997, 1998) and is the most
common class of medications used to treat PE currently.
The correct answer is: Fluoxetine

023
The drug alprazolam acts on

Select one:
NMDA receptors
GABA
D1 receptors
Histaminergic receptors
M1 muscarinic receptors

Alprazolam is a benzodiazepine; it acts via GABA mechanisms.


The correct answer is: GABA

024
The mechanism of action for Sodium valproate

Select one:
Dopamine stabilisation
NMDA antagonism

Benzodiazepine antagonism
GABA potentiation
Membrane stabilising

820
Valproate acts at sodium channels, at several important steps in GABA metabolism leading to
GABA potentiation, and on the activity of histone deacetylase (which influences gene
transcription).
The correct answer is: GABA potentiation

025
The pharmacological activity of benzodiazepines depends on its action on which of the
following receptors?

Select one:
Beta Adrenergic receptor complex
GABA-a and GABA-b receptor complex
5HT1-A receptor complex
GABA-b receptor complex
GABA-a receptor complex

Benzodiazepines act on GABA-a receptor complex and potentiates GABAergic function.


The correct answer is: GABA-a receptor complex

026
Which one of the following has antiepileptic activity?

Select one:
Fluoxetine
Buspirone
Buspirone
Bupropion
Gabapentin

Gabapentin has antiepileptic property.


The correct answer is: Gabapentin

027
Which of the following antidementia drugs acts directly on nicotine receptors?

821
Select one:
Donepezil
Tacrine
Rivastigmine

Gallantamine
Memantine

Galantamine, unlike the other anticholinesterases in clinical use, is derived from the alkaloids
from the daffodil and snowdrop family. It is a reversible, competitive inhibitor of
acetylcholinesterase with some inhibitory action on butyrylcholinesterase. It is also an agonist at
nicotinic receptor sites. Although a clinically effective drug, galantamine frequently causes
gastrointestinal side effects.
The correct answer is: Gallantamine

028
Mechanism of action of acamprosate is

Select one:
Opioid anatagonism
Glutamate antagonism
Inhibits alcohol dehydrogenase
Opioid agonism
Inhibits aldehyde dehydrogenase

Acamprosate's mechanism is not well understood but is thought to involve interference with
excitatory amino acids such as glutamate that may be involved in alcoholic relapse.
The correct answer is: Glutamate antagonism

029
The mechanism of action of cyproheptadine is

Select one:
Histamine H1 receptor blockade
Serotonin 5HT3 receptor blockade
Serotonin 5HT2 receptor stimulation

822
Histamine H2 receptor blockade
Histamine H1 receptor stimulation

Anorgasmia related to SSRI use may be treated by cyproheptadine one to two hours before
sexual activity.Cyproheptadine is a sedating antihistamine with antimuscarinic,5HT2A
serotonin-antagonist and calcium-channel blocking properties. It competes for H1-receptor
sites on effector cells in the GIT, blood vessels and respiratory tract.
The correct answer is: Histamine H1 receptor blockade

030
The mechanism of action of barbiturates is by

Select one:
Increasing the frequency of chloride channels opening
Increasing the duration of chloride channel opening
Closing down the potassium channels
Increasing the number of chloride channels

Closing down the sodium channels

Barbiturates act on GABA receptors and serve to increase the duration of chloride channel
opening.
The correct answer is: Increasing the duration of chloride channel opening

031
Which of the following statements best describes the receptor profile of trazodone?

Select one:
It is a serotonin antagonist
It is a mixed serotonin antagonist/agonist.
It is a serotonin, dopamine and norepinephrine antagonist
It is a serotonin, dopamine and norepinephrine agonist

It is a serotonin agonist

823
Trazodone is a multifunctional drug: it is a hypnotic at low doses and an antidepressant at high
doses due to the manner in which different multifunctional binding properties apply at different
doses. Trazodone's most potent binding property is 5-HT2A antagonism. Its ability to block
Serotonin reuptake transporters (SERT) is 100 fold less potent than its ability to block 5- HT2A
receptors. Trazadone is categorized as 'serotonin antagonist-reuptake inhibitor'. At lower doses,
it acts as 5HT2A antagonist; though at higher doses it acts as SERT blocker. (Retrieved from
CNS Spectrums: Mechanism of Action of Trazodone.
http://www.cnsspectrums.com/aspx/articledetail.aspx?articleid=2373 (accessed April 11,
2015)).
The correct answer is: It is a mixed serotonin antagonist/agonist.

032
Which of the following street drugs is called a dissociative anaesthetic?

Select one:
Ketamine
Cannabis
Heroin
LSD
Cocaine

Ketamine is used as a veterinary anesthesia and produces a state of dissociative anaesthesia


(not same as psychological dissociation).
The correct answer is: Ketamine

033
The enzyme/s that metabolise/s most of the body dopamine:

Select one:
MAO-A and COMT
COMT only

MAO-A and MAO-B


MAO-B and COMT
MAO-A and Tyrosine hydroxylase

824
Dopamine is metabolized in the liver, kidney, and plasma by MAO and COMT. Therefore,
inhibition of one or both of these enzymes would decrease dopamine metabolism and result in
increased levels of dopamine in the brain
The correct answer is: MAO-B and COMT

034
Excitotoxicity that is secondary to glutamatergic overstimulation results in neuronal damage.
This is the basis of treating patients with neurodegenerative disorders with which of the
following medications?

Select one:
Selegiline
Donepezil
Risperidone
Galantamine
Memantine

The hypothesis that glutamate-mediated neurotoxicity is involved in the pathogenesis of AD is


increasingly being favoured; Glutamate receptors are overactive, and N­methyl­D­aspartate
(NMDA) receptor antagonists have therapeutic potential for the treatment of AD and other
neurological disorders. Memantine is a noncompetitive NMDA antagonist that is considered a
neuroprotective drug
The correct answer is: Memantine

035
Which drug acts as a full agonist at Mu receptor and has a long half-life?

Select one:
Acamprosate
Buspirone

Methadone
Buprenorphine
Disulfiram

Methadone is a full mu- opioid agonist. Its half-life is approximately 24 hours with regular use.
The correct answer is: Methadone

825
036
Which one of the following has been postulated as the pharmacological basis of clozapine
related hypersalivation?

Select one:
Reduced cholinesterase activity
Activation of the swallowing reflex.
Muscarinic M-4 antagonism
Adrenergic alpha-2 stimulation
Muscarinic M-4 agonism

Clozapine is well known to be associated with hypersalivation (drooling particularly at night).


This seems to be problematic in the early stages of treatment and is probably dose-related. M4
agonism may be an underlying mechanism though this is not clear. Loss of swallowing reflex
may also contribute.
The correct answer is: Muscarinic M-4 agonism

037
Which one among the following is the least sedative of tricyclic antidepressants?

Select one:
Dothiepin
Nortriptyline
Amitryptyline
Lofepramine
Imipramine

Amitryptyline and dothiepin are more sedative, with trimipramine being the most sedative,
Imipramine and Lofepramine are less sedative, and Nortriptyline is the least sedative.
The correct answer is: Nortriptyline

038
The mechanism of action of varenicline tartrate is

826
Select one:
Nicotinic antagonist
Partial muscarinic agonism
Inhibition of nitric oxide synthesis

Phosphodiesterase inhibition
Partial nicotinic agonism

Varenicline is an a4ß2 nicotinic acetylcholine receptor partial agonist that relieves symptoms of
nicotine withdrawal and cigarette craving through its agonist actions while blocking the
reinforcing effects of continued nicotine use through an antagonist action.
The correct answer is: Partial nicotinic agonism

039
The endocrine effects of neuroleptics would include all except

Select one:
Gynaecomastia
Primary amenorrhoea
Weight gain
Lactorrhoea
Impotence

The endocrine effects are probably largely due to elevated prolactin, whose release is
normally inhibited by dopamine. The endocrine effects would include secondary
amenorrhoea and not primary amenorrhoea.
The correct answer is: Primary amenorrhoea

040
A patient with Parkinson's disease experiences visual hallucinations on levodopa/carbidopa
therapy. The most appropriate intervention would be to initiate treatment with which of the
following medications?

Select one:
Aripiprazole
Amisulpride

827
Olanzapine
Risperidone
Quetiapine

Atypical antipsychotic, especially quetiapine, have a reduced likelihood of causing adverse


drug-induced parkinsonism and therefore a possible role in treating psychotic symptoms in
patients with Parkinson's, including the hallucinations induced by levodopa/carbidopa.
The correct answer is: Quetiapine

041
Poor response to lithium prophylaxis is predicted by

Select one:
classic mania
family loading of bipolar spectrum disorders
rapid cycling
secondary mani

obsessional symptoms

The phenotype of lithium responders tend to differ from that of other mood stabilisers. Patients
with episodic pattern, no comorbidity (especially no panic attacks and substance abuse),
significantly higher familial risk of bipolar disorder and absence of rapid cycling pattern are some
of the indicators for therapeutic response to lithium.
The correct answer is: rapid cycling

042
Which of the following is a noradrenaline reuptake inhibitor?

Select one:
Risperidone
Reboxetine
Agomelatine
Fluoxetine
Paroxetine

828
Reboxetine, the first selective norepinephrine reuptake inhibitor used in the treatment of
depression, mainly acts by binding to the norepinephrine transporter and blocking the
reuptake of extracellular norepinephrine
The correct answer is: Reboxetine

043
Which of the following has no evidence-based support when prescribing antidementia drug
donepezil? It

Select one:
Reduces caregiver burden
Improves the amount of daily activities in some patients
Improves neuropsychiatric symptoms
Reduces the progression of cognitive decline
Reduces time to institutionalisation

There is strong evidence that donepezil has efficacy against the three major domains of
Alzheimer's disease symptoms, namely functional ability, behavior, and cognition. The strongest
evidence is for improvement or less deterioration in global outcomes and cognition in the short
to medium term. There is limited evidence that improved global outcomes are maintained in the
long term and clear evidence to support long-term maintenance of cognitive benefits. Also,
donepezil appears to maintain function in the long term, and there is some level 1 and two
evidence of improved or limited deterioration in behavior or mood in the short to medium term.
Despite donepezil's effects on major symptoms of Alzheimer's disease, its impact on patients'
quality of life has not been consistently demonstrated, perhaps reflecting the difficulty of
assessing this aspect in this patient population. Donepezil may also lessen caregiver burden.
There is limited and conflicting weak evidence that long-term donepezil treatment delays time to
institutionalization. Birks JS, Harvey R. Donepezil for dementia due to Alzheimer's disease.
Cochrane Database Syst Rev. 2003;(3): CD001190.
The correct answer is: Reduces time to institutionalisation

044
Which one among the following has high D2/high 5HT2 activity?

Select one:
Clozapine
Quetiapine
Risperidone

829
Haloperidol
Amisulpride

Haloperidol has a High D2/low 5HT2 activity. High D2/high 5HT2 activity is characteristic of
atypicals such as olanzapine, risperidone and also loxapine. Low D2/high 5HT2 is specific to
clozapine while some argue that quetiapine has a lowD2/low 5HT2 activity.
The correct answer is: Risperidone

045
Which of the following groups of medications are associated with bruxism more often than the
others listed?

Select one:
Stimulants
Antidementia drugs
Antidepressants
Antipsychotics
Benzodiazepines

Teeth grinding in children is otherwise known as bruxism. It is associated with ADHD


especially when stimulants are sued for its treatment.
The correct answer is: Stimulants

046
Which one of the following is a pure D2 antagonist?

Select one:
Aripiprazole
Clozapine

Risperidone
Amisulpride
Sulpiride

830
Sulpiride is a pure D2 antagonist. Whereas, Amisulpride is a D2/D3 antagonist. At low doses
blocks autoreceptors thus increasing the synaptic dopamine levels and thought to be how it
improves negative symptoms. In the limbic region, blockade of postsynaptic D2/D3 receptors
reduces positive symptoms.
The correct answer is: Sulpiride

047
Which of the following increases the risk of withdrawal reactions to therapeutic drugs?

Select one:
The drug having anticholinergic properties
Taking the drug after food
Having multiple active metabolites with slow clearance
Slow withdrawal of the drug
Drugs being a partial agonist instead of full agonist

Paroxetine has anticholinergic properties; withdrawal causes cholinergic rebound symptoms.


The correct answer is: The drug having anticholinergic properties

048
Which of the following drugs is not suitable for treating acute mania?

Select one:
Topiramate
Lithium

Carbamazapine
Olanzapine
Valproate

Though initial data from open trials suggested that topiramate may possess antimanic
properties; this has not been borne out by later more rigorous trials.
The correct answer is: Topiramate

049

831
Receptor blockade in which dopaminergic pathway results in increased prolactin levels in the
body?

Select one:
Mesocortical pathway
Nigrostriatal pathway
Mesolimbic pathway

Ultra short ganglionic pathway


Tuburoinfundibular pathway.

Blockade of dopamine-2 receptors in the mesolimbic pathway leads to the therapeutic effects.
Blockade of dopamine-2 receptors in the mesocortical pathway leads to production or
worsening of negative symptoms. Blockade of dopamine-2 receptors in the nigrostriatal pathway
leads to EPSEs. Blockade of dopamine-2 receptors in the tuberoinfundibular pathway leads
features of hyperprolactinaemia such as galactorrhoea, sexual dysfunction, gynaecomastia,
infertility, amenorrhoea, and possibly accelerated osteoporosis.
The correct answer is: Tuburoinfundibular pathway.

050
Drug X produces improvement in depression when given at a dose of 10mg. Drug Y which acts
via the same mechanism as X produces the same improvement when given at a dose of 20mg.
When X and Y are given together Y binds to more receptors in a quicker and stronger fashion
than X. Which of the following is true?

Select one:
Y has lesser affinity then X
X is less potent than Y
X and Y are equally efficacious
X and Y have equal potency but different affinity
X and Y have equal affinity but different potency

Efficacy refers to how well the drug produces the expected response i.e. the maximum clinical
response produced by a drug (‘productivity'). Efficacy depends on affinity, potency, duration of
receptor action in some cases and kinetic properties such as half life, among other factors
The correct answer is: X and Y are equally efficacious

832
833
Home HiYield Paper Pharmacology

HiYield Paper A(2)

001
What is the risk of developing fatal agranulocytosis in patients treated with clozapine in the
UK?

Select one:
1 in 10000

1 in 1300

1 in 4500

1 in 5000

1 in 1000

Fatal agranulocytosis­ 1 in 5000 patients; fatal pulmonary embolism­ 1 in 4500 patients


treated; fatal myocarditis or fatal cardiomyopathy­1 in 1300 patients treated.
The correct answer is: 1 in 5000

002
Which one among the following statements about neuroleptic malignant syndrome (NMS) is
false?.

Select one:
Patients on Tricyclic antidepressants can develop NMS

2-3% of all patients treated with conventional antipsychotics develop NMS

It lasts for 10-14 days if left untreated.

Fluctuating blood pressure is a feature of NMS

834
It evolves rapidly over 24-72 hours

The risk is estimated to be less than 0.5-1%. It evolves rapidly over 24-72 hours and lasts for
10-14 days if left untreated. NMS is also seen very rarely with other drugs such as antipsychotics
and lithium. The important clinical features of NMS would include Fluctuating consciousness,
Fluctuating blood pressure, tachycardia, fever, confusion, diaphoresis, rigidity. Signs of NMS
would include elevated creatinine kinase, leucocytosis, and altered liver function tests.

The correct answer is: 2-3% of all patients treated with conventional antipsychotics develop
NMS

003
What percentage of patients develop Tardive Dyskinesia with every year of typical
antipsychotic exposure?

Select one:
More than 50%

2-5%

5-10%

20-25%

10-20%

TD­2­5% ; Dystonia­2­10%; Pseudo­parkinsonism­20%; Akathisia­ 25%


The correct answer is: 2-5%

004
The risk of Ebstein's anomaly is increased by how many times in children of mothers taking
lithium while pregnant?

Select one:
20 times

10 times

1000 times

2 times

100 times

835
The most common teratogenic effect of lithium involves cardiac valves especially Ebstein's
anomaly of the tricuspid valves. The risk of Ebstein's malformation in lithium-exposed fetuses is
1 of 1,000 (20 times the risk in the general population).
The correct answer is: 20 times

005
Which of the following statements concerning CATIE is incorrect?

Select one:
First-generation drugs do as well as second generation drugs

Haloperidol was not included in the trial

Patients taking perphenazine showed more EPSEs

Quetiapine group had less EPSEs

25% of patients taking olanzapine discontinued the drug

In the CATIE trial, irrespective of the prescribed drug - 74% discontinued treatment in 18
months (surprisingly high despite naturalistic design). Median time to discontinue was 4.6
months. Olanzapine had lowest discontinuation rate (still 64%) - but highest side effect
burden. 64% discontinued olanzapine; 75%, perphenazine; 82%, quetiapine; 74%,
risperidone; and 79%, ziprasidone.
The correct answer is: 25% of patients taking olanzapine discontinued the drug

006
The patients who are prescribed clozapine or olanzapine should have their serum lipids
measured every

Select one:
6 days whilst on treatment

One year whilst on treatment

3 months for the first year of treatment

6 weeks for the first year of treatment

6 months for the first year of treatment

836
837
Clozapine, Olanzapine, quetiapine and phenothiazine-suggested monitoring- fasting lipids
and cholesterol at baseline then every 3 months for a year, then annually according to
national guidelines. Other antipsychotics-fasting lipids and cholesterol at baseline and 3
months and then annually.
The correct answer is: 3 months for the first year of treatment

007
The upper limit of safe QTc interval for men is

Select one:
440 msec

470 msec

500 msec

490 msec

410 msec

The safe QTc interval for men is 440 msec and for women is 470 msec. If QTc is greater than
440 ms (men) or 470 ms (women) when taking antipsychotics, consider a switch to drug of
lower effect; reperform ECG and consider referral to cardiologist. If the Qtc is greater than
500ms in both genders, immediately stop the suspected causative drugs and refer to a
cardiologist.
The correct answer is: 440 msec

008
The sexual side effects caused by SSRI antidepressants are related to the consequence of
stimulating which of the following receptors?

Select one:
5HT-3 receptors

5HT-1 and 2 receptors

5HT-1 receptors

5HT-2 receptors

5HT-2 and 3 receptors

838
Clinically significant consequences of stimulating 5HT1-a receptors leads to antidepressant
actions, anti obsessive-compulsive disorder properties, anti-panic and anti-phobic properties and
anti-bulimic properties. Clinically significant consequences of stimulating 5HT2 receptors includes
agitation, akathisia, anxiety, panic attacks, insomnia and sexual dysfunction.
The correct answer is: 5HT-2 receptors

009
A patient with a known diagnosis of depression does not want to get obese. Drugs acting on
which of the following receptors should be avoided?

Select one:
5HT-2A

5HT3

Dopamine D2 receptors

Alpha-adrenergic

5HT-2C

The mechanism by which weight gain occurs during treatment with psychotropics is poorly
understood, but the broader receptor affinities of the agents and their antagonism of histamine
H1 and serotonin 5-HT2C receptors have been implicated.
The correct answer is: 5HT-2C

010
Weight gain as a side effect of antipsychotics is caused due to all of the following mechanisms
except

Select one:
H-1 antagonism

Hyperprolactinaemia

Increased serum leptin leading to leptin desensitisation.

5HT2-a antagonism

5HT2-c antagonism

The suggested mechanisms for weight gain as a side effect of antipsychotics include 5HT2-c
antagonism, H-1 antagonism, hyperprolactinaemia and increased serum leptin as a result of

839
secondary leptin desensitisation.

840
The correct answer is: 5HT2-a antagonism

011
In treating serotonin syndrome which of the following receptor antagonism is useful for
controlling neurological signs?

Select one:
Alpha

H1

5HT2A

5HT7

5HT3

Any agent that increases serotonergic function can put patients at risk of a central
serotonergic syndrome. This syndrome most commonly affects patients on multiple
serotonergic drugs and involves multiple systems. The CNS effects are thought to be related
to 5HT-2A stimulation.
The correct answer is: 5HT2A

012
A patient presents to you on the ward with a 'muscle spasm' on his neck. He was started on
5mg Haloperidol (bd) few days ago. What is the most likely problem?

Select one:
Neuroleptic malignant syndrome

Acute dystonia

Parkinsonism

Akathisia

Tardive dyskinesia

An acute dystonic reaction consists of sustained, often painful muscular spasms, producing
twisting abnormal postures. 50% occur within 48 hours of administering an antipsychotic for the
first time.
The correct answer is: Acute dystonia

841
013
Which of the following is not a dose-dependent effect when using clozapine?

Select one:
Agranulocytosis

Seizures

Sedation

Weight gain

Hypersalivation

Agranulocytosis is idiosyncratic and can occur at any dose; unlike sedation, seizures and to
some extent weight gain, it is not dose dependent. Note that weight gain may also be
relatively dose-independent in a number of patients.
The correct answer is: Agranulocytosis

014
A 45-year-old gentleman stated on risperidone complained of feeling anxious and was
constantly found crossing/uncrossing legs on the ward. What is the most likely cause among
the following?

Select one:
Akathisia

Dystonia

Pseudoparkinsonism

Tardive dystonia

Tardive dyskinesia

Akathisia is characterised by a subjectively unpleasant state of inner restlessness when there is


a strong desire or compulsion to move. e.g., foot stamping when seated, constantly
crossing/uncrossing legs, rocking from foot to foot and constantly pacing up and down.
The correct answer is: Akathisia

015
Postural hypotension as a side effect of tricyclic antidepressants are related to

Select one:

842
Alpha-adrenergic blockade

5-HT2 Blockade

Anticholinergic effect

Antihistaminergic effect

Beta blockade

Postural hypotension, dizziness, tachycardia and arrhythmias are caused due to alpha-
adrenergic blockade. 5-HT2c Blockade is related to weight gain.
The correct answer is: Alpha-adrenergic blockade

016
Which one among the following benzodiazepines is more toxic than others in overdose?

Select one:
Nitrazepam

Alprazolam

Lorazepam

Temazepam

Diazepam

Toxic dose of benzodiazepines is probably more than 100 mg diazepam equivalents. Fatality is
unusual if taken alone. Alprazolam is the most toxic agent. Isbister GK et al. Alprazolam is
relatively more toxic than other benzodiazepines in overdose. Br J Clin Pharmacol
2004;58:88­95.
The correct answer is: Alprazolam

017
Which of the following diuretics can be used to control lithium-induced polyuria without
causing lithium toxicity?

Select one:
Amiloride
Caffein Frusemide

843
Ethacrynic acid

Thiazide

One of the major side effects of lithium is nephrogenic diabetes insipidus. The established
treatment for the disorder is thiazide diuretics, but these are associated with hypokalemia and
reduced lithium excretion, predisposing the patient to lithium toxicity. Amiloride is a
potassium-sparing diuretic that reduces lithium-induced polyuria without affecting lithium or
potassium levels. ( Treatment of severe lithium-induced polyuria with amiloride. Retrieved from
http://www.ncbi.nlm.nih.gov/pubmed/3538913)
The correct answer is: Amiloride

018
A reduction in akathisia symptoms may not be seen on treatment with

Select one:
Propranolol

Cyproheptadine

Anticholinergics

Clonazepam (low dose)

Trazadone

A reduction in akathisic symptoms may be seen with Propranolol, Clonazepam (low dose),
Cyproheptadine, Trazadone, and Mianserin. Anticholinergics are generally unhelpful.
The correct answer is: Anticholinergics

019
A 73-year-old lady was admitted to the medical ward with a history of increasing confusion,
lethargy and dizziness over the last three days. She is on Fluoxetine 30 mg, and her other
medications would include Furosemide, Aspirin and Ramipril. She has a history of chronic renal
failure. Her vital signs were within normal limits. What do you think would be the most likely
diagnosis?

Select one:
Serotonin syndrome
Tyramine Reactio
Acute Dystonic reaction

844
Neuroleptic malignant syndrome

Antidepressant-induced hyponatraemia

Most antidepressants have been associated with hyponatraemia. Serotonergic


antidepressant drugs are relatively more likely to cause hyponatraemia. Signs of
hyponatraemia would include nausea, lethargy, dizziness, confusion, seizures, cramps and
coma.
The correct answer is: Antidepressant-induced hyponatraemia

020
What percentage of patients with Tardive Dyskinesia may show recovery within a year with
antipsychotic reduction?

Select one:
Approximately 1-2%

Approximately 20-25%

Approximately 2-4%

Approximately 50-55%

Approximately 10-20%

Approximately 50% of cases with TD are reversible. As a first line intervention, stop
anticholinergics if these were prescribed previously. Reduce the dose of antipsychotic and
consider changing to an atypical drug. Clozapine is the antipsychotic most associated with
resolution of symptoms.
The correct answer is: Approximately 50-55%

021
Which one among the following is associated with a lower risk of weight gain than the others?

Select one:
Aripiprazole

Clozapine

Quetiapine

Chlorpromazine

Olanzapine

845
Ziprasidone has been associated with minimal weight gain, which could distinguish it among
other second-generation antipsychotics. Similarly, aripiprazole appears to cause little or no
weight gain. During long-term treatment, clozapine and olanzapine have the largest effects on
weight gain; risperidone produces intermediate weight gain. Weight gain does not appear to be
dose-dependent, tends to plateau between 6 and 12 months after initiation of treatment, and is
mainly due to an increase in body fat. Conventional agents such as chlorpromazine can cause
as much weight gain as atypicals.
The correct answer is: Aripiprazole

022
Floppy baby syndrome is a recognised complication associated with the use of which of the
following psychotropic drugs in pregnancy?

Select one:
Antipsychotics

Anti-dementia drugs

Methadone

Antidepressants

Benzodiazepines

First- trimester exposure to benzodiazepines appears to be associated with an increased risk of


oral clefts in newborns while late third-trimester use is commonly associated with the floppy baby
syndrome.
The correct answer is: Benzodiazepines

023
A specific side effect of mianserin that requires regular monitoring is

Select one:
anticholinergic activity

Risk of serotonin syndrome

cardiotoxicity
QT prolongation
Bone marrow suppression

846
Mianserin was the first of the second-generation antidepressants to be developed. It lacked
the amine reuptake inhibitory and MAOI actions of the first-generation drugs and also lacked
the cardiotoxicity and anticholinergic activity of the TCAs. However, it was sedative
(antihistaminic), caused postural hypotension (alpha-1 blockade) and also caused blood
dyscrasias and agranulocytosis in a small number of patients. This has limited the use of
mianserin in recent years.
The correct answer is: Bone marrow suppression

024
The principal cause of death following tricyclic antidepressant (TCA) overdose is

Select one:
cerebral oedema

seizures

malignant hypertension

Cardiac arrhythmia.

respiratory failure

A membrane stabilizing effect can precipitate dysrhythmias and altered myocardial conduction.
This activity occurs from blockade of inward/fast sodium channels, resulting in altered
repolarization and conduction. This effect occurs distal to the AV node, producing a depression
of the His-Purkinje conduction system and a direct negative inotropic effect. Intravenous sodium
bicarbonate is the preferred pharmacologic treatment of choice to reverse TCA-induced cardiac
conduction defects. This is the commonest cause of death in an overdose with TCAs.

The correct answer is: Cardiac arrhythmia.

025
A patient recently started on clozapine developed chest pain dizziness, vomiting, fever and
some pedal oedema. He is short of breath, and ECG is abnormal. What is the likely side
effect?

Select one:
Pneumonic consolidation

Cardiomyopathy

Agranulocytosis Ventricular tachycardia

847
Myocardial infarction

Clozapine therapy is known to be associated with potentially fatal myocarditis and


cardiomyopathy in physically healthy young adults with schizophrenia. The clinical features
includes asymptomatic patients who may have electro­cardiographic abnormalities; some
patients may have signs and symptoms of clinical heart failure and ventricular dilatation. Non-
specific flu-like symptoms, tachycardia, elevated cardiac enzymes are the key features. ESR
and eosinophils may be higher.
The correct answer is: Cardiomyopathy

026
Patients on tranylcypromine should not take an excess of

Select one:
Cheese

Swede

Potatoes

Banana

Salt

Cheeses, particularly any type of ripe cheese, Broad beans (fava beans) Sausages, Salami,
Pickled fish, etc must be avoided due to the fear of tyramine induced crisis. It appears that
foods containing more than 10mg of tyramine must be consumed in order to produce a
significant rise in blood pressure.
The correct answer is: Cheese

027
Which one of the following SSRIs is relatively unsafe for post-MI patients?

Select one:
Paroxetine

Citalopram

Fluoxetine
Fluvoxamine
Sertraline

848
In normal doses, citalopram has no effect on QTc, but one if its metabolite may prolong QTc
interval. Sertraline is an SSRI with some safety data in post-MI patients (SADHART trial).
The correct answer is: Citalopram

028
Which one among the following drug is not advocated for the treatment of clozapine-induced
hypersalivation?

Select one:
Amitriptyline

Hyoscine

Benzhexol (trihexyphenidyl)

Pirenzepine

Clomipramine

Extensive clinical experience suggests that Pirenzepine is effective for the treatment of
Clozapine induced hypersalivation. Pirenzepine is a selective M1, M4 antagonist. Other drugs of
use include benzhexol (trihexyphenidyl), hyoscine, Amitriptyline, and combination of benztropine
and terazosin.
The correct answer is: Clomipramine

029
Which of the following is NOT a common side effect of rivastigmine?

Select one:
Drowsiness

Nausea

Constipation

Vomiting

Weight loss

849
Rivastigmine have experienced nausea, vomiting, diarrhea, high blood pressure and
hallucinations but constipation is very rare.
The correct answer is: Constipation

030
Which of the following is the most common adverse effect of valproate?

Select one:
Ataxia

Persistent elevation of hepatic transaminases

Reversible thrombocytopenia

Diarrhea

Hair loss

Nausea, vomitting, dyspepsia and diarrhea are the most common adverse effects of valproate.
The correct answer is: Diarrhea

031
A trainee doctor working at a medical ward asks your advice while on call. She is requesting an
assessment for an agitated and hallucinating inpatient on the medical ward. She reports that
he appears flushed with a hot and dry skin, mydriasis, rapid pulse and diminished bowel
sounds. Which of the following is the best suggestion for you to make at this stage?

Select one:
Administer atropine

Prescribe IM olanzapine

Discontinue anticholinergic drugs

Prescribe diazepam

Prescribe haloperidol.

Anticholinergic delirium constitutes a medical emergency. Symptoms of anticholinergic delirium


include hot, dry skin; dry mucous membranes; dilated pupils; absent bowel sounds; and
tachycardia. Physicians must first determine and remove the offending agent because patients
are at a high risk for a cholinergic crisis. Atropine can be used to treat anticholinergic delirium
symptoms once the agent has been removed.

850
The correct answer is: Discontinue anticholinergic drugs

032
A 36-year-old woman with schizoaffective disorder, living in a psychiatric rehabilitation unit is
admitted to a medical ward for treatment of a suspected intestinal infection resulting in
dehydration, vomiting and fever. Her psychiatric condition has been managed with a stable
dose of clozapine for the last nine months. At the medical ward, she appears confused with
reduced orientation to time and appears lethargic. She also has stiffness and some rigidity in
movements. The most appropriate recommendation at this stage would be to

Select one:
Add a benzodiazepine

Add an antiepileptic

Switch to a typical antipsychotic

Add benztropine

Discontinue clozapine

Atypical NMS can present in this manner.


The correct answer is: Discontinue clozapine

033
A 34-year-old man is being treated with carbamazepine and levothyroxine for a rapid cycling
bipolar disorder. As his illness was poorly controlled, clozapine was added after which his
condition has stabilized. The patient's most recent white blood cell count is below 3,000/cu
mm. The most appropriate immediate intervention is

Select one:
discontinue the carbamazepine

discontinue the thyroxine

.decrease the dose of clozapine

discontinue the clozapine.

decrease the dose of carbamazepine

If there is a red alert when taking clozapine, it must be immediately discontinued.


The correct answer is: discontinue the clozapine.

851
034
Which of the following tricyclics is most toxic in overdoses?

Select one:
Nortriptyline

Imipramine

Clomipramine

Dosulepin

Lofepramine

Dosulepin and amitriptyline are the most toxic (seizures and cardiac arrhythmia) tricyclics.
The correct answer is: Dosulepin

035
The tricyclic with highest antihistaminic activity is

Select one:
Amitriptyline

Imipramine

Amoxapine

clomipramine

Doxepine

has a lower risk of orthostatic hypotension but has a higher antihistaminergic effect.
The correct answer is: Doxepine

036
The anticholinergics should be prescribed for the treatment of patients with which of the
following effects of neuroleptic use?

Select one:
Pseudoparkinsonism
Dystonia Tardive dystonia

852
Tardive dyskinesia

Akathisia

While dystonia generally responds well to anticholinergics, other EPSEs do not. Parkinsonian
rigidity but not tremor may be reduced with anticholinergic use.
The correct answer is: Dystonia

037
Which of the following is true concerning the side effects of escitalopram when compared to
citalopram?

Select one:
Escitalopram is associated with greater gastrointestinal effects.

Escitalopram is associated with greater weight gain.

Escitalopram is associated with fewer sexual side effects

Escitalopram and citalopram have similar side-effect profiles

Escitalopram is associated with greater cardiovascular effects

Citalopram is a racemate consisting of a 1:1 mixture of the R(-)- and S(+)-enantiomers. Non-
clinical studies show that the serotonin reuptake inhibitory activity of citalopram is attributable to
the S-enantiomer, escitalopram. Escitalopram is the active S- isomer of the antidepressant
citalopram. It is claimed that escitalopram has more efficacy and a faster onset of effect than
citalopram. The side effect profile of escitalopram is similar to that of other SSRIs. (Retrieved
from http://www.ncbi.nlm.nih.gov/pubmed/15160261)
The correct answer is: Escitalopram and citalopram have similar side-effect profiles

038
Which one of the following is a common and early side effect of lithium?

Select one:
Hypothyroidism

Weight gain

Fine tremor

Coarse Tremor

Nephrogenic diabetes insipidus

853
Lithium has been shown to be effective in the treatment of bipolar affective disorder where it
reduces both the number and severity of relapses. The mechanism of action of lithium remains
unclear but appears to reduce the neurotransmitter-induced activation of second messenger
systems. The effect may be via G-proteins and can affect adenylate cyclase and
phosphatidylinositol cycle. Early and common side effects include fine tremors, polyuria,
polydipsia, nausea, and bad metallic taste and transient raised leucocyte and platelet count.
Propranolol can be useful in the treatment of lithium-induced tremors.
The correct answer is: Fine tremor

039
Agranulocytosis as a side effect of Clozapine occurs most commonly during the

Select one:
First 48 weeks of treatment

First 8 days of treatment

First 18 weeks of treatment

First 8 weeks of treatment

First 28 weeks of treatment

80% of the cases develop during the first 18 weeks of treatment but may occur at any time.
The correct answer is: First 18 weeks of treatment

040
Which one among the following SSRIs is present in high concentrations in breast milk?

Select one:
Sertraline

Paroxetine

Fluvoxamine

Fluoxetine

Citalopram

854
855
Antidepressants are excreted in varying degrees in breast milk, so breastfeeding in
antidepressant- treated women should be done with caution. Fluoxetine's long half-life and
potential for accumulation in breast milk have prompted some recommendations to avoid its
use in women who are breast-feeding young infants. (Retrieved from
http://www.aafp.org/afp/2001/0701/p119.html (accessed April 11, 2015).)
The correct answer is: Fluoxetine

041
Which of the following is an important difference between NMS and serotonin syndrome
clinically?

Select one:
Presence of myoclonus in NMS

Subacute onset in serotonin syndrome

Hyperreflexia in serotonin syndrome

CPK elevation in serotonin syndrome

Hypomanic presentation in NMS

Symptoms such as hyperreflexia and myoclonus are attributed to the enhanced release of
serotonin in serotonin syndrome and are not seen in NMS.
The correct answer is: Hyperreflexia in serotonin syndrome

042
Choose one side effect NOT seen with tricyclics commonly

Select one:
Sedation

QT prolongation

Delirium on overdose

Weight gain

Hypothyroidism

The most frequent and treatment-limiting side effect of TCAs is orthostatic hypotension, followed
by sedation, dry mouth and constipation. Hypothyroidism is not a side effect of TCAs.
The correct answer is: Hypothyroidism

856
043
Commonest side effect of methylphenidate is

Select one:
Tics

Psychosis

Insomnia

Diarrhoea

Headaches

The most common side effects with methylphenidate are nervousness, agitation, anxiety, and
insomnia.
The correct answer is: Insomnia

044
A 27-year-old man with bipolar depression was started on one of the mood stabilisers. He
developed a rash immediately, which subsequently which progressed to causing Steven
Johnson syndrome. Which one of the following is most likely to be the mood stabiliser that
was prescribed to this man?

Select one:
Topiramate

Lamotrigine

Valproate

Zonisamide

Lithium

Lamotrigine stabilises neuronal membranes and reduces the release of excitatory amino acids
(especially glutamate) by blocking voltage dependent sodium channels. Lamotrigine is
associated with Stevens-Johnson syndrome in 1% of the population with the risk being greatest
in the first 8 weeks. The risk is reduced with slow dose titration and special dosing guidelines
especially when valproate is used concomitantly as is often the case in psychiatric usage.

The correct answer is: Lamotrigine

857
Choose an antibiotic that can cause serotonin syndrome if combined with MAOIs:

Select one:
Ciprofloxacin

Linezolid

Amoxicillin

Erythromycin

Trimethoprim

Linezolid, an antibiotic of the oxazolidone family, is a reversible, nonselective MAOI used in


the treatment of methicillin-resistant Staphylococcus aureus. Case reports of serotonin
syndrome resulting from the interaction of linezolid and the SSRIs citalopram, sertraline, and
paroxetine have been published.
The correct answer is: Linezolid

046
Which of the following psychotropics is relatively contraindicated when ACE inhibitors are
prescribed?

Select one:
Lithium

Olanzapine

Lorazepam

Valproate sodium

Risperidone

A number of drugs may interact with ACE inhibitors. In particular, non-steroidal anti-
inflammatory drugs (NSAIDs), diuretics and lithium (leads to increased serum levels of
lithium).
The correct answer is: Lithium

047
Which of the following drugs is associated with a 'creeping creatinine levels' on long term
use?

Select one:

858
Duloxetine

Lamotrigine

Carbamazepine

Lithium

Valproate

Lithium dosage will require a compensatory increase because glomerular filtration rate
increases gradually. But long term use can cause both reversible and irreversible renal
damage.
The correct answer is: Lithium

048
Which one among the following is not a potentially high-risk factor for developing neuroleptic
malignant syndrome (NMS)?

Select one:
Low potency typical drugs

Abrupt withdrawal of anticholinergics

Recent or rapid dose increase of antipsychotics

High-potency typical drugs

Rapid dose reduction of antipsychotics

The independent risk factors would include psychomotor agitation, mental retardation,
psychosis, organic brain disease, Parkinson's disease, hyperthyroidism, and alcoholism.
The correct answer is: Low potency typical drugs

049
Moclobemide is an antidepressant that requires dietary restrictions when prescribed. The
mechanism of action for the drug Moclobemide is

Select one:
MAOI - irreversible

5HT 1A agonist

Noradrenaline reuptake blockade

859
5HT2A antagonist

MAOI- reversible

Moclobemide is a reversible monoamine oxidase A (MAO-A) inhibitor.


The correct answer is: MAOI- reversible

050
Cheese Reaction is one of the most common and dangerous side-effects of

Select one:
TCAs

SNRIs

Atypical Antipsychotics

SSRIs

MAOIs

The gut MAO-A generally breaks down the tyramine in the diet. If tyramine enters the circulation
in large amounts, this results in a release of noradrenaline from endogenous storage sites,
causing a sudden and fatal rise in blood pressure. This can take place when an individual taking
MAO-A inhibitors (that reduces the activity of gut MAO) ingests large quantities of foods
containing tyramine.
The correct answer is: MAOIs

051
Which of the following drugs is contraindicated for a patient receiving a monoamine oxidase
(MAO) inhibitor?

Select one:
Trifluoperazine

Meperidine

Atropine

Chlorpromazine Budesonide

860
Meperidine is an opioid that can interact with monoamine oxidase inhibitors resulting in
serotonin syndrome.
The correct answer is: Meperidine

052
Sustained abnormal postures or positions called tardive dystonias are sometimes seen during
neuroleptic treatment. Tardive dystonia occurs after

Select one:
Months to years of antipsychotic treatment

Days to weeks of starting antipsychotics

Within hours to weeks of starting antipsychotics

Within hours of starting antipsychotics

Within minutes of starting antipsychotics

Generally, acute dystonia occurs within hours of starting antipsychotics. Pseudo-parkinsonism


occurs days to weeks of starting antipsychotics. Akathisia occurs within hours to weeks of
starting antipsychotics. Tardive Dyskinesia and tardive dystonia occurs after months to years of
antipsychotic treatment. (tardy = slow)
The correct answer is: Months to years of antipsychotic treatment

053
Which of the following receptors is involved in hypersalivation that occurs during clozapine
therapy?

Select one:
Beta adrenergic

Alpha adrenergic

Muscarinic

Nicotinic

Histaminic

861
The pharmacological basis of clozapine-related hypersalivation remains unclear. Suggested
mechanisms include muscarinic M4 agonism, adrenergic alpha 2 antagonism and inhibition of
the swallowing reflex.
The correct answer is: Muscarinic

054
The toxic confusional state caused by antipsychotics is mainly due to which of the following
mechanisms?

Select one:
Nicotinic receptor blockade

Alpha adrenergic blockade

Muscarinic receptor blockade

Histaminergic receptor blockade

All of the above

The toxic confusional state caused by antipsychotics is due to muscarinic receptor blockade. It
also causes dry mouth, difficulty urinating, constipation, and blurred vision.
The correct answer is: Muscarinic receptor blockade

055
A 31-year-old patient was started on antipsychotic medication for the first time. He developed
fluctuating BP, hyperthermia, confusion, urinary incontinence and has raised creatinine kinase.
The diagnosis is

Select one:
Parkinsonism

Acute dystonia

Neuroleptic malignant syndrome

Tardive dystonia

Tardive dyskinesia

The neuroleptic malignant syndrome (NMS) is rare, but life-threatening, idiosyncratic reaction to
a neuroleptic medication. The syndrome is characterized by fever, muscular rigidity, altered
mental status, and autonomic dysfunction.
The correct answer is: Neuroleptic malignant syndrome

862
056
A 50 yr old man who had a red alert with clozapine 10 years ago is now having a severe
relapse where all treatment strategies have failed. What is the best management choice?

Select one:
Clozapine and lithium can be given together

No clozapine should be given

Clozapine can be given with more frequent monitoring

Clozapine can be given in low doses

Clozapine can be given on alternate days

According to the central clozapine monitoring regulations, (CPMS: Clozaril patient monitoring
service), a 'green' alert indicates satisfactory count, an 'amber' alert requires a repeat FBC test
while clozapine can be continued, and a 'red' alert warrants immediate cessation of clozapine.
Re-introduction must be avoided unless there is a specialist supervision.
The correct answer is: No clozapine should be given

057
Which one among the following is not a significantly high-risk factor for developing drug-
induced hyponatraemia?

Select one:
Reduced renal function

Extreme Old Age

Obesity

Co-therapy with diuretics

Medical co-morbidity such as diabetes

The high risk factors would include extreme old age, female sex, warm weather, low body
weight, reduced renal function, co-therapy with diuretics, NSAIDs, carbamazepine, cancer
chemotherapy, medical co-morbidity such as diabetes, hypertension, COPD, hypothyroidism,
head injury, CVA.
The correct answer is: Obesity

863
058
The CNS side effects like anxiety and agitation in the initial few weeks of treatment with SSRIs
are proposed to be due to

Select one:
Under stimulation of 5HT3 receptors in the limbic system

Over stimulation of 5HT3 receptors in the limbic system

Over stimulation of 5HT2 receptors in the limbic system

Under stimulation of 5HT2 receptors in the limbic system

Under stimulation of 5HT1 receptors in the limbic system

This effect may be due to over stimulation of 5HT-2 receptors in the limbic system though this is
far from being conclusive. Fluoxetine has a high anxiogenic effect while citalopram has the least
propensity to induce anxiety.
The correct answer is: Over stimulation of 5HT2 receptors in the limbic system

059
Which antidepressant is contraindicated in closed-angle glaucoma?

Select one:
Bupropion

Mirtazapine

Paroxetine

Buspirone

Sertraline

Anticholinergic side effects include dry mouth, constipation, urinary retention, blurred vision,
and less commonly narrow-angle glaucoma. These effects are usually dose-related and
worse in patients with pre-existing defects. Paroxetine has significant anticholinergic
properties
The correct answer is: Paroxetine

060
Which one among the following is associated with erectile dysfunction more often than the
other SSRIs?

864
Select one:
Fluvoxamine

Fluoxetine

Citalopram

Paroxetine

Sertraline

Sexual dysfunction due to activation of post synaptic 5-HT 2 receptors is more prominent with
paroxetine than with the other SSRIs.
The correct answer is: Paroxetine

061
Which group of patients are not associated with an increased risk of developing Tardive
Dyskinesia?

Select one:
Patients with organic brain damage

Females

Patients on atypical antipsychotics

Patients with affective disorders

Older patients

Risk factors for TD-Older patients, females, patients with organic brain damage and patients
with affective disorders, and those who have had acute EPSEs (Extra pyramidal side effects)
early on treatment.
The correct answer is: Patients on atypical antipsychotics

062
In managing the hypertensive crisis associated with monoamine oxidase (MAO) inhibitors and
the ingestion of food with tyramine, the agent of choice is:

Select one:
propranolol

chlorpromazine

phentolamine

865
carvedilol

reserpine

Hypertensive crisis during MAOI use can be managed using phentolamine or


phenoxybenzamine.
The correct answer is: phentolamine

063
Which one among the following is not a feature of hyperprolactinaemia?

Select one:
Polymenorrhoea

Galactorrhoea

Reductions in bone mineral density

Amenorrhoea

Gynaecomastia

Persistent hyperprolactinaemia can lead to galactorrhoea, amenorrhoea, hypogonadism,


Gynaecomastia, sexual dysfunction and an increased risk of osteoporosis. Risperidone,
Amisulpride and Zotepine have potent prolactin-elevating effects, similar to conventional
drugs.
The correct answer is: Polymenorrhoea

064
Which one of the following is a common side effect of MAOIs

Select one:
Hepatocellular jaundice

Bone marrow suppression

Cardiac arrhythmias

Postural hypotension

Peripheral neuropathy

866
867
Common side effects of MAOIs would include insomnia, postural hypotension, peripheral
oedema, restlessness, nausea, dizziness, sexual difficulties, sweating and tremor. Rare side
effects would include hepatocellular jaundice, Peripheral neuropathy, bone marrow
suppression, and cardiac arrhythmias.
The correct answer is: Postural hypotension

065
From the following list choose the pair of medications where the former producing side-effects
that are ameliorated by the second drug.

Select one:
clozapine-frusemide

reboxetine-doxazosin

citalopram-hyoscine

Zopiclone-atomoxetine

Phenytoin-danazol

Urinary hesitancy can be an uncomfortable side effect during treatment with antidepressants
such as the selective noradrenaline reuptake inhibitor reboxetine. The use of selective a1A-
adrenoceptor antagonists such as doxazosin or tamsulosin can treat the urinary hesitancy -
especially in the elderly with associated prostate enlargement.
The correct answer is: reboxetine-doxazosin

066
The cardiotoxic effects of tricyclic antidepressants would include all of the following except

Select one:
Arrhythmias

ST elevation or depression

Atrioventricular Block

Reduction in cardiac output

QT Prolongation

TCAs affect cardiac conduction; as a result, they have antiarrhythmic properties and can slow
cardiac conduction. These effects are responsible for the cardiotoxicity of TCAs and their

868
danger in overdose. TCAs do not affect cardiac output. However, they should not be used in

869
patients with a preexisting conduction delay greater than a first-degree block or in patients
immediately after myocardial infarction.
The correct answer is: Reduction in cardiac output

067
Which one among the following as a side effect of tricyclic antidepressants is not related as
due to anticholinergic receptor blockade?

Select one:
Urinary retention

Constipation

Dry mouth

Blurred vision

Sedation

Anticholinergic effect: Dry mouth, constipation, urinary retention, blurred vision, and
glaucoma. Antihistaminic effect: sedation, weight gain.
The correct answer is: Sedation

068
Which of the following psychotropics is associated with an increased risk of GI bleed in the
elderly?

Select one:
Neuroleptics

SSRIs

Z group hypnotics

Atypical antipsychotics

Methadone

Risk of GI bleeds in the elderly can be increased by SSRIs.By increasing central and
peripheral cholinergic stimulation cholinesterase inhibitors, may also increase gastric acid
secretion, increasing the risk of GI bleeding especially in patients with ulcer disease or those
taking anti-inflammatories.
The correct answer is: SSRIs

870
069
Which of the following skin problems can prove to be fatal in those started on Lamotrigine?

Select one:
Steven Johnson's syndrome

Psoriasis

Jarisch-Herxheimer reaction

Acanthosis nigricans

Lichen planus

Rzany B et al. Risk of Stevens-Johnson syndrome and toxic epidermal necrolysis during first
weeks of antiepileptic therapy: a case-control study. Study Group of the International Case
Control Study on Severe Cutaneous Adverse Reactions. Lancet 1999;353:2190­4.
The correct answer is: Steven Johnson's syndrome

070
Which of the following is a prominent side effect of venlafaxine controlled by terazosin, alpha-
adrenergic blocker?

Select one:
Nausea

Headaches

Tremors

Postural drop in blood pressure

Sweating

Terazosin, a blocker of alpha-1 receptors, is observed to be effective in reducing excessive


sweating caused by antidepressant treatment, especially venlafaxine and SSRIs.
The correct answer is: Sweating

071
Botulinum toxin may have a role in the treatment of

Select one:
Tardive Dyskinesia

871
Akathisia

Tardive dystonia

Parkinsonism

Acute Dystonias

Although a large number of agents have been studied for their therapeutic effects on TD, there
is no definitive drug treatment for it. Botulinum may help in some but not all cases.
The correct answer is: Tardive dystonia

072
Which drug has been licensed for the treatment of Tardive Dyskinesia in the UK?

Select one:
Clonazepam

Vitamin E

Clonazepam

Tetrabenazine

Amitryptiline

Tetrabenazine has been licensed for the treatment of TD in UK. Other drugs that may be
useful include vitamin E, clonazepam and diazepam.
The correct answer is: Tetrabenazine

073
Which one among the following statements regarding drug interactions is false?

Select one:
TCAs interact with MAOIs and can cause serious toxicity like serotonergic
syndrome
Cimetidine increases TCAs levels

Warfarin action is potentiated by TCAs

The plasma levels of tricyclics and antipsychotics are decreased if co-


administered.

872
TCAs
reduce the
antihyperte
nsive action
of clonidine
and
guanethidin
e.

873
TCAs potentiate the effects of alcohol, hypnotics, anxiolytics and antipsychotics. The plasma
levels of tricyclics and antipsychotics are increased if co-administered.
The correct answer is: The plasma levels of tricyclics and antipsychotics are decreased if co-
administered.

074
Which one of the following drugs has a pronounced effect on QTc Prolongation?

Select one:
Zotepine

Olanzapine

Thioridazine

Sulpiride

Haloperidol

Drugs with a high effect on QTc include sertindole, thioridazine, pimozide and droperidol.
There have been reports of sudden death with Pimozide due to drug-induced QT
prolongation.
The correct answer is: Thioridazine

075
Which of the following drugs is associated with nephrolithiasis?

Select one:
Lithium

Carbamazepine

Topiramate

Valproate

Quetiapine

874
Topiramate is a weak inhibitor of carbonic anhydrase and can promote the development of
renal stones. It is weight neutral and can even cause weight loss.
The correct answer is: Topiramate

076
Which one among the following drugs may cause appetite suppression and weight loss?

Select one:
Gabapentin

Valproate

Lamotrigine

Lithium

Topiramate

Topiramate causes appetite suppression leading to weight loss. Topiramate blocks glutamate at
non-NMDA receptors. Gabapentin is effective against anxiety symptoms but not effective as a
mood stabiliser in bipolar disorder. It is not associated with hepatic or haematological problems.

The correct answer is: Topiramate

077
Priapism is a dreaded side effect associated with

Select one:
Imipramine

Phenelzine

Fluoxetine

Trazodone

Clomipramine

Trazodone has been associated with priapism in rare cases; this must be treated urgently to
avoid long-term impairment. Risperidone is also associated with the rare side effect of

875
priapism.
The correct
answer is:
Trazodone

876
078
Which class of antidepressant drugs must be avoided while treating depression in a patient
who does not want to gain weight?

Select one:

Tricyclics

Reboxetine

SSRIs

Venlafaxine

MAOIs

Any psychotropic can cause weight gain; in the given list tricyclics have a higher risk than
others. Note that up to 30% of patients on SSRIs also experience this side effect, with
increases of 8 kg or more.
The correct answer is: Tricyclics

079
Which of the following is not a recognised side effect of acetylcholinesterase inhibitors?

Select one:

Nausea

Anorexia

Urinary retention

Muscle cramps

Seizures

The most common cholinergic side effects of AChEIs involve the gastrointestinal tract. These
side effects are usually mild and have been reported to occur in approximately 20% of patients
taking these medications. Among the side effects reported in the package inserts of currently
available AChEIs are nausea (11%-47%), vomiting (10%-31%), diarrhea (5%-19%), and
anorexia (4%-17%). These can be minimized with the use of longer titration periods and the
administration of these medications with food.
The correct answer is: Urinary retention

080Which one among the following is not a specific risk factor for QTc prolongation?

877
Select one:
Anorexia nervosa
Extremes of age
Bradycardia

Myocarditis
Use of atypical antipsychotics rather than typicals

The effect of many but not all antipsychotic drugs on the QT interval appears to be dose-
related. Several antipsychotic drugs have infrequently been associated with malignant
arrhythmias such as torsade de pointes though atypicals have been rarely implicated in this
specific fatal effect of Qtc prolongation. Risk factors for QTc prolongation include female
gender, extremes of age, extreme physical exertion, stress or shock, anorexia nervosa, left
ventricular hypertrophy, myocardial infarction, hypokalemia, hypocalcaemia and
hypomagnesemia, bradycardia, myocarditis and congenital long QT syndrome.
The correct answer is: Use of atypical antipsychotics rather than typicals

081
Bupropion is associated with which of the following side effects?

Select one:

Anticholinergic action

Weight loss

Sedation

Loss of libido

Increase seizure threshold

Bupropion is an antidepressant that induces weight loss and even seems to be effective in
obesity when combined with calorie-restricted diets. It is contraindicated in anorexia and other
eating disorders.
The correct answer is: Weight loss

Flag question 082


Which one of the following statements regarding antidepressants is false?

Select one:

878
Orofacial
Dyskinesia
has been
reported with
Paroxetine

879
Hyponatraemia has been reported with Fluvoxamine

Hypoglycaemia has been reported on using Fluoxetine

Fluovaxamine causes less sexual dysfunction than other SSRIs

Withdrawal syndrome has been reported most commonly with Fluoxetine

Hyponatraemia has been reported with Fluvoxamine and Fluoxetine. Withdrawal syndrome has
been reported most commonly with Paroxetine. Dystonias and Orofacial Dyskinesia have been
reported with Paroxetine.
The correct answer is: Withdrawal syndrome has been reported most commonly with
Fluoxetine

083
Lithium induced hypothyroidism is much more common in

Select one:

Children

Elderly females

Young males

Young females

Elderly males

Lithium induced hypothyroidism is much more common in middle-aged women (up to 3-4%)
The correct answer is: Young females

880
 Home Mock Paper Practice Tests

Mock Paper A(2)

Started on Saturday, 27 June 2015, 11:49 PM
State Finished
Completed on Saturday, 27 June 2015, 11:49 PM
Time taken 26 secs
Grade 0.00 out of 100.00
Feedback This is a bit worrying. You need to prioritize your revision urgently.

Question 1 MoK Adverse Effects 001
Not answered What percentage of patients on clozapine develop agranulocytosis during the first year of
treatment?
Marked out of 1.00

Flag question Select one:
0.15
0.1
0.05
0.01
0.2

Your answer is incorrect.
In an epidemiologic drug registry study in the United Kingdom, it was found that clozapine
causes neutropenia in 1.5% to 2.9% of patients treated over a 1­year period, with progression
to agranulocytosis in 0.8% of patients
The correct answer is: 0.01

Question 2 MoK Adverse Effects 002
Not answered The annual incidence of tardive dyskinesia in patients with chronic schizophrenia who are on
any antipsychotic treatment is
Marked out of 1.00

Flag question Select one:
0.05
0.03

0.1
0.01

881
0.2

Your answer is incorrect.
Longitudinal incidence data with first generation antipsychotics (FGAs) as a class suggest that
the annual incidence of tardive dyskinesia risk is ~4% per year of exposure. Thus, of 100
patients started on haloperidol or chlorpromazine, four would have some manifestations of
tardive dyskinesia at the end of 1 year. That rate increases after the second or third year. It is
unknown for how long this rate remains linear and if it would extend to 100% (Retrieved from
http://primarypsychiatry.com/in­session­with­jeffrey­a­lieberman­md­tardive­dyskinesia/). A
systematic review published in 2004 (Corell et al., 2004) suggested that weighted mean
annual incidence of tardive dyskinesia for second­generation antipsychotics was 0.8%
(range=0.0%­1.5%) in adults and 5.3% in patients aged 54 years and older, compared to
5.4% in adults treated with haloperidol. A large prospective study of 9298 patients (SOHO
study) suggested that overall, the risk of TD in patients taking any antipsychotic is likely to be
around 3% (Tenback et al., 2006).
The correct answer is: 0.03

Question 3 MoK Adverse Effects 004
Not answered The risk of fetal malformations with sodium valproate is approximately

Marked out of 1.00
Select one:
Flag question
7%

28%
1%
70%
90%

Your answer is incorrect.
Epilepsy itself increases the risk of foetal malformations  two to three times when compared to
the general population. 7.2% of pregnant mothers using valproate will have children showing
some form of fetal malformation.  In particular, the incidence of neural tube defects is 1% to
2%. This risk increases with increasing doses of valproic acid especially when the dose is
more than 1gm/day.
NICE. Technology Appraisal 76.
The correct answer is: 7%

Question 4 MoK Adverse Effects 005
Not answered Which of the following drugs is currently thought to cause least QT prolongation of all
antipsychotics?
Marked out of 1.00

Flag question Select one:
Olanzapine

882
Quetiapine
Haloperidol
Clozapine
Aripiprazole

Your answer is incorrect.
Current available evidence favours aripiprazole though this may be revised in future with
more data becoming available.
The correct answer is: Aripiprazole

Question 5 MoK Adverse Effects 009
Not answered According to the CATIE study of patients with schizophrenia, the number of patients meeting
criteria for metabolic syndrome ranges between what percentages?
Marked out of 1.00

Flag question Select one:
10­20%
31­40%
51­60%

21­30%
41­50%

Your answer is incorrect.
In the CATIE study, the metabolic syndrome prevalence was found to be 51.6% for females
and 36.0% for males with schizophrenia.
The correct answer is: 41­50%

Question 6 MoK Adverse Effects 010
Not answered The upper normal QTc level in female subjects is

Marked out of 1.00
Select one:
Flag question
440 ms
450 ms
500 ms

470 ms
420 ms

Your answer is incorrect.

883
Normal QTc for men is 440 ms and 470 ms for women. There is a clearly increased risk of
arrhythmias in QTc values over 500 ms.
The correct answer is: 470 ms

Question 7 MoK Adverse Effects 011
Not answered A 44­year­old woman on clozapine has gained 15Kgs in 3 months after starting the treatment.
This antipsychotic induced weight gain is most likely to be due to;
Marked out of 1.00

Flag question Select one:
Hypoprolactinaemia
Leptin sensitisation
5­HT2C antagonism
M4 blockade
H2 antagonism

Your answer is incorrect.
Agonists at the 5HT2 family of receptors cause hypophagia and can cause weight loss in
animals and human beings. In rats, 5HT2C antagonists cause an increase in feeding.
Clozapine is a potent 5HT2C antagonist. Autoradiographic studies have shown that 5HT2C
receptors are localised in high density in the ventromedial and dorsomedial nuclei within the
hypothalamic satiety control centres. 5HT2C knockout mice are overweight and exhibit
increased feeding as opposed to metabolic changes.3 Clozapine may disrupt serotonergic
pathways, resulting in weight gain in predisposed people.
The correct answer is: 5­HT2C antagonism

Question 8 MoK Adverse Effects 013
Not answered Which of the following is true with regard to acetylcholine (Ach) ?
Marked out of 1.00
Select one:
Flag question
Acetylcholinergic projections are more predominant than serotonergic
projections in the cortex
Tricyclics increase acetylcholine levels
Antidementia drugs reduce acetylcholine concentration
Acetylcholine agonists are useful in mild to moderate dementia of
Alzheimer's type
Acetylcholine is synthesised by acetyl cholinesterase

Your answer is incorrect.
The distribution of cholinergic innervation in the human cerebral cortex reveals that all
cytoarchitectonic regions and layers of the cerebral cortex display a dense cholinergic
innervation The density of cholinergic axons is higher in the more superficial layers (layers I

884
and II and the upper parts of layer III) of the cerebral cortex. Distinct patterns of lamination
exist in individual cytoarchitectonic regions ( Structure and Function of Cholinergic Pathways.
http://www.acnp.org/g4/GN401000012/CH012.html (accessed April 13, 2015)).
The correct answer is: Acetylcholinergic projections are more predominant than serotonergic
projections in the cortex

Question 9 MoK Adverse Effects 014
Not answered Which of the following antidepressants is least likely to be associated with sexual dysfunction
in male depressed patients?
Marked out of 1.00

Flag question Select one:
Agomelatine
Trazodone
Venlafaxine
Fluoxetine
Amitriptyline

Your answer is incorrect.
The overall side­effect profile of agomelatine appears relatively mild. For example,
agomelatine has no discontinuation syndrome and is generally weight neutral. Agomelatine
appears to be relatively free of sexual dysfunction and related side effects. (Sansone &
Sansone, 2011)
The correct answer is: Agomelatine

Question 10 MoK Adverse Effects 017
Not answered A 66 year old man taking antidepressant therapy develops dizziness every morning. He gives
a history of palpitations accompanied by the dizziness. There is no history of vertigo or falls
Marked out of 1.00
but the dizziness improves spontaneously on reclining. Which of the following receptor
Flag question blockade can explain the above symptoms?

Select one:
Alpha 1 adrenergic
Cholinergic nicotinic
Cholinergic muscarinic
Alpha 2 adrenergic
Beta adrenergic

Your answer is incorrect.
Alpha 1 blockade can cause dizziness, postural hypotension, ejaculatory disturbance and
dependent edema.
The correct answer is: Alpha 1 adrenergic

885
Question 11 MoK Adverse Effects 018
Not answered Mr. X is taking trazodone for his depressive symptoms. He complains of excessive sedation.
This effect is most likely to be due to the blockade of which of the following receptors?
Marked out of 1.00

Flag question Select one:
Nicotinic
Beta adrenergic
Alpha adrenergic
Muscarinic

Histaminic

Your answer is incorrect.
Trazodone's potent alpha1­adrenergic blockade (~3­fold lower relative to 5­HT2A) may cause
some side effects like sedation and orthostatic hypotension. Conversely, along with 5­HT2A
antagonism, it may underlie trazodone's efficacy as a hypnotic. This seems possible as
trazodone's antihistamine activity is relatively weak and probably clinically insignificant;
hence, it cannot explain trazodone's sleep­inducing/enhancing effects. 
The correct answer is: Alpha adrenergic

Question 12 MoK Basic Pharmacology 001
Not answered Choose the most accurate statement about cannabis
Marked out of 1.00
Select one:
Flag question
Cannabis commonly causes physical dependence.
9­delta­tetrahydrocannabinol is the active substance
Fatal overdose is reported in several countries

Urine tests usually remain positive for up to a week after regular use has
ceased
Anandamide is a specific cannabinoid receptor and a naturally occurring
antagonist at this receptor

Your answer is incorrect.
There are 60 psychoactive cannaboids, the most important of which is 9­delta­
tetrahydrocannabinol (THC). Anandamide is a specific cannabinoid receptor and a naturally
occurring agonist at this receptor. Cannabis is not usually associated with physical
dependence. Urine tests usually remain positive for up to 4weeks after regular use has
ceased. Unlike most other drugs of abuse, there are no reports of fatal overdose from
cannabis.
The correct answer is: 9­delta­tetrahydrocannabinol is the active substance

886
Question 13 MoK Basic Pharmacology 002
Not answered Mr.Z is a 26 year old gentleman who was diagnosed with paranoid schizophrenia 4 years ago
and has had 6 hospitalisations over last 3 year period. Most of his relapses have been
Marked out of 1.00
associated with non adherence to medication. Which of the following interventions are most
Flag question likely to improve compliance for this patient?

Select one:
Insight orientated psychotherapy
Cognitive motivational intervention
Supportive psychotherapy
Family therapy
Psycho educational intervention

Your answer is incorrect.
Studies on psycho social intervention for improving medication adherence showed that
psycho education alone and family therapy alone were ineffective and programmes that focus
on the attitudinal and behavioural aspects of taking medications have better outcomes. There
is no evidence based data available for supportive psychotherapy and insight orientated
psychotherapy for improving medication adherence.
The correct answer is: Cognitive motivational intervention

Question 14 MoK Basic Pharmacology 003
Not answered Carlsson is associated with which of the following?

Marked out of 1.00
Select one:
Flag question
learned helplessness hypothesis of depression
hairline triggers in borderline personality
endocrine hypothesis of depression
kindling hypothesis of depression
dopamine hypothesis of schizophrenia

Your answer is incorrect.
Arvid Carlsson made the seminal discovery in the late 1950's that dopamine is a transmitter in
the mammalian brain.
The correct answer is: dopamine hypothesis of schizophrenia

Question 15 MoK Basic Pharmacology 004
Not answered The risk factor predicting poor response to lithium prophylaxis in bipolar illness is

Marked out of 1.00
Select one:
Flag question

887
Dominant personality
Social support
High social status
High age of illness onset
High number of previous hospitalizations

Your answer is incorrect.
Predictors of poor response to lithium in bipolar prophylaxis include High number of previous
hospitalization, stress, high expressed emotion, and high number of life events, episodic
patterns of depression­ mania interval, continuous cycling, and unemployment and neurotic
personality.
The correct answer is: High number of previous hospitalizations

Question 16 MoK Basic Pharmacology 005
Not answered Which of the following antipsychotics is recommended for mixed affective states in bipolar
disorder?
Marked out of 1.00

Flag question Select one:
perphenazine
amisulpride
sulpride
quetiapine
Olanzapine

Your answer is incorrect.
Though based on a limited amount of information, a Cochrane review (and British Association
of Psychopharmacology guidelines) suggests that there is evidence that olanzapine may
prevent further mood episodes in patients who have responded to olanzapine during an index
manic or mixed episode and who have not previously had a satisfactory response to lithium or
valproate (http://www.ncbi.nlm.nih.gov/pubmedhealth/PMH0012734/ )
The correct answer is: Olanzapine

Question 17 MoK Basic Pharmacology 007
Not answered A 36­year­old cocaine user notices that the longer he uses the drug, the more of it he requires
to achieve the same effect. This is best described as
Marked out of 1.00

Flag question Select one:
Addiction
Dependence

888
Tolerance
Sensitisation
Compliance

Your answer is incorrect.
Tolerance refers to the pharmacological adaptation because of which a larger dose of a drug
becomes necessary over time to achieve the same effect.
The correct answer is: Tolerance

Question 18 MoK Pharmacodynamics 001
Not answered Which of the following is true with regard to nicotinic receptors?

Marked out of 1.00
Select one:
Flag question
Blockade causes problems with fine motor movements
Mediate anti­parkinsonian effects of amantadine
All nicotinic receptors are excitatory
Mediate sympathomimetic actions of dopamine
Anticholinergic effects of antipsychotics are mediated by nicotinic receptors

Your answer is incorrect.
All nAChR are excitatory. Nicotinic acetylcholine receptors (nAChRs) are excitatory
neurotransmitter­gated ion channels and members of a superfamily that also includes
ionotropic receptors such as GABA­A. The other type of cholinergic receptors, muscarinic
group, mediate anti­cholinergic effects of psychotropics and disturbances of fine motor
movements seen in certain neurological disorders. Nicotinic receptors when blocked produce
gross neuromuscular paralysis.
The correct answer is: All nicotinic receptors are excitatory

Question 19 MoK Pharmacodynamics 002
Not answered SSRIs, on stimulating which receptors, will induce sleep disturbances?
Marked out of 1.00
Select one:
Flag question
5­HT2A
5­HT3
5­HT1A
H­1

5­HT1D

889
Your answer is incorrect.
5­HT2A stimulation by SSRIs will lead to insomnia
The correct answer is: 5­HT2A

Question 20 MoK Pharmacodynamics 003
Not answered Aripiprazole has an antagonistic effect on which of the following receptors?
Marked out of 1.00
Select one:
Flag question
D2 receptors
5­HT3 receptors
5­HT2A receptors

5­HT1A receptors
D1 receptors

Your answer is incorrect.
Aripiprazole is a D2 partial agonist, an action that is unique to aripiprazole; it is also a 5­HT2A
antagonist and 5­HT1A partial agonist. Minor actions include H1 antagonism, alpha­1, alpha­
2 antagonism and to some extent diffuse muscarininc antagonism. (M1, M2, M3, M4, M5
antagonist)
The correct answer is: 5­HT2A receptors

Question 21 MoK Pharmacodynamics 004
Not answered Agonistic effect of fluoxetine on which of the following receptors results in anorgasmia?

Marked out of 1.00
Select one:
Flag question
Alpha­1 adrenergic receptors
5HT­1A stimulation
D­2 receptors
5HT­2C stimulation
Mu opiod receptors

Your answer is incorrect.
5­HT2A/2C stimulation by SSRIs can cause delayed ejaculation and Anorgasmia.
The correct answer is: 5HT­2C stimulation

Question 22 MoK Pharmacodynamics 005
Not answered Which of the following is a noncompetitive NMDA blocker?

Marked out of 1.00

890
Flag question Select one:
Lamotrigine
Bupropion
Acamprosate
Disulfiram
Varenicline

Your answer is incorrect.
Acamprosate has a structure similar to that of GABA and is active at the N­methyl­D­aspartate
(NMDA) receptor (Knopfel et al 1987), via non­competitive NMDA blockade. But this action is
supposed to be weak in therapeutic doses.
The correct answer is: Acamprosate

Question 23 MoK Pharmacokinetics 001
Not answered The number of half­lives normally taken by drugs to reach steady plasma levels is;

Marked out of 1.00
Select one:
Flag question
7
9
2
3
5

Your answer is incorrect.
In clinical practice, after regular administration, a drug takes 4 to 5 times the half­life for its
serum concentration to reach steady state.
The correct answer is: 5

Question 24 MoK Pharmacokinetics 002
Not answered Which of the following is correct with regard to benzodiazepine metabolism?

Marked out of 1.00
Select one:
Flag question
Desmethyldiazepam is an inactive metabolit
Diazepam achieves nearly 10 times greater plasma level in the elderly

Diazepam is around 95% protein bound
Duration of action depends primarily on elimination half­life
Elimination half­life is reduced in the elderly

891
Your answer is incorrect.
Diazepam produces an active metabolite desmethyldiazepam (also called as nordiazepam ­
t1/2 71 hours) via CYP3A4 and 2C19. Diazepam is heavily protein bound (around 95%). This
plasma protein binding is significantly reduced in the elderly subjects, but the plasma free
(unbound) diazepam concentrations do not significantly exceed when compared to young
adults. The elimination half­life is increased by 100% in elderly ( around 90 hrs) as clearance
is reduced, leading to a much prolonged duration of action than in younger adults (around 46
hrs). Duration of action of benzodiazepines depends on rate and extent of distribution, in
addition to half­life. Most long­acting agents have long elimination half­life.
The correct answer is: Diazepam is around 95% protein bound

Question 25 MoK Pharmacokinetics 004
Not answered Mr Y is a 27 year old gentleman who has recently been started on lithium for treatment of his
acute phase of his manic episode. What is the therapeutic blood lithium level you would aim
Marked out of 1.00
for?
Flag question
Select one:
0.6­ 0.8 M Eq/ L
0.4­ 0.5 M Eq/ L
0.2­ 0.4 M Eq/ L
1.5 ­2.0 M Eq/ L
0.8­1.2 M Eq/ L

Your answer is incorrect.
Lithium is currently the drug of choice for treatment of the acute phase of a manic episode as
well as for prevention of further episodes of bipolar mood disorder. The treatment is closely
monitored by repeated blood levels as the difference between therapeutic and lethal blood
levels is not very wide. Therapeutic blood levels for antimanic or antidepressant effect range
between 0.8 and 1.2 MEq/L; Prophylactic blood lithium levels vary from 0.6 to 1.2 MEq/L. A
blood lithium level of more than 2 milliequivalents per litre may be associated with toxicity
while a level of more than 2.5 may be lethal.
The correct answer is: 0.8­1.2 M Eq/ L

Question 26 MoK Pharmacokinetics 005
Not answered The minimum effective dose of Aripiprazole in treating a psychotic episode is
Marked out of 1.00
Select one:
Flag question
20mg
15mg
10mg
25mg
5mg

892
Your answer is incorrect.
Aripiprazole was shown to be superior to placebo at doses of 15 mg/day, 20 mg/day, and 30
mg/day in at least two clinical trials for each dose and at 10 mg/day in another trial. Two
studies that evaluated the efficacy of aripiprazole at lower doses (2 and 5mg/day) in
schizophrenia, failed to demonstrate the superiority of aripiprazole versus placebo. The
available trial evidences indicate that Aripiprazole 10 mg/day is the minimum effective dose
for the treatment of patients with schizophrenia experiencing acute relapse. Aripiprazole 5
mg/day produced improvements in some outcome measures but not in the primary efficacy
measure, suggesting that it may lack efficacy for treating acute psychosis. CNS Spectr.
2006;11(9):691­702
The correct answer is: 10mg

Question 27 MoK Pharmacokinetics 006
Not answered The minimum duration of gap recommended between stopping venlafaxine and starting
duloxetine is
Marked out of 1.00

Flag question Select one:
2 weeks
2 months
2 days
1 day
1 week

Your answer is incorrect.
According to product info, a 2 weeks gap is recommended though popular prescribing
guidelines suggest 2 weeks of tapering for venlafaxine till stopping it, followed by the alternate
day prescription of duloxetine.
The correct answer is: 2 weeks

Question 28 MoK Pharmacokinetics 013
Not answered The rate­limiting enzyme in the metabolism of alcohol is;
Marked out of 1.00
Select one:
Flag question
Alcohol dehydrogenase
Acetyl co A dehydrgenase
Aldehyde dehydrogenase
Acetic acid
Acetyl dehydrogenase

893
Your answer is incorrect.
The major site of alcohol metabolism is the liver. Alcohol dehydrogenase is the rate­limiting
enzyme in the metabolism of alcohol which makes it impossible to speed up alcohol
metabolism.
The correct answer is: Alcohol dehydrogenase

Question 29 MoK Pharmacokinetics 014
Not answered Which of the following drugs is least dependent on hepatic metabolism?

Marked out of 1.00
Select one:
Flag question
Chlorpromazine
Clozapine
Olanzapine
Amisulpride
Risperidone

Your answer is incorrect.
Renal elimination without significant liver breakdown is seen with amisulpride and sulpride
(90% elimination via renal route)
The correct answer is: Amisulpride

Question 30 MoK Pharmacokinetics 016
Not answered Which of the following drugs have a half­life of 3 days?

Marked out of 1.00
Select one:
Flag question
Carbamazepine
Aripiprazole
Quetipaine
Reboxetine
Fluoxetine

Your answer is incorrect.
Aripiprazole displays linear kinetics and has a mean terminal half­life of 60 hours. (Range 46­
68 hours)
The correct answer is: Aripiprazole

Question 31 MoK Applied Neuroscience 001
Not answered Which of the following is a localising sign for frontal lesions?

894
Marked out of 1.00 Select one:
Flag question Postural headache
Ptosis
Nystagmus

Olfactory disturbances
Myoclonus

Your answer is incorrect.
The most classic presentation of frontal lobe dysfunction is an olfactory groove meningioma
characterized by anosmia, loss of inhibition, memory impairment, headaches, and visual
symptoms. But anosmia itself is not a good localising sign as often the cause of olfactory
disturbance is peripheral ­ related to nostrils (Frontal Lobe Syndromes Clinical Presentation,
http://emedicine.medscape.com/article/1135866­clinical (accessed April 13, 2015).).
The correct answer is: Olfactory disturbances

Question 32 MoK Applied Neuroscience 002
Not answered National adult reading test is used to assess

Marked out of 1.00
Select one:
Flag question
Visuospatial ability
Premorbid intelligence

Attentional bias
Nonverbal intelligence
Frontal lobe functions

Your answer is incorrect.
The validity of NART as a measure of premorbid ability rests upon the assumptions that
reading ability (of irregular words) is relatively independent of brain damage, and that it is a
strong predictor of intelligence in the normal population.The test requires subjects to read out
loud a set of 50 words which are irregular in terms of their grapheme­phoneme
correspondences (Coltheart et al., 1987). The responses are individually scored as correct or
incorrect, according to their pronunciation. This score can then be used to derive a premorbid
IQ estimate. Journal of the International Neuropsychological Society (2002), 8, 847­854.
The correct answer is: Premorbid intelligence

Question 33 MoK Applied Neuroscience 003
Not answered A 34­year­old man unable to remember any of the events that happened at the hospital during
the time of his admission for a head injury even after two years after recovery. He has no other
Marked out of 1.00
memory problems. This is
Flag question
Select one:

895
Confabulation
Anterograde amnesia
Retrograde amnesia
Episodic memory loss
Dissociative memory loss

Your answer is incorrect.
After the head injury, he had a period of anterograde amnesia during which he did not form
any memory trace for a period of time. As a result, he has no memory of these events even
after two years.
The correct answer is: Anterograde amnesia

Question 34 MoK Applied Neuroscience 004
Not answered A 70­year­old man presents with sudden onset of right arm weakness following severe
headache and inability to understand speech. His speech problem is accurately described as
Marked out of 1.00

Flag question Select one:
Aphasia
Alexia
Aphemia
Agraphia
Amusia

Your answer is incorrect.
This is receptive aphasia (Wernicke's area damage)
The correct answer is: Aphasia

Question 35 MoK Applied Neuroscience 005
Not answered On routine examination of eyes, accommodation reflexes were present but pupils were small
irregular with absent light reflexes. This type of pupil is called 
Marked out of 1.00

Flag question Select one:
Marcus Gunn pupil
Senile pupil
Anosognosia
Kayser­Fleischer pupil
Argyll Robertson pupils

896
Your answer is incorrect.
Douglas Argyll Robertson's (1837­1909) conducted experimental work with physostigmine in
1863. He observed patients with spinal cord disease who had lost the response to light even
though accommodation to near objects was normal. This phenomenon, known subsequently
as the Argyll Robertson pupil came to be considered pathognomonic of tabes dorsalis,
general paresis and neurovascular syphilis.
The correct answer is: Argyll Robertson pupils

Question 36 MoK Applied Neuroscience 006
Not answered Abnormalities in Stroop test suggests
Marked out of 1.00
Select one:
Flag question
Short term memory disturbance
Damage to occipital cortex
Attentional bias
Cortical blindness
Lesion of optic chiasma

Your answer is incorrect.
An abnormal Stroop effect with the loss of appropriate inhibitory control is seen in frontal lobe
lesions. Abnormalities in modified emotional Stroop interference effects (e.g. on threat words
in anxious subjects on the emotional Stroop task) have generally been interpreted as
evidence for mood­congruent attentional bias in various mental states.
The correct answer is: Attentional bias

Question 37 MoK Applied Neuroscience 007
Not answered Which of the following instruments can be used to screen for cognitive deficits after head
injury?
Marked out of 1.00

Flag question Select one:
Bender visual motor gestalt test
National adult reading test
Glasgow coma scale
Weschler adult intelligence scale
Raven's progressive matrices

Your answer is incorrect.
The Bender­Gestalt is used to evaluate the maturity of the visual­motor system and to screen
children for developmental delays. The test is also used to assess brain damage and
neurological deficits. Individuals who have suffered a traumatic brain injury may be given the

897
Bender­Gestalt as part of a battery of neuropsychological measures or tests.
The correct answer is: Bender visual motor gestalt test

Question 38 MoK Applied Neuroscience 008
Not answered A 64­year­old man has behavioural changes with irritability. He is found wandering in the
streets near his home. On neurological examination, there is evidence of mild aphasia and
Marked out of 1.00
cognitive dysfunction, but motor function is preserved. He seems to have no awareness
Flag question regarding his difficulties and has a silly, jocular affect. Which of the following is most likely to
be seen in CT/MRI scan?

Select one:
Dilated ventricles without cerebral atrophy
Predominant occipito parietal atrophy
Diffuse cortical atrophy with apparently dilated ventricles
Bilateral frontal atrophy
Space occupying lesion obstructing third ventricle

Your answer is incorrect.
The presence of reduced verbal fluency, loss of insight and having a silly affect points towards
frontal lesions.
The correct answer is: Bilateral frontal atrophy

Question 39 MoK Applied Neuroscience 009
Not answered Nonfluent aphasia is associated with lesions in or near which of the following brain regions?
Marked out of 1.00
Select one:
Flag question
Parietal operculum
Anterior cingulate
Superior temporal gyrus
Insula
Broca's area

Your answer is incorrect.
Nonfluent aphasia is associated with lesions in or near Broca's area in motor cortex. Temporal
and parietal lobes are usually not impaired. Therefore, receptive language and
comprehension are not usually impaired in nonfluent aphasia.
The correct answer is: Broca's area

Question 40 MoK Molecular Genetics 001

898
Not answered What is the concordance rate of schizophrenia in dizygotic twins?
Marked out of 1.00
Select one:
Flag question
0.22
0.48
0.18
0.14
0.08

Your answer is incorrect.
Most studies suggest that the concordance rates are 44­50% for MZ twins and 10­15% for DZ
twins. On the average, most studies show that the concordance rate for schizophrenia is three
times higher in monozygotic twins than in dizygotic twins.
The correct answer is: 0.48

Question 41 MoK Molecular Genetics 002
Not answered The gene implicated in Huntington's disease is present on the chromosome

Marked out of 1.00
Select one:
Flag question
21
14
7
4
1

Your answer is incorrect.
Huntington's disease­related gene is located on the short arm of chromosome 4; it contains a
variable stretch of repeating CAG trinucleotides which code for a glutamine residue in the
huntingtin protein. Individuals with 40 or more repeats will develop Huntington's disease;
those with 35 or fewer will not. But how this polyglutamine expansion in huntingtin causes the
disease is uncertain. Am J Psychiatry 164:1318, September 2007
The correct answer is: 4

Question 42 MoK Molecular Genetics 003
Not answered Which of the following is associated with narcolepsy?

Marked out of 1.00
Select one:
Flag question
Waxy flexibility
Unrefreshing sleep attacks

899
2% prevalence
Prolonged REM latency
HLA­DR2

Your answer is incorrect.
Narcolepsy is associated with cataplexy (NOT catalepsy / waxy flexibility ­ the latter are
catatonic features). It is a disabling sleep disorder affecting 0.02% of adults worldwide. It is
characterised by severe, irresistible daytime sleepiness and sudden loss of muscle tone
(cataplexy), and can be associated with sleep­onset or sleep­offset paralysis and
hallucinations, frequent movement and awakening during sleep, and weight gain. Sleep
monitoring during night and day shows rapid sleep onset and abnormal, shortened rapid­eye­
movement sleep latencies. The onset of narcolepsy with cataplexy is usually during teenage
and young adulthood and persists throughout the lifetime. Pathophysiological studies have
shown that the disease is caused by the early loss of neurons in the hypothalamus that
produce hypocretin, a wakefulness­associated neurotransmitter present in the cerebrospinal
fluid. The cause of neural loss could be autoimmune since most patients (more than 90%)
have the HLA DQB1*0602 allele that predisposes individuals to the disorder. The frequency
of HLA­DR2 is also increased in narcolepsy though not as high as HLA DQB1*0602.
Treatment is with stimulant drugs to suppress daytime sleepiness, antidepressants for
cataplexy, and gamma­hydroxybutyrate for both symptoms (Narcolepsy with cataplexy,
http://www.ncbi.nlm.nih.gov/pubmed/17292770).
The correct answer is: HLA­DR2

Question 43 MoK Molecular Genetics 005
Not answered Mr. Y has a well­established diagnosis of bipolar affective disorder. His younger brother, who
is also his main carer, is aware that this disorder has a strong genetic loading and is worried.
Marked out of 1.00
What is the prevalence rate in first­degree relatives of patients with bipolar affective disorder?
Flag question
Select one:
15­20%
0.4
1­2%
0.05
0.79

Your answer is incorrect.
Bipolar disorder has a multifactorial aetiology. Both family studies and twin studies have
demonstrated a strong genetic loading for it. A prevalence rate of 15­20% has been
demonstrated in first­degree relatives of patients with the bipolar affective disorder. Twin
studies have shown a concordance rate of up to 79% in Monozygotic twins and 19% in
dizygotic twins. Ref: Pandarakalam, JP (2008) Diagnostic conundrums of bipolar disorder.
Progress in Neurology and Psychiatry, 12, 6­11.
The correct answer is: 15­20%

900
Question 44 MoK Molecular Genetics 006
Not answered Mr. Y is a 40­year­old gentleman with history of callous unemotional traits. What is the
heritability (genetic contribution) of these traits?
Marked out of 1.00

Flag question Select one:
5­10%
50­60%
1­2%
10­20%
30­40%

Your answer is incorrect.
Results from a meta­analysis of behaviour genetic studies on children with callous­
unemotional traits and antisocial behaviour estimates, that 41% of the variance is due to
genetic factors, 16% to shared environmental factors and 43% to non­shared environmental
factors. Data from other studies appear to indicate moderate heritability for violent behaviour
as well. (Ref: Viding E. On the Nature and Nurture of Antisocial Behavior and Violence.
Retrieved from http://www.socialbehavior.uzh.ch/teaching/semsocialneurosciencews07/)
The correct answer is: 30­40%

Question 45 MoK Molecular Genetics 010
Not answered Which of the following substances is found in reduced levels in patients with high impulsive
suicidality?
Marked out of 1.00

Flag question Select one:
CYP4503A4
COMT

5­HIAA
BDNF
Neuregulin

Your answer is incorrect.
Decreased 5­HIAA levels are associated with suicidal, antisocial and violent behaviours.
[Halaban et al. 1996]. The psychological mechanism underlying this relationship is unclear,
but some current models speculate that reductions in serotonin increase impulsivity and
disinhibit behavior, thereby increasing the likelihood of engaging in risky and antisocial acts.
The correct answer is: 5­HIAA

Question 46 MoK Molecular Genetics 012
Not answered Which of the following types of chromosomes undergo Robertsonian translocation?

901
Marked out of 1.00 Select one:
Flag question submetacentric
Metacentric
telocentric
Holocentric
acrocentric

Your answer is incorrect.
Robertsonian translocation is a non­reciprocal (i.e. unequal exchange) that results in a single
fused chromosome from 2 acrocentric (non­homologous) chromosomes. A metacentric
chromosome has centromere right in the middle. So p and q arms are of equal length. Most
chromosomes are submetacentric where the arms are of unequal length. If the centromere is
at the tail of a chromosome, it is called telocentric. With holocentric chromosomes, the entire
length of the chromosome acts as the centromere. These latter two types are not seen in
humans.
The correct answer is: acrocentric

Question 47 MoK Molecular Genetics 013
Not answered Which of the following statements is true about sex linked recessive disorders?

Marked out of 1.00
Select one:
Flag question
25% of sons of heterozygous mothers are affected
25% of daughters of heterozygous mothers are affected
Females more affected than males
All daughters of affected fathers are carriers of the disease
Affected fathers pass disease to sons

Your answer is incorrect.
In sex­linked recessive disorders, an affected male can transmit the disease­causing mutation
to his daughters, who remains normal phenotypically but carries and transmits the disease­
causing allele to her sons. As females have two copies of the X chromosome, they need a
double identical mutation for disease expression that is extremely rare.
The correct answer is: All daughters of affected fathers are carriers of the disease

Question 48 MoK Molecular Genetics 014
Not answered In Down's syndrome with Alzheimer's disease, the gene located is on the

Marked out of 1.00
Select one:
Flag question
Amyloid precursor protein
Presenilin 1

902
Presenilin 2
Apolipoprotein e3
Apolipoprotien e 4

Your answer is incorrect.
Individuals with Down syndrome develop beta­amyloid deposition characteristics of early
onset Alzheimer's disease in mid­life, presumably due to an extra copy of the chromosome
21­located APP gene. By the age of 60, between 50 and 70% of people with Down syndrome
develop dementia. Triplication of a short segment of APP in people without Down syndrome
has also been associated with early­onset Alzheimer's disease (Retrieved from
http://www.cdadc.com/ds/dementia.htm).
The correct answer is: Amyloid precursor protein

Question 49 MoK Molecular Genetics 017
Not answered In which of the following types of inheritance, 75% of children inherit a disease when both
parents have the disease and share a similar genotype?
Marked out of 1.00

Flag question Select one:
Mitochondrial
Autosomal dominant
X linked recessive
X linked dominant

Autosomal recessive

Your answer is incorrect.
If both parents are diseased and have an autosomal recessive condition, then all children will
be diseased (100%). If both are diseased with heterozygous genotype, an autosomal
dominant disease will be transmitted 3 out of 4 occasions to their children.
The correct answer is: Autosomal dominant

Question 50 MoK Molecular Genetics 018
Not answered Which of the following specific phobias has a strongly genetic aetiology?
Marked out of 1.00
Select one:
Flag question
Spider phobia
Acrophobia
Animal phobia
Space phobia
Blood injury injection phobia

903
Your answer is incorrect.
Blood injury injection phobia is oddly different from other phobias in that the response to
exposure is not tachycardia and sympathetically driven heart rate, etc. Instead, a fainting
response occurs where the patient may drop fainting with low BP and bradycardia.
The correct answer is: Blood injury injection phobia

Question 51 MoK Neuroanatomy 001
Not answered Which of the following structures is involved in addictive behaviour?

Marked out of 1.00
Select one:
Flag question
Suprachiasmatic nucleus
Nucleus tractus solitarius
Substantia nigra
Ventral tegmentum
Preoptic nucleus

Your answer is incorrect.
All forms of additive behaviors appear to be mediated through at least three structures: the
ventral tegmental area VTA, the nucleus accumbens NA and the septal nucleus. VTA and NA
contain neurons that use dopamine as their major neurotransmitter.
The correct answer is: Ventral tegmentum

Question 52 MoK Neuroanatomy 002
Not answered Which brain structure is spared during normal aging?

Marked out of 1.00
Select one:
Flag question
Cerebellum
Motor neuron

Cerebral cortex
Sub thalamic nuclei
Cranial nerves nuclei

Your answer is incorrect.
Cranial nerve nuclei are spared during the normal ageing process. All the other structures
could be affected.
The correct answer is: Cranial nerves nuclei

904
Question 53 MoK Neuroanatomy 003
Not answered Loss of taste sensation in the anterior part of tongue is related to

Marked out of 1.00
Select one:
Flag question
5th Nerve Palsy
9th Nerve Palsy
8th Nerve Palsy
6th Nerve Palsy
7th Nerve Palsy

Your answer is incorrect.
The facial nerve supplies the taste sensation in the anterior tongue. The chorda tympani (the
branch mediating this sensation) leaves the parent nerve, crossing through the middle ear,
where it can also be damaged by severe infections, etc.
The correct answer is: 7th Nerve Palsy

Question 54 MoK Neuroanatomy 004
Not answered The Kluver­Bucy syndrome is typically associated with lesions of which of the following
structures?
Marked out of 1.00

Flag question Select one:
Amygdala
Medial dorsal thalamus

Lateral hypothalamus
Globus pallidus
Subthalamic nucleus

Your answer is incorrect.
This syndrome is the result of lesions of the temporal lobe in which parts of the amygdala are
involved. Lesions of other regions such as the hypo­ thalamus, cingulate cortex, or septal area
do not produce the Kluver­Bucy syndrome.
The correct answer is: Amygdala

Question 55 MoK Neuroanatomy 005
Not answered The last part of the brain to mature is

Marked out of 1.00
Select one:
Flag question
Posterior parietal pole
Occipital pole

905
Anterior parietal pole
Anterior frontal pole
Hippocampus

Your answer is incorrect.
The last area of the brain to develop is the frontal lobe. The last region of the brain to mature is
pre­frontal cortex (anterior frontal pole). While it may be fully formed in size, various parts
within the structure of the brain continue to develop well into a person's twenties. The
adolescent brain is immature and in a dynamic state of development. Frontal brain
development is also a factor related to impulsive behavior among teens.
The correct answer is: Anterior frontal pole

Question 56 MoK Neurochemistry 001
Not answered Which of the following is the precursor of serotonin?

Marked out of 1.00
Select one:
Flag question
5 hydroxy dopamine
5 Hydroxy indole acetic acid
5 Hydroxy tryptophan
5 Hydroxy tryptamine
Anandamide

Your answer is incorrect.
Note that 5­hydroxy tryptamine is nothing but serotonin itself.
The correct answer is: 5 Hydroxy tryptophan

Question 57 MoK Neurochemistry 002
Not answered The somatodendritic inhibition of 5­HT release is regulated by
Marked out of 1.00
Select one:
Flag question
5­HT1A receptors
5­HT3 receptors
5­HT4 receptors
5­HT5 receptors
5­HT2 receptors

Your answer is incorrect.

906
The 5­HT1a receptor is found at both somatodendritic and post synaptic sites.
Somatodendritic 5­HT1A receptors in the dorsal and median raphe nuclei are autoreceptors
regulating 5­HT neuronal firing. Post synaptic 5HT1A receptors are found in the hippocampus
and periaqueductal grey matter where they regulate behaviours such as resilience,
impulsivity and restraint of excessive response to stress. The 5­HT1A receptor may be an
important target in the action of antidepressants. (Ref: Fundamentals of clinical
psychopharmacology­ 18)
The correct answer is: 5­HT1A receptors

Question 58 MoK Neurochemistry 003
Not answered A 70­year­old woman awakens with inability to walk and is found on the floor by her family.
Her examination shows right face, arm, and leg weakness and numbness. A MRI shows no
Marked out of 1.00
recent changes. What is the most likely cause of her gait problems?
Flag question
Select one:
Parkinsonism
A right hemisphere infarction
A right cerebellar infarction
A left hemisphere infarction
Drug Toxicity

Your answer is incorrect.
The presentation is suggestive of a cerebral infarct, most likely to be on the left side as the
woman is presenting with right sided weakness
The correct answer is: A left hemisphere infarction

Question 59 MoK Neurochemistry 004
Not answered Mr. P is a 77­year­old gentleman suffering from gradually progressive cognitive decline for
last 6 months. He has history of Parkinson's disease for last 3 years and suffers from visual
Marked out of 1.00
hallucinations. The chemical that is mainly involved and deficient in this condition is
Flag question
Select one:
Dopamine
Acetyl choline
Noradenaline
GABA
Serotonin

Your answer is incorrect.

907
The diagnosis is Parkinson's disease dementia. In both Lewy body dementia and Parkinson's
disease dementia, brain acetyl choline levels are reduced similar to Alzheimer's disease.
Therefore acetylcholinesterases (Anti­dementia drugs) could be used in the treatment of Lewy
body dementia.
The correct answer is: Acetyl choline

Question 60 MoK Neurochemistry 009
Not answered Hypothetical use of cholecystokinin has been studied in which of the following disorders?

Marked out of 1.00
Select one:
Flag question
Bulimia
Conduct disorder
Childhood schizophrenia
Autism
ADHD

Your answer is incorrect.
CCK can produce a sense of satiation bringing binges to a conclusion earlier than usual.
The correct answer is: Bulimia

Question 61 MoK Neuropathology 001
Not answered Hirano bodies seen in which one of the following types of dementia?

Marked out of 1.00
Select one:
Flag question
Vascular
Lewy body type
Pick's
Frontotemporal
Alzheimer's

Your answer is incorrect.
Hirano bodies were first described by Hirano in 1965. These are intracellular eosinophilic rod­
shaped structures seen in the neurons of individuals with Alzheimer's and some forms of
Creutzfeldt­Jacob disease (Cartier et al., 1985).
The correct answer is: Alzheimer's

Question 62 MoK Neuropathology 002
Not answered

908
Marked out of 1.00 Tom is a 12­year­old boy with reduced social interactions, poor communication abilities and
repetitive stereotypical behaviour. The macroscopic changes involved in the pathogenesis of
Flag question
this condition would be hypoplasia of the

Select one:
Hippocampus
Fusiform gyrus
Cerebellar vermis

Amygdala
Angular gyrus

Your answer is incorrect.
Reduced social interactions, poor communication abilities and repetitive stereotypical
behaviour are core features of autistic spectrum disorder. In children with autism, hypoplasia
of cerebellar vermis and to some extent the cerebellar hemispheres is documented. Purkinje
cell count in cerebellum is significantly lower
The correct answer is: Cerebellar vermis

Question 63 MoK Neuropathology 003
Not answered In CJD, the amyloid plaques are prominent in which part of the brain?

Marked out of 1.00
Select one:
Flag question
Cerebellum
Raphe nuclei

Corpus callosum
Occipital cortex
Hippocampus

Your answer is incorrect.
CJD is a type of multifocal dementia involving the cerebellar structures, where amyloid
plaques are prominent.
The correct answer is: Cerebellum

Question 64 MoK Neurophysiology 001
Not answered Normal adult enters deep sleep from wakefulness before entering REM sleep. But infants
enter REM sleep directly from wakefulness. When does the adult pattern of sleep develop in a
Marked out of 1.00
child?
Flag question
Select one:
12 months

909
3 years
18 months
5 years
3 months

Your answer is incorrect.
Newborns sleep for 16 ­ 18 hours daily though the longest sleep episode lasts only 2.5 to 4
hours. Newborns have three types of sleep: quiet sleep (similar to NREM), active sleep
(analogous to REM), and indeterminate sleep. Sleep onset occurs through REM, unlike adults
where it occurs through NREM. In infants, each sleep episode is made of only one or two
cycles (instead of 4 or more in adults). This infantile sleep architecture disappears when
circadian rhythms are fully entrained. Circadian rhythm development in the first 1 month
involves the emergence of the 24­hour core body temperature cycle; by 2, months
progression of nocturnal sleeping is noted and in 3 months, melatonin and cortisol rhythms
are established along with NREM in the sleep onset phase.
http://www.ncbi.nlm.nih.gov/books/NBK19956/
The correct answer is: 3 months

Question 65 MoK Neurophysiology 002
Not answered A 40 year old man presents with gradually developing darkening of the skin most visible on
scars, skin folds and pressure points. Following a trivial driving accident, he notices extreme
Marked out of 1.00
fatigue, muscle weakness and weight loss.The most commonly used definitive test for this
Flag question condition is

Select one:
Skin Biopsy
ELISA serology
Prolactin Test
ACTH stimulation test
Fasting glucose

Your answer is incorrect.
This patient has Addison's disease which, due to insidious onset, often goes unnoticed until
something else brings the symptoms to notice. The ACTH stimulation test is the most
commonly used test for diagnosing adrenal insufficiency. In this test, blood cortisol, urine
cortisol, or both are measured before and after a synthetic form of ACTH is given by injection.
The normal response after an ACTH injection is a rise in blood and urine cortisol levels.
People with Addison's disease or long­standing secondary adrenal insufficiency have little or
no increase in cortisol levels.
The correct answer is: ACTH stimulation test

Question 66 MoK Neurophysiology 003

910
Not answered During relaxed periods of wakefulness, a normal individual's EEG will display which of the
Marked out of 1.00
following rhythms?

Flag question
Select one:
Delta rhythm
Gamma rhythm
Alpha rhythm
Beta rhythm
Theta rhythm

Your answer is incorrect.
During states of quiet wakefulness or drowsiness, the EEG pattern becomes slower (8 to 13
Hz with an average amplitude of 50 V) than what is seen during an alert state. This constitutes
the alpha rhythm.
The correct answer is: Alpha rhythm

Question 67 MoK Pharmacology EMI001
Not answered Adverse effects
For each of the following drugs, identify the most important side effect associated with its
Marked out of 3.00
therapeutic use
Flag question

Hypersalivation
Amenorrhoea
Nasal stuffiness
Procyclidine
Polydipsia
Blurred vision
Diastolic Hypertension
Weight loss
Prolactin elevation
Polydipsia
QTc prolongation
Hypersalivation
Glucose dysregulation
Prolactin elevation
Ziprasidone Bradycardia
Diastolic Hypertension
Glucose dysregulation
Blurred vision
Amenorrhoea
Weight loss
Amenorrhoea
QTc prolongation
Glucose dysregulation
Nasal stuffiness
Diastolic Hypertension
Bradycardia
Prolactin elevation
Nasal stuffiness
Olanzapine
Blurred vision
Bradycardia
QTc prolongation
Weight loss
Hypersalivation
Polydipsia

911
Your answer is incorrect.
Procyclidine, Benztropine, Orphenadrine and Biperiden are anticholinergic drugs that can
cause dry mouth, constipation, urinary retention and blurred vision. The latter is associated
with impaired visual accommodation, causing difficulty in reading. 
There are concerns over the use of Ziprasidone, as it can increase QTc. However with this
drug, EPSE is equivalent to placebo, no appreciable weight gain, and no appreciable
anticholinergic side effects are noted. 
Although possible glucose dysregulation and metabolic syndrome are reported with most
atypical antipsychotics, Olanzapine is the most common offender. Metabolic syndrome is a
cluster of disorders comprising of obesity(central and abdominal), dyslipidaemias, glucose
intolerance, insulin resistance (or hyperinsulinaemia) and hypertension.
The correct answer is: Procyclidine – Blurred vision, Ziprasidone – QTc prolongation,
Olanzapine – Glucose dysregulation

Question 68 MoK Pharmacology EMI002
Not answered Mechanism of drug action 
Find the drug that matches each of the following mechanisms of action;
Marked out of 3.00

Flag question
Aripiprazole
Agomelatine
Sildenafil
Phosphodiesterase­5 inhibitor
Rivastigmine
Bupropion
Reboxetine
Duloxetine
Galantamine
Donepezil
Donepezil
Aripiprazole
Mirtazapine
Reboxetine
Butyrylcholinesterase inhibitor
Sildenafil
Mirtazapine
Galantamine
Duloxetine
Bupropion
Duloxetine
Rivastigmine
Rivastigmine
Agomelatine
Bupropion
Dopamine reuptake inhibitor
Mirtazapine
Agomelatine
Reboxetine
Donepezil
Aripiprazole
Galantamine
Your answer is incorrect. Sildenafil
Sildenafil is a potent and selective inhibitor of c­GMP­ specific phosphodiesterase type 5
(PDE5), which is responsible for degradation of c­GMP in corpus cavernosum of the penis.
Rivastigmine inhibits both acetyl and butyrylcholinesterases. Bupropion is a DARI ­ Dopamine
reuptake inhibitor

912
The correct answer is: Phosphodiesterase­5 inhibitor – Sildenafil, Butyrylcholinesterase
inhibitor – Rivastigmine, Dopamine reuptake inhibitor – Bupropion

Question 69 MoK Pharmacology EMI003
Not answered Swapping antidepressants
For each of the following change of antidepressants, choose ONE method of cross tapering
Marked out of 3.00
from options. Each option may be used once, more than once or not at all.
Flag question

Halve the dose of the first drug and start the second drug at ha
Withdraw
Venlafaxine to mirtazapine in a Keep full dose of the first drug and start the second drug at half
patient who has developed severe
Withdraw both the agents
gastrointestinal intolerance.
Withdraw the first and start the second immediately after stoppi
wait for two weeks and then start the second drug
Cross taper the two drugs
Keep full dose of the first drug and start the second drug simult
Withdraw both the agents
Offer combined therapy
Halve the dose of the first drug and start the second drug at ha
Fluoxetine to tranylcypromine due to
Halve the dose of the first drug and start the second drug at full
Offer combined therapy
the inefficacy of the former. wait for five weeks and then start the second drug
Withdraw
Withdraw the first and start the second immediately after stoppi
wait for two weeks and then start the second drug
Cross taper the two drugs
wait for five weeks and then start the second drug
Keep full dose of the first drug and start the second drug at half
Halve the dose of the first drug and start the second drug at full
wait for five weeks and then start the second drug
Paroxetine to fluoxetine due to Keep full dose of the first drug and start the second drug simult
Cross taper the two drugs
anticholinergic side effects of the Keep full dose of the first drug and start the second drug at half
Withdraw the first and start the second immediately after stoppi
former
wait for two weeks and then start the second drug
Keep full dose of the first drug and start the second drug simult
Halve the dose of the first drug and start the second drug at full
Withdraw both the agents
Offer combined therapy
Your answer is incorrect. Withdraw
Halve the dose of the first drug and start the second drug at ha
All answers are based on Maudsley Prescribing Guidelines, 10th edition. 
Venlafaxine has a short half­life. Incases where side effects are intolerable, it must be
withdrawn promptly but the second agent mirtazapine should be started immediately. The risk
of serotonin syndrome is not a significant problem with this approach with VFX and MZP being
often used together as combination therapy. A similar approach can also be used when
stopping one SSRI and starting another, as in case 3. When administered together MAOI and
SSRI can increase the risk of the serotonin syndrome manifold. 
Fluoxetine has a long half­life, and its metabolite stays in the system for a much longer time
(2­4 weeks). To be safe, it is recommended that a five weeks waiting period is applied before
MAOIs are started after a patient has discontinued fluoxetine.
The correct answer is: Venlafaxine to mirtazapine in a patient who has developed severe
gastrointestinal intolerance. – Withdraw the first and start the second immediately after
stopping it, Fluoxetine to tranylcypromine due to the inefficacy of the former. – Withdraw, wait
for five weeks and then start the second drug, Paroxetine to fluoxetine due to anticholinergic
side effects of the former – Withdraw the first and start the second immediately after stopping it

913
Question 70 MoK Neurosciences EMI001
Not answered Research studies in genetics
Select the most appropriate type of study for each description given below
Marked out of 3.00

Flag question
An investigator is interested in
Transgenic studies
calculating the risk of illness within Population studies
families among first, second and third Association studies
degree relatives and compare the Outcome studies
rates with the established risk in the Adoption studies
general population. Genome­wide association studies
Treatment studies
Linkage studies
Association studies
An investigator wants to compare the Family studies
Population studies
likelihood of the observed rate of Pedigree studies
Treatment studies
illness in a given pedigree with those Segregation analysis
Transgenic studies
families that have a differing mode of
transmission
Genome­wide association studies
Linkage studies
Segregation analysis
A researcher has no candidate genes Adoption studies
Association studies
of a priori interest but wants to study Pedigree studies
Adoption studies
the whole genome in large samples Outcome studies
Family studies
of patients and controls to investigate Family studies
Segregation analysis
the most commonly observed
polymorphic variations in the
Outcome studies
patients. Linkage studies
Population studies
Transgenic studies
Genome­wide association studies
Your answer is incorrect. Pedigree studies
Treatment studies
Family studies aid in determining the likelihood of illness among relatives. Segregation
analysis is used to investigate whether familial data for a disorder or trait suggests any
particular modes of inheritance. Both monogenic (Mendelian) or polygenic models can be
tested in this fashion. In addition, age of onset and sex effects can be taken into account
besides the specific genetic hypothesis under consideration. In Genome­Wide Association
Studies (GWAS), a 'candidate gene' approach is not used. Instead, several thousands of
single nucleotide polymorphisms are assayed in thousands of individuals.
The correct answer is: An investigator is interested in calculating the risk of illness within
families among first, second and third degree relatives and compare the rates with the
established risk in the general population. – Family studies, An investigator wants to compare
the likelihood of the observed rate of illness in a given pedigree with those families that have
a differing mode of transmission – Segregation analysis, A researcher has no candidate
genes of a priori interest but wants to study the whole genome in large samples of patients
and controls to investigate the most commonly observed polymorphic variations in the
patients. – Genome­wide association studies

Question 71 MoK Neurosciences EMI002
Not answered Bedside tests of cognition
For each of the following options below, choose the most appropriate test from the given list:
Marked out of 3.00

Flag question

914
A test of visual memory where a child Paired associates test
is shown ten geometric designs, one
Vocabulary test
at a time, and asked to reproduce
each one as exactly as possible on
Trail making test
plain paper from memory. Stanford­Binet test
Raven's progressive matrices
Stroop test
Wisconsin card sorting test
Digit span test.
Raven's progressive matrices
Rey­Osterreith test
Stroop test
A test to check inhibitory control Benton test
Digit span test.
aspect of executive function National adult reading test
Benton test
Rey­Osterreith test
Paired associates test
Wisconsin card sorting test
Stanford­Binet test
Benton test
This test measures the recall of Trail making test
National adult reading test
learned material. It is a complex Vocabulary test
Wisconsin card sorting test
figure test, which distinguishes National adult reading test
Paired associates test
between brain damage and mental
retardation.
Trail making test
Stanford­Binet test
Digit span test.
Raven's progressive matrices
Stroop test
Your answer is incorrect. Vocabulary test
Rey­Osterreith test
Rey­Osterrieth complex figure test is one of the non verbal memory tests. Here the subject is
first asked to copy a complex geometric figure and then to draw from memory after an interval
of 30 minutes. The recall is impaired in patients with dementia and amnesic syndrome. 
The Benton Visual Retention Test (aka Benton Test) is used for individuals aged 8 onwards to
measure visual perception and visual memory. It can also help evaluating possible learning
disabilities. The subject is asked to draw from memory each one of the ten designs that are
shown one at a time. 
When the name of a colour (e.g., "blue," "green," or "red") is written in a colour that is different
from the name (e.g., the word "red" printed in blue ink instead of red ink), naming the actual
colour of the colour­incongruent word is challenging, takes longer and is more error­prone
than namign the colour of a congruent word. This is called Stroop Effect and is utilised in
testing the executive faculty of inhibition (suppression of interferences) and error correction.
The correct answer is: A test of visual memory where a child is shown ten geometric designs,
one at a time, and asked to reproduce each one as exactly as possible on plain paper from
memory. – Benton test, A test to check inhibitory control aspect of executive function – Stroop
test, This test measures the recall of learned material. It is a complex figure test, which
distinguishes between brain damage and mental retardation. – Rey­Osterreith test

Question 72 MoK Neurosciences EMI003
Not answered Localizing signs
Choose two symptoms for each of the following conditions:
Marked out of 6.00

Flag question

915
Bilateral 6th nerve palsy
A malnourished alcoholic. (TWO)
Gerstmann Syndrome
Left 4th nerve palsy
Ipsilateral optic atrophy
Agraphia
Constructional Apraxia
Anosognosia
Gait Ataxia
Constructional Apraxia
Anosmia
Agraphia
A right­handed person with right Anosognosia
parietal lobe lesion. (TWO) Gerstmann Syndrome
Anosmia
Gait Ataxia
Ipsilateral optic atrophy
Bilateral 6th nerve palsy
Ipsilateral optic atrophy
Left 4th nerve palsy
Constructional Apraxia
Gerstmann Syndrome
A frontal meningioma. (TWO)
Bilateral 6th nerve palsy
Agraphia
Anosmia
Anosognosia
Gait Ataxia
Left 4th nerve palsy
Your answer is incorrect.
Case 1: This patient is most likely to have Wernicke's encephalopathy. The classic triad
includes ophthalmoplegia, ataxia and confusion. The most common ophthalmoplegia here is
a bilateral 6th nerve (not 4th nerve) lesion. The 4th nerve may be affected in few cases. 
Case 2: The non­dominant parietal lobe has a significant role in constructional praxis and
also in awareness of illness states. 
Case 3: Frontal meningiomas are associated with anosmia (often present due to the common
olfactory groove distribution of tumours causing pressure) and ipsilateral optic atrophy, on the
side of the tumour. Anosmia with ipsilateral optic atrophy and contralateral papilledema is
also known as Kennedy­Foster syndrome.
The correct answer is: A malnourished alcoholic. (TWO) – Gait Ataxia , Bilateral 6th nerve
palsy, A right­handed person with right parietal lobe lesion. (TWO) – Anosognosia,
Constructional Apraxia, A frontal meningioma. (TWO) – Ipsilateral optic atrophy , Anosmia

Question 73 MoK Neurosciences EMI004
Not answered Inheritance of psychosis
A 26­year­old woman has been admitted to hospital for bizarre delusions, auditory
Marked out of 5.00
hallucinations and disorganized speech. She is also flat in her affect, unkempt and is self­
Flag question neglecting. There is no history of mania or depression. For each question below, choose the
correct answer from the given list:

916
What is the chance of her 12­year­old
11­15%
son inheriting the disorder?
41­45%
6­10%
36­40%
46­50%
1­5%
26­30%
20­25%
11­15%
31­35%
6­10%
She has a sister who has a 6­year­
15­20%
15­20%
old son with Down's syndrome. What
is the chance of him developing the 31­35%
same psychotic disorder? 36­40%
20­25%
46­50%
26­30%
46­50%
41­45%
6­10%
What is the chance of her identical 1­5%
15­20%
twin sister to inherit the disorder? 20­25%
31­35%
41­45%
11­15%
36­40%
31­35%
26­30%
1­5%
If her child's father is also suffering
from the same psychotic disorder,
1­5%
15­20%
what is the risk of inheritance in the 11­15%
child? 41­45%
20­25%
36­40%
46­50%
11­15%
6­10%
20­25%
What is the risk of her non­identical 26­30%
15­20%
twin brother to inherit the disorder? 36­40%
26­30%
31­35%
6­10%
41­45%
1­5%
Your answer is incorrect. 46­50%
Siblings and offspring of patients with schizophrenia have about a 10­fold increase in their
lifetime risk of developing the disorder compared to the general population, whereas in
second­ degree relatives (e.g. nephews, nieces, grandchildren) the risk is about 3­4 times the
population risk. Children with one parent affected by schizophrenia have a 13% risk of
developing the illness. The risk increases with the number of relatives affected, rising from 9%
in a sibling to 16% if a parent and a sibling are affected and 45% if both parents are affected.
A meta­analysis of a large number of European twin studies has produced a monozygotic
concordance rate of 46% compared with a dizygotic concordance rate of 14%. Patients with
Down's syndrome have lower that the average population risk for schizophrenia (choose the
lowest number ­ option A).
The correct answer is: What is the chance of her 12­year­old son inheriting the disorder? – 11­
15%, She has a sister who has a 6­year­old son with Down's syndrome. What is the chance of

917
him developing the same psychotic disorder? – 1­5%, What is the chance of her identical twin
sister to inherit the disorder? – 41­45%, If her child's father is also suffering from the same
psychotic disorder, what is the risk of inheritance in the child? – 41­45%, What is the risk of
her non­identical twin brother to inherit the disorder? – 11­15%

Question 74 MoK Neurosciences EMI005
Not answered Chromosomal profiles 
Identify the karyotypic / genetic profile for each of the following conditions:
Marked out of 3.00

Flag question
Deletion of part of chromosome 22
Mr. X is a 21­year­old tall, thin man47 XXY
with a moderate level of learning Deletion of part of chromosome 5
disabilities especially with reading.
Deletion of part of chromosome 15
He has diabetes and presents with
behavioural outbursts.
46 YY
Triplication of part of chromosome 22
Trisomy 18
Deletion of part of chromosome 7
47 XXY
Miss. Y is a 19­year­old woman with Deletion of part of chromosome 21
46 YY
a moderate learning disability and Deletion of part of chromosome 22
has a developed a schizophrenia­like
Deletion of part of chromosome 15
psychosis. She has cardiac problems
and cleft palate
Deletion of part of chromosome 5
Deletion of part of chromosome 7
Trisomy 18
Mr. Z is an 11­year­old boy with Deletion of part of chromosome 21
growth retardation who is very Trisomy 18
Triplication of part of chromosome 22
sociable with markedly better verbal Triplication of part of chromosome 22
than spatial intelligence. He has 47 XXY
cardiac problems and sensory­motor
Deletion of part of chromosome 15
impairments
Deletion of part of chromosome 21
Deletion of part of chromosome 22
46 YY
Deletion of part of chromosome 5
Deletion of part of chromosome 7

Your answer is incorrect.
Case 1 describes a man with Klinefelter syndrome. Klinefelter syndrome (KS) (47, XXY) is the
most common allosomal disorder in man and is a relatively frequent cause of male infertility
and hypogonadism. Men with 47XXY makeup have an increased risk of diabetes and
metabolic syndrome related to adiponectin dysregulation. 
Case 2 is descriptive of Velocardio Facial Syndrome (VCFS). VCFS was first described by
Kirkpatrick and DiGeorge in 1968 as a constellation of immunologic deficiencies (hence the
name DiGeorge syndrome). It was renamed Shprintzen Syndrome following a thorough
description of the common presenting symptoms by Shprintzen, et al., in 1978 who included
palate anomalies (Velo), congenital cardiovascular defects (Cardio), and mild facial
dysmorphism (Facial) as characteristic features of VCFS . Nearly 25% develop psychosis. It is
associated with a microdeletion in the long arm of chromosome 22 at band 22q11. 
Williams's syndrome (case 3) is a developmental disorder classically characterized by the
triad of typical facial features, infantile hypercalcemia, and supra valvular aortic stenosis.
Deletion of elastin gene at chromosome 7q11 is associated with this syndrome. 
Ref: Morbidity and mortality in Klinefelter syndrome (47,XXY),
http://www.ncbi.nlm.nih.gov/pubmed/21414026 (accessed April 5, 2015).

918
The correct answer is: Mr. X is a 21­year­old tall, thin man with a moderate level of learning
disabilities especially with reading. He has diabetes and presents with behavioural outbursts.
– 47 XXY, Miss. Y is a 19­year­old woman with a moderate learning disability and has a
developed a schizophrenia­like psychosis. She has cardiac problems and cleft palate –
Deletion of part of chromosome 22, Mr. Z is an 11­year­old boy with growth retardation who is
very sociable with markedly better verbal than spatial intelligence. He has cardiac problems
and sensory­motor impairments – Deletion of part of chromosome 7

Question 75 MoK Neurosciences EMI014
Not answered Neurotransmitters & disorders
Identify the disorders associated with each of the neurotransmitters listed below
Marked out of 5.00

Flag question
Schizophrenia
Huntington's disease
Parkinson's disease
Substance P
Feeding
OCD
Circadian function disorders
Depression
Alzheimer's disease
Alzheimer's disease
Migraine
Migraine
Circadian function disorders
Neurotensin
Parkinson's disease
Depression
Huntington's disease
OCD
Feeding
Alzheimer's disease
Schizophrenia
Depression
Migraine
Neuropeptide Y
Schizophrenia
Parkinson's disease
Feeding
Huntington's disease
Circadian function disorders
Migraine
OCD
Schizophrenia
Depression
Feeding
Alzheimer's disease
OCD
Corticotropin­releasing factor Parkinson's disease
Circadian function disorders
Huntington's disease

919
Endocannabinoids
OCD
Circadian function disorders
Huntington's disease
Migraine
Depression
Parkinson's disease
Alzheimer's disease
Schizophrenia
Feeding
Your answer is incorrect.
Substance P is the primary neurotransmitter in most primary afferent sensory neurons (Pain)
and is also seen in the striatonigral pathway. Levels of substance P have been reported to be
markedly reduced in patients with Huntington's chorea. 
Neurotensin coexists with dopamine in some axon terminals. Neurotensin­related peptides
are being investigated in the treatment of psychotic symptoms. 
Neuropeptide Y may play an important role in feeding disorders through its effects on energy
balance. It is secreted by the hypothalamus and in addition to increasing food intake, it
increases the proportion of energy stored as fat and blocks nociceptive signals to the brain. 
Corticotropin releasing factor (CRF) is the neurotransmitter associated with anxiety and
depression, and CRF receptor antagonists are currently under clinical evaluation as
anxiolytics and antidepressants. 
The endocannabinoids are associated with schizophrenia and psychotic disorders.
Anandamide (weak ligand) and 2­arachnidonylglycerol (strong) are formed from arachidonic
acid and ethanolamine ­ belong to endocannabinoid family. The two types of cannabinoid
receptors, central (CB1) and peripheral (CB2), bind tetrahydrocannabinol (THC), the active
ingredient of marijuana. CB1 receptors are found in the brain and are potential targets for the
treatment of both pain and psychosis. CB2 receptors are associated with immune regulation.
The correct answer is: Substance P – Huntington's disease, Neurotensin – Schizophrenia,
Neuropeptide Y – Feeding, Corticotropin­releasing factor – Depression, Endocannabinoids –
Schizophrenia

Finish review

920
 Home Mock Paper Practice Tests

Mock Paper A(2)

Started on Wednesday, 13 May 2015, 2:24 PM
State Finished
Completed on Monday, 18 May 2015, 7:57 AM
Time taken 4 days 17 hours
Overdue 4 days 16 hours
Grade 0.00 out of 100.00
Feedback You are on the right track. Could you spend some more time on your revision?

Question 1 MoK Adverse Effects 003
Not answered What is the incidence of seizures in patients taking clozapine at a dose of more than
600mg/day?
Marked out of 1.00

Flag question Select one:
0.15
0.05
0.02

0.01
0.005

Your answer is incorrect.
Risk of seizures at a dose less than 300 mg/d: 1% to 2%. Dose 300 to 600 mg/d: 3% to 4%.
Dose more than 600 mg/d: 5%.
The correct answer is: 0.05

Question 2 MoK Adverse Effects 007
Not answered Neuroleptic malignant syndrome (NMS) is associated with a mortality rate of

Marked out of 1.00
Select one:
Flag question
15­20%.
5­10%

25­50%
1­2%
2­5%

921
Your answer is incorrect.
5­20% of patients on oral medication and up to 30% of patients on depot formulations who
develop the syndrome will die from the condition if untreated.
The correct answer is: 15­20%.

Question 3 MoK Adverse Effects 015
Not answered Which one among the following is not a recognised side effect of Valproate?

Marked out of 1.00
Select one:
Flag question
Agranulocytosis
Hair loss with curly regrowth

Tremor
Weight gain
Thrombocytopenia

Your answer is incorrect.
Common side effects of sodium valproate include weight gain, nausea, vomiting, diarrhoea,
sedation, tremor and hair loss with curly regrowth.
The correct answer is: Agranulocytosis

Question 4 MoK Adverse Effects 016
Not answered Mr.Z is a 25 year old gentleman being treated for first episode schizophrenia. He was initially
started on Risperidone 2mgs daily which was subsequently increased to 4mgs by week 4.
Marked out of 1.00
Although he was noted to show good improvement with his delusions and hallucinations, the
Flag question nurses on the ward noticed he is generally restless, constantly pacing up and down, foot
stamping when seated, rocking back and forth and fidgeting in his chair. The nurses
wondered whether this could be a side effect of his medication. Which of the following side
effects of treatment is he experiencing?

Select one:
Neuroleptic malignant syndrome
Akathisia
Tardive dyskinesia
Restless leg syndrome
Pseudo Parkinsonism

Your answer is incorrect.

922
Akathisia is subjectively an unpleasant state of motor restlessness, which is seen in 20­25%
of patients prescribed with anti­psychotic medication. It is a subjectively unpleasant state of
inner restlessness where there is a strong desire or compulsion to move. It may become
prominent as the dose of antipsychotic medication is increased. Akathisia can be mistaken for
psychotic agitation and has been linked with suicide and aggression towards others.
The correct answer is: Akathisia

Question 5 MoK Adverse Effects 019
Not answered A significant prolactin elevating effect is seen with one of the following
Marked out of 1.00
Select one:
Flag question
Ziprasidone
Clozapine
Quetiapine
Aripiprazole
Amisulpiride

Your answer is incorrect.
Maudsley Prescribing Guidelines. 9th ed. p 10.
The correct answer is: Amisulpiride

Question 6 MoK Adverse Effects 020
Not answered The antidepressant most associated with QTc prolongation is

Marked out of 1.00
Select one:
Flag question
Venlafaxine
Paroxetine
Mirtazapine
Amitriptyline
Moclobemide

Your answer is incorrect.
The antidepressant class most associated with QTc prolongation is Tricyclic antidepressants
such as Amitriptyline. Moclobemide, citalopram, and venlafaxine have a low effect on QTc
prolongation. Mirtazapine and SSRIs (except citalopram) have no effect on QTc prolongation
The correct answer is: Amitriptyline

Question 7 MoK Adverse Effects 021
Not answered

923
Marked out of 1.00 Which of the following psychotropics has a tendency to reduce gastrointestinal transit with a
Flag question
resultant slow absorption when given orally?

Select one:
Amitriptyline
Risperidone
Diazepam

Fluoxetine
Lithium carbonate

Your answer is incorrect.
Amitriptyline, due to its atropine­like anticholinergic action, can slow GI transit with a resultant
slow absorption. However, nearly 95% of the orally administered drug gets absorbed.
The correct answer is: Amitriptyline

Question 8 MoK Adverse Effects 022
Not answered Mrs. Y is a 28­year­old pregnant woman with severe depression. The ideal anti­depressant
treatment of choice would be
Marked out of 1.00

Flag question Select one:
Venlafaxine
Reboxetine

Paroxetine
Fluvaxamine
Amitryptyline

Your answer is incorrect.
The antidepressants recommended for treatment of depression in pregnancy are
Amitriptyline, nortryptyline, imipramine and Fluoxetine
The correct answer is: Amitryptyline

Question 9 MoK Adverse Effects 023
Not answered A 65­year­old man develops recurrent UTI since starting an antidepressant. He also gives a
history suggestive of new onset prostatism. But prostate examination shows no enlargement.
Marked out of 1.00
The most likely offending agent is
Flag question
Select one:
Nefazadone
Reboxetine

924
Trazodone
Mirtazapine
Amitryptyline

Your answer is incorrect.
Drugs with significant anticholinergic properties such as amitriptyline are notorious to cause
signs of prostatism in elderly males. They may produce urinary retention and sometimes
bladder distension.
The correct answer is: Amitryptyline

Question 10 MoK Adverse Effects 024
Not answered Tom is an inpatient on a psychiatric ward. He has a history of psychotic depression and is on
combination treatment with antipsychotics and antidepressants. He appears slightly flushed,
Marked out of 1.00
with dry mouth, dry skin, feeling hot, and confused. He is exhibiting features of
Flag question
Select one:
SSRI discontinuation syndrome
Neuroleptic malignant syndrome
Idiopathic parkinsonism
Anticholinergic syndrome

Serotonin syndrome

Your answer is incorrect.
Anticholinergic syndrome results from the inhibition of cholinergic neurotransmission at
muscarinic receptor sites. Psychotropics such as phenothiazines, tricyclic antidepressants
and antiparkinsonian agents have anticholinergic properties. When they are administered in
combination, the action of these drugs becomes additive resulting in anticholinergic
syndrome. Features of a mild organic brain syndrome, which includes disorientation, difficulty
in concentrating and impaired short­term memory are seen, especially noticeable at nighttime.
Other physical symptoms include flushing, dry skin and mucosa, mydriasis with loss of
accommodation and fever (Ref: Kaplan & Sadock Synopsis of Psychiatry 9th ed. Pg 1170­
1172)
The correct answer is: Anticholinergic syndrome

Question 11 MoK Adverse Effects 025
Not answered Which drug increases the risk of bleeding in patients on SSRIs?
Marked out of 1.00
Select one:
Flag question
Paracetamol
Aspirin
Naproxen

925
Ibuprofen

Diclofenac

Your answer is incorrect.
There is an increased risk of bleeding when SSRIs are taken with antiplatelet drugs such as
low­dose aspirin or clopidogrel and non­steroidal anti­inflammatory drugs (NSAIDs).
However, the risk is particularly high with aspirin than other NSAIDs. A large Canadian study
found patients taking both aspirin and an SSRI had a 42% increased risk of a bleeding
episode ­ gastrointestinal bleeding, hemorrhagic stroke or other forms of bleeding that
required hospital admission ­ compared with those taking aspirin alone. (Labos et al., 2011:
http://www.cmaj.ca/content/early/2011/09/26/cmaj.100912)
The correct answer is: Aspirin

Question 12 MoK Adverse Effects 026
Not answered A 33 year old man is admitted to Psychiatric Intensive Care Unit with an acute relapse of
schizophrenia. He is started on a typical antipsychotic medication after which his symptoms
Marked out of 1.00
reduce in intensity. But since this new treatment was started, he complains of feeling restless,
Flag question and is seen pacing up and down a lot. When you interview him, he appears to be shuffling
often and not able to stay for long in the interview room. If you want to prescribe propranolol to
this patient, which of the following medical condition will be a contraindication for the
prescription?

Select one:
Asthma
Obesity
Hypertension

Akathisia
Elevated eosinophils

Your answer is incorrect.
Propranolol and other beta­blockers are recommended to treat akathisia, but beta­receptor
blockade causes bronchospasm, and so are contraindicated in asthma.
The correct answer is: Asthma

Question 13 MoK Adverse Effects 027
Not answered Which of the following is not a feature of Neuroleptic Malignant Syndrome?

Marked out of 1.00
Select one:
Flag question
Incontinence
Bradycardia

Tremor

926
Rigidity
Diaphoresis

Your answer is incorrect.
Bradycardia is not a feature of NMS. Neuroleptic Malignant Syndrome can occur at anytime
during treatment with neuroleptics. It consists of the tetrad of extreme hyperthermia, severe
muscular rigidity and confusion, and autonomic fluctuations (BP and pulse rate). Other clinical
features of neuroleptic malignant syndrome include the following diaphoresis, dysphagia,
tremor, shuffling gait, tachycardia, dyspnoea, incontinence, delirium progressing to lethargy,
stupor, coma
The correct answer is: Bradycardia

Question 14 MoK Adverse Effects 028
Not answered Which of the following is the most likely change seen in the ECG of patients taking lithium?
Marked out of 1.00
Select one:
Flag question
Flattened trace
ST elevation
Prolonged QRS
Prolonged QTc
Changes in T wave morphology

Your answer is incorrect.
The most frequently observed change during treatment was flattening or inversion of the T
wave. Heart rate decreased, and P­Q interval increased slightly during therapy. There was no
change in the S­T segment or the Q­T interval corrected for heart rate. Well monitored lithium
treatment of patients with serum lithium concentrations of about 0.6 mmol/l does not cause
serious ECG abnormalities. In the absence of symptoms or signs of heart disease, routine
monitoring of ECG is not necessary during lithium treatment. (Ref: Bucht G, Smigan L, Wahlin
A, Eriksson P. ECG changes during lithium therapy. A prospective study. Retrieved from
http://onlinelibrary.wiley.com/doi/10.1111/j.0954­6820.1984.tb03777.x/)
The correct answer is: Changes in T wave morphology

Question 15 MoK Adverse Effects 029
Not answered Patients on tranylcypromine should not take an excess of
Marked out of 1.00
Select one:
Flag question
Swede
Banana
Cheese
Potatoes

927
Salt

Your answer is incorrect.
Cheese can induce tyramine related reaction in those on MAOIs.
The correct answer is: Cheese

Question 16 MoK Basic Pharmacology 006
Not answered Which among the following antipsychotic drugs causes prolongation of QT­c interval on the
ECG?
Marked out of 1.00

Flag question Select one:
Clozapine
Olanzapine
Haloperidol
Chlorpromazine

Thioridazine

Your answer is incorrect.
The effect on QTC prolongation is high for some psychotropic drugs like Thioridazine,
Pimozide, Sertindole, and any intravenous antipsychotic. There is a moderate effect on QT­c
prolongation for psychotropic drugs such as chlorpromazine, quetiapine, ziprasidone and
tricyclic antidepressants.
The correct answer is: Thioridazine

Question 17 MoK Pharmacodynamics 006
Not answered Which of the following is not true regarding drugs used to prevent relapse in alcohol
dependence?
Marked out of 1.00

Flag question Select one:
Acamprosate interacts with Disulfiram
Disulfiram inhibits metabolism of warfarin
Metronidazole can cause disulfiram like reactions with alcohol
Alcohol should be avoided for 1week following cessation of disulfiram
Acamprosate acts on NMDA receptors

Your answer is incorrect.

928
Acamprosate does not interact with Disulfiram. It is believed to act through enhancing GABA
transmission in the brain. It is licensed to prevent relapse to alcohol use and has been found
to have a modest treatment effect.
The correct answer is: Acamprosate interacts with Disulfiram

Question 18 MoK Pharmacodynamics 007
Not answered Choose one drug that modulates melatonin system directly.

Marked out of 1.00
Select one:
Flag question
Trazodone
Zopiclone
Agomelatine
Perphenazine
Galantamine

Your answer is incorrect.
Agomelatine is a new antidepressant with a novel mechanism of action. It acts as an agonist
to both the melatonin receptors MT1 and MT2. It is also an antagonist to the 5HT2C serotonin
receptor.
The correct answer is: Agomelatine

Question 19 MoK Pharmacodynamics 008
Not answered Which of the following accurately reflects the activity of benzodiazepines?
Marked out of 1.00
Select one:
Flag question
Reduce GABA transmission
Block sodium reuptake
Allosteric modification of GABA receptor
Substitute for GABA
Facilitate action potential

Your answer is incorrect.
Benzodiazepines (BDZs) bind to the gamma sub­unit of the GABA­A receptor. Their binding
causes an allosteric (structural) modification of the receptor that results in an increase in
GABA­A receptor activity. BDZs do not substitute for GABA, which bind at the alpha sub­unit,
but increase the frequency of channel opening events which leads to an increase in chloride
ion conductance and inhibition of the action potential. (Ref: From CNSforum: Basic and
clinical pharmacology, 8th edition. Katzung BG. USA: The McGraw Hill Companies, Inc,
2001:364­381.)
The correct answer is: Allosteric modification of GABA receptor

929
Question 20 MoK Pharmacodynamics 009
Not answered A 62 year old patient with insomnia and depression develops priapism when trazodone is
prescribed. This is possibly mediated via
Marked out of 1.00

Flag question Select one:
Beta 1 agonism
Alpha 2 antagonism
Alpha 1 antagonism
Alpha 2 agonism
Alpha 1 agonism

Your answer is incorrect.
Alpha 1 blockade causes priapism when using trazodone or risperidone.
The correct answer is: Alpha 1 antagonism

Question 21 MoK Pharmacodynamics 010
Not answered Impotence as a side effect of antipsychotics is caused due to which of the following
mechanisms?
Marked out of 1.00

Flag question Select one:
Alpha­1 adrenergic blockade
Histaminergic receptor blockade

Nicotinic receptor blockade
Alpha­1 adrenergic blockade
Muscarinic receptor blockade

Your answer is incorrect.
Other side effects caused due to alpha 1 adrenergic blockade includes sedation, postural
hypotension, and ejaculatory failure.Alpha 1 adrenergic blockade leads to a moderately high
reported incidence of ejaculatory problems such as retrograde ejaculation and rarely
priapism. Sexual dysfunction has been reported as a side effect of all antipsychotics and up to
45% of people taking conventional antipsychotics experience sexual dysfunction.
The correct answer is: Alpha­1 adrenergic blockade

Question 22 MoK Pharmacodynamics 011
Not answered The mechanism of action of Varenicline is

Marked out of 1.00
Select one:
Flag question
Alpha­2, beta­4, nicotinic acetylcholine receptor partial agonist

930
Alpha­2, beta­2, nicotinic acetylcholine receptor partial agonist
Alpha­4, beta­2, nicotinic acetylcholine receptor partial antagonist
Alpha­4, beta­2, nicotinic acetylcholine receptor partial agonist
Alpha­2, beta­4, nicotinic acetylcholine receptor partial antagonist

Your answer is incorrect.
Varenicline is a partial agonist at the alpha­4 beta­2 unit of nicotinic acetylcholine receptor. It
assists smoking cessation by relieving nicotine withdrawal symptoms and reducing the
rewarding properties of nicotine
The correct answer is: Alpha­4, beta­2, nicotinic acetylcholine receptor partial agonist

Question 23 MoK Pharmacodynamics 012
Not answered Which of the following medications have dual action in prophylaxis of influenza as well as in
treatment of Parkinson's disease?
Marked out of 1.00

Flag question Select one:
Carbidopa
Levodopa
Orphenadrine
Bromocriptine
Amantadine

Your answer is incorrect.
Amantadine is an antiviral agent and prevents viral uncoating. It also induces dopamine
release from nerve terminals and so is used in Parkinsonism.
The correct answer is: Amantadine

Question 24 MoK Pharmacokinetics 007
Not answered What is the half­life of risperidone?

Marked out of 1.00
Select one:
Flag question
2 hours
5 hours
150 hours

8 hours
24 hours

931
Your answer is incorrect.
The half­life of risperidone and of its active metabolite is about 24 hours.
The correct answer is: 24 hours

Question 25 MoK Pharmacokinetics 008
Not answered Mr. Z is a 43 year old gentleman who has a 15 year history of schizophrenia and poor
adherence with antipsychotic treatments. He has been stabilised on 15mgs of Olanzapine in
Marked out of 1.00
the hospital and he has previously done well on Flupentixol injections. He has agreed to
Flag question switch to Flupentixol injection once a fortnight. The test dose was administered. What is the
most likely amount of time he will need to continue taking the oral Olanzapine?

Select one:
3­ 6 months
2­ 3 months
2­3 weeks
1 month
1­ 2 weeks

Your answer is incorrect.
The depot medication may require 3­6 months to reach a steady plasma level, and, therefore,
oral antipsychotic medication should be continued until steady state plasma level is achieved.
Attainment of peak plasma levels, therapeutic effect and steady state plasma levels are all
delayed with depot injection. The use of adjunctive oral medication to assess depot
requirements may be helpful and oral anti psychotic cover is sometimes required for longer
periods of time. Moreover, in most clinical trials relapse seems only to occur six months after
withdrawing depot therapy; roughly the time required to clear steady state depot drug levels
from the blood.
The correct answer is: 3­ 6 months

Question 26 MoK Pharmacokinetics 009
Not answered The dosage of duloxetine used for the treatment of diabetic neuropathy is

Marked out of 1.00
Select one:
Flag question
60­120 mg
5­10 mg
10­20 mg
120­240 mg
30­60 mg

Your answer is incorrect.

932
Duloxetine inhibits the re­uptake of both serotonin and noradrenaline. It is used for the
treatment of major depressive disorder (60 mg once daily), generalized anxiety disorder (60­
120 mg) and the treatment of diabetic neuropathy (60­120 mg in divided doses).
The correct answer is: 60­120 mg

Question 27 MoK Pharmacokinetics 010
Not answered Aripiprazole has an elimination half life of
Marked out of 1.00
Select one:
Flag question
3.5 hours
2 hours
12 hours
24 hours
75 hours

Your answer is incorrect.
Aripiprazole is metabolized extensively in humans, with a mean terminal half­life of 60 hours
(up to 76 hours) after 14 days of administration. The half­life of its major metabolite,
dehydroaripiprazole, is 96 hours.
The correct answer is: 75 hours

Question 28 MoK Pharmacokinetics 011
Not answered Which of the following is considered to be the therapeutic blood level of clozapine?

Marked out of 1.00
Select one:
Flag question
Above 550 ng/mL
above 450 ng/mL
above 350 ng/mL
Above 250 ng/mL
Above 150 ng/mL

Your answer is incorrect.
Blood levels above 350 ng/mL are believed to be correlated with therapeutic response.
The correct answer is: above 350 ng/mL

Question 29 MoK Pharmacokinetics 012
Not answered Which of the following follows zero order kinetics?

Marked out of 1.00
Select one:

933
Flag question
Sodium valproate
Lorazepam
Alcohol

Haloperidol
Amitriptyline

Your answer is incorrect.
A drug with zero order kinetics is more likely to achieve toxic levels than a drug with first order
kinetics because its rate of elimination cannot increase beyond a certain level. E.g. Alcohol.
The correct answer is: Alcohol

Question 30 MoK Pharmacokinetics 015
Not answered Which of the following antipsychotics is primarily renally excreted?

Marked out of 1.00
Select one:
Flag question
Olanzapine
Amisulpride
Clozapine
Haloperidol
Risperidone

Your answer is incorrect.
Amisulpride is primarily renally excreted. Nevertheless, there is a limited experience of using
any antipsychotics it is in renal disease and, therefore, best to avoid them if possible in well
established renal failure.
The correct answer is: Amisulpride

Question 31 MoK Pharmacokinetics 017
Not answered A 23 year old patient is observed to consume high amounts of coffee whilst in the ward. She is
being treated for a depressive episode. Which of the following statement is likely to be true in
Marked out of 1.00
this case?
Flag question
Select one:
Caffeine is poorly absorbed orally
Caffeine increases sodium retention
Caffeine reduces renal clearance of lithium
Caffeine can interact with drugs that are metabolised by CYP1A2
Caffeine suppresses weight gain

934
Your answer is incorrect.
Controlling caffeine intake when investigating renal function using lithium clearance is
important as caffeine has a natriuretic effect and increases lithium clearance as well. It is
metabolised by CYP1A2 and can interact with drugs sharing this mechanism Clin Sci (Lond).
2002 Nov;103(5):461­6.
The correct answer is: Caffeine can interact with drugs that are metabolised by CYP1A2

Question 32 MoK Pharmacokinetics 018
Not answered Which of the following drugs when combined with lithium, do NOT cause an increase in
lithium levels per se but still cause increased incidence of neurotoxicity?
Marked out of 1.00

Flag question Select one:
ACE inhibitors
Fluoxetine
Frusemide
Calcium channel blockers
NSAIDs

Your answer is incorrect.
See course notes on mood stabilisers, paper 2 pharmacology. Haloperidol and clozapine can
produce increased neurotoxic effects with lithium without altering the plasma levels.
The correct answer is: Calcium channel blockers

Question 33 MoK Pharmacokinetics 003
Not answered Which of the following statement is correct with regard to pharmacokinetics?
Marked out of 1.00
Select one:
Flag question
Chlorpromazine decreases levels of tricyclics
Most orally administered antipsychotics undergo first pass metabolism
Depots achieve peak plasma levels on the third day after injection
First pass metabolism enables easier entry through blood brain barrier
Extensive first pass metabolism increases bioavailability

Your answer is incorrect.
First pass metabolism will serve to reduce bioavailability. It does not influence passage via
BBB. CPZ inhibits TCA metabolism thereby increasing TCA levels.
The correct answer is: Most orally administered antipsychotics undergo first pass metabolism

935
Question 34 MoK Applied Neuroscience 010
Not answered Which one of the following is not a frontal lobe function test?

Marked out of 1.00
Select one:
Flag question
Tower of London
Verbal fluency
Luria's motor tests.
California Verbal Learning Test
Cognitive estimates

Your answer is incorrect.
Frontal lobe tests include Stroop test, Halstead trial making, cognitive estimates, Tower of
London, Wisconsin card sorting test, verbal fluency, Luria's motor tests and multiple errands
tests. The California Verbal Learning Test (CVLT) is useful to asses verbal memory. Though it
may be affected in patients with frontal lesions, CVLT is not a specific frontal test.
The correct answer is: California Verbal Learning Test

Question 35 MoK Applied Neuroscience 011
Not answered Aura is a commonly occurring phenomenon seen in which type of seizures?

Marked out of 1.00
Select one:
Flag question
Grandmal seizures
Simple partial seizures
Complex partial seizures
Petitmal seizures
Myoclonic seizures

Your answer is incorrect.
Auras are particularly common in temporal lobe complex partial epilepsy, generally last only a
few seconds. Auras are typically stereotyped, e.g., autonomic or visceral aura (epigastric
sensations) and may consist of depersonalization, derealisation and affective changes (like
anxiety, euphoria), perceptual experiences like illusions and hallucinations.
The correct answer is: Complex partial seizures

Question 36 MoK Applied Neuroscience 012
Not answered In Broca's area lesion, which of the following features is seen?

Marked out of 1.00
Select one:
Flag question
Naming is preserved

936
Comprehension is preserved
Inflection is preserved
Repetition is preserved
Fluency is preserved

Your answer is incorrect.
Patients with Broca's aphasia produce slow, halting speech that is rarely grammatical. Typical
Broca's aphasics eliminate inflections such as ­ed and words not central to the meaning of the
sentence, such as the and and
The correct answer is: Comprehension is preserved

Question 37 MoK Applied Neuroscience 013
Not answered A 50­year­old woman began complaining of double vision and blurry vision 3 months ago and
has since had diminishing interaction with her family, a paucity of thought and expression,
Marked out of 1.00
and unsteadiness of gait. Her whole body appears to jump in the presence of a loud noise. An
Flag question MRI scan and routine CSF examination are unremarkable. What is the most appropriate
diagnosis?

Select one:
Creutzfeldt­Jakob disease
Alzheimer's disease
Normal­pressure hydrocephalus
Parkinson's disease
Transient global amnesia

Your answer is incorrect.
The neurological com­ plaints occurring early in the course of Creutzfeldt­Jakob disease are
often cerebellar or visual. Patients may complain of ataxia, clumsiness, or dysarthria, as well
as diplopia, distorted vision, blurred vision, field defects, changes in color perception, and
visual agnosia. Ultimately cortical blindness may occur.
The correct answer is: Creutzfeldt­Jakob disease

Question 38 MoK Applied Neuroscience 014
Not answered Raclopride is a compound used in imaging brain receptors. Which of the following receptors
are blocked by raclopride?
Marked out of 1.00

Flag question Select one:
D2 receptors
D1 autoreceptors
5HT2C receptors

937
5HT3 receptors
5HT2A receptors

Your answer is incorrect.
Raclopride is a D2 antagonist that can be radiolabelled with the carbon­11 radioisotope for
use in positron emission tomography (PET) scanning to assess the distribution of D2
receptors and the degree of endogenous dopamine binding to this receptor.
The correct answer is: D2 receptors

Question 39 MoK Applied Neuroscience 015
Not answered Performance sub­sets of WAIS­R include all but
Marked out of 1.00
Select one:
Flag question
Digit span
Digit symbol
Picture arrangement
Picture completion
Block design

Your answer is incorrect.
Verbal subtests include similarities, arithmetic, digit span, vocabulary, information and
comprehension tests. Performance subtests include picture arrangement, block design,
picture completion, digit symbol, matrix reasoning (replaces object assembly).
The correct answer is: Digit span

Question 40 MoK Applied Neuroscience 016
Not answered Which one of the following tests measures non­verbal IQ?

Marked out of 1.00
Select one:
Flag question
Digit span
Digit Symbol
Vocabulary
Arithmetic
Information

Your answer is incorrect.

938
WAIS has both verbal and performance tests. Verbal tests = similarities, arithmetic, digit span,
vocabulary, information and comprehension. Performance tests = picture arrangement, block
design, picture completion, digit symbol, matrix reasoning (replaces object assembly in newer
versions)
The correct answer is: Digit Symbol

Question 41 MoK Applied Neuroscience 017
Not answered Patients with Gerstmann's syndrome usually demonstrate which of the following?
Marked out of 1.00
Select one:
Flag question
Dominant parietal lobe signs
Aphasia
Hemineglect
Cortical blindness
Dressing apraxia

Your answer is incorrect.
Damage to the inferior parietal lobule of the dominant hemisphere results in Gerstmann's
syndrome. It is characterized by finger agnosia, right­left disorientation, dysgraphia and
dyscalculia.
The correct answer is: Dominant parietal lobe signs

Question 42 MoK Applied Neuroscience 018
Not answered Epileptic automatisms usually lasts for

Marked out of 1.00
Select one:
Flag question
Few minutes to hours
Few days to months
Few seconds to minutes
Few hours to days
Few months to years

Your answer is incorrect.
Epileptic automatism is a state of clouding of consciousness which occurs during or
immediately after a seizure. The impairment of awareness varies. The individual retains
control of posture and muscle tone but performs simple or complex movements without being
aware of what is happening. To the onlooker, the patient appears confused, and there is
subsequent amnesia for the episode. Simple stereotyped behaviours (gesturing, grasping, lip­
smacking and chewing movements) are often exhibited lasting few seconds to minutes. Very

939
occasionally, automatisms are prolonged (fugue states), or complex actions are carried out. If
violent, these are never premeditated, seldom goal­directed, rarely involves use of complex
tools/weapons and are especially likely if restraining was attempted
The correct answer is: Few seconds to minutes

Question 43 MoK Molecular Genetics 004
Not answered If a 34 year old mother suffers from schizophrenia, what is the lifetime morbid risk for her only
child for schizophrenia?
Marked out of 1.00

Flag question Select one:
5 to 10%
10 to 15%
30 to 40%
20 to 30%
Less than 1%

Your answer is incorrect.
Results of the Copenhagen High­Risk Study (207 children of schizophrenic mothers and 104
control children followed since 1962) indicated that 16.2% of the high­risk children versus
1.9% of the control group developed schizophrenia, and another 4.6% developed a related
illness (versus 0.9%. 10­15% is the closest answer to this question.
The correct answer is: 10 to 15%

Question 44 MoK Molecular Genetics 007
Not answered Mrs. X has adopted a child from family of alcohol dependent parents. The risk of her adopted
child suffering from alcohol dependence is increased by
Marked out of 1.00

Flag question Select one:
12 times
8 times
4 times
20 times
10 times

Your answer is incorrect.
Adoptees have the risk of alcoholism proportional to the biological family (1.6 to 3.6 times
greater in adopted away children of alcoholics). Close family members have 4­fold increased
risk of alcoholism.
The correct answer is: 4 times

940
Question 45 MoK Molecular Genetics 008
Not answered The odds of having panic disorder is increased in first degree relatives of patients with panic
disorder compared to general population by a factor of
Marked out of 1.00

Flag question Select one:
18­20 times
2­3 times
10­12 times
4­8 times
No increase in prevalence

Your answer is incorrect.
Several family studies have documented an increased panic disorder risk among affected
individuals' relatives. Among relatives, the prevalence varies between 6 to 17%, which
translates to an average of 4­8 times increase in the risk. If the index cases have onset ages
below 20, the risk appears to be much more (17­times).
(http://www.ncbi.nlm.nih.gov/pmc/articles/PMC3102861/)
The correct answer is: 4­8 times

Question 46 MoK Molecular Genetics 009
Not answered A 24 year old woman suffers from bipolar affective disorder. The risk of her twin brother
developing bipolar affective disorder is
Marked out of 1.00

Flag question Select one:
Above 90%
40­50%
0­5%

5­15%
65­70%

Your answer is incorrect.
The risk in dizygotic twins is 5­15%. ('Dizygotic' as it is a twin 'brother'. Twin sister could have
been either monozygotic or dizygotic)
The correct answer is: 5­15%

Question 47 MoK Molecular Genetics 015
Not answered Patients with Down's syndrome have higher risk of Alzheimer's disease. Which of the
following gene is implicated in this relationship?
Marked out of 1.00

Flag question Select one:

941
Presenilin 1
Presenilin 2
Amyloid Precursor Protein

Tau
Apo E4

Your answer is incorrect.
A pathological cascade in both Alzheimer's and Down's ­related dementia could be triggered
by alterations in APP processing. Several genes that might play a role in the development of
AD are found in chromosome 21. Among them are the amyloid precursor protein and the
overexpression of the APP gene might be related to the overproduction of the major protein
observed in the senile plaque, the A­beta (1­42) peptide, which is considered to be one of the
important factors leading to the development of the pathology in Down syndrome with
dementia (Alzheimer Disease in Down Syndrome ­ Medscape Reference. Retrieved from
http://emedicine.medscape.com/article/1136117­overview)
The correct answer is: Amyloid Precursor Protein

Question 48 MoK Molecular Genetics 019
Not answered The isoforms A and B of enzyme Monoamine oxidase are encoded on chromosomes

Marked out of 1.00
Select one:
Flag question
21 and 22 respectively
Chromosomes 1 and 14 respectively
X and 21 respectively
Both encoded on chromosome X
Both encoded on chromosome 14

Your answer is incorrect.
Both isoforms of MAO are encoded on chromosome X.
http://www.nature.com/nrg/journal/v6/n10/full/nrg1687.html
The correct answer is: Both encoded on chromosome X

Question 49 MoK Molecular Genetics 020
Not answered A 25­year­old gentleman has moderate level of learning disability with expressive language
disorder, social anxiety, impulsivity, marked inattention. Molecular genetic testing may show
Marked out of 1.00
an expansion of which repeats;
Flag question
Select one:
CAG
CCCG

942
CTG
CGG
GCC

Your answer is incorrect.
These are behavioural phenotypes seen in Fragile X syndrome. The syndrome is associated
with a large sequence of triplet repeats (CGG) at a fragile site on the X chromosome (Xq27.3)
The correct answer is: CGG

Question 50 MoK Molecular Genetics 021
Not answered The presence of 2 or more cell lines from different zygotes in a single individual is called
Marked out of 1.00
Select one:
Flag question
Mosaicism
Chimerism
Translocation
Transcription

Aneuploidy

Your answer is incorrect.
A chimera or chimaera is a single organism (usually an animal) that is made up of various
genetically distinct cells derived from different zygotes. In contrast to mosaic organisms who
have different cellular lineages with distinct genetic composition, but all cells originating from
the same zygote, chimeric organisms are formed from more than one zygote. Chimeras do not
occur naturally; they are formed from four parent cells (two fertilized eggs or early embryos)
often fused together in laboratory conditions.
The correct answer is: Chimerism

Question 51 MoK Molecular Genetics 022
Not answered Transcriptome is a

Marked out of 1.00
Select one:
Flag question
Statistical test
Type of plasmid
Collection of expressed m­RNA
Positional cloning

Stretch of DNA

943
Your answer is incorrect.
A transcriptome is different from a genome, the latter being the entire DNA sequence of an
organism. A transcriptome represents the very small percentage of the genome ­ less than 5
percent in humans ­ that is transcribed into RNA molecules. A gene may produce many
different types of mRNA molecules, so a transcriptome is much more complex than the
genome that encodes it.
The correct answer is: Collection of expressed m­RNA

Question 52 MoK Molecular Genetics 023
Not answered A nonsense mutation refers to

Marked out of 1.00
Select one:
Flag question
A point mutation
Substitution of stop codons
Destruction of stop codons
Deletion of stop codons
Creation of stop codons

Your answer is incorrect.
A nonsense mutation prematurely introduces a stop codon in the transcribed mRNA resulting
in an incomplete and often nonfunctional protein.
The correct answer is: Creation of stop codons

Question 53 MoK Molecular Genetics 024
Not answered The gene most strongly implicated in ADHD is

Marked out of 1.00
Select one:
Flag question
Dopamine 2 receptor gene
Dopamine 3 receptor gene
Dopamine 4 receptor gene
Dopamine 1 receptor gene
Dopamine 5 receptor gene

Your answer is incorrect.
The gene most strongly implicated in ADHD is the D4 dopamine receptor gene DRD4. The
distribution of DRD4 mRNA in the brain suggests that it plays a role in cognitive and executive
functions. These functions are implicated in the pathophysiology of ADHD (Patterson 1999)
[Retrieved from http://apt.rcpsych.org/content/aptrcpsych/16/2/96.full.pdf]
The correct answer is: Dopamine 4 receptor gene

944
Question 54 MoK Neuroanatomy 006
Not answered The structure of the brain involved in taste sensation is

Marked out of 1.00
Select one:
Flag question
Amygdala
Hippocampus
Prefrontal cortex
Entorhinal cortex
Anterior insula

Your answer is incorrect.
The primary gustatory cortex is responsible for taste perception. It consists of two
substructures: the anterior insula on the insular lobe and the frontal operculum on the inferior
frontal gyrus of the frontal lobe. In each hemisphere of the mammalian brain, the insular cortex
(often called insular lobe) is a portion of the cerebral cortex folded deep within the lateral
sulcus between the temporal lobe and the frontal lobe.
The correct answer is: Anterior insula

Question 55 MoK Neuroanatomy 008
Not answered Which of the following areas of the brain does not lack blood brain barrier?

Marked out of 1.00
Select one:
Flag question
Median eminence
Posterior pituitary

Area postrema
Subfornical organ
Anterior pituitary

Your answer is incorrect.
The anterior pituitary does not lack BBB. There are some areas of the brain that do not have a
blood­ brain barrier. These are so­called circum­ventricular organs e.g. subfornical organ,
area postrema (chemo receptor trigger zone), median eminence and posterior pituitary. Here
the capillaries are fenestrated like those in peripheral tissues
The correct answer is: Anterior pituitary

Question 56 MoK Neuroanatomy 009
Not answered The white matter tract connecting Wernicke's are and Broca's area is called

Marked out of 1.00
Select one:
Flag question
Cingulate gyrus

945
Anterior commissure
Arcuate fasciculus
Habenular commissure

Planum temporale

Your answer is incorrect.
In the dominant hemisphere, arcuate fasciculus connects Wernicke's area and Broca's area.
The function of the arcuate fasciculus in right hemisphere is unclear. Lesions of dominant
arcuate fasciculus can cause conduction aphasia.'
http://www.lib.mcg.edu/edu/eshuphysio/program/section8/8ch15/s8c15_14.htm
The correct answer is: Arcuate fasciculus

Question 57 MoK Neuroanatomy 010
Not answered Which hypothalamic nucleus regulates prolactin secretion?
Marked out of 1.00
Select one:
Flag question
Medial dorsal nucleus
Suprachiasmatic nucleus
Preoptic nucleus
Subthalamic nucleus
Arcuate nucleus

Your answer is incorrect.
The prolactin­releasing factor is found in the arcuate n ucleus of the hypothalamus. This
activates the lactotrophic cells of the anterior pituitary. Dopamine, via the tuberoinfundibular
dopaminergic system, regulates prolactin secretion through direct projection to the pituitary.
Prolactin­secreting microadenomas can be treated by the drug bromocriptine, a dopamine
agonist (often used in the treatment of Parkinson's ). By giving a dopamine agonist, which acts
as Prolactin Inhibiting Factor, the tumour cells are inhibited, and eventually the tumour
shrinks.
The correct answer is: Arcuate nucleus

Question 58 MoK Neuroanatomy 011
Not answered Which of the following cells help in regulating potassium ion concentration in the extracellular
space around neuronal cell body?
Marked out of 1.00

Flag question Select one:
Purkinje cells
Astrocytes
Microglia

946
Schwann cells
Oligodendrocytes

Your answer is incorrect.
Astrocytes are the most numerous type of glial cell. They insulate and separate neurons from
one another, and regulate the potassium ion concentration in the extracellular space.'
The correct answer is: Astrocytes

Question 59 MoK Neurochemistry 005
Not answered Acetylcholine is a substrate of which of the following substances?

Marked out of 1.00
Select one:
Flag question
Choline Acetyl transferase
Acetyl cholinesterase
Monoamine oxidase
Acetyl Co A
Choline

Your answer is incorrect.
Acetylcholine is produced by the combination of acetyl coA with choline in the presence of
ChAT ­ choline acetyl transferase. Ach is a substrate of the acetylcholinesterase enzyme.
The correct answer is: Acetyl cholinesterase

Question 60 MoK Neurochemistry 006
Not answered Which enzyme is required for the metabolism of the transmitter at the neuromuscular junction?

Marked out of 1.00
Select one:
Flag question
Choline acetyltransferase
Alpha ketoglutarate
Glutamine synthetase
Acetylcholinesterase
Glutaminase

Your answer is incorrect.
The enzyme involved is degrading acetylcholine is acetylcholinesterase it breaks down ACh
into acetate and choline. Choline is then taken up by the presynaptic terminal.
The correct answer is: Acetylcholinesterase

947
Question 61 MoK Neurochemistry 007
Not answered The rate limiting enzyme in alcohol metabolism is
Marked out of 1.00
Select one:
Flag question
Acetaldehyde dehydrogenase
Alanine dehydrogenase
DOPA dehydrogenase
Alcohol dehydrogenase
Aldehyde dehydrogenase

Your answer is incorrect.
The first step in the metabolism of alcohol is the oxidation of ethanol to acetaldehyde
catalyzed by alcohol/dehydrogenase containing the coenzyme NAD+. The acetaldehyde is
further oxidized to acetic acid and finally CO2 and water through the citric acid cycle. A
number of metabolic effects from alcohol are directly linked to the production of an excess of
both NADH and acetaldehyde
The correct answer is: Alcohol dehydrogenase

Question 62 MoK Neurochemistry 008
Not answered Which among the following statements favour the dopamine hypothesis of schizophrenia?

Marked out of 1.00
Select one:
Flag question
Amphetamines release Dopamine into the synapse and cause positive
schizophrenic symptoms
Disulfiram stimulates Dopamine metabolism and may exacerbate
schizophrenia
Postal Emission Tomography (PET) shows decreased Dopamine receptor
binding in drug naïve patients.

Phencyclidine (PCP) causes negative schizophrenic symptoms
Both alpha and beta isomers of Flupenthixol are effective anti psychotic
drugs

Your answer is incorrect.
Only the alpha isomer of Flupentixol is a Dopamine antagonist and an effective anti psychotic
drug. Phencyclidine causes both positive and negative symptoms of schizophrenia but via
antagonisms at the NMDA receptor. Disulfiram actually inhibits Dopamine metabolism and
may exacerbate schizophrenia. PET studies show increased Dopamine receptor binding in
drug­naive patients. All these findings support the Dopamine hypothesis of schizophrenia.
The correct answer is: Amphetamines release Dopamine into the synapse and cause positive
schizophrenic symptoms

948
Question 63 MoK Neurochemistry 010
Not answered Release of transmitters synthesized by neurons in the central nervous system is directly
regulated by the concentration of which of the following electrolytes?
Marked out of 1.00

Flag question Select one:
Calcium influx
Chloride influx
Potassium influx

Magnesium influx
Sodium influx

Your answer is incorrect.
Action potentials at the presynaptic axon terminals open up calcium channels, permitting
calcium influx. This leads to movement of synaptic vesicles and subsequent release of stored
neurotransmitters.
The correct answer is: Calcium influx

Question 64 MoK Neuropathology 004
Not answered A 75­year­old gentleman with memory problems, disinhibition and lack of insight. What
microscopic findings will be expected in this case?
Marked out of 1.00

Flag question Select one:
Spongiform encephalopathy
Abnormally phosphorlyated tau protein
Intraneuronal inclusions
Beta amyloid peptides
Cytoplasmic inclusions

Your answer is incorrect.
This man is exhibiting symptoms of fronto­temporal dementia. The syndrome of FTD has been
associated with two types of pathological change, which in individual cases cannot be
predicted on clinical grounds. Tau positive inclusions account for the majority of cases, and
most familial (but not sporadic) cases with tau pathology carry mutations in the microtubule­
associated tau protein on chromosome 17. The other main pathological finding in FTD is so­
called ubiquitin­only pathology with cytoplasmic and nuclear inclusions (i.e. negative for tau
or alpha­synuclein), suggesting a link with Amyotrophic lateral sclerosis.
The correct answer is: Cytoplasmic inclusions

Question 65 MoK Neuropathology 005

949
Not answered A 37­year­old male presents with episodes of waxing and waning neurological deficits. The
most likely pathological change in brain is
Marked out of 1.00

Flag question
Select one:
Gliosis
Demyelination
Lewy bodies

Neuritic plaques
Amyloid plaques

Your answer is incorrect.
This patient has a diagnosis of Multiple Sclerosis. This is caused by the gradual
demyelination that leads to delayed and inhibited signal conduction.
The correct answer is: Demyelination

Question 66 MoK Neuropathology 006
Not answered A newborn suffers from seizures and hydrocephalus; on examination midline facial anomaly
and spasticity are evident. Further investigations reveal agenesis of corpus callosum. Which
Marked out of 1.00
of the following underlying pathology is most likely?
Flag question
Select one:
Tissue loss secondary to obstetric trauma
Disorder of neuronal migration
Congential transmission of infection
Excessive synaptic pruning
Neurodegeneration

Your answer is incorrect.
Agenesis of CC is a disorder of neuronal migration.
http://www.ninds.nih.gov/disorders/neuronal_migration/neuronal_migration.htm
The correct answer is: Disorder of neuronal migration

Question 67 MoK Neurophysiology 004
Not answered Which of the following EEG wavelets have low amplitude and high frequency?

Marked out of 1.00
Select one:
Flag question
Sigma waves
Theta waves
Beta waves

950
Spike waves
Alpha waves

Your answer is incorrect.
Beta waves have high (more than 13Hz) frequency but low amplitude
The correct answer is: Beta waves

Question 68 MoK Neurophysiology 006
Not answered Which of the following is a gut hormone released from small intestine in response to fats and
proteins and serves to regulate food intake?
Marked out of 1.00

Flag question Select one:
Neurokinin
Serotonin
Capsaicin
Substance P
Cholecystokinin

Your answer is incorrect.
CCK inhibits food intake by stimulating the vagal nerve and inhibiting gastric emptying.
Central CCK receptors in the brain can inhibit eating.
The correct answer is: Cholecystokinin

Question 69 MoK Neurophysiology 007
Not answered The suprachiasmatic nucleus plays an important role in which of the following functions?
Marked out of 1.00
Select one:
Flag question
Attention
Long term memory
Circadian rhythms
Sexual behaviour
Water and hunger

Your answer is incorrect.
The suprachiasmatic nucleus controls the biologic clock of internal circadian rhythms.
The correct answer is: Circadian rhythms

951
Question 70 MoK Pharmacology EMI004
Not answered Pharmacokinetics of antipsychotics
Identify the antipsychotic drug using descriptions below:
Marked out of 4.00

Flag question
Olanzapine
Clozapine
Risperidone
Half life more than 50 hours
Promazine
Chlorpromazine
Amisulpride
Sertindole
Haloperidol
Clozapine
Aripiprazole
Aripiprazole
Antipsychotics with a 'hit and run'
Quetiapine
Olanzapine
profile (choose two drugs) Haloperidol
Sertindole
Amisulpride
Quetiapine
Chlorpromazine
Clozapine
Promazine
Amisulpride
Risperidone
Chlorpromazine
D2 antagonistic effect
Quetiapine
Sertindole
Promazine
Haloperidol
Olanzapine
Aripiprazole
Your answer is incorrect. Risperidone
The half­life of antipsychotics varies greatly. Shorter half­life medications, such as quetiapine
and ziprasidone, have 7­12 hours of half­life. Risperidone has a 3­hour half­life, but it is
metabolized to 9­hydroxy risperidone (aka paliperidone), which has a 24­28­hour half­life
while olanzapine has a 30­hour half­life. The longest half­life, about three days, is seen with
aripiprazole. 
Clozapine and quetiapine are unique atypical antipsychotic agents that 'hit' the D2 receptor
(blockade) only for a short while to cause antipsychotic effects and then 'run' (dissociate)
before they cause extrapyramidal side effects. As a result, they are recommended for use in
Parkinson's disease with psychosis. 
Amisulpride functions primarily as a D2 and D3 receptor antagonist. It may also have a unique
5HT­7 function, which is not fully understood. 
Excerpts from Correll, CU. Psychiatry Weekly: Practical Dosing Strategies. Retrieved from
http://www.psychweekly.com/aspx/article/articledetail.aspx?articleid=1107 (accessed April 6,
2015).
The correct answer is: Half life more than 50 hours – Aripiprazole, Antipsychotics with a 'hit
and run' profile (choose two drugs) – Clozapine , Quetiapine, D2 antagonistic effect –
Amisulpride

Question 71 MoK Pharmacology EMI005
Not answered Uncommon adverse effects 
For each of the adverse effects given below, identify the best drug associated with it;
Marked out of 3.00

952
Flag question
SSRIs
Lithium
Bupropion
Severe sweating
Lorazepam
Acamprosate
Droperidol
Carbamazepine
Mirtazapine
Lithium
Sildenafil
Mirtazapine
Chlorpromazine
Sildenafil
Torsades de pointes Lamotrigine
Bupropion
Chlorpromazine
Acamprosate
Lamotrigine
Droperidol
SSRIs
Carbamazepine
Bupropion
Allergic dermatitis and Lorazepam
Acamprosate
photosensitivity SSRIs
Mirtazapine
Lorazepam
Chlorpromazine
Sildenafil
Droperidol
Lamotrigine
Your answer is incorrect. Carbamazepine
Lithium
SSRIs: Severe sweating especially nocturnally is seen in some patients; Terazosin is effective
in counteracting sweating. 
Droperidol: Neuroleptics can decrease cardiac contractility, increase circulating levels of
catecholamines, and prolong atrial and ventricular conduction time. Low­potency drugs are
more cardiotoxic than high­potency drugs. ECG shows QT and PR prolongation, blunting of
the T waves, and ST depression. Thioridazine and droperidol, in particular, can cause torsade
de pointes. Antipsychotic related sudden death may be due to cardiac arrhythmias or even
seizures asphyxiation or malignant hyperthermia. Drugs causing QT prolongation are
associated with more sudden deaths. 
Chlorpromazine: Allergic dermatitis and photosensitivity can occur with low­potency agents
such as Chlorpromazine. Long­term chlorpromazine use can cause blue­gray discoloration of
skin areas exposed to sunlight. This is reversible.
The correct answer is: Severe sweating – SSRIs, Torsades de pointes – Droperidol, Allergic
dermatitis and photosensitivity – Chlorpromazine

Question 72 MoK Pharmacology EMI006
Not answered Drug­related adverse effects
For the drugs given below, identify the most important adverse effect leading to highly
Marked out of 3.00
restricted use and requirements for appropriate monitoring
Flag question

953
Pimozide Visual field defects
Blood dyscrasias
Guillain­Barre syndrome
Hypersensitivity reactions
Hepatotoxicity
Haemolytic anaemia
Agranulocytosis
Cheese reactions
Hepatotoxicity
Aplastic anemia
Haemolytic anaemia
QT prolongation on ECG
Blood dyscrasias
Vigabatrin
QT prolongation on ECG
Visual field defects
Aplastic anemia
Cheese reactions
Guillain­Barre syndrome
Aplastic anemia
Agranulocytosis
QT prolongation on ECG
Hypersensitivity reactions
Hepatotoxicity
Clozapine
Agranulocytosis
Guillain­Barre syndrome
Visual field defects
Cheese reactions
Hypersensitivity reactions
Blood dyscrasias
Haemolytic anaemia
Your answer is incorrect.
Pimozide is rarely used a first line treatment of schizophrenia. Its use has been associated
with ventricular arrhythmias and sudden unexplained death. Sertindole is another
antipsychotic drug associated with sudden cardiac deaths. Droperidol and Thioridazine are
also associated with significant QT prolongation on ECG. 
Vigabatrin is associated with visual field defects. 
Clozapine induced agranulocytosis is 15 to 30 times higher than the risk associated with
phenothiazines. At least 0.7% of Clozapine treated patients develop agranulocytosis, which is
potentially fatal. The maximum risk is between 4 and 18 weeks, and after a year the risk is
same as with phenothiazines. Weekly monitoring of the white cell count is required for 26
weeks in most countries, with the frequency decreasing to biweekly or monthly after that.
The correct answer is: Pimozide – QT prolongation on ECG, Vigabatrin – Visual field defects,
Clozapine – Agranulocytosis

Question 73 Mok Pharmacology EMI007
Not answered Plasma steady state
For each of the following drugs, identify the time taken to achieve steady state plasma levels
Marked out of 3.00

Flag question
2­3 days
3 months
15­16 days
Lithium
1 day
1 week
5 days
2 weeks
2 months

954
4 weeks
2­3 days
Clozapine
1 day
15­16 days
1 week
2 months
5 days
2 weeks
3 months
3 months
15­16 days
1 week
Sodium valproate
5 days
4 weeks
2 weeks
1 day
2­3 days
2 months
Your answer is incorrect.
Lithium takes up to five days to reach steady plasma level. Clozapine takes 2­3 days, the
same time as sodium valproate. (Ref: Maudsley prescribing guidelines­ Pg 3)
The correct answer is: Lithium – 1 week, Clozapine – 2­3 days, Sodium valproate – 2­3 days

Question 74 MoK Neurosciences EMI006
Not answered Types of Mutations 
For the following mutations, select the most appropriate term
Marked out of 3.00

Flag question
Missense
Nonsense
Expansion
Creation of a new stop codon
Transition
Transversion
In frame
Silent
Frameshift
In frame
Frameshift
This mutation does not alter the Missense
amino acid product Nonsense
Transition
Expansion
Transversion
Silent
Transversion
Nonsense
Frameshift
Transition
Substitution of a purine for a purine Silent
Missense
In frame
Expansion

955
Your answer is incorrect.
A silent mutation causes no change in protein product ­ this is possible because a single
amino acid is often coded by more than one triplet sequence. In a silent mutation, one triplet
sequence is replaced by a different sequence but without changing amino acid product. 
In missense mutation, the new mutant codon specifies a different amino acid with variable
effects on final protein product. For example, haemophilia, sickle cell anaemia. In nonsense
mutation, the new codon is UUA UGA or UAG, which signals 'stop' to the amino acid
sequence resulting in a nonfunctional protein. In frame shift mutations, the deletion or
insertion is not in multiples of three codons e.g. a segment of 5 bases deletion mutations. 
Point mutations are usually substitutions where one base is replaced by another. It could be
termed as transition if a purine is replaced by another purine or a pyrimidine replaced by
another pyrimidine (e.g. A to G). It is called transversion if a purine is replaced by a pyrimidine
or vice versa (e.g. A to T).
The correct answer is: Creation of a new stop codon – Nonsense, This mutation does not alter
the amino acid product – Silent, Substitution of a purine for a purine – Transition

Question 75 MoK Neurosciences EMI007
Not answered Chromosomal loci
Select the most appropriate chromosomal locus for each of the scenario given below
Marked out of 3.00

Flag question A 7­year­old girl has a large
mandible and an open­mouthed Chromosome 7
expression which reveals the tongue. Chromosome 1
She has severe learning difficulties, Mitochondrial DNA
with poor communication skills and Chromosome X
little or absent speech. She has an Chromosome 5
unusual manner of laughing when Chromosome 22
stimulated. Chromosome 15
A 3­year­old girl repeatedly presents Chromosome 4
with respiratory infections. On Chromosome Y
examination, her larynx appears to
Chromosome 21
Chromosome 11
be maldeveloped, and she has an Chromosome 5
Chromosome 21
unusually small head with a round Chromosome 22
face and a small chin with folds of Chromosome Y
skin over her eyes. The mother Chromosome 7
recalls a distinctive crying sound that Chromosome X
became less noticeable after age 2. Chromosome 15
Chromosome 1
A 4­year­old girl is brought to the A&E Chromosome 11
with severe breathing problems. On Chromosome 4
examination, there is excessive Chromosome X
Mitochondrial DNA
drooling, with reduced muscle tone in Mitochondrial DNA
her limbs. Developmental history is Chromosome 1
normal up to the age of 18 months
Chromosome 21
apart from repeated problems related
to gastroesophageal reflux. Growth
Chromosome 5
charts reveal a slowing of head Chromosome Y
growth beginning at approximately 5 Chromosome 7
­ 6 months of age. Chromosome 22
Chromosome 4
Chromosome 11
956
Your answer is incorrect.
Case 1 describes Angelman syndrome. A small deletion of part of chromosome 15 inherited
from mother accounts for about 70 percent of those with Angelman syndrome. A similar
genetic fault to the chromosome inherited from the father is found in another genetic condition,
Prader­Willi syndrome.
Case 2 exemplifies Cri­du­chat syndrome. Cri­du­chat is caused by a variable length
microdeletion on the short arm of chromosome 5. One of the deleted genes codes for TERT
(telomerase reverse transcriptase) enzyme. 
Case 3 is an example of Rett syndrome associted with a defect in the methyl­CpG­binding
protein 2 (MeCP2) genes on the X chromosome. As females have two X chromosomes, even
when one has this significant defect, the other X chromosome is normal enough for the child
to survive. But most of the male fetuses die and so Rett's syndrome is almost exclusively
restricted to female children. 
Rett syndrome: MedlinePlus Medical Encyclopedia,
http://www.nlm.nih.gov/medlineplus/ency/article/001536.htm (accessed April 5, 2015).
The correct answer is: A 7­year­old girl has a large mandible and an open­mouthed
expression which reveals the tongue. She has severe learning difficulties, with poor
communication skills and little or absent speech. She has an unusual manner of laughing
when stimulated. – Chromosome 15, A 3­year­old girl repeatedly presents with respiratory
infections. On examination, her larynx appears to be maldeveloped, and she has an
unusually small head with a round face and a small chin with folds of skin over her eyes. The
mother recalls a distinctive crying sound that became less noticeable after age 2. –
Chromosome 5, A 4­year­old girl is brought to the A&E with severe breathing problems. On
examination, there is excessive drooling, with reduced muscle tone in her limbs.
Developmental history is normal up to the age of 18 months apart from repeated problems
related to gastroesophageal reflux. Growth charts reveal a slowing of head growth beginning
at approximately 5 ­ 6 months of age. – Chromosome X

Question 76 MoK Neurosciences EMI009
Not answered Neuropathology 
For the following clinical conditions, identify the most common neuropathological changes in
Marked out of 6.00
the brain.
Flag question

Cystic formation with lacunar changes
Neurofibrillary tangles and amyloid plaques in neocortex
A 70­year­old gentleman with history
Presence of e­4 allele
of parkinsonism, confusion and visual
Atrophy of frontal poles
hallucinations (Choose TWO)
Maldevelopment of cerebellum
Sensitivity to antipsychotics
Intraneuronal synuclein deposits in cortex and substantia nigra
An 85­year­old lady with progressive Mutation in Tau gene
Presence of e­4 allele
memory loss for recent events and Atrophy of parietal poles
dysphasia, which has become more
Gliosis
Gliosis
Atrophy of frontal poles
generalized. She also has a 20­years
history of alcohol misuse (Choose Atrophy of parietal poles
TWO) Mutation in Tau gene
Maldevelopment of cerebellum
Intraneuronal synuclein deposits in cortex and substantia nigra
Sensitivity to antipsychotics
Neurofibrillary tangles and amyloid plaques in neocortex
Cystic formation with lacunar changes

957
A 57­year­old gentleman presenting
Maldevelopment of cerebellum
with altered behaviour, apathy,
Cystic formation with lacunar changes
disinhibition and word finding
difficulties. His mother and aunt died
Gliosis
from a similar condition. (Choose Atrophy of parietal poles
TWO) Presence of e­4 allele
Sensitivity to antipsychotics
Atrophy of frontal poles
Mutation in Tau gene
Intraneuronal synuclein deposits in cortex and substantia nigra
Your answer is incorrect. Neurofibrillary tangles and amyloid plaques in neocortex
Case 1: This is a case of Lewy body dementia characterised by neuroleptic sensitivity and the
presence of Lewy bodies on pathological examination. Lewy bodies are intraneuronal
deposits. 
Case 2 is an example of Alzheimer's disease characterized by amyloid plaques,
neurofibrillary tangles and apo­E4 gene. 
Case 3 describes frontotemporal dementia characterized by frontal and temporal lobe atrophy
and mutations in the tau gene.
The correct answer is: A 70­year­old gentleman with history of parkinsonism, confusion and
visual hallucinations (Choose TWO) – Sensitivity to antipsychotics, Intraneuronal synuclein
deposits in cortex and substantia nigra, An 85­year­old lady with progressive memory loss for
recent events and dysphasia, which has become more generalized. She also has a 20­years
history of alcohol misuse (Choose TWO) – Presence of e­4 allele, Neurofibrillary tangles and
amyloid plaques in neocortex, A 57­year­old gentleman presenting with altered behaviour,
apathy, disinhibition and word finding difficulties. His mother and aunt died from a similar
condition. (Choose TWO) – Atrophy of frontal poles, Mutation in Tau gene

Question 77 MoK Neurosciences EMI010
Not answered Prefrontal syndromes
A 65­year­old man has presented to the clinic with features of frontal lobe deficits. Identify the
Marked out of 6.00
two common clinical features commonly seen in each type of pre­frontal syndrome
Flag question

Paucity of spontaneous behaviour
Diminished planning
Poverty of speech
Medial prefrontal syndrome
Executive dysfunction
Poor impulse control
Flexible and adjustable pattern of behaviour
Explosive outbursts
Generalised seizures
Socially appropriate behaviour
Socially appropriate behaviour
Executive dysfunction
Flexible and adjustable pattern of behaviour
Orbitofrontal syndrome
Explosive outbursts
Diminished planning
Paucity of spontaneous behaviour
Generalised seizures
Poverty of speech
Poor impulse control

Dorsolateral prefrontal syndrome

958
Paucity of spontaneous behaviour
Generalised seizures
Explosive outbursts
Socially appropriate behaviour
Diminished planning
Executive dysfunction
Poor impulse control
Poverty of speech
Flexible and adjustable pattern of behaviour
Your answer is incorrect.
Medial (ventromedial) prefrontal syndrome, also called apathetic, or pseudo depressive type
is featured by a paucity of spontaneous behaviours, sparse verbal output, reduced social
knowledge and judgement of harmful intent. 
The orbitofrontal syndrome is also called disinhibited or pseudopsychopathic type and is
characterised by poor impulse control, explosive outburst, and inappropriate behavior. 
Dorsolateral is called as dysexecutive or disorganized type is featured by cognitive
dysfunction, diminished judgment, and planning and poor insight. The patients are also
described to be concrete and inflexible in nature.
The correct answer is: Medial prefrontal syndrome – Poverty of speech , Paucity of
spontaneous behaviour, Orbitofrontal syndrome – Poor impulse control, Explosive outbursts,
Dorsolateral prefrontal syndrome – Executive dysfunction, Diminished planning

Finish review

959
 Home Mock Paper Practice Tests

Mock Paper A(2)

Started on Thursday, 25 June 2015, 5:52 AM
State Finished
Completed on Thursday, 25 June 2015, 5:53 AM
Time taken 38 secs
Grade 0.00 out of 100.00
Feedback You are on the right track. Could you spend some more time on your revision?

Question 1 MoK Applied Neuroscience 019
Not answered A hypertensive patient develops a small focal lesion of one inferior olivary nucleus. Which of
the following clinical signs can be expected?
Marked out of 1.00

Flag question Select one:
Right Left Disorientation
Visual Inattention
Anosognosia
Oculomotor Apraxia
Finger Nose Ataxia

Your answer is incorrect.
Inferior Olivary Nucleus has a role in motor coordination by providing CLIMBING FIBERS to
the cerebellum.Lesion results in motor incoordination of the contralateral arm and leg (unlike
other cerebellar lesions which cause ipsilateral dysfunction)
The correct answer is: Finger Nose Ataxia

Question 2 MoK Applied Neuroscience 020
Not answered The Wisconsin Card SortingTest (WCST) assess which of the following functions?

Marked out of 1.00
Select one:
Flag question
category fluency
personality traits

visuomotor performance
flexibility in problem solving
Attention and concentration.

960
Your answer is incorrect.
The Wisconsin Card Sorting Test (WCST) is a neuropsychological test of 'set­shifting', i.e. the
ability to display flexibility in the face of changing schedules of reinforcement
The correct answer is: flexibility in problem solving

Question 3 MoK Applied Neuroscience 022
Not answered A patient was asked to name as many animals as possible in one minute. This test is useful to
detect deficits involving which of the following structures?
Marked out of 1.00

Flag question Select one:
Occipital lobe
Cerebellum

Temporal lobe
Parietal lobe
Frontal lobe

Your answer is incorrect.
'Category fluency test' is a frontal lobe function test for lexical fluency. The task involves listing
names of animals or listing the items to buy in a supermarket. It tests not only the speed and
accuracy but also the ability to shift from a narrow set to the next.
The correct answer is: Frontal lobe

Question 4 MoK Applied Neuroscience 023
Not answered A 65­year­old lady has been diagnosed with normal pressure hydrocephalus. Which of the
following symptoms is more likely to improve following shunting?
Marked out of 1.00

Flag question Select one:
Executive functions

Visuospatial deficits
Cognitive impairment
Urinary incontinence
Gait impairment

Your answer is incorrect.
Gait disturbance is more likely to improve following ventriculo­peritoneal shunting in NPH.
The correct answer is: Gait impairment

961
Question 5 MoK Applied Neuroscience 024
Not answered Raven's progressive matrices is used to assess

Marked out of 1.00
Select one:
Flag question
Frontal lobe functions
Premorbid intelligence
Visuospatial ability
General intellectual ability
Attentional bias

Your answer is incorrect.
Raven's Progressive Matrices (often referred to simply as Raven's Matrices) are multiple­
choice intelligence tests of abstract reasoning that provide culturally unbiased IQ estimates.
Rey Osterrieth test comprises of copying a complex figure initially and then reproducing it from
immediate and delayed recall that tests both visual memory and constructional ability
The correct answer is: General intellectual ability

Question 6 MoK Applied Neuroscience 025
Not answered In a psychological test, the clinican reads an incomplete sentence 'The captain wanted to stay
with the sinking?' and asks the patient to complete the sentence. Which of the following test is
Marked out of 1.00
being carried out?
Flag question
Select one:
Wisconsin Test
Hayling's Test
Goldstein's Test
Brixton Test
Stroop test

Your answer is incorrect.
The Hayling Sentence Completion Test takes about five minutes to administer and yields
three different measures of executive functioning. It consists of two sets of 15 sentences each
having the last word missing. In the first section, the examiner reads each sentence aloud,
and the participant has to simply complete the sentences (response initiation speed).The
Brixton Spatial Anticipation Test measures the ability to detect rules in sequences of stimuli. It
is perceptually simple and does not require a verbal response. It is thus appropriate for
people who are suffering from a wide range of associated deficits such as those involving
speech production or reading.
The correct answer is: Hayling's Test

Question 7 MoK Applied Neuroscience 026

962
Not answered Lesions of the dominant parietal lobe may produce Gerstmann's syndrome. Which of the
Marked out of 1.00
following is not a feature of this syndrome?

Flag question
Select one:
Hemi­neglect
Acalculia
Left­right confusion

Finger agnosia
Agraphia

Your answer is incorrect.
Hemi neglect is not a part of the tetrad described in Gerstmann syndrome.
The correct answer is: Hemi­neglect

Question 8 MoK Applied Neuroscience 027
Not answered Which of the following movement disorders is characterized by involuntary flinging
movements of the extremities?
Marked out of 1.00

Flag question Select one:
Chorea
Hemiballismus
Writer's cramp
Dysdiodochokinesia
Athetosis

Your answer is incorrect.
Hemiballismus is usually a unilateral, nonrhythmic, rapid, nonsuppressible, violent, flinging
movement of the proximal arm. Hemiballismus is caused by a lesion, usually an infarct,
around the contralateral subthalamic nucleus. Although disabling, hemiballismus is usually
self­limited, lasting 6 to 8 wk. Treatment with antipsychotics is often effective.
The correct answer is: Hemiballismus

Question 9 MoK Applied Neuroscience 028
Not answered A 45­year­old gentleman suffers from anterograde memory loss with preserved insight. What
is the most likely diagnosis?
Marked out of 1.00

Flag question Select one:
Lesions involving orbitofrontal cortex
Psychogenic amnesia

963
Picks disease
Diencephalic amnesia
Hippocampal amnesia

Your answer is incorrect.
Anterograde amnesia is seen most frequently in patients who have sustained injuries to the
medial temporal lobes or the diencephalon (Graff­Radford, Tranel, Von Hoesen, and Brandt,
1990). The medial temporal lobe contains the amygdala and the hippocampal formation, and
animal experiments in primates have shown that damage to these two areas causes more
severe anterograde amnesia. These two areas also send fibers to the diencephalon, and it is
unclear if damage to the diencephalon affects the hippocampus and amygdala also.
The correct answer is: Hippocampal amnesia

Question 10 MoK Applied Neuroscience 029
Not answered Many medical conditions are associated with autistic behavioural phenotype. Which one of
the following is NOT associated with autism?
Marked out of 1.00

Flag question Select one:
Fragile X Syndrome
Tuberous sclerosis
Huntington's disease
Neurofibromatosis
Down Syndrome

Your answer is incorrect.
Fragile­X, tuberous sclerosis, neurofibromatosis, hypomelanosis of Ito, Moebius syndrome,
Rett syndrome, and Down syndrome are some of the medical conditions associated with
autism.
The correct answer is: Huntington's disease

Question 11 MoK Applied Neuroscience 030
Not answered The negative symptoms of schizophrenia are related to which of the following neurobiological
phenomena?
Marked out of 1.00

Flag question Select one:
Hyperactivation of corpus striatum
Hypoactivation of mesolimbic pathway
Hyperactivation of dorsolateral prefrontal cortex
Hypoactivation of dorsolateral prefrontal cortex

964
Hypoactivation of corpus striatum

Your answer is incorrect.
It has been suggested that positive symptoms could be associated with excessive DA
transmission in subcortical regions, whereas negative symptoms could be related to a
concomitant deficit in cortical DA transmission. Earlier fMRI investigations, without
pharmacologic challenge, have found some evidence for the concept of frontal hypoactivity,
by showing reduced activation during prefrontal cognitive tasks (working memory, attention,
and executive functions) in the ventrolateral prefrontal cortex (VLPFC), dorsolateral prefrontal
cortex (DLPFC), and anterior cingulate in nonmedicated or medication­nave patients, relative
to healthy controls.
The correct answer is: Hypoactivation of dorsolateral prefrontal cortex

Question 12 MoK Molecular Genetics 025
Not answered You are seeing a patient in the gender identity clinic. The patient would like to obtain
temporary membership of the opposite sex. The most probable diagnosis is;
Marked out of 1.00

Flag question Select one:
Body dysmorphic disorder

Transexualism
Dual role transvestism
Frotteurism
Fetishistic transvestism

Your answer is incorrect.
Dual role transvestism is a milder form of gender dysphoria. The individual wears clothes of
the opposite sex in order to experience temporary membership in the opposite sex. The
individual experiences a sense of appropriateness by wearing clothes of the other gender.
There is no sexual motivation for the cross­dressing. The individual has no desire for a
permanent change to the opposite sex.
The correct answer is: Dual role transvestism

Question 13 MoK Molecular Genetics 026
Not answered The presence of apolipoprotein­e4 allele increases the chance of developing Alzheimer's
disease by around
Marked out of 1.00

Flag question Select one:
Twenty fold
Two fold
Fifty fold
Ten fold

965
Five fold

Your answer is incorrect.
In 1993, a polymorphism in the apolipoprotein E4 allele (apoE4) was discovered to be a
major risk factor for late­ onset Alzheimer disease. This gene promotes the deposition of beta­
amyloid protein in the brain, contributing to the formation of senile plaques. People who have
inherited a single E4 allele of apolipoprotein (APOE) experience up to a three to fivefold
increase in risk of developing Alzheimer disease.
The correct answer is: Five fold

Question 14 MoK Molecular Genetics 027
Not answered Culturing cells in conditions of thymidine deprivation can aid in the diagnosis of which of the
following disorders?
Marked out of 1.00

Flag question Select one:
Fragile X syndrome
Asperger's disease
Cri­du­chat syndrome
Autism
Down's syndrome

Your answer is incorrect.
Thymidine deprivation can make the fragile sites of the chromosome obvious for detection
The correct answer is: Fragile X syndrome

Question 15 MoK Molecular Genetics 028
Not answered Social anxiety and gaze avoidance are features of which of the following trinucleotide
expansion syndrome?
Marked out of 1.00

Flag question Select one:
Fredreich's Ataxia
Leber's Optic Neuropathy
Myotonic dystrophy
Fragile X syndrome
Huntington's disease

Your answer is incorrect.
Eye­gaze avoidance is a striking phenotypic feature in fragile X syndrome.

966
The correct answer is: Fragile X syndrome

Question 16 MoK Molecular Genetics 029
Not answered One of the following disorders do not involve genetic deletion. Choose the correct answer.
Marked out of 1.00
Select one:
Flag question
William syndrome
Angelman syndrome
Cri­du­chat syndrome
DiGeorge syndrome
Frederich's ataxia

Your answer is incorrect.
Friedreich's ataxia is caused by a defect in a gene called Frataxin (FXN), which is located on
chromosome 9. Changes in this gene cause the body to produce too much of a part of DNA
called trinucleotide repeat (GAA). Normally, the body contains about 8 to 30 copies of GAA.
Those with Individuals with Friedreich's ataxia have as many as 1,000 copies. The more
copies of GAA a patient has, the earlier in life the disease starts and the faster it gets worse.
Friedreich's ataxia is an autosomal recessive genetic disorder [Friedreich's ataxia:
MedlinePlus Medical Encyclopedia,
http://www.nlm.nih.gov/medlineplus/ency/article/001411.htm].
The correct answer is: Frederich's ataxia

Question 17 MoK Molecular Genetics 030
Not answered Which one of the following is a primary proponent of endophenotypes concept?
Marked out of 1.00
Select one:
Flag question
Gottesmann
Freud
Kraeplin
Fromm­Reichmann
Kendler

Your answer is incorrect.
Gottesman and Shields re­introduced the endophenotypes concept.
The correct answer is: Gottesmann

Question 18 MoK Molecular Genetics 031
Not answered The process of meiosis produces

967
Marked out of 1.00 Select one:
Flag question Diploid somatic cells
Haploid germ cells
Tetraploid cells
Triploid cells
Aneuploid cells

Your answer is incorrect.
The outcome of meiosis is four (genetically unique) haploid cells, compared with the two
(genetically identical) diploid cells produced from mitosis. Meiosis is thus a reduction division
where one diploid cell with two copies of each chromosome yields four haploid cells
containing a single copy of each chromosome.
The correct answer is: Haploid germ cells

Question 19 MoK Molecular Genetics 032
Not answered Tay­Sachs disease is caused by deficiency of

Marked out of 1.00
Select one:
Flag question
Neuropeptide Y
Glucoronidase
Monoamine oxidase
Hexosaminidase A
Hypoxanthine phosphoribosyl transferase

Your answer is incorrect.
Tay­Sachs disease is caused by a deficiency of the enzyme Hexosaminidase. It is inherited as
an autosomal recessive disorder. Clinical features are apparent by around six months of age
and usually results in death by the age of four.
The correct answer is: Hexosaminidase A

Question 20 MoK Molecular Genetics 033
Not answered Which of the following is a characteristic feature of autosomal recessive transmission?

Marked out of 1.00
Select one:
Flag question
Pure maternal transmission
Everyone with mutant gene is diseased

High male to male transmission
Phenotype not expressed in females

968
High risk with consanguinity

Your answer is incorrect.
As carriers are possible in AR transmission, 'hidden relatives' may give birth to unwell
children.
The correct answer is: High risk with consanguinity

Question 21 MoK Molecular Genetics 034
Not answered The anticipation in paternal transmission is most likely to show up in which of the following
conditions?
Marked out of 1.00

Flag question Select one:
Noonan syndrome
Hurler syndrome
Lesch­nyhan syndrome
Huntington's disease
Tuberous sclerosis

Your answer is incorrect.
Anticipation refers to an inheritance pattern in which individuals in the most recent
generations of a pedigree develop a disease at an earlier age or with greater severity than
does those in previous generations. This is mostly due to the gradual expansion of
trinucleotide repeat polymorphisms (this instability is called dynamic mutation). The
phenomenon of anticipation is seen in Huntington's disease. The inheritance of HD from the
father is associated with a greater expansion of the repeats and an earlier age of onset. In
approximately one­third of cases in which the father passes on the gene to his offspring, there
is an expansion resulting in juvenile­onset HD. Characteristic protein deposits form nuclear
inclusions in neurons of HD patients.
The correct answer is: Huntington's disease

Question 22 MoK Molecular Genetics 036
Not answered If working memory deficit is inherited as a multifactorial trait then its recurrence risk will be
Marked out of 1.00
Select one:
Flag question
same in all races
not dependent on number of affected family members
influenced by the severity of deficit
same in all families
same in both sexes

969
Your answer is incorrect.
Involvement of more family members and higher severity of expression of a multifactorial trait
may mean higher polygenic contribution to the inheritance. Ethnic and gender differences are
well noted in multifactorial inheritances.
The correct answer is: influenced by the severity of deficit

Question 23 MoK Molecular Genetics 037
Not answered Which of the following is most likely with regard to Fragile X syndrome?
Marked out of 1.00
Select one:
Flag question
Locus q25 is deleted
Microcephaly is often seen
Clinical features are apparent only during adolescence
CTG repeats are notable
Intelligence is correlated to the load of chromosomal damage

Your answer is incorrect.
CGG repeats at q27 of X chromosome is associated with macrocephaly and mental
retardation during childhood.
The correct answer is: Intelligence is correlated to the load of chromosomal damage

Question 24 MoK Molecular Genetics 038
Not answered Which of the following refers to the stage between two successive cell divisions wherein the
cell replicates its genetic material?
Marked out of 1.00

Flag question Select one:
Metaphase
Interphase
Prophase
Anaphase
Telophase

Your answer is incorrect.
Interphase is the 'holding' stage
The correct answer is: Interphase

Question 25 MoK Neuroanatomy 007
Not answered

970
Marked out of 1.00 When experiencing disgust, which of the following parts of brain show consistent changes in
functional imaging?
Flag question

Select one:
Anterior insular region
Supplementary motor area
Dorso medial thalamus

Posterior hippocampus
Anterior cingulate cortex

Your answer is incorrect.
The involvement of insula in disgust recognition has been reported in behavioural and
functional imaging studies. Both strong and mild expressions of disgust activated the same
sites in the anterior insula.
The correct answer is: Anterior insular region

Question 26 MoK Neuroanatomy 013
Not answered Which of the following is seen in anterior cerebral artery occlusion?

Marked out of 1.00
Select one:
Flag question
Contralateral lower limb weakness
Sensory aphasia
Unilateral sensory loss
Ipsilateral facial palsy
Bulbar palsy

Your answer is incorrect.
Occlusion of the anterior cerebral artery may result in weakness and sensory loss of the
contralateral foot and leg; gait apraxia and urinary incontinence (with bilateral damage in the
acute phase). This sometimes resembles the triad of Normal Pressure Hydrocephalus where
identical anatomical structures are implicated albeit presenting with an insidious or subacute
onset. In left­sided lesions, some cases present with transcortical motor aphasia.
The correct answer is: Contralateral lower limb weakness

Question 27 MoK Neuroanatomy 014
Not answered A 56­year­old gentleman suffers from ataxia and nystagmus. His CT scan shows ischemia in
cerebellum. Which part of cerebellum is most likely to be affected?
Marked out of 1.00

Flag question Select one:
Vermis

971
Anterior lobe
Flocculo­nodular lobe
Posterior lobe
Lateral lobe

Your answer is incorrect.
Damage to the flocculonodular lobe (the vestibular part) may show up as a loss of equilibrium
and, in particular, an altered walking gait, with a wide stance that indicates difficulty in
balancing. Damage to the lateral zone, or the cerebrocerebellum, results in problems with
skilled voluntary and planned movements. This can cause manifestations like hypotonia,
dysarthria and dysdiadochokinesia and tremors.
The correct answer is: Flocculo­nodular lobe

Question 28 MoK Neuroanatomy 015
Not answered Which area of the brain is implicated with recognition of faces?

Marked out of 1.00
Select one:
Flag question
Amygdala
Fusiform gyrus
Nucleus accumbens
Prefrontal cortex
Left inferior frontal area

Your answer is incorrect.
Face perception is an ability that involves many areas of the brain. Brain imaging studies have
typically shown a great deal of activity in the area of the temporal lobe known as fusiform
gyrus, an area also known to cause prosopagnosia when damaged bilaterally.
The correct answer is: Fusiform gyrus

Question 29 MoK Neuroanatomy 016
Not answered Communication between the frontal lobes of each side of the brain is mediated through which
of the following structures?
Marked out of 1.00

Flag question Select one:
Fornix
Genu of corpus callosum
Parahippocampal gyrus
Splenium of corpus callosum
Arcuate fasciculus

972
Your answer is incorrect.
The corpus callosum forms the major connection by which the hemispheres communicate
with one another. The genu of the corpus callosum contains fibers that pass from the frontal
lobe of one side to that of the other.
The correct answer is: Genu of corpus callosum

Question 30 MoK Neurochemistry 011
Not answered Which of the following enzymes is responsible for the degradation of noradrenaline?

Marked out of 1.00
Select one:
Flag question
acetylcholinesterase
Choline Acetyl Transferase
Catechol­O­Methyl transferase
Tryptophan hydroxylase
Tyrosine hydroxylase

Your answer is incorrect.
Catechol­O­methyltransferase and monoamine oxidase are critical for the metabolic
degradation of catecholamines
The correct answer is: Catechol­O­Methyl transferase

Question 31 MoK Neurochemistry 012
Not answered Which of the following substance is an active ingredient in the Ethiopian plant Catha edulis,
whose leaves on chewing provide amphetamine­like stimulation?
Marked out of 1.00

Flag question Select one:
Coumarin
Cathine
Ketamine
Ephedrine
Cathinone

Your answer is incorrect.
The stimulant effect of the Khat plant was originally attributed to cathine, but this was disputed
by reports showing the plant extracts from fresh leaves contained another substance more
active than cathine ­ called cathinone. Cathinone is not very stable and breaks down to
produce cathine and norephedrine. This explains why fresh leaves are more stimulating, and
the habit of chewing Khat is more or less confined to the regions where it is cultivated.

973
The correct answer is: Cathinone

Question 32 MoK Neurochemistry 013
Not answered Hyperpolarization of the neuron is influenced by which of the following?

Marked out of 1.00
Select one:
Flag question
chloride and sodium
calcium and chloride
sodium and calcium
potassium and sodium
chloride and potassium

Your answer is incorrect.
In CNS neurons, inhibitory transmitters open chloride channels. In addition, second
messengers that mediate inhibition work by operating on potassium channels.
The correct answer is: chloride and potassium

Question 33 MoK Neurochemistry 014
Not answered The receptors which have both pre and postsynaptic presence are

Marked out of 1.00
Select one:
Flag question
5­HT2C receptors
D2 receptors
D1 receptors
D5 receptors
5­HT1A receptors

Your answer is incorrect.
D2­like receptors (D­2, D­3 and D­4) have both pre­synaptic and post­synaptic inhibitory
effect. D1 like receptors (D1 and D5) have only postsynaptic inhibition effect.
The correct answer is: D2 receptors

Question 34 MoK Neurochemistry 015
Not answered Which of the following is the most powerful endogenous opioid peptide?
Marked out of 1.00
Select one:
Flag question
ß­Endorphin
Morphine

974
Dynorphin
Enkephalin (?­endorphin)
Neoendorphin

Your answer is incorrect.
Dynorphin, a natural brain endorphin, is far more potent than morphine or any other known
endorphin. With such high potency, dynorphin can bind with great specificity to pain receptors.
Dynorphins, endorphins, and enkephalins arise from different biosynthetic pathways.
The correct answer is: Dynorphin

Question 35 MoK Neuropathology 007
Not answered All of the following intrauterine infections cause learning difficulties except
Marked out of 1.00
Select one:
Flag question
CMV
Toxoplasmosis
Treponema pallidum
Rubella
Epstein Barr Virus

Your answer is incorrect.
EBV (Infectious Mononucleosis) has not been implicated in learning disability syndromes. In
pregnant women, measles (rubeola) unlike German measles (rubella) does not cause any
congenital anomalies / learning disabilities.
The correct answer is: Epstein Barr Virus

Question 36 MoK Neuropathology 008
Not answered Which of the following is least likely to occur in schizophrenia?

Marked out of 1.00
Select one:
Flag question
Increase in neuron density
Cytoarchitectural anomalies
Reduced cell size affecting neurons
Gliosis
Reduced cell number affecting neurones

Your answer is incorrect.

975
In terms of histological changes, no evidence for astrogliosis in schizophrenia has been
reported. Reduced cell numbers or cell size has been described especially affecting neurons
in the hippocampus and dorsolateral prefrontal cortex. The Increase in neuronal density,
which may relate to the observed decrease in neurone size has been reported. Subtle
cytoarchitectural anomalies were described in the hippocampal formation and frontal cortex.
Synaptic studies in the hippocampus and DLPFC in schizophrenia show decrements in
presynaptic markers. (Ref: Harrison PJ. The neuropathology of schizophrenia. A critical
review of the data and their interpretation. Brain 1999; 122: 593­624)
The correct answer is: Gliosis

Question 37 MoK Neuropathology 009
Not answered An intravenous heroin user presents with significant slowing of cognitive ability. He has
fatuous affect and impaired concentration that has developed insidiously. Which of the
Marked out of 1.00
following is a likely diagnosis?
Flag question
Select one:
Opioid intoxication
Bacterial encephalitis
Heroin withdrawal
HIV dementia
Depression

Your answer is incorrect.
HIV dementia can present with the above features. See Neuroscience section on HIV
The correct answer is: HIV dementia

Question 38 MoK Neurophysiology 005
Not answered The term used to describe EEG more than 13Hz is
Marked out of 1.00
Select one:
Flag question
Mu waves
Beta waves
Gamma waves
Theta waves
Alpha waves

Your answer is incorrect.
more than 13 = beta; 8 ­ 13 = alpha; 4­8 = theta; less than 4 = delta
The correct answer is: Beta waves

976
Question 39 MoK Neurophysiology 008
Not answered The disorder associated with female pseudo­hermaphroditism is

Marked out of 1.00
Select one:
Flag question
Criduchat syndrome
Cushing's syndrome
Conn's syndrome
Congenital adrenal hyperplasia

Cotard's syndrome

Your answer is incorrect.
Therefore, female pseudohermaphroditism is a condition in which the gonadal sex is female,
but the external genitalia are masculinized, caused by high androgen levels. This is most
commonly caused by hyperplasia of the adrenal gland as a result of the genetic defects of
steroidogenesis, called the virilizing congenital adrenal hyperplasias (CAHs)
The correct answer is: Congenital adrenal hyperplasia

Question 40 MoK Neurophysiology 009
Not answered A 58 year old man presents with postural hypotension, anorexia, nausea and weakness. He
appears to be extremely apathetic, with significant anhedonia compounded by fatigue and
Marked out of 1.00
depressed mood.Which of the following hormonal imbalance is most likely?
Flag question
Select one:
Vasopressin
Insulin
Thyroxine
Corticosteroids
Adrenaline

Your answer is incorrect.
Addison's syndrome may result from adrenal destruction by autoimmune disease, metastatic
cancer, tuberculosis, or human immunodeficiency virus infection or secondary to suppression
of ACTH secretion by chronic corticosteroid therapy. Symptoms include postural hypotension,
anorexia, nausea and vomiting, and weakness. Psychiatric symptoms include apathy,
anhedonia, fatigue, and depressed mood. Adrenal insufficiency can be mistaken for
depressive disorder. Though a loss of appetite is seen in both, vomiting is almost never due to
isolated depression. http://www.primarypsychiatry.com/aspx/articledetail.aspx?articleid=329
The correct answer is: Corticosteroids

Question 41 MoK Adverse Effects 006
What is the incidence of fatal hepatotoxicity with valproate?

977
Not answered Select one:
Marked out of 1.00 1 in 250
Flag question
1 in 100
1 in 100,000
1 in 1,000
1 in 10,000

Your answer is incorrect.
In those taking valproate alone, 1 in 100,000 patients. Risk factors for hepatotoxicity include
young age (less than ten years old), sensitivity during the first few months of treatment, severe
epilepsy, and use of multiple drugs.
The correct answer is: 1 in 100,000

Question 42 MoK Adverse Effects 023
Not answered A 65­year­old man develops recurrent UTI since starting an antidepressant. He also gives a
history suggestive of new onset prostatism. But prostate examination shows no enlargement.
Marked out of 1.00
The most likely offending agent is
Flag question
Select one:
Nefazadone
Reboxetine
Trazodone
Mirtazapine
Amitryptyline

Your answer is incorrect.
Drugs with significant anticholinergic properties such as amitriptyline are notorious to cause
signs of prostatism in elderly males. They may produce urinary retention and sometimes
bladder distension.
The correct answer is: Amitryptyline

Question 43 MoK Adverse Effects 024
Not answered Tom is an inpatient on a psychiatric ward. He has a history of psychotic depression and is on
combination treatment with antipsychotics and antidepressants. He appears slightly flushed,
Marked out of 1.00
with dry mouth, dry skin, feeling hot, and confused. He is exhibiting features of
Flag question
Select one:
SSRI discontinuation syndrome
Neuroleptic malignant syndrome

978
Idiopathic parkinsonism
Anticholinergic syndrome
Serotonin syndrome

Your answer is incorrect.
Anticholinergic syndrome results from the inhibition of cholinergic neurotransmission at
muscarinic receptor sites. Psychotropics such as phenothiazines, tricyclic antidepressants
and antiparkinsonian agents have anticholinergic properties. When they are administered in
combination, the action of these drugs becomes additive resulting in anticholinergic
syndrome. Features of a mild organic brain syndrome, which includes disorientation, difficulty
in concentrating and impaired short­term memory are seen, especially noticeable at nighttime.
Other physical symptoms include flushing, dry skin and mucosa, mydriasis with loss of
accommodation and fever (Ref: Kaplan & Sadock Synopsis of Psychiatry 9th ed. Pg 1170­
1172)
The correct answer is: Anticholinergic syndrome

Question 44 MoK Adverse Effects 029
Not answered Patients on tranylcypromine should not take an excess of
Marked out of 1.00
Select one:
Flag question
Swede
Banana
Cheese
Potatoes
Salt

Your answer is incorrect.
Cheese can induce tyramine related reaction in those on MAOIs.
The correct answer is: Cheese

Question 45 MoK Adverse Effects 030
Not answered Which of the following antipsychotics is associated with photosensitivity more than others
listed?
Marked out of 1.00

Flag question Select one:
Olanzapine
Quetiapine
Haloperidol
Chlorpromazine

979
Risperidone

Your answer is incorrect.
Allergic dermatitis and photosensitivity can occur with low­potency agents. Long­term
chlorpromazine use can cause blue­gray discoloration of skin areas exposed to sunlight. This
is reversible. Irreversible retinal pigmentation is associated with the use of high dose
thioridazine (above 1000 mg a day).
The correct answer is: Chlorpromazine

Question 46 MoK Adverse Effects 033
Not answered A 56­year­old gentleman suffers from a severe depressive episode. He has a history of
bipolar affective disorder. He is on maintenance treatment with Lithium for five years. What is
Marked out of 1.00
the most appropriate drug to treat his depression?
Flag question
Select one:
Amitriptyline
Venlafaxine
Phenelzine
Citalopram
Duloxetine

Your answer is incorrect.
The risk of inducing a switch to mania is less with SSRIs and more with other classes of
antidepressants such as TCAs, MAOIs and SNRIs.
The correct answer is: Citalopram

Question 47 MoK Adverse Effects 034
Not answered Which of the following drugs is used in the treatment of akathisia?
Marked out of 1.00
Select one:
Flag question
Procyclidine
Atrophine
Trihexyphenidyl
Clonazepam
Benperidol

Your answer is incorrect.

980
Treatments for akathisia include reducing the antipsychotic drug or change to an atypical
drug. A reduction in symptoms of akathisia may be seen with propranolol, clonazepam,
mirtazapine, trazodone, mianserin and cyproheptadine
The correct answer is: Clonazepam

Question 48 MoK Adverse Effects 035
Not answered Which of the following drugs can cause severe rebound hypertension when stopped?

Marked out of 1.00
Select one:
Flag question
risperidone
venlafaxine
duloxetine
paroxetine
clonidine

Your answer is incorrect.
Clonidine is an Alpha­2 adrenergic receptor agonist. Agonist effects at these receptors, which
are located presynaptically in adrenergic neurons, reduce the release of epinephrine from
these neurons.
The correct answer is: clonidine

Question 49 MoK Adverse Effects 036
Not answered Tom is a 10­year­old boy with Tourettes' syndrome. He has had a medication for treating his
condition, which has lead to significant weight gain. Choose the best alternative?
Marked out of 1.00

Flag question Select one:
Risperidone
Clonidine
Haloperidol
Pimozide
Olanzapine

Your answer is incorrect.
Weight gain may be more pronounced in children than adults with antipsychotics, and this
may limit their use. Clonidine has been shown in open studies to reduce the severity and
frequency of tics and could bring a substantial reduction in tics. (Ref; Leckman JF et al.; Arch
Gen Psychiatry;1991: 42: 324­328)
The correct answer is: Clonidine

981
Question 50 MoK Adverse Effects 037
Not answered Which of the following statement regarding lithium is NOT true?

Marked out of 1.00
Select one:
Flag question
Oral preparations may have different bioavailability
Hypothyroidism is more common in middle­aged women talking lithium
ECG changes can be seen even at therapeutic doses

Coarse tremor is a common side effect
Peripheral limb oedema is a recognised side effect of lithium use

Your answer is incorrect.
Lithium chloride causes a concentration dependent block of the peak sodium current in
cardiac pacemakers, but even at therapeutic doses, abnormal ECG can be documented, often
with no clinical implications. Fine hand tremor is a common side effect often seen at lower
doses; coarse tremor indicates toxicity.
The correct answer is: Coarse tremor is a common side effect

Question 51 MoK Pharmacodynamics 013
Not answered Co­agonists glycine and d­serine act on which of the following receptors?

Marked out of 1.00
Select one:
Flag question
AMPA
Serotonergic
Adenosine
Muscarinic
Kainate

Your answer is incorrect.
D­serine is an endogenous ligand for the glycine site of the N­methyl­D­aspartate receptor.
Some recent studies suggest that there is a low affinity and stereo­selective site at the agonist­
binding pocket of AMPA receptors as well for D­amino acids such as d­serine.
The correct answer is: AMPA

Question 52 MoK Pharmacodynamics 014
Not answered The constituents of Khat have action that has greatest similarity to which of the following
drugs?
Marked out of 1.00

Flag question Select one:
Cocaine

982
Phencyclidine
Amphetamine
LSD
Cannabis

Your answer is incorrect.
The fresh leaves of Catha edulis from East Africa were used as a stimulant for amphetamine­
like effects from the main ingredient cathinone. Cathinone has most of the CNS and
peripheral actions of amphetamine. It is typically absorbed buccally after chewing the leaf and
being an alkaloid, gets metabolized rapidly with low chances of toxicity.
The correct answer is: Amphetamine

Question 53 MoK Pharmacodynamics 015
Not answered A 56­year­old woman on treatment with Clozapine has some residual symptoms and a BMI of
27. The ideal drug of choice for adjunctive treatment is
Marked out of 1.00

Flag question Select one:
Olanzapine
Citalopram
Fluoxetine

Quetiapine
Aripiprazole

Your answer is incorrect.
The addition of Aripiprazole to Clozapine may be particularly effective in reversing metabolic
effects (such as weight gain) and could be used as an augmenting agent to Clozapine.
The correct answer is: Aripiprazole

Question 54 MoK Pharmacodynamics 016
Not answered The mechanism of action of Cyproterone acetate is

Marked out of 1.00
Select one:
Flag question
Blocks testosterone receptors
LHRH antagonism
Reduces Prolactin levels

Agonist at testosterone receptors
17­ alpha hydroxy catabolism

983
Your answer is incorrect.
The mechanism of action of Cyproterone acetate is to block testosterone receptors. The direct
antiandrogenic effect of Cyproterone is to block the binding of dihydrotestosterone to the
specific receptors in the prostatic carcinoma cell. In addition, cyproterone exerts a negative
feed­back on the hypothalamo­pituitary axis, by inhibiting the secretion of LH resulting in
diminished production of testicular testosterone (Retrieved from Archives of Product Info:
http://www.drugs.com/mmx/cyproterone­acetate.html).
The correct answer is: Blocks testosterone receptors

Question 55 MoK Pharmacodynamics 017
Not answered Which of the following drugs acts as a Partial Mu receptor agonist?

Marked out of 1.00
Select one:
Flag question
Acamprosate
Buprenorphine
Disulfiram
Methadone
Buspirone

Your answer is incorrect.
Buprenorphine is a partial agonist at the mu­opioid receptor and has a reduced risk of fatal
respiratory depression.
The correct answer is: Buprenorphine

Question 56 MoK Pharmacodynamics 018
Not answered Which of the following atypical antidepressants have dopaminergic and noradrenergic
actions?
Marked out of 1.00

Flag question Select one:
Bupropion
Mirtazapine
Venlafaxine
Reboxetine
Mianserin

Your answer is incorrect.
Bupropion is an atypical antidepressants have dopaminergic and noradrenergic actions, and
it has been advocated by NICE for smoking cessation
The correct answer is: Bupropion

984
Question 57 MoK Pharmacodynamics 019
Not answered The drug used in treatment of REM sleep behavioural disorder is

Marked out of 1.00
Select one:
Flag question
Citalopram
Sildenafil
Amitryptyline

Olanzapine
Clonazepam

Your answer is incorrect.
Treatment with clonazepam has shown to be effective in REM sleep behavioural disorders.
The correct answer is: Clonazepam

Question 58 MoK Pharmacodynamics 020
Not answered Which of the following acts on presynaptic receptors as main mechanism of action?

Marked out of 1.00
Select one:
Flag question
Phenelzine
Propronalol
Donepezil
Diazepam
Clonidine

Your answer is incorrect.
Clonidine is a sympatholytic medication, which is classified as a centrally acting a2
adrenergic agonist. It has specificity towards the presynaptic a2 receptors in the vasomotor
center in the brainstem.
The correct answer is: Clonidine

Question 59 MoK Pharmacokinetics 019
Not answered A patient has chronic schizophrenia and is on clozapine therapy. If you want to try
augmentation, which one should be avoided?
Marked out of 1.00

Flag question Select one:
Carbamazepine
Sodium valproate

Aripiprazole

985
Fluoxetine
Lithium

Your answer is incorrect.
The combination of carbamazepine and clozapine is generally contraindicated. First,
carbamazepine and clozapine each produce hematologic side effects at higher frequencies
than do most other available drugs. Another consideration is that carbamazepine lowers the
level of clozapine, possibly more than it lowers levels of traditional neuroleptics. Am J
Psychiatry 155:12­21, January 1998
The correct answer is: Carbamazepine

Question 60 MoK Pharmacokinetics 020
Not answered Liz was started on a mood stabiliser when she was diagnosed with bipolar affective disorder
type 2. Her psychiatrist says that the dose needs to be carefully titrated as the prescribed
Marked out of 1.00
mood stabiliser induces its own metabolism. Which of the following has been prescribed to
Flag question Liz?

Select one:
Carbamazepine
Semisodium valproate
Lamotrigine
Lithium
Sodium valproate

Your answer is incorrect.
Carbamazepine has a tendency to induce its own metabolism leading to reduced plasma
steady state levels.
The correct answer is: Carbamazepine

Question 61 MoK Pharmacokinetics 021
Not answered Which of the following does not increase tricyclic antidepressant concentrations?
Marked out of 1.00
Select one:
Flag question
Cigarette smoking
Haloperidol
Quetiapine
Methylphenidate
Risperidone

986
Your answer is incorrect.
Antipsychotics, methylphenidate, acetazolamide, aspirin, cimetidine, thiazides, fluoxetine and
sodium bicarbonate increase tricyclic antidepressant (TCA) concentrations. Cigarette smoking
decreases their concentration through its action on the 1A2 enzyme
The correct answer is: Cigarette smoking

Question 62 MoK Pharmacokinetics 022
Not answered The antidepressant with the longest mean half­life from the given list is

Marked out of 1.00
Select one:
Flag question
fluvoxamine
venlafaxine
Citalopram
paroxetine

trazodone

Your answer is incorrect.
Fluoxetine has the longest half­life of all the SSRIs, one to three days, because of a
metabolite. Paroxetine, the SSRI with a long t1/2 compared to the others, has a half­life of only
21 hours
The correct answer is: Citalopram

Question 63 MoK Pharmacokinetics 023
Not answered A patient who is prescribed disulfiram to aid alcohol abstinence, presents to Accident and
Emergency room with facial flushing, hypotension, tachycardia, nausea, and vomiting. She
Marked out of 1.00
denies any recent ingestion of alcohol. Which of the following could have caused her
Flag question symptoms?

Select one:
Walnuts
Mature cheese
Banana
Chocolate
Cough syrup

Your answer is incorrect.
Cough and cold medications bought over the counter often contain alcohol and can
precipitate such Disulfiram reaction. Severe Antabuse reaction (also known as disulfiram
reaction) is characterized by respiratory depression, seizures, cardiovascular collapse, and
even death.

987
The correct answer is: Cough syrup

Question 64 MoK Pharmacokinetics 024
Not answered A barbiturate­dependent patient shows little pharmacological response to administration of
benzodiazepines. This is called 
Marked out of 1.00

Flag question Select one:
Tolerance
Dependence
Down regulation
Sensitisation
Cross tolerance

Your answer is incorrect.
Cross tolerance refers to decreased response to one medication due to exposure to a
different medication. Here exposure to barbiturates has produced cross­tolerance to
benzodiazepines.
The correct answer is: Cross tolerance

Question 65 MoK Pharmacokinetics 025
Not answered The benzodiazepines with the longest half life in a young adult is;

Marked out of 1.00
Select one:
Flag question
Alprazolam
Diazepam
Clonazepam
Nitrazepam
Temazepam

Your answer is incorrect.
Diazepam is a benzodiazepine which has a very long elimination half­life (20 ­ 100 hours with
an active metabolite 36 ­ 200 hours); as a result withdrawal symptoms are not very common
with diazepam.
The correct answer is: Diazepam

Question 66 MoK Pharmacokinetics 026
Not answered When administered orally, the benzodiazepine that produces the most rapid onset of action is
Marked out of 1.00
Select one:

988
Flag question
Alprazolam
Lorazepam
Diazepam.
Chlordiazepoxide
Nitrazepam

Your answer is incorrect.
Diazepam has the most rapid onset of action among orally administered benzodiazepines.
The correct answer is: Diazepam.

Question 67 MoK Pharmacokinetics 027
Not answered You are treating an Afro­Caribbean man with Schizophrenia using clozapine. What is the
biggest risk factor for developing reduced white cell count?
Marked out of 1.00

Flag question Select one:
Male sex
Ethnicity
Dose related
Duration of treatment
Treatment setting

Your answer is incorrect.
There is a 77% increase in risk for Afro­Caribbean people to develop clozapine­induced
neutropenia. Other risk factors include young age and having a low baseline white cell count.
The correct answer is: Ethnicity

Question 68 MoK Pharmacokinetics 028
Not answered A patient who needs a prescription for a new antidepressant is almost certain that he will miss
one or two tablets during his treatment due to the nature of his job. What is the most suitable
Marked out of 1.00
choice of medications?
Flag question
Select one:
escitalopram
paroxetine

citalopram
venlafaxine
Fluoxetine

989
Your answer is incorrect.
Due to its longer half­life, missing fluoxetine for a day or two will not usually cause acute
withdrawal symptoms. This is a safe bet for the patient described in this question.
The correct answer is: Fluoxetine

Question 69 MoK Pharmacology EMI008
Not answered Choosing psychiatric medications
For each of the following descriptions choose the most appropriate antidepressant from the
Marked out of 3.00
given list
Flag question

A 24­year­old woman has had two
episodes of irritable mood, increased
energy, decreased need for sleep, Lithium
and pressured speech. She was Paroxetine
recently started on medication by her Reboxetine
psychiatrist to control these
Fluoxetine
symptoms. She presents to her GP
now complaining of a significant
Methylphenidate
worsening of her acne since starting Clozapine
this new medication. What drug was Citalopram
she started on? Escitalopram
Modafinil
A 24­year­old man has multiple
problems at his workplace due to Agomelatine
'extreme tiredness' during office Modafinil
hours. For several years, he has had Clozapine
severe daytime sleepiness, episodes
Reboxetine
of falling asleep without warning, and
hearing his name being called out
Agomelatine
loud on many occasions when no Paroxetine
one is around. Which of the following Escitalopram
treatments may be of benefit to this Fluoxetine
patient? Citalopram
Lithium
Citalopram
Methylphenidate
Which of the following drugs carried a Reboxetine
risk of thromboembolism especially Clozapine
during the first three months of Methylphenidate
treatment? Lithium
Paroxetine
Fluoxetine
Modafinil
Escitalopram
Your answer is incorrect. Agomelatine
Several cutaneous side effects are possible with lithium including acne, follicular and
maculopapular eruptions. Alopecia has also been reported. With the exception of lithium, the
other choices do not worsen acne. 
Case 2: The description in the questions fits with a diagnosis of narcolepsy. Stimulants such
as methylphenidate can reduce daytime sleepiness. Medications that reduce REM sleep,
such as TCAs and SSRIs, are used if cataplexy is also present. Modafinil can reduce the
number of sleep attacks and can improve psychomotor performance.

990
Clozapine's adverse effect of thromboembolism may be related to its effect on
antiphospholipid antibodies; though rare, if it occurs, this is usually in the first three months of
the treatment.
The correct answer is: A 24­year­old woman has had two episodes of irritable mood,
increased energy, decreased need for sleep, and pressured speech. She was recently started
on medication by her psychiatrist to control these symptoms. She presents to her GP now
complaining of a significant worsening of her acne since starting this new medication. What
drug was she started on? – Lithium, A 24­year­old man has multiple problems at his
workplace due to 'extreme tiredness' during office hours. For several years, he has had severe
daytime sleepiness, episodes of falling asleep without warning, and hearing his name being
called out loud on many occasions when no one is around. Which of the following treatments
may be of benefit to this patient? – Methylphenidate, Which of the following drugs carried a
risk of thromboembolism especially during the first three months of treatment? – Clozapine

Question 70 MoK Neurosciences EMI016
Not answered Neuropsychological measurement 
Identify the test, which could be used in each of the following situations
Marked out of 3.00

Flag question
WAIS ­R
Which of the above tests use the Rivermead Behavioural memory test
following question: 'How many Wisconsin card sorting test
camels are there in England?' National Adult Reading Test
(Choose one) Halstead­Reitan test battery
Mini­Mental State Examination
Luria­Nebraska battery
Ravens progressive matrices
Luria­Nebraska battery
Stanford­Binet test
Halstead­Reitan test battery
Scores are given for pronouncing the
words DRACHM, SUPERFLUOUS
Trail making test B
Trail making test A
and PLACEBO correctly (Choose Digit span
Stanford­Binet test
one) Trail making test A
Weschler Memory scale
Weschler Memory scale
Mini­Mental State Examination
Cognitive estimates test
Trail making test B
National Adult Reading Test
Mini­Mental State Examination
Ravens progressive matrices
Trail making test B
The subject is asked to connect
scattered numbers and alphabets in
Digit span
Halstead­Reitan test battery
the sequence 1A2B3C4D5E6F..., Wisconsin card sorting test
Trail making test A
using a pen and paper. (Choose one) Rivermead Behavioural memory test
Ravens progressive matrices
Cognitive estimates test
National Adult Reading Test
WAIS ­R
Luria­Nebraska battery
WAIS ­R
Weschler Memory scale
Your answer is incorrect. Cognitive estimates test
Wisconsin card sorting test
In cognitive estimates test, factual questions that require abstract processing are asked. This
Digit span
is a test of frontal function. In NART, scores are given for correct pronunciation of complex
Stanford­Binet test
phonetic words.­ In trail making test A, the subject has to simply connect 1­2­3­4­5­6...etc as a
Rivermead Behavioural memory test
sequence. In trail making test B, the subject has to connect the sequence
1A2B3C4D5E6F...etc.
The correct answer is: Which of the above tests use the following question: 'How many camels
are there in England?' (Choose one) – Cognitive estimates test, Scores are given for
pronouncing the words DRACHM, SUPERFLUOUS and PLACEBO correctly (Choose one) –

991
National Adult Reading Test, The subject is asked to connect scattered numbers and
alphabets in the sequence 1A2B3C4D5E6F..., using a pen and paper. (Choose one) – Trail
making test B

Question 71 MoK Pharmacology EMI010
Not answered Partial agonism 
Identify the predominant receptor level of action for the following partial agonistic drugs.
Marked out of 3.00

Flag question
Benzodiazepine receptor
D­2 receptor
Mu opioid receptor
Buspirone
5HT2 receptor
D­5 receptor
5 HT1A receptor
5HT3 receptor
D­1 receptor
D­1 receptor
Kappa opioid receptor
Kappa opioid receptor
Omega opioid receptor
Mu opioid receptor
Clonazepam
Omega opioid receptor
5HT2 receptor
D­5 receptor
5 HT1A receptor
Benzodiazepine receptor
D­1 receptor
D­2 receptor
Omega opioid receptor
5HT3 receptor
5 HT1A receptor
Buprenorphine
5HT3 receptor
D­5 receptor
Kappa opioid receptor
Mu opioid receptor
Benzodiazepine receptor
5HT2 receptor
D­2 receptor
Your answer is incorrect.
Partial agonist­ Aripiprazole ­ D2; Buspirone 5HT1A; Clonazepam ­ BDZ receptor;
Buprenorphine ­ opioid receptor mu.
Aripiprazole carries a low risk of extrapyramidal symptoms despite high levels of D2
occupancy (> 90%) at therapeutic doses; this is probably due to its partial­agonist activity at
D2 receptors. 
Buspirone acts as a partial agonist on serotonin 5­HT1A receptors ­ presynaptic agonism
leads to inhibition of release of serotonin, with consequent antianxiety effects. Postsynaptic
agonism leads to antidepressant activity. 
Benzodiazepines act via a specific site called omega site in GABA­A complex. All are agonists
except clonazepam, which is a partial agonist.
The correct answer is: Buspirone – 5 HT1A receptor, Clonazepam – Benzodiazepine
receptor, Buprenorphine – Mu opioid receptor

Question 72 MoK Pharmacology EMI011
Not answered Chemical class of psychotropic drugs 
Match each chemical class below with the most appropriate drug from the given list:

992
Marked out of 3.00
Clozapine
Flag question Olanzapine
Loxapine
Thienobenzodiazepine
Zuclopenthixol
Molindone
Amisulpride
Risperidone
Haloperidol
Quetiapine
Quetiapine
Haloperidol
Chlorpromazine
Amisulpride
Dibenzodiazepine Trifluoperazine
Molindone
Loxapine
Chlorpromazine
Trifluoperazine
Zuclopenthixol
Zuclopenthixol
Clozapine
Chlorpromazine
Olanzapine
Trifluoperazine
Substituted benzamides Risperidone
Olanzapine
Haloperidol
Molindone
Amisulpride
Quetiapine
Clozapine
Your answer is incorrect. Risperidone
Loxapine
Thienobenzodiazepine­ Olanzapine; 
Dibenzodiazepine­ Clozapine; Substituted benzamides include Amisulpride, Sulpiride. 
Dibenzothiazepine­ Quetiapine. 
Benzisoxazole derivatives include Risperidone; 
Thioxanthenes include Flupenthixol and Zuclopenthixol.
The correct answer is: Thienobenzodiazepine – Olanzapine, Dibenzodiazepine – Clozapine,
Substituted benzamides – Amisulpride

Question 73 MoK Neurosciences EMI011
Not answered Cerebral arteries & lesions
Using the clinical descriptions below, identify the artery most likely to be involved:
Marked out of 3.00

Flag question
Right anterior cerebral artery
A 58­year­old gentleman presents Subclavian artery proximal to ipsilateral vertebral artery
with diplopia, ataxia, nystagmus on Posterior inferior cerebellar artery
looking to the side of the lesion and Vertebral artery
Horner's syndrome Basilar artery
Left anterior cerebral artery
Posterior cerebral artery
Right middle cerebral artery
A 57­year­old gentleman who is a Left middle cerebral artery
right handed individual presenting Bilateral anterior cerebral artery
with right hemiparesis, dysphasia

993
and sensory loss involving face &
Bilateral anterior cerebral artery
arms on the right side.
Left middle cerebral artery
Posterior cerebral artery
Right middle cerebral artery
Vertebral artery
Subclavian artery proximal to ipsilateral vertebral artery
Right anterior cerebral artery
Left anterior cerebral artery
Basilar artery
A 67­year­old gentleman with Basilar artery
Bilateral anterior cerebral artery
presenting with right­sided weakness Posterior inferior cerebellar artery
Right anterior cerebral artery
especially in the legs. The weakness
Left anterior cerebral artery
is milder in the arm, and the face is
spared.
Right middle cerebral artery
Posterior inferior cerebellar artery
Subclavian artery proximal to ipsilateral vertebral artery
Left middle cerebral artery
Vertebral artery
Your answer is incorrect. Posterior cerebral artery
The lateral medullary syndrome or posterior inferior cerebellar artery (PICA) thrombosis, or
Wallenberg's syndrome, is a brain­stem infarction presenting as acute vertigo with cerebellar
signs. It follows thromboembolism in the PICA or its branches, vertebral artery
thromboembolism or dissection. On the side of the lesion, there could be facial numbness (5th
nerve), diplopia (6th nerve), nystagmus, ataxia (cerebellar), Horner's syndrome, 9th& 10th
nerve lesions. On the contralateral side, there could be spinothalamic sensory loss and mild if
any, hemiparesis. 
Middle cerebral artery lesions are characterized by hemiplegia, hemianesthesia. If the lesion
involves dominant hemisphere, the patients often present with other symptoms such as
dysphasia and dyspraxia. If the lesion involves non­dominant hemisphere, then contralateral
neglect is highly likely. 
Anterior cerebral artery lesions involve contralateral hemiplegia/hemiparesis with weakness
involving more of the legs than the arms, and the face will usually be spared.
The correct answer is: A 58­year­old gentleman presents with diplopia, ataxia, nystagmus on
looking to the side of the lesion and Horner's syndrome – Posterior inferior cerebellar artery, A
57­year­old gentleman who is a right handed individual presenting with right hemiparesis,
dysphasia and sensory loss involving face & arms on the right side. – Left middle cerebral
artery, A 67­year­old gentleman with presenting with right­sided weakness especially in the
legs. The weakness is milder in the arm, and the face is spared. – Left anterior cerebral artery

Question 74 MoK Neurosciences EMI013
Not answered Familial risk rates
Choose the most appropriate risk in each of the following situations:
Marked out of 3.00

Flag question
70­79%
1­5%
60­69%
30­39%
The risk of inheritance in a 30­year­ 40­49%
old man whose mother and maternal 10­19%
grandfather developed Alzheimer's 80­89%
disease in their 50s 50­59%
5­10%

994
10­19%
A 25­year­old woman with a history of 20­29%
bipolar disorder. What is the chance 1­5%
that one of her children may develop 80­89%
a bipolar spectrum disorder? 30­39%
70­79%
60­69%
50­59%
50­59%
A 36­year­old woman with unipolar
40­49%
20­29%
depression. What is the chance that 5­10%
1­5%
her daughter inherits the same 30­39%
illness? 40­49%
5­10%
80­89%
10­19%
70­79%
Your answer is incorrect. 60­69%
For the common late­onset form of Alzheimer's, the risk for those with no first degree relatives
(parents or siblings) with a diagnosis of Alzheimer's disease is 7% or less (given normal life
expectancy). The risk for those with one first­degree family member is about 15­20%. For
second degree relatives (e.g., grandparents) the risk may be about 10%. For monozygotic
twins, the risk is 80­85%. But in early­onset cases (<65) strong autosomal dominant tendency
is noted increasing the risk to >50%. 
Compared with the offspring of control parents, children of parents with bipolar disorder had
an increased risk of having a bipolar spectrum disorder (Odds Ratio 13.4). A meta­analysis
and more recent studies have reported rates of BP between 4% and 15% (B is the closest
answer) in the offspring of parents with BP and between 0% and 2% in the offspring of healthy
parents.(Lapalme et al. Can J Psychiatry. 1997;42(6):623­631). Children in families where
both parents had bipolar disorders also were more likely than those in families containing one
parent with bipolar disorder to develop the condition (Odds Ratio: 3.6) [Birmaher et al. Arch
Gen Psychiatry. 2009;66(3):287­296. Children of parents with bipolar disorder have an
increased risk of unipolar (11.4%) depression too. Children of parents with unipolar
depression have a 5.9 to 18.4 % (median 9.1%: the closest answer is 10­19%) risk of unipolar
depression and no increased risk of bipolar disorder.
The correct answer is: The risk of inheritance in a 30­year­old man whose mother and
maternal grandfather developed Alzheimer's disease in their 50s – 50­59%, A 25­year­old
woman with a history of bipolar disorder. What is the chance that one of her children may
develop a bipolar spectrum disorder? – 5­10%, A 36­year­old woman with unipolar
depression. What is the chance that her daughter inherits the same illness? – 10­19%

Question 75 MoK Neurosciences EMI015
Not answered Neuroimaging findings 
For each of the condition given below choose ONE appropriate finding from the above list
Marked out of 3.00

Flag question

995
Medial temporal atrophy (MRI)
Geriatric depression Increase in the volume of the brain
Bilateral high pulvinar signals (MRI)
Primary auditory cortex activation (fMRI)
Impaired dorso­lateral prefrontal function (fMRI)
Diffuse oedema of cerebral cortex (MRI)
Periventricular white matter changes (MRI)
Focal atrophy of occipital lobes
Deep white matter hyperintensities (MRI)
Deep white matter hyperintensities (MRI)
Impaired dorso­lateral prefrontal function (fMRI)
Focal frontal atrophy (MRI)
Focal frontal atrophy (MRI)
Variant CJD
Diffuse oedema of cerebral cortex (MRI)
Increase in the volume of the brain
Periventricular white matter changes (MRI)
Primary auditory cortex activation (fMRI)
Focal atrophy of occipital lobes
Bilateral high pulvinar signals (MRI)
Bilateral high pulvinar signals (MRI)
Focal frontal atrophy (MRI)
Auditory hallucinations in
Medial temporal atrophy (MRI)
Increase in the volume of the brain
schizophrenia Medial temporal atrophy (MRI)
Deep white matter hyperintensities (MRI)
Focal atrophy of occipital lobes
Periventricular white matter changes (MRI)
Impaired dorso­lateral prefrontal function (fMRI)
Primary auditory cortex activation (fMRI)
Your answer is incorrect. Diffuse oedema of cerebral cortex (MRI)
In geriatric depression the common MRI findings include atrophy, ventricular enlargement,
lesions in basal ganglia and deep white matter hyper intensities (WMH). WMH occur
particularly in the deep subcortical white matter and to a lesser extent in the basal ganglia and
periventricular white matter (Harrison, 2002). In major depression, WMH are linked to risk
factors for vascular disease, and thus are often seen in the elderly subjects. 
In variant CJD, MRI is the most useful supportive diagnostic test. A characteristic abnormality
seen in the posterior thalamic region (pulvinar sign) is highly sensitive and specific for variant
CJD. The pulvinar sign has been found in more than 90% of pathologically proven vCJD
cases. FLAIR sequences of MRI are most likely to show the abnormality. 
In patients with auditory hallucinations in Schizophrenia, primary auditory cortex activation is
seen in the f­MRI scan (Dierks et al­1999). Neuroimaging studies have shown that auditory
hallucinations in schizophrenic patients were associated with increased metabolic activity in
the left primary auditory cortex and the right middle temporal gyrus. 

Harrison, PJ (2002). The neuropathology of primary mood disorder. Brain,
http://brain.oxfordjournals.org/content/125/7/1428
The correct answer is: Geriatric depression – Deep white matter hyperintensities (MRI),
Variant CJD – Bilateral high pulvinar signals (MRI), Auditory hallucinations in schizophrenia –
Primary auditory cortex activation (fMRI)

Question 76 MoK Neurosciences EMI017
Not answered Personality tests
For each of the following tests given below, choose their correct names from the list given:
Marked out of 4.00

Flag question

996
This test is a way of measuring
Rey­Osterreith test
personality along the dimensions of
Stroop test
extroversion­introversion and
neuroticism­stability
Paired associates test
Wisconsin card sorting test
National adult reading test
Eysenck Personality Inventory
Rorschach inkblot test
Thematic apperception test
Thematic apperception test
Trail making test
Rey­Osterreith test
It is one of the objective personality Minnesota multiphasic personality inventory
Trail making test
tests that includes hypochondriasis Raven's progressive matrices
Minnesota multiphasic personality inventory
as a personality factor
National adult reading test
Paired associates test
Raven's progressive matrices
Eysenck Personality Inventory
Thematic apperception test
Rorschach inkblot test
Raven's progressive matrices
A projective personality test with Wisconsin card sorting test
Minnesota multiphasic personality inventory
pictures of ambiguous persons and Stroop test
Paired associates test
scenes to make up a story
Stroop test
Wisconsin card sorting test
Eysenck Personality Inventory
Rorschach inkblot test
Rey­Osterreith test
Rey­Osterreith test
Raven's progressive matrices
This test is useful to assess
National adult reading test
National adult reading test
premorbid intelligence. Trail making test
Trail making test
Stroop test
Rorschach inkblot test
Thematic apperception test
Minnesota multiphasic personality inventory
Paired associates test
Your answer is incorrect. Eysenck Personality Inventory
Wisconsin card sorting test
Eysenck's approach: Eysenck used second order analysis (orthogonal factor analysis) that
identified small number of powerful independent factors relevant to measuring personality.
This method yielded three­dimensional traits. These are neuroticism (vs. stability),
psychoticism and extraversion (vs. introversion). Eysenck's personality questionnaire contains
a lie scale. Eysenck maintained that no fundamental discontinuity exists between normal and
abnormal personalities (dimensional). 
Minnesota multiphasic personality inventory MMPI is a popular inventory for measuring
personality. It has ten scales with clinical labels including hypochondriasis and hypomania. It
is NOT a projective test. 
Rorschach Inkblot Test and Murray's Thematic Apperception Test are projective tests; so are
sentence completion and Draw a Person tests. Projective tests use ambiguous stimuli, and
the patient is expected to generate narratives based on them; rating scales may or may not be
used in conjunction to rate such measures. 
NART ­ National adult reading test: Taps on previous word knowledge before becoming ill ­
hence used to estimate premorbid IQ.
The correct answer is: This test is a way of measuring personality along the dimensions of
extroversion­introversion and neuroticism­stability – Eysenck Personality Inventory, It is one of
the objective personality tests that includes hypochondriasis as a personality factor –
Minnesota multiphasic personality inventory, A projective personality test with pictures of

997
ambiguous persons and scenes to make up a story – Thematic apperception test, This test is
useful to assess premorbid intelligence. – National adult reading test

Question 77 MoK Neurosciences EMI019
Not answered Derivatives of neural tube
For each of the following organs choose the neural tube vesicle of origin from the list
Marked out of 3.00
provided:
Flag question

Myelencephalon
Telencephalon
Mesencephalon
Thalamus
Diencephalon
Metencephalon

Mesencephalon
Telencephalon
Diencephalon
Hippocampus
Myelencephalon
Metencephalon

Metencephalon
Mesencephalon
Diencephalon
Cerebellum
Telencephalon
Myelencephalon

Your answer is incorrect.
The telencephalon (cerebrum) together with the diencephalon constitute the forebrain. The
diencephalon ("interbrain") is the region of the brain that includes the thalamus,
metathalamus, hypothalamus, epithalamus, prethalamus or subthalamus and pretectum.The
telencephalon or the cerebrum is composed of the following sub­regions:cerebral cortices,
basal ganglia and the limbic system. Cerebellum is part of the metencephalon, which also
includes the pons; the metencephalon is the upper part of the rhombencephalon or
"hindbrain".
The correct answer is: Thalamus – Diencephalon, Hippocampus – Telencephalon,
Cerebellum – Metencephalon

Question 78 MoK Neurosciences EMI027
Not answered Neuropathological features
You are working in a neuroscience department, and you have a special interest in dementias.
Marked out of 4.00
For each of the definitions given below, identify the neuropathological terms used to describe
Flag question them.

998
Spongiform encephalopathy
Lewy bodies
Weakly eosinophilic spherical
Pick cells
cytoplasmic inclusions
Hirano bodies
Microvacuolation
Alpha­synuclein
Prion proteins
Pick bodies
Pick bodies
Senile plaques
Alpha­synuclein
This substance accelerates reuptake Hirano bodies
of dopamine in neurons that might be
Lewy bodies
toxic
Senile plaques
Spongiform encephalopathy
Prion proteins
Pick cells
Microvacuolation
Microvacuolation
Pick cells
Abnormal swollen neuronal cells,
with loss of Nissl's substance, oval inAlpha­synuclein
shape with peripherally displaced Senile plaques
nucleus Lewy bodies
Pick bodies
Spongiform encephalopathy
Hirano bodies
Senile plaques
Prion proteins
Alpha­synuclein
These are masses of cytoskeletal
elements, which are argentophilic, Hirano bodies
tau and ubiquitin reactive filamentous Microvacuolation
inclusions. Prion proteins
Lewy bodies
Spongiform encephalopathy
Pick cells
Pick bodies
Your answer is incorrect.
Lewy bodies are weakly eosinophilic, spherical, cytoplasmic inclusions. In Parkinson's
disease they are confined to substantia nigra; in Lewy body dementia, they are also present in
many areas of the cerebrum including the temporal lobe, the cingulate gyrus and the frontal
lobes. Lewy bodies in Parkinson's disease and DLB contain accumulations of alpha­
synuclein. 
Alpha­synuclein accelerates reuptake of dopamine in neurons, and this dopamine overload
might be toxic. Lewy neurites are nerve cell processes that contain aggregates of alpha­
synuclein. These abnormal structures can occur in both DLB/dementia of Parkinson's disease
and idiopathic Parkinson's disease and are most numerous in the CA2/3 region of the
hippocampus and the substantia nigra.
A disproportionate atrophy of the frontotemporal regions than other brain regions is seen in
Pick's disease. These areas also show a reduction in large cortical nerve cells, abundant
gliosis, and neuronal Pick's bodies, which are masses of cytoskeletal elements. Pick's bodies
are argentophilic, tau and ubiquitin reactive filamentous inclusions. Abnormal swollen
neuronal cells, with a loss of Nissl's substance, oval in shape with peripherally displaced
nucleus are called Pick cells. 

Ref: Love S., (2005) Neuropathological investigation of dementia: a guide for neurologists.
Retrieved from http://jnnp.bmj.com/content/76/suppl_5/v8.full

999
The correct answer is: Weakly eosinophilic spherical cytoplasmic inclusions – Lewy bodies,
This substance accelerates reuptake of dopamine in neurons that might be toxic – Alpha­
synuclein, Abnormal swollen neuronal cells, with loss of Nissl's substance, oval in shape with
peripherally displaced nucleus – Pick cells, These are masses of cytoskeletal elements, which
are argentophilic, tau and ubiquitin reactive filamentous inclusions. – Pick bodies

Finish review

1000
 Home Mock Paper Practice Tests

Mock Paper A(2)

Started on Thursday, 25 June 2015, 5:53 AM
State Finished
Completed on Thursday, 25 June 2015, 5:54 AM
Time taken 22 secs
Grade 0.00 out of 100.00
Feedback You are on the right track. Could you spend some more time on your revision?

Question 1 MoK Applied Neuroscience 031
Not answered A man is unable to mimic the action of hammering a nail as instructed by the examiner. He is
exhibiting:
Marked out of 1.00

Flag question Select one:
Constructional apraxia
Finger agnosia
Ideomotor apraxia
Dressing apraxia
Visual object agnosia

Your answer is incorrect.
Ideomotor apraxia is a disorder of goal­directed movement. The patient knows what to do but
not how to do it. Tool use is more affected than spontaneous actions/gestures. There is an
impairment of pantomiming ability.
The correct answer is: Ideomotor apraxia

Question 2 MoK Applied Neuroscience 032
Not answered Which of the following tests of WAIS is resistant to brain damage?

Marked out of 1.00
Select one:
Flag question
Digit span
Similarities

Digit symbol
Block design
Information

1001
Your answer is incorrect.
The WAIS is composed of 11 subtests made up of six verbal subtests and five performance
subtests, which yield a verbal IQ, a performance IQ, and a combined or full­scale IQ. In WAIS,
hold tests are vocabulary, information, object assembly and picture completion. Non hold tests
are block design, digit span, similarities and digit symbol. A deterioration quotient is derived
from the difference between 'don't hold' and 'hold' test scores. Hold tests are supposed to be
resistant to age­related decline; so they may be sensitive to organic brain damage such as
dementia. But this is disputed.
The correct answer is: Information

Question 3 MoK Applied Neuroscience 033
Not answered Which of the following statements is true about neural correlates of sign language in deaf
patients?
Marked out of 1.00

Flag question Select one:
Occipital lobe takes over language processing functions in deaf patients
Comprehension of sign language does not require involvement of language
areas in brain
Right sided dominance for language is seen in most sign language users
Left temporal lesions can produce comprehension deficits in sign language
users
Aphasias are not seen in sign language users

Your answer is incorrect.
Deaf patients show a neural organisation of language cortex that is similiar to the non­deaf
population. Left temporal cortex is important for comprehending sign language while Broca's
area serves in the production of sign language.
The correct answer is: Left temporal lesions can produce comprehension deficits in sign
language users

Question 4 MoK Applied Neuroscience 034
Not answered Damage to which of the following regions unilaterally will produce an upper quadrantanopia?

Marked out of 1.00
Select one:
Flag question
Superior frontal gyrus

Medial temporal lobe
Inferior Parietal Lobule
Lower bank of calcarine fissure
Parietal operculum

1002
Your answer is incorrect.
The primary visual cortex lies on the banks of the calcarine fissure. Neurons located on the
lower bank are connected to the lateral geniculate nucleus carrying visual information from
the nasal or temporal upper visual fields. A lesion of this region can induce an upper
quadrantanopia (i.e., loss of one­quarter of the visual field).
The correct answer is: Lower bank of calcarine fissure

Question 5 MoK Applied Neuroscience 035
Not answered A patient has spent some time in hospital, and the team asked you to review him as his IQ
seems to have deteriorated. The best test to measure the patient's premorbid IQ is
Marked out of 1.00

Flag question Select one:
Beck's inventory
Paired associate learning test
Rey Osterrieth test
Luria motor test
National Adult Reading Test

Your answer is incorrect.
The National Adult Reading Test (NART) is a commonly used clinical method to estimate
premorbid intelligence levels in English­speaking patients with dementia and other organic
brain lesions. It was developed by Hazel Nelson in 1982.
The correct answer is: National Adult Reading Test


Question 6 MoK Applied Neuroscience 036
Not answered Broca's aphasia is also known as

Marked out of 1.00
Select one:
Flag question
Fluent aphasia
Non­fluent aphasia
Conduction aphasia
Nominal aphasia
Jargon aphasia

Your answer is incorrect.
Fluency and repetition are affected while comprehension is preserved in Broca's aphasia.
The correct answer is: Non­fluent aphasia

1003
Question 7 MoK Applied Neuroscience 037
Not answered Which of the following cognitive tests is useful to check semantic memory?

Marked out of 1.00
Select one:
Flag question
Copying pentagons and colouring them
Category fluency and digit­symbol test
Serial subtraction and proverb interpretation

Object naming and defining the meaning of words
Letter fluency

Your answer is incorrect.
Semantic memory involves memory for word meaning and general knowledge, the key neural
substrate being the anterior temporal lobe. The bedside tests used include object naming and
defining the meaning of words. Patients with semantic breakdown typically complain of loss of
words. Vocabulary diminishes, and patients substitute object names with stock­words e.g.
''thing''. There is a parallel impairment in appreciating the meaning of individual words, which
first involves infrequent or unusual words.
The correct answer is: Object naming and defining the meaning of words

Question 8 MoK Applied Neuroscience 038
Not answered Which one among the following is a neuropsychological theory of ADHD?

Marked out of 1.00
Select one:
Flag question
Low central coherence
Reversal learning

Orbitofrontal striatal executive dysfunction
Selective attention bias
Poor external locus of control

Your answer is incorrect.
Attention Deficit Hyperactivity Disorder has been implicated in dysfunction of neural reward
circuitry controlling motivation, reward, and impulsivity, including orbitofrontal cortex systems.
This condition has increasingly been defined as a disorder resulting from impaired behavioral
inhibition leading to executive function deficits.
The correct answer is: Orbitofrontal striatal executive dysfunction

Question 9 MoK Applied Neuroscience 039
Not answered Lesion of which lobe causes nominal aphasia?

Marked out of 1.00
Select one:
Flag question

1004
Frontal
Cerebellum
Parietal

Occipital
Temporal

Your answer is incorrect.
Although many experts have believed that damage to either Broca's or Wernicke's area is the
main cause of anomia, current studies have shown that damage in the left parietal lobe is the
epicenter of anomic aphasia. (Ref: Fridriksson J, Kjartansson O, Morgan PS, et al. (2010).
Impaired speech repetition and left parietal lobe damage. J. Neurosci. 30 (33): 11057­61.)
The correct answer is: Parietal

Question 10 MoK Applied Neuroscience 040
Not answered Jane experiences sensory aura with tingling, numbness with vague pain sensations, body
image distortions, vertigo and visual illusions. The most likely seat of epileptic focus is
Marked out of 1.00

Flag question Select one:
Brain stem
Frontal lobe

Temporal lobe
Parietal lobe
Occipital lobe

Your answer is incorrect.
Pain is rare as an epileptic phenomenon, but one­quarter of parietal lobe seizures have pain
as prominent symptom. Somatosensory seizures, somatic illusions, visual distortion, vertigo
and body­image distortions are other symptoms of parietal seizures. Ref:
http://www.med.nyu.edu/cec/epilepsy/types/parietal_lobe.html
The correct answer is: Parietal lobe

Question 11 MoK Applied Neuroscience 044
Not answered A lesion in which of the following sites is known to result in bitemporal hemianopia?

Marked out of 1.00
Select one:
Flag question
Frontal lobe tumour
Pituitary tumour

Temporal lobe tumour
Parietal lobe tumour

1005
Occipital lobe tumour

Your answer is incorrect.
While most children with pituitary tumours present with the clinical picture of raised
intracranial pressure,the majority o f the adults present with visual failure and optic atrophy.
Bitemporal hemianopia is fairly frequently found (one­third of patients at the time of diagnosis)
but is usually asymmetrical and unpredictable in its evolution. Br J Ophthalmol. 1975
Dec;59(12):689­95.
The correct answer is: Pituitary tumour

Question 12 MoK Applied Neuroscience 045
Not answered Alexia without agraphia (pure alexia) is seen in lesions of

Marked out of 1.00
Select one:
Flag question
Middle meningeal artery

Posterior cerebral territory
Anterior cerebral territory
Middles cerebral territory
Posterior inferior cerebellar artery

Your answer is incorrect.
Alexia without agraphia almost always involves an infarct to the left posterior cerebral artery
affecting splenium of the corpus callosum and left visual cortex. So the affected person, who is
still able to see with right visual cortex, cannot undertake lexical word processing making him
unable to read. But he can still write normally.
The correct answer is: Posterior cerebral territory

Question 13 MoK Applied Neuroscience 046
Not answered A major use of Glasgow Coma Scale is to
Marked out of 1.00
Select one:
Flag question
Predict mortality
Predict development of late complications
Predict time course for recovery
Predict development of co­morbidities
Predict development of early complications

Your answer is incorrect.

1006
The Glasgow Coma Scale (GCS) is routinely used in the acute care setting after traumatic
brain injury (TBI) to guide decisions in triage, based on its ability to predict mortality. Although
the GCS has been previously demonstrated to predict mortality, efficacy in the prediction of
functional outcome is not well established. GCS less than 8 can mean a mortality rate as high
as 70%. Balestreri, M., Czosnyka, M., Chatfield, D., Steiner, L., Schmidt, E., Smielewski, P., et
al. (2004). Predictive value of Glasgow Coma Scale after brain trauma: Change in trend over
the past ten years. Journal of Neurology, Neurosurgery, and Psychiatry, 75(1), 161?162.
The correct answer is: Predict mortality

Question 14 MoK Applied Neuroscience 047
Not answered A 43­year­old man can speak normally and comprehend what is spoken. He can also write
spontaneously and to dictation, but reading comprehension is impaired.
Marked out of 1.00

Flag question Select one:
Pure word deafness
Jargon aphasia
Broca's aphasia
Pure word blindness
Wernicke's aphasia

Your answer is incorrect.
Pure word blindness (alexia but no agraphia or agnosic alexia) is one form of alexia where
patients suffer from severe reading problems while other language­related skills such as
naming, oral repetition, auditory comprehension or writing are typically intact.
The correct answer is: Pure word blindness

Question 15 MoK Applied Neuroscience 049
Not answered Immediate repetition of 3 items in MMSE is used to test
Marked out of 1.00
Select one:
Flag question
Recognition
Recall
Digit span
Fluency
Registration

Your answer is incorrect.
Delayed recall can be tested by asking to repeat the items after a distraction test
The correct answer is: Registration

1007
Question 16 MoK Molecular Genetics 016
Not answered Alzheimer's disease is strongly associated with:
Marked out of 1.00
Select one:
Flag question
Presenile­1 gene on chromosome 1
Presenilin­2 gene on chromosome 2
COMT gene on chromosome 21
CAG repeats on chromosome 4
APO E4 on chromosome 19

Your answer is incorrect.
ApoE4 on Chr 19 is associated with amyloid precursor protein
The correct answer is: APO E4 on chromosome 19

Question 17 MoK Molecular Genetics 035
Not answered Which of the following is caused by trinucleotide repeat expansion?
Marked out of 1.00
Select one:
Flag question
Down's syndrome
Huntington's disease
Parkinson's disease
Schizophrenia
Lewy body dementia

Your answer is incorrect.
Trinucleotide repeat expansion is the underlying cause for fragile X syndrome and
Huntington's disease.
The correct answer is: Huntington's disease

Question 18 MoK Molecular Genetics 039
Not answered Which of the following components of transcribed RNA is removed before translation occurs?

Marked out of 1.00
Select one:
Flag question
Introns
Ribosomal attachment sites
Cap of RNA
Aminoacid coding portions

1008
Tail of RNA

Your answer is incorrect.
An intron is any nucleotide sequence within a gene that is removed before generating the
final mature RNA product of a gene. Introns are removed by a process called splicing before
protein translation begins. The ribonucleoprotein assembly that removes the intervening
sequences from pre­mRNAs through splicing is called the spliceosome.
The correct answer is: Introns

Question 19 MoK Molecular Genetics 040
Not answered Which of the following genetic element is implicated in Huntington's disease?

Marked out of 1.00
Select one:
Flag question
Presenilin­1 gene on chromosome 14
IT­15 chromosome 4
IT­4 on chromosome 15
APO E4 on chromosome 19
IT­1 gene on chromosome 1

Your answer is incorrect.
The mutation in the IT­15 gene ("interesting transcript 15") on chromosome 4 with an unstable
region of the harbouring an expanded trinucleotide repeat motif, is associated with
Huntington's disease.
The correct answer is: IT­15 chromosome 4

Question 20 MoK Molecular Genetics 041
Not answered The term used for purpose to identify locus of a gene on the chromosome by studying the
extent to which it, co­segregates with a marker gene is called as;
Marked out of 1.00

Flag question Select one:
Mutagenesis
Physical mapping
Association studies
Linkage analysis
Polymerase chain reaction

Your answer is incorrect.

1009
Linkage analysis is used to obtain a coarse chromosomal location of the genes associated
with a phenotype under investigation, e.g. a genetic disease or an important quantitative trait
The correct answer is: Linkage analysis

Question 21 MoK Molecular Genetics 042
Not answered Two parents are both affected with the same autosomal recessive disorder give birth to a
clinically normal child. Which of the following can explain the above?
Marked out of 1.00

Flag question Select one:
Incomplete penetrance
Variable expressivity
Pleiotropy
Locus heterogeneity
Allelic heterogeneity

Your answer is incorrect.
Locus heterogeneity refers to mutations at several different loci producing identical
phenotypes. A good example is presenilin1, 2 and Beta APP mutations all causing
phenotypically similar presenile Alzheimer's dementia. In this example, each parent may have
had a different mutation, so the child is a heterozygote carrier at both loci.
The correct answer is: Locus heterogeneity

Question 22 MoK Molecular Genetics 043
Not answered The term used to describe a point mutation that leads to failure of expression of alleles is
Marked out of 1.00
Select one:
Flag question
Dominant negative mutation
Lethal mutation
Apoptotic function
Loss of function
Lesion mutation

Your answer is incorrect.
Loss­of­function mutations are the result of gene product having less or no function. When the
allele has a complete loss of function, it is often called an amorphic mutation. Phenotypes
associated with such mutations are most often recessive (Mutation ­ DNA Wiki,
http://dnawiki.wikispot.org/Mutation).
The correct answer is: Loss of function

1010
Question 23 MoK Molecular Genetics 044
Not answered Following a Robertsonian translocation in phenotypically normal individual which of the
following types of chromosomes is produced?
Marked out of 1.00

Flag question Select one:
Holocentric
acrocentric
Metacentric
telocentric
submetacentric

Your answer is incorrect.
Following a Robertsonian translocation, the small 'p' arms are discarded, and a metacentric
fusion chromosome results.
The correct answer is: Metacentric

Question 24 MoK Molecular Genetics 045
Not answered A family with three affected children has a higher recurrence risk for a disease than another
family with only two affected children. This disease is most likely to be inherited via
Marked out of 1.00

Flag question Select one:
Mitochondrial
Autosomal dominant

Autosomal recessive
Multifactorial
X linked dominant

Your answer is incorrect.
An increasing recurrence risk according to the number of relatives affected is seen in
multifactorial diseases. Recurrence risks for Mendelian disorders are unaffected by number of
offsprings already affected.
The correct answer is: Multifactorial

Question 25 MoK Molecular Genetics 046
Not answered Which of the following disorders is associated with HLA­DQB1*­0602?

Marked out of 1.00
Select one:
Flag question
Huntington's disease
Narcolepsy

1011
Pick's disease
Schizophrenia
Bipolar disorder

Your answer is incorrect.
There is a strong association between HLA­DR2 haplotypes coded on chromosome 6 and
narcolepsy. HLA­DQB1*0602 and DQA *0102 are found in 85­95% of individuals, compared
to 12­38% in the general population.
The correct answer is: Narcolepsy

Question 26 MoK Molecular Genetics 047
Not answered A patient with schizophrenia is less likely to get married and produce children due to the
nature of the illness. This will serve to reduce the gene pool of schizophrenia causing genes.
Marked out of 1.00
This is called as
Flag question
Select one:
Pleiotropy
Natural selection
Assortative mating
Genetic drift
Negative eugenics

Your answer is incorrect.
Natural selection serves to maintain a low pool of certain disorders. Due to impaired fertility
and affliction at reproductive age, schizophrenia genes may be maintained at a lower level of
constancy in the population.
The correct answer is: Natural selection

Question 27 MoK Neuroanatomy 012
Not answered A man is brought to the A& E department with akinetic mutism. He has suffered a stroke.
Which artery is most likely to be involved?
Marked out of 1.00

Flag question Select one:
Basilar artery
Subclavian artery
Posterior inferior cerebellar artery
Middle cerebral artery
Bilateral anterior cerebral artery

1012
Your answer is incorrect.
The cingulate gyrus is supplied bilaterally by the anterior cerebral artery. An occlusion
involving bilateral anterior cerebral arteries could damage the cingulate gyri, leading to an
akinetic mute state.
The correct answer is: Bilateral anterior cerebral artery

Question 28 MoK Neuroanatomy 016
Not answered Communication between the frontal lobes of each side of the brain is mediated through which
of the following structures?
Marked out of 1.00

Flag question Select one:
Fornix
Genu of corpus callosum
Parahippocampal gyrus
Splenium of corpus callosum
Arcuate fasciculus

Your answer is incorrect.
The corpus callosum forms the major connection by which the hemispheres communicate
with one another. The genu of the corpus callosum contains fibers that pass from the frontal
lobe of one side to that of the other.
The correct answer is: Genu of corpus callosum

Question 29 MoK Neuroanatomy 018
Not answered Which of the following structures appears larger on the left side than the right in healthy right­
handed man?
Marked out of 1.00

Flag question Select one:
Frontal lobe
Hippocampus
Inferior temporal gyrus
Caudate nucleus
Heschl's gyrus

Your answer is incorrect.
In normal healthy right­handed subjects, Left larger than right symmetry is seen with structures
related to language processing such as planum temporale and Heschl's gyrus (primary
auditory cortex)
The correct answer is: Heschl's gyrus

1013
Question 30 MoK Neuroanatomy 019
Not answered A 40­year­old man develops depressed mood, anhedonia, initial and terminal insomnia, loss
of appetite, a 10­lb weight loss, and difficulty with sexual arousal. The clinical features of the
Marked out of 1.00
patient's psychiatric illness suggest dysfunction of the
Flag question
Select one:
Hippocampus
Corpus callosum
Frontal lobes
Pituitary
Hypothalamus

Your answer is incorrect.
Clinical studies of patients with major depressive disorders indicate that an intrinsic regulatory
defect involving the hypothalamus underlies the disorder. It also involves the monoamine
pathways. The hypothalamic modulation of neuroendocrine activity has been implicated, as
have been the neurotransmitter systems of serotonin and norepinephrine.
The correct answer is: Hypothalamus

Question 31 MoK Neuroanatomy 020
Not answered Which of the following statements about microglia is correct?

Marked out of 1.00
Select one:
Flag question
It is mainly responsible for myelination
It constitutes 70% of glial population in brain
It is sensitive to sodium channel stimulation
It is mesodermal in origin
It is present outside brain and spinal cord

Your answer is incorrect.
Some authors claim that microglia originate from the neuroepithelium, although most now
believe that microglial cells are of mesodermal origin, and probably belong to the
monocyte/macrophage cell line. These cells must enter the developing central nervous
system (CNS) from the blood stream, the ventricular space or the meninges. Afterward,
microglial cells are distributed more or less homogeneously through the entire nervous
parenchyma. Prog Neurobiol. 1998 Oct;56(2):173­89.
The correct answer is: It is mesodermal in origin

Question 32 MoK Neurochemistry 016
Not answered Which of the following is the primary mechanism involved in removal of the transmitter at the
neuromuscular junction?

1014
Marked out of 1.00 Select one:
Flag question Reuptake
Enzymatic degradation
Diffusion

Immune complex formation
Renal clearance

Your answer is incorrect.
A transmitter can be removed from the synaptic cleft through one of the following three modes:
(1) enzymatic degradation, (2) reuptake, and (3) diffusion. In neuromuscular junctions
acetylcholine, the primary neurotransmitter, undergoes enzymatic degradation.
The correct answer is: Enzymatic degradation

Question 33 MoK Neurochemistry 017
Not answered Removal of noradrenaline from the region of the synaptic cleft may be achieved by which of
the following mechanisms?
Marked out of 1.00

Flag question Select one:
reuptake only
diffusion only
conversion to adrenaline
enzymatic degradation only
enzymatic degradation, diffusion, and reuptake

Your answer is incorrect.
The most common of these mechanisms is reuptake, in which mediated by high­affinity
transmitter­specific transporter molecules.
The correct answer is: enzymatic degradation, diffusion, and reuptake

Question 34 MoK Neurochemistry 018
Not answered The metabotropic receptors are coupled to

Marked out of 1.00
Select one:
Flag question
Ligand non­dependent regulators of nuclear transcription
G proteins
Ligand dependent regulators of nuclear transcription
Proteins that have intrinsic enzyme activity
Ligand gated channels

1015
Your answer is incorrect.
Receptors are categorized into: (1) ionotropic: ligand­gated channels, in which binding of a
chemical messenger alters the probability of opening of transmembrane pores or channels;
(2) metabotropic: those in which the receptor proteins are coupled to intracellular G proteins
as transducing elements (3) intrinsic enzymes: those consisting of single membrane­spanning
protein units that have intrinsic enzyme activity (for example, having tyrosine kinase activity;
some authors classify them alongside metabotropic receptors); and (4) ligand­dependent
regulators of nuclear transcription (including receptors for corticosteroids such as
testosterone)
The correct answer is: G proteins

Question 35 MoK Neurochemistry 019
Not answered The neurotransmitter involved in the long­term potentiation of memory is

Marked out of 1.00
Select one:
Flag question
Noradrenaline
Dopamine
Acetyl choline
Glutamate
Serotonin

Your answer is incorrect.
Long­term potentiation is a long­lasting enhancement of signal transmission between 2
neurotransmitters that results from stimulating them synchronously. It is considered as one of
the major cellular mechanisms that underlie learning and memory. Glutamate, the major
excitatory neurotransmitter has NMDA receptors which mediate long­term potentiation.
The correct answer is: Glutamate

Question 36 MoK Neurochemistry 020
Not answered The neurotransmitter which is most widely distributed in the brain is

Marked out of 1.00
Select one:
Flag question
Dopamine
Glutamate
Acetylcholine
Norepinephrine
Serotonin

Your answer is incorrect.

1016
The most prevalent neurotransmitter in the brain is glutamate. It is the major excitatory
neurotransmitter with wide distribution in cerebral cortex, hippocampus and neocortical
projections in the brain
The correct answer is: Glutamate

Question 37 MoK Neuropathology 010
Not answered In a patient being evaluated for neuropsychiatric disturbances, CT scan shows atrophy of
head of caudate nucleus. Which of the following is a likely diagnosis?
Marked out of 1.00

Flag question Select one:
Alzheimer's disease
Wilson's disease
Fahr's disease
Huntington's disease
Pick's disease

Your answer is incorrect.
Huntington's is characterised by this finding especially in late stages; in Fahr's disease
calcified basal ganglia are noted.
The correct answer is: Huntington's disease

Question 38 MoK Neuropathology 011
Not answered Which of the following neuroimaging finding is most suggestive of new variant CJD?
Marked out of 1.00
Select one:
Flag question
Occipital hypoperfusion on HMPAO­ SPECT
Activation of primary auditory cortex on functional MRI
Impaired dorsolateral cortex function
Medial temporal atrophy on structural MRI scan
Increased signal in the pulvinar nucleus of thalamus bilaterally

Your answer is incorrect.
The MRI is the most useful supportive diagnostic test in variant CJD. A characteristic
abnormality seen in the posterior thalamic region (pulvinar sign) is highly sensitive and
specific for variant CJD.
The correct answer is: Increased signal in the pulvinar nucleus of thalamus bilaterally

Question 39 MoK Neuropathology 012
Not answered

1017
Marked out of 1.00 A patient develops viral encephalitis. He exhibits increased oral exploratory behaviour and
inappropriate sexuality. This syndrome can be called 
Flag question

Select one:
Kluver Bucy syndrome
Korsakoff syndrome
Lennox Gastaut syndrome
Gerstmann syndrome
Klein Levine syndrome

Your answer is incorrect.
Hypermetamorphopsia (compulsion to attend to all visual stimuli), hyperorality,
hypersexuality, blunted emotional reactivity is called Kluver­Bucy syndrome; the full syndrome
is rarely seen in man, but partial syndrome can occur with viral encephalitis affecting temporal
lobe.
The correct answer is: Kluver Bucy syndrome

Question 40 MoK Neurophysiology 010
Not answered A 44 year old man is treated for long standing asthma with a combination of oral theophylline,
prednisolone, inhaled salbutamol and inhaled budesonide. He appears heavy with purplish
Marked out of 1.00
striae on abdomen and excessive fat on the nape of his neck. His asthma is under control but
Flag question he exhibits behaviours and mental state consistent with hypomania. Which of the following is
a possible cause?

Select one:
Late onset bipolar disorder
Theophylline induced epileptiform activity

Hypothyroidism
Cushing's syndrome
Secondary diabetes mellitus

Your answer is incorrect.
The story is consistent with Cushing's syndrome. It is most likely due to oral prednisolone, not
inhaled budesonide.
The correct answer is: Cushing's syndrome

Question 41 MoK Neurophysiology 011
Not answered During EEG recording a resting subject is asked to focus his attention on a cognitive task.
Which of the following will be observed?
Marked out of 1.00

Flag question Select one:

1018
No change in resting EEG
Unilateral change in resting EEG at dominant hemisphere
Appearance of alpha waves on focussing attention
Disappearance of alpha waves on focussing attention
Disappearance of theta waves on focussing attention

Your answer is incorrect.
The disappearance of alpha waves on focussing attention is called as an alpha block; this
occurs when attention is directed.(Ganong's Review of Physiology 21e, Chapter 11., Lange).
The correct answer is: Disappearance of alpha waves on focussing attention

Question 42 MoK Neurophysiology 012
Not answered In the dementia associated with Alzheimer's disease, the EEG will usually show which of the
following?
Marked out of 1.00

Flag question Select one:
Periodic frontal lobe discharges
Generalized background slowing
Focal slowing
An isoelectric record
Spike­and­wave discharges

Your answer is incorrect.
In Alzheimer's disease, the frequency of background alpha rhythm may slow down with
reduced duration. Periodic discharges in the form of sharp waves or spikes may develop
during Creutzfeldt­Jakob disease. EEG is otherwise not especially helpful in distinguishing
between the common causes of dementia.
The correct answer is: Generalized background slowing

Question 43 MoK Adverse Effects 031
Not answered Which of the following drugs can cause whitish­brown granular deposits in the cornea and
lens ?
Marked out of 1.00

Flag question Select one:
Thioridazine
Amisulpride
Chlorpromazine
Quetiapine

1019
Olanzapine

Your answer is incorrect.
Irreversible pigmentation (similar to retinitis pigmentosa) with thioridazine, whitish­brown
granular deposits in the cornea and lens with chlorpromazine, and rarely glaucoma (and
other eye changes) with quetiapine are some of the ocular side effects of neuroleptics.
The correct answer is: Chlorpromazine

Question 44 MoK Adverse Effects 038
Not answered Risk factors for tardive dyskinesia includes all of the following except

Marked out of 1.00
Select one:
Flag question
Using high potency drugs
Having affective disorder diagnosis
Continuous use without drug holidays
Being Afro Caribbean
Having neurological damage

Your answer is incorrect.
Drug 'holidays' are associated with more frequent relapses, leading to higher than usual
doses of antipsychotics being used cumulatively This is associated with higher risk of Tardive
Dyskinesia.
The correct answer is: Continuous use without drug holidays

Question 45 MoK Adverse Effects 039
Not answered Rabbit syndrome presenting with mouth tremors is a result of

Marked out of 1.00
Select one:
Flag question
D1 receptor blockade
5HT2C receptor blockade
D2 receptor blockade
5HT2A receptor blockade
D4 receptor blockade

Your answer is incorrect.
The rabbit syndrome is a tremor affecting the lips and perioral muscles and occurs late in the
course of antipsychotic treatment. It is considered as a form of extrapyramidal symptom
related to D2 receptor blockade.

1020
The correct answer is: D2 receptor blockade

Question 46 MoK Adverse Effects 040
Not answered A patient with schizophrenia has been functioning considerably better since he started taking
clozapine. Currently he is on a dose of 600 mg/day, but he has experienced an episode of
Marked out of 1.00
new onset seizures. Which of the following is the most appropriate next step?
Flag question
Select one:
Add carbamazepine to clozapine
Add valproate to the same dose of clozapine
Add amisulpride
Decrease the dose of clozapine
Stop clozapine for 6 weeks

Your answer is incorrect.
Seizures related to clozapine are dose­/dose increase­related. In those patients who
experience seizures on clozapine, prophylactic valproate can be considered if on a high dose
or with high plasma level (more than 500 ng/l). After a seizure, withhold clozapine for one day;
restart at reduced dose; give sodium valproate. EEG abnormalities are common in those on
clozapine and does not warrant alteration in the treatment automatically. (Maudsley
Prescribing Guidelines 9thEd)
The correct answer is: Decrease the dose of clozapine

Question 47 MoK Adverse Effects 041
Not answered Benzodiazepine withdrawal is associated with:

Marked out of 1.00
Select one:
Flag question
Increased brain GABA function
Decreased brain dopamine function
Decreased brain GABA function
Increased brain serotonin function
Increased brain dopamine function

Your answer is incorrect.
People become tolerant to benzodiazepines probably because their nerve cells respond by
producing fewer receptors for GABA/benzodiazepines. This phenomenon, known as 'down­
regulation', means that the number of 'high affinity' GABA receptors decreases in response to
the enhancement of GABA caused by the drug. The body responds to the continued presence
of the drug with a series of adjustments that tend to overcome the drug effects. In the case of
benzodiazepines, compensatory changes occur in the GABA and benzodiazepine receptors

1021
which become less responsive, so that the inhibitory actions of GABA and benzodiazepines
are decreased (Ref: CH Ashton. The role of GABA and GABA receptors in benzodiazepine
withdrawal. http://www.benzosupport.org/gaba.htm)
The correct answer is: Decreased brain GABA function

Question 48 MoK Adverse Effects 042
Not answered A 48­year­old gentleman suffers from schizophrenia and is on Olanzapine treatment. He has
a BMI of 38 and suffers from drowsiness and erectile dysfunction. His diagnosis would be
Marked out of 1.00

Flag question Select one:
Kluver bucy syndrome
Cushing's syndrome
Addison's disease
Klein Levin syndrome
Diabetes mellitus

Your answer is incorrect.
Erectile dysfunction (ED) is a common problem amongst men who have diabetes affecting 35­
75% of male diabetics, which could be induced by Olanzapine. The other side effects of
Olanzapine include drowsiness and weight gain. Antipsychotic induced sedation and weight
gain may reduce sexual desire.
The correct answer is: Diabetes mellitus

Question 49 MoK Adverse Effects 043
Not answered Which of the following drugs can markedly increase blood acetaldehyde concentration?

Marked out of 1.00
Select one:
Flag question
Acamprosatwe
Morphine
Disulfiram
Naltrexone
Biperiden

Your answer is incorrect.
Aldehyde dehydrogenase (ALDH) is the enzyme that facilitates the catabolism of
acetaldehyde to acetic acid. When the action of ALDH is inhibited by disulfiram, acetaldehyde
is not converted to acetate but accumulates in the blood. Most of the symptoms of the
disulfiram­ethanol are due to the increased circulating acetaldehyde.
The correct answer is: Disulfiram

1022
Question 50 MoK Adverse Effects 044
Not answered A 33­year­old woman with a history of recurrent depressive disorder and is on fluoxetine 20
mg/day. She wishes to conceive and if she takes fluoxetine during pregnancy, what is the risk
Marked out of 1.00
of it causing major malformations in the foetuses?
Flag question
Select one:
Trebles the risk
Does not appear to significantly increase the risk
Doubles the risk
Quadruples the risk

More than quadruples the risk

Your answer is incorrect.
Tricyclics and fluoxetine do not have a known teratogenic effect in humans. Miscarriages do
not seem to be increased by fluoxetine
The correct answer is: Does not appear to significantly increase the risk

Question 51 MoK Adverse Effects 045
Not answered The drug of choice for treatment of restless legs syndrome is
Marked out of 1.00
Select one:
Flag question
Antidepressant
Dopamine agonist
Baclofen
Clonazepam
Antipsychotic

Your answer is incorrect.
Currently, dopamine agonists such as ropinirole, pramipexole, carbidopa/levodopa or
pergolide are commonly used for the treatment of restless legs syndrome. Clonazepam may
also be beneficial.
The correct answer is: Dopamine agonist

Question 52 MoK Adverse Effects 046
Not answered In patients receiving lithium therapy, the following can be observed even in normal
therapeutic doses:
Marked out of 1.00

Flag question Select one:
ECG changes

1023
Coarse tremors
Bradycardia
Pulmonary oedema
Ataxia

Your answer is incorrect.
Reversible flattening and inversion of T­waves are seen even at therapeutic doses, but
usually produce no clinical consequences. Br. J. Clin. Pharmac. (1980), 9, 599­604
The correct answer is: ECG changes

Question 53 MoK Adverse Effects 047
Not answered One among the following is not a feature of serotonin syndrome

Marked out of 1.00
Select one:
Flag question
Hyperreflexia
Confusion
Myoclonic Jerks
Electric shock like sensations
Autonomic instability

Your answer is incorrect.
Serotonin syndrome: Confusion, pyrexia, sweating, Myoclonic Jerks, Hyperreflexia,
Autonomic instability, gastrointestinal symptoms, mood changes and mania. Note that shock­
like sensations are seen in SSRI discontinuation syndrome.
The correct answer is: Electric shock like sensations

Question 54 MoK Adverse Effects 048
Not answered Which of the following is a potentially serious adverse effect associated with tacrine?

Marked out of 1.00
Select one:
Flag question
Elevated liver enzymes
Blood dyscrasias
Respiratory arrest

Retinal damage
Renal failure

Your answer is incorrect.

1024
Elevated liver enzymes (potentially serious), anorexia, nausea, rash, myalgia, loss of weight,
and increased sweating are some of the side effects of tacrine, the first introduced anti­
dementia drug. Donepezil is specific for cholinesterase inhibition in the CNS while tacrine is
not. Donepezil, therefore, has less peripheral adverse effects.
The correct answer is: Elevated liver enzymes

Question 55 MoK Adverse Effects 050
Not answered Which of the following is a predisposing factor for QT prolongation?

Marked out of 1.00
Select one:
Flag question
Hyperkalemia
Hypermagnesemia
Male gender
Hypercalcemia
Female gender

Your answer is incorrect.
New cases of long QT syndrome are diagnosed more in female patients (60­70% of cases)
than male patients. The female predominance may be related to the relatively prolonged QTc
(as determined by using the Bazett formula) in women compared with men and to a relatively
higher mortality rate in young men ( Long QT Syndrome ­ Medscape Reference,
http://emedicine.medscape.com/article/157826­overview (accessed April 13, 2015).).
Electrolyte abnormalities (especially hypokalaemia or hypomagnesaemia) are also
associated.
The correct answer is: Female gender

Question 56 MoK Pharmacodynamics 021
Not answered Which of the following antipsychotics is the fastest to dissociate from the Dopamine D2
receptor?
Marked out of 1.00

Flag question Select one:
Aripiprazole
Xanomeline
Clozapine
Paliperidone
Risperidone

Your answer is incorrect.
Animal data show that a rapid dissociation from the D(2) receptor at a molecular level
produces the atypical antipsychotic effect. In vitro data show that the single most powerful
predictor of atypicality for the current generation of atypical antipsychotics is fast dissociation

1025
from the D(2) receptor, not its high affinity at 5­HT(2), D(4), or another receptor. It has also
been postulated that clozapine, as well as quetiapine, bind more loosely to the Dopamine
receptor, and show a fast dissociation (Kapur & Seeman, 2001. Retrieved from
http://ajp.psychiatryonline.org/doi/abs/10.1176/appi.ajp.158.3.360).
The correct answer is: Clozapine

Question 57 MoK Pharmacodynamics 022
Not answered Which of the following street drugs has local anaesthetic properties?

Marked out of 1.00
Select one:
Flag question
Ketamine
Cannabis
LSD
Cocaine
Heroin

Your answer is incorrect.
Cocaine has membrane stabilising, and thus local anaesthetic effects.
The correct answer is: Cocaine

Question 58 MoK Pharmacodynamics 023
Not answered A 41­year­old woman is taking disulfiram to aid abstinence from alcohol. She present to A and
E with sickness and feeling unwell. On examination she appears to have facial flushing with
Marked out of 1.00
low BP but high pulse rate. She denies having drunk alcohol. Which of the following might
Flag question explain her symptoms?

Select one:
Mature cheese
2 mugs of black coffee
Cough syrup
Banana yoghurt
Tramadol tablets

Your answer is incorrect.
Many cough and common cold ailments contain a significant amount of alcohol sufficient to
precipitate an alcohol­disulfiram reaction.
The correct answer is: Cough syrup

Question 59 MoK Pharmacodynamics 025

1026
Not answered The antiemetic property of chlorpromazine is due to
Marked out of 1.00
Select one:
Flag question
D2 blockade
5HT2A blockade
Antiadrenergic effect

Anticholinergic effect
5HT3 blockade

Your answer is incorrect.
D2 blockade due to chlorpromazine in chemoreceptor trigger zone explains its antiemetic
property.
The correct answer is: D2 blockade

Question 60 MoK Pharmacodynamics 026
Not answered Sally is taking Amisulpride and is complaining of amenorrhea, galactorrhoea and loss of
sexual desire. This effect is caused by
Marked out of 1.00

Flag question Select one:
5HT1A antagonism
D1 blockade

D2 blockade
D3 blockade
5HT2A antagonism

Your answer is incorrect.
This patient is suffering from clinical features of hyperprolactinemia (amenorrhea,
galactorrhoea and loss of sexual desire) caused by antipsychotics. Hyperprolactinemia
caused by antipsychotics is due to D2 blockade and/or 5HT1A stimulation. Dopamine inhibits
prolactin release and so dopamine antagonists can be expected to increase prolactin levels.
The correct answer is: D2 blockade

Question 61 MoK Pharmacodynamics 027
Not answered Which of the following is a known psychiatric effect of the drug clonidine?

Marked out of 1.00
Select one:
Flag question
Psychosis
Mania
Anorexia

1027
Depression
Dementia

Your answer is incorrect.
Other drugs that can cause depression include propranolol, digoxin, frusemide, reserpine,
cimetidine, methyldopa, bromocriptine, and phenothiazines.
The correct answer is: Depression

Question 62 MoK Pharmacodynamics 028
Not answered The term "intrinsic activity" of a drug­receptor complex refers to
Marked out of 1.00
Select one:
Flag question
The efficiency of a drug in reaching the appropriate receptors
Potency of the drug to cause a functional response
Affinity of the drug to the receptor
Efficacy of the resulting functional response
The receptor action in the absence of the drug molecule

Your answer is incorrect.
Intrinsic activity refers to the capability of a drug­receptor complex to produce the highest
functional response (i.e. efficacy).
The correct answer is: Efficacy of the resulting functional response

Question 63 MoK Pharmacodynamics 029
Not answered Gamma hydroxybutyrate (GHB) acts on which of the following receptors?

Marked out of 1.00
Select one:
Flag question
Dopamine
Opiod
GABA­B
Serotonin
Noradrenaline

Your answer is incorrect.
GHB is primarily a GABAB agonist, naturally occurring in the brain, as well as in wine, beef,
small citrus fruits, and almost all animals in small amounts. It was synthesized as an
anesthetic initially; later banned due to abuse by body builders (as it was thought to boost

1028
growth hormone release) and use as a date rape drug (colourless liquid, easily concealed). It
was also abused for euphoric effects that are short lived. It is used as substitute for ecstasy in
dance parties (raves)­ called liquid ecstasy
The correct answer is: GABA­B

Question 64 MoK Pharmacokinetics 029
Not answered A patient who has had a renal transplant is currently on cyclosporine for immunosuppression.
He appears to be clinically depressed. The renal physician says it has been difficult titrating
Marked out of 1.00
the cyclosporine levels. Which of the following drugs must be avoided as the first line
Flag question intervention?

Select one:
amitriptyline
fluoxetine
venlafaxine
citalopram
phenelzine

Your answer is incorrect.
Fluoxetine, fluvoxamine and nefazodone are potent CYP3A4 inhibitors and can significantly
affect cyclosporine levels in patients.
The correct answer is: fluoxetine

Question 65 MoK Pharmacokinetics 030
Not answered A 60­year­old gentleman has a diagnosis of bipolar affective disorder and is on Lithium
carbonate. He has recently developed systemic hypertension. Which one among the following
Marked out of 1.00
is the most appropriate drug to treat his hypertension?
Flag question
Select one:
Bendroflumethiazide
Amiloride
Captopril
Furosemide
Chlorthalidone

Your answer is incorrect.
Diuretics can increase serum lithium levels markedly by decreasing its clearance. Thiazides
are the worst culprits while loop diuretics are somewhat safer. ACE Inhibitors decrease the
excretion of Lithium and can also precipitate renal failure.
The correct answer is: Furosemide

1029
Question 66 MoK Pharmacokinetics 031
Not answered The safest diuretic to be used with lithium is

Marked out of 1.00
Select one:
Flag question
Bendrofluomethiazide
Furosemide
Xipamide

Chlorthalidone
Indapamide

Your answer is incorrect.
The following are loop diuretics­ furosemide, bumetanide and torasemide. Although there are
case reports of lithium toxicity induced by loop diuretics, many patients receive this
combination of drugs without apparent problems. Patients taking blood diuretics may also
have been advised to restrict their salt intake, which may contribute to the risk of lithium
toxicity in these individuals. Thiazide diuretics (bendroflumethiazide, chlorthalidone,
indapamide, metolazone, xipamide), care is required, as these classes of drugs can reduce
the renal clearance of lithium and precipitate toxicity. (Ref: The Maudsley prescribing
guidelines)
The correct answer is: Furosemide

Question 67 MoK Pharmacokinetics 032
Not answered A patient on MAOI can consume

Marked out of 1.00
Select one:
Flag question
Grape fruit juice
Broad beans

Red wine
Pickled herring
Cheese

Your answer is incorrect.
Unlike the others listed, grapefruit juice is not rich in tyramine
The correct answer is: Grape fruit juice

Question 68 MoK Pharmacokinetics 034
Not answered Which of the following is true regarding diazepam?

Marked out of 1.00
Select one:
Flag question
Has 95% protein binding

1030
Has an inactive metabolite called desmethyldiazepam
Does not cause blurred vision
Half­life is 12 hours
Presence of food increases the extent of absorption

Your answer is incorrect.
Diazepam is a 1,4­benzodiazepine. Diazepam has an active metabolite desmethyldiazepam.
It is rapidly absorbed with nearly 100% oral and close to 95% is bound in plasma. Food may
inhibit absorption. The half­life of diazepam is 43±13 hours but ranges from 40­100 hours if
the contribution from active metabolites is included. Side effects may include dry mouth,
blurred or double vision, headache, vertigo, urinary retention, excessive perspiration, nausea
and vomiting, ataxia, tremor, depression, hypotension and diminished reflexes ( Drugs and
Human Performance FACT SHEETS ­ Diazepam. Retrieved from
http://www.nhtsa.gov/People/injury/research/job185drugs/diazepam.htm).
The correct answer is: Has 95% protein binding

Question 69 MoK Pharmacokinetics 035
Not answered Which one among the following statements about mianserin is true?

Marked out of 1.00
Select one:
Flag question
Inhibits uptake of both Noradrenaline and 5­HT
Has considerable alpha 1 adrenoreceptor blocking properties
Has a plasma half life that allows once daily dosage

Has strong antimuscarinic effects
Has significant sedative effects

Your answer is incorrect.
Mianserin is a potent inhibitor of Noradrenaline uptake with little effect on 5­HT. It is an alpha
2 antagonist and has weak alpha 1 antagonism. It is mildly sedative and has weak
antimuscarinic effects. It is rapidly absorbed from the gut with the peak plasma concentration
appearing in less than 3 hours after a single oral dose. The plasma half­life is 10­20 hours.
The correct answer is: Has a plasma half life that allows once daily dosage

Question 70 MoK Pharmacokinetics 036
Not answered Which of the following is NOT a phase 2 metabolic reaction in drug elimination?

Marked out of 1.00
Select one:
Flag question
N acetylation
Conjugation
Methylation

1031
Hydrolysis
Glucuronidation

Your answer is incorrect.
Conjugation, N­acetylation, methylation and glucuronidation are phase 2 reactions. Oxidation
and hydrolysis are phase 1 reactions.
The correct answer is: Hydrolysis

Question 71 MoK Adverse Effects 051
Not answered Thromboembolism as a possible side effect of clozapine treatment seems most likely to occur
in the
Marked out of 1.00

Flag question Select one:
Within 6­8 weeks of starting treatment
First 3 months of treatment
First 4 days of treatment
First 4 months of treatment
First 4 weeks of treatment

Your answer is incorrect.
Clozapine's adverse effect of Thromboembolism may be related to its observed effect on
Antiphospholipid antibodies; though rare, if it occurs, this is usually in the first three months of
the treatment.
The correct answer is: First 3 months of treatment

Question 72 MoK Pharmacokinetics 037
Not answered Which of the following antidepressants' plasma levels correlates with therapeutic response?

Marked out of 1.00
Select one:
Flag question
Imipramine
Mirtazapine
Amoxapine
Sertraline
Duloxetine

Your answer is incorrect.

1032
The clinical outcome when using imipramine is unequivocally related to the plasma level.
Patients with levels less than 180 ng/ml had a low probability of recovery while levels above
180 ng/ml were associated with a high probability of recovery. Unlike nortriptyline, there is no
tight therapeutic window for the efficacy of imipramine.
The correct answer is: Imipramine

Question 73 MoK Pharmacokinetics 038
Not answered Which of the following truly reflects the effects of smoking?

Marked out of 1.00
Select one:
Flag question
increases clozapine levels
no tolerance develops to physical effects
inhibits CYP1A2
increases caffeine metabolism
reduces risk of depression in adults

Your answer is incorrect.
Smoking reduces blood levels of clozapine and is associated with poor therapeutic responses
to clozapine. Smokers metabolize caffeine twice as fast as non­smokers. Smoking can
substantially increase CYP1A2 activity.
The correct answer is: increases clozapine levels

Question 74 MoK Pharmacology EMI012
Not answered Pharmacodynamics
Select the mechanism of action for each of the following drugs using the given list:
Marked out of 3.00

Flag question
5HT1A partial agonist
GABA­A agonist
NMDA receptor antagonist
Memantine
Butyryl­cholinesterase inhibitor
Presynaptic alpha­2 agonist
MAO­B Inhibitor
D­2 receptor partial agonist
GABA­B agonist
MAO­A Inhibitor
MAO­A Inhibitor
5­HT2 antagonist
5­HT2 antagonist
D­2 receptor partial agonist
Buspirone
GABA­B agonist
MAO­B Inhibitor
Presynaptic alpha­2 agonist
5HT1A partial agonist
NMDA receptor antagonist
GABA­A agonist
Butyryl­cholinesterase inhibitor
Lofexidine

1033
5­HT2 antagonist
Butyryl­cholinesterase inhibitor
GABA­B agonist
Presynaptic alpha­2 agonist
MAO­B Inhibitor
MAO­A Inhibitor
GABA­A agonist
D­2 receptor partial agonist
NMDA receptor antagonist
5HT1A partial agonist
Your answer is incorrect.
Memantine is an antidementia drug that acts through blockade of N­methyl­d­aspartate
(NMDA) glutamate receptors. Unlike ketamine that is a high­affinity noncompetitive blocker,
memantine is a non­competitive blocker with low affinity and binds only to actively open
NMDA channels. Its receptor dissociation rate is relatively fast, and so it does not accumulate
and interfere with normal NMDA activity.
Buspirone is a partial agonist on serotonin 5­HT1A receptors. At presynaptic levels, it is mostly
a full agonist, which inhibits the release of serotonin, with consequent antianxiety effects.
Partial agonist action at postsynaptic receptors appears to account for the antidepressant
activity. 
Lofexidine is a presynaptic alpha two agonist ­ leads to reduced central sympathetic tone.
Opioid receptors on locus coeruleus projections reduce noradrenergic tone on long­term use.
The cellular machinery compensates via up­regulation of adenylate cyclase and maintains
normal sympathetic tone in a chronic user. Sudden withdrawal leads to increased adrenergic
firing rate (withdrawal symptoms); hence alpha two autoreceptor stimulation that reduces
central sympathetic tone helps in treating opioid withdrawal.
The correct answer is: Memantine – NMDA receptor antagonist, Buspirone – 5HT1A partial
agonist, Lofexidine – Presynaptic alpha­2 agonist

Question 75 MoK Pharmacology EMI014
Not answered Adverse reactions
Choose one option from the given list for each description below:
Marked out of 3.00

Flag question A 22­year­old man is admitted in a
secure unit following an assault on Synergistic action
two policemen. He is very agitated Withdrawal effect
and dehydrated as a result of the
Pharmacokinetic adverse interaction
long struggle while arresting him. He
Dose­dependent side effect
was given rapid tranquillisation, soon
after which he shows signs of
Idiosyncratic reaction
hyperthermia, fluctuant BP and Pharmacodynamic adverse reaction
rigidity. Toxicity
Time­dependent side effect
A 40­year­old man on lithium Time­dependent side effect
Nocebo effect
treatment has returned from a trip to Nocebo effect
Greece where he suffered from
Pharmacokinetic adverse interaction
traveller's diarrhoea. He made a
Withdrawal effect
good recovery but when he flew back
he noticed tremors and soon
Pharmacodynamic adverse reaction
developed nausea. Synergistic action
Idiosyncratic reaction
Toxicity
Dose­dependent side effect

1034
A 45­year­old man is on quetiapine. Nocebo effect
Due to lack of efficacy and poor
Idiosyncratic reaction
compliance he is transferred to
risperidone depot. During the
Pharmacodynamic adverse reaction
transition, he was switched to oral Synergistic action
risperidone and simultaneous depot Time­dependent side effect
administration. He develops a stiff Toxicity
back and some difficulty in rolling Pharmacokinetic adverse interaction
over in his bed. Withdrawal effect
Dose­dependent side effect

Your answer is incorrect.
The first patient has developed Neuroleptic Malignant Syndrome, which is an idiosyncratic
reaction, which can occur at anytime during treatment with neuroleptics. It consists of the
tetrad of extreme hyperthermia, severe muscular rigidity and confusion, and autonomic
fluctuations (varying BP and pulse rate). 
The second patient has developed signs of lithium toxicity, which occurs in conditions of
overdose or dehydration due to vomiting or diarrhoea. Gastrointestinal symptoms are followed
by neurological symptoms. Immediate stopping of lithium and medical attention is required. 
The third patient has developed dose­dependent side effects of risperidone due to cumulative
drug use leading to extra pyramidal side effects (stiff back is due to axial dystonia).
The correct answer is: A 22­year­old man is admitted in a secure unit following an assault on
two policemen. He is very agitated and dehydrated as a result of the long struggle while
arresting him. He was given rapid tranquillisation, soon after which he shows signs of
hyperthermia, fluctuant BP and rigidity. – Idiosyncratic reaction, A 40­year­old man on lithium
treatment has returned from a trip to Greece where he suffered from traveller's diarrhoea. He
made a good recovery but when he flew back he noticed tremors and soon developed
nausea. – Toxicity, A 45­year­old man is on quetiapine. Due to lack of efficacy and poor
compliance he is transferred to risperidone depot. During the transition, he was switched to
oral risperidone and simultaneous depot administration. He develops a stiff back and some
difficulty in rolling over in his bed. – Dose­dependent side effect

Question 76 MoK Pharmacology EMI015
Not answered Classifying adverse reactions
Choose one option from the given list for each description below:
Marked out of 3.00

Flag question
A 47­year­old gentleman with a Synergistic action
history of chronic opioid misuse is Time­dependent side effect
unable to get his drugs for last threeNocebo effect
days. He develops insomnia, Idiosyncratic reaction
restlessness, diarrhoea and Dose­dependent side effect
tachycardia Pharmacodynamic adverse reaction
Toxicity
Pharmacokinetic adverse interaction
A 45­year­old woman with a history of
Withdrawal effect
bipolar depression was commenced
on Lamotrigine recently. She has
developed a severe rash and
diagnosed with Stevens­ Johnson
syndrome.

1035
Withdrawal effect
Pharmacokinetic adverse interaction
Toxicity
Time­dependent side effect
Nocebo effect
Dose­dependent side effect
Idiosyncratic reaction
Pharmacodynamic adverse reaction
Withdrawal effect
A 67­year­old lady with Synergistic action
Time­dependent side effect
schizoaffective disorder is on depot
Dose­dependent side effect
antipsychotic for several years. She
Nocebo effect
presents with an odd orofacial
movement disorder.
Idiosyncratic reaction
Synergistic action
Pharmacokinetic adverse interaction
Pharmacodynamic adverse reaction
Toxicity
Your answer is incorrect.
The first patient suffers from opioid withdrawal effects, related to abrupt discontinuation
characterised by insomnia, restlessness, diarrhoea and tachycardia. 
In the second case, lamotrigine is known to cause idiosyncratic side effects like Steven
Johnson syndrome that is non­dose related. 
Tardive dyskinesia and other related ('tardive' ­ slow onset) syndromes can be classified as
time­dependent side effects.
The correct answer is: A 47­year­old gentleman with a history of chronic opioid misuse is
unable to get his drugs for last three days. He develops insomnia, restlessness, diarrhoea and
tachycardia – Withdrawal effect, A 45­year­old woman with a history of bipolar depression
was commenced on Lamotrigine recently. She has developed a severe rash and diagnosed
with Stevens­ Johnson syndrome. – Idiosyncratic reaction, A 67­year­old lady with
schizoaffective disorder is on depot antipsychotic for several years. She presents with an odd
orofacial movement disorder. – Time­dependent side effect

Question 77 MoK Neurosciences EMI012
Not answered Cerebral circulation disorders
Identify 3 common symptoms seen in following types of cerebral circulation disorders.
Marked out of 6.00

Flag question
Hemiparesis
Myoclonus
Horner syndrome
Carotid system TIA
Nystagmus
Dysarthria
Ataxia
Agraphia
Diplopia
Horner syndrome
Amaurosis fugax
Aphasia
Aphasia
Nystagmus
Hemiparesis
Amaurosis fugax
Vertebrobasilar system TIA
Agraphia
Myoclonus
Ataxia

1036
Your answer is incorrect.
Carotid system TIA is also called as anterior circulation TIA. It is characterised by amaurosis
fugax, aphasia, hemiparesis, hemisensory loss and hemianopic visual loss due to the
involvement of watershed or highly oxygen dependent cortical regions. 
Vertebrobasilar system TIA is also called as posterior circulation TIA. It affects brain stem
cranial nerve nuclei leading to diplopia, vertigo, vomiting, choking, dysarthria, and ataxia,
alexia without agraphia, hemisensory loss, hemianopic visual loss, transient global amnesia,
quadriparesis and loss of consciousness (rare).
The correct answer is: Carotid system TIA – Amaurosis fugax, Hemiparesis, Aphasia,
Vertebrobasilar system TIA – Dysarthria, Ataxia, Diplopia

Question 78 MoK Neurosciences EMI018
Not answered Neuroimaging
Choose ONE most relevant investigation for each scenario described below.
Marked out of 3.00

Flag question
Functional MRI
Contrast CT scan.
Differentiation of drug­induced
Iomazenil (I123)
parkinsonism from Parkinson's
Volumetric MRI.
disease in an elderly woman.
ß­CIT SPECT
11C raclopride
Technetium HMPAO SPECT (99mTc).
T2 weighted MRI.
Volumetric MRI.
A researcher is interested in
Fluoro­deoxyglucose (18F)
Fluoro­deoxyglucose (18F)
comparing the degree of dopamine Iomazenil (I123)
release after cocaine use and Contrast CT scan.
amphetamine use using PET scan. Functional MRI
11C raclopride
Technetium HMPAO SPECT (99mTc).
T2 weighted MRI.
Iomazenil (I123)
ß­CIT SPECT
ß­CIT SPECT
Indirect measurement of cerebral Fluoro­deoxyglucose (18F)
metabolism using blood flow 11C raclopride
measures.
Volumetric MRI.
Functional MRI
Contrast CT scan.
Technetium HMPAO SPECT (99mTc).
T2 weighted MRI.

Your answer is incorrect.
Beta CIT and FP­CIT (DATscan) are SPECT ligands that bind to dopamine transporters found
on nerve endings. In Parkinson's disease where there is a loss of dopaminergic neurons,
there is decreased uptake of beta CIT, in contrast to drug­induced Parkinsonism, where the

1037
uptake is normal. 11C Raclopride is a PET ligand that binds to D2 dopaminergic receptors.
This can be used to determine the amount of dopamine bound to D2 receptors. If there is
excess dopamine bound to D2 receptors, there will be less raclopride uptake. Because blood
flow to the brain is tightly coupled with local brain metabolism and energy use, the 99mTc­
HMPAO is a SPECT tracer used to assess brain metabolism regionally, in an attempt to
diagnose causal pathology such as dementia.
The correct answer is: Differentiation of drug­induced parkinsonism from Parkinson's disease
in an elderly woman. – ß­CIT SPECT, A researcher is interested in comparing the degree of
dopamine release after cocaine use and amphetamine use using PET scan. – 11C raclopride,
Indirect measurement of cerebral metabolism using blood flow measures. – Technetium
HMPAO SPECT (99mTc).

Question 79 MoK Neurosciences EMI020
Not answered Neuropeptides
For each of the following descriptions choose ONE correct option from the above list.
Marked out of 3.00

Flag question
Bombesin
Gastrin
Produced in response to fat levels in Vasoactive Intestinal Peptide
adipose tissue Cholecystokinin
Somatostatin
Leptin
Oxytocin
Neuropeptide Y
Ghrelin
Neurotensin
Neuropeptide X
Produced by stomach and pancreas Ghrelin
Vasoactive Intestinal Peptide
following food intake Neuropeptide X
Oxytocin
Hypocretin
Neurotensin
Neuropeptide Y
Hypocretin
Somatostatin
Neuropeptide X
Leptin
Somatostatin
Found in hypothalamus and Bombesin
Bombesin
observed to be abnormal in some Gastrin
Cholecystokinin
subjects with obesity Cholecystokinin
Neuropeptide Y
Neurotensin
Hypocretin
Vasoactive Intestinal Peptide
Ghrelin
Your answer is incorrect. Leptin
Gastrin
Leptin is an adipose­derived hormone that circulates at levels proportional to body fat and
Oxytocin
controls food intake and energy expenditure by acting on the mediobasal hypothalamus.
Ghrelin stimulates hunger and is usually produced by P cells lining the fundus of the stomach
and epsilon cells in the pancreas. The levels increase before meals and decrease after
meals.
Leptin binds to neuropeptide Y neurons in the arcuate nucleus so that it decreases the activity
of neuropeptide Y. The main effect of increased NPY is increased food intake and decreased
physical activity. 
Kaplan & Sadock Synopsis of Psychiatry 10th edition; pg108.

1038
The correct answer is: Produced in response to fat levels in adipose tissue – Leptin, Produced
by stomach and pancreas following food intake – Ghrelin, Found in hypothalamus and
observed to be abnormal in some subjects with obesity – Neuropeptide Y

Question 80 MoK Neurosciences EMI021
Not answered EEG wave forms
Identify the wave forms present during different phases of sleep
Marked out of 3.00

Flag question
Single waves
Low voltage theta activity
Less than 50% delta waves
REM sleep
More than 50% delta waves
Sleep spindles and K­complexes
Mu waves
Saw tooth waves
Lambda waves
Saw tooth waves
More than 50% delta waves
Single waves
Stage 2 NREM Sleep
Mu waves
Lambda waves
Sleep spindles and K­complexes
Less than 50% delta waves
Low voltage theta activity
Saw tooth waves
Single waves
Less than 50% delta waves
Stage 4 NREM Sleep
Low voltage theta activity
More than 50% delta waves
Sleep spindles and K­complexes
Mu waves
Lambda waves

Your answer is incorrect.
REM sleep (25% sleep in adults) is characterized by a high level of brain activity and
physiological activity similar to those in wakefulness. EEG shows low­voltage, mixed­
frequency (theta and slow alpha) activity similar to the awake state. Sawtooth waves are seen.
The REM episodes increase in length as the night unfolds. 
NREM sleep: (75% sleep in adults). Most physiological functions are markedly lower than in
wakefulness (decreased muscle tone, respiration, temperature and heart rate). It is classified
as stages 1 to 4 with increasing amplitude and decreasing frequency of EEG activity. Stage 1
sleep (the drowsy period ­ 5%) ­ very light; when awoken from this stage one denies being
asleep. Low voltage theta activity with sharp waves. Stage 2 sleep (45% ­ most common)
shows the development of sleep spindles and K complexes. Stage 3 sleep (12% of sleep
time) is comprised of <50% delta waves. Stage 4 sleep (13% of sleep time) is comprised of
>50% delta waves. Physiological functions are at the lowest during stage 4. Stage 3 & 4
together constitute slow wave sleep (SWS). SWS dominates the initial part of the sleep.
The correct answer is: REM sleep – Saw tooth waves, Stage 2 NREM Sleep – Sleep spindles
and K­complexes, Stage 4 NREM Sleep – More than 50% delta waves

1039
Question 81 MoK Neurosciences EMI022
Not answered Basal Ganglia Disorders 
Match the disorders given below with most commonly reported changes in the basal ganglia;
Marked out of 3.00

Flag question
Copper deposits in the lenticular nucleus
Depigmentation of substantia nigra
A 10­year­old boy with multiple tics Increased caudate blood flow
and coprolalia Progressive calcium deposition in the basal ganglia
Acute bilateral anoxic damage to basal ganglia
Subthalamic nucleus damage
Striatal dopaminergic dysfunction
Degeneration of the corpus striatum
Subthalamic nucleus damage
Selective loss of GABA­ergic neurons
Copper deposits in the lenticular nucleus
A 67­year­old gentleman with Degeneration of the corpus striatum
tremors, rigidity and bradykinesia Acute bilateral anoxic damage to basal ganglia
Striatal dopaminergic dysfunction
Selective loss of GABA­ergic neurons
Increased caudate blood flow
Progressive calcium deposition in the basal ganglia
Depigmentation of substantia nigra
Depigmentation of substantia nigra
Acute bilateral anoxic damage to basal ganglia
A 46­year­old lady with recurrent
obsessional thoughts of Copper deposits in the lenticular nucleus
contamination and compulsive hand Degeneration of the corpus striatum
washing repetitively Progressive calcium deposition in the basal ganglia
Selective loss of GABA­ergic neurons
Striatal dopaminergic dysfunction
Subthalamic nucleus damage
Increased caudate blood flow
Your answer is incorrect.
Multiple tics with coprolalia in a young boy is suggestive of Tourette's syndrome, which is
believed to be caused by striatal dopaminergic dysfunction.
In Parkinson's disease, depigmentation of the substantia nigra is the common finding.
Subcortical Lewy bodies are seen. Striatal overactivity may lead to bradykinesia (excessive
motor­gating).
In OCD, both reduced and increased volumes of caudate nuclei are reported.Patients have
higher caudate blood flow. Increased caudate metabolism has been found to reduce after
effective treatment of the OCD (Ring & Serra­Mestres, 2002). 
 Ring HA and Serra­Mestres J. (2002), Neuropsychiatry of the basal ganglia. Retrieved from
http://jnnp.bmj.com/content/72/1/12.full
The correct answer is: A 10­year­old boy with multiple tics and coprolalia – Striatal
dopaminergic dysfunction, A 67­year­old gentleman with tremors, rigidity and bradykinesia –
Depigmentation of substantia nigra, A 46­year­old lady with recurrent obsessional thoughts of
contamination and compulsive hand washing repetitively – Increased caudate blood flow

Finish review

1040
 Home Mock Paper Practice Tests

Mock Paper A(2)

Started on Thursday, 25 June 2015, 5:57 AM
State Finished
Completed on Thursday, 25 June 2015, 5:57 AM
Time taken 16 secs
Grade 0.00 out of 100.00
Feedback You are on the right track. Could you spend some more time on your revision?

Question 1 MoK Adverse Effects 052
Not answered Nomifensine is an antidepressant that has noradrenaline, dopamine and also serotonin
reuptake inhibition properties. It has been withdrawn due to its propensity to cause which of
Marked out of 1.00
the following side effects?
Flag question
Select one:
Haemolytic anaemia
Agranulocytosis
Seizures

Thrombocytopenia
Hepatitis

Your answer is incorrect.
Nomifensine has noradrenaline, dopamine and serotonin reuptake inhibition properties. It
was withdrawn due to haemolytic anaemia in a small number of patients. It was a particularly
effective antidepressant for patients with epilepsy as, it did not alter the seizure threshold.
The correct answer is: Haemolytic anaemia

Question 2 MoK Adverse Effects 053
Not answered Which of the following medications is preferred option to treat a 75 year old man with a
relapse of psychotic depression and has been taking treatment for prostatic hypertrophy in
Marked out of 1.00
addition to high cholesterol?
Flag question
Select one:
Thioridazine
Chlorpromazine
Haloperidol

1041
Olanzapine
Clozapine

Your answer is incorrect.
High­potency neuroleptics, such as haloperidol have lower anticholinergic effects and less
likely to cause urinary retention compared to low­potency medications such as
chlorpromazine in elderly patients with prostatic hypertrophy. Clozapine is not recommended
because of its powerful anticholinergic effects, its tendency to cause hypotension, and the risk
for agranulocytosis. Thioridazine can cause fatal arrhythmias by prolonging the QT interval, in
addition to the high anticholinergic profile. Finally, olanzapine may not be appropriate due to
cholesterol related issues.
The correct answer is: Haloperidol

Question 3 MoK Adverse Effects 054
Not answered A bipolar disorder patient on lithium treatment developed signs of severe lithium toxicity. What
is the best immediate treatment option?
Marked out of 1.00

Flag question Select one:
Peritoneal dialysis
Forced alkaline diuresis
Hemodialyisis
Volume repletion
Arteriovenous hemodynamic filtration

Your answer is incorrect.
Moderate and severe lithium toxicity require aggressive approaches. Initial general
antipoisoning measures like gastric lavage may be helpful, but ultimate success depends
upon elimination of lithium from the body. Hemodialysis will rapidly eliminate lithium from the
body.
The correct answer is: Hemodialyisis

Question 4 MoK Adverse Effects 055
Not answered Tardive dyskinesia induced by psychotropics is attributed to which of the following
mechanisms?
Marked out of 1.00

Flag question Select one:
Low D­2 receptor occupancy
High D­2 receptor occupancy
High D­2 receptor sensitivity
Low D­1 receptor sensitivity

1042
Low D­2 receptor sensitivity

Your answer is incorrect.
Theories explaining tardive dyskinesia include dopaminergic/cholinergic imbalance,
upregulation/supersensitivity of postsynaptic D2 receptors in the basal ganglia following
chronic blockade and GABA hypofunction leading to enhanced DA transmission.

The correct answer is: High D­2 receptor sensitivity

Question 5 MoK Adverse Effects 056
Not answered A 67­year­old gentleman has been on treatment for depression. He went to the local beach for
sunbathing on a summer day afternoon and presented to A and E complaining of nausea,
Marked out of 1.00
dizziness and leg cramps.
Flag question
Select one:
Hypoglycemia
Hyponatraemia
Hyperkalemia
Hypokalemia
Hypernatraemia

Your answer is incorrect.
Most antidepressants are strongly linked with hyponatraemia. The clinical features of
hyponatraemia include nausea, dizziness, lethargy, confusion, cramps and seizures. Other
risk factors, in this case, includes old age, warm weather and drug treatments such as
diuretics.
The correct answer is: Hyponatraemia

Question 6 MoK Adverse Effects 057
Not answered A 65 year old woman who is being treated for depression comes to the Medical Emergency
Unit with lethargy, cramps, poor memory and weight loss on a hot afternoon. She is taking
Marked out of 1.00
some unknown psychotropic medication for a long time. Also, more recently, she started
Flag question taking 'water tablets' for swelling in her legs. Which of the following is a likely cause of her
symptoms? 

Select one:
Hypokalemia
Hypernatremia
Hyponatremia
Neuroleptic malignant syndrome
Serotonin syndrome

1043
Your answer is incorrect.
The story is suggestive of hyponatremia induced by diuretics and antidepressants.
The correct answer is: Hyponatremia

Question 7 MoK Adverse Effects 060
Not answered What is the main risk involved in treating a patient with combination of clozapine and
carbamazepine?
Marked out of 1.00

Flag question Select one:
Neuroleptic malignant syndrome
Diabetes mellitus
Seizures
Leucopenia
Weight gain

Your answer is incorrect.
Clozapine can cause neutropenia. Carbamazepine causes a chronic low white blood cell
count.
The correct answer is: Leucopenia

Question 8 MoK Adverse Effects 061
Not answered Whilst having ECT, which of the following medications may require lower than normal current
to be selected?
Marked out of 1.00

Flag question Select one:
Sodium valproate
Nitrazepam
Lithium
Zolpidem
Lamotrigine

Your answer is incorrect.
Lithium could reduce seizure threshold even on normal therapeutic doses. So patients on
Lithium, while having ECT may require lower than normal current.
The correct answer is: Lithium

Question 9 MoK Adverse Effects 062
Not answered

1044
Marked out of 1.00 Which of the following agents is likely to have caused a newborn to have a defect of the
tricuspid valve?
Flag question

Select one:
Valproate
Paroxetine
Lithium

Zopiclone
Naltrexone

Your answer is incorrect.
Valproate induced neural tube effects are predominantly lumbosacral, but anencephaly is
also seen occasionally. Tricuspid defects are often seen in lithium­treated patients, with
extreme cases having full­blown Ebstein's anomaly.
The correct answer is: Lithium

Question 10 MoK Adverse Effects 064
Not answered Which one among the following drug usage has been associated with hyperparathyroidism?

Marked out of 1.00
Select one:
Flag question
Sodium Valproate
Gabapentin
Carbamazepine
Lithium
Lamotrigine

Your answer is incorrect.
Hypocalcaemia and hyperparathyroidism are uncommon side effects associated with lithium
therapy. The common medical symptoms include nausea, polydipsia and constipation.
The correct answer is: Lithium

Question 11 MoK Adverse Effects 067
Not answered A depressed patient experiences constipation and dry mouth whilst on amitriptyline. This is
mainly due to
Marked out of 1.00

Flag question Select one:
M1 receptor blockade
Alpha­1 receptor blockade

Alpha 1 agonism

1045
M 2 receptor blockade
M3 receptor blockade

Your answer is incorrect.
Amitriptyline and other tricyclic antidepressants can induce anticholinergic side effects such
as constipation, dry mouth and urinary retention. Muscarinic receptor blockade leads to
anticholinergic side effects, especially through M3 receptor blockade.
The correct answer is: M3 receptor blockade

Question 12 MoK Adverse Effects 069
Not answered Which of the following antidepressants is associated with a significant risk of agranulocytosis
compared to the others listed?
Marked out of 1.00

Flag question Select one:
Mirtazapine
Bupropion
Escitalopram
Fluoxetine
Venlafaxine

Your answer is incorrect.
All drugs ending with a suffix ­zapine can cause agranulocytosis (mirtazapine, olanzapine,
clozapine, carbamazepine, etc.). In addition, mianserin can also cause this effect
The correct answer is: Mirtazapine

Question 13 MoK Adverse Effects 070
Not answered Which of the following antidepressants has a recognised side effect of elevated cholesterol?

Marked out of 1.00
Select one:
Flag question
fluoxetine
imipramine
venlafaxine
paroxetine
mirtazapine

Your answer is incorrect.
Mirtazapine ­ similar to olanzapine ­ is associated with blood dyscrasias and hyperlipidemia.

1046
The correct answer is: mirtazapine

Question 14 MoK Adverse Effects 071
Not answered Which of the following is NOT a dystonia?
Marked out of 1.00
Select one:
Flag question
Writer's cramp
Oculogyric crisis
Motor tic
Torticollis
Blepharospasm

Your answer is incorrect.
Tics are not classified as dystonia
The correct answer is: Motor tic

Question 15 MoK Adverse Effects 072
Not answered Which one among the following is not a feature of SSRI discontinuation syndrome
Marked out of 1.00
Select one:
Flag question
Increased dreams
Dizziness
Insomnia
Myoclonic Jerks
Electric shock like sensations

Your answer is incorrect.
SSRI discontinuation syndrome: Sensory abnormalities include electric shock­like sensations,
symptoms of a generalised disequilibrium including dizziness, insomnia, increased dreams,
and general somatic symptoms such as headache, lethargy, gastrointestinal symptoms and
mood changes such as anxiety, agitation and depression.
The correct answer is: Myoclonic Jerks

Question 16 MoK Pharmacodynamics 030
Not answered Which of the following agents exert an allosteric action on the nicotinic receptors?
Marked out of 1.00
Select one:
Flag question
Memantine

1047
Gallantamine
Rivastigmine
Tacrine
Donepezil

Your answer is incorrect.
The two most common approaches to correcting cholinergic deficits in dementia are to
increase the synaptic availability of acetylcholine (ACh) by inhibiting acetylcholinesterase
(AChE), or to mimic the effects of ACh by acting directly on nicotinic receptors. AChE inhibitors
produce only short­term symptomatic improvement while long­term use of nicotinic agonists
desensitizes nicotinic receptors limiting the duration of efficacy. Allosteric modulation of
nAChR is a novel approach, which circumvents the development of tolerance. Allosteric
modulators bind to a site on nAChR that is different to the binding site of the natural agonist,
ACh. This allosteric interaction amplifies the actions of ACh at post­ and presynaptic nAChR.
In particular, presynaptic nAChR are capable of modulating the release of ACh and other
neurotransmitters, such as glutamate, serotonin and GABA, which may contribute to
symptoms of the illness. Allosteric modulation of nAChR could, therefore, produce significant
therapeutic benefit in AD. As well as modulating nAChR, galantamine inhibits AChE. The
extent to which the clinical benefits of galantamine are attributable specifically to its nicotinic
effects is uncertain and requires further investigation (Excerpt retrieved from Maelick A.,
Dement Geriatr Cogn Disord 2000 Sep;11 Suppl 1:11­80).
The correct answer is: Gallantamine

Question 17 MoK Pharmacodynamics 031
Not answered Antipsychotic drug induced akathisia is due to;
Marked out of 1.00
Select one:
Flag question
Dopamine D2 receptor activation in the hippocambus
Dopamine receptor supersensitivity
Low dopamine D2 receptor occupancy in striatum
High dopamine D2 receptor occupancy in striatum
Serotonin 5­HT2 receptor blockade

Your answer is incorrect.
Antipsychotic drug­induced akathisia is thought to be due to high dopamine D2 receptor
occupancy in the striatum. Higher doses of antipsychotic medications are correlated with
higher D2 receptor occupancy and worse subjective experience of akathisia. Most
extrapyramidal symptoms are related to D2 receptor occupancy and, in particular, akathisia
has an important subjective component.
The correct answer is: High dopamine D2 receptor occupancy in striatum

Question 18 MoK Pharmacodynamics 032

1048
Not answered A patient with depression is being treated by antidepressant medications. Which of the
following changes may be observed?
Marked out of 1.00

Flag question
Select one:
Increasing corticotropin­releasing hormone mRNA
Increasing cortisol
Enhanced activity of HPA axis

Increasing adrenocorticotropic hormone
Increasing expression of corticosteroid receptors in brain

Your answer is incorrect.
Antidepressants reduce HPA axis activity by increasing negative feedback from steroid
hormone.. This is made possible via increased corticosteroid receptors.
The correct answer is: Increasing expression of corticosteroid receptors in brain

Question 19 MoK Pharmacodynamics 033
Not answered Which of the following is the most likely mode of action of disulfiram?

Marked out of 1.00
Select one:
Flag question
Inhibition of sulphuric acid
Inhibition of alcohol decarboxylase

Inhibition of alcohol dehyrogenase
Inhibition of acetic acid
Inhibition of aldehyde dehyrogenase

Your answer is incorrect.
Disulfiram causes an accumulation of acetaldehyde by blocking oxidation of alcohol after
drinking. This action is mediated via the inhibition of hepatic aldehyde dehydrogenase
enzyme. Upon alcohol consumption while taking antabuse, the patient experiences colicky
abdominal pain, flushing, anxiety, dizziness, tachycardia, vomiting and headache. Symptoms
are generally acute in onset, starting 5­15 minutes after drinking alcohol and last for several
hours.
The correct answer is: Inhibition of aldehyde dehyrogenase

Question 20 MoK Pharmacodynamics 034
Not answered Which symptom is acknowledged as part of cocaine withdrawal?

Marked out of 1.00
Select one:
Flag question
Intense craving
Increased energy

1049
Euphoria
Diminished need for sleep
Formication

Your answer is incorrect.
Cocaine Withdrawal: Effects of cocaine are short­lived ­ rapid cessation occurs due to rapid
metabolism ­ this can lead to repeated use. Dysphoria (crash), anhedonia, anxiety, irritability
hypersomnolence, and sometimes agitation usually end within 18 hours though in heavy
dependent users this can last up to a week, peaking in 3 days. The characteristic feature is
intense cravings for cocaine with a striking lack of much physical withdrawal symptoms.
The correct answer is: Intense craving

Question 21 MoK Pharmacodynamics 035
Not answered With respect to the neurotransmitter glutamate, which of the following is true?

Marked out of 1.00
Select one:
Flag question
It is an inhibitory neurotransmitter
All receptors are ionotropic
It acts via chloride channels
It acts via NMDA receptors
It is synthesized by the enzyme glutamate decarboxylase

Your answer is incorrect.
Glutamate decarboxylase (GAD) is an enzyme that catalyzes the decarboxylation of glutamate
to GABA ­ thus GABA, not Glutamate, is synthesized by GAD.
The correct answer is: It acts via NMDA receptors

Question 22 MoK Pharmacodynamics 036
Not answered Xanomeline has been advocated as a novel approach to treat schizophrenia. It acts via
Marked out of 1.00
Select one:
Flag question
Muscarinic antagonism
D4 partial agonism
5HT7 blocakde
D4 agonism
M1/M4 agonism

Your answer is incorrect.

1050
Xanomeline is undergoing Phase 3 trials as a treatment option for schizophrenia. It acts via
M1/M4 agonism. http://ajp.psychiatryonline.org/cgi/content/full/165/8/1033
The correct answer is: M1/M4 agonism

Question 23 MoK Pharmacodynamics 037
Not answered Tom is on clozapine for treatment resistant psychosis. He is embarrassed about
hypersalivation and wet­pillows every morning. The most likely mechanism of this adverse
Marked out of 1.00
event is
Flag question
Select one:
M4 receptor agonism and alpha 2 receptor antagonism
M4 receptor agonism and alpha 1 receptor antagonism
M4 receptor antagonism and alpha 2 receptor antagonism
M4 receptor antagonism and alpha 2 receptor antagonism
M4 receptor antagonism and alpha 1 receptor antagonism

Your answer is incorrect.
Clozapine increases salivation through its muscarinic M4 receptor activation and/or blockade
of alpha2­adrenoceptors, or by causing a distortion in swallowing reflex (possibly via M3). The
M1 and M4 receptor subtypes predominate in various secretory glands. Clozapine is
antagonistic at M1, M2, M3, and M5 receptor subtypes, but in vitro findings indicate that it is a
full agonist at the M4 receptor. As a result, clozapine's stimulation of the M4 receptors in
salivary glands leads to an increase in secretions. Agonism at the M4 receptor seems unique
to clozapine and does not occur with other atypical antipsychotics. Treatment options include
chewing gum, reducing the dosage of clozapine, or prescribing pharmacologic agents such
as anticholinergics or alpha2­adrenoceptor agonists. (Ref; Rogers et al.,Clozapine­induced
hypersalivation, Pharmacotherapy. 2000; 20(9):
http://www.ncbi.nlm.nih.gov/pubmed/10852096)
The correct answer is: M4 receptor agonism and alpha 2 receptor antagonism

Question 24 MoK Pharmacodynamics 039
Not answered Which antidepressant is a presynaptic alpha­2 antagonist and now classified as NaSSA?
Marked out of 1.00
Select one:
Flag question
Bupropion
Venlafaxine
Mirtazapine
Reboxetine
Buspirone

Your answer is incorrect.

1051
Mirtazapine is a presynaptic alpha­2 antagonist that increases central neurotransmission by
noradrenaline and serotonin. It is classified as Noradrenergic and specific serotonergic
antagonist­ NaSSA.
The correct answer is: Mirtazapine

Question 25 MoK Pharmacodynamics 040
Not answered Which one among the following has very limited potential for causing a hypertensive crisis?

Marked out of 1.00
Select one:
Flag question
Cocaine
Phenelzine
Isocarboxazid
Tranylcypromine
Moclobemide

Your answer is incorrect.
Tyramine interactions are rare and mild with moclobemide but possible if high doses (more
than 600 mg/day) used or if large quantities of tyramine ingested
The correct answer is: Moclobemide

Question 26 MoK Pharmacokinetics 033
Not answered Which of the following is acceptable to be taken along with MAO inhibitors?

Marked out of 1.00
Select one:
Flag question
Mature cheese
Broad bean pods
Red wine
Pickled Herring
Grapefruit juice

Your answer is incorrect.
Except grapefruit juice, the others on the list are rich in tyramine and so can produce cheese
reaction.
The correct answer is: Grapefruit juice

Question 27 MoK Pharmacokinetics 039
Not answered Which routes of administration leads to nicotine reaching the brain fast?

Marked out of 1.00
Select one:

1052
Flag question
Sublingual
Inhalational
Intravenous
Cutaneous
Oral

Your answer is incorrect.
Drugs administered via the inhalational route reaches the brain quickly.
The correct answer is: Inhalational

Question 28 MoK Pharmacokinetics 040
Not answered Which one of the following statements about reboxetine is correct?

Marked out of 1.00
Select one:
Flag question
It is highly cardiotoxic.
Reboxetine is a hepatic enzyme inhibitor

Reboxetine is a hepatic enzyme inducer
It does not affect hepatic cytochrome P450 enzymes.
It is completely excreted via kidneys

Your answer is incorrect.
Reboxetine is rapidly absorbed after oral doses. It is mainly metabolised in the liver and has
an elimination half­life of about 13 hours. In clinical trials, reboxetine has been shown to be as
effective as the SSRIs in the treatment of depression but, unlike the SSRIs, reboxetine does
not inhibit any of the cytochrome P450 enzymes in the liver.
The correct answer is: It does not affect hepatic cytochrome P450 enzymes.

Question 29 MoK Pharmacokinetics 041
Not answered An orally administered medication requires five half­life periods to reach a steady state
concentration. Which of the following statements is most likely to be correct with respect to this
Marked out of 1.00
drug?
Flag question
Select one:
Doses up to five times the normal are non­toxic
It follows first order kinetics
A loading dose must be provided for adequate therapeutic activity
Its rate of metabolism is independent of its concentration
This medication must be administered only once a day

1053
Your answer is incorrect.
Under conditions of first­order kinetics, given that a drug is rapidly and evenly distributed
throughout the body and no loading dose is given, at least 5 half­lives are required to achieve
a steady state concentration. Drugs following zero order kinetics show a concentration­
independent elimination.
The correct answer is: It follows first order kinetics

Question 30 MoK Pharmacokinetics 042
Not answered Which of the following is true with regard to sodium valproate?

Marked out of 1.00
Select one:
Flag question
Intravenous preparation is available for acute mania
It is excreted unchanged via kidney
It has no significant interactions with other mood stabilisers
Should not be combined with antidepressants
It increases carbamazepine levels

Your answer is incorrect.
Valproate appears to inhibit the glucuronidation of carbamazepine, leading to an increase in
its peak levels. This may be associated with the clinically significant dose related effects of
carbamazepine such as ataxia. Interactions are also seen with other psychotropics (especially
lamotrigine). A study involving the administration of 100 to 300 mg/ day of chlorpromazine to
schizophrenic patients already receiving valproate (200 mg BID) revealed a 15% increase in
trough plasma levels of valproate.
The correct answer is: It increases carbamazepine levels

Question 31 MoK Pharmacokinetics 043
Not answered Regarding pharmacokinetics and properties of flumazenil, which of the following is true?

Marked out of 1.00
Select one:
Flag question
It is an inverse agonist at GABA receptor
It has an elimination half­life of 4 hours
It reduces panic attacks instantaneously
It is an epileptogenic drug
It is a partial agonist at GABA receptor

Your answer is incorrect.

1054
Flumazenil is a competitive antagonist of benzodiazepines at benzodiazepine sites of GABA­
A receptors. Hence it is an epileptogenic drug. It is an imidazobenzodiazepine derivative with
weak partial agonist action in some animal models but has no agonist activity in humans. By
blocking GABA A neurotransmission, it can induce panic attacks. This has some diagnostic
value too. It has a very short elimination t O and so must be given as a continuous infusion. It
is 40 to 50 percent protein bound with significant tissue uptake. It is primarily metabolized by
the liver with t1/2 of 40 ­ 80 minutes.
The correct answer is: It is an epileptogenic drug

Question 32 MoK Pharmacokinetics 044
Not answered Which of the following is hydroxylated as one of the steps in dopamine synthesis?

Marked out of 1.00
Select one:
Flag question
Tryptophan hydroxylase
L­tyrosine
L­dopa
L­tryptophan
Phenylalanine hydroxylase

Your answer is incorrect.
Tyrosine is converted to dopamine by the enzymes tyrosine hydroxylase (TH) and L­amino
acid decarboxylase (AADC) also called dihydroxyphenylalanine (DOPA) decarboxylase
(DDC).
The correct answer is: L­tyrosine

Question 33 MoK Pharmacokinetics 045
Not answered A 63­year­old man has a long­standing history of mood disorder. He is also taking Ramipril for
hypertension. Which of the following drugs is most likely to cause a possible interaction?
Marked out of 1.00

Flag question Select one:
Venlafaxine
Amitriptyline
Fluoxetine
Lithium
Bupropion

Your answer is incorrect.
Ramipril is an Angiotensin converting enzyme inhibitors (ACEI). ACEI can reduce thirst, which
can lead to mild dehydration, and increase renal sodium loss, leading to increased sodium
reabsorption by the kidney causing an increase in lithium plasma levels and, therefore,

1055
precipitate lithium toxicity. ACE inhibitors can also precipitate renal failure. Therefore, if ACEI
are co­prescribed with lithium, more frequent monitoring of e­GFR and plasma lithium is
required. (Ref: The Maudsley prescribing guidelines­ 11th edition­Pg. 164)
The correct answer is: Lithium

Question 34 MoK Pharmacokinetics 046
Not answered Which of the following is correct with regard to the pharmacology of lithium?

Marked out of 1.00
Select one:
Flag question
Lithium is unsafe in the elderly
Lithium is contraindicated in patients with sick sinus syndrome
Obesity is associated with lower rates of lithium clearance
It is not excreted in breast milk
Lithium cardiotoxicity is more prevalent in people on a high­salt diet

Your answer is incorrect.
Lithium is safe and effective in the elderly; it is excreted in the breast milk and must be
carefully considered in lactating mothers.BMI does not affect lithium clearance as long as
renal function is normal; Lithium can cause a shift in intracellular potassium and is
contraindicated in sick sinus syndrome. The cardiotoxicity is more common in those on low
salt diet and those with renal insufficiency.
The correct answer is: Lithium is contraindicated in patients with sick sinus syndrome

Question 35 MoK Applied Neuroscience 041
Not answered What part of the Weschler's Adult Intelligence Scale is most likely to be resistant to brain
damage?
Marked out of 1.00

Flag question Select one:
Digit Symbol
Digit Span
Picture completion
Object assembly
Matrix assembly

Your answer is incorrect.
Tests on WAIS are either 'hold' or 'no hold'. The so called 'hold tests' in WAIS are thought to
reflect the use of old knowledge and are relatively resistant to the effects of brain damage and
ageing. These include vocabulary, information, object assembly, and picture completion. 'No
hold' tests require speed of response, working memory, or the creation of new relations
between unrelated items. These are more likely to show an early decline with ageing and

1056
cognitive impairment. These include digit symbol, digit span, similarities, and block design.
This explains why WAIS is not the best test to measure premorbid intelligence. A deterioration
index is calculated using the composite scores of HOLD and NON­HOLD subtests.
The correct answer is: Picture completion

Question 36 MoK Applied Neuroscience 048
Not answered A 43­year­old man with pure word deafness is likely to show which of the following?

Marked out of 1.00
Select one:
Flag question
Reading is unimpaired
Comprehension of speech is intact
Writing is impaired
Meaning of words is intact when asked
All of the above

Your answer is incorrect.
Pure word deafness: Patient can speak read & write fluently, but comprehension is impaired
only for spoken language. Bilateral (or left sided with disrupted connections to non­dominant
circuit) damage to the superior temporal pole is suspected. In Wernicke's aphasia, the
comprehension is impaired for both spoken and written language.
The correct answer is: Reading is unimpaired

Question 37 MoK Applied Neuroscience 050
Not answered Which one of the following tests are not correctly linked with the main function tested?

Marked out of 1.00
Select one:
Flag question
Benton test­ Visual memory.
Paired associates test­verbal memory
Stroop test­executive functioning
Halstead­Reitan test­ Intelligence
Rey­Osterreith test­ To detect prefrontal lobe pathology

Your answer is incorrect.
Rey­Osterreith test measures the recall of learned material. It is a test of visual memory.
Paired associate test tests for verbal memory using paired and unpaired words. Wisconsin
card sorting test is used to detect prefrontal lobe pathology.
The correct answer is: Rey­Osterreith test­ To detect prefrontal lobe pathology

Question 38 MoK Applied Neuroscience 051

1057
Not answered There is a time gradient notable when patients develop retrograde amnesia, so that recent
memories are more likely to be lost than the more remote memories. This is called
Marked out of 1.00

Flag question
Select one:
Interference theory
Ribot's Law
Law of effect
Yerkes Dodson law
Decay theory

Your answer is incorrect.
Ribot in 1881 wrote a book (Les Maladies de la Memoire (The Diseases of Memory)) in which
he claimed on the basis of clinical observations, that brain injury affects premorbid memories
in the reverse order of their formation such that newly formed memories (i.e., those formed just
prior to brain injury) are impaired to a greater extent than older memories. Today, Ribot's law
is referred to as temporally graded retrograde amnesia, and much evidence involving both
human and animal subjects supports its validity (Ref: Wixted, J. T. (2004). On Common
Ground: Jost's (1897) law of forgetting and Ribot's (1881) law of retrograde amnesia.
Psychological review, 111(4), 864.)
The correct answer is: Ribot's Law

Question 39 MoK Applied Neuroscience 052
Not answered The most obviously affected neuropsychological measure in those with post concussion
syndrome is
Marked out of 1.00

Flag question Select one:
Speed of processing
Procedural memory
Block designing
Verbal fluency
Picture matching

Your answer is incorrect.
Processing speed is the most obviously affected neuropsychological measure in those with
post concussion syndrome.
The correct answer is: Speed of processing

Question 40 MoK Applied Neuroscience 053
Not answered A patient has with UMN lesion of cranial nerves 9, 10, 11 and 12. Which of the following
dysfunction is likely to be seen?
Marked out of 1.00

1058
Flag question Select one:
Facial droop
Tongue fasciculations
Swallowing problems
Chewing problems
Loss of touch sensation on left side of face

Your answer is incorrect.
Swallowing ­ IX and X; Chewing and facial sensation: ­ V. Facial muscle tone­ VII.
Fasciculations are seen in LMN lesion
The correct answer is: Swallowing problems

Question 41 MoK Applied Neuroscience 054
Not answered You want to measure the impairment of everyday memory functioning to facilitate
rehabilitation of individuals with acquired brain damage. The most suitable test is
Marked out of 1.00

Flag question Select one:
Wechsler memory scale
Halstead­Reitan Neuropsychological Battery
The Rivermead behavioral memory test
Ravens progressive matrices
Thematic apperception test

Your answer is incorrect.
The Rivermead Behavioral Memory Test detects impairment of everyday memory functioning
and also monitors changes following the treatment for memory difficulties. The aim of the test
is to provide task analogues of everyday memory problems that appear challenging for
patients with memory impairment. It can be used in adults with memory problems; individuals
with traumatic or acquired brain injury, dementia, or MS.
The correct answer is: The Rivermead behavioral memory test

Question 42 MoK Applied Neuroscience 055
Not answered A 45­year­old gentleman has history of alcohol dependence. He has no memeory problems
or neurological signs but appears disorganised in day to day tasks. If a neuropsychologist
Marked out of 1.00
tests him he is most likely to show impaired performance in which of the following tests?
Flag question
Select one:
Orientation
Clock drawing test

1059
Trail making tests
Digit span
Picture completion test

Your answer is incorrect.
Trial making tests is a test of frontal lobe function. Patients with alcohol dependence have
decrements in executive function (frontal lobe), and it could be used.
The correct answer is: Trail making tests

Question 43 MoK Applied Neuroscience 056
Not answered A 76­year­old man was brought to Emergency Unit by his wife. He had breakfast that morning
and appeared unable to remember that he and his wife have planned a day out. He
Marked out of 1.00
repeatedly enquired where she was taking him. She reports that he appeared bewildered, but
Flag question was alert and had no sensory and motor deficits throughout. He returned to normal after 10
hours. What is the most likely diagnosis?

Select one:
Creutzfeldt­Jakob disease
Transient global amnesia
Alzheimer's disease
Normal­pressure hydrocephalus
Parkinson's disease

Your answer is incorrect.
Transient global amnesia (TGA) refers to an episode of complete and reversible anterograde
and retrograde memory loss lasting up to 24 h. Patients have a persistent loss of memory for
the time of the attack. During the episode, patients often appear bewildered and ask repeated
questions.
The correct answer is: Transient global amnesia

Question 44 MoK Molecular Genetics 048
Not answered Which of the following cannot be termed as a reliable endophenotype for schizophrenia?

Marked out of 1.00
Select one:
Flag question
Glial cell changes
Working memory defects
Prepulse inhibition
Smooth pursuit defects
Negative symptoms

1060
Your answer is incorrect.
The suggested criteria that define an endophenotype include: the marker is associated with
the illness in the population, heritable, state­independent, co­segregates with the illness
within families and is present at a higher rate in unaffected family members of the proband
compared with the general population. Clinical symptom clusters do not fit these criteria.
The correct answer is: Negative symptoms

Question 45 MoK Molecular Genetics 050
Not answered Which one among the following is the least likely statement about Huntington's disease?

Marked out of 1.00
Select one:
Flag question
Huntington genes code for production of protein called huntingtin
CAG repeat size and age are inversely correlated
Late onset Huntington's disease is usually inherited from mother

Paternally derived gene may be more unstable than maternally derived
gene
Normal Huntingtin gene has 35­45 CAG repeats

Your answer is incorrect.
Huntington's disease is a fully penetrant autosomal disorder. The affected gene, at 4p16,
codes for huntingtin. In Huntington's disease, there are increased CAG repeats. Normal
repeat lengths range from about 7 to 28 triplets. Individuals with 29­35 triplets will not develop
HD but may pass an expanded allele to an offspring, while individuals with 40 or more triplets
will develop HD. (Ref: New Oxford Textbook­ 372)
The correct answer is: Normal Huntingtin gene has 35­45 CAG repeats

Question 46 MoK Molecular Genetics 051
Not answered Where does transcription take place in human cells?

Marked out of 1.00
Select one:
Flag question
Rough endoplasmic reticulum
Smooth endoplasmic reticulum

Cytoplasm
Golgi cmplex
Nucleus

Your answer is incorrect.
Active transcription units are clustered in the nucleus, in discrete sites called transcription
factories or euchromatin.

1061
The correct answer is: Nucleus

Question 47 MoK Molecular Genetics 052
Not answered Probandwise concordance rate is calculated as

Marked out of 1.00
Select one:
Flag question
Number of total pairs divided the affected population
Number of twin pair divided by the total number of sib pairs
Number of affected twins divided by total number of co­twins
Number of twin pairs divided by the total relatives
Number of twin pair divided by the total population

Your answer is incorrect.
A pairwise concordance rate is estimated as the number of twin pairs who both have the
disorder divided by the total number of pairs. However, where there has been systematic
ascertainment, one can report a probandwise concordance rate that is calculated as the
number of affected twins divided by the total number of cotwins.
The correct answer is: Number of affected twins divided by total number of co­twins

Question 48 MoK Molecular Genetics 053
Not answered Which of the following gene, when knocked out, produces mice with phenotype resembling
narcolepsy?
Marked out of 1.00

Flag question Select one:
Neuropeptide Y
Vasopressin
Orexin
Substance P
Hydrocortisone

Your answer is incorrect.
Hypocretin­1 and ­2 (also called orexin­A and ­B) are newly discovered neuropeptides
processed from a common precursor, preprohypocretin. Hypocretin­containing cells are
located exclusively in the lateral hypothalamus, with widespread projections to the entire
neuraxis.The discovery that hypocretins are involved in narcolepsy, a disorder associated
with excessive daytime sleepiness, cataplexy and unusually rapid transitions to rapid­eye­
movement sleep, opens a new field of investigation in the area of sleep control physiology
(Hypocretin/orexin, sleep and narcolepsy, http://www.ncbi.nlm.nih.gov/pubmed/11340621).
The correct answer is: Orexin

1062
Question 49
MoK Molecular Genetics 054
The technique used to assemble correct DNA fragments from a gene library into a continuous
Not answered
overlapping stretch, which will contain genes of interests is;
Marked out of 1.00

Flag question Select one:
Physical mapping
Polymerase chain reaction
Association studies
Mutagenesis
Linkage analysis

Your answer is incorrect.
Physical maps of DNA give the physical, DNA­base­pair distances from one landmark to
another. By using the overlapping DNA sequences at the ends of the genome pieces to keep
track of where the pieces came from, physical mapping is accomplished.
The correct answer is: Physical mapping

Question 50 MoK Molecular Genetics 056
Not answered Polyploidy refers to which of the following phenomenon?

Marked out of 1.00
Select one:
Flag question
Fusion of male and female gametes producing 2n chromosome numbers in
an embryo
An inversion mutation that includes centromere of chromosomes
Clubbing together of mitochondria and other cell organelles during
interphase

Presence of one or more extra sets of chromosomes
Reduplicated nuclear material before cell division

Your answer is incorrect.
Polyploidy refers to the presence of extra sets of chromosomes. Triploidy refers to 3n
numbers; tetraploidy refers to 4n numbers. Fusion of male and female gametes producing 2n
chromosome numbers in an embryo is the normal diploid state seen in human cells.
The correct answer is: Presence of one or more extra sets of chromosomes

Question 51 MoK Molecular Genetics 057
Not answered A 48­year­old man has dementia. Which gene could be responsible for this condition?

Marked out of 1.00
Select one:
Flag question
COMT

1063
Progranulin
Presenilin

PARK­2
Dysbindin

Your answer is incorrect.
The commonest cause of early onset dementia (<65 years) is Alzheimer's dementia. It is
caused by a mutation in one of at least three genes: presenilin 1, presenilin 2 and amyloid
precursor protein.
The correct answer is: Presenilin

Question 52 MoK Molecular Genetics 058
Not answered Recombination occurs during which phase of meiosis?

Marked out of 1.00
Select one:
Flag question
Prophase 1
Anaphase 1

Interphase
Telophase 1
Metaphase 1

Your answer is incorrect.
Chromosomal crossover is an exchange of genetic material between homologous
chromosomes. It is one of the final phases of genetic recombination, which occurs during
prophase 1 of meiosis in a process called as synapsis. Synapsis is not completed until near
the end of prophase I. Crossover usually occurs when matching regions on matching
chromosomes break and then reconnect to the other chromosome.
The correct answer is: Prophase 1

Question 53 MoK Molecular Genetics 059
Not answered Ribosome is the site of

Marked out of 1.00
Select one:
Flag question
Posttranslational modification of protein

DNA storage
Protein synthesis
Synthesis of DNA
Energy production via respiratory cycle

1064
Your answer is incorrect.
Translation is carried out by more than one ribosome simultaneously.
The correct answer is: Protein synthesis

Question 54 MoK Molecular Genetics 060
Not answered A 53­year­old gentleman presents to the memory clinic with history of progressive short­term
memory loss, which was gradual in onset. His MMSE score was 17/30. The gene, which is
Marked out of 1.00
most likely involved in this condition, is
Flag question
Select one:
ApoE4
Neuronal sortilin related receptor SORL1
PSEN­1
Dysbindin
DISC­1

Your answer is incorrect.
The diagnosis is Alzheimer's disease (early­onset). AD is caused by mutations in the
Presenilin­1 (aka PSEN) gene. Mutations in the PSEN1 gene are the most common cause of
early­onset AD, accounting for up to 70% of cases
The correct answer is: PSEN­1

Question 55 MoK Neuroanatomy 017
Not answered Which of the following brain regions is supplied by the anterior cerebral artery?

Marked out of 1.00
Select one:
Flag question
cuneus
Inferior temporal gyrus
Lingual gyrus
Genu of corpus callosum
planum temporale

Your answer is incorrect.
MCA supplies blood to the cortical areas involved in speech, swallowing and language,
including the lateral motor strip, lateral sensory strip, Broca's area, Wernicke's area, Heschl's
gyrus, planum temporale and the angular gyrus. In addition, it provides most of the blood
supply to the corpus striatum. PCA supplies the cuneus, inferior temporal gyrus and lingual
gyrus. Genu and 3/4ths of corpus callosum are supplied by ACA.
The correct answer is: Genu of corpus callosum

1065
Question 56 MoK Neuroanatomy 021
Not answered Which region of cerebral blood flow correlates with hallucinations and delusions?

Marked out of 1.00
Select one:
Flag question
Right medial temporal cortex
Left medial temporal cortex
Caudate nucleus

Left prefrontal cortex
Right prefrontal cortex

Your answer is incorrect.
Patients with schizophrenia who experience frequent auditory hallucinations show reduced
grey matter density and neuronal activation. Similarly, f­MRI images have shown reduced
activation in the left temporal lobe and anterior cingulate cortex. The brain structural and
functional abnormalities seem to converge in the superior gyrus of the left temporal lobe.
The correct answer is: Left medial temporal cortex

Question 57 MoK Neuroanatomy 022
Not answered The role of ependymal cells is

Marked out of 1.00
Select one:
Flag question
Lining the walls of the ventricular systems
Nutrition of neurons

Produce myelin sheaths
Clear neuronal debris
Maintains the blood brain barrier

Your answer is incorrect.
Ependymal cells are a special type of glial cells that covers the lining in the walls of the
ventricular systems and enable CSF circulation.
The correct answer is: Lining the walls of the ventricular systems

Question 58 MoK Neuroanatomy 023
Not answered The disengagement of attention is mediated by

Marked out of 1.00
Select one:
Flag question
Medial temporal cortex
Medial parietal cortex
Lateral temporal cortex

1066
Prefrontal cortex
Occipital cortex

Your answer is incorrect.
The parietal lobe is postulated to be involved in engagement and disengagement of attention.
In conditions like Alzheimer's disease, the medial parietal cortex is affected in early stages,
and attentional deficits are seen before language and visuospatial deficits.
The correct answer is: Medial parietal cortex

Question 59 MoK Neuroanatomy 024
Not answered Which neuroglial cells act as scavenger cells at the sites of CNS injury?

Marked out of 1.00
Select one:
Flag question
Schwann cells
Microglia
Satellite cells
Astrocytes
Ependymal cells

Your answer is incorrect.
Microglia acts as scavenger cells at sites of CNS injury; Ependymal cells aids the flow of CSF.
Schwann cells are responsible for the myelin sheath formation in peripheral nervous system
(neurilemma formation). Satellite cells provide support to neurons in sensory and autonomic
ganglia.
The correct answer is: Microglia

Question 60 MoK Neurochemistry 021
Not answered Which of the following enzymes is seen only in the neuron and not in the glial cells?

Marked out of 1.00
Select one:
Flag question
Glutamic Acid Decarboxylase
monoamine oxidase
dopadecarboxylase
cholinesterase
Glutamine Synthase

Your answer is incorrect.

1067
Glutamic acid decarboxylase [GAD] is a neuron­specific enzyme that is not found in glial cells.
Hence, antibodies to GAD can be used to identify neurons that release GABA. Glutamine
synthase is a glial enzyme that metabolizes glutamate into glutamine.
The correct answer is: Glutamic Acid Decarboxylase

Question 61 MoK Neurochemistry 022
Not answered MAO­A is available primarily in which of the following body tissues?

Marked out of 1.00
Select one:
Flag question
Brain
Lungs
Brain
Kidney
Gut

Your answer is incorrect.
Monoamine oxidases are of two types: the brain predominantly has monoamine oxidase
inhibitor ­B and in the liver/GI tract, predominantly monoamine oxidase inhibitor­A is noted.
The correct answer is: Gut

Question 62 MoK Neurochemistry 023
Not answered The most common side effect when using sildenafil is:

Marked out of 1.00
Select one:
Flag question
Headache
Insomnia
Double vision
Hypotension
Bradycardia

Your answer is incorrect.
Viagra is an indirect vasodilator, resulting in headaches
The correct answer is: Headache

Question 63 MoK Neurochemistry 024
Not answered Low CSF concentration of 5­Hydroxy Indole Acetic Acid is associated with which of the
following?
Marked out of 1.00

Flag question Select one:

1068
Cheese Reaction
Serotonin Syndrome
Impulsivity

Visual Hallucinations
Coronary Artery Disease

Your answer is incorrect.
Animal and human research suggests that the central serotonin system is involved in the
inhibition of impulsive behaviour A low CSF 5­HIAA concentration was primarily associated
with impulsivity. http://archpsyc.ama­assn.org/cgi/content/abstract/51/1/20
The correct answer is: Impulsivity

Question 64 MoK Neurochemistry 025
Not answered Nitric oxide is synthesized from which of the following?

Marked out of 1.00
Select one:
Flag question
Toluene
Benzene

Phosphokinase
L­arginine
L­tryptophan

Your answer is incorrect.
Nitric oxide is synthesized from L­arginine when stimulated by nitric oxide synthase.
The correct answer is: L­arginine

Question 65 MoK Neuropathology 013
Not answered Which of the following human spongiform encephalopathy has the slowest course?

Marked out of 1.00
Select one:
Flag question
Sporadic CJD
Variant CJD

Iatrogenic CJD
Gerstmann­ Straussler­ scheinker syndrome
Kuru

1069
Your answer is incorrect.
Kuru is a type of prion disease which has the slowest course. The incubation period was
prolonged­ up to 40 years before disease onset, which was then rapid and fatal. It is thought
to have been transmitted by ritual cannibalism. Symptoms include personality and
behavioural disturbances, progressive cerebellar ataxia, dementia and death.
The correct answer is: Kuru

Question 66 MoK Neuropathology 014
Not answered Mr. X is a 73­year­old gentleman with 5­year history of Parkinson's disease, cognitive decline
and visual hallucinations. Post mortem evidence suggests alpha syn­nuclein deposits and
Marked out of 1.00
cytoplasmic inclusions. The common microscopic finding expected in this case would be
Flag question
Select one:
Pick cells
Tau proteins
Hirano bodies
Lewy bodies
Pick bodies

Your answer is incorrect.
The clinical features, in this case, are suggestive of Parkinson's disease dementia. In
Parkinson's disease, the Lewy bodies are confined to the substantia nigra. The Lewy bodies
are weak eosinophilic, spherical, cytoplasmic inclusions. Parkinson's disease and Lewy body
dementia contains an accumulation of alpha­synuclein and is classified under
synucleinopathies.
The correct answer is: Lewy bodies

Question 67 MoK Neuropathology 018
Not answered Which of the following neuroimaging finding is most suggestive of Alzheimer's disease?
Marked out of 1.00
Select one:
Flag question
Occipital hypoperfusion on HMPAO­ SPECT
Impaired dorsolateral cortex function
Activation of primary auditory cortex on functional MRI
Increased signal in the Pulvinar nucleus of thalamus bilaterally
Medial temporal atrophy on structural MRI scan

Your answer is incorrect.
MRI often shows atrophy of grey matter involving the hippocampus, entorhinal cortex and
amygdala in the medial temporal lobe.

1070
The correct answer is: Medial temporal atrophy on structural MRI scan

Question 68 MoK Neurophysiology 013
Not answered Which of the following neural structures is strongly implicated in the regulation of P50
suppression, a proposed endophenotype in schizophrenia?
Marked out of 1.00

Flag question Select one:
Frontal cortex
Angular gyrus
Basal ganglia
Hippocampus
Ventral tegmentum

Your answer is incorrect.
P50 suppression is regulated by wide­ranging neural circuitry, prominently involving
hippocampal structures.
The correct answer is: Hippocampus

Question 69 MoK Neurophysiology 014
Not answered A 56­year­old lady was admitted with depression following a suicide attempt by self­induced
skin laceration. She was increasingly isolating herself and complained of somatic pain
Marked out of 1.00
symptoms in her back. She also had constipation for few weeks. No past psychiatric history
Flag question was noted. Routine blood tests revealed an isolated anomaly of high serum calcium level
(15.8 mg/dL). Which of the following is the most likely diagnosis?

Select one:
Chronic renal failure
Hyperparathyroidism
Hypothyroidism
Hyperprolactinemia
Hypoparathyroidism

Your answer is incorrect.
Primary hyperparathyroidism is generally related to parathyroid adenomas. Most patients are
asymptomatic but can have recurrent kidney stones, peptic ulcers, dehydration, constipation,
and altered mental status. Diagnosis can be confirmed with measurement of parathyroid
hormone levels. (Ref: http://psy.psychiatryonline.org/cgi/content/full/40/6/486)
The correct answer is: Hyperparathyroidism

1071
Question 70 MoK Neurophysiology 015
Not answered Which of the following statements with regard to alpha waves in the EEG is correct?

Marked out of 1.00
Select one:
Flag question
Antipsychotics abolish alpha waves
It is maximal over occipital region
It reduces with eye closure
Excess of alpha rhythm is seen in Huntington's disease
It is a higher frequency band than beta waves

Your answer is incorrect.
The alpha brain waves oscillate about 10 times per second, and the range is 8­13 cycles per
second. The brain waves called 'Alpha' were the first to be discovered (around 1908, by an
Austrian Psychiatrist named Hans Berger).Neuroleptics commonly produce increased alpha
rhythm without obvious clinical manifestations. In Huntington's the EEG will be low voltage,
with poorly formed or absent alpha waves.
The correct answer is: It is maximal over occipital region

Question 71 MoK Neurosciences EMI023
Not answered Blood supply to the brain
Match the specific areas of the brain with their blood supply from the list provided:
Marked out of 3.00

Flag question
Superior cerebellar artery
Posterior inferior cerebellar artery
Internal auditory artery
Most of cerebellar hemisphere
Medial cerebral artery
Subclavian artery
Posterior cerebral artery
Anterior communicating artery
Anterior inferior cerebellar artery
Subclavian artery
Anterior cerebral artery
Anterior cerebral artery
Posterior cerebral artery
Posterior limb of internal capsule
Superior cerebellar artery
Internal auditory artery
Anterior inferior cerebellar artery
Anterior communicating artery
Medial cerebral artery
Subclavian artery
Posterior inferior cerebellar artery
Medial cerebral artery
Anterior cerebral artery
Internal auditory artery
Cerebellum­vermis Superior cerebellar artery
Posterior inferior cerebellar artery
Anterior inferior cerebellar artery
Anterior communicating artery
Posterior cerebral artery

1072
Your answer is incorrect.
Most of the cerebellar hemispheres including superior and middle cerebellar peduncles and
cerebellar nuclei are supplied by the superior cerebellar artery. The vermis of the cerebellum
is supplied by the posterior inferior cerebellar artery. 
The posterior limb of the internal capsule is supplied by the posterior cerebral artery. 

The correct answer is: Most of cerebellar hemisphere – Superior cerebellar artery, Posterior
limb of internal capsule – Posterior cerebral artery, Cerebellum­vermis – Posterior inferior
cerebellar artery

Question 72 MoK Neurosciences EMI024
Not answered Neurological symptoms
Find one term each to describe the following presentations:
Marked out of 3.00

Flag question
A 65­year­old gentleman was unable Right ­ left disorientation
Agraphaesthesia
to identify familiar objects such as key
or coins by touch with eyes closed. Prosopagnosia
He has difficulty in describing their Visuospatial agnosia
shape and texture. However, he Finger agnosia
identifies them when eyes are open. Dressing apraxia
Astereognosis
Constructional apraxia
Visuospatial agnosia
Anosognosia
Astereognosis
A 67­year­old woman was unable to Finger agnosia
recognize what number is scratched
Right ­ left disorientation
into her skin without seeing
Constructional apraxia
Dressing apraxia
Prosopagnosia
Anosognosia
Dressing apraxia
Agraphaesthesia
Finger agnosia
A man has left side hemiplegia after a Anosognosia
stroke. He is unaware of his deficit. Right ­ left disorientation
Prosopagnosia
Agraphaesthesia
Constructional apraxia
Visuospatial agnosia
Astereognosis
Your answer is incorrect.
Agraphaesthesia is defined as inability to recognize what number or alphabet is scratched
on one's skin without seeing. Astereognosis is defined as inability to recognize objects by
palpation, and without visual inspection. Anosognosia refers to the inability to identify one's
neurological deficits.
The correct answer is: A 65­year­old gentleman was unable to identify familiar objects such as
key or coins by touch with eyes closed. He has difficulty in describing their shape and texture.
However, he identifies them when eyes are open. – Astereognosis, A 67­year­old woman was

1073
unable to recognize what number is scratched into her skin without seeing –
Agraphaesthesia, A man has left side hemiplegia after a stroke. He is unaware of his deficit. –
Anosognosia

Question 73 MoK Neurosciences EMI026
Not answered Receptor localization 
For each of these areas of the brain, identify the main neurotransmitter pathways localized to
Marked out of 5.00
the specified regions:
Flag question

GABA
Acetylcholine
Dopamine
Median raphe nuclei
Norepinephrine
Glutamate
Histamine
Serotonin
Opiate receptors
Histamine
Glycine
GABA
Acetylcholine
Nucleus basalis of Meynert
Opiate receptors
Glutamate
Glycine
Serotonin
Dopamine
Norepinephrine
Norepinephrine
Opiate receptors
Acetylcholine
Substantia nigra
Serotonin
Histamine
GABA
Glutamate
Glycine
Histamine
Dopamine
Serotonin
Opiate receptors
Dorsal raphe nuclei
GABA
Glycine
Acetylcholine
Dopamine
Norepinephrine
Acetylcholine
Glutamate
Norepinephrine
Histamine
Locus coeruleus
Glycine
Glutamate
Opiate receptors
Serotonin
GABA
Dopamine
Your answer is incorrect.
The major site of serotonergic cell bodies is in the upper pons and midbrain ­ in the median
and dorsal raphe nuclei. 

1074
A group of acetylcholine producing neurons in the nucleus basalis of Meynert project to the
cerebral cortex, limbic system, hypothalamus and thalamus. 
The major concentration of norepinephrine is in the cell bodies that project upward in the
brain is in the locus coeruleus in the pons. 
Dopamine mainly localized in nigrostriatal pathways, mesolimbic and mesocortical pathways
and tuberoinfundibular pathways.
The correct answer is: Median raphe nuclei – Serotonin, Nucleus basalis of Meynert –
Acetylcholine, Substantia nigra – Dopamine, Dorsal raphe nuclei – Serotonin, Locus
coeruleus – Norepinephrine

Question 74 MoK Neurosciences EMI028
Not answered Lobar dysfunction
For each of the clinical descriptions given below, identify the area of the brain damaged.
Marked out of 5.00

Flag question
Frontal lobe
Cerebellum
Alexia without agraphia, colour Dominant parietal lobe
agnosia and visual object agnosia Dominant occipital lobe
Non­dominant parietal lobe
Non­dominant occipital lobe
Non­dominant temporal lobe
Bilateral temporal lobe lesions
Non­dominant parietal lobe
Dominant temporal lobe
Non­dominant occipital lobe
Profound changes in personality, Frontal lobe
problems with verbal expression and
Dominant occipital lobe
forced utilization behaviour
Non­dominant temporal lobe
Cerebellum
Dominant temporal lobe
Dominant parietal lobe
Frontal lobe
Bilateral temporal lobe lesions
Non­dominant occipital lobe
Dominant occipital lobe
Profound amnesia
Bilateral temporal lobe lesions
Non­dominant parietal lobe
Cerebellum
Dominant temporal lobe
Non­dominant temporal lobe
Dominant temporal lobe
Dominant parietal lobe
Dominant parietal lobe
Frontal lobe
Non­dominant temporal lobe
Visuospatial agnosia, prospagnosia Non­dominant occipital lobe
and complex visual hallucinations Bilateral temporal lobe lesions
Cerebellum
Dominant occipital lobe
Non­dominant parietal lobe

1075
Dressing apraxia, prospagnosia,
Dominant occipital lobe
hemisomatognosia
Cerebellum
Dominant temporal lobe
Dominant parietal lobe
Non­dominant occipital lobe
Frontal lobe
Non­dominant temporal lobe
Non­dominant parietal lobe
Bilateral temporal lobe lesions
Your answer is incorrect.
Lesions in the dominant occipital lobe result in alexia without agraphia, colour agnosia and
visual object agnosia. 
Lesions in the non­dominant occipital lobe result in visuospatial agnosia, metamorphopsia
(image distortion) and complex visual hallucinations. 
Damage to the frontal lobes results in a change in one's personality, with poor motivation,
poor judgement and impaired social awareness. If the orbital frontal region (just above the
eye sockets) is damaged, forced utilization behaviour may be seen. Here, when objects are
placed in front of the subject, they will be picked up and used appropriately even when the
subject is told not to do so. 
Profound amnesia results from bilateral medial temporal lobe lesions. The causes of such
lesions would include infections, tumour and epilepsy. Both anterograde and retrograde
memory will be impaired with intact immediate memory (i.e. unimpaired digit span) and
preserved executive task performance. 
Lesions in the non­dominant parietal lobe result in anosognosia, hemisomatagnosia,
dressing apraxia and prosopagnosia. Gerstmann syndrome is associated with lesions of the
dominant parietal lobe. 
(Ref: Core Psychiatry: pg 16­20)
The correct answer is: Alexia without agraphia, colour agnosia and visual object agnosia –
Dominant occipital lobe, Profound changes in personality, problems with verbal expression
and forced utilization behaviour – Frontal lobe, Profound amnesia – Bilateral temporal lobe
lesions, Visuospatial agnosia, prospagnosia and complex visual hallucinations – Non­
dominant occipital lobe, Dressing apraxia, prospagnosia, hemisomatognosia – Non­dominant
parietal lobe

Question 75 MoK Pharmacology EMI013
Not answered Pharmacodynamics
Select one mechanism of action each for the following drugs
Marked out of 3.00

Flag question
5HT1A partial agonist
Inhibitor of norepinephrine and serotonin re­uptake
Presynaptic alpha­2 agonist
Rivastigmine
GABA­A agonist
GABA­B agonist
D­2 receptor partial agonist
NMDA receptor antagonist
MAO­A Inhibitor
Selective norepinephrine re­uptake inhibitor
Butyryl­cholinesterase inhibitor
Selegeline MAO­B Inhibitor

1076
GABA­B agonist
Butyryl­cholinesterase inhibitor
MAO­A Inhibitor
D­2 receptor partial agonist
Inhibitor of norepinephrine and serotonin re­uptake
Selective norepinephrine re­uptake inhibitor
GABA­A agonist
Presynaptic alpha­2 agonist
GABA­B agonist
5HT1A partial agonist
D­2 receptor partial agonist
NMDA receptor antagonist
Inhibitor of norepinephrine and serotonin re­uptake
Diazepam MAO­B Inhibitor
Presynaptic alpha­2 agonist
Butyryl­cholinesterase inhibitor
NMDA receptor antagonist
MAO­A Inhibitor
5HT1A partial agonist
GABA­A agonist
Selective norepinephrine re­uptake inhibitor
Your answer is incorrect.
MAO­B Inhibitor
Rivastigmine is a cholinesterase Inhibitor. It acts by inhibiting acetylcholinesterase enzyme
that breaks down acetylcholine centrally. Rivastigmine inhibits both the acetyl and butyl­
cholinesterase while donepezil and galantamine are acetyl specific. 
Selegiline is a monoamine oxidase Inhibitor ­ selective for the enzyme MAO­B at normal
therapeutic doses; selectivity is lost when the drug is administered (usually as a patch) at
higher doses, leading to some antidepressant action.
Diazepam is a type of benzodiazepine which acts via a specific site called omega site in
GABA­A complex. All benzodiazepines are full agonists except clonazepam which is a partial
agonist. They facilitate GABA action on GABA­A complex ­ thus inhibitory neurotransmission
via chloride ion flow is facilitated. Benzodiazepines have no direct agonistic action in the
absence of GABA.
The correct answer is: Rivastigmine – Butyryl­cholinesterase inhibitor, Selegeline – MAO­B
Inhibitor, Diazepam – GABA­A agonist

Question 76 MoK Pharmacology EMI016
Not answered Pharmacokinetics (1)
Match each characteristic below with the most likely drug from the list given
Marked out of 3.00

Flag question
Imipramine
Diazepam
This drug undergoes no metabolism, Fluoxetine
and it is excreted unchanged by the
Lorazepam
kidney
Lamotrigine
Nitrazepam
Temazepam
Carbamazepine
Imipramine
This drug accelerates its own
metabolism and the metabolism of
Sodium valproate
Phenytoin
other drugs by inducing the hepatic Phenytoin
Lorazepam
microsomal enzymes Lithium
Lamotrigine
Carbamazepine
Diazepam
Nitrazepam
Lithium
1077
Temazepam
Fluoxetine
The metabolism of this drug follows Lamotrigine
zero order kinetics Sodium valproate
Lorazepam
Carbamazepine
Diazepam
Lithium
Phenytoin
Your answer is incorrect. Imipramine
Nitrazepam
Lithium is orally well absorbed but not metabolised by the liver; it is renally excreted. 
Carbamazepine is metabolized by the hepatic cytochrome P450 2D6 system, which it also
robustly induces. As a result of this autoinduction, the rate of metabolism of carbamazepine
(and other P450 substrates) usually increases over the first several weeks of treatment. The
initial steady state may be attained within 4 to 5 days, but autoinduction may delay final steady
state until 3 to 4 weeks after treatment initiation. Hence, the level of carbamazepine must be
monitored, and its dose often must be raised during this early phase of treatment. 
Slow release preparations e.g. lithium MR, depot preparations follow zero­order absorption
kinetics; drugs that rapidly saturate enzymes such as alcohol and phenytoin follow zero­order
elimination kinetics
The correct answer is: This drug undergoes no metabolism, and it is excreted unchanged by
the kidney – Lithium, This drug accelerates its own metabolism and the metabolism of other
drugs by inducing the hepatic microsomal enzymes – Carbamazepine, The metabolism of this
drug follows zero order kinetics – Phenytoin

Question 77 MoK Pharmacology EMI017
Not answered Drug interactions (1)
Choose agents that will have clinically significant interaction for each description below
Marked out of 3.00

Flag question
St John's wort
A 45­year­old patient is an Clozapine
enthusiastic naturalist and is keen on Pimozide
drinking grapefruit juice for breakfast. Clomipramine
(Choose TWO) Phenytoin
Lithium
Carbamazepine
Amitryptyline
Phenytoin
Clomipramine
Lithium
A 35­year­old patient is on long­term St John's wort
NSAID therapy for an ankle injury. Pimozide
(Choose ONE) Amitryptyline
Carbamazepine
Clozapine

Your answer is incorrect.

1078
The "grapefruit juice effect" is due to components of grapefruit juice (bergamottin, 6, 7­
dihydroxybergamottin, and naringenin ­ these significantly increase drug oral bioavailability
by selectively and rapidly down­regulating intestinal (but not liver) CYP3A4 and to a lesser
extent, 1A2. This effect is greatest in drugs with high first­pass metabolism such as the
dihydropyridine calcium antagonists, felodipine and nimodipine, terfenadine, carbamazepine,
triazolam and midazolam (to some extent diazepam), simvastatin and methylprednisone.
Buspirone and Pimozide are also significantly affected by grapefruit. (Ref: Psychosomatics;
2005: 46). 
The class of medications that could increase serum lithium levels and precipitate lithium
toxicity includes diuretics (Loop and Thiazide diuretics), NSAIDs and ACE inhibitors.
The correct answer is: A 45­year­old patient is an enthusiastic naturalist and is keen on
drinking grapefruit juice for breakfast. (Choose TWO) – Carbamazepine, Pimozide, A 35­year­
old patient is on long­term NSAID therapy for an ankle injury. (Choose ONE) – Lithium

Question 78 MoK Pharmacology EMI023
Not answered Antipsychotics & side effects
For each of the following situations, identify the most likely medication(s) associated with it.
Marked out of 5.00

Flag question
Clozapine
Aripiprazole
Trifluoperazine
Constipation
Sulpiride
Thioridazine
Olanzapine
Haloperidol
Amisulpride
Thioridazine
Quetiapine
Amisulpride
Aripiprazole
Paralytic Ileus
Clozapine
Sulpiride
Trifluoperazine
Olanzapine
Haloperidol
Haloperidol
Quetiapine
Amisulpride
Thioridazine
Orthostatic Hypotension
Quetiapine
Aripiprazole
Sulpiride
Clozapine
Olanzapine
Haloperidol
Trifluoperazine
Thioridazine
Sulpiride
Post­injection delirium Olanzapine
Trifluoperazine
Clozapine
Amisulpride
Quetiapine
Aripiprazole

1079
Nause and Vomiting
Thioridazine
Sulpiride
Quetiapine
Olanzapine
Aripiprazole
Amisulpride
Trifluoperazine
Haloperidol
Clozapine
Your answer is incorrect.
Clozapine has pronounced anticholinergic properties. It can cause severe constipation.
Effective treatment or prevention of constipation is essential, as this can prove fatal by
progressing to paralytic ileus and bowel ischemia, followed by sepsis and perforation. 
Clozapine and Risperidone can cause severe orthostatic hypotension due to alpha one
adrenergic blockade. 
Nausea, vomiting and gastric side effects are common with Aripiprazole due to 5­HT1A
receptor partial agonistic actions. 
Olanzapine if administered as long­acting depot injection (Olanzapine Pamoate) could
produce symptoms suggestive of antipsychotic overdose in a small number of subjects (<
0.1%), a phenomenon that has been termed "post­injection delirium/sedation syndrome"
(PDSS). PDSS is characterized by symptoms related to excessive sedation and delirium soon
after the injection, consistent with some of the symptoms of oral olanzapine overdose. 
Ref: McDonnell et al., BMC Psychiatry. 2010; 10: 45. Retrieved from
http://www.ncbi.nlm.nih.gov/pmc/articles/PMC2895590/
The correct answer is: Constipation
– Clozapine, Paralytic Ileus
– Clozapine, Orthostatic Hypotension
– Clozapine, Post­injection delirium – Olanzapine, Nause and Vomiting
– Aripiprazole

Finish review

1080
 Home Mock Paper Practice Tests

Mock Paper A(2)

Started on Thursday, 25 June 2015, 5:58 AM
State Finished
Completed on Thursday, 25 June 2015, 5:59 AM
Time taken 18 secs
Grade 0.00 out of 100.00
Feedback You are on the right track. Could you spend some more time on your revision?

Question 1 MoK Adverse Effects 032
Not answered Phenothiazines that are associated with pigment deposits in anterior lens capsule include

Marked out of 1.00
Select one:
Flag question
Chlorpromazine and promazine
perphenazine and trifluoperazine
Chlorpromazine and perphenazine

promazine and thioridazine
Chlorpromazine and thioridazine

Your answer is incorrect.
Aliphatic phenothiazines chlorpromazine and thioridazine are associated with pigmentary
retinopathy and lens pigmentation.
The correct answer is: Chlorpromazine and thioridazine

Question 2 MoK Adverse Effects 049
Not answered Which of the following is a risk factor for neuroleptic­induced akathisia?

Marked out of 1.00
Select one:
Flag question
Non­affective psychosis
Afro­Caribbean ethnicity
Younger age
lipid abnormalities
Female gender

1081
Your answer is incorrect.
Female gender and older age are risk factors for akathisia.
The correct answer is: Female gender

Question 3 MoK Adverse Effects 058
Not answered Which of the following can provide an acute symptom relief for a patient with tardive
dyskinesia?
Marked out of 1.00

Flag question Select one:
IV amantadine
Bromocriptine

Increase neuroleptic dose
IV amphetamine
Stopping all neuroleptics

Your answer is incorrect.
Among the given options, increasing neuroleptic dose can provide acute relief but can worsen
TD in the long run and so is not recommended. Stopping all neuroleptics suddenly can
worsen TD acutely.
The correct answer is: Increase neuroleptic dose

Question 4 MoK Adverse Effects 059
Not answered The most well­established risk factor in terms of developing late onset dyskinesia when using
antipsychotics is
Marked out of 1.00

Flag question Select one:
brain pathology
mood disorder
increasing age
Afro Caribbean ethnicity
female gender

Your answer is incorrect.
One of the risk factors for TD is age. The majority of those with tardive dyskinesia are over the
age of 55.
The correct answer is: increasing age

Question 5 MoK Adverse Effects 063

1082
Not answered Fetal Ebstein's anomaly is associated with which of the following psychotropic medication?
Marked out of 1.00
Select one:
Flag question
Lithium
Sodium valproate
Lamotrigine

Carbamazepine
Gabapentin

Your answer is incorrect.
Lithium use during pregnancy has a well­known association with the cardiac malformation
Ebstein's anomaly (relative risk is 10­20 times more than control, but the absolute risk is low at
1:1000). The period of maximum risk to the foetus on exposure is 2­6 weeks after conception.
The correct answer is: Lithium


Question 6 MoK Adverse Effects 065
Not answered A woman comes into Emergency Department with lethargy, nausea and diarrhoea with weight
loss on a hot afternoon. She is taking some unknown psychotropic medication for a long time.
Marked out of 1.00
She is recently started on 'water tablets' for swelling in her legs. Which of the following is a
Flag question likely cause of her symptoms?

Select one:
Hypokalemia
Lithium toxicity

Hyponatremia
Neuroleptic malignant syndrome
Serotonin syndrome

Your answer is incorrect.
The story here is suggestive of lithium overdose or toxicity.
The correct answer is: Lithium toxicity

Question 7 MoK Adverse Effects 068
Not answered The factors that increase the risk of agranulocytosis in patients treated with clozapine include
all of the following except
Marked out of 1.00

Flag question Select one:
Female gender
Male gender

1083
Increasing age
Low baseline WBC count
Asian ethnicity

Your answer is incorrect.
The factors that increase the risk for agranulocytosis in patients treated with clozapine are
Jewish descent, female gender (male­to­female ratio of 1:2), increasing age, first 6 months of
treatment, and a low white blood count at baseline.
The correct answer is: Male gender

Question 8 MoK Adverse Effects 073
Not answered Which one of the following is the least likely feature associated with Neuroleptic malignant
syndrome?
Marked out of 1.00

Flag question Select one:
Muscle rigidity
Hyperthermia
Autonomic lability
myoclonus

Raised CPK levels

Your answer is incorrect.
Myoclonus is a feature that could be seen in serotonin syndrome and not in NMS.
The correct answer is: myoclonus

Question 9 MoK Adverse Effects 074
Not answered The most common side effect of sildenafil is
Marked out of 1.00
Select one:
Flag question
Bradycardia
Hypotension
Nasal stuffiness
Insomnia

Hair loss

Your answer is incorrect.

1084
Common side effects of sildenafil citrate (occurring in more than 2 percent of people) includes
headache 16%, facial flushing­10 %, indigestion­ 7 %, nasal congestion­ 4 %, Urinary tract
Infections (UTI) ­3 %, vision problems­ 3%.
The correct answer is: Nasal stuffiness

Question 10 MoK Adverse Effects 075
Not answered Tom recently underwent an alcohol detoxification programme. Upon achieving abstinence, he
was keen on anti­craving treatment and has been started on Acamprosate. Which one among
Marked out of 1.00
the following is a common side effect of Acamprosate?
Flag question
Select one:
Constipation
Hair loss
Nausea
Headache
Dizziness

Your answer is incorrect.
Acamprosate is well tolerated though side effects include diarrhoea, abdominal pain, nausea
and vomiting, also pruritus. It should be initiated as soon as abstinence has been achieved
and should be continued for up to 6 months, with regular monthly supervision. (Ref: The
Maudsley Prescribing Guidelines, 11th ed. Pg. 370)
The correct answer is: Nausea

Question 11 MoK Adverse Effects 076
Not answered Which one among the following has a high hepatotoxic potential?
Marked out of 1.00
Select one:
Flag question
Trazadone
Amitryptyline
Dothiepin
Imipramine
Nefazodone

Your answer is incorrect.
Because of concerns of hepatotoxicity, the sale of the antidepressant nefazodone
hydrochloride was discontinued by the manufacturer in 2003.
The correct answer is: Nefazodone

1085
Question 12 MoK Adverse Effects 077
Not answered Low serum iron is associated with which of the following side effects related to neuroleptic
medication treatment?
Marked out of 1.00

Flag question Select one:
sedation
orthostatic hypotension
weight gain
neuroleptic malignant syndrome
hyperglycaemia

Your answer is incorrect.
Low serum iron is associated with the neuroleptic malignant syndrome and extrapyramidal
reactions. Correcting with supplemental iron may reduce the risk for these conditions.
The correct answer is: neuroleptic malignant syndrome

Question 13 MoK Adverse Effects 078
Not answered A 45­year­old gentleman with depression has been started on Tranylcypromine. Which drug
is safe to be taken?
Marked out of 1.00

Remove flag Select one:
Nifedipine
Pethidine

Pseudoephedrine
Buprenorphine
Salbutamol

Your answer is incorrect.
Tranylcypromine is a MAOI Inhibitor. MAOIs should not be combined with other psychoactive
substances (antidepressants, painkillers, stimulants, both legal and illegal etc.). Certain
combination of drugs are potentially fatal with MAOIs: examples include SSRIs, tricyclics,
MDMA, meperidine, tramadol and dextromethorphan.
The correct answer is: Nifedipine

Question 14 MoK Adverse Effects 079
Not answered According to CATIE study, which drug had the lowest drop out rate when treating
schizophrenia?
Marked out of 1.00

Flag question Select one:
Amisulpride

1086
Quetiapine
Olanzapine
Risperidone

Ziprasidone

Your answer is incorrect.
CATIE stands for Clinical Antipsychotic Trials of Intervention Effectiveness. The study design
was double­blind pragmatic RCT. 1493 patients with chronic schizophrenia were included
and randomised to one of the following: Olanzapine, Quetiapine, Risperidone, Ziprasidone
(added later in the trial), Perphenazine. The primary outcome was a 'real­world' measure ­
discontinuation for any reason (irrespective of whether patient­initiated or physician­initiated).
Irrespective of the prescribed drug ­ 74% discontinued treatment in 18 months (surprisingly
high despite naturalistic design). Median time to discontinue was 4.6 months. Olanzapine had
lowest discontinuation rate (still 64%) ­ but highest side effect burden. 64% discontinued
olanzapine; 75%, perphenazine; 82%, quetiapine; 74%, risperidone; and 79%, ziprasidone.
The correct answer is: Olanzapine

Question 15 MoK Adverse Effects 080
Not answered Which of the following adverse effect of clozapine is mediated via alpha adrenoceptor
blockade?
Marked out of 1.00

Flag question Select one:
sedation
weight gain
constipation

sialorrhea
orthostatic hypotension

Your answer is incorrect.
Alpha 1 blockade leads to a postural drop in BP.
The correct answer is: orthostatic hypotension

Question 16 MoK Pharmacodynamics 038
Not answered Mr Y is a 45­year­old gentleman with history of schizophrenia. He suffers from akathisia due to
taking risperidone and also has some depressive symptoms. The drug of choice in his case
Marked out of 1.00
would be
Flag question
Select one:
Venlafaxine
Amitrytyline

1087
Paroxetine
Fluoxetine
Mirtazapine

Your answer is incorrect.
In the treatment of akathisia, a reduction in symptoms may be seen with mirtazapine. Other
drugs, which are helpful, include propranolol (30­80 mg/day), low dose clonazepam, 5­HT2
antagonists such as cyproheptadine, mianserin, trazodone and diphenhydramine.
The correct answer is: Mirtazapine

Question 17 MoK Pharmacodynamics 041
Not answered Buprenorphine acts on which receptor?
Marked out of 1.00
Select one:
Flag question
Sigma
Mu
Kappa
Delta

Alpha

Your answer is incorrect.
Buprenorphine is a µ­opioid receptor agonist with high affinity, but a low intrinsic activity.
Compared with morphine which behaves as a full µ­opioid agonist, buprenorphine is usually
defined as a partial µ­opioid agonist that shows high affinity for and slow dissociation from the
µ­opioid receptor.
The correct answer is: Mu

Question 18 MoK Pharmacodynamics 042
Not answered A 40­year­old woman with a past history of psychotic depression is having a relapse but is
managed by the Crisis and Home Treatment team in the community. She is currently on
Marked out of 1.00
lofepramine 180 mg and risperidone 2 mg at bedtime. After a week, she reports to the
Flag question community worker who visited her at home that her nipples are leaking. She is very
concerned and denies she could be pregnant. Which class of medications is most commonly
known to cause this problem?

Select one:
Antidepressants of SSRI class
Tricyclic antidepressants
Mood stabilisers
Antidepressants of MAO inhibitors class

1088
Neuroleptics

Your answer is incorrect.
Dopamine receptor blockade causes hyperprolactinemia, which can cause breast
enlargement. This is associated with galactorrhea in women and suppression of testosterone
production in men. The typical neuroleptics such as haloperidol and chlorpromazine are
particularly prone to cause an increase in prolactin.
The correct answer is: Neuroleptics

Question 19 MoK Pharmacodynamics 043
Not answered A 50 year old man is treated for erectile dysfunction using sildenafil as and when required.
Availability of which of the following vasodilatory substances is increased by sildenafil?
Marked out of 1.00

Flag question Select one:
Nitric oxide

Endothelin
Phosphodiesterase
Angiotensin
Prostacyclin

Your answer is incorrect.
By inhibiting phosphodiesterase, sildenafil increases the availability of nitric oxide, a second
messenger that acts as vasodilator of penile smooth muscle producing an erection.
The correct answer is: Nitric oxide

Question 20 MoK Pharmacodynamics 044
Not answered Which of the following substrate/receptors blockade explains the clinical efficacy of riluzole in
motor neuron disease?
Marked out of 1.00

Flag question Select one:
Acetyl cholinesterase enzyme
AMPA receptors
NMDA receptors
GABA receptors
Dopamine D5 receptor

Your answer is incorrect.
Riluzole is an NMDA antagonist, similar to memantine

1089
The correct answer is: NMDA receptors

Question 21 MoK Pharmacodynamics 045
Not answered The mechanism of action of Memantine is
Marked out of 1.00
Select one:
Flag question
Non competitive NMDA receptor agonist
Non competitive NMDA receptor antagonist
Competitive NMDA receptor antagonist
Competitive NMDA receptor agonist
Acetylcholinesterase inhibitor

Your answer is incorrect.
The mechanism of memantine involves blockade of N­methyl­d­aspartate (NMDA) glutamate
receptors. Unlike ketamine which is a high­affinity noncompetitive blocker, memantine is a
non­competitive blocker with low affinity and binds only to actively open NMDA channels. Its
receptor dissociation rate is relatively fast, and so it does not accumulate and interfere with
normal NMDA activity
The correct answer is: Non competitive NMDA receptor antagonist

Question 22 MoK Pharmacodynamics 046
Not answered What is the mechanism of action of atomoxetine when used in the treatment of ADHD?
Marked out of 1.00
Select one:
Flag question
Dopaminergic agonism
Noradrenaline reuptake inhibition
Glutamatergic antagonism
Serotonin reuptake inhibition
Dopamine reuptake inhibition

Your answer is incorrect.
The exact mechanism of action is unknown, but atomoxetine selectively inhibits the pre­
synaptic noradrenaline transporter (NARI ­ Noradrenaline reuptake inhibitor)
The correct answer is: Noradrenaline reuptake inhibition

Question 23 MoK Pharmacodynamics 047
Not answered Which one among the following properties is not a significant effect of Trazodone?
Marked out of 1.00
Select one:

1090
Flag question
Serotonin reuptake blockade
Antihistamine properties
Strong alpha­1 antagonism
Norepinephrine reuptake blockade
Serotonin 2 antagonism

Your answer is incorrect.
Nefazadone achieves its clinical effects by inhibiting the reuptake of primarily serotonin but
also norepinephrine and by antagonizing the postsynaptic 5­HT2 receptor and a1­adrenergic
receptors. Trazadone works via serotonin 2 antagonism and serotonin re­uptake blockade
with strong alpha­1 antagonism and antihistamine properties.
The correct answer is: Norepinephrine reuptake blockade

Question 24 MoK Pharmacokinetics 047
Not answered The effect of which of the following groups of drugs is least influenced by their
pharmacokinetics?
Marked out of 1.00

Flag question Select one:
SSRIs
MAO Inhibitors

Benzodiazepines
Neuroleptics
Tricyclics

Your answer is incorrect.
MAOIs are also well absorbed from the gastrointestinal tract. Their short half­life of one to two
hours is not particularly relevant as they bind irreversibly with MAO. Thus, the activity of these
drugs depends less on pharmacokinetics and more on the synthesis of new MAO to restore
normal enzyme activity. This synthesis requires approximately two weeks. This class of drugs
is little used due to their potentially dangerous interactions with sympathomimetics and foods
containing tyramine.
The correct answer is: MAO Inhibitors

Question 25 MoK Pharmacokinetics 048
Not answered Which of the following psychotropic drugs could be potentially affected by acetylator status?

Marked out of 1.00
Select one:
Flag question
Phenothiazines
Lithium

1091
Tricyclic antidepressants
Acetylcholinesterase inhibitor
MAO inhibitors

Your answer is incorrect.
Though potentially MAO inhibitors can be affected acetylator status, in practical use this is not
usually a problem.
The correct answer is: MAO inhibitors

Question 26 MoK Pharmacokinetics 049
Not answered Which one among the following antidementia drugs is mainly renally excreted?

Marked out of 1.00
Select one:
Flag question
Tacrine
Donepezil
Rivastigmine
Galantamine
Memantine

Your answer is incorrect.
Memantine is mainly renally excreted. But as robust data from renal impaired patients are not
available, drug dose should be reduced in patients with mild to moderate renal impairment.
Memantine is not recommended for patients with severe renal impairment (Jann & Small
2005).
The correct answer is: Memantine

Question 27 MoK Pharmacokinetics 050
Not answered Which drug acts as a full agonist at Mu receptor and has a long half life?

Marked out of 1.00
Select one:
Flag question
Buspirone
Buprenorphine
Disulfiram
Methadone
Acamprosate

Your answer is incorrect.

1092
Methadone is a full mu­ opioid agonist. Its half­life is approximately 24 hours with regular use.
The correct answer is: Methadone

Question 28 MoK Pharmacokinetics 051
Not answered When swapping MAOIs to SSRIs, what is the optimum wash out period?
Marked out of 1.00
Select one:
Flag question
Minimum 2 days is essential
Minimum 5 days is essential
Minimum 2 months is essential
Minimum 2 weeks is essential
Minimum 5 weeks is essential

Your answer is incorrect.
MAOI to SSRI­ 2 weeks SSRI to MAOI­2 weeks for all, 5 weeks for Fluoxetine TCA to MAOI­ 2
weeks MAOI to TCA­ 1­2 weeks.
The correct answer is: Minimum 2 weeks is essential

Question 29 MoK Pharmacokinetics 052
Not answered Apparent volume of distribution of haloperidol is

Marked out of 1.00
Select one:
Flag question
1l/kg
0.5l/kg
0.1l/kg
0.25l/kg
more than 2l/kg

Your answer is incorrect.
The apparent volume of distribution is the theoretical volume of fluid into which the total drug
administered would have to be diluted to produce the concentration in plasma. For example, if
1000 mg of a drug is given and the subsequent plasma concentration is 10 mg/L, that 1000
mg seems to be distributed in 100 L (dose/volume = concentration; 1000 mg/x L = 10 mg/L;
therefore, x = 1000 mg/10 mg/L = 100 L). Many acidic drugs (e.g., warfarin) are highly protein­
bound and thus have a small apparent volume of distribution. Many basic drugs (e.g.,
amphetamine) are extensively taken up by tissues and thus have an apparent volume of
distribution larger than the volume of the entire body. Haloperidol is very extensively bound to
plasma proteins (90%). It has a volume of distribution ranging from 9.5 to 21.7 liters/kilogram
(Kudo and Ishizaki, 1999) [Drug Distribution to Tissues ­ Pharmacokinetics ­ Merck. Retrieved
from http://www.merckmanuals.com/professional/clinical­pharmacology/pharmacokinetics/].
The correct answer is: more than 2l/kg

1093
Question 30 MoK Pharmacokinetics 053
Not answered Among Caucasians who are social drinkers, what percentage of alcohol is metabolized by
alcohol dehydrogenase?
Marked out of 1.00

Flag question Select one:
less than 9
60­79
more than 90%
75­85
40­59

Your answer is incorrect.
The major site of alcohol metabolism or breakdown (biotransformation) is in the liver. Alcohol
dehydrogenase is the rate­limiting enzyme in the metabolism of alcohol and thus limits the
amount of alcohol metabolized. More than 90% of consumed alcohol in Caucasians is
metabolized via this route. Another significant metabolic pathway in the liver is called
microsomal ethanol oxidizing system (MEOS). This consists of the induction of the liver
enzyme cytochrome P­450 (CYP2E1) which biotransforms a number of substances including
alcohol and barbiturates
The correct answer is: more than 90%

Question 31 MoK Pharmacokinetics 054
Not answered Which of the following over­the­counter preparations must be avoided by a patient who is
prescribed phenelzine for resistant depression with social anxiety?
Marked out of 1.00

Flag question Select one:
Nasal decongestant sprays
Artificial tears
Antacids
Nicotine inhalers
Heat patches

Your answer is incorrect.
If MAOIs are taken with decongestants, the patient may experience a life­threatening rise in
blood pressure due to synergistic sympathetic drive.
The correct answer is: Nasal decongestant sprays

Question 32 MoK Pharmacokinetics 056
Which of the following statements about lithium is correct?

1094
Not answered Select one:
Marked out of 1.00 Lithium levels fall within 10 days of starting a thiazide diuretic
Flag question
NSAIDs can increase the plasma levels of lithium to >40%
A third of patients treated with lithium experience at least one side effect

70% of middle aged women develop hypothyroidism after 6­12 months of
treatment
ACE inhibitors cause a reduction in lithium levels

Your answer is incorrect.
NSAIDs increase the plasma levels of lithium (>10 to >40% in some cases) by inhibiting the
synthesis of renal prostaglandins and reducing renal blood flow. This increases renal
reabsorption of both sodium and lithium. The levels of lithium usually increase within ten days
of a thiazide diuretic being prescribed. ACE inhibitors can reduce thirst, which can lead to
dehydration, and increase renal sodium loss, leading to increased sodium reabsorption by
the kidneys, causing an increase in lithium plasma levels. About 20% of middle­aged women
develop hypothyroidism after 6­12 months of treatment. Approximately two­thirds of patients
treated with lithium experience at least one side effect.
The correct answer is: NSAIDs can increase the plasma levels of lithium to >40%

Question 33 MoK Applied Neuroscience 042
Not answered Which of the following subtests of Wechsler Adult Intelligence Scale (WAIS) is noted to resist
any decline with aging?
Marked out of 1.00

Flag question Select one:
Digit symbol
Block design
Digit span
similiarities
Picture completion

Your answer is incorrect.
Hold tests' are supposed to be resistant to age­related decline; so they may be sensitive to
organic brain damage such as dementia
The correct answer is: Picture completion

Question 34 MoK Applied Neuroscience 057
Not answered Hayling test in neuropsychological assessments is used to test which of the following?

Marked out of 1.00
Select one:
Flag question
Digit span and executive function

1095
Verbal fluency and performace IQ
Verbal fluency and executive function
Premorbid IQ
Verbal memory

Your answer is incorrect.
The Hayling test (Burgess and Shallice, 1997) is an instrument that asks the subject to
complete sentences by saying a final word to complete the sentence. The aim of Part A of the
test is evaluate the preprogrammed organizing process because in this phase the constructs
to be examined are concentrated attention, verbal initiation, processing speed, and the
strategy of a well­succeeded search for automated words related to the preactivation of
semantic networks. Part B assesses more complex Executive Function components, such as
verbal inhibition and planning, because the subject must inhibit the content of the sentence
(semantics) and develop alternative strategies in his lexical search to complete the syntactic
stimulus (Chan et al., 2008) (Retrieved from
http://www.psycneuro.org/index.php/path/article/viewFile/104/393).
The correct answer is: Verbal fluency and executive function

Question 35 MoK Applied Neuroscience 058
Not answered A 49­year­old patient with schizophrenia scored poorly on Brixton test. This shows deficits in
Marked out of 1.00
Select one:
Flag question
Attention and concentration
Verbal memory
Visuospatial sequencing
Abstraction
Language

Your answer is incorrect.
The Brixton test is a visuospatial sequencing task that measures the ability to learn rules from
sequences of stimuli. It is relatively free from educational and sociocultural bias as it does not
require verbal responses. It can be administered in five and ten minutes. It is often used to
assess executive functioning in people with neurological disorders such as strokes, dementia
and psychiatric disorders such as schizophrenia.
The correct answer is: Visuospatial sequencing

Question 36 MoK Applied Neuroscience 059
Not answered During extended cognitive assessment by a neuropsychologist, the subject is asked to read a
short story containing 25 elements and the psychologist tested both immediate and delayed
Marked out of 1.00
recall after 30 minutes. The test done by the neuropsychologist is called 
Flag question
Select one:

1096
Halstead ­ Reitan scale
Rivermead behavioural memory test
Wechsler memory scale
Wechsler adult intelligence scale
Rey­Osterrieth test

Your answer is incorrect.
Here the subject is asked to read a short story from the Wechsler Memory scale containing 25
elements and both immediate and delayed recall after an interval of 30 minutes is tested. This
is one of the verbal subtests used in Wechsler memory scale. WMS­3 consists of tests
grouped under two domains; verbal and nonverbal, all of which have immediate and delayed
recall and or recognition components. It also has a third group of tests involve .mental control
subtests'.
The correct answer is: Wechsler memory scale

Question 37 MoK Applied Neuroscience 060
Not answered A psychologist is performing neuropsychological testing in a 45 year old man. Which one of
the following is a test of set shifting?
Marked out of 1.00

Flag question Select one:
Wechsler memory scale
Ravens progressive matrices
Boston naming test
Wisconsin Card sorting test
Clock drawing task

Your answer is incorrect.
Wisconsin Card Sorting test is a neuropsychological test of 'set shifting'. Shifting set refers to
the process of updating or "shifting" cognitive strategies in response to changes in the
environment. For example, in WCST patients are first asked to perform according to some rule
(sort cords by number). Next, patients are asked to switch to a new rule (sort cords by color).
Successful performance requires the ability to abandon an old strategy and start responding
according to a new rule. Patients with frontal lobe damage are often impaired at tasks, which
require set­shifting abilities.
The correct answer is: Wisconsin Card sorting test

Question 38 MoK Applied Neuroscience 061
Not answered A 50­year­old man on a neurology ward has non­dominant parietal lobe dysfunction. Identify
the test that he is least likely to be able to do?
Marked out of 1.00

Flag question Select one:

1097
Reading a sentence
Serial subtraction of 7s from 100
Copying intersecting pentagons
Writing a sentence
Correctly stating current location

Your answer is incorrect.
The cognitive process tested by asking the patient to copy intersecting pentagons is
constructional praxis, which is a feature of non­dominant parietal lobe lesions. The effects of
lesion involving non­dominant parietal lobe include visuospatial dysfunction, topographical
memory loss, and constructional / dressing apraxia. These disorders could be seen in lesions
of either hemisphere but observed more frequently and are of greater severity with lesions of
the non­dominant parietal lobe.
The correct answer is: Copying intersecting pentagons

Question 39 MoK Applied Neuroscience 062
Not answered Mr. Smith was seen by a clinical psychologist at a memory clinic and was noted to have
features of executive dysfunction. Executive function is tested using which of the following
Marked out of 1.00
tests?
Flag question
Select one:
National Adult Reading Test
Raven's Progressive Matrices
Wisconsin Card Sorting Test
Vocabulary Test
Rey­Osterrieth Test

Your answer is incorrect.
Executive function includes planning, initiation, sequencing, coordinating, error detection,
error correction, set shifting, and termination. It is closely allied to other frontal functions such
as judgement, problem­solving, impulse control, and abstract reasoning. These can be tested
formally using Wisconsin Card Sorting Test.
The correct answer is: Wisconsin Card Sorting Test

Question 40 MoK Applied Neuroscience 063
Not answered Non­fluent expressive dysphasia is seen in which of the following conditions

Marked out of 1.00
Select one:
Flag question
Frontotemporal dementia
Lewy body dementia

1098
Progressive supranuclear palsy
Parkinson's disease
Cruetzfeldt Jacob disease

Your answer is incorrect.
Broca's aphasia is also called non­fluent expressive dysphasia. There is reduced frequency
and fluency of speech and presents with few disjointed words. Comprehension is relatively
preserved. It occurs with damage to Broca's area in the left frontal lobe. It is commonly seen in
frontotemporal dementia (Pick's disease).
The correct answer is: Frontotemporal dementia

Question 41 MoK Applied Neuroscience 064
Not answered Digit span test is administered to patient with suspected cognitive impairment. This test
detects problem in which of the following cognitive faculties?
Marked out of 1.00

Flag question Select one:
Semantic memory
Procedural memory
Premorbid IQ
Working memory
Calculation ability

Your answer is incorrect.
Digit span is a test of working memory (immediate), and attention
The correct answer is: Working memory

Question 42 MoK Molecular Genetics 049
Not answered Which of the following can be termed as an intermediate phenotype with high occurrence in
relatives but cannot be classified as a subclinical endophenotype for schizophrenia?
Marked out of 1.00

Flag question Select one:
Impaired attentional performance
Working memory deficits
Eye movement dysfunctions
Impaired p50 suppression
Neurological soft signs

1099
Your answer is incorrect.
Neurological soft signs are observable intermediate phenotypes unlike the rest of the listed
which are endophenotypes.
The correct answer is: Neurological soft signs

Question 43 MoK Molecular Genetics 055
Not answered What is the best test for diagnosing Huntington's disease?

Marked out of 1.00
Select one:
Flag question
CT scan
Serum Protein Assay
Polymerase Chain Reaction
Karyotyping
MRI scan

Your answer is incorrect.
Serum polymerase chain reaction (PCR) is the test of choice to examine the number of
trinucleotide repeats (More than35 in adults and More than50 in children) in order to diagnose
Huntington's disease (HD). The causative gene resides on the short arm of chromosome 4 at
4p16.3. A chromosomal karyotype can reveal only macroscopic defects in chromosomes such
as deletions, translocations, or trisomies.
The correct answer is: Polymerase Chain Reaction

Question 44 MoK Molecular Genetics 061
Not answered The form of Down's syndrome, which is as likely to have come from father as that of mother's
chromosome, is;
Marked out of 1.00

Flag question Select one:
Mosaicism
Duplication
Robertsonian translocation
Deletion
Full trisomy 21

Your answer is incorrect.
In a Robertsonian translocation, two acrocentric chromosomes break at their centromeres.
The resulting long arms fuse to form a single chromosome with a single centromere. The short
arms also join to form a reciprocal product, which typically contains nonessential genes and is

1100
usually lost after a few cell divisions. When chromosome 21 is involved in a Robertsonian
translocation (often with chromosome 14), this may result in Down's syndrome, which is as
likely to have come from the father as that of the mother's chromosome.
The correct answer is: Robertsonian translocation

Question 45 MoK Molecular Genetics 062
Not answered Where does translation take place in human cells?

Marked out of 1.00
Select one:
Flag question
Nucleus
Mitochondria
Smooth endoplasmic reticulum
Rough endoplasmic reticulum
Golgi complex

Your answer is incorrect.
This answer is not precise, but best of the rest. In fact it takes place in ribosomes ­ Rough ER
harbours these ribosomes on its outer surface when translation takes place.
The correct answer is: Rough endoplasmic reticulum

Question 46 MoK Molecular Genetics 063
Not answered Smooth pursuit eye tracking dysfunction is proposed as a strong endophenotype marker for
which of the following group of disorders?
Marked out of 1.00

Flag question Select one:
Personality disorders
OCD spectrum disorders
Mania
Schizophrenia
Anxiety disorders

Your answer is incorrect.
Several studies have shown that eye tracking dysfunction is present in a high percentage of
subjects with schizophrenia and their non­schizophrenic relatives
http://ajp.psychiatryonline.org/cgi/content/full/162/3/466
The correct answer is: Schizophrenia

Question 47 MoK Molecular Genetics 064
Not answered

1101
Marked out of 1.00 Which of the following personality disorders has the strongest support for a genetic
Flag question
contribution?

Select one:
Dependent perosnality
Schizoid disorder
Anankastic disorder
Antisocial disorder
Schizotypal disorder

Your answer is incorrect.
Though schizotypal, antisocial, dependent and avoidant the with familial risk, molecular and
twin study evidences so far strongly implicate genetic aetiology for the schizotypal disorder.
The correct answer is: Schizotypal disorder

Question 48 MoK Molecular Genetics 065
Not answered A child with a deficiency of Hypoxanthine­guanine phosphoribosyltransferase developed
increased acid. Which of the following behavior is consistent with this syndrome?
Marked out of 1.00

Flag question Select one:
Self Injury
Stereotypes
Tangential speech
Visual neglect
Isolated performance IQ defect

Your answer is incorrect.
Lesch­Nyhan syndrome (LNS), is a rare disorder caused by a deficiency of the enzyme
hypoxanthine­guanine phosphoribosyl transferase (HGPRT). Persons affected are cognitively
impaired; the condition is characterized by the overproduction and accumulation of uric acid,
a waste product of normal chemical processes that is found in blood and urine. The
uncontrollable self­injury a usually begins at three years of age. The self­injury begins with
biting of the lips and tongue.
The correct answer is: Self Injury

Question 49 MoK Molecular Genetics 066
Not answered Mutations that occur in body cells may not get transmitted via gametes. These are called 

Marked out of 1.00
Select one:
Flag question
polymorphisms

1102
missense mutations
Somatic mutations
pseudomutations
Nonsense mutations

Your answer is incorrect.
Mutations that occur in somatic cells are somatic mutations and unlike germ cell mutations
these may not be transmitted to next generation.
The correct answer is: Somatic mutations

Question 50 MoK Molecular Genetics 067
Not answered Trinucleotide repeat expansions in DNA can be detected using

Marked out of 1.00
Select one:
Flag question
Northern blot
ELISA
Polymerase chain reaction

Western blot
Southern blot

Your answer is incorrect.
As trinucleotide repeats are DNA abnormalities, the Southern blot can be used to detect them.
The correct answer is: Southern blot

Question 51 MoK Molecular Genetics 068
Not answered ApoE polymorphisms are associated with risk of Alzheimer's disease. Apo E4 alleles are also
implicated in following disorders except
Marked out of 1.00

Flag question Select one:
Sporadic CJD
Parkinson's disease
Intracerebral haemorrhage
Recovery from head injury
Herpes simplex encephalitis

Your answer is incorrect.

1103
Associations between the E4 allele and Alzheimer disease, Parkinson disease, susceptibility
to herpes simplex virus infection, poor recovery from head injury, intracerebral haemorrhage,
and elective cardiac bypass surgery have been shown (Wright et al. Neurogenetics II:
complex disorders, JNNP; 76 (5): 623, http://jnnp.bmj.com/content/76/5/623.full ).
The correct answer is: Sporadic CJD

Question 52 MoK Molecular Genetics 069
Not answered Which of the following statements about epidemiology of Autism is true?

Marked out of 1.00
Select one:
Flag question
It is more common in children who had MMR vaccine
The heritability of this disorder is greater than 90%
The monozygotic twin concordance is more than 95%
It is seen among children of parents from high socioeconomic status
It is more common in girls than boys

Your answer is incorrect.
Twin studies have reported 60% concordance for classic autism in monozygotic twins versus
0 in dizygotic twins. If a broader autistic phenotype (that included communication and social
disorders) is considered, the concordance increases remarkably from 60% to 92% in MZ twins
and from 0 to 10% in DZ twins. This translates to 90% heritability.
The correct answer is: The heritability of this disorder is greater than 90%

Question 53 MoK Neuroanatomy 025
Not answered Which of the following cells can be described as macrophages of the central nervous system?

Marked out of 1.00
Select one:
Flag question
Microglia
Oligodendrocytes
Amacrine cells
Endothelium
Astrocytes

Your answer is incorrect.
Microglia are the macrophages of the brain; they are non­neuronal cells derived from
mesoderm and help in scavenging the CNS.
The correct answer is: Microglia

Question 54 MoK Neuroanatomy 026

1104
Not answered Which of the following arteries supplies Broca's area?
Marked out of 1.00
Select one:
Flag question
Posterior cerebral artery
Anterior cerebral artery
Vertebral artery
Middle cerebral artery
Lenticulostriate artery

Your answer is incorrect.
Broca's area lies in anterior aspect supplied by MCA. Wernicke's area is also supplied by the
middle cerebral artery.
http://www.csuchico.edu/~pmccaffrey//syllabi/CMSD%20320/362unit11.html
The correct answer is: Middle cerebral artery

Question 55 MoK Neuroanatomy 028
Not answered The part of the brain involved in reward processing is

Marked out of 1.00
Select one:
Flag question
Amygdala
Brain stem
Nucleus accumbens
Prefrontal cortex
Hippocampus

Your answer is incorrect.
The amygdala ­ an almond­shaped group of nuclei at the heart of the telencephalon ­ has
been associated with a range of cognitive functions, including emotion, learning, memory,
attention and perception. Most current views of amygdala function emphasize its role in
negative emotions, such as fear, and in linking negative emotions with other aspects of
cognition, such as learning and memory. However, recent evidence supports a role for the
amygdala in processing positive emotions as well as negative ones, including learning about
the beneficial biological value of stimuli. Indeed, the amygdala's role in stimulus­reward
learning might be just as important as its role in processing negative affect and fear
conditioning (Baxter and Murray., The amygdala and reward. Nature Reviews Neuroscience.
(2002). Retrieved from http://www.nature.com/nrn/journal/v3/n7/abs/nrn875.html).
Comparative studies have implicated the nucleus accumbens (NAcc) in the anticipation of
incentives, but the relative responsiveness of this neural substrate during anticipation of
rewards versus punishments remains unclear. At the highest reward level, NAcc activation
was correlated with individual differences in self­reported happiness elicited by the reward
cues. These findings suggest that whereas other striatal areas may code for expected

1105
incentive magnitude, a region in the NAcc codes for expected positive incentive value
(Excerpt from Knutson et al. J Neurosci. 2001 Aug 15;21(16):RC159. Retrieved from
http://www.ncbi.nlm.nih.gov/pubmed/11459880).
The correct answer is: Nucleus accumbens

Question 56 MoK Neuroanatomy 029
Not answered Which of the following is the only cranial nerve without thalamic relay?

Marked out of 1.00
Select one:
Flag question
Trigeminal
Optic
Abducens
Oculomotor
Olfactory

Your answer is incorrect.
The olfactory nerve does not pass through the thalamus, unlike other cranial nerves.
The correct answer is: Olfactory

Question 57 MoK Neurochemistry 026
Not answered The rate limiting step in the synthesis of serotonin is mediated by which of the following
enzymes?
Marked out of 1.00

Flag question Select one:
L­tryptophan hydroxylase
Monoamine oxidase A
Tyrosine Hydroxylase
Aminoacid decarboxylase
Acetyl CO­A Synthetase

Your answer is incorrect.
The biochemical pathway for serotonin synthesis initially involves the conversion of L­
tryptophan to 5­hydroxytryptophan by the enzyme L­tryptophan hydroxylase. This enzyme
provides the rate limiting step in serotonin synthesis, in the same manner that norepinephrine
and dopamine synthesis in adrenergic and dopaminergic neurons is controlled by the ability
of the related enzyme, L­tyrosine hydroxylase, to convert L­tyrosine to L­dihydroxyphenyl­
alanine (L­DOPA).
The correct answer is: L­tryptophan hydroxylase

1106
Question 58 MoK Neurochemistry 028
Not answered In schizophrenia, dopamine deficiency is seen in which of the following brain regions?

Marked out of 1.00
Select one:
Flag question
All of the above
Mesocortical projections
Tuberoinfundibular pathway

Nigrostriatum
Mesolimbic pathway

Your answer is incorrect.
Excess dopamine is postulated as a cause of positive symptoms in the mesolimbic pathway.
The correct answer is: Mesocortical projections

Question 59 MoK Neurochemistry 029
Not answered Which of the following is an agonist at the mu­opioid receptor?

Marked out of 1.00
Select one:
Flag question
Disulfiram
Naltrexone

Buprenorphine
Methadone
Naloxone

Your answer is incorrect.
Buprenorphine is a partial opioid agonist.
The correct answer is: Methadone

Question 60 MoK Neurochemistry 030
Not answered The enzyme involved in degradation of noradrenaline in presynaptic neurons is

Marked out of 1.00
Select one:
Flag question
GABA transaminase
Monoamine oxidase

Homovanillic acid
Vanillyl mandelic acid
3­methoxy, 4­hydroxy, phenylglycol

1107
Your answer is incorrect.
The breakdown enzymes involved in degradation of noradrenaline in presynaptic neurons
are monoamine oxidase (MAO­A) and catechol­o­methyl­transferase (COMT)
The correct answer is: Monoamine oxidase

Question 61 MoK Neurochemistry 031
Not answered Which of the following receptors requires the simultaneous binding of two different agonists
for activation?
Marked out of 1.00

Flag question Select one:
GABA B

Muscarinic M3
GABA A
Mu Opiate
NMDA

Your answer is incorrect.
NMDA receptors require simultaneous binding of two different agonists for their activity.
NMDA ion channels open in response to the presence of both glutamate and glycine (co­
activation molecules).
The correct answer is: NMDA

Question 62 MoK Neuropathology 015
Not answered Which of the following pathological bodies is made up of ubiquitin and alpha synuclein and is
associated with loss of dopaminergic neurons in basal ganglia?
Marked out of 1.00

Flag question Select one:
Hirano bodies
Lewy bodies
Neurofibrillary tangles
Barr bodies
Prion bodies

Your answer is incorrect.
Lewy bodies are seen Parkinsonian disorders and they are associated with a reduction of
dopaminergic neurons (KC 1228)
The correct answer is: Lewy bodies

1108
Question 63 MoK Neuropathology 019
Not answered A 55 year old lady presents with weakness of her legs and hands. She feels clumsy with a
history of falls. On examination she has fasiculations in her tongue and memory deficits. She
Marked out of 1.00
is diagnosed to have amyotrophic lateral sclerosis. The primary pathological change will be
Flag question noted in

Select one:
Motor neurons
Neuromuscular junction
Myocytes
Spinal autonomic ganglion
Sensory neurons

Your answer is incorrect.
ALS is a motor neuron disease where upper motor neurons are specifically damaged leading
o loss of cells in anterior horns of cord and brain stem nuclei.
The correct answer is: Motor neurons

Question 64 MoK Neuropathology 020
Not answered Which of the following accounts for spongiform appearance in CJD?

Marked out of 1.00
Select one:
Flag question
Accumulation of prion proteins
Neuronal vacuolation
Trapped air in parenchymal cells
Neuronal loss
Neurophil loss

Your answer is incorrect.
Microscopically, CJD shows a shows a spongiform encephalopathy, secondary to neutrophil
vacuolisation.
The correct answer is: Neuronal vacuolation

Question 65 MoK Neurophysiology 016
Not answered Miss. X suffers from eating disorder characterized by binge eating, body image disturbances
and measures taken to reduce weight . This condition is associated with reduced levels of
Marked out of 1.00

Flag question Select one:
Dopamine

1109
Glycine
Vasopressin
Leptin
Endorphins

Your answer is incorrect.
Miss X is exhibiting symptoms of bulimia nervosa. Patients with bulimia nervosa as well as
individuals who had recovered from bulimia nervosa had significantly lower serum leptin
levels than healthy controls matched for BMI and percent body fat. The results are consistent
with the hypothesis that decreased leptin function contributes to impaired postingestion
satiety, neuroendocrine abnormalities, and abnormally low resting metabolic rate in bulimia
nervosa. (Ref: (J Clin Endocrinol Metab 85: 4511­4514, 2000) and
http://www.primarypsychiatry.com/aspx/articledetail.aspx?articleid=3532)
The correct answer is: Leptin

Question 66 MoK Neurophysiology 017
Not answered REM sleep disorder is associated with

Marked out of 1.00
Select one:
Flag question
Alzheimer's disease
Fronto temporal dementia
Wilson's disease

Lewy body disease
Huntington's disease

Your answer is incorrect.
REM sleep disorder is associated with disorders such as Parkinson's disease, multi­system
atrophy and Gullian Barre syndrome. RBD may be the prodrome of neurodegenerative
disorders such as DLB and Parkinson's disease.
The correct answer is: Lewy body disease

Question 67 MoK Neurophysiology 018
Not answered A 46­year­old lady suffers from excessive daytime somnolence and acute episodes of
collapse on laughing. What is the most likely risk factor?
Marked out of 1.00

Flag question Select one:
Binge drinking
Low hypocretin (orexin) levels
SSRI drug use

1110
Schizophrenia
Gamma hydroxy butyric acid abuse

Your answer is incorrect.
This is a case of narcolepsy, where low orexin levels can be one of the common risk factors.
Low CSF hypocretin­1 is highly specific (99.1%) and sensitive (88.5%) for narcolepsy with
cataplexy
The correct answer is: Low hypocretin (orexin) levels

Question 68 MoK Pharmacology EMI018
Not answered Adverse effects 
For each of the following drugs, identify the most important side effect associated with its
Marked out of 3.00
therapeutic use
Flag question

Prolactin elevation
Nasal stuffiness
Polydipsia
Galantamine
Blurred vision
Glucose dysregulation
Diastolic Hypertension
Hypersalivation
QTc prolongation
Polydipsia
Weight loss
Prolactin elevation
Amenorrhoea
Glucose dysregulation
Venlafaxine Bradycardia
Blurred vision
Hypersalivation
Diastolic Hypertension
Amenorrhoea
Bradycardia
Bradycardia
Nasal stuffiness
Amenorrhoea
QTc prolongation
Weight loss
Topiramate Weight loss
Prolactin elevation
Diastolic Hypertension
QTc prolongation
Hypersalivation
Glucose dysregulation
Nasal stuffiness
Blurred vision
Your answer is incorrect.
Polydipsia
Antidementia drugs like galantamine can produce bradycardia, especially in those with
supraventricular conduction delay. 
A significant number of patients receiving doses above 300mg/day of venlafaxine experience
an increase in diastolic blood pressure. This risk is not restricted to those with preexisting
hypertension.
Topiramate is a weak inhibitor of renal carbonic anhydrase and can promote the development
of renal stones. It is weight neutral and can even cause weight loss.
The correct answer is: Galantamine – Bradycardia, Venlafaxine – Diastolic Hypertension,

1111
Topiramate – Weight loss

Question 69 MoK Pharmacology EMI020
Not answered Adverse effects 
For each of the adverse effects given below, identify the drug most likely associated with it:
Marked out of 3.00

Flag question
Droperidol
Lamotrigine
Acamprosate
Renal failure
Lithium
SSRIs
Chlorpromazine
Sildenafil
Lorazepam
Sildenafil
Bupropion
Lithium
Carbamazepine
Droperidol
Seizures Mirtazapine
Carbamazepine
Mirtazapine
Lorazepam
Bupropion
Lamotrigine
Lithium
Acamprosate
Lorazepam
Chlorpromazine
Lamotrigine
Nocturnal myoclonus SSRIs
SSRIs
Acamprosate
Sildenafil
Chlorpromazine
Droperidol
Carbamazepine
Mirtazapine
Your answer is incorrect.
Bupropion
Lithium: Renal damage may occur in severe, prolonged toxicity ­ but cumulative lithium use
leads to renal failure more commonly. It takes at least ten years to develop interstitial fibrosis ­
the pathology behind the chronic renal damage. 
SSRIs: Severe sweating especially nocturnally is seen in some patients; Terazosin is effective
in counteracting sweating. Nocturnal myoclonus is reported with SSRIs. The repetitive leg
movements are characterised by extensions of the large toe and flexion of all major leg joints
(ankle, knee, and hips) and occur every 20 to 60 seconds. Benzodiazepines and levodopa
may be tried. 
Bupropion is associated with seizures and exacerbation of eating disorders due its
anorexigenic properties.
The correct answer is: Renal failure – Lithium, Seizures – Bupropion, Nocturnal myoclonus –
SSRIs

Question 70 HiY Neurosciences EMI019
Not answered Cell biology
Using descriptions given below, identify the type of enzymes from the list
Marked out of 3.00

Flag question

1112
RNA Polymerase
pre m­RNA
Used in experiments to decrease
Double stranded DNA
gene expression
Si­ RNA
DNA Polymerase
Single­stranded DNA
Aminoacyl t­RNA synthetase
m­RNA
Double stranded DNA
r­RNA
Si­ RNA
Provides a bridge between m­RNA t­RNA
r­RNA
and amino acids DNA Polymerase
pre m­RNA
RNA Polymerase
m­RNA
Single­stranded DNA
Aminoacyl t­RNA synthetase
Aminoacyl t­RNA synthetase
Single­stranded DNA
t­RNA
Enzymes which attaches amino acids Double stranded DNA
to t­RNA Si­ RNA
r­RNA
pre m­RNA
t­RNA
DNA Polymerase
m­RNA
Your answer is incorrect. RNA Polymerase
Explanation: 
Small interfering RNA (Si­RNA) is a class of double­stranded RNA molecule, involving in RNA
interference pathway, where it interferes with expression of a specific gene.
T­RNA is a small RNA molecule that transfers a specific active aminoacid to growing
polypeptide chain at the ribosomal site of protein synthesis during translation. It also has a
terminal site for amino acid attachment.
Aminoacyl t­RNA synthetase is an enzyme that catalyze the esterification of a specific amino
acid or its precursors to one of all its compatible cognate t­RNA to form an aminoacyl t­RNA.
The correct answer is: Used in experiments to decrease gene expression – Si­ RNA, Provides
a bridge between m­RNA and amino acids – t­RNA, Enzymes which attaches amino acids to
t­RNA – Aminoacyl t­RNA synthetase

Question 71 MoK Pharmacology EMI021
Not answered Selectivity of Tricyclics (TCAs)
Identify the most suitable drug, for the descriptions given below:
Marked out of 3.00

Flag question
Doxepin
Imipramine
This drug is the most serotonergic
Protriptyline
reuptake selective of all tricyclic
Desipramine
antidepressants
Maprotiline
Amitryptyline
Clomipramine
Nortriptyline
Amoxapine

1113
This drug is the most noradrenergic Imipramine
selective of all tricyclic Nortriptyline
antidepressants Amoxapine
Amitryptyline
Clomipramine
Maprotiline
Protriptyline
Doxepin
Amitryptyline
Desipramine
Amoxapine
This drug has the most antihistaminic Desipramine
activity. Doxepin
Maprotiline
Imipramine
Clomipramine
Nortriptyline
Protriptyline
Your answer is incorrect.
Clomipramine more serotonergic reuptake selective; desipramine is the most noradrenergic
selective of TCAs. Amoxapine, nortriptyline, desipramine, and maprotiline (secondary amines)
have the least anticholinergic activity; doxepin has the most antihistaminic activity.
The correct answer is: This drug is the most serotonergic reuptake selective of all tricyclic
antidepressants – Clomipramine, This drug is the most noradrenergic selective of all tricyclic
antidepressants – Desipramine, This drug has the most antihistaminic activity. – Doxepin

Question 72 MoK Pharmacology EMI022
Not answered Receptor mechanisms (1)
Using the definitions below, identify the terminology best suited:
Marked out of 3.00

Flag question
Competitive antagonist
This refers to the amount of the drug Affinity
needed to produce a particular effect Partial agonist
compared to another standard drug Inverse agonist
with similar receptor profile Full agonist
Efficacy
Potency
Non­competitive antagonists
Irreversible Antagonists
Irreversible Antagonists
Inverse agonist
Semi agonist
Efficacy
This refers to the ability of the drug to
bind to its appropriate receptor Full agonist
Affinity
Semi agonist
Non­competitive antagonists
Potency
Partial agonist
Competitive antagonist
This refers to how well the drug
produces the expected response

1114
Inverse agonist
Partial agonist
Non­competitive antagonists
Competitive antagonist
Semi agonist
Affinity
Efficacy
Irreversible Antagonists
Full agonist
Your answer is incorrect. Potency
Potency of a drug with receptor binding action refers to the amount of the drug needed to
produce a particular effect compared to another standard drug with similar receptor profile
('vigor'). 
Affinity refers to the ability of the drug to bind to its appropriate receptor ('affection'). 
Efficacy refers to how well the drug produces the expected response i.e. the maximum clinical
response produced by a drug ('productivity'). 
Haloperidol is more potent than chlorpromazine ­ 5 mg of haloperidol can achieve the same
effect as 100 mg of chlorpromazine. These drugs, however, produce equivalent maximal
clinical response i.e. equally efficacious but not equipotent.
The correct answer is: This refers to the amount of the drug needed to produce a particular
effect compared to another standard drug with similar receptor profile – Potency, This refers to
the ability of the drug to bind to its appropriate receptor – Affinity, This refers to how well the
drug produces the expected response – Efficacy

Question 73 MoK Neurosciences EMI025
Not answered Blood supply to the brain
Match the specific areas of the brain with their blood supply from the list above
Marked out of 4.00

Flag question
Anterior communicating artery
Posterior cerebral artery
Medial cerebral artery
Hypothalamus
Anterior communicating artery
Subclavian artery
Anterior inferior cerebellar artery
Internal auditory artery
Posterior inferior cerebellar artery
Internal auditory artery
Superior cerebellar artery
Anterior cerebral artery
Anterior cerebral artery
Anterior inferior cerebellar artery
Basal ganglia
Posterior inferior cerebellar artery
Anterior communicating artery
Superior cerebellar artery
Posterior cerebral artery
Subclavian artery
Subclavian artery
Medial cerebral artery
Anterior cerebral artery
Anterior limb of internal capsule (2 Anterior inferior cerebellar artery
choices) Internal auditory artery
Posterior cerebral artery
Medial cerebral artery
Anterior communicating artery

1115
Your answer is incorrect.
Anterior communicating and anterior medial arteries supply the hypothalamus. 
The basal ganglia are supplied by the medial cerebral artery. 
Anterior cerebral and anterior communicating arteries supply the anterior limb of the internal
capsule. 

The correct answer is: Hypothalamus – Anterior communicating artery, Basal ganglia – Medial
cerebral artery, Anterior limb of internal capsule (2 choices) – Anterior cerebral artery, Anterior
communicating artery

Question 74 HiY Psychology EMI013
Not answered Memory types
Identify the type of memory which is being referred to, for each of the following situations.
Marked out of 4.00

Flag question
A 45­year­old man was involved in a
Working memory
road traffic accident and was Semantic memory
unconscious for a week. On False memory
examination, he has no memory for Infantile memory
any events for a period of a week Declarative memory
prior to the accident. Procedural memory
Episodic memory
Anterograde memory
Retrograde memory
A 76­year­old man was found
Retrograde memory
Declarative memory
wandering and was brought to the Episodic memory
A&E by the police. On mini­mental False memory
state examination, when you ask him Procedural memory
to remember the three objects, he is Anterograde memory
unable to immediately repeat them Working memory
Semantic memory
Infantile memory
An 83­year­old woman has been
Declarative memory
diagnosed with Alzheimer's disease. Infantile memory
She enjoys playing piano, which she Retrograde memory
used to do for a long time even Episodic memory
before her illness began. Which type Anterograde memory
of memory remains intact? Working memory
Semantic memory
False memory
Retrograde memory
Procedural memory
Declarative memory
Episodic memory
Semantic memory
When answering the question, 'What Anterograde memory
is the capital of France?' What is the Procedural memory
type of memory Mr. P was using? Working memory
False memory
Infantile memory

1116
Your answer is incorrect.
Explanation: Retrograde amnesia involves the loss of memories that were created before the
actual trauma or neurological damage. 
Anterograde amnesia involves an inability to form new memories after the condition
developed. 
The immediate memory test in MMSE is thought to be tapping on to the domain of working
memory. 
Remembering a specific piano lesson is an example of explicit memory, while playing the
piano as a result of this learning is an example of implicit procedural memory.
Semantic memory is the portion of long­term memory which is concerned with ideas,
meanings, and concepts that are not related to personal experiences.
The correct answer is: A 45­year­old man was involved in a road traffic accident and was
unconscious for a week. On examination, he has no memory for any events for a period of a
week prior to the accident. – Retrograde memory, A 76­year­old man was found wandering
and was brought to the A&E by the police. On mini­mental state examination, when you ask
him to remember the three objects, he is unable to immediately repeat them – Working
memory, An 83­year­old woman has been diagnosed with Alzheimer's disease. She enjoys
playing piano, which she used to do for a long time even before her illness began. Which type
of memory remains intact? – Procedural memory, When answering the question, 'What is the
capital of France?' What is the type of memory Mr. P was using? – Semantic memory

Question 75 MoK Neurosciences EMI029
Not answered Neuroimaging tests in clinical practice
Identify the most likely neuroimaging findings for the following neuropsychiatric conditions
Marked out of 4.00

Flag question
A 22­year­old gentleman with Medial temporal atrophy
persecutory delusions and 3rd Lower prefrontal metabolism
person auditory hallucinations. In Ventricular enlargement
addition to reduced frontotemporal
Increased frontal lobe volumes
and insular grey matter, what other
feature can be seen in structural
White matter hyperintensities
imaging? Progressive increase in brain volume in first few years of diagno
Loss of white matter in the dorsolateral prefrontal cortex
White matter hypointensities
Medial temporal atrophy
A 69­year­old woman with recurrent Ventricular enlargement
episodes of low mood, anhedonia Loss of white matter in the dorsolateral prefrontal cortex
and poor sleep since the age of 60.
Increased frontal lobe volumes
What feature can be seen in
structural imaging?
Progressive increase in brain volume in first few years of diagno
Lower prefrontal metabolism
White matter hypointensities
White matter hyperintensities
A 77­year­old man with history of
short­term memory loss and poor
concentration. In addition to
ventricular enlargement, what other
feature can be seen in structural
imaging?

1117
White matter hypointensities
Progressive increase in brain volume in first few years of diagno
Increased frontal lobe volumes
Medial temporal atrophy
Loss of white matter in the dorsolateral prefrontal cortex
Lower prefrontal metabolism
Ventricular enlargement
White matter hyperintensities
White matter hypointensities
White matter hyperintensities
A 33­year­old woman with several Increased frontal lobe volumes
waxing and waning neurological Lower prefrontal metabolism
deficits involving ocular movements Medial temporal atrophy
and sensorimotor system but intact Loss of white matter in the dorsolateral prefrontal cortex
cognitive abilities.
Ventricular enlargement
Progressive increase in brain volume in first few years of diagno

Your answer is incorrect.
Explanation: 
Case 1: The diagnosis is schizophrenia. Neuroimaging findings in schizophrenia include
ventricular enlargement and loss of grey matter ­ especially insula, anterior cingulate cortex,
medial temporal lobe and dorsolateral pre­frontal cortex with some progressive loss of brain
volume in the first few years of diagnosis. fMRI shows poor dorsolateral pre­frontal cortex
activation in executive tasks, while decreased N­acetyl aspartate in prefrontal cortex is seen in
spectroscopy. 
Case 2: The diagnosis is depressive disorder (geriatric depression). Neuroimaging findings
include periventricular and deep WM hyperintensities, subcortical thalamic and striatal
hyperintensities, decreased frontal and basal ganglia volumes, and decreased metabolism in
prefrontal cortex. 
Case 3: The diagnosis is Alzheimer's dementia. Neuroimaging findings in Alzheimer's
disease include ventricular enlargement, reduced medial temporal lobe volume ­ especially
hippocampus and a reduction in parieto­temporal fMRI activation and SPECT blood flow.
Case 4: The diagnosis is Multiple Sclerosis. Neuroimaging findings in this disease include
features of demyelination, especially white matter hypointensities affecting corpus callosum. If
fluid­attenuated inversion recovery (FLAIR) MRI is used,  periventricular high­signal intensity
lesions can be observed.
The correct answer is: A 22­year­old gentleman with persecutory delusions and 3rd person
auditory hallucinations. In addition to reduced frontotemporal and insular grey matter, what
other feature can be seen in structural imaging?
– Ventricular enlargement, A 69­year­old woman with recurrent episodes of low mood,
anhedonia and poor sleep since the age of 60. What feature can be seen in structural
imaging? – White matter hyperintensities, A 77­year­old man with history of short­term
memory loss and poor concentration. In addition to ventricular enlargement, what other
feature can be seen in structural imaging? – Medial temporal atrophy, A 33­year­old woman
with several waxing and waning neurological deficits involving ocular movements and
sensorimotor system but intact cognitive abilities.
– White matter hypointensities

1118
Question 76 MoK Neurosciences EMI030
Not answered Neuroanatomical lesions in CNS disorders 
For each of the condition below choose the most specific option/s from given list
Marked out of 4.00

Flag question
Supramarginal gyrus
Hypothalamus
Inferior frontal gyrus
Korsakoff's psychosis
Third ventricle
Occipitotemporal junction
Dorsolateral prefrontal cortex
Dorsomedial thalamus
Hippocampus
Inferior frontal gyrus
Corpus callosum
Hypothalamus
Dorsomedial thalamus
Gerstmann syndrome
Occipitotemporal junction
Dorsolateral prefrontal cortex
Hippocampus
Third ventricle
Corpus callosum
Hypothalamus
Supramarginal gyrus
Supramarginal gyrus
Inferior frontal gyrus
Prosopagnosia
Dorsolateral prefrontal cortex
Dorsomedial thalamus
Occipitotemporal junction
Third ventricle
Hippocampus
Occipitotemporal junction
Corpus callosum
Hippocampus
Hypothalamus
Alzheimer's disease
Supramarginal gyrus
Inferior frontal gyrus
Corpus callosum
Dorsolateral prefrontal cortex
Third ventricle
Dorsomedial thalamus
Your answer is incorrect.
Explanation: 
About 80 per cent of alcoholic patients recovering from Wernicke's encephalopathy develop
Korsakoff's amnesic syndrome. Diagnosis of Korsakoff's syndrome correlates best with the
presence of lesions in the dorsomedial thalamus. But lesions in the mammillary bodies, the
mammillothalamic tract and the anterior thalamus may be more relevant to memory
dysfunction than lesions in the medial dorsal nucleus of the thalamus (Kopelamn et al., 2009).
Gerstmann syndrome is characterized by four primary symptoms: Dysgraphia/agraphia,
Dyscalculia/acalculia, Finger agnosia and Left­right disorientation. This is often associated
with brain lesions in the dominant (usually left) side of the angular and supramarginal gyri
(parietal lobe). 
Prosopagnosics cannot recognise familiar faces. Face processing is a bilateral function; more
vital areas may be present on the right hemisphere. Acquired prosopagnosia is usually
associated with bilateral or right­sided lesions of the occipital ­ temporal junction (fusiform
gyrus). 

1119
In Alzheimer's disease, the main areas affected are the hippocampus and medial temporal
lobe. Atrophic changes are mainly observed in these two areas in even in very early stages of
Alzheimer's disease.

Barton, JJ., (2008). Prosopagnosia associated with a left occipitotemporal lesion. Excerpt
retrieved from http://www.ncbi.nlm.nih.gov/pubmed/18374372 
Kopeman et al., (2009). The Korsakoff Syndrome: Clinical Aspects, Psychology and
Treatment. Excerpt retrieved from http://alcalc.oxfordjournals.org/content/44/2/148
The correct answer is: Korsakoff's psychosis
– Dorsomedial thalamus, Gerstmann syndrome – Supramarginal gyrus, Prosopagnosia –
Occipitotemporal junction, Alzheimer's disease
– Hippocampus

Question 77 MoK Neurosciences EMI031
Not answered Neurohistology 
Match the following neuroglial cells with their specific functions from the list provided:
Marked out of 2.00

Flag question
Neuronal support in sensory and autonomic ganglia
Neurilemma formation
Covering the ventricles
Astrocytes 
Act as scavenger cells at sites of CNS injury
CNS myelin sheath formation
Nutrition of neuronal cells
Peripheral myelin sheath formation
Apoptotic clearance
Neurilemma formation
Peripheral myelin sheath formation
Neuronal support in sensory and autonomic ganglia
Ependymal cells
Nutrition of neuronal cells
Covering the ventricles
Act as scavenger cells at sites of CNS injury
Apoptotic clearance
CNS myelin sheath formation

Your answer is incorrect.
Explanation: Astrocytes are the most numerous of all three types of glial cells. These are star­
shaped cells that enable nutrition of neurons, breakdown of some neurotransmitters, and
maintaining the blood­brain barrier. 
Oligodendrocytes are seen in CNS (not in peripheral nerves, where Schwann cells replace
them). They produce myelin sheaths that help in the saltatory conduction of an action potential
(pole to pole jumping), which quicken the process of signal transmission. 
The microglia are descendants of macrophages. They are scavenger cells that clear neuronal
debris following cell death. 
Ependymal cells are a particular type of glia that cover the ventricles and enable CSF
circulation. 
Schwann cells are seen in the peripheral nervous system where they produce myelin
sheaths.
The correct answer is: Astrocytes 
– Nutrition of neuronal cells, Ependymal cells – Covering the ventricles

1120
Finish review

1121
 Home Mock Paper Practice Tests

Mock Paper A(1)

Started on Tuesday, 30 June 2015, 2:45 AM
State Finished
Completed on Tuesday, 30 June 2015, 2:46 AM
Time taken 22 secs
Grade 0.00 out of 100.00

Question 1 MoK Classification 001
Not answered How many episodes of mania and/or depression should be seen in 12 months time to
diagnose rapid cycling bipolar disorder?
Marked out of 1.00

Flag question Select one:
3
4
2

6
8

Rapid cycling bipolar disorder is defined in the DSM­5 as a type of bipolar illness in which the
patient experiences four or more episodes of mania and/or major depression per year ( Rapid
cycling bipolar disorder, http://www.ncbi.nlm.nih.gov/pubmed/11825328).
The correct answer is: 4

Question 2 MoK Classification 002
Not answered Axis V of DSM­IV recorded Global Assessment of Functioning. What is the maximum score in
the GAF scale?
Marked out of 1.00

Flag question Select one:
7
100

10
60
80

1122
The GAF is a scale used by clinicians to describe a patient's level of functioning in three
areas: social (i.e., marriage, friends), occupational, and psychological. The GAF scale ranges
from 100 (highest functioning) to 0. The GAF is recorded on Axis V.
The correct answer is: 100

Question 3 MoK Classification 003
Not answered The term simple schizophrenia was added as to the categories of existing schizophrenias by
Marked out of 1.00
Select one:
Flag question
Bleuler
Hubbard
Hecker
Angst
Liddle

The diagnostic category called simple schizophrenia has a long history; it was one of the
traditional schizophrenic subtypes identified by Bleuler and was later accepted by Kraepelin.
The term has been removed from the official American nosology but has been retained in ICD
system.
The correct answer is: Bleuler

Question 4 MoK Classification 004
Not answered Which of the following is correct with regard to a diagnosis of post schizophrenic depression?

Marked out of 1.00
Select one:
Flag question
The patient must not have a previous history of depression
The patient must not be on any antipsychotic medication
Some positive symptoms must be present along with depression
It can be diagnosed only 12 months after the last psychotic episode
It is classified along with psychotic disorders

Post­schizophrenic depression forms F20.4 ­ it is classified as a type of schizophrenia along
with other psychotic disorders in ICD10. While the general criteria for schizophrenia must
have been met within the past twelve months, they must not be met in full at present. But one
of either positive/negative symptoms/formal thought disorder/catatonia symptoms must be
present. The depressive symptoms must be sufficiently prolonged, severe and extensive to
meet criteria for at least a mild depressive episode (F32.0). The patient may or may not be on
treatment with antipsychotics or antidepressants. Previous history of depression does not
preclude a diagnosis
The correct answer is: It is classified along with psychotic disorders

1123
Question 5 MoK Classification 006
Not answered Which of the following features seen in a person suffering from grief reaction 1 year after the
loss suggests depression?
Marked out of 1.00

Flag question Select one:
Mummification
Guilt regarding omission of care
Physical health problems

Preoccupation with the relative
Anniversary reaction

Signs of difficulty in assimilating the loss vary from the slight (e.g., expecting the person to
return home from work and setting two places for dinner) to the extreme of mummification
(e.g., retaining a child's room exactly as it was on the day of death for years). These signs may
be harbingers of depression if they persist for more than one year. Mummification was first
described by Gorer in 1965.
The correct answer is: Mummification

Question 6 MoK Classification 008
Not answered In order to make a ICD­10 diagnosis of delusional disorder, the delusions must be present for
at least
Marked out of 1.00

Flag question Select one:
3 days
1 month
6 months
2 weeks
3 months

Persistent Delusional Disorder includes a variety of disorders in which long­standing (more
than three months) delusions constitute the only, or the most conspicuous, clinical
characteristic. Delusional disorders that have lasted for less than three months should be
classified, at least temporarily, under schizophreniform illnesses.
The correct answer is: 3 months

Question 7 MoK Classification 010
Not answered The most common co­morbid psychiatric condition associated with panic disorder is

Marked out of 1.00
Select one:
Flag question
Depression

1124
Schizophrenia
Agoraphobia
Borderline personality disorder
Social phobia

Agoraphobia is the most frequent comorbid diagnosis, with 30 ­ 60% of panic disorder
patients satisfying the diagnostic criteria for agoraphobia.
The correct answer is: Agoraphobia

Question 8 MoK Classification 011
Not answered A person drinks alcohol as soon as he wakes up in the morning. During working hours, he
again drinks in the lunch time and after getting home he needs to have at least 6 units of
Marked out of 1.00
alcohol. His behaviour is best described as
Flag question
Select one:
Compulsive behaviour
Alcohol dependence
Rumination

Impulsivity
Obsessions

Compulsive behaviours seen in addictions are associated with 'pleasure seeking' i.e. seeking
the drug to obtain 'chemical' pleasure. This is typical of drug dependence states.
The correct answer is: Alcohol dependence

Question 9 MoK Classification 013
Not answered What is least likely with regard to night terrors?

Marked out of 1.00
Select one:
Flag question
Associated with organised automatic behaviours
Amnesia of the episode
Occurs in deep slow wave sleep

Often seen during early nocturnal sleep
Increased autonomic arousal is noted during the episode

It is rarely associated with complex motor activities.
The correct answer is: Associated with organised automatic behaviours

1125
Question 10 MoK Classification 014
Not answered Which of the following clinical features suggests a diagnosis of social phobia rather than
agoraphobia?
Marked out of 1.00

Flag question Select one:
Being house bound
Having a fear of getting into situations of no escape
Having a fear of being under continuous scrutiny from others
Avoiding the use of public transport
Avoiding crowded places

Both social phobia and agoraphobia can present with house­bound behaviour and avoiding
public places. But in patients with social phobia the underlying fear is that of embarrassing
oneself by 'doing something stupid' while others are watching them.
The correct answer is: Having a fear of being under continuous scrutiny from others

Question 11 MoK Classification 015
Not answered Marie presents with a first episode of depression. She is started on an SSRI. Seven days later,
she is found to be overactive, easily distractible, irritable and sleepless, which lasts for about 5
Marked out of 1.00
days. The symptoms resolve when the SSRI was stopped. Most appropriate diagnosis among
Flag question the following list is

Select one:
Unipolar mania
Bipolar type 2 disorder
Mixed affective disorder
Bipolar type 1 disorder
Substance induced mood disorder

SSRI­induced mood switch or destabilisation is not a recognised classification in ICD or DSM.
But as this lady has had a period of hypomania and past history of depression, she can be
diagnosed with bipolar disorder type 2.
The correct answer is: Bipolar type 2 disorder

Question 12 MoK Classification 016
Not answered A 23­year­old woman is referred by her GP. She states that her eyes are set wide apart and
people call her 'fish­face'. She spends hours in front of the mirror examining her eyes. She has
Marked out of 1.00
seen surgeons twice to get plastic surgery done to her eye folds. However there is no obvious
Flag question deformity. The most appropriate diagnosis is

1126
Select one:
Body dysmorphic disorder
Social phobia
Panic disorder

Panic disorder
Generalized anxiety disorder

In body dysmorphic disorder, there is an excessive concern (overvalued idea) about trivial or
non­existent physical abnormalities, which are perceived to be deformities
The correct answer is: Body dysmorphic disorder

Question 13 MoK Classification 017
Not answered A 36­year­old woman complains of severe tiredness not relieved by rest for over 6 months
and is unable to work. There is no identifiable organic pathology and she denies any pain in
Marked out of 1.00
her body. The most appropriate diagnosis is
Flag question
Select one:
Fibromyalgia
Generalized anxiety disorder
Chronic Fatigue Syndrome
Social phobia
Depression

Chronic fatigue syndrome is characterized by persistent or relapsing unexplained chronic
fatigue for at least 6 months, is of new or definite onset, is not the result of an organic disease
or of continuing exertion and is not alleviated by rest. Fatigue results in a substantial reduction
in previous occupational, educational, social, and personal activities. Four or more of the
following symptoms, concurrently present for more than 6 months: impaired memory or
concentration, sore throat, tender cervical or axillary lymph nodes, muscle pain, pain in
several joints, new headaches, unrefreshing sleep, or malaise after exertion
The correct answer is: Chronic Fatigue Syndrome

Question 14 MoK Classification 018
Not answered Unstable affect and impulsive behaviour is characterised by which of the following personality
disorders, according to DSM­5 Criteria?
Marked out of 1.00

Flag question Select one:
Cluster E
Cluster D

Cluster C

1127
Cluster A
Cluster B

There are four Cluster B personality disorders: antisocial, borderline, narcissistic, and
histrionic. The DSM  views these as a subset of personality disorders that are characterized by
dramatic, emotional or erratic behavior.
The correct answer is: Cluster B

Question 15 MoK Classification 019
Not answered A 23­year­old lady has a history of olfactory hallucinations along with flushing. The most likely
diagnosis is
Marked out of 1.00

Flag question Select one:
Stuporous excitement
Delirium
Frontal partial seizures
Complex partial seizures
Occipital seizures

Wilder Penfield reported that fewer than 1% of patients with partial­onset seizures
experienced olfactory symptoms. The majority of patients with epilepsy and olfactory auras
will have clear complex partial or secondary generalized seizures in addition to their sensory
symptoms although these episodes may occur at separate times.
The correct answer is: Complex partial seizures

Question 16 MoK Clinical Examination 001
Not answered Which of the following enzymes mediate alcohol related brain injury?
Marked out of 1.00
Select one:
Flag question
Alcohol dehydrogenase
Lactate dehydrogenase
Transketolase
Aldehyde dehydrogenase
Alpha ketoglutarate

Thiamine (vitamin B1) in its active form, thiamin pyrophosphate (TPP), is a co­enzyme for
several enzymes, including transketolase. Transketolase is an important enzyme in the
pentose phosphate pathway (PPP), essential for energy transduction and for generating
ribose for nucleic acid synthesis. Transketolase also links the PPP to glycolysis, allowing a

1128
cell to adapt to a variety of energy needs, depending on its environment. Abnormal
transketolase expression and/or activity have been implicated in a number of diseases where
thiamin availability is low, including Wernicke­Korsakoff's Syndrome and other forms of
alcohol­related brain injuries. Alexander­Kaufman, K. and Harper, C.(2009): The International
Journal of Biochemistry and Cell Biology 41, 717­720.
The correct answer is: Transketolase

Question 17 MoK Clinical Examination 002
Not answered Length of time cannabis can be detected in the urine of a person who has used it only once is
Marked out of 1.00
Select one:
Flag question
3 months
24 hours
1 month
7 days
3 days

With regular use, it may be present for weeks.
The correct answer is: 3 days

Question 18 MoK Clinical Examination 003
Not answered Which of the following reflexes may NOT be exaggerated in upper motor neuron lesions?
Marked out of 1.00
Select one:
Flag question
Abdominal reflex
Ankle jerk reflex
Biceps reflex
Knee jerk reflex
Jaw jerk reflex

Deep tendon reflexes are exaggerated. Abdominal reflex is not a deep tendon reflex; it is a
superficial reflex.
The correct answer is: Abdominal reflex

Question 19 MoK Clinical Examination 004
Not answered Which of the following is the most important discriminating feature between mild cognitive
impairment and dementia?
Marked out of 1.00

Flag question Select one:

1129
Educational attainment
Duration of symptoms of memory disturbances
Family history of Alzheimer's
Activities of daily living
MMSE score

Mild Cognitive Impairment (MCI) is a borderline state between age­associated cognitive
decline and mild dementia. MCI is separated from mild dementia by an absence of global
intellectual deterioration and the preservation of activities of daily living (ADL).
The correct answer is: Activities of daily living

Question 20 MoK Clinical Examination 005
Not answered Which one of the following is best suited for an assessment of global cognitive functioning?

Marked out of 1.00
Select one:
Flag question
Mini­COG
Clock drawing test
MMSE
Montreal cognitive assessment guide
Addenbrookes cognitive examination

Addenbrookes cognitive examination is a cognitive screening instrument used in the
assessment of global cognitive functioning. It tests various cognitive domains such as
Orientation, Attention, Memory, Verbal fluency, and Language and Visuospatial ability. It
incorporates the questions on the MMSE and expands on the domains of memory, language
and visuospatial concepts and adds tests of verbal fluency.
The correct answer is: Addenbrookes cognitive examination

Question 21 MoK Clinical Examination 006
Not answered A woman presents to A&E with the following features: weight loss, fatigue, hyperkalaemia and
hyponatraemia. What is the most likely diagnosis?
Marked out of 1.00

Flag question Select one:
Addison's disease
Hypothyroidism
Cushing's disease
Diabetes insipidus
Diabetes mellitus

1130
Hyperkalaemia and hyponatraemia are seen in people with Addison's disease. The most
common symptoms are fatigue, muscle weakness, fever, weight loss, darkening
(hyperpigmentation) of the skin, including areas not exposed to the sun and orthostatic
hypotension.
The correct answer is: Addison's disease

Question 22 MoK Clinical Examination 007
Not answered Mr. Y has a history of alcohol dependence. He is admitted to a medical ward as he developed
memory difficulties following Wernicke's encephalopathy. The bedside test most likely to be
Marked out of 1.00
impaired is
Flag question
Select one:
Digit span
Address test
Months of the year backwards
Spelling world backwards
Serial sevens

Korsakoff syndrome is a preventable memory disorder that usually emerges (although not
always) in the aftermath of an episode of Wernicke's encephalopathy. Episodic memory is
severely affected in the Korsakoff syndrome, and the learning of new semantic memories is
variably affected. 'Implicit' aspects of memory are preserved.
The correct answer is: Address test

Question 23 MoK Clinical Examination 008
Not answered On ocular examination of a 74­year­old patient, constriction of pupils is noted on looking at
near objects but no reaction to light. This is called
Marked out of 1.00

Flag question Select one:
Senile pupil
Adie's pupil
Anisocoria
Argyll Robertson Pupil
Marcus Gunn pupil

Argyll Robertson Pupil (ARP): The pupils are bilaterally small and reduce in size when the
patient focuses on a near object (i.e. Accommodation Reflexes Present ­ ARP again!) but do
not constrict when exposed to bright light (i.e. pupillary reflexes absent). They are seen in
patients with neurosyphilis (hence the name "Prostitute's Pupil") and diabetic neuropathy.
The correct answer is: Argyll Robertson Pupil

1131
Question 24 MoK Descriptive Psychopathology 001
Not answered Which of the following syndromes is named after the person who initially described them?

Marked out of 1.00
Select one:
Flag question
Othello syndrome
Fregoli syndrome
Munchaussen syndrome
Rabbit syndrome
Cotard's syndrome

Cotard syndrome is named after Jules Cotard (1840­1889), a French neurologist who first
described the condition, which he called le delire de negation.
The correct answer is: Cotard's syndrome

Question 25 MoK Descriptive Psychopathology 002
Not answered You are interviewing a newly admitted patient in your ward. When asked what he had for
breakfast, he says 'cereals'. Then when you asked 'what is your name?' he says 'cereals'.
Marked out of 1.00
When you asked 'what is his date of birth?' he continues to say 'cereals'. The most likely
Flag question symptom exhibited here is

Select one:
Mannerism
Automatism
Perseveration
Stuttering
Stereotypy

Perseveration is the uncontrolled repetition or continuation of a response (e.g., behavior,
word, thought, activity, strategy, or emotion) in the absence of an ongoing occasion or
rationale for that behavior or emotion (e.g., the topic or task requirements have changed).
The correct answer is: Perseveration

Question 26 MoK Descriptive Psychopathology 003
Not answered Which of the following is not a feature of Kubler­Ross model of grief?

Marked out of 1.00
Select one:
Flag question
Agitation
Bargaining

Denial

1132
Depression
Acceptance

The Kubler­Ross model, commonly referred to as the "five stages of grief", is an explanatory
model for grief reaction introduced by Elisabeth Kubler­Ross. When a person is faced with the
reality of impending death or other extreme, awful fate, he or she will experience a series of
emotional stages. This sequence can be remembered using the acronym DABDA, which
include: denial, anger, bargaining, depression, and acceptance.
The correct answer is: Agitation

Question 27 MoK Descriptive Psychopathology 004
Not answered Inability to verbalize one's strong emotions is termed as

Marked out of 1.00
Select one:
Flag question
Anhedonia
Avolition
Apathy
Ambivalence
Alexithymia

Alexithymia refers to difficulties in emotionalizing, fantasizing, verbalizing, identifying, and
analyzing emotions.
The correct answer is: Alexithymia

Question 28 MoK Descriptive Psychopathology 005
Not answered Which of the following may indicate poor response to psychotherapeutic interventions?
Marked out of 1.00
Select one:
Flag question
alexithymia
insomnia
apraxia
anhedonia
amnesia

Alexithymia is associated with poor outcome in both traditional psychodynamic psychotherapy
and supportive therapy. This negative effect is found in individual and group psychotherapies.
In the context of group therapy, higher levels of alexithymic features elicit negative reactions
from one's therapist, which partially contribute to the poor outcome experienced by such

1133
patients. This negative reaction of therapists towards alexithymic patients appears to be in
response to the lack of positive emotional expression. Ref:
http://www.ncbi.nlm.nih.gov/pubmed/20471096
The correct answer is: alexithymia

Question 29 MoK Descriptive Psychopathology 007
Not answered A patient with schizophrenia protrudes his tongue repeatedly in and out when asked to do it
once. He is exhibiting which of the following symptoms?
Marked out of 1.00

Flag question Select one:
Chorea
Stereotypy
Ambivalence
Hemiballism
Ambitendence

Ambitendency can be regarded as a mild variety of negativism. The patient appears to be in
conflict regarding moving their body. The presence of opposing tendencies to action is
described as 'jack in the box' feature of catatonia.
The correct answer is: Ambitendence

Question 30 MoK Descriptive Psychopathology 009
Not answered Ben is worried that he has to shout 'Willy Wonka' whenever he sees an attractive woman,
despite his best efforts to keep calm. This is best termed as
Marked out of 1.00

Flag question Select one:
A first rank symptom of schizophrenia
A mental compulsion
An obsessional impulse
Somatic passivity
A made act

Obsessions can present as thoughts, images, impulses or 'sticky words'. In the above
scenario, obsessions are presenting as impulses i.e. the rush to do something.
The correct answer is: An obsessional impulse

Question 31 MoK Descriptive Psychopathology 010
Not answered Which one of the following terms describes fantasy thinking?
Marked out of 1.00
Select one:

1134
Flag question
Autistic thinking
Conceptualism
Rationalism
Concrete thinking
Magical thinking

Normal thinking is of three functional types: a) Fantasy/dereistic thinking or autistic thinking:
There is no goal direction, unrealistic like daydreaming. Predominant in cluster A personality,
dissociation and pseudologia fantastica. b). Imaginative thinking: Again fantasy elements but
admixed with memory, involving abstract concepts but goal­directed and does not cross
boundaries of possibility and realism. Determining tendency of thoughts is preserved e.g.
creative lateral thinking. c). Rational or conceptual thinking: based on material reality and
uses logic.
The correct answer is: Autistic thinking

Question 32 MoK Descriptive Psychopathology 011
Not answered Mr Y has a cardiac arrest. He is in the resuscitation room and experiences seeing himself
outside of his body. This experience is called
Marked out of 1.00

Flag question Select one:
Somatic hallucination
Lilliputian hallucination
Reflex hallucination
Autoscopic hallucination
Extracampine hallucination

Autoscopic hallucination is the visual experience of seeing oneself projected external to the
body space. Males predominate in a 2:1 ratio; impaired consciousness is a common
accompaniment and depression is the commonest psychiatric cause. They are also called
phantom mirror images and may take the form of pseudohallucinations. Schizophrenia, TLE,
parietal lesions are also implicated. In negative autoscopy, one looks into a mirror and sees
no image at all.
The correct answer is: Autoscopic hallucination

Question 33 MoK Descriptive Psychopathology 012
Not answered You note tonic spasm in a woman who cannot resist closing her right eye while talking to you.
She is able to carry out most of her daily activities including watching television normally. This
Marked out of 1.00
is best described as
Flag question
Select one:
Tonic seizures

1135
Blepharospasm
Normal phenomenon
Catatonic posturing
Multiple sclerosi

Blepharospasm is often idiopathic and benign.
The correct answer is: Blepharospasm

Question 34 MoK Descriptive Psychopathology 013
Not answered On listening to a joke shared by a friend, a woman loses her muscle tone and collapses
suddenly. This is described as
Marked out of 1.00

Flag question Select one:
Drop attacks
Cataplexy
Syncopy
Catatonia
Catalepsy

In a state of cataplexy, an intense emotional state triggers objective transient muscle
weakness (often with areflexia). The clinical manifestations are varied, ranging from
involuntary eye closure and neck weakness to a subtle buckling of the knees to generalized
muscle weakness that causes the patient to collapse. Consciousness and awareness of the
environment are preserved throughout the episode.
The correct answer is: Cataplexy

Question 35 MoK Descriptive Psychopathology 014
Not answered A patient with schizophrenia is observed in the waiting room with various semi­purposive
movements, with an inability to change the movements at will or make them goal­directed.
Marked out of 1.00
This is best described as
Flag question
Select one:
Mannerism
Agitation
Restlenessness
Catatonic impulsiveness
Akathisia

1136
Catatonia often presents with immobility, stupor, posturing, rigidity, staring, grimacing, and
withdrawal, The alternative presentation of catatonia (described here) is an excited state
(impulsivity and associated agitation), along with combativeness, and autonomic instability.
The correct answer is: Catatonic impulsiveness

Question 36 MoK Descriptive Psychopathology 015
Not answered The following is the most common obsession in OCD:
Marked out of 1.00
Select one:
Flag question
Rumination
Fear of contamination
Symmetry
Checking
Magical thinking

The most common obsession is fear of contamination, followed by pathological doubt, a need
for symmetry, and aggressive obsessions. The most common compulsion is checking, which
is followed by washing, symmetry, the need to ask or confess, and counting. Swedo SE, et al.
Obsessive­Compulsive Disorder in children and adolescents: Clinical phenomenology of 70
consecutive cases. Arch Gen Psychiatry 1989;46:335­41
The correct answer is: Fear of contamination

Question 37 MoK Descriptive Psychopathology 016
Not answered When asked what he did for a living, a patient stated that he worked at a shop selling clothes.
He went on to say "I don't like the messages on t­shirts. I wish I was a sailor". He is exhibiting
Marked out of 1.00

Flag question Select one:
Circumstantial speech
Tangential speech
Stilted speech
Pressured speech
Clang associations

The presence of excessive unwanted details suggests circumstantial speech, though the
patient responds to the question appropriately.
The correct answer is: Circumstantial speech

Question 38 MoK Basic Psychology 001
Not answered With regard to the psychology of attention, all of the following are correct except

1137
Marked out of 1.00 Select one:
Flag question Focused attention is modality specific
Sustained attention refers to vigilance
Divided attention is the ability to attend to multiple task demands
Alternating attention is the highest form of attention
Shadowing is a test for selective attention

Sohlberg and Mateer proposed a hierarchic model based on the recovering of attention
processes of brain­damaged patients when coming out of a coma. 1. Focused attention: The
ability to respond discretely to specific visual, auditory or tactile stimuli. This is the most
primitive form of attention 2. Sustained attention (vigilance): The ability to maintain a
consistent behavioral response during continuous and repetitive activity. 3. Selective
attention: The ability to maintain a behavioral or cognitive set in the face of distracting or
competing stimuli. Shadowing is a test for selective attention in which simultaneously two
different inputs are provided, one to each ear, and the subject is required to 'shadow' or follow
one of the inputs. 4. Alternating attention: The ability of mental flexibility that allows individuals
to shift their focus of attention and move between tasks having different cognitive
requirements. 5. Divided attention: This is the highest level of attention, and it refers to the
ability to respond simultaneously to multiple tasks or multiple task demands. In dual­task
technique which tests divided attention, the subject is asked to attend to both inputs in an
attempt to divide attention. Thus the ability to attend to multiple task demands is tested.
The correct answer is: Alternating attention is the highest form of attention

Question 39 MoK Basic Psychology 002
Not answered Cognitive theory suggests the following as the underlying factor for anxiety disorders:

Marked out of 1.00
Select one:
Flag question
Somatisation
Catastrophisation

Nihilism
Depersonalisation
Personalisation

The term catastrophizing was used by Albert Ellis, the founder of rational­emotional therapy,
almost four decades ago. Catastrophizing is related to anxiety (e.g., anxiety is associated with
the tendency to overemphasize the probability of a catastrophic outcome and the possible
consequences of such an outcome). During the past decade, cognitive models of panic
disorder have emphasized the role of catastrophizing in panic attacks (Retrieved from
http://journals.lww.com/clinicalpain/Fulltext/2001/03000)
The correct answer is: Catastrophisation

Question 40 MoK Basic Psychology 003

1138
Not answered Visual or iconic stimuli in immediate memory lasts for
Marked out of 1.00
Select one:
Flag question
20­30 minutes
0.5 seconds
2­3 seconds
20­30 seconds
0.05 seconds

Visual or iconic­ 0.5 seconds; Auditory or echoic ­2­3 seconds.
The correct answer is: 0.5 seconds

Question 41 MoK Basic Psychology 006
Not answered Which one of the following is an example of non homeostatic motives?

Marked out of 1.00
Select one:
Flag question
All of the above
Temperature control

Achievement
Hunger
Thirst

There are two types of motivating forces; homeostatic and non homeostatic. Homeostatic
motives arise from physiological needs such as thirst, hunger and temperature control and are
concerned with basic survival. Non homeostatic motives refer to psychological drives such as
curiosity, adventure, achievement, etc.
The correct answer is: Achievement

Question 42 MoK Basic Psychology 007
Not answered Perceptual constancy can be demonstrated for

Marked out of 1.00
Select one:
Flag question
All of the above
Size
Brightness constancies
Colour
Location

1139
Perceptual constancy: It refers to the ability to perceive the appearance of objects as constant,
even though, their retinal images keep changing. It includes several components: size, shape,
colour, location and brightness constancies.
The correct answer is: All of the above

Question 43 MoK Basic Psychology 012
Not answered One of the local newspaper reports that drug addicts carry out muggings. An 83­year­old lady
was mugged and the public concluded the attacker was a drug addict. Select the cognitive
Marked out of 1.00
heuristic in this case.
Flag question
Select one:
Framing
Availability
Simulation
Representation
Anchoring

Availability­ a heuristic for judging frequency and probability. Higgins and King (1981)
described this phenomenon as a readiness with which a particular schema is used in
information processing.
The correct answer is: Availability

Question 44 MoK Basic Psychology 013
Not answered In a program aimed at cessation of smoking, a 45­year­old woman is required to smoke many
cigarettes in a small booth and over a short period of time. What is the process that is
Marked out of 1.00
employed in this approach?
Flag question
Select one:
Aversive conditioning
Stimulus control
Systematic desensitization
Variable ratio reinforcement
Habituation

The smoker is being exposed to an aversive stimulus. The physiologic and psychological
trauma is so severe and so aversive that the patient may choose to avoid smoking in the
future.
The correct answer is: Aversive conditioning

1140
Question 45 MoK Basic Psychology 014
Not answered A rat in the cage initially got electric shocks whenever goes to a particular area of the cage.
Now it stays in electric shock free area. What is the learning process involved?
Marked out of 1.00

Flag question Select one:
Avoidance conditioning
Classical conditioning
Reciprocal inhibition
Shaping
Higher order conditioning

Avoidance learning is a type of learning where an organism learns to avoid certain responses
or situations. In avoidance conditioning, the response or act prevents the aversive stimulus
from occurring. For example, people with posttraumatic stress after a road traffic accident may
take a different road to work that avoids cues associated with the accident and, therefore, the
anxiety that may be associated with the exposure. Avoidance is a powerful reinforcer that
produces a strong form of learning that is difficult to extinguish.
The correct answer is: Avoidance conditioning

Question 46 MoK Basic Psychology 019
Not answered A forensic investigator tries to reconstruct the crime scene and encourages visits to scene of
crime by witnesses. Which recall phenomenon is replicated in this study?
Marked out of 1.00

Flag question Select one:
Blocking
Failure of prospective memory
Encoding failure
State dependent memory
Context dependent memory

Context­dependent memory refers to the situation or context in which you find yourself is a
determinant of the ability to retrieve memories (Godden and Baddeley 1975). It suggests that
contextual and environmental cues (aspects of the environment) that are effective in activating
recall of information. This phenomenon has been useful particularly in crime detection.
Reconstructing the crime scene and visits to scenes of crime has been found to be effective in
triggering recall of details of the crime. Similarly, returning to a childhood home village/ town
or old school/college brings long forgotten memories flooding back. It is sometimes not even
necessary to visit a place to evoke memories but simply imagining the place or situation may
be sufficient to retrieve a flood of memories.
The correct answer is: Context dependent memory

1141
Question 47 MoK Basic Psychology 020
Not answered Which of the following is a test that primarily measures sustained attention?

Marked out of 1.00
Select one:
Flag question
Tower of London test
Wisconsin card sorting test
Rey verbal learning test
Continuous performance test
Trail making test B

A Continuous Performance Task/Test, or CPT, is a psychological test which measures a
person's sustained and selective attention and impulsivity. Sustained attention is the ability to
maintain a consistent focus on some continuous activity or stimuli and is associated with
impulsivity. Selective attention is the ability to focus on relevant stimuli and ignore competing
stimuli. This skill is associated with distractibility.
The correct answer is: Continuous performance test

Question 48 MoK Basic Psychology 021
Not answered A person with a fear of heights takes to parachute jumping for a hobby. This is best described
as
Marked out of 1.00

Flag question Select one:
Counterphobic behaviour
Reciprocal inhibition
Desensitisation
reaction formation
Projection

Counterphobic behaviour is any behavior that seeks out experiences that are consciously or
unconsciously feared in an attempt to master those fears. An example would be parachute
jumping for a person with a fear of heights.
The correct answer is: Counterphobic behaviour

Question 49 MoK Social Psychology 001
Not answered Fundamental attribution error refers to

Marked out of 1.00
Select one:
Flag question
Attributing personal factors for one's own behaviours
Attributing external factors for others behaviours

1142
Failure to make attributions for observed behaviours
Attributing personal factors for others' behaviours
Attributing external factors for one's own behaviours

Often when we try to understand and explain what happens in social settings, we tend to
explain behaviour of others in terms of internal disposition, such as personality traits, abilities,
motives, etc. as opposed to external situational factors. This bias is more pronounce for
external behaviours of others rather than self.
The correct answer is: Attributing personal factors for others' behaviours

Question 50 MoK Social Psychology 002
Not answered According to Allport, the denial of opportunity to the minority group due to the majority group
practicing their prejudice is termed as
Marked out of 1.00

Flag question Select one:
Avoidance
Discrimination
Antilocution
Physical attack
Extermination

Allport identified the following processes operating during conflicts between the minority and
the majority: 1. Antilocution 2. Avoidance 3. Discrimination: Minority group is discriminated
against by denying them opportunity and equality. 4. Physical attack 5. Extermination.
The correct answer is: Discrimination

Question 51 MoK Sociocultural Psychiatry 001
Not answered Which of the following is NOT true with regard to research ethics?

Marked out of 1.00
Select one:
Flag question
Written approval must be obtained from ethics committees
Advertisements can be used to recruit subjects
Subjects can be recruited by contacting medical colleagues
A subject is ethically bound to complete a study after giving written informed
consent

Financial incentives can be offered for subjects to participate in research

1143
Subjects, both healthy controls and patients, are reimbursed for their participation in research.
But signing a consent form does not mean that they cannot discontinue the study during its
course.
The correct answer is: A subject is ethically bound to complete a study after giving written
informed consent

Question 52 MoK Sociocultural Psychiatry 002
Not answered Which of the following is true with regard to aetiological theories of suicide?

Marked out of 1.00
Select one:
Flag question
Sensational media reports of suicide reduce acute risk of suicide
Serotonin receptor binding is decreased in suicidal patients
The stressors in the stress­diathesis model can include genetic factors,
physical illness or childhood experiences
Serotonin transporter binding is increased in suicidal patients
Durkheim proposed social models of suicide

According to stress­diathesis model, a typical stressor includes the acute worsening of a
psychiatric disorder or an acute psychosocial crisis. Pessimism and aggression/impulsivity
are components of the diathesis for suicidal behaviour; sex, religion, familial/genetic factors,
childhood experiences and various other factors influence this diathesis. In the prefrontal
cortex of people who committed suicide, serotonin transporter binding was decreased but
serotonin receptor binding was increased. Mann (2003). Nature Reviews Neuroscience 4,
819­828.
The correct answer is: Durkheim proposed social models of suicide

Question 53 MoK Sociocultural Psychiatry 004
Not answered A patient is willing to stop alcohol use and is accepting the need for a positive change, though
she has not taken any steps yet. According to Prochaska and DiClemente she in the stage of
Marked out of 1.00

Flag question Select one:
Precontemplation
Ready for Action
Preparation
Maintenance
Acting out

The Stages of Change model shows that, for most persons, a change in behaviour occurs
gradually, with the patient moving from being uninterested, unaware or unwilling to make a
change (precontemplation), to considering a change (contemplation), to deciding and
preparing to make a change.During the preparation stage, patients prepare to make a specific

1144
change. They may experiment with small changes as their determination to change increases.
Genuine, determined action is then taken and over time, attempts to maintain the new
behaviour occur. Relapses are almost inevitable in the process of working towards a life­long
change.
The correct answer is: Ready for Action

Question 54 MoK Sociocultural Psychiatry 005
Not answered What is the number of women that have been reported being amputated in the genital area?

Marked out of 1.00
Select one:
Flag question
1 million
10 millions
100 millions
100, 000
10000

WHO has estimated that 100 to 140 million girls and women worldwide are currently living
with the consequences of FGM.
The correct answer is: 100 millions

Question 55 MoK Sociocultural Psychiatry 007
Not answered Schizophrenic patients were more likely to relapse if discharged to families exhibiting 'high
expressed' emotion for more than:
Marked out of 1.00

Flag question Select one:
55 hours per week
33 hours per week
15 hours per week
24 hours per week
45 hours per week

Brown et al in 1962 found that schizophrenic patients were more likely to relapse if
discharged to families exhibiting high expressed emotion (hostility, critical comments and
emotional over­involvement) especially if they were exposed to these families for more than
35 hours per week. Relapse was less likely in low expressed emotion families that expressed
warmth and positive remarks. Leff et al in 1985 demonstrated the interaction between
expressed emotion and antipsychotic medication in a schizophrenic relapse and in a
controlled trial they also showed the effectiveness of family intervention therapy consisting of
education about schizophrenia and its treatment, communication training and support for
relatives, in alleviating the effects of high expressed emotion. Ref; Core Psychiatry; pg 259­
293.
The correct answer is: 33 hours per week

1145
Question 56 MoK Sociocultural Psychiatry 008
Not answered All of the following are potential boundary violations between a therapist and a patient except

Marked out of 1.00
Select one:
Flag question
A private therapist expecting payment for each session of therapy
A therapist colludes with a patient against a third party
A clinician accepts gifts from one patient but not others
Influencing patient for political causes

A clinician tries to be impressive often by disclosing personal information

If the therapist expects all patients to pay for each session, then he may not be violating any
boundaries by asking a patient do so.
The correct answer is: A private therapist expecting payment for each session of therapy

Question 57 MoK Sociocultural Psychiatry 009
Not answered Which of the following conditions is NOT strongly associated with social disadvantage?

Marked out of 1.00
Select one:
Flag question
Depression
Alzheimer's disease
Schizophrenia

Bipolar disorder
Alcohol use disorders

Early life socioeconomic level is related to level of cognition in late life but not to rate of
cognitive decline or risk of AD.Neuroepidemiology. 2005;25(1):8­14. Epub 2005 Apr 25.
The correct answer is: Alzheimer's disease

Question 58 MoK Sociocultural Psychiatry 011
Not answered A Malaysian man shows sudden excitement with marked violence after a period of
withdrawal. He returns to premorbid state after a violent incident. This is called as
Marked out of 1.00

Flag question Select one:
Latah
Cathard
Windigo

1146
Amok
Piblokto

Amok is a culture­bound syndrome seen in Malaysia and South East Asia. It is characterised
by initial sullen period, followed by an outburst of violent, sometimes homicidal behaviour and
a return to premorbid state after the episode. Though mostly dissociative, rarely amok may
occur during a brief psychotic episode or constitute the onset or an exacerbation of a chronic
psychotic process.
The correct answer is: Amok

Question 59 MoK Sociocultural Psychiatry 016
Not answered Which of the following is most associated with an increased risk of developing schizophrenia?

Marked out of 1.00
Select one:
Flag question
Being frequently placed in a 'double­blind situation' as a child
Being born in an urban area
Maternal cannabis use during pregnancy
Experiencing high expressed emotion in the family home
Maternal alcohol use during pregnancy

There is robust and consistent evidence from epidemiological studies showing that urban
birth is associated with an increased risk of developing schizophrenia. Evidence suggests that
this exposure may be associated with a sizeable proportion of cases. (McGrath 2006)
The correct answer is: Being born in an urban area

Question 60 MoK HumanDevelopment 001
Not answered Who proposed theories on cognitive development?

Marked out of 1.00
Select one:
Flag question
Sullivan
Ainsworth
Munroe
Piaget
Fromm

Swiss cognitive psychologist Jean Piaget contributed immensely to our understanding of the
development of cognitive faculties in children. His key concepts assimilation, accommodation,
equilibration, and schemas are applicable to learning at any age.
The correct answer is: Piaget

1147
Question 61 MoK HumanDevelopment 002
Not answered Which of the following is a normal stage of grief?
Marked out of 1.00
Select one:
Flag question
Mourning
Protest
Acting Out
Ambivalence
Bargaining

The classic work on stages of grief came from Erich Lindemann at Massachusetts General
Hospital, who studied 101 bereaved people. He wrote an article, published in 1944, titled
'Symptomology and Management of Acute Grief.' In this article he described a set pattern:
After an unexpected death, there is an initial shock that lasts 10­14 days. After the initial shock
comes a period of intense sadness, and the grieving person may withdraw from social
contact. Next comes anger, as the grieving person seems to 'protest' the unexpected death.
Finally, within a year or so, the grief is resolved, and the person returns to normal.
The correct answer is: Protest

Question 62 MoK HumanDevelopment 003
Not answered Which period refers to the autistic phase of Mahler's stages of early development?

Marked out of 1.00
Select one:
Flag question
6­36 months
0­2 months
2­6 Months
6­12 months
12­24 months

According to Mahler, the first few weeks of infancy is termed as the 'Normal Autistic Phase'
(she later discarded this designation). A newborn infant is blissfully unaware of anything but
its own needs, and the mother needs to be available to meet those needs in this stage.
The correct answer is: 0­2 months

Question 63 MoK HumanDevelopment 004
Not answered Average age of puberty in boys is

Marked out of 1.00
Select one:
Flag question
11­12 years

1148
10­11 years
8­9 years

9­10 years
14­15 years

In the UK the average age of onset of puberty in males is 11.2 years; for females it is 11 years.
Menarche on average is at 12.5 years for females.
The correct answer is: 11­12 years

Question 64 MoK HumanDevelopment 005
Not answered The strange­situation study of attachment was carried out on

Marked out of 1.00
Select one:
Flag question
Children aged 6­9 years
Infants between 1­6 months

Newborn babies
Toddlers
Children aged 12­24 months

The strange­situation study of attachment was carried out on children between 12­24 Months.
It is a procedure designed to test the quality of the attachment bond between mother and their
infants. It has three phases with the infant's reactions being noted in each of the phases; 1.
Reaction to a stranger in the presence of the mother 2. Reaction when left alone with a
stranger 3. Reaction when re united with the mother
The correct answer is: Children aged 12­24 months

Question 65 MoK HumanDevelopment 007
Not answered At what age can a child build a tower of 6 cubes?

Marked out of 1.00
Select one:
Flag question
2 years
6 ­10 months
5 years
12­18 months
4 years

1149
At age 2, a child can run and build a tower of 6 cubes. At age 3, a child can make a tower of 9­
10 cubes.
The correct answer is: 2 years

Question 66 MoK HumanDevelopment 011
Not answered Mastering intimate relationships despite the demands of work is a successful developmental
task for ages between
Marked out of 1.00

Flag question Select one:
60 to 80
40 to 60
20 to 40
10 to 15
80 to 90

The Intimacy vs. Isolation conflict is emphasized around the ages of 30. At the start of this
stage, identity vs. role confusion is coming to an end, though it still lingers at the foundation of
the stage (Erikson, 1950).
The correct answer is: 20 to 40

Question 67 MoK HumanDevelopment 014
Not answered Which of the following is correct with regard to day­care for children?
Marked out of 1.00
Select one:
Flag question
4 months day care before age 1 for more than 20hours/week may be
detrimental
Day care reduces attachment behaviour
Day care induces neurotic clinging
Regular day care affects cognitive development
Day care increases maternal bonding

A series of studies by Belsky and Rovine (1988) claim that insecure attachments were more
likely to develop if the child had been receiving care of at least 20 hours per week for 4
months or more before its first birthday.
The correct answer is: 4 months day care before age 1 for more than 20hours/week may be
detrimental

Question 68 MoK Sociocultural Psychiatry 017
Not answered

1150
Marked out of 1.00 When a psychiatric patient is unwell, family and friends report their actions to mental health
professionals in an attempt to get them admitted to an institution. What was the term used by
Flag question
Goffman to describe this?

Select one:
Institutional neurosis
Role stripping
Mortification

Batch living
Betrayal funnel

Goffman depicted the sequence of events through which the understandings of a 'pre­patient'
would be betrayed using the process of referral and admission. Goffman described this as a
"betrayal funnel" wherein during each stage of the admission process the family and friends
might try to give patients the impression that they would not lose any more of the rights that
they enjoyed in the community. (Goffman 1961, p. 140).
The correct answer is: Betrayal funnel

Question 69 MoK Assessment EMI001
Not answered Features of defence mechanisms
For each of the characteristic features described below chose the most appropriate defence
Marked out of 3.00
mechanism suitable from the above list
Flag question

Passive aggression
Identification
Ascribing unacceptable motivations Acting out
to others Projection
Suppression
Regression
Humour
Denial
Projection
Repression
Acting out
Altruism
Regression
Resentment by noncooperation Turning into self
Passive aggression
Humour
Repression
Identification
Denial
Altruism
Turning into self
Regression
Suppression
Suppression
Altruism
Humour
Failure to accept or face conflicts Identification
Denial
Passive aggression
Projection
Repression
Acting out

1151
Explanation: In projection, intolerable feelings, impulses or thoughts are falsely attributed to
other people.  Passive aggression can manifest itself as repeated postponements, rigidity in
task performance, resentment, sullenness, or repeated failure to take up the responsibility to
complete tasks. Denial is characterised by a total failure to accept a problem even when faced
with the facts contrary to the negation.Passive aggression
The correct answer is: Ascribing unacceptable motivations to others – Projection, Resentment
by noncooperation – Passive aggression, Failure to accept or face conflicts – Denial

Question 70 MoK Assessment EMI002
Not answered Supportive techniques
For each of the following examples, identify the type of supportive techniques aimed at
Marked out of 3.00
eliciting information
Flag question

Reassurance
Sometimes when people are upset Validation
and low in their mood, they think of Positive reinforcement
hurting themselves. Has this been Advice
true for you? Postponement
Partnering
Acknowledgment of affect
Statement of respect
Positive reinforcement
Disapproval
Validation
I think it is best for you to consider
Disapproval
psychotherapy at this time. If I am
Partnering
you, I will give this a serious thought
Advice
Acknowledgment of affect
Statement of respect
Reassurance
Partnering
This depression may be very difficult
Postponement
Disapproval
for you. But I think it is very likely with Positive reinforcement
the proper treatment you can get Reassurance
back to your job. Validation
Advice
Statement of respect
Postponement
Acknowledgment of affect
Explanation: 
Validation/normalization: It helps to decrease a patient's sense of embarrassment about a
feeling or behaviour. It can be done by quoting how it is normal for people to have different
emotions/ reactions/ behaviours, etc. 
Advice: Many patients seek help directly; it is acceptable to provide advice but based on
sound understanding of the context. Premature advice can be obstructive than facilitative. 
Ref: Adapted from Kay J & Tasman A. Essentials of Psychiatry, 2nd edition, 2006. John Wiley
& Sons, Ltd.
The correct answer is: Sometimes when people are upset and low in their mood, they think of
hurting themselves. Has this been true for you? – Validation, I think it is best for you to
consider psychotherapy at this time. If I am you, I will give this a serious thought – Advice, This

1152
depression may be very difficult for you. But I think it is very likely with the proper treatment
you can get back to your job. – Reassurance

Question 71 MoK Assessment EMI004
Not answered Rating scales and their uses
For each description given below, select the relevant rating scale from the list above;
Marked out of 4.00

Flag question
Hospital anxiety and depression scale (HADS)
Young mania rating scale (YMRS)
This scale is useful for rating severity General health questionnaire (GHQ)
in patients with a pre­existing
Global assessment of functioning scale (GAF)
diagnosis of OCD
Yale­Brown obsessive compulsive scales (YBOCS)
Brief psychiatric rating scale (BPRS)
Positive and negative symptoms scale (PANSS)
Mini­mental state examination (MMSE)
Mini­mental state examination (MMSE)
Minnesota multiphasic personality inventory (MMPI)
Clinical Global impressions scale (CGI)
Commonly used screening tool used Clinical Global impressions scale (CGI)
Hospital anxiety and depression scale (HADS)
in primary care and general
Positive and negative symptoms scale (PANSS)
population studies
Minnesota multiphasic personality inventory (MMPI)
Global assessment of functioning scale (GAF)
Yale­Brown obsessive compulsive scales (YBOCS)
Brief psychiatric rating scale (BPRS)
Minnesota multiphasic personality inventory (MMPI)
General health questionnaire (GHQ)
Positive and negative symptoms scale (PANSS)
This scale measures overall
Young mania rating scale (YMRS)
Clinical Global impressions scale (CGI)
psychosocial functioning Young mania rating scale (YMRS)
Global assessment of functioning scale (GAF)
Yale­Brown obsessive compulsive scales (YBOCS)
Hospital anxiety and depression scale (HADS)
General health questionnaire (GHQ)
Brief psychiatric rating scale (BPRS)
Mini­mental state examination (MMSE)
Yale­Brown obsessive compulsive scales (YBOCS)
7­point scale useful to assess global Brief psychiatric rating scale (BPRS)
Global assessment of functioning scale (GAF)
observation of severity of psychiatric
Minnesota multiphasic personality inventory (MMPI)
illness
Young mania rating scale (YMRS)
General health questionnaire (GHQ)
Positive and negative symptoms scale (PANSS)
Clinical Global impressions scale (CGI)
Hospital anxiety and depression scale (HADS)
Explanation:  Mini­mental state examination (MMSE)
Yale­Brown obsessive­compulsive scale (YBOCS) is a clinician­administered semi­structured
interview, allowing rating of severity in patients with a pre­existing diagnosis of OCD. 
General Health Questionnaire (GHQ) is a self­rated screening instrument for the presence of
psychiatric illness. It is a commonly used screening tool used in primary care and general
population studies. 
GAF of Global assessment of functioning scale is a 100­item, self­report rating scale,
measuring overall psychosocial functioning. 
CGI or Clinical global impression scale is a 7­point scale useful for clinicians to assess the
global observed severity of psychiatric illness.
The correct answer is: This scale is useful for rating severity in patients with a pre­existing

1153
diagnosis of OCD – Yale­Brown obsessive compulsive scales (YBOCS), Commonly used
screening tool used in primary care and general population studies – General health
questionnaire (GHQ), This scale measures overall psychosocial functioning – Global
assessment of functioning scale (GAF), 7­point scale useful to assess global observation of
severity of psychiatric illness – Clinical Global impressions scale (CGI)

Question 72 MoK Assessment EMI005
Not answered Dangerous complications of eating disorders
Identify the metabolic abnormalities commonly seen in the following clinical situations
Marked out of 3.00

Flag question
Hyperhomocystinemia
A 16­year­old girl with history of Hyperkalemia
significant weight loss, body image Low T3 and high growth hormone
distortions and morbid fear of fatness
Hypothyroidism
had shown some endocrine
abnormalities in her blood
Hyperphosphatemia
Hyponatremia
Hypokalemia
Hypercortisolemia
Hyperhomocystinemia
High T3 and high growth hormone
Hyperphosphatemia
A 23­year­old woman with history of Hypophosphatemia
High T3 and high growth hormone
bulimia nervosa and is determined to
Hyperkalemia
reduce weight by repetitive vomiting
Hypothyroidism
Hypercortisolemia
Hypokalemia
A 20­year­old woman uses regular Hypophosphatemia
Hypophosphatemia
laxatives for last two years in order to Low T3 and high growth hormone
Hypokalemia
reduce weight. In addition to Hyponatremia
Hyperphosphatemia
hypokalemia associated with
Hypercortisolemia
acidosis, what other electrolyte
Hyperkalemia
abnormalities results from chronic
laxative abuse? Hyponatremia
High T3 and high growth hormone
Hypothyroidism
Low T3 and high growth hormone
Hyperhomocystinemia
Explanation: The diagnosis is anorexia nervosa. The common endocrine changes in anorexia
include low concentrations of luteinising hormone, follicle stimulating hormone and oestradiol,
low T3 but T4 in low normal range, average concentrations of thyroid stimulating hormone
(low T3 syndrome), mild increase in plasma cortisol and raised growth hormone level. 
Electrolyte disturbances can be of variable nature and are present in those who frequently
vomit or misuse large quantities of laxatives or diuretics. Vomiting results in metabolic
alkalosis and hypokalemia. In repetitive vomiting, loss of hydrochloric acid from gastric juices
lead to metabolic alkalosis (loss of acid ­ alkalosis). Laxative misuse results in metabolic
acidosis, hyponatraemia, hypokalemia. During laxative induced diarrhoea, a considerable
amount of bicarbonate may be lost in the stool. With healthy kidneys, the lost bicarbonate is
replaced adequately, and a serious base deficit does not develop. When there is poor renal
blood flow due to hypovolaemia/starvation, base deficit and acidosis develop rapidly. Acidosis
also results from excessive production of lactic acid when patients have severe diarrhoea.
The correct answer is: A 16­year­old girl with history of significant weight loss, body image
distortions and morbid fear of fatness had shown some endocrine abnormalities in her blood
– Low T3 and high growth hormone, A 23­year­old woman with history of bulimia nervosa and
is determined to reduce weight by repetitive vomiting – Hypokalemia, A 20­year­old woman

1154
uses regular laxatives for last two years in order to reduce weight. In addition to hypokalemia
associated with acidosis, what other electrolyte abnormalities results from chronic laxative
abuse?
– Hyponatremia

Question 73 MoK Assessment EMI006
Not answered Clinically notable motor disorders
Choose one option for each of the following descriptions
Marked out of 4.00

Flag question
Fibrillation
Chorea
Sustained abnormalities of posture Tardive dyskinesia
occurring focally in writer's cramp or
Rigidity
torticollis
Fasciculations
Tics
Asterixis
Hemiballismus
Tics
Brief jerky downward movements of Myoclonus
Hemiballismus
the wrist, when the patient extend Dystonia
Asterixis
both arms with the wrists dorsiflexed,
Dystonia
palms facing forward, and eyes
closed
Fibrillation
Myoclonus
Fasciculations
Chorea
Fibrillation
Tardive dyskinesia
Hemiballismus
Quasi­purposeful movements Rigidity
Dystonia
affecting multiple joints with a distal
Myoclonus
preponderance
Chorea
Fasciculations
Asterixis
Tics
Tics
Rigidity
Tardive dyskinesia
Tardive dyskinesia
Chorea
Asterixis
Quivering of the muscle seen under Dystonia
the skin in neuromuscular disorder
Rigidity
Hemiballismus
Fasciculations
Fibrillation
Myoclonus

Explanation: 
Writer's cramp is a form of focal dystonia, which is an involuntary, sustained muscle
contraction causing twisting movements and abnormal postures. Writer's cramp is the most
typical dystonia occurring in the setting of repetitive movement disorders. 
The term chorea means dance. These are quasi­purposeful movements affecting multiple
joints with a distal preponderance. 

1155
Asterixis represents the failure to maintain a position actively and is "caused by the abnormal
function of diencephalic motor centres that regulate the tone of the agonist and antagonist
muscles involved in maintaining posture" (Asterixis ­ NEJM ­ The New England Journal of
Medicine.., http://www.nejm.org/doi/full/10.1056/NEJMicm0911157 (accessed April 21, 2015). 
Fasciculations may be seen under the skin as quivering of the muscle. Although fasciculations
are typically benign (particularly when they occur in the calf), if widespread, they can be
associated with neuromuscular disease, including amyotrophic lateral sclerosis (ALS). 
Neurological History and Physical Examination,
http://emedicine.medscape.com/article/1147993­overview (accessed March 31, 2015).
The correct answer is: Sustained abnormalities of posture occurring focally in writer's cramp or
torticollis – Dystonia, Brief jerky downward movements of the wrist, when the patient extend
both arms with the wrists dorsiflexed, palms facing forward, and eyes closed – Asterixis,
Quasi­purposeful movements affecting multiple joints with a distal preponderance – Chorea,
Quivering of the muscle seen under the skin in neuromuscular disorder – Fasciculations

Question 74 MoK Assessment EMI007
Not answered Medical disorders & psychiatry 
For each description below choose ONE option from the above list
Marked out of 3.00

Flag question
A 44­year­old homeless man is Alcoholic hallucinosis
brought into A&E with agitation and Fahr's syndrome
features of psychosis and Complex partial seizures
depression. An EEG reveals diffuse
Hepatic encephalopathy
attenuation in alpha activity, and his
gait is abnormal. MRI shows atrophy
Alcohol withdrawal
of the striatum. Metachromatic leucodystorphy
Neuroacanthosis
A 25­year­old man recently returned Alcoholic delirium
from Kansas. He presents with Systemic Lupus Erythematosis
unprovoked aggression. Of late he Metachromatic leucodystorphy
Herpes simplex encephalitis
experienced recurrent seizures Hepatic encephalopathy
Huntington's disease
followed by periods of impaired Alcohol withdrawal
mentation. His full blood profile and Alcoholic delirium
hepatic functions are normal, and a
Fahr's syndrome
lumbar puncture is unremarkable.
Neuroacanthosis
Complex partial seizures
Alcoholic hallucinosis
Herpes simplex encephalitis
Alcoholic hallucinosis
Huntington's disease
A 30­year­old woman has two­sided Alcoholic delirium
Systemic Lupus Erythematosis
basal ganglia calcification. She is Hepatic encephalopathy
being reviewed for kidney stones and Fahr's syndrome
polyuria. Systemic Lupus Erythematosis
Metachromatic leucodystorphy
Herpes simplex encephalitis
Alcohol withdrawal
Complex partial seizures
Huntington's disease
Explanation: Case 1 is suggestive of Huntington's disease, which can present with
nonspecific psychiatric disturbances. Neuroacanthosis
Case 2 is descriptive of complex partial seizures. These begin as simple partial seizures and
progress to impairment of consciousness. There may also be an impairment of consciousness
at the onset. Aggressive behaviour is also a possibility in patients with complex partial

1156
seizures. The lead about America is a red herring. 
Case 3 describes Fahr's disease, which is caused by idiopathic progressive calcium
deposition in the basal ganglia. Early­onset type (between the ages of 20 and 40) is
associated with schizophreniform psychoses and catatonic symptoms. Onset between the
ages of 40 and 60 is associated with dementia and choreoathetosis. Depression is also very
common, but mania much less so. 50% of patients have psychiatric symptoms, which
correlate with the degree of calcification. The most frequent clinical features are Parkinsonism,
dystonia, tremor, gait disturbance and dysarthria with some cases exhibiting chorea, seizures
and myoclonus. The term Fahr's syndrome is preferred if specific causes of calcium deposition
in the basal ganglia such as hypoparathyroidism are diagnosed. Radiological basal ganglia
calcification on CT without clinical features may occur at a rate of about 0.9% MRI shows
hypointensity of the striatum. EEG is non­specific ­ may show diffuse changes. 
Neuropsychiatry of the basal ganglia ­ Re: Cognition Health, http://www.re­
cognitionhealth.com/dynamicdata/MovementDisorders.pdf (accessed March 31, 2015)
The correct answer is: A 44­year­old homeless man is brought into A&E with agitation and
features of psychosis and depression. An EEG reveals diffuse attenuation in alpha activity,
and his gait is abnormal. MRI shows atrophy of the striatum. – Huntington's disease, A 25­
year­old man recently returned from Kansas. He presents with unprovoked aggression. Of late
he experienced recurrent seizures followed by periods of impaired mentation. His full blood
profile and hepatic functions are normal, and a lumbar puncture is unremarkable. – Complex
partial seizures, A 30­year­old woman has two­sided basal ganglia calcification. She is being
reviewed for kidney stones and polyuria. – Fahr's syndrome

Question 75 MoK Psychology EMI001
Not answered Principles of Learning Theory
Which type of learning process is involved in each of the following situations?
Marked out of 3.00

Flag question
Extinction
Stimulus preparedness
The fear that a child with a spider Trace conditioning
phobia shows towards insects in
Stimulus generalisation
general.
Positive reinforcement
Habituation
Punishment
Negative reinforcement
Positive reinforcement
Decay
Habituation
Humans usually react with a fear Stimulus generalisation
response to snakes Extinction
Decay
Punishment
Stimulus preparedness
Trace conditioning
Habituation
Negative reinforcement
Extinction
Cars have the habit of flashing a seat
belt warning sign until you tighten
Trace conditioning
your seat belt. Punishment
Decay
Stimulus preparedness
Negative reinforcement
Stimulus generalisation
Positive reinforcement

1157
Stimulus preparedness (Seligman) explains why snake and spider phobia are commoner
than shoe phobia or watch phobia. In evolutionary terms, the stimuli that were threatening to
hunter­gatherer man has been hard­wired in our system to respond immediately ­ and phobia
develops more readily for such 'prepared stimuli'. 
Pavlovs' dog experiments were conducted using human subjects by Watson & Rayner and
paved the way for founding the school of behaviourism. Watson produced 'phobia' in an infant
called Little Albert. By exposing him to loud, frightening noise whenever he was shown a
white rat, eventually Albert became fearful of the white rat, even when he heard no loud noise.
A similar fear response was seen when any furry white object was shown to Albert. This
'spread' of associative learning from one stimulus to other is called stimulus generalisation. 
Reinforcement can be positive or negative according to the way reinforcers are handled i.e. if
something is given or taken away. If a particular behaviour increases by providing a
reinforcer, then this is positive reinforcement. If a particular behaviour increases by removal of
an aversive stimulus then this is negative reinforcement. Similarly punishment can also be
positive (smack) or negative (take the chocolate away) though it is often used in positive
sense.
The correct answer is: The fear that a child with a spider phobia shows towards insects in
general. – Stimulus generalisation, Humans usually react with a fear response to snakes –
Stimulus preparedness, Cars have the habit of flashing a seat belt warning sign until you
tighten your seat belt. – Negative reinforcement

Question 76 MoK Assessment EMI010
Not answered Eponymous syndromes
Find one most appropriate diagnosis/eponym each for the syndromes described below:
Marked out of 3.00

Flag question
Binswanger's disease
Mr. Yardley is a prisoner awaiting trial Ekbom's syndrome
and when he was asked 'What is the Dhat syndrome
capital of England, he answered Diogenes syndrome
'Paris'. Cotard's syndrome
Couvade syndrome
Fregoli syndrome
De Clerembault syndrome
Ganser's syndrome
Ganser's syndrome
Ekbom's syndrome
A 34­year­old gentleman is
convinced that his wife is not his real Formication
De Clerembault syndrome
wife and has been replaced by an Capgras syndrome
Cotard's syndrome
imposter who looks just like her. Capgras syndrome
Binswanger's disease
Formication
Progressive global cognitive Dhat syndrome
impairment with a clinical course Capgras syndrome
Couvade syndrome
Couvade syndrome
characterised by gradual intellectual
Fregoli syndrome
Formication
decline, generalised slowing, and
Diogenes syndrome
Diogenes syndrome
motor problems. MRI scan shows
multiple microvascular infarcts of Cotard's syndrome
perforating vessels and white matter Ekbom's syndrome
leukoaraiosis De Clerembault syndrome
Fregoli syndrome
Binswanger's disease
Explanation: 
Ganser's syndrome
Dhat syndrome

1158
Ganser symptom refers to the production of approximate solutions as seen in case 1. Here the
patient gives repeated wrong answers to questions, which are nonetheless approximately
right. 
Capgras syndrome (case 2) is a delusional misidentification resulting in the belief that a
person known to a patient has been replaced by a 'double' who appears identical though not
the real person. Capgras syndrome is sometimes referred to as the illusion of doubles though
it is a delusion. This syndrome was originally reported by Kahlbaum (1866) but most
subsequent research is attributed to Capgras and colleagues (1923, 1924). 
Case 3 is Binswanger's disease (progressive small vessel disease) ­ a subcortical dementia
with a clinical course characterised by slow intellectual decline and generalised slowing. The
clinical picture ma be dominated by the dementia, and there may be concomitant physical
problems such as gait disorders or dysarthria. MRI scan shows small distinct infarcts
(Lacunae) or more generalised white matter changes (Leukoariasis).
The correct answer is: Mr. Yardley is a prisoner awaiting trial and when he was asked 'What is
the capital of England, he answered 'Paris'. – Ganser's syndrome, A 34­year­old gentleman is
convinced that his wife is not his real wife and has been replaced by an imposter who looks
just like her. – Capgras syndrome, Progressive global cognitive impairment with a clinical
course characterised by gradual intellectual decline, generalised slowing, and motor
problems. MRI scan shows multiple microvascular infarcts of perforating vessels and white
matter leukoaraiosis – Binswanger's disease

Question 77 MoK Assessment EMI011
Not answered Descriptive features in psychiatry
Identify the terms used to describe each of the following:
Marked out of 3.00

Flag question
A 70­year­old man lives in a Mitgehen
supported accommodation. He goes Alexithymia
out for shopping on his own but Jamais Vu
forgets a few things on his shopping
Perseveration
list. When confronted he says that he
has already got enough of these
Reflex hallucination
items in stock at his flat. Deja vu
Functional hallucination
Paraphasia
Mitgehen
Vorbeigehen
Jamais Vu
Suggestibility
Reflex hallucination
A 72­year­old male admitted to A&E, Confabulation
Alexithymia
when asked what the month was,
Deja vu
said 'April' and when asked what the
Perseveration
year was, said 'April.'
Vorbeigehen
Suggestibility
Functional hallucination
Paraphasia
A patient with a moderate learning Confabulation
disability is unable to express her Vorbeigehen
emotional problems verbally. When Functional hallucination
asked if she feels depressed, she Perseveration
says 'yes'. When a picture book with
Jamais Vu
various emotional states is shown,
she admits having whatever emotion
Confabulation
is shown first in the booklet at Alexithymia
different times. Paraphasia
Deja vu

1159
Explanation: 
Confabulation may be spontaneous where often elements of fantasy are found. It can also
occur in an attempt to avoid embarrassment as in this case.
Perseveration is repetitive goal­directed verbal or motor behaviour where the response set
does not change despite changes in the stimuli.
Case 3: This patient is highly suggestible as she answers yes to closed questions and admits
to the emotions in any random picture shown to her.
The correct answer is: A 70­year­old man lives in a supported accommodation. He goes out
for shopping on his own but forgets a few things on his shopping list. When confronted he
says that he has already got enough of these items in stock at his flat. – Confabulation, A 72­
year­old male admitted to A&E, when asked what the month was, said 'April' and when asked
what the year was, said 'April.' – Perseveration, A patient with a moderate learning disability is
unable to express her emotional problems verbally. When asked if she feels depressed, she
says 'yes'. When a picture book with various emotional states is shown, she admits having
whatever emotion is shown first in the booklet at different times. – Suggestibility

Question 78 MoK Assessment EMI016
Not answered Personality disorders ­ diagnosis
Choose one option each from above list:
Marked out of 3.00

Flag question A 35­year­old man refuses to provide
answers to standard questions during Schizotypal personality disorder
an initial clerking and threatens to Dependent personality disorder
stop the interview if recording his Borderline personality disorder
telephone number is insisted. It
Paranoid personality disorder
seems that he has taken many
hospitals to court for suspicions about
Narcissistic personality disorder
how his personal data has been Histrionic personality disorder
handled. Obsessive­compulsive personality disorder
Antisocial personality disorder
A 33­year­old night security at a local Dependent personality disorder
Schizoid personality disorder
hospital prefers to be alone Antisocial personality disorder
Avoidant personality disorder
whenever possible. He has no Schizoid personality disorder
friends, and he does not socialise. He
Obsessive­compulsive personality disorder
does not keep update with current
affairs and has no sexual interests.
Avoidant personality disorder
He spends most time day dreaming. Histrionic personality disorder
Schizotypal personality disorder
A 21­year­old lady spends excessive Borderline personality disorder
Narcissistic personality disorder
time seeking attention by making Narcissistic personality disorder
Avoidant personality disorder
herself attractive. She wears the Paranoid personality disorder
Antisocial personality disorder
'fashion of the week' all the time and
Histrionic personality disorder
show shallow but labile affect. She
Schizoid personality disorder
comes across as being very
impressionistic. Borderline personality disorder
Obsessive­compulsive personality disorder
Schizotypal personality disorder
Paranoid personality disorder
Explanation:  Dependent personality disorder

1160
Paranoid personality disorder is "characterised by pervasive suspiciousness, mistrust, and
hypersensitivity to criticism and hostility". As a rule, paranoid people are ready to counter­
attack, provoking repeated confrontations and induce hostility and resentment in others.
Schizoid personality disorder is "characterised by a persistent pattern of social withdrawal,
discomfort in social interactions and are introverted, isolated and lonely". 
Histrionic personality disorder is "characterised by excessive emotionality and attention
seeking, and by dramatic, colourful, and extroverted behaviour that is egocentric, dependent,
and demanding in interpersonal relationships". (All descriptions from New Oxford Textbook of
Psychiatry)
The correct answer is: A 35­year­old man refuses to provide answers to standard questions
during an initial clerking and threatens to stop the interview if recording his telephone number
is insisted. It seems that he has taken many hospitals to court for suspicions about how his
personal data has been handled. – Paranoid personality disorder, A 33­year­old night
security at a local hospital prefers to be alone whenever possible. He has no friends, and he
does not socialise. He does not keep update with current affairs and has no sexual interests.
He spends most time day dreaming. – Schizoid personality disorder, A 21­year­old lady
spends excessive time seeking attention by making herself attractive. She wears the 'fashion
of the week' all the time and show shallow but labile affect. She comes across as being very
impressionistic. – Histrionic personality disorder

Finish review

1161
 Home Mock Paper Practice Tests

Mock Paper A(1)

Started on Tuesday, 30 June 2015, 2:43 AM
State Finished
Completed on Tuesday, 30 June 2015, 2:44 AM
Time taken 12 secs
Grade 0.00 out of 100.00
Feedback You are on the right track. Could you spend some more time on your revision?

Question 1 MoK Classification 005
Not answered With respect to depression seen in patients with schizophrenia, which of the following is
correct?
Marked out of 1.00

Flag question Select one:
It is seen only in post­psychotic period
It is not an outcome indicator
It often coexists with negative symptoms
It is often a side effect of antipsychotic use
It does not respond to antidepressants

Your answer is incorrect.
Depression can be seen in either acute or postpsychotic phases of schizophrenia. Post­
schizophrenic depression is diagnosed if the general criteria for schizophrenia were met
within the past twelve months, but not in full at the present time. But one of either
positive/negative symptoms/formal thought disorder/catatonia symptoms must be present.
The correct answer is: It often coexists with negative symptoms

Question 2 MoK Classification 007
Not answered Which of the following is true with regard to posttraumatic stress disorder?

Marked out of 1.00
Select one:
Flag question
Diagnostic EEG findings often noted
Not seen in children

Patients show decreased startle reaction
Onset is within 6 months of trauma

1162
Trauma must be near fatal in nature

Your answer is incorrect.
The onset of PTSD is generally within six months of the trauma. Retrospective reports of war
veterans reveal that delayed­onset PTSD is extremely rare one year post­trauma, and there
was no evidence of PTSD symptom onset 6 or more years after trauma exposure. Br J
Psychiatry. 2009 Jun;194(6):515­20.
The correct answer is: Onset is within 6 months of trauma

Question 3 MoK Classification 009
Not answered Select one typical feature of hypomania

Marked out of 1.00
Select one:
Flag question
Presence of bizarre delusions
Presence of mood incongruent delusions
Absence of flight of ide
Absence of significant psychosocial dysfunction
Presence of hallucinations

Your answer is incorrect.
Hypomania produces some but not significant psychosocial dysfunction according to ICD
The correct answer is: Absence of significant psychosocial dysfunction

Question 4 MoK Classification 020
Not answered What is the most common psychiatric disorder in children with mild learning disability?

Marked out of 1.00
Select one:
Flag question
Conduct disorder
Psychosis
Autistic spectrum disorder
Personality disorder
Depression

Your answer is incorrect.
Disruptive and conduct­disorder behaviours occurred more commonly in the mildly disabled
group, which represents approximately 85 percent of persons with learning disability. ADHD

is often a prominent feature in children with LD (up to 20%). Those with severe mental

1163
retardation have a particularly high rate of autistic disorder and pervasive developmental
disorders.
The correct answer is: Conduct disorder

Question 5 MoK Classification 021
Not answered A 45­year­old woman believes that his neighbours are plotting to kill her for last 6 months.
There is no hallucination. The diagnosis is
Marked out of 1.00

Flag question Select one:
Adjustment disorder
Dementia
Schizophrenia
Bipolar disorder
Delusional disorder

Your answer is incorrect.
Delusional disorder is an illness characterized by the presence of nonbizarre delusions in the
absence of other mood or psychotic symptoms
The correct answer is: Delusional disorder

Question 6 MoK Classification 024
Not answered Mrs. Chomes has recently lost her husband after his protracted fight with stomach cancer. The
following is indicative of a pathological grief reaction:
Marked out of 1.00

Flag question Select one:
Signing up for counselling
Denial 4 months after the death
Joining a charity for fighting prostate cancer
Clearing his room of his belongings
Visual hallucinations of her husband

Your answer is incorrect.
Pathological grief can be delayed grief (not occurring for a long period), inhibited grief
(symptoms of normal grief not shown), prolonged grief (grief more than 6­12 months) or frank
depressive reaction (psychosis, suicidality, inappropriate guilt outside the death event etc).
Some degree of mummification (preserving the belongings of the deceased as they were
before death), visual hallucinations of the deceased ('sighting the dead'), altruistic charity and
seeking help through counseling are normal reactions.
The correct answer is: Denial 4 months after the death

1164
Question 7 MoK Classification 026
Not answered In ICD­10 astasia­abasia is described under:

Marked out of 1.00
Select one:
Flag question
Dissociative fugue disorder
Dissociative motor disorder
Factitious disorder
Malingering
Dissociative stupor

Your answer is incorrect.
In ICD10 dissociative (conversion) disorders are classified into dissociative amnesia,
dissociative fugue, trance /possession, disorders of movement (includes motor disorders),
dissociative stupor, dissociative convulsions (pseudoseizures) and dissociative sensory loss /
anaesthesia. Astasia­abasia is discussed under dissociative motor disorders.
The correct answer is: Dissociative motor disorder

Question 8 MoK Classification 028
Not answered A university student has been diagnosed with Kleine Levine syndrome after presenting with
recurring periods of excessive drowsiness and sleep. Which of the following feature is NOT a
Marked out of 1.00
part of this diagnosis?
Flag question
Select one:
Irritability
Excessive food intake
Drop attacks

Uninhibited sex drive
Amnesia following episodes

Your answer is incorrect.
Drop attacks are not a part of Kleine Levine syndrome. Kleine­Levin syndrome (KLS) is a rare
disorder characterised by periodic episodes of hypersomnolence and hyperphagia.
Associated features of the disorder include a lack of concentration, mood changes, sometimes
hypersexuality and anxiety. Laboratory tests may show slight changes in the
electroencephalogram. However, clinical presentation and laboratory tests are normal during
asymptomatic intervals. KLS most often presents in adolescent males, with complete recovery
by the 3rd to the 4th decade of life. Possible precipitating factors include excessive workload,
febrile illness, and respiratory infections ( Kleine­Levin syndrome: a unique cause of fatigue.
Retrieved from http://bjsm.bmj.com/content/39/2/e7.full.html).
The correct answer is: Drop attacks

1165
Question 9 MoK Classification 029
Not answered Which of the following is highly characteristic of somatisation disorder?
Marked out of 1.00
Select one:
Flag question
Somatic delusions is a core feature
Usually presents after the age of 40
Less observable clinical association with childhood illness in the patient
Excessive use of medications and non compliance with medical advice
Occurs commonly in males

Your answer is incorrect.
Somatisation disorder can be associated with a great deal of stigma; there is a risk that
patients may be dismissed by their physicians as having problems that are 'all in their head'.
Doctors' explanations of their symptoms are often at odds with these patients' own thinking.
The correct answer is: Excessive use of medications and non compliance with medical advice

Question 10 MoK Classification 030
Not answered Mr Richmond obtains gratification by rubbing against women travelling on the crowded
Dockland Railways. What is this condition called?
Marked out of 1.00

Flag question Select one:
Exhibitionism
Voyeurism
Frotteurism
Fetishism
Masochism

Your answer is incorrect.
Frotteurism is a paraphilia (sexual deviancy) classified in ICD10. In this disorder, non­
consensual rubbing of genital organs against another person to achieve arousal is seen.
The correct answer is: Frotteurism

Question 11 MoK Classification 031
Not answered When asked how many legs a cat has, a 42­year­old male prisoner answers 'five legs'. Which
of the following conditions is likely?
Marked out of 1.00

Flag question Select one:
Dementia
Autistic Spectrum Disorder

1166
Schizophrenia
Learning Disability
Ganser syndrome

Your answer is incorrect.
This is a rare condition of uncertain or variable aetiology. It was first described by the
psychiatrist Sigbert Ganser in 1898 nad characterised by approximate answers. The essential
feature of approximate answers is that while the patient gives an incorrect response, the
approximately correct nature of the response suggests that he or she understands the
question well. Thus, the responses in Ganser's are not nonsensical, perseverative or
otherwise inappropriate.
The correct answer is: Ganser syndrome

Question 12 MoK Classification 033
Not answered A middle aged man presents with 2 year history of having abnormal limb movements and
memory problems. He has no past history of psychiatric problems and physical investigations
Marked out of 1.00
are unremarkable. His father died at the age of 50 and he had similar 'limb shakes'. The likely
Flag question diagnosis is

Select one:
Wilson's disease
Catatonic disorder
Crutzfeld Jakob disease
Huntington's disease

Lewy body dementia

Your answer is incorrect.
Huntington's disease onset is defined by the beginning of motor symptoms, and most often the
initial complaint that leads patients to seek medical attention is 'clumsiness', 'tremor', 'balance
trouble', or 'jerkiness'. The primary involuntary movement abnormality, and often the earliest
symptom, is chorea or choreoathetosis, continuous and irregular writhing and jerking
movements. The limbs and trunk are most prominently affected
The correct answer is: Huntington's disease

Question 13 MoK Classification 035
Not answered Which of the following is a characteristic feature of schizoid personality disorder?
Marked out of 1.00
Select one:
Flag question
Labile affect
Insensitivity to prevailing social norms
Highly sensitive and intolerability to setbacks

1167
Poor frustration tolerance

Rigidity and stubbornness

Your answer is incorrect.
Taking pleasure in few, if any, activities, indifference to social norms and conventions,
preoccupation with fantasy and introspection are features of schizoid personality.
The correct answer is: Insensitivity to prevailing social norms

Question 14 MoK Classification 036
Not answered According to DSM­5 criteria, which of the following is not included in the diagnostic criteria for
a manic episode?
Marked out of 1.00

Flag question Select one:
Increased energy levels
Flight of ideas
Much more talkative than usual
Insomnia/hypersomnia
Inflated self esteem/grandiosity

Your answer is incorrect.
The DSM­5 gives a number of criteria that must be met before a disorder is classified as
mania. The first one is that an individual's mood must be elevated, expansive or irritable.
Reduced need for sleep, not insomnia, is a feature of mania according to DSM­5.
The correct answer is: Insomnia/hypersomnia

Question 15 MoK Classification 040
Not answered Which of the following is true with regard to classification of psychiatric disorders?

Marked out of 1.00
Select one:
Flag question
Every symptom used to diagnose a disorder can be reliably elicited
ICD­10 is theoretical
Most diagnoses are dependent on few identified core symptoms
Diagnosis obtained using psychodynamic formulation is the most reliable
DSM­IV is theoretical

Your answer is incorrect.

1168
Most diagnoses in descriptive systems are based on few core symptoms and some ancillary
symptoms. But not all of the listed symptoms in a category could be reliably diagnosed. For
example, loss of libido may be more reliable than anhedonia. Both ICD and DSM are
atheoretical. Theoretical assumptions such as psychodynamic theory does not make
classifications reliable.
The correct answer is: Most diagnoses are dependent on few identified core symptoms

Question 16 MoK Clinical Examination 009
Not answered A 76­year­old man presents to the memory clinic with a MMSE score of 23/30. What will you
do in terms of management?
Marked out of 1.00

Flag question Select one:
Wait and watch for 6 months
Repeat memory test in 3 months time
Prescribe antidepressant drugs
Discharge back to GP
Arrange for dementia screen work up

Your answer is incorrect.
MMSE score of 23/30 suggests a possibility of dementia. Consider dementia screen work up,
which includes blood tests, urine tests, ECG and CT scan.
The correct answer is: Arrange for dementia screen work up

Question 17 MoK Clinical Examination 010
Not answered Physical findings that point to a metabolic cause of mental status change include
Marked out of 1.00
Select one:
Flag question
Hyporeflexia
Positive Rhomberg's sign
Positive Babinski sign
Asterixis
Dilated pupils

Your answer is incorrect.
It is helpful to look for asterixis, tremor and myoclonic jerks in patients who are confused and
agitated. These are helpful signs that point to a metabolic or toxic cause of altered mental
status. In the absence of myoclonus, asterixis, or other definitive localizing neurologic signs
on an exam, the examiner should consider various conditions that could explain
bihemispheric involvement. http://www.aan.com/go/education/curricula/internal/chapter4
The correct answer is: Asterixis

1169
Question 18 MoK Clinical Examination 011
Not answered On examination, there is a repeated fluctuant movement of agonist and antagonist muscles
when the patient tries to reach something. Where is the lesion?
Marked out of 1.00

Flag question Select one:
Frontal lobe
Basal ganglia
Parietal lobe

Pons
Cerebellum

Your answer is incorrect.
The description refers to intentional tremor, which is a feature of Cerebellar damage.
Intentional tremors are the result of the lack of the cerebellum's ability to coordinate the timing
of alternately contracting and inhibiting antagonistic muscles, and are often associated with
dysdiadochokinesia.
The correct answer is: Basal ganglia

Question 19 MoK Clinical Examination 012
Not answered Which one of the following eye signs is pathognomonic of multiple sclerosis?

Marked out of 1.00
Select one:
Flag question
Argyll Robertson pupil
Kayser Fleischer ring
Bilateral internuclear ophthalmoplegia
Holmes Adie Pupil
Horner's syndrome

Your answer is incorrect.
In young patients with bilateral internuclear ophthalmoplegia, multiple sclerosis is often the
underlying cause. In older patients with one­sided ophthalmoplegia, stroke is a more common
cause. Internuclear ophthalmoplegia (INO) strems from a dysfunction of the medial
longitudinal fasciculus (MLF), a tract that contains axons projecting from the VI nucleus to the
medial rectus subnuclei of the contralateral III nuclear complex. The cardinal findings in INO
are impaired adduction and abducting nystagmus during conjugate version movements. The
resulting ocular divergence leads to horizontal diplopia. (Ref: Barton JS (2008). Retrieved
from http://www.neuroophthalmology.ca/textbook/disorders­of­eye­movements/)
The correct answer is: Bilateral internuclear ophthalmoplegia

Question 20 MoK Clinical Examination 013

1170
Not answered A 64 year old lady has a history suggestive of frontal lobe dementia. Which of the following
bedside cognitive tests is appropriate to assess frontal lobe function?
Marked out of 1.00

Flag question
Select one:
Rey Osterreith test
Wisconsin card sorting test
Visual field test
Mini mental status examination
Category fluency test

Your answer is incorrect.
Wisconsin card sorting test is not a bed side cognitive test. It is a more detailed
neuropsychological assessment.
The correct answer is: Category fluency test

Question 21 MoK Clinical Examination 014
Not answered Intention tremor is characteristically seen in which of the following conditions?

Marked out of 1.00
Select one:
Flag question
Essential tremor
Cerebellar disease
Anxiety
Lithium toxicity
Frontal lobe disorders

Your answer is incorrect.
Intention tremor is an oscillating tremor that accelerates in pace on approaching the target. It
is one of the neurological signs seen in disorders involving the cerebellum.
The correct answer is: Cerebellar disease

Question 22 MoK Clinical Examination 015
Not answered Which of the following is the single most useful tool for diagnosing dementia?

Marked out of 1.00
Select one:
Flag question
Clinical interview of patient and carer
MMSE
MRI scan
Neuropsychological testing

1171
CT scan

Your answer is incorrect.
Nothing can replace a careful clinical interview for diagnostic evaluation of most disorders
The correct answer is: Clinical interview of patient and carer

Question 23 MoK Clinical Examination 019
Not answered Read the strip of ECG shown below and identify the relevant abnormality. P waves are
marked in the figure.
Marked out of 1.00

Flag question

Select one:
Mobitz type II
Type I second degree heart block
Mobitz type I
Complete heart block
Normal ECG

Your answer is incorrect.
There is a complete interruption of conduction between atria and ventricles so that the two
appear to be working independently. The atrial P waves bear no relationship to the ventricular
QRS complex, which usually arise as the result of a ventricular escape rhythm. (Ref:
Houghton & Gray, Making sense of the ECG. Pg 120­122).
The correct answer is: Complete heart block

Question 24 MoK Descriptive Psychopathology 006
Not answered Which one of the following is not a negative symptom of schizophrenia?
Marked out of 1.00
Select one:
Flag question
Alexithymia
Attentional impairment
Affective flattening or blunting
Anhedonia
Avolition

Your answer is incorrect.

1172
The prominent negative symptoms include affective flattening or blunting, attentional
impairment, avolition­apathy (lack of initiative), anhedonia, asociality, alogia (lack of speech
output).
The correct answer is: Alexithymia

Question 25 MoK Descriptive Psychopathology 008
Not answered Simultaneous presence of contrasting attitudes towards a person, an action or an idea is
called 
Marked out of 1.00

Flag question Select one:
Automatic obedience
Avolition
Ambivalence
Anhedonia
Avoidance

Your answer is incorrect.
Ambivalence is a state of having simultaneous, conflicting feelings toward a person or thing. It
is a term coined by Bleuler, who differentiated between Affective ambivalence­ e.g., To love
and hate the same person at the same time and Intellectual ambivalence­e.g., Assertion and
denial of the same idea. In contrast, ambitendence is the equivalent of the ambivalence of the
will­ (Ambitendence is a catatonic symptom). An ambitendent schizophrenia patient may try to
bring the spoon to his mouth number of times but never completes the act.
The correct answer is: Ambivalence

Question 26 MoK Descriptive Psychopathology 017
Not answered A man with epilepsy answers a question correctly, but goes well past the point before moving
on to the next response. This form of speech is called
Marked out of 1.00

Flag question Select one:
Circumstantiality
Clang association
Verbigeration
Tangentiality
Vorbereiden

Your answer is incorrect.
In circumstantiality, thinking proceeds slowly, with many unnecessary details and digressions,
before returning to the point. It is seen in some patients with temporal lobe epilepsy or
alcohol­induced persisting dementia, learning difficulty and in obsessional personalities.

1173
Circumstantiality must be differentiated from tangentiality ­ the patient never reaches the point
in tangentiality, whereas they do reach the point of discussion in circumstantiality.
The correct answer is: Circumstantiality

Question 27 MoK Descriptive Psychopathology 019
Not answered The most common form of synesthesia is

Marked out of 1.00
Select one:
Flag question
Word taste type
Colour sound type
Colour number type
Touch colour type
Flavour colour type

Your answer is incorrect.
Colour number type is the most common type ­ it is also known as Grapheme colour
synesthesia
The correct answer is: Colour number type

Question 28 MoK Descriptive Psychopathology 020
Not answered Mr. Smith experiences repetitive intrusive images of corpses of his family members in coffins.
He had to imagine the same people alive to reduce the anxiety associated with the
Marked out of 1.00
experience. This compensatory attempt is best described as
Flag question
Select one:
Ruminations
Disaster image
Obsessional image
Disruptive image
Compulsive image

Your answer is incorrect.
Two types of recurrent images are described in OCD. Obsessive image refers to recurrent
images that are recognised as irrelevant, senseless appearing in one's own mind but with no
wilful control. Sometimes patients may voluntarily produce images that are counter­intuitive in
order to negate the feared effects of such obsessive images ­ these are called as compulsive
images.
The correct answer is: Compulsive image

1174
Question 29 MoK Descriptive Psychopathology 021
Not answered On a nice sunny day you are driving along a country road where you have never been before.
You suddenly have a sense of familiarity for the place. This is known as:
Marked out of 1.00

Flag question Select one:
Transient amnesia
Derealisation
Reduplicative paramnesia
Jamais vu
Deja vu

Your answer is incorrect.
Deja vu is classified as a pathology of familiarity. Deja vu is the feeling of having seen or
experienced an event, which is being experienced for the first time. It occurs more frequently
under stress and fatigue, while it declines with age. Deja vu is reported more frequently in
temporal lobe epileptics.
The correct answer is: Deja vu

Question 30 MoK Descriptive Psychopathology 022
Not answered A 49­year­old lady repeatedly presents to police asking them to arrest her for a murder
despite never having committed this crime. This condition is known as:
Marked out of 1.00

Flag question Select one:
Anhedonia
Delusion of guilt
Delusion of reference
Depersonalisation
Nihilistic delusion

Your answer is incorrect.
This lady probably has a delusion of guilt and expects some punishment for an uncommitted
wrongful act. This presentation is common in psychotic depression.
The correct answer is: Delusion of guilt

Question 31 MoK Descriptive Psychopathology 023
Not answered "Today I took a train from Derby to Matlock with Jeff, my dog. I carried my knife and pepper
spray as we walk through the town, as I am convinced someone will abduct and mutilate me
Marked out of 1.00
and kill Jeff". This can be described as a
Flag question
Select one:

1175
Nihilistic delusion
Delusion of misidentification
Delusion of persecution
Delusion of reference
Delusion of control

Your answer is incorrect.
Persecutory delusions are among the most common features of the psychotic illness. Here the
person believes that he/she is being persecuted e.g. being spied upon or being poisoned by
known or unknown others.
The correct answer is: Delusion of persecution

Question 32 MoK Descriptive Psychopathology 024
Not answered A 45­year­old woman in a psychiatric inpatient unit complains that someone is injecting blood
into her body on a regular basis. On further questioning she points to a black mole in her hand
Marked out of 1.00
as an explanation for her belief. This is best termed as
Flag question
Select one:
Delusional perception
Delusional elaboration
Persecutory delusion
Somatic hallucination
Delusional infestation

Your answer is incorrect.
A patient with schizophrenia often actively devotes his or her attention to
understanding/explaining the profound but inscrutable relationships among events and
things, to reduce perplexity. This process of secondary "delusional elaboration" (Hamilton
1984, Slater and Roth 1969) can itself be experienced with the same intensity and conviction
of other primary mental phenomena (e.g. hallucinations).
The correct answer is: Delusional elaboration

Question 33 MoK Basic Psychology 004
Not answered The average capacity of short term memory is

Marked out of 1.00
Select one:
Flag question
7+/_2
9+/_2

5+/_2

1176
3+/_2
6+/_2

Your answer is incorrect.
Short term memory: It holds a small amount of information. The capacity of STM according to
Miller is 7+/­ 2, as evident while testing digit span. By chunking, larger information could be
sorted into 7+/­ 2 items and effectively stored. Chunks are bits of information coordinated
together with the help of long­term memory. Unaided, STM lasts 15 to 30 seconds. By
maintenance rehearsal (repetition of items in one's mind) this duration can be increased
further up to indefinite periods. STM largely uses acoustic coding (mostly) or visual coding.
Recall of information is effortless and usually error­free. Information is held in STM by the
process of rehearsal. Loss of information from STM occurs mainly through displacement
(newly acquired items entering STM displaces existing material) and decay (recently acquired
material has a higher trace strength than older items)
The correct answer is: 7+/_2

Question 34 MoK Basic Psychology 008
Not answered Which one among the following is a mode of retrieval from long­term memory?

Marked out of 1.00
Select one:
Flag question
Recall
Re­learning
Recognition
All of the above
Reintegration

Your answer is incorrect.
Modes of retrieval (LTM store being passed onto STM) are through recognition (solving
MCQs), recall (actively searching and reproducing), re­learning, confabulation and
reintegration (recollection of past experiences based on certain cues). An eyewitness
testimony is a reconstructive memory, which is a mode of retrieval from long­term memory.
However, a reconstructive memory of events as in eyewitness testimony is affected by the type
of questioning asked to elicit the information.
The correct answer is: All of the above

Question 35 MoK Basic Psychology 010
Not answered With regard to attention which of the following is incorrect?

Marked out of 1.00
Select one:
Flag question
Divided attention is paying attention to multiple tasks at a time.
Concentration is sustained attention.

1177
Stroop test is a test for selective attention
Alternating attention is the highest level of attention.
Selective attention is the ability to avoid distractions

Your answer is incorrect.
Divided attention is the highest level of attention, and it refers to the ability to respond
simultaneously to multiple tasks or multiple task demands.
The correct answer is: Alternating attention is the highest level of attention.

Question 36 MoK Basic Psychology 015
Not answered Incorporating "applied tension" is necessary when doing exposure therapy with what
particular group of phobias?
Marked out of 1.00

Flag question Select one:
Blood Injury type
Claustrophobia
Agoraphobia
Situational phobia

Social Phobia

Your answer is incorrect.
Blood/illness/injury phobia is unique in that a paradoxical drop in blood pressure occurs
during exposure to a particular stimulus, leading to fainting. Applied tension involves
contraction of large muscle groups to maintain a normal blood pressure during exposure. This
practice is contrary to the progressive muscle relaxatin that is employed when treating other
forms of anxiety.
The correct answer is: Blood Injury type

Question 37 MoK Basic Psychology 016
Not answered A 45­year­old female patient is undergoing regular chemotherapy for her breast cancer; at her
last visit she became nauseated and vomited as she entered the clinic room. This is likely to
Marked out of 1.00
be related to
Flag question
Select one:
Positive reinforcement
Classical conditioning
Modeling
Negative reinforcement
Habituation

1178
Your answer is incorrect.
Classical conditioning occurs when a patient associates some aspect of the oncology clinic
(e.g., the room, smell, the sight of the intravenous infusion bottle) with the nausea previously
experienced during the intravenous process itself. Thus, the mere sight or smell of one or
more components of the experience can produce the conditioned response of nausea and
vomiting.
The correct answer is: Classical conditioning

Question 38 MoK Basic Psychology 022
Not answered A 45 year old man was admitted to the acquired brain injury unit with severe memory
impairment. Which of the common clinical tests is used in testing his immediate memory?
Marked out of 1.00

Flag question Select one:
Recall of recent topics in news
Recall of distant personal events
Recall of items after 5 minutes
Recall of last meal
Digit span

Your answer is incorrect.
The terms used in psychology and psychiatry are somewhat different which could lead to
some confusion. The terms used in psychology are short term memory (immediate memory in
psychiatry) and long­term memory (recent memory and remote memory in psychiatry). In
psychology, short term memory refers to immediate memory, which is tested by the recall of
digits immediately after their presentation (Digit span). However, in psychiatry it is often
referred to as a test of immediate memory. Recollection of a name and an address after 5
minutes is seen as a test of recent memory (which, in fact, is a part of LTM and should not be
considered as a test of STM)
The correct answer is: Digit span

Question 39 MoK Basic Psychology 024
Not answered According to Aaron Beck, depression results from

Marked out of 1.00
Select one:
Flag question
high expressed emotions
aggression turned inwards
genetic risk
distorted negative thoughts
low expressed emotions

1179
Your answer is incorrect.
Aaron Beck first proposed the theory behind cognitive distortions that are associated with
depression.
The correct answer is: distorted negative thoughts

Question 40 MoK Basic Psychology 026
Not answered With regards to exposure based treatment approaches which of the following is true?

Marked out of 1.00
Select one:
Flag question
Exposure should be sufficiently long for response to occur
Direct exposure to feared object is detrimental
An overtly safe environment should not be created
Experience of anxiety during the first session must be avoided
Patients must be discouraged from being exposed outside the treatment
context

Your answer is incorrect.
Exposure therapy has been shown to be an evidence­based treatment component for
Phobias, GAD, OCD, Panic disorder. There are several variations of exposure therapy (in
vivo, graded, flooding, imaginal, interoceptive, systematic desensitisation). Over time, people
find that their reactions to feared objects or situations decrease. Exposure can help weaken
previously learned associations between feared objects, activities, or situations and bad
outcomes.
The correct answer is: Exposure should be sufficiently long for response to occur

Question 41 MoK Basic Psychology 032
Not answered 2 Antidepressants A and B were introduced. Both A and B had similar mechanism of action
and similar efficacy. A shows 50% failure rate. B shows 50% success rate. Drug B reported
Marked out of 1.00
much better sales when marketed than drug A. Select the cognitive heuristic in this case.
Flag question
Select one:
Anchoring
Simulation
Representation
Availability
Framing

Your answer is incorrect.

1180
Framing is a cognitive heuristic in which people tend to reach conclusions based on the
'framework' within which a situation was presented. Framing­ with reliance on how information
is presented, a judgment is made on the benefit of a choice
The correct answer is: Framing

Question 42 MoK Basic Psychology 034
Not answered Which brain region plays a predominant role in working memory?
Marked out of 1.00
Select one:
Flag question
Hippocampus
Amygdala
Occipital lobe
Frontal cortex
Cerebellum

Your answer is incorrect.
Dorsolateral prefrontal cortex is considered to be a significant seat of working memory
capacity. Impairment of fronto­parietal networks in schizophrenia is thought to be related to
working memory deficits seen in this disorder.
The correct answer is: Frontal cortex

Question 43 MoK Basic Psychology 035
Not answered Which theory of perception is most aligned with the view that "the whole is more than the sum
of the parts"?
Marked out of 1.00

Flag question Select one:
Kohut
Mahler
Freud
Adler
Gestalt

Your answer is incorrect.
When the perceptual system forms a percept or gestalt, the whole thing has a reality of its
own, independent of the parts. The Gestalt psychologist Kurt Koffka made a famous statement
about this: "The whole is other than the sum of its parts." This statement is often translated into
English as, "The whole is greater than the sum of the parts." Koffka did not like that translation.
He firmly corrected students who substituted "greater" for "other" (Heider, 1977). "This is not a
principle of addition," he said. The statement as originally worded was supposed to mean that

1181
the whole had an independent existence in the perceptual system (Excerpt from Dewey RA.
The Whole is Other than the Sum of the Parts. Retrieved from
http://www.intropsych.com/ch04_senses/).
The correct answer is: Gestalt

Question 44 MoK Basic Psychology 036
Not answered Motivational interviewing is associated with

Marked out of 1.00
Select one:
Flag question
Inventiveness
Increasing cognitive dissonance
Negative reinforcement
Premack's principle
Learned resourcefulness

Your answer is incorrect.
Motivational interviewing attempts to encourage people to take on responsibility for their own
life and to deal with their problems themselves. The nature, principles and techniques of MI
are, without exception, found to relate to one or more of the principles of cognitive dissonance.
In creating a cognitive dissonance, however subtly or gently you may do that, you are
inevitably directing the client to where, in many ways, you as the therapist want him to be, to
get him to be open to change. (Ref: "MOTIVATIONAL INTERVIEWING Preparing People to
Change Addictive Behaviour" by W.R. Miller & S. Rollnick, The Guildford Press, New York,
London, 1991)
The correct answer is: Increasing cognitive dissonance

Question 45 MoK Basic Psychology 037
Not answered Sternberg's triarchic theory is a theory of

Marked out of 1.00
Select one:
Flag question
Temperament
Adolescent development
Sociocultural adapatation
Intelligence
Psychosexual development

Your answer is incorrect.
The tendency for attributers to underestimate the impact of situational factors and to
overestimate the role of dispositional factors in controlling behavior' (Ross, 1977, p.183) is
called the fundamental attribution error.
The correct answer is: Intelligence

1182
Question 46 MoK Basic Psychology 038
Not answered The Stanford­Binet test is a test of
Marked out of 1.00
Select one:
Flag question
Behaviour
Personality
Intelligence
Attitude
Memory

Your answer is incorrect.
It is a test of intelligence. Around 50% of the population are said to have an IQ between 90
and 110. In the Stanford­Binet test, IQ is Calculated as, IQ = Measured Age/Chronological Age
multiplied by 100.
The correct answer is: Intelligence

Question 47 MoK Basic Psychology 039
Not answered Which of the following statements about Eysenck's three­factor theory is true?

Marked out of 1.00
Select one:
Flag question
Extraverts is related to autonomic lability
Introversion is associated with autonomic lability
Introverts have a more reactive reticular activating system
Extraverts have a more reactive reticular activating system
Extraversion is related to cortisol levels

Your answer is incorrect.
In introverts, the ascending reticular activating system is over­aroused and excited. In
Extraverts, the ARAS is under­aroused and inhibited.
The correct answer is: Introverts have a more reactive reticular activating system

Question 48 MoK Basic Psychology 040
Not answered According to Yerkes­Dodson Law, the level of arousal is related with performance of an act
by;
Marked out of 1.00

Flag question Select one:
J­shaped curve

1183
U­shaped curve
Straight line
Inverted U­shaped curve
Inverted J­shaped curve

Your answer is incorrect.
Yerkes­Dodson Law: An inverted U­shaped curve relates the level of arousal with the
performance of an act. Optimum arousal (moderate) is required for best performance; too low
or too high arousal proves to be a hindrance and can lead to a poorer performance. The
optimum level of arousal for best performance is also influenced by the nature of the task. A
simple well­rehearsed task will require a higher level of optimum arousal than more complex,
less well­rehearsed tasks.
The correct answer is: Inverted U­shaped curve

Question 49 MoK Social Psychology 003
Not answered Which of the following is a behavioural component of Allport's concept of prejudice?

Marked out of 1.00
Select one:
Flag question
Circumlocution
Stereotype
Discrimination

Hostility
Moral attack

Your answer is incorrect.
Prejudice is essentially an attitude. Hence, it has 1. Cognitive component ­ stereotypes; 2.
Affective component ­ hostility; 3. Behavioural component ­ which according to Allport can be
of several types in terms of increasing severity starting from anti­locution, avoidance,
discrimination, physical attack and in some cases, ending in extermination .
The correct answer is: Discrimination

Question 50 MoK Social Psychology 005
Not answered A police officer arrives at a place to investigate a reported crime. He notices a suspicious
looking man who looks at the officer and runs way without talking to the officer. The police
Marked out of 1.00
team arrests him out of suspicion and is not ready to listen to any of his explanations. But he
Flag question turns out to be not guilty as the investigation proceeds. Which of the following is a likely
explanation?

Select one:
Primacy effect
Hawthorne effect

1184
Halo effect

Barnum effect
Pygmalion effect

Your answer is incorrect.
Halo effect is the tendency to perceive other persons as wholly good or bad based on few
observed traits (e.g. physical attractiveness). Thus a person's positive or negative traits 'spill
over' from one area to influence the total perception of their personality. Investigators
evaluating suspects are susceptible for halo effect (to be accurate ­ reversed halo effect or
devil effect or association fallacy). For example, a policeman may conclude someone is guilty
by association with attributes he has previously seen in other criminals. Mere similarity of a
person to a suspect often causes the police to wrongly associate the innocent with a guilty act.
The correct answer is: Halo effect

Question 51 MoK Sociocultural Psychiatry 003
Not answered Autonomy, beneficience, non­maleficinece in medical ethics are described as

Marked out of 1.00
Select one:
Flag question
Utilitarianism
Teleology

Enshrinement
Principlism
Deontology

Your answer is incorrect.
Principlism refers to Childress and Beauchamp's four prima facie principles in ethics.
The correct answer is: Principlism

Question 52 MoK Sociocultural Psychiatry 006
Not answered In infants born prematurely, after which week of pregnancy are we legally able to resuscitate
an infant irrespective of parental wishes?
Marked out of 1.00

Flag question Select one:
20 wks

16 wks
22 wks
21 wks
24 wks

1185
Your answer is incorrect.
In the United Kingdom women have access to termination of pregnancy for maternal reasons
until 24 weeks' completed gestation, but it is accepted practice for children born at or beyond
25 weeks' gestation to be treated according to the child's perceived best interests even if this
is not in accordance with parental wishes (Retrieved from Campbell AV. Viability and the
moral status of the fetus. Ciba Found Symp. 1985;115:228­43.
http://www.ncbi.nlm.nih.gov/pubmed/3849416). There is no legislation imposing an age limit
in UK for resuscitating premature babies as of now. If this is a choice, this must be chosen.
Boyle et al. Ethics of refusing parental requests to withhold or withdraw treatment from their
premature baby. J Med Ethics.2004; 30: 402­405
The correct answer is: 24 wks

Question 53 MoK Sociocultural Psychiatry 010
Not answered Social deprivation has no role in the aetiology of which of the following disorders?

Marked out of 1.00
Select one:
Flag question
Bipolar Disorder
ADHD
Depression
Alzheimer's disease
Schizophrenia

Your answer is incorrect.
There is no significant causal association between increased risk of Alzheimer's disease (AD)
and socioeconomic status (SES). There is an association between low education and
increased AD risk, but this is not mediated by adult SES or socioeconomic mobility. (Karp et
al., 2003; http://aje.oxfordjournals.org/content/159/2/175)
The correct answer is: Alzheimer's disease

Question 54 MoK Sociocultural Psychiatry 012
Not answered A 29­year­old man with a long history of schizophrenia and currently being effectively treated
in the community decides to donate blood. His treating team feels he has the capacity to make
Marked out of 1.00
decision with respect to this issue. Which of the following principle the treating team has
Flag question adhered to?

Select one:
Maleficence
Non­maleficence
Beneficence
Autonomy
Equality

1186
Your answer is incorrect.
Individual freedom is based on the principle of autonomy in modern medical ethics. It is the
principle that a person should be free to make his or her own decisions.
The correct answer is: Autonomy

Question 55 MoK Sociocultural Psychiatry 018
Not answered An Asian immigrant in England is observed to practice his religion and cultural traditions at
home but adapts well especially at work with good degree of fluency in both English and his
Marked out of 1.00
native language. This type of enculturation is called
Flag question
Select one:
Melting Pot
Bisection
Assimiliation
Accommodation
Biculturalism

Your answer is incorrect.
The acculturative process involves acquisition and retention as well as relinquishing
attitudes/values and practices in both majority and minority populations that continues over
several generations. The rate of change and the circumstances that influence it vary greatly,
both between and within groups. For these reasons, studies of groups experiencing
acculturative change often divide the groups by temporal experience into first­, second­, and
third­generation immigrants. Families within such groups have been categorized as
traditional, transitional, or bicultural. Traditional families are characterized as using their
native tongues rather than English, living in ethnic enclaves, avoiding interaction with majority
cultural institutions, and maintaining preimmigration values and behaviors. Transitional
families are characterized by greater fluency in the language of the host culture and by
children who are becoming familiar with the values and social behaviors of the dominant
majority population through attendance at school and school­related activities. Bicultural
families are defined as those with a high degree of language fluency in their native languages
as well as English, economic stability, and residence in multiethnic settings. Biculturalism
appears to be more adaptive and associated with minimal acculturation stress.
The correct answer is: Biculturalism

Question 56 MoK Sociocultural Psychiatry 020
Not answered Theories of social origins of depression is related to which one of the following pair of
individuals?
Marked out of 1.00

Flag question Select one:
Crow and Johnstone
Ingvar and Franzen
Cerletti and Bini

1187
Watson and Crick
Brown and Harris

Your answer is incorrect.
In 1978, as the result of two community surveys carried out in Camberwell between 1969 and
1975 of women aged 18 and 65 and a series of comparable psychiatric patients, Brown and
Harris published their influential book, 'Social Origins of Depression'.
The correct answer is: Brown and Harris

Question 57 MoK Sociocultural Psychiatry 021
Not answered Which of the following instruments could be used to measure the discriminant attitudes and
level of stigma in a community?
Marked out of 1.00

Flag question Select one:
Changing minds scale
Helsinki Scale of Stigma Burden
Community attitudes to mental illness scale
Openness and discrimination scale
Stigma and Social prejudice scale

Your answer is incorrect.
Community Attitudes to Mental Illness (CAMI) measures attitudes across a number of key
parameters. It was developed specifically for use in the community (in the context of the
emptying of the asylums) and refined to quantify attitudes across four scales: authoritarianism,
benevolence, social destructiveness and community mental health ideology. It was developed
by Taylor and Dear in 1981 ( Psychiatric stigma. The British Journal of Psychiatry,
http://bjp.rcpsych.org/content/178/3/281).
The correct answer is: Community attitudes to mental illness scale

Question 58 MoK Sociocultural Psychiatry 023
Not answered According to Holmes and Rahe's social readjustment rating scale (SRRS) which one was
rated as the most serious life event?
Marked out of 1.00

Flag question Select one:
Death of a spouse
Loss of a job
Violation of law
Moving home
Marriage

1188
Your answer is incorrect.
Holmes and Rahe created a list of 43 life events of carrying severity collected from the
analysis of data from 5000 patient records. Death of a spouse was arbitrarily rated as the most
serious life event with minor violations of law rated as the least serious.
The correct answer is: Death of a spouse

Question 59 MoK Sociocultural Psychiatry 028
Not answered Koro can be classified as a/an

Marked out of 1.00
Select one:
Flag question
Somatisation phenomenon
Autoscopic phenomenon
Desomatisation phenomenon
Heautoscopic phenomenon
Derealisation phenomenon

Your answer is incorrect.
Koro is a culture­bound syndrome. The fear of shrinking genitalia seen in Koro can be
phenomenologically classified as a desomatisation phenomenon.
The correct answer is: Desomatisation phenomenon

Question 60 MoK Sociocultural Psychiatry 029
Not answered A Sri Lankan man presents with depression and fatigue, and attributes it to excessive
masturbation and losing semen in urine. What is the name of his condition?
Marked out of 1.00

Flag question Select one:
Koro
Susto
Dhat
Latah
Piblokto

Your answer is incorrect.
This is called as Dhat syndrome. It is seen in India and some parts of South East Asia. Dhat
refers to the severe anxiety and hypochondriacal concerns associated with the seminal
discharge accompanied by feeling weak and exhausted. Patients may show guilt for
excessive masturbation.
The correct answer is: Dhat

1189
Question 61 MoK HumanDevelopment 006
Not answered Ainsworth constructed a strange situation experiment with
Marked out of 1.00
Select one:
Flag question
1 separation and 3 re­union episodes
2 separation and 3 re­union episodes
2 separation and 2 re­union episodes
3 separation and 1 re­union episodes
1 separation and 2 re­union episodes

Your answer is incorrect.
Ainsworth constructed a strange situation experiment with 2 separation and 2 reunion
episodes. It is a controlled observation carried out in a comfortable room equipped with toys.
An infant is observed in the presence and absence of its mother and a stranger in the vicinity
in seven different combinations. According to the infant's behaviour it is classified as type A, B
or C. A­ Anxious avoidant, B­ Secure, C­Anxious resistant
The correct answer is: 2 separation and 2 re­union episodes

Question 62 MoK HumanDevelopment 008
Not answered Gender identity in a normal child develops around

Marked out of 1.00
Select one:
Flag question
Puberty
Early schooling
4 years of age

2 years of age
1 year of age

Your answer is incorrect.
Gender identity refers to the unshakable self­conviction of being male or female that begins
around 18 months and gets fixed by 24 to 30 months.
The correct answer is: 2 years of age

Question 63 MoK HumanDevelopment 009
Not answered At what age can a child copy a circle with crayon?

Marked out of 1.00
Select one:
Flag question
6 months­ 12 months
3­4 years

1190
4­5 years
1­ 2 years
2­3 years

Your answer is incorrect.
Age 2­3: In terms of fine motor skills, a child can copy a straight line with a crayon. (By 30
months, he or she tries to copy a circle and a plus sign.)
The correct answer is: 2­3 years

Question 64 MoK HumanDevelopment 010
Not answered The range of pubertal time period in a boy lasts for duration of

Marked out of 1.00
Select one:
Flag question
5­6 yrs
6­7yrs
7­8yrs
2­3 yrs
3­5yrs

Your answer is incorrect.
Boys enter puberty at an average age of 11­12 (one year later than girls) but this may occur
anytime. One of the largest cohorts of the pubertal adolescents reported so far has estimated
the duration of puberty in boys to be around 2.4 years; in girls about 1.9 years. In this study,
duration was defined by the difference in ages at onset of a pubertal growth spurt (OGS) and
peak height velocity (PHV). Depending on the method used, the duration of puberty varies
between 2 to 3 years in both sexes, with boys having longer duration than girls consistently
(refer to Tanner's method and Gasser's method for further discussion).
The correct answer is: 2­3 yrs

Question 65 MoK HumanDevelopment 016
Not answered Which of the following is INCORRECT with regard to child development?

Marked out of 1.00
Select one:
Flag question
A 2 years old child can speak more than 50 words
A 5 years old child can tell time by quarter of an hour accuracy
A 12 months old child can speak up to 3 words
A child starts babbling at 3 months of age
A 4 years old child speaks with correct grammar

1191
Your answer is incorrect.
Reception class children (ages 4­5) should be beginning to read the time to the hour, year 1
children (age 5­6) should be able to read the time to the hour or the half hour on analogue
clocks and year 2 children (age 6­7) should be able to read the time to the hour, half hour or
quarter hour on analogue clocks.
The correct answer is: A 5 years old child can tell time by quarter of an hour accuracy

Question 66 MoK HumanDevelopment 017
Not answered When children are hospitalised for physical problems, a short period of separation from
primary care giver ensues. This temporary separation from the loved ones is termed as
Marked out of 1.00

Flag question Select one:
Anaclitic depression
Privation
Deprivation
Pathological separation anxiety
Childhood Greif reaction

Your answer is incorrect.
Anaclitic depression or hospitalism: Spitz described this. When children are hospitalised for
physical problems, a short period of separation from primary caregiver ensues; this loss
produces Anaclitic (object loss) depression. It is thought to be counterproductive to a child's
development. But recovery can be very good if maternal deprivation is kept minimum i.e. less
than three months. To some extent, surrogate mothering can help an infant experiencing
anaclitic depression .
The correct answer is: Anaclitic depression

Question 67 MoK HumanDevelopment 018
Not answered A 4­year­old child asks 'When do clouds sleep, papa?' Which of the following features is she
exhibiting?
Marked out of 1.00

Flag question Select one:
Semiotic thinking
Lack of conservation
Animism
Conservation
Egocentricity

Your answer is incorrect.
Animism refers to the attribution of life and consciousness to certain inanimate objects.

1192
The correct answer is: Animism

Question 68 MoK HumanDevelopment 021
Not answered Mahler is associated with
Marked out of 1.00
Select one:
Flag question
Autistic phase
Conventional morality
Anal phase
Operational stage
Individuality vs. inferiority

Your answer is incorrect.
Margaret Mahler proposed a separation individuation theory in which a normal autistic phase
was described.
The correct answer is: Autistic phase

Question 69 MoK Assessment EMI003
Not answered Obstructive techniques
For each of the following examples, identify the type of obstructive techniques that may
Marked out of 4.00
hamper the progress of information sharing;
Flag question

Suggestive questions
Negative non­verbal gestures
Do you take a proper break from work Postponement
every six months and are you able to
Polythematic questions
relax?
Setting traps
Negative reinforcement
Disapproval
Why questions
Disapproval
Statement of disrespect
Polythematic questions
These voices, which you are Postponement
describing, are not from your head.
Negative non­verbal gestures
Am I right?
Negative reinforcement
Setting traps
Statement of disrespect
Suggestive questions
Negative non­verbal gestures
Why questions
When the patient stated, 'Over the last Postponement
four weeks I have had trouble with my Why questions
sleep', the doctor replied 'We are Disapproval
here to talk about your mood.' Negative reinforcement
Suggestive questions
Polythematic questions
Setting traps
Statement of disrespect

1193
You wanted to see me as nothing
Disapproval
had gone well for you. But you just
Postponement
mentioned that you have got a new
job and entered into a new Why questions
relationship Setting traps
Suggestive questions
Negative reinforcement
Statement of disrespect
Polythematic questions
Negative non­verbal gestures
Your answer is incorrect.
Explanation: 
Compound or polythematic questioning is adding two or more questions in a single
statement. This interview technique confuses the patient and will lead to either a vague
response or non­response.
In  suggestive questions, the answers are contained in the question. It misleads both the
patient and the doctor. The patient is left with little choice. 
Disapproval is expressing unhappiness with a topic that the patient wants to discuss; may
lead to withdrawal and not revealing the important problem faced by the patient. Setting traps
is tricking the patient using his words. Often seen as doctor's attempt to negate patient's
problems.
The correct answer is: Do you take a proper break from work every six months and are you
able to relax? – Polythematic questions, These voices, which you are describing, are not from
your head. Am I right? – Suggestive questions, When the patient stated, 'Over the last four
weeks I have had trouble with my sleep', the doctor replied 'We are here to talk about your
mood.' – Disapproval, You wanted to see me as nothing had gone well for you. But you just
mentioned that you have got a new job and entered into a new relationship – Setting traps

Question 70 MoK Assessment EMI009
Not answered Assessment instruments 
For each description below choose an option from the list of scales given here.
Marked out of 4.00

Flag question
Health of Nations Outcome Scale
General Health Questionnaire
Yale­Brown Obsessive compulsive scale
A 34­year­old woman complains of International Personality Disorder Examination
loss of weight and appetite for last Global assessment of functioning
two months. She has poor sleep and Beck's depression scale
has multiple somatic complaints that Positive and Negative symptoms scale
started when she and her husband Present status examination
separated three months ago. She
Diagnostic interview scale
denies using alcohol.
Minnesota multiphasic personality inventory
CAGE questionnaire

A researcher is keen to evaluate the
degree of self­rated psychiatric
symptoms in a sample of 125 diabetic
outpatients attending an insulin clinic.

1194
CAGE questionnaire
General Health Questionnaire
Global assessment of functioning
Beck's depression scale
Positive and Negative symptoms scale
Yale­Brown Obsessive compulsive scale
Health of Nations Outcome Scale
As a part of a new World Mental International Personality Disorder Examination
Health initiative, it is planned to
Health of Nations Outcome Scale
Present status examination
conduct a nationwide survey of CAGE questionnaire
Minnesota multiphasic personality inventory
mental health burden. It is intended to Present status examination
Diagnostic interview scale
Global assessment of functioning
administer a fully structured
instrument through trained non­ Yale­Brown Obsessive compulsive scale
clinical research workers to ascertain International Personality Disorder Examination
lifetime diagnosis of mental illness. Minnesota multiphasic personality inventory
Diagnostic interview scale
General Health Questionnaire
Positive and Negative symptoms scale
A newly constituted mental health Health of Nations Outcome Scale
General Health Questionnaire
trust wants to adopt a standard scale Minnesota multiphasic personality inventory
Beck's depression scale
Yale­Brown Obsessive compulsive scale
for monitoring clinical recovery on
discharge from different adult units. Global assessment of functioning
Present status examination
Positive and Negative symptoms scale
International Personality Disorder Examination
CAGE questionnaire
Your answer is incorrect. Diagnostic interview scale
Beck's depression scale
Explanation: Beck's depression scale (BDI) is a self­rated questionnaire containing 21
statements with four possible responses for each. 
The General Health Questionnaire (GHQ) is a self­rated screening instrument for the
presence of psychiatric illness. 
The diagnostic interview schedule (DIS) is a non­clinician administered fully structured
interview and used in Epidemiological catchment area study. 
The Health of Nations Outcome Scale (HONOS) is used for monitoring clinical recovery and is
helpful to measure clinical outcomes in mental health.
The correct answer is: A 34­year­old woman complains of loss of weight and appetite for last
two months. She has poor sleep and has multiple somatic complaints that started when she
and her husband separated three months ago. She denies using alcohol. – Beck's depression
scale, A researcher is keen to evaluate the degree of self­rated psychiatric symptoms in a
sample of 125 diabetic outpatients attending an insulin clinic. – General Health
Questionnaire, As a part of a new World Mental Health initiative, it is planned to conduct a
nationwide survey of mental health burden. It is intended to administer a fully structured
instrument through trained non­clinical research workers to ascertain lifetime diagnosis of
mental illness. – Diagnostic interview scale, A newly constituted mental health trust wants to
adopt a standard scale for monitoring clinical recovery on discharge from different adult units.
– Health of Nations Outcome Scale

Question 71 MoK Assessment EMI012
Not answered Psychodynamic reactions
Identify the defence mechanisms involved in each of the following situations:
Marked out of 5.00

Flag question

1195
A 39­year­old woman is diagnosed Repression
with metastatic cervical cancer. She
Isolation
exhibits no emotions with respect to
her physical health but discusses at
Splitting
great length about the pathology of Denial
cervical cancer and anatomy of Regression
cervix. Suppression
Idealisation
Rationalisation
Idealisation
A 34­year­old man is served debt
Intellectualisation
Displacement
collection notice for unpaid bills. He Omnipotence
Isolation
does not react to this note but soon Sublimation
Rationalisation
gets furious at his wife for no Displacement
Intellectualisation
apparent fault of hers. Reaction Formation
Denigration
Denigration
Sublimation
Regression
Regression
A 20­year­old woman is diagnosed Repression
Repression
with borderline personality disorder. Splitting
Displacement
She finds it difficult to trust people Omnipotence
Reaction Formation
and cannot accept that people can Denial
Denial
have both good and bad qualities. Reaction Formation
Splitting
She thinks that everyone in this world Suppression
Isolation
must be either good or bad with no
moderation in between.
Sublimation
Idealisation
Suppression
Denigration
Sublimation
The above­described woman Omnipotence
Reaction Formation
undergoes psychotherapy. She calls Rationalisation
Denigration
her therapist as "the best person in Intellectualisation
Idealisation
the world" within the first few weeks of
uninterrupted sessions.
Repression
Rationalisation
Isolation
Regression
Denial
The same woman calls her therapist
Omnipotence
Omnipotence
the "the worst person in the world"
and demands a change in therapist
Denial
Splitting
Intellectualisation
when the therapist announces a short Repression
break in sessions due to a planned Suppression
Isolation
holiday. Splitting
Sublimation
Displacement
Regression
Displacement
Suppression
Your answer is incorrect. Intellectualisation
Explanation: 
Rationalisation
Denigration
In intellectualization, excessive abstract thinking or factual information are used to control
Idealisation
threatening emotions.
Reaction Formation
Displacement refers to choosing inappropriate but softer targets to vent one's frustrations.
Splitting is said to occur when an individual is unable to see others in moderation, i.e.,
between black and white ­ as grey. Hence, anybody's action is seen as either all good or all
bad. Idealisation refers to classifying someone as 'all good'. Denigration refers to classifying
someone as 'all bad.'

1196
The correct answer is: A 39­year­old woman is diagnosed with metastatic cervical cancer. She
exhibits no emotions with respect to her physical health but discusses at great length about
the pathology of cervical cancer and anatomy of cervix. – Intellectualisation, A 34­year­old
man is served debt collection notice for unpaid bills. He does not react to this note but soon
gets furious at his wife for no apparent fault of hers. – Displacement, A 20­year­old woman is
diagnosed with borderline personality disorder. She finds it difficult to trust people and cannot
accept that people can have both good and bad qualities. She thinks that everyone in this
world must be either good or bad with no moderation in between. – Splitting, The above­
described woman undergoes psychotherapy. She calls her therapist as "the best person in
the world" within the first few weeks of uninterrupted sessions. – Idealisation, The same
woman calls her therapist the "the worst person in the world" and demands a change in
therapist when the therapist announces a short break in sessions due to a planned holiday. –
Denigration

Question 72 MoK Assessment EMI015
Not answered Gait abnormalities in clinical practice
Choose best option for each of the following:
Marked out of 4.00

Flag question
Broad­based and unsteady gait
Myopathic gait
Stiff legged scissoring gait
Parkinson's disease
Festinant gait (shuffling gait)
Lurching chaotic gait
Stomping gait
High stepping gait (Steppage gait)
Pigeon gait
Waddling gait
Waddling gait
Pigeon gait
Failure of gait ignition
Stiff legged scissoring gait
Cerebellar disease
High stepping gait (Steppage gait)
Stomping gait
Lurching chaotic gait
Myopathic gait
Festinant gait (shuffling gait)
Lurching chaotic gait
Failure of gait ignition
Stomping gait
Broad­based and unsteady gait
High stepping gait (Steppage gait)
Upper motor neuron dysfunction
Stiff legged scissoring gait
Festinant gait (shuffling gait)
Pigeon gait
Waddling gait
Broad­based and unsteady gait
Myopathic gait
Myopathic gait
Lurching chaotic gait
Failure of gait ignition
Broad­based and unsteady gait
Foot drop Festinant gait (shuffling gait)
Pigeon gait
Failure of gait ignition
Waddling gait
Stomping gait
High stepping gait (Steppage gait)
Stiff legged scissoring gait
Your answer is incorrect.

1197
Explanation: In shuffling gait, the individual takes short steps to the point of practically not
moving forward. In some patients, the steps vary with a tendency for the patient to accelerate
(festinating gait) as he or she walks. This is seen in Parkinson disease. 
A patient with ataxia may spread his or her legs apart to widen the base of support to offset the
imbalance while standing or walking. The heel­to­toe or tandem walking manoeuvres and
standing on one leg uncover subtle forms of ataxia / broad­based gait. 
In diplegia (UMN damage) associated with cerebral palsy, the legs are held in adduction
while hip and the thighs brush against each other upon walking. A spasm of the inner thigh
muscles make the knees tend to slide over each other like the blades of scissors (Scissoring
gait). 
Steppage gait is seen in chronic peripheral neuropathies and can be the result of the
functional elongation of the legs due to bilateral drop foot. The gait is made of high steps as if
climbing a flight of stairs while walking on a level surface. 

Ref: Neurological History and Physical Examination,
http://emedicine.medscape.com/article/1147993­overview (accessed March 31, 2015).
The correct answer is: Parkinson's disease – Festinant gait (shuffling gait), Cerebellar disease
– Broad­based and unsteady gait, Upper motor neuron dysfunction – Stiff legged scissoring
gait, Foot drop – High stepping gait (Steppage gait)

Question 73 MoK Assessment EMI017
Not answered Disturbed thinking processes
For each description given below choose the most appropriate term from the list above;
Marked out of 5.00

Flag question
Logoclonia
Replies to questions are irrelevant or Thought blocking
oblique. The response usually refers Flight of Ideas
to the relevant topic but fails to give a Clanging
complete answer Pressure of speech
Derailment
Perseveration
Neologisms
Neologisms
Circumstantiality
Flight of Ideas
The individual cannot express ideas
Tangentiality
Pressure of speech
as quickly as they come into his
head, presenting with fragmented Logoclonia
thoughts, abrupt changes in the topic Derailment
and general incoherence. Circumstantiality
Thought blocking
Clanging
Perseveration
Thought blocking
Tangentiality
Neologisms
When answering questions, the Flight of Ideas
patient gets stuck on the last syllable
Logoclonia
of a word and repeats it
Circumstantiality
Clanging
Derailment
Pressure of speech
Perseveration
Tangentiality

1198
Abrupt interruption in a train of
Logoclonia
thinking before an idea is complete
Tangentiality
Clanging
Perseveration
Derailment
Thought blocking
Pressure of speech
Flight of Ideas
Derailment
Neologisms
Circumstantiality
Rapid production of speech output Circumstantiality
Clanging
with a subjective feeling of racing
Tangentiality
thoughts
Flight of Ideas
Pressure of speech
Thought blocking
Neologisms
Perseveration
Your answer is incorrect. Logoclonia
Explanation: In tangentiality the replies to questions are irrelevant or oblique. The response
usually refers to the appropriate topic but fails to give a complete answer. The patient never
reaches the point in tangentiality, whereas they do reach the point in circumstantiality. In flight
of ideas, the individual cannot express ideas as quickly as they come into his head, leading to
fragmented thoughts, abrupt changes in the topic and general incoherence. It is characteristic
of mania. In logoclonia, the patient gets stuck on the last syllable of a word and repeats it. It is
often a symptom of Parkinson's disease. Thought blocking (or deprivation) is the abrupt
interruption in a train of thinking before a thought or idea is finished. Following a pause, the
patient may report no recall of what was being said or what was going to be said. Pressure of
speech refers to the rapid production of speech output with a subjective feeling of racing
thoughts often seen in mania.
The correct answer is: Replies to questions are irrelevant or oblique. The response usually
refers to the relevant topic but fails to give a complete answer – Tangentiality, The individual
cannot express ideas as quickly as they come into his head, presenting with fragmented
thoughts, abrupt changes in the topic and general incoherence. – Flight of Ideas, When
answering questions, the patient gets stuck on the last syllable of a word and repeats it –
Logoclonia, Abrupt interruption in a train of thinking before an idea is complete – Thought
blocking, Rapid production of speech output with a subjective feeling of racing thoughts –
Pressure of speech

Question 74 MoK Assessment EMI018
Not answered Varieties of Delusions
For each of the descriptions below choose the most closely associated psychopathological
Marked out of 4.00
terms from the given list.
Flag question

Delusions of infestations
Delusions of grandiosity
Delusions of control
Autochthonous delusions
A 32­year­old psychotic patient with
persistent auditory hallucinations is
Delusional perception
now developing a belief that a Delusional elaboration
transmitter has been placed in his Persecutory delusions
ear. Delusions of reference
Delusional memory

1199
A 21­year­old psychotic patient
Persecutory delusions
remembers his parents taking him for Delusions of control
an operation as a child becoming Delusional perception
convinced that he had been Autochthonous delusions
implanted with monitoring devices, Delusional elaboration
which have become active in his life. Delusional misidentification
Delusions of reference
Delusions of grandiosity
Delusional memory
Delusional memory
Delusional perception
A 29­year­old psychotic patient with
an odd belief that certain individuals
Delusions of infestations
Delusional elaboration
are not who they externally appear to Autochthonous delusions
be. Delusions of grandiosity
Persecutory delusions
Delusions of reference
Delusions of control
Delusions of reference
Delusions of infestations
Delusional misidentification
A 33­year­old psychotic patient is
Delusional misidentification
Autochthonous delusions
convinced that the Mafia is after him
and are conspiring to ruin his life Persecutory delusions
Delusions of grandiosity
Delusional perception
Delusional memory
Delusions of infestations
Delusions of control
Your answer is incorrect. Delusional elaboration
Explanation: 
Secondary delusions that arise when a patient attempts to find an explanation for primary
psychopathological processes (in case 1, a hallucination) are called delusional elaborations.
In the above scenario, a patient with auditory hallucinations is now developing a belief that a
transmitter has been placed in his ear. 
Delusional memory is a primary delusion, which is recalled as arising as a result of a memory.
In the above scenario, a psychotic patient remembers his parents taking him for an operation
as a child (memory). However, in a delusional manner he is now convinced that control and
monitoring devices were implanted. Delusional memory can also be a retrospective delusion
where something that never happened and so false, irrational or bizarre is reported as if it has
occurred in the past. 
Delusional misidentification is a delusional belief that certain individuals are not who they
externally appear to be. It includes Capgras and Fregoli syndrome). 
A persecutory delusion is a belief that one's life is being interfered with in a harmful way. 
(Ref: Oxford Handbook of Psychiatry 1e­Pg 84­87)
The correct answer is: A 32­year­old psychotic patient with persistent auditory hallucinations
is now developing a belief that a transmitter has been placed in his ear. – Delusional
elaboration, A 21­year­old psychotic patient remembers his parents taking him for an
operation as a child becoming convinced that he had been implanted with monitoring
devices, which have become active in his life. – Delusional memory, A 29­year­old psychotic
patient with an odd belief that certain individuals are not who they externally appear to be. –
Delusional misidentification, A 33­year­old psychotic patient is convinced that the Mafia is
after him and are conspiring to ruin his life – Persecutory delusions

1200
Question 75 MoK Assessment EMI020
Not answered Sensory disturbances
For each of the description below choose one correct option from the given list:
Marked out of 3.00

Flag question
Illusions
Reflex hallucinations
A 22­year­old man can smell music Hyperesthesia
after consuming an unknown drug
Somatic passivity
cocktail
Delusional misrepresentation
Hypoaesthesia
Delusional misperception
Referential delusion
Delusional misperception
Allodynia
Somatic passivity
A 44­year­old man walking down a Synaesthesia
Reflex hallucinations
dimly lit road perceives fearsome Delusional perception
Synaesthesia
shadows beneath trees
Delusional misrepresentation
Hypoaesthesia
Referential delusion
Hyperesthesia
Delusional misperception
Delusional perception
Allodynia
A 77­year­old man admitted with
delirium has altered sensory Allodynia
Synaesthesia
threshold and the nurses have to Illusions
Referential delusion
shout aloud to communicate with him Reflex hallucinations
Delusional perception
Delusional misrepresentation
Hypoaesthesia
Somatic passivity
Your answer is incorrect. Illusions
Hyperesthesia
Explanation: Synaesthesia is characterised by object perception in a modality different from
that of the original stimulus. It can occur in subjects using hallucinogens. Illusion described in
question 2 refers to affect illusion where a fearful emotional state contributes to
misperceptions. In some cases of altered sensorium (delirium), hypoacusis is commonly seen.
The correct answer is: A 22­year­old man can smell music after consuming an unknown drug
cocktail – Synaesthesia, A 44­year­old man walking down a dimly lit road perceives fearsome
shadows beneath trees – Illusions, A 77­year­old man admitted with delirium has altered
sensory threshold and the nurses have to shout aloud to communicate with him –
Hypoaesthesia

Question 76 MoK Assessment EMI035
Not answered Jung and Analytical Psychology
For each description given below, select the most appropriate Jungian concept from the list
Marked out of 3.00
above
Flag question

1201
A concept regarded as universally
Collective unconscious
common to mankind. It contains
Self
various archetypes described by
Jung.
Analytical perception
Archetypes
Animus
Shadow
Introversion
Anima
Self
Personal unconscious
Extroversion
Images and symbols that represent Extroversion
Analytical perception
human experience common to Persona
Animus
different cultures
Persona
Anima
Introversion
Personal unconscious
Personal unconscious
Shadow
Archetypes
Archetypes
The unconscious feminine aspects of Shadow
man Collective unconscious
Collective unconscious
Animus
Anima
Persona
Introversion
Self
Your answer is incorrect. Extroversion
Analytical perception
Explanation: Jung founded analytic psychology and introduced the terms extraversion and
introversion. He described collective unconscious ­ a term to describe all of the mankind's
collective symbolic past, and archetypes ­ like images and symbols that constitute the
collective unconscious. These representational images have universal symbolic meaning
(e.g. Hero, Old Wise Man, Tree, etc.). Other terms described by Jung include Persona which is
the mask covering one's personality and presented to outside world, Anima ­ unconscious
feminine aspect of a man, Animus ­ unconscious masculine character of a woman, Shadow ­
an archetype ­ a personification of unacceptable aspects of oneself symbolized as a dark
internal alien and Individuation ­ ultimate goal of life where an individual develops a sense of
self­ identity.
The correct answer is: A concept regarded as universally common to mankind. It contains
various archetypes described by Jung. – Collective unconscious, Images and symbols that
represent human experience common to different cultures – Archetypes, The unconscious
feminine aspects of man – Anima

Finish review

1202
 Home Mock Paper Practice Tests

Mock Paper A(1)

Started on Wednesday, 1 July 2015, 5:07 PM
State Finished
Completed on Wednesday, 1 July 2015, 5:07 PM
Time taken 13 secs
Grade 0.00 out of 100.00
Feedback You are on the right track. Could you spend some more time on your revision?

Question 1 MoK Classification 012
Not answered Which of the following is a cardinal feature of depression?

Marked out of 1.00
Select one:
Flag question
Anhedonia
Pessimism
Suicidality

Somatisation
Guilt

Your answer is incorrect.
The role of anhedonia in the diagnosis of melancholia of the past years was considered to be
crucial. Klein regarded anhedonia to be a symptom of depressive illness and probably the
best clinical marker predicting response to antidepressant drugs. In 1980, the revised DSM
presented the concept of anhedonia as one of the two cardinal symptoms of major depression
( Anhedonia: a neglected symptom of psychopathology,
http://www.ncbi.nlm.nih.gov/pubmed/8134519).
The correct answer is: Anhedonia

Question 2 MoK Classification 022
Not answered Which of the following clinical features suggests a diagnosis of mania rather than hypomania?

Marked out of 1.00
Select one:
Flag question
Decreased need for sleep
Psychomotor agitation

Inflated self esteem
Flight of ideas

1203
Delusions of grandiosity

Your answer is incorrect.
Psychotic symptoms are suggestive of mania rather than hypomania.
The correct answer is: Delusions of grandiosity

Question 3 MoK Classification 025
Not answered Mrs Pinto lost her husband 9 months ago. She had continued attending her bingo sessions
and cathedral groups since his death, but recently she has had pervasive guilt and suicidal
Marked out of 1.00
thoughts. What is the most appropriate diagnosis?
Flag question
Select one:
Depression
Paraphrenia

Delusional disorder
Bereavement
Dementia

Your answer is incorrect.
The presence of delusional guilt or intense suicidal ideas indicates a major depressive
episode rather than bereavement, especially after nine months of loss.
The correct answer is: Depression

Question 4 MoK Classification 027
Not answered A 37­year­old woman was admitted to the medical ward with history of sudden onset aphonia.
Physical examination and investigations were normal. Her boyfriend has recently invited his
Marked out of 1.00
mother to stay with them permanently, as his mother is feeling lonely and isolated. What is the
Flag question most likely diagnosis according to ICD­10 criteria?

Select one:
Adjustment disorder

Munchausen syndrome
Hypochondriasis
Somatisation disorder
Dissociative motor disorder

Your answer is incorrect.

1204
Functional weakness describes weakness which is inconsistent and incongruent with a
neurological disease. It is also referred to as motor conversion disorder (DSM­IV), dissociative
motor disorder (ICD­10) and 'psychogenic' paralysis. Studies of aetiology have focused on
risk factors such as childhood adversity and life events.(Stone et al. 2011)
The correct answer is: Dissociative motor disorder

Question 5 MoK Classification 032

Not answered A soldier is referred with symptoms of PTSD following return from Iraq. Which of the following
feature is not consistent with the diagnosis of PTSD?
Marked out of 1.00

Flag question Select one:
He is not able to enjoy watching television
He is not able to share his war experience with his close friends
He sleeps in his day time clothes
He is keen to go to Iraq again
He has sustained only mild injuries physicall

Your answer is incorrect.
Avoidance is a part of PTSD ­ wanting to go back seems odd.
The correct answer is: He is keen to go to Iraq again

Question 6 MoK Classification 034
Not answered Mrs Broadbent has been diagnosed with compensation neurosis. Which of the following is
true regarding this?
Marked out of 1.00

Flag question Select one:
Severe difficulties with sleep are noted
Most patients do not return to work even after settlement
Higher the severity of injury, greater the symptoms of compensation
neurosis
Absence of frontal headaches
Improvements tend to occur after financial settlement

Your answer is incorrect.
Compensation neurosis is not an ICD diagnosis; its validity is questionable. Patients show
scant improvement even after financial settlement. A follow­up study of 760 litigants found
that, of the 264 subjects who were not working at the time of conclusion of litigation and who
could be traced, 198 (75%) were not working after an average of 23 months following the
finalization of their cases. (Mendelson, 1995. Compensation neurosis revisited. Journal of
Psychosomatic Research, 39, pp. 695­706)
The correct answer is: Most patients do not return to work even after settlement

1205
Question 7 MoK Classification 037
Not answered An elderly man presents with fluctuation in consciousness and visual hallucinations. His wife
reports that he is not able to turn in his bed properly. You suspect
Marked out of 1.00

Flag question Select one:
Vascular dementia
Huntington's disease
Lewy body dementia
Wilson disease
Alzheimer's dementia

Your answer is incorrect.
Not able to turn in bed indicates axial rigidity that may be related to Parkinsonism. This patient
has Lewy body dementia.
The correct answer is: Lewy body dementia

Question 8 MoK Classification 039
Not answered Which of the following is consistent with a diagnosis of bipolar affective disorder, currently
hypomanic episode?
Marked out of 1.00

Flag question Select one:
Mood incongruent delusions
2 weeks duration of symptoms
Fleeting hallucinations

Minimal disturbance of psychosocial function
Mood congruent delusions

Your answer is incorrect.
The presence of delusions or hallucinations should prompt to make a diagnosis of mania, not
hypomania. By definition, the impairment in psychosocial functioning is of higher degree in
mania than hypomania.
The correct answer is: Minimal disturbance of psychosocial function

Question 9 MoK Classification 041
Not answered Which of the following features is included in ICD­10 diagnosis for generalised anxiety
disorder?
Marked out of 1.00

Flag question Select one:

1206
Nightmares
Panic attacks
Depression

Low self confidence
Motor tension

Your answer is incorrect.
To diagnose GAD, the sufferer must have primary symptoms of anxiety most days for at least
several weeks at a time, and usually for several months. These symptoms should usually
involve elements of 1. apprehension (worries about future misfortunes, feeling 'on edge',
difficulty in concentrating, etc.); 2. motor tension (restless fidgeting, tension headaches,
trembling, inability to relax); and 3. autonomic overactivity (lightheadedness, sweating,
tachycardia or tachypnoea, epigastric discomfort, dizziness, dry mouth, etc.).
The correct answer is: Motor tension

Question 10 MoK Classification 042
Not answered A 33­year­old lady presents with excessive day time sleepiness and auditory hallucinations
when waking from sleep. She also recalls episodes of feeling 'pinned down' to her bed even
Marked out of 1.00
after waking up. The likely diagnosis is
Flag question
Select one:
Schizophrenia
REM behavioural disorder

Narcolepsy
Sleep terror
Primary hypersomnia

Your answer is incorrect.
The symptoms described here are hypnopompic hallucinations and sleep paralysis. These
are characteristic of narcolepsy.
The correct answer is: Narcolepsy

Question 11 MoK Classification 043
Not answered Which one of the following is a feature of atypical Anorexia Nervosa?

Marked out of 1.00
Select one:
Flag question
Depressed mood
Normal menstruation

Binge eating

1207
Purging behaviour
Restricting behaviour

Your answer is incorrect.
Atypical anorexia refers to an absence of one or more core symptoms needed to diagnose
anorexia nervosa i.e. BMI criteria, body image criteria or amenorrhea.
The correct answer is: Normal menstruation

Question 12 MoK Classification 044
Not answered Bleuler discarded the term dementia praecox primarily because
Marked out of 1.00
Select one:
Flag question
Mania was commonly coexistent with dementia praecox
Not all cases began during adolescence
Praecox affect was rarely seen in the early cases
No treatment was available for dementia praecox
No cognitive impairment was noted in the early cases

Your answer is incorrect.
Kraepelin's original formulation of 'dementia praecox' was rejected by Bleuler, who favoured
'the group of schizophrenias' to describe the psychotic condition. Bleuler argued that the term
dementia praecox was based on the notions of the age of onset and the illness course, for
which convincing evidence was lacking. Bleuler emphasized the multiplicity of symptoms and
considered the most important symptom to be fragmentation in the formulation and expression
of thought, referred to as 'loosening of associations.'
The correct answer is: Not all cases began during adolescence

Question 13 MoK Classification 045
Not answered Mrs. Z arrives at work at least an hour earlier than anyone else to plan the day. She becomes
upset if the cleaners have moved anything on her desk and in her office, which is very tidy.
Marked out of 1.00
Her home and car are spotlessly clean. This is best described as;
Flag question
Select one:
Obsessional personality trait
Normal phenomenon
Hoarding behaviour
Obsessive compulsive disorder
Compulsive behaviour

1208
Your answer is incorrect.
Obsessive­compulsive personality disorder (OCPD) is a personality disorder characterized by
excessive perfectionism and a need for control over one's environment. People with OCPD
tend to be reliable and orderly, but also inflexible and unable to surrender control. This makes
it hard for people with this condition to express their feelings, have close relationships and
enjoy their successes. This lady is exhibiting some of the above traits.
The correct answer is: Obsessional personality trait

Question 14 MoK Classification 046
Not answered Which of the following is true with respect to adjustment disorder?
Marked out of 1.00
Select one:
Flag question
usually requires prolonged treatment
correlates with the severity of the stressor
occurs more often in males than in females
occurs in all age groups

it is a type of bereavement

Your answer is incorrect.
Although an adjustment disorder can occur at any age, it is more common at times in life when
major transitions occur, such as adolescence, mid­life, and late­life (Mental Health:
Adjustment Disorder ­ WebMD, http://www.webmd.com/mental­health/mental­health­
adjustment­disorder).
The correct answer is: occurs in all age groups

Question 15 MoK Classification 047
Not answered With respect to models of diagnostic classification, which of the following is correct?
Marked out of 1.00
Select one:
Flag question
Ontological theories suggest that mental state does not exist by virtue of its
abstract nature
Fulford claims that psychiatric diagnoses are not value laden
ICD­10 is entirely atheoretical
Pheneticists stress on the natural existence of hierarchical structure among
disease entities
Categorical entities are not used in modern classification

Your answer is incorrect.

1209
In contrast to the essentialist strategy that treats diseases as entities that exist in nature and
form natural groups in essence, phenetic approach, (also called numerical taxonomy) shifts
the emphasis to the systematic description of the observed disease presentations to enable
manual classification. This approach treats all characters and attributes as having equal
weight. While ICD and DSM are atheoretical in terms of assuming aetiology and causal
models, they are not entirely atheoretical in descriptions used (e.g. dissociative disorder,
conversion, etc. employ psychodynamic concepts).
The correct answer is: Ontological theories suggest that mental state does not exist by virtue
of its abstract nature

Question 16 MoK Clinical Examination 016
Not answered Which of the following assessment is not a part of MMSE?
Marked out of 1.00
Select one:
Flag question
Repeating phrase
Clock Drawing
Motor sequence
Testing attention
Copying polygons

Your answer is incorrect.
Clock drawing provides complementary information, but not a part of MMSE.
The correct answer is: Clock Drawing

Question 17 MoK Clinical Examination 018
Not answered Which of the following is a frontal lobe test?
Marked out of 1.00
Select one:
Flag question
Repertory Grid
Line bisection test
Star cancellation test
Cognitive Estimates Test
Bar chart test

Your answer is incorrect.
Cognitive estimates test is a part of the frontal assessment battery. It includes questions such
as 'What is the average height of a British woman?'
The correct answer is: Cognitive Estimates Test

1210
Question 18 MoK Clinical Examination 020
Not answered Subtracting 7 out of 100 repeatedly (serial sevens test) is most useful to test one's
Marked out of 1.00
Select one:
Flag question
Attention
Alertness
Concentration
Working memory
Motivation

Your answer is incorrect.
Concentration is also known as sustained attention.
The correct answer is: Concentration

Question 19 MoK Clinical Examination 023
Not answered A 56­year­old woman is brought to the hospital with a stroke. A CT scan shows evidence of
left occipital lobe damage. Which of the following is most likely to be seen?
Marked out of 1.00

Flag question Select one:
Quadrantanopsia
Cortical blindness
Anosmia
Bitemporal hemianopsia
Apraxia

Your answer is incorrect.
Cortical blindness is the total or partial loss of vision in a normal­appearing eye caused by
damage to the occipital cortex of the brain. Cortical blindness can be acquired or congenital,
and may also be transient in certain instances. Acquired cortical blindness is most often
caused by loss of blood flow to the occipital cortex from either unilateral or bilateral posterior
cerebral artery blockage due to ischemic stroke. In most cases, the patient may recover some
of their vision
The correct answer is: Cortical blindness

Question 20 MoK Clinical Examination 024
Not answered A boxer is knocked out by a blow to his lateral skull. Despite recovering after a few minutes,
and being asymptomatic for the next 12 hours, he develops severe headache, vomiting and
Marked out of 1.00
reduced consciousness. Which of the following is the most indicated investigation?
Flag question
Select one:

1211
X ray skull
SPECT scan
EEG
MRI brain
CT scan brain

Your answer is incorrect.
In cases of suspected bony fracture and fresh hemorrhage, a CT scan is more useful than
plain X­ray, which cannot pick up the bleed or MRI brain which is slower to obtain and may
miss fresh bleeding.
The correct answer is: CT scan brain

Question 21 MoK Clinical Examination 025
Not answered Hyperhomocysteinemia is associated with increased risk for which of the following
conditions?
Marked out of 1.00

Flag question Select one:
Alcoholism
Borderline personality disorder

Panic disorder
Self mutilation
Dementia

Your answer is incorrect.
An increased plasma homocysteine level is a strong, independent risk factor for the
development of dementia and Alzheimer's disease. N Engl J Med 2002; 346:476­483
The correct answer is: Dementia

Question 22 MoK Clinical Examination 026
Not answered Which of following is not commonly seen in nicotinic acid deficiency?

Marked out of 1.00
Select one:
Flag question
Insomnia
Diarrhoea
Glossitis
Dermatitis
Dementia

1212
Your answer is incorrect.
Dementia occurs only in an extreme form of nicotinic acid deficiency. Commonly, acute
deficiency causes pellagra characterized by 3 Ds ­ diarrhoea, dermatitis and disturbed mental
functions.
The correct answer is: Dementia

Question 23 MoK Clinical Examination 027
Not answered Which of the following best describes ideomotor apraxia?
Marked out of 1.00
Select one:
Flag question
Inability to coordinate and carry out facial and lip movements
Inability to construct elements to a meaningful whole
Difficulty in learning new information
Inability to perform a multiple­step task.
Difficulty in copying skilled movements

Your answer is incorrect.
Ideomotor apraxia is a disorder of goal­directed movement. The patient knows what to do but
not how to do it. There is an inability to mimic a learned motor task or behavior, such as
combing one's hair. Ideomotor apraxia results from damage to the dominant parietal lobe.
Abnormalities include (i) abnormal amplitude of motor movements (ii) body­part­as­object
substitution, e.g. the patient uses his own finger to represent a toothbrush when asked to
brush his teeth; (iii) abnormal orientation of body parts when attempting to perform an action.
The correct answer is: Difficulty in copying skilled movements

Question 24 MoK Clinical Examination 028
Not answered While screening for alcohol use problems, which of the following questions is the best one to
be asked first?
Marked out of 1.00

Flag question Select one:
Have you ever felt bad or guilty about your drinking?
Have people annoyed you by getting on your case about your drinking?
Do you enjoy a drink now and then?
Have you ever needed to take a drink first thing in the morning?
Have you felt you should cut down on your drinking?

Your answer is incorrect.
This question is not about CAGE. It is about sensitive interview techniques. A sensitive topic
must be gradually introduced with non­judgemental tone.
The correct answer is: Do you enjoy a drink now and then?

1213
Question 25 MoK Clinical Examination 029
Not answered A patient with schizoaffective disorder is on both lithium and risperidone. He develops tremors
as a side effect of his treatment. Which of the following will help detecting the offending agent?
Marked out of 1.00

Flag question Select one:
Finger nose test
Peripheral visual field
Examination of tone of joints
Frontal release sign
Physical examination for tendon reflexes

Your answer is incorrect.
EPSE with risperidone can cause tremors (Parkinsonian) and rigidity, unlike Lithium tremor
where one would not expect rigidity.
The correct answer is: Examination of tone of joints

Question 26 MoK Descriptive Psychopathology 018
Not answered When asked where do he lived when he was a child, a patient replied "Winchester.
Manchester City was my favourite football team." The type of speech abnormality exhibited is
Marked out of 1.00

Flag question Select one:
Clang association
Pressured speech
Tangential
Coprolalia
Circumstantial

Your answer is incorrect.
This is an example of clang association, which is an abnormality of speech where the
connection between words is their sound rather than their meaning. It may often occur in
patients with mania.
The correct answer is: Clang association

Question 27 MoK Descriptive Psychopathology 025
Not answered Capgras syndrome is:

Marked out of 1.00
Select one:
Flag question
A form of negative autoscopy

1214
A form of autoscopy
Also called the phantom mirror image
A perceptual abnormality
Delusional misidentification

Your answer is incorrect.
Capgras syndrome is included in delusional misidentification syndromes along with
reduplicative paramnesia, Fregoli's syndrome and intermetamorphosis. It is not a perceptual
error. Phantom mirror image refers to autoscopy which is a perceptual phenomenon.
The correct answer is: Delusional misidentification

Question 28 MoK Descriptive Psychopathology 027
Not answered A 20­year­old man is unable to express his problem coherently but appears very confused
and feels that 'something is happening but not sure what it is'. This sense of perplexity and
Marked out of 1.00
uncertainty that exists during a prodrome of psychosis may be linked most appropriately to
Flag question which of the following experiences?

Select one:
Negative symptoms
Depression
Delusional memory
Delusional mood
Autochthonous delusions

Your answer is incorrect.
Delusional mood refers to a state of perplexity in which the patient has some sense of some
inexplicable change in his environment. He senses 'something going on' which he cannot
identify, but which has a peculiar significance for him.
The correct answer is: Delusional mood

Question 29 MoK Descriptive Psychopathology 030
Not answered A 23­year­old man notices the inscribed image of the Queen on a coin, and says "I know now,
my mother is an alien". This is an example of
Marked out of 1.00

Flag question Select one:
Ideas of reference
Delusional mood
Delusional memory
Delusional misidentification

1215
Delusional perception

Your answer is incorrect.
This is delusional perception that is a primary delusion, which is recalled as having arisen as
a result of a perception. The percept is a real external object, not a hallucinatory experience.
The correct answer is: Delusional perception

Question 30 MoK Descriptive Psychopathology 033
Not answered A 43­year­old woman, who was involved in a serious flight accident, felt at the time of the
accident as if the world had ceased to exist. For some time she lost her sense of time
Marked out of 1.00
perception and felt detached from the rescue proceedings around her. This is known as:
Flag question
Select one:
Nihilistic delusion
Derealisation
Suicidal thought
Depersonalisation
None of the above

Your answer is incorrect.
In depersonalisation, an unpleasant sense of detachment can occur. Disturbed time
perception and feelings of 'unreality' are also noted. In isolated derealisation (which is rare in
the absence of depersonalisation), one may not have a personal sense of detachment but
may feel that the world around them is unreal, sometimes 'robotic', made of cardboards!
The correct answer is: Depersonalisation

Question 31 MoK Descriptive Psychopathology 034
Not answered One among the following statements is NOT true about depersonalisation?

Marked out of 1.00
Select one:
Flag question
Depersonalisation is easily distinguished from derealizatio
Depersonalisation is always subjective
Depersonalisation is frequently accompanied by the symptom of
derealization
Depersonalisation is a frequent experience of normal healthy people
Affect is invariably involved in Depersonalisation

Your answer is incorrect.

1216
It is hard to accurately distinguish the two clinically.
The correct answer is: Depersonalisation is easily distinguished from derealizatio

Question 32 MoK Descriptive Psychopathology 035
Not answered Of various thought disorders classified by Andreasen, which of the following are more
common in schizophrenia than mania?
Marked out of 1.00

Flag question Select one:
Derailment
Circumstantiality
Neologisms
Clanging
Flight of ideas

Your answer is incorrect.
Of various thought disorders classified by Andreasen, clanging and flight are more common in
mania while derailment (loosening) and thought blocking and to some extent tangentiality,
poverty of content of speech are often seen in schizophrenia ­ other items were largely non­
specific
The correct answer is: Derailment

Question 33 MoK Descriptive Psychopathology 038
Not answered The phenomenon in which a subject perceives other people's heads to be as big as their
actual body size can be termed as
Marked out of 1.00

Flag question Select one:
Pareidolia
Dysmegalopsia
Lilliputian Hallucinations
Micropsia
Macropsia

Your answer is incorrect.
Dysmegalopsia is characterised by alterations in the symmetry/size of perceived objects.
The correct answer is: Dysmegalopsia

Question 34 MoK Descriptive Psychopathology 039
Not answered A patient repeats whatever the examiner says. He is exhibiting

1217
Marked out of 1.00 Select one:
Flag question Perseveration
Logoclonia
Parapraxis
Stereotypy
Echolalia

Your answer is incorrect.
Echolalia is an echophenomenon seen in catatonia. Other echophenomena are echopraxia
(repeating what you do) and mirror movements ( doing as you do at same time)
The correct answer is: Echolalia

Question 35 MoK Descriptive Psychopathology 040
Not answered The nature of speech produced in Broca's aphasia is

Marked out of 1.00
Select one:
Flag question
Fluent and intelligible
Impaired speech comprehension
Fluent but unintelligible
Effortful but agrammatism is seen
Effortful but with grammatical use of language

Your answer is incorrect.
Broca's aphasia is characterised by retained use of nouns and verbs, but the patient may
have lost all forms of pronouns, articles, and conjunctions and hence may struggle to speak
more than one word at a time even with enormous effort.
The correct answer is: Effortful but agrammatism is seen

Question 36 MoK Descriptive Psychopathology 041
Not answered Mr X was admitted to the psychiatric unit with a history of elated mood, overactivity,
restlessness, grandiose delusions, disinhibited behaviour and was diagnosed with mania. A
Marked out of 1.00
student nurse observed that he also experienced third person auditory hallucinations and
Flag question wondered whether first rank symptoms could occur in manic patients. He wanted to clarify this
with the treating psychiatric doctor in the team. Choose the most appropriate statement from
the following:

Select one:
First rank symptoms never occur in manic patients
First rank symptoms may present in 10­25% of manic patients

1218
First rank symptoms are very rarely seen in manic patients
First rank symptoms are present in all cases of mania with psychosis
First rank symptoms are present in nearly half of all patients with mania

Your answer is incorrect.
First rank symptoms are not pathognomonic of schizophrenia. They are observed to occur on
bipolar mania, epilepsy (TLE) and also in organic psychotic states.
The correct answer is: First rank symptoms may present in 10­25% of manic patients

Question 37 MoK Dynamic Psychopathology 001
Not answered Which of the following defence mechanisms is most prominent in subjects with narcissism?

Marked out of 1.00
Select one:
Flag question
Idealisation
Discrimination
Acting out
Anticipation
Projection

Your answer is incorrect.
Projection is commonly seen in narcissism. It involves taking one's own unacceptable
qualities or feelings and ascribing them to other people. For example, a narcissist with a
strong dislike for someone, might instead believe that the other person does not like him.
The correct answer is: Projection

Question 38 MoK Dynamic Psychopathology 002
Not answered Avoidance is a defence mechanism most commonly used in
Marked out of 1.00
Select one:
Flag question
OCD
Agoraphobia
Hypochondriasis
Somatoform disorder
Depression

Your answer is incorrect.

1219
Avoidance is a powerful cognitive and behavioural strategy that serves as a negative
reinforcer to reduce anxiety associated with agoraphobia. (Most questions on defence
mechanism/cognitive styles are based on psychotherapeutic theoretical grounding; not on
quantitative evidence regarding frequency and prevalence.)
The correct answer is: Agoraphobia

Question 39 MoK Dynamic Psychopathology 003
Not answered You are a CT1 psychiatric trainee. One of your patients on the ward who is a 62­year­old man
has an acute onset chest pain and sweating. Accompanying ECG showed ST elevation in
Marked out of 1.00
leads V1­4. What is the abnormality?
Flag question
Select one:
Sick sinus syndrome
Anterior MI
First degree block
Inferior MI
Prolonged QTc

Your answer is incorrect.
ST elevation and/or Q waves in V1­4/V5 is suggestive of anterior wall MI
The correct answer is: Anterior MI

Question 40 MoK Dynamic Psychopathology 004
Not answered Which of the following defence mechanisms involves realistic planning for future discomfort?
Marked out of 1.00
Select one:
Flag question
Depreciation
Anticipation
Altruism
Reaction formation
Denial

Your answer is incorrect.
Anticipation is a mature defence mechanism
The correct answer is: Anticipation

Question 41 MoK Dynamic Psychopathology 005
Not answered The expression of ideas, thoughts, and suppressed material that is accompanied by an
emotional response producing a state of relief in the patient is called as

1220
Marked out of 1.00 Select one:
Flag question Democratisation
Modelling
Mirroring
Rapprochement
Catharsis

Your answer is incorrect.
Catharsis is the expression of ideas, thoughts and suppressed material that is accompanied
by an emotional response that produces a state of relief in the patient
The correct answer is: Catharsis

Question 42 MoK Dynamic Psychopathology 008
Not answered Which of the following ego defence mechanism has been put forth as an explanation for
phobia?
Marked out of 1.00

Flag question Select one:
Denial
Intellectualization
Reaction formation
Splitting
Displacement

Your answer is incorrect.
Freud invoked the mechanism of displacement to explain the fear of horses in Little Hans. The
fear of his father was displaced on to horses.
The correct answer is: Displacement

Question 43 MoK Basic Psychology 005
Not answered The span of immediate memory is estimated to be around

Marked out of 1.00
Select one:
Flag question
8±2
9±2

5±2
3±2
7±2

1221
Your answer is incorrect.
The 'magical number seven' (G. A. Miller; PA, Vol 31:2914, ) demonstrated the concept of
limited channel capacity and specified 7 (+/­2) as the immediate memory span. Though the
span is limited, the immediate memory capacity is relatively insensitive to the amount of
information per item. Miller, and later Baddeley emphasized the importance of the recoding of
information (the concept of chunking).
The correct answer is: 7±2

Question 44 MoK Basic Psychology 009
Not answered Which one of the following theories is not associated with encoding failure of forgetting?

Marked out of 1.00
Select one:
Flag question
State dependent memory
Blocking
Failure of prospective memory
Context dependent memory
All of the above

Your answer is incorrect.
All the above theories are associated with retrieval failure of forgetting and not due to
encoding failure. Encoding failure: Forgetting over time may occur due to defective or
impaired initial encoding of information into memory. It may also be due to shallow processing
of information. Retrieval failure; This is the commonest cause of failure to access long term
memory. Although the memory trace is there, the route to retrieving is blocked, which can be
facilitated by cues.
The correct answer is: All of the above

Question 45 MoK Basic Psychology 011
Not answered Abnormalities in Stroop test suggests

Marked out of 1.00
Select one:
Flag question
Cortical blindness
Lesion of optic chiasma
Attentional bias
Short term memory disturbance
Damage to occipital cortex

Your answer is incorrect.

1222
Abnormal Stroop effect with the loss of appropriate inhibitory control is seen in frontal lobe
lesions. Abnormalities in modified emotional stroop interference effects (e.g. on threat words
in anxious subjects on the emotional Stroop task) have generally been interpreted as
evidence for mood­congruent attentional bias in various mental states.
The correct answer is: Attentional bias

Question 46 MoK Basic Psychology 017
Not answered Tom is a 4­year­old boy. He has a rabbit at home. When he reached out to stroke the rabbit, a
hammer is struck against an iron rod causing a loud noise, which frightens him. This is
Marked out of 1.00
repeated 5 times. As a result of this, the sight of the rabbit on its own now frightens Tom. This
Flag question type of conditioning is called

Select one:
Classical conditioning
Operant conditioning
Observational learning
Stimulus discrimination
Trace conditioning

Your answer is incorrect.
Classical conditioning is a process of behaviour modification by which a subject comes to
respond in a desired manner to a previously neutral stimulus that has been repeatedly
presented along with an unconditioned stimulus that elicits the desired response
The correct answer is: Classical conditioning

Question 47 MoK Basic Psychology 018
Not answered The process which leads to the transfer of information from short­term memory to long­term
memory is called:
Marked out of 1.00

Flag question Select one:
Rehearsal
Encoding
Consolidation
Displacement
Interference

Your answer is incorrect.
Rehearsal facilitates the transfer of items in short term memory to long term storage.
The correct answer is: Consolidation

1223
Question 48 MoK Basic Psychology 023
Not answered All of the following are tests of recent memory except
Marked out of 1.00
Select one:
Flag question
Name and address recall task
Rey­osterrieth complex figure test
Three word learning task
Wechsler memory scale
Digit span

Your answer is incorrect.
Digit span is the commonest test of auditory, verbal working memory. Both forward and
backward digit span are tested in routine clinical practice. The normal range of digit forwards
is 6+/­1 and for reverse digit span is 5+/­1 Three word learning task (E.g. apple, table, penny)
is a test of Anterograde memory and learning. Name and address recall task (7 items) is the
commonest test of recent memory. Here the subject is asked to recall as many items, without
prompts, in five or ten minutes.
The correct answer is: Digit span

Question 49 MoK Basic Psychology 028
Not answered A middle­aged woman treated for agoraphobia has improved with repeated exposure to
crowds. Which psychological process is involved here?
Marked out of 1.00

Flag question Select one:
Instrumental conditioning
Massed practices
Cognitive restructuring
Counter­conditioning
Extinction

Your answer is incorrect.
Extinction is the conditioning phenomenon in which a previously learned response to a cue is
reduced when the cue is presented in the absence of the previously paired stimulus
The correct answer is: Extinction

Question 50 MoK Basic Psychology 029
Not answered If a parent intentionally ignores a child's misbehavior, this is an example of which operant
conditioning principle?
Marked out of 1.00

Flag question Select one:

1224
Positive reinforcement
Punishment
Extinction.
Negative reinforcement
Shaping

Your answer is incorrect.
Negative reinforcement, by definition, is the application of an event that increases the
probability of a target behavior occurring in the future.
The correct answer is: Extinction.

Question 51 MoK Basic Psychology 043
Not answered Whose theory of emotions proposed that emotion is secondary to physiological changes and
that there is a distinct pattern of changes for different emotions?
Marked out of 1.00

Flag question Select one:
Cannon baird theory
Cognitive appraisal theory
James Lange theory
Schema theory
None of the above

Your answer is incorrect.
According to James­Lange theory emotion, is secondary to physiological changes and that
there is a distinct pattern of changes for different emotions. The Cannon­Baird theory of
emotion states that the subjective experience of emotions is independent of physiological
changes. Any stimulus is processed by the thalamus that sends impulses to both the cortex
and the hypothalamus. The cortex is responsible for emotions while the hypothalamus is
responsible for physiological changes. The cognitive appraisal conscious or unconscious
always precedes emotional experience. Lazarus proposed this theory.
The correct answer is: James Lange theory

Question 52 MoK Basic Psychology 044
Not answered Who postulated that thinking occurs peripherally in the speech apparatus?

Marked out of 1.00
Select one:
Flag question
Sternberg
John Watson
Whorf

1225
Spearman
Maslow

Your answer is incorrect.
The Linguistic relativity hypothesis also called as the Whorfian hypothesis, places language
as primary and thoughts as secondary. But in peripheralism, John Watson postulated that
thinking occurs peripherally in the speech apparatus.
The correct answer is: John Watson

Question 53 MoK Basic Psychology 045
Not answered The following are correctly paired with the concepts they introduced:
Marked out of 1.00
Select one:
Flag question
Freud : negative and positive symptoms
Jung : introversion
Eugen Bleuler : dementia praecox
Manfred Bleuler : dementia praecox
Charcot : dissociation

Your answer is incorrect.
Jung developed a personality typology that distinguished between introversion and
extroversion. Introverts are people who prefer their internal world of thoughts, feelings,
fantasies, dreams, and so on, while extroverts prefer the external world of things and people
and activities.
The correct answer is: Jung : introversion

Question 54 MoK Social Psychology 004
Not answered Which one among the following is not one of the Gestalt principles of perception?

Marked out of 1.00
Select one:
Flag question
Proximity
Similarity
Closure
Continuity
Familiarity

Your answer is incorrect.

1226
Gestalt law of perceptual organization includes Proximity: Objects close to each other are
perceived as one figure, Closure: Incompletely closed figures are perceived as fully closed,
Continuity: Continuous items are perceived together, Similarity: Similar items are grouped
together based on colour or shape, etc. and Common fate: Things moving together are
perceived together.
The correct answer is: Familiarity

Question 55 MoK Social Psychology 006
Not answered Beautiful people are generally viewed as more kind, convincing and effective. This is called

Marked out of 1.00
Select one:
Flag question
Barnum effect
Fundamental attribution error
Halo effect
Pygmalion effect
Hawthorne effect

Your answer is incorrect.
Halo effect refers to our tendency to allow one specific trait or our overall impression of a
person to influence our judgment of their other traits. This is also called the physical
attractiveness stereotype or "what is beautiful is good" principle.
The correct answer is: Halo effect

Question 56 MoK Sociocultural Psychiatry 014
Not answered Which of the following ethical principles does the issue of capacity to make treatment
decisions uphold?
Marked out of 1.00

Flag question Select one:
Non maleficience
Justice
Confidentiality
Autonomy
Beneficience

Your answer is incorrect.
Individual freedom is based on the principle of autonomy in modern medical ethics. It is the
principle that a person should be free to make his or her own decisions.
The correct answer is: Autonomy

1227
Question 57 MoK Sociocultural Psychiatry 019
Not answered Under the principle of necessity which of the following can be done without patient's consent?

Marked out of 1.00
Select one:
Flag question
Covert administration of medication in a competent non consenting adult
A psychiatric inpatient who needs overnight detention as he attempts to
leave
Sterilisation of a patient with repeated postpartum mania

Liver transplantation to a patient who consumed overdose of paracetamol
Blood sample drawn to confirm suspected myocardial infarction in a
schizophrenia patient

Your answer is incorrect.
Emergency interventions for life­saving measures could be carried out under common law or
necessity principle. Irreversible or long term procedures cannot be carried out without
formalised legal procedures.
The correct answer is: Blood sample drawn to confirm suspected myocardial infarction in a
schizophrenia patient

Question 58 MoK Sociocultural Psychiatry 022
Not answered Tarasoff case is related to which of the following ethical principles?

Marked out of 1.00
Select one:
Flag question
Autonomy
Beneficience

Confidentiality
Justice
Capacity

Your answer is incorrect.
Tarasoff case refers to US federal law and highlights the duty to warn third parties who may be
directly harmed by a patient, in addition to a doctor's duty of care towards patients. Tatiana
Tarasoff was killed by Prasanjit Poddar, who divulged his intentions to his therapist before the
homicide. Though the therapist arranged to admit him, no one directly warned Tatiana
Tarasoff. Following his discharge from a brief hospitalization, Poddar killed Tarasoff. Turner M,
Kennedy M. Tarasoff and the duty to warn third parties. Psychiatric Bull 1997; 21: 465­466;
The correct answer is: Confidentiality

Question 59 MoK Sociocultural Psychiatry 024
Not answered

1228
Marked out of 1.00 According to the Holmes and Rahe Social readjustment scale, which of the following has the
highest 'Life Change' value?
Flag question

Select one:
Pregnancy
Retirement
Death of a spouse

Marriage
Minor violation of law

Your answer is incorrect.
The Holmes and Rahe stress scale is a list of 43 stressful life events that can contribute to
mental/physical illness. When using Holmes and Rahe Stress Scale, the number of "Life
Change Units" for relevant events in the past year of one's life are summed up to provide a
final score of total stress exposure. The top one on the list is the death of a spouse followed by
divorce.
The correct answer is: Death of a spouse

Question 60 MoK Sociocultural Psychiatry 027
Not answered Which of the following ethical theories concerns itself with the actions of an individual, rather
than the thinking that lead up to it or the consequences?
Marked out of 1.00

Flag question Select one:
Deontology
Teleology

Consequentialism
Utilitarianism
Virtue theory

Your answer is incorrect.
Deontological ethics derives the rightness or wrongness of one's conduct from the character
of the behaviour itself rather than the outcomes of the conduct.
The correct answer is: Deontology

Question 61 MoK Sociocultural Psychiatry 031
Not answered A patient demands to have a CT scan of her head to evaluate her migrainous headaches.
Your clinical judgment suggests that the pain is due to a migraine and there is no necessity for
Marked out of 1.00
a CT scan.She does not agree and repeatedly asks for the scan. The best option in this
Flag question situation is

Select one:

1229
Do not order the CT scan and ask her to seek medical care elsewhere
Do not order the CT scan, but discharge her from your care
Order the CT scan so that the patient will not lodge a complaint with
hospital managers.

Do not order the CT scan, but offer to refer her for a second opinion if she
wishes to consider this.
Ask her to arrange for a privately paid CT scan if she is willing to pay for
this.

Your answer is incorrect.
Ordering unnecessary tests is both inappropriate utilization of health care resources and
needlessly expensive. The CT scan should not be ordered for this patient if it is not indicated,
regardless of their request.Provided the professional adheres to an existing level of standard
of care to this patient, then a malpractice or negligence should not be of concern. There is not
a clear reason to discharge the patient from the caseload completely and to do so may be
construed as patient abandonment. The best choice, in this case, is to offer the patient an
opportunity to get a second opinion on her symptoms.
The correct answer is: Do not order the CT scan, but offer to refer her for a second opinion if
she wishes to consider this.

Question 62 MoK Sociocultural Psychiatry 032
Not answered In some families, contradictory parental demands on children lead to confusion and
frustration, increasing the risk of emotional difficulties. Which of the following term best
Marked out of 1.00
captures this situation?
Flag question
Select one:
Learned Helplessness
Rapproachment

Marital Schism
Double bind
Projective identification

Your answer is incorrect.
The essential hypothesis of the double bind theory is that the 'victim'­the person who becomes
psychotically unwell­finds him or herself in a communicational matrix, in which messages
contradict each other. These are described as injunctions as below: Primary Negative
Injunction: 'I'll kill you if you stay.' i.e. 'Do not stay.' Secondary Negative Injunction: 'I am used
to people leaving me.' i.e. 'Do not abandon me.' Tertiary Negative Injunction: 'That's right. It is
so typical of you to irritate me and then leave me. No guts to face your problems. I'm prepared
to stick here'. i.e. Don't leave.
The correct answer is: Double bind

1230
Question 63 MoK Sociocultural Psychiatry 033
Not answered Which of the following is not a risk factor for ADHD?

Marked out of 1.00
Select one:
Flag question
Large size of the family
Family history of ADHD
Low birth weight

Low educational attainment in parents
Food additives

Your answer is incorrect.
There is no proven link between food additives and ADHD. ADHD and its common
complications are associated with low social class, large family size, paternal antisocial
personality, and maternal mental disorder.
The correct answer is: Food additives

Question 64 MoK Sociocultural Psychiatry 037
Not answered All of the following factors commonly influence the basic features of health explanatory belief
models except
Marked out of 1.00

Flag question Select one:
history of good physical health
age

time­point and context of assessment
educational status
gender

Your answer is incorrect.
Kleinman's work in the development of a health explanatory belief model has led to an
increased appreciation of the role of a patients' understanding of the causes of an illness
when attempting to treat the illness. Age, gender, cultural background, time and context of
assessment and educational status can influence one's explanatory models. Ref: Kramer et
al., West J Med. 2002 Sep; 176(4): 227­231.
The correct answer is: history of good physical health

Question 65 MoK HumanDevelopment 012
Not answered The average age of the earliest retrieved memory is

Marked out of 1.00
Select one:
Flag question
2.5 years

1231
3.5 years
5 years
4.5 years
2 years

Your answer is incorrect.
The average age of the earliest retrieved memory is 3.5 years. There is a total lack of
memories for events occurring during the first few years of life, and there is amnesia for events
that occurred in the first 2 to 5 years. This is also termed as infantile amnesia.
The correct answer is: 3.5 years

Question 66 MoK HumanDevelopment 013
Not answered Infants exhibit the capacity for colour vision and accommodation at the age of
Marked out of 1.00
Select one:
Flag question
4 months
2 months
1 month
6 months
Birth

Your answer is incorrect.
At birth, infants are able to track and scan objects, can discriminate levels of brightness, able
to fix objects, fixed focus at about 20 cm, figure­ground discrimination and can have slightly
impaired visual acuity. At 1month, they can differentiate faces with preference shown for
complex stimuli. At 2 months­ possesses depth perception, prefers 3 dimensional rather than
2 dimensional representations of a face. At 4 months, colour vision and accommodation
develops. At 6 months accurate acuity is achieved(6:6).
The correct answer is: 4 months

Question 67 MoK HumanDevelopment 019
Not answered In a Strange Situation experiment, a 2­year­old girl continues to play without getting disturbed
when her mother leaves the room and does not show any response when she comes back.
Marked out of 1.00
What type of attachment is seen here?
Flag question
Select one:
Absence of attachment
Disorganised attachment
Resistant attachment

1232
Secure attachment
Anxious avoidant attachment

Your answer is incorrect.
Anxious Avoidant attachment is characterized by anxiety and fright within the child and signs
of not trusting the mother resulting in an indifferent attitude towards mother leaving the room
or entering the room. Distress, when left alone, can be comforted by a stranger rather easily.
The correct answer is: Anxious avoidant attachment

Question 68 MoK HumanDevelopment 022
Not answered A boy can identify that two balls are of the same size even though they appear to be of
different sizes due to a distance illusion. This corresponds to which stages of Paget's cognitive
Marked out of 1.00
development?
Flag question
Select one:
Formal operational
Operational stage
Sensorimotor
Pre­operational
Concrete operational

Your answer is incorrect.
In concrete operational stage children start achieving 'conservation'. They are successful in
conservation tasks involving numbers, quantity and volume by 7­8 years. The child realizes
that certain properties like volume, number, length and mass of objects remain unchanged
even when the objects appearances are altered.
The correct answer is: Concrete operational

Question 69 MoK Assessment EMI008
Not answered Interview techniques
For each of the following statements given below, identify the interview technique used.
Marked out of 5.00

Flag question
Question rephrasing
Summation
You seem very upset when talking Limit setting
about your girlfriend. Are you having
Confrontation
any problems with your relationship?
Symptom expectation
Symptom exaggeration
Redirection
Facilitation
Interpretation
Clarification

1233
The psychiatrist uses approval nods, Interpretation
leaning forwards slightly to express
Symptom expectation
interest, using responses such as
'Yes. And then?' 'Yeah, go on..', 'Uh­
Limit setting
huh', etc. Confrontation
Summation
Question rephrasing
Symptom exaggeration
Clarification
Redirection
Facilitation
Summation
You mentioned having taken
overdoses since your last admission.
Redirection
Limit setting
How many times have you taken Question rephrasing
overdoses in total? Seven...? Eight...? Clarification
Symptom exaggeration
Confrontation
Facilitation
Symptom expectation
Symptom expectation
Limit setting
Interpretation
Facilitation
Clarification
What sort of recreational drugs do Summation
you usually use when you are Redirection
drinking? Symptom exaggeration
Confrontation
Interpretation
Question rephrasing

The patient said 'It is not good if one's
Symptom expectation
parents are divorced even before one Clarification
goes to school.' The doctor replied 'I'd Redirection
like to hear more about your parents, Question rephrasing
but first let me get a picture of what's Summation
happening to you of late.' Interpretation
Confrontation
Facilitation
Limit setting
Your answer is incorrect. Symptom exaggeration
Explanation: Interpretation is clarifying certain associations or relationships that the patient
may not see. It is a sophisticated technique and should be used only after the doctor has
established some rapport. Should be stated as a hypothesis after sufficient collection of
evidence from the interview. 
Facilitation is helping patients continue the interview by providing both verbal and nonverbal
encouragement. It helps patient to feel that the doctor is listening actively. Encourages flow of
information. 
Symptom exaggeration: When deception or minimisation is expected, overstating a guessed
frequency in order to elicit a true answer. It is also helpful in reducing guilt to a certain extent
as the patient feels that the doctor has expected a higher amount of problem that what she/he
has brought. 
Symptom expectation: Without a formal admission from the patient, asking about details of
problem behaviour. The doctor assumes (rightly) that the patient is involved in the act. 

1234
Redirection is gently reorienting patient towards the topic of discussion. The motive is to keep
the patient back on track. 
Ref: Adapted from Kay J & Tasman A. Essentials of Psychiatry, 2nd edition, 2006. John Wiley
& Sons, Ltd.
The correct answer is: You seem very upset when talking about your girlfriend. Are you having
any problems with your relationship? – Interpretation, The psychiatrist uses approval nods,
leaning forwards slightly to express interest, using responses such as 'Yes. And then?' 'Yeah,
go on..', 'Uh­huh', etc. – Facilitation, You mentioned having taken overdoses since your last
admission. How many times have you taken overdoses in total? Seven...? Eight...? –
Symptom exaggeration, What sort of recreational drugs do you usually use when you are
drinking? – Symptom expectation, The patient said 'It is not good if one's parents are divorced
even before one goes to school.' The doctor replied 'I'd like to hear more about your parents,
but first let me get a picture of what's happening to you of late.' – Redirection

Question 70 MoK Assessment EMI013
Not answered Maladaptive defences
For each of the disorders below choose the most prominent defence mechanisms used.
Marked out of 3.00

Flag question
Introjection
Splitting
Denigration
Fugue reactions (one)
Rationalisation
Idealisation
Suppression
Dissociation
Conversion
Regression
Isolation
Denigration
Reaction Formation
Isolation
Depression (one) Denial
Rationalisation
Regression
Sublimation
Omnipotence
Introjection
Sublimation
Splitting
Idealisation
Suppression
Reaction Formation
Rationalisation
Conversion
Isolation
Pseudoseizures (one) Omnipotence
Denial
Suppression
Introjection
Dissociation
Idealisation
Denial
Omnipotence
Splitting
Reaction Formation
Your answer is incorrect.Dissociation
Regression
Explanation: Dissociation is the predominant defence seen in dissociative fugue and
Conversion
amnesia. This involves temporarily but drastically modifying one's sense of personal identity
Sublimation
or mental functions to avoid emotional distress.Moving back into childish or earlier
Denigration
developmental phase to avoid confronting a conflict is called as regression. Regression may
explain the withdrawal and tearfulness seen in depression.Conversion refers to the switching
of mental conflicts into somatic symptoms.
The correct answer is: Fugue reactions (one) – Dissociation, Depression (one) – Regression,
Pseudoseizures (one) – Conversion

1235
Question 71 MoK Assessment EMI014
Not answered Rating scales in clinical use
Identify the relevant rating scales for each of the following conditions
Marked out of 4.00

Flag question
General Health Questionnaire
ChooseTWO observer­rated tests to Simpson­Angus Scale
monitor depression in a woman Edinburgh postnatal depression scale
recently diagnosed with postnatal Scale for Assessment of Positive Symptoms
depression (Choose Two) Hamilton Depression scale
Present state examination
Beck's depression inventory
Brief Psychiatric Rating Scale
General Health Questionnaire
Montgomery Asberg Depression Rating Scale
Simpson­Angus Scale
Choose the most appropriate tool to Edinburgh postnatal depression scale
screen for depression in mothers
Brief Psychiatric Rating Scale
following childbirth
Beck's depression inventory
Hamilton Depression scale
Montgomery Asberg Depression Rating Scale
Scale for Assessment of Positive Symptoms
Edinburgh postnatal depression scale
Present state examination
Brief Psychiatric Rating Scale
Choose one scale to assess EPSEs Scale for Assessment of Positive Symptoms
due to antipsychotic use. General Health Questionnaire
Hamilton Depression scale
Present state examination
Montgomery Asberg Depression Rating Scale
Simpson­Angus Scale
Beck's depression inventory
Your answer is incorrect.
Explanation: 
Hamilton and Montgomery Asberg depression rating scales are observer­rated tests used to
monitor depression. 
The Edinburgh Postnatal Depression Scale (EPDS) have been used to screen for depression
in women during the antepartum and postpartum periods. Edinburgh Postnatal Depression
Scale was specially developed for assessing postpartum depression and relies much less on
somatic questions. The questions on the Edinburgh scale (10 items, can be self or clinician­
rated) are framed within the "past seven days", and the response format is frequency­based.
Routine use of EPDS during the postpartum period has been shown to increase the detection
of postnatal depression compared with usual care, but it is not sensitive to detect a change in
the severity. T
he Simpson­Angus Scale is helpful to assess EPSEs due to the use of antipsychotic
medications. It is less sensitive than AIMS (Abnormal Involuntary Movement Scale) scale to
detect tardive dyskinesia. 

Screening for Depression Across the Lifespan: A Review ..,
http://www.aafp.org/afp/2002/0915/p1001.html (accessed March 31, 2015).
The correct answer is: ChooseTWO observer­rated tests to monitor depression in a woman
recently diagnosed with postnatal depression (Choose Two) – Hamilton Depression scale,
Montgomery Asberg Depression Rating Scale, Choose the most appropriate tool to screen for

1236
depression in mothers following childbirth – Edinburgh postnatal depression scale, Choose
one scale to assess EPSEs due to antipsychotic use. – Simpson­Angus Scale

Question 72 MoK Assessment EMI018
Not answered Varieties of Delusions
For each of the descriptions below choose the most closely associated psychopathological
Marked out of 4.00
terms from the given list.
Flag question

Delusional elaboration
A 32­year­old psychotic patient with Delusions of infestations
persistent auditory hallucinations is Delusions of reference
now developing a belief that a
Delusions of grandiosity
transmitter has been placed in his
ear.
Persecutory delusions
Delusional perception
Autochthonous delusions
Delusions of control
Delusional perception
A 21­year­old psychotic patient
Delusional misidentification
Autochthonous delusions
remembers his parents taking him for
an operation as a child becoming Delusional memory
Delusions of grandiosity
convinced that he had been Delusional memory
implanted with monitoring devices, Delusions of reference
which have become active in his life. Persecutory delusions
Delusional elaboration
Delusions of infestations
Persecutory delusions
Delusions of control
Delusions of reference
A 29­year­old psychotic patient with
an odd belief that certain individuals
Delusional misidentification
Delusional memory
are not who they externally appear to Delusions of infestations
be. Delusional misidentification
Delusional perception
Delusions of control
Delusions of grandiosity
Persecutory delusions
Delusional elaboration
Autochthonous delusions
A 33­year­old psychotic patient is Autochthonous delusions
Delusional elaboration
convinced that the Mafia is after him
Delusional memory
and are conspiring to ruin his life
Delusions of infestations
Delusions of control
Delusional perception
Delusional misidentification
Delusions of grandiosity
Your answer is incorrect. Delusions of reference
Explanation: 
Secondary delusions that arise when a patient attempts to find an explanation for primary
psychopathological processes (in case 1, a hallucination) are called delusional elaborations.
In the above scenario, a patient with auditory hallucinations is now developing a belief that a
transmitter has been placed in his ear. 
Delusional memory is a primary delusion, which is recalled as arising as a result of a memory.
In the above scenario, a psychotic patient remembers his parents taking him for an operation
as a child (memory). However, in a delusional manner he is now convinced that control and

1237
monitoring devices were implanted. Delusional memory can also be a retrospective delusion
where something that never happened and so false, irrational or bizarre is reported as if it has
occurred in the past. 
Delusional misidentification is a delusional belief that certain individuals are not who they
externally appear to be. It includes Capgras and Fregoli syndrome). 
A persecutory delusion is a belief that one's life is being interfered with in a harmful way. 
(Ref: Oxford Handbook of Psychiatry 1e­Pg 84­87)
The correct answer is: A 32­year­old psychotic patient with persistent auditory hallucinations
is now developing a belief that a transmitter has been placed in his ear. – Delusional
elaboration, A 21­year­old psychotic patient remembers his parents taking him for an
operation as a child becoming convinced that he had been implanted with monitoring
devices, which have become active in his life. – Delusional memory, A 29­year­old psychotic
patient with an odd belief that certain individuals are not who they externally appear to be. –
Delusional misidentification, A 33­year­old psychotic patient is convinced that the Mafia is
after him and are conspiring to ruin his life – Persecutory delusions

Question 73 MoK Assessment EMI021
Not answered Disorders of perception
Which of the listed descriptive psychopathological terms refers to the following symptoms?
Marked out of 5.00

Flag question
Kinaesthetic hallucination
A 34­year­old gentleman in the Reflex hallucination
psychiatric outpatient clinic Hypnagogic hallucination
mentioned 'I heard a voice calling my
Affect illusions
name which then startled me back to
wakefulness to find no one there.'
Hypnopompic hallucination
Hygric hallucinations
Complete illusion
A 20­year­old university student Pareidolic illusion
Hypnopompic hallucination
mentioned that while she was Charles Bonnet syndrome
Charles Bonnet syndrome
walking across a lonely park at night; Hypnagogic hallucination
Haptic hallucinations
she became anxious on mistaking
Affect illusions
bushes rustling in the wind as a
Reflex hallucination
hidden attacker but later calmed
down Pareidolic illusion
Hygric hallucinations
Complete illusion
Kinaesthetic hallucination
Haptic hallucinations
Hypnagogic hallucination
A woman with bilateral cataracts says Kinaesthetic hallucination
Hygric hallucinations
that she sees a number of her dead
Charles Bonnet syndrome
relatives walking in the house
Reflex hallucination
Haptic hallucinations
Hypnopompic hallucination
Affect illusions
Hypnagogic hallucination
Complete illusion
Kinaesthetic hallucination
A 46­year­old woman on the
psychiatric ward says 'All my blood
Pareidolic illusion
Hypnopompic hallucination
has dropped into my legs, and I can Reflex hallucination
feel a water level in my chest.' Affect illusions
Haptic hallucinations
Complete illusion
Pareidolic illusion
Charles Bonnet syndrome

1238
A 37­year­old gentleman on the
Charles Bonnet syndrome
psychiatric ward described 'I thought
Complete illusion
my life was outside my feet and made
them vibrate.'
Pareidolic illusion
Reflex hallucination
Haptic hallucinations
Hypnopompic hallucination
Kinaesthetic hallucination
Hypnagogic hallucination
Affect illusions
Your answer is incorrect. Hygric hallucinations
Explanation: 
Hypnagogic hallucinations: A transient false perception experienced while on the verge of
falling asleep. (Go for gogic) It is usually auditory. The same phenomenon if experienced
when waking up is called hypnopompic hallucinations. Both can occur in healthy individuals. 
In affect illusion, there is a mixture of heightened emotions and misperception. 
Charles Bonnet Syndrome: Elderly patients, with normal consciousness and no brain
pathology, with reduced visual acuity due to ocular problems, experience vivid, distinct,
usually coloured hallucinations. The images are mostly humans; at times animated cartoons
are reported. The vividness of this experience is a striking contrast to the real sensory
percepts which appear blurred due to the visual impairment. Insight about unreality is usually
preserved ­ though they may evoke emotions including fear and joy. 
Case 4 is a description of a hygric hallucination (fluid­wetness), which is a type of superficial
somatic hallucination. 
Kinaesthetic or proprioceptive hallucinations is also a form of somatic hallucination. It refers to
joint or muscle sense, often linked to bizarre somatic delusions. They are also seen in
benzodiazepine withdrawal and alcohol intoxication
The correct answer is: A 34­year­old gentleman in the psychiatric outpatient clinic mentioned 'I
heard a voice calling my name which then startled me back to wakefulness to find no one
there.' – Hypnagogic hallucination, A 20­year­old university student mentioned that while she
was walking across a lonely park at night; she became anxious on mistaking bushes rustling
in the wind as a hidden attacker but later calmed down – Affect illusions, A woman with
bilateral cataracts says that she sees a number of her dead relatives walking in the house –
Charles Bonnet syndrome, A 46­year­old woman on the psychiatric ward says 'All my blood
has dropped into my legs, and I can feel a water level in my chest.' – Hygric hallucinations, A
37­year­old gentleman on the psychiatric ward described 'I thought my life was outside my
feet and made them vibrate.' – Kinaesthetic hallucination

Question 74 MoK Assessment EMI023
Not answered Speech disorder
Identify the correct terms that represent each of the following speech disorders
Marked out of 5.00

Flag question
Hysterical mutism
Stammering
Decrease in spontaneous talking Palilalia
occurring in depression Logorrhoea
Elective mutism
Alogia
Akinetic mutism
Syntactical aphasia
Logoclonia

1239
Increased quantity of speech seen in Alogia
early manic states Logoclonia
Stammering
Logorrhoea
Nominal aphasia
Elective mutism
Syntactical aphasia
Hysterical mutism
Palilalia
Palilalia
Logoclonia
The patient is mute but remain aware Akinetic mutism
Alogia
of the environment though cannot
Stammering
move or respond
Logorrhoea
Elective mutism
Hysterical mutism
Syntactical aphasia
Alogia
Nominal aphasia
Syntactical aphasia
Difficulty in finding the correct name Akinetic mutism
Palilalia
for an object Elective mutism
Nominal aphasia
Akinetic mutism
Logoclonia
Stammering
Palilalia
Hysterical mutism
Nominal aphasia
Often seen in children who refuse to
Logorrhoea
Stammering
speak to certain people Logoclonia
Elective mutism
Akinetic mutism
Alogia
Hysterical mutism
Logorrhoea
Your answer is incorrect. Syntactical aphasia
Explanation: 
Alogia is a term used to denote poverty of speech and a decrease in spontaneous talking; it
occurs in depression and schizophrenia. 
Logorrhoea is increased quantity of speech, without pressure of speech or formal thought
disturbances and seen especially in early manic states. 
Nominal aphasia is also called as amnestic aphasia as it manifests as a difficulty in finding
the correct name for an object.
Akinetic mutism is associated with lesions of the upper midbrain or posterior diencephalon.
Here the patient is mute but remains aware of the environment though cannot move or
respond to commands.
Elective mutism: Mostly seen in children who refuse to speak to certain people; for example,
the child may not speak at school but may speak at home.
The correct answer is: Decrease in spontaneous talking occurring in depression – Alogia,
Increased quantity of speech seen in early manic states – Logorrhoea, The patient is mute but
remain aware of the environment though cannot move or respond – Akinetic mutism, Difficulty
in finding the correct name for an object – Nominal aphasia, Often seen in children who refuse
to speak to certain people – Elective mutism

1240
Question 75 MoK Psychology EMI005
Not answered Cloninger's Theory of Personality
Identify the correct behavioural traits to match the dimensions of temperament described by
Marked out of 6.00
Cloninger.
Flag question

Amotivation
Perseverance
Sentimentality
Harm avoidance traits (Choose TWO)
Set­shifting
Fear of uncertainty
Impulsivity
Praise dependence
Frustration Avoidance
Praise dependence
Pessimism
Impulsivity
Self directedness
Fear of uncertainty
Novelty seeking traits (Choose TWO)
Amotivation
Perseverance
Set­shifting
Self directedness
Frustration Avoidance
Impulsivity
Pessimism
Perseverance
Reward dependence traits (Choose
Sentimentality
Self directedness
TWO) Fear of uncertainty
Amotivation
Praise dependence
Pessimism
Set­shifting
Frustration Avoidance
Your answer is incorrect. Sentimentality
Cloninger's psychobiological model of personality includes four dimensions of temperament
(each 50 to 60 % heritable), which manifest early in life and 3 components of character, which
are shaped by environment. The temperamental dimensions include 

(1) Novelty­seeking (includes frustration avoidance, impulsive decision­making) 
(2) Harm­avoidance (pessimistic worry about the future, passive avoidant behaviour, fear
of uncertainty); 
(3) Reward­dependence (sentimentality, social attachment, and dependence on praise
and approval) 
(4) Persistence (high perseverance and tolerance of frustration). The character
dimensions are self­directedness, cooperativeness, and self­transcendence.

The correct answer is: Harm avoidance traits (Choose TWO) – Fear of uncertainty, Pessimism,
Novelty seeking traits (Choose TWO) – Frustration Avoidance, Impulsivity, Reward
dependence traits (Choose TWO) – Praise dependence, Sentimentality

Finish review

1241
1242
 Home Mock Paper Practice Tests

Mock Paper A(1)

Started on Wednesday, 1 July 2015, 5:05 PM
State Finished
Completed on Wednesday, 1 July 2015, 5:06 PM
Time taken 1 min 3 secs
Grade 0.00 out of 100.00
Feedback You are on the right track. Could you spend some more time on your revision?

Question 1 MoK HumanDevelopment 015
Not answered At what age does theory of mind develop?

Marked out of 1.00
Select one:
Flag question
4 years
9 years
5 year

3 years
2 months

Your answer is incorrect.
At about age 4, children begin to understand that what people think and believe, as well as
what they desire, crucially affects how they behave. 4­year­old children pass the false belief
task because they understand that people's beliefs interpret the world, rather than are direct
copies.
The correct answer is: 4 years

Question 2 MoK HumanDevelopment 020
Not answered Which one of the following infants do not attempt to interact with the mother when she is
present, nor do they appear particularly upset when left with the stranger?
Marked out of 1.00

Flag question Select one:
Disorganised infants
Anxious/resistant infants

All of the above
Anxious/avoidant infants

1243
Securely attached infants

Your answer is incorrect.
Anxious/avoidant infants do not attempt to interact with the mother when with her nor do they
appear particularly upset when left with the stranger. When the mother returns, the infant may
not make any attempt to get close to her. There is a lack of proximity or interaction during the
reunion.
The correct answer is: Anxious/avoidant infants

Question 3 MoK HumanDevelopment 023
Not answered Which of the following experiments is not correctly matched with Piaget's stages of cognitive
development?
Marked out of 1.00

Flag question Select one:
Classification of objects according to rules­concrete operational stage

Hiding objects­ Sensorimotor stage
Pendulum experiment­ Formal operational stage
Mountain tasks­ Pre­operational stage
Conservation tasks­ Sensorimotor stage

Your answer is incorrect.
Piaget illustrated egocentric thinking with his famous 'mountain task' and conservation tasks to
demonstrate the deficiencies in pre­operational thinking. Using a series of experiments Piaget
demonstrated that during this stage the child has not grasped the concept of conservation and
does not realise that certain properties of objects such as mass, volume, number, length, and
weight remain unchanged even when the objects' appearances are altered. Piaget and
Inhelder demonstrated that children's understanding of the physical world changes with
growth by using pendulum experiment. At the start of formal operational thinking, children can
construct experiments to deduct facts. They can work out the factor that determines swinging
duration as string length and not weight or speed, etc.
The correct answer is: Conservation tasks­ Sensorimotor stage

Question 4 MoK HumanDevelopment 025
Not answered A 4­year­old boy is playing a hide­and­seek game. He 'hides' the sought object on his head.
He assumes that since he cannot see the object, others cannot see it too. In Piaget's terms,
Marked out of 1.00
this is best described as
Flag question
Select one:
Artificialism
Egocentrism
Propositional thought

1244
Precausal reasoning
Circular reaction

Your answer is incorrect.
Egocentrism refers to the restricted ability to view the world from a single point of view at a
preoperational developmental stage. This was demonstrated using the Mountains task where
a child at this age group could not say what a person would see from another side of the desk
when only one side of a mountain was visible from each view.
The correct answer is: Egocentrism

Question 5 MoK HumanDevelopment 026
Not answered Which one among the following fears is present at birth?

Marked out of 1.00
Select one:
Flag question
Fear of loud noises
Fear of strangers
Fear of animals
Fear of dark
Fear of supernatural things

Your answer is incorrect.
At birth, the newborn infant shows an innate reflex to sudden or loud noises. They also show a
similar reaction to a sense of falling or sudden loss of physical support.
The correct answer is: Fear of loud noises

Question 6 MoK HumanDevelopment 029
Not answered Family instability affects development of children at home. Which of the following is correct in
this regard?
Marked out of 1.00

Flag question Select one:
There is no effect on cognitive development
Boys are less affected than girls
Hyperactive children are more affected than easy­going children
Younger children are less affected than older children
Psychosis is the most common effect

Your answer is incorrect.

1245
Family instability can affect children to various extent depending on sex (boys affected more
than girls), age (younger affected more than older children), and temperament hyperactive
affected more than placid). This has demonstrable effect on a child's cognitive achievements;
the most common psychopathology noted is behavioural difficulty.
The correct answer is: Hyperactive children are more affected than easy­going children

Question 7 MoK HumanDevelopment 030
Not answered Which of the following is NOT an effect of parental separation?
Marked out of 1.00
Select one:
Flag question
Guilt and accepting responsibility for separation
Suicidal ideas
Physical aggression
Increased motivation for academic performance
Changes in bowel habits

Your answer is incorrect.
All age groups prone to behavioural difficulties after parental divorce ­this is evident even in
infants who may show changes in eating, sleeping and bowel patterns, with fearful or anxious
responses. Academic and social aptitude suffers due to divorce; asthma, injuries, headaches
and speech defects are more common in divorced families.
The correct answer is: Increased motivation for academic performance

Question 8 MoK HumanDevelopment 031
Not answered According to Erik Erikson, between the ages of 6 and 11, the crisis to be resolved is
Marked out of 1.00
Select one:
Flag question
Autonomy vs shame
Initiative vs guilt
Industry vs inferiority
Trust vs mistrust
Identity vs role confusion

Your answer is incorrect.
During 6 to 12 years of age we are capable of learning, creating and accomplishing
numerous new skills and knowledge, thus developing a sense of industry. This is also a very
social stage of development, and if we experience unresolved feelings of inadequacy and
inferiority among our peers, we can have serious problems in terms of competence and self­
esteem.
The correct answer is: Industry vs inferiority

1246
Question 9 MoK HumanDevelopment 036
Not answered Which of the following programme is very appropriate for the needs of children and adults with
communication and learning difficulties?
Marked out of 1.00

Flag question Select one:
British Sign Language 
Makaton
Sensory therapy

Phonetic training
Habit reversal

Your answer is incorrect.
The Makaton Vocabulary was originally devised in the 1970's by Margaret Walker a speech
and language therapist and two of her colleagues Katharine and Tony, the name Makaton is
made up of syllables from the beginning of each of their names Ma­Ka­Ton. Makaton consists
of a core vocabulary of specially selected concepts and ideas. These are considered to be
most appropriate for the needs of children and adults with communication and language
difficulties. The initial stages introduce the vocabulary required to express basic ideas. More
complex concepts are introduced in the later stages. There is also an additional resource
vocabulary grouped into topics.
The correct answer is: Makaton

Question 10 MoK Basic Psychology 025
Not answered Amnesia refers to marked impairment in

Marked out of 1.00
Select one:
Flag question
Procedural memory
Episodic memory

All of the above
Working memory
Semantic memory

Your answer is incorrect.
Amnesia refers to a marked impairment in episodic memory, although other types of memory
such as working memory, semantic memory and procedural memory may remain relatively
intact. It is useful to remember two types of amnesia here; 1. Anterograde Amnesia: In
hippocampal damage, a lack memory for the immediate future is seen. The subject cannot
learn anything new as no information can be moved from STM to LTM. 2. Retrograde
Amnesia: Following head injury, a lack memory for immediately preceding events is seen. The
subject never consolidates the info (the information moves from STM to LTM, but cannot be
retrieved when needed; thus the stored memory is 'lost' or inaccessible).
The correct answer is: Episodic memory

1247
Question 11 MoK Basic Psychology 027
Not answered A 5­year­old girl loved to help her mother in kitchen. She was scared of going near the cooker
after she burnt her hand. This is explained by
Marked out of 1.00

Flag question Select one:
Reciprocal inhibition
Extinction
Negative reinforcement
Positive reinforcement
Operant conditioning

Your answer is incorrect.
When the conditioned stimulus no longer produces conditioned response, this is known as
extinction. In this case, the extinction is associated with a 'punishing' experience of being
burnt.
The correct answer is: Extinction

Question 12 MoK Basic Psychology 030
Not answered Which type of reinforcement is the least resistant to extinction?

Marked out of 1.00
Select one:
Flag question
Fixed interval
Variable ratio.

Continuous
Fixed ratio
Variable interval

Your answer is incorrect.
Reinforcement schedules may be continuous, fixed interval, fixed ratio, variable interval or
variable ratio. Variable ratio reinforcement is the most resistant e.g., gambling. Continuous
reinforcement is the one that can be easily made extinct. Intermittent reinforcement takes the
longest to establish.
The correct answer is: Fixed ratio

Question 13 MoK Basic Psychology 031
Not answered A psychologist takes a 31 year old patient with significant fear of heights to the top of a 21
storey­building. Which of the following behavioural technique is he employing?
Marked out of 1.00

Flag question Select one:
Relaxation training

1248
Desensitisation
Flooding
Implosion

In vitro flooding

Your answer is incorrect.
Flooding: It is an operant conditioning technique where actual exposure to feared stimulus
takes place for a substantial amount of time, so the accompanying anxiety response fades
away while the stimulus is continuously present. This will lead to an extinction of the fear.
When a similar technique is attempted with imagined not actual exposure, then it is called as
implosion.
The correct answer is: Flooding

Question 14 MoK Basic Psychology 033
Not answered In patients with moderate head injury, post traumatic amnesia is usually expected to last
Marked out of 1.00
Select one:
Flag question
From 1 week to 1 month
From 1 day to 1 week
Less than 1 hour
From 1 to 24 hours
Less than 1 minute

Your answer is incorrect.
Post traumatic amnesia is the time between the injury and recovery of normal continuous
memory, seen in head injury patients. The longer the PTA, the more severe the brain damage
and poorer the prognosis for the recovery. PTA grading system is as follows; PTA less than 1
hour­ mild head injury PTA from 1 to 24 hours­moderate head injury PTA longer than 24
hours­severe injury. Retrograde amnesia is also possible after a head injury. In most cases,
the amnesic gap is short and lasts for less than one minute. It is not a good indicator of
prognosis.
The correct answer is: From 1 to 24 hours

Question 15 MoK Basic Psychology 041
Not answered Which of the following statements regarding operant conditioning is correct?

Marked out of 1.00
Select one:
Flag question
It cannot be used in patients with learning disability as the concept of
reward is non­existent in most patients
Punishment is the most effective form of operant learning

1249
It is a form of associative learning
It was first described by Albert Bandura
Negative reinforcement refers to introducing an aversive stimulus to reduce
the frequency of a behaviour

Your answer is incorrect.
Operant conditioning and classical conditioning are two types of associative learning.
Habituation and sensitisation are described as non­associative learning. Punishment is not
an effective conditioning technique; it helps to learn how not to behave but not how to behave.
On most occasions, it only serves to suppress a response ­ not abolishing it completely.
The correct answer is: It is a form of associative learning

Question 16 MoK Basic Psychology 042
Not answered Which statement among the following about short­term memory is false?
Marked out of 1.00
Select one:
Flag question
The retrieval process in short term memory is error free.
Information is held in the STM for 15 to 30 seconds.
It can be extended by rehearsal
Chunking of material can effectively increase the capacity of short term
memory
It is mainly stored in the visual modality

Your answer is incorrect.
The information in the short­term memory is mainly stored in the auditory modality. In STM, the
encoding is acoustic, and the retrieval is error­free.
The correct answer is: It is mainly stored in the visual modality

Question 17 MoK Basic Psychology 046
Not answered Which one of the following is associated with classifying personality according to body build?
Marked out of 1.00
Select one:
Flag question
Kretschmer
Rutter
Kraeplin
Russell

Fairburn

1250
Your answer is incorrect.
Kretschmer classified body types into pyknic, asthenic and athletic types and proposed that
personality traits vary with body type.
The correct answer is: Kretschmer

Question 18 MoK Basic Psychology 047
Not answered Which type of learning behaviour meant that there is a preformed mental representation even
without reinforcement;
Marked out of 1.00

Flag question Select one:
Modelling
None of the above
Insight learning
Latent learning
Observational learning

Your answer is incorrect.
Latent Learning: For example if an animal is allowed to wander around a maze without being
reinforced for doing so, then later on, when placed in the maze, it will learn it faster than other
animals with no previous experience, suggesting that a mental representation of the maze is
being made even without any reinforcement­based learning.
The correct answer is: Latent learning

Question 19 MoK Basic Psychology 048
Not answered Which one of the following statements about memory loss following ECT is incorrect?

Marked out of 1.00
Select one:
Flag question
Memory impairment is more pronounced with unilateral ECT
There is no lasting memory impairment

A third of patients report persistent memory loss
There may be both Anterograde and retrograde amnesia
Most studies did not measure autobiographical memory over the long term

Your answer is incorrect.
Memory loss following ECT; The impairment is usually temporary, and there is no lasting
memory impairment. There may be both Anterograde and retrograde amnesia, both of which
shrink rapidly. A third of patients report persistent memory loss following ECT (Rose et al
2003). Memory impairment is less pronounced with unilateral ECT. Most studies on ECT have
not measured autobiographical memory over the long term.
The correct answer is: Memory impairment is more pronounced with unilateral ECT

1251
Question 20 MoK Basic Psychology 049
Not answered Which of the following instruments assess personality as a construct spread over nine
different psychopathological domains?
Marked out of 1.00

Flag question Select one:
International personality disorders examination
Eysenck's personality questionnaire
NEO personality inventory
Thematic appreception test
Minnesota multiphasic personality inventory

Your answer is incorrect.
MMPI assesses personality across nine psychopathological domains using more than 500
questions.
The correct answer is: Minnesota multiphasic personality inventory

Question 21 MoK Basic Psychology 050
Not answered Which one among the following is an objective personality test?

Marked out of 1.00
Select one:
Flag question
Roschach ink blot test
Sentence completion test.
Draw a person test
Thematic apperception test
Minnesota multiphasic personality inventory

Your answer is incorrect.
MMPI is an objective personality test devised initially from a clinical population. It includes a
lie scale, frequency and correction scales. Cattell 16 PF scale is an objective personality test
devised from a nonclinical population. It includes a lie scale, frequency and correction scales.
Non­objective or projective tests include Rorschach inkblot test (10 cards), TAT (Thematic
Apperception Test­ with pictures of ambiguous persons and scenes, make up a story), DAPT
(Draw a person test) and sentence completion test.
The correct answer is: Minnesota multiphasic personality inventory

Question 22 MoK Basic Psychology 053
Not answered Which one among the following is a feature of attentional processing according to Shiffrin and
Schneider's divided attention theory?
Marked out of 1.00

1252
Flag question Select one:
Automatic through extensive training
Not limited
Needs more mental effort.
Unconscious
Resistant to change.

Your answer is incorrect.
According to Shiffrin and Schneider's divided attention theory (1977), automatic processing is
unconscious, not limited, automatic through extensive training, and once learnt are resistant to
change. But Attentional (controlled) processing is consciously directed, needs focussed
attention and more mental effort and also has limited capacity.
The correct answer is: Needs more mental effort.

Question 23 MoK Basic Psychology 054
Not answered Which of the following offers a psychological explanation regarding the aetiology of OCD?

Marked out of 1.00
Select one:
Flag question
Reciprocal inhibition
Punishment

Negative reinforcement
Positive reinforcement
Reward orientation

Your answer is incorrect.
Compulsions provide short­term relief of obsessional anxiety via negative reinforcement.
When practicing compulsive rituals, anxiety is reduced acutely. This provides a reinforcement
to practice the compulsions repeatedly. In other words the termination of the aversive anxiety
cued by obsessions, increases or stamps in the compulsive behaviour that removed the
anxiety, without addressing the core of obsessions.
The correct answer is: Negative reinforcement

Question 24 MoK Basic Psychology 057
Not answered Star chart is based on the principles of

Marked out of 1.00
Select one:
Flag question
Avoidance conditioning
Classical conditioning

Operant conditioning

1253
Higher order conditioning
Trace conditioning

Your answer is incorrect.
Both positive and negative reinforcement increase the desired response e.g., the use of a
"star chart," with a variable interval schedule (2 or 3 stars are administered per day depending
on the frequency of desired behaviour).
The correct answer is: Operant conditioning

Question 25 MoK Basic Psychology 059
Not answered Which part of the brain is involved in mediating performances in short term memory?

Marked out of 1.00
Select one:
Flag question
Left and right ventricles
Parietal lobes
Prefrontal lobes
Entorhinal cortex of the medial temporal lobe
Limbic lobe

Your answer is incorrect.
Brain imaging and neuropsychological studies provide strong evidence that a.The brain areas
mediating performances in STM are principally the pre­frontal lobes b.The phonological STM
system is mediated by the left hemisphere regions of Broca's' area and prefrontal cortex. c.The
visuospatial STM system is mediated by the parietal and prefrontal areas of the right
hemisphere. d.The brain areas responsible for LTM includes the regions of the limbic system
especially the hippocampus and the Entorhinal cortex of the medial temporal lobe.
The correct answer is: Prefrontal lobes

Question 26 MoK Social Psychology 007
Not answered A candidate who has appeared for a mock exam before the actual exam does well than a
candidate who has not. This is called
Marked out of 1.00

Flag question Select one:
Regression of mean
Hawthorne effect
Observer effect
Halo effect
Practice effect

1254
Your answer is incorrect.
Practice effects occur when a participant in an experiment performs a task after having
performed the same task earlier. Subjects can either have a positive (subjects become better
at performing the task) or negative (subjects become worse at performing the task) effect.
Repeated measures designs are almost always affected by practice effects.
The correct answer is: Practice effect

Question 27 MoK Social Psychology 008
Not answered The most common effect of cognitive dissonance is

Marked out of 1.00
Select one:
Flag question
Psychological distress
Cognitive distortions
Memory disturbances
Formal thought disorder

Behavioural modification

Your answer is incorrect.
'Cognitive dissonance' is an aversive psychological state aroused when there is a
discrepancy between actions and attitudes (Festinger, 1957). In situations when actions
cannot be reversed, or when doing so requires great effort, adjusting attitudes to be in line
with decisions often reduces this discrepancy.
The correct answer is: Psychological distress

Question 28 MoK Sociocultural Psychiatry 013
Not answered Compulsory treatment cannot be justified under which of the following ethical principles?

Marked out of 1.00
Select one:
Flag question
Deontology
Beneficience
Autonomy
Teleology
Non maleficience

Your answer is incorrect.
A deontologist could justify detention as a duty; a teleologist can defend it based on intended
outcome. But detention is against the principle of autonomy.
The correct answer is: Autonomy

1255
Question 29 MoK Sociocultural Psychiatry 015
Not answered The following is true about the prima facie (main) principles of medical ethics:

Marked out of 1.00
Select one:
Flag question
Virtue theory: principle based on the characteristics of moral agents
Autonomy: Respecting patient's views despite disagreement
Utilitarianism: principle based on the greatest benefit for the greater number
of people
Deontology: principle based on the act as stemming from duties irrespective
of the moral agent characteristics
Teleology: principle based on consequences of the act

Your answer is incorrect.
Note that, even though, the pairs are correct with respect to the explanations attached, except
autonomy other ethical principles stated here belong to 'higher­order' values.
The correct answer is: Autonomy: Respecting patient's views despite disagreement

Question 30 MoK Sociocultural Psychiatry 030
Not answered The life event with highest rating in Holmes Rahe scale is

Marked out of 1.00
Select one:
Flag question
Marriage
Childbirth
Divorce
Job loss
Jail term

Your answer is incorrect.
In the absence of 'death of a spouse' among the given choices, divorce can be chosen as the
answer.
The correct answer is: Divorce

Question 31 MoK Sociocultural Psychiatry 034
Not answered Which of the following research materials is covered by the Helsinki Declaration?

Marked out of 1.00
Select one:
Flag question
Identifiable human data
Anonymised human data

1256
Animal data
Environmental data
Non­clinical digital data

Your answer is incorrect.
The World Medical Association (WMA) has developed the Declaration of Helsinki as a
statement of ethical principles for medical research involving human subjects, including
research on identifiable human material and data.
The correct answer is: Identifiable human data

Question 32 MoK Sociocultural Psychiatry 038
Not answered A man who works full time at the local council office has headache, abdominal pain and
dizziness. He feels tired and has a fever. Which of the following best describes the above
Marked out of 1.00
situation?
Flag question
Select one:
Health behaviour
Illness
Sick role
Abnormal illness behaviour
Illness behaviour

Your answer is incorrect.
The description given here fits the illness itself rather than the patient's response or social
changes that take place.
The correct answer is: Illness

Question 33 MoK Sociocultural Psychiatry 040
Not answered A 45 year old man holds out his arm to allow measurement of blood pressure as soon as the
staff nurse walks towards him following the daily routine of medication dispensing. Which of
Marked out of 1.00
the following is true?
Flag question
Select one:
As the patient is competent consent is not needed
Explicit consent is not required as this is a situation of necessity
Explicit consent need not be sought for routine procedures
Blood pressure is measured under principle of best interests
Implied consent can be assumed

1257
Your answer is incorrect.
Implied consent is assumed here. This is not a situation of necessity, and best interest
decisions do not apply here.
The correct answer is: Implied consent can be assumed

Question 34 MoK Sociocultural Psychiatry 042
Not answered Which of the following is true with regards to Changing Minds campaign?
Marked out of 1.00
Select one:
Flag question
It is focussed on geriatric mental health issues
It is an initiative from World Health Organisation
It is not a part of Care Programme Approach
It provides a legal clause against stigma
It is an ongoing programme against stigma

Your answer is incorrect.
Concern about the stigma of mental illness culminated in the Royal College of Psychiatrists' 5­
year 'Changing Minds' campaign (1998­2003) whose aim was to promote positive images of
mental illness, challenge misrepresentations and discrimination, encourage patient advocacy
and educate the public about the real nature and treatability of mental disorder (Crisp et al,
2000) [Excerpt retrieved from: Effectiveness of Changing Minds campaign factsheets,
http://pb.rcpsych.org/content/pbrcpsych/31/10/377.full.pdf].
The correct answer is: It is not a part of Care Programme Approach

Question 35 MoK Sociocultural Psychiatry 043
Not answered Exaggerated startle response is one of the important features of which culture bound
syndrome?
Marked out of 1.00

Flag question Select one:
Koro
Amok
Dhat syndrome
Latah
Windigo

Your answer is incorrect.
Latah is a condition where, triggered by the startle reflex; victims fall into a trance in which he
or she engages in repetitive speech or movements. Often these take the form of echopraxic
and/or echolaliac automatisms. Latah is found only in certain world cultures and is therefore
commonly considered a culture­specific syndrome.

1258
The correct answer is: Latah

Question 36 MoK Sociocultural Psychiatry 045
Not answered With respect to vulnerability for depression, poor coping style is best described as which of the
following?
Marked out of 1.00

Flag question Select one:
Modifying factor
Maintaining factor
Precipitating factor
Predisposing factor
Protective factor

Your answer is incorrect.
Brown et al. identified the following factors as maintaining factors for depression: a. Further
negative life events b. Persistent poor quality social support c.Poor coping style characterized
by self­blame and helplessness, denial of problems, poor problem­solving ability, blaming
others or external forces d. Inability to obtain adequate social support due to fear of intimacy
and e. Low educational level.
The correct answer is: Maintaining factor

Question 37 MoK Sociocultural Psychiatry 046
Not answered Which of the following is true with regard to discrimination experienced by patients with
mental health difficulties?
Marked out of 1.00

Flag question Select one:
Majority of service users experience discrimination at workplace
Most of them experience no discrimination
Discriminant behaviour against mentally ill leads to enhanced public safety
Health professionals do not make service users feel discriminated
No discrimination is experienced when attending one's own general
practitioner

Your answer is incorrect.
A report from 556 service users in UK shows that 70% have experienced discrimination in
some form: 47% in the workplace, 44% from general practitioners and 32% from other health
professionals (Mental Health Foundation, 2000).
The correct answer is: Majority of service users experience discrimination at workplace

1259
Question 38 MoK Classification 023
Not answered Mania is often characterized by the presence of

Marked out of 1.00
Select one:
Flag question
Delusions of control
Delusions of reference
Delusions of love
Delusions of grandiosity
Delusions of pregnancy

Your answer is incorrect.
Grandiose delusions in the absence of mania are relatively uncommon though schizophrenia
is the commonest condition after mania in which this type of delusion can be seen.
The correct answer is: Delusions of grandiosity

Question 39 MoK Classification 038
Not answered Mr. Y is a 83 year old man seen in the outpatients clinic. He is complaining of vivid dreams.
On performing a cognitive examination, he shows a decline in attention and visuo­spatial
Marked out of 1.00
ability. The most likely diagnosis is;
Flag question
Select one:
Alzheimer's dementia
Fronto­temporal dementia
Lewy body dementia
Chronic subdural haematoma
Huntington's dementia

Your answer is incorrect.
In Dementia of Lewy Body, memory impairment appears to result from deficits in alertness and
attention rather than in memory acquisition; thus, short­term recall is affected less than digit
span memory (ability to repeat 7 digits forward and 5 backward). Patients may stare into
space for long periods. Excessive daytime drowsiness is common. Visuospatial and
visuoconstructional abilities (tested by block design, clock drawing, or figure copying) are
affected more than other cognitive deficits. Extreme sensitivity to antipsychotics is typical.
Many patients have rapid eye movement (REM) sleep behavior disorder, a parasomnia
characterized by vivid dreams without the usual physiologic paralysis of skeletal muscles
during REM sleep. As a result, dreams may be acted out, sometimes injuring the bed partner.
The correct answer is: Lewy body dementia

Question 40 MoK Classification 048

1260
Not answered Neurasthenia is a diagnosis included under which of the following topics in 'Neurotic, stress
Marked out of 1.00
related and somatoform disorders' in ICD 10?

Flag question
Select one:
Adjustment disorders
Dissociative disorders
Other anxiety disorders
Somatoform disorders
Other neurotic disorders

Your answer is incorrect.
Neurasthenia is defined in the ICD­10 as a psychiatric disorder whose main feature is
'persistent and distressing complaints of increased fatigue after mental effort, or persistent and
distressing complaints of bodily weakness and exhaustion after minimal effort' (Chronic
fatigue syndrome or neurasthenia?, http://bjp.rcpsych.org/content/181/4/350.2.full).
The correct answer is: Other neurotic disorders

Question 41 MoK Classification 049
Not answered Excessive doubt and distrust in relationships is a characteristic feature of which of the
following personality disorder?
Marked out of 1.00

Flag question Select one:
Schizoid
Dissocial
Schizotypal
Borderline
Paranoid

Your answer is incorrect.
Paranoid PD is characterized by distrust in relationships.
The correct answer is: Paranoid

Question 42 MoK Classification 050
Not answered A 40­year­old man demanded to see you in the outpatient clinic. He says that he does not
want to see any member of the community mental health team as they have been late for visits
Marked out of 1.00
on 3 occasions. He is unhappy with the council in terms of rubbish collections and is in the
Flag question process of writing his 12th compliant letter. What is the most likely personality disorder?

Select one:
Borderline

1261
Paranoid
Dissocial
Histrionic
Schizoid

Your answer is incorrect.
Paranoid personality disorder (PPD) is characterized by a pervasive mistrust of other people.
Other common features of the disorder include quarrelsomeness, hostility, emotional
coldness, hypersensitivity to slights or criticism, stubbornness, and rigidly held maladaptive
beliefs of others' intents (Paranoid Personality Disorder. Retrieved from
http://www.sagepub.com/upm­data/15198_Chapter3.pdf).
The correct answer is: Paranoid

Question 43 MoK Classification 053
Not answered Which one among the following can produce both positive and negative schizophrenic
symptoms in non schizophrenic individuals?
Marked out of 1.00

Flag question Select one:
LSD
Amphetamine
PCP or Phencyclidine
Levodopa
Psilocybin

Your answer is incorrect.
Amphetamines and Levodopa releases dopamine at the synapse and can cause positive
schizophrenic symptoms but do not produce negative symptoms. LSD and Psilocybin are
5HT­2A/2C receptor agonist, and both can cause positive symptoms of schizophrenia in non­
schizophrenic individuals. LSD also produces visual hallucinations which are uncommon in
schizophrenia. A single dose of phencyclidine which is a non­competitive NMDA receptor
antagonist can produce both positive and negative schizophrenic symptoms in non­
schizophrenic individuals and exacerbations of schizophrenia in schizophrenic patients.
The correct answer is: PCP or Phencyclidine

Question 44 MoK Classification 054
Not answered Paraphrenia is included in ICD 10 under which of the following descriptions?

Marked out of 1.00
Select one:
Flag question
Induced delusional disorders
Acute polymorphic psychosis

1262
Persistent delusional disorder
Paranoid schizophrenia
Schizo affective disorder

Your answer is incorrect.
Paraphrenia refers to the delusional disorder that occurs in the elderly. It is more common in
those with sensory impairments like deafness or visual problems and is also more common in
those who live on their own. In this condition personality, integration is better than with early
onset schizophrenia.
The correct answer is: Persistent delusional disorder

Question 45 MoK Classification 055
Not answered A 40 year­old chef presents with a 4­year history of pain involving the left side of the body.
There is no pathological evidence for a cause of such pain. The most likely diagnosis is
Marked out of 1.00

Flag question Select one:
Body dysmorphic disorder
Persistent somatoform pain disorder
Hypochondriasis
Somatisation disorder
Somatoform autonomic dysfunction

Your answer is incorrect.
Persistent somatoform pain syndrome is characterised by unexplained, usually isolated pain
symptom of the musculoskeletal system.
The correct answer is: Persistent somatoform pain disorder

Question 46 MoK Classification 057
Not answered Which one of the following is not considered to be a melancholic feature of major depressive
disorder?
Marked out of 1.00

Flag question Select one:
Loss of interest or pleasure
Early morning awakening
Phobic symptoms
Psychomotor agitation

Psychomotor retardation

1263
Your answer is incorrect.
Melancholic features (DSM­IV) includes loss of interest or pleasure in usual activities, lack of
reactivity to pleasurable stimuli, plus at least three of the following: distinct quality of mood
(unlike normal sadness); morning worsening of mood; early morning waking; psychomotor
agitation or retardation; significant anorexia or weight loss; excessive guilt and marked loss of
libido.
The correct answer is: Phobic symptoms

Question 47 MoK Classification 058
Not answered A patient with schizophrenia was treated for an acute episode and left hospital seven months
ago. For the last 2 weeks, he has been having symptoms of social isolation, poor eating and
Marked out of 1.00
sleeping. The most likely diagnosis is
Flag question
Select one:
Major depressive disorder
Undifferentiated schizophreni
Post schizophrenic depression

Residual schizophrenia
Hebephrenic schizophrenia

Your answer is incorrect.
Postschizophrenic depression is an ICD 10 diagnosis ­ it refers to depressive episode less
than 12 months of an acute episode of schizophrenia
The correct answer is: Post schizophrenic depression

Question 48 MoK Clinical Examination 017
Not answered Which of the following tests can detect visual hemi­neglect?

Marked out of 1.00
Select one:
Flag question
Head tilting test
Abbreviated Mental Test
Calorie stimulation test
Clock drawing test
Mini Mental State examination

Your answer is incorrect.
Clocks drawn by patients with hemi­neglect often have no numbers on one side (often the left
side)
The correct answer is: Clock drawing test

1264
Question 49 MoK Clinical Examination 022
Not answered In MMSE, intersecting pentagons primarily assesses which of the following functions?

Marked out of 1.00
Select one:
Flag question
Orientation
Constructional praxis
Language

Copying skills
Visual retention

Your answer is incorrect.
Visuospatial functioning and constructional praxis.
The correct answer is: Constructional praxis

Question 50 MoK Clinical Examination 030
Not answered A 57­year­old man came to the clinic with a history of personality changes, sexual
disinhibition and tremors. Which of the following is crucial to elicit in the history?
Marked out of 1.00

Flag question Select one:
Family history
Forensic history

Social history
Psychosexual history
Medication history

Your answer is incorrect.
The scenario is suggestive of frontal lobe dementia. There is a strong family history present in
50% of cases.
The correct answer is: Family history

Question 51 MoK Clinical Examination 031
Not answered A 32­year­old patient with an acute psychotic episode is being treated with antipsychotics. He
has developed muscular stiffness. Which of the following neurological signs would raise
Marked out of 1.00
suspicions of a cause other than drug induced side effects?
Flag question
Select one:
Blepharospasm
Tremors
Fasciculation

1265
Rigidity
Bradykinesia

Your answer is incorrect.
Fasciculations are seen in lower motor nerve lesions
The correct answer is: Fasciculation

Question 52 MoK Clinical Examination 032
Not answered Which of the following is not a feature of upper motor neuron lesion?

Marked out of 1.00
Select one:
Flag question
Hypertonia

Rigidity
Mild disuse atrophy
Fasciculation
Exaggerated reflexes

Your answer is incorrect.
Fasciculations are characteristic of LMN lesions
The correct answer is: Fasciculation

Question 53 MoK Clinical Examination 033
Not answered Select one investigation for a patient on antipsychotics presenting with weight loss and thirst

Marked out of 1.00
Select one:
Flag question
U and Es
FBC
Fasting glucose
Li level
LFTs

Your answer is incorrect.
Type 2 diabetes may be present in this individual (but note weight gain is more common in
type 2 diabetes).
The correct answer is: Fasting glucose

1266
MoK Clinical Examination 034
Question 54
A 49­year­old man presented to the clinic with jerky movements, clumsiness and history of
Not answered
memory decline for the last year but no family history of similar illnesses. How could we
Marked out of 1.00 identify his disorder?
Flag question
Select one:
Genetic testing
Medication history
CT scan
Drug and alcohol history
Family history

Your answer is incorrect.
The diagnosis is Huntington's disease. A genetic test can then confirm the diagnosis by
determining if the person indeed has inherited the HD gene mutation (an expansion of the
CAG triplet)
The correct answer is: Genetic testing

Question 55 MoK Clinical Examination 035
Not answered You instruct the patient to tap once in response to your single tap, and to withhold the tapping
response for your double taps. This is test is known as
Marked out of 1.00

Flag question Select one:
N­back test
Fluency Test
Go No­Go Test
continuous performance test

oddball test

Your answer is incorrect.
Impulsivity is thought to reflect a failure of response inhibition, and is seen in inferior frontal
pathology. It can be assessed using the Go­No­Go task. The examiner instructs the patient to
tap once in response to a single tap and to withhold a response for two taps. This test can be
made more difficult by changing the initial rule after several trials (for example, ''tap once
when I tap twice, and not at all when I tap once'')(Kipps and Hodges, 2006. Retrieved from
http://jnnp.bmj.com/content/76/suppl_1/i22.full).
The correct answer is: Go No­Go Test

Question 56 MoK Clinical Examination 036
Not answered Which of the following can test abstract reasoning?

Marked out of 1.00

1267
Flag question Select one:
Clock drawing test
Digit symbol substitution test
Goldstein's card sorting test
Trail making test
n­back test

Your answer is incorrect.
The Goldstein­Scheerer Color Form Sorting Test was initially designed to test abstract
reasoning (concept formation rather than the simple identification of abstract rules). Wisconsin
card sorting test is a descendant of Goldstein task nd it tests concept identification rather than
formation
The correct answer is: Goldstein's card sorting test

Question 57 MoK Clinical Examination 037
Not answered Which of the following complications of anorexia is unlikely to correct itself on weight
restoration?
Marked out of 1.00

Flag question Select one:
hypothyroidism
salivary enlargement

growth restriction
lipid abnormalities
osteoporosis

Your answer is incorrect.
Impaired linear growth and the possibility of permanent short stature are significant medical
complications in adolescents with Anorexia Nervosa. There are reports in the pediatric
literature of children and adolescents with anorexia presenting with growth failure or short
stature. Reports regarding catch­up growth after weight restoration include failure to gain any
height or incomplete catch­up growth, with very rare instances where complete catch­up
growth occurs. (Int J Eat Disord 2005; 37: S52­S59)
The correct answer is: growth restriction

Question 58 MoK Clinical Examination 038
Not answered Whilst on call, you see a psychotic patient on a long term inpatient unit. The patient has a
tremor, is ataxic and restless. During the interview the patient vomits. The nurse tells you he
Marked out of 1.00
has been having diarrhea and has been urinating very frequently. What question would be
Flag question most useful to ask the patient?

Select one:

1268
Can you subtract 7 serially from 100?
What place is this?
Who is the current prime minister?
Have you been drinking a lot of water recently?
Where are you from?

Your answer is incorrect.
The case presented in this question is a common description of water intoxication. Symptoms
include tremor, ataxia, restlessness, diarrhea, vomiting, polyuria, and eventual stupor. This is
a problem that has been reported in up to 20% of some cohorts of patients with chronic
schizophrenia.
The correct answer is: Have you been drinking a lot of water recently?

Question 59 MoK Clinical Examination 039
Not answered A 55­year­old man who was admitted to the hospital yesterday has become combative and
wants to leave. His wife reports that he has episodic irritability and altered sleep wake cycle.
Marked out of 1.00
She feels that he has completely changed as a person in last 1 year. He also has poor
Flag question memory and has palmar erythema and multiple subcutaneous bleeds over his back. He has
bilateral plantar extensor reflexes. The most probable diagnosis is

Select one:
Haemophilia
Huntington's disease
Wilson's disease
Hepatic failure
Idiopathic thrombocytopenic purpura

Your answer is incorrect.
Hepatic failure can produce asterixis, tremors, exaggerated deep tendon reflexes and altered
consciousness with preceding history of changes in personality, abrupt mood swings, and
cognitive deterioration. Peripheral signs may include subcutaneous bleeds, pedal edema,
ascites, palmar erythema, etc.
The correct answer is: Hepatic failure

Question 60 MoK Clinical Examination 040
Not answered Presence of lymphocytic pleiocytosis with many red cells in the CSF, along with hyperintense
appearance of left temporal lobe in T2 weighted MRI imaging and lateralised periodic
Marked out of 1.00
discharges in EEG in a semi­conscious patient suggests
Flag question
Select one:
Pneumococcal meningitis

1269
Crutzfeldt Jakob disease
Temporal lobe malignancy
Subdural haemorrhage
Herpes encephalitis

Your answer is incorrect.
MRI is the preferred modality for evaluating the brain.[14, 15, 16] However, early in the clinical
course of the disease, MRI results may be negative. A negative MRI does not rule out HSV
encephalitis. In adults, T2­weighted MRI reveals hyperintensity corresponding to edematous
changes in the temporal lobes (see the first two images below), inferior frontal lobes, and
insula, with a predilection for the medial temporal lobes. Foci of hemorrhage occasionally can
be observed on MRI (Imaging in Herpes Encephalitis ­ Medscape Reference,
http://emedicine.medscape.com/article/341142­overview).
The correct answer is: Herpes encephalitis

Question 61 MoK Clinical Examination 041
Not answered Degeneration of head of caudate nucleus is associated with

Marked out of 1.00
Select one:
Flag question
Huntington chorea
OCD
Tourete's syndrome
Fahr's disease
Wilson disease

Your answer is incorrect.
Degeneration of the striatum (mainly caudate nucleus) and selective loss of GABAergic
neurons is seen in Huntington's disease.
The correct answer is: Huntington chorea

Question 62 MoK Descriptive Psychopathology 026
Not answered Which one of the following is not a type of primary delusion?

Marked out of 1.00
Select one:
Flag question
Delusional memory
Delusional misidentification
Delusional atmosphere
Delusional perception
Autochthonous delusions

1270
Your answer is incorrect.
The four types of primary delusions are an autochthonous delusion, delusional mood,
delusional memory and delusional perception.
The correct answer is: Delusional misidentification

Question 63 MoK Descriptive Psychopathology 028
Not answered A 19 year­old university student, seen for first time by a psychiatrist, is very perplexed and
claims that he could feel something odd happening around him. This phenomenon is termed
Marked out of 1.00
as
Flag question
Select one:
Sensitive ideas of reference
Delusional mood
Oneiroid state
Vorbeireden
Persecutory delusion

Your answer is incorrect.
The sense of perplexity that could precede frank psychotic reaction I s called as delusional
mood or atmosphere. At this stage, the patient will not have any fully formed delusions. But
once delusions arrive, this will provide a sense of relief from perplexity, and everything will
seem to 'fall in place'. Delusional mood is a type of Jasperian primary delusion.
The correct answer is: Delusional mood

Question 64 MoK Descriptive Psychopathology 031
Not answered 'I saw a blue caravan on the road and I immediately realised that the South Pole is going to
sink soon'. This is consistent with
Marked out of 1.00

Flag question Select one:
Delusional perception
Delusional memory
Delusional atmosphere
Delusional mood
Autochthonous delusion

Your answer is incorrect.
Note that there are two distinct processes here ­ visual perception and delusional
misjudgment.

1271
The correct answer is: Delusional perception

Question 65 MoK Descriptive Psychopathology 036
Not answered Cameron described the following form of thought disorder:

Marked out of 1.00
Select one:
Flag question
Omission
Derailment
Fusion
Drivelling
Desultory thinking

Your answer is incorrect.
Cameron's analysis of formal thought disorder in 25 'unmistakably schizophrenia' patients led
to the concept of asyndetic thinking that included derailment, loss of the central determining
idea, and loose associations.
The correct answer is: Derailment

Question 66 MoK Descriptive Psychopathology 037
Not answered A patient describes that everything around him appears hazy, dull and lifeless. This can be
termed as
Marked out of 1.00

Flag question Select one:
Anhedonia
Aphemia
Depersonalisation
Derealisation
Agnosia

Your answer is incorrect.
The description here suits derealisation. Depersonalisation is characterised by a profound
disruption of the self­awareness mainly characterised by feelings of disembodiment and
subjective emotional numbing (Depersonalization: a selective impairment of self­awareness,
http://www.ncbi.nlm.nih.gov/pubmed/21087873).
The correct answer is: Derealisation

Question 67 MoK Descriptive Psychopathology 042
Not answered

Marked out of 1.00

1272
Flag question When asked about his favorite sport the patient answers, "I like soccer. The players wear
shirts. Shirts are clothes. Clothes are dirty." The speech disturbance noted here is best
described as

Select one:
Loosening of association
Paragrammatism
Tangentiality
Flight of ideas
Circumstantiality

Your answer is incorrect.
Flight of ideas refers to speech that often (but not always) occurs at a rapid rate and involves
fragmented or unrelated ideas, connected by distracting cues rather than the goal of the
discourse. This is common in mania.
The correct answer is: Flight of ideas

Question 68 MoK Descriptive Psychopathology 044
Not answered The nature of conversational speech in jargon dysphasia is

Marked out of 1.00
Select one:
Flag question
Non­fluent and intelligible
Fluent and unintelligible
The rhythm of speech is not retained
Non­fluent and unintelligible
Fluent and intelligible

Your answer is incorrect.
Wernicke's aphasia is fluent but unintelligible
The correct answer is: Fluent and unintelligible

Question 69 MoK Psychology EMI002
Not answered Reinforcement & Learning
Lead in: Which type of learning process is involved in each of the following situations?
Marked out of 3.00

Flag question

This principle can be used to teach
toilet training in the learning disabled,
after having the process broken down
into simple steps that link to each
other

1273
Stimulus generalisation
Punishment
Shaping
Variable ratio reinforcement
Stimulus preparedness
Habituation
Modelling
Chaining
Positive reinforcement
Operant learning
Modelling
Fixed ratio reinforcement
Stimulus preparedness
Gambling is an example of this. Positive reinforcement
Shaping
Variable ratio reinforcement
Habituation
Punishment
Chaining
Stimulus preparedness
Fixed ratio reinforcement
Modelling
This type of learning occurs through
Operant learning
Shaping
observation alone Stimulus generalisation
Chaining
Fixed ratio reinforcement
Habituation
Positive reinforcement
Operant learning
Variable ratio reinforcement
Your answer is incorrect. Stimulus generalisation
Punishment
Chaining: This refers to reinforcing a series of related behaviours, each of which provides the
cue for the next to obtain a reinforcer. Chaining can be forward or backward chaining
according to where the first nodal point of learning starts. If it started from the end behaviour
and connected backwards, it is called backward chaining e.g. in completing a jigsaw puzzle a
child can be taught by starting from the whole puzzle and then removing pieces one by one.
Forward chaining is used in situations such as toilet training. Individual behaviours such as
defecating, removing one's trousers, calling for parents, running to the toilet may be seen as
'pieces' in a child ­ by chaining they can be made into a sequence leading to effective toilet
habits. 
In variable ratio reinforcement, random rewarding is seen i.e. it is not possible to predict when
the reward will occur following a response. Variable schedules generate a constant rate of
response as the chance of obtaining a reward stays the same at any time and any instance of
behaviour ­ so the organism keeps responding till it gets the reward. In contrast, with fixed
schedules where there is either a fixed time interval between rewards or fixed ratio between
rewards and responses, the organism can somewhat predict the reward. So, a reduction in
the response rate immediately after obtaining a reward is seen in fixed interval reinforcements
(if you will get the reward only after 10 minute gaps, why bother responding during the first 8­9
minutes?). 
According to Bandura's social learning theory, people can learn by observing the behaviour of
others and the outcomes of those behaviours without undergoing conditioning paradigms.
Learning is defined as the relatively permanent change in behaviour, according to
behaviourists. Cognition plays a role in learning. 
The correct answer is: This principle can be used to teach toilet training in the learning
disabled, after having the process broken down into simple steps that link to each other –
Chaining, Gambling is an example of this. – Variable ratio reinforcement, This type of learning
occurs through observation alone – Modelling

1274
Question 70 MoK Assessment EMI022
Not answered Hallucinatory phenomena
Which of the listed descriptive psychopathological terms is the most appropriate descriptor for
Marked out of 5.00
each of the  symptoms below?
Flag question

Reflex hallucination
Hypnagogic hallucination
Hypnopompic hallucination
Visual experience of seeing oneself
Lilliputian hallucination
Ictal hallucinations
Haptic hallucinations
Olfactory hallucinations
Kinaesthetic hallucination
Autoscopic hallucinations
Autoscopic hallucinations
Lilliputian hallucination
An individual complains seeing small Hypnagogic hallucination
Migrainous hallucination
sized figures such as dwarfs or
Kinaesthetic hallucination
midgets
Olfactory hallucinations
Ictal hallucinations
Reflex hallucination
Hypnopompic hallucination
Migrainous hallucination
Haptic hallucinations
Olfactory hallucinations
Migrainous hallucination
Hypnagogic hallucination
Bugs are crawling over one's skin
Hypnopompic hallucination
Lilliputian hallucination
Kinaesthetic hallucination
Haptic hallucinations
Reflex hallucination
Kinaesthetic hallucination
Ictal hallucinations
Ictal hallucinations
Seeing flashes of light as part of a Autoscopic hallucinations
Haptic hallucinations
seizure activity Hypnagogic hallucination
Migrainous hallucination
Hypnopompic hallucination
Lilliputian hallucination
Reflex hallucination
Migrainous hallucination
Olfactory hallucinations
Haptic hallucinations
Autoscopic hallucinations
Hypnopompic hallucination
Simple visual hallucinations of
Hypnagogic hallucination
geometric patterns with micropsia Autoscopic hallucinations
and macropsia Ictal hallucinations
Olfactory hallucinations
Reflex hallucination
Lilliputian hallucination
Kinaesthetic hallucination

Your answer is incorrect.
Explanation: 

1275
Autoscopic hallucinations are hallucinations of one's physical self. It is predominantly seen in
males at a ratio of 2:1, with depression being the commonest psychiatric cause. In negative
autoscopy, one looks into a mirror and sees no image at all. 
Lilliputian hallucinations can occur in visual mode ­ they usually involve seeing tiny people or
animals and are seen in delirium tremens and unlike other organic visual hallucinations.
Lilliputian hallucinations can often be amusing though while withdrawing from alcohol,
anxiety and fear predominate. These are not the same as micropsia. 
Case 3: This is formication, a form of haptic hallucination. Simple haptic hallucinations such
as the feeling that bugs are crawling over one's skin are common in alcohol withdrawal
syndrome and cocaine intoxication. 
Ictal hallucinations can occur as a part of seizure activity. They may be simple images, such
as flashes of light or even elaborate ones such as visual recollections of past experiences.
They last only seconds to minutes and are typically brief. 
Migrainous hallucinations are commonly reported by patients with migraine (50%). These are
simple visual hallucinations of geometric patterns with micropsia and macropsia.
The correct answer is: Visual experience of seeing oneself – Autoscopic hallucinations, An
individual complains seeing small sized figures such as dwarfs or midgets – Lilliputian
hallucination, Bugs are crawling over one's skin – Haptic hallucinations, Seeing flashes of
light as part of a seizure activity – Ictal hallucinations, Simple visual hallucinations of
geometric patterns with micropsia and macropsia – Migrainous hallucination

Question 71 MoK Assessment EMI024
Not answered Memory related phenomenon
Choose the appropriate terms associated with each of the following descriptions:
Marked out of 5.00

Flag question
Hyperesthesia
Synaesthesia
Exaggerated degree of retention and Paramnesia
recall Hypoaesthesia
Lethologica
Regression
Eidetic images
Suppression
Synaesthesia
Hypermnesia
Lethologica
Temporary inability to remember a
Repression
Suppression
name or proper noun Hyperesthesia
Eidetic images
Paramnesia
Hypermnesia
Hypoaesthesia
Lethologica
Regression
Paramnesia
Repression
Repression
Regression
Suppression
Confusion of facts and fantasies
Synaesthesia
Eidetic images
Hypoaesthesia
Hypermnesia
Hyperesthesia

1276
Hyperesthesia
Hypermnesia
Visual memories with almost Suppression
hallucinatory vividness Lethologica
Eidetic images
Synaesthesia
Regression
Hypoaesthesia
Repression
Repression
Suppression
Unconscious forgetting of
Paramnesia
Eidetic images
unacceptable ideas Lethologica
Regression
Paramnesia
Hypoaesthesia
Hypermnesia
Synaesthesia
Your answer is incorrect. Hyperesthesia
Explanation: Hypermnesia is an ability to remember material that ordinarily is not retrievable
and is an exaggerated degree of retention and recall. Lethologica is the temporary inability to
remember a name or proper noun. Paramnesia refers to the falsification of memory by
distortion of recall leads to a confusion of facts and fantasies. Eidetic imagery refers to the
phenomenon of experiencing visual memories of almost hallucinatory vividness.Repression
is a type of defence mechanisms, which refers to unconscious forgetting of unacceptable
ideas or impulses
The correct answer is: Exaggerated degree of retention and recall – Hypermnesia, Temporary
inability to remember a name or proper noun – Lethologica, Confusion of facts and fantasies –
Paramnesia, Visual memories with almost hallucinatory vividness – Eidetic images,
Unconscious forgetting of unacceptable ideas – Repression

Question 72 MoK Assessment EMI025
Not answered Personality disorders
Choose one option each from above list:
Marked out of 5.00

Flag question A 49­year­old woman came to A&E
department and insisted on seeing
an expert. He looks much younger
than his years. He tells that he has
recently been made bankrupt, and he
Borderline personality disorder
is facing fraud charges, which he Antisocial personality disorder
dismisses as just the ordinary sort of Dependent personality disorder
thing. His second wife has left him; he Schizoid personality disorder
has three children with whom he has Avoidant personality disorder
no contact. He tells that he cannot Hysterical personality disorder
cope living with them anymore. He Schizotypal personality disorder
would like to escape this mess with
Narcissistic personality disorder
the most dignity, and a clean suicide
Obsessive­compulsive personality disorder
would be the way forward.
Paranoid personality disorder

1277
A 23­year­old woman was seen in
Schizotypal personality disorder
the clinic. She tells you that she was
Antisocial personality disorder
abused by her stepfather and has
often thought of killing him and Paranoid personality disorder
herself. She thinks she might be Schizoid personality disorder
Obsessive­compulsive personality disorder
lesbian but is not sure. She feels tired
and empty all the time Borderline personality disorder
Dependent personality disorder
Narcissistic personality disorder
Avoidant personality disorder
Hysterical personality disorder
A 42­year­old woman has always Borderline personality disorder
been impeccable and conscientious; Schizoid personality disorder
She often stays long after regular Avoidant personality disorder
working hours to check on
Antisocial personality disorder
punctuation errors. She had to do all
jobs by herself and could not
Schizotypal personality disorder
delegate work. Narcissistic personality disorder
Obsessive­compulsive personality disorder
A 35­year­old man refuses to provide Hysterical personality disorder
Schizotypal personality disorder
answers to standard questions during Paranoid personality disorder
an initial clerking and threatens to
Obsessive­compulsive personality disorder
Dependent personality disorder
Antisocial personality disorder
stop the interview if recording his
telephone number is insisted upon. It Borderline personality disorder
seems that he has taken many Narcissistic personality disorder
hospitals to court for suspicions about Dependent personality disorder
how his personal data has been Paranoid personality disorder
handled. Avoidant personality disorder
Schizoid personality disorder
A 33­year­old night security at a local Obsessive­compulsive personality disorder
hospital prefers to be alone Hysterical personality disorder
Paranoid personality disorder
whenever possible. He has no Antisocial personality disorder
friends, and he does not socialise. He
Dependent personality disorder
does not keep update with current
affairs and has no sexual interests.
Hysterical personality disorder
He spends most time daydreaming. Narcissistic personality disorder
Avoidant personality disorder
Schizotypal personality disorder
Schizoid personality disorder
Your answer is incorrect. Borderline personality disorder
Explanation:
Narcissistic personality disorder­Exaggerated sense of own importance, frequently self­
centered, Intolerant of other people, grandiose plans and ideas, cravings for attention and
admiration. 
Borderline personality disorder­ poor self­image, unstable personal relationships, Impulsive
behaviour in areas such as personal safety and substance misuse, may self­harm, feel
suicidal and act on these feelings, experience instability of mood, , Have episodes of micro­
psychosis, feelings of chronic emptiness and fears of abandonment ­ rejection sensitivity
hence form intense but short lasting relations. 
The cardinal features of Obsessive­Compulsive personality disorder are "an exaggerated and
pervasive attempt to control those who are close to them, to control every uncertainty, and to
control their thoughts and emotions".
Paranoid personality disorder: Suspicious of other people and their motives, hold
longstanding grudges against people, believe others are not trustworthy, emotionally
detached and feel other people are deceiving, threatening, or making plans against them. 

1278
Schizoid personality disorder: Have difficulties in expressing emotions, particularly around
warmth or tenderness, Prefer loneliness, aloof or remote, have difficulty in developing or
maintaining social relationships, remain unaware of social trends, unresponsive to praise or
criticism.
The correct answer is: A 49­year­old woman came to A&E department and insisted on seeing
an expert. He looks much younger than his years. He tells that he has recently been made
bankrupt, and he is facing fraud charges, which he dismisses as just the ordinary sort of thing.
His second wife has left him; he has three children with whom he has no contact. He tells that
he cannot cope living with them anymore. He would like to escape this mess with the most
dignity, and a clean suicide would be the way forward. – Narcissistic personality disorder, A
23­year­old woman was seen in the clinic. She tells you that she was abused by her
stepfather and has often thought of killing him and herself. She thinks she might be lesbian
but is not sure. She feels tired and empty all the time – Borderline personality disorder, A 42­
year­old woman has always been impeccable and conscientious; She often stays long after
regular working hours to check on punctuation errors. She had to do all jobs by herself and
could not delegate work. – Obsessive­compulsive personality disorder, A 35­year­old man
refuses to provide answers to standard questions during an initial clerking and threatens to
stop the interview if recording his telephone number is insisted upon. It seems that he has
taken many hospitals to court for suspicions about how his personal data has been handled. –
Paranoid personality disorder, A 33­year­old night security at a local hospital prefers to be
alone whenever possible. He has no friends, and he does not socialise. He does not keep
update with current affairs and has no sexual interests. He spends most time daydreaming. –
Schizoid personality disorder

Question 73 MoK Assessment EMI026
Not answered Conflicts and Defences
Choose the best option from the list provided for each description below.
Marked out of 5.00

Flag question
Isolation
Which defence mechanisms are used Passive­aggressive
in patients with obsessive­ Possession
compulsive personality disorder Dissociation
(Choose three answers) Projection
Reaction formation
Undoing
Acting out
Projection
Denial
Displacement
Introjection
Acting out
Displacement
Passive­aggressive
Isolation
Which defence mechanism is used
when a person refuses to accept the
Introjection
impact of his actions?
Undoing
Possession
Denial
Reaction formation
Dissociation

1279
Which defence mechanisms are used
Introjection
in patients with impulse control
Denial
disorder?
Undoing
Projection
Reaction formation
Possession
Displacement
Passive­aggressive
Isolation
Your answer is incorrect. Dissociation
Acting out
Explanation: Isolation (obsessional thoughts), undoing (compulsive rituals) and reaction
formation (obsessive personality disorder) are three defence mechanisms employed in
patients with OCD. Reaction Formation­ When someone is confronted with disturbing desires
or impulses, he or she may actively express the opposite impulse. This involves openly
displaying a particular attitude that is opposite of disturbing repressed traits. Undoing­ When a
person acts inappropriately, it sometimes produces anxiety. To counter this anxiety, the
person may try to negate the original behaviour. For example, a child who becomes unruly at
the dinner table but then offers to help during cleanup. Acting out refers to an expression of an
unconscious wish or impulse, through action, to avoid being conscious of an accompanying
affect. The unconscious fantasy is lived out impulsively in behaviour, thereby gratifying the
impulse instead of prohibiting it, seen in impulse control disorders. 
Ref: Defense Mechanisms ­ What are Defense Mechanisms?,
http://panicdisorder.about.com/od/symptoms/a/DefMech.htm (accessed April 1, 2015).
The correct answer is: Which defence mechanisms are used in patients with obsessive­
compulsive personality disorder (Choose three answers) – Undoing, Isolation, Reaction
formation, Which defence mechanism is used when a person refuses to accept the impact of
his actions?
– Denial, Which defence mechanisms are used in patients with impulse control disorder? –
Acting out

Question 74 MoK Assessment EMI030
Not answered Substance misuse­ signs & symptoms
Identify the type of illicit substance misuse associated with each of the following clinical signs
Marked out of 5.00
& symptoms
Flag question

Ecstasy
LSD
Nasal mucosal atrophy and septal Heroin
perforation Cannabis
Ketamine
Amphetamine
Crack cocaine
Benzodiazepines
Cannabis
Crack cocaine
Ecstasy
LSD
Benzodiazepines
Vivid afterimages & recurring intense Ketamine
sensations Heroin
Amphetamine

1280
LSD
Benzodiazepines
Amphetamine
Apathy and amotivational syndrome
Crack cocaine
Ketamine
Heroin
Ecstasy
Cannabis
Ecstasy
Crack cocaine
Cannabis
Water intoxication
Ketamine
Amphetamine
Benzodiazepines
Heroin
LSD
LSD
Crack cocaine
Ketamine
Pinpoint pupils
Cannabis
Heroin
Amphetamine
Ecstasy
Benzodiazepines

Your answer is incorrect.
Explanation: 
Cocaine snorting produces nasal mucosal atrophy and nasal perforation.
Long after ingesting a hallucinogen like LSD, a person can experience a recurrence of
hallucinogenic symptoms. This condition can be diagnosed as hallucinogen persisting
perception disorder using DSM­5. This reexperiencing of perceptual symptoms following
cessation of hallucinogen use is characterized by seeing geometric hallucinations, false
perceptions of movement in the peripheral visual fields, flashes or intensified colours, trails of
images of moving objects, positive afterimages, halos around objects, macropsia, and
micropsia. This is also called flashback phenomenon. 
Chronic cannabis use is can cause lethargy, apathy, loss of interest, anergia, reduced drive
and lack of ambition, which is collectively known as the amotivational syndrome. Some
authors do not agree with the validity of this syndrome. 
Water intoxication associated with drug abuse is typically caused by MDMA ("ecstasy"), often
secondary to excessive sweating. MDMA is commonly used as a party drug; so abusers often
dance for several hours, after preparing themselves by drinking a lot of water beforehand. As
a lot of salts is lost through the sweat but does not get replenished by water loading, the
osmotic balance is affected, resulting in water intoxication. 
Acute opioid intoxication (e.g. through the use of heroin) is characterized by bradycardia,
hypotension, respiratory depression, subnormal body temperature and pin­point pupils. 
[Ref: Gowing LR et al. (2002) The health effects of ecstasy: a literature review. Drug Alcohol
Rev 21: 53­63]
The correct answer is: Nasal mucosal atrophy and septal perforation – Crack cocaine, Vivid
afterimages & recurring intense sensations – LSD, Apathy and amotivational syndrome –

1281
Cannabis, Water intoxication – Ecstasy, Pinpoint pupils – Heroin

Question 75 MoK Assessment EMI033
Not answered Culture bound syndromes ­ Descriptions
What is the most appropriate diagnosis in the following cases?
Marked out of 4.00

Flag question
Susto
A 25­year­old Srilankan man Dhat
presenting with lethargy, sleep Brain Fag
disturbance and impotence which he Amok
attributes to the loss of semen Latah
Frigophobia
Windigo
Taijin Kyofusho
Koro
A 4­­year­old Malaysian woman Zar
Susto
presenting with dissociation, Koro
Windigo
automatic obedience and echolalia in
Frigophobia
response to the news of her son's
death
Taijin Kyofusho
Zar
Latah
Dhat
Brain Fag
Brain Fag
A 25­year­old Japanese woman who Susto
has a persistent and overwhelming
Amok
Windigo
fear that his appearance and bodily Taijin Kyofusho
odour is offensive to other people Zar
Amok
Dhat
Frigophobia
Dhat
A 50­year­old Ethiopian man has an Latah
Windigo
experience of spirit possession. His Koro
Taijin Kyofusho
symptoms include episodes of
Zar
laughing inappropriately, shouting
and hitting his head against a wall
Brain Fag
Latah
Susto
Amok
Frigophobia
Your answer is incorrect. Koro
Explanation: 
Dhat/Jiryan/Sukra­Prameha refers to semen­loss syndrome noted in the Indian subcontinent.
Young adult males present with excessive fatigue and impotence attributed to loss of vitality
through ejaculated semen.
Latah is seen in Malaysia and Indonesia. It includes hypersensitivity to sudden fright, often
with echopraxia, echolalia, command obedience, and dissociative or trance­like behavior.
This condition is more frequent in middle­aged women. 
Taijin kyofusho (Japan) is "a syndrome of intense fear that one's body, body parts, or bodily
functions are displeasing, embarrassing, or offensive to other people in appearance, odor,
facial expressions, or movements". 

1282
Zar is an exclusively North African syndrome: where subjects present with an experience of
spirit possession. Symptoms may include "dissociative episodes with laughing, shouting,
hitting the head against a wall, singing, or weeping". Individuals may show withdrawal,
refusing to eat or carry out daily tasks, or may develop a long­term relationship with the
possessing spirit. 
Ref: Timothy Hall., Index of Culture­Bound Syndromes By Culture,
http://mccajor.net/cbs_cul.html (accessed April 1, 2015).
The correct answer is: A 25­year­old Srilankan man presenting with lethargy, sleep
disturbance and impotence which he attributes to the loss of semen – Dhat, A 4­­year­old
Malaysian woman presenting with dissociation, automatic obedience and echolalia in
response to the news of her son's death – Latah, A 25­year­old Japanese woman who has a
persistent and overwhelming fear that his appearance and bodily odour is offensive to other
people – Taijin Kyofusho, A 50­year­old Ethiopian man has an experience of spirit
possession. His symptoms include episodes of laughing inappropriately, shouting and hitting
his head against a wall – Zar

Finish review

1283
 Home Mock Paper Practice Tests

Mock Paper A(1)

Started on Thursday, 2 July 2015, 1:22 AM
State Finished
Completed on Thursday, 2 July 2015, 5:38 AM
Time taken 4 hours 15 mins
Overdue 2 hours 45 mins
Grade 0.00 out of 100.00
Feedback You are on the right track. Could you spend some more time on your revision?

Question 1 MoK HumanDevelopment 024
Not answered Which of the following attachment behaviour pattern has been commonly reported in children
subjected to abuse?
Marked out of 1.00

Flag question Select one:
Anxious/avoidant attachment
Secure attachment
Multiple attachment

Disorganised attachment
Ambivalent/resistant attachment

Your answer is incorrect.
In addition to the 3 major types of attachment noted by Ainsworth, Main and Solomon in 1990
described a fourth category of attachment for children who did not fit neatly into the three
patterns described by Ainsworth. This fourth category is disorganised/ disorientated type. The
children with this pattern appear insecure, fearful, with a dazed look, act as if it is frightened of
the mother and show overly conflicted behaviour during the strange situation procedure. 15%
of children in middle­class non­clinical groups show this type of attachment behaviour pattern.
This is seen more often in maltreated or maternally deprived children. May be a precursor to
personality difficulties later or dissociative experiences. The mother may have been herself
abused as a child. Some studies have reported the much higher proportion of this category of
attachment behaviour pattern in children subjected to abuse.
The correct answer is: Disorganised attachment

Question 2 MoK HumanDevelopment 027
Not answered Fears that show no particular age trend include:

Marked out of 1.00
Select one:

1284
Flag question
Fear of open spaces
Fear of snakes
Fear of the dark
Fear of animals
Fear of sex

Your answer is incorrect.
Fear of animals ­ age 3; fear of the dark ­ age 4 or 5; fear of imaginary creatures and
supernatural things ­ 5 plus; Fear of bodily injury, disasters ­ 5 to 8 years; fears that arrive in
late childhood or adult life ­ fear of sex/open spaces, fear that occurs from teenage onwards
includes fear of failure, illness and death. Fears that show no particular age trend include fear
of snakes or storms.
The correct answer is: Fear of snakes

Question 3 MoK HumanDevelopment 028
Not answered Deductive reasoning is achieved in which stage of Piaget's cognitive development theory?

Marked out of 1.00
Select one:
Flag question
Operational stage
Concrete operational stage
Pre­operational stage

Sensorimotor stage
Formal operational stage

Your answer is incorrect.
Formal operational stage is characterized by the emergence of abstract reasoning, logical
thinking and hypothesis testing. Hypotheticodeductive reasoning is achieved in a proportion
of children after age 12.
The correct answer is: Formal operational stage

Question 4 MoK HumanDevelopment 032
Not answered An 8­year­old boy is most likely to fall under which stage of Erikson's psychosocial
development?
Marked out of 1.00

Flag question Select one:
Trust vs mistrust
Identity vs role confusion
Initiative vs guilt
Autonomy vs shame

1285
Industry vs inferiority

Your answer is incorrect.
Between 6­11 years, the life crisis described by Erikson was industry vs. inferiority. The
outcome expected is a sense of achievement/inadequacy.
The correct answer is: Industry vs inferiority

Question 5 MoK HumanDevelopment 033
Not answered In Ainsworth's strange situation experiment, a child showed no signs of distress at leaving the
mother and ignored her on return. Which type of attachment behaviour is this?
Marked out of 1.00

Flag question Select one:
Disorganized attachment behaviour

Insecurely attached anxious ambivalent behaviour
Insecurely attached anxious resistant behaviour
Insecurely attached anxious avoidant behaviour
Securely attachment behaviour

Your answer is incorrect.
Avoidant children (about 20%) don't appear too distressed by the separation, and, upon
reunion, actively avoid seeking contact with their parent, sometimes turning their attention to
play objects on the laboratory floor.
The correct answer is: Insecurely attached anxious avoidant behaviour

Question 6 MoK HumanDevelopment 037
Not answered The mode of language designed to provide a means of communication for those with a
learning disability is
Marked out of 1.00

Flag question Select one:
COPE method
BASDEC method
Jarmen system
Makaton system
Barthel method

Your answer is incorrect.

1286
Makaton is a method of sign language designed to provide a means of communication to
individuals who cannot communicate efficiently by speaking. It has been used effectively in
individuals with learning disabilities, autistic spectrum disorder and multisensory impairment
The correct answer is: Makaton system

Question 7 MoK HumanDevelopment 038
Not answered Who proposed a theory of identity development in children, independent of their mothers?
Marked out of 1.00
Select one:
Flag question
Freud
Rutter
Margaret Mahler
Spitz
Winnicott

Your answer is incorrect.
Margaret Mahler described the development of a sense of identity in young children,
independent of their mothers. This is called separation­individuation theory, and the proposed
stages are supposed to be universal in all children.
The correct answer is: Margaret Mahler

Question 8 MoK HumanDevelopment 041
Not answered Which of the following type of parents will not set any limit for child's behaviour but show poor
impulse control and act aggressively if boundaries are violated?
Marked out of 1.00

Flag question Select one:
Authoritarian
Authoritative
Permissive
Good enough parent
Neglectful

Your answer is incorrect.
Authoritarian style parents impose strict rules that can be associated with low self­esteem and
social withdrawal in the children. Permissive style parents set no limits, leading to poor
impulse control and aggression. The neglectful style of parenting can lead to poor self­
esteem, impaired self­control, and increased aggression. Parents with an authoritative style
set firm rules but share decision­making with their children, creating a warm, loving
environment, which promotes self­reliance and positive self­esteem.
The correct answer is: Permissive

1287
Question 9 MoK HumanDevelopment 042
Not answered A 4­year­old child now recognizes she's a girl and wants to spend more time with her dad.
According to Freud she's at what phase of psychosexual development?
Marked out of 1.00

Flag question Select one:
Genital phase
Oral phase
Anal phase

Latency phase
Phallic phase

Your answer is incorrect.
In phallic phase (3­5 years), the main site of gratification is genital area. The girl develops
penis envy (discontent with female genitalia following a fantasy that they result from a loss of
penis). This is theorized by Freud to lead to a wish to 'receive' the penis and to bear a child.
The resolution occurs by identification with the mother. This phase has been called as 'Electra
complex'.
The correct answer is: Phallic phase

Question 10 MoK Basic Psychology 051
Not answered Which one of the following is an objective test of personality?

Marked out of 1.00
Select one:
Flag question
Rorschach inkblot test
Minnesota multiphasic personality inventory
Sentence completion test
Draw a person test
Thematic apperception test

Your answer is incorrect.
Minnesota multiphasic personality inventory MMPI is a popular inventory for measuring
personality. It has ten scales with clinical labels. It is NOT a projective test. Rorschach inkblot
test and Murray's Thematic Apperception Test are projective tests; so are sentence completion
and draw­a­person tests. Projective tests use ambiguous stimuli, and the patient is expected
to generate narratives based on them; rating scales may or may not be used in conjunction to
rate such measures.
The correct answer is: Minnesota multiphasic personality inventory

Question 11 MoK Basic Psychology 052
Not answered The best neuropsychological test of semantic memory is

1288
Marked out of 1.00 Select one:
Flag question Naming objects and describing their meanings
Address recall
Rivermead behavioural memory test

Vocabulary test
Weschler's Memory Scale

Your answer is incorrect.
Semantic Memory is the accumulation of facts and experience gained over a lifetime. Tasks
dependent on semantic memory include object naming, generation of definitions for spoken
words, word­picture and picture­picture matching and the generation of exemplars on
category fluency tests (e. g. animals). Weschler's Memory Scale reports on Auditory Memory,
Visual Memory, Visual Working Memory, Immediate Memory, and Delayed Memory ­ but not
specifically on Semantic Memory (Excerpt from Hodges et al., Brain (1992), 115, 1783­1806).
The correct answer is: Naming objects and describing their meanings

Question 12 MoK Basic Psychology 055
Not answered Which of the following scales utilizes the theory of Big Five personality constructs?

Marked out of 1.00
Select one:
Flag question
Eysenck's Personality Inventory
IPDE

NEO Inventory
Thematic Apperception Test
Minnesota Multiphasic Personality Inventory

Your answer is incorrect.
The NEO Personality Inventory, or NEO PI, is a 240­item measure of the Big Five personality
traits: Extraversion, Agreeableness, Conscientiousness, Neuroticism, and Openness to
Experience.
The correct answer is: NEO Inventory

Question 13 MoK Basic Psychology 056
Not answered In Skinner box experiments, rats receive food pellets in response to pressing a lever. What
type of learning is this?
Marked out of 1.00

Flag question Select one:
Backward conditioning
Operant conditioning

1289
Forced conditioning
Classical conditioning
Forward conditioning

Your answer is incorrect.
Operant conditioning also known as instrumental conditioning was first demonstrated by
Skinner using a box in which a rat received food pellets in response to pressing a lever.
Operant conditioning refers to a type of learning in which the likelihood of a behaviour being
repeated depends on the associated consequences. The ABC refers to antecedents,
behaviour and consequences.
The correct answer is: Operant conditioning

Question 14 MoK Basic Psychology 058
Not answered Star charts are effective in managing certain behavioural problems. They work on the basis of

Marked out of 1.00
Select one:
Flag question
Modelling
Social learning
Operant conditioning
Classical conditioning
Covert conditioning

Your answer is incorrect.
Star charts are used to positively reinforce each dry night in children who have a delay in
developing bladder control. The charts are often used along with a no­blaming parental
attitude towards the enuresis.
The correct answer is: Operant conditioning

Question 15 MoK Basic Psychology 060
Not answered A child was told to finish homework tasks before he can play outside. This is based on
Marked out of 1.00
Select one:
Flag question
Reciprocal inhibition
Classical conditioning
Cognitive mapping
Premack's principle
Operant conditioning

1290
Your answer is incorrect.
Premack's principle states that more probable behaviors will reinforce less probable
behaviors. Premack's Principle suggests that if a person wants to perform a given activity, the
person will perform a less desirable activity to get at the more desirable activity.
The correct answer is: Premack's principle

Question 16 MoK Basic Psychology 061
Not answered With respect to memorising a list of words, which one among the following statements is true?
Marked out of 1.00
Select one:
Flag question
In immediate recall the primacy effect is stronger than the recency effect
The retroactive inhibition refers to prior learning interfering with subsequent
learning
Primacy effect is related to initial items receiving most consolidation

Elaborative rehearsal helps to store information in STM.
Retrieval of material from semantic long term memory is an implicit process
which requires clear knowledge of the time of acquisition of the semantic
knowledge

Your answer is incorrect.
An accurate recollection of an item is more likely if it is one of the first (primacy effect) or last
items (recency effect) to be learnt. Primacy is because initial items receive most consolidation
and recency is because immediate information is still in STM. The Proactive inhibition of
'interference theory' refers to prior learning interfering with subsequent learning. Retroactive
inhibition is based on the fact that over time, new memories interfere with existing ones and
this interference effect is increased if there is a considerable similarity between the new
memory and existing ones. The other hypothesis for forgetting is decay theory due to the trace
of the memory fading with time.
The correct answer is: Primacy effect is related to initial items receiving most consolidation

Question 17 MoK Basic Psychology 062
Not answered Which of the following is a type of non­declarative memory?

Marked out of 1.00
Select one:
Flag question
Semantic memory
Autobiographical memory
Procedural memory
All of the above
Episodic memory

1291
Your answer is incorrect.
There are two forms of LTM, which includes declarative (explicit) and non­declarative (implicit)
memory and their subdivisions. Declarative memory has two components; episodic and
semantic memory. Non declarative memory is made of procedural memory for skills and
habits, priming, classical conditioning and non­associative learning.
The correct answer is: Procedural memory

Question 18 MoK Basic Psychology 064
Not answered A family has got a new dog. A 3 year old child in the family is afraid of dogs and would stay in
the same room with the dog only if he could sit on his mother's lap and had his favourite ice
Marked out of 1.00
cream. Select the learning theory principle employed here
Flag question
Select one:
Avoidance learning
Extinction
Operant conditioning
Reciprocal inhibition
Classical conditioning

Your answer is incorrect.
The modification of a behavioural pattern by the conditioning of responses that are
incompatible with the response to be eliminated is called as reciprocal inhibition.
The correct answer is: Reciprocal inhibition

Question 19 MoK Basic Psychology 066
Not answered Identify the term that best denotes the feeling of familiarity that accompanies the return of
stored material to consciousness;
Marked out of 1.00

Flag question Select one:
Jamais vu
Retrieval
Dejavu
Recognition
Recollection

Your answer is incorrect.
The feeling of knowing that characterizes recognition is a "fringe conscious" phenomenon,
that is, an event that has high accuracy but low reported conscious content (Baars, 2002).
There is considerable evidence that people are conscious of retrieved memories in a recall,
but not necessarily in a recognition task (e.g., Gardiner et al., 1998). For pioneering memory

1292
researchers like Ebbinghaus, indeed, the term "recall" meant retrieval to consciousness
(Excerpt from Franklin et al., The Role of Consciousness in Memory, Brain Minds and Media,
2005;93;1505). In numerous experiments, these differences result in striking dissociations
between subjective reports in "remember" vs. "know" ­types of retrieval.
The correct answer is: Recognition

Question 20 MoK Social Psychology 009
Not answered Which of the following can reduce groupthink?

Marked out of 1.00
Select one:
Flag question
Strong chairperson
Financial incentives
Unplanned meetings of the group
Seeking external opinion
Pairing up group members

Your answer is incorrect.
Various strategies that can reduce groupthink include encouraging open debate,
acknowledging the presence of groupthink, seeking external opinion, splitting the group into
smaller units for discussion, holding last chance meetings to encourage challenges and
reserving leader's opinions until after the groups discussion has been completed.
The correct answer is: Seeking external opinion

Question 21 MoK Social Psychology 010
Not answered People prefer relationships that appear to offer an optimum cost benefit ratio and with
expectations that our actions towards others will be reciprocated in some way. Which of the
Marked out of 1.00
following theories suggest this?
Flag question
Select one:
Cannon Baird theory
Functional attribution error
James Lange theory
Social Cognitive Theory
Social exchange theory

Your answer is incorrect.
Social exchange theory is a theory of social interaction based on the proposition that people
expect rewards and costs from a social exchange to be equitable. Most of our social
behaviour is influenced by the expectations that our actions towards others will be

1293
reciprocated in some way and relationships are based on a mutual exchange of benefits. The
US sociologist George Caspar Homans developed this theory. (Ref: Oxford Dictionary of
Psychology­ Pg 709)
The correct answer is: Social exchange theory

Question 22 MoK Sociocultural Psychiatry 025
Not answered The ethics to determine goodness from acts and intentions behind them rather than their
consequences is
Marked out of 1.00

Flag question Select one:
Teleologic ethics
Liberation ethics
Utilitarian ethics
Deontologic ethics
Virtual ethics

Your answer is incorrect.
Consequentialist moral theories are teleological: they aim at some goal state and evaluate the
morality of actions in terms of progress toward that state. The best­known version of
consequentialism is utilitarianism. As most clearly stated by Mill a proponent of the
utilitarianism (teleology), the basic principle of utilitarianism is: Actions are right to the degree
that they tend to promote the greatest good for the greatest number. In contrast, deontological
principles are duty based. Kant's theory is an example of a deontological or duty­based ethics:
it judges morality by examining the nature of actions and the will of agents rather than goals
achieved. A deontological theory looks at inputs rather than outcomes (Retrieved from Kay, C.
Utilitarianism, http://sites.wofford.edu/kaycd/utilitarianism/ ).
The correct answer is: Deontologic ethics

Question 23 MoK Sociocultural Psychiatry 026
Not answered A patient is next on waiting list to receive CBT. A new referral comes up which needs more
urgent treatment. The psychotherapist refuses to displace the next patient on the list as this
Marked out of 1.00
would be against his sense of duty and responsibility. Which ethical principle is he
Flag question upholding?

Select one:
Teleology
Beneficience
Deontology
Non maleficience
Autonomy

Your answer is incorrect.

1294
Deontology and Teleology are two alternative higher­order ethical principles concerning
medical practice. The term Deontology derives from the Greek 'Deon' for 'duty' indicating the
centrality of rules in governing medical practice. Accordingly, rights and duties determine
action and so it is also called as absolutism. The term Teleology derives its name from the
Greek 'Teleon', meaning 'purpose' and the central concept is that rather than rights, people
have interests, whether these are concerns, desires or needs. Accordingly, a broad judgment
of benefits and harm determine medical practice.
The correct answer is: Deontology

Question 24 MoK Sociocultural Psychiatry 035
Not answered Which of the following offers a guideline for principles of ethical research?
Marked out of 1.00
Select one:
Flag question
Hippocartic oath
Mt. Sinai declaration
Helsinki declaration
Bristol code
Warsaw pact

Your answer is incorrect.
The World Medical Association (WMA) has developed the Declaration of Helsinki as a
statement of ethical principles for medical research involving human subjects, including
research on identifiable human material and data.
The correct answer is: Helsinki declaration

Question 25 MoK Sociocultural Psychiatry 039
Not answered A 40 year old man has chest pain and dizziness. His bloods and ECG are normal. He is
advised to take medications, fat restricted diet, exercise. This is called 
Marked out of 1.00

Flag question Select one:
Illness behaviour
Illness
Sick role
Worried well
Depression

Your answer is incorrect.
The concept of illness behavior was largely defined and adopted during the second half of the
twentieth century. Broadly speaking, it is any behavior undertaken by an individual who feels
ill to relieve that experience or to better define the meaning of the illness experience.
The correct answer is: Illness behaviour

1295
Question 26 MoK Sociocultural Psychiatry 041
Not answered The first written account on ethical aspects of medicine was authored by

Marked out of 1.00
Select one:
Flag question
Burton
Childress
Hippocrates
Charaka
Ishaq bin Ali al­Ruhawi

Your answer is incorrect.
The first written book on medical ethics was authored by Ishaq bin Ali Rahawi. This book,
called Adab al­Tabib (Conduct of a Physician), is thought to be first published in 9th century
The correct answer is: Ishaq bin Ali al­Ruhawi

Question 27 MoK Sociocultural Psychiatry 044
Not answered Which of the following model of reducing stigma demands equal rights and criminal
responsibility for the mentally ill?
Marked out of 1.00

Flag question Select one:
Disability inclusion model
No fault approach
Libertarian model
Brain disease model
Individual growth model

Your answer is incorrect.
The libertarian approach advocates equal rights with no discrimination ­ both positive and
negative. It has the risk of inducing serial losses for the mentally ill at workplaces or courts.
The correct answer is: Libertarian model

Question 28 MoK Sociocultural Psychiatry 047
Not answered Tearooom Trade Study is an example of ethically controversial research practices. Which of
the following was the subject of investigation in this study?
Marked out of 1.00

Flag question Select one:
Response to hepatitis vaccine

1296
Response to injected cancer cells
Male sexual behaviours
Natural history of syphilis

Female contraception

Your answer is incorrect.
During 1960s, a sociologist called Laud Humphries followed up many men who had
anonymous sex in public places by tracing their number plates after falsely befriending them.
The research was conducted without explicit informed consent and became a matter of
debate, highlighting the important of ethics in scientific research in non­medical fields of
enquiry.
The correct answer is: Male sexual behaviours

Question 29 MoK Sociocultural Psychiatry 048
Not answered The nurses in your ward do not like a newly admitted patient. They are angry that she is trying
to split them up by giving conflicting messages at ward rounds. Which of the following
Marked out of 1.00
phenomenon is likely?
Flag question
Select one:
Introjection
Identification
Elimination
Malignant alienation
Transference

Your answer is incorrect.
Morgan (1979) coined the term malignant alienation to describe a process characterised by a
progressive deterioration in the relationship between carers (staff in a ward) and a patient,
including loss of sympathy and support from members of staff, who tended to construe these
patients' behaviour as provocative, unreasonable, or overdependent. In some instances, such
alienation may precede suicide/ attempted suicide of the patient (Excerpt from Watts &
Morgan, Br J Psychiatry. 1994 Jan;164(1):11­5.
http://www.ncbi.nlm.nih.gov/pubmed/8137091)
The correct answer is: Malignant alienation

Question 30 MoK Sociocultural Psychiatry 049
Not answered Which of the following is not a central antipsychiatry belief?

Marked out of 1.00
Select one:
Flag question
Medication and hospitalisation are harmful to the individual so treated
The scientific method cannot explain the subjective abnormalities of mental

1297
disorder as no direct observation can take place
Mental disorder can be best explained by social, ethical and political factors
Mind is a bodily organ and can be diseased
The labelling of individuals as 'ill' is an artificial device used by society to
maintain its stability in the face of challenge

Your answer is incorrect.
Antipsychiatry refers to the many different individuals and organizations who have criticised
the consideration of psychiatry as a scientific discipline ever since it became a recognized
medical specialty in the 1800s. Antipsychiatry advocates claim that psychiatry presents itself
as a healing art yet actually controls behaviour deemed abnormal, irrational or socially
unacceptable.
The correct answer is: Mind is a bodily organ and can be diseased

Question 31 MoK Sociocultural Psychiatry 050
Not answered The most common reason for doctors to be taken to the court is

Marked out of 1.00
Select one:
Flag question
Problems in the doctor/patient relationship
Misuse and abuse
Negligence
Not obtaining informed consent
Breaking confidentiality

Your answer is incorrect.
Negligence: This requires the plaintiff to prove three things 1. That the doctor owed a duty of
care to the particular patient 2. That the doctor was in breach of the appropriate standard of
care imposed by the law. 3. That the breach in duty of care caused the patient harm, meriting
compensation.
The correct answer is: Negligence

Question 32 MoK Sociocultural Psychiatry 051
Not answered With respect to population studies, cultural analysis refers to

Marked out of 1.00
Select one:
Flag question
Racial profile of groups
Norms in a group
Beliefs among the group members
Lingual diversity in a group

1298
Geographical clusters of separation

Your answer is incorrect.
Cultural analysis refers to the study of norms or behavior patterns that are typical of specific
cultural groups. These norms are often passed down from generation to generation by
observational learning by the groups gatekeepers ­ parents, teachers, religious leaders, and
peers. Cultural norms include how people choose marriage partners, attitudes toward alcohol
consumption, and acceptance (or rejection) of spanking children.
The correct answer is: Norms in a group

Question 33 MoK Sociocultural Psychiatry 053
Not answered Which of the following form the basis of official social classification in Britain?

Marked out of 1.00
Select one:
Flag question
Residential status

Land ownership
Financial status
Educational status
Occupational status

Your answer is incorrect.
Social class is determined by occupation, financial status (paying capacity), education, type of
residence, and leisure activities. In Britain, occupational classification forms the main mode of
social classification.
The correct answer is: Occupational status

Question 34 MoK Classification 051
Not answered Which personality disorder is the most likely in a 23­year­old man who is sensitive to rebuffs
and avoids close contact as a result?
Marked out of 1.00

Flag question Select one:
Paranoid personality disorder
Dissocial personality disorder
Narcissistic personality disorder
Schizoid personality disorder
Schizotypal disorder

Your answer is incorrect.

1299
A pervasive distrust and suspiciousness of others such that their motives are interpreted as
malevolent, beginning by early adulthood and present with (1) suspicion without sufficient
basis (2) preoccupation with unjustified doubts about the loyalty or trustworthiness of friends
or associates (3) reluctance to confide in others (4) reads hidden demeaning or threatening
meanings into benign remarks or events (5) persistently bears grudges, i.e., is unforgiving of
insults, etc.
The correct answer is: Paranoid personality disorder

Question 35 MoK Classification 052
Not answered A 28­year­old man has complained to local council numerous times about his neighbours
throwing rubbish into his garden. Being vengeful, he emptied his weekly rubbish collection
Marked out of 1.00
into their backyard before going in person to council office to make a complaint. He has
Flag question always had trouble with different neighbours and has changed 9 houses in last 3 years.

Select one:
Borderline personality disorder
Schizotypal personality disorder
Paranoid personality disorder
Schizoid personality disorder
Anankastic personality disorder

Your answer is incorrect.
Paranoid personality disorder: Suspicious of other people and their motives, hold
longstanding grudges against people, believe others are not trustworthy, emotionally
detached and feel other people are deceiving, threatening, or making plans against them.
Tenacious sense of personal rights, excessive self­importance with self­referential nature are
seen in paranoid personality disorder.
The correct answer is: Paranoid personality disorder

Question 36 MoK Classification 059
Not answered An 8 year old child present with multiple tics. Which of the following must be explored during
mental state examination?
Marked out of 1.00

Flag question Select one:
Presence of delusions
Presence of formal thought disorder
Absence of insight
Presence of obsessions
Psychomotor retardation

Your answer is incorrect.

1300
Obsessions commonly accompany tics especially in children. This is particularly true if the
diagnosis is Tourette's syndrome.
The correct answer is: Presence of obsessions

Question 37 MoK Classification 060
Not answered Which of the following is treated as a measurement of severity of depression in ICD 10?

Marked out of 1.00
Select one:
Flag question
Presence of psychomotor retardation
Presence of somatic symptoms
Presence of psychotic features
History of mania
Presence of suicidal ideas

Your answer is incorrect.
Irrespective of the total symptom count, the presence of psychotic symptoms contribute
towards the classification of a depressive episode as severe.
The correct answer is: Presence of psychotic features

Question 38 MoK Classification 061
Not answered Which of the following is a type of narcissistic defences?

Marked out of 1.00
Select one:
Flag question
Acting out
Repression
Projection
Suppression
Splitting

Your answer is incorrect.
People with narcissistic thinking and behavior strive to defend their fragile self­esteem
through projection and denial. They also use other primitive defenses.
The correct answer is: Projection

Question 39 MoK Classification 062
Not answered Select one typical feature of depressive episode with psychotic symptoms
Marked out of 1.00
Select one:
Flag question

1301
Prominent affective symptoms and mood congruent delusions
Systemised delusions
Persecutory delusions
Bizzare delusions
Non prominent affective symptoms and mood congruent delusions

Your answer is incorrect.
Affective symptoms of depression are prominent, and delusions are almost always mood­
congruent though the mere presence of mood incongruence need not negate a diagnosis of
psychotic depression. Bizarre delusions point to schizophrenia. In mania, delusions are often
fleeting and not systematized.
The correct answer is: Prominent affective symptoms and mood congruent delusions

Question 40 MoK Classification 064
Not answered In DSM IV axis 4 is a description of which of the following?

Marked out of 1.00
Select one:
Flag question
Psychiatric disorder
Psychosocial functioning
General medical health
Personality traits
Intellectual ability

Your answer is incorrect.
Psychosocial and environmental factors contributing to a psychiatric disorder were described
in axis 4. DSM­5 has abolished the multiaxial system.
The correct answer is: Psychosocial functioning

Question 41 MoK Classification 065
Not answered A 27­year­old soldier returns from war field with distressing feelings, flashbacks, nightmares,
and hypervigilance. Which of the following diagnosis is likely?
Marked out of 1.00

Flag question Select one:
Generalised anxiety disorder
Schizophrenia
Panic disorder
PTSD
Dissociative disorder

1302
Your answer is incorrect.
PTSD generally starts within six months; delayed onset is rare.
The correct answer is: PTSD

Question 42 MoK Classification 066
Not answered In comparison to DSM, ICD10 has the following features, except:

Marked out of 1.00
Select one:
Flag question
Includes schizotypal disorders under schizophrenia

Has acute polymorphic psychosis as a diagnosis
Post­psychotic depression is included under schizophrenia
Has dissocial personality disorder as a diagnostic entity
Requires 6 months criteria for schizophrenia

Your answer is incorrect.
ICD10 is somewhat more lenient in diagnosing schizophrenia. One month of symptom
duration is enough to diagnose schizophrenia using ICD10 where as six months duration
criteria is required for DSM­IV.
The correct answer is: Requires 6 months criteria for schizophrenia

Question 43 MoK Classification 067
Not answered Which of the following features validate a diagnosis of OCD when a patient presents with
repetitive thoughts?
Marked out of 1.00

Flag question Select one:
Thoughts spoken aloud
Resisting the thoughts
Possessing full control of the thoughts
Thoughts coming from an external agent
Repeating thoughts take a variety of new themes each day

Your answer is incorrect.
Ego dystonicity and resulting resistance is characteristic of OCD
The correct answer is: Resisting the thoughts

Question 44 MoK Classification 068

1303
Not answered Which of the following distinguishes depressive reaction of adjustment disorder from major
Marked out of 1.00
depressive disorder?

Flag question
Select one:
Resolution is seen within 6 months after termination of the stressor
Symptoms develop within 1 month of the onset of the stressor
Marked psychosocial and occupational impairment in functioning is seen
Symptoms are distinctly different from those seen in depressive episode
Follows a stressful life event

Your answer is incorrect.
Symptoms of adjustment disorder (depressive reaction) are phenomenologically same as
those seen in depression. In both disorders, these can follow a stressful life event. But in
adjustment reactions, one expects resolution of symptoms within 6 months of the cessation of
the stressor.
The correct answer is: Resolution is seen within 6 months after termination of the stressor

Question 45 MoK Classification 069
Not answered A patient complains of poor sleep due to paraesthesia in her legs. The most probable
diagnosis is
Marked out of 1.00

Flag question Select one:
Akathisia
Anxiety
Periodic limb movement disorder
Dyskinesia
Restless leg syndrome

Your answer is incorrect.
Patients with restless legs syndrome experience tingling, burning, itching (paresthesia) or
cramping of calf muscles; often relieved by moving or rubbing their legs.
The correct answer is: Restless leg syndrome

Question 46 MoK Classification 070
Not answered Who first described Bulimia Nervosa?

Marked out of 1.00
Select one:
Flag question
Klermann and Weissmann
Greisinger

1304
Russell
Liddle
Crow

Your answer is incorrect.
Russell sign is a classical skin marker of induced vomiting. Russell described bulimia nervosa
in 1979
The correct answer is: Russell

Question 47 MoK Classification 071
Not answered Which of the following personality disorders is not included in both ICD 10 and DSM 5?

Marked out of 1.00
Select one:
Flag question
Borderline
Paranoid
Histrionic
Sadistic
Schizoid

Your answer is incorrect.
Though described in the research manual, DSM does not include sadistic PD in its clinical
manual. It is not included in ICD either. The DSM Research criteria describe Sadistic
Personality Disorder as a pervasive pattern of cruel, demeaning and aggressive behaviour,
beginning by early adulthood.
The correct answer is: Sadistic

Question 48 MoK Classification 073
Not answered Mr Wells has spent most of his life researching about a time machine. He is unmarried and
has poor communication skills. He has extensive theories about extraterrestrial species. He
Marked out of 1.00
admits to unusual experiences wherein he is able to communicate with some extraterrestrial
Flag question species using clairvoyance. He is most likely to have

Select one:
Paranoid schizophrenia
Schizoid personality disorder
Schizotypal personality disorder
Cyclothymic personality

Paranoid personality disorder

1305
Your answer is incorrect.
This is a classical description of schizotypal disorder (a personality disorder in DSM­IV).
Patients may appear odd or eccentric, may have illusions, magical thinking, obsessions
without resistance, thought disorders and paranoia. Some of them are often members of
quasi­cultural groups and may believe in ESP, clairvoyance, etc. They may even claim
themselves to be mediums and may have transient psychotic features.
The correct answer is: Schizotypal personality disorder

Question 49 MoK Clinical Examination 021
Not answered Clock drawing test can be used to detect

Marked out of 1.00
Select one:
Flag question
Abnormal time perception
Episodic memory loss
Poor attention
Constructional dyspraxia
Disorientation

Your answer is incorrect.
A number of cognitive, motor and perceptual functions are required simultaneously for
successful completion of the clock drawing test. Visual­spatial organization (constructional
praxis), memory and executive function, auditory comprehension, visual memory, motor
programming, numerical knowledge, semantic instruction, inhibition of distracting stimuli, and
concentration and have all been highlighted as contributing to the successful drawing of
clock.
The correct answer is: Constructional dyspraxia

Question 50 MoK Clinical Examination 042
Not answered A 42­year­old man presents with motor clumsiness, poor coordination and unsteadiness of
gait. He is apathetic and his thinking process is slowed. He has a strong family history of
Marked out of 1.00
similar presentations. What is the most likely diagnosis?
Flag question
Select one:
CJD
Lewy body dementia
Huntington's disease
Alzheimer's dementia
Wilson's disease

Your answer is incorrect.

1306
A strong family history with motor disturbances in a patient with psychosis and cognitive
impairment suggests Huntington's.
The correct answer is: Huntington's disease

Question 51 MoK Clinical Examination 043
Not answered Which of the following is a good opening question when discussing compliance with
treatment?
Marked out of 1.00

Flag question Select one:
It is very important that you take your tablets
Are you taking your tablets?
How many times have you missed your tablets so far?
Why are you not taking your tablets?
Is there any difficulty with your tablets?

Your answer is incorrect.
Asking "Is there any difficulty with your tablets?" is less judgmental and least confrontational
among all the other given options.
The correct answer is: Is there any difficulty with your tablets?

Question 52 MoK Clinical Examination 046
Not answered Mr Z is a 49­year­old gentleman with history of memory loss and personality changes. He also
presents with wing beating tremors. The biochemical abnormality to be identified in this case
Marked out of 1.00
would be
Flag question
Select one:
Low serum ceruloplasmin
Low albumin
Low serum calcium
Low serum thyroxine
Low serotonin

Your answer is incorrect.
The diagnosis is Wilson's disease, which is caused by copper deposits in the lenticular nuclei
in the brain due to deficiency of serum ceruloplasmin. Wing beating tremors is a characteristic
feature of Wilson's disease.
The correct answer is: Low serum ceruloplasmin

Question 53 MoK Clinical Examination 047

1307
Not answered A patient is suspected to have parietal damage related to a vascular lesion. Which of the
Marked out of 1.00
following signs may be elicited in neurological examination?

Flag question
Select one:
Bitemporal hemianopia
Homonymous hemianopia
One sided blindness
Upper quadrantonopia
Lower quadrantanopia

Your answer is incorrect.
Lower homonymous quadrantanopias are usually caused by damage to the optic radiation as
it passes through the parietal lobes.
The correct answer is: Lower quadrantanopia

Question 54 MoK Clinical Examination 048
Not answered An anorexic patient repeatedly purges with laxatives. Which of the following is a possible
complication?
Marked out of 1.00

Flag question Select one:
Hyponatraemia
None of the above
Hyperkalaemia
Metabolic alkalosis
Metabolic acidosis

Your answer is incorrect.
In laxative induced diarrhoea, a large amount of bicarbonate may be lost in the stool. With
normal kidneys, the lost bicarbonate is replaced effectively, and a serious base deficit does
not develop. When there is poor renal blood flow due to hypovolaemia/starvation, base deficit
and acidosis develop rapidly. Acidosis also results from excessive production of lactic acid
when patients have severe diarrhoea.
The correct answer is: Metabolic acidosis

Question 55 MoK Clinical Examination 049
Not answered Which test would you do to screen the cognitive functions of elderly at bedside?

Marked out of 1.00
Select one:
Flag question
MOCA
Addenbrookes cognitive examination

1308
MMSE
Mini­COG
Clock drawing test

Your answer is incorrect.
The Mini­Mental State Examination (MMSE) is the standard screening instrument for
dementia. It takes 5­10 minutes to administer. It is a brief tool used to quickly assess the
cognitive functions of elderly at bedside, for grading cognitive impairment in elderly and
screening for dementia.
The correct answer is: MMSE

Question 56 MoK Clinical Examination 050
Not answered Read the strip of ECG shown below and identify the relevant abnormality. Blue lines indicate
PR intervals. A 'P wave' is also marked in the figure.
Marked out of 1.00

Flag question

Select one:
Complete heart block
Type I second degree heart block
Mobitz type II
Mobitz type I
Normal ECG

Your answer is incorrect.
Mobitz type I AV block: The PR interval shows progressive lengthening until one P wave fails
to be conducted and fails to produce a QRS complex. The PR interval resets to normal, and
the cycle repeats. (Ref: Houghton & Gray, Making sense of the ECG. Pg 120­122)
The correct answer is: Mobitz type I

Question 57 MoK Clinical Examination 051
Not answered Read the strip of ECG shown below and identify the relevant abnormality. Blue lines indicate
PR intervals. A P wave is also marked in the figure.
Marked out of 1.00

Flag question

Select one:
Type I second degree heart block

1309
Normal ECG
Mobitz Type II
Complete heart block
Mobitz type I

Your answer is incorrect.
Mobitz type 2 block: If the PR interval is fixed and normal but occasionally a P wave fails to
produce a QRS complex, the patient has Mobitz type 2 block. (Ref: Houghton & Gray, Making
sense of the ECG. Pg 120­122)
The correct answer is: Mobitz Type II

Question 58 MoK Clinical Examination 052
Not answered Choose one advantage that a MRI scan has over a CT scan of the brain:

Marked out of 1.00
Select one:
Flag question
MRI scan is cheaper to obtain than CT scans
MRI scan can get multiple views of brain anatomy
MRI scan is very useful for calcified lesions
MRI scan can be done in emergencies
MRI scan produces less claustrophobia

Your answer is incorrect.
Using MRI one can obtain multiple slices of different views that can be useful in analysing
brain pathology in greater detail.
The correct answer is: MRI scan can get multiple views of brain anatomy

Question 59 MoK Clinical Examination 053
Not answered A man was admitted with severe depression and on examination has bilateral ptosis and
reduced muscle tone. Which of the following medical condition is most likely?
Marked out of 1.00

Flag question Select one:
Myasthenia Gravis
Addison's disease
Carcinoid tumors

Diabetes
Hypothyroidism

1310
Your answer is incorrect.
Myasthenia often presents with ptosis with fatigue and reduced muscle tone upon
examination. The two classic features of myasthenic weakness are variability, in that
weakness changes over minutes or days, or even shifts between different muscles, and
fatigability, which means that the weakness worsens with repeated use and improves with
rest. Many people (15­20%) who are diagnosed with myasthenia develop depression
(Retrieved from http://www.neuroophthalmology.ca/textbook/disorders­of­eye­movements/iii­
neuromuscular­junction/i­myasthenia­gravis).
The correct answer is: Myasthenia Gravis

Question 60 MoK Clinical Examination 054
Not answered Mr. Y is a 63 year old man who presents with a dementia characterised by disinhibition, lack
of concern towards others and recklessness. Which of the following language deficits is he
Marked out of 1.00
most likely to experience first?
Flag question
Select one:
Receptive dysphasia
Dyslexia
Non­fluent expressive dysphasia
Dysarthria
Fluent expressive dysphasia

Your answer is incorrect.
In frontal lobe type of dementia the following frontal lobe behavioral abnormalities occur over
time: disinhibition, impulsivity, impersistence, inertia, loss of social awareness, neglect of
personal hygiene, mental rigidity, stereotyped behavior, and utilization behavior (ie, tendency
to pick up and manipulate any object in the environment). These descriptions included
language abnormalities such as reduced speech output, mutism, echolalia, and
perseveration.
The correct answer is: Non­fluent expressive dysphasia

Question 61 MoK Clinical Examination 055
Not answered Which of the following is NOT correct with regard to body language during a psychiatric
interview?
Marked out of 1.00

Flag question Select one:
Reduced eye contact suggests passive behaviour
Sitting on the edge of the chair suggests anxiety
Lack of congruent facial expression can occur in schizophrenia
Non­invasive but direct eye contact suggests aggressiveness
Lack of voice modulation is seen in depression

1311
Your answer is incorrect.
Reduced eye contact may suggest passive behaviour with a lack of assertiveness, but direct
eye contact does not mean aggressiveness if non­invasive. Eye contact is also subject to the
cultural background of the therapist and the patient.
The correct answer is: Non­invasive but direct eye contact suggests aggressiveness

Question 62 MoK Clinical Examination 056
Not answered A characteristic magnetic gait is seen in which of the following disorders?

Marked out of 1.00
Select one:
Flag question
Fronto­temporal dementia
Parkinson's disease
Lewy Body Dementia
Normal Pressure Hydrocephalus
Corticobasal degeneration

Your answer is incorrect.
Normal­pressure hydrocephalus is caused by a deficiency in the resorption of CSF and is
responsible for 2% to 5% of all dementias. Normal­pressure hydrocephalus causes a
characteristic triad of dementia, gait disturbance, and (later) incontinence. The dementia is
frontal­subcortical in nature, characterized by deficits in attention, visuospatial skills,
abstraction, judgment, and new learning. The gait slows, strides become shorter, and step
height decreases, producing a characteristic 'magnetic gait.' Incontinence is more likely to be
urinary than fecal.
The correct answer is: Normal Pressure Hydrocephalus

Question 63 MoK Clinical Examination 057
Not answered 'Sometimes when people are very depressed, they think of hurting themselves. Has this been
true for you?' What technique is being used in this question?
Marked out of 1.00

Flag question Select one:
Facilitation
Deception
Normalisation
Symptom exaggeration
Suggestion

Your answer is incorrect.
Normalisation helps in reducing guilt.
The correct answer is: Normalisation

1312
Question 64 MoK Clinical Examination 058
Not answered Which of the following features correlate well with the severity of Alzheimer's dementia?

Marked out of 1.00
Select one:
Flag question
Age of onset
Number of plaques
Size of plaques
Family history of dementia

Gender of the patient

Your answer is incorrect.
Nevertheless, the burden of neurofibrillary tangles is a much better correlate of severity than
the burden of plaques.
The correct answer is: Number of plaques

Question 65 MoK Clinical Examination 059
Not answered A patient revealed his drug dependency to you after an eye drop test failed to dilate his pupils.
Which of the following drug dependency will produce this effect?
Marked out of 1.00

Flag question Select one:
Cocaine
Alcohol
Cannabis
Opioids
Amphetamines

Your answer is incorrect.
It has been shown that tolerance develops to the miotic effects more than to constipation when
using µ­opioid agonists. It can be readily observed that patients taking large doses of opioids
over a long term do not have miotic pupils, unless they are measured in bright light or have
other conditions that produce miosis. Hence, dark pupillography can be useful to study opioid
dependence.Senay E, Shick J: Pupillography response to methadone challenge: Aid to
diagnosis of opioid dependence. Drug Alcohol Depend 1978; 3:133­8
The correct answer is: Opioids

Question 66 MoK Clinical Examination 060
Not answered Which of the following symptoms are observed secondary to hyperventilation?

Marked out of 1.00
Select one:

1313
Flag question
Nystagmus
Hyper­reflexia
Vertigo

Ataxia
Paraesthesia

Your answer is incorrect.
Patients with acute hyperventilation (often in the context of panic attack in psychiatry) often
present with agitation, hyperpnea and tachypnea, chest pain, dyspnea, wheezing, dizziness,
palpitations, tetanic cramps (carpopedal spasm), paresthesias, generalized weakness, and
syncope. They may complain of a sense of suffocation. Paresthesias occur more commonly in
the upper extremity and are usually bilateral. Unilateral paresthesias are left­sided in
approximately 80% of cases. Perioral numbness is very common.
http://www.emedicine.com/EMERG/topic270.htm
The correct answer is: Paraesthesia

Question 67 MoK Clinical Examination 061
Not answered Which of the following is the most prominent risk factor for depression that can be identified in
a clinical interview?
Marked out of 1.00

Flag question Select one:
Sub­average educational achievement
Lack of positive life events
Anxious personality traits
Family history of depression
Past history of depression

Your answer is incorrect.
The most prominent factor for depression that could be identified during a clinical interview is
having a past history of depression. If measured using validated scales, neuroticism could be
a risk factor too, but generally cross­sectional clinical interviews are not reliable in diagnosing
enduring personality traits.
The correct answer is: Past history of depression

Question 68 MoK Assessment EMI019
Not answered Delusions in clinical practice
Which one of the following descriptions below correctly refers to the psychopathological terms
Marked out of 4.00
above?
Flag question

1314
A 34­year­old psychotic patient sees Delusional perception
two white cars pulling up in front of
Delusional memory
his home. He is convinced that he is
about to be accused of being a
Delusional mood
paedophile. Delusions of infestations
Delusions of grandiosity
Delusional jealousy
Delusional elaboration
Delusions of control
Delusional perception
Delusions of reference
Delusions of grandiosity
A 40­year­old psychotic patient who Autochthonous delusions
Delusions of infestations
believes that parts of the Bible are
Delusional jealousy
about him directly.
Delusional memory
Delusions of reference
Delusional elaboration
A 33­year­old man requested Delusions of control
Delusions of grandiosity
admission to a hospital because he Delusional mood
Autochthonous delusions
had become convinced that God had Autochthonous delusions
Delusional perception
granted him the greatest possible sort
Delusions of reference
of mind and that coming into contact
with him would cure others of mental
Delusional elaboration
illnesses. Delusional memory
Delusional mood
Delusional jealousy
Delusional perception
A 34­year­old man with psychotic Delusions of infestations
Autochthonous delusions
patient on the ward recalled being Delusions of control
Delusional elaboration
anxious and a sense of 'something
Delusions of reference
about to happen' prior to his
admission Delusional mood
Delusions of grandiosity
Delusions of control
Delusions of infestations
Delusional memory
Your answer is incorrect. Delusional jealousy
Explanation: 
In delusional perception, a normally perceived object is given a new meaning, usually in the
sense of self­reference ­ the conclusion being entirely unwarranted, the perception is normal.
Hence, it is a two­staged process ­ ordinary perception preceding the attachment of
delusional significance. The percept is a real external object and not a hallucinatory
experience. The only type of delusion included in Schneider's first­rank symptoms is
delusional perception. 
Delusions of reference are beliefs that external events or situations are arranged in such a
way as to have a particular significance for or to convey a message to the affected individual.
In the above scenario, the patient believes that parts of the Bible 'refer' to him directly. 
Grandiose delusion is a delusional belief that one has special powers, is unusually rich or
powerful or that one has an exceptional destiny. It occurs in all psychotic illnesses but
particularly in manic illness. 
Delusional mood refers to the sense of perplexity and uncertainty that exists during a
prodrome of psychosis, usually ending in an autochthonous delusion, which will make sense
of the perplexity on arrival. Delusional mood/atmosphere can precede other primary
delusions. It is the only psychiatric phenomenon that can directly precede and be causally

1315
related to primary autochthonous delusions. Note that delusional mood is a specific affective
experience ­ not a thought or an idea. This abnormal mood state is characterized by
anticipatory anxiety and a sense of 'something about to happen'. 
(Ref: Oxford Handbook of psychiatry­Pg 84­87)
The correct answer is: A 34­year­old psychotic patient sees two white cars pulling up in front
of his home. He is convinced that he is about to be accused of being a paedophile. –
Delusional perception, A 40­year­old psychotic patient who believes that parts of the Bible are
about him directly. – Delusions of reference, A 33­year­old man requested admission to a
hospital because he had become convinced that God had granted him the greatest possible
sort of mind and that coming into contact with him would cure others of mental illnesses. –
Delusions of grandiosity, A 34­year­old man with psychotic patient on the ward recalled being
anxious and a sense of 'something about to happen' prior to his admission – Delusional mood

Question 69 MoK Assessment EMI027
Not answered Defence mechanisms in operation
Identify the defence mechanism used in each of the following situations:
Marked out of 4.00

Flag question
Rationalization
Suppression
A boy who is being bullied at school Denial
takes on the identity of Superman Regression
Intellectualization
Sublimation
Reaction formation
Dissociation
Regression
Repression
Repression
A woman is not able to recall the Reaction formation
abuse she sustained as a child Rationalization
Suppression
Denial
Dissociation
Sublimation
Reaction formation
An idle woman who is not performing Intellectualization
Repression
well at her job is fired. She states this Dissociation
is because her colleague is bad­
Intellectualization
mouthing her rather than admitting
her poor performance.
Suppression
Rationalization
Regression
Sublimation
Intellectualization
Denial
Repression
A man who was diagnosed to have Denial
cancer says, "This cannot be true... I
Reaction formation
follow a healthy lifestyle."
Regression
Dissociation
Sublimation
Rationalization
Suppression
Your answer is incorrect.
Explanation: 

1316
Reaction formation is the transformation of unacceptable thoughts and feelings into their
opposite.
Repression refers to the process by which impulses, thoughts and memories are pushed out
or excluded from the conscious mind. 
Rationalization involves offering rational explanations in an attempt to justify attitudes, beliefs,
or behaviour that may otherwise be unacceptable. Such underlying motives are usually
instinctually determined. It often involves finding excuses that will justify unacceptable
behaviours when self­esteem is threatened. 
Denial is the explicit refusal to acknowledge a threatening reality. It may persist despite
persistent explanation of the facts.
The correct answer is: A boy who is being bullied at school takes on the identity of Superman
– Reaction formation, A woman is not able to recall the abuse she sustained as a child –
Repression, An idle woman who is not performing well at her job is fired. She states this is
because her colleague is bad­mouthing her rather than admitting her poor performance. –
Rationalization, A man who was diagnosed to have cancer says, "This cannot be true... I
follow a healthy lifestyle." – Denial

Question 70 MoK Assessment EMI029
Not answered Diagnosing neurotic disorders
Choose the correct diagnosis for each of the following clinical situations;
Marked out of 3.00

Flag question
Adjustment disorder
The medical teams have reviewed a Acute stress reaction
woman several times over four years Malingering
for symptoms of abdominal pain and
Factitious disorder
shortness of breath. No organic
cause has ever been found
Conversion disorder
Somatisation
Post­traumatic stress disorder
Moderate depressive episode
Malingering
Normal adaptive reaction
Moderate depressive episode
A man presents with low mood and Somatisation
anhedonia. He also has fatigue, poor
Acute stress reaction
sleep and headache
Normal adaptive reaction
Factitious disorder
Conversion disorder
Post­traumatic stress disorder
Factitious disorder
A man presents with an abscess in
Adjustment disorder
Conversion disorder
his left groin. He requires constant
reassurance from the nursing staff. A Adjustment disorder
member of the night staff observes Post­traumatic stress disorder
him injecting from a soiled syringe Somatisation
into the area adjacent to the abscess. Normal adaptive reaction
Acute stress reaction
Malingering
Moderate depressive episode
Your answer is incorrect.
Explanation: 
Case 1: It is a case of somatisation, which is the experience of bodily symptoms with no, or no
sufficient, physical cause for them, with presumed psychological causation. 

1317
Case 2: This man has core features of depression such as low mood, anhedonia and low
energy levels. 
Case 3: It is a case of factitious disorder in which a person deliberately produces, feigns, or
exaggerates symptoms.
The correct answer is: The medical teams have reviewed a woman several times over four
years for symptoms of abdominal pain and shortness of breath. No organic cause has ever
been found – Somatisation, A man presents with low mood and anhedonia. He also has
fatigue, poor sleep and headache – Moderate depressive episode, A man presents with an
abscess in his left groin. He requires constant reassurance from the nursing staff. A member of
the night staff observes him injecting from a soiled syringe into the area adjacent to the
abscess. – Factitious disorder

Question 71 MoK Assessment EMI034
Not answered Diagnosing culture­bound syndromes
For each of the descriptions given below chose one most appropriate culture­bound
Marked out of 3.00
syndrome as a diagnosis from the list above
Flag question

Koro
Dhat syndrome
A 32­year­old Greek woman presents Windigo
with extreme sorrow, nausea,
Brain fag
insomnia and fatigue
Susto
Nerfiza
Pa­leng
Latah
Nerfiza
A 44­year­old Mexican woman Amok
Dhat syndrome
presents feeling despondent with Tajin
Amok
involuntary tics and diarrhoea since
Susto
her husband met with a sudden
death in her presence
Koro
Brain fag
Pa­leng
A 22­year­old Nepalese man Latah
Windigo
presents with the fear that he is losing Windigo
Brain fag
semen in the urine. He describes Tajin
symptoms suggestive of Urinary Tract
Nerfiza
infection along with tiredness. Pa­leng
Tajin
Koro
Dhat syndrome
Latah
Amok
Your answer is incorrect. Susto
Explanation:
Nerfiza (also called as nerves, nevra, and nervios) is seen in Egypt, northern Europe, Greece,
Mexico, Central and South America. It involves recurrent or unremitting episodes of despair
often with anxiety, and a plethora of somatic complaints including headaches, sluggishness,
gastrointestinal intolerance, sleeplessness, fatigue and in some cases, psychomotor agitation.
The syndrome is more common in women than in men. 

1318
Susto (also called as espanto) is seen in Mexico, Central and South America. It has a highly
diverse presentation all of which are considered to be related to loss of one's soul that follows
a severe fright reaction involving supernatural beliefs. In some cases, individuals may be
stricken when others suffer a fright/sudden death. Symptoms include agitation, anorexia,
insomnia, fever, diarrhoea, mental confusion and apathy, depression and introversion. 
Ref: Christmas, D. Culture Specific Disorders,
http://old.trickcyclists.co.uk/pdf/Culture_Specific_Disorders.PDF (accessed April 21, 2015).
The correct answer is: A 32­year­old Greek woman presents with extreme sorrow, nausea,
insomnia and fatigue – Nerfiza, A 44­year­old Mexican woman presents feeling despondent
with involuntary tics and diarrhoea since her husband met with a sudden death in her
presence – Susto, A 22­year­old Nepalese man presents with the fear that he is losing semen
in the urine. He describes symptoms suggestive of Urinary Tract infection along with
tiredness. – Dhat syndrome

Question 72 Mok Assessment EMI038
Not answered Subtypes of OCD 
A 35­year­old gentleman has been diagnosed with Obsessive Compulsive Disorder (OCD).
Marked out of 5.00
He enquires about the various forms of this illness. Choose the most appropriate descriptor for
Flag question the questions below using the given list:

Sexual/religious obsessional thoughts
Trichotillomania
Which one symptom­based subtype Counting rituals
confers a higher genetic risk to first­
Aggressive/checking rituals
degree relatives?
Contamination/washing rituals
Excessive checking of facial defects
Hoarding rituals
Symmetry/ordering rituals
Aggressive/checking rituals
Counting rituals
Which TWO types predict a poor Hoarding rituals
response to treatment with SSRI
Excessive checking of facial defects
antidepressants?
Symmetry/ordering rituals
Contamination/washing rituals
Trichotillomania
Sexual/religious obsessional thoughts
Aggressive/checking rituals
Sexual/religious obsessional thoughts
Which TWO types predict a poor Contamination/washing rituals
response to CBT? Counting rituals
Excessive checking of facial defects
Trichotillomania
Symmetry/ordering rituals
Hoarding rituals

Your answer is incorrect.
Explanation: Symmetry/ordering rituals tend to be chronic and confers a higher genetic risk on
first­degree relatives.Sexual/religious obsessional thoughts and hoarding rituals predict a
poor response to SSRIs and CBT. Compulsive hoarding may be a neurobiologically distinct
form of obsessive­compulsive disorder. Hoarding is notoriously difficult to treat by either
psychological or pharmacological means.
The correct answer is: Which one symptom­based subtype confers a higher genetic risk to

1319
first­degree relatives? – Symmetry/ordering rituals, Which TWO types predict a poor response
to treatment with SSRI antidepressants? – Sexual/religious obsessional thoughts, Hoarding
rituals, Which TWO types predict a poor response to CBT? – Sexual/religious obsessional
thoughts, Hoarding rituals

Question 73 Mok Assessment EMI039
Not answered Catatonic signs
From the given list chose the most appropriate terms describing each of the following clinical
Marked out of 6.00
observations;
Flag question

Mannerisms
Mitgehen
A patient moves his arm on slightest Ambitendency
pressure regardless of any instruction
Catalepsy
given
Automatic obedience
Stereotypy
Waxy flexibility
Gegenhalten
Gegenhalten
Cataplexy
Automatic obedience
Despite being asked to comply with, Stereotypy
a patient resists with same pressure
Cataplexy
an attempt to move his arm.
Mannerisms
Catalepsy
Waxy flexibility
Mitgehen
Catalepsy
When asked to put out his hand, a Ambitendency
Cataplexy
patient extends, withdraws and then Mannerisms
extends his hand repeatedly for 23
Gegenhalten
times before allowing it to rest on the
examiner's hand.
Stereotypy
Mitgehen
Ambitendency
Automatic obedience
Cataplexy
Waxy flexibility
Ambitendency
On hearing a sudden noise, a patient Gegenhalten
feels that his knees are buckling. He
Automatic obedience
falls and hurts himself.
Waxy flexibility
Stereotypy
Mannerisms
Mitgehen
Waxy flexibility
A patient allows his arms to be Catalepsy
Automatic obedience
manipulated freely without much Cataplexy
resistance and maintains the posture
Catalepsy
in which the examiner leaves him for
few minutes. (TWO)
Stereotypy
Ambitendency
Mannerisms
Mitgehen
Gegenhalten
Your answer is incorrect.
Explanation: 

1320
Mitgehen or "angle­poise lamp" sign: The patient yields to slightest of pressures, without much
resistance, similar to an angle­poise lamp that bends easily. This happens even if the patient
is instructed to resist any manipulation. 
Gegenhalten is a milder form of resistance/negativism. Patients with negativism resist or
oppose all passive movements attempted by the examiner. A mild form of this resistance is
Gegenhalten or opposition. 
In ambitendency, the patient makes a series of tentative, opposing alternate movements that
do not reach the intended goal. This becomes evident when the patient is asked to carry out a
motor act. Ambivalence is the inability to make a decision ­ a dilemma of volition. 
Cataplexy is the abrupt loss of muscle tone resulting in a collapse. It occurs following
emotional stress in patients with narcolepsy. 
In catalepsy the patient's limbs can be passively moved to any posture which will then be held
for prolonged periods of time. It is also called as waxy flexibility (flexibilitas cerea) here the
patient shows wax­like plastic 'mouldable' quality. His limbs can be moved by the examiner to
occupy certain postures, which are then maintained, even if these are uncomfortable and
bizarre.
The correct answer is: A patient moves his arm on slightest pressure regardless of any
instruction given – Mitgehen, Despite being asked to comply with, a patient resists with same
pressure an attempt to move his arm. – Gegenhalten, When asked to put out his hand, a
patient extends, withdraws and then extends his hand repeatedly for 23 times before allowing
it to rest on the examiner's hand. – Ambitendency, On hearing a sudden noise, a patient feels
that his knees are buckling. He falls and hurts himself. – Cataplexy, A patient allows his arms
to be manipulated freely without much resistance and maintains the posture in which the
examiner leaves him for few minutes. (TWO) – Catalepsy, Waxy flexibility

Question 74 Mok Assessment EMI040
Not answered Drug Detection in Urine
How long could each of the following drugs be detected in urine?
Marked out of 5.00

Flag question
28 days
48 days
1­3 days
Cannabis (chronic user)
3 days or more
One week
6­8 hours
60 days
Up to 48 hours
48 days
Up to 24 hrs
28 days
6­8 hours
Cocaine
60 days
One week
Up to 48 hours
Up to 24 hrs
3 days or more
60 days
1­3 days
One week
1­3 days
Up to 48 hours
Amphetamine
Up to 24 hrs
48 days
6­8 hours
28 days

1321
48 days
One week
60 days
Heroin
Up to 24 hrs
6­8 hours
3 days or more
1­3 days
28 days
Up to 48 hours
Up to 48 hours
One week
1­3 days
Methadone
6­8 hours
48 days
60 days
28 days
3 days or more
Up to 24 hrs
Your answer is incorrect.
Explanation: Occasional use of cannabis could be detected in urine for up to 3 days. High
daily use for long time would last up to 4 weeks. The cocaine could be found in urine for 6­8
hours; metabolites found up to 2­4 days. The half­life of methadone is longer than heroin and
could be detected for three days or more in the urine.
The correct answer is: Cannabis (chronic user) – 28 days, Cocaine – 6­8 hours, Amphetamine
– Up to 48 hours, Heroin – 1­3 days, Methadone – 3 days or more

Question 75 MoK Assessment EMI037
Not answered Supportive techniques
For each of the following examples, identify the type of supportive techniques aimed at
Marked out of 3.00
eliciting information
Flag question

I note that these side effects are
Reassurance
rather inconvenient for you. But you Acknowledgment of affect
have shown some notable Advice
improvement, and if you continue Postponement
with the treatment it is likely that you Positive reinforcement
will achieve sustained response. Partnering
Disapproval
Statement of respect
Acknowledgment of affect
Validation
Reassurance
I can see that you look anxious and Postponement
upset when talking about those
Positive reinforcement
voices
Advice
Disapproval
Validation
Partnering
Statement of respect

1322
You have been through a lot. I am
Statement of respect
impressed at how you have hung in
Positive reinforcement
there and you must be a strong
person
Postponement
Advice
Disapproval
Acknowledgment of affect
Reassurance
Validation
Partnering
Your answer is incorrect.
Explanation: Reassurance: Used to instil positive hope and avoid or reduce despair. Must not
be falsely reassuring. Acknowledgement of affect: Making a remark about patient's affect can
facilitate disclosure.Statement of respect: Affirmative statements (must be genuine and
appropriate) indicating respect and dignity along with positive reinforcement. Ref: Adapted
from Kay J & Tasman A. Essentials of Psychiatry, 2nd edition, 2006. John Wiley & Sons, Ltd.
The correct answer is: I note that these side effects are rather inconvenient for you. But you
have shown some notable improvement, and if you continue with the treatment it is likely that
you will achieve sustained response. – Reassurance, I can see that you look anxious and
upset when talking about those voices – Acknowledgment of affect, You have been through a
lot. I am impressed at how you have hung in there and you must be a strong person –
Statement of respect

Finish review

1323
 Home Mock Paper Practice Tests

Mock Paper A(1)

Started on Tuesday, 30 June 2015, 12:20 PM
State Finished
Completed on Tuesday, 30 June 2015, 12:20 PM
Time taken 12 secs
Grade 0.00 out of 100.00
Feedback You are on the right track. Could you spend some more time on your revision?

Question 1 MoK Classification 056
Not answered Which of the following traits is characteristic of anxious (avoidant) personality disorder,
according to ICD­10 criteria?
Marked out of 1.00

Flag question Select one:
Chronic feelings of emptiness
Fears of abandonment
Pervasive feelings of tension
Excessive feelings of doubt and caution
Inability to make everyday decisions

Your answer is incorrect.
Anxious (Avoidant) Personality Disorder is characterized by (a) persistent and pervasive
feelings of tension and apprehension; (b) belief that one is socially inept, personally
unappealing, or inferior to others; (c) excessive preoccupation with being criticized or rejected
in social situations and other similar features.
The correct answer is: Pervasive feelings of tension

Question 2 MoK Classification 063
Not answered Which of the following is consistent with bipolar affective disorder, current episode mania with
psychotic symptoms:
Marked out of 1.00

Flag question Select one:
Bizarre delusional content
Persecutory delusions with resultant affective change

Non prominent affective symptoms but mood congruent delusions
Prominent affective symptoms and mood congruent delusions

1324
Systematised delusions with no direct functional impairment

Your answer is incorrect.
Prominent affective symptoms and mood­congruent delusions can be seen in mania. Mood
incongruent delusions do not negate diagnosis of mania, but they are not characteristic either.
Bizarre delusions point to schizophrenia.
The correct answer is: Prominent affective symptoms and mood congruent delusions

Question 3 MoK Classification 072
Not answered Which of the following is not an ICD10 personality disorder?

Marked out of 1.00
Select one:
Flag question
Anankastic personality
Paranoid personality

Schizoid personality
Schizotypal personality
Emotionally unstable borderline personality

Your answer is incorrect.
It is classified with F20 ­ schizophrenia.
The correct answer is: Schizotypal personality

Question 4 MoK Classification 074
Not answered Which of the following disorders increases the risk of panic disorder and agoraphobia in later
life?
Marked out of 1.00

Flag question Select one:
conduct disorder

ADHD
Separation anxiety disorder
Post­traumatic stress disorder
smoking as an adolescent

Your answer is incorrect.
Female (less so in male) agoraphobics report significantly more childhood separation anxiety
than those with other anxiety disorder
The correct answer is: Separation anxiety disorder

1325
Question 5 MoK Classification 075
Not answered A lady believes that she is dead and her body needs to be cremated. Her husband reports
that she has been feeling very low over the past few months. Most appropriate diagnosis
Marked out of 1.00
would be
Flag question
Select one:
Somatoform disorder
Schizophrenia
Mild depressive episode

Hypochondriasis
Severe depression with psychosis

Your answer is incorrect.
This is a nihilistic delusion in the presence of depressive psychosis
The correct answer is: Severe depression with psychosis

Question 6 MoK Classification 076
Not answered A woman with nausea, abdominal pain, dysmenorrhoea, had several investigations. No
specific cause is identified. The diagnosis is;
Marked out of 1.00

Flag question Select one:
Munchausen syndrome
Factitious disorder
Hypochondriasis
Somatisation disorder
Conversion disorder

Your answer is incorrect.
Somatization disorder is a long­term (chronic) condition in which a person has physical
symptoms that involve more than one part of the body, but no physical cause can be found.
The pain and other symptoms people with this disorder feel are real and are not created or
faked on purpose (malingering) [Somatization disorder,
http://www.nlm.nih.gov/medlineplus/ency/article/000955.htm].
The correct answer is: Somatisation disorder

Question 7 MoK Classification 077
Not answered A middle aged woman has been presenting recurrently to her GP for the last 3 years for
various symptoms such as chest pain, muscular pain, and digestive problems. Despite
Marked out of 1.00
various investigations, no physical causes were found for any of her complaints. She is most
Flag question likely to have a diagnosis of

1326
Select one:
Somatization disorder
Hypochondriasis
Somatoform disorder

Malingering
Factitious disorder

Your answer is incorrect.
Somatization occurs where a psychological problem turns into physical and subconscious
symptoms. Somatisation disorder is characterised by persistent somatic complaints spanning
more than one organ system.
The correct answer is: Somatization disorder

Question 8 MoK Classification 078
Not answered Mr Hanks has been sent to you by his GP with a 10­month history of sweating, tremor,
palpitation and a persistent precordial  pain. He has been excessively investigated and seen
Marked out of 1.00
by a cardiologist, and there is no possibility of a heart disease. He continues to believe that he
Flag question has a serious heart condition. What is his diagnosis?

Select one:
Generalised anxiety disorder
Somatisation disorder

Somatoform autonomic dysfunction
Binswanger's disease
Conversion disorder

Your answer is incorrect.
Somatoform autonomic dysfunction refers to symptoms of autonomic arousal that are
attributed by the patient to a physical disorder of cardiovascular / gastrointestinal / respiratory
or genitourinary system. There must be no underlying physical disorder explaining the
symptoms, and these symptoms are not due to other mental illnesses such as phobia or panic
disorder. It is different from Hypochondriasis as there are true autonomic symptoms present
though in both disorders the patient may be preoccupied with an underlying physical disorder.
The correct answer is: Somatoform autonomic dysfunction

Question 9 MoK Classification 079
Not answered Which of the following diagnostic criteria for schizophrenia considers age of onset and family
history but can diagnose schizophrenia even in the absence of delusions and hallucinations?
Marked out of 1.00

Flag question Select one:
CATEGO

1327
Scheiderian criteria
ICD­10
Research Diagnostic Criteria
St. Louis criteria

Your answer is incorrect.
This is also called Feighner's criteria.
The correct answer is: St. Louis criteria

Question 10 MoK Classification 080
Not answered The following features are more often found in non­epileptic than true epileptic seizures,
except:
Marked out of 1.00

Flag question Select one:
Deliberate self­harm
Stereotyped pattern of motor symptoms
Left­sided somato­sensory symptoms
Presence of affective disorder
Family history of psychiatric disorder

Your answer is incorrect.
There are many differences between non­epileptic attacks (aka Pseudoseizures) and true
epileptic seizures. But in practice it is often difficult to separate them.
The correct answer is: Stereotyped pattern of motor symptoms

Question 11 MoK Classification 081
Not answered Atypical anorexia nervosa is characterised by
Marked out of 1.00
Select one:
Flag question
Significant weight gain with anorexia
Hyponatremia
Menarche is never achieved
Induced vomiting is a feature
Symptoms suggestive of anorexia nervosa but in milder form

Your answer is incorrect.

1328
Atypical anorexia is, in fact, a partial syndrome of the full­blown eating disorder.
The correct answer is: Symptoms suggestive of anorexia nervosa but in milder form

Question 12 MoK Classification 082
Not answered Which pair of features is most likely to be present in delusional disorders?
Marked out of 1.00
Select one:
Flag question
Prominent affective symptoms; bizarre delusions
Mood congruent delusions; prominent affective symptoms
Systematised delusions; poor psychosocial functioning
Systematised delusions; well preserved functioning
Systematised delusions; prominent affective symptoms

Your answer is incorrect.
Unfortunately, patients with the delusional disorder do not have good insight into their
pathological experiences. Interestingly, despite significant delusions, many other
psychosocial abilities remain intact as if the delusions are circumscribed. Indeed, this is one of
the key differences between delusional disorder and other primary psychotic disorders (Ref:
Delusional Disorder ­ Medscape Reference. Retrieved from
http://emedicine.medscape.com/article/292991­overview).
The correct answer is: Systematised delusions; well preserved functioning

Question 13 MoK Clinical Examination 044
Not answered Which of the following can be used as an objective measure of compliance with prescribed
medications?
Marked out of 1.00

Flag question Select one:
Pill counting
Change in symptom scores
Satisfaction questionnaire
Report of side effects
Clinic attendance rates

Your answer is incorrect.
Adherence can be evaluated using sensitive questions that ask about problems with
medication, and focus specifically on missed doses and plans for continuation. More
objective, direct methods include the use of pill counting or measuring serum or urine drug
levels. (Ref: http://apt.rcpsych.org/content/13/5/336#sec­21)
The correct answer is: Pill counting

1329
Question 14 MoK Clinical Examination 045
Not answered Which of the following techniques are useful during a clinical interview?
Marked out of 1.00
Select one:
Flag question
Restricted affect
Linking statements
Frequent interpretations
Long pauses
Polythematic questions

Your answer is incorrect.
Reflective statement during an interview is a helpful non­directive approach. One type of
refection is a linking statement. Here the interviewer tries to link feelings to experiences and
behaviours: e.g. "you feel [accurately name the patients feeling] because [accurately name the
experiences and behaviour that gave rise to the feeling]".
The correct answer is: Linking statements

Question 15 MoK Clinical Examination 062
Not answered Mr Kahn presents to GP with a history of depression. Objective evidence of his mood can be
inferred from all of the following except
Marked out of 1.00

Flag question Select one:
Patient's stated feeling
Speech
Motor activity
Facial expressions
Behaviour during the interview

Your answer is incorrect.
While recording mental state of a patient, the mood is described to have an objective and
subjective component by some authors. Objective mood can be discerned from psychomotor
activity, behaviour during the interview, facial reactivity and speech tone, rate and rhythm.
Subjective mood can only be discerned from asking the patient to describe how he/she feels.
The correct answer is: Patient's stated feeling

Question 16 MoK Clinical Examination 063
Not answered Which one among the following statements does not support the neurodevelopmental
hypothesis of schizophrenia?
Marked out of 1.00

Flag question Select one:

1330
Patients with schizophrenia have an increased frequency of cranio­facial
and dermatoglyphic minor physical anomalies
Patients with schizophrenia have a lower than average IQ
Patients with schizophrenia have subtle psychomotor, behavioural,
personality and social abnormalities
Patients with schizophrenia have an excess of progressive structural brain
abnormalities
Patients with schizophrenia do exhibit non specific neurological soft signs

Your answer is incorrect.
Patients with schizophrenia exhibit non­progressive abnormalities and presence of
progressive abnormalities would imply an ongoing disease process rather than
schizophrenia. The cause of neurodevelopmental schizophrenia is unknown. The hypothesis
states that a proportion of schizophrenia commences with impaired foetal or neonatal
development rather than with the onset of psychotic symptoms in early adulthood. This
dominant hypothesis is supported by the findings described by the statements described
above.Core Psychiatry; pg 259­293
The correct answer is: Patients with schizophrenia have an excess of progressive structural
brain abnormalities

Question 17 MoK Clinical Examination 064
Not answered A 42­year­old woman has paroxysmal palpitations and raised blood pressure. She was
referred to psychiatrist for severe anxiety and panic attacks. Which of the following medical
Marked out of 1.00
conditions must be investigated in this case?
Flag question
Select one:
Hypothyroidism
Steven Johnson Syndrome
Pancreatitis
Porphyria cutanea tarda
Phaeochromocytoma

Your answer is incorrect.
Phaeochromocytoma is characterized by increased heart rate, increased blood pressure,
palpitations, and anxiety often resembling a panic attack, along with headaches and
diaphoresis.
The correct answer is: Phaeochromocytoma

Question 18 MoK Clinical Examination 065
Not answered Loss of normal asymmetry of which of the following structure is implicated in schizophrenia?

1331
Marked out of 1.00 Select one:
Flag question Planum temporale
Parietal lobule
Corpus callosum
Cingulate cortex
Stria terminalis

Your answer is incorrect.
The planum temporale, the posterior superior surface of the superior temporal gyrus, is a
highly lateralized brain structure involved with language. In schizophrenic patients, there is a
somewhat consistent report of reversal of the normal left­larger­than­ right asymmetry (Ref:
Planum temporale asymmetry reversal in schizophrenia. Retrieved from
http://www.ncbi.nlm.nih.gov/pubmed/9137122.)
The correct answer is: Planum temporale

Question 19 MoK Clinical Examination 066
Not answered Presence of which of the following will prompt investigations for a medical cause for recurrent
anxiety?
Marked out of 1.00

Flag question Select one:
Presence of avoidance behavior
Family history of anxiety disorders
Significant anticipatory anxiety
Onset of symptoms before the age of 25 years
Poor response to anxiolytic agents

Your answer is incorrect.
Factors implying organic etiology of anxiety include ­ onset of symptoms after the age of 35
years, lack of anticipatory anxiety, lack of avoidance behavior, and lack of personal or family
history of anxiety disorders and poor response to short term anxiolytics
The correct answer is: Poor response to anxiolytic agents

Question 20 MoK Clinical Examination 067
Not answered Vocabulary test is helpful to assess which of the following?

Marked out of 1.00
Select one:
Flag question
Language
Visuospatial ability
Premorbid intelligence

1332
Set shifting
Executive functions

Your answer is incorrect.
Vocabulary test is helpful to assess premorbid intelligence. It is a type of reading test. Reading
ability is generally resistant to the effects of brain damage and, therefore, can be relied upon
when assessing premorbid intelligence levels.
The correct answer is: Premorbid intelligence

Question 21 MoK Clinical Examination 068
Not answered Which of the following is a finding seen in upper motor neuron lesion?
Marked out of 1.00
Select one:
Flag question
Marked hypotonia of anti­gravity muscles
Exaggerated superficial reflexes
Absence of ankle jerk
Marked atrophy of extensor muscles
Presence of clonus

Your answer is incorrect.
Exaggerated deep tendon reflexes may present as ankle clonus in UMN lesions.
The correct answer is: Presence of clonus

Question 22 MoK Clinical Examination 069
Not answered A 56 year old patient has been drinking excessive alcohol for last 31 years. Which of the
following signs on neurological examination is NOT due to cerebellar involvement?
Marked out of 1.00

Flag question Select one:
Hypotonia
Pupillary mydriasis
Dysarthria
Positional nystagmus
Impaired arm coordination

Your answer is incorrect.
In severe form of alcohol withdrawal (DTs) autonomic disturbances such as diaphoresis,
hypertension and mydriasis can be seen.

1333
The correct answer is: Pupillary mydriasis

Question 23 MoK Clinical Examination 070
Not answered Which of the following clinical findings is most consistent with a diagnosis of Parkinson's
disease?
Marked out of 1.00

Flag question Select one:
Action tremor, usually marked on one side
Increased frequency of spontaneous blinking
Wide based gait
Vertical gaze palsy
Raised limb rigidity, usually more marked on one side

Your answer is incorrect.
In the early stages of Parkinson's, rigidity may be present in one arm and later progress to the
leg on the same side, and then move to the trunk and the other side of the body.
The correct answer is: Raised limb rigidity, usually more marked on one side

Question 24 MoK Clinical Examination 071
Not answered While interviewing a patient, your consultant says, "I now have enough information about your
family. Can we now move on to talk about your job." She is employing which of the following
Marked out of 1.00
techniques?
Flag question
Select one:
Redirection
Interpretation
Summation
Validation
Positive facilitation

Your answer is incorrect.
If a patient focused excessively on one area or drifts into providing some non essential details,
the interviewing clinician can redirect him or her to aid clinical interviewing.
The correct answer is: Redirection

Question 25 MoK Clinical Examination 073
Not answered A 45­year­old woman has a long­standing history of bipolar disorder and has been on
maintenance treatment with lithium carbonate for 8 years. On testing her thyroid functions, her
Marked out of 1.00
TSH is 6.4 and T4 is normal. What is the next step of management?

1334
Flag question Select one:
Add levothyroxine
Repeat TSH in a months time
Reduce lithium carbonate

Stop lithium immediately
Add carbimazole

Your answer is incorrect.
This lady has a subclinical thyroid disease. This is estimated to be present in 4 to 8.5% of the
general population, and up to 20% of women older than 60 years, and about 2% general
population are thought to be subclinically hyperthyroid. As she is on lithium, the TSH levels
must be monitored in addition to symptoms.
The correct answer is: Repeat TSH in a months time

Question 26 MoK Clinical Examination 074
Not answered An interviewer says, 'I am impressed by what you have achieved despite all these difficulties'.
This is best described as
Marked out of 1.00

Flag question Select one:
Summation
Respectful statement
Inquiry
Facilitation
Reflection

Your answer is incorrect.
Such statements convey the mutual respect maintained by the therapist in the wake of an
emotionally important disclosure. This can facilitate further conversations.
The correct answer is: Respectful statement

Question 27 MoK Clinical Examination 075
Not answered Visual memory and constructional ability can be tested using

Marked out of 1.00
Select one:
Flag question
Sally and Ann test
Raven's Progressive Matrices
Rey Auditory Verbal Learning Test
Rey Osterrieth Complex Figure Test

1335
Clock drawing test

Your answer is incorrect.
Rey­Osterrieth Complex Figure Test (CFT) is a standard measure of perceptual organization
and visual memory in brain­injured individuals.
The correct answer is: Rey Osterrieth Complex Figure Test

Question 28 MoK Clinical Examination 076
Not answered Which of the following is true with regard to Addenbrooke's Cognitive Examination?

Marked out of 1.00
Select one:
Flag question
Visuospatial domain receives disproportionately higher distribution of scores

It includes activities of daily living
Frontal tests are not included
Score less than 82 has high sensitivity for dementia
A score less than 82 has a high specificity for dementia

Your answer is incorrect.
The normative data provided with ACE­R (revised version) states that there are two defined
cut­offs (less than 88: sensitivity=0.94, specificity=0.89; less than 82: sensitivity=0.84,
specificity=1.0). The likelihood ratio for a positive test of dementia at a cut­off of 82 is 100:1.
Language domain receives the major share of the scoring. Frontal tests such as verbal
fluency are included in the ACE­R. (Hodges R et al., 2000)
The correct answer is: A score less than 82 has a high specificity for dementia

Question 29 MoK Descriptive Psychopathology 029
Not answered Which of the following terminology is used to refer to a psychotic patient feeling uncomfortable
and perplexed?
Marked out of 1.00

Flag question Select one:
Delusional mood
Primary delusions
Delusional perception
Secondary delusions
Delusional memory

Your answer is incorrect.

1336
Psychotic prodrome may initially manifest as affective symptoms­ anxiety, depression and
then develop into a state of delusional mood ­ a perplexed individual who presents worried,
unsure and disturbed about something non­specific in the external world ( Retrieved from
http://www.bmj.com/rapid­response/2011/10/31/schizophrenia­prodromal­symptoms­and­
clinical­dilemma).
The correct answer is: Delusional mood

Question 30 MoK Descriptive Psychopathology 032
Not answered "When I saw the man holding a glass of wine, I knew my father was dead". This is an example
of
Marked out of 1.00

Flag question Select one:
Reflex hallucination
Functional hallucination
Delusional perception
Delusional mood
Delusional memory

Your answer is incorrect.
Delusional perception is a Schneiderian first­rank symptom in which a person believes that a
normal percept (product of perception) has a special meaning for him or her. For example, a
cloud in the sky may be misinterpreted as meaning that someone has sent that person a
message to save the world (Delusional perception ­ Oxford Reference,
http://www.oxfordreference.com/view/10.1093).
The correct answer is: Delusional perception

Question 31 MoK Descriptive Psychopathology 043
Not answered A patient says 'I went to the mental health centre. Centre is the gravity. Newton discovered
gravity. Gravity stays with the apple. My psychiatrist took the apple away. Psychiatrists and
Marked out of 1.00
Pediatricians are doctors'. The speech disturbance noted here is best described as
Flag question
Select one:
Loosening of association
Flight of ideas
Derailment
Verbigeration
Vorbireiden

Your answer is incorrect.

1337
Flight of ideas is characteristic of mania. The speech often (but not always) occurs at a rapid
rate and involves fragmented or unrelated ideas, connected by distracting cues rather than
the goal of the discourse. Chance associations that arise from distractions in the environment
or from one's own speech connect successive ideas.
The correct answer is: Flight of ideas

Question 32 MoK Descriptive Psychopathology 045
Not answered Which of the following statements describes a characteristic feature of primary delusions?

Marked out of 1.00
Select one:
Flag question
Occurs secondary to auditory hallucinations
Not preceded by delusional mood
Primary delusions can be understood as having arisen in response to other
psychiatric conditions
Characteristically persecutory
Frequently systematised to secondary delusions

Your answer is incorrect.
Primay delusional experiences are often regarded as the starting point of the development of
unclear but fully formed and affect­laden beliefs to a scheme of systematized delusions.
The correct answer is: Frequently systematised to secondary delusions

Question 33 MoK Descriptive Psychopathology 046
Not answered A psychotic patient heard hallucinatory voices only when water flowed from a tap. This
phenomenon is known as a
Marked out of 1.00

Flag question Select one:
Hygric hallucination
Reflex hallucination
Delusional misinterpretation
Delusional perception
Functional hallucination

Your answer is incorrect.
In a functional hallucination, stimulus in one modality elicits a hallucination in the same
modality (the percept will NOT correspond to the stimulus ­ hence it is a hallucination).
The correct answer is: Functional hallucination

Question 34 MoK Descriptive Psychopathology 047

1338
Not answered Which of the following is more common in mania than schizophrenia?
Marked out of 1.00
Select one:
Flag question
Passivity symptoms
Persecutory delusions
Tangentiality
Grandiose delusions
Circumstantiality

Your answer is incorrect.
Though seen in both disorders, grandiose delusions are more common in mania with
psychosis.
The correct answer is: Grandiose delusions

Question 35 MoK Descriptive Psychopathology 048
Not answered Formication is a type of

Marked out of 1.00
Select one:
Flag question
Deep hallucination
Visceral hallucination

Delusional infestation
Haptic hallucination
Kinaesthetic hallucination

Your answer is incorrect.
Formication is a type of somatic hallucination of haptic (superficial touch sensation) variety. It
is seen in cocaine intoxication, alcohol withdrawal and in some cases of delusional
parasitosis.
The correct answer is: Haptic hallucination

Question 36 MoK Descriptive Psychopathology 050
Not answered Prof. Smith was honoured with Knighthood. On the day of the ceremony he said that the red
carpet appeared to be more vivid and bright, and that the cheers from the crowd were 'louder
Marked out of 1.00
than thousands of pneumatic drills'. This is known as:
Flag question
Select one:
Dysmegalopsia
Oneiroid state
Hyperaesthesia

1339
Macropsia
Illusion

Your answer is incorrect.
In hyperaesthesia, intense perceptions could occur without any sensory distortions. Note that
the intensity, not the quality of the percept, changes in hyperaesthesia (unlike macropsia,
dysmegalopsia or illusions where the reverse is true).
The correct answer is: Hyperaesthesia

Question 37 MoK Descriptive Psychopathology 051
Not answered An elderly patient in a medical ward has reduced orientation to time and appears confused.
When talking to him, a nurse notices that she has to raise her voice more than usual to get him
Marked out of 1.00
to understand what she is saying. Which of the following is the best explanation?
Flag question
Select one:
Hypoaesthesia
Hypoalgesia
Hyperesthesia
Paresthesia
Hyperalgesia

Your answer is incorrect.
Reduced sensory perception (hypoaesthesia) may be seen especially in the hypoactive type
of delirium. This may be experienced by patients as decreased volume of spoken words
(hypoacusis) and reduced tactile sensations. In some, hyperaesthesia can also be seen.
The correct answer is: Hypoaesthesia

Question 38 MoK Descriptive Psychopathology 052
Not answered Obsessions can occur in the form of

Marked out of 1.00
Select one:
Flag question
Hallucinations
Delusions
Panic
Craving
Images

Your answer is incorrect.

1340
Obsessions can be distressing thoughts, images or impulses that occur on a stereotyped
basis
The correct answer is: Images

Question 39 MoK Descriptive Psychopathology 053
Not answered All are true about pseudohallucinations except

Marked out of 1.00
Select one:
Flag question
Perception is retained for a reasonable time
Images are incomplete with ill defined outlines
Images are located in inner subjective space
The perception itself can cause relevant emotional changes
Images cannot be dismissed at will

Your answer is incorrect.
Pseudohallucinations resemble true hallucinations in most respects except for (i) retained
insight in pseudohallucinations and (ii) perceptions occurring in inner subjective space rather
than outer objective space in pseudohallucinations. So pseudohallucinations are usually
complete with well­defined borders similar to true hallucinations. They can produce congruent
emotional changes (e.g. a derogatory pseudohallucinatory voice can make a patient
depressed).
The correct answer is: Images are incomplete with ill defined outlines

Question 40 MoK Descriptive Psychopathology 054
Not answered Which of the following can differentiate depersonalisation seen in normal persons and
depersonalisation experienced by patients with a psychiatric disorder?
Marked out of 1.00

Flag question Select one:
Sense of time is not altered in the former
Intense affective change is seen in the latter
No difference is notable between the two groups
Feelings of detachment is not seen in the former
Shorter duration in the latter

Your answer is incorrect.
According to Sims, pathological depersonalisation is associated with intense affective
change.
The correct answer is: Intense affective change is seen in the latter

1341
Question 41 MoK Descriptive Psychopathology 055
Not answered A patient believes that she can see others change into someone else temporarily. Identify the
type of delusional misidentification syndrome.
Marked out of 1.00

Flag question Select one:
Subjective doubles delusion
Capgras delusion
Fregoli delusion

Autoscopic syndrome
Intermetamorphosis delusion

Your answer is incorrect.
In Intermetamorphosis delusion, the patient believes that they can see others change into
someone else both in their external appearance and internal personality, usually on a
temporary basis. In subjective doubles delusion, the patient believes that there is a double
who exists and functions independently. In Autoscopic syndrome, the patient sees a double of
themselves projected onto other people or objects nearby.
The correct answer is: Intermetamorphosis delusion

Question 42 MoK Descriptive Psychopathology 057
Not answered Which of the following is true with regard to loss of insight seen in psychosis?

Marked out of 1.00
Select one:
Flag question
It is a trait marker in schizophrenia
It has no direct relevance to treatment acceptance

It is strongly associated with other psychopathology in schizophrenia
It is similar to anosognosia in neurological states
It is seen constantly irrespective of treatment response

Your answer is incorrect.
Anosognosia closely resembles the loss of insight in phenomenological terms. As insight
fluctuates significantly, it cannot be termed as a trait factor; no consistent correlations are
noted between various domains of psychosis and insight.
The correct answer is: It is similar to anosognosia in neurological states

Question 43 MoK Descriptive Psychopathology 058
Not answered A 33 year old man says 'I can feel, taste and hear the bright red flowers whenever I walk past
my garden'. Which one of the following statements is true about his experience?
Marked out of 1.00

Flag question Select one:

1342
It is more common in males
It is treated using mescaline
It is often a pathological phenomenon
It is the perception of a stimulus in one modality as a sensation in another
modality.
It does not occur in children

Your answer is incorrect.
Synaesthesia is the experience of a stimulus in one sense modality producing a sensory
experience in another. Synaesthesia can occur under the influence of hallucinogenic drugs
such as LSD or mescaline
The correct answer is: It is the perception of a stimulus in one modality as a sensation in
another modality.

Question 44 MoK Descriptive Psychopathology 059
Not answered 'I thought that my life was outside my feet and made my feet vibrate.' This is known as:

Marked out of 1.00
Select one:
Flag question
Somatic passivity
Made feeling
Haptic hallucination
Delusion of control
Kinaesthetic hallucination

Your answer is incorrect.
In kinaesthetic hallucinations, perceptual alterations occur in the joint position, muscular
movement and vibration senses (proprioception). The above statement is a classical example
quoted from Sims ­ Symptoms in the Mind. The first statement 'I thought that my life was
outside my feet' should not be confused with passivity delusion as it is expressed as a thought
than a fully formed belief.
The correct answer is: Kinaesthetic hallucination

Question 45 MoK Descriptive Psychopathology 060
Not answered The term blunted affect refers to

Marked out of 1.00
Select one:
Flag question
Lack of emotional reactivity when responding to others
Inability to enjoy pleasurable things in life
Sudden rapid shift in affect

1343
Complete loss of control over emotions
Incongruity of affect

Your answer is incorrect.
Blunted affect refers to dulling of normal emotional responses marked by a severe reduction
in the intensity of affective expression.
The correct answer is: Lack of emotional reactivity when responding to others

Question 46 MoK Descriptive Psychopathology 061
Not answered When asked about his dinner, a patient with a prominent movement disorder answers, "I had
some.. eh..chicken today...ay... ay... ay". What is this phenomenon called?
Marked out of 1.00

Flag question Select one:
Echolalia
Catalepsy
Perseveration
Palilalia
Logoclonia

Your answer is incorrect.
The term logoclonia refers to the repetition of the last syllable of every word.
The correct answer is: Logoclonia

Question 47 MoK Descriptive Psychopathology 063
Not answered Which of the following is most commonly seen in schizophrenia?

Marked out of 1.00
Select one:
Flag question
Ambidexterity
Mitgehen
Simultanagnosia
Cataplexy
Apraxia

Your answer is incorrect.
Catatonic features are more common than focal neurological deficits. Some studies suggest
that left laterality and ambidexterity are common in schizophrenia, though this is disputed.
The correct answer is: Mitgehen

1344
Question 48 MoK Descriptive Psychopathology 064
Not answered A patient propels fully on to the front on being merely touched on his back, despite being
asked to resist a push. He is exhibiting
Marked out of 1.00

Flag question Select one:
Verstimmung
Gegenhalten
Witzelschut
Mitgehen
Vorbeigehen

Your answer is incorrect.
This is called as Mitgehen / Anglepoise lamp sign of catatonia. Here, a gentle push will be
enough to make a patient tilt fully in the direction of push. This will happen despite instructions
to resist the push. It may be considered as a milder form of automatic obedience.
The correct answer is: Mitgehen

Question 49 MoK Descriptive Psychopathology 065
Not answered A psychiatrist attempts to move a patient's arm. Before doing so he instructs the patient to
resist moving and not to let him manipulate his arm. But the patient continues to move his arm
Marked out of 1.00
in the direction of force. Once the psychiatrist removes application of force, the patient's arm
Flag question comes back to original position. This phenomenon is called

Select one:
Echophenomenon
Mitgehen
Catalepsy
Posturing
Waxy flexibility

Your answer is incorrect.
Mitgehen is the 'Anglepoise lamp' arm raising in response to light finger pressure, despite
instructions to the contrary.
The correct answer is: Mitgehen

Question 50 MoK Descriptive Psychopathology 066
Not answered A young African­Caribbean man is being interviewed at the emergency unit. He sat facing
away from clinical team throughout the interview. He turned away from all attempts to speak to
Marked out of 1.00
him, without any accompanying emotional expression. This is known as:

1345
Flag question Select one:
Resistance
Somatic passivity
Obstruction
Stereotypy
Negativism

Your answer is incorrect.
Negativism is an apparently motiveless resistance to all interference and may or may not be
associated with an outspoken defensive attitude. It is found in catatonia, severe learning
disability and dementia. Negativism may be passive when all interference is resisted and
orders are not carried out, or it may manifest as active or command negativism when the
patient does the exact opposite of what they are asked to do, in a reflex way. (Fish
Psychopathology)
The correct answer is: Negativism

Question 51 MoK Descriptive Psychopathology 067
Not answered An elderly lady reports 'my intestines have blocked and blood has stopped flowing to my
brain'. Which of the following phenomena is she describing?
Marked out of 1.00

Flag question Select one:
Nihilism
Misidentification
Grandiosity
Delusion of reference
Hypochondriasis

Your answer is incorrect.
Nihilism is an extreme form of delusion of hypochondriasis or catastrophe where the patient
claims that termination of existence is impending e.g. 'world is coming to an end', 'my
intestines have stopped working'; 'my brain is rotting away', etc.
The correct answer is: Nihilism

Question 52 MoK Basic Psychology 063
Not answered Which of the following is least likely to accompany lesions causing predominantly an
impairment of working memory?
Marked out of 1.00

Flag question Select one:
Procedural memory impairment
Inability to learn new information

1346
Impaired performance on mathematical calculations
Episodic memory impairment
Impaired performance on tests of attention and concentration

Your answer is incorrect.
Procedural memories are spontaneously retrieved without a conscious effort whenever the
execution of tasks involving the learnt skills are required; e.g. driving a car. The access and
use of the implicit procedural memories require no conscious control or attention. It is not
usually affected in conditions causing amnesia.
The correct answer is: Procedural memory impairment

Question 53 MoK Basic Psychology 065
Not answered A patient with phobia for spiders is gradually exposed to various grades of anxiety provoking
stimuli with simultaneous relaxation training. Which of the following principles are primarily
Marked out of 1.00
employed in this therapy?
Flag question
Select one:
Discriminant learning
Reciprocal inhibition
Classical conditioning
Covert desensitisation
Stimulus generalisation

Your answer is incorrect.
The acquisition of anxiety disorders (e.g., phobias) was proposed to be mediated by classical
conditioning processes by Wolpe, 1958 ( Wolpe and Rowan, 1989, Psychotherapy by
reciprocal inhibition. Behaviour Research and Therapy, 27, 583­585). The success of
exposure therapy is possibly a consequence of extinction based on the principle of reciprocal
inhibition.
The correct answer is: Reciprocal inhibition

Question 54 MoK Basic Psychology 067
Not answered During psychometric testing by a psychologist, the subject is first asked to copy a complex
geometric figure and then to draw from memory after an interval of 30 minutes? What is this
Marked out of 1.00
test called as?
Flag question
Select one:
Wechsler Adult Intelligence Scale
Rivermead Behavioural Memory Test
Wechsler Memory Scale

1347
Halstead­Reitan scale
Rey­Osterrieth test

Your answer is incorrect.
Rey­Osterrieth complex figure test is one of non­verbal memory test. Here the subject is first
asked to copy a complex geometric figure and then to draw from memory after an interval of
30 minutes. Recall is impaired in patients with dementia and amnesic syndrome.
The correct answer is: Rey­Osterrieth test

Question 55 MoK Basic Psychology 068
Not answered Most patients engage in certain subtle behaviours while in a fearful situation which serve to
maintain the fear. These behaviours are called
Marked out of 1.00

Flag question Select one:
Compulsions
Safety behaviours
Mental imagery
Obsessions
Phobic behaviours

Your answer is incorrect.
In­situation safety behaviours (Salkovskis) refer to a variety of subtle behaviours/internal
mental processes that most patients engage in while in a fearful situation. These are actually
intended to prevent feared outcome e.g., a panic disorder patient bows his head down and
gently leans leftwards when having a panic, with a hope to increase heart's circulation.
The correct answer is: Safety behaviours

Question 56 MoK Basic Psychology 069
Not answered During cognitive examination, a patient correctly states that bicycles have two wheels. What
type of memory has been tested?
Marked out of 1.00

Flag question Select one:
Semantic memory
Episodic memory
Procedural memory
Working memory
Autobiographical memory

1348
Your answer is incorrect.
Episodic or autobiographic memory involves recalling an 'episode' ­ the time associated
memory. Semantic memory is not tagged to the time of learning but involves factual recalls.
The correct answer is: Semantic memory

Question 57 MoK Basic Psychology 070
Not answered When answering the question, "What is the capital of France?" one is using

Marked out of 1.00
Select one:
Flag question
Autobiographical memory
Flashbulb memories
Episodic memory
Procedural memory
Semantic memory

Your answer is incorrect.
The semantic memory includes factual knowledge of the world and is concerned with
remembering facts, ideas and concepts. It is also called factual memory and includes all
general knowledge, including the meaning of words. It refers to what is known rather than
when and how the knowledge was acquired. Semantic memory contains memory about facts
and concepts that are not linked to a specific time or place. For example you do not know how
you know that an elephant is an animal or when you learned it. But episodic memory is
specific to a time or place, as each episode occurred at a specific time and place and is tied to
that time and place.
The correct answer is: Semantic memory

Question 58 MoK Social Psychology 011
Not answered Which theory states that people are basically selfish while deciding on relationships?

Marked out of 1.00
Select one:
Flag question
Social exchange theory
Equity theory
Contingency theory
Group think
Drive reduction theory

Your answer is incorrect.
The social exchange theory states that people are basically selfish while deciding on
relationships. According to exchange theory, preference is for relationships that offer the
greatest gains with least expense. Equity theory introduces the concept of fairness with

1349
approximately equal gains in the relationships for both individuals.
The correct answer is: Social exchange theory

Question 59 MoK Social Psychology 012
Not answered The Sally­Anne task is associated with which of the following concepts

Marked out of 1.00
Select one:
Flag question
Object permanence
Theory of mind
Attachment behaviour
Temperament
Schemas

Your answer is incorrect.
Sally­Ann test is a false belief test, which is useful to demonstrate that children under the age
of 4 and autism have a specific theory of mind deficit
The correct answer is: Theory of mind

Question 60 MoK Sociocultural Psychiatry 036
Not answered The current international agreement on the research of human subjects is

Marked out of 1.00
Select one:
Flag question
Tuskegee syphilis study
Geneva declaration
Hippocratic oath
Nuremburg code
Helsinski declaration

Your answer is incorrect.
The Declaration of Helsinki (DoH) is the World Medical Association's best­known policy
statement applicable for research ethics. The first version was adopted in 1964 and has been
amended six times since, most recently at the General Assembly in October 2008.
The correct answer is: Helsinski declaration

Question 61 MoK Sociocultural Psychiatry 054
Not answered An Inuit woman suffers from sudden attacks of shouting, becoming frightened and impulsive
running away. She appears dissociated during the attacks and has total amnesia for the
Marked out of 1.00
episodes. Probable diagnosis is , June 2009)
Flag question

1350
Select one:
Windig
Dhat
Latah

Fag
Piblokto

Your answer is incorrect.
Piblokto is a culture­specific syndrome occurring among traditional Eskimo women,
characterized by an outburst of cries or screams, the removal of clothing, and seeming
possession by a bird or animal spirit.
The correct answer is: Piblokto

Question 62 MoK Sociocultural Psychiatry 055
Not answered A 17­year­old man is using cannabis for last 6 months. His parents worry about his
dependence but he is not worried. Which stage of Prochaska & DiClemente's transtheoretical
Marked out of 1.00
model of change is he presently in?
Flag question
Select one:
Pre contemplation
Maintenance
Contemplation
Action
Ready for action

Your answer is incorrect.
The most appropriate stage of change in the transtheoretical model is precontemplation
stage, as he is not even ready to think about change seriously.
The correct answer is: Pre contemplation

Question 63 MoK Sociocultural Psychiatry 056
Not answered Which one of the following statements regarding Hippocratic Oath is true?

Marked out of 1.00
Select one:
Flag question
Originated from Helsinki declaration
Allows breach of confidentiality if required
Allows abortion in extreme case scenarios
Prohibits adultery with patients

1351
Allows therapeutic euthanasia

Your answer is incorrect.
Hippocratic Oath prohibits adultery with patients, prohibits abortion and euthanasia in any
form.
The correct answer is: Prohibits adultery with patients

Question 64 MoK Sociocultural Psychiatry 057
Not answered Which of the following statements is true with regard to public and professional perception of
depressive illness?
Marked out of 1.00

Flag question Select one:
Professionals do not perceive antidepressants to be very helpful for
depression

Health professionals and public hold concordant views on management of
depression
Public have greater faith in antidepressants than the professionals
Public does not support special diet or nutrition to treat depression
Public perceive psychotropics to be more harmful than helpful

Your answer is incorrect.
Health professionals give higher ratings than the public to the helpfulness of antidepressants
for depression, and of antipsychotics and admission to a psychiatric ward for schizophrenia.
Conversely, the public tended to give much more favourable ratings to vitamins and minerals
and special diets for both depression and schizophrenia, and to reading self­help books for
schizophrenia. The beliefs that health practitioners hold about mental disorders differ greatly
from those of the general public. (Retrieved from Jorm AF, Korten AE, et al. Helpfulness of
interventions for mental disorders: beliefs of health professionals compared with the general
public. Br J Psychiatry 1997; 171: 233­237)
The correct answer is: Public perceive psychotropics to be more harmful than helpful

Question 65 MoK HumanDevelopment 034
Not answered Which one among the following is the last stage of Erik Erikson's model of psychosocial
development?
Marked out of 1.00

Flag question Select one:
Initiative vs guilt
Identity vs identity confusion
Industry vs inferiority
Integrity vs despair

1352
Intimacy vs isolation

Your answer is incorrect.
Erik Erikson developed an alternative model of psychosocial development based on the crisis
at each developmental stage.The different stages of development start with a Trust vs. Mistrust
phase at infancy and ends with Integrity vs. Despair phase at the old age.
The correct answer is: Integrity vs despair

Question 66 MoK HumanDevelopment 039
Not answered Which of the following factors protects against child abuse in families?

Marked out of 1.00
Select one:
Flag question
Personality disorder in parents
History of childhood maltreatment in parents
Network of relatives
High expectation of parents
Young age of parents

Your answer is incorrect.
Protective factors against child abuse include having emotionally satisfying relationships with
a network of relatives or friends. Parents who were abused as children are less likely to abuse
their own children if they have resolved internal conflicts and pain related to their history of
abuse and if they have an intact, stable, supportive, and nonabusive relationship with their
partner.
The correct answer is: Network of relatives

Question 67 MoK HumanDevelopment 040
Not answered According to Margaret Mahler's original theory of separation, what phase of development
applies to a newborn child?
Marked out of 1.00

Flag question Select one:
Seperation­individuation phase
Rapprochement phase
Object constancy phase
Normal Autism
Symbiosis

Your answer is incorrect.

1353
Normal autism (0 to 2 months)­ child spends most time in sleep as if the intrauterine aloofness
continues beyond birth.
The correct answer is: Normal Autism

Question 68 MoK HumanDevelopment 043
Not answered Which of the following statements is true of adolescence?

Marked out of 1.00
Select one:
Flag question
Fears of rejection for the peer group peaks around the age of 17
Negative identity is more common in societies that mark transition to
adulthood by initiation ceremonies
Risk taking behaviour is less common in societies that mark transition to
adulthood by initiation ceremonies
The cultural ideal hypothesis suggests that boys move further away from
their physical ideal earlier than girls
Delinquency or offending behaviour reaches a peak in early teens and
declines thereafter

Your answer is incorrect.
Initiation is a rite of passage ceremony marking entrance or acceptance into a group or
society. It could also be a formal admission to adulthood in a community or one of its formal
components. Risk­taking behaviour and negative identity are less common in societies that
mark the transition to adulthood by initiation ceremonies. Delinquency or offending behaviour
reaches a peak in late teens (17 years) and declines after that.
The correct answer is: Risk taking behaviour is less common in societies that mark transition
to adulthood by initiation ceremonies

Question 69 MoK HumanDevelopment 044
Not answered The most common type of attachment behaviour in the Western world is

Marked out of 1.00
Select one:
Flag question
Anxious resistant
Secure
Anxious avoidant
Absence of attachment
Disorganised

Your answer is incorrect.
Most children are securely attached.
The correct answer is: Secure

1354
Question 70 MoK HumanDevelopment 045
Not answered The game of peek­a­boo becomes characteristically evident during which stage of cognitive
development in a child?
Marked out of 1.00

Flag question Select one:
Informal operations
Sensorimotor
Preoperational
Formal operations
Concrete operations

Your answer is incorrect.
In the sensorimotor stage of cognitive development, object permanence is achieved in about
nine months. This refers to an understanding object that disappears from the field of
perception has not ceased to exist; if searched for this object can be found, or it will reappear.
Hence, peek­a­boo games are understood and enjoyed.
The correct answer is: Sensorimotor

Question 71 MoK Assessment EMI028
Not answered Delusion syndromes
Identify the psychopathological term used for each of the following:
Marked out of 3.00

Flag question
Capgras syndrome
Intermetamorphosis
A woman telephoned the police as
Fregoli syndrome
she believed her husband was an
Syndrome of subjective doubles
imposter
Folie a deux
Doppelganger
Mirror sign
Phantom Boarder syndrome
Fregoli syndrome
Folie a deux
A woman walking down the street Phantom Boarder syndrome
recognized a stranger as her
Mirror sign
husband
Doppelganger
Capgras syndrome
Intermetamorphosis
A 76­year­old woman with
Syndrome of subjective doubles
Phantom Boarder syndrome
Alzheimers dementia has a false Fregoli syndrome
belief that some guests are living in Doppelganger
her house Intermetamorphosis
Syndrome of subjective doubles
Folie a deux
Capgras syndrome
Mirror sign

1355
Your answer is incorrect.
Explanation: 
In Capgras syndrome, the delusion may be that familiar people have been replaced with
outwardly identical strangers. 
In Fregoli's syndrome, false identification of previously known individuals occurs when
meeting strangers (i.e. friends/family members are coming in disguise). 
The Phantom Boarder Syndrome is a false belief that guests are living in the person's house,
often seen in patients with dementia.
The correct answer is: A woman telephoned the police as she believed her husband was an
imposter – Capgras syndrome, A woman walking down the street recognized a stranger as
her husband – Fregoli syndrome, A 76­year­old woman with Alzheimers dementia has a false
belief that some guests are living in her house – Phantom Boarder syndrome

Question 72 MoK Assessment EMI031
Not answered Defence mechanisms
Identify the defence mechanisms used in each of the following clinical situations;
Marked out of 5.00

Flag question
Altruism
Passive aggression
A man who was reprimanded by his Identification
supervisor at work picks a fight with
Displacement
his wife at home over a trivial cause
Repression
Sublimation
Regression
Denial
Regression
Intellectualization
Identification
A hostage victim falls in love with the
Reaction formation
Denial
perpetrator Displacement
Altruism
Sublimation
Passive aggression
Reaction formation
Denial
Intellectualization
Displacement
Repression
Sublimation
Regression
Repression
A man with numerous difficulties at
Intellectualization
work spends most evenings playing
squash and finds that this helps his
Passive aggression
temper Altruism
Reaction formation
Identification

1356
A 50­year­old woman was
Displacement
overlooked for promotion at work.
Denial
She said to her colleagues that she
was never in the fray and did not
Identification
expect this promotion at such an Altruism
early stage of her career. Intellectualization
Regression
Sublimation
Passive aggression
Intellectualization
An 18­year­old man who was
Repression
Regression
abandoned by his parents later Reaction formation
Displacement
joined a humanitarian organization Reaction formation
that helps orphans Sublimation
Passive aggression
Denial
Altruism
Repression
Your answer is incorrect. Identification
Explanation: 
Displacement refers to deflection of anger on a substitute target instead of the source that
triggered the anger. 
Identification with the aggressor is often seen in victims kidnapped by terrorists (also known
as Stockholm syndrome). One way of seeking refuge from the pain of being badly treated is to
identify with the aggressor and to treat another person in a similar way. 
Sublimation refers to channelization of aggressive impulses into creative activities.
Denial refers to the exclusion of unpleasant or painful reality from conscious awareness.
Altruism refers to the process of turning a negative experience into a socially useful or
positive one. It often involves using constructive and gratifying service to others to receive a
vicarious satisfaction. This does not necessarily mean giving up one's pleasures (i.e. altruism
is not he same as asceticism).
The correct answer is: A man who was reprimanded by his supervisor at work picks a fight
with his wife at home over a trivial cause – Displacement, A hostage victim falls in love with
the perpetrator – Identification, A man with numerous difficulties at work spends most
evenings playing squash and finds that this helps his temper – Sublimation, A 50­year­old
woman was overlooked for promotion at work. She said to her colleagues that she was never
in the fray and did not expect this promotion at such an early stage of her career. – Denial, An
18­year­old man who was abandoned by his parents later joined a humanitarian organization
that helps orphans – Altruism

Question 73 MoK Assessment EMI032
Not answered Disorders and Freudian defences
For each of the following disorders given below, identify the defences commonly used.
Marked out of 4.00

Flag question
Projection
Idealization
Splitting
Paranoid delusions
Denial
Dissociation
Sublimation
Denigration
Displacement
1357
Regression
Acting out
Dissociation
Fugue
Denial
Sublimation
Displacement
Denigration
Idealization
Dissociation
Splitting
Projection
Projection
Regression
Dissocial personality Isolation
Idealization
Denigration
Isolation
Acting out
Sublimation
Isolation
Displacement
Sublimation
Splitting
Denigration
Phobia Denial
Denial
Dissociation
Regression
Projection
Displacement
Idealization
Splitting
Your answer is incorrect.
Acting out
Freud formulated that patients with paranoid delusions perceived and reacted to
unacceptable inner impulses as though they originated outside the self (refer to Schreber's
case as interpreted by Freud). For example, the person who attributes hostility to others may
be unconsciously projecting their own hostility. Thus projection is a defense associated with
paranoia.
Temporarily but drastically modifying one's sense of personal identity to avoid emotional
distress is called dissociation. Fugue states and hysterical conversion reactions are various
manifestations of dissociation. 
Acting out refers to the expression of an unconscious wish or impulse through action to avoid
being conscious of an accompanying affect. The unconscious fantasy is lived out impulsively
in behaviour, thereby gratifying the impulse instead of prohibiting it, as in antisocial
personality disorder patients.
Displacement is the process by which interest and emotion are shifted from one object to
another less threatening, often less­retaliating one. For example, one who is told off by her
consultant during clinical supervision may displace the anger felt onto her spouse or dog
(though the reaction may be remarkably different from the two!). Freud formulated phobia to
be related to displacement where the fear originating from a different object/person is
expressed against a less threatening, easily avoidable object.
The correct answer is: Paranoid delusions – Projection, Fugue – Dissociation, Dissocial
personality – Acting out, Phobia – Displacement

Question 74 MoK Psychology EMI003
Not answered Schools of Psychology
Match each of the following with the relevant person from the list above;
Marked out of 3.00

Flag question

1358
Alfred Adler
Carl Gustav Jung
Founder of the school of individual Sigmund Freud
psychology Johann Weyer
John Watson
Francis Galton
Wilhelm Greisinger
Gustav Theodore Fechner
John Watson
Frederic Skinner
Francis Galton
Founder of operant conditioning Wilhelm Greisinger
model of learning Alfred Adler
Carl Gustav Jung
Sigmund Freud
Frederic Skinner
Gustav Theodore Fechner
Carl Gustav Jung
Johann Weyer
Gustav Theodore Fechner
Alfred Adler
Founder of the school of behaviorism
John Watson
Francis Galton
Wilhelm Greisinger
Frederic Skinner
Johann Weyer
Sigmund Freud
Your answer is incorrect.
Sigmund Freud is credited with the foundation of Psychoanalysis as an intervention in
psychiatry though many of his predecessors employed a variety of similar techniques. 
Alfred Adler, a Viennese physician, initially followed Freudian ideas but later differed with him
and found the school of individual psychology. He also coined several phrases such as organ
inferiority, masculine protest and striving for superiority
(http://webspace.ship.edu/cgboer/adler.html). 
Behaviorist B.F. Skinner coined operant conditioning, so it is also referred to as Skinnerian
conditioning. 
Analytical Psychology is a school of psychology based on the concepts of Carl Gustav Jung.
Jung gave a fully­developed theory of the structure and dynamics of the psyche, and a
detailed theory of personality types. 
The correct answer is: Founder of the school of individual psychology – Alfred Adler, Founder
of operant conditioning model of learning – Frederic Skinner, Founder of the school of
behaviorism – John Watson

Question 75 Mok Psychology EMI004
Not answered Books and Authors
Match each of the proponents below with the book authored from the given list
Marked out of 3.00

Flag question

1359
The Myth of Mental Illness
Kraepelin
Battie
Bleuler
Foucault
Szasz
Linehan
Laing
Freud
Frankl
Pinel
Bleuler
Bowlby
Laing
The Divided Self Frankl
Linehan
Battie
Pinel
Foucault
Kraepelin
Bleuler
Bowlby
Bowlby
Szasz
Foucault
A Secure Base Freud
Kraepelin
Frankl
Linehan
Freud
Szasz
Laing
Your answer is incorrect. Pinel
Battie
The Myth Of Mental Illness: Foundations of a Theory of Personal Conduct is a controversial
anti­psychiatry book by Thomas Szasz published in 1961.
The Divided Self: An Existential Study in Sanity and Madness was written by the Glasgow­
born psychiatrist RD Laing, who promoted a non­treatment approach to mental illnesses
including psychosis. 
Bowlby's classic work in attachment theory is summarized in his book 'A Secure Base'
The correct answer is: The Myth of Mental Illness – Szasz, The Divided Self
– Laing, A Secure Base – Bowlby

Question 76 MoK Assessment EMI041
Not answered DSM Axes
Identify the descriptions from the given list that are best suited for each of the following DSM­
Marked out of 3.00
IV axes.
Flag question

Schizoid traits
Alcohol intoxication
Unemployment
Axis­2
Narcolepsy
Recurrent self­harm
Hypertension
Substance induced psychosis
Hopelessness
Anorexia nervosa
Global assessment of functioning=60

1360
Axis­3 Anorexia nervosa
Hypertension
Global assessment of functioning=60
Substance induced psychosis
Narcolepsy
Hopelessness
Unemployment
Recurrent self­harm
Hypertension
Schizoid traits
Substance induced psychosis
Alcohol intoxication
Schizoid traits
Axis­5
Unemployment
Hopelessness
Narcolepsy
Global assessment of functioning=60
Alcohol intoxication
Anorexia nervosa
Recurrent self­harm
Your answer is incorrect.
The multi­axial system of DSM­IV used five axes. Though these axes are now not employed in
DSM­5, the multiaxial concept is still being used by researchers and health service analysts.  
Axis I ­ Clinical Disorders (e.g. anorexia, alcohol intoxication, psychosis due to medical
condition), Axis II ­ Personality Disorders/ Mental Retardation (e.g. moderate Mental
Retardation, anxious personality traits), Axis III ­ General Medical Conditions (e.g. epilepsy,
hypertension), Axis IV ­ Psychosocial and Environmental Problems (e.g. Unemployment), Axis
V ­ Global Assessment of Functioning (e.g. GAF score of 60)
The correct answer is: Axis­2
– Schizoid traits, Axis­3 – Hypertension, Axis­5 – Global assessment of functioning=60

Question 77 MoK Assessment EMI042
Not answered Clinical neurological signs
Identify the neurological sign seen in each of the following situations:
Marked out of 3.00

Flag question
Straight leg rising sign
Ataxia
A 60­year­old man develops alcohol Neck stiffness
related hepatic failure, which results
Lhermitte's sign
in anoxic brain damage. 
Beevor sign
Extensor plantar reflex
Perseveration
Optic atrophy
Upper limb rigidity
A 35­year­old man is a known binge Upper limb rigidity
Straight leg rising sign
drinker with significant Flapping tremor
Perseveration
malnourishment. He present with Intention tremor
Flapping tremor
acute onset confusion and
Intention tremor
disorientation.
Neck stiffness
Lhermitte's sign
Beevor sign
Optic atrophy
Ataxia
Extensor plantar reflex

1361
An 38­year­old woman developed Upper limb rigidity
opthalmoplegia. A T1­weighted MRI
Ataxia
reveals multiple hypointense lesions
in the corpus callosum. Lhermitte's sign
Straight leg rising sign
Perseveration
Intention tremor
Beevor sign
Extensor plantar reflex
Optic atrophy
Your answer is incorrect. Flapping tremor
Neck stiffness
A flapping tremor is seen in cases of respiratory, hepatic or cardiac failure with
encephalopathy. 
Ataxia and ophthalmoplegia with nystagmus in a background of impaired consciousness are
characteristic of Wernicke's encephalopathy. 
Lhermitte's sign is characterised by a sudden unpleasant  (but not painful) electric shock­like
sensation starting in the neck and radiating along the spine towards distal limbs. It is seen in
patients with multiple sclerosis.
The correct answer is: A 60­year­old man develops alcohol related hepatic failure, which
results in anoxic brain damage. 
– Flapping tremor, A 35­year­old man is a known binge drinker with significant
malnourishment. He present with acute onset confusion and disorientation.
– Ataxia, An 38­year­old woman developed opthalmoplegia. A T1­weighted MRI reveals
multiple hypointense lesions in the corpus callosum. – Lhermitte's sign

Question 78 MoK Assessment EMI043
Not answered Alcohol­induced physical symptoms
Choose the symptoms which would best support each diagnosis in an alcoholic,
Marked out of 3.00
malnourished middle­aged man who becomes confused two days after emergency surgery
Flag question for a femoral shaft fracture.

Gait ataxia
Clonus
Kernig's sign
Bacterial septicaemia
Elevated central venous pressure
Exophtholmos
Jaundice
Ophthalmoplegia
Neck stiffness
Kernig's sign
Exophtholmos
Jaundice
Wernicke syndrome
Gait ataxia
Ophthalmoplegia
Clonus
Neck stiffness
Elevated central venous pressure

Acute hepatic failure

1362
Elevated central venous pressure
Neck stiffness
Gait ataxia
Kernig's sign
Clonus
Exophtholmos
Ophthalmoplegia
Jaundice

Your answer is incorrect.
Please note that this patient has recently had an emergency surgery of the femur. Don't try to
do Kernig's test on him; also gait ataxia may not be demonstrable. Petechial rashes and neck
stiffness may be observed when septicaemia is present. 
Wernicke's syndrome is an acute condition characterized by the well­known triad of
ophthalmoplegia, ataxia and global confusional state. The classic triad is only present in 10%
of patients, and 80% of patients are not diagnosed before postmortem. The eye signs are
often bilateral though need not be always symmetrical. 
Jaundice and flapping tremor (Asterixis) are features of hepatic failure. Asterixis can be
elicited by having the patient extend both arms with the wrists dorsiflexed and palms facing
forward, and eyes closed. Brief jerky downward movements of the wrist are considered a
positive sign. Asterixis is commonly seen with metabolic encephalopathies. (Note pronator
drift is elicited by having the patient extend both arms with the wrists supinated and palms
facing upwards and eyes closed ­ slow unequal drift towards pronation indicates hemiparetic
weakness). Elevated JVP is not seen in hepatic failure that is due to non­cardiac causes.
The correct answer is: Bacterial septicaemia – Neck stiffness, Wernicke syndrome –
Ophthalmoplegia, Acute hepatic failure – Jaundice

Question 79 MoK Assessment EMI044
Not answered Dementia
Choose salient features for each item below from the list shown:
Marked out of 3.00

Flag question
Striking loss of insight
Tactile hallucinations
A salient features of Lewy Body Seizures
dementia Rapidly progressing dementia
Personality change
Emotional blunting
Auditory hallucinations
History of TIAs
Demential with prominent but short lucid intervals
Demential with prominent but short lucid intervals
History of TIAs
Weight­loss
Personality change
A salient feature of CJD
Emotional blunting
Seizures
Striking loss of insight
Rapidly progressing dementia
Auditory hallucinations
Weight­loss
Tactile hallucinations
A salient feature of dementia with
Parkinson's disease

1363
Tactile hallucinations
Seizures
Striking loss of insight
History of TIAs
Weight­loss
Personality change
Emotional blunting
Auditory hallucinations
Rapidly progressing dementia
Your answer is incorrect. Demential with prominent but short lucid intervals
Dementia with Lewy Bodies is characterised by spontaneous motor features of Parkinsonism,
along with a fluctuating cognition, recurrent visual hallucinations of well formed and detailed
imageries. The cognitive decline is severe enough to affect normal social and occupational
functioning. In Parkinson's disease dementia, affective blunting is prominent often due to well
established Parkinson's disease. 
Creutzfeldt­Jakob disease: The clinical picture is one of rapidly deteriorating dementia,
myoclonus, cerebellar and extrapyramidal signs leading to death within a year. Patients may
present with non­specific symptoms such as lethargy, depression and fatigue. Within weeks,
more fulminant symptoms develop, including progressive cortical­pattern dementia,
myoclonus and pyramidal and extrapyramidal signs. Myoclonus becomes prominent as the
disease progresses. Patients may develop cortical blindness.
The correct answer is: A salient features of Lewy Body dementia
– Demential with prominent but short lucid intervals, A salient feature of CJD
– Rapidly progressing dementia, A salient feature of dementia with Parkinson's disease –
Emotional blunting

Finish review

1364
 Home HiYield Paper A(1) Psychology

HiYield Paper A(1)

Started on Saturday, 27 June 2015, 2:18 PM
State Finished
Completed on Saturday, 27 June 2015, 2:30 PM
Time taken 12 mins 26 secs
Marks 1.00/123.00
Grade 0.81 out of 100.00

Question 1 HiY Psychology EMI001
Partially correct Learning theory in practice
For each of the following situations, identify the learning theory used in practice.
Mark 1.00 out of
4.00

Flag question
Chaining
Classical conditioning
A cat bit a young child when she
Discriminative learning
attempted to feed it. After that, the girl
Intermittent reinforcement.
remains afraid of approaching the
animal.
Partial reinforcement
Shaping
Modelling
Reciprocal inhibition
Extinction
Discriminative learning
A man with spider phobia is asked to Variable reinforcement
Reciprocal inhibition
relax and listen to music and
Intermittent reinforcement.
presented initially with pleasant
Modelling
visual images. Later he is shown a
Extinction
picture of a spider.
Shaping
Variable reinforcement
A rat is presented with a lever. Partial reinforcement
Initially, he was given a food pellet
Extinction
Chaining
when he moved towards the lever Discriminative learning
Classical conditioning
every time. Later the food pellets Chaining
were delivered only upon touching Intermittent reinforcement.
the lever. Later, the delivery of food Variable reinforcement
only occurred when the lever was Shaping
pressed. Classical conditioning
Reciprocal inhibition
Intermittent reinforcement.
Partial reinforcement
Extinction
Modelling
Discriminative learning
A 6­year­old boy imitates the
Partial reinforcement
aggressive behaviour of his father,
which alarms his mother Classical conditioning
Chaining
Reciprocal inhibition
Modelling

1365
Check

Explanation: 
Vignette 1 ­ Classical conditioning explains some of the fear acquisitions seen during a child's
development. 
Vignette 2 refers to reciprocal inhibition. When anxiety and a relaxed state are co­existent,
then anxiety reduces ­ reciprocal inhibition. Here the spider phobia is the anxiety state. The
relaxed state is listening to music and pleasant visual images. 
Vignette 3 refers to shaping, a form of operant conditioning in which a desirable behaviour
pattern is learnt by the successive reinforcement of behaviours closer to the desired one. Note
that unlike chaining, shaping is used when the target behaviour does not exist yet. In shaping,

some approximation of the target behaviour is reinforced in steps. 
Vignette 4 refers to modeling, which is a form of observational learning. In participant
modeling, subjects learn a new behavior by imitation, primarily by observation, without having
to perform the behavior until they feel ready.
The correct answer is: A cat bit a young child when she attempted to feed it. After that, the girl
remains afraid of approaching the animal. – Classical conditioning, A man with spider phobia
is asked to relax and listen to music and presented initially with pleasant visual images. Later
he is shown a picture of a spider. – Reciprocal inhibition, A rat is presented with a lever.
Initially, he was given a food pellet when he moved towards the lever every time. Later the
food pellets were delivered only upon touching the lever. Later, the delivery of food only
occurred when the lever was pressed. – Shaping, A 6­year­old boy imitates the aggressive
behaviour of his father, which alarms his mother – Modelling

Question 2 HiY Psychology EMI002
Not answered Types of Learning
Using the definitions given below, identify the types of learning principles applied
Marked out of 5.00

Flag question
Habituation
Learning results from consequences Operant conditioning
of one's actions and the resultant Sensitization
effect on the environment. The
Insight learning
learning organism is active and
Discrimination learning
instrumental on the environment.
Cross sensitization
Classical conditioning
Social learning
Latent learning
Learning takes place through Latent learning
Social learning
repeated temporal association of two Pseudo­conditioning
Sensitization
events. The learning organism is
Insight learning
passive, respondent but not
instrumental.
Operant conditioning
Cross sensitization
Pseudo­conditioning
Classical conditioning
Habituation
Although new learning occurs slowly,
in some situations solution to a Discrimination learning
problem comes in an instant, with a

1366
sudden grasp of a concept Classical conditioning
Insight learning
Pseudo­conditioning
Cross sensitization
Social learning
Discrimination learning
Operant conditioning
Sensitization
Social learning
Latent learning
Habituation
Habituation
Latent learning
This type of learning combines both Insight learning
classic and operant models of
Cross sensitization
learning and includes cognitive
Operant conditioning
processes and social interactions to
Pseudo­conditioning
be relevant in human learning
Discrimination learning
Classical conditioning
Sensitization

Cross sensitization
Reinforcement may be necessary for Operant conditioning
a performance of learned response Pseudo­conditioning
but not necessary for the learning Social learning
itself to occur Classical conditioning
Insight learning
Habituation
Latent learning
Check
Sensitization
Discrimination learning

Explanation: 
Classic conditioning: learning takes place through repeated temporal association of two
events. The learning organism is passive, respondent (e.g. salivation is a reflexive response)
but not instrumental. 
In operant conditioning, learning results from consequences of one's actions ­ operations. The
learning organism actively operates (instrumental) on the environment. 
Insight learning: Kohler first demonstrated this learning phenomenon. In most situations, new
learning occurs slowly leading to gradual improvement in performance. In some situations,
the solution to a problem comes in an instant, with a sudden grasp of a concept. This sudden
awareness of a solution to a problem is termed as insight learning. Kohler in 1952 observed
that one of the chimpanzees was able to reach a banana outside the cage using a stick and
even join tow sticks to manoeuvre the banana towards him. 
Social learning theory combines both classic and operant models of learning and includes
cognitive processes and social interaction to be relevant in human learning. 
According to Tolman, reinforcement may be necessary for a performance of learned response
but not necessary for the learning itself to occur (Latent learning). He inferred that rats could
make cognitive maps of mazes ­ called place learning, which consists of cognitive
expectations as to what comes next, even without overt reinforcement.
The correct answer is: Learning results from consequences of one's actions and the resultant
effect on the environment. The learning organism is active and instrumental on the
environment. – Operant conditioning, Learning takes place through repeated temporal

1367
association of two events. The learning organism is passive, respondent but not instrumental.
– Classical conditioning, Although new learning occurs slowly, in some situations solution to a
problem comes in an instant, with a sudden grasp of a concept – Insight learning, This type of
learning combines both classic and operant models of learning and includes cognitive
processes and social interactions to be relevant in human learning – Social learning,
Reinforcement may be necessary for a performance of learned response but not necessary
for the learning itself to occur – Latent learning

Question 3 HiY Psychology EMI003
Not answered Cancer chemotherapy and behavioural psychology
A middle­aged man is diagnosed with a skin tumour for which he is undergoing six months
Marked out of 5.00
long radiotherapy and chemotherapy, administered at the same hospital every time by
Flag question uniformed nurses and radiotherapists. This treatment has resulted in severe nausea and
vomiting. Match one term from the given list of learning concepts to each of the following
situations.

Avoidance learning
He develops nausea as soon as he Stimulus generalization
enters the hospital, without even Negative reinforcement
taking the drugs or seeing his therapy Aversive conditioning
team. Chaining
Classical conditioning
Modelling
Operant conditioning
Stimulus generalization
Extinction
Negative reinforcement
The nausea worsens as soon as he
sees anyone in a white coat, Sensitisation
Operant conditioning
including the lab workers and Discrimination
Chaining
pharmacist. Aversive conditioning
Modelling
Sensitisation
Extinction
Aversive conditioning
Discrimination
Operant conditioning
Whenever possible he avoids using Classical conditioning
Classical conditioning
the travel route via the hospital, to Avoidance learning
Extinction
avoid nausea.
Discrimination
Modelling
Sensitisation
Negative reinforcement
Discrimination
Avoidance learning
He experiences nausea whenever he Classical conditioning
sees any tablet, even if it is not for his Chaining
Avoidance learning
cancer, but he is not nauseous when Stimulus generalization
Aversive conditioning
taking them as capsules. Sensitisation
Negative reinforcement
Operant conditioning
Modelling
Stimulus generalization
His treatment is stopped, but he is
followed up for next six months by the
Chaining
same team. Gradually his nausea Extinction
subsides

1368
Aversive conditioning
Avoidance learning
Classical conditioning
Discrimination
Extinction
Modelling
Chaining
Operant conditioning
Check Negative reinforcement
Stimulus generalization
Sensitisation
Explanation: Nausea is an autonomic reflex. In the above scenario, it gets paired in the
following fashion. Receiving chemotherapy is the unconditioned stimulus (UCS); suffering
from nausea due to chemotherapy is the unconditioned response (UCR). The UCS is now
paired with CS (hospital) for few diets. Following conditioning, the CS (hospital) produces
conditioned response (CR), which is feeling nauseous on visiting hospital itself. The 'spread'
of associative learning from one stimulus to other is called stimulus generalisation. In the
above scenario, the nausea worsens as soon as he sees anyone in a white coat.
Discrimination is the opposite process where learned responses are made only to specific
stimuli and not to other similar stimuli. For example, a child may be afraid of a barking dog but
as the child can discriminate other four­legged animals from dogs, he or she may not be afraid
of a cat. Extinction refers to the reduction or disappearance of a learned response when the
UCS ­ CS pairing (or the reinforcer in operant conditioning) is not available any more. Faster
extinction may mean weaker learning.
The correct answer is: He develops nausea as soon as he enters the hospital, without even
taking the drugs or seeing his therapy team. – Classical conditioning, The nausea worsens as
soon as he sees anyone in a white coat, including the lab workers and pharmacist. – Stimulus
generalization, Whenever possible he avoids using the travel route via the hospital, to avoid
nausea. – Avoidance learning, He experiences nausea whenever he sees any tablet, even if
it is not for his cancer, but he is not nauseous when taking them as capsules. – Discrimination,
His treatment is stopped, but he is followed up for next six months by the same team.
Gradually his nausea subsides – Extinction

Question 4 HiY Psychology EMI004
Not answered Psychological mechanisms
Identify one main mechanism involved for each of the following situations.
Marked out of 4.00

Flag question
Stimulus preparedness
Stimulus generalization
Tom is a seven­year­old boy who is
Classical conditioning
told to finish his homework before he
Discriminative learning
can go and play with his mates
Reciprocal inhibition
Cognitive mapping
Extinction
Miss. Y is a 23­year­old woman who Shaping
has a history of spider phobia. She is Chaining
put into a relaxed state by soothing Premack principle
music, suggestion and visualizing
pleasant images. While she is in a
relaxed state, she is shown a picture
of a spider.

1369
Chaining
Premack principle
Cognitive mapping
Discriminative learning
Stimulus generalization
Extinction
Reciprocal inhibition
Stimulus preparedness
Shaping
Chaining
Lucy is a 7­year­old girl who is Classical conditioning
Extinction
seeing a speech therapist for mutism. Shaping
She only speaks in the presence of Premack principle
the therapist but not others Reciprocal inhibition
Discriminative learning
Cognitive mapping
Classical conditioning
Sam is a 3­year­old boy, who is Stimulus generalization
learning to write. His mother Stimulus preparedness
appreciates him for holding his pencil Extinction
correctly, then for scribbling
Premack principle
something on the paper and later for
writing something, which resembles a
Reciprocal inhibition
letter of the alphabet. Discriminative learning
Chaining
Cognitive mapping
Check
Shaping
Classical conditioning

Explanation: Vignette 1 refers to Premack principle, which states that high­frequency
behaviour can be used to reinforce a low­frequency behaviour. This is also called as
Grandma's rule ­ eat the spinach to get the dessert. This is nothing but positive reinforcement
but instead of a material reinforcer, an already existing behaviour itself (eating dessert) is
linked to desired response (eating spinach). 
Vignette 2 refers to reciprocal inhibition, where a stimulus with the desired response and a
stimulus with the undesired response are presented together repeatedly; then incompatibility
leads to reduced undesired response. 
Vignette 3 refers discriminative learning, a process where learned responses are made only
to specific stimuli and not to other similar stimuli. 
Vignette 4 refers to shaping which is a form of operant conditioning where a desirable
behaviour pattern is learnt by the successive reinforcement of behaviours closer to the
desired one.
The correct answer is: Tom is a seven­year­old boy who is told to finish his homework before
he can go and play with his mates – Premack principle, Miss. Y is a 23­year­old woman who
has a history of spider phobia. She is put into a relaxed state by soothing music, suggestion
and visualizing pleasant images. While she is in a relaxed state, she is shown a picture of a
spider. – Reciprocal inhibition, Lucy is a 7­year­old girl who is seeing a speech therapist for
mutism. She only speaks in the presence of the therapist but not others – Discriminative
learning, Sam is a 3­year­old boy, who is learning to write. His mother appreciates him for
holding his pencil correctly, then for scribbling something on the paper and later for writing
something, which resembles a letter of the alphabet. – Shaping

1370
HiY Psychology EMI005
Question 5
Operant conditioning
Not answered
Identify the type of reinforcement using the descriptions given below.
Marked out of 4.00

Flag question Continuous reinforcement
Secondary reinforcers
This type of reinforcement is the least Latent learning
resistant to extinction Fixed interval reinforcement
Variable ratio reinforcement
Fixed ratio reinforcement
Intermittent Reinforcement
Variable interval reinforcement
Secondary reinforcers
Primary reinforcers
Fixed interval reinforcement
This type of reinforcement is the most Fixed ratio reinforcement
resistant to extinction Variable interval reinforcement
Intermittent Reinforcement
Variable ratio reinforcement
Continuous reinforcement
Latent learning
Primary reinforcers
Primary reinforcers
Variable ratio reinforcement
This type of reinforcement takes the Secondary reinforcers
longest to establish. Fixed interval reinforcement
Variable interval reinforcement
Fixed ratio reinforcement
Intermittent Reinforcement
Latent learning
Primary reinforcers
Continuous reinforcement
Intermittent Reinforcement
This type of reinforcement leads to Fixed interval reinforcement
highest rate of responding Variable ratio reinforcement
Latent learning
Continuous reinforcement
Secondary reinforcers
Fixed ratio reinforcement
Check
Variable interval reinforcement

Explanation: 
In operant conditioning reinforcement schedules can be continuous or partial (also called
intermittent). In full/ continuous reinforcement schedules (also called contingency
reinforcement), every behaviour is followed by a reinforcer. This makes learning quicker, but
extinction occurs soon after few occasions when a reinforcer is not provided as the organism
learns that there is no use in doing the same behaviour as it is not going to get anything.
Vending machines where everytime you press a button after placing a pound coin in the slot
are examples of continuous reinforcement. It takes only a short time to learn, but if once you
fail to get the drink from a machine, you will become wary and stop using it soon. In partial or
intermittent reinforcement schedules, only some instances of the behaviour get reinforced
during the learning phase. This may be done in a ratio schedule where every nth behaviour
can be reinforced or interval schedule where every nth second the behaviour gets reinforced.
In ratio schedules, higher the ratio, higher the response rate. 

1371
Partial schedules can be of fixed ratio/interval or variable ratio/interval. In fixed­ratio (FR)
schedule, a reward is given after a certain number of responses are made. For example, the
locum on­calls are paid according to number of hours worked. The example of a rat receiving
food pellet for every 3rd lever­press is also a fixed ratio (a constant 1:3 ratio). As more
responses yield more reinforcement, only short pauses occur after each response. So, FR
schedules produce the highest rate of responding. In fixed interval schedules, the time interval
is fixed beforehand and is constant e.g. every 4th minute a pellet is given to the rat. Another
example is your monthly salary. You get paid once a month (reward) no matter how many
hours you work (response). As a result, most salaried workers tend to work slow and steady,
only doing as much as they have to essentially do! In fixed interval schedules, a longer pause
may be seen after reinforcement as the organism knows the reinforcement will not be
happening for some reasonable time or attempts after this. Variable schedules generate a
constant rate of response as the chance of obtaining a reward stays the same at any time and
any instance of behaviour. Examples include buying lottery, gambling on horses, etc. This is
difficult to extinguish, as rewards are unpredictable. Even when the organism goes without
reward for long periods, the expectations will stay high and responses may even increase in
vigour. 
When we interpret an operation to be under control, we learn more quickly ­ as seen in fixed
schedules, compared to intermittent schedules where learning is slower. But intermittent
schedules are more resistant to extinction with the variable­ratio being the most resistant.
The correct answer is: This type of reinforcement is the least resistant to extinction –
Continuous reinforcement, This type of reinforcement is the most resistant to extinction –
Variable ratio reinforcement, This type of reinforcement takes the longest to establish. –
Intermittent Reinforcement, This type of reinforcement leads to highest rate of responding –
Fixed ratio reinforcement

Question 6 HiY Psychology EMI006
Not answered Terms related to memory 
For each of the description below chose one most appropriate term from the given list
Marked out of 4.00

Flag question
Recapitulation
Delusional elaboration
Reintegrating events using a variety Retrospective falsification
of components Registration
Confabulation
Déjà vu
Recognition
Recollection
Jamais vu
Jamais vu
Retrospective falsification
Modifying memory in terms of Recognition
attitudes and emotions Registration
Confabulation
Recollection
Recapitulation
Déjà vu
Registration
Delusional elaboration
Delusional elaboration
Absence of the normal feeling of
familiarity that accompanies memory Recapitulation
when visiting a place for the second Jamais vu
time Confabulation
Retrospective falsification
Déjà vu
Recognition

1372
Confabulation
Feeling of familiarity that Déjà vu
accompanies return of stored Registration
material to consciousness Recognition
Recollection
Jamais vu
Delusional elaboration
Retrospective falsification
Check Recapitulation

Explanation: Recollection is the reintegration of a complete event from a variety of different
components and is a function of recall. For example, to remember the sequence of events on
your wedding day requires recalling a number of separate items of information by exploring
many different avenues. 
Retrospective falsification involves applying 'emotional colour' to the recall of events. 
Jamais vu refers to a pathology of familiarity that is the phenomenological antagonism of déjà
vu. It is a form of paramnesia characterised by an erroneous belief of having never before
seen or experienced something that one has in reality encountered before. 
Recognition is the feeling of familiarity, which accompanies the return of stored material to
consciousness. It is therefore not strictly part of the process of memory but is intimately
connected.
The correct answer is: Reintegrating events using a variety of components – Recollection,
Modifying memory in terms of attitudes and emotions – Retrospective falsification, Absence of
the normal feeling of familiarity that accompanies memory when visiting a place for the
second time – Jamais vu, Feeling of familiarity that accompanies return of stored material to
consciousness – Recognition

Question 7 HiY Psychology EMI007
Not answered Theories of Attention
For each of the statements given below, identify the names of the theory from the given list.
Marked out of 4.00

Flag question
Equity theory
Group think
'People can only attend to one
Shiffrin and Schneider's divided attention theory
physical channel of information at a
Cannon Bard theory
time'
Social exchange theory
Attenuator model of selective attention
Broadbent's filter theory of attention
Dichotic listening theory
Schema theory
Schema theory
Attenuator model of selective attention
Cocktail party phenomenon
Cocktail party phenomenon
Cannon Bard theory
'Alternative information is Dichotic listening theory
simultaneously processed and can Group think
be attended to if required.' Shiffrin and Schneider's divided attention theory
Broadbent's filter theory of attention
Equity theory
Social exchange theory

1373
Schema theory
Group think
'The ability of people to switch their
Dichotic listening theory
attention rapidly to a non­processed
Attenuator model of selective attention
message.'
Shiffrin and Schneider's divided attention theory
Social exchange theory
Broadbent's filter theory of attention
Cocktail party phenomenon
Schema theory
Equity theory
Shiffrin and Schneider's divided attention theory
Cannon Bard theory
Social exchange theory
'Irrelevant stimuli are attenuated'
Group think
Dichotic listening theory
Broadbent's filter theory of attention
Cocktail party phenomenon
Attenuator model of selective attention
Check
Equity theory
Cannon Bard theory

Explanation: 
Broadbent's early selection filter theory suggests that our ability to process information is
capacity limited. A temporary buffer system receives all information and passes it to a
selective filter. The selection is based on physical characteristics of the information ­ one
source is selected, and others are rejected. Processing two different information will take
longer and will be less efficient as switching takes a substantial period. 
Dichotic listening refers to feeding one message into the right ear and a different message
simultaneously into left ear. Participants have to repeat one message aloud. This process is
called Shadowing (first used by Cherry). This is a method to study selective attention. Divided
attention can be tested using a dual­task technique whereby the individual is asked to attend
and respond to both or all incoming messages. 
Cocktail party effect is a concept related to selective attention. It is a term used to describe "the
ability of people to be able to switch their attention rapidly to a non­processed message"
amidst the abundance of multiple other distracting stimuli. Certain types of stimuli e.g. the
location of the speaker, the pitch of the voice or the use of familiar stimuli such as the listener's
name, can provoke the swift switch between messages (also called the Lunch­Queue effect).
The correct answer is: 'People can only attend to one physical channel of information at a
time' – Broadbent's filter theory of attention, 'Alternative information is simultaneously
processed and can be attended to if required.' – Dichotic listening theory, 'The ability of
people to switch their attention rapidly to a non­processed message.' – Cocktail party
phenomenon, 'Irrelevant stimuli are attenuated' – Attenuator model of selective attention

Question 8 HiY Psychology EMI008
Not answered Theories of Emotion
For each of the following statements, choose the most appropriate names of theory from the
Marked out of 4.00
attached list.
Flag question

1374
'Emotion is secondary to
Shiffrin and Schneider's theory
physiological changes and that there
Dichotic listening theory
is a distinct pattern of changes for
different emotions'
Cognitive appraisal theory
James­Lange theory
Schema theory
Broadbent's filter theory
Contingency theory
Cognitive enhancement theory
Cognitive enhancement theory
Schachter­Singer theory
Schema theory
The subjective experience of Cannon­Bard theory
Dichotic listening theory
emotions is independent of Social exchange theory
James­Lange theory
physiological changes.
Shiffrin and Schneider's theory
Schachter­Singer theory
Cognitive appraisal theory
Social exchange theory
Dichotic listening theory
Cannon­Bard theory
Cognitive appraisal theory
The cognitive appraisal conscious or Broadbent's filter theory
Social exchange theory
unconscious always precedes Contingency theory
Cognitive enhancement theory
emotional experience.
Schema theory
James­Lange theory
Cannon­Bard theory
Contingency theory
Contingency theory
Shiffrin and Schneider's theory
Shiffrin and Schneider's theory
Schachter­Singer theory
Dichotic listening theory
On perception of a stimulus, both Broadbent's filter theory
Cognitive enhancement theory
physiological changes and a
Cognitive appraisal theory
conscious experience of general
James­Lange theory
arousal take place simultaneously
which is then interpreted to either Schachter­Singer theory
positive or negative valence and Social exchange theory
labelled appropriately Broadbent's filter theory
Cannon­Bard theory
Schema theory

Check

Explanation: 
James­Lange theory of emotions: Perception of a stimulus leads to bodily (skeletal and
visceral) changes. The peripheral responses send feedback to the cortex via thalamus
leading to the perception of the emotion.
Cannon­Bard theory: On the perception of a stimulus, thalamus coordinates signals to cortex
leading to conscious experience and simultaneously sends signals to hypothalamus leading
to physiological changes.
Schachter­singer labelling theory: On the perception of a stimulus, both physiological
changes and a conscious experience of general arousal take place simultaneously. This
generic arousal is then interpreted to either positive or negative and labelled appropriately
according to the situational cues. This is also called jukebox theory or two­factor theory. If an
appropriate label is not found, by default, negative appreciation of arousal occurs.

1375
Lazarus cognitive appraisal theory states that appraisal precedes affective reaction ­ hence
affective primacy cannot be true.
The correct answer is: 'Emotion is secondary to physiological changes and that there is a
distinct pattern of changes for different emotions' – James­Lange theory, The subjective
experience of emotions is independent of physiological changes. – Cannon­Bard theory, The
cognitive appraisal conscious or unconscious always precedes emotional experience. –
Cognitive appraisal theory, On perception of a stimulus, both physiological changes and a
conscious experience of general arousal take place simultaneously which is then interpreted
to either positive or negative valence and labelled appropriately – Schachter­Singer theory

Question 9 HiY Psychology EMI009
Not answered Group Processes
For each of the following definitions, identify the right term used to describe them from the list
Marked out of 4.00
shown.
Flag question

When individuals express their Pluralistic ignorance
opinions separately and then group Conformity
to decide upon the same matter the Deindividuation.
eventual outcome is likely to be more Risky shift phenomenon
extreme than that of the group Sociogram
average. Obedience
Diffusion of responsibility
Bystander intervention
Deindividuation.
Groupthink
Bystander intervention
Polarization
Groupthink
Sociogram
People tend to make riskier decisions
Polarization
when working as members of a group
Pluralistic ignorance
than they would make when making
the same decision as individuals.
Conformity
Obedience
Diffusion of responsibility
Risky shift phenomenon

Polarization
Risky shift phenomenon
It is the desire to achieve consensus Deindividuation.
and avoid dissent in group decisions. Diffusion of responsibility
Bystander intervention
Conformity
Sociogram
Obedience
Diffusion of responsibility
Groupthink
Deindividuation.
It is a graphical representation of Pluralistic ignorance
Polarization
relationships in a group Obedience
Conformity
Pluralistic ignorance
Bystander intervention
Risky shift phenomenon
Groupthink
Sociogram

1376
Check

Explanation: There are various processes that influence individuals when making decisions
as a part of a group. The group can make more risky decisions than what an individual
him/herself is inclined to. This is called risky shift. A group discussion process can strengthen
average individual inclinations and polarise the group in the direction where most individuals
were heading already. This is called group polarisation. While making extreme decisions, the
desire to agree with other members of a group can override rational judgment applicable in
individual decision­making. This is called groupthink.
The correct answer is: When individuals express their opinions separately and then group to
decide upon the same matter the eventual outcome is likely to be more extreme than that of
the group average. – Polarization, People tend to make riskier decisions when working as
members of a group than they would make when making the same decision as individuals. –
Risky shift phenomenon, It is the desire to achieve consensus and avoid dissent in group
decisions. – Groupthink, It is a graphical representation of relationships in a group –
Sociogram

Question 10 HiY Psychology EMI010
Not answered Altruism
For each of the following descriptions, identify the right term used to describe the
Marked out of 4.00
psychological phenomena from the list provided.
Flag question

Diffusion of responsibility
Pluralistic ignorance
The tendency of not to intervene in a Risky shift phenomenon
situation due to the presence of
Groupthink
others.
Sociogram
Polarization
Obedience
Bystander apathy
Polarization
Deindividuation.
Sociogram
People feel that the responsibility is Conformity
Deindividuation.
not theirs, and someone else will do
Bystander apathy
something.
Risky shift phenomenon
Conformity
Groupthink
Diffusion of responsibility
Pluralistic ignorance
Obedience
Sociogram
Pluralistic ignorance
Obedience
Members of a group convince each
other that there is no problem Groupthink
Bystander apathy
Conformity
Polarization
Risky shift phenomenon
Diffusion of responsibility
The individual behaviour of group Deindividuation.
members is less important than that
of the whole group.

1377
Polarization
Groupthink
Diffusion of responsibility
Obedience
Pluralistic ignorance
Sociogram
Deindividuation.
Risky shift phenomenon
Check
Bystander apathy
Conformity

Explanation: 
Bystander apathy: When alone, individuals will typically intervene if another person is in need
of help: this is called bystander intervention. But intervention becomes less likely to an extent
that no single person will intervene from a crowd or group of observers when someone is in
need of help. This is called bystander apathy or Genovese effect.
Pluralistic ignorance: This refers to members of a crowd looking at each other for signs of
distress, but himself/herself remaining calm, leading to misappraisal of the situation being
safe leading to lack of intervention. 
Diffusion of responsibility: Similar to social loafing ­ 'I have some responsibility, but so do
others; let someone else help' 
Dissolution of responsibility: If not knowing what others are doing, rationalizes that someone
would have helped the victim.
Deindividuation (Leon Festinger 1952) represents the situation where anti­normative
behaviour appears in group settings in which "individuals are not seen or paid attention to as
individuals". It is immersion in a group to the point at which the individual ceases to be seen
as such.
The correct answer is: The tendency of not to intervene in a situation due to the presence of
others. – Bystander apathy, People feel that the responsibility is not theirs, and someone else
will do something. – Diffusion of responsibility, Members of a group convince each other that
there is no problem – Pluralistic ignorance, The individual behaviour of group members is
less important than that of the whole group. – Deindividuation.

Question 11 HiY Psychology EMI011
Not answered Social Psychology
For each of the following descriptions, identify the right term used to describe the
Marked out of 3.00
psychological phenomena from the list provided;
Flag question

Classic studies by Milgram in which Polarization
the majority of participants delivered Groupthink
the maximum shock voltage, showing
Pluralistic ignorance
that people would carry out orders
Bystander apathy
under certain conditions even if these
exceeded the bounds of their usual
Deindividuation.
beliefs. Diffusion of responsibility
Risky shift phenomenon
Conformity
The normative social influence that
Social loafing
forces an individual to agree with the
Obedience
group view despite holding a different

1378
personal view.
Conformity
Diffusion of responsibility
Social loafing
Risky shift phenomenon
Polarization
Groupthink
Deindividuation.
Pluralistic ignorance
Polarization
Larger a group is, lesser the Bystander apathy
Bystander apathy
individual performance as one thinks Obedience
Obedience
the others will do the job and bring Diffusion of responsibility
the results Conformity
Deindividuation.
Social loafing
Pluralistic ignorance
Check
Groupthink
Risky shift phenomenon

Explanation: 
Conformity is a process where no explicit requirement is made to do a certain task. But peer
influence and the need for acceptance pushes one to carry out the task. 
Obedience refers to conditions where the individual is explicitly asked to do a task, and this
instruction comes from an authority.
Social loafing: This is also called Ringelmann's effect. It is seen in games such as tug­of­war
and clapping hands after a performance. Larger a group is, lesser the individual performance
as one thinks the others will do the job and bring the results.
The correct answer is: Classic studies by Milgram in which the majority of participants
delivered the maximum shock voltage, showing that people would carry out orders under
certain conditions even if these exceeded the bounds of their usual beliefs. – Obedience, The
normative social influence that forces an individual to agree with the group view despite
holding a different personal view. – Conformity, Larger a group is, lesser the individual
performance as one thinks the others will do the job and bring the results – Social loafing

Question 12 HiY Psychology EMI012
Not answered Social Power
With the descriptions given below, identify the type of social power from the list of options
Marked out of 4.00
provided.
Flag question

Democratic power
Autocratic power
The leader has a position of formal Informational power
authority Coercive power
Reward power
Legitimate power
Expert power
Referent power

1379
The follower identifies with the leader
Expert power
and consequently grants him or her
Autocratic power
influence
Reward power
Democratic power
Legitimate power
Coercive power
Referent power
Informational power
Referent power
Legitimate power
The leader can administer or threaten Reward power
punishments for 'unacceptable
Democratic power
behaviours.'
Informational power
Coercive power
Autocratic power
Expert power
Reward power
Democratic power
Followers will allow themselves to be Informational power
influenced because they perceive the
Coercive power
leader to have expertise
Autocratic power
Legitimate power
Referent power
Expert power
Reward power
Autocratic power
This is based on the perception that Democratic power
someone has the right to prescribe
Informational power
behaviour due to election
Referent power
Coercive power
Expert power
Legitimate power
Check

Explanation: 
Reward Power is based on the "perceived ability to give positive consequences or remove
negative ones". 
Coercive Power is the "perceived ability to punish those who not conform to your ideas or
demands". 
Legitimate Power is "based on the perception that someone has the right to prescribe
behaviour due to election or appointment to a position of responsibility". 
Referent Power is through association with others who possess the power (e.g., a colleague
who is closer to the management authorities).
Expert Power is "based on having distinctive knowledge, expertness, ability or skills)".
Information Power is "based on controlling the information needed by others in order to reach
an important goal". 
Excerpt from Power, Essence and the Organisation. (n.d.). Retrieved from
http://www.iasag.ch/docs/article.lammers.power.essence.pdf
The correct answer is: The leader has a position of formal authority – Legitimate power, The
follower identifies with the leader and consequently grants him or her influence – Referent
power, The leader can administer or threaten punishments for 'unacceptable behaviours.' –

1380
Coercive power, Followers will allow themselves to be influenced because they perceive the
leader to have expertise – Expert power, This is based on the perception that someone has
the right to prescribe behaviour due to election – Democratic power

Question 13 HiY Psychology EMI013
Not answered Memory types
Identify the type of memory which is being referred to, for each of the following situations.
Marked out of 4.00

Flag question
A 45­year­old man was involved in a
Declarative memory
road traffic accident and was Working memory
unconscious for a week. On False memory
examination, he has no memory for Semantic memory
any events for a period of a week Episodic memory
prior to the accident. Anterograde memory
Procedural memory
Retrograde memory
Infantile memory
Infantile memory
Procedural memory
Retrograde memory
A 76­year­old man was found
Working memory
wandering and was brought to the False memory
A&E by the police. On mini­mental Episodic memory
state examination, when you ask him Semantic memory
to remember the three objects, he is Declarative memory
unable to immediately repeat them Anterograde memory

An 83­year­old woman has been Working memory
diagnosed with Alzheimer's disease. Retrograde memory
She enjoys playing piano, which she False memory
used to do for a long time even Declarative memory
before her illness began. Which type Semantic memory
of memory remains intact? Anterograde memory
Episodic memory
Infantile memory
Retrograde memory
Procedural memory
Anterograde memory
When answering the question, 'What Episodic memory
is the capital of France?' What is the Infantile memory
type of memory Mr. P was using? Working memory
Procedural memory
Declarative memory
False memory
Semantic memory
Check

Explanation: Retrograde amnesia involves the loss of memories that were created before the
actual trauma or neurological damage. 

1381
Anterograde amnesia involves an inability to form new memories after the condition
developed. 
The immediate memory test in MMSE is thought to be tapping on to the domain of working
memory. 
Remembering a specific piano lesson is an example of explicit memory, while playing the
piano as a result of this learning is an example of implicit procedural memory.
Semantic memory is the portion of long­term memory which is concerned with ideas,
meanings, and concepts that are not related to personal experiences.
The correct answer is: A 45­year­old man was involved in a road traffic accident and was
unconscious for a week. On examination, he has no memory for any events for a period of a
week prior to the accident. – Retrograde memory, A 76­year­old man was found wandering
and was brought to the A&E by the police. On mini­mental state examination, when you ask
him to remember the three objects, he is unable to immediately repeat them – Working
memory, An 83­year­old woman has been diagnosed with Alzheimer's disease. She enjoys
playing piano, which she used to do for a long time even before her illness began. Which type
of memory remains intact? – Procedural memory, When answering the question, 'What is the
capital of France?' What is the type of memory Mr. P was using? – Semantic memory

Question 14 HiY Psychology EMI014
Not answered Ethical bodies
Which of the above organisations/consensus are responsible for each of the following
Marked out of 3.00
practice?
Flag question

General Medical Council
National Institute of Mental Health
National Institute of Clinical excellence
Good medical practice
Nuremberg Code
Department of Health
Helsinki Declaration
Royal College of Psychiatrists
Nuremberg Code
National Institute of Clinical excellence
Ethical human research (Choose General Medical Council
TWO). Department of Health
Royal College of Psychiatrists
National Institute of Mental Health
Helsinki Declaration

Check

Explanation: Good Medical Practice is a key document issued by the General Medical Council
highlighting the duties and professional responsibilities of medical practitioners in the UK.
While ethical human research is a responsibility emphasized by multiple ethical bodies, the
most relevant ones in this list are Helsinki declaration, which enshrines the current research
ethics code, and Nuremberg code which highlighted the importance of voluntary consent in
medical research for the first time in modern medical history.
The correct answer is: Good medical practice – General Medical Council, Ethical human
research (Choose TWO). – Nuremberg Code, Helsinki Declaration

1382
Question 15 HiY Psychology EMI015
Not answered Sociology of Psychiatry
Select one option for each of the following.
Marked out of 3.00

Flag question
Self­actualisation
A patient who had been living in an Rationalisation
institution for many years now cannot Institutionalization
cook for himself and live in a flat Aversion
independently. Normalisation
Overgeneralization
Residual rule breaking
Overgeneralization
Self­actualisation
A member of the staff believes all
Residual rule breaking
mentally ill people are not able to
Rationalisation
decide about what they want.
Institutionalization
Aversion
Normalisation
Normalisation
Self­actualisation
A passenger on a coach laughs to
Aversion
himself and argues with himself in a
Residual rule breaking
loud manner
Rationalisation
Overgeneralization
Institutionalization
Check

Explanation: 
Long­term hospitalization is often associated with "deskilling, where patients might lose social
and vocational skills and the ability to set goals. This is also known as institutionalization. 
Question 2 highlights overgeneralization ('lumping tendency') a common cognitive distortion
in inexperienced professionals. 
Thomas Scheff proposed a labelling theory of mental illness and suggested that persons
labelled mentally disturbed ('deviant') are in fact breaking "residual" rules ­ a set of unwritten
norms of social life. According to him 'residual rules' included diverse kinds of violations for
which the culture provides no explicit label. For the convenience of the society, these
violations get lumped together into a residual category e.g. witchcraft and spirit possession in
the past and mental illness in the contemporary society (Scheff 1999; 55)
The correct answer is: A patient who had been living in an institution for many years now
cannot cook for himself and live in a flat independently. – Institutionalization, A member of the
staff believes all mentally ill people are not able to decide about what they want. –
Overgeneralization, A passenger on a coach laughs to himself and argues with himself in a
loud manner – Residual rule breaking

Question 16 HiY Psychology EMI016
Not answered

1383
Marked out of 3.00 Learning Theory ­ vignettes
For each of the following clinical situations, choose the one option, which describes it more
Flag question
appropriately.

Premack's principle
A rat is presented with a lever. When Backward chaining
he pressed the lever accidentally, the Forward chaining
food was delivered. Impressed by the
Operant conditioning
food rewards, the rat now presses the
lever frequently.
Latent learning
Avoidance learning
Reciprocal inhibition
A cat was placed in a big cage. When Stimulus preparedness
Latent learning
it started to explore the cage, it Cognitive mapping
Shaping
received electric shocks when it Shaping
Avoidance learning
walked around certain parts of the
Backward chaining
cage. After few minutes, the cat
started moving around only in those
Operant conditioning
areas that were shock­free. Stimulus preparedness
Forward chaining
Reciprocal inhibition
Stimulus preparedness
The mother of a 4­year­old child Cognitive mapping
Cognitive mapping
helped her in solving a jigsaw puzzle. Premack's principle
Latent learning
She showed the girl how to start from
Backward chaining
the whole puzzle and then removed
pieces one by one.
Avoidance learning
Reciprocal inhibition
Operant conditioning
Shaping
Check
Premack's principle
Forward chaining

Explanation: 
Operant conditioning stems from 'law of effect' and trial­and­error learning described by
Thorndike. In the latter, an organism learns a behaviour after many trials because the right
behaviour is followed by appropriate consequence. 
Avoidance learning is a better and more specific option than operant learning for case 2. 
Avoidance learning is a type of operant conditioning where an organism learns to avoid
certain responses or situations. A special form of avoidance is escape conditioning seen in
agoraphobia where places in which panic occurs are avoided / escaped from leading to a
housebound state eventually. It is a strong form of learning as avoidance is a powerful
reinforcer and is difficult to extinguish. 
Chaining refers to reinforcing a series of related behaviours, each of which provides the cue
for the next to obtain a reinforcer. Chaining can be forward or backward chaining according to
where the first nodal point of learning starts. If it started from the end behaviour and connected
backwards, it is called backward chaining e.g. in completing a jigsaw puzzle a child can be
taught by starting from the whole puzzle and then removing pieces one by one.
The correct answer is: A rat is presented with a lever. When he pressed the lever accidentally,
the food was delivered. Impressed by the food rewards, the rat now presses the lever
frequently. – Operant conditioning, A cat was placed in a big cage. When it started to explore
the cage, it received electric shocks when it walked around certain parts of the cage. After few
minutes, the cat started moving around only in those areas that were shock­free. – Avoidance
learning, The mother of a 4­year­old child helped her in solving a jigsaw puzzle. She showed
the girl how to start from the whole puzzle and then removed pieces one by one. – Backward

1384
chaining

Question 17 HiY Psychology EMI017
Not answered Cultural Psychology
With the descriptions given below, identify which of the following best describes this process
Marked out of 4.00
during resettlement in UK from overseas?
Flag question

Disintegration
A 45­year­old Sri Lankan refugee Separation
wants to retain his cultural identity Integration
while maintaining contact with Marginalisation
members of the new culture Socialization
Acclimatization
Assimilation
Accommodation
Socialization
Union
Disintegration
A 36­year­old Indian woman does not
want to seek contact with the new Acclimatization
culture and would like to maintain the Accommodation
original cultural identity Union
Marginalisation
Separation
Integration
Disintegration
A 46­year­old Polish gentleman living Assimilation
Socialization
in England takes on the values and Separation
attitudes of the English culture and
Marginalisation
retains only minimal values from his
original culture.
Union
Integration
Acclimatization
Accommodation
Disintegration
A 33­year­old South African woman Assimilation
Separation
is not only shedding her original Union
identity and cultural values but also
Acclimatization
not seeking contact with new cultural
group
Accommodation
Marginalisation
Integration
Socialization
Check Assimilation

Explanation: This question refers to Berry's model of cultural change. 
Assimilation: This refers to a partial adaptation of a new culture (seen in migrants or refugees)
without retaining or giving up all of one's culture of origin completely.
Integration refers to both high retentions of one's own cultural values and high adoption of the
practices of the new culture. 
Separation refers to a high retention of one's own cultural values and low adoption of the
practices of the new culture. 

1385
Marginalisation refers to both low retentions of one's own cultural values and low adoption of
the practices of the new culture. Members of both cultures of origin and adoption may
marginalize these individuals.
The correct answer is: A 45­year­old Sri Lankan refugee wants to retain his cultural identity
while maintaining contact with members of the new culture – Integration, A 36­year­old Indian
woman does not want to seek contact with the new culture and would like to maintain the
original cultural identity – Separation, A 46­year­old Polish gentleman living in England takes
on the values and attitudes of the English culture and retains only minimal values from his
original culture. – Assimilation, A 33­year­old South African woman is not only shedding her
original identity and cultural values but also not seeking contact with new cultural group –
Marginalisation

Question 18 HiY Psychology EMI018
Not answered Behaviourism
Identify the learning theory principles used in each of the following situations.
Marked out of 3.00

Flag question
Aversive conditioning
A boy with learning disabilities is Chaining
toilet trained by successively Blocking
rewarding behaviours that Covert reinforcement
approximate final desired outcome Covert sensitisation
Classical conditioning
Social learning
Reciprocal inhibition
Covert sensitisation
Flooding
Flooding
A boy who is fond of Spiderman
dresses like Spiderman and attempts Operant conditioning
Covert reinforcement
adventurous sports to be like his Punishment
Operant conditioning
favourite hero. Shaping
Shaping
Aversive conditioning
Punishment
Chaining
Aversive conditioning
A man with a desire to touch women Blocking
Shaping
in a crowded tube keeps this impulse Reciprocal inhibition
Reciprocal inhibition
under control by imagining that he Classical conditioning
Punishment
will be arrested and imprisoned for Social learning
sexual offence.
Covert reinforcement
Classical conditioning
Operant conditioning
Blocking
Check
Social learning
Covert sensitisation
Flooding
Chaining
Explanation: 
Shaping is a form of operant conditioning where a desirable behaviour pattern is learnt by the
successive reinforcement of responses closer to the desired one. 
Modelling is a mode of social learning behaviour often seen in school going children. 
Reduction in the frequency of a behaviour (often undesirable behaviour) by associating it with
imagined unpleasant consequences is called covert sensitisation.
The correct answer is: A boy with learning disabilities is toilet trained by successively
rewarding behaviours that approximate final desired outcome – Shaping, A boy who is fond of

1386
Spiderman dresses like Spiderman and attempts adventurous sports to be like his favourite
hero. – Social learning, A man with a desire to touch women in a crowded tube keeps this
impulse under control by imagining that he will be arrested and imprisoned for sexual offence.
– Covert sensitisation

Question 19 HiY Psychology EMI019
Not answered Learning theories ­ illustrations
Identify the learning theory principles used in each of the following situations.
Marked out of 3.00

Flag question
extinction
positive reinforcement
To train a 4­year­girl not to bite her flooding
nails, her mother applies bitter oil to
aversive conditioning
her lips whenever she bites her nails.
modelling
desensitization
punishment
reciprocal inhibition
Disulfiram is prescribed to a 36­ year­ modelling
Variable interval reinforcement
old alcohol dependent patient. Twice desensitization
when lapsed into drinking, he
negative reinforcement
Variable interval reinforcement
developed severe physical continuous reinforcement
continuous reinforcement
discomfort. He now remains abstinent variable ratio reinforcement
negative reinforcement
for last two years. avoidance
reciprocal inhibition
hypnosis
aversive conditioning
hypnosis
aversive conditioning
flooding
continuous reinforcement
To train a 4­year­old boy to say
"goodbye," his mother gives him a variable ratio reinforcement
flooding
candy whenever he waves goodbye punishment
punishment
to someone. positive reinforcement
hypnosis
extinction
modelling
avoidance
desensitization
avoidance
Check
reciprocal inhibition
extinction
Variable interval reinforcement
negative reinforcement
Explanation: Punishment is used to reduce undesirable behaviour by introducing an aversive
variable ratio reinforcement
stimulus (bitter oil).
positive reinforcement
Subsequent to the experience of an aversive reaction to the disulfiram­alcohol combination,
patients on disulfiram reduce their behaviour of drinking. This can be termed as conditioned
avoidance resulting from aversive conditioning. 
Candy is used as a reward that positively reinforces and brings about a desirable behaviour.
This is called positive reinforcement.
The correct answer is: To train a 4­year­girl not to bite her nails, her mother applies bitter oil to
her lips whenever she bites her nails. – punishment, Disulfiram is prescribed to a 36­ year­old
alcohol dependent patient. Twice when lapsed into drinking, he developed severe physical
discomfort. He now remains abstinent for last two years. – aversive conditioning, To train a 4­
year­old boy to say "goodbye," his mother gives him a candy whenever he waves goodbye to
someone. – positive reinforcement

1387
Question 20 HiY Psychology EMI020
Not answered Learning principles
Identify the learning theory principles used in each of the following situations.
Marked out of 3.00

Flag question
An IT firm introduces an escalating
Fixed ratio reinforcement
bonus system whereby workers who Backward conditioning
achieve certain targets are rewarded Fixed interval reinforcement
with rising bonuses irrespective of theTrace conditioning
time taken by them to attain the Reciprocal inhibition
target. Shaping
Variable ratio reinforcement
Variable interval reinforcement
An angler on average gets five meaty Full reinforcement
Continuous reinforcement
Trace conditioning
fishes everyday when he goes out
Full reinforcement
fishing. Sometimes all of these fishes Backward conditioning
are caught within an hour; on other Fixed ratio reinforcement
days he may have to wait for nearly 9 Reciprocal inhibition
hours for a single catch. Shaping
Variable interval reinforcement
Fixed interval reinforcement
Fixed interval reinforcement
Continuous reinforcement
Continuous reinforcement
NHS employees are paid every Variable ratio reinforcement
Backward conditioning
month irrespective of targets
Trace conditioning
achieved or competitive performance.
Fixed ratio reinforcement
Shaping
Variable interval reinforcement
Variable ratio reinforcement
Check Reciprocal inhibition
Full reinforcement

Explanation: In fixed ratio schedule every nth (e.g., fifth, tenth) response is rewarded. Hence,
target­oriented bonuses are in line with fixed­ratio schedules.In variable ratio reinforcement,
every nth response is rewarded on average, but the gap between two rewarded responses is
highly variable. This schedule is also seen in gambling, apart from fishing.This refers to
consistently regular reinforcement at specific time intervals. The desired behaviour may
become sluggish immediately after reinforcement, catching up during the later part of the
interval.
The correct answer is: An IT firm introduces an escalating bonus system whereby workers
who achieve certain targets are rewarded with rising bonuses irrespective of the time taken by
them to attain the target. – Fixed ratio reinforcement, An angler on average gets five meaty
fishes everyday when he goes out fishing. Sometimes all of these fishes are caught within an
hour; on other days he may have to wait for nearly 9 hours for a single catch. – Variable ratio
reinforcement, NHS employees are paid every month irrespective of targets achieved or
competitive performance. – Fixed interval reinforcement

Question 21 HiY Psychology EMI021
Not answered Applied learning theory
For each of the following clinical situations, choose one most appropriate description.
Marked out of 4.00

Flag question

1388
A 6­year­old girl usually plays with Cognitive mapping
her dog without fear. One day the dog
Operant conditioning
accidentally bites the child. She then
becomes fearful of approaching the
Stimulus generalization
dog Classical conditioning
Shaping
Chaining
Reciprocal inhibition
Stimulus preparedness
Cognitive mapping
Premack's principle
Classical conditioning
A 7­year­old child is told to complete Latent learning
Stimulus generalization
his homework before he can play
Chaining
with his friends
Stimulus preparedness
Premack's principle
Latent learning
Operant conditioning
Classical conditioning
Reciprocal inhibition
Reciprocal inhibition
Shaping
Cognitive mapping
Premack's principle
A 15­year­old boy with spider Stimulus generalization
phobias is put into a relaxed state by
Stimulus preparedness
suggestion and soothing music.
While he is in this relaxed state, he is
Shaping
shown the photograph of a spider Latent learning
Operant conditioning
Chaining

A 4­year­old child is learning to write. Operant conditioning
She is appreciated by her parents for Stimulus generalization
holding the pen properly, then for Shaping
scribbling something on the notepad,
Cognitive mapping
later writing something that closely
resembles a letter and finally writes
Latent learning
the letter clearly Chaining
Reciprocal inhibition
Premack's principle
Check Stimulus preparedness
Classical conditioning

Explanation: 
In case 1, the young girl has learnt an association between the pain response and the dog
after a single instance of stimulus­response pairing. 
Case 2 exemplifies the Premack's principle, which states that a high­frequency behaviour can
be used to reinforce a low­frequency behaviour. This is also called as Grandma's rule ­ eat the
spinach to get the dessert. This is nothing but positive reinforcement but instead of a material
reinforcer, an already existing behaviour (eating dessert) is linked to desired response (eating
spinach). 
Case 3 exemplifies reciprocal inhibition (Wolpe). If a stimulus with a desired response and a
stimulus with an undesired response are presented together repeatedly, then incompatibility
leads to a reduction in the undesired response. This is evident when your dog barks at your

1389
friend; try hugging him in front of your dog every time the dog barks (!), slowly the dog will stop
barking at your friend. This is used in relaxation therapy for anxiety and systematic
desensitisation. 
Case 4 describes shaping, a form of operant conditioning in which a desirable behaviour
pattern is learnt by the successive reinforcement of behaviours closer to the desired one
(successive approximation). This is used in training animals in circus where the dog is
reinforced when it runs towards the circle, then when it jumps near the circle, then when it
jumps through the circle and at last when it jumps through the circle of fire.
The correct answer is: A 6­year­old girl usually plays with her dog without fear. One day the
dog accidentally bites the child. She then becomes fearful of approaching the dog – Classical
conditioning, A 7­year­old child is told to complete his homework before he can play with his
friends – Premack's principle, A 15­year­old boy with spider phobias is put into a relaxed state
by suggestion and soothing music. While he is in this relaxed state, he is shown the
photograph of a spider – Reciprocal inhibition, A 4­year­old child is learning to write. She is
appreciated by her parents for holding the pen properly, then for scribbling something on the
notepad, later writing something that closely resembles a letter and finally writes the letter
clearly – Shaping

Question 22 HiY Psychology EMI022
Not answered Theories in Psychology
For each of the following statements, choose the most appropriate theories from the list
Marked out of 3.00
provided.
Flag question

Broadbent's filter theory of attention
This theory states that people are Shiffrin and Schneider's theory
selfish while deciding on Cannon­Bard theory
relationships; preference is for
Schema theory
relationships that offer the 'greatest
gains' with 'least expense'.
Cocktail party phenomenon
Equity theory
Cognitive appraisal theory
Group think
Contingency theory
Contingency theory
Social exchange theory
James­Lange theory
Schema theory
Social exchange theory
Shiffrin and Schneider's theory
This theory introduces the concept of
'fairness' with approximately equal Broadbent's filter theory of attention
gains in relationships between both James­Lange theory
individuals. Group think
Cannon­Bard theory
Cocktail party phenomenon
Equity theory
Cognitive appraisal theory
Cocktail party phenomenon
The letter "O" will automatically grab Contingency theory
attention in a page full of X's. This Broadbent's filter theory of attention
suggests that attention involves Group think
automatic processes. James­Lange theory
Cognitive appraisal theory
Schema theory
Social exchange theory
Equity theory
Shiffrin and Schneider's theory

1390
Check

Explanation: 
Social exchange theory claims that much, if not most, of our social behaviour is influenced by
the expectation that our action towards others will be reciprocated in some way. Relationships
are based on a mutual exchange of benefits. 
Equity theory suggests that people try to maintain a balance between what they put into a
relationship and what they get out of it. Most people would expect to receive rewards from a
relationship, which are roughly proportional to the contributions they put into it. 
Shiffrin and Schneider's theory of attention summarized evidence pointing towards two types
of processing ­ controlled and automatic processing. Automatic processing involves attention
to patterns and deviations. Controlled processing is employed when evaluating a situation in
more detail.
The correct answer is: This theory states that people are selfish while deciding on
relationships; preference is for relationships that offer the 'greatest gains' with 'least expense'.
– Social exchange theory, This theory introduces the concept of 'fairness' with approximately
equal gains in relationships between both individuals. – Equity theory, The letter "O" will
automatically grab attention in a page full of X's. This suggests that attention involves
automatic processes. – Shiffrin and Schneider's theory

Question 23 HiY Psychology EMI023
Not answered Personality tests
For each of the following tests given below, choose their correct names from the list provided.
Marked out of 4.00

Flag question
Wisconsin card sorting test
This test is a way of measuring Minnesota multiphasic personality inventory
personality along the dimensions of National adult reading test
extroversion­introversion and Raven's progressive matrices
neuroticism­stability Paired associates test
Rorschach ink blot test
Eysenck Personality Inventory
Thematic apperception test
Eysenck Personality Inventory
Rey­Osterreith test
Minnesota multiphasic personality inventory
It is one of the objective personality Trail making test
Rey­Osterreith test
tests that includes hypochondriasis Stroop test
Raven's progressive matrices
as a personality factor
Wisconsin card sorting test
National adult reading test
Thematic apperception test
Rorschach ink blot test
National adult reading test
Paired associates test
Trail making test
A projective personality test with Stroop test
Minnesota multiphasic personality inventory
pictures of ambiguous persons and Trail making test
Rey­Osterreith test
scenes to make up a story
Stroop test
Eysenck Personality Inventory
Paired associates test
Raven's progressive matrices
Wisconsin card sorting test
Thematic apperception test

1391
This test is useful to assess
National adult reading test
premorbid intelligence.
Stroop test
Paired associates test
Eysenck Personality Inventory
Rey­Osterreith test
Rorschach ink blot test
Wisconsin card sorting test
Raven's progressive matrices
Check Minnesota multiphasic personality inventory
Trail making test
Thematic apperception test
Explanation: 
Eysenck's approach: Eysenck used second order analysis (orthogonal factor analysis) that
identified small number of powerful independent factors relevant to measuring personality.
This method yielded three­dimensional traits. These are neuroticism (vs. stability),
psychoticism and extraversion (vs. introversion). Eysenck's personality questionnaire contains
a lie scale. Eysenck maintained that no fundamental discontinuity exists between normal and
abnormal personalities (dimensional). 
Minnesota multiphasic personality inventory MMPI is a popular inventory for measuring
personality. It has ten scales with clinical labels including hypochondriasis and hypomania. It
is NOT a projective test. 
Rorschach Inkblot Test and Murray's Thematic Apperception Test are projective tests; so are
sentence completion and Draw a Person tests. Projective tests use ambiguous stimuli, and
the patient is expected to generate narratives based on them; rating scales may or may not be
used in conjunction to rate such measures.
NART ­ National Adult Reading Test: Taps on previous word knowledge before becoming ill ­
hence it can be used to estimate premorbid IQ.
The correct answer is: This test is a way of measuring personality along the dimensions of
extroversion­introversion and neuroticism­stability – Eysenck Personality Inventory, It is one of
the objective personality tests that includes hypochondriasis as a personality factor –
Minnesota multiphasic personality inventory, A projective personality test with pictures of
ambiguous persons and scenes to make up a story – Thematic apperception test, This test is
useful to assess premorbid intelligence. – National adult reading test

Question 24 HiY Psychology EMI024
Not answered Memory
Identify the type of memory which is being tested.
Marked out of 4.00

Flag question
Semantic memory
Free recall memory
Mr.X is shown a list of words and was Repressed memory
asked to remember as many words
False memory
as possible
Infantile memory
Flashbulb memory
Cued memory
Procedural memory
Recognition memory
Mr.Y is shown a list of words and
asked to remember a supermarket
grocery item in the list that starts with

1392
the letter 'B'.
Infantile memory
Procedural memory
Semantic memory
Recognition memory
Free recall memory
Repressed memory
Cued memory
False memory
Infantile memory
Mr. Z is shown a list of words. He is
Flashbulb memory
Repressed memory
then shown an another list of words Semantic memory
and asked which of those appeared Flashbulb memory
on the first list Free recall memory
False memory
Cued memory
Recognition memory
Procedural memory
Procedural memory
Free recall memory
Mrs. Q has early dementia. She is still Recognition memory
able to drive his car False memory
Repressed memory
Cued memory
Infantile memory
Semantic memory
Check
Flashbulb memory

Explanation: 
In free recall memory paradigm, subjects are required to study a list of items that are later
recalled in any order (e.g. the word recall test in MMSE). 
Cued recall is similar to free recall except that subjects are given hints (cues) at the time of
recall. 
A very different way of testing memory involves merely showing subjects the previously learnt
words in a list format and asking them whether they recognize the items. To keep the subjects
honest, these target words are mixed in with distractors (or foils) that they have not seen. 
Remembering a specific swimming lesson is a form of explicit memory, while an improvement
in swimming that results from the lesson is a form of implicit procedural memory.
The correct answer is: Mr.X is shown a list of words and was asked to remember as many
words as possible – Free recall memory, Mr.Y is shown a list of words and asked to remember
a supermarket grocery item in the list that starts with the letter 'B'. – Cued memory, Mr. Z is
shown a list of words. He is then shown an another list of words and asked which of those
appeared on the first list – Recognition memory, Mrs. Q has early dementia. She is still able to
drive his car – Procedural memory

Question 25 HiY Psychology EMI025
Not answered Medical Ethics ­ Principles
Identify one principle for each of the following.
Marked out of 5.00

Flag question

1393
During the second world war, the
Autonomy
Nazi medico­military programme
Justice
prisoners were shot in order to study
gunshot wounds
Non­maleficence
Paternalism
Conscience
Empiricism
Balanced decision
Beneficence
Empiricism
You decide that a patient who has
Utilitarianism
Paternalism
previously developed renal
impairment on Lithium Carbonate Non­maleficence
should not be treated with it for a Beneficence
second time due to the risk of Utilitarianism
developing the condition again Autonomy
Justice
Conscience
Balanced decision
A general manager of a regional
Balanced decision
Justice
psychiatric service has had her
budget significantly reduced. She has Autonomy
to choose to cut either a day hospital Beneficence
service or an early intervention Paternalism
service Utilitarianism
Empiricism
A woman has a probable bone Non­maleficence
Balanced decision
cancer discovered on a routine bone Conscience
Utilitarianism
scan to detect osteoporosis. She is Justice
not informed about the suspected
Empiricism
malignant tumour but is followed up
with serial bone scans every few
Non­maleficence
months to see if it is growing Autonomy
Beneficence
Conscience
Autonomy
A 76­year­old man was diagnosed
Paternalism
Justice
with Parkinson's disease dementia.
He is due to be treated with Non­maleficence
antidementia drugs. He has been Utilitarianism
offered a choice to choose oral Empiricism
tablets or transdermal patch. Beneficence
Paternalism
Balanced decision
Check Conscience

Explanation: 
The unacceptable Nazi medical experiments can be considered as an instance of extreme
utilitarianism. In Nazi medico­military programme, prisoners were shot to study gunshot
wounds, the consequentialist argument being the greater good for number of people in the
future as a result of these experiments.
Non­maleficence is often the reason for avoid rechallenging with drugs causing adverse
effects. 
Justice is the primary ethical consideration in resource allocation; it is concerned with fairness
or equity in distributing goods to people. Justice means ensuring that as far as possible all
sections of the community get their 'fair share'of those resources. 

1394
As highlighted by Sullivan et al., "In certain terminal illnesses, questions often arise
concerning how much truth to tell. When, if ever, is a physician justified in withholding
information? Can too much information be harmful? These conflicts between benevolent
paternalism on the part of the physician and an increasing interest on the part of the patient to
preserve autonomy are not easily resolved". 
Offering choices among medications is a way of respecting patient's autonomy. 
Ref: Sullivan et al., Truth­telling and patient diagnoses. Retrieved from
http://jme.bmj.com/content/27/3/192.full
The correct answer is: During the second world war, the Nazi medico­military programme
prisoners were shot in order to study gunshot wounds – Utilitarianism, You decide that a
patient who has previously developed renal impairment on Lithium Carbonate should not be
treated with it for a second time due to the risk of developing the condition again – Non­
maleficence, A general manager of a regional psychiatric service has had her budget
significantly reduced. She has to choose to cut either a day hospital service or an early
intervention service – Justice, A woman has a probable bone cancer discovered on a routine
bone scan to detect osteoporosis. She is not informed about the suspected malignant tumour
but is followed up with serial bone scans every few months to see if it is growing –
Paternalism, A 76­year­old man was diagnosed with Parkinson's disease dementia. He is due
to be treated with antidementia drugs. He has been offered a choice to choose oral tablets or
transdermal patch. – Autonomy

Question 26 HiY Psychology EMI026
Not answered Attention
Choose the type of attentional model involved in each of the following.
Marked out of 4.00

Flag question
Divided attention
Sustained attention
This requires conscious attention and Selective attention
is heavily demanding Automatic processing
Controlled processing
Alternating attention
Stroop effect
Focused attention
Automatic processing
Focused attention
Ability to focus on two different tasks Divided attention
at the same time Sustained attention
Selective attention
Controlled processing
Stroop effect
Alternating attention
Stroop effect
Divided attention
Difficulty in overriding automatic
processing with conscious visual
Alternating attention
control Selective attention
Focused attention
Automatic processing
Sustained attention
Controlled processing

1395
Ability to maintain a constant Selective attention
behavioural response during
Divided attention
continuous and repetitive activity
Alternating attention
Focused attention
Sustained attention
Stroop effect
Controlled processing
Automatic processing
Check

Explanation: 
Controlled processing requires conscious attention, places heavy demands, and is capacity
limited. 
Divided attention is the highest level of attention that is much more difficult to achieve within
the same (e.g. visual) compared to multiple modalities (visual and auditory). This effect is
seen during Stroop colour word test. The test is based on the observation that individuals can
read words faster through automatic processing, than when they are asked to identify and
name colours of the presented words. It requires two strategies; selective attention to the
primary stimulus (colour) and inhibition of the predominant tendency to name the word or the
distracter stimulus. It is thought to measure interference control and requires the inhibition of
an over­learned automatic response. (Ref: Thambirajah MS. Psychological Basis of
Psychiatry (2005); pg 145). 
Sustained attention is the ability to maintain a consistent behavioral response during
continuous and repetitive activity. It is also called vigilance or concentration.
The correct answer is: This requires conscious attention and is heavily demanding –
Controlled processing, Ability to focus on two different tasks at the same time – Divided
attention, Difficulty in overriding automatic processing with conscious visual control – Stroop
effect, Ability to maintain a constant behavioural response during continuous and repetitive
activity – Sustained attention

Question 27 HiY Psychology EMI027
Not answered Social Psychiatry
Identify the correct terms used in social psychiatry to describe following situations.
Marked out of 5.00

Flag question
Conformity
Overt aggression
A man standing alone in a lonely Bystander intervention
road helped another person in
Obedience
trouble
Social facilitation
Risky shift phenomenon
Social loafing
A man stole a bottle of water from a Associative learning
Overt aggression
shop. He was sentenced for one Minority influence
month "so that all other offenders Obedience
know the consequences of shop­ Minority influence
lifting." Bystander intervention
Social facilitation
Risky shift phenomenon
Associative learning
Conformity

1396
Obedience
An affable person becomes violent Social facilitation
when he participates in a rally along Bystander intervention
with a number of charged­up Risky shift phenomenon
individuals
Conformity
Overt aggression
Social loafing
Minority influence
During a committee meeting, some of Obedience
Associative learning
Social loafing
the employees did not make much of
an effort and had few suggestions. Social facilitation
These employees had made a much Conformity
greater effort on other tasks in which Associative learning
they worked alone. Minority influence
Risky shift phenomenon
Bystander intervention
Overt aggression
A politician is giving a talk to Overt aggression
Social loafing
persuade the public regarding his Associative learning
views about economic repression
Social facilitation
and the need for more borrowing to
improve it.
Bystander intervention
Obedience
Conformity
Risky shift phenomenon
Check
Minority influence

Explanation: 
When alone, individuals often intervene if another person is in need of help: this is called
bystander intervention. But intervention becomes less likely to an extent that no single person
will intervene from a crowd or group of observers when someone is in need of help. This is
called bystander apathy. 
Obedience refers to conditions where the individual is explicitly asked to do a task, and this
instruction comes from an authority. 
Conformity is a process where no explicit requirement is made to do a certain task. But peer
influence and a need for acceptance pushes one to carry out the task. 
Social loafing is also called Ringelmann's effect. It is seen in games such as tug­of­war and
clapping hands after a performance. Larger a group is, lesser the individual performance as
one thinks the others will do the job and bring the results. Minority influence usually involves a
personal shift in private opinion and is also a central component of identity politics.
Excerpted from Social Loafing, Including Definition and Examples. (n.d.). Retrieved from
http://www.psychologyandsociety.com/socialloafing.html
The correct answer is: A man standing alone in a lonely road helped another person in
trouble – Bystander intervention, A man stole a bottle of water from a shop. He was sentenced
for one month "so that all other offenders know the consequences of shop­lifting." –
Obedience, An affable person becomes violent when he participates in a rally along with a
number of charged­up individuals – Conformity, During a committee meeting, some of the
employees did not make much of an effort and had few suggestions. These employees had
made a much greater effort on other tasks in which they worked alone. – Social loafing, A
politician is giving a talk to persuade the public regarding his views about economic
repression and the need for more borrowing to improve it. – Minority influence

1397
Question 28 HiY Psychology EMI028
Not answered Societal Influence
Match each of the proponents below with the concept proposed from the given list.
Marked out of 3.00

Flag question
Biopsychosocial model
Illness
Illness behaviour
Goffman
Moral Insanity
Primary Deviance
Role stripping
Impairment
Institutional Neurosis
Impairment
Handicap
Illness behaviour
Sick role
Abnormal illness behaviour
Mechanic Abnormal illness behaviour
Moral Insanity
Institutional Neurosis
Sick role
Primary Deviance
Handicap
Illness
Biopsychosocial model
Impairment
Role stripping
Biopsychosocial model
Parsons Illness
Handicap
Institutional Neurosis
Moral Insanity
Abnormal illness behaviour
Role stripping
Check Sick role
Illness behaviour
Primary Deviance
Explanation: In his book 'Asylums', Goffman illustrated the process of stripping the inmates (of
an institution such as psychiatric hospital) of all their social roles and privileges in order to
subdue them into accepting the new demands made upon them. This was termed as Role
Stripping. 
David Mechanic, a pioneer in the study of social influence on illnesses, summarised the
complex causes that predispose to illness behaviors, "including biological predispositions, the
nature of symptomatology, learned patterns of response, attributional predispositions,
situational influences, and the organization and incentives characteristic of the health care
system and the availability of secondary benefits" (Mechanic, 1995).
The correct answer is: Goffman – Role stripping, Mechanic – Illness behaviour, Parsons –
Sick role

Question 29 HiY Psychology EMI029
Not answered Stigma
Match each of the descriptions below with the concepts from the given list
Marked out of 3.00

Flag question A 43­year­old woman applied for a
secretarial job after two years of
unemployment secondary to
depression. At the job interview, she

1398
was asked very detailed questions
Courtesy Stigma
about the Mental Health Act invoked
Status Frustration
during her hospitalization.
Negotiated order
Positive Discrimination
Directed stigma
Felt stigma
Enacted stigma
Stereotype
NIMBY stigma
NIMBY stigma
Courtesy Stigma
A hospitalized patient in a female
ward is allowed to go out on short Positive Discrimination
breaks daily if she returns a urine Directed stigma
sample for drug tests. Felt stigma
Stereotype
Status Frustration
Enacted stigma
Felt stigma
Negotiated order
Negotiated order
A highly disabling fear of Directed stigma
discrimination perceived by patients
Stereotype
and carers
Positive Discrimination
Status Frustration
Enacted stigma
Courtesy Stigma
Check
NIMBY stigma

Explanation: A discreditable state is one where a person has a stigmatised characteristic, but
this is not generally known. There is, therefore, a potential to be discredited or rejected if the
characteristic becomes apparent. Patients carrying a history of mental illnesses are said to be
in a discreditable state. In this state, there is a high degree of 'Felt Stigma' ­ the shame felt by
the patients secondary to the fear of discovery and subsequent discrimination. They reach a
discredited state if the stigmatised condition has become visible or known. Following this, they
may experience an overt social rejection (or discriminatory response) as a result being
discredited. This is called enacted stigma. Enacted stigma is the actual discrimination on the
basis of the illness (case 1). Scambler (1990) made the distinction between felt stigma and
enacted stigma. 
Strauss proposed that hospitals work in a 'Negotiated Order' where a give­and­take
interaction takes place within settings predefined by broader, and more formal norms or
expectations, in order to secure preferred ends. This is demonstrated in case 2.
The correct answer is: A 43­year­old woman applied for a secretarial job after two years of
unemployment secondary to depression. At the job interview, she was asked very detailed
questions about the Mental Health Act invoked during her hospitalization. – Enacted stigma, A
hospitalized patient in a female ward is allowed to go out on short breaks daily if she returns a
urine sample for drug tests. – Negotiated order, A highly disabling fear of discrimination
perceived by patients and carers – Felt stigma

Question 30 HiY Psychology EMI030
Not answered Effect of Culture
Match each of the descriptions below with the most appropriate effect of cultural processes
Marked out of 3.00
from the given list.
Flag question

1399
Pathophysiologic
A Malaysian man humiliated in public Pathodiscriminating
takes a weapon and kills people
Pathoplastic
indiscriminately to prove his
Pathologic
manhood.
Pathoelaborating
Pathofacilitating
Pathognomonic
A 23­year­old woman is prone to a Pathoreactive
Pathophysiologic
sudden onset of a transient Pathoselective
Pathoreactive
dissociative attacks induced by Pathogenetic
Pathoselective
startling. She is often provoked on
Pathodiscriminating
social occasions and acts like a
clown, providing social
Pathoplastic
entertainment. Pathognomonic
Pathoelaborating
Pathogenetic
Pathophysiologic
Pathologic
Pathoselective
Pathofacilitating
Pathognomonic
At an international health convention, Pathologic
a health minister form one of the Pathofacilitating
members nations argued that cultural Pathoelaborating
permission to consume alcohol freely Pathogenetic
may increase the prevalence of Pathoplastic
drinking problems. Pathodiscriminating
Pathoreactive

Check

Explanation: Culture substantially influences psychopathology (Tseng and Streltzer 1997).
The various ways that culture contributes to psychopathology have been termed
pathogenetic, pathoplastic, pathoelaborating, pathofacilitating, pathodiscriminating, and
pathoreactive effects (Tseng 2001). Tseng argued that culture could have pathogenetic (i.e.
culture is a direct causative factor in forming psychopathology) or pathoselective effect (i.e.
culturally influenced behavioural selection resulting in the manifestation of specific
psychopathologies e.g. family suicides in Japan or amok in Malaysia [case 1]) in many
culture­bound disorders. It may be pathoplastic too (modelling of manifestations of
psychopathology). The content of delusions, auditory hallucinations, obsessions, or phobias
is subject to the environmental context in which the pathology is manifested. Culture may also
be pathoelaborating (behaviour patterns get exaggerated to the extreme). This is well
illustrated by the unique mental phenomenon of Latah (case 2), which is mainly observed in
Malaysia, where the universal dissociative reaction to stress becomes exaggerated to the
extreme through cultural reinforcement (Simon, 1996). Culture can also be either
pathofacilitative (some conditions are more common in some cultures e.g. alcohol abuse) or
pathoreactive (culture influences people's reactions to distress and illnesses).
The correct answer is: A Malaysian man humiliated in public takes a weapon and kills people
indiscriminately to prove his manhood. – Pathoselective, A 23­year­old woman is prone to a
sudden onset of a transient dissociative attacks induced by startling. She is often provoked on
social occasions and acts like a clown, providing social entertainment. – Pathoelaborating, At
an international health convention, a health minister form one of the members nations argued

1400
that cultural permission to consume alcohol freely may increase the prevalence of drinking
problems. – Pathofacilitating

Question 31 HiY Psychology EMI031
Not answered Medical Ethics and Principles
Identify one suitable principle applicable to each of the following description.
Marked out of 5.00

Flag question
Beneficence
During the second world war, the Justice
Nazi medico­military programme Autonomy
prisoners were shot in order to study Empiricism
gunshot wounds Balanced decision
Conscience
Utilitarianism
Paternalism
Paternalism
You decide that a patient who has
Non­maleficence
Conscience
previously developed renal
impairment on Lithium carbonate Justice
should not be treated with it for a Autonomy
second time due to the risk of Empiricism
developing the condition again Balanced decision
Beneficence
Utilitarianism
Balanced decision
A general manager of a regional
Non­maleficence
Non­maleficence
psychiatric service has had her
budget significantly reduced. She has Paternalism
to choose to cut either a day hospital Empiricism
service or an early intervention Conscience
service Beneficence
Justice
A woman has a probable bone Utilitarianism
Utilitarianism
cancer discovered on a routine bone Autonomy
Justice
scan to detect osteoporosis. She is Empiricism
not informed about the suspected
Balanced decision
malignant tumour, but is followed up
with serial bone scans every few
Paternalism
months to see if it is growing Beneficence
Conscience
Non­maleficence
Justice
A 76­year­old man was diagnosed
Autonomy
Beneficence
with Parkinson's disease dementia.
He is due to be treated with Non­maleficence
antidementia drugs. He has been Empiricism
offered a choice to choose oral Utilitarianism
tablets or transdermal patch. Paternalism
Conscience
Autonomy
Check Balanced decision

Explanation: The unacceptable Nazi medical experiments can be considered as an instance
of extreme utilitarianism. In Nazi medico­military programme, prisoners were shot to study
gunshot wounds, the consequentialist argument being the greater good for more number of

1401
people in the future as a result of these experiments. 
Non­maleficience is often the reason for avoid re­challenging with drugs causing adverse
effects.
Justice is the primary ethical consideration in the area of health care resource allocation since
justice is concerned with fairness or equity in distributing goods to people. Justice means
ensuring that as far as possible all sections of the community get their 'fair share' of those
resources. 
In certain terminal illnesses, questions often arise concerning how much truth to tell. When, if
ever, is a physician justified in withholding information? Can too much information be
harmful? These conflicts between benevolent paternalism on the part of the physician and an
increasing interest on the part of the patient to preserve autonomy are not easily resolved. 
Offering choices among medications is a way of respecting patient's autonomy.
The correct answer is: During the second world war, the Nazi medico­military programme
prisoners were shot in order to study gunshot wounds – Utilitarianism, You decide that a
patient who has previously developed renal impairment on Lithium carbonate should not be
treated with it for a second time due to the risk of developing the condition again – Non­
maleficence, A general manager of a regional psychiatric service has had her budget
significantly reduced. She has to choose to cut either a day hospital service or an early
intervention service – Justice, A woman has a probable bone cancer discovered on a routine
bone scan to detect osteoporosis. She is not informed about the suspected malignant tumour,
but is followed up with serial bone scans every few months to see if it is growing –
Paternalism, A 76­year­old man was diagnosed with Parkinson's disease dementia. He is due
to be treated with antidementia drugs. He has been offered a choice to choose oral tablets or
transdermal patch. – Autonomy

Question 32 HiY Psychology EMI032
Not answered Sternberg's Theory of Intelligence
Identify the correct cognitive faculties to match the three levels of abilities described by
Marked out of 5.00
Sternberg's Triarchic Theory of Intelligence from the given list.
Flag question

Applying knowledge
Set­shifting
Consciousness
Analytical Abilities (Choose TWO)
Attention
Evaluation
Comparison
Aspiration
Motivation
Discovery
Discovery
Comparison
Attention
Creative Abilities (Choose ONE)
Consciousness
Set­shifting
Motivation
Aspiration
Applying knowledge
Evaluation

Practical Abilites (Choose ONE)

1402
Consciousness
Set­shifting
Motivation
Applying knowledge
Comparison
Discovery
Evaluation
Aspiration
Check Attention

Triarchic theory of intelligence: Sternberg's Triarchic Theory of (Successful) Intelligence
contends that intelligent behaviour arises from a balance between analytical, creative and
practical abilities, and that these abilities function collectively to allow individuals to achieve
success within particular sociocultural contexts (Sternberg, 1999). Analytical abilities enable
the individual to evaluate, analyze, compare and contrast information. Creative abilities
generate invention, discovery, and other creative endeavours. Practical abilities tie everything
together by allowing individuals to apply what they have learned in the appropriate setting.
The correct answer is: Analytical Abilities (Choose TWO) – Comparison, Evaluation, Creative
Abilities (Choose ONE) – Discovery, Practical Abilites (Choose ONE) – Applying knowledge

Finish review

1403
 Home HiYield Paper A(1) Assessment

HiYield Paper A(1)

Started on Saturday, 27 June 2015, 2:43 PM
State Finished
Completed on Saturday, 27 June 2015, 2:43 PM
Time taken 12 secs
Marks 0.00/228.00
Grade 0.00 out of 100.00

Question 1 HiY Assessment EMI001
Not answered Self harm and defence mechanisms
A 22­year­old woman was admitted to the psychiatric ward with history of depression and
Marked out of 5.00
recurrent attempts to harm herself using razors. She is complaining of chronic feelings of
Flag question emptiness and constant fear of abandonment. She has no friends and is not in any
Suppression
relationship. From the list provided, match the most likely defence mechanisms being
Splitting
described below. Each option can be used once, more than once or not at all.
Denial
Idealisation
She tells that the consultant who Turning against the self
wants her to stay on the ward is an Denigration
excellent professional, but the nurses Isolation
are being irrational and insist that she Projection
should be discharged. (Choose Regression
THREE)
Projective identification

Idealisation
Turning against the self
Her arms and legs show superficial Denial
lacerations of self­cutting (Choose Suppression
ONE) Splitting
Regression
Isolation
Idealisation
Projective identification
Splitting
Denigration
Regression
Projection
Denial
She has had psychotherapy in the
past. The trainee therapist started
Projection
feeling depressed, helpless and went Isolation
on protracted medical leave after Denigration
seeing her for a few sessions. Projective identification
(Choose ONE) Suppression
Turning against the self

Check

1404
The diagnosis is borderline personality disorder. The defence mechanisms used in borderline
PD patients are splitting, idealisation and denigration. Splitting is often seen in patients with
borderline personality disorder. Here qualities of an object or person are split into black and
white i.e. either good or bad without any middle ground or grey area in between. 
Idealisation and denigration: This is often accompanied by splitting in those with borderline
traits. Here an object is either glorified, and supremacy is ascribed (idealised, omnipotence
ascribed) or considered very negatively and cursed! (Denigration). 
The defence mechanism behind the features described in question 2 is 'Turning against the
self' which is an unconscious deflection of hostility towards another person onto oneself,
resulting in lowered self­esteem, self­criticism and at times, injury to self. It is seen in patients
with severe depression, deliberate self­harm and suicide. 
Projective identification occurs when an aspect of self is projected onto someone else. The
projector influences the recipient to identify with what has been projected and projector
herself now believes that the aspect originated from the reactor. This process may result in the
recipient behaving in a manner similar to the projector. Here the therapist is the recipient, and
the patient is the projector. The feelings of depression have now been projected onto the
therapist.
The correct answer is: She tells that the consultant who wants her to stay on the ward is an
excellent professional, but the nurses are being irrational and insist that she should be
discharged. (Choose THREE)
– Splitting, Denigration, Idealisation, Her arms and legs show superficial lacerations of self­
cutting (Choose ONE) – Turning against the self, She has had psychotherapy in the past. The
trainee therapist started feeling depressed, helpless and went on protracted medical leave
after seeing her for a few sessions. (Choose ONE) – Projective identification

Question 2 HiY Assessment EMI002
Not answered Disorders and Freudian defences 
Match the following condition with the most likely defence mechanisms provided in the list.
Marked out of 8.00
Each option can be used once, more than once or not at all.
Flag question

Projection
Denial
Introjection
Obsessive compulsive disorder 
Turning against the self
Idealization
Isolation
Repression
Acting out
Repression
Dissociation
Projection
Denial
Introjection
Dissociation
Acting out
Impulse control disorders  Idealization
Turning against the self
Isolation

1405
Dissociation
Projection
Introjection
Grief
Repression
Idealization
Turning against the self
Isolation
Acting out
Introjection
Denial
Repression
Projection
Suicide
Acting out
Turning against the self
Dissociation
Idealization
Isolation
Denial
Denial
Acting out
Dissociation
Fugue
Isolation
Repression
Turning against the self
Introjection
Projection
Check
Idealization 

Isolation: Splitting or separating an idea from the affect that accompanies it but is repressed.
Splitting is the defence mechanism used in patients with obsessional thoughts. Isolation of
affect is also seen in grief reaction, and the patient can discuss a major loss or stressful events
without the associated disturbing emotions, with the passage of time. 
Patients with OCD exhibit isolation (of affect), reaction formation and undoing. Reaction
formation involves transforming an unacceptable impulse into its exact opposite. If these
defenses are frequently used at any early stage of ego development, it can become a
permanent character trait, contributing to an obsessional personality. 
Acting out refers to an expression of an unconscious wish or impulse through action to avoid
being conscious of an accompanying affect; it is seen in impulse control disorders. 
Introjection refers to unconscious internalization of qualities of an object or person. It is seen
in depression, grief reaction and is featured by identification with the aggressor (e.g. in victims
kidnapped by terrorists, also called as Stockholm syndrome). 
Turning against the self is unconscious deflection of hostility towards another person onto
oneself resulting in lowered self­esteem, self­criticism and at times injury to self. It is seen in
patients with severe depression, deliberate self­harm and suicide. 
Both fugue states and conversion reactions are seen as manifestations of dissociation.
Dissociation also accompanies a counterphobic behaviour; here a person with fear of heights
takes up parachute diving and experiences dissociation during the act.
The correct answer is: Obsessive compulsive disorder 
– Isolation, Impulse control disorders 
– Acting out, Grief
– Introjection, Suicide

1406
– Turning against the self, Fugue
– Dissociation

Question 3 HiY Assessment EMI003
Not answered Defence mechanisms in clinical practice
From the list given here, match the most likely defence mechanisms being described below
Marked out of 5.00

Flag question
Identification with the aggressor
A 45­year­old man was frustrated Splitting
caused by problems at work. He Isolation
became violent at home towards his Reaction formation
wife and children Sublimation
Denial
Regression
Displacement
Projection
Repression
Isolation
A 65­year­old woman has been
diagnosed with breast cancer. She is
Projection
Regression
refusing to accept that she has Identification with the aggressor
cancer Denial
Displacement
Sublimation
Splitting
Displacement
Repression
Projection
Reaction formation
Identification with the aggressor
Denial
A 25­year­old man tells 'I find Regression
gardening a good outlet for my pent­ Isolation
up aggression.' Repression
Sublimation
Splitting
Reaction formation

Splitting
Since the death of her long­term Projection
partner, a 45­year­old woman has Denial
been acting like a child. She is losing
Identification with the aggressor
her temper and is eating excessively
for comfort
Regression
Repression
Displacement
Reaction formation
Sublimation
Sublimation
Splitting
A 55­year old woman was talking
about her car accident when she lost Isolation
Regression
her partner, without any display of Reaction formation
emotions Denial
Displacement
Repression
Projection
Isolation

1407
Check

1. This scenario refers to a displacement­ transferring our feelings from their true target onto a
harmless substitute target. 
2. This scenario refers to Denial, which is failing/refusing to acknowledge/perceive some
aspect of reality. In this case, refusing to accept that they have a serious illness. 
3. Sublimation is a mature defence mechanism in which a socially positive substitute activity is
found for expressing some unacceptable impulse. 
4. Regression is reverting to behaviour characteristic of an earlier stage of development such
as losing your temper; comfort eating, sleeping more when depressed. 
5. Isolation is separating contradictory thoughts and feelings into logic­tight compartments like
the example above talking about some traumatic experience without any display of emotions
or even giggling about it.
The correct answer is: A 45­year­old man was frustrated caused by problems at work. He
became violent at home towards his wife and children – Displacement, A 65­year­old woman
has been diagnosed with breast cancer. She is refusing to accept that she has cancer –
Denial, A 25­year­old man tells 'I find gardening a good outlet for my pent­up aggression.' –
Sublimation, Since the death of her long­term partner, a 45­year­old woman has been acting
like a child. She is losing her temper and is eating excessively for comfort – Regression, A 55­
year old woman was talking about her car accident when she lost her partner, without any
display of emotions – Isolation

Question 4 HiY Assessment EMI004
Not answered Defence mechanisms in daily life
From the list above, match the most likely defence mechanisms being described below
Marked out of 5.00

Flag question
Projection
A 27­year old man made a voluntary Repression
decision to not to think about an Displacement
argument with his girlfriend he had Reaction formation
earlier when going for an interview Regression
Denial
Suppression
Sublimation
Denial
Identification
Reaction formation
A 45­year­old man hit his wife and Rationalisation
Regression
tells he is cruel 'for her own sake'. 'I
Repression
only did it because I love you.'
Rationalisation
Displacement
Identification
Suppression
Repression
A 16­year­old girl hated her Projection
Denial
classmate and disliked him. Sublimation
Suppression
However, she is very polite and even
Displacement
went out of her way to being nice to
him.
Regression
Rationalisation
Projection
Identification

1408
Displacement
Regression
Denial
A 24­year­old woman with Suppression
personality disorder tells her distress Projection
is the fault of others whom she Identification
blames and has never done anyone Repression
any harm. Reaction formation
Sublimation
Rationalisation

Denial
A 5­year­old boy started to behave Suppression
like his father and seemed to have Rationalisation
acquired the mannerisms and Repression
qualities of his dad. Displacement
Sublimation
Identification
Reaction formation
Check Regression
Projection

1. This is suppression, which is "consciously or semiconsciously postponing the attention to a
conscious impulse or conflict. Issues may be deliberately cut off, but they are not avoided.
Discomfort is acknowledged but minimised" (DSM­IV). 
2. The defence mechanism used here is rationalisation, which involves the use of rational
explanations in order to justify one's otherwise unacceptable attitudes or behaviours. Such
underlying motives are usually instinctually determined. It often involves finding excuses that
will justify unacceptable behaviours when self­esteem is threatened, often seen in teenagers
and those who abuse alcohol and drugs. 
3. This scenario describes reaction formation which is being considerate/polite to someone
you strongly dislike and even going out of her way to be nice to them. Reaction formation
involves transforming an unacceptable impulse into its exact opposite. 
4. The defence mechanism used here is projection, which is displacing your unacceptable
feelings/characteristics onto someone else. 
5. This scenario refers to identification, which is incorporating another person into one's
personality and making them part of oneself
The correct answer is: A 27­year old man made a voluntary decision to not to think about an
argument with his girlfriend he had earlier when going for an interview – Suppression, A 45­
year­old man hit his wife and tells he is cruel 'for her own sake'. 'I only did it because I love
you.' – Rationalisation, A 16­year­old girl hated her classmate and disliked him. However, she
is very polite and even went out of her way to being nice to him. – Reaction formation, A 24­
year­old woman with personality disorder tells her distress is the fault of others whom she
blames and has never done anyone any harm. – Projection, A 5­year­old boy started to
behave like his father and seemed to have acquired the mannerisms and qualities of his dad.
– Identification

Question 5 HiY Assessment EMI005

1409
Not answered Defence mechanisms ­ definitions
For each of the definitions given below, identify the most likely defence mechanisms from the
Marked out of 4.00
list provided:
Flag question

Reaction formation
Unconsciously motivated acts which Repression
magically or symbolically counteract Acting out
unacceptable thoughts, impulses or Sublimation
acts. Projection
Identification with the aggressor
Undoing
Denial
Undoing
Isolation
Sublimation
Unconscious viewing of self or others Splitting
Reaction formation
as good or bad without considering
Repression
the whole range of qualities
Projection
Acting out
Isolation
Identification with the aggressor
Identification with the aggressor
Splitting
Unconscious gradual channelization Denial
of unacceptable infantile impulses
Denial
Reaction formation
into personally satisfying and socially Sublimation
valuable behaviour patterns Repression
Splitting
Projection
Acting out
Acting out
Separation of the idea of an Isolation
Undoing
unconscious impulse from its Undoing
Repression
appropriate affect thus allowing only
Projection
the idea and not the associated affect
Denial
to enter awareness
Identification with the aggressor
Isolation
Sublimation
Check
Reaction formation
Splitting

Sublimation: Achieving impulse gratification but only after altering a socially objectionable
impulse to a socially acceptable one. Sublimation allows instincts to be channelled, rather
than blocked. 
Isolation: Splitting or separating an idea from the affect that accompanies (the affect gets
repressed). Isolation is often seen in OCD. 
Splitting: It is seen most often in those with borderline personality. Here qualities of an object
or person are split into black and white i.e. either good or bad with no grey area in between. 
Undoing in psychoanalysis is a defence mechanism whereby "an action which carries out a
certain injunction is immediately succeeded by another action which stops or undoes the first
one even if it does not go quite so far as to carry out its opposite". Sigmund Freud introduced
in his famous case study of 'Rat Man', where he describes compulsive actions in 2 stages, of
which the second neutralises the first. The Rat Man initially hurt his feet on a stone lying in the
road. Fearing the safety of his loved one who may pass the same road in a carriage, he
removed the stone. Soon he realized the irrationality of his behaviour and replaced the stone

1410
back on the road. (Freud's Ego and the Mechanisms of Defence, 1937). 
Undoing ­ Oxford Reference,
http://www.oxfordreference.com/viewbydoi/10.1093/oi/authority.20110803110632745
(accessed March 31, 2015).
The correct answer is: Unconsciously motivated acts which magically or symbolically
counteract unacceptable thoughts, impulses or acts. – Undoing, Unconscious viewing of self
or others as good or bad without considering the whole range of qualities – Splitting,
Unconscious gradual channelization of unacceptable infantile impulses into personally
satisfying and socially valuable behaviour patterns – Sublimation, Separation of the idea of an
unconscious impulse from its appropriate affect thus allowing only the idea and not the
associated affect to enter awareness – Isolation

Question 6 HiY Assessment EMI006
Not answered Neurological signs
Match the following clinical signs correctly:
Marked out of 4.00

Flag question
Chvostek's sign
A 74­year­old man presents with Kayser­Fleischer ring
difficulty while turning on the bed, Myerson Sign
tremors even while at rest, stiffness of
Ash leaf macules
both forearm muscles and
Milkmaid's grip
depression.
Tuft of hair on midline
Shuffling gait
Frontal baldness
Milkmaid's grip
A 24­year­old man presents with
Argyll Robertson pupil
Ash leaf macules
bizarre behaviour, anxiety and
depression. He has a bird­like wing
Gower's sign
Shuffling gait
beating tremor. On examination, liver Chvostek's sign
functions are abnormal, and a Tuft of hair on midline
genetic disease is suspected. Kayser­Fleischer ring
Frontal baldness
A 4­year­old boy has poor motor Myerson Sign
development in spite of excellent
Shuffling gait
Gower's sign
Milkmaid's grip
cognitive development. He has quite
Argyll Robertson pupil
bulky muscles on inspection, and no Ash leaf macules
facial dysmorphism is noted. His Chvostek's sign
brother had suffered from a similar Tuft of hair on midline
problem before he died of Argyll Robertson pupil
cardiomyopathy at the age of 16. Gower's sign
Frontal baldness
Milkmaid's grip
Kayser­Fleischer ring
Frontal baldness
A 73­year old gentleman with resting Myerson Sign
Ash leaf macules
tremor, cog­ wheal rigidity continues
Tuft of hair on midline
to blink with repeated glabellar taps
Gower's sign
Argyll Robertson pupil
Kayser­Fleischer ring
Shuffling gait
Check
Chvostek's sign
Myerson Sign

1411
Shuffling gait: The patient exhibits features of Parkinson's disease with tremors and axial
rigidity leading to difficulty while turning on a bed. Patients have short­stepped shuffling gait.
The pace may vary with a tendency for the patient to accelerate i.e. progressively short
(festinating gait). 
Kayser­Fleischer (KF) ring is a brownish ring around the limbus of the cornea. It is best
demonstrated during an ophthalmologic slit lamp examination. This patient exhibits features
of Wilson's disease. KF rings (copper deposits on Descemet's membrane of the cornea) are
present in 95% of patients with neurological symptoms, in 50­60% of patients without
neurological symptoms and in only 10% of asymptomatic siblings. Personality disturbances,
mood abnormalities, and cognitive dysfunction can be seen in Wilson's disease. A wing­
beating tremor is another important feature of this condition. 
Gower sign is seen in severe myopathies. It occurs when the patient attempts to stand up from
the floor. Patients first sit up, then assume a quadruped position, and then climb up their legs
by using their arms to push themselves up. This is an important feature of Duchenne's
muscular dystrophy which is an X­linked recessive disorder. Due to muscle degeneration,
patients experience difficulty when walking and breathing, and may die eventually. The
incidence is 1 in 3,000. Only males are affected while females are carriers. 
Myerson sign: Patients with Parkinson disease and those experiencing bilateral frontal
lesions, will continue to blink with renewed glabellar taps. 
Ref: Yiu, E; Kornberg, A. "Duchenne muscular dystrophy". Neurology India 2008;56(3):236­47
The correct answer is: A 74­year­old man presents with difficulty while turning on the bed,
tremors even while at rest, stiffness of both forearm muscles and depression. – Shuffling gait,
A 24­year­old man presents with bizarre behaviour, anxiety and depression. He has a bird­
like wing beating tremor. On examination, liver functions are abnormal, and a genetic disease
is suspected. – Kayser­Fleischer ring, A 4­year­old boy has poor motor development in spite
of excellent cognitive development. He has quite bulky muscles on inspection, and no facial
dysmorphism is noted. His brother had suffered from a similar problem before he died of
cardiomyopathy at the age of 16. – Gower's sign, A 73­year old gentleman with resting tremor,
cog­ wheal rigidity continues to blink with repeated glabellar taps – Myerson Sign

Question 7 HiY Assessment EMI007
Not answered Physical signs
For each of the description given below, identify the physical examination findings from the list
Marked out of 6.00
provided.
Flag question

Sialorrhea
An 18­year­old girl hides food in Xanthelasma
cupboards, refuses to sit with others Lanugo hair
while eating and has lost Argyll Robertson Pupil
considerable weight over last six Lid lag
months. She is a high flyer at school
Mask like face
but recently her performance is
Goiter
declining She does not induce
vomiting but exercises for 3 hours a
Lemon stick appearance
day.  Piloerection
Russell sign
Constricted pupil
A 22­year­old woman who has a
previous history of eating disorder not
otherwise specified attends clinic with
a history of uncontrollable episodes
of overeating that culminates in

1412
purging and vomiting. She has Lemon stick appearance
maintained reasonable weight but
Goiter
feels she has lost control over her
eating again. 
Mask like face
Constricted pupil
Piloerection
Xanthelasma
Sialorrhea
Russell sign
A 43­year­old heroine user presents
Sialorrhea
Lanugo hair
to the A&E demanding a strong Lid lag
Lid lag
painkiller for stomach ache. He Lanugo hair
Argyll Robertson Pupil
threatens to bleed himself to death if Mask like face
he is not given a parenteral opioid Xanthelasma
analgesic. Goiter
Piloerection
Constricted pupil
Russell sign
Argyll Robertson Pupil
Piloerection
A 33­year­old man has been started
on clozapine three weeks ago but Lemon stick appearance
Sialorrhea
has developed pneumonia recently. Russell sign
Xanthelasma
His WBC count is normal. Lanugo hair
Goiter
Constricted pupil
Lid lag
Check Mask like face
Argyll Robertson Pupil
Lemon stick appearance
Lanugo hair is the growth of fine, downy hair on the face and body of those with anorexia or
starvation. It starts first on the face, later spreading to the back. The distribution is in areas
where there is typically very little hair growth. 
In question 2, the patient is presenting with features of bulimia nervosa. Russell's sign refers
to calluses on the back of the hand that result from chronic and recurrent self­induction of
vomiting in bulimics. Bilateral­and occasionally unilateral­parotid gland swelling has been
reported to occur in 10 to 15% of patients.
In question 3, the patient is presenting with features of opioid withdrawal, which is usually not
fatal in a healthy adult. The withdrawal syndrome usually begins 6 to 8 hours after the last
dose of heroin, peaks in two days and reduces in a week usually. The features are dysphoric
mood, nausea or vomiting, muscle aches, lacrimation or rhinorrhea, pupillary dilation,
piloerection (gooseflesh), or sweating, diarrhea, yawning, fever and insomnia. Sialorrhea is a
common side effect of treatment with clozapine which induces hypersalivation (particularly at
night time). It seems to be chiefly problematic in the early stages of treatment and is probably
dose­related.
The correct answer is: An 18­year­old girl hides food in cupboards, refuses to sit with others
while eating and has lost considerable weight over last six months. She is a high flyer at
school but recently her performance is declining She does not induce vomiting but exercises
for 3 hours a day. 
– Lanugo hair, A 22­year­old woman who has a previous history of eating disorder not
otherwise specified attends clinic with a history of uncontrollable episodes of overeating that
culminates in purging and vomiting. She has maintained reasonable weight but feels she has
lost control over her eating again. 
– Russell sign, A 43­year­old heroine user presents to the A&E demanding a strong painkiller
for stomach ache. He threatens to bleed himself to death if he is not given a parenteral opioid
analgesic. – Piloerection, A 33­year­old man has been started on clozapine three weeks ago

1413
but has developed pneumonia recently. His WBC count is normal. – Sialorrhea

Question 8 HiY Assessment EMI008
Not answered General examination findings
For each of the following clinical situations, identify the most likely findings on physical
Marked out of 5.00
examination
Flag question

Lanugo hair
Dilated pupil
A 45­year­old homosexual drug Xanthelasma
abuser presenting to A&E with a
Jaundice
history of mania and weight loss
Sialorrhoea
Argyll­ Robertson pupil
Generalised lymph node enlargement
Checkerboard abdomen
Xanthelasma
Goitre
Dilated pupil
A 67­year­old woman with 20­year
Kayser­Fleischer ring
Argyll­ Robertson pupil
history of diabetes and depression Sialorrhoea
Goitre
Kayser­Fleischer ring
Jaundice
Generalised lymph node enlargement
Sialorrhoea
Lanugo hair
A 57­year­old gentleman with chronic Dilated pupil
Checkerboard abdomen
schizophrenia and is on maintenance Goitre
treatment with Olanzapine for 12 Checkerboard abdomen
years Generalised lymph node enlargement
Kayser­Fleischer ring
Xanthelasma
Jaundice
Generalised lymph node enlargement
A 45­year­old woman presenting Lanugo hair
Checkerboard abdomen
repeatedly to A& E with history of Argyll­ Robertson pupil
Xanthelasma
chronic abdominal pain and self­
Dilated pupil
inflicted injuries
Sialorrhoea
Jaundice
Lanugo hair
Goitre
Sialorrhoea
Kayser­Fleischer ring
Goitre
A 35­year­old gentleman with a
Argyll­ Robertson pupil
Jaundice
history of heavy alcohol use Argyll­ Robertson pupil
Generalised lymph node enlargement
Checkerboard abdomen
Dilated pupil
Kayser­Fleischer ring
Check Xanthelasma
Lanugo hair

This is a case of HIV illness, and generalised lymph node enlargement is a common finding. 

1414
The more common cause of Argyll­Robertson pupil is diabetes mellitus, and the other
common cause is neurosyphilis, which is rare to see these days. 
Xanthelasma is due to lipid accumulation, which is often related to long­term treatment with
antipsychotic medications such as Olanzapine. 
Multiple surgical scars are often seen in patients with factitious disorder leading to a checker
board abdomen. 
Heavy alcohol use can cause impairment of liver functions liver leading to Jaundice, hepatitis,
fatty liver and cirrhosis.
The correct answer is: A 45­year­old homosexual drug abuser presenting to A&E with a
history of mania and weight loss – Generalised lymph node enlargement, A 67­year­old
woman with 20­year history of diabetes and depression – Argyll­ Robertson pupil, A 57­year­
old gentleman with chronic schizophrenia and is on maintenance treatment with Olanzapine
for 12 years – Xanthelasma, A 45­year­old woman presenting repeatedly to A& E with history
of chronic abdominal pain and self­inflicted injuries – Checkerboard abdomen, A 35­year­old
gentleman with a history of heavy alcohol use – Jaundice

Question 9 HiY Assessment EMI009
Not answered Interview techniques
For each of the following statements given below, identify the interview technique used;
Marked out of 5.00

Flag question
Summation
Repetition
Open­ended question
What brings you to the clinic today?
Facilitation
Closed questions
Redirection
Clarification
Restatement
Open­ended question
Confrontation
Summation
'You said you are feeling low in mood Closed question
Closed question
ever since you can remember. When
Clarification
do you feel most depressed?'
Restatement
Closed questions
Confrontation
Facilitation
Confrontation
Repetition
Clarification
The patient said, 'I was having bad
dreams last night'. The doctor said Redirection
Restatement
'So, you are getting disturbed by the Repetition
dreams you have'. Redirection
Closed questions
Closed question
Facilitation
Open­ended question
Summation
'So from what you told so far, you are Restatement
anxious for the last three months and Open­ended question
Redirection
not sleeping well, and your job is at Closed questions
risk. Right'? Facilitation
Clarification
Confrontation
Closed question
Summation

1415
The patient said, 'I was having bad Facilitation
dreams last night'. The doctor said
Summation
'So, you were having bad dreams last
night'. Restatement
Open­ended question
Closed question
Repetition
Clarification
Confrontation
Check Closed questions
Redirection

Open­ended question s a non­directive technique used at the opening stage of a clinical
interview, to allow free narration. It is especially preferred when the patients are highly
suggestible; not very useful if interviewing an over­talkative or extremely poor historian. Open­
ended questions start with 'tell me', 'describe', etc. 
Clarification: Doctor tries to get details from patients about what the patient has already said. It
helps in avoiding misconceptions by the clinician. It also shows clinician's interest in knowing
more. 
Restatement: Similar to repetition but phrases rearranged. It helps patient to feel that the
doctor is listening actively. 
Summation: Brief summarisation of what the patient has said up to a point in the interview. It
helps patient to check if he has said what he intended to say and also helps the doctor to form
an idea of the narration so far. Repetition is repeating the exact words of the patient. It also
helps patient to feel that the doctor is listening actively. Ref: Adapted from Kay J & Tasman A.
Essentials of Psychiatry, 2nd edition, 2006. John Wiley & Sons, Ltd.
The correct answer is: What brings you to the clinic today? – Open­ended question, 'You said
you are feeling low in mood ever since you can remember. When do you feel most
depressed?' – Clarification, The patient said, 'I was having bad dreams last night'. The doctor
said 'So, you are getting disturbed by the dreams you have'. – Restatement, 'So from what you
told so far, you are anxious for the last three months and not sleeping well, and your job is at
risk. Right'? – Summation, The patient said, 'I was having bad dreams last night'. The doctor
said 'So, you were having bad dreams last night'. – Repetition

Question 10 HiY Assessment EMI010
Not answered Psychiatric Interview Skills
For each of the following statements, identify the interview technique used;
Marked out of 5.00

Flag question
Transition
You seem not to have gained any Redirection
weight in the last three months. Is it Closed question
possible that your eating has been Interpretation
poor again? Confrontation
Clarification
Summation
Self­revelation
Limit setting
I am going to interrupt you as there Question rephrasing
are few important things we need to
cover today

1416
Redirection
Limit setting
Self­revelation
Confrontation
Clarification
Summation
Interpretation
Transition
Question rephrasing
Question rephrasing
Closed question
Closed question
Self­revelation
Did you sleep well last night?
Redirection
Limit setting
Clarification
Confrontation
Transition
Closed question
Summation
Interpretation
Do you like Sir Cliff Richards? I was a Interpretation
Limit setting
mad fan while at school Question rephrasing
Self­revelation
Confrontation
Summation
Redirection
Question rephrasing
Transition
Limit setting
Clarification
Summation
Interpretation
You mentioned that your mother is a Self­revelation
receptionist. What about yourself?
Redirection
What job do you do?
Clarification
Confrontation
Transition
Closed question

Check

Confrontation is pointing out to a patient something to which the doctor thinks the patient is
missing or denying. It must be done in a respectful way. The aim is to help patients face a
difficult aspect rather than dismissing patients by showing a negative aspect. 
Limit setting is useful to manage time pressure, especially in garrulous patients. It should be
used cautiously, overuse may detach patient from the doctor. The motive is to use time
effectively. 
Closed questions: When, where, how many, which and what questions. Answers can only be
'yes or no', in most occasions. When clubbed with non­facilitative gestures, can be detrimental
to interview process. 
Stating a presumption followed by tags can be very directive. It is better avoided in early parts
of an interview as they can produce prescribed answers lacking in detail. Also, avoid in highly
suggestible patients. A good technique is to start with open questions and moving to closed
questions by the end of an interview. Closed questions are useful to rule out less likely
symptoms.

1417
Self­Revelation is limited, discreet self­disclosure by physicians. It helps the physician to feel
at­ease sometimes. Excessive self­revelation is a boundary violation. 
The transition is moving from one to another topic ­ this is a special skill and preferably must
be done as smoothly as possible to keep the patient interested. Smooth transitions ­ uses the
cue off something the patient just said. 
Ref: Adapted from Kay J & Tasman A. Essentials of Psychiatry, 2nd edition, 2006. John Wiley
& Sons, Ltd.
The correct answer is: You seem not to have gained any weight in the last three months. Is it
possible that your eating has been poor again? – Confrontation, I am going to interrupt you as
there are few important things we need to cover today – Limit setting, Did you sleep well last
night? – Closed question, Do you like Sir Cliff Richards? I was a mad fan while at school –
Self­revelation, You mentioned that your mother is a receptionist. What about yourself? What
job do you do? – Transition

Question 11 HiY Assessment EMI011
Not answered Investigating alcoholism
For the statements given below, identify from the given list the most appropriate test for
Marked out of 4.00
patients with alcohol abuse.
Flag question

Serotonin deficient transferring
Aspartate aminotransferase
This is a sensitive marker of alcohol Gamma glutamyl transpeptidase
abuse Alanine aminotransferase
Mean corpuscular volume
Carbohydrate­deficient transferrin
AUDIT
Uric acid
Serotonin deficient transferring
Triglycerides
Triglycerides
This test accurately predicts recent Mean corpuscular hemoglobin concentration
Aspartate aminotransferase
alcohol abuse and can be useful in
AUDIT
detecting relapse
Mean corpuscular hemoglobin concentration
Gamma glutamyl transpeptidase
Carbohydrate­deficient transferrin
Uric acid
Carbohydrate­deficient transferrin
Mean corpuscular volume
Alanine aminotransferase
This test is more specific and is less Alanine aminotransferase
Mean corpuscular hemoglobin concentration
useful in detecting relapse AUDIT
Gamma glutamyl transpeptidase
Triglycerides
Uric acid
Serotonin deficient transferring
Mean corpuscular volume
Aspartate aminotransferase
Aspartate aminotransferase
AUDIT
This is the best screening tool in Serotonin deficient transferring
primary care Gamma glutamyl transpeptidase
Carbohydrate­deficient transferrin
Triglycerides
Mean corpuscular volume
Alanine aminotransferase
Mean corpuscular hemoglobin concentration
Uric acid
1418
Check

Gamma­glutamyl transpeptidase (GGT) is a sensitive marker of alcohol abuse. Levels return
to normal 2­4 weeks after abstinence and, therefore, helpful in identifying those who have
relapsed after treatment. 
Mean corpuscular volume (MCV): Elevated MCV is more specific but less sensitive than GGT
for alcohol abuse. Due to the long lifespan of RBCs, MCVs may not return to normal for 2­4
months after abstinence and so is less useful in detecting relapse. 
Carbohydrate­deficient transferrin (CDT) accurately predicts recent alcohol abuse and can be
useful in detecting relapse. No single lab test can dependably diagnose alcohol abuse.
Carbohydrate­deficient transferrin is the single most specific and sensitive test for detecting
heavy alcohol use over last ten days. But due to a high degree of intersubject variability it is
best to compare CDT levels with patient's baseline. 
In primary care, AUDIT is the best screening tool (Alcohol Use Disorders Identification Test)
This is a 10­item questionnaire, covering quantity, frequency, inability to control drinking,
withdrawal relief, loss of memory, injury and concern by others. A score of 8 or higher
indicates drinking to a degree that is harmful or hazardous, whereas a score of 13 or more in
women and 15 or more in men is indicative of dependent drinking. It is a very useful and
widely used scale, recommended by WHO for primary care use.
The correct answer is: This is a sensitive marker of alcohol abuse – Gamma glutamyl
transpeptidase, This test accurately predicts recent alcohol abuse and can be useful in
detecting relapse – Carbohydrate­deficient transferrin, This test is more specific and is less
useful in detecting relapse – Mean corpuscular volume, This is the best screening tool in
primary care – AUDIT

Question 12 HiY Assessment EMI012
Not answered Appropriate use of rating scales
Identify the most appropriate rating scale for the following conditions.
Marked out of 6.00

Flag question
Barnes Akathisia Scale
Mrs.X suffers from OCD. She has Present state examination
recently delivered a baby. The health Beck depression inventory
visitor would like to screen her for CATEGO
depression Montgomery and Asberg depression rating scale
Edinburgh postnatal depression scale
Hospital anxiety and depression scale
Abnormal involuntary movements scale
Simpson­Angus scale
Hamilton depression rating scale
CATEGO
Simpson­Angus scale
General health questionnaire
General health questionnaire
Hospital anxiety and depression scale
Mr.Y is taking a high dose of Hamilton depression rating scale
haloperidol. You wish to monitor the Montgomery and Asberg depression rating scale
extent of parkinsonian side effects Edinburgh postnatal depression scale
Barnes Akathisia Scale
Abnormal involuntary movements scale
Present state examination
Beck depression inventory

1419
Mr. Z is a new trainee in psychiatry Barnes Akathisia Scale
who would like to interview a patient Edinburgh postnatal depression scale
in a standardized manner, but still Hamilton depression rating scale
retaining the features of a clinical General health questionnaire
examination Present state examination
Simpson­Angus scale
Montgomery and Asberg depression rating scale
Hospital anxiety and depression scale
Beck depression inventory
Beck depression inventory
Simpson­Angus scale
Mr. A is a GP registrar who would like CATEGO
Present state examination
to use a computer programme to Abnormal involuntary movements scale
General health questionnaire
generate psychiatric diagnosis
Abnormal involuntary movements scale
Hamilton depression rating scale
Edinburgh postnatal depression scale
Montgomery and Asberg depression rating scale
Edinburgh postnatal depression scale
Mrs. P has been diagnosed with Hospital anxiety and depression scale
Abnormal involuntary movements scale
postnatal depression and has been CATEGO
General health questionnaire
commenced on antidepressants. You Barnes Akathisia Scale
Hamilton depression rating scale
wish to monitor her response to
treatment (Choose TWO)
Hospital anxiety and depression scale
Beck depression inventory
Montgomery and Asberg depression rating scale
Present state examination
Check
Simpson­Angus scale
CATEGO
Barnes Akathisia Scale
Edinburgh Postnatal Depression Scale was specially developed for assessing postpartum
depression and relies much less on somatic questions than traditional depression scales.
Questions on the Edinburgh scale (10 items, can be self or clinician­rated) are framed within
the "past seven days", and the response format is frequency­based. Routine use of EPDS
during the postpartum period has been shown to increase the detection of postpartum
depression compared with usual care. 
The present state examination is clinician administered semi­structured clinical interview,
which provides a clinical diagnosis. 
CATEGO­ it is a computer programme for processing data from the schedules of clinical
assessment in neuropsychiatry.
Both Montgomery­Asberg depression rating scale and Hamilton rating scale for depression
are useful to measure the change in depression, especially during treatment trials. These
scales are helpful to monitor response to treatment.
The correct answer is: Mrs.X suffers from OCD. She has recently delivered a baby. The health
visitor would like to screen her for depression – Edinburgh postnatal depression scale, Mr.Y is
taking a high dose of haloperidol. You wish to monitor the extent of parkinsonian side effects –
Simpson­Angus scale, Mr. Z is a new trainee in psychiatry who would like to interview a
patient in a standardized manner, but still retaining the features of a clinical examination –
Present state examination, Mr. A is a GP registrar who would like to use a computer
programme to generate psychiatric diagnosis – CATEGO, Mrs. P has been diagnosed with
postnatal depression and has been commenced on antidepressants. You wish to monitor her
response to treatment (Choose TWO) – Montgomery and Asberg depression rating scale,
Hamilton depression rating scale

1420
Question 13 HiY Assessment EMI013
Not answered Rating scales ­ descriptions
For each description given below, select the relevant rating scale used for depressive
Marked out of 5.00
disorders from the list provided.
Flag question

Positive and negative symptoms scale (PANSS)
Hamilton depression scale (Ham D)
Self­rating, 20 items scale used in General health questionnaire (GHQ)
depression research, insensitive to
Clinical Global impressions scale (CGI)
change
Montgomery Asberg depression rating scale (MADRS)
Zung depression rating scale
Beck depression inventory (BDI)
Yale­Brown obsessive compulsive scales (YBOCS))
Montgomery Asberg depression rating scale (MADRS)
Observer­rated, ten items rated on a Hospital anxiety and depression scale (HADS)
Young mania rating scale
four­point scale, used in depression Young mania rating scale
Clinical Global impressions scale (CGI)
research and is useful to measure Brief psychiatric rating scale (BPRS)
Hospital anxiety and depression scale (HADS)
change in depression especially
during treatment trials
Brief psychiatric rating scale (BPRS)
Hamilton depression scale (Ham D)
General health questionnaire (GHQ)
Zung depression rating scale
Hamilton depression scale (Ham D)
Beck depression inventory (BDI)
Hospital anxiety and depression scale (HADS)
Observer­rated, 17 items, widely
used in depression research to Positive and negative symptoms scale (PANSS)
Clinical Global impressions scale (CGI)
assess the severity of depressive Yale­Brown obsessive compulsive scales (YBOCS))
Beck depression inventory (BDI)
syndrome Yale­Brown obsessive compulsive scales (YBOCS))
Young mania rating scale
Montgomery Asberg depression rating scale (MADRS)
Zung depression rating scale
Clinical Global impressions scale (CGI)
Brief psychiatric rating scale (BPRS)
General health questionnaire (GHQ)
Self­rated, 14 items, widely used in
Positive and negative symptoms scale (PANSS)
Montgomery Asberg depression rating scale (MADRS)
anxiety and depression General health questionnaire (GHQ)
Hospital anxiety and depression scale (HADS)
Positive and negative symptoms scale (PANSS)
Hamilton depression scale (Ham D)
Beck depression inventory (BDI)
Zung depression rating scale
General health questionnaire (GHQ)
A self­rated questionnaire containing Young mania rating scale
Clinical Global impressions scale (CGI)
21 statements with four possible Yale­Brown obsessive compulsive scales (YBOCS))
Zung depression rating scale
responses for each, one of which is Brief psychiatric rating scale (BPRS)
Positive and negative symptoms scale (PANSS)
chosen as best describing the
Young mania rating scale
symptom at the time.
Montgomery Asberg depression rating scale (MADRS)
Brief psychiatric rating scale (BPRS)
Hospital anxiety and depression scale (HADS)
Check Hamilton depression scale (Ham D)
Beck depression inventory (BDI)
Yale­Brown obsessive compulsive scales (YBOCS))
Beck Depression Inventory is a self­rated questionnaire containing 21 statements with four
possible responses for each. The maximum score is 63; BDI lacks discriminatory power
among the very severely ill and in addition, BDI includes more psychological than somatic
factors. 
Montgomery Asberg depression rating scale (MADRS) is a 10­item observer­rated scale,
which is very sensitive to measure the change in depression scores especially during

1421
treatment trials. 
Hamilton depression scale (HamD) is a 17­item rating scale for depressive illness. It's not a
diagnostic instrument and is used to measure changes (e.g. as a result of drug treatment). The
17 items are scored according to severity, producing the total score. 
Hospital Anxiety and Depression Scale, when used with a cut­off score of ≥8 , offers optimal
sensitivity and specificity to detect cases among unselected patients in primary care. Hospital
Anxiety and Depression Scale (HADS) and Patient Health Questionnaire (PHQ­9) are self
rated scales that serve as useful screening tools for non­psychiatric clinicians. 
Zung self­rating scale has 20 items, avoids imbalance towards psychological factors seen in
Beck's and is insensitive to change
The correct answer is: Self­rating, 20 items scale used in depression research, insensitive to
change – Zung depression rating scale, Observer­rated, ten items rated on a four­point scale,
used in depression research and is useful to measure change in depression especially during
treatment trials – Montgomery Asberg depression rating scale (MADRS), Observer­rated, 17
items, widely used in depression research to assess the severity of depressive syndrome –
Hamilton depression scale (Ham D), Self­rated, 14 items, widely used in anxiety and
depression – Hospital anxiety and depression scale (HADS), A self­rated questionnaire
containing 21 statements with four possible responses for each, one of which is chosen as
best describing the symptom at the time. – Beck depression inventory (BDI)

Question 14 HiY Assessment EMI014
Not answered Neuroimaging tests in psychiatry
Identify 2 common neuroimaging findings for the following psychiatric conditions
Marked out of 6.00

Flag question
Decreased metabolism in Prefrontal and Anterior cingulate cort
Loss of temporal lobe volume ­ especially hippocampus
A 22­year­old gentleman with Progressive increase in brain volume in first few years of diagno
persecutory delusions and 3rd
Ventricular enlargement
person auditory hallucinations
Loss of white matter in the dorsolateral pre­frontal cortex
Increased blood flow in parieto­temporal areas in SPECT
Increased frontal lobe volumes
Lower prefrontal metabolism
Poor activation of dorsolateral prefrontal cortex in executive tas
Poor activation of dorsolateral prefrontal cortex in executive tas
Increased frontal lobe volumes
Periventricular and deep white matter hyperintensities
A 29­year­old woman with low mood, Loss of temporal lobe volume ­ especially hippocampus
anhedonia and poor sleep. Periventricular and deep white matter hyperintensities
Lower prefrontal metabolism
Loss of white matter in the dorsolateral pre­frontal cortex
Progressive increase in brain volume in first few years of diagno
Ventricular enlargement
Poor activation of dorsolateral prefrontal cortex in executive tas
Increased blood flow in parieto­temporal areas in SPECT
Increased frontal lobe volumes
A 77­year­old man with history of Decreased metabolism in Prefrontal and Anterior cingulate cort
Lower prefrontal metabolism
short­term memory loss and poor
Progressive increase in brain volume in first few years of diagno
concentration
Ventricular enlargement
Periventricular and deep white matter hyperintensities
Loss of temporal lobe volume ­ especially hippocampus
Increased blood flow in parieto­temporal areas in SPECT
Check Decreased metabolism in Prefrontal and Anterior cingulate cort
Loss of white matter in the dorsolateral pre­frontal cortex

1422
The diagnosis for case 1 is most likely to be schizophrenia. Ventricular enlargement, Loss of
greymatter ­ especially around cingulate cortex, insula and medial temporal lobe, along with
fMRI evidence for poor dorsolateral pre­frontal cortex activation in executive tasks, decreased
N­acetyl aspartate in prefrontal cortex in Magnetic Resonance Spectroscopy are some of the
neuroimaging findings in schizophrenia.
The diagnosis for case 2 is likely to be depressive disorder. Neuroimaging in depressed
patients shows periventricular and deep WM hyperintensities, subcortical hyperintensities,
decreased frontal and basal ganglia volumes, decreased metabolism in the prefrontal cortex. 
The diagnosis for case 3 is likely to be Alzheimer's dementia. Neuroimaging in Alzheimers
disease reveals ventricular enlargement related to cortical atrophy, reduced medial temporal
lobe volume (especially hippocampal atrophy) and decreased parieto­temporal fMRI
activation and SPECT blood flow.
The correct answer is: A 22­year­old gentleman with persecutory delusions and 3rd person
auditory hallucinations – Ventricular enlargement, Poor activation of dorsolateral prefrontal
cortex in executive tasks during f­MRI, A 29­year­old woman with low mood, anhedonia and
poor sleep. – Periventricular and deep white matter hyperintensities, Decreased metabolism
in Prefrontal and Anterior cingulate cortex, A 77­year­old man with history of short­term
memory loss and poor concentration – Ventricular enlargement, Loss of temporal lobe volume
­ especially hippocampus

Question 15 HiY Assessment EMI015
Not answered EEG wave forms
For each of the following waveforms, identify the frequency range.
Marked out of 5.00

Flag question
Spike waves
Single sharp wave
8­13 Hz
Alpha
More than 13 Hz
Triangular waves
4­8 Hz
7­11 Hz
Less than 4 Hz
4­8 Hz
More than 13 Hz
8­13 Hz
Beta
Spike waves
Single sharp wave
7­11 Hz
Triangular waves
Less than 4 Hz
Spike waves
Single sharp wave
7­11 Hz
Theta
More than 13 Hz
4­8 Hz
8­13 Hz
Triangular waves
Less than 4 Hz

1423
Mu Less than 4 Hz
Single sharp wave
Triangular waves
More than 13 Hz
7­11 Hz
Spike waves
8­13 Hz
4­8 Hz
More than 13 Hz
Triangular waves
7­11 Hz
Delta
Single sharp wave
Spike waves
4­8 Hz
8­13 Hz
Less than 4 Hz
Check

Alpha waves have a frequency of 8­13 Hz. Alpha waves are the dominant brainwave
frequency when eyes are closed. Alpha disappears with anxiety, arousal, eye opening or
focused attention. Beta waves (more than 13 Hz) are seen at frontal, central position in the
normal waking EEG. Theta waves (4­8 Hz) are seen in wake EEG at frontotemporal area;
prominent in drowsy or sleep EEG. Excessive theta in awake EEG is a sign of pathology.
Delta waves (less than 4 Hz) are not seen in waking EEG. It is common in deeper stages of
sleep; the presence of focal/generalized delta in awake EEG is a sign of pathology. Mu waves
(7­11 Hz) occur over the motor cortex. Mu is related to motor activity, characterized by arch
like waves.
The correct answer is: Alpha – 8­13 Hz, Beta – More than 13 Hz, Theta – 4­8 Hz, Mu – 7­11
Hz, Delta – Less than 4 Hz

Question 16 HiY Assessment EMI016
Not answered Agnosias & Apraxias
Find one term each to describe the following presentations.
Marked out of 5.00

Flag question
Finger Agnosia
Agraphaesthesia
A 76­year­old gentleman is having
Hemisomatognosia
difficulty in recognising complex
Visuospatial Agnosia
pictures but can describe the parts.
Astereognosia
Ideomotor apraxia
Prosopagnosia
Anosognosia
Prosopagnosia
An 83­year­old woman having Simultagnosia
Astereognosia
difficulty in copying a wire cube on Dressing apraxia
Visuospatial Agnosia
extended cognitive examination Hemisomatognosia
Anosognosia
Simultagnosia
Agraphaesthesia
Dressing apraxia
Ideomotor apraxia

1424
A 73­year­old gentleman with a
Ideomotor apraxia
history of a major stroke suffered from
Anosognosia
left sided hemiplegia. However, he
denies having any physical problems Hemisomatognosia
including his gait disturbances. Astereognosia
Prosopagnosia
Agraphaesthesia
Dressing apraxia
Simultagnosia
Anosognosia
Visuospatial Agnosia
Prosopagnosia
Finger Agnosia
Visuospatial Agnosia
Dressing apraxia
A 78­year­old gentleman with Hemisomatognosia
Alzheimer's dementia has difficulty in
Agraphaesthesia
recognizing the faces of his children,
grandchildren and close friends,
Finger Agnosia
However, he recognizes their voices Ideomotor apraxia
Simultagnosia
Astereognosia

Hemisomatognosia
Ideomotor apraxia
A 76­year­old woman complains of a
Astereognosia
feeling that the limbs on one side of
Simultagnosia
the body are missing
Visuospatial Agnosia
Anosognosia
Prosopagnosia
Finger Agnosia
Check
Dressing apraxia
Agraphaesthesia

Simultagnosia is the inability to recognize complex pictures. 
Visuospatial agnosia is the inability to construct or copy figures. Copying three­dimensional
shapes such as a wire cube, interlocking pentagons or drawing a clock­face with numbers are
useful tests of constructional ability and may also highlight neglect if it is present. 
Anosognosia is failure to recognize a disabled limb. The patient does not recognise functional
deficits of their body. 
Prosopagnosics cannot identify familiar faces. "Often other clues, such as gait, voice or
distinctive clothing, are used to aid identification. The deficit may not be entirely selective to
faces, and often fine­grained identification within categories may also be impaired (for
example, makes of cars, and types of flowers)" (Kipps & Hodges). 
Hemisomatognosia is a term used when part of the body felt to be absent. 

Cognitive assessment for clinicians ­­ Kipps and Hodges 76 ..,
http://jnnp.bmj.com/content/76/suppl_1/i22.full (accessed April 1, 2015).
The correct answer is: A 76­year­old gentleman is having difficulty in recognising complex
pictures but can describe the parts. – Simultagnosia, An 83­year­old woman having difficulty
in copying a wire cube on extended cognitive examination – Visuospatial Agnosia, A 73­year­
old gentleman with a history of a major stroke suffered from left sided hemiplegia. However,
he denies having any physical problems including his gait disturbances. – Anosognosia, A

1425
78­year­old gentleman with Alzheimer's dementia has difficulty in recognizing the faces of his
children, grandchildren and close friends, However, he recognizes their voices –
Prosopagnosia, A 76­year­old woman complains of a feeling that the limbs on one side of the
body are missing – Hemisomatognosia

Question 17 HiY Assessment EMI017
Not answered Localising signs
Choose two likely symptoms for each of the following conditions:
Marked out of 6.00

Flag question
Anosognosia and constructional apraxia
Ipsilateral optic atrophy and nystagmus
Agraphia and anosognosia
A malnourished alcoholic.
Ipsilateral optic atrophy and anosmia
Ataxia and 6th nerve palsy
Ataxia and 4th nerve palsy
Anosmia and anosognosia
Ataxia and constructional apraxia
Anosmia and anosognosia
Ipsilateral optic atrophy and anosmia
A right­handed person with right Ataxia and 4th nerve palsy
parietal lobe lesion. Anosognosia and constructional apraxia
Agraphia and anosognosia
Ataxia and 6th nerve palsy
Ipsilateral optic atrophy and nystagmus
Ataxia and constructional apraxia
Ataxia and 4th nerve palsy
Agraphia and anosognosia
Ipsilateral optic atrophy and anosmia
A frontal meningioma.
Ataxia and constructional apraxia
Anosognosia and constructional apraxia
Anosmia and anosognosia
Ipsilateral optic atrophy and nystagmus
Ataxia and 6th nerve palsy
Check

This patient is most likely to have Wernicke's encephalopathy. The classic triad includes
ophthalmoplegia, ataxia and confusion. The most common ophthalmoplegia here is a
bilateral 6th nerve (not 4th nerve) lesion. The 4th nerve may be affected in few cases. The
non­dominant parietal lobe has a significant role in constructional praxis and also in
awareness of illness states. Frontal meningiomas are associated with anosmia (often present
due to the common olfactory groove distribution of tumours causing pressure) and ipsilateral
optic atrophy, on the side of the tumour. Anosmia with ipsilateral optic atrophy and
contralateral papilledema is also known as Kennedy­Foster syndrome.
The correct answer is: A malnourished alcoholic.
– Ataxia and 6th nerve palsy, A right­handed person with right parietal lobe lesion.
– Anosognosia and constructional apraxia, A frontal meningioma.
– Ipsilateral optic atrophy and anosmia

Question 18 HiY Assessment EMI018

1426
Not answered Rating scales in old age psychiatry
Identify the best fitting rating scales that could be used in each of the following situations.
Marked out of 5.00

Flag question
Hamilton Depression Rating Scale
This scale is more useful for the MMSE
elderly as it focuses on cognitive and BASDEC
subjective symptoms more than MADRS
physical symptoms AMTS
IQ­CODE
Cornell scale for depression
CAMDEX
Hamilton Depression Rating Scale
Geriatric depression scale
MADRS
This scale is helpful to detect Cornell scale for depression
depression in people with dementia AMTS
CAMDEX
Geriatric depression scale
IQ­CODE
MMSE
CAMDEX
BASDEC
MMSE
This scale is particularly useful for Geriatric depression scale
older hearing impaired subjects Cornell scale for depression
AMTS
BASDEC
MADRS
Hamilton Depression Rating Scale
Hamilton Depression Rating Scale
IQ­CODE
Cornell scale for depression
IQ­CODE
This scale is helpful for screening Geriatric depression scale
people with dementia
MADRS
BASDEC
MMSE
AMTS
CAMDEX
Geriatric depression scale
Hamilton Depression Rating Scale
This scale has a number of somatic
CAMDEX
items, which renders it less
MADRS
appropriate for older subjects.
MMSE
IQ­CODE
AMTS
Cornell scale for depression
Check BASDEC

Geriatric Depression Scale: Originally a 30­item test, it now has 15­items which enable most
subjects to be scored for depression in four or five minutes. It avoids somatic questions and so
remains useful for older patients. An overall score of five or more suggests the possibility of a
depressive episode. 

1427
Cornell scale for depression is the best­validated scale for detecting depression in dementing
patients. It is an interviewer­administered scale that uses information both from the patient and
an outside informant.
BASDEC (Brief assessment schedule depression cards): It was initially designed for use in
liaison psychiatry and is particularly useful with deaf subjects. It consists of a series of
statements in large print on cards, which are shown to the patient, one at a time and answers
'true' or 'false'.
The Mini­Mental State Examination (MMSE) is the standard screening instrument for
dementia. It takes 5­10minutes to administer and has a median positive Likelihood Ratio of
6.3 and a medianNegativeLikelihood Ratio of 0.19. It is a brief tool for grading cognitive
impairment in elderly and screening for dementia. It is not very sensitive to change, but used
in the clinical trials for dementia. It is a 30 point scale. It does not pick up frontal executive
defects. 
Hamilton rating scale: It has a number of somatic items, which render it less appropriate for
older subjects. It is more useful for working­age adults than the elderly in quantifying
depression.
The correct answer is: This scale is more useful for the elderly as it focuses on cognitive and
subjective symptoms more than physical symptoms – Geriatric depression scale, This scale is
helpful to detect depression in people with dementia – Cornell scale for depression, This
scale is particularly useful for older hearing impaired subjects – BASDEC, This scale is helpful
for screening people with dementia – MMSE, This scale has a number of somatic items, which
renders it less appropriate for older subjects. – Hamilton Depression Rating Scale

Question 19 HiY Assessment EMI019
Not answered Biochemical Investigations
Which one of the listed investigations is associated with the assessment of each of the
Marked out of 5.00
psychiatric conditions given below?
Flag question

Narcoanalysis
Catecholamine levels
Polysomnography
Panic disorder
ECG
Serum creatinine phosphokinase
Dexamethasone suppression test
Urinalysis
Mean corpuscular volume
Narcoanalysis
Serum prolactin levels
Dexamethasone suppression test
To differentiate pseudo seizures from
Lactate provocation test
Catecholamine levels
true epileptic attacks Lactate provocation test
Serum creatinine phosphokinase
Serum prolactin levels
Polysomnography
Urinalysis
Lactate provocation test
Mean corpuscular volume
Urinalysis
ECG
Polysomnography
Neuroleptic malignant syndrome
Narcoanalysis
Serum prolactin levels
ECG
Dexamethasone suppression test
Catecholamine levels
Serum creatinine phosphokinase

1428
Polysomnography
Syndrome of inappropriate ADH
Mean corpuscular volume
secretion induced by antidepressants
Serum creatinine phosphokinase
Urinalysis
Dexamethasone suppression test
Catecholamine levels
Lactate provocation test
ECG
Catecholamine levels
Serum prolactin levels
Dexamethasone suppression test
Narcoanalysis
Serum creatinine phosphokinase
Depression
Lactate provocation test
ECG
Serum prolactin levels
Polysomnography
Narcoanalysis
Check Mean corpuscular volume
Urinalysis

Lactate infusion: Nearly 72% patients with panic disorder have a panic attack when
administered IV injections of sodium lactate. Therefore, lactate provocation is used to confirm
a diagnosis of panic disorder. Hyperventilation and CO2 inhalation have been used.
Serum prolactin levels are usually elevated within 15­20 minutes of true seizures, which is not
the case in pseudo seizures. 
Creatinine phosphokinase (CPK) is raised in neuroleptic malignant syndrome.
Urine analysis may be necessary to differentiate SIADH induced by antidepressants or
antipsychotics from psychogenic polydipsia where excessive water consumption occurs
without apparent organic illness or Diabetes inspidus due to lithium(nephrogenic) or head
injury (central). The urine osmolality is high in SIADH induced by antidepressants despite a
hyponatremia. 
Continuous production of endogenous cortisol despite administration of exogenous steroid
(dexamethasone) suggests a failure of feedback suppression of ACTH/CRH. Normally
administering dexamethasone must reduce cortisol in plasma. This is because of intact HPA
function leading to reduced ACTH and CRH. In depression and other psychiatric
hypercortisolemic states (also in organic hypercortisolemic states such as Cushing's), this
does not occur. The dexamethasone suppression test (DST) is used to demonstrate this
hypercortisolemia (feedback failure).
The correct answer is: Panic disorder – Lactate provocation test, To differentiate pseudo
seizures from true epileptic attacks – Serum prolactin levels, Neuroleptic malignant syndrome
– Serum creatinine phosphokinase, Syndrome of inappropriate ADH secretion induced by
antidepressants – Urinalysis, Depression – Dexamethasone suppression test

Question 20 HiY Assessment EMI020
Not answered Jungian concepts in dynamic psychopathology
For each description given below, select the most appropriate Jungian concept from the list
Marked out of 3.00
provided:
Flag question

1429
The unconscious masculine aspects
Shadow
of a woman.
Extroversion
Analytical Psychology
Personal unconscious
Self
Persona
Animus
Anima
Anima
Introversion
Collective unconscious
Unacknowledged aspects of oneself Collective unconscious
Introversion
that includes both creative impulses Archetypes
Analytical Psychology
and destructive urges.
Animus
Personal unconscious
Extroversion
Persona
Anima
Self
Introversion
The outer concealed aspects of
Archetypes
Collective unconscious
oneself Shadow
Animus
Persona
Archetypes
Self
Shadow
Check
Extroversion
Personal unconscious
Analytical Psychology
Jung founded analytic psychology and introduced the terms extraversion and introversion. He
described collective unconscious ­ a term describing all of the mankind's collective symbolic
past, and archetypes ­ a term describing images and symbols that constitute the collective
unconscious. These representational images have universal symbolic meaning (e.g. Hero,
Old Wise Man, Tree, etc.). Other terms described by Jung include Persona which is the mask
covering one's personality and presented to outside world, Anima ­ unconscious feminine
aspect of a man, Animus ­ unconscious masculine character of a woman, Shadow ­ an
archetype ­ a personification of unacceptable aspects of oneself symbolized as a dark internal
alien and Individuation ­ ultimate goal of life where an individual develops a sense of self­
identity.
The correct answer is: The unconscious masculine aspects of a woman. – Animus,
Unacknowledged aspects of oneself that includes both creative impulses and destructive
urges. – Shadow, The outer concealed aspects of oneself – Persona

Question 21 HiY Assessment EMI021
Not answered Eye signs
Given the clinical situation, identify the eye signs commonly seen:
Marked out of 5.00

Flag question

1430
Sluggish light and accommodation
Kayser­Fleischer ring
reflexes with persistent reduction in
Anosognosia
pupil size
Marcus Gunn pupil
Argyll­Robertson pupil
Horner's syndrome
Holmes­ Adie pupil
Hutchinson pupil
Senile pupil
Dolls eye manoeuvre
Marcus Gunn pupil
Anisocoria
Hutchinson pupil
Small and irregular pupil that does Anosognosia
not react to light but does
Argyll­Robertson pupil
accommodate
Holmes­ Adie pupil
Kayser­Fleischer ring
Dolls eye manoeuvre
Senile pupil
Horner's syndrome
Horner's syndrome
Kayser­Fleischer ring
As the flashlight swings from the
normal eye to the abnormal eye, the
Anisocoria
Anosognosia
abnormal pupil dilates. No anisocoria Marcus Gunn pupil
is seen. Holmes­ Adie pupil
Anisocoria
Argyll­Robertson pupil
Senile pupil
Argyll­Robertson pupil
Dolls eye manoeuvre
Kayser­Fleischer ring
The pupil on one side constricts and
Hutchinson pupil
then widely dilates. Then the pupil of Anosognosia
the other eye goes through the same Hutchinson pupil
sequence Senile pupil
Marcus Gunn pupil
Dolls eye manoeuvre
Horner's syndrome
Dolls eye manoeuvre
Holmes­ Adie pupil
Senile pupil
Anisocoria
Horner's syndrome
Anosognosia
Ptosis, anhidrosis and enophthalmos
Hutchinson pupil
with normal light and accommodation
reflex
Anisocoria
Kayser­Fleischer ring
Argyll­Robertson pupil
Marcus Gunn pupil
Holmes­ Adie pupil

Check

Senile pupil: Note that with aging both accommodation and light reflexes become sluggish.
This is accompanied by a persistent reduction in pupil size, called senile miosis. Senile miosis
is better observed when the eye is completely dark­adapted i.e. in darkness. The senile pupil
is the result of age­related atrophy of the radiator dilator muscle of the pupil despite relatively
preserved sphincter constrictor muscle strength. 

1431
Argyll­Robertson pupil (ARP), seen in neurosyphilis, is irregular and small; it does not react to
light, but does accommodate. (Mnemonic: A­R­P: Accommodation Reflex Present).
Marcus­Gunn pupil: This sign is elicited using a swinging flashlight test. As the flashlight
swings from one eye to the other, the abnormal pupil dilates when the light turns away from
the normal eye, onto the affected side. At rest, anisocoria (unequal pupils) is not seen in this
condition. This phenomenon indicates an afferent pupillary defect arising from optic nerve
damage. It also goes by the name paradoxical pupillary reflex.
Hutchinson pupil results from rapidly rising unilateral intracranial pressure, as in sudden
massive intracranial haemorrhage.
Horner's syndrome refers to ptosis, anhydrosis and enophthalmos with normal light and
accommodation reflex. 
Ref: Neurological History and Physical Examination. (n.d.). Retrieved from
http://emedicine.medscape.com/article/1147993­overview
The correct answer is: Sluggish light and accommodation reflexes with persistent reduction in
pupil size – Senile pupil, Small and irregular pupil that does not react to light but does
accommodate – Argyll­Robertson pupil, As the flashlight swings from the normal eye to the
abnormal eye, the abnormal pupil dilates. No anisocoria is seen. – Marcus Gunn pupil, The
pupil on one side constricts and then widely dilates. Then the pupil of the other eye goes
through the same sequence – Hutchinson pupil, Ptosis, anhidrosis and enophthalmos with
normal light and accommodation reflex – Horner's syndrome

Question 22 HiY Assessment EMI022
Not answered Gait abnormality
Choose the best option for each of the following;
Marked out of 4.00

Flag question
Stiff legged
scissoring gait
Festinant gait (shuffling gait)
Huntington's disease
Stomping gait
Myopathic gait (Waddling gait)
Pigeon gait
Broad­based
High stepping gait (Steppage gait)
Festinant gait (shuffling gait)
Lurching chaotic gait
Myopathic gait (Waddling gait)
unsteady gait
Lurching chaotic gait
Pregnancy Failure of gait ignition
Failure of gait ignition
Pigeon gait
High stepping gait (Steppage gait)
Stiff legged
unsteady gait
Stiff legged
Stomping gait
Myopathic gait (Waddling gait)
Broad­based
Festinant gait (shuffling gait)
Hip dysplasia scissoring gait
Broad­based
High stepping gait (Steppage gait)
Lurching chaotic gait
unsteady gait
Stomping gait
Failure of gait ignition
Pigeon gait
scissoring gait

1432
Friedreich's ataxia
Broad­based
Pigeon gait
High stepping gait (Steppage gait)
scissoring gait
Festinant gait (shuffling gait)
Myopathic gait (Waddling gait)
Lurching chaotic gait
Stiff legged
Check
Stomping gait
unsteady gait
Failure of gait ignition
A chaotic lurching gait is seen in Huntington's disease. 
Waddling gait is a broad­based gait with a duck­like waddle to the swing phase seen in
pregnancy and proximal myopathy. 
Torsional difficulties lead to pigeon gait in hip dysplasia. 
In dorsal column sensory disturbances such as subacute combined cord degeneration and
Friedreich's ataxia, a stomping gait is noted.
The correct answer is: Huntington's disease – Lurching chaotic gait, Pregnancy – Myopathic
gait (Waddling gait), Hip dysplasia – Pigeon gait, Friedreich's ataxia – Stomping gait

Question 23 HiY Assessment EMI023
Not answered Validated rating scales for clinical use
Choose one rating scale for each of the following;
Marked out of 4.00

Flag question
CATEGO
A primary care research student CAN
would like to use a computer SANS
programme to generate psychiatric HONOS
diagnosis HADS
SCAN
PANSS
HAMD
HAMD
A CT1 trainee would like to use a
BPRS
BPRS
semi­structured interview, which CATEGO
retains the features of a clinical HADS
examination HONOS
SCAN
PANSS
SANS
CAN
A rehabilitation nurse would like to
CAN
PANSS
carry out a needs assessment HADS
looking at various domains of an CATEGO
individual's life SCAN
SANS
HONOS
HAMD
BPRS

1433
The commissioners of the local
HADS
catchment area would like mental
HONOS
health professionals to use an
instrument, which could measure
SCAN
clinical outcomes in mental health. CATEGO
PANSS
BPRS
HAMD
SANS
Check
CAN

CATEGO is a computer programme for processing data from the schedules of clinical
assessment in neuropsychiatry. 
SCAN stands for Schedules for Clinical Assessment in Neuropsychiatry. SCAN includes a set
of tools made by the WHO to aid diagnosis and measurement of adult mental illness. While
SCAN was not purpose­built for either ICD­10 or DSM­IV, it is compatible with both systems.
The original name for SCAN was PSE or Present State Examination, but since version 10
(PSE­10), the name SCAN has been commonly used. 
CAN refers to the Camberwell Assessment of Need. It is a 'needs assessment for the severely
mentally ill'. It is a 22­item objective instrument that includes assessments of various domains
of an individual's life. 
HONOS was developed to measure clinical outcomes in mental health. 
Mental Health Outcome Assessment Tools (n.d.). Retrieved from
http://www.whatisglobelife.com/html/mental­health­outcome­assessment­tools.html
The correct answer is: A primary care research student would like to use a computer
programme to generate psychiatric diagnosis – CATEGO, A CT1 trainee would like to use a
semi­structured interview, which retains the features of a clinical examination – SCAN, A
rehabilitation nurse would like to carry out a needs assessment looking at various domains of
an individual's life – CAN, The commissioners of the local catchment area would like mental
health professionals to use an instrument, which could measure clinical outcomes in mental
health. – HONOS

Question 24 HiY Assessment EMI024
Not answered Epilepsy & clinical features
According to the most important features seen in various types of epilepsy, identify the clinical
Marked out of 12.00
syndrome described.
Flag question

Dissociative seizures
Simple partial seizures
It may start with a focal lesion in the Absence seizures
brain and may later have a
Myoclonic seizures
secondary generalization
Grandmal seizures
Infantile spasms
Complex partial seizures

Seizures differ from attack to attack
with features such as arching of the
back and asymmetrical jerking of
limbs. Intensity of seizure influenced
by the presence of an observer

1434
Grandmal seizures
Myoclonic seizures
Dissociative seizures
Simple partial seizures
Infantile spasms
Complex partial seizures
Absence seizures
Complex partial seizures
Myoclonic seizures
Loss of awareness is an important Dissociative seizures
feature. EEG shows multispikes
Simple partial seizures
followed by spike and slow waves
Infantile spasms
Absence seizures
Grandmal seizures
Complex partial seizures
It may start with a focal lesion in the Dissociative seizures
brain with secondary generalization. Simple partial seizures
Loss of awareness is an important Myoclonic seizures
feature. Infantile spasms
Grandmal seizures
Absence seizures
Simple partial seizures
Infantile spasms
EEG shows generalised 3 Hz spike Absence seizures
and slow wave activity. Loss of
Complex partial seizures
awareness is an important feature
Grandmal seizures
Myoclonic seizures
Dissociative seizures

Check

A simple partial seizure arises in a localized area of the cerebral cortex, and it evolves into a
secondarily generalized seizure. 
Dissociative seizures are non­epileptic events that have a psychogenic origin and resemble
epileptic seizures. Here the patients can present with bizarre and irregular movements such
as arching of the limbs and asymmetrical jerking of the limbs. The behaviour is also
influenced by the presence of an observer, by restraint or by suggestion during periods of
apparent unresponsiveness. 
In grandmal seizures, loss of awareness is a key feature and EEG shows multispikes followed
by spike and slow waves. 
Complex partial seizures often start as simple partial seizures and progress to impairment of
consciousness. 
Absence seizures occur without warning and consist of a sudden interruption of
consciousness. The patient abruptly ceases whatever activity he or she is engaged in, and the
eyes stare vacantly ahead. It usually lasts 2­15 seconds following which normal behaviour
resumes and the patient is often unaware of what has happened. Typical absence seizures
are accompanied by a characteristic EEG abnormality of generalized spike and wave activity
at a frequency of 3 Hz (2­4 Hz spike and wave activity)
The correct answer is: It may start with a focal lesion in the brain and may later have a

1435
secondary generalization – Simple partial seizures, Seizures differ from attack to attack with
features such as arching of the back and asymmetrical jerking of limbs. Intensity of seizure
influenced by the presence of an observer – Dissociative seizures, Loss of awareness is an
important feature. EEG shows multispikes followed by spike and slow waves – Grandmal
seizures, It may start with a focal lesion in the brain with secondary generalization. Loss of
awareness is an important feature. – Complex partial seizures, EEG shows generalised 3 Hz
spike and slow wave activity. Loss of awareness is an important feature
– Absence seizures

Question 25 HiY Assessment EMI025
Not answered Speech difficulties in children 
Identify the type of speech difficulties seen in each of the following situations:
Marked out of 3.00

Flag question
Elective mutism
A 7­year­old girl can't speak but can Expressive language disorder
utter incomprehensible sounds. She Dysphonia
can make herself understood using
Schizophrenia
non­verbal communication and can
read and understand.
Speech fluency deficit
Autism
Dyslexia
Dysarthria
Dyslexia
Expressive language disorder
A 7­year­old boy says, "Me want a Autism
drink" rather than "I would like a
Dysarthria
drink."
Elective mutism
Dysphonia
Speech fluency deficit
Schizophrenia
Autism
Dysphonia
Speech fluency deficit
An 11­year­old boy says, "In deep Schizophrenia
space nine soldiers are fronting" Dyslexia
when asked if he wants some food. Dysarthria
He appears to stare vacantly in Expressive language disorder
response to other questions. Elective mutism

Check

In expressive language disorder, the use of non­verbal cues and internal language, as
reflected in imaginative or make­believe play, are intact, and the capacity to interact socially
without words is almost preserved. The child shows interest in communicating despite its
limitations and uses demonstration, gesture or non­speech vocalization to compensate for
lack of verbal language (ICD­10; 238). 
Autism­ Speech is not only limited in autism but is pervaded by echolalia, stereotypic
utterances and avoidance of the personal pronoun like I, etc. 

1436
Case3: This boy appears to have childhood­onset schizophrenia, with thought disorder
characterised by "breaks or interpolations in the train of thought, resulting in incoherence or
irrelevant speech or neologisms." (ICD­10) 
Schizophrenia and Public Health. (n.d.). Retrieved from
http://www.who.int/mental_health/media/en/55.pdf
The correct answer is: A 7­year­old girl can't speak but can utter incomprehensible sounds.
She can make herself understood using non­verbal communication and can read and
understand. – Expressive language disorder, A 7­year­old boy says, "Me want a drink" rather
than "I would like a drink." – Autism, An 11­year­old boy says, "In deep space nine soldiers are
fronting" when asked if he wants some food. He appears to stare vacantly in response to other
questions. – Schizophrenia

Question 26 HiY Assessment EMI026
Not answered Psychiatric syndromes associated with stroke
For each of the following clinical conditions below, match the associated psychiatric syndrome
Marked out of 3.00
from the given list:
Flag question

Depression
After an acute haemorrhagic stroke, a Emotional lability
76­year­old woman becomes Emotionalism
anxious and tearful whenever she Psychosis
attempts to solve times crossword. Bipolar affective disorder
Anxiety disorder
Mania
Cognitive impairment
Anxiety disorder
Catastrophic Reaction
Mania
An 80­year­old gentleman with right­ Psychosis
Apathy without depression
sided hemiparesis presents with
Cognitive impairment
impairment in the control of crying.
Catastrophic Reaction
Emotionalism
Bipolar affective disorder
Depression
Catastrophic Reaction
A 67­year­old man is recovering after
Emotional lability
Bipolar affective disorder
a significant thromboembolic stroke.
He appears unconcerned about his
Apathy without depression
Depression
memory loss and not motivated to Anxiety disorder
participate in the physical Psychosis
rehabilitation activities. Apathy without depression
Mania
Cognitive impairment
Check Emotional lability
Emotionalism

A catastrophic reaction is seen in dementia when a cognitively impaired patient is asked
undertake tasks beyond their performance ability. Patients may appear agitated, angry and
even violent. The situation described as case 2 is termed emotionalism ­ a recurrent crying (or
sobbing) or laughing episodes that mark a change in previous behaviour. Post­stroke
emotionalism has a high prevalence rate varying between 15% to 35%. Apathy is dull
emotional tone associated with detachment, which mimics like depression. Often there is
avolition and reduced motivation. No other overt symptoms of depression are present. This is
seen in 20% of patients with stroke.

1437
The correct answer is: After an acute haemorrhagic stroke, a 76­year­old woman becomes
anxious and tearful whenever she attempts to solve times crossword. – Catastrophic
Reaction, An 80­year­old gentleman with right­sided hemiparesis presents with impairment in
the control of crying. – Emotionalism, A 67­year­old man is recovering after a significant
thromboembolic stroke. He appears unconcerned about his memory loss and not motivated to
participate in the physical rehabilitation activities. – Apathy without depression

Question 27 HiY Assessment EMI027
Not answered Somatoform and Dissociative Disorders
For each of the following descriptions choose the most appropriate diagnosis from the list
Marked out of 4.00
provided:
Flag question

A 47­year­old woman presented with
Possession disorder
a 4­year history of multiple vague Hypochondriacal disorder
physical symptoms not explained by Neurasthenia
any physical causes. She seeks Dissociative fugue
repeated consultations with her GP Munchausen syndrome
and hospital specialists. Somatisation disorder
Dissociative amnesia
Ganser's syndrome
Munchausen syndrome
Multiple personality disorder
Somatisation disorder
Neurasthenia
Dissociative fugue
A 35­year­old gentleman who
Hypochondriacal disorder
presents himself 30 miles away from
his residence for no apparent reason
Ganser's syndrome
with well­maintained self­care. Possession disorder
Multiple personality disorder
Dissociative amnesia

Multiple personality disorder
Somatisation disorder
A 43­year­old prisoner awaiting trial Neurasthenia
when asked how many legs a cat
Possession disorder
has, answers 'five legs.'
Dissociative amnesia
Munchausen syndrome
Dissociative fugue
Ganser's syndrome
Possession disorder
A 40­year­old woman who is always Hypochondriacal disorder
Munchausen syndrome
preoccupied that she has breast Ganser's syndrome
cancer despite all her relevant
Dissociative fugue
investigations were proved to be
negative.
Hypochondriacal disorder
Dissociative amnesia
Somatisation disorder
Neurasthenia
Check
Multiple personality disorder

1438
Somatisation is the experience of bodily symptoms with no, or no sufficient, physical cause for
them, with presumed psychological causation.
Fugue is a dissociative reaction to intolerable stress. This could happen following a significant
external stressor that the affected individual develops global amnesia and may wander to a
distant location. Consciousness is unimpaired. 
Ganser syndrome involves the production of approximate answers. Here the patient gives
repeated wrong answers to questions, which are nonetheless in the right ballpark. Sometimes
this is associated with organic brain illness though it is much more common as a form of
malingering (especially in prisoners anticipating trial). 
In hypochondriasis, patient looks for diagnosis, not symptom relief; s/he often names a
particular suspect physical disorder and may be more or less convinced about having the
disorder. In a patient with somatisation musculoskeletal symptoms are common; while in
hypochondriasis gastrointestinal problems are the most common ones. Family history of
alcohol use and antisocial personality may be more common in women with somatisation
disorder
The correct answer is: A 47­year­old woman presented with a 4­year history of multiple vague
physical symptoms not explained by any physical causes. She seeks repeated consultations
with her GP and hospital specialists. – Somatisation disorder, A 35­year­old gentleman who
presents himself 30 miles away from his residence for no apparent reason with well­
maintained self­care. – Dissociative fugue, A 43­year­old prisoner awaiting trial when asked
how many legs a cat has, answers 'five legs.' – Ganser's syndrome, A 40­year­old woman who
is always preoccupied that she has breast cancer despite all her relevant investigations were
proved to be negative. – Hypochondriacal disorder

Question 28 HiY Assessment EMI028
Not answered Tropical diseases
Choose the possible infective agent for each of the following presentations.
Marked out of 3.00

Flag question
Leishmania Donavanii
An 18­year­old girl develops a Trichomonas
circular bull's eye with lethargy andBorrelia burgdorferi
myalgia along with memory and
Tropical yaws
mood disturbances after returning
from a camping trip in America.
Giardia lamblia
Leptospirosis
Epstein­Barr Virus
Cytomegalovirus
Leishmania Donavanii
A 36­year­old man running an animal
Falciparum malaria
Borrelia burgdorferi
and bird rescue centre presents with
high fever, joint pains along with Chlamydia psitaccii
Cytomegalovirus
diarrhoea. On examination, he has Treponema pallidum
Trichomonas
conjunctivitis and macular spots on Tropical yaws
his face. Leptospirosis
Falciparum malaria
Treponema pallidum
Trichomonas
A 65­year­old Vietnam war veteran
Epstein­Barr Virus
Falciparum malaria
presents with manic features and
cognitive impairment. He has poor
Giardia lamblia
Borrelia burgdorferi
balance and complains of lightning Chlamydia psitaccii
Chlamydia psitaccii
pains in the lower extremities when Cytomegalovirus
examined. Epstein­Barr Virus
Leptospirosis
Tropical yaws

1439
Check

Borrelia Burgdorferi causes Lyme's disease characterised by erythema migraines followed by
neurological symptoms and depression. A zoonotic illness seen in pet handlers (especially
tropical birds) is Psittacosis, caused by Chlamydia psittaci. The presentation is similar to
typhoid fever ­ with macular rashes and splenomegaly. Tertiary syphilis can present with
insidious neurological illness characterised by tabes dorsalis and later general paresis of
insane characterised by grandiosity and affective disturbances.
The correct answer is: An 18­year­old girl develops a circular bull's eye with lethargy and
myalgia along with memory and mood disturbances after returning from a camping trip in
America. – Borrelia burgdorferi, A 36­year­old man running an animal and bird rescue centre
presents with high fever, joint pains along with diarrhoea. On examination, he has
conjunctivitis and macular spots on his face. – Chlamydia psitaccii, A 65­year­old Vietnam war
veteran presents with manic features and cognitive impairment. He has poor balance and
complains of lightning pains in the lower extremities when examined. – Treponema pallidum

Question 29 HiY Assessment EMI029
Not answered Visual defects
Using the descriptions given below, identify the type of visual defect
Marked out of 5.00

Flag question
Adie pupil
Tunnel vision
A 79­year­old man with a recent Quadrantanopia
history of major stroke describes
Vertical hemianopia
having a keyhole vision
Funnel vision
Horizontal hemianopia
Cortical blindness
Argyll Robertson pupil
Horizontal hemianopia
Anton's syndrome
Argyll Robertson pupil
A 45­year­old woman with sudden Adie pupil
onset blindness describes patchy
Quadrantanopia
spirals of field loss
Tunnel vision
Cortical blindness
Vertical hemianopia
Funnel vision
Quadrantanopia
Anton's syndrome
Anton's syndrome
A 67­year­old man has a history of
glaucoma. He reports that the field Vertical hemianopia
projected at 2 meters is larger than Argyll Robertson pupil
the one plotted at 1 metre. Horizontal hemianopia
Cortical blindness
Adie pupil
Funnel vision
Adie pupil
Tunnel vision
Anton's syndrome
A 60­year­old woman who is blind Funnel vision
denies her visual defect Argyll Robertson pupil
Vertical hemianopia
Horizontal hemianopia
Quadrantanopia

1440
A 70­year­old woman with optic Funnel vision
nerve lesions describes only one­ Cortical blindness
fourth of the field is affected on one Horizontal hemianopia
side Adie pupil
Quadrantanopia
Vertical hemianopia
Anton's syndrome
Argyll Robertson pupil
Check Tunnel vision

Cortical blindness is most often due to posterior cerebral artery occlusion. In effect, it is a
bilateral homonymous hemianopia. Patients experience either a small central field around the
point of fixation (macular sparing or keyhole vision) or complete blindness. 
For case 2, the diagnosis is conversion disorder. Tunnel vision refers to the absence of
projection distance disparity in the field of vision and reporting patchy spirals of field loss.
Tunnel vision is seen in hysteria or malingering.
Funnel vision: In the presence of an organic field defect, the field projected at two metres is
larger than the field plotted at one metre (funnel vision). This is seen in glaucoma, retinitis
pigmentosa, in the presence of hyaline bodies in the disc, post papilledema optic atrophy,
bilateral occipital infarcts with macular sparing, and feigned visual loss. 
Occasionally, patients with cortical blindness deny their visual defect, which is called Anton's
syndrome. 
When only a one­fourth of the field is affected, the resulting deficit is called quadrantanopia.
Hemianopia is a field defect that encompasses roughly half of the field. Vertical hemianopia
can be nasal or temporal. Horizontal or altitudinal hemianopia can be superior or inferior.
Bilateral field defects are said to be homonymous when they are similarly located in both
visual fields.
The correct answer is: A 79­year­old man with a recent history of major stroke describes
having a keyhole vision – Cortical blindness, A 45­year­old woman with sudden onset
blindness describes patchy spirals of field loss – Tunnel vision, A 67­year­old man has a
history of glaucoma. He reports that the field projected at 2 meters is larger than the one
plotted at 1 metre. – Funnel vision, A 60­year­old woman who is blind denies her visual defect
– Anton's syndrome, A 70­year­old woman with optic nerve lesions describes only one­fourth
of the field is affected on one side – Quadrantanopia

Question 30 HiY Assessment EMI030
Not answered Investigations of choice
Choose the most appropriate investigations of choice for following clinical situations.
Marked out of 2.00

Flag question
SPECT scan
CT scan
A 35­year­old gentleman presenting
ECG
with fever, abrupt onset confusion
Ultrasound scan
and behavioral changes
EEG
Lumbar puncture
MRI scan
F­MRI scan
MRS scan

1441
A 75­year­old gentleman, who suffers EEG
from late onset psychosis, has been
ECG
on olanzapine. He develops
parkinsonian symptoms, and the MRI scan
neurologist says that it could be SPECT scan
antipsychotic­induced. MRS scan
Lumbar puncture
Ultrasound scan
F­MRI scan
Check
CT scan

Case 1: This presentation is suggestive of herpes simplex encephalitis (HSE) characterized
by a fairly abrupt onset of confusion, memory impairment, and often seizures. 70% show
psychiatric disturbances including acute confusion, depression and psychosis. Fever is
common but not invariable. Magnetic resonance imaging (MRI) provides the most sensitive
method of detecting early lesions and is the imaging of choice in HSE. Cerebrospinal fluid
(CSF) examination commonly reveals a lymphocytosis and raised protein. The gold standard
for in vivo diagnosis is CSF polymerase chain reaction (PCR) for herpes viruses, which has a
sensitivity and specificity of ~95%. 
Case 2: This could be a case of Lewy body dementia presenting initially with psychotic
symptoms and later developed other features like motor symptoms of parkinsonism and
neuroleptic sensitivity. Alternatively the patient might be now developing Parkinson's disease
on top of functional psychotic illness. In either case, the investigation useful would be DAT­
SCAN (I­123­FP­CITscan). This uses SPECT technology and can be undertaken in most
imaging departments. This demonstrated abnormal images in 86% of DLB cases and also
helps to differentiate idiopathic Parkinson's from drug­induced parkinsonism.
The correct answer is: A 35­year­old gentleman presenting with fever, abrupt onset confusion
and behavioral changes – MRI scan, A 75­year­old gentleman, who suffers from late onset
psychosis, has been on olanzapine. He develops parkinsonian symptoms, and the
neurologist says that it could be antipsychotic­induced. – SPECT scan

Question 31 HiY Assessment EMI031
Not answered DSM Axes
Identify the descriptions from the given list that are best suited for each of the following DSM­
Marked out of 5.00
IV axes.
Flag question

Suicidality
Recurrent self­harm
Global assessment of functioning=60
Axis­2
Alcohol intoxication
Unemployment
Psychotic disorder due to a general medical condition
Anorexia nervosa
Narcolepsy
Complex partial seizures
Complex partial seizures
Psychotic disorder due to a general medical condition
Moderate mental retardation
Global assessment of functioning=60
Alcohol intoxication
Axis­3 Anorexia nervosa
Narcolepsy
Unemployment
Recurrent self­harm

1442
Alcohol intoxication
Global assessment of functioning=60
Unemployment
Axis­4
Suicidality
Narcolepsy
Anorexia nervosa
Psychotic disorder due to a general medical condition
Moderate mental retardation
Check
Recurrent self­harm
Complex partial seizures

The multi­axial system of DSM­IV used five axes. Though these axes are now not employed in
DSM­5, the multiaxial concept is still being used by researchers and health service analysts.  
Axis I ­ Clinical Disorders (e.g. anorexia, alcohol intoxication, psychosis due to medical
condition), Axis II ­ Personality Disorders/ Mental Retardation (e.g. moderate Mental
Retardation, anxious personality traits), Axis III ­ General Medical Conditions (e.g. epilepsy,
hypertension), Axis IV ­ Psychosocial and Environmental Problems (e.g. Unemployment), Axis
V ­ Global Assessment of Functioning (e.g. GAF score of 60)
The correct answer is: Axis­2
– Moderate mental retardation, Axis­3 – Complex partial seizures, Axis­4 – Unemployment

Question 32 HiY Assessment EMI032
Not answered Cognitive impairment in the elderly
For each of the following descriptions choose the most appropriate diagnosis from the list
Marked out of 2.00
provided.
Flag question

A 67­year­old gentleman presenting Sporadic CJD
with memory problems and urinary Multi­infarct dementia
incontinence. On neurological Charles Bonnet Syndrome
examination, he has a broad­based, Depressive pseudodementia
short stepped gait. A CT brain scan Wilson's' disease.
shows ventricular enlargement. Normal pressure hydrocephalus
Alzheimer's disease
Dementia in Huntington's disease
Lewy body Dementia
83­year­old lady with bilateral
Dementia in HIV diseases
Dementia in Huntington's disease
cataracts sees animals and people in Lewy body Dementia
Multi­infarct dementia
the house. She is independent with Dementia in Pick's disease
Sporadic CJD
all her daily activities and enjoys life. Normal pressure hydrocephalus
Wilson's' disease.
Charles Bonnet Syndrome
Alzheimer's disease
Check
Dementia in HIV diseases
Depressive pseudodementia
Dementia in Pick's disease
Case 1 is suggestive of Normal Pressure Hydrocephalus, which is a syndrome associated
with dilatation of cerebral ventricles (especially 3rd ventricle) but normal CSF pressure at the
lumbar puncture. It typically presents with the triad of dementia, gait ataxia, and urinary

1443
incontinence. The diagnosis rests on clinical suspicion. Disturbance of balance and gait is the
cardinal feature and patients walk with a mildly broad­based, short stepped gait, which is also
symmetrical. Gait disturbance always precedes the development of other symptoms and
worsens insidiously over months. 
Case 2 is Charles Bonnet syndrome, which is a condition in which individuals with partial
visual impairment experience extremely vivid visual hallucinations. These occur persistently
and repetitively without any delusions or hallucinations in other modality, and without
disturbance of consciousness. Insight is preserved. The condition is most often seen in
women of advanced age. A similar phenomenon can also occur in the auditory modality in
those who have a hearing impairment. 
Confabulation occurs due to a double lesion ­ frontal deficits and diencephalic deficits.
The correct answer is: A 67­year­old gentleman presenting with memory problems and
urinary incontinence. On neurological examination, he has a broad­based, short stepped gait.
A CT brain scan shows ventricular enlargement. – Normal pressure hydrocephalus, 83­year­
old lady with bilateral cataracts sees animals and people in the house. She is independent
with all her daily activities and enjoys life. – Charles Bonnet Syndrome

Question 33 HiY Assessment EMI033
Not answered Eponymous syndromes in psychiatry
Identify the syndromes in psychiatry based on the descriptions given below
Marked out of 3.00

Flag question
Anton's syndrome
Klein­Levine syndrome
Demyelination and necrosis of Gerstmann syndrome
corpus callosum associated with red
Marchiafava­Bignami disease
wine consumption.
Kluver­Bucy syndrome
Geschwind syndrome
Briquet's syndrome
Balint's syndrome
Anton's syndrome
Balint's syndrome
Lesion of occipital lobes with cortical Kluver­Bucy syndrome
blindness. Briquet's syndrome
Marchiafava­Bignami disease
Geschwind syndrome
Klein­Levine syndrome
Gerstmann syndrome
Balint's syndrome
Anton's syndrome
Geschwind syndrome
Briquet's syndrome
Marchiafava­Bignami disease
Lesion of the dominant parietal lobe. Klein­Levine syndrome
Kluver­Bucy syndrome
Gerstmann syndrome

Check

1444
Marchiafava­Bignami disease is due to demyelination and necrosis of corpus callosum and
adjacent anterior commissure seen in alcoholics, especially using red wine excessively.
Anton's syndrome is a visual agnosia, in which the patient denies any deficit and may attempt
to negotiate the environment, invariably without success.
Gerstmann syndrome is characterized by four primary symptoms: Dysgraphia/agraphia,
Dyscalculia/acalculia, Finger agnosia, Left­right disorientation. It is seen in dominant parietal
lesions.
The correct answer is: Demyelination and necrosis of corpus callosum associated with red
wine consumption. – Marchiafava­Bignami disease, Lesion of occipital lobes with cortical
blindness. – Anton's syndrome, Lesion of the dominant parietal lobe. – Gerstmann syndrome

Question 34 HiY Assessment EMI034
Not answered Communication skills
Identify the interview techniques used in each of the following situations:
Marked out of 5.00

Flag question
Patient says: 'It is not good if one's
normalisation
parents are divorced even before one acknowledgement
clarification
goes to school.' Doctor says: 'I'd like
to hear more about your parents, but limit setting
first let me get a picture of what's symptom expectation
happening to you of late'. restatement
self­revelation
When discussing alcohol use, repetition
repetition
suddenly the clinician asks 'What redirection
limit setting
sorts of drugs do you usually use facilitation
restatement
when you're drinking?' summation
redirection
self­revelation
acknowledgement
normalisation
clarification
limit setting
summation
clarification
'Sometimes when people are very
depressed, they think of hurting symptom expectation
acknowledgement
themselves. Has this been true for facilitation
redirection
you?' restatement
symptom expectation
facilitation
repetition
repetition
normalisation
limit setting
summation
'I can see that you look anxious when normalisation
talking about those voices'. self­revelation
acknowledgement
facilitation
clarification
self­revelation
Clinician says 'In fact when I took
symptom expectation
fluoxetine some years ago, I had all restatement
these side effects for first few days. summation
But they disappeared in few weeks redirection
time.'

1445
acknowledgement
self­revelation
repetition
redirection
facilitation
symptom expectation
restatement
clarification
Check limit setting
summation
normalisation
Redirection is changing the direction of conversation to elicit clinical information. In symptom
expectation, the clinician expects the patient to use drugs after listening to some information
regarding alcohol use. In Normalisation, the clinician normalises / validates suicidal ideas in
the context of depression before further questioning. Acknowledgement refers to the clinician
acknowledging the patient's affect. This can facilitate further disclosure. Self­revelation refers
to the doctor revealing some of his or her experiences to the patient.
The correct answer is: Patient says: 'It is not good if one's parents are divorced even before
one goes to school.' Doctor says: 'I'd like to hear more about your parents, but first let me get a
picture of what's happening to you of late'. – redirection, When discussing alcohol use,
suddenly the clinician asks 'What sorts of drugs do you usually use when you're drinking?' –
symptom expectation, 'Sometimes when people are very depressed, they think of hurting
themselves. Has this been true for you?' – normalisation, 'I can see that you look anxious
when talking about those voices'. – acknowledgement, Clinician says 'In fact when I took
fluoxetine some years ago, I had all these side effects for first few days. But they disappeared
in few weeks time.' – self­revelation

Question 35 HiY Assessment EMI035
Not answered Clinical neurological signs
Identify the neurological sign seen in each of the following situations:
Marked out of 4.00

Flag question
Perseveration
Lhermitte's sign
A 60­year­old man develops
Nystagmus
respiratory failure, which results in
Upper limb rigidity
anoxic brain damage. (Select ONE).
Straight leg rising sign
Flapping tremor
Beevor sign
Extensor plantar reflex
Upper limb rigidity
Intention tremor
Nystagmus
Optic atrophy
Lhermitte's sign
Neck stiffness
Perseveration
A 35­year­old man is a known binge
drinker with significant Intention tremor
malnourishment. He present with Beevor sign
acute onset confusion, ataxia and Straight leg rising sign
disorientation. Extensor plantar reflex
Neck stiffness
Flapping tremor
Optic atrophy

1446
Upper limb rigidity
Nystagmus
An 18­year­old man sustains
Flapping tremor
motorbike accident with spinal injury
Straight leg rising sign
at the level of T5. 
Perseveration
Intention tremor
Neck stiffness
Optic atrophy
Check
Lhermitte's sign
Beevor sign
Extensor plantar reflex
A flapping tremor is seen in cases of respiratory, hepatic or cardiac failure with
encephalopathy.Ataxia and ophthalmoplegia with nystagmus in a background of impaired
consciousness are characteristic of Wernicke's encephalopathy.Beevor's sign is characterised
by an upward movement of umbilicus when raising the head from a supine position due to
lower abdominal paralysis.
The correct answer is: A 60­year­old man develops respiratory failure, which results in anoxic
brain damage. (Select ONE). – Flapping tremor, A 35­year­old man is a known binge drinker
with significant malnourishment. He present with acute onset confusion, ataxia and
disorientation.
– Nystagmus, An 18­year­old man sustains motorbike accident with spinal injury at the level of
T5. 
– Beevor sign

Question 36 HiY Assessment EMI036
Not answered Alcohol­induced physical symptoms
Choose the symptoms which would best support each diagnosis in an alcoholic,
Marked out of 6.00
malnourished middle­aged man who becomes confused two days after emergency surgery
Flag question for a femoral shaft fracture.

Petechial rash
Nystagmus
Kernig's sign
Bacterial septicaemia
Exophtholmos
Flapping tremor
Elevated central venous pressure
Gait ataxia
Clonus
Gait ataxia
Clonus
Petechial rash
Wernicke syndrome
Flapping tremor
Exophtholmos
Kernig's sign
Nystagmus
Elevated central venous pressure

Acute hepatic failure

1447
Kernig's sign
Clonus
Gait ataxia
Exophtholmos
Nystagmus
Elevated central venous pressure
Petechial rash
Flapping tremor
Check

Please note that this patient has recently had an emergency surgery of the femur. Don't try to
do Kernig's test on him; also gait ataxia may not be demonstrable. 
Wernicke's syndrome is an acute condition characterized by the well­known triad of
ophthalmoplegia, ataxia and global confusional state. The classic triad is only present in 10%
of patients, and 80% of patients are not diagnosed before postmortem. The eye signs are
often bilateral though need not be always symmetrical. 
Jaundice and flapping tremor (Asterixis) are features of hepatic failure. Asterixis can be
elicited by having the patient extend both arms with the wrists dorsiflexed and palms facing
forward, and eyes closed. Brief jerky downward movements of the wrist are considered a
positive sign. Asterixis is commonly seen with metabolic encephalopathies. (Note pronator
drift is elicited by having the patient extend both arms with the wrists supinated and palms
facing upwards and eyes closed ­ slow unequal drift towards pronation indicates hemiparetic
weakness). Elevated JVP is not seen in hepatic failure that is due to non­cardiac causes.
The correct answer is: Bacterial septicaemia – Petechial rash, Wernicke syndrome –
Nystagmus, Acute hepatic failure – Flapping tremor

Question 37 HiY Assessment EMI038
Not answered Complications of Eating Disorders
Identify the metabolic abnormalities commonly seen in the following clinical situations
Marked out of 2.00

Flag question
Hypercortisolemia
A 16­year­old girl was admitted to a Hyponatraemia
psychiatric unit for treatments as her
Hypokalemia
weight was dangerously low. She
Hypothyroidism
died suddenly when she was
administered parenteral nutrition. Low T3 and high growth hormone
Hypernatraemia
Hyperphosphatemia
High T3 and high growth hormone
Low T3 and high growth hormone
A 15­year­old girl with a history of Hyperkalemia
Hypernatraemia
anorexia was admitted to a medical Hypophosphatemia
Hyperkalemia
ward for severe dehydration. During
Hypothyroidism
fluid resuscitation, she went into a
state of coma. Hypercortisolemia
High T3 and high growth hormone
Hypophosphatemia
Hyponatraemia
Check
Hypokalemia
Hyperphosphatemia

1448
Refeeding syndrome is a complication that can occur within four days of restarting regular
feeding in an anorexic subject. This syndrome is due to the sudden shift from starvation­
induced fat metabolism to feeding­triggered carbohydrate metabolism. This results in a
sudden increase in insulin levels leading to increased cellular uptake of phosphate and a
relative scarcity of glucose and thiamine. Refeeding increases body's basal metabolic rate.
Following refeeding, electrolytes move inside cells causing a fall in the serum levels of
phosphate (hypophosphatemia), potassium and magnesium. This may progress from
confusion, coma, convulsions, to death. This shifting of electrolytes and fluid balance
increases the cardiac workload and heart rate increasing the likelihood of acute heart failure
compounded by previous starvation­induced weakening of cardiac strength. Oxygen
consumption is also increased which strains the respiratory system and can make weaning
from ventilation more difficult. 
If hypovolemia is rapidly corrected, it can cause hyponatraemia leading to central pontine
myelinolysis and coma. Apart from this, patients can also experience neuromuscular and
hematologic complications.
The correct answer is: A 16­year­old girl was admitted to a psychiatric unit for treatments as
her weight was dangerously low. She died suddenly when she was administered parenteral
nutrition. – Hypophosphatemia, A 15­year­old girl with a history of anorexia was admitted to a
medical ward for severe dehydration. During fluid resuscitation, she went into a state of coma.
– Hyponatraemia

Question 38 HiY Assessment EMI039
Not answered Bipolar Affective disorder (BPAD)
Select one diagnostic option for each of the following scenario
Marked out of 4.00

Flag question
Mixed affective state
A patient, who has been diagnosed Cyclothymia.
with depression and was started on Hypomania
venlafaxine, now presents a history of
Bipolar disorder­unspecified.
feeling high with 'unstoppable
energy'.
Bipolar type 1
Rapid cycling bipolar disorder.
Bipolar type 2
A patient who is known to have mood Affective instability
disturbances is taking lithium. In last Dysthymia
six months, he has had one episode
Cyclothymia.
each of low mood and mania,
followed by another episode of high
Affective instability
energy, sleep loss and euphoric Bipolar disorder­unspecified.
mood. After an intervening period of Mixed affective state
normal mood, he developed another Bipolar type 1
episode of mania. Hypomania
Bipolar type 2
A 33­year­old man presents with a Rapid cycling bipolar disorder.
history of numerous periods with
Cyclothymia.
hypomanic symptoms and numerous
periods with depressive symptoms in Rapid cycling bipolar disorder.
last two years that do not meet criteria Affective instability
for a mania or a major depressive Dysthymia
episode. Mixed affective state
Bipolar type 2
Bipolar type 1
Hypomania
Bipolar disorder­unspecified.

1449
A 25­year­old woman has episodes Bipolar disorder­unspecified.
of depressed mood and irritability. Affective instability
She described her energy levels to
Bipolar type 1
be high and is sleeping only for 4
Rapid cycling bipolar disorder.
hours a day.
Mixed affective state
Dysthymia
Bipolar type 2
Cyclothymia.
Check Hypomania

Case 1 can be described as bipolar 3; it is a variant used to describe minimal depression
complicated by antidepressant­induced hypomania ­ these patients fall into bipolar spectrum
(in ICD: this is coded as unspecified type). 
Case 2 is suggestive of rapid cycling, where at least four episodes of bipolar disorder occur
within a period of one year. 70­80% of rapid cyclers are women. Some of the factors
associated with the rapid cycling include the use of tricyclic anti­depressant, low thyroxin
level, being a female patient and having bipolar two rather than type 1illness. Ultra­rapid
cycling refers to the situation when fluctuations are over days or even hours. 
Case 3 is suggestive of cyclothymia in which an individual may experience oscillating high
and low moods (numerous periods of mild depression and mild elation), without ever having a
manic or depressive episode that is sufficiently severe or prolonged to fulfill the criteria for
BPAD or recurrent depressive disorder. The mood swings are usually as being unrelated to
life events. 
Mixed states are instances where manic and depressive symptoms occur simultaneously, as
seen in case 4 above. The occurrence of both manic/hypo manic and depressive symptoms in
a single episode, present every day for at least 1 week (DSM­5) or 2 weeks (ICD­10)
The correct answer is: A patient, who has been diagnosed with depression and was started
on venlafaxine, now presents a history of feeling high with 'unstoppable energy'. – Bipolar
disorder­unspecified., A patient who is known to have mood disturbances is taking lithium. In
last six months, he has had one episode each of low mood and mania, followed by another
episode of high energy, sleep loss and euphoric mood. After an intervening period of normal
mood, he developed another episode of mania. – Rapid cycling bipolar disorder., A 33­year­
old man presents with a history of numerous periods with hypomanic symptoms and
numerous periods with depressive symptoms in last two years that do not meet criteria for a
mania or a major depressive episode. – Cyclothymia., A 25­year­old woman has episodes of
depressed mood and irritability. She described her energy levels to be high and is sleeping
only for 4 hours a day. – Mixed affective state

Question 39 HiY Assessment EMI040
Not answered Diagnosis of anxiety disorders
Identify the most likely diagnosis using the clinical descriptions given below.
Marked out of 3.00

Flag question
Narcissistic personality disorder
Post­traumatic stress disorder
Adjustment disorder
A patient with long­standing anxiety Generalised Anxiety Disorder
describes living on the edge all his Bipolar Affective Disorder
life. He has recently deteriorated. He Anankastic personality disorder
is performing poorly at work. Acute stress reaction
Schizophrenia

1450
A 42­year­old mother with no Bipolar Affective Disorder
previous psychiatric history presents Narcissistic personality disorder
with confusion and highly anxious. Obsessive compulsive disorder
Her son developed an acute Adjustment disorder
neurological illness and had been Generalised Anxiety Disorder
hospitalised since the previous day. Acute stress reaction
Schizophrenia
Post­traumatic stress disorder
Generalised Anxiety Disorder
Anankastic personality disorder
Schizophrenia
A 45 year­old­woman is noted to Obsessive compulsive disorder
suffer from anxiety, depression, poor Post­traumatic stress disorder
concentration, irritability and is Anankastic personality disorder
unable to work for last four weeks. Acute stress reaction
She divorced her husband recently. Narcissistic personality disorder
Bipolar Affective Disorder
Adjustment disorder

Check

Generalised anxiety disorder (case 1) is characterised by prominent tension, excessive worry
with generalised free­floating persistent anxiety ('living on the edge') and feelings of
apprehension about everyday events leading to significant stress and functional impairment. 
Acute stress reaction (case 2) usually starts within an hour of the exposure to the stressful
event; resolution starts within 8 hours if the stress is temporary or after 48 hours if it is
prolonged. The presence of physical exhaustion, organic factors or physical disease states
increases the risk. The stressor is usually one that poses a serious threat to security, integrity
and social position. It has a polymorphic presentation, and the patient may initially be dazed
with narrowed attention; disorientation is not uncommon as a result.
Adjustment disorder (case 3) is a condition that refers to the psychological reactions arising in
relation to adapting to new circumstances and occurs in someone who has been exposed to a
psychosocial stressor like divorce, separation etc, which is not catastrophic in nature. The
usual presentations include anxiety, depression, poor concentration, irritability, anger, etc.
with physical symptoms caused by autonomic arousals such as tremor and palpitations. The
onset is more gradual than that of acute stress reaction, with course more prolonged. Onset
must be within one month of the stress event according to ICD and three months in DSM, with
some psychosocial dysfunction being required for the diagnosis to be made.
The correct answer is: A patient with long­standing anxiety describes living on the edge all his
life. He has recently deteriorated. He is performing poorly at work. – Generalised Anxiety
Disorder, A 42­year­old mother with no previous psychiatric history presents with confusion
and highly anxious. Her son developed an acute neurological illness and had been
hospitalised since the previous day. – Acute stress reaction, A 45 year­old­woman is noted to
suffer from anxiety, depression, poor concentration, irritability and is unable to work for last
four weeks. She divorced her husband recently. – Adjustment disorder

Question 40 HiY Assessment EMI041

1451
Not answered Diagnostic features of psychotic disorders
For each of the following diagnosis choose the most characteristic diagnostic feature that aids
Marked out of 8.00
in clinical diagnosis.
Flag question

Apathy
Symptoms last for only two weeks
Catatonic symptoms
Paranoid schizophrenia
Amnesia
Relatively preserved functioning
Behavioural and personality changes
Prominent somatisation
Prominent delusions and hallucinations
Symptoms last for only two weeks
Young adult or adolescent onset
Young adult or adolescent onset
Behavioural and personality changes
Persistent delusional disorder
Prominent delusions and hallucinations
Amnesia
Prominent somatisation
Apathy
Relatively preserved functioning
Prominent somatisation
Catatonic symptoms
Behavioural and personality changes
Apathy
Hebephrenic schizophrenia 
Catatonic symptoms
Amnesia
Symptoms last for only two weeks
Young adult or adolescent onset
Relatively preserved functioning
Behavioural and personality changes
Prominent delusions and hallucinations
Catatonic symptoms
Amnesia
Frontotemporal dementia
Relatively preserved functioning
Apathy
Prominent delusions and hallucinations
Symptoms last for only two weeks
Prominent somatisation
Check Young adult or adolescent onset

Paranoid schizophrenia: Prominent delusions and hallucinations are the most prominent
features of F20 paranoid schizophrenia; others in the list are mostly supplementary. Though
the age of onset is not a criterion for paranoid schizophrenia (unlike hebephrenia), it is a well­
known clinical observation that paranoid schizophrenia has a later age of onset compared to
hebephrenia or catatonia. 
A persistent delusional disorder is characterised by a later age of onset (mean 40yrs), and
symptoms must be present for more that three months for a diagnosis to be made. Social
functioning is somewhat preserved when compared to schizophrenia (according to DSM). 
Hebephrenic schizophrenia is characterised by emotional disturbances (incongruence or a
fatuous affect), abnormal premorbid personality and a younger age of onset.
The correct answer is: Paranoid schizophrenia
– Prominent delusions and hallucinations, Persistent delusional disorder
– Relatively preserved functioning, Hebephrenic schizophrenia 
– Young adult or adolescent onset, Frontotemporal dementia

1452
– Behavioural and personality changes

Question 41 HiY Assessment EMI042
Not answered Syndromes in psychopathology 
Find one most appropriate diagnosis/eponym each for the syndromes described below:
Marked out of 3.00

Flag question
Ekbom's syndrome
A 73­year­old gentleman who has Folie a Deux
been grossly neglecting himself, his Diogenes syndrome
flat being untidy and filthy, hoards up Cotard's syndrome
objects of no practical value. Dhat syndrome
Capgras syndrome
Couvade syndrome
De Clerembault syndrome
Cotard's syndrome
Fregoli syndrome
De Clerembault syndrome
A 46­year­old man is convinced that Formication
Ganser's syndrome
is skin is infested with multiple, tiny Ganser's syndrome
Formication
mites and insects.
Dhat syndrome
Capgras syndrome
Diogenes syndrome
Couvade syndrome
Mrs. Zakaria is a 30­year­old woman Cotard's syndrome
who has recently become pregnant. Fregoli syndrome
Dhat syndrome
Folie a Deux
Her husband has developed morning De Clerembault syndrome
sickness, abdominal pain and Ekbom's syndrome
Ganser's syndrome
unusual food cravings after he found Capgras syndrome
out that she was pregnant Ekbom's syndrome
Diogenes syndrome
Fregoli syndrome
Check Folie a Deux
Formication
Couvade syndrome
Diogenes syndrome (case 1) is characterized by hoarding of objects, usually of no practical
use, and the neglect of one's home or the environment. It may present as a behavioural
manifestation of various conditions including organic disorders, schizophrenia, depressive
disorder, or obsessive­compulsive disorder, or reflect a reaction late in life to stress in a
certain type of personality. 
Ekbom syndrome (case 2) is a monosymptomatic delusional disorder where the core delusion
is a delusion of infestation. 
A conversion reaction with symptoms mimicking pregnancy (nausea, vomiting, abdominal
pain, and food cravings) sometimes seen in partners of pregnant mothers is called Couvade
syndrome (case 3). It is not delusional in nature; the affected individual does not believe they
are pregnant (c.f. pseudocyesis).
The correct answer is: A 73­year­old gentleman who has been grossly neglecting himself, his
flat being untidy and filthy, hoards up objects of no practical value. – Diogenes syndrome, A
46­year­old man is convinced that is skin is infested with multiple, tiny mites and insects. –
Ekbom's syndrome, Mrs. Zakaria is a 30­year­old woman who has recently become pregnant.
Her husband has developed morning sickness, abdominal pain and unusual food cravings
after he found out that she was pregnant – Couvade syndrome

1453
Question 42 HiY Assessment EMI043
Not answered Investigations in the elderly
For each of the following clinical conditions below, match the correct investigations to be
Marked out of 6.00
performed from the given list.
Flag question

EEG
77­year­old lady admitted to the ward MRI Scan
with a history of an acute confusional Mid stream urine
state. The patient passed urine
Chest radiography
several times at nighttime and was
foul smelling. 
Liver function tests
SPECT Scan
Urea and electrolytes
CT scan
MRI Scan
79­year­old lady who lives on her ECG
Liver function tests
own admits to drinking three glasses Thyroid function tests
SPECT Scan
of wine everyday when CPN visited
Thyroid function tests
her. She is admitted to the ward for
depression and alcohol misuse. 
Chest radiography
Mid stream urine
CT scan
83­year­old lady with a history of ECG
EEG
diabetes and hypertension was EEG
SPECT Scan
admitted to the ward with a history of Urea and electrolytes
ECG
gradual intellectual decline,
Thyroid function tests
depression and cognitive impairment.
She is unsteady on her feet and is
Mid stream urine
also dysarthric.  Urea and electrolytes
Liver function tests
CT scan
SPECT Scan
77­year­old lady who is on lithium MRI Scan
Thyroid function tests
augmentation for her depressive Chest radiography
MRI Scan
symptoms is constipated, lethargic
Chest radiography
and has gained weight despite poor
appetite in the last six months.
CT scan
Mid stream urine
Liver function tests
Urea and electrolytes
Check ECG
EEG

Case 1 is suggestive of a woman possibly suffering from urinary tract infection. Full blood
count (Leucocytosis may indicate the seriousness of the UTI) and mid­stream urine are
investigations of choice. 
Case 2 refers to chronic alcoholism. Raised Gamma­glutamyl transferase (GGT) and raised
mean corpuscular volume (MCV) are important features seen in alcoholic subjects. 
Case 3 refers to vascular dementia (Binswanger's disease). MRI is the investigation of choice
for patients with vascular dementia, subcortical type. 
Case 4 describes a patient who has developed hypothyroidism, which occurs on long­term
treatment with Lithium carbonate. Abnormal thyroid function test, especially raised TSH levels,
is seen in patients with lithium­induced hypothyroidism.
The correct answer is: 77­year­old lady admitted to the ward with a history of an acute
confusional state. The patient passed urine several times at nighttime and was foul smelling. 

1454
– Mid stream urine, 79­year­old lady who lives on her own admits to drinking three glasses of
wine everyday when CPN visited her. She is admitted to the ward for depression and alcohol
misuse. 
– Liver function tests, 83­year­old lady with a history of diabetes and hypertension was
admitted to the ward with a history of gradual intellectual decline, depression and cognitive
impairment. She is unsteady on her feet and is also dysarthric. 
– MRI Scan, 77­year­old lady who is on lithium augmentation for her depressive symptoms is
constipated, lethargic and has gained weight despite poor appetite in the last six months. –
Thyroid function tests

Question 43 HiY Assessment EMI044
Not answered Common features of personality disorders
Choose one option from the list above for each question below.
Marked out of 4.00

Flag question A 28­year­old man has complained to
local council numerous times about
his neighbours throwing rubbish into Schizotypal
his garden. Being vengeful, he Anankastic
emptied his weekly rubbish collection Narcissistic
into their backyard before going in
Paranoid
person to the council office to make a
complaint. He has always had trouble
Emotionally unstable: borderline type
with different neighbours and has Dissocial
changed nine houses in last three Anxious/avoidant
years. Emotionally unstable: impulsive type
Histrionic
A 34­year­old lady who is attending
outpatient clinic regularly becomes Dependent
Anxious/avoidant
angry when asked to see a registrar Schizoid
Histrionic
during a visit as the consultant was Paranoid
on holiday. She swears vehemently
Dissocial
at the receptionist and kicks a flower
vase in the waiting room. She
Emotionally unstable: impulsive type
threatens to kill herself if the Anankastic
consultant does not see her. Dependent
Schizoid
A 37­year­old lady believes in
Narcissistic
Emotionally unstable: impulsive type
clairvoyance and sixth sense. She
and her sister belong to an occult
Emotionally unstable: borderline type
Anxious/avoidant
group, which believes in Schizotypal
Narcissistic
communicating with 'medium' to Anankastic
enhance one's spirit. She collects Schizotypal
carcases of animals that die on Paranoid
country roads to preserve their hides. Dissocial
A 41­year­old man insists his GP on Emotionally unstable: borderline type
being referred to a psychiatric Schizoid
consultant. He presented with Dependent
Emotionally unstable: impulsive type
depression but has not responded to Narcissistic
Histrionic
initial pharmacotherapy. He has a Schizotypal
personal history of financial
Emotionally unstable: borderline type
bankruptcy. He maintains that all his
seven children born to his divorced
Anankastic
wife are in very high positions, and Histrionic
they would not want to see him 'being Dependent
so down.' Anxious/avoidant
Paranoid

1455
Check

The first case is suggestive of paranoid personality disorder where patients are suspicious of
the motives of others, hold longstanding grudges against people, and believe others are not
trustworthy. 
In emotionally unstable personalities (case 2), minimal ability to plan ahead may be seen,
similar to some patients with antisocial traits. In the impulsive variant, lack of self­control and
emotional instability are prominent; in the borderline variant, poor self­image, and chronic
emptiness are prominent. 
Patients with schizotypal personality disorder such as the one described in case 3 appear odd
or eccentric, may have illusions and magical thinking, obsessions without resistance, may be
members of quasi­cultural groups, may believe in extra­sensory perceptions (paranormality),
clairvoyance etc, may have transient psychotic features. 
Case 4 is an example of narcissistic personality disorder, where patients show exaggerated
sense of own importance, frequently self­centered, and being intolerant of other people, with a
lot of grandiose plans and ideas and seek admiration. Fear of dependency is the core
conflictual theme in narcissism with injuries to self­pride leading to rage reactions.
The correct answer is: A 28­year­old man has complained to local council numerous times
about his neighbours throwing rubbish into his garden. Being vengeful, he emptied his weekly
rubbish collection into their backyard before going in person to the council office to make a
complaint. He has always had trouble with different neighbours and has changed nine
houses in last three years. – Paranoid, A 34­year­old lady who is attending outpatient clinic
regularly becomes angry when asked to see a registrar during a visit as the consultant was on
holiday. She swears vehemently at the receptionist and kicks a flower vase in the waiting
room. She threatens to kill herself if the consultant does not see her. – Emotionally unstable:
impulsive type, A 37­year­old lady believes in clairvoyance and sixth sense. She and her
sister belong to an occult group, which believes in communicating with 'medium' to enhance
one's spirit. She collects carcases of animals that die on country roads to preserve their hides.
– Schizotypal, A 41­year­old man insists his GP on being referred to a psychiatric consultant.
He presented with depression but has not responded to initial pharmacotherapy. He has a
personal history of financial bankruptcy. He maintains that all his seven children born to his
divorced wife are in very high positions, and they would not want to see him 'being so down.' –
Narcissistic

Question 44 HiY Assessment EMI045
Not answered Childhood & developmental disorders
For each of the following vignette below choose one correct answer from the given list:
Marked out of 4.00

Flag question A 7­year­old boy is frequently
skipping school with complaints of
tummy ache, diarrhea, back pain and Disintegrative disorder
tiredness. He is increasingly refusing Schizophrenia ­ childhood onset
to sleep on his own and wants to Separation anxiety disorder
share his parents' bedroom. He does ADHD
not want his mum to leave him at Pervasive developmental disorder
home and go out on her own as he is Conduct disorder
scared of 'black monsters' in the Truancy
house. School refusal
Normal adolescence
Depressive episode
Juvenile delinquency

1456
A family with three children aged 11, Truancy
15 and 18 has recently migrated to
Depressive episode
Leeds from London. The 11­year­old
boy has developed troublesome Normal adolescence
stomach ache and back pain since Conduct disorder
the relocation. The symptoms ease Schizophrenia ­ childhood onset
off during the day if he stays at homeBipolar depression
taking bed­rest. The GP could not findSeparation anxiety disorder
any physical cause. He seems free of Disintegrative disorder
these problems over the weekends. School refusal
A 15­year­old girl has changed a lot ADHD
in last six months according to her Juvenile delinquency
mother. She isolates herself in her Pervasive developmental disorder
room most of the time, talking to few
Truancy
selected friends on her mobile phone Bipolar depression
most of the time. She spends much Normal adolescence
time on the internet chat rooms and Depressive episode
social networks. She has little Juvenile delinquency
conversation if any with her 8­year­ Pervasive developmental disorder
old brother and her parents, though Schizophrenia ­ childhood onset
her academic progress has been Conduct disorder
satisfactory.
Separation anxiety disorder
School refusal
Truancy
A 14­year­old girl with anhedonia
Disintegrative disorder
Conduct disorder
and poor concentration, poor
appetite, and insomnia lacks
ADHD
Separation anxiety disorder
motivation to do anything and is ADHD
participating poorly during activities Pervasive developmental disorder
at her school. Juvenile delinquency
Depressive episode
Disintegrative disorder
Check Schizophrenia ­ childhood onset
School refusal
Bipolar depression
Normal adolescence
Multiple somatic complaints in children is most often due to an internalizing emotional
problem that can present as school refusal (case 2). In separation anxiety (case 1), school
refusal is present along with an anxiety of leaving the caregiver. The latter is often not overtly
described, but various reasons such as fear of dark or monsters are invoked to ensure the
presence of the caregiver in proximity. 
It is normal to stay aloof from family and indulge in one's age­specific interests during
adolescence (case 3).
Major depression is the most severe condition in adolescents, with either "sad or irritable
mood, or anhedonia, along with at least five other symptoms, such as social withdrawal,
worthlessness, guilt, suicidal thoughts or behaviour, sleep increase or decrease, decreased
motivation and/or concentration, and increased or decreased appetite" (DSM­5). A crucial
distinction between depression as an illness and the normal mood variations of childhood
and adolescence is the association of functional impairment with depression, in addition to a
lack of responsiveness of the depressed mood to contextual changes.
The correct answer is: A 7­year­old boy is frequently skipping school with complaints of tummy
ache, diarrhea, back pain and tiredness. He is increasingly refusing to sleep on his own and
wants to share his parents' bedroom. He does not want his mum to leave him at home and go
out on her own as he is scared of 'black monsters' in the house. – Separation anxiety disorder,
A family with three children aged 11, 15 and 18 has recently migrated to Leeds from London.
The 11­year­old boy has developed troublesome stomach ache and back pain since the

1457
relocation. The symptoms ease off during the day if he stays at home taking bed­rest. The GP
could not find any physical cause. He seems free of these problems over the weekends. –
School refusal, A 15­year­old girl has changed a lot in last six months according to her
mother. She isolates herself in her room most of the time, talking to few selected friends on her
mobile phone most of the time. She spends much time on the internet chat rooms and social
networks. She has little conversation if any with her 8­year­old brother and her parents,
though her academic progress has been satisfactory. – Normal adolescence, A 14­year­old
girl with anhedonia and poor concentration, poor appetite, and insomnia lacks motivation to
do anything and is participating poorly during activities at her school. – Depressive episode

Question 45 HiY Assessment EMI046
Not answered Classification of disorders in the elderly
For each of the following clinical conditions below, match the associated psychiatric syndrome
Marked out of 3.00
from the given list:
Flag question

A 77­year­old widowed gentleman Late onset paraphrenia
became increasingly isolated over Acute confusional state
the last two years and starting to Diogenes syndrome
Adjustment disorder
neglect himself. He lives in filthy, poor
conditions. Although necessitating Old age depression
intervention, he was resistant to help.
Pseudodementia
Chronic confusional state
A 78­year­old lady with hearing Hypomania
Diogenes syndrome
impairment and who lives on her own Korasakoff's psychosis
Korasakoff's psychosis
believes that the neighbours are Charles Bonnet syndrome
spying on her. She can also hear
Pseudodementia
neighbours talking about her all the Adjustment disorder
time through the walls. Charles Bonnet syndrome
Chronic confusional state
Hypomania
An 87­year­old lady with a history of Late onset paraphrenia
Pseudodementia
falls and was brought to the A&E by Acute confusional state
Korasakoff's psychosis
the paramedics. She is agitated, Old age depression
Chronic confusional state
paranoid, confused and also has
Diogenes syndrome
visual hallucinations. She was
previously reported to be a high
Acute confusional state
functioning lady Old age depression
Charles Bonnet syndrome
Adjustment disorder
Check
Late onset paraphrenia
Hypomania

Case 1 is an example of Diogenes syndrome (or 'senile squalor syndrome') which refers to
severe self­neglect seen in somewhat eccentric and reclusive elderly individuals. They
become increasingly isolated and neglect themselves, living in filthy, poor conditions. It can
be seen as the response of someone with a particular personality type to the hardships of old
age and loneliness (Howard & Bergman 1993). They are often oblivious to their condition and
resistant to help, making management particularly challenging. 
Case 2 depicts late onset paraphrenia. Persecutory delusions are the most common
symptoms of late paraphrenia; they are found in around 90% of patients. Auditory
hallucinations occur in approximately 75% of cases. Visual hallucinations are observed in up
to 60% of patients. Few patients may present with delusions only (10­20%). Partition

1458
delusions (attack through the wall, neighbours spying via any 'partition') are
common.According to ICD patients, must either be diagnosed as having delusional disorder
or schizophrenia ­ no separate diagnosis exists for paraphrenia. 
Case 3 refers to delirium. The salient features rapid onset of clinical features with fluctuations
in severity over minutes and hours (even back to apparent normality), clouding of
consciousness with reduced attention span and distractibility, global impairment of cognition
with disorientation, and impairment of recent memory and abstract thinking.
The correct answer is: A 77­year­old widowed gentleman became increasingly isolated over
the last two years and starting to neglect himself. He lives in filthy, poor conditions. Although
necessitating intervention, he was resistant to help. – Diogenes syndrome, A 78­year­old lady
with hearing impairment and who lives on her own believes that the neighbours are spying on
her. She can also hear neighbours talking about her all the time through the walls. – Late
onset paraphrenia, An 87­year­old lady with a history of falls and was brought to the A&E by
the paramedics. She is agitated, paranoid, confused and also has visual hallucinations. She
was previously reported to be a high functioning lady – Acute confusional state

Question 46 HiY Assessment EMI047
Not answered Drug misuse and clinical presentations
Choose one option for each question below
Marked out of 3.00

Flag question
A 22­year­old university student
Korsakoff's psychosis.
develops intense pounding Cannabis intoxication.
headaches, marked fatigue anxiety Cannabis withdrawal.
and nausea immediately after his Opiate intoxication
exams. He denies using alcohol or Amphetamine use
smoking. Opiate withdrawal
Caffeine withdrawal
Cocaine intoxication
A 30­year­old man has panic attacks Cocaine intoxication
Cocaine withdrawal.
Opiate withdrawal
and recurrent cough. He has a red
eye on examination but has no Korsakoff's psychosis.
needle marks. He appears quite Cannabis withdrawal.
apathetic and unconcerned. Caffeine withdrawal
Cocaine withdrawal.
Amphetamine use
Opiate intoxication
Opiate intoxication
Cannabis intoxication.
Cocaine intoxication
A 44­year­old truck driver presents
with intense paranoia and anxiety; he Caffeine withdrawal
has tachycardia and reports having Opiate withdrawal
ants crawling under his clothes. Korsakoff's psychosis.
Amphetamine use
Cannabis withdrawal.
Cannabis intoxication.
Check Cocaine withdrawal.

Caffeine Withdrawal is not recognised in DSM but is a likely diagnosis in a student who has
just appeared for an exam (case 1). DSM research criteria lists headache, marked fatigue or
drowsiness, marked anxiety or depression, nausea or vomiting as the prominent features that
can be seen even at low doses (from 100mg/day), beginning within 12 hours, peaking at 24­
48 hours and lasting up to 1 week. 

1459
Cannabis Intoxication (case 2) is characterised by impaired motor coordination, euphoria,
sensation of slowed time, social withdrawal, conjunctival injection (red eye), increased
appetite (munchies), dry mouth and tachycardia. Impairment in motor skills remains after the
euphoriant effects are gone. 
Some patients (case 3) under cocaine intoxication report a sensation of bugs crawling
beneath the skin (formication). It is associated with increased energy, increased confidence,
heightened self­esteem, and perceived improvement on mental and physical tasks, euphoria,
and diminished need for sleep. With high doses agitation, irritability, impaired judgment,
impulsive aggression, some symptoms of mania with associated tachycardia, hypertension,
and mydriasis are noted.
The correct answer is: A 22­year­old university student develops intense pounding
headaches, marked fatigue anxiety and nausea immediately after his exams. He denies using
alcohol or smoking. – Caffeine withdrawal, A 30­year­old man has panic attacks and recurrent
cough. He has a red eye on examination but has no needle marks. He appears quite
apathetic and unconcerned. – Cannabis intoxication., A 44­year­old truck driver presents with
intense paranoia and anxiety; he has tachycardia and reports having ants crawling under his
clothes. – Cocaine intoxication

Question 47 HiY Assessment EMI048
Not answered Supportive techniques while interviewing
For each of the following examples, identify the type of supportive techniques aimed at
Marked out of 4.00
eliciting information
Flag question

Postponement
Encouragement
Reassurance
Let's plan on working on this together
Statement of respect
Acknowledgment of affect
Support
Partnering
Advice
Postponement
Validation
Support
I would like to offer whatever help I Advice
can Acknowledgment of affect
Statement of respect
Partnering
Reassurance
Validation
Validation
I can see that you are finding it Encouragement
difficult to tell me about your
Advice
relationships. It is fine, and we can Encouragement
come back to this when you feel Partnering
ready to discuss this with me Support
Statement of respect
Postponement
Reassurance
Acknowledgment of affect
In the past, you have managed this
difficult situation very well, and I think
you can manage it again.

1460
Statement of respect
Validation
Partnering
Reassurance
Encouragement
Acknowledgment of affect
Postponement
Support
Check Advice

Partnering: The interviewer encourages the patient to ask questions and to express any
concerns, encouraging team working. Support: Emphasising the interviewer's role in helping.
Postponement: Conscious and deliberate postponement of delicate issues; but must be
opened at an appropriate time. Encouragement: to inspire with the courage or confidence to
do something.
The correct answer is: Let's plan on working on this together – Partnering, I would like to offer
whatever help I can – Support, I can see that you are finding it difficult to tell me about your
relationships. It is fine, and we can come back to this when you feel ready to discuss this with
me – Postponement, In the past, you have managed this difficult situation very well, and I think
you can manage it again. – Encouragement

Question 48 HiY Assessment EMI049
Not answered Defining defence mechanisms
For each of the definitions given below, identify the most likely defence mechanisms from the
Marked out of 4.00
list provided.
Flag question

Identification with the aggressor
Reaction formation
Unconscious attribution of one's Repression
attitudes and urges to other persons
Denial
due to the painful affect aroused.
Projection
Isolation
Acting out
Splitting
Repression
Introjection
Introjection
Sublimation
Sublimation
Isolation
Projection
Unconscious transformation of Reaction formation
unacceptable impulses into exactly Denial
opposing attitudes, impulses, feelings
Splitting
or behaviours
Acting out
Identification with the aggressor

1461
Expression of an unconscious Repression
impulse through an action, thereby Reaction formation
gratifying the impulse Introjection
Splitting
Sublimation
Identification with the aggressor
Denial
Projection
Sublimation
Isolation
Introjection
Unconscious internalisation of the Acting out
Repression
qualities of an object or person Identification with the aggressor
Reaction formation
Splitting
Acting out
Denial
Check
Isolation
Projection

Projection: This refers to perceiving and reacting to unacceptable inner impulses as though
they originated outside the self. For example, the person who attributes hostility to others may
be unconsciously projecting their hostility. Thus, internal threats become externalised and
then are easier to handle. 
Expressing an unconscious impulse through explicit acts to avoid the unpleasant awareness
of the accompanying affect is called Acting Out. In other words, an unconscious wish is
expressed impulsively through a behavioural act, thus gratifying instead of prohibiting the
impulse. 
Reaction formation: The transforming of an unacceptable wish into its exact antithesis is
called Reaction Formation. It is seen in patients with neuroses such as OCD, and with
recurrent use from any early stage of ego development, it can also result in obsessional
personality. 
Introjection refers to the unconscious internalization of qualities of an object or person. It is
seen in depression, grief reaction and is featured by identification with the aggressor (Eg
sometimes seen in victims kidnapped by terrorists, also called as Stockholm syndrome).
The correct answer is: Unconscious attribution of one's attitudes and urges to other persons
due to the painful affect aroused. – Projection, Unconscious transformation of unacceptable
impulses into exactly opposing attitudes, impulses, feelings or behaviours – Reaction
formation, Expression of an unconscious impulse through an action, thereby gratifying the
impulse – Acting out, Unconscious internalisation of the qualities of an object or person –
Introjection

Question 49 HiY Assessment EMI050
Not answered Motor disorders in psychiatric clinic
Choose one option for each of the following descriptions
Marked out of 4.00

Flag question

1462
Jerky movements which can be Stupor
voluntarily suppressed for a brief
Chorea
period and is habitual
Tardive dyskinesia
Pill rolling tremor
Dystonia
Rigidity
Intention tremor
Tics
Hemiballismus
Hemiballismus
Stupor
Myoclonus
Shock­like muscle contractions, Rigidity
which can be local or generalised Pill rolling tremor
Intention tremor
Tics
Dystonia
Chorea
Tardive dyskinesia
Myoclonus
Myoclonus
Rhythmic alternating contractions of Tardive dyskinesia
Pill rolling tremor
agonist and antagonist muscles
Stupor
occur with arms outstretched
Hemiballismus
Dystonia
Rigidity
Tics
Pill rolling tremor
Intention tremor
Tics
Violent flinging movement of half of Chorea
Dystonia
the body Stupor
Myoclonus
Intention tremor
Tardive dyskinesia
Chorea
Check Rigidity
Hemiballismus

Tics are rapid, repetitive, involuntary contractions of a group of muscles. Motor tics can affect
any part of the body, but they often involve the muscles of the face, eyes, head and neck. 
Myoclonus refers to sudden, involuntary jerking of a muscle or group of muscles. Myoclonic
twitches or jerks usually are caused by sudden muscle contractions. 
The pill­rolling tremor has a frequency typically between 4 and 7 Hz and consists of coarse,
alternating agonist­antagonist muscle movements. It is a rest tremor that is present when the
patient is alert but not moving volitionally. With purposeful movements, the tremor decreases. 
Violent twitching and spasms involving one­half of the body (hemiballismus) can result from a
lesion of the contralateral subthalamic nucleus.
From Tourette Syndrome. (n.d.). Retrieved from
http://my.clevelandclinic.org/health/diseases_conditions/hic_Tourette_Syndrome
The correct answer is: Jerky movements which can be voluntarily suppressed for a brief
period and is habitual – Tics, Shock­like muscle contractions, which can be local or
generalised – Myoclonus, Rhythmic alternating contractions of agonist and antagonist
muscles occur with arms outstretched – Pill rolling tremor, Violent flinging movement of half of
the body – Hemiballismus

1463
Question 50 HiY Assessment EMI037
Not answered Clinical rating scales 
For each description given below, select the relevant rating scale from the list above;
Marked out of 4.00

Flag question
General health questionnaire (GHQ)
Yale­Brown obsessive compulsive scales (YBOCS)
This scale is useful to assess the Positive and negative symptoms scale (PANSS)
severity of various mental illnesses Mini mental state examination (MMSE)
Eysenck personality questionnaire (EPQ)
Clinical Global impressions scale (CGI)
Young mania rating scale
Hospital anxiety and depression scale (HADS)
Hospital anxiety and depression scale (HADS)
Minnesota multiphasic personality inventory (MMPI)
Positive and negative symptoms scale (PANSS)
Brief psychiatric rating scale (BPRS)
Clinical Global impressions scale (CGI)
11 item observer scale used in mania
Minnesota multiphasic personality inventory (MMPI)
Yale­Brown obsessive compulsive scales (YBOCS)
Mini mental state examination (MMSE)
Young mania rating scale
Brief psychiatric rating scale (BPRS)
Young mania rating scale
Eysenck personality questionnaire (EPQ)
Yale­Brown obsessive compulsive scales (YBOCS)
Clinician administered rating scale
for assessment of severity and General health questionnaire (GHQ)
Hospital anxiety and depression scale (HADS)
monitoring the change of symptoms Positive and negative symptoms scale (PANSS)
in patients with schizophrenia Mini mental state examination (MMSE)
General health questionnaire (GHQ)
Minnesota multiphasic personality inventory (MMPI)
Brief psychiatric rating scale (BPRS)
Hospital anxiety and depression scale (HADS)
Eysenck personality questionnaire (EPQ)
Brief psychiatric rating scale (BPRS)
Clinical Global impressions scale (CGI)
Yale­Brown obsessive compulsive scales (YBOCS)
Young mania rating scale
19­item test to assess cognitive
functions
Eysenck personality questionnaire (EPQ)
Mini mental state examination (MMSE)
Positive and negative symptoms scale (PANSS)
General health questionnaire (GHQ)
Clinical Global impressions scale (CGI)
Minnesota multiphasic personality inventory (MMPI)
Check

BPRS: This scale is useful to assess the severity of severe mental illness. It is an 18­item
questionnaire widely used measure of psychotic symptoms and not just schizophrenia
(BPRS). 
PANNS: It is a Clinician administered rating scale for assessment of severity and monitoring of
change of symptoms in patients with a diagnosis of schizophrenia. 
YMRS: Young mania rating scale is an 11­item observer scale useful in the assessment of
mania. 
The Mini­Mental State Examination (MMSE) is the standard screening instrument for
dementia. It takes 5­10minutes to administer and introduced by Folstein. It is a brief tool for
grading cognitive impairment in elderly and screening for dementia. It is not very sensitive for

1464
change but has been used in dementia drug trials. It does not pick up frontal executive
defects.
The correct answer is: This scale is useful to assess the severity of various mental illnesses –
Brief psychiatric rating scale (BPRS), 11 item observer scale used in mania – Young mania
rating scale, Clinician administered rating scale for assessment of severity and monitoring the
change of symptoms in patients with schizophrenia – Positive and negative symptoms scale
(PANSS), 19­item test to assess cognitive functions – Mini mental state examination (MMSE)

Finish review

1465
 Home HiYield Paper Pharmacology

HiYield Paper A(2)

Started on Sunday, 10 May 2015, 11:59 AM
State Finished
Completed on Saturday, 16 May 2015, 6:49 PM
Time taken 6 days 6 hours
Marks 85.65/95.00
Grade 90.16 out of 100.00

Question 1 HiY Pharmacology EMI051
Correct History of Pharmacology (2)
For each of the following historical figures choose one most appropriate discovery made from
Mark 5.00 out of
the above list;
5.00

Flag question
Diazepam
Fluphenazine depot
Lithium
The first antidepressant
Risperidone Depot
agent to be discovered
Iproniazid
Haloperidol
Zimeldine
Chlordiazepoxide
Chlorpromazine
Diazepam
Fluoxetine
Fluphenazine depot
The first antipsychotic Lithium
shown to have Risperidone Depot
therapeutic effects Iproniazid
Haloperidol
Zimeldine
Chlordiazepoxide
Chlorpromazine
Diazepam
Fluoxetine
Fluphenazine depot
Lithium
Risperidone Depot
The first depot
Iproniazid
injectable forms of
antipsychotic Haloperidol
medication introduced Zimeldine
in the market. Chlordiazepoxide
Chlorpromazine
Fluoxetine

1466
Diazepam
Fluphenazine depot
Lithium
The first SSRI to be
Risperidone Depot
launched
Iproniazid
Haloperidol
Zimeldine
Chlordiazepoxide
Chlorpromazine
Diazepam
Fluoxetine
Fluphenazine depot
The first Lithium
benzodiazepine to be Risperidone Depot
introduced Iproniazid
Haloperidol
Zimeldine
Chlordiazepoxide
Chlorpromazine
Check
Fluoxetine


Explanation: 
Cade in Australia (1949) discovered the use of lithium compounds in mania. 
In 1950­52 presurgical antihistamine agent chlorpromazine was shown to have antipsychotic
effects independently by Delay and Deniker's team, and Charpentier from Rhône­Poulenc in
France.
In 1952, the first true antidepressant was discovered. Iproniazid was being used in treating
tuberculosis and was found to have mood­lifting properties. But hypertensive reactions
precluded large­scale use. Imipramine manufactured as chlorpromazine derivative came to
market soon. 
In 1954 the first benzodiazepine, chlordiazepoxide (Librium) was discovered serendipitously
by the Austrian scientist Leo Sternbach. 
Carlssen synthesized purpose made SSRI Zimeldine ­ but this was later withdrawn due to an
unacceptably high incidence of hypersensitivity syndrome and demyelinating disease.
In1970s Fluoxetine was tested as a noradrenaline reuptake inhibitor but was discarded as it
had a poor activity in this regard. Later it was rediscovered as serotonin reuptake inhibitor,
reaching market in 1987. 
The first depot injectable forms of antipsychotic medication, Fluphenazine enanthate
(Moditen) and decanoate (modecate) were introduced in 1968 and stimulated the
development of community psychiatric nursing services.
The correct answer is: The first antidepressant agent to be discovered – Iproniazid, The first
antipsychotic shown to have therapeutic effects – Chlorpromazine, The first depot injectable
forms of antipsychotic medication introduced in the market. – Fluphenazine depot, The first
SSRI to be launched – Zimeldine, The first benzodiazepine to be introduced –
Chlordiazepoxide

Question 2 HiY Pharmacology EMI052
Correct Adverse effects (4)
For each of the following description of adverse effects, choose the most likely offending
Mark 4.00 out of
agent.

1467
4.00
Olanzapine
Flag question A 40 year­old­ Mirtazapine
gentleman developed Mianserin
prolonged painful
Sodium valproate
erection on
antidepressant
Carbamazepine
treatment Paroxetine
Clozapine
Reboxetine
Trazodone
Olanzapine
Lithium
Mirtazapine
A 23­year­old
gentleman commenced Amisulpride
Mianserin
on an antipsychotic has Sodium valproate
a significantly raised Carbamazepine
prolactin. Paroxetine
Clozapine
Reboxetine
Trazodone
Olanzapine
A 20­year­old
Lithium
Mirtazapine
gentleman with first
episode psychosis Amisulpride
Mianserin
develops significant Sodium valproate
weight gain on Carbamazepine
treatment with an Paroxetine
antipsychotic. Clozapine
Reboxetine
Trazodone
Olanzapine
A 45­year­old lady on Lithium
Mirtazapine
suddenly stopping the Amisulpride
Mianserin
antidepressant became
Sodium valproate
anxious, felt electric
Carbamazepine
shock sensations and
became dizzy Paroxetine
Clozapine
Reboxetine
Trazodone
Check
Lithium
Amisulpride

Explanation: 
Trazodone is associated with priapism, which can be serious if unattended and is due to due
to alpha­1 receptor antagonism. The first step in the emergency management of priapism is
an intracavernosal injection of an alpha1 agonist such as metaraminol or epinephrine.
Amisulpride can cause dose­dependent EPSE and prolactinaemia at higher doses. PET
studies have indicated that 60%­80% occupation of D2 receptors is associated with
antipsychotic efficacy. Higher occupancy levels are associated with an increased risk of acute
extrapyramidal symptoms as well as hyperprolactinemia from the blocking of D2 receptors on
anterior­pituitary mammotrophic cells (these cells are normally tonically inhibited by
dopamine produced in the hypothalamic arcuate nucleus). 
Atypical antipsychotics such as olanzapine are recommended for the treatment of first episode
psychosis. Olanzapine is most commonly associated with significant weight gain. 

1468
The abrupt withdrawal of SSRI especially paroxetine or fluvoxamine (shorter half­life), is
associated with dizziness, nausea, headache, depression, anxiety, insomnia, poor
concentration, flu­like symptoms, paresthesias, electric shock­like symptoms and migraine­
like symptoms.
The correct answer is: A 40 year­old­gentleman developed prolonged painful erection on
antidepressant treatment – Trazodone, A 23­year­old gentleman commenced on an
antipsychotic has a significantly raised prolactin. – Amisulpride, A 20­year­old gentleman with
first episode psychosis develops significant weight gain on treatment with an antipsychotic. –
Olanzapine, A 45­year­old lady on suddenly stopping the antidepressant became anxious, felt
electric shock sensations and became dizzy – Paroxetine

Question 3 HiY Pharmacology EMI053
Correct Adverse effects (5)
Which of the drugs in the given list most likely fits the adverse effect profile described below?
Mark 5.00 out of
5.00

Flag question
Amitriptyline
An 83­year­old lady Trazodone
suffered dizziness and Fluoxetine
incurred falls on Chlorpromazine
starting this Venlafaxine
antidepressant Olanzapine
Sodium valproate
Clozapine
Thioridazine
Amitriptyline
A 45­year­old patient Mirtazapine
Trazodone
with bipolar disorder on Fluoxetine
Carbamazepine
treatment develops
Chlorpromazine
acute abdominal pain
Venlafaxine
and significantly raised
serum amylase levels Olanzapine
Sodium valproate
Clozapine
Thioridazine
Amitriptyline
A 45­year­old
Mirtazapine
Trazodone
gentleman with
treatment­resistant Carbamazepine
Fluoxetine
schizophrenia started Chlorpromazine
on a new antipsychotic Venlafaxine
developed chest pain, Olanzapine
tachycardia, Sodium valproate
tachypnoea and
Clozapine
palpitations
Thioridazine
Mirtazapine
Carbamazepine
A 45­year­old chronic
schizophrenia patient
Amitriptyline
on longstanding
Trazodone
treatment with an Fluoxetine
antipsychotic Chlorpromazine
medication has Venlafaxine
pigmentary retinopathy Olanzapine
on ophthalmic Sodium valproate
examination. Clozapine

1469
A 50­ year­old
gentleman with chronic Amitriptyline
schizophrenia is on an Trazodone
antipsychotic Fluoxetine
medication for a long
Chlorpromazine
period. He has
Venlafaxine
developed blue gray
discolouration of skin Olanzapine
when exposed to Sodium valproate
sunlight. Clozapine
Thioridazine
Check Mirtazapine
Carbamazepine

Explanation: 
Tricyclics like Amitriptyline can cause sedation and postural hypotension leading to falls. 
Sodium valproate can induce pancreatitis, leucopenia, thrombocytopenia and red cell
hypoplasia. 
Clozapine is associated with myocarditis and cardiomyopathy. The symptoms include fever,
tachycardia, flu­like symptoms, fatigue, chest pain and dyspnoea. 
Irreversible retinal pigmentation is associated with the use of high dose thioridazine (above
1000 mg a day). An early symptom of the side effect can sometimes be nocturnal confusion
related to difficulty with night vision. Allergic dermatitis and photosensitivity can occur with
low­potency antipsychotic agents. 
Long­term chlorpromazine use can cause blue­gray discoloration of skin areas exposed to
sunlight. This is usually reversible.
The correct answer is: An 83­year­old lady suffered dizziness and incurred falls on starting
this antidepressant – Amitriptyline, A 45­year­old patient with bipolar disorder on treatment
develops acute abdominal pain and significantly raised serum amylase levels – Sodium
valproate, A 45­year­old gentleman with treatment­resistant schizophrenia started on a new
antipsychotic developed chest pain, tachycardia, tachypnoea and palpitations – Clozapine, A
45­year­old chronic schizophrenia patient on longstanding treatment with an antipsychotic
medication has pigmentary retinopathy on ophthalmic examination. – Thioridazine, A 50­
year­old gentleman with chronic schizophrenia is on an antipsychotic medication for a long
period. He has developed blue gray discolouration of skin when exposed to sunlight. –
Chlorpromazine

Question 4 HiY Pharmacology EMI054
Correct Pharmacokinetics (2)
Match each characteristic below with the most likely drug from the list above;
Mark 4.00 out of
4.00

Flag question
Diazepam
Paroxetine
This drug undergoes N­ Sodium valproate
dealkylation during Nitrazepam
phase 1 metabolism, Lamotrigine
which results in
Lithium
desmethyl derivative
Fluoxetine
Temazepam
Oxazepam

1470
that has a longer half­
life than the parent
compound.

Diazepam
This drug undergoes Paroxetine
direct phase 2 Sodium valproate
metabolism without Nitrazepam
undergoing phase 1 Lamotrigine
metabolism Lithium
Fluoxetine
Temazepam
Oxazepam
Diazepam
Lorazepam
Paroxetine
The active metabolite of Imipramine
Sodium valproate
this drug has an
Nitrazepam
elimination half­life of 6
days
Lamotrigine
Lithium
Fluoxetine
Temazepam
Oxazepam
Diazepam
Lorazepam
Paroxetine
Imipramine
Sodium valproate
This drug has narrow
Nitrazepam
therapeutic index
Lamotrigine
Lithium
Fluoxetine
Temazepam
Oxazepam
Check
Lorazepam
Imipramine

Explanation: 
Fluoxetine is metabolized to norfluoxetine, which acts on 5­HT reuptake in a similar fashion as
fluoxetine. The half­life of norfluoxetine is 4­16 days while that of fluoxetine itself is 4­6 days.
In total, fluoxetine may take up to 4 ­5 weeks to be cleared from the body substantially.
Certain drugs such as lithium, carbamazepine and phenytoin have a narrow range of plasma
levels within which the efficacy is optimum and toxicity is less evident; crossing this range on
higher side will increase toxicity while on the lower range will reduce efficacy. Drugs with a
low therapeutic index or narrow therapeutic range will require plasma monitoring. 
Diazepam has an oral bioavailability nearly 100% and is widely distributed due to its highly
lipophilic nature(so CSF concentration more or less equals plasma concentration) with 95­
99% plasma protein binding and a slow elimination t1/2 30 h (ranges between 20 ­ 100 h). It
gets extensively metabolised in the liver with 3 active metabolites: Nordiazepam or
desmethyldiazepam (principal metabolite ­ could accumulate due to long t1/2). 
It is not essential that a drug must undergo phase 1 metabolism in order to undergo phase 2
metabolism e.g. lorazepam, temazepam and oxazepam undergo direct phase 2 reactions. (As
a result, in patients with alcoholic liver disease, oxazepam is a favoured drug for alcohol
detoxification instead of chlordiazepoxide)

1471
The correct answer is: This drug undergoes N­dealkylation during phase 1 metabolism, which
results in desmethyl derivative that has a longer half­life than the parent compound. –
Diazepam, This drug undergoes direct phase 2 metabolism without undergoing phase 1
metabolism – Oxazepam, The active metabolite of this drug has an elimination half­life of 6
days – Fluoxetine, This drug has narrow therapeutic index – Lithium

Question 5 HiY Pharmacology EMI055
Correct Effect of receptor action (3)
Match the receptor below with its most significant effect from the list above
Mark 4.00 out of
4.00

Flag question
Production of negative symptoms
Withdrawal symptoms caused by MAOIs
Consequences of Withdrawal symptoms caused by benzodiazepines
stimulating 5HT1­a Therapeutic effects of antidepressants
receptor Gastrointestinal side­effects of SSRIs
Insomnia and sexual side effects of SSRIs
Leads to features of hyperprolactinaemia
Excessive weight gain
Leads to extra­pyramidal side effects.
Production of negative symptoms
Orthostatic hypotension
Withdrawal symptoms caused by MAOIs
Consequences of Withdrawal symptoms caused by SSRIs
Withdrawal symptoms caused by benzodiazepines
stimulating 5HT2 Therapeutic effects of antidepressants
receptors Gastrointestinal side­effects of SSRIs
Insomnia and sexual side effects of SSRIs
Leads to features of hyperprolactinaemia
Excessive weight gain
Leads to extra­pyramidal side effects.
Production of negative symptoms
Orthostatic hypotension
Withdrawal symptoms caused by MAOIs
Consequences of Withdrawal symptoms caused by SSRIs
Withdrawal symptoms caused by benzodiazepines
stimulating 5HT3 Therapeutic effects of antidepressants
receptors Gastrointestinal side­effects of SSRIs
Insomnia and sexual side effects of SSRIs
Leads to features of hyperprolactinaemia
Excessive weight gain
Leads to extra­pyramidal side effects.
Production of negative symptoms
Orthostatic hypotension
Withdrawal symptoms caused by MAOIs
Withdrawal symptoms caused by SSRIs
Withdrawal symptoms caused by benzodiazepines
Therapeutic effects of antidepressants
Gastrointestinal side­effects of SSRIs
Decreased brain GABA
Insomnia and sexual side effects of SSRIs
function
Leads to features of hyperprolactinaemia
Excessive weight gain
Leads to extra­pyramidal side effects.
Orthostatic hypotension
Withdrawal symptoms caused by SSRIs

Check

1472
Explanation: The stimulation of the 5HT1A receptor leads to its therapeutic effects and often
used for the treatment of depression, anxiety, phobias, OCD, etc. 
The stimulation of 5HT2 receptors cause CNS side effects (anxiety, insomnia, agitation, etc.)
and sexual side effects (reduced libido, impotence, ejaculatory dysfunction, and anorgasmia,
etc.)
The consequences of stimulating 5HT3 receptors include gastrointestinal side effects
(nausea, vomiting, anorexia, and diarrhoea). 
Benzodiazepine withdrawal is characterized by anxiety, diaphoresis, restlessness, irritability,
light­headedness, tremor, insomnia, and weakness. This syndrome can occur after 1 or 2
weeks after stopping long­acting benzodiazepines and is related to a reduction in GABA
functions.
The correct answer is: Consequences of stimulating 5HT1­a receptor – Therapeutic effects of
antidepressants, Consequences of stimulating 5HT2 receptors – Insomnia and sexual side
effects of SSRIs, Consequences of stimulating 5HT3 receptors – Gastrointestinal side­effects
of SSRIs, Decreased brain GABA function – Withdrawal symptoms caused by
benzodiazepines

Question 6 HiY Pharmacology EMI056
Correct Side effects and receptors (4)
Choose one explanatory option for each of the following conditions:
Mark 4.00 out of
4.00

Flag question
Serotonin 5HT3 blockade
Decreased nicotinic receptor density
Serotonin 5HT3 activation
Late onset Alzheimer's
M4 agonism
disease.
Dopamine D4 activation
High 5HT2 Serotonin receptor sensitivity
Low Dopamine receptor sensitivity
Serotonin 5­HT2 receptor blockade
Low D2 receptor occupancy in striatum
Serotonin 5HT3 blockade
Apolipoprotein E receptors
Decreased nicotinic receptor density
Serotonin 5HT3 activation
Clozapine induced
M4 agonism
hypersalivation
Dopamine D4 activation
High 5HT2 Serotonin receptor sensitivity
Low Dopamine receptor sensitivity
Serotonin 5­HT2 receptor blockade
Low D2 receptor occupancy in striatum
Serotonin 5HT3 blockade
Apolipoprotein E receptors
Decreased nicotinic receptor density
Serotonin 5HT3 activation
M4 agonism
Dopamine D4 activation
SSRI­induced nausea High 5HT2 Serotonin receptor sensitivity
Low Dopamine receptor sensitivity
Serotonin 5­HT2 receptor blockade
Low D2 receptor occupancy in striatum
Apolipoprotein E receptors

1473
Serotonin 5HT3 blockade
Decreased nicotinic receptor density
Lower incidence of Serotonin 5HT3 activation
EPSEs of newer M4 agonism
atypical antipsychotics Dopamine D4 activation
High 5HT2 Serotonin receptor sensitivity
Low Dopamine receptor sensitivity
Serotonin 5­HT2 receptor blockade
Low D2 receptor occupancy in striatum
Check
Apolipoprotein E receptors

Explanation: 
Apolipoprotein E receptors are involved in Alzheimer's disease. 
The pharmacological basis of clozapine related hypersalivation remains unclear. Suggested
mechanisms include M4 agonism, adrenergic alpha two antagonism and inhibition of the
swallowing reflex. 
The consequences of stimulating 5HT3 receptors by SSRIs causes gastro intestinal side
effects like nausea, vomiting, anorexia, and diarrhoea. 
Lower incidence of EPSEs of newer atypical antipsychotics is due to 5­HT2 receptor
blockade. PET and SPECT studies have shown that newer atypicals like olanzapine,
risperidone, quetiapine and clozapine have a higher degree of 5­HT2A receptor occupancy
(more than 90%) that leads to a reduced incidence of EPSEs.
The correct answer is: Late onset Alzheimer's disease. – Apolipoprotein E receptors,
Clozapine induced hypersalivation – M4 agonism, SSRI­induced nausea – Serotonin 5HT3
activation, Lower incidence of EPSEs of newer atypical antipsychotics – Serotonin 5­HT2
receptor blockade

Question 7 HiY Pharmacology EMI057
Correct Pharmacodynamics (3)
Choose the drug that best fits the description below
Mark 4.00 out of
4.00

Flag question
Moclobemide
Mirtazapine
This drug weakly Fluvoxamine
blocks the synaptic Venlafaxine
reuptake of dopamine Escitalopram
Dothiepin
Citalopram
Paroxetine
Fluoxetine
Moclobemide
Sertraline
Mirtazapine
Fluvoxamine
Venlafaxine
Escitalopram
This drug stimulates 5­ Dothiepin
HT2C receptors Citalopram
Paroxetine
Fluoxetine

1474
Moclobemide
Mirtazapine
Fluvoxamine
This SSRI drug has an
Venlafaxine
anticholinergic action
Escitalopram
Dothiepin
Citalopram
Paroxetine
Fluoxetine
Moclobemide
Sertraline
Mirtazapine
This drug is a Fluvoxamine
reversible inhibitor of Venlafaxine
monoamine oxidase A Escitalopram
Dothiepin
Citalopram
Paroxetine
Fluoxetine
Check
Sertraline

Explanation: 
Paroxetine has greater anxiolytic effects among SSRIs but can cause anticholinergic side
effects (e.g. dry mouth). 
Sertraline weakly inhibits noradrenaline and dopamine reuptake. This may be beneficial in
the treatment of depressed patients with psychomotor retardation. 
Fluoxetine binds to 5­HT2C receptors and, therefore, does not cause weight gain and aids in
the treatment of bingeing and bulimia. 
Moclobemide: Reversible selective inhibitor of MAO­A. The potential for interaction of MAOIs
with indirect sympathomimetics is greatly reduced with moclobemide as unlike irreversible
MAOIs; this drug is displaced from the enzyme when rising tyramine levels compete for the
site. Moclobemide has a low potential for cheese reaction because of this selectivity for MAO­
A and reversibility.
The correct answer is: This drug weakly blocks the synaptic reuptake of dopamine –
Sertraline, This drug stimulates 5­HT2C receptors – Fluoxetine, This SSRI drug has an
anticholinergic action – Paroxetine, This drug is a reversible inhibitor of monoamine oxidase
A – Moclobemide

Question 8 HiY Pharmacology EMI058
Partially correct Pharmacodynamics (4)
Choose the drug that best fits the description below.
Mark 3.00 out of
4.00

Flag question

1475
This drug carries the
Bupropion
least risk of
Fluvoxamine
pharmacodynamic drug
interactions among
Citalopram
SSRIs Paroxetine
Fluoxetine
Escitalopram
Venlafaxine
Mirtazapine
Reboxetine
Bupropion
Moclobemide
Fluvoxamine
A preferential blockade Citalopram
of dopamine is seen Paroxetine
with this drug Fluoxetine
Escitalopram
Venlafaxine
Mirtazapine
Reboxetine
Bupropion
At high doses of
administration of this
Moclobemide
Fluvoxamine
drug, there is a Citalopram
blockade of the Paroxetine
reuptake of serotonin, Fluoxetine
noradrenaline and Escitalopram
dopamine. Venlafaxine
Mirtazapine
Reboxetine
Bupropion
Moclobemide
Fluvoxamine
This drug has a Citalopram
negligible serotonergic
Paroxetine
effect and has a safe
Fluoxetine
cardiovascular profile.
Escitalopram
Venlafaxine
Mirtazapine
Reboxetine
Check
Moclobemide

Explanation: Escitalopram is the most selective and carries the least risk of pharmacodynamic
drug interactions among SSRIs. It has no pharmacodynamic effects other than the 5­HT
reuptake inhibition. Bupropion is not licensed in UK for depression. It acts via dopamine and
noradrenaline reuptake inhibition. It is available in sustained release form too. Smoking and
cocaine dependence can be treated using bupropion though the latter use is not widely
practiced. Venlafaxine has low protein binding; it has t1/2 around 3.5 hours. Venlafaxine is
well absorbed per orally. The metabolite O­desmethyl venlafaxine (ODV) has a half­life of 9
hours. It is metabolized by hepatic cytochrome P450 (CYP) 2D6. Venlafaxine has no enzyme­
inducing properties. At average doses of administration of this drug between 75­300 mg/day,
there is a blockade of the reuptake of serotonin and noradrenaline. At high doses of
administration of this drug more than 300 mg/day, there is a blockade of the reuptake of
serotonin, noradrenaline and dopamine. Reboxetine is a noradrenaline reuptake inhibitor
(NARI) with negligible serotonergic effects. It has a safe cardiovascular profile and can be
used in the elderly.
The correct answer is: This drug carries the least risk of pharmacodynamic drug interactions

1476
among SSRIs – Escitalopram, A preferential blockade of dopamine is seen with this drug –
Bupropion, At high doses of administration of this drug, there is a blockade of the reuptake of
serotonin, noradrenaline and dopamine. – Venlafaxine, This drug has a negligible
serotonergic effect and has a safe cardiovascular profile. – Fluvoxamine

Question 9 HiY Pharmacology EMI059
Correct Mechanism of action (2)
Find the drug, which matches each of the following mechanisms of action;
Mark 4.00 out of
4.00

Flag question
Galantamine
Buspirone
Selective Paliperidone
Norepinephrine Duloxetine
reuptake inhibitor Aripiprazole
Reboxetine
Agomelatine
Fluoxetine
Bupropion
Galantamine
Molindone
Buspirone
Paliperidone
Melatonin agonist Duloxetine
Aripiprazole
Reboxetine
Agomelatine
Fluoxetine
Bupropion
Galantamine
Molindone
Buspirone
Paliperidone
D2 partial agonist Duloxetine
Aripiprazole
Reboxetine
Agomelatine
Fluoxetine
Bupropion
Galantamine
Molindone
Buspirone
Paliperidone
Duloxetine
Aripiprazole
5­HT1A partial agonist Reboxetine
Agomelatine
Fluoxetine
Bupropion
Molindone

Check

1477
Explanation: 
Reboxetine: This is a noradrenaline reuptake inhibitor (NARI) with negligible serotonergic
effects. It has a safe cardiovascular profile and can be used in the elderly. Reboxetine has a
specific side­effect profile linked to the noradrenergic system. Urinary hesitancy has been
observed in around 10% of male patients taking part in the clinical­trials. Relief from this side
effect could be achieved by using either tamsulosin, a peripheral alpha1­receptor blocker or
doxazosin with a similar mechanism of action as tamsulosin. 
Agomelatine: Agomelatine enhances norepinephrine and dopamine neurotransmission
through 5­HT2C antagonism. It is also a direct agonist at melatonin (MT1 and MT2) receptors.
GABA interneurons tonically inhibit noradrenergic circuits (from locus coeruleus) and
dopaminergic circuits (from ventral tegmentum) projecting to the prefrontal cortex. Serotonin
via 5HT2C stimulation drives these GABA interneurons. Thus, norepinephrine and dopamine
circuits are inhibited by the normal tonic release of serotonin onto 5­HT2C receptors (Stahl,
2007). Thus agomelatine, through 5HT2C inhibition, acts as norepinephrine and dopamine
disinhibitor (NDDI). 
Aripiprazole: Partial D2 agonist and 5HT2A antagonist; thought to demonstrate a Goldilocks'
Phenomenon or stabilising action wherein at sites of excessive dopamine such as mesolimbic
zones, it antagonises DA while at dopamine deficient zones such as mesocortical areas that
cause negative symptoms, it produces DA agonism. Hence, it does not produce much change
in tuberoinfundibular where regular DA levels are expected in schizophrenia. 
Buspirone: Partial agonist on serotonin 5­HT1A receptors. At presynaptic levels, it is mostly a
full agonist, which inhibits the release of serotonin, with consequent antianxiety effects. Partial
agonist action at postsynaptic receptors appears to account for antidepressant activity
The correct answer is: Selective Norepinephrine reuptake inhibitor – Reboxetine, Melatonin
agonist – Agomelatine, D2 partial agonist – Aripiprazole, 5­HT1A partial agonist – Buspirone

Question 10 HiY Pharmacology EMI060
Partially correct Receptor action and effects (5)
For the following side effects exhibited by the drugs, identify the causative receptor action.
Mark 6.40 out of
8.00

Flag question
D2 stimulation
A 45­year­old 5­HT2A stimulation
gentleman develops H2­ antagonism
akathisia and
5­HT1A stimulation
restlessness on
treatment with
H3­ antagonism
risperidone D2 blockade
H1 antagonism
5­HT3 agonism
5­HT3 antagonism
D2 stimulation
5­HT2C stimulation
5­HT2A stimulation
H2­ antagonism
5­HT1A stimulation
H3­ antagonism
A 45­year­old D2 blockade
gentleman develops H1 antagonism
insomnia on taking 5­HT3 agonism
paroxetine
5­HT3 antagonism
5­HT2C stimulation

1478
D2 stimulation
A 25­year­old 5­HT2A stimulation
gentleman complains H2­ antagonism
of delayed ejaculation 5­HT1A stimulation
on treatment with H3­ antagonism
fluoxetine  D2 blockade
H1 antagonism
5­HT3 agonism
5­HT3 antagonism
D2 stimulation
5­HT2C stimulation
5­HT2A stimulation
A 56­year­old woman H2­ antagonism
complains of gaining
5­HT1A stimulation
weight on treatment
H3­ antagonism
with amitriptyline
D2 blockade
H1 antagonism
5­HT3 agonism
5­HT3 antagonism
D2 stimulation
A 49­year­old lady has 5­HT2C stimulation
5­HT2A stimulation
shown improvement on H2­ antagonism
her depressive
5­HT1A stimulation
symptoms by taking
sertraline for more than
H3­ antagonism
four weeks D2 blockade
H1 antagonism
5­HT3 agonism
5­HT3 antagonism
Check
5­HT2C stimulation

Explanation: 
The side effects of antipsychotic medications like akathisia, EPSEs and hyperprolactinaemia
are caused primarily by D2 receptor blockade. Akathisia and agitation are also induced by
5HT2A stimulation (hence some data on mirtazapine­ 5HT2A antagonist reducing akathisia). 
Patients on SSRIs may develop insomnia due to 5­HT2A stimulation and can develop
delayed ejaculation due to 5HT2A/2C stimulation. 
The weight gain induced by psychotropics is due to antihistaminic effects and 5HT2C
antagonism. 
5HT1A stimulation by SSRIs such as sertraline indirectly produces antidepressant action
The correct answer is: A 45­year­old gentleman develops akathisia and restlessness on
treatment with risperidone
– D2 blockade, A 45­year­old gentleman develops insomnia on taking paroxetine – 5­HT2A
stimulation, A 25­year­old gentleman complains of delayed ejaculation on treatment with
fluoxetine 
– 5­HT2C stimulation, A 56­year­old woman complains of gaining weight on treatment with
amitriptyline
– H1 antagonism, A 49­year­old lady has shown improvement on her depressive symptoms
by taking sertraline for more than four weeks – 5­HT1A stimulation

1479
Question 11 HiY Pharmacology EMI061
Correct Addiction Pharmacology
Choose the drug that best fits descriptions below
Mark 4.00 out of
4.00

Flag question
Acamprosate
Naloxone
Aldehyde Oxazepam
dehydrogenase Buprenorphine
inhibitor Disulfiram
Oxycodone
Chlordiazepoxide
Naltrexone
Acamprosate
Naloxone
Non­selective long­ Oxazepam
acting opiate Buprenorphine
antagonist Disulfiram
Oxycodone
Chlordiazepoxide
Naltrexone
Acamprosate
Naloxone
Oxazepam
Mu opioid receptor
Buprenorphine
partial agonist
Disulfiram
Oxycodone
Chlordiazepoxide
Naltrexone
Acamprosate
Naloxone
Oxazepam
Shorter acting mu
Buprenorphine
opioid antagonist
Disulfiram
Oxycodone
Chlordiazepoxide
Naltrexone
Check

Explanation: 
Disulfiram: Inhibits aldehyde dehydrogenase. Accumulation of acetaldehyde takes place if
alcohol is consumed while taking disulfiram, producing unpleasant reactions. Disulfiram is
used as a prophylactic adjunctive deterrent agent in alcohol dependent patients. 
Naltrexone is a longer acting opioid mu antagonist whereas naloxone is a shorter acting
opioid mu antagonist. 
Naltrexone is used to help prevent relapse in formerly opioid­dependent patients who are
detoxified and have remained opioid­free for at least a week. 

1480
Buprenorphine is a partial opioid agonist. At lower doses, it has mild agonistic effects while at
higher doses antagonistic effects are noted. It is used sublingually (subitex) as an adjunct in
managing opioid dependence.
The correct answer is: Aldehyde dehydrogenase inhibitor – Disulfiram, Non­selective long­
acting opiate antagonist – Naltrexone, Mu opioid receptor partial agonist – Buprenorphine,
Shorter acting mu opioid antagonist – Naloxone

Question 12 HiY Pharmacology EMI062
Correct Adverse effects (3)
For each of the drugs given below, identify the main side effect
Mark 5.00 out of
5.00

Flag question
Hair loss
Hepatomegaly
Hepatitis
Paroxetine  
Worsening of psoriasis
Weight loss
Tics
Pruritus
Stevens­Johnson Syndrome
Hair loss
Agranulocytosis
Hepatomegaly
Amenorrhoea
Hepatitis
Lithium Anticholinergic effects  
Worsening of psoriasis
Weight loss
Tics
Pruritus
Stevens­Johnson Syndrome
Hair loss
Agranulocytosis
Hepatomegaly
Amenorrhoea
Hepatitis
Amitriptyline Anticholinergic effects  
Worsening of psoriasis
Weight loss
Tics
Pruritus
Stevens­Johnson Syndrome
Hair loss
Agranulocytosis
Hepatomegaly
Amenorrhoea
Hepatitis
Anticholinergic effects
Worsening of psoriasis  
Lamotrigine Weight loss
Tics
Pruritus
Stevens­Johnson Syndrome
Agranulocytosis
Amenorrhoea
Anticholinergic effects
Hair loss
Hepatomegaly
Hepatitis
Sodium valproate  
Worsening of psoriasis
Weight loss
Tics
Pruritus
Stevens­Johnson Syndrome

1481
Check

Explanation: Paroxetine can often cause dry mouth due to its anticholinergic property.
Exacerbation of acne and psoriasis associated with lithium therapy. Anticholinergic side
effects such as dry mouth, constipation, urinary retention and blurred vision are commonly
seen in patients taking tricyclic such as amitryptiline. Lamotrigine can produce a rash; some of
these rashes may be the first sign of an imminentSteven­Johnson syndrome. Hair loss with
curly regrowth can occur in patients on sodium valproate.
The correct answer is: Paroxetine – Anticholinergic effects, Lithium – Worsening of psoriasis,
Amitriptyline – Anticholinergic effects, Lamotrigine – Stevens­Johnson Syndrome, Sodium
valproate – Hair loss

Question 13 HiY Pharmacology EMI063
Correct Adverse effects (4)
For each of the adverse effects given below, identify the best drug associated with it;
Mark 6.00 out of
6.00

Flag question
Methylphenidate
Lamotrigine
Phenelzine
Insomnia  
Clozapine
Carbamazepine
Lithium
Sildenafil
Lorazepam
Methylphenidate
Acamprosate
Lamotrigine
Mirtazapine
Phenelzine
Pedal edema Citalopram  
Clozapine
Carbamazepine
Lithium
Sildenafil
Lorazepam
Methylphenidate
Acamprosate
Lamotrigine
Mirtazapine
Phenelzine
Blue tinged vision Citalopram  
Clozapine
Carbamazepine
Lithium
Sildenafil
Lorazepam
Methylphenidate
Acamprosate
Lamotrigine
Mirtazapine
Phenelzine
Delayed orgasm Citalopram  
Clozapine
Carbamazepine
Lithium
Sildenafil
Lorazepam
Acamprosate
Mirtazapine
Citalopram

1482
Rash Methylphenidate
Lamotrigine
Phenelzine
Clozapine  
Carbamazepine
Lithium
Sildenafil
Lorazepam
Methylphenidate
Acamprosate
Lamotrigine
Mirtazapine
Phenelzine
Fine tremor Citalopram  
Clozapine
Carbamazepine
Lithium
Sildenafil
Lorazepam
Check Acamprosate
Mirtazapine
Citalopram
Explanation: Some sildenafil users experience tinted blue vision (cyanopsia). Pedal edema is
a side effect of phenelzine therapy. Sexual difficulties such as reduced libido, impotence,
ejaculatory dysfunction, delayed orgasm and anorgasmia are reported with SSRIs. The
incidence of sexual dysfunction is seen in nearly 1 in 3 patients treated. The most common
adverse effects of methylphenidate are anxiety, irritability, insomnia, tachycardia, cardiac
arrhythmias, and dysphoria with decreased appetite. Lamotrigine can cause a rash in 10% of
patients. Risk factors for rash include rapid initial dose escalation, concurrent Valproate use,
and age less than 16 years. As these rashes are potentially serious in at least some
individuals, discontinuation of the drug is warranted along with full blood count.
The correct answer is: Insomnia – Methylphenidate, Pedal edema – Phenelzine, Blue tinged
vision – Sildenafil, Delayed orgasm – Citalopram, Rash – Lamotrigine, Fine tremor – Lithium

Question 14 HiY Pharmacology EMI064
Partially correct Agonistic agents (1)
Match the following receptors with the drugs which have an agonistic effect on the receptors
Mark 5.25 out of
7.00

Flag question
Carbamazepine
Diazepam
Sildenafil
Dopamine D2  
Clozapine
receptor 
Zolpidem
Isocarboxazid
Oxycodone
Sodium valproate
Ropinirole
Carbamazepine
Fluvoxamine
Diazepam
Sildenafil
Clozapine  
Zolpidem
Mu receptors Isocarboxazid
Oxycodone
Sodium valproate

1483
Carbamazepine
Diazepam
Sildenafil
Sigma receptors Clozapine  
Zolpidem
Isocarboxazid
Oxycodone
Sodium valproate
Ropinirole
Carbamazepine
Fluvoxamine
Diazepam
Sildenafil
Omega­1 receptors Clozapine  
Zolpidem
Isocarboxazid
Oxycodone
Sodium valproate
Ropinirole
Check
Fluvoxamine

Explanation: The examples of dopamine agonists include bromocriptine pramipexole,
pergolide, Ropinirole and Apomorphine. Bromocriptine and Ropinirole are selective
dopamine two like agonists. Pure Mu agonists include oxycodone, hydrocodone, morphine,
fentanyl and methadone. Z­drugs (Zopiclone, Zolpidem and Zaleplon) act via GABA A
complex but act differently than benzodiazepines. In contrast to benzodiazepines which
occupy all 3 subunits of ω (omega) receptor, the Z­drugs only occupy specific subunits
resulting in unique effects e.g., zolpidem and zopiclone acts on ω­1 receptors producing no
muscle relaxant, anxiolytic or anticonvulsant effects. Also, slow wave sleep is unaffected when
using Z­drugs. Zaleplon occupies all three omega receptors. Zopiclone occurs as a racemic
mixture with only s­isomer being active (eszopiclone). Among all antidepressants,
fluvoxamine is the most potent sigma­1 receptor agonist.
The correct answer is: Dopamine D2 receptor 
– Ropinirole, Mu receptors
– Oxycodone, Sigma receptors – Fluvoxamine, Omega­1 receptors
– Zolpidem

Question 15 HiY Pharmacology EMI065
Partially correct Agonistic agents (2)
Match the following agonists with their neurochemical systems in the list.
Mark 3.00 out of
5.00

Flag question

1484
Oxycodone Dopamine 1 receptor
Serotonin 2A
Serotonin 1A
Histamine­1  
NMDA
Mu receptor
Serotonin 3
Dopamine 2 receptor
Dopamine 1 receptor
GABA­A
Serotonin 2A
Sigma
Serotonin 1A
Zolpidem  
Histamine­1
NMDA
Mu receptor
Serotonin 3
Dopamine 2 receptor
Dopamine 1 receptor
GABA­A
Serotonin 2A
Sigma
Serotonin 1A
Bromocriptine  
Histamine­1
NMDA
Mu receptor
Serotonin 3
Dopamine 2 receptor
Dopamine 1 receptor
GABA­A
Serotonin 2A
Sigma
Serotonin 1A
Pramipexole  
Histamine­1
NMDA
Mu receptor
Serotonin 3
Dopamine 2 receptor
Dopamine 1 receptor
GABA­A
Serotonin 2A
Sigma
Serotonin 1A
Aripiprazole  
Histamine­1
NMDA
Mu receptor
Serotonin 3
Dopamine 2 receptor
Check GABA­A
Sigma

Explanation: Oxycodone is a full opioid agonist with no antagonistic properties. It has an
affinity for kappa, mu and delta opioid receptors in the brain. Zolpidem binds with high affinity
and acts as a full agonist at GABA­A receptor. Bromocriptine, Pergolide, Pramipexole and
Ropinirole are all dopamine two receptor agonists. Aripiprazole is an atypical antipsychotic
agent with partial dopaminergic and serotonin 5HT1A receptor agonist activity. It has
anxiolytic effect and may be particularly useful when used in conjunction with standard
antidepressants in treatment­resistant depression
The correct answer is: Oxycodone – Mu receptor, Zolpidem – GABA­A, Bromocriptine –
Dopamine 2 receptor, Pramipexole – Dopamine 2 receptor, Aripiprazole – Serotonin 1A

1485
Question 16 HiY Pharmacology EMI066
Partially correct Post­marketing surveillance
For each of the following drugs, identify the adverse effects originally reported during post­
Mark 4.00 out of
marketing surveillance of the agents.
5.00

Flag question
Sudden cardiac death
Phocomelia
Cheese reactions
Sertindole  
QT prolongation on ECG
Blood dyscrasias
Agranulocytosis
Aplastic anemias
Guillain­Barre syndrome
Sudden cardiac death
Hepatotoxicity
Phocomelia
Cheese reactions
Mianserin  
QT prolongation on ECG
Blood dyscrasias
Agranulocytosis
Aplastic anemias
Guillain­Barre syndrome
Sudden cardiac death
Hepatotoxicity
Phocomelia
Cheese reactions
MAOIs  
QT prolongation on ECG
Blood dyscrasias
Agranulocytosis
Aplastic anemias
Guillain­Barre syndrome
Sudden cardiac death
Hepatotoxicity
Phocomelia
Cheese reactions
Thalidomide  
QT prolongation on ECG
Blood dyscrasias
Agranulocytosis
Aplastic anemias
Guillain­Barre syndrome
Sudden cardiac death
Hepatotoxicity
Phocomelia
Cheese reactions
Zimeldine  
QT prolongation on ECG
Blood dyscrasias
Agranulocytosis
Aplastic anemias
Guillain­Barre syndrome
Check
Hepatotoxicity

Explanation: 
Cheese reaction: MAOIs interact with tyramine­rich foods to cause cheese reaction or
tyramine reaction. Tyramine has both direct and indirect (via vesicular release)
sympathomimetic actions that develop 20 min to 1 h following ingestion of food. Cheese
reaction is a sympathetic overdrive syndrome characterized by high BP, nausea, occasional

1486
chills, sweating, restlessness with occipital headache, palpitations, and vomiting along with
piloerection, dilated pupils and fever. If severe, this can lead to cerebral haemorrhage and
death. 
Sertindole was associated with sudden cardiac deaths and lead to product withdrawal from
the market before being reinstated. 
Leucopenia, fatal aplastic anaemia and bone marrow suppression has been reported with the
use of Mianserin (and also mirtazapine that has a similar mechanism of drug action). 
Zimelidine was the first marketed SSRI. It is a pyridyl allylamine, structurally different from
other antidepressants. Zimelidine has been banned worldwide due to serious, sometimes
fatal, cases of central and/or peripheral neuropathy known as Guillain­Barre syndrome and
due to a peculiar hypersensitivity reaction involving many organs including skin exanthema,
flu­like symptoms, arthralgia. 
Thalidomide, an analgesic, was withdrawn due to the Teratogenic effect of phocomelias in
babies.
The correct answer is: Sertindole – Sudden cardiac death, Mianserin – Blood dyscrasias,
MAOIs – Cheese reactions, Thalidomide – Phocomelia, Zimeldine – Guillain­Barre syndrome

Question 17 HiY Pharmacology EMI067
Correct Receptor mechanisms (2)
From the descriptions below identify the pharmacological terminology referred to in the given
Mark 5.00 out of
list
5.00

Flag question
Non­competitive antagonist
Irreversible Antagonist
These are agents that Full agonist
produce a maximal Efficacy
response Competitive antagonist
Semi agonist
Affinity
Partial agonist
Potency
Non­competitive antagonist
Inverse agonist
Irreversible Antagonist
Full agonist
These are agents with
Efficacy
a ceiling effect
Competitive antagonist
Semi agonist
Affinity
Partial agonist
Potency
Non­competitive antagonist
Inverse agonist
Irreversible Antagonist
Full agonist
Efficacy
These agents bind to
Competitive antagonist
the same receptor but
produce the opposite
Semi agonist
pharmacological effect Affinity
Partial agonist
Potency
Inverse agonist

1487
Non­competitive antagonist
These alter the receptor Irreversible Antagonist
site in some way so Full agonist
increasing the dose of
Efficacy
the agonist drug can
reverse the effects only
Competitive antagonist
partially. Semi agonist
Affinity
Partial agonist
Potency
Non­competitive antagonist
Inverse agonist
Irreversible Antagonist
These agents not only Full agonist
shift the curve to the
Efficacy
right but also reduce
Competitive antagonist
the maximum effect
Semi agonist
Affinity
Partial agonist
Potency
Check
Inverse agonist

Explanation:Full agonists produce a maximal response. Examples are benzodiazepines at
GABA­A complex, Bromocriptine for dopamine. Partial agonists cannot elicit a maximal
response and are less effective than full agonists. Examples are Aripiprazole, buspirone and
buprenorphine. Hence, these partial agonists have a ceiling effect. An inverse agonist is an
agent that binds to the same receptor but produces the opposite pharmacological effect.
Antagonists are drugs that interact with receptors to interfere with their activation by
neurotransmitter or other agonistic molecules. If the dose of the agonist drug is increased, the
effect of competitive antagonism can be overcome almost completely. Thus competitive
antagonists can only reduce the potency but not the efficacy of agonists. Examples of
competitive antagonism include atropine at muscarinic receptors and propranolol at beta­
adrenergic receptors. The effects of a noncompetitive antagonist can only be partially
reversed by increasing the dose of the agonist drug as these agents alter the configuration of
the receptor site. Non­competitive antagonism reduces both the potency and the efficacy of
agonists. Therefore, non­competitive antagonists not only shift the curve to the right but also
reduce the maximum effect. For example, ketamine and phencyclidine are noncompetitive
NMDA antagonists. Irreversible antagonists bind irreversibly to the target site e.g. most
traditional MAOIs. Body cells must freshly synthesize the blocked enzyme before a releasing
effect from the drug can be produced.
The correct answer is: These are agents that produce a maximal response – Full agonist,
These are agents with a ceiling effect – Partial agonist, These agents bind to the same
receptor but produce the opposite pharmacological effect – Inverse agonist, These alter the
receptor site in some way so increasing the dose of the agonist drug can reverse the effects
only partially. – Non­competitive antagonist, These agents not only shift the curve to the right
but also reduce the maximum effect – Non­competitive antagonist

Question 18 HiY Pharmacology EMI068
Correct Receptor mechanisms (4)
For the following side effects exhibited by the drugs, identify the receptor action responsible
Mark 4.00 out of
for it
4.00

Flag question

1488
H3­ antagonism
A 40­year­old woman H1­ antagonism
on fluoxetine M3 receptor blockade
developed nausea and
5­HT1 agonism
vomiting
M2 receptor blockade
5­HT2 agonism
5­HT1 antagonism
H2­ antagonism
5­HT3 antagonism
H3­ antagonism
M1 receptor blockade
H1­ antagonism
A 65­year­old woman 5­HT3 agonism
M3 receptor blockade
complained of
5­HT1 agonism
excessive sedation on
taking trazodone
M2 receptor blockade
5­HT2 agonism
5­HT1 antagonism
H2­ antagonism
5­HT3 antagonism
H3­ antagonism
M1 receptor blockade
H1­ antagonism
A 33­year­old 5­HT3 agonism
M3 receptor blockade
gentleman suffered
5­HT1 agonism
from dry mouth on
taking amitriptyline
M2 receptor blockade
5­HT2 agonism
5­HT1 antagonism
H2­ antagonism
5­HT3 antagonism
H3­ antagonism
A 23­year­old woman
M1 receptor blockade
H1­ antagonism
has developed weight 5­HT3 agonism
M3 receptor blockade
gain and is overly 5­HT1 agonism
sedated on taking M2 receptor blockade
mirtazapine 5­HT2 agonism
5­HT1 antagonism
H2­ antagonism
5­HT3 antagonism
Check
M1 receptor blockade
5­HT3 agonism

Explanation: 
SSRIs like Fluoxetine causes GI side­ effects due to 5­HT3­receptor stimulation. 
Trazodone causes sedation through H1 receptor antagonism. It can also induce priapism
through alpha one receptor antagonism. 
Dry mouth is an anticholinergic side­effect of tricyclics like amitriptyline and is due to M3
receptor blockade. 
Mirtazapine can cause sedation and weight gain through H1 receptor antagonism.
The correct answer is: A 40­year­old woman on fluoxetine developed nausea and vomiting –
5­HT3 agonism, A 65­year­old woman complained of excessive sedation on taking trazodone
– H1­ antagonism, A 33­year­old gentleman suffered from dry mouth on taking amitriptyline –
M3 receptor blockade, A 23­year­old woman has developed weight gain and is overly
sedated on taking mirtazapine – H1­ antagonism

1489
Question 19 HiY Pharmacology EMI069
Partially correct Side effects and receptors
Choose one explanatory option for each of the following adverse effects.
Mark 2.00 out of
3.00

Flag question
Decreased nicotinic receptor density
Low Dopamine receptor sensitivity
Serotonin 5HT3 activation
Antipsychotic­induced
Serotonin 5HT3 blockade
Parkinsonism
High 5HT2 Serotonin receptor sensitivity
Low D2 receptor occupancy in striatum
High Dopamine receptor sensitivity
Dopamine D4 activation
Decreased nicotinic receptor density
Low Dopamine receptor sensitivity
Serotonin 5HT3 activation
Tardive dyskinesia Serotonin 5HT3 blockade
High 5HT2 Serotonin receptor sensitivity
Low D2 receptor occupancy in striatum
High Dopamine receptor sensitivity
Dopamine D4 activation
Decreased nicotinic receptor density
Low Dopamine receptor sensitivity
Serotonin 5HT3 activation
SSRI­induced nausea Serotonin 5HT3 blockade
High 5HT2 Serotonin receptor sensitivity
Low D2 receptor occupancy in striatum
High Dopamine receptor sensitivity
Dopamine D4 activation
Check

Explanation: Low D2 receptor occupancy in striatum due to antipsychotics causes
parkinsonian side­effects and dopamine receptor supersensitivity leads to tardive dyskinesia.
The consequences of stimulating 5HT3 receptors by SSRIs causes gastrointestinal side
effects like Nausea, vomiting, anorexia, and diarrhoea.
The correct answer is: Antipsychotic­induced Parkinsonism – Low D2 receptor occupancy in
striatum, Tardive dyskinesia – High Dopamine receptor sensitivity, SSRI­induced nausea –
Serotonin 5HT3 activation

Question 20 HiY Pharmacology EMI070
Partially correct Drugs & mechanism of action
Select one mechanism of action for each of the following drugs
Mark 4.00 out of
5.00

Flag question

1490
Rivastigmine Acetylcholinesterase inhibitor
Inhibitor of norepinephrine and serotonin re­uptake
MAO­A Inhibitor
NMDA receptor antagonist  
Presynaptic alpha­2 agonist
GABA­A agonist
Selective norepinephrine reuptake inhibitor
GABA­B agonist
Acetylcholinesterase inhibitor
MAO­B Inhibitor
Inhibitor of norepinephrine and serotonin re­uptake
D­2 receptor partial agonist
MAO­A Inhibitor
Selegeline 5HT1A partial agonist  
NMDA receptor antagonist
Presynaptic alpha­2 agonist
GABA­A agonist
Selective norepinephrine reuptake inhibitor
GABA­B agonist
Acetylcholinesterase inhibitor
MAO­B Inhibitor
Inhibitor of norepinephrine and serotonin re­uptake
D­2 receptor partial agonist
MAO­A Inhibitor
Diazepam 5HT1A partial agonist  
NMDA receptor antagonist
Presynaptic alpha­2 agonist
GABA­A agonist
Selective norepinephrine reuptake inhibitor
GABA­B agonist
Acetylcholinesterase inhibitor
MAO­B Inhibitor
Inhibitor of norepinephrine and serotonin re­uptake
D­2 receptor partial agonist
MAO­A Inhibitor
Reboxetine 5HT1A partial agonist  
NMDA receptor antagonist
Presynaptic alpha­2 agonist
GABA­A agonist
Selective norepinephrine reuptake inhibitor
GABA­B agonist
Acetylcholinesterase inhibitor
MAO­B Inhibitor
Inhibitor of norepinephrine and serotonin re­uptake
D­2 receptor partial agonist
MAO­A Inhibitor
Duloxetine 5HT1A partial agonist  
NMDA receptor antagonist
Presynaptic alpha­2 agonist
GABA­A agonist
Selective norepinephrine reuptake inhibitor
GABA­B agonist
Check
MAO­B Inhibitor
D­2 receptor partial agonist
5HT1A partial agonist
Explanation: Rivastigmine is a Cholinesterase Inhibitors. It acts by inhibiting
acetylcholinesterase enzyme that breaks down acetylcholine centrally. Rivastigmine inhibits
both the acetyl and butyl­cholinesterase while donepezil and galantamine are acetyl specific. 
Selegiline is a monoamine Oxidase Inhibitors ­ selective for B at normal therapeutic doses;
selectivity lost when a patch is applied at higher doses, leading to some antidepressant
action. 
Diazepam is a type of Benzodiazepine which acts via a specific site called omega site in
GABA­A complex. All are agonists except clonazepam which is a partial agonist. They
facilitate GABA action on GABA­A complex ­ thus inhibitory neurotransmission via chloride ion
flow is facilitated. They have no direct agonistic action in the absence of GABA. 
Reboxetine is a Noradrenergic specific reuptake inhibitor (NARI). 

1491
Duloxetine is an SNRI (Serotonin and Noradrenaline re­uptake inhibitor) similar to
venlafaxine. Said to have a better profile for psychosomatic pain and neuropathic pain
The correct answer is: Rivastigmine – Acetylcholinesterase inhibitor, Selegeline – MAO­B
Inhibitor, Diazepam – GABA­A agonist, Reboxetine – Selective norepinephrine reuptake
inhibitor, Duloxetine – Inhibitor of norepinephrine and serotonin re­uptake

Finish review

1492
 Home HiYield Paper Pharmacology

HiYield Paper A(2)

Started on Sunday, 10 May 2015, 9:06 AM
State Finished
Completed on Saturday, 16 May 2015, 9:22 AM
Time taken 6 days
Marks 198.42/229.00
Grade 86.64 out of 100.00

Question 1 HiY Pharmacology EMI001
Correct Receptors and side effects
Identify the receptors responsible for each of the following side effects induced by
Mark 5.00 out of
psychotropic medications
5.00

Flag question
Alpha­1 antagonism
5HT2C Antagonism
5HT­1D stimulation
Weight gain caused by
5HT3 stimulation
antipsychotics
Alpha­1 stimulation
5HT2C stimulation
5HT2A stimulation
D­2 blockade
5HT2A blockade
Alpha­1 antagonism
GABA­A stimulation
5HT2C Antagonism
Hyperprolactinaemia 5HT­1D stimulation
induced by 5HT3 stimulation
antipsychotics Alpha­1 stimulation
5HT2C stimulation
5HT2A stimulation
D­2 blockade
5HT2A blockade
Alpha­1 antagonism
GABA­A stimulation
5HT2C Antagonism
5HT­1D stimulation
5HT3 stimulation
Alpha­1 stimulation
Extrapyramidal side
5HT2C stimulation
effects of antipsychotics
5HT2A stimulation
D­2 blockade
5HT2A blockade
GABA­A stimulation

1493
Alpha­1 antagonism
5HT2C Antagonism
Gastrointestinal 5HT­1D stimulation
discomfort caused by 5HT3 stimulation
SSRIs Alpha­1 stimulation
5HT2C stimulation
5HT2A stimulation
D­2 blockade
5HT2A blockade
Alpha­1 antagonism
GABA­A stimulation
5HT2C Antagonism
Postural hypotension 5HT­1D stimulation
induced by 5HT3 stimulation
antipsychotics Alpha­1 stimulation
5HT2C stimulation
5HT2A stimulation
D­2 blockade
5HT2A blockade
Check
GABA­A stimulation

Explanation: EPSEs caused by antipsychotics are due to D2 blockade, the severity of which
reduces with 5HT2A antagonistic properties of atypical compounds. Hyperprolactinaemia
caused by antipsychotics is also due to D2 blockade, while 5HT1A stimulation can also
increase prolactin levels when using certain psychotropics. Weight gain is due to both 5HT2C
antagonism and antihistaminic effects. 5HT3 stimulation by SSRIs induces gastrointestinal
side effects such as nausea and vomiting. 
The correct answer is: Weight gain caused by antipsychotics – 5HT2C Antagonism,
Hyperprolactinaemia induced by antipsychotics – D­2 blockade, Extrapyramidal side effects of
antipsychotics – D­2 blockade, Gastrointestinal discomfort caused by SSRIs – 5HT3
stimulation, Postural hypotension induced by antipsychotics – Alpha­1 antagonism

Question 2 HiY Pharmacology EMI002
Partially correct Drugs and adverse effects 
For each of the given adverse effect that leads to restricted use and requirements for
Mark 3.00 out of
appropriate monitoring, choose the most likely offending agent.
4.00

Flag question
Lithium
Risperidone
Sodium Valproate
Seizures Bupropion
Methylphenidate
Olanzapine
Ziprasidone
Carbamazepine
Haloperidol
Acamprosate

Ovarian cyst

1494
Lithium
Risperidone
Sodium Valproate
Bupropion
Methylphenidate
Olanzapine
Ziprasidone
Carbamazepine
Haloperidol
Lithium
Acamprosate
Risperidone
Sodium Valproate
Steven Johnson
Bupropion
syndrome
Methylphenidate
Olanzapine
Ziprasidone
Carbamazepine
Haloperidol
Lithium
Acamprosate
Risperidone
Sodium Valproate
QTc prolongation Bupropion
Methylphenidate
Olanzapine
Ziprasidone
Carbamazepine
Haloperidol
Check
Acamprosate

Explanation: 
Seizures may occur with Bupropion at doses above 400 mg per day. 
Ziprasidone prolongs QTc to a greater extent than quetiapine, risperidone, olanzapine and
haloperidol (Taylor, 2003); so it is contraindicated in patients with a history of cardiac
arrhythmias. 
Nearly 10­15% of patients who are on carbamazepine develop a benign maculopapular rash
within first three weeks of treatment and usually leads to resolution of symptoms. Some
patients may develop severe life threatening syndrome called Steven Johnson syndrome.
This complication is also seen with patients on Lamotrigine. 
Valproate can cause hyperandrogenism in women and has been linked with the development
of polycystic ovaries. Epilepsy, for which valproate is widely used, is itself tipped to increase
PCOD occurrence.10% woman on valproate have new onset PCOD. The relative risk is 7.5
for PCOD. On stopping, most people remit from PCOD. The mechanism by which valproate
causes PCOS is thought to be related to the stimulation of ovarian androgen production,
weight gain and insulin resistance. 
http://www.ncbi.nlm.nih.gov/pubmed/12697001
The correct answer is: Seizures – Bupropion, Ovarian cyst – Sodium Valproate, Steven
Johnson syndrome – Carbamazepine, QTc prolongation – Ziprasidone

Question 3 HiY Pharmacology EMI003
Correct

1495
Mark 3.00 out of Psychiatric side effects of non­psychiatric medications
3.00 For each of the side effects profile given below identify the possible offending drug
Flag question

A 74­year­old
Prednisolone
gentleman being Amiodarone
treated for resting Bendroflumethiazide
tremors develops acute Fluoxetine
confusional state with Ramipril
paranoia and visual Levodopa
hallucinations Digoxin
Lithium
Alpha Methyldopa
Prednisolone
A 67­year­old lady
taking treatment for
Isoniazid
Amiodarone
swollen legs complains Bendroflumethiazide
of visual halos, which Fluoxetine
are green in colour and Ramipril
appears to be Levodopa
confused. Digoxin
Lithium
Alpha Methyldopa
Prednisolone
A 56­year­old lady on Isoniazid
Amiodarone
treatment for
Bendroflumethiazide
polymyalgia
Fluoxetine
rheumatica becomes
hypomanic and
Ramipril
confused. Levodopa
Digoxin
Lithium
Alpha Methyldopa
Check
Isoniazid

Explanation: The patient in scenario 1 is likely to be taking treatment for Parkinson's disease
using levodopa. Levodopa is associated with high rates of visual hallucinations. Other
psychiatric side effects of Levodopa consist of psychosis, hallucinations, confusion, euphoria,
depression, insomnia and abnormal dreams. 
The patient in scenario 2 is likely to be suffering from cardiac failure, and is taking digoxin.
One of the side effects of cardiac glycosides such as digoxin is a change in vision
characterized by seeing a halo or light around objects. 
The patient in scenario 3 is likely to be taking steroids for her autoimmune condition.
Corticosteroids are commonly implicated in iatrogenic depression, mania or hypomania and
confusion.
The correct answer is: A 74­year­old gentleman being treated for resting tremors develops
acute confusional state with paranoia and visual hallucinations – Levodopa, A 67­year­old
lady taking treatment for swollen legs complains of visual halos, which are green in colour
and appears to be confused. – Digoxin, A 56­year­old lady on treatment for polymyalgia
rheumatica becomes hypomanic and confused. – Prednisolone

Question 4 HiY Pharmacology EMI004
Correct

1496
Mark 3.00 out of Sexual side effects 
3.00 For each of the following side effects, choose the most likely associated cause from the given
Flag question
list

A 34­year­old patient
Risperidone
presents to an Paroxetine
emergency unit with Mirtazapine
painfully prolonged Citalopram
erection. He is being Duloxetine
treated for low mood Trazodone
and insomnia. Childhood trauma
Venlafaxine
A 41­year­old man Fluoxetine
complains of
Risperidone
decreased sexual Paroxetine
interest, as he is not Mirtazapine
able to ejaculate Citalopram
normally. He has been Duloxetine
maintained on a Trazodone
treatment for chronic Childhood trauma
auditory hallucinations. Venlafaxine
Fluoxetine
Risperidone
A 32­year­old woman Paroxetine
complains of new onset
Mirtazapine
pain during intercourse.
Citalopram
She recently started
treatment for
Duloxetine
depression. Trazodone
Childhood trauma
Venlafaxine
Fluoxetine
Check

Explanation: 
Priapism is reported in <1 in 1000 cases taking trazodone. 
From the given list of drugs, risperidone can cause dry ejaculation or retrograde ejaculation
leading to loss of interest. Phenothiazines are the worst offenders in this regard. 
Paroxetine produces anticholinergic dryness in vagina leading to vaginismus. Paroxetine is a
worse offender when compared to other SSRIs in this regard.
The correct answer is: A 34­year­old patient presents to an emergency unit with painfully
prolonged erection. He is being treated for low mood and insomnia. – Trazodone, A 41­year­
old man complains of decreased sexual interest, as he is not able to ejaculate normally. He
has been maintained on a treatment for chronic auditory hallucinations. – Risperidone, A 32­
year­old woman complains of new onset pain during intercourse. She recently started
treatment for depression. – Paroxetine

Question 5 HiY Pharmacology EMI005
Correct

Mark 3.00 out of

1497
3.00 Teratogenicity of psychotropics
Flag question
Choose the most likely offending agent for each of the following teratogenic effects from the
given list.

Diazepam
Reboxetine
Lamotrigine
Newborn with
Mirtazapine
anencephaly
Paroxetine
Carbamazepine
Lithium
Valproate
Zopiclone
Diazepam
Venlafaxine
Reboxetine
Lamotrigine
Newborn with defect of
Mirtazapine
the tricuspid valve
Paroxetine
Carbamazepine
Lithium
Valproate
Zopiclone
Diazepam
Venlafaxine
Reboxetine
Lamotrigine
Newborn with spina
Mirtazapine
bifida
Paroxetine
Carbamazepine
Lithium
Valproate
Zopiclone
Check
Venlafaxine

Explanation: The prevalence of spina bifida is approximately 1 to 2% with valproate exposure
and 0.5% with carbamazepine. Sodium valproate is the most teratogenic of all antiepileptics.
The risk for any birth defect is quoted as 7.2% in Maudsley & NICE guidelines. Congenital
anomalies include risk of neural tube defect (1­2 %), risk of spina bifida (10 fold increase),
anencephaly, digital and limb defects, heart defects (VSD, Pulmonary stenosis etc. 4 fold
increase), urogenital malformations, low birth weight and psychomotor slowness. A greater
frequency of cognitive impairment in children is associated with valproate in mothers. Lithium
induced Ebstein's anomaly presents with cyanotic heart disease characterized by tricuspid
valve malformation.
The correct answer is: Newborn with anencephaly – Valproate, Newborn with defect of the
tricuspid valve – Lithium, Newborn with spina bifida – Valproate

Question 6 HiY Pharmacology EMI006
Correct Drug Interactions 
Identify the antidepressant drug that is most likely to interact and should be avoided in the
Mark 3.00 out of
following situations
3.00

Flag question

1498
A 33­year­old woman
Fluvoxamine
treated for mixed
Fluoxetine
anxiety & depressive
disorder. She is on
Citalopram
diazepam, started by Reboxetine
her GP. Mirtazapine
Agomelatine
Paroxetine
Escitalopram
Venlafaxine
Fluvoxamine
Mianserin
Fluoxetine
A 21­year­old asthmatic Citalopram
on theophylline is
Reboxetine
suffering from
Mirtazapine
depression
Agomelatine
Paroxetine
Escitalopram
Venlafaxine
Fluvoxamine
A 67­year­old Mianserin
Fluoxetine
gentleman with a Citalopram
previous history of
Reboxetine
stroke and is now
depressed. He is on
Mirtazapine
regular warfarin. Agomelatine
Paroxetine
Escitalopram
Venlafaxine
Check
Mianserin

Explanation: 
Fluvoxamine reduces the clearance of both diazepam and its active metabolite, N­
desmethyldiazepam, there is a high likelihood of substantial accumulation of both.
Accordingly, diazepam and fluvoxamine should not ordinarily be co­administered.
Fluvoxamine reduces the clearance of theophylline approximately 3­fold via CYP1A2
inhibition. Therefore, if theophylline is co­administered with fluvoxamine, its dose should be
reduced to one­third of the usual daily maintenance dose and plasma concentrations of
theophylline should be monitored. No dosage adjustment is required for fluvoxamine. When
fluvoxamine is administered with warfarin, warfarin plasma levels increases by 98% and
prothrombin times are prolonged. Hence, anticoagulant dose must be adjusted accordingly. 
Ref: Luvox (Fluvoxamine Maleate Tablets) Drug Information. Retrieved from
http://www.rxlist.com/luvox­drug/warnings­precautions.htm
The correct answer is: A 33­year­old woman treated for mixed anxiety & depressive disorder.
She is on diazepam, started by her GP. – Fluvoxamine, A 21­year­old asthmatic on
theophylline is suffering from depression – Fluvoxamine, A 67­year­old gentleman with a
previous history of stroke and is now depressed. He is on regular warfarin. – Fluvoxamine

Question 7 HiY Pharmacology EMI007
Correct Side effects of treating bipolar disorder
The following patients with bipolar disorder have all presented with side effects. Please
Mark 4.00 out of
choose the most likely offending agent from the list shown here.
4.00

1499
Flag question
A 38­year­old man Lithium
comes to see you with Risperidone
severe central Lamotrigine
abdominal pain. It is Topiramate
associated with Quetiapine
vomiting. An amylase
Valproate
level is highly elevated.
Olanzapine
Lorazepam
Fluoxetine
Lithium
A 52­year­old woman Risperidone
complains of thirst. She Lamotrigine
is also passing a lot of
Topiramate
urine. On examination,
Quetiapine
she is well hydrated but
has a fine tremor. Valproate
Olanzapine
Lorazepam
A 25­year­old man with Fluoxetine
bipolar disorder starts
on a second drug. The Lithium
dose is increased Risperidone
quickly. He develops a Lamotrigine
rash on his skin and
Topiramate
blistering on his lips
Quetiapine
and gums and a rash
around his eyes. On
Valproate
examination, you find Olanzapine
target­like lesions on Lorazepam
his trunk. Fluoxetine
A 40­year­old man
comes to see you, as
Lithium
he is worried about his Risperidone
libido. He is also Lamotrigine
concerned as he thinks Topiramate
he is developing Quetiapine
breasts. On Valproate
examination, you find Olanzapine
gynaecomastia. Lorazepam
Fluoxetine
Check

Explanation: 
Valproate can cause pancreatitis associated with abdominal pain. 
Lithium is associated with polyuria and polydipsia. The tremor is an additional clue in this
case. 
Lamotrigine has a well­established association with rashes. 
Risperidone loses its atypicality at higher doses and can produce elevations in prolactin
levels, leading to gynaecomastia.
The correct answer is: A 38­year­old man comes to see you with severe central abdominal
pain. It is associated with vomiting. An amylase level is highly elevated. – Valproate, A 52­
year­old woman complains of thirst. She is also passing a lot of urine. On examination, she is

1500
well hydrated but has a fine tremor. – Lithium, A 25­year­old man with bipolar disorder starts
on a second drug. The dose is increased quickly. He develops a rash on his skin and
blistering on his lips and gums and a rash around his eyes. On examination, you find target­
like lesions on his trunk. – Lamotrigine, A 40­year­old man comes to see you, as he is worried
about his libido. He is also concerned as he thinks he is developing breasts. On examination,
you find gynaecomastia. – Risperidone

Question 8 HiY Pharmacology EMI008
Correct Drug Toxicity (1)
Choose the most important treatment option for the drug­induced toxic symptoms given below
Mark 4.00 out of
4.00

Flag question
Diazepam
An 83­year­old woman Acidification of urine
presenting with signs of Peritoneal dialysis
lithium toxicity following Propranalol
diarrhoea and presents Haemodialysis
with tremors and ataxia Rehydration
Naloxone
Thiopentone
Alkalinisation of urine
Diazepam
A 23­year­old
Flumazenil
Acidification of urine
gentleman who took an Phentolamine
Peritoneal dialysis
overdose of methadone Propranalol
prescription and has Haemodialysis
pinpoint pupils. Rehydration
Naloxone
Thiopentone
Alkalinisation of urine
Diazepam
Flumazenil
Acidification of urine
A 56­year­old Phentolamine
Peritoneal dialysis
gentleman who is on Propranalol
phenelzine therapy Haemodialysis
inadvertently consumes Rehydration
large quantities of Naloxone
cheese and wine and Thiopentone
has a very high blood
Alkalinisation of urine
pressure.
Flumazenil
Phentolamine

A 56­year­old Diazepam
gentleman with Acidification of urine
depression took a Peritoneal dialysis
dangerous overdose of
Propranalol
barbiturates needs
Haemodialysis
additional treatment to
enhance drug Rehydration
clearance. Naloxone
Thiopentone
Alkalinisation of urine
Check
Flumazenil
Phentolamine

1501
Explanation: 
Lithium toxicity occurs in conditions of overdose or dehydration. Gastrointestinal symptoms
are followed by neurological symptoms. Immediate stopping of lithium and medical attention
is required. Hemodialysis is recommended in conditions of severe lithium toxicity. 
Methadone: In the event of methadone overdose, naloxone should be administered, and the
preferred route is the intramuscular route. According to Maudsley Prescribing Guidelines,
"Naloxone is short acting and, therefore, the effect may reverse within 20 minutes to 1 hour,
meaning that a patient can revert to an overdose state". Therefore, ongoing monitoring should
be provided after naloxone administration. 
Phenelzine: An MAOI­induced hypertensive crisis can be treated with alpha­adrenergic
antagonists such as phentolamine or even chlorpromazine, which is immediately available in
most psychiatric wards. This can lower blood pressure in few minutes. 
Barbiturates: the Ionized drug is more or less trapped in the glomerular filtrate and does not
get reabsorbed. Hence, renal clearance is more for ionized drugs. A weak base can be
ionised by acidifying urine; a weak acid by alkalinising urine. Hence for salicylate (aspirin)
overdose, and barbiturate overdose, alkalinisation helps. Acidification may help in the
elimination of amphetamines and phencyclidine, but often complications of such a procedure
override any benefits. 

The correct answer is: An 83­year­old woman presenting with signs of lithium toxicity following
diarrhoea and presents with tremors and ataxia – Haemodialysis, A 23­year­old gentleman
who took an overdose of methadone prescription and has pinpoint pupils. – Naloxone, A 56­
year­old gentleman who is on phenelzine therapy inadvertently consumes large quantities of
cheese and wine and has a very high blood pressure. – Phentolamine, A 56­year­old
gentleman with depression took a dangerous overdose of barbiturates needs additional
treatment to enhance drug clearance. – Alkalinisation of urine

Question 9 HiY Pharmacology EMI009
Partially correct Drug Toxicity (2)
Choose an appropriate treatment option for the toxic symptoms produced by psychotropic
Mark 3.00 out of
drugs
4.00

Flag question
A 66­year­old woman Flumazenil
who took an overdose Naloxone
of aspirin and presents
Haemodialysis
with altered levels of
Phentolamine
consciousness requires
Alkalinisation of urine
additional treatment to
enhance drug Thiopentone
clearance Diazepam
Propranalol
A 21­year­old drug Peritoneal dialysis
Flumazenil
addict consumed high­ Rehydration
dose amphetamine in a Naloxone
Acidification of urine
Haemodialysis
nightclub accidentally
and went to a state of Phentolamine
coma. He requires Alkalinisation of urine
additional treatment to Thiopentone
enhance drug Diazepam
clearance
Propranalol
Peritoneal dialysis

1502
An 18­year­old girl Flumazenil
presented to A&E after Naloxone
consuming an Haemodialysis
overdose of diazepam Phentolamine
tablets and presents Alkalinisation of urine
with drowsiness and
Thiopentone
confusion
Diazepam
Propranalol
A 56­year­old woman Peritoneal dialysis
Flumazenil
took an overdose of Rehydration
Naloxone
amitriptyline due to Acidification of urine
Haemodialysis
chronic pain presented
Phentolamine
to A&E with
Alkalinisation of urine
hyperreflexia and
altered levels of Thiopentone
consciousness. Diazepam
Propranalol
Peritoneal dialysis
Check
Rehydration
Acidification of urine

Explanation:
Aspirin & amphetamines: the ionized forms are more or less trapped in the glomerular filtrate
and do not get reabsorbed. Hence, renal clearance is more for ionized drugs. A weak base
can be ionised by acidifying urine; a weak acid by alkalinising urine. Hence for salicylate
(aspirin) overdose, and barbiturate overdose, alkalinisation helps. Acidification may help in
the elimination of amphetamines and phencyclidine, but often complications of such a
procedure override any benefits. 
Diazepam: Benzodiazepines are not considered fatal on their own even in cases of a
considerable overdose. Nevertheless, diazepam overdose is a medical emergency. If
respiratory depression or cardiovascular complications are present, then an antidote for
benzodiazepines called flumazenil can be used. Unlike diazepam that stays in the body for
days, flumazenil is a short­acting drug, and thus multiple doses of flumazenil may be required
to reverse the effect of overdose fully. 
Amitriptyline in higher doses can cause hypotension, hyperreflexia, and QT prolongation and
precipitate seizures. Diazepam is usually administered to prevent seizures, as amitriptyline
can reduce seizure threshold significantly.
The correct answer is: A 66­year­old woman who took an overdose of aspirin and presents
with altered levels of consciousness requires additional treatment to enhance drug clearance
– Alkalinisation of urine, A 21­year­old drug addict consumed high­dose amphetamine in a
nightclub accidentally and went to a state of coma. He requires additional treatment to
enhance drug clearance – Acidification of urine, An 18­year­old girl presented to A&E after
consuming an overdose of diazepam tablets and presents with drowsiness and confusion –
Flumazenil, A 56­year­old woman took an overdose of amitriptyline due to chronic pain
presented to A&E with hyperreflexia and altered levels of consciousness. – Diazepam

Question 10 HiY Pharmacology EMI010
Correct Clozapine­related adverse effects 
For each of the following side effects caused by clozapine, identify the best plan of action
Mark 5.00 out of
5.00

1503
Flag question
Consider prophylactic valproate treatment
A 34­year­old woman Increase the dose
started on clozapine Recommend high fibre diet
suffers from high Advise gradual postural change
temperature
Avoid fluids before bed time
Check full blood count
Perform EEG
Dietary counselling
Hyoscine 300 micrograms at night time
Consider prophylactic valproate treatment
A 65­year old Titrate the dose upwards quickly
Increase the dose
gentleman with one Give smaller dose in the morning
Recommend high fibre diet
episode of seizures ten
Advise gradual postural change
years ago now requires
high dose of clozapine
Avoid fluids before bed time
to treat psychosis Check full blood count
Perform EEG
Dietary counselling
Hyoscine 300 micrograms at night time
Consider prophylactic valproate treatment
Titrate the dose upwards quickly
Increase the dose
A 33­year­old woman Give smaller dose in the morning
Recommend high fibre diet
has gained weight on
Advise gradual postural change
clozapine after taking it
Avoid fluids before bed time
for three months
Check full blood count
Perform EEG
Dietary counselling
Hyoscine 300 micrograms at night time
Consider prophylactic valproate treatment
Titrate the dose upwards quickly
Increase the dose
A 45­year­old woman Give smaller dose in the morning
Recommend high fibre diet
complains of excessive
Advise gradual postural change
salivation especially at
night time
Avoid fluids before bed time
Check full blood count
Perform EEG
Dietary counselling
Hyoscine 300 micrograms at night time
Consider prophylactic valproate treatment
A 56­year­old woman
Titrate the dose upwards quickly
Increase the dose
recently started on Give smaller dose in the morning
Recommend high fibre diet
clozapine complaints of Advise gradual postural change
dizzy spells on waking Avoid fluids before bed time
up from bed Check full blood count
Perform EEG
Dietary counselling
Hyoscine 300 micrograms at night time
Check
Titrate the dose upwards quickly
Give smaller dose in the morning

Explanation: 
Fever is common in the first three weeks after starting on clozapine treatment. Clozapine
induces inflammatory response (increased C­ reactive protein and interleukin 6). The first step
is to give antipyretics and check full blood count. 

1504
Seizures are dose­related side effects of clozapine treatment. Consider prophylactic valproate
if on a high dose or with high plasma levels. EEG abnormalities are common in those on
clozapine. 
For clozapine­induced weight gain, dietary counselling is essential. 
Hypersalivation due to clozapine treatment is very troublesome at night and last for many
months. Hyoscine 300 micrograms sucked and swallowed at night may be an option though
not much evidence base exists in this regard. Pirenzepine can also be tried. 
Hypotension usually occurs in the early phase of treatment (first four weeks). It is important to
advise patient to take time when standing up. If hypotension occurs, reduce dose or slow rate
of increase. If very severe, consider Fludrocortisone.
The correct answer is: A 34­year­old woman started on clozapine suffers from high
temperature – Check full blood count, A 65­year old gentleman with one episode of seizures
ten years ago now requires high dose of clozapine to treat psychosis – Consider prophylactic
valproate treatment, A 33­year­old woman has gained weight on clozapine after taking it for
three months – Dietary counselling, A 45­year­old woman complains of excessive salivation
especially at night time – Hyoscine 300 micrograms at night time, A 56­year­old woman
recently started on clozapine complaints of dizzy spells on waking up from bed – Advise
gradual postural change

Question 11 HiY Pharmacology EMI011
Correct Plasma levels
For each of the following drugs, identify the most appropriate range of target plasma levels.
Mark 6.00 out of
6.00

Flag question
350­500 micrograms/L
50­150 micrograms/L
150­210 micrograms/L
Sodium valproate  
200­320 micrograms/L
0.6­1 mmol/L
100­200 micrograms/L
More than 7 mg/L
50­100 mg/L
350­500 micrograms/L
10­20 mg/L
50­150 micrograms/L
20­60 micrograms/L
150­210 micrograms/L
Lithium  
200­320 micrograms/L
0.6­1 mmol/L
100­200 micrograms/L
More than 7 mg/L
50­100 mg/L
350­500 micrograms/L
10­20 mg/L
50­150 micrograms/L
20­60 micrograms/L
150­210 micrograms/L
200­320 micrograms/L  

Carbamazepine 0.6­1 mmol/L
100­200 micrograms/L
More than 7 mg/L
50­100 mg/L
10­20 mg/L
20­60 micrograms/L

1505
Clozapine 350­500 micrograms/L
50­150 micrograms/L
150­210 micrograms/L
200­320 micrograms/L  
0.6­1 mmol/L
100­200 micrograms/L
More than 7 mg/L
50­100 mg/L
350­500 micrograms/L
10­20 mg/L
50­150 micrograms/L
20­60 micrograms/L
150­210 micrograms/L
Phenytoin  
200­320 micrograms/L
0.6­1 mmol/L
100­200 micrograms/L
More than 7 mg/L
50­100 mg/L
350­500 micrograms/L
10­20 mg/L
50­150 micrograms/L
20­60 micrograms/L
150­210 micrograms/L
Amitriptyline  
200­320 micrograms/L
0.6­1 mmol/L
100­200 micrograms/L
More than 7 mg/L
50­100 mg/L
Check
10­20 mg/L
20­60 micrograms/L

Explanation: 
Clozapine: 350­500 micrograms/L: 
Carbamazepine:  More than 7 mg/L. 
Lithium: 0.6­1 mmol/L. 
Sodium valproate: 50­100 mg/L. Uncertainty exists over the most appropriate levels in
epilepsy and bipolar disorder. Some evidence that levels up to 125 mg/L is tolerated and
more effective than lower levels in mania. 
Phenytoin: 10­20 mg/L. Phenytoin follows zero­order kinetics. Free rather than plasma­bound
levels of phenytoin is required to correlate with clinical features. 
Amitriptyline: 100­200 micrograms/L. 
Ref: Maudsley prescribing guidelines 9e­ Pg 3.
The correct answer is: Sodium valproate – 50­100 mg/L, Lithium – 0.6­1 mmol/L,
Carbamazepine – More than 7 mg/L, Clozapine – 350­500 micrograms/L, Phenytoin – 10­20
mg/L, Amitriptyline – 100­200 micrograms/L

Question 12 HiY Pharmacology EMI012
Partially correct Mechanism of action 
Select the primary mechanism of action for each of the following drugs from the list shown
Mark 4.00 out of
here.
5.00

Flag question

1506
Presynaptic alpha­2 agonist
Ramelteon MT3/MT4 receptor agonist
GABA­A agonist
NMDA glutamate receptor antagonist  
MT1/MT2 receptor agonist
D­2 receptor partial agonist
5 ­HT1­a partial agonist
Partial agonist at the α4β2 unit of nicotinic acetylcholine receptor
Presynaptic alpha­2 agonist
MAO­B Inhibitor
MT3/MT4 receptor agonist
Selective presynaptic norepinephrine reuptake inhibition
GABA­A agonist
Riluzole  
NMDA glutamate receptor antagonist
MT1/MT2 receptor agonist
D­2 receptor partial agonist
5 ­HT1­a partial agonist
Partial agonist at the α4β2 unit of nicotinic acetylcholine receptor
Presynaptic alpha­2 agonist
MAO­B Inhibitor
MT3/MT4 receptor agonist
Selective presynaptic norepinephrine reuptake inhibition
GABA­A agonist
Atomoxetine  
NMDA glutamate receptor antagonist
MT1/MT2 receptor agonist
D­2 receptor partial agonist
5 ­HT1­a partial agonist
Partial agonist at the α4β2 unit of nicotinic acetylcholine receptor
Presynaptic alpha­2 agonist
MAO­B Inhibitor
MT3/MT4 receptor agonist
Selective presynaptic norepinephrine reuptake inhibition
GABA­A agonist
Clonidine  
NMDA glutamate receptor antagonist
MT1/MT2 receptor agonist
D­2 receptor partial agonist
5 ­HT1­a partial agonist
Partial agonist at the α4β2 unit of nicotinic acetylcholine receptor
Presynaptic alpha­2 agonist
MAO­B Inhibitor
MT3/MT4 receptor agonist
Selective presynaptic norepinephrine reuptake inhibition
GABA­A agonist
NMDA glutamate receptor antagonist  
MT1/MT2 receptor agonist
Varenicline D­2 receptor partial agonist
5 ­HT1­a partial agonist
Partial agonist at the α4β2 unit of nicotinic acetylcholine receptor
MAO­B Inhibitor
Selective presynaptic norepinephrine reuptake inhibition

Check

Explanation: 
Ramelteon: This is a novel MT1/MT2 receptor agonist promoted as a hypnotic. It decreases
sleep latency and increases sleep time across all ages; the dose­response curve is flat with
no significant difference in efficacy between the 16­mg or 64­mg doses. Hence, it may have

1507
lower abuse potential than other hypnotics but its endocrinological effects need to be fully
evaluated yet. 
Riluzole: This is approved for use in motor neuron disorder. It is unclear whether this would
help features of frontotemporal dementia associated with MND. Riluzole's mechanism of
action is via sodium channel blockade, high­voltage calcium channel blockade and NMDA­
glutamate receptor antagonism. 
Atomoxetine causes selective inhibition of the presynaptic norepinephrine transporter, in a
fashion not too dissimilar to reboxetine. It is used as a second line drug for ADHD, being
preferred in those with tics, substance abuse history and refractory ADHD. 
Lofexidine & clonidine: In opioid detoxification, if a short duration of treatment is desirable, α­2
adrenergic agonists such as lofexidine and clonidine are preferred to methadone. 
Varenicline is a partial agonist at the α4β2 unit of nicotinic acetylcholine receptor. It is shown
to assist smoking cessation by relieving nicotine withdrawal symptoms and reducing the
rewarding properties of nicotine.
The correct answer is: Ramelteon – MT1/MT2 receptor agonist, Riluzole – NMDA glutamate
receptor antagonist, Atomoxetine – Selective presynaptic norepinephrine reuptake inhibition,
Clonidine – Presynaptic alpha­2 agonist, Varenicline – Partial agonist at the α4β2 unit of
nicotinic acetylcholine receptor

Question 13 HiY Pharmacology EMI013
Correct Psychotropic induced ECG changes
Psychotropic use is often associated with ECG changes, necessitating a good working
Mark 3.00 out of
knowledge of ECG findings for a psychiatric trainee. For each of the following clinical
3.00
conditions, identify the possible ECG changes.
Flag question

A 65­year­old smoker
with schizophrenia and
IDDM suffers from chest
Prolonged QRS interval
pain and shortness of Corrected QT of 440 ms
breath. The heart rate is P­R interval of 500 ms
40 per minute. In R­R interval of 1500ms
addition to a marked ST P­R interval of 100 ms
elevation in inferior P­R interval of 280 ms
leads, what other R­R interval of 500 ms
feature can be Diffuse ST elevation with saddle formation
expected?
P waves absent before every QRS
A 27­year­old woman Corrected QT of 510 ms
suffers from depression
and is on Amitriptyline.
Prolonged QRS interval
She develops
infectious diarrhoea Corrected QT of 440 ms
and collapses. ECG P­R interval of 500 ms
shows polymorphic R­R interval of 1500ms
ventricular tachycardia. P­R interval of 100 ms
What feature could be P­R interval of 280 ms
seen in her ECG R­R interval of 500 ms
recorded prior to Diffuse ST elevation with saddle formation
developing
P waves absent before every QRS
tachycardia?
Corrected QT of 510 ms

1508
A 40­year­old woman is
Prolonged QRS interval
receiving Transcranial
Corrected QT of 440 ms
Magnetic Stimulation
for resistant P­R interval of 500 ms
depression. She is also R­R interval of 1500ms
taking imipramine. P­R interval of 100 ms
P­R interval of 280 ms
R­R interval of 500 ms
Diffuse ST elevation with saddle formation
P waves absent before every QRS
Check
Corrected QT of 510 ms

Explanation: 
Question 1 describes a case of acute myocardial infarction presenting with bradycardia. ST
elevation is a tell­tale sign of ischemia and inferior wall is commonly involved; RR interval is
prolonged reflecting the bradycardia. 
Accordig to OBrien (2003), "Tricyclic antidepressants delay cardiac conduction mainly by
prolonging the second part of the QRS interval without significantly affecting the first. The
effect is dose­dependent, occurring at both therapeutic and toxic doses, particularly with
amitriptyline, imipramine and nortriptyline, but not doxepin". (Ref:
http://apt.rcpsych.org/content/9/6/414)
TCAs (especially amitriptyline) have a moderate effect on QTc prolongation. There is some
controversy over the exact association between QTc and risk of arrhythmias. Rather strong
evidence links QTc values over 500 ms to an apparently increased risk of polymorphic
ventricular arrhythmia (torsades de pointes).
The correct answer is: A 65­year­old smoker with schizophrenia and IDDM suffers from chest
pain and shortness of breath. The heart rate is 40 per minute. In addition to a marked ST
elevation in inferior leads, what other feature can be expected?
– R­R interval of 1500ms, A 27­year­old woman suffers from depression and is on
Amitriptyline. She develops infectious diarrhoea and collapses. ECG shows polymorphic
ventricular tachycardia. What feature could be seen in her ECG recorded prior to developing
tachycardia?
– Corrected QT of 510 ms, A 40­year­old woman is receiving Transcranial Magnetic
Stimulation for resistant depression. She is also taking imipramine.
– Prolonged QRS interval

Question 14 HiY Pharmacology EMI014
Correct Adverse effects (1)
Identify the type of side effects caused by antipsychotics, using descriptions given below.
Mark 3.00 out of
3.00

Flag question
Parkinsonism
An 18­year­old Hyponatraemia
gentleman commenced Serotonin syndrome
on haloperidol presents
Cerebellar ataxia
dramatically with eyes
rolled upwards, and a
Cholinergic rebound syndrome
twisted neck. Acute dystonia
Neuroleptic malignant syndrome
SIADH
Akathisia
Tardive dyskinesia

1509
25­year­old woman on Parkinsonism
the ward commenced
Hyponatraemia
on risperidone is
constantly pacing up Serotonin syndrome
and down, rocking from Cerebellar ataxia
foot to foot and always Cholinergic rebound syndrome
crossing/uncrossing Acute dystonia
legs when seated. Neuroleptic malignant syndrome
SIADH
Akathisia
Parkinsonism
A 33­year­old
Tardive dyskinesia
gentleman was started Hyponatraemia
on trifluoperazine five Serotonin syndrome
days ago. He is now Cerebellar ataxia
complaining of tremors, Cholinergic rebound syndrome
stiffness and inability to Acute dystonia
initiate body movement. Neuroleptic malignant syndrome
SIADH
Akathisia
Check
Tardive dyskinesia

Explanation: 
Dystonias are brief or prolonged contractions of specific groups of muscles resulting in
symptoms such as oculogyric crises, tongue protrusion, trismus, torticollis, blepharospasm.
Dystonias are often seen in younger men receiving a high dose of high­potency medications. 
Akathisia includes both subjective and objective ­ feelings and signs of restlessness. Patients
may exhibit inability to relax, jitteriness, pacing, rocking with alternation of sitting and standing.
Drug­induced parkinsonism is seen in 15­20% of patients treated with antipsychotics, seen
within 90 days of treatment (5 to 90) and is characterized by muscle stiffness, cogwheel
rigidity, shuffling gait, stooped posture, and drooling.
The correct answer is: An 18­year­old gentleman commenced on haloperidol presents
dramatically with eyes rolled upwards, and a twisted neck. – Acute dystonia, 25­year­old
woman on the ward commenced on risperidone is constantly pacing up and down, rocking
from foot to foot and always crossing/uncrossing legs when seated. – Akathisia, A 33­year­old
gentleman was started on trifluoperazine five days ago. He is now complaining of tremors,
stiffness and inability to initiate body movement. – Parkinsonism

Question 15 HiY Pharmacology EMI015
Correct Adverse effects (2)
Identify the type of side effects, using descriptions given below.
Mark 3.00 out of
3.00

Flag question A 36­year­old woman
Cerebellar ataxia
treated on fluoxetine Hyponatraemia
and phenelzine for her Parkinsonism
depression has now Neuroleptic malignant syndrome
developed diarrhoea, Tardive dyskinesia
excessive sweating Acute dystonia
and myoclonus. Cholinergic rebound syndrome
Serotonin syndrome
Antidepressant discontinuation syndrome

1510
A 76­year­old
gentleman admitted to
hospital is on Cerebellar ataxia
paroxetine for more Hyponatraemia
than five years. He has Parkinsonism
not taken his Neuroleptic malignant syndrome
antidepressant Tardive dyskinesia
medication for last five Acute dystonia
days. He has Cholinergic rebound syndrome
developed flu­like
Serotonin syndrome
symptoms,
Antidepressant discontinuation syndrome
paresthesias and
complains of electric
shock­like sensations

A 73­year­old woman Cerebellar ataxia
started on fluoxetine Hyponatraemia
recently was noticed by Parkinsonism
her CPN to be
Neuroleptic malignant syndrome
confused and is
Tardive dyskinesia
complaining of
dizziness, nausea and Acute dystonia
lethargy Cholinergic rebound syndrome
Serotonin syndrome
Antidepressant discontinuation syndrome
Check

Explanation: 
Serotonin syndrome is a result of excessive serotonergic transmission in the brain. Although
no single receptor appears to be responsible, it is possibly mediated via 5HT 2A receptor
stimulation. Diarrhea, myoclonus, diaphoresis, hyperactive reflexes, ataxia, hypomanic or
labile mood, tremors and disorientation are the characteristic features. It may mimic NMS or
anticholinergic syndrome in those receiving psychotropics. 
The abrupt withdrawal of SSRI especially paroxetine, fluvoxamine or venlafaxine (shorter half­
life), is associated with dizziness, nausea, headache, depression, anxiety, insomnia, poor
concentration, flu­like symptoms, paresthesias, and migraine­like symptoms. It usually
requires at least 6 weeks of treatment before discontinuation and resolves spontaneously in 3
weeks. 
Most antidepressants are associated with hyponatraemia. The onset is usually within 30 days
of treatment and is probably not dose­related. The mechanism of this adverse effect is likely to
be mediated via the syndrome of inappropriate ADH secretion (SIADH). Maudsley Prescribing
Guidelines suggests that serotonergic drugs are relatively more likely to cause
hyponatraemia. The clinical symptoms of hyponatraemia include dizziness, lethargy, nausea,
confusion, cramps and seizures.
The correct answer is: A 36­year­old woman treated on fluoxetine and phenelzine for her
depression has now developed diarrhoea, excessive sweating and myoclonus. – Serotonin
syndrome, A 76­year­old gentleman admitted to hospital is on paroxetine for more than five
years. He has not taken his antidepressant medication for last five days. He has developed
flu­like symptoms, paresthesias and complains of electric shock­like sensations –
Antidepressant discontinuation syndrome, A 73­year­old woman started on fluoxetine recently
was noticed by her CPN to be confused and is complaining of dizziness, nausea and lethargy
– Hyponatraemia

1511
Question 16 HiY Pharmacology EMI016
Partially correct Prescription for unlicensed indications
Choose an appropriate drug that could be used in the management of the following
Mark 4.00 out of
conditions.
5.00

Flag question
Olanzapine
Sodium valproate
Self­injurious
Modafinil
behaviour in people
Lamotrigine
with learning
disabilities
Fluoxetine
Naltrexone
Cyproheptadine
Melatonin
Methylphenidate
Olanzapine
Sodium valproate
Treatment and Modafinil
prophylaxis of bipolar Lamotrigine
mania Fluoxetine
Naltrexone
Cyproheptadine
Melatonin
Methylphenidate
Olanzapine
Sodium valproate
Modafinil
Treatment of bipolar
Lamotrigine
depression
Fluoxetine
Naltrexone
Cyproheptadine
Melatonin
Methylphenidate
Olanzapine
Sodium valproate
Attention deficit Modafinil
hyperactivity disorder in Lamotrigine
people over 18 Fluoxetine
Naltrexone
Cyproheptadine
Melatonin
Methylphenidate
Olanzapine
Sodium valproate
Modafinil
Akathisia Lamotrigine
Fluoxetine
Naltrexone
Cyproheptadine
Melatonin
Methylphenidate
Check

Explanation: 

1512
Although there is limited evidence­base, naltrexone has been used in the management of
self­injurious behaviour in people with learning disabilities. 
There is established clinical practice in the treatment and prophylaxis of bipolar disorder with
sodium valproate. 
RCT evidence supports efficacy of lamotrigine for the treatment of bipolar depression. 
There is established clinical practice and well­supported evidence base for the use of
methylphenidate in the treatment of ADHD in children. In some cases, if required it can also
be used for adults over the age of 18. 
There is some evidence to support the efficacy of cyproheptadine in the treatment of akathisia
induced by antipsychotic medication though this is rare in practice.
The correct answer is: Self­injurious behaviour in people with learning disabilities –
Naltrexone, Treatment and prophylaxis of bipolar mania – Sodium valproate, Treatment of
bipolar depression – Lamotrigine, Attention deficit hyperactivity disorder in people over 18 –
Methylphenidate, Akathisia – Cyproheptadine

Question 17 HiY Pharmacology EMI017
Partially correct Management of drug­induced ECG changes
Mr. Y is a 55­year­old gentleman treated for psychosis and is on 600 mg of quetiapine. You
Mark 6.75 out of
are monitoring his cardiac functions by recording an ECG. What actions will be appropriate in
9.00
the following situations?
Flag question

Arrange for an echocardiogram
Refer to cardiologist immediately
Reduce the dose and repeat ECG
QTc more than 500ms Prescribe diltiazem
No action required
Refer to a cardiologist immediately
Increase the dose of quetiapine
Alter the time of administration
Arrange for treadmill test
Arrange for an echocardiogram
Refer to cardiologist immediately
Reduce the dose and repeat ECG
QTc less than 440ms Prescribe diltiazem
No action required
Refer to a cardiologist immediately
Increase the dose of quetiapine
Alter the time of administration
Arrange for treadmill test
Arrange for an echocardiogram
Refer to cardiologist immediately
Reduce the dose and repeat ECG
Prescribe diltiazem
No action required
QTc interval of 470 ms
Refer to a cardiologist immediately
Increase the dose of quetiapine
Alter the time of administration
Arrange for treadmill test

1513
Arrange for an echocardiogram
Refer to cardiologist immediately
Reduce the dose and repeat ECG
Abnormal T wave
Prescribe diltiazem
morphology
No action required
Refer to a cardiologist immediately
Increase the dose of quetiapine
Alter the time of administration
Arrange for treadmill test
Check

Explanation: 
Many psychotropic drugs are associated with ECG changes, and it is probable that certain
drugs are causally linked to serious ventricular arrhythmias and sudden cardiac death.
Therefore measure QTc in all patients prescribed antipsychotics. If QTc interval is less than
440 ms in men or less than 470 ms in women, no action is generally required unless
abnormalities are noted in T­wave morphology in which case the treatment must be reviewed
and urgent advice sought from the cardiology team. QTc of more than 440 ms in men or more
than 470 ms in women but less than 500 ms must prompt consideration for reducing dose or
switching to drug of lower effect; also repeat the ECG and consider referral to a cardiologist.
QTc interval of more than 500 ms necessitates stopping the suspected causative drug and
switching to a drug with lower QTc effect; this also requires a prompt referral to the cardiologist
immediately.
The correct answer is: QTc more than 500ms
– Refer to cardiologist immediately, QTc less than 440ms – No action required, QTc interval of
470 ms
– Reduce the dose and repeat ECG, Abnormal T wave morphology
– Refer to a cardiologist immediately

Question 18 HiY Pharmacology EMI018
Correct Remedial measures following rapid tranquillization
Mr. Y is a 45­year­old gentleman admitted to the psychiatric intensive care unit for agitation
Mark 7.00 out of
and extremely disturbed behaviour. He was given 1 mg of lorazepam and 5 mg of haloperidol
7.00
intramuscularly. For each of the following problems, identify the important action/s to be taken.
Flag question

Give Oxygen
Ensure patient is not lying face­down
Give procyclidine 5­10 mg IM
Increased temperature
Initiate CPR
(Choose ONE)
Attempt cardioversion
Refer to specialist medical care immediately
Make the patient to lie flat
Give Flumazenil
Ventilate mechanically on the psychiatric ward
Check creatinine kinase urgently

Fall in blood pressure
by more than 30 mm
Hg 

1514
Give Oxygen
Ensure patient is not lying face­down
Give procyclidine 5­10 mg IM
Initiate CPR
Attempt cardioversion
Refer to specialist medical care immediately
Make the patient to lie flat
Give Flumazenil
Ventilate mechanically on the psychiatric ward
Give Oxygen
Check creatinine kinase urgently
Ensure patient is not lying face­down
Give procyclidine 5­10 mg IM
Acute dystonia  Initiate CPR
Attempt cardioversion
Refer to specialist medical care immediately
Make the patient to lie flat
Give Flumazenil
Ventilate mechanically on the psychiatric ward
Give Oxygen
Check creatinine kinase urgently
Ensure patient is not lying face­down
Give procyclidine 5­10 mg IM
Irregular or slow pulse
Initiate CPR
(less than 50/minute) 
Attempt cardioversion
Refer to specialist medical care immediately
Make the patient to lie flat
Give Flumazenil
Ventilate mechanically on the psychiatric ward
Give Oxygen
Check creatinine kinase urgently
Ensure patient is not lying face­down
Benzodiazepine Give procyclidine 5­10 mg IM
induced respiratory Initiate CPR
depression  Attempt cardioversion
Refer to specialist medical care immediately
Make the patient to lie flat
Give Flumazenil
Ventilate mechanically on the psychiatric ward
Check
Check creatinine kinase urgently

Explanation: 
Increased temperature following Rapid Tranquillization (RT) must prompt a blood test for
creatinine kinase urgently, due to a high risk of developing the neuroleptic malignant
syndrome.
Fall in blood pressure during RT must be considered significant if more than 30 mm Hg
orthostatic drop occurs or if the diastolic BP is less than 50 mm Hg. In this case, make the
patient lie flat, tilt bed towards head and monitor closely.
Acute dystonia including oculogyric crisis can be managed by giving procyclidine 5­10mg IM
or benztropine 1­2 mg IM. 
Irregular or slow pulse (less than 50/minute) is a sinister sign; it necessitates a referral to
specialist medical care immediately. 
Reduced respiratory rate of less than 10/min or oxygen saturation less than 90% must be
managed by giving oxygen, raising the legs, ensuring patient is not lying face down, and
giving flumazenil if benzodiazepine induced respiratory depression suspected. If any other

1515
sedative agent induces such a respiratory arrest, transfer to a medical bed immediately and
ventilate mechanically.
The correct answer is: Increased temperature (Choose ONE) – Check creatinine kinase
urgently, Fall in blood pressure by more than 30 mm Hg 
– Make the patient to lie flat, Acute dystonia 
– Give procyclidine 5­10 mg IM, Irregular or slow pulse (less than 50/minute) 
– Refer to specialist medical care immediately, Benzodiazepine induced respiratory
depression 
– Give Flumazenil

Question 19 HiY Pharmacology EMI019
Correct Drug Treatments
Select the drug that could be used, for the treatment of each of the following situations;
Mark 7.00 out of
7.00

Flag question
Dantrolene
Venlafaxine
Benzodiazepines
Catatonia Selegiline
Agomelatine
Ropinirole
Demeclocycline
Thiamine
Clomipramine
Dantrolene
Venlafaxine
Benzodiazepines
Neuroleptic malignant
Selegiline
syndrome
Agomelatine
Ropinirole
Demeclocycline
Thiamine
Clomipramine
Dantrolene
Venlafaxine
Benzodiazepines
SIADH induced by
Selegiline
antipsychotics
Agomelatine
Ropinirole
Demeclocycline
Thiamine
Clomipramine
Dantrolene
Venlafaxine
Recurrent Benzodiazepines
antidepressant­induced Selegiline
hyponatraemia  Agomelatine
Ropinirole
Demeclocycline
Thiamine
Clomipramine

1516
Wernicke's Dantrolene
encephalopathy
Venlafaxine
Benzodiazepines
Selegiline
Agomelatine
Ropinirole
Demeclocycline
Thiamine
Clomipramine
Check

Explanation: 
Catatonia is characterized by movement abnormalities usually associated with schizophrenia
and mood disorders. Several reports indicate that benzodiazepines have a rapid action, are
highly effective and safe and thus regarded as the first line treatment for catatonia. 
Neuroleptic malignant syndrome: Symptomatic management for vital signs instability, fluid
replacement and prevention of renal failure are the main treatment methods after immediate
stopping of the causative psychotropic agent. Dantrolene, bromocriptine or amantadine can
be used in the management. Dantrolene is a peripheral muscle relaxant, which inhibits the
intracellular calcium release from the sarcoplasmic reticulum. It was originally applied to treat
cases of malignant hyperthermia. 
For antipsychotic­induced SIADH, if mild, fluid restriction with careful monitoring of serum
sodium is suggested. Antidepressant­induced hyponatremia can be managed by water
restriction and prescribing non­SSRI agents, especially noradrenergic drugs such as
reboxetine, lofepramine or an MAOI such as Moclobemide. Demeclocycline may be useful in
recurrent cases of both SIADH and hyponatremia. 
Wernicke's encephalopathy: Parenteral thiamine should be given as a treatment.
The correct answer is: Catatonia – Benzodiazepines, Neuroleptic malignant syndrome –
Dantrolene, SIADH induced by antipsychotics – Demeclocycline, Recurrent antidepressant­
induced hyponatraemia 
– Demeclocycline, Wernicke's encephalopathy – Thiamine

Question 20 HiY Pharmacology EMI020
Partially correct Receptor Pharmacology
Which of the listed terms best describes each of the following receptors that are relevant for
Mark 4.00 out of
pharmacological actions of various psychotropics?
5.00

Flag question
Seven transmembrane receptor
Allosteric binding site
Heteroreceptor
Dopamine D2 receptor Oligomer
Extrasynaptic receptor
Heterodimer
Inhibitory auto­receptor
Voltage­gated ion channel
Nuclear receptor

1517
The glucocorticoid
Seven transmembrane receptor
receptor
Allosteric binding site
Heteroreceptor
Oligomer
Extrasynaptic receptor
Heterodimer
Inhibitory auto­receptor
Voltage­gated ion channel
Nuclear receptor
Seven transmembrane receptor
Allosteric binding site
Depolarization triggers Heteroreceptor
activation in this Oligomer
receptor Extrasynaptic receptor
Heterodimer
Inhibitory auto­receptor
Voltage­gated ion channel
Nuclear receptor
Seven transmembrane receptor
Allosteric binding site
Heteroreceptor
GABA­A receptor
Oligomer
complex
Extrasynaptic receptor
Heterodimer
Inhibitory auto­receptor
Voltage­gated ion channel
Nuclear receptor
Seven transmembrane receptor
Allosteric binding site
A receptor that
Heteroreceptor
regulates the synthesis
Oligomer
of mediators other than
its own ligand
Extrasynaptic receptor
Heterodimer
Inhibitory auto­receptor
Voltage­gated ion channel
Nuclear receptor
Check

Explanation: 
The dopamine D2 receptor belongs to the family of seven transmembrane domain G­protein­
coupled receptors and is highly expressed in the central nervous system and the pituitary
gland. 
The glucocorticoid receptor (GR, or GCR) also known as NR3C1 (nuclear receptor subfamily
3, group C, member 1) is the receptor that cortisol and other glucocorticoids bind to. Its
primary mechanism of action is the regulation of gene transcription. 
Voltage­gated ion channels are transmembrane ion channels that are opened or closed by
electrical activity in their proximity. 
When certain modulating agents bind to a regulatory site on a receptor complex that is
different from the site where the ligand binds to, this can bring about an allosteric modulation
that enhances or inhibits the effects of the endogenous ligand. GABA­A has two active sites

1518
for endogenous GABA binding, but also has regulatory binding sites for benzodiazepines and
general anaesthetic agents. These allosteric sites result in positive allosteric modulation,
potentiating GABA transmission. 
A heteroreceptor is a receptor that regulates the synthesis and release of mediators other than
its own ligand. An often cited example is that of epinephrine, which influences the release of
acetylcholine from parasympathetic nerve endings by acting on alpha­2A, alpha­2B, and
alpha­2C receptors. Another example is acetylcholine that influences the release of
norepinephrine by acting on muscarinic­2 and muscarinic­4 receptors on sympathetic
neurons. An example that is more relevant to psychiatric practice is the presence of alpha 2­
adrenergic heteroreceptors on serotonergic nerve cell terminals in the hippocampus.
Mirtazapine acts as an antagonist at these presynaptic alpha 2­adrenergic heteroreceptors
leading to a facilitation of serotonergic transmission.
The correct answer is: Dopamine D2 receptor – Seven transmembrane receptor, The
glucocorticoid receptor – Nuclear receptor, Depolarization triggers activation in this receptor –
Voltage­gated ion channel, GABA­A receptor complex – Allosteric binding site, A receptor that
regulates the synthesis of mediators other than its own ligand – Heteroreceptor

Question 21 HiY Pharmacology EMI021
Correct Newer antidepressants
Select mechanism of action for the following drugs
Mark 5.00 out of
5.00

Flag question
NMDA receptor antagonism
Alpha­2 adrenoreceptor antagonism
5­HT2C receptor antagonism
Mirtazapine   
MAOB­B inhibitor
MAOI­A inhibitor
GABA­A Agonist
5­HT2A receptor stimulation
Norepinephrine and serotonin reuptake reduction
NMDA receptor antagonism
Presynaptic alpha­2 agonist
Alpha­2 adrenoreceptor antagonism
D2 receptor partial agonist
5­HT2C receptor antagonism
Agomelatine  
MAOB­B inhibitor
MAOI­A inhibitor
GABA­A Agonist
5­HT2A receptor stimulation
Norepinephrine and serotonin reuptake reduction
NMDA receptor antagonism
Presynaptic alpha­2 agonist
Alpha­2 adrenoreceptor antagonism
D2 receptor partial agonist
5­HT2C receptor antagonism
MAOB­B inhibitor  
MAOI­A inhibitor
Duloxetine GABA­A Agonist
5­HT2A receptor stimulation
Norepinephrine and serotonin reuptake reduction
Presynaptic alpha­2 agonist
D2 receptor partial agonist

1519
Selegiline NMDA receptor antagonism
Alpha­2 adrenoreceptor antagonism
5­HT2C receptor antagonism
MAOB­B inhibitor  
MAOI­A inhibitor
GABA­A Agonist
5­HT2A receptor stimulation
Norepinephrine and serotonin reuptake reduction
Check Presynaptic alpha­2 agonist
D2 receptor partial agonist

Explanation: 
Mirtazapine acts via 5HT2A antagonism and alpha 2 antagonism in addition to displaying
antihistaminic and anti 5HT3 properties. Mianserin has similar profile except for antihistaminic
properties, but it has more anticholinergic properties than mirtazapine. 
Agomelatine enhances norepinephrine and dopamine neurotransmission through 5­HT2C
antagonism. It is also a direct agonist at melatonin (MT1 and MT2) receptors. GABA
interneurons tonically inhibit noradrenergic circuits (from locus coeruleus) and dopaminergic
circuits (from ventral tegmentum) projecting to prefrontal cortex. Serotonin via 5HT2C
stimulation drives these GABA interneurons. Thus, norepinephrine and dopamine circuits are
inhibited by the normal tonic release of serotonin onto 5­HT2C receptors (Stahl, 2007). Thus
agomelatine, through 5HT2C inhibition, acts as norepinephrine and dopamine disinhibitor
(NDDI). 
Duloxetine is an SNRI similar to venlafaxine. It has a favourable effect on psychosomatic pain
and neuropathic pain. 
Selegiline is a Monoamine Oxidase Inhibitor that is selective for MAO­B at normal therapeutic
doses; this selectivity is lost when a transdermal selegiline patch is applied at higher doses,
leading to notable antidepressant action.
The correct answer is: Mirtazapine 
– Alpha­2 adrenoreceptor antagonism, Agomelatine – 5­HT2C receptor antagonism,
Duloxetine – Norepinephrine and serotonin reuptake reduction, Selegiline – MAOB­B
inhibitor

Question 22 HiY Pharmacology EMI022
Correct Effect of receptor­level actions (1)
Match the receptors below with its most significant effect from the list shown here.
Mark 5.00 out of
5.00

Flag question
Sedation and weight gain
Sedation and orthostatic hypotension
Blockade of D­2 Gastrointestinal side effects of SSRIs
receptors in the Dry mouth and constipation
mesolimbic pathway Insomnia and sexual side effects of SSRIs
Therapeutic effects of antidepressants
Therapeutic effects of antipsychotics
Leads to tardive dyskinesia
Leads to features of hyperprolactinaemia
Leads to extrapyramidal side effects
Production of negative symptoms

1520
Blockade of D­2 Sedation and weight gain
receptors in the
Sedation and orthostatic hypotension
mesocortical pathway
Gastrointestinal side effects of SSRIs
Dry mouth and constipation
Insomnia and sexual side effects of SSRIs
Therapeutic effects of antidepressants
Therapeutic effects of antipsychotics
Leads to tardive dyskinesia
Leads to features of hyperprolactinaemia
Sedation and weight gain
Leads to extrapyramidal side effects
Sedation and orthostatic hypotension
Blockade of D­2 Production of negative symptoms
Gastrointestinal side effects of SSRIs
receptors in the Dry mouth and constipation
nigrostriatal pathway Insomnia and sexual side effects of SSRIs
Therapeutic effects of antidepressants
Therapeutic effects of antipsychotics
Leads to tardive dyskinesia
Leads to features of hyperprolactinaemia
Sedation and weight gain
Leads to extrapyramidal side effects
Sedation and orthostatic hypotension
Blockade of D­2 Production of negative symptoms
Gastrointestinal side effects of SSRIs
receptors in the
Dry mouth and constipation
tuberoinfundibular
pathway
Insomnia and sexual side effects of SSRIs
Therapeutic effects of antidepressants
Therapeutic effects of antipsychotics
Leads to tardive dyskinesia
Leads to features of hyperprolactinaemia
Sedation and weight gain
Leads to extrapyramidal side effects
Sedation and orthostatic hypotension
Production of negative symptoms
Gastrointestinal side effects of SSRIs
Dopamine receptor
Dry mouth and constipation
supersensitivity
Insomnia and sexual side effects of SSRIs
Therapeutic effects of antidepressants
Therapeutic effects of antipsychotics
Leads to tardive dyskinesia
Leads to features of hyperprolactinaemia
Check
Leads to extrapyramidal side effects
Production of negative symptoms

Explanation: Blockade of D­2 receptors in the mesolimbic pathway is responsible for
therapeutic effects of antipsychotics. The blockade of D­2 receptors in the mesocortical
pathway leads to the production of negative symptoms. The blockade of D­2 receptors in the
nigrostriatal pathway causes EPSEs. It is thought that higher than 80% receptor occupancy of
brain D2 by antipsychotics can cause EPSEs. The blockade of D­2 receptors in the
tuberoinfundibular pathway produces symptoms of hyperprolactinaemia. PET studies have
indicated that 60%­80% occupation of D2 receptors is associated with antipsychotic efficacy.
Higher occupancy levels are associated with an increased risk of acute extrapyramidal
symptoms as well as hyperprolactinemia from the blocking of D2 receptors on anterior
pituitary mammographic cells (these prolactin­producing cells are normally inhibited by
dopamine produced in the hypothalamic arcuate nucleus ­ hence dopamine is sometimes
referred to as Prolactin Inhibiting Substance (PIS)). The exact mechanism of tardive
dyskinesia is poorly understood; it is suspected to be related to dopamine receptor
supersensitivity.
The correct answer is: Blockade of D­2 receptors in the mesolimbic pathway – Therapeutic

1521
effects of antipsychotics, Blockade of D­2 receptors in the mesocortical pathway – Production
of negative symptoms, Blockade of D­2 receptors in the nigrostriatal pathway – Leads to
extrapyramidal side effects, Blockade of D­2 receptors in the tuberoinfundibular pathway –
Leads to features of hyperprolactinaemia, Dopamine receptor supersensitivity – Leads to
tardive dyskinesia

Question 23 HiY Pharmacology EMI023
Partially correct Effect of receptor­level actions (2)
Match the receptor below with its most significant effect from the list above
Mark 3.00 out of
5.00

Flag question
Dry mouth
constipation
Blockade of Neuroleptic malignant syndrome
postsynaptic alpha­1 Production of negative symptoms
adrenoreceptor Myocarditis
Therapeutic effects of Antidepressants
Leads to tardive dyskinesia
Tolerance to hallucinogenic drugs
Insomnia and sexual side effects of SSRIs
Dry mouth
Sedation and weight gain
constipation
QT prolongation
Neuroleptic malignant syndrome
Blockade of M3 Sedation and orthostatic hypotension
Production of negative symptoms
cholinergic receptors
Myocarditis
Therapeutic effects of Antidepressants
Leads to tardive dyskinesia
Tolerance to hallucinogenic drugs
Insomnia and sexual side effects of SSRIs
Dry mouth
Sedation and weight gain
constipation
QT prolongation
Neuroleptic malignant syndrome
Blockade of H1 Sedation and orthostatic hypotension
Production of negative symptoms
histamine receptors
Myocarditis
Therapeutic effects of Antidepressants
Leads to tardive dyskinesia
Tolerance to hallucinogenic drugs
Insomnia and sexual side effects of SSRIs
Dry mouth
Sedation and weight gain
constipation
QT prolongation
Neuroleptic malignant syndrome
Sedation and orthostatic hypotension
Production of negative symptoms
Myocarditis
Downregulation of 5­
HT2A receptors.
Therapeutic effects of Antidepressants
Leads to tardive dyskinesia
Tolerance to hallucinogenic drugs
Insomnia and sexual side effects of SSRIs
Sedation and weight gain
QT prolongation
Sedation and orthostatic hypotension

1522
D­2 receptor blockade
Dry mouth
constipation
Neuroleptic malignant syndrome
Production of negative symptoms
Myocarditis
Therapeutic effects of Antidepressants
Leads to tardive dyskinesia
Tolerance to hallucinogenic drugs
Insomnia and sexual side effects of SSRIs
Check
Sedation and weight gain
QT prolongation
Sedation and orthostatic hypotension
Explanation: 
Psychotropic medications such as antipsychotics & antidepressants often cause postural
hypotension, which is due to alpha­1 receptor blockade, while sedation & weight gain are due
to histamine H1 receptor blockade. 
Antipsychotics and antidepressants can cause anticholinergic side effects such as dry mouth,
constipation, urinary retention, blurred vision and worsening of confusion due to blockade of
M3 cholinergic receptors. 
Downregulation of 5­HT2A receptors can cause tolerance to hallucinogenic drugs such as
LSD. 
D­2 receptor blockade caused by antipsychotics leads to the development of the neuroleptic
malignant syndrome. NMS is more common in young men, after agitation and when using
high potency drugs especially after rapid tranquillization. Dopaminergic drugs on withdrawal
can produce NMS. Mechanism may be related to dopamine blockade or hypothalamic
sympathetic dysregulation.
The correct answer is: Blockade of postsynaptic alpha­1 adrenoreceptor – Sedation and
orthostatic hypotension, Blockade of M3 cholinergic receptors – Dry mouth, constipation,
Blockade of H1 histamine receptors – Sedation and weight gain, Downregulation of 5­HT2A
receptors. – Tolerance to hallucinogenic drugs, D­2 receptor blockade – Neuroleptic
malignant syndrome

Question 24 HiY Pharmacology EMI024
Correct Side effects and receptors
Choose one explanatory option for each of the following conditions:
Mark 3.00 out of
3.00

Flag question
Dopamine D4 activation
Apolipoprotein E receptors
Serotonin 5HT2 receptor blockade
Clozapine induced
Decreased nicotinic receptor density
hypersalivation
Serotonin 5HT3 blockade
High 5HT2 Serotonin receptor sensitivity
Low D1 receptor sensitivity
M4 agonism
Low D2 receptor occupancy in striatum
Serotonin 5HT3 activation

SSRI­induced nausea

1523
Dopamine D4 activation
Apolipoprotein E receptors
Serotonin 5HT2 receptor blockade
Decreased nicotinic receptor density
Serotonin 5HT3 blockade
High 5HT2 Serotonin receptor sensitivity
Low D1 receptor sensitivity
M4 agonism
Low D2 receptor occupancy in striatum
Dopamine D4 activation
Serotonin 5HT3 activation
Apolipoprotein E receptors
Lower incidence of Serotonin 5HT2 receptor blockade
EPSEs of newer Decreased nicotinic receptor density
atypical antipsychotics Serotonin 5HT3 blockade
High 5HT2 Serotonin receptor sensitivity
Low D1 receptor sensitivity
M4 agonism
Low D2 receptor occupancy in striatum
Check
Serotonin 5HT3 activation

Explanation: 
The pharmacological basis of clozapine related hypersalivation remains unclear. Suggested
mechanisms include M4 agonism, adrenergic alpha 2 antagonism and inhibition of the
swallowing reflex. 
The consequences of stimulating 5HT3 receptors by SSRIs causes gastrointestinal side
effects like nausea, vomiting, anorexia, and diarrhoea. 
Lower incidence of EPSEs of newer atypical antipsychotics is due to 5­HT2 receptor
blockade. PET and SPECT studies have shown that newer atypicals like olanzapine,
risperidone, quetiapine and clozapine have a higher degree of 5­HT2A receptor occupancy
(more than 90%) that leads to a reduced incidence of EPSEs.
The correct answer is: Clozapine induced hypersalivation – M4 agonism, SSRI­induced
nausea – Serotonin 5HT3 activation, Lower incidence of EPSEs of newer atypical
antipsychotics – Serotonin 5HT2 receptor blockade

Question 25 HiY Pharmacology EMI025
Correct Mechanism of action
Identify the correct drug based on their mechanism of action
Mark 5.00 out of
5.00

Flag question
Topiramate
Sodium Valproate
Enhances GABAergic Gabapentin
function and inhibits
Lithium
GABA
aminotransferase
Vigabatrin
Pregabalin
Carbamazepine
Lamotrigine
Levetiracetam

1524
Effects on intracellular
Topiramate
second messenger
Sodium Valproate
systems
Gabapentin
Lithium
Vigabatrin
Pregabalin
Carbamazepine
Lamotrigine
Levetiracetam
Topiramate
Sodium Valproate
Neuronal membrane Gabapentin
stabiliser and blocks
Lithium
voltage­dependent
sodium channels
Vigabatrin
Pregabalin
Carbamazepine
Lamotrigine
Levetiracetam
Topiramate
Sodium Valproate
Decrease GABA Gabapentin
metabolism by
Lithium
inhibition of glial GABA
transaminase
Vigabatrin
Pregabalin
Carbamazepine
Lamotrigine
Levetiracetam
Topiramate
Sodium Valproate
Enhances GABA and Gabapentin
blocks glutamate at Lithium
NMDA receptors Vigabatrin
Pregabalin
Carbamazepine
Lamotrigine
Levetiracetam
Check

Explanation: 
Sodium valproate enhances GABAergic function, inhibits GABA aminotransferase, and
increases GABA binding in some brain structures like the hippocampus. 
Lithium is thought to act via intracellular second messenger systems. It inhibits Na induced c­
AMP activity, limits inositol triphosphate formation and reduces the activity of protein kinases. 
Lamotrigine has a novel mechanism of action stabilising neuronal membranes, reduce the
release of excitatory amino acids like glutamate by blocking voltage dependent sodium
channels. 
Vigabatrin is an antiepileptic drug that inhibits the catabolism of GABA by irreversibly
inhibiting GABA transaminase (Vi GABA TRansaminaseINhibitor). 
Topiramate is used as an adjunctive therapy for partial seizures. It enhances GABA and
blocks glutamate at NMDA receptors
The correct answer is: Enhances GABAergic function and inhibits GABA aminotransferase –
Sodium Valproate, Effects on intracellular second messenger systems – Lithium, Neuronal
membrane stabiliser and blocks voltage­dependent sodium channels – Lamotrigine,

1525
Decrease GABA metabolism by inhibition of glial GABA transaminase – Vigabatrin, Enhances
GABA and blocks glutamate at NMDA receptors – Topiramate

Question 26 HiY Pharmacology EMI026
Partially correct Management of mood disorder
Choose the best treatment option from the shown list for each of the following clinical
Mark 2.00 out of
situations
4.00

Flag question
Venlafaxine
Rob is a 40­year­old Quetiapine
man with bipolar Sodium valproate
disorder currently
Mirtazapine
depressed. He has
been on lithium for five
Haloperidol
years. Lamotrigine
Citalopram
Lithium
Lucy is a 25­year­old Carbamazepine
Venlafaxine
woman who has a Quetiapine
history of bipolar Sodium valproate
disorder. She is now
Mirtazapine
presenting with a mixed
Haloperidol
affective state. She is
also pregnant for the Lamotrigine
first time. Citalopram
Lithium
Carbamazepine
Venlafaxine
Andy suffers from Quetiapine
recurrent depression Sodium valproate
treated by his GP with
Mirtazapine
venlafaxine. He has not
made much
Haloperidol
improvement. Lamotrigine
Citalopram
Lithium
Linda has a history of Carbamazepine
recurrent depressive
Venlafaxine
episodes followed by
hypomania often due to
Quetiapine
her antidepressant Sodium valproate
drugs. She recovers Mirtazapine
from the most recent Haloperidol
episode of depression. Lamotrigine
Her GP is keen to start Citalopram
her on prophylactic Lithium
treatment
Carbamazepine

Check

Explanation: 
SSRIs should be tried as the first line treatment option for depressive episodes in bipolar
disorder. 

1526
Mixed affective states should be treated as a manic episode. Although the patient in the
question is pregnant, she will need treatment for bipolar disorder. A low dose of older first­
generation antipsychotics such as haloperidol could be used and have minimal risk of
teratogenicity. There is little evidence that is available for the use of other antipsychotics in a
pregnant woman with bipolar disorder. 
In the management of recurrent depression showing partial improvement, lithium should be
tried as the first line drug for an augmentation strategy. 
Antidepressant­induced mania/hypomania has been reported in all major antidepressant
classes in 20­40% of bipolar patients (Goldberg & Truman, 2003). Lithium is supposed to
confer better protection against switching when compared with other mood stabilizers,
although comparable switch rates have been reported on or off mood stabilizers. Mood
switches were less frequent in patients receiving lithium (15%) than in patients not treated
with lithium (44%) in a small naturalistic study. Lithium is probably effective against both
manic and depressive relapse, although it is more effective in preventing mania. Lithium is
also associated with a reduced risk of suicide in bipolar disorder. 

Ref: Goldberg, JF., Truman, CJ. (2003). Antidepressant­induced mania: an overview of current
controversies. Bipolar Disorders Dec;5(6):407­20. Retrieved from
http://www.ncbi.nlm.nih.gov/pubmed/14636364
The correct answer is: Rob is a 40­year­old man with bipolar disorder currently depressed. He
has been on lithium for five years. – Citalopram, Lucy is a 25­year­old woman who has a
history of bipolar disorder. She is now presenting with a mixed affective state. She is also
pregnant for the first time. – Haloperidol, Andy suffers from recurrent depression treated by his
GP with venlafaxine. He has not made much improvement. – Lithium, Linda has a history of
recurrent depressive episodes followed by hypomania often due to her antidepressant drugs.
She recovers from the most recent episode of depression. Her GP is keen to start her on
prophylactic treatment – Lithium

Question 27 HiY Pharmacology EMI027
Partially correct Pharmacological properties
Identify the correct drug for each of the following descriptions;
Mark 4.00 out of
6.00

Flag question
Gabapentin
Carbamazepine
An antidepressant with Venlafaxine
high anticholinergic Clomipramine
effect Duloxetine
Sodium valproate
Reboxetine

Gabapentin
Carbamazepine
Venlafaxine
Excreted predominantly Clomipramine
via renal route
Duloxetine
Sodium valproate
Reboxetine

1527
A drug with
Gabapentin
pronounced
Carbamazepine
antihistaminic effect
Venlafaxine
Clomipramine
Duloxetine
Sodium valproate
Reboxetine

Check

Explanation: 
Tricyclic antidepressants like Amitriptyline and Clomipramine could cause sedation (H1
receptor blockade), orthostatic hypotension (alpha­1 receptor blockade) and anticholinergic
side effects. 
Gabapentin is removed from the systemic circulation by renal excretion in an unchanged form.
Lithium is excreted almost entirely by the kidneys. Lithium is freely filtered by the glomerulus
as it is free from protein binding in serum.
The correct answer is: An antidepressant with high anticholinergic effect – Clomipramine,
Excreted predominantly via renal route – Gabapentin, A drug with pronounced antihistaminic
effect – Clomipramine

Question 28 HiY Pharmacology EMI028
Correct History of psychiatric treatments
Match each of the following biological treatments with the pioneers associated from the given
Mark 5.00 out of
list
5.00

Flag question
Kuhn
Janssen
Egas Moniz and Almeida Lima
Electroconvulsive
John Cade
therapy
Hugo Sternbach
F M Berger
Manfred Sakel
Ganesh Sen and Kartik Bose
Ugo Cereletti and Lucio Bini
Kuhn
Delay and Deniker
Janssen
Egas Moniz and Almeida Lima
Lithium John Cade
Hugo Sternbach
F M Berger
Manfred Sakel
Ganesh Sen and Kartik Bose
Ugo Cereletti and Lucio Bini
Delay and Deniker

Chlorpromazine

1528
Kuhn
Janssen
Egas Moniz and Almeida Lima
John Cade
Hugo Sternbach
F M Berger
Manfred Sakel
Ganesh Sen and Kartik Bose
Ugo Cereletti and Lucio Bini
Kuhn
Delay and Deniker
Janssen
Egas Moniz and Almeida Lima
Haloperidol John Cade
Hugo Sternbach
F M Berger
Manfred Sakel
Ganesh Sen and Kartik Bose
Ugo Cereletti and Lucio Bini
Kuhn
Delay and Deniker
Janssen
Egas Moniz and Almeida Lima
Chlordiazepoxide John Cade
Hugo Sternbach
F M Berger
Manfred Sakel
Ganesh Sen and Kartik Bose
Ugo Cereletti and Lucio Bini
Check
Delay and Deniker

Explanation: 
According to case notes recovered from 1934, Laszlo Meduna, a 38­year­old Hungarian
psychiatrist administered IM injections of 10ml of 20% camphor to five in Budapest to treat
schizophrenia by inducing epileptic seizures. 
The prototype electroconvulsive machine was first designed by an Italian psychiatrist Ugo
Cerletti and his student Lucio Bini in 1938. 
A historical misconception regarding the connection between mania and urea formed the
grounds for a series of experiments designed by Cade in which he injected uric acid (as
lithium urate) into guinea pigs. Rather surprisingly, he observed a calming effect instead of
increased excitation. This observation later paved the way for developing lithium as an
antimanic compound. 
Chlordiazepoxide was the first benzodiazepine to be discovered, albeit serendipitously in
1954 by the Austrian scientist Leo Sternbach while he was working for the pharmaceutical
company Hoffmann­La Roche. 
Chlorpromazine (CPZ) was synthesized in1951 by Paul Charpentier and released for clinical
investigation in 1952 as a possible potentiator of general anesthesia. Clinical investigations
with CPZ at Saint­Anne's hospital ­ at Pierre Deniker's service in Jean Delay's department ­ in
Paris began in 1952. The six publications of Delay and Deniker during the six months that
followed set the stage for the introduction of CPZ in psychiatry (Olie 2004). 

Ref: Ban, TA.(2007). Fifty years chlorpromazine: a historical perspective Neuropsychiatric
Disease and Treatment 2007:3(4) 495­500. Retrieved from

1529
http://europepmc.org/articles/PMC2655089/reload=0
The correct answer is: Electroconvulsive therapy – Ugo Cereletti and Lucio Bini, Lithium –
John Cade, Chlorpromazine – Delay and Deniker, Haloperidol – Janssen, Chlordiazepoxide
– Hugo Sternbach

Question 29 HiY Pharmacology EMI029
Correct Side effects of medication
Lead In:Match the side effects with the drugs most commonly associated with it
Mark 5.00 out of
5.00

Flag question
Citalopram
Carbamazepine
Sodium valproate
Thrombocytopenia  
Reboxetine
Clozapine
Thioridazine
Risperidone
Trazodone
Citalopram
Lithium
Carbamazepine
Chlorpromazine
Sodium valproate
Gastric bleeding  
Reboxetine
Clozapine
Thioridazine
Risperidone
Trazodone
Citalopram
Lithium
Carbamazepine
Chlorpromazine
Sodium valproate
Hypothyroidism  
Reboxetine
Clozapine
Thioridazine
Risperidone
Trazodone
Citalopram
Lithium
Carbamazepine
Chlorpromazine
Sodium valproate
Reboxetine  
Clozapine
Cardiomyopathy Thioridazine
Risperidone
Trazodone
Lithium
Chlorpromazine

Citalopram
Carbamazepine
Sodium valproate
Urinary hesitancy  
Reboxetine
Clozapine
Thioridazine
Risperidone
Trazodone

1530
Check

Explanation: 
The use of Sodium Valproate is associated with thrombocytopenia, leucopenia, red cell
hypoplasia and pancreatitis. Thrombocytopenia is a dose­related effect of valproate; a
decrease in dose is required if bruising or bleeding gums is noted. 
Several database studies have found that patients who take SSRIs are at significantly
increased risk of being hospitalized with an upper gastrointestinal bleeding compared with
age and sex­matched controls (Maudsley Prescribing Guidelines). The risk is greatest with
SSRIs with high affinity for the serotonin transporter. 
Lithium can cause a variety of thyroid problems ­ the most common being a benign
hypothyroid state. 5% patients may develop goiter and overt hyperthyroidism is also reported
in some cases. Thyroid deficiency is common in those with high risk for preexisting antithyroid
antibodies such as middle­aged women. The risk is 3­4:1 in women and is high in first two
years of treatment. Rapid cycling patients are at higher risk. High TSH is seen in nearly 1/3rd
of chronic lithium­treated patients ­ without clinical hypothyroidism. In resistant depression or
non­responsive cyclers with bipolar it may be useful treating this subclinical state with
thyroxine. 
It has been suggested that clozapine is associated with myocarditis and cardiomyopathy.
Myocarditis seems to occur within 6­8 weeks of starting clozapine and cardiomyopathy may
occur later in treatment (median­9 months) but both may occur at any time.
Urinary hesitancy has been observed in around 10% of male patients taking part in the
clinical trials of reboxetine. Relief from this side effect could be achieved by using peripheral
alpha1­receptor blocker tamsulosin or doxazosin.
The correct answer is: Thrombocytopenia – Sodium valproate, Gastric bleeding – Citalopram,
Hypothyroidism – Lithium, Cardiomyopathy – Clozapine, Urinary hesitancy – Reboxetine

Question 30 HiY Pharmacology EMI030
Partially correct Classification of psychotropic drugs 
Match each chemical class below with the most appropriate drug from the list.
Mark 4.00 out of
5.00

Flag question
Phenelzine
Tranylcypromine
Zolpidem
Pyrazolopyrimidine Diazepam
Isocarboxazid
Clonazepam
Moclobemide
Zopiclone
Phentolamine
Phenelzine
Zaleplon
Tranylcypromine
Zolpidem
Diazepam
Isocarboxazid
Non hydrazine MAOI Clonazepam
Moclobemide
Zopiclone
Phentolamine

1531
Phenelzine
Tranylcypromine
Zolpidem
Reversible MAOI Diazepam
Isocarboxazid
Clonazepam
Moclobemide
Zopiclone
Phentolamine
Phenelzine
Zaleplon
Tranylcypromine
Zolpidem
Full agonist at the BDZ
Diazepam
receptor
Isocarboxazid
Clonazepam
Moclobemide
Zopiclone
Phentolamine
Phenelzine
Zaleplon
Tranylcypromine
Zolpidem
Partial agonist at the
Diazepam
BDZ receptor
Isocarboxazid
Clonazepam
Moclobemide
Zopiclone
Phentolamine
Check
Zaleplon

Explanation: Cyclopyrrolone ­ Zopiclone, Imidazopyridine ­Zolpidem. Pyrazolopyrimidine­
Zaleplon. 
Monoamine oxidase inhibitors include both hydrazine and non­hydrazine derivatives.
Hydrazine derivatives include phenelzine and isocarboxazid (greater hepatotoxicity than
tranylcypromine, a non hydrazine compound). Moclobemide is a RIMA ( a reversible inhibitor
of monoamine oxidase A). 
Among benzodiazepines, there are full agonists like diazepam and partial agonists like
clonazepam, which act at the BDZ receptor to enhance the action of GABA.
The correct answer is: Pyrazolopyrimidine – Zaleplon, Non hydrazine MAOI –
Tranylcypromine, Reversible MAOI – Moclobemide, Full agonist at the BDZ receptor –
Diazepam, Partial agonist at the BDZ receptor – Clonazepam

Question 31 HiY Pharmacology EMI031
Partially correct Antidepressant side effects
Choose the most likely offending drug for each of the following situations.
Mark 3.00 out of
5.00

1532
Flag question
Fluoxetine
A 61­year­old lady
Amitriptyline
develops atropine
Mirtazapine
psychosis when taking
this medication for
Lofepramine
depression. Imipramine
Clomipramine
Phenelzine
Dosulepin
Nomifensine
Fluoxetine
A 44­year­old man Moclobemide
Amitriptyline
develops septicaemia, Methylphenidate
Mirtazapine
and an FBC shows Venlafaxine
Lofepramine
agranulocytosis
following anti­
Imipramine
depressant treatment. Clomipramine
Phenelzine
Dosulepin
Nomifensine
Fluoxetine
An 11­year­old boy
Moclobemide
Amitriptyline
develops sudden Methylphenidate
Mirtazapine
involuntary jerky Venlafaxine
Lofepramine
movements in her face Imipramine
with vocal grunts. Clomipramine
Phenelzine
Dosulepin
A 64­year­old woman is Nomifensine
prescribed amitriptyline Moclobemide
for many years by her
Fluoxetine
Methylphenidate
GP. She has a relapse
Amitriptyline
Venlafaxine
in depression, and the
GP changes her
Mirtazapine
prescription to another Lofepramine
tricyclic agent. But she Imipramine
takes three weeks Clomipramine
supply of this Phenelzine
antidepressant as an Dosulepin
overdose. She Nomifensine
develops arrhythmias
Moclobemide
and despite intensive
Methylphenidate
cardiac monitoring,
dies three days later.
Venlafaxine

A 62­year­old man is Fluoxetine
on treatment for Amitriptyline
intractable OCD. He Mirtazapine
develops myoclonus,
Lofepramine
nausea, diarrhoea and
disinhibition when
Imipramine
lithium is added to his Clomipramine
antidepressant. Phenelzine
Dosulepin
Nomifensine
Check
Moclobemide
Methylphenidate
Venlafaxine

1533
Explanation: Amitriptyline has a very high anticholinergic activity. Mirtazapine can cause
agranulocytosis. Methylphenidate can induce tics in some children who are treated for ADHD.
Dosulepin has higher cardiotoxic potential than most other TCAs. Clomipramine can cause
serotonin syndrome when combined with lithium.
The correct answer is: A 61­year­old lady develops atropine psychosis when taking this
medication for depression. – Amitriptyline, A 44­year­old man develops septicaemia, and an
FBC shows agranulocytosis following anti­depressant treatment. – Mirtazapine, An 11­year­
old boy develops sudden involuntary jerky movements in her face with vocal grunts. –
Methylphenidate, A 64­year­old woman is prescribed amitriptyline for many years by her GP.
She has a relapse in depression, and the GP changes her prescription to another tricyclic
agent. But she takes three weeks supply of this antidepressant as an overdose. She develops
arrhythmias and despite intensive cardiac monitoring, dies three days later. – Dosulepin, A
62­year­old man is on treatment for intractable OCD. He develops myoclonus, nausea,
diarrhoea and disinhibition when lithium is added to his antidepressant. – Clomipramine

Question 32 HiY Pharmacology EMI032
Correct Receptor mechanisms 
Identify the drugs & receptors that display the following features:
Mark 4.00 out of
4.00

Flag question
Atropine at muscarinic receptors
Propranalol at beta adrenergic receptors
Benzodiazepines at GABA A complex
Non­competitive
Buspirone
antagonist 
Aripiprazole
Phencyclidine at NMDA receptors
MAOIs
Buprenorphine
Atropine at muscarinic receptors
Propranalol at beta adrenergic receptors
Benzodiazepines at GABA A complex
Irreversible antagonists Buspirone
Aripiprazole
Phencyclidine at NMDA receptors
MAOIs
Buprenorphine
Atropine at muscarinic receptors
Propranalol at beta adrenergic receptors
Benzodiazepines at GABA A complex
Full agonists  Buspirone
Aripiprazole
Phencyclidine at NMDA receptors
MAOIs
Buprenorphine

Check

Explanation: 

1534
Full agonists produce a maximal response. Examples are benzodiazepines at GABA­A
complex, Bromocriptine for dopamine. 
When the dose of the agonist drug is increased sufficiently, this reverses competitive
antagonism almost completely. In other words, competitive antagonists only reduce the
potency (the minimal dose needed for an effect to become apparent) but not the efficacy (the
degree of response produced) of agonist drugs. Examples of competitive antagonism include
atropine at muscarinic receptors and propranolol at beta­adrenergic receptors. 
Noncompetitive antagonists modify the receptor configuration such that the effects can be
reversed only partially by increasing the dose of the agonist drug. Non­competitive
antagonism reduces both the potency and the efficacy of agonists. Therefore, non­competitive
antagonists not only shift the curve to the right but also reduce the maximum effect. For
example, ketamine and phencyclidine are noncompetitive NMDA antagonists. Irreversible
antagonists bind irreversibly to the target site e.g. most traditional MAOIs.
The correct answer is: Non­competitive antagonist 
– Phencyclidine at NMDA receptors, Irreversible antagonists
– MAOIs, Full agonists 
– Benzodiazepines at GABA A complex

Question 33 HiY Pharmacology EMI033
Partially correct Swapping antidepressants 
For the clinical situations given below, choose the best management option.
Mark 3.00 out of
6.00

Flag question A 45­year­old woman Withdraw and then start new drug straightaway


on paroxetine for four Discontinue over a month
weeks has now Co­administer for a brief period
decided to take It may cause withdrawal symptoms
citalopram, as she Withdraw and wait one week before starting new drug
responded well to this Withdraw and wait two weeks before starting new drug
medication in the past.  Withdraw and wait five weeks before starting new drug
Stop drug immediately
A 52­year­old woman Cross taper with caution
Withdraw and then start new drug straightaway
on Venlafaxine Discontinue over a month
developed insomnia & Co­administer for a brief period
dizzy spells. She is not
It may cause withdrawal symptoms
keen to continue taking
this antidepressant.
Withdraw and wait one week before starting new drug
She is now willing to Withdraw and wait two weeks before starting new drug
take mirtazapine.  Withdraw and wait five weeks before starting new drug
Stop drug immediately
Cross taper with caution
Withdraw and then start new drug straightaway
A 57­year­old woman Discontinue over a month
on fluoxetine has Co­administer for a brief period
atypical depression.
It may cause withdrawal symptoms
Now you have decided
to treat her on
Withdraw and wait one week before starting new drug
moclobemide. Withdraw and wait two weeks before starting new drug
Withdraw and wait five weeks before starting new drug
Stop drug immediately
Cross taper with caution
A 46­year­old man with
depression on
tranylcypromine has

1535
decided to switch over
Withdraw and then start new drug straightaway
to sertraline
Discontinue over a month
Co­administer for a brief period
It may cause withdrawal symptoms
Withdraw and wait one week before starting new drug
Withdraw and wait two weeks before starting new drug
Withdraw and wait five weeks before starting new drug
Stop drug immediately
Cross taper with caution
Check

Explanation: 
When swapping from one antidepressant to another, abrupt withdrawal should usually be
avoided. The dose of the currently used drug with poor efficacy or intolerance can be slowly
reduced, and a new drug can be slowly introduced (cross­tapering). 
When you switch from fluoxetine to moclobemide, withdraw and wait at least five weeks, as
fluoxetine has the longest half­life. 
When you switch from MAOI like tranylcypromine to SSRI, withdraw and wait for two weeks.
MAOIs should not be co­administered with SSRIs, as this combination could precipitate
serotonin syndrome.
The correct answer is: A 45­year­old woman on paroxetine for four weeks has now decided to
take citalopram, as she responded well to this medication in the past. 
– Cross taper with caution, A 52­year­old woman on Venlafaxine developed insomnia & dizzy
spells. She is not keen to continue taking this antidepressant. She is now willing to take
mirtazapine. 
– Cross taper with caution, A 57­year­old woman on fluoxetine has atypical depression. Now
you have decided to treat her on moclobemide.
– Withdraw and wait five weeks before starting new drug, A 46­year­old man with depression
on tranylcypromine has decided to switch over to sertraline
– Withdraw and wait two weeks before starting new drug

Question 34 HiY Pharmacology EMI034
Partially correct Drug action on neurotransmitters
For each of the examples given below, identify the level of drug action on neurotransmitters;
Mark 4.00 out of
5.00

Flag question
Synthesis of neurotransmitter
Pre­synaptic receptor activity
The suspected Re­uptake
mechanism of
Release from storage
therapeutic action of
lithium via inositol
Partial agonist
Antagonists
Postsynaptic receptors
Storage of neurotransmitter
Full agonist
Degradation
Second messengers

Benzodiazepines at
GABA­A complex

1536
Synthesis of neurotransmitter
Pre­synaptic receptor activity
Re­uptake
Release from storage
Partial agonist
Antagonists
Postsynaptic receptors
Storage of neurotransmitter
Full agonist
Synthesis of neurotransmitter
Degradation
Pre­synaptic receptor activity
Amphetamine on Second messengers
Re­uptake
dopamine and Release from storage
noradrenaline. Partial agonist
Antagonists
Postsynaptic receptors
Storage of neurotransmitter
Full agonist
Synthesis of neurotransmitter
Degradation
Pre­synaptic receptor activity
Therapeutic Second messengers
Re­uptake
mechanism of
Release from storage
monoamine oxidase
inhibitors
Partial agonist
Antagonists
Postsynaptic receptors
Storage of neurotransmitter
Full agonist
Synthesis of neurotransmitter
Degradation
Pre­synaptic receptor activity
Second messengers
Re­uptake
Lofexidine at alpha­2
Release from storage
receptors
Partial agonist
Antagonists
Postsynaptic receptors
Storage of neurotransmitter
Full agonist
Check
Degradation
Second messengers

Explanation: Presynaptic receptors on stimulation lead to a reduction in the output of
respective neurotransmitter from the presynaptic neuron. Examples of drugs exploiting this
mechanism include clonidine and lofexidine at alpha2 receptors. Most agonist/antagonist
actions refer to post­synaptic receptors. e.g., Most antipsychotics that act at D2 receptors.
Examples of partial agonists include aripiprazole at D2, buspirone at 5HT1A, clonazepam at
BDZ receptor and buprenorphine at opioid receptor mu. Antagonists include flumazenil at
GABA­A for benzodiazepines, antipsychotics at D2 for dopamine. Full agonists include
benzodiazepines at GABA­A complex, bromocriptine at dopamine receptors, etc.
The correct answer is: The suspected mechanism of therapeutic action of lithium via inositol –
Second messengers, Benzodiazepines at GABA­A complex – Full agonist, Amphetamine on
dopamine and noradrenaline. – Release from storage, Therapeutic mechanism of
monoamine oxidase inhibitors – Degradation, Lofexidine at alpha­2 receptors – Pre­synaptic
receptor activity

1537
Question 35 HiY Pharmacology EMI035
Incorrect Common adverse effects 
For the drugs given below identify the most common side effects from the list.
Mark 0.00 out of
4.00

Flag question
Ployuria
Diplopia
Postural hypotension
Lithium Nausea
Hair loss
Ataxia
Sedation
Insomnia
Weight gain
Ployuria
Agranulocytosis
Diplopia
Postural hypotension
Venlafaxine Nausea
Hair loss
Ataxia
Sedation
Insomnia
Weight gain
Ployuria
Agranulocytosis
Diplopia
Postural hypotension
Carbamazepine in the
Nausea
elderly
Hair loss
Ataxia
Sedation
Insomnia
Weight gain
Check
Agranulocytosis

Explanation: SNRIs such as venlafaxine are often associated with gastrointestinal side effects
and sexual side effects. Polyuria is the most common side effect of lithium therapy and is
reportedly seen in 20 to 25% of patients. The common side effects of lithium include polyuria,
polydipsia, fine tremors and bad metallic taste in the mouth. In elderly patients, sedation is
reportedly the most common side effect of carbamazepine, and mild transient leukopenia
develops in approximately 10% of patients.
The correct answer is: Lithium
– Ployuria, Venlafaxine
– Nausea, Carbamazepine in the elderly
– Sedation

Question 36 HiY Pharmacology EMI036
Partially correct Treatments in the elderly
Identify the drug for each of the following descriptions
Mark 3.00 out of
5.00

Flag question

1538
This drug, when St. John's wort
combined with SSRIs,
Gingko biloba
can induce serotonin
syndrome Primrose oil
Lemon grass
Atropine
Aromatherapy
Vitamin E
Peppermint oil
Lemon balm
St. John's wort
Gingko biloba
This drug inhibits Primrose oil
mucosal secretions and
Lemon grass
relaxes involuntary
muscles
Atropine
Aromatherapy
Vitamin E
Peppermint oil
Lemon balm
St. John's wort
Gingko biloba
Primrose oil
Used for the treatment
Lemon grass
of agitation in dementia
Atropine
Aromatherapy
Vitamin E
Peppermint oil
Lemon balm
St. John's wort
Gingko biloba
Used for the treatment Primrose oil
of cognitive symptoms Lemon grass
of dementia Atropine
Aromatherapy
Vitamin E
Peppermint oil
Lemon balm
St. John's wort
Gingko biloba
Primrose oil
Lemon grass
Usually administered
Atropine
by skin massage to
relieve tension, Aromatherapy
increase circulation Vitamin E
and aid relaxation Peppermint oil
Lemon balm

Check

Explanation: 

1539
St. John's wort may interact with tricyclic antidepressants, SSRIs, and monoamine oxidase
inhibitors (MAOIs) and potentially lead to the dangerous condition of serotonin syndrome. 
Atropine is anticholinergic; it reduces secretions and relaxes smooth muscles, reducing
frequent contractions in these muscles. 
In a recent open labelled trial on people with dementia, the use of a range of essential oils,
including ylang ylang, patchouli, rosemary, peppermint and others, produced a marked
decrease in disturbed behaviour in the majority of participants (Beshara et al. Int. J.
Aromatherapy. 2003; 12: 207­12). 
In folklore, linen bags were filled with lavender flowers and placed under pillows in order to
facilitate sleep: one trial showed that the use of lavender increased sleep patterns of dementia
patients who were in residential care.[Henry et al. Int. J. Aromatherapy. 1994; 6:2, 28­30.] In a
trial involving 122 non­demented patients in intensive care, massage aromatherapy using
lavender oil was well received, the greatest improvements being in mood and reduction in
anxiety [Dunn et al. Adv. Nurs. 1995; 21: 34­40.]. In another trial, lavender, geranium and
mandarin essential oils in an almond oil base were applied to the skin of 39 patients over an
unspecified period. This resulted in increased alertness, contentment and sleeping at night;
and reduced levels of agitation, withdrawal and wandering.[Kilstoff et al. Int. J. Nurs. Pract.
1998; 4: 70­83]. 
Excerpts above retrieved from http://alzheimers.org.uk/site/scripts
The correct answer is: This drug, when combined with SSRIs, can induce serotonin syndrome
– St. John's wort, This drug inhibits mucosal secretions and relaxes involuntary muscles –
Atropine, Used for the treatment of agitation in dementia – Peppermint oil, Used for the
treatment of cognitive symptoms of dementia – Gingko biloba, Usually administered by skin
massage to relieve tension, increase circulation and aid relaxation – Aromatherapy

Question 37 HiY Pharmacology EMI037
Correct Theories of drug action & side effects
Select one option for each description below.
Mark 3.00 out of
3.00
A 34­year­old patient is
Flag question recently started on 5HT1A antagonism
risperidone. He Skeletal muscle calcium channel blockade
develops dizziness on D1 receptor stimulation
the mornings and has
CYP450 enzyme inhibition
had two falls since the
D2 receptor antagonism
initiation of risperidone.
He is not on any co­ Alpha 2 blockade
prescribed CYP450 enzyme induction
psychotropics. Direct ovarian stimulation via LH/FSH
Alpha­1 blockade
A patient with bipolar 5HT1A antagonism
disorder type 2 is
5HT2A stimulation
Skeletal muscle calcium channel blockade
Downregulation of alpha­2 adrenergic receptors
treated with
Carbamazepine. She
D1 receptor stimulation
becomes pregnant in CYP450 enzyme inhibition
spite of adhering strictly D2 receptor antagonism
to her oral Alpha 2 blockade
contraceptive CYP450 enzyme induction
prescription. Direct ovarian stimulation via LH/FSH
A patient attending a Alpha­1 blockade
Regional Affective 5HT2A stimulation
Disorder Services has Downregulation of alpha­2 adrenergic receptors
been prescribed lithium

1540
to augment the effect of 5HT1A antagonism
fluoxetine for
Skeletal muscle calcium channel blockade
depression. She
experiences tremors,
D1 receptor stimulation
hyperthermia, sweating CYP450 enzyme inhibition
and confusion. On D2 receptor antagonism
examination, Alpha 2 blockade
hyperreflexia is noted. CYP450 enzyme induction
Direct ovarian stimulation via LH/FSH
Alpha­1 blockade
5HT2A stimulation
Check Downregulation of alpha­2 adrenergic receptors

Explanation: Postural hypotension is commonly associated with antipsychotic drugs that are
antagonists at postsynaptic adrenergic a one receptors. Examples include clozapine,
chlorpromazine, quetiapine and risperidone. (Ref: Maudsley Prescribing Guidelines. 10th
edition. Pg: 94). Carbamazepine is a potent inducer of hepatic cytochrome P450 enzymes.
Patients requiring contraception should either receive a preparation containing not less than
50 μg oestrogen or use a non­hormonal method (Ref: Maudsley Prescribing Guidelines. 10th
edition. Pg: 136). Lithium may enhance the pharmacologic effects of selective serotonin
reuptake inhibitors (SSRIs) and potentiate the risk of serotonin syndrome, a condition thought
to result from hyperstimulation of brainstem 5­HT1A and 2A receptors. (Sandson NB. An
Overview of Psychotropic Drug­Drug Interactions.Psychosomatics 46:464­494, September­
October 2005)
The correct answer is: A 34­year­old patient is recently started on risperidone. He develops
dizziness on the mornings and has had two falls since the initiation of risperidone. He is not
on any co­prescribed psychotropics. – Alpha­1 blockade, A patient with bipolar disorder type
2 is treated with Carbamazepine. She becomes pregnant in spite of adhering strictly to her
oral contraceptive prescription. – CYP450 enzyme induction, A patient attending a Regional
Affective Disorder Services has been prescribed lithium to augment the effect of fluoxetine for
depression. She experiences tremors, hyperthermia, sweating and confusion. On
examination, hyperreflexia is noted. – 5HT2A stimulation

Question 38 HiY Pharmacology EMI038
Partially correct Drug interactions with combination therapies 
For each of the following drug combinations, choose the most characteristic adverse effect
Mark 2.67 out of
from the list above.
4.00

Flag question
Dry mouth
A combination of Tardive dyskinesia
imipramine and Rhinorrhoea
chlorpromazine is
Hypersalivation
prescribed for
depression with
Absence seizure
psychosis Fever
Anaemia
Gynaecomastia
A combination of Presbyopia
tranylcypromine and
clomipramine
prescribed for

1541
treatment­resistant
Dry mouth
depression at a
Tardive dyskinesia
specialized regional
affective disorders unit Rhinorrhoea
Hypersalivation
Absence seizure
Fever
Anaemia
Gynaecomastia
Presbyopia
Dry mouth
A combination of Tardive dyskinesia
amisulpride and Rhinorrhoea
clozapine prescribed Hypersalivation
for treatment­resistant Absence seizure
schizophrenia Fever
Anaemia
Gynaecomastia
Presbyopia
Check

Explanation: Both chlorpromazine and imipramine have significant antimuscarinic action,
which can lead to dry mouth and blurred vision.Clomipramine is the most serotonergic TCA.
Combination with tranylcypromine can cause severe 5HT toxicity, which can be fatal.While
hypersalivation appears less frequent upon adding amisulpride to clozapine in several case
series, it is by far the most common side effect reported in the literature for the combination
therapy (for example, in 27% of cases in Ziegenbin et al., 2006). 
The correct answer is: A combination of imipramine and chlorpromazine is prescribed for
depression with psychosis
– Dry mouth, A combination of tranylcypromine and clomipramine prescribed for treatment­
resistant depression at a specialized regional affective disorders unit
– Fever, A combination of amisulpride and clozapine prescribed for treatment­resistant
schizophrenia
– Hypersalivation

Question 39 HiY Pharmacology EMI039
Correct Receptor mechanisms
For the following side effects exhibited by the drugs, identify the receptor action responsible
Mark 4.00 out of
for it
4.00

Flag question
H2­ antagonism
5­HT2 agonism
A 40­year­old woman 5­HT3 antagonism
on fluoxetine
M2 receptor blockade
developed nausea and
vomiting
M1 receptor blockade
5­HT1 antagonism
5­HT1 agonism
5­HT3 agonism
H1­ antagonism
H3­ antagonism
M3 receptor blockade

1542
A 65­year­old woman H2­ antagonism
complained of
5­HT2 agonism
excessive sedation on
taking trazodone 5­HT3 antagonism
M2 receptor blockade
M1 receptor blockade
5­HT1 antagonism
5­HT1 agonism
5­HT3 agonism
H1­ antagonism
H2­ antagonism
H3­ antagonism
5­HT2 agonism
A 33­year­old M3 receptor blockade
5­HT3 antagonism
gentleman suffered
M2 receptor blockade
from dry mouth on
taking amitriptyline
M1 receptor blockade
5­HT1 antagonism
5­HT1 agonism
5­HT3 agonism
H1­ antagonism
H2­ antagonism
A 23­year­old woman
H3­ antagonism
5­HT2 agonism
has developed weight M3 receptor blockade
5­HT3 antagonism
gain and is overly M2 receptor blockade
sedated on taking M1 receptor blockade
mirtazapine 5­HT1 antagonism
5­HT1 agonism
5­HT3 agonism
H1­ antagonism
Check
H3­ antagonism
M3 receptor blockade

Explanation: SSRIs such as fluoxetine cause GI side­effects due to 5HT3 receptor stimulation.
Trazodone causes sedation through H1 receptor antagonism. It can also induce priapism
through alpha one receptor antagonism. Dry mouth is an anticholinergic side effect of
tricyclics like amitriptyline and is due to M3 receptor blockade. Mirtazapine can cause
sedation and weight gain through H1 receptor antagonism.
The correct answer is: A 40­year­old woman on fluoxetine developed nausea and vomiting –
5­HT3 agonism, A 65­year­old woman complained of excessive sedation on taking trazodone
– H1­ antagonism, A 33­year­old gentleman suffered from dry mouth on taking amitriptyline –
M3 receptor blockade, A 23­year­old woman has developed weight gain and is overly
sedated on taking mirtazapine – H1­ antagonism

Question 40 HiY Pharmacology EMI040
Correct Antidepressants
For each of the following descriptions choose the most appropriate antidepressant from the
Mark 4.00 out of
given list
4.00

Flag question

1543
Which of the following Venlafaxine
drugs is associated
Trazadone
with a specific risk of
hypertension?
Fluoxetine
Reboxetine
Mirtazapine
Minocycline
Agomelatine
Citalopram
Paroxetine
Escitalopram
Venlafaxine
Trazadone
Which of the following Fluoxetine
drugs is selective for
Reboxetine
noradrenergic reuptake
inhibition?
Mirtazapine
Minocycline
Agomelatine
Citalopram
Paroxetine
Venlafaxine
Which of the following Escitalopram
Trazadone
drugs is called as a Fluoxetine
noradrenergic and Reboxetine
specific serotonergic Mirtazapine
antagonist? Minocycline
Agomelatine
Citalopram
Stimulation of certain Paroxetine
serotonergic receptors Escitalopram
leading to the adverse
effects such as
Venlafaxine
insomnia, agitation, Trazadone
sexual dysfunction and Fluoxetine
nausea is seen Reboxetine
commonly during the Mirtazapine
use of antidepressants. Minocycline
Which of the following Agomelatine
antidepressant has a Citalopram
low propensity for these
Paroxetine
effects?
Escitalopram
Check

Explanation: 
There is no evidence that venlafaxine, used in prescription doses, is as cardiotoxic as the
TCAs. But if taken in an overdose, it is at least threefold more likely than the SSRIs to result in
death. Hypertension occurs in some patients on venlafaxine especially in those who are
predisposed to be hypertensive. 
Both mirtazepine and mianserin enhance noradrenaline release (alpha­2 blockade) and 5­
HT2A receptor antagonism.These two drugs are often grouped as (NaSSA) noradrenergic
and specific serotonergic antidepressants. Mirtazapine causes a blockade of both 5­HT­2 and

1544
5­HT­3 receptors ­ hence the side effects resulting from the stimulation of these receptors is
rarely seen with the use of mirtazapine. 
Reboxetine molecule has a chemical similarity to fluoxetine. However, it is relatively selective
for inhibiting the reuptake of noradrenaline and is described as an NRI.
The correct answer is: Which of the following drugs is associated with a specific risk of
hypertension? – Venlafaxine, Which of the following drugs is selective for noradrenergic
reuptake inhibition? – Reboxetine, Which of the following drugs is called as a noradrenergic
and specific serotonergic antagonist? – Mirtazapine, Stimulation of certain serotonergic
receptors leading to the adverse effects such as insomnia, agitation, sexual dysfunction and
nausea is seen commonly during the use of antidepressants. Which of the following
antidepressant has a low propensity for these effects? – Mirtazapine

Question 41 HiY Pharmacology EMI041
Correct Partial agonism 
Identify the predominant receptor level of action for the following partial agonistic drugs.
Mark 4.00 out of
4.00

Flag question
5HT2 receptor
Kappa opioid receptor
5HT3 receptor
Buspirone  
D­1 receptor
D­2 receptor
D­5 receptor
Benzodiazepine receptor
5 HT1A receptor
5HT2 receptor
Mu opioid receptor
Kappa opioid receptor
Omega opioid receptor
5HT3 receptor
Clonazepam  
D­1 receptor
D­2 receptor
D­5 receptor
Benzodiazepine receptor
5 HT1A receptor
5HT2 receptor
Mu opioid receptor
Kappa opioid receptor
Omega opioid receptor
5HT3 receptor
Buprenorphine  
D­1 receptor
D­2 receptor
D­5 receptor
Benzodiazepine receptor
5 HT1A receptor
5HT2 receptor
Mu opioid receptor
Kappa opioid receptor
Omega opioid receptor
5HT3 receptor
Aripiprazole  
D­1 receptor
D­2 receptor
D­5 receptor
Benzodiazepine receptor
5 HT1A receptor
Check
Mu opioid receptor
Omega opioid receptor

Explanation: 

1545
Partial agonists include Aripiprazole on D2; Buspirone on 5HT1A; Clonazepam on BDZ
receptor; Buprenorphine on opioid receptor mu. 
Aripiprazole has a particularly low risk of acute extrapyramidal symptoms despite high levels
of occupation of D2 receptors (> 90%) at therapeutic doses; this is probably due to its partial­
agonist activity at D2 receptors. 
Buspirone acts as a partial agonist on serotonin 5­HT1A receptors ­ presynaptic agonism
leads to inhibition of release of serotonin, with consequent antianxiety effects. Postsynaptic
agonism leads to antidepressant activity. 
Benzodiazepines act via a specific site called omega site in GABA­A complex. All are agonists
except clonazepam, which is a partial agonist. 
Buprenorphine is a partial opioid agonist. Lower doses produce mild agonism; higher doses
produce antagonistic effects. 
Excerpt retrieved from 'Gardner et al. (2005). Modern antipsychotic drugs: a critical overview',
http://www.cmaj.ca/content/172/13/1703.full (accessed April 6, 2015).
The correct answer is: Buspirone – 5 HT1A receptor, Clonazepam – Benzodiazepine
receptor, Buprenorphine – Mu opioid receptor, Aripiprazole – D­2 receptor

Question 42 HiY Pharmacology EMI042
Correct Antidementia drugs
For the antidementia drugs given below, identify their mode of action from list shown here:
Mark 6.00 out of
6.00

Flag question
Nicotine receptor agonist
Nicotine receptor antagonist
Affects dopamine transmission
Accessory effect of
NMDA receptor antagonist
galantamine
NMDA receptor agonist
Irreversible competitive inhibitor of AchEIs
Irreversible non­competitive inhibitor of AchEIs
Butyrylcholinesterase inhibitor
Nicotine receptor agonist
Nicotine receptor antagonist
Affects dopamine transmission
Memantine NMDA receptor antagonist
NMDA receptor agonist
Irreversible competitive inhibitor of AchEIs
Irreversible non­competitive inhibitor of AchEIs
Butyrylcholinesterase inhibitor
Nicotine receptor agonist
Nicotine receptor antagonist
Affects dopamine transmission
NMDA receptor antagonist
NMDA receptor agonist
Accessory effect of
rivastigmine
Irreversible competitive inhibitor of AchEIs
Irreversible non­competitive inhibitor of AchEIs
Butyrylcholinesterase inhibitor

1546
Check

Explanation: Binding to the Acetl Cholinersterase enzyme (AChE) sites may be either
reversible or irreversible, and may be competitive or noncompetitive with acetylcholine.
Rivastigmine is a non­selective inhibitor of both acetyl cholinesterase and
butyrylcholinesterase. Galantamine is a competitive drug. It is a selective and reversible
inhibitor of acetyl cholinesterase. It is also a nicotine modulator. Memantine is an NMDA
receptor antagonist and a weak 5­HT3 receptor antagonist.
The correct answer is: Accessory effect of galantamine
– Nicotine receptor agonist, Memantine
– NMDA receptor antagonist, Accessory effect of rivastigmine
– Butyrylcholinesterase inhibitor

Question 43 HiY Pharmacology EMI043
Partially correct Adverse effects 
For each of the following drugs, identify the most important side effect associated with its
Mark 4.00 out of
therapeutic use
6.00

Flag question
Nausea and vomiting
Word finding difficulties
Bradycardia
Galantamine  
Weight loss
Hypersalivation
Hypothyroidism
Diastolic Hypertension
Nasal stuffiness
Nausea and vomiting
QTc prolongation
Word finding difficulties
Polydipsia
Bradycardia
Venlafaxine Blurred vision  
Weight loss
Hypersalivation
Hypothyroidism
Diastolic Hypertension
Nasal stuffiness
Nausea and vomiting
QTc prolongation
Word finding difficulties
Polydipsia
Bradycardia
Blurred vision
Weight loss  
Hypersalivation
Hypothyroidism
Topiramate Diastolic Hypertension
Nasal stuffiness
QTc prolongation
Polydipsia
Blurred vision

1547
Lithium Nausea and vomiting
Word finding difficulties
Bradycardia
Weight loss  
Hypersalivation
Hypothyroidism
Diastolic Hypertension
Nasal stuffiness
Nausea and vomiting
QTc prolongation
Word finding difficulties
Polydipsia
Bradycardia
Aripiprazole Blurred vision  
Weight loss
Hypersalivation
Hypothyroidism
Diastolic Hypertension
Nasal stuffiness
Nausea and vomiting
QTc prolongation
Word finding difficulties
Polydipsia
Bradycardia
Topiramate Blurred vision  
Weight loss
Hypersalivation
Hypothyroidism
Diastolic Hypertension
Nasal stuffiness
Check QTc prolongation
Polydipsia
Blurred vision
Explanation: 
Antidementia drugs like galantamine can produce bradycardia, especially in those with
supraventricular conduction delay. 
Significant numbers of patients receiving doses above 300mg/day of venlafaxine experience
an increase in diastolic blood pressure. This risk is not restricted to those with preexisting
hypertension.
Topiramate: Several studies have documented language impairment due to the use of
topiramate. Word finding difficulties are noted in 7.2% of epileptic patients taking topiramate,
especially those with a positive family history of epilepsy, or simple partial seizures and left
temporal seizures. Topiramate is a weak inhibitor of renal carbonic anhydrase and can
promote the development of renal stones. It is weight neutral and can even cause weight loss.
Lithium can cause a variety of thyroid problems ­ the most common being a benign
hypothyroid state. 5% patients may develop goiter, and overt hyperthyroidism is also reported
in some cases. Thyroid deficiency is common in those with high risk for preexisting antithyroid
antibodies such as middle­aged women. The risk is 3­4:1 in women and is high in first two
years of treatment. Rapid cycling patients are at higher risk. High TSH is seen in nearly 1/3rd
of chronic lithium­treated patients ­ without clinical hypothyroidism. In resistant depression or
non­responsive cyclers with bipolar it may be useful treating this subclinical state with
thyroxine. 
Aripiprazole: It is a D2 receptor partial agonist and, therefore, more likely to induce side effects
like nausea, vomiting, etc. 

The correct answer is: Galantamine – Bradycardia, Venlafaxine – Diastolic Hypertension,
Topiramate – Weight loss, Lithium – Hypothyroidism, Aripiprazole – Nausea and vomiting,
Topiramate – Word finding difficulties

1548
Question 44 HiY Pharmacology EMI044
Correct Teratogenicity of psychotropics 
Choose the most likely offending agent for each of the following teratogenic effects from the
Mark 5.00 out of
given list.
5.00

Flag question
Diazepam
Zopiclone
Mirtazapine
Newborn with cleft
Carbamazepine
palate
Paroxetine
Valproate
Reboxetine
Lithium
Diazepam
Newborn with Zopiclone
symptoms of Mirtazapine
antidepressant Carbamazepine
discontinuation Paroxetine
reaction Valproate
Reboxetine
Lithium
Diazepam
Zopiclone
Mirtazapine
Cognitive impairment in
Carbamazepine
children
Paroxetine
Valproate
Reboxetine
Lithium
Check

Explanation: 
Benzodiazepine use in pregnancy is associated with congenital anomalies such as cleft lip
and cleft palate in the new born. Lamotrigine use in pregnancy may be associated with cleft
palate. Lamotrigine monotherapy is associated with a 3.2% frequency of malformations.
Paroxetine and venlafaxine are the most common offending drugs to precipitate
antidepressant discontinuation symptoms in the newborn due to their short half­lives. 
Sodium valproate is the most teratogenic of all antiepileptics.Congenital anomalies include
risk of neural tube defect (1­2 %), risk of spina bifida (10 fold increase),digital and limb
defects, heart defects ( VSD, Pulmonary stenosis etc. 4 fold increase), urogenital
malformations, low birth weight and psychomotor slowness. A greater frequency of cognitive
impairment in children is associated with valproate in mothers.
The correct answer is: Newborn with cleft palate – Diazepam, Newborn with symptoms of
antidepressant discontinuation reaction
– Paroxetine, Cognitive impairment in children – Valproate

1549
Question 45 HiY Pharmacology EMI045
Correct Adverse effects 
Identify the type of side effects caused by antipsychotics, using descriptions given below
Mark 2.00 out of
2.00
A 25­year­old
Flag question gentleman with an Hyponatraemia
acute episode of Cerebellar ataxia
psychosis was started SIADH
on trifluoperazine 3 Neuroleptic malignant syndrome
days ago. He has now
Cholinergic rebound syndrome
developed a high fever,
rigidity, profuse Akathisia
sweating and Tardive dyskinesia
confusion. Serotonin syndrome
Parkinsonism
A 69­year­old lady on Hyponatraemia
Acute dystonia
long­term treatment Cerebellar ataxia
with depixol injection SIADH
presents with lip Neuroleptic malignant syndrome
smacking, chewing and Cholinergic rebound syndrome
protrusion of the Akathisia
tongue. Tardive dyskinesia
Serotonin syndrome
Parkinsonism
Check
Acute dystonia

Explanation: 
Neuroleptic Malignant Syndrome can occur at anytime during treatment with neuroleptics. It
consists of a tetrad of extreme hyperthermia, severe muscular rigidity and confusion, and
autonomic fluctuations (BP and pulse rate). Patients may be akinetic and mute.Increased
WBC count, creatinine phosphokinase, liver enzymes, plasma myoglobin, and myoglobinuria
are noted.The onset is generally subacute i.e. in 24 to 72 hours, and if untreated lasts 10 to 14
days. It is more common in young men, especially after prolonged agitation and when using
high potency drugs during rapid tranquillization. 
Tardive dyskinesia is a late side effect occurring in nearly 25% patients usually only after (at
least six months) 1 ­ 2 years of treatment. It presents as abnormal, involuntary, irregular
choreoathetotic movements of the muscles of the head, limbs, and trunk. Perioral movements
are the most common.
The correct answer is: A 25­year­old gentleman with an acute episode of psychosis was
started on trifluoperazine 3 days ago. He has now developed a high fever, rigidity, profuse
sweating and confusion. – Neuroleptic malignant syndrome, A 69­year­old lady on long­term
treatment with depixol injection presents with lip smacking, chewing and protrusion of the
tongue. – Tardive dyskinesia

Question 46 HiY Pharmacology EMI046
Correct Receptor mechanisms 
Identify the drugs & receptors that display the following features;
Mark 5.00 out of
5.00

Flag question

1550
MAOIs
Propranalol at beta adrenergic receptors
Partial agonist  Ketamine at NMDA receptors
Phencyclidine at NMDA receptors
Buprenorphine
Benzodiazepines at GABA A complex

MAOIs
Propranalol at beta adrenergic receptors
Ketamine at NMDA receptors
Competitive antagonist  Phencyclidine at NMDA receptors
Buprenorphine
Benzodiazepines at GABA A complex

MAOIs
Propranalol at beta adrenergic receptors
Ketamine at NMDA receptors
Noncompetitive Phencyclidine at NMDA receptors
antagonist Buprenorphine
Benzodiazepines at GABA A complex

Check

Explanation: 
Full agonists produce a maximal response. Examples are benzodiazepines at GABA­A
complex and bromocriptine for dopamine. Partial agonists cannot elicit the maximal response
and are less effective than full agonists. Examples include aripiprazole, buspirone and
buprenorphine. These partial agonists have a ceiling effect. The degree of response of a
partial agonist depends on availability of physiological neurotransmitter in the vicinity; i.e.
when maximal dopamine is available, a partial agonist can actually inhibit the dopaminergic
transmission as a less efficient molecule competes with more efficient molecule. In dopamine
deficient states, the same partial agonist can enhance dopaminergic effects. (Aripiprazole).
Hence, these partial agonists have a ceiling effect. 
Antagonists are drugs that interact with receptors to interfere with their activation by
neurotransmitter or other agonistic molecules. Competitive antagonism can be reversed
completely by increasing the dose of the agonist drug. Competitive antagonists reduce the
potency (minimal dose needed to produce an effect) but not the efficacy (maximal response
produced) of agonists. Examples of competitive antagonism include atropine at muscarinic
receptors and propranolol at beta­adrenergic receptors. Noncompetitive antagonists alter the
receptor site in some way so increasing the dose of the agonist drug can reverse the effects
only partially e.g. Ketamine at NMDA receptors.
The correct answer is: Partial agonist 
– Buprenorphine, Competitive antagonist 
– Propranalol at beta adrenergic receptors, Noncompetitive antagonist – Ketamine at NMDA
receptors

1551
Question 47 HiY Pharmacology EMI047
Correct Chemical class of psychotropic drugs
Match each chemical class below with the most appropriate drug from the given list:
Mark 4.00 out of
4.00

Flag question
Trifluoperazine
Loxapine
Haloperidol
Aliphatic  
Thioridazine
phenothiazine
Chlorpromazine
Risperidone
Zuclopenthixol
Quetiapine
Ziprasidone
Trifluoperazine
Molindone
Loxapine
Haloperidol
Thioridazine  
Butyrophenones
Chlorpromazine
Risperidone
Zuclopenthixol
Quetiapine
Ziprasidone
Trifluoperazine
Molindone
Loxapine
Haloperidol
Thioridazine  
Piperazine derivatives
Chlorpromazine
Risperidone
Zuclopenthixol
Quetiapine
Ziprasidone
Trifluoperazine
Molindone
Loxapine
Haloperidol
Thioridazine  
Piperidine derivatives
Chlorpromazine
Risperidone
Zuclopenthixol
Quetiapine
Ziprasidone
Check
Molindone

Explanation: Aliphatic phenothiazine includes Chlorpromazine, promazine, and
triflupromazine. Butyrophenones include Haloperidol, Droperidol. Piperazine derivatives
include Trifluoperazine, Fluphenazine, Perphenazine. Piperidine derivatives include
thioridazine.
The correct answer is: Aliphatic phenothiazine – Chlorpromazine, Butyrophenones –
Haloperidol, Piperazine derivatives – Trifluoperazine, Piperidine derivatives – Thioridazine

Question 48 HiY Pharmacology EMI048

1552
Correct Chemical class of psychotropic drugs 
Match each drug below with the most appropriate chemical class from the given list:
Mark 5.00 out of
5.00

Flag question Butyrophenones
Imidazopyridine
Hydrazine derivatives
Zopiclone  
Triazolopyridine
Cyclopyrrolone
Tertiary amine
Secondary amine
Aliphatic phenothiazine
Butyrophenones
Piperazine derivatives
Imidazopyridine
Piperidine derivatives
Hydrazine derivatives
Imipramine  
Triazolopyridine
Cyclopyrrolone
Tertiary amine
Secondary amine
Aliphatic phenothiazine
Butyrophenones
Piperazine derivatives
Imidazopyridine
Piperidine derivatives
Hydrazine derivatives
Nortryptyline  
Triazolopyridine
Cyclopyrrolone
Tertiary amine
Secondary amine
Aliphatic phenothiazine
Butyrophenones
Piperazine derivatives
Imidazopyridine
Piperidine derivatives
Hydrazine derivatives
Phenelzine  
Triazolopyridine
Cyclopyrrolone
Tertiary amine
Secondary amine
Aliphatic phenothiazine
Butyrophenones
Piperazine derivatives
Imidazopyridine
Piperidine derivatives
Hydrazine derivatives
Zolpidem  
Triazolopyridine
Cyclopyrrolone
Tertiary amine
Secondary amine
Aliphatic phenothiazine
Check Piperazine derivatives
Piperidine derivatives

Explanation: Cyclopyrrolone ­ Zopiclone, Imidazopyridine ­Zolpidem. Tertiary amines include
imipramine, amitriptyline, clomipramine, dothiepin, trimipramine. Secondary amines include
desipramine, amoxapine, nortriptyline and protriptyline. Hydrazine derivatives include
phenelzine and Isocarboxazid (greater hepatotoxicity than non hydrazine compounds such as
tranylcypromine)
The correct answer is: Zopiclone – Cyclopyrrolone, Imipramine – Tertiary amine, Nortryptyline
– Secondary amine, Phenelzine – Hydrazine derivatives, Zolpidem – Imidazopyridine

1553
Question 49 HiY Pharmacology EMI049
Correct Mechanism of action (1)
Identify the drug that matches each of the following mechanisms of action.
Mark 4.00 out of
4.00

Flag question
Mirtazapine
Lactate
Alpha autoreceptor and Lithium
selective serotonergic Glutamate
antagonist. Sildenafil
Aspartate
Sodium mono glutamate
Fluoxetine
Duloxetine
Mirtazapine
GABA
Lactate
Serotonin and Lithium
norepinephrine Glutamate
reuptake inhibitor. Sildenafil
Aspartate
Sodium mono glutamate
Fluoxetine
Duloxetine
Mirtazapine
GABA
Lactate
Lithium
An inhibitory metabolite
Glutamate
of glutamate
Sildenafil
Aspartate
Sodium mono glutamate
Fluoxetine
Duloxetine
Mirtazapine
GABA
Lactate
Lithium
Phosphodiesterase­5
Glutamate
inhibitor
Sildenafil
Aspartate
Sodium mono glutamate
Fluoxetine
Duloxetine
Check
GABA

Explanation: Duloxetine is similar to venlafaxine ­ an SNRI. It is said to have a better profile for
psychosomatic pain and neuropathic pain. Mirtazapine is known as NaSSA ­ Noradrenergic
and specific serotonergic antagonist.5HT2A antagonism, alpha two antagonism, anti
histaminic and anti 5HT3 properties are noted with Mirtazapine. Of these, alpha­2
autoreceptor and heteroreceptor (adrenergic receptors on serotonergic neurons) blockade
releases noradrenaline and serotonin, respectively, from pre­synaptic neurones. GABA is the
most common inhibitory amino acid neurotransmitter, seen in 60% of the brain synapses.
GABA is broken down to glutamate, and then eventually to succinic acid. Sildenafil [Viagra] is
a Phosphodiesterase­5 Inhibitor, used for the treatment of erectile dysfunction.
The correct answer is: Alpha autoreceptor and selective serotonergic antagonist. –

1554
Mirtazapine, Serotonin and norepinephrine reuptake inhibitor. – Duloxetine, An inhibitory
metabolite of glutamate – GABA, Phosphodiesterase­5 inhibitor – Sildenafil

Question 50 HiY Pharmacology EMI050
Partially correct History of pharmacology (1)
For each of the following historical figures choose one most appropriate discovery made from
Mark 4.00 out of
the above list;
5.00

Flag question
Chlorpromazine for psychosis
Carbamazepine for mood disorders
Haloperidol for schizophrenia
Osterloh Electroconvulsive therapy  
Sildenafil for erectile dysfunction
Clozapine for resistant schizophrenia
Lithium for mania
Imipramine for depression
Chlorpromazine for psychosis
Carbamazepine for mood disorders
Haloperidol for schizophrenia
Takezaki &  
Electroconvulsive therapy
Hanaoka
Sildenafil for erectile dysfunction
Clozapine for resistant schizophrenia
Lithium for mania
Imipramine for depression
Chlorpromazine for psychosis
Carbamazepine for mood disorders
Haloperidol for schizophrenia
Cade Electroconvulsive therapy  
Sildenafil for erectile dysfunction
Clozapine for resistant schizophrenia
Lithium for mania
Imipramine for depression
Chlorpromazine for psychosis
Carbamazepine for mood disorders
Haloperidol for schizophrenia
Electroconvulsive therapy  
Sildenafil for erectile dysfunction
Clozapine for resistant schizophrenia
Kuhn Lithium for mania
Imipramine for depression

1555
Cereletti & Bini Chlorpromazine for psychosis
Carbamazepine for mood disorders
Haloperidol for schizophrenia
Electroconvulsive therapy  
Sildenafil for erectile dysfunction
Clozapine for resistant schizophrenia
Lithium for mania
Imipramine for depression
Check

Explanation: 
Kuhn in 1958 discovered that among the various different psychiatric disturbances
'endogenous' depression responded best to imipramine. 
In 1971, Drs. Takezaki and Hanaoka initially employed carbamazepine to treat mania in
patients refractory to antipsychotics (as lithium was not available in Japan at that time). Dr.
Okuma, working independently, did the same intervention with similar success. 
Sildenafil was initially studied for use in hypertension and angina. The first clinical trials were
conducted in Swansea. Phase 1 clinical trials directed by Ian Osterloh suggested that the drug
had little effect on angina, but it can produce marked penile erections and is now used for the
treatment of erectile dysfunction. 
Cade, an Australian psychiatrist, had a theory that mania was due to a metabolic disorder,
indicated by excessive urea in the urine of those with bipolar disorder. This historic
misconception regarding the connection between mania and urea formed the grounds for a
series of experiments designed by Cade in which he injected uric acid (as lithium urate) into
guinea pigs. Rather surprisingly, this produced a calming effect instead of increased
excitation. This observation later paved the way for developing lithium as an antimanic
compound. 
Roland Kuhn first administered Imipramine as an antidepressant to 'Paula JF', and on the
basis of her clinical recovery claimed that Imipramine­like agents work better in vital (or
endogenous) rather than reactive or neurotic depression. 
In 1938, Ugo Cereletti and Lucio Bini successfully employed electrical stimulation in the
treatment of a mute catatonic patient and ECT was introduced.
The correct answer is: Osterloh – Sildenafil for erectile dysfunction, Takezaki & Hanaoka –
Carbamazepine for mood disorders, Cade – Lithium for mania, Kuhn – Imipramine for
depression, Cereletti & Bini – Electroconvulsive therapy

Finish review

1556
 Home HiYield Paper A(1) Assessment

HiYield Paper A(1)

Started on Saturday, 27 June 2015, 2:44 PM
State Finished
Completed on Saturday, 27 June 2015, 2:44 PM
Time taken 9 secs
Marks 0.00/200.00
Grade 0.00 out of 100.00

Question 1 HiY Assessment EMI051
Not answered Dysarthria
Choose one aetiology for each of the following clinical situations;
Marked out of 4.00

Flag question
Extrapyramidal lesions
Mrs. W is a 55­year­old woman who Conversion disorder
presents with slow scanning speech.
Upper motor neuron lesions
She speaks with varying force and
Lesion's of Wernicke's area
appears to be broken down into
syllables.
Lower motor neuron lesions
Laryngeal lesions
Cerebellar lesions
Lesions of Broca's area
Vocal cord paralysis
Vocal cord paralysis
Upper motor neuron lesions
Conversion disorder
Mr. X is a 63­year­old man who Extrapyramidal lesions
presents with slow, slurred and Lesion's of Wernicke's area
monotonous speech. The pitch is Laryngeal lesions
high. On examination, his tongue is
Lower motor neuron lesions
small and contracted.
Lesions of Broca's area
Cerebellar lesions

Lower motor neuron lesions
Lesion's of Wernicke's area
Mr. Y is a 69­year­old man who Extrapyramidal lesions
presents with a speech of nasal
Laryngeal lesions
quality. It is slurred and indistinct.
Vocal cord paralysis
Cerebellar lesions
Upper motor neuron lesions
Conversion disorder
Lesions of Broca's area
Miss. Z is a 45­year­old woman with
sudden loss of speech following an
argument with her boyfriend

1557
Upper motor neuron lesions
Cerebellar lesions
Lesion's of Wernicke's area
Extrapyramidal lesions
Vocal cord paralysis
Lower motor neuron lesions
Conversion disorder
Laryngeal lesions
Check
Lesions of Broca's area

In cerebellar lesion scanning or ataxic speech is seen. Intonation, pitch and volume and also
be affected, as well as difficulty with alternate tongue movements. 
Flaccid dysarthria can be of various qualities depending on which motor neuron is affected in
a LMN type of lesion. The tongue shows loss of tone if XII nerve is involved and lies flaccidly
on the floor of the mouth. In LMN lesions, speech could be of nasal quality if IX and X nerves
are involved. 
In UMN lesions (e.g. pseudobulbar palsy), small and contracted tongue accompanies a high
pitched speech tone.
In hysterical aphonia, the examination is usually normal. Sudden loss of voice following a
stressor, but preserved vocal cord activity is notable.
The correct answer is: Mrs. W is a 55­year­old woman who presents with slow scanning
speech. She speaks with varying force and appears to be broken down into syllables. –
Cerebellar lesions, Mr. X is a 63­year­old man who presents with slow, slurred and
monotonous speech. The pitch is high. On examination, his tongue is small and contracted.
– Upper motor neuron lesions, Mr. Y is a 69­year­old man who presents with a speech of
nasal quality. It is slurred and indistinct. – Lower motor neuron lesions, Miss. Z is a 45­year­
old woman with sudden loss of speech following an argument with her boyfriend –
Conversion disorder

Question 2 HiY Assessment EMI052
Not answered Schizophrenic speech disturbance
For each description given below choose the most appropriate term from the list provided:
Marked out of 4.00

Flag question
Tangentiality
Flight of Ideas 
A disorder in the logical progression
of thoughts in which unrelated and Neologisms
unconnected ideas shift from one Derailment
subject to another. Circumstantiality
Clanging
Perseveration
Echolalia
Clanging
Logoclonia
Circumstantiality
A pattern of speech, which is indirect Thought blocking
Derailment
and delayed in reaching its goal idea Neologisms
Perseveration
Logoclonia
Thought blocking
Tangentiality
Echolalia

1558
Words are chosen for common Thought blocking
sounds of initial syllabi rather than Perseveration
the meaning Neologisms
Echolalia
Circumstantiality
Logoclonia
Flight of Ideas
Derailment
Tangentiality
Tangentiality
Neologisms
Making up a totally new word that is Clanging
Thought blocking
not in the dictionary Clanging
Logoclonia
Flight of Ideas
Perseveration
Circumstantiality
Check Echolalia
Derailment

Derailment is a symptom of schizophrenic thought disorder in which there is a total break in
the chain of association among succeeding thoughts. The relationship between the two
sequential ideas expressed in the speech is unclear to both the patient and the examiner. In
circumstantiality, thinking proceeds slowly, with many unnecessary details and digressions,
before returning to the point. It is seen in some patients with temporal lobe epilepsy or
alcohol­induced persisting dementia, learning difficulty and in obsessional personalities. The
verbal associations (due to words spoken) are called clang associations where thoughts are
associated with the structure of words rather than their meaning. In schizophrenic FTD,
clanging occurs more often with the first syllables as opposed to clangs seen in poetry,
humour and manic speech where they occur more at the end syllables. Neologisms refers to
making up a totally new word that is not in dictionary or using a known word with an entirely
different meaning e.g. 'Inkur' for pen (new) or 'roast' for pen (different).
The correct answer is: A disorder in the logical progression of thoughts in which unrelated and
unconnected ideas shift from one subject to another. – Derailment, A pattern of speech, which
is indirect and delayed in reaching its goal idea – Circumstantiality, Words are chosen for
common sounds of initial syllabi rather than the meaning – Clanging, Making up a totally new
word that is not in the dictionary – Neologisms

Question 3 HiY Assessment EMI053
Not answered Differential diagnoses
Choose the most likely diagnosis for each of the following situations;
Marked out of 3.00

Flag question
Acute stress reaction
Adjustment disorder
A 40­year­old woman was robbed in Schizophrenia
an underground station five years Depression
ago. Since then, she is fearful to Post­traumatic stress
leave her flat. When she goes to Normal grief reaction
supermarkets, she freezes and feels Bipolar disorder
that she might even die. Agoraphobia with panic disorder
Conversion disorder
Social phobia

1559
A 23­year­old woman works in a
warehouse. She is very anxious to Post­traumatic stress
speak even to her colleagues and Depression
boss. She does not socialise, never Adjustment disorder
been to a pub and isolates herself. Social phobia
She is worried about causing Conversion disorder
embarrassment when talking in front Schizophrenia
of others Bipolar disorder
Normal grief reaction
Normal grief reaction
A 26­year­old woman lost her dad in Acute stress reaction
a car accident three weeks ago. She Schizophrenia
Agoraphobia with panic disorder
has not taken the car since this Bipolar disorder
incident and doesn't want to use the Social phobia
same route. She feels as if her dad is Depression
still with her. Acute stress reaction
Agoraphobia with panic disorder
Post­traumatic stress
Check
Conversion disorder
Adjustment disorder

Case 1 describes panic disorder and agoraphobia. Case 2 is a clear example of social
phobia. In question 3, normal grief ­ not post­traumatic stress ­ is described. Unlike
depression, normal but intense grief is characterized by the absence of shame or significant
guilt. According to Ng (2005), "guilt, if it exists, is that of omission (what the bereaved person
may have failed to do) rather than commission (the recognition of malevolent intent)". Grief
often fluctuates, and, even in intense grief, distraction is often possible, and pleasant, happy
memories are intermixed with more painful ones. Although life may seem empty, there is often
a cognitive awareness of the experience as a time­limited process. Suicidal thoughts are
passive when they exist; less intense and suicidal acts are rare in the grieving person. 
(Ref: Am J Psychiatry 152:1, January 1995 and
http://www.ncbi.nlm.nih.gov/pubmed/16021224).
The correct answer is: A 40­year­old woman was robbed in an underground station five years
ago. Since then, she is fearful to leave her flat. When she goes to supermarkets, she freezes
and feels that she might even die. – Agoraphobia with panic disorder, A 23­year­old woman
works in a warehouse. She is very anxious to speak even to her colleagues and boss. She
does not socialise, never been to a pub and isolates herself. She is worried about causing
embarrassment when talking in front of others – Social phobia, A 26­year­old woman lost her
dad in a car accident three weeks ago. She has not taken the car since this incident and
doesn't want to use the same route. She feels as if her dad is still with her. – Normal grief
reaction

Question 4 HiY Assessment EMI054
Not answered Rating scales ­ Properties
Which of the listed items describes the attributes of the scale that you are testing?
Marked out of 4.00

Flag question

1560
You ask patients to complete an Concurrent validity
anxiety rating scale six months apart
Concussion validity
Predictive validity
Criterion validity
Content validity
Ecological validity
Test re­test reliability
Internal consistency reliability
Interrater reliability
Criterion validity
You look back at all your patients Interrater reliability
whom you have treated for Test re­test reliability
depression to see if their baseline
Content validity
rating scales are related to their
Internal consistency reliability
current mental state
Ecological validity
Concussion validity
Predictive validity
Concussion validity
You asked a set of nurses, doctors Concurrent validity
Interrater reliability
and former patients to give their Internal consistency reliability
opinion as to whether you are asking
Ecological validity
the appropriate questions about
levels of satisfaction on the ward
Predictive validity
Concurrent validity
Test re­test reliability
Content validity
Internal consistency reliability
You are administering a putative test Criterion validity
Content validity
of intelligence and WAIS (Weschler Predictive validity
adult intelligence scale) at the same
Interrater reliability
time, and the correlation between the
two is computed
Concussion validity
Concurrent validity
Criterion validity
Test re­test reliability
Check
Ecological validity

Test­retest correlation to assess reliability involves administering an instrument twice to the
same population. 
Predictive validity refers to the ability of a test to predict future group differences according to
current group differences in the score. e.g., high aggression score in childhood and high
criminal incidents in adult life.
Content validity refers to whether the contents i.e. each subscale, items or elements of the
test are in line with the general objectives or specifications the test was originally designed to
measure. This often cannot be statistically tested, but experts are called for comments on this
aspect of validity. 
Convergent validity refers to an agreement between instruments that measure same
construct e.g. between BDI and HAMD for depression. This agreement can be tested in
contrasted groups i.e. depressed and non­depressed, both groups showing a high correlation
between the two scales. Sometimes this is referred to as concurrent validity in a broader
sense.
The correct answer is: You ask patients to complete an anxiety rating scale six months apart –
Test re­test reliability, You look back at all your patients whom you have treated for depression
to see if their baseline rating scales are related to their current mental state – Predictive

1561
validity, You asked a set of nurses, doctors and former patients to give their opinion as to
whether you are asking the appropriate questions about levels of satisfaction on the ward –
Content validity, You are administering a putative test of intelligence and WAIS (Weschler
adult intelligence scale) at the same time, and the correlation between the two is computed –
Concurrent validity

Question 5 HiY Assessment EMI055
Not answered Sleep disturbances
Choose one option for each of the following descriptions
Marked out of 4.00

Flag question
This is a type of sleep disorder Sleep apnoea
featured by stereotyped, repetitive Recurrent hypersomnia
movements usually involving the Advanced sleep phase syndrome
head and neck. It occurs immediately Non 24 hour sleep wake disorder
prior to sleep which are sustained Circadian rhythm disorder­not otherwise specified
into light sleep Shift work sleep disorder
Hypnic jerks
Klein Levin syndrome
Klein Levin syndrome
This is a type of sleep disorder in
Rhythmic movement disorder
which the individuals have more than Non 24 hour sleep wake disorder
24 hour sleep­wake period, which
Narcolepsy syndrome
Circadian rhythm disorder­not otherwise specified
leads to a steady pattern of 1­2 hour Hypnic jerks
daily delays in sleep onset and wake Narcolepsy syndrome
times Advanced sleep phase syndrome
Sleep apnoea
Recurrent hypersomnia
Circadian rhythm disorder­not otherwise specified
Mr. A's partner is concerned that he
Rhythmic movement disorder
Non 24 hour sleep wake disorder
presents with sudden abrupt
contractions of muscle groups mainly Shift work sleep disorder
Rhythmic movement disorder
in his legs but sometimes involves Sleep apnoea
the arms, which occurs mainly at Advanced sleep phase syndrome
sleep onset Narcolepsy syndrome
Hypnic jerks
Shift work sleep disorder
Shift work sleep disorder
Recurrent hypersomnia
Recurrent hypersomnia
Mr. A is complaining of evening Klein Levin syndrome
Circadian rhythm disorder­not otherwise specified
sleepiness, early sleep onset by 7 pm
and early morning wakening
Advanced sleep phase syndrome
Sleep apnoea
Non 24 hour sleep wake disorder
Rhythmic movement disorder
Narcolepsy syndrome
Check Hypnic jerks
Klein Levin syndrome

Rhythmic movement disorder (RMD) is characterized by recurrent, well­stereotyped episodes
of rhythmic motor activity when drowsy or having light NREM sleep. The movements may
involve the head, trunk, and limbs either alone or in combination at a typical frequency of 0.5
to 2 Hz (Fig. 1). Several distinct variations have been described, including head banging
(jactatio capitis nocturna) and body rocking, in which affected children rise upon the hands
and knees while vigorously rocking the entire body in an anteroposterior direction. 

1562
Non­24­Hour Sleep­Wake Disorder (N24HSWD) refers to a lack the light sensitivity that is
required to reset the biological rhythm. Consequently, patients with this disorder experience
cyclic insomnia and sleep deprivation that affects their concentration and memory, leading to
an elevated risk of errors and accidents. It was initially observed in individuals who are blind
but later reported in fully sighted individuals as well. 
Hypnic jerk or myoclonus is an involuntary muscle spasm that happens as people fall asleep. 
Advanced sleep phase syndrome is "a disorder in which the major sleep episode is advanced
in relation to the desired clock time, which results in symptoms of compelling evening
sleepiness, an early sleep onset, and an awakening that is earlier than desired". [Advanced
Sleep Phase Syndrome ­ Stanford University. (n.d.). Retrieved from
http://web.stanford.edu/~dement/advanced.html]
The correct answer is: This is a type of sleep disorder featured by stereotyped, repetitive
movements usually involving the head and neck. It occurs immediately prior to sleep which
are sustained into light sleep – Rhythmic movement disorder, This is a type of sleep disorder
in which the individuals have more than 24 hour sleep­wake period, which leads to a steady
pattern of 1­2 hour daily delays in sleep onset and wake times – Non 24 hour sleep wake
disorder, Mr. A's partner is concerned that he presents with sudden abrupt contractions of
muscle groups mainly in his legs but sometimes involves the arms, which occurs mainly at
sleep onset – Hypnic jerks, Mr. A is complaining of evening sleepiness, early sleep onset by 7
pm and early morning wakening – Advanced sleep phase syndrome

Question 6 HiY Assessment EMI056
Not answered Motor symptoms of schizophrenia
Identify the terms used to describe the following presentations;
Marked out of 4.00

Flag question
Waxy flexibility
While examining a 45­year­old man Mannerism
with schizophrenia, the psychiatrist Automatic obedience
places the patient's arms in a folded
Stereotypy
position which the patient then holds
Psychological pillow
in the same position for a long period
Cataplexy
Gegenhalten
Mitgehen
Psychological pillow
While examining a 36­year­old man, Catalepsy
Waxy flexibility
he resists passive movement with Mitgehen
same degree of force as applied by Cataplexy
the examiner Mannerism
Stereotypy
Gegenhalten
Catalepsy
Cataplexy
Automatic obedience
Mitgehen
While examining a 30­year­old man Psychological pillow
with schizophrenia, his limbs may be
Stereotypy
moved with minimal pressure
Waxy flexibility
Gegenhalten
Catalepsy
Automatic obedience
Mannerism

1563
A 43­year­old man in a rehabilitation
Cataplexy
unit was observed to be saluting
Mitgehen
passers­by all the time
Gegenhalten
Mannerism
Waxy flexibility
Automatic obedience
Catalepsy
Psychological pillow
Check
Stereotypy

In waxy flexibility, the examiner can position the patient to assume very awkward postures,
which would be maintained by the patient for a considerable period. 
Patients with gegenhalten resist passive movement of the limbs in a manner proportionate to
the force used. The patient deliberately opposes the examiner's attempts to move or
manipulate. 
In Mitgehen (in German, 'to go with'), the interviewer can manipulate the patient's limbs with
slightest pressure.
Performing trivial tasks in a manner that is idiosyncratic to an individual is called mannerism
(e.g, a bending one's arm in a peculiar way to scratch one's ears).

Catatonia Clinical Presentation ­ Medscape Reference. (n.d.). Retrieved from
http://emedicine.medscape.com/article/1154851­clinical
The correct answer is: While examining a 45­year­old man with schizophrenia, the psychiatrist
places the patient's arms in a folded position which the patient then holds in the same position
for a long period – Waxy flexibility, While examining a 36­year­old man, he resists passive
movement with same degree of force as applied by the examiner – Gegenhalten, While
examining a 30­year­old man with schizophrenia, his limbs may be moved with minimal
pressure – Mitgehen, A 43­year­old man in a rehabilitation unit was observed to be saluting
passers­by all the time – Mannerism

Question 7 HiY Assessment EMI057
Not answered Clinical features and psychopathology
Identify the type of psychopathology in each of the following situations;
Marked out of 5.00

Flag question A 46­year­old woman has been
admitted to the psychiatric ward. This No delusions
is her first admission. She has low Capgras delusion
self­confidence and is reported to be
Delusions of persecution
withdrawn. She worries that she has
Delusions of infidelity
no money. She states that she
wouldn't be surprised if her boyfriend
Delusions of reference
is having an affair at all, as she is not Delusions of grandiosity
attractive Fregoli delusion
Nihilistic delusions
A 42­year­old man was admitted with Delusional mood
a relapse of his psychotic illness. He
tells that there are hidden television

1564
cameras in his room, and he has
Fregoli delusion
seen the full interview being
Delusions of reference
broadcast on the television
Delusions of grandiosity
Delusional mood
Capgras delusion
No delusions
Delusions of infidelity
A 45­year­old man with chronic Delusions of persecution
schizophrenia believes that his Nihilistic delusions
neighbour can take on different
guises. He claims that the London
No delusions
Metropolitan police are suspecting Fregoli delusion
him of initiating riots in London and Delusions of reference
following him all the time. He Nihilistic delusions
believes that his neighbour has Delusions of grandiosity
played a crucial role in disseminating Delusions of persecution
this information on the Television, as Delusions of infidelity
there have been constant references
Capgras delusion
to riots. (Choose three answers)
Delusional mood

Check

Most of the beliefs expressed by patient 1 may be true and are not expressed with absolute
conviction and distress. They are unlikely to be delusions. The belief regarding television
cameras is a delusion of reference. 
Fregoli delusion is a misidentification phenomenon in which unknown person are
inappropriately identified as being familiar. 
The correct answer is: A 46­year­old woman has been admitted to the psychiatric ward. This is
her first admission. She has low self­confidence and is reported to be withdrawn. She worries
that she has no money. She states that she wouldn't be surprised if her boyfriend is having an
affair at all, as she is not attractive – No delusions, A 42­year­old man was admitted with a
relapse of his psychotic illness. He tells that there are hidden television cameras in his room,
and he has seen the full interview being broadcast on the television – Delusions of reference,
A 45­year­old man with chronic schizophrenia believes that his neighbour can take on
different guises. He claims that the London Metropolitan police are suspecting him of initiating
riots in London and following him all the time. He believes that his neighbour has played a
crucial role in disseminating this information on the Television, as there have been constant
references to riots. (Choose three answers) – Fregoli delusion, Delusions of persecution,
Delusions of reference

Question 8 HiY Assessment EMI058
Not answered Research measures in psychiatry 
For each of the following descriptions choose the appropriate instrument from the list
Marked out of 4.00
provided.
Flag question

1565
A scale used as a measure of
MADRS
psychiatric caseness in research
Clinical Interview Schedule
studies
CAGE
SF­12
Schedule for Assessment in Neuropsychiatry
MMSE
Diagnostic interview schedule
GHQ­12
Geriatric Mental Score
Geriatric Mental Score
SF­12
A scale used as a measure of CAGE
depression in research studies Schedule for Assessment in Neuropsychiatry
GHQ­12
Diagnostic interview schedule
MADRS
MMSE
Diagnostic interview schedule
Clinical Interview Schedule
SF­12
A scale used as a screening CAGE
instrument for alcoholism Schedule for Assessment in Neuropsychiatry
MADRS
Clinical Interview Schedule
Geriatric Mental Score
GHQ­12
SF­12
MMSE
A scale used as fully structured, non­ Schedule for Assessment in Neuropsychiatry
clinician administered tool for making MADRS
lifetime diagnoses in epidemiological Clinical Interview Schedule
studies. Geriatric Mental Score
Diagnostic interview schedule
GHQ­12
CAGE
Check
MMSE

The General Health Questionnaire (GHQ) screens for non­psychotic psychiatric disorders and
is available in a number of versions. GHQ 60, 30, 28 and 12. GHQ­12 is a quick­to­use shorter
form, ideal for research studies.
The Montgomery­Asberg Depression Rating Scale (abbreviated MADRS) is a ten­item
questionnaire used to measure the severity and change in the severity of depression over
time. 
The CAGE questionnaire is a widely used method of screening for alcoholism. It is a 4 item
scale, with each item scored 1. A CAGE test scores >=2 is thought to have a sensitivity of 93%
and a specificity of 76% for the identification of problem drinkers.
The Diagnostic Interview Schedule is a structured interview schedule that covers more than
30 mental disorders and uses specially trained interviewers (including non ­ clinicians) to
assign a DSM diagnosis.
The correct answer is: A scale used as a measure of psychiatric caseness in research studies
– GHQ­12, A scale used as a measure of depression in research studies – MADRS, A scale
used as a screening instrument for alcoholism – CAGE, A scale used as fully structured, non­
clinician administered tool for making lifetime diagnoses in epidemiological studies. –
Diagnostic interview schedule

1566
Question 9 HiY Assessment EMI059
Not answered Scenarios related to psychodynamic defences
Choose the best option from the list provided for each description below.
Marked out of 3.00

Flag question
Isolation
A 23­year­old woman studying at a Passive­aggressive
university in Newcastle finds herself Displacement
in London without any recollection of Projection
how she got there. Dissociation
Intellectualization
Rationalization
Acting out
Isolation
A 24 yr old lady waiting for
Denial
mediastinoscopy has read everything Rationalization
about the procedure and its Introjection
Acting out
evidence. She is not concerned Dissociation
about the pain and impact of Displacement
diagnosis on her emotional state. Projection
Passive­aggressive
Soon after the death of her husband, Intellectualization
Introjection
Denial
a 48­year­old woman suddenly starts Denial
becoming quite flirtatious. She shows Introjection
Dissociation
inappropriate behaviour towards her Passive­aggressive
dead husband's friends and appears Rationalization
to have had no grief or mourning. On
Displacement
confrontation, she could not explain
her behaviour.
Intellectualization
Acting out
Projection
Check Isolation

Dissociation is the defense mechanism used here. Dissociation is temporarily but drastically
modifying one's sense of personal identity to avoid emotional distress. Fugue states and
hysterical conversion reactions are common manifestations of dissociation. 
Intellectualization is employed here. This refers to excessively using intellectual processes to
avoid affective expression or experience. When faced with a stressful situation, the use of
logic helps to focus closely on external reality and to avoid the expression of inner feelings
(e.g., fear). 
Expressing an unconscious impulse through explicit acts to avoid the unpleasant awareness
of the accompanying affect is called Acting Out. In other words, an unconscious wish is
expressed impulsively through a behavioural act, thus gratifying instead of prohibiting the
impulse.
The correct answer is: A 23­year­old woman studying at a university in Newcastle finds herself
in London without any recollection of how she got there. – Dissociation, A 24 yr old lady
waiting for mediastinoscopy has read everything about the procedure and its evidence. She is
not concerned about the pain and impact of diagnosis on her emotional state. –
Intellectualization, Soon after the death of her husband, a 48­year­old woman suddenly starts
becoming quite flirtatious. She shows inappropriate behaviour towards her dead husband's
friends and appears to have had no grief or mourning. On confrontation, she could not explain
her behaviour. – Acting out

1567
Question 10 HiY Assessment EMI060
Not answered Psychodynamic defences
Choose the best option from the list above for each description below.
Marked out of 3.00

Flag question A 42­year­old man has been
diagnosed with poor prognostic
Intellectualization
variant carcinoma of the lung. When Denial
breaking the bad news, the patient Isolation
proceeds to tell his physician how Acting out
excited he is about attending his Altruism
son's wedding anniversary two Projection
months later. Rationalization
Displacement
Rationalization
Dissociation
A mother, whose son was killed in a Intellectualization
Passive­aggressive
road rage, quits her job and joins a Passive­aggressive
voluntary organization working Acting out
towards safer driving. Projection
Dissociation
Isolation
Altruism
Denial
After having an argument with his Denial
Passive­aggressive
wife, Mark gets into an altercation Displacement
and fight at the local pub, quite
Altruism
uncharacteristic of him.
Dissociation
Rationalization
Intellectualization
Isolation
Acting out
Check Displacement
Projection

The defence mechanism used here is denial, which is the explicit refusal to acknowledge a
threatening reality. It may persist despite constant explanation of the facts. It is not same as
conscious avoidance of painful topics or thoughts. 
Altruism is using constructive and gratifying service to others to receive a vicarious
satisfaction. This does not involve giving up one's pleasures. 
The defense mechanism employed here is displacement. It is the process by which interest
and/or emotion is shifted from one object onto another less threatening, often less­retaliating
one. In this scenario, Mark who had an argument with his wife has displaced the anger felt
onto someone in the pub and gets into a fight.
The correct answer is: A 42­year­old man has been diagnosed with poor prognostic variant
carcinoma of the lung. When breaking the bad news, the patient proceeds to tell his physician
how excited he is about attending his son's wedding anniversary two months later. – Denial, A
mother, whose son was killed in a road rage, quits her job and joins a voluntary organization
working towards safer driving. – Altruism, After having an argument with his wife, Mark gets
into an altercation and fight at the local pub, quite uncharacteristic of him. – Displacement

Question 11 HiY Assessment EMI061
Not answered The Language of Psychopathology
Identify the terms commonly used in psychopathology using the definitions given in the list.

1568
Marked out of 5.00
Alexithymia
Flag question Euphoria
Pervasive emotional state prevailing Agitation
over a longer period Anhedonia
Anxiety
Ecstasy
Euthymia
Mood
Anxiety
Blunted affect
Euphoria
Inability to verbally describe Affect
Affect
emotional feelings Agitation
Euthymia
Alexithymia
Ecstasy
Anhedonia
Euphoria
Blunted affect
Alexithymia
Mood
Mood
Inability to experience pleasure
Blunted affect
Euthymia
Anhedonia
Ecstasy
Agitation
Mood
Anxiety
Agitation
Subjective and immediate
Affect
Alexithymia
experience of emotion at a given time Euthymia
Anhedonia
Euphoria
Affect
Ecstasy
Mood
Anxiety
Euthymia
Blunted affect
Anxiety
Normal range of mood, with absence
of depressed or elevated mood Blunted affect
Affect
Euphoria
Anhedonia
Agitation
Alexithymia
Check Ecstasy

Mood is the pervasive feeling tone, which is sustained and colours the total experience of the
person. It is an emotional state persisting over a longer period than affect. It is subjectively
experienced and reported by the patient and can be observed by others. 
Alexithymia is the inability to verbally describe emotionally feelings or being aware of
emotions or mood. It is especially seen in psychosomatic illnesses, somatoform disorders,
depression, PTSD, personality disorders and paraphilias. 
Anhedonia is inability to experience pleasure and is one of the core diagnostic features of
depression; it is also one of the various negative symptoms of psychosis. 
Affect is subjective and immediate (cross­sectional) experience of emotion at a given time. 

1569
Euthymia is the normal range of mood, with an absence of depressed or elevated mood.
The correct answer is: Pervasive emotional state prevailing over a longer period – Mood,
Inability to verbally describe emotional feelings – Alexithymia, Inability to experience pleasure
– Anhedonia, Subjective and immediate experience of emotion at a given time – Affect,
Normal range of mood, with absence of depressed or elevated mood – Euthymia

Question 12 HiY Assessment EMI062
Not answered Affective disturbances 
Identify the terms commonly used in psychopathology using the definitions listed.
Marked out of 4.00

Flag question
Alexithymia
Incongruent Affect
Severe reduction in the intensity of Euthymia
affect Labile Affect
Elation
Agitation
Blunted affect
Anhedonia
Blunted affect
Euphoria
Alexithymia
Rapid and abrupt changes in affect Anxiety
Agitation
unrelated to external stimuli Anhedonia
Euphoria
Elation
Incongruent Affect
Labile Affect
Agitation
Anxiety
Anhedonia
Objective impression that the Euthymia
Euphoria
displayed affect is not consistent with
Anxiety
the current thoughts or actions
Labile Affect
Blunted affect
Euthymia
Alexithymia
Agitation
Incongruent Affect
Elation
An unpleasant emotional state
Elation
Alexithymia
associated with psychophysiological
changes in response to an Euthymia
intrapsychic conflict Blunted affect
Anhedonia
Anxiety
Labile Affect
Incongruent Affect
Euphoria
Check

Blunted affect refers to a severe reduction in the intensity of affect. If the reactivity is
conspicuously absent, then this is called blunted affect or parathymia, according to Bleuler.
Bleuler considered blunted affect as a primary schizophrenic symptom. 

1570
Labile affect refers to the rapid and abrupt changes in affect that occurs unrelated to external
stimuli. The absence of stability manifests as a sudden, unprovoked change in affect; the
patient may break down to tears for no reason or appear enlightened with apparently no
environmental cues. This lability is seen in histrionic personality, borderline personality, and
sometimes in PTSD. 
One expects affect to be reactive to cues in the environment; we laugh on hearing a joke,
blush when embarrassed, etc. The incongruity of affect refers to an objective impression that
the displayed affect is not consistent with the current thoughts or actions. Incongruent affect
may be seen in hebephrenic schizophrenia and learning disability. For example, a patient
might maintain a silly, jocular affect in spite of hearing bad news. 
Anxiety is an unpleasant emotional state associated with psychophysiological changes in
response to an intrapsychic conflict
The correct answer is: Severe reduction in the intensity of affect – Blunted affect, Rapid and
abrupt changes in affect unrelated to external stimuli – Labile Affect, Objective impression that
the displayed affect is not consistent with the current thoughts or actions – Incongruent Affect,
An unpleasant emotional state associated with psychophysiological changes in response to
an intrapsychic conflict – Anxiety

Question 13 HiY Assessment EMI063
Not answered Disturbances of attention
Choose the most appropriate term that illustrates each of the following disturbances of
Marked out of 5.00
attention
Flag question

Sundowning
Selective inattention
Narrowly focused attention and Coma
altered consciousness seen in
Somnolence
dissociative states
Trance
Twilight state
Distractibility
Hypervigilance
Distractibility
Twilight state
Trance
Abnormal drowsiness
Selective inattention
Hypervigilance
Somnolence
Sundowning
Coma
Twilight state
Sundowning
Drowsiness, confusion and falls at
night time in older people often with
Coma
reduced cognitive reserve Somnolence
Trance
Hypervigilance
Selective inattention
Distractibility

Disturbed consciousness with
hallucinations

1571
Distractibility
Sundowning
Somnolence
Twilight state
Coma
Trance
Selective inattention
Hypervigilance
Sundowning
Selective inattention
Excessive attention to all internal and Distractibility
external stimuli Coma
Trance
Hypervigilance
Somnolence
Twilight state
Check

Trance refers to the focused attention and altered consciousness seen in dissociative states,
hypnosis and ecstatic experiences. Somnolence is same as abnormal drowsiness.
Sundowning is a syndrome in older people which occurs at night time and is characterised by
drowsiness, confusion and falls at night time in older people often as a result of being overly
sedated with medications.Twilight state refers to disturbed consciousness with hallucinations
seen in organic or drug induced states. Hypervigilance is excessive attention and focuses on
all internal and external stimuli, usually seen secondary to delusional or paranoid states. (Ref:
Kaplan & Sadock Synopsis of Psychiatry­ 8th edition; pg 84)
The correct answer is: Narrowly focused attention and altered consciousness seen in
dissociative states – Trance, Abnormal drowsiness – Somnolence, Drowsiness, confusion
and falls at night time in older people often with reduced cognitive reserve – Sundowning,
Disturbed consciousness with hallucinations – Twilight state, Excessive attention to all
internal and external stimuli – Hypervigilance

Question 14 HiY Assessment EMI064
Not answered Phobias
Identify the terms used for different specific phobias
Marked out of 5.00

Flag question
Cynophobia
Ophidiophobia
Agoraphobia
Fear of open spaces
Thalassophobia
Claustrophobia
Amathophobia
Acrophobia
Triskaidekaphobia
Gynophobia
Odynophobia

Fear of enclosed spaces

1572
Triskaidekaphobia
Odynophobia
Claustrophobia
Gynophobia
Agoraphobia
Acrophobia
Cynophobia
Thalassophobia
Agoraphobia
Ophidiophobia
Cynophobia
Amathophobia
Acrophobia
Fear of heights
Ophidiophobia
Triskaidekaphobia
Gynophobia
Amathophobia
Thalassophobia
Ophidiophobia
Odynophobia
Cynophobia
Claustrophobia
Amathophobia
Fear of snakes
Acrophobia
Agoraphobia
Thalassophobia
Odynophobia
Claustrophobia
Triskaidekaphobia
Gynophobia
Ophidiophobia
Triskaidekaphobia
Gynophobia
Fear of number thirteen
Cynophobia
Acrophobia
Claustrophobia
Odynophobia
Thalassophobia
Check
Amathophobia
Agoraphobia

Fear of open spaces­ agoraphobia.Fear of enclosed spaces­ claustrophobia.Fear of heights­
acrophobia.Fear of snakes­ Ophidiophobia.Fear of number thirteen­ Triskaidekaphobia. Other
common specific phobias include Amathophobia­ fear of dust, Astrapophobia­ fear of
lightning, Odynophobia­ fear of pain, Cynophobia­ fear of dogs and Arachnophobia­ fear of
spiders.
The correct answer is: Fear of open spaces – Agoraphobia, Fear of enclosed spaces –
Claustrophobia, Fear of heights – Acrophobia, Fear of snakes – Ophidiophobia, Fear of
number thirteen – Triskaidekaphobia

Question 15 HiY Assessment EMI065
Not answered Pathology of familiarity
Identify the terms used to describe each of the following
Marked out of 4.00

Flag question

1573
Feeling of unfamiliarity with a Derealization
previously experienced situation Déjà vu
Déjà entendu
Déjà vecu
Jamais vu
Derailment
Depersonalization
Confabulation
Déjà pensee
Déjà pensee
Depersonalization
Anterograde amnesia
Derealization
Illusion of auditory recognition
Déjà entendu
Déjà vu
Derailment
Anterograde amnesia
Déjà vecu
Confabulation
Confabulation
Déjà pensee
Regarding a new situation as a
Jamais vu
Déjà vu
repetition of a previous experience Anterograde amnesia
Depersonalization
Déjà vecu
Derealization
Derailment
Jamais vu
Jamais vu
Déjà vecu
Regarding a new thought as a
Déjà entendu
Confabulation
repetition of a previous thought Anterograde amnesia
Déjà pensee
Déjà vu
Derealization
Déjà entendu
Check Derailment
Depersonalization

Jamais vu is a feeling of strangeness while attending an environment to which one has been
exposed in the past. In other words, a well­known place all of a sudden stops providing the
feelings of familiarity. It can occur in normal people, and also can occur pathologically in
Temporal Lobe Epilepsy. 
Déjà entendu is a pathological familiarity for someone's voice. 
Déjà vu is the feeling of having seen or experienced an event, which is in fact being
experienced for the first time. Reports of déjà vu are more frequent in temporal lobe (TL)
epileptics, with some association also noted in psychotic disorders. 
Déjà pensee refers to the experience of pathological familiarity towards a thought or idea.
The correct answer is: Feeling of unfamiliarity with a previously experienced situation –
Jamais vu, Illusion of auditory recognition – Déjà entendu, Regarding a new situation as a
repetition of a previous experience – Déjà vu, Regarding a new thought as a repetition of a
previous thought – Déjà pensee

Question 16 HiY Assessment EMI066
Not answered

1574
Marked out of 4.00 Content of delusions
Flag question
Identify the type of delusions for each of the examples given below

Folie a deux
De Clerambault's delusion
A 46­year­old depressed woman Fregoli delusion
says 'I am dead.' Paranoid delusions
Capgras delusion
Othello syndrome
Mirrored self­identification
Ekbom's syndrome
Capgras delusion
Autochthonous delusions
Paranoid delusions
A 76­year­old woman with dementia,
on meeting her husband says 'You Cotard delusion
Cotard delusion
are not my husband. You are an Othello syndrome
impostor who looks like him.' Mirrored self­identification
Autochthonous delusions
Folie a deux
De Clerambault's delusion
Othello syndrome
Fregoli delusion
De Clerambault's delusion
Ekbom's syndrome
Ekbom's syndrome
A 66­year­old man with psychosis
mentions, 'I am constantly followed by Capgras delusion
people I know, but I can't recognise Fregoli delusion
them as they are always in disguise.' Paranoid delusions
Autochthonous delusions
Cotard delusion
Mirrored self­identification
Folie a deux
De Clerambault's delusion
A 47­year­old depressed alcoholic is Ekbom's syndrome
agitated and is convinced that his Fregoli delusion
wife is having an affair with his Paranoid delusions
neighbour. Cotard delusion
Folie a deux
Othello syndrome
Capgras delusion
Check Autochthonous delusions
Mirrored self­identification

The diagnosis for case 1 is Cotard's syndrome. It occurs in severe depression with nihilistic
and hypochondriacal delusions tinged with a negative attitude. Cotard's syndrome is seen in
schizophrenia and depressive psychosis in the elderly. 
Case 2 refers to a Capgras delusion. Here a person believes that a person usually close to
him has been replaced by an exact double. Capgras syndrome is sometimes referred to as
the illusion of doubles though it is a delusion. It is now thought to be mostly due to organic
brain damage (>50%, Lishman) apart from being seen as a part of schizophrenia or isolated
delusional disorder including brain injury and schizophrenia. 
Case 3 fits the description of a Fregoli delusion where a false identification of familiar persons
occurs on meeting strangers. As a result, a familiar person is thought to be taking various
disguises. 

1575
Othello syndrome is also called as pathological jealousy. It can occur in various forms ­
delusion, overvalued idea, in depression and anxiety states; it is not a misidentification
syndrome. It is common in alcoholics and has a potential of violence.
The correct answer is: A 46­year­old depressed woman says 'I am dead.' – Cotard delusion, A
76­year­old woman with dementia, on meeting her husband says 'You are not my husband.
You are an impostor who looks like him.' – Capgras delusion, A 66­year­old man with
psychosis mentions, 'I am constantly followed by people I know, but I can't recognise them as
they are always in disguise.' – Fregoli delusion, A 47­year­old depressed alcoholic is agitated
and is convinced that his wife is having an affair with his neighbour. – Othello syndrome

Question 17 HiY Assessment EMI067
Not answered Examples of formal thought disorders
Using the descriptions given below, identify the correct psychiatric terms used to describe
Marked out of 3.00
them from the list shown.
Flag question

Incoherence (word salad)
Perseveration
A 43­year­old lady with bipolar Echolalia
disorder says 'I ate my food, mood,
Clanging
rude, nude.'
Derailment
Neologisms
Circumstantiality
Tangentiality
Flight of ideas
Flight of ideas
Tangentiality
A 33­year­old schizophrenic says 'All Incoherence (word salad)
is nothing and under nothing twists.' Derailment
Perseveration
Echolalia
Clanging
Neologisms
Incoherence (word salad)
Circumstantiality
Derailment
A 19­year­old gentleman with Perseveration
paranoid psychosis says ' I have a
Neologisms
helopantic under my foot.'
Circumstantiality
Flight of ideas
Clanging
Tangentiality
Check Echolalia

Clanging: A person with flight of ideas may associate words based on the sound similarity
rather than the meaning or logical sequence of the idea. The words used have no logical
connections and may include examples of rhyming and punning in addition to clanging. 
Incoherence (word salad) is an extreme form of schizophrenic thought disorder ­ patients
speak spontaneously and remains responsive to queries, but no connection could be made
between sequential words and phrases used. It is also called schizophasia. 
Neologisms are new words created by the patient, often by combining syllables of other
words, for idiosyncratic psychological reasons. (Ref: Kaplan & Sadock Synopsis of Psychiatry­
8th edition; pg 85)

1576
The correct answer is: A 43­year­old lady with bipolar disorder says 'I ate my food, mood,
rude, nude.' – Clanging, A 33­year­old schizophrenic says 'All is nothing and under nothing
twists.' – Incoherence (word salad), A 19­year­old gentleman with paranoid psychosis says ' I
have a helopantic under my foot.' – Neologisms

Question 18 HiY Assessment EMI068
Not answered Motor behaviour
Choose the name of the motor phenomenon from the explanation given below:
Marked out of 5.00

Flag question
Catatonia
Akathisia
Pathological imitation of movements Waxy flexibility
of one person by another Mutism
Mannerism
Automatic obedience
Negativism
Stereotypy
Akathisia
Echopraxia
Waxy flexibility
Increased resting muscle tone, not Echolalia
Negativism
present during active or passive
Catatonia
movements
Mannerism
Echolalia
Echopraxia
Mutism
Waxy flexibility
Automatic obedience
Catatonia
Ingrained habitual voluntary Stereotypy
Mannerism
movement which appear to be goal­
Stereotypy
directed
Mutism
Echolalia
Echopraxia
Negativism
Negativism
Automatic obedience
Echopraxia
Akathisia
Akathisia
Automatic obedience
Subjective feeling of muscular Catatonia
tension secondary to antipsychotic
Stereotypy
use
Mutism
Waxy flexibility
Mannerism
Echolalia

Echopraxia
Catatonia
Repetitive, fixed pattern of physical Mutism
action which is not goal directed Automatic obedience
Akathisia
Stereotypy
Echolalia
Waxy flexibility
Mannerism

1577
Check

Echopraxia is the pathological mimicking of examiner's movements. Echolalia is mimicking of
examiner's speech. This is seen in catatonia, in Latah (a culture­bound syndrome) and also in
Tourette's syndrome. 
Catatonia is defined as rigidity during involuntary movements while movement on volition is
carried out normally. It is increased resting muscle tone, not present during active or passive
movements (in contrast to rigidity present in Parkinson's disease and extrapyramidal side
effects) Note that in neurological spasticity the tone is increased irrespective of passive or
active movements. A patient with catatonia can use the affected limb or muscle group when
needed with completely normal tone ­ for example, running out when there is a fire. Catatonia
persists in sleep and can continue for weeks without improvement. 
Mannerisms are odd, but purposeful movements (hopping, saluting passers­by or other
mundane movements). They are different from stereotypes as mannerisms appear as goal­
directed movements. 
Stereotypies are non­goal directed motor activity (e.g., spinning one's hands, repeated
touching, patting, rubbing self). These are seen in catatonia and also in pervasive
developmental disorder and severe learning disabilities. 
Akathisia is the subjective feeling of muscular tension experienced by patients taking
antipsychotics; it is associated with restlessness, pacing and repeated sitting & standing.
The correct answer is: Pathological imitation of movements of one person by another –
Echopraxia, Increased resting muscle tone, not present during active or passive movements –
Catatonia, Ingrained habitual voluntary movement which appear to be goal­directed –
Mannerism, Subjective feeling of muscular tension secondary to antipsychotic use –
Akathisia, Repetitive, fixed pattern of physical action which is not goal directed – Stereotypy

Question 19 HiY Assessment EMI069
Not answered Perceptual errors in psychiatric disorders
Which of the listed descriptive psychopathological terms refers to the following symptoms?
Marked out of 5.00

Flag question
Pareidolic illusion
A 43­year­old lady with psychotic Haptic hallucinations.
depression mentions 'I hear the voice Charles Bonnet syndrome
of my long dead father, as if he were
Extracampine hallucinations
talking to me now, when I hear water
running from the bath tap'
Complete illusion
Hygric hallucinations
Synaesthesia
Reflex hallucination
Charles Bonnet syndrome
Functional hallucinations
A 48­year­old schizophrenic inpatient Extracampine hallucinations
says that he can hear his fathers'
Gedankenlautwerden
Functional hallucinations
voice who is talking from 20 miles Gedankenlautwerden
from the ward' Reflex hallucination
Synaesthesia
Hygric hallucinations
Complete illusion
Haptic hallucinations.
Pareidolic illusion

1578
A 22­year­old woman with Pareidolic illusion
schizophrenia describes the
Complete illusion
sensation that somebody is touching
her body in intimate places. Reflex hallucination
Haptic hallucinations.
Synaesthesia
Gedankenlautwerden
Functional hallucinations
Hygric hallucinations
Extracampine hallucinations
Charles Bonnet syndrome
Hygric hallucinations
A 45­year­old schizophrenic lady Extracampine hallucinations
Synaesthesia
says that she sees red flames when
Reflex hallucination
she hears a song
Pareidolic illusion
Complete illusion
Charles Bonnet syndrome
Functional hallucinations
Gedankenlautwerden
Gedankenlautwerden
Extracampine hallucinations
A 47­year­old woman with Haptic hallucinations.
Hygric hallucinations
schizophrenia says that she is Complete illusion
hearing her own thoughts aloud
Reflex hallucination
Functional hallucinations
Pareidolic illusion
Synaesthesia
Check Haptic hallucinations.
Charles Bonnet syndrome

Functional hallucinations: An external stimulus provokes hallucination, and both hallucination
and stimulus are in same modality but separately perceived. e.g voices heard whenever the
noise of water running through the tap is heard.
Extracampine hallucinations occur outside the normal field of perception e.g. images seen
behind your back, under your sternum, etc. They occur in schizophrenia and epilepsy. 
Haptic hallucination (Touch) is a type of superficial somatic hallucination. 
When hallucinations in one modality are provoked reflexively by a stimulus in another
modality. e.g., seeing an angel or flames whenever listening to music, the term reflex
hallucination is used. The scenario in the question is unlikely to be synaesthesia as both
precepts are experienced (it is not the song/music that is experienced as colour red, but after
the song is heard, the flames are perceived ­ therefore there is no synchronous occurrence). 
One special type of hallucination, which is characteristic of schizophrenia, is hearing one's
own thoughts being spoken aloud. This is known as 'Gedankenlautwerden' in German and
'Echo de la pense' in French.
The correct answer is: A 43­year­old lady with psychotic depression mentions 'I hear the voice
of my long dead father, as if he were talking to me now, when I hear water running from the
bath tap' – Functional hallucinations, A 48­year­old schizophrenic inpatient says that he can
hear his fathers' voice who is talking from 20 miles from the ward' – Extracampine
hallucinations, A 22­year­old woman with schizophrenia describes the sensation that
somebody is touching her body in intimate places. – Haptic hallucinations., A 45­year­old
schizophrenic lady says that she sees red flames when she hears a song – Reflex
hallucination, A 47­year­old woman with schizophrenia says that she is hearing her own
thoughts aloud – Gedankenlautwerden

1579
Question 20 HiY Assessment EMI070
Not answered Illusions
Which of the listed descriptive psychopathological terms refers to the following symptoms?
Marked out of 3.00

Flag question
Synaesthesia
Affect illusion
A 48­year­old man says that he can Reflex hallucination
see a ghost's picture on his carpet
Complete illusion
that has plenty of cigarette burns
Kinaesthetic hallucination
Hypnagogic hallucination
Pareidolic illusion
Thermic hallucinations
Complete illusion
Functional hallucinations
Thermic hallucinations
A 47­year­old depressed woman is
Hypnapompic hallucination
Hypnagogic hallucination
reading the word 'deed' as 'dead.' Synaesthesia
Kinaesthetic hallucination
Reflex hallucination
Hypnapompic hallucination
Affect illusion
Hypnagogic hallucination
Pareidolic illusion
Complete illusion
A 26­year­old man hurrying to post Functional hallucinations
Hypnapompic hallucination
office reads 'LOCK' as 'where Reflex hallucination
Kinaesthetic hallucination
Thermic hallucinations
Affect illusion
Pareidolic illusion
Check Functional hallucinations
Synaesthesia

Explanations: In pareidolic illusions, the objects are formed from ambiguous stimuli, coloured
by prevailing emotion; not entirely due to inattention or affective change. On paying extra
effort, the object intensifies and does not disappear.In affect illusion, the prevailing emotional
state leads to misperceptions. It disappears on focussing the object with additional
concentration. In completion illusions, the stimulus that does not form a complete object might
be perceived to be complete, due to inattention. As a rule, this illusion disappears on
concentration.
The correct answer is: A 48­year­old man says that he can see a ghost's picture on his carpet
that has plenty of cigarette burns – Pareidolic illusion, A 47­year­old depressed woman is
reading the word 'deed' as 'dead.' – Affect illusion, A 26­year­old man hurrying to post office
reads 'LOCK' as 'where – Complete illusion

Question 21 HiY Assessment EMI071
Not answered Disorders of volition/action
Which of the listed descriptive psychopathological terms refers to the following symptoms?
Marked out of 4.00

Flag question

1580
A patient says 'someone is using Made impulse
telepathy to make my hands tremble'
Delusional misrepresentation
though no obvious tremors are noted.
Delusional perception
Made affect
Somatic passivity
Somatic hallucination
Compulsion
Referential delusion
Made affect
Made volition
Delusional misrepresentation
'I know it's wrong to do it, and in fact I Made impulse
do not want to do it. However, despite
Somatic hallucination
my resistance, they force me to do it.'
Referential delusion
Made volition
Somatic passivity
Compulsion
Compulsion
Delusional perception
Delusional misrepresentation
'When I heard the station master's Made impulse
whistle, I came to know that there is a
Somatic passivity
plot to finish me off.'
Delusional perception
Made volition
Referential delusion
Somatic hallucination
Made impulse
Made affect
Made volition
A patient says 'I feel unpleasant due Made affect
to insects crawling under my skin.' Compulsion
Somatic hallucination
Delusional perception
Somatic passivity
Delusional misrepresentation
Check Referential delusion

Case 1 describes somatic passivity, which is an experience of sensations (trembling) on the
body caused by an external agency (telepathy). It is one of the first rank symptoms of
schizophrenia. 
Case 2 describes made impulse (someone controlling the desire to act­ not completed act but
the drive. If the action has been carried out, the patient admits to ownership of act, not the
impulse behind it).
Case 3 is an example of delusional perception, which is to attribute a new meaning, usually in
the sense of self­reference, to a typically perceived object, provided that the new meaning
cannot be explained by the person's mood or disturbed perception. 
Case 4 describes a type of superficial somatic hallucinations called as formication.
Formication (formic acid ­ from ants) is a special type of haptic hallucination where an
unpleasant sensation of little animals or insects crawling under one's skin is perceived. It is
seen in delirium tremens and cocaine intoxication.
The correct answer is: A patient says 'someone is using telepathy to make my hands tremble'
though no obvious tremors are noted. – Somatic passivity, 'I know it's wrong to do it, and in fact
I do not want to do it. However, despite my resistance, they force me to do it.' – Made impulse,
'When I heard the station master's whistle, I came to know that there is a plot to finish me off.' –
Delusional perception, A patient says 'I feel unpleasant due to insects crawling under my skin.'

1581
– Somatic hallucination

Question 22 HiY Assessment EMI072
Not answered Thought disorders
Identify one or more specific disorders of thought for each category given below
Marked out of 9.00

Flag question
Crowding of thought
Tangentiality
Disorders of stream of thought Delusional perception
(Choose two answers) Poverty of speech
Thought alienation
Loosened association
Formication
Thought alienation
Crowding of thought
Disorders of the possession of Formication
thoughts Poverty of speech
Loosened association
Delusional perception
Tangentiality
Thought alienation
Tangentiality
Poverty of speech
Disorders of the content of thought
Crowding of thought
Loosened association
Formication
Delusional perception
Loosened association
Formication
Tangentiality
Poverty of speech
Thought alienation
Delusional perception
Disorders of form of thought (choose
Crowding of thought
two answers)

Check

Disorders of the stream of thought includes crowding, poverty and pressured thoughts. 
Disorders of the possession of thoughts include obsessive thoughts and thought alienation. 

1582
Disorders of the content of thoughts include primary delusions (delusional mood, delusional
perception, sudden delusional ideas), secondary delusions, suicidal thoughts, etc. 
Disorders of the form of thought include loosened associations, tangentiality, etc.
The correct answer is: Disorders of stream of thought (Choose two answers) – Crowding of
thought, Poverty of speech, Disorders of the possession of thoughts
– Thought alienation, Disorders of the content of thought
– Delusional perception, Disorders of form of thought (choose two answers) – Loosened
association, Tangentiality

Question 23 HiY Assessment EMI074
Not answered Disorders of speech
Identify the correct term that represents each of the following speech disorders
Marked out of 5.00

Flag question
Logoclonia
Aphonia
Akinetic mutism
Inability to vocalize.
Dysarthria
Elective mutism
Alogia
Palilalia
Stammering
Aphonia
Logorrhoea
Logorrhoea
Elective mutism
Disorders of articulation
Alogia
Akinetic mutism
Logoclonia
Palilalia
Dysarthria
Dysarthria
Stammering
Aphonia
The normal flow of speech is Elective mutism
interrupted by pauses or by the
Stammering
repetition of fragments of the word.
Logoclonia
Logorrhoea
Alogia
Akinetic mutism
Dysarthria
Palilalia
Alogia
Repetition of last uttered word, Elective mutism
without any apparent purpose Aphonia
Stammering
Palilalia
Logoclonia
Akinetic mutism
Elective mutism
Logorrhoea
Dysarthria
Logoclonia
Repetition of last syllable of a word
Alogia
Logorrhoea
Aphonia
Akinetic mutism
Stammering
Palilalia

1583
Check

Aphonia refers to the inability to vocalize. It refers to sound production rather than sound
manipulation ­ disturbance of the latter being dysarthria. In aphonia, whispering occurs; it may
be due to paralysed vocal cords or due to hysteria. 
Dysarthria refers to disorders of articulation; it may be due to lesions in the brain stem
(bulbar), cortex (pseudobulbar), cerebellum or extrapyramidal system. Dysarthria can also be
drug induced in schizophrenia. 
Stammering: In stammering the normal flow of speech is interrupted by pauses or by the
repetition of fragments of the word. Tics often accompany stammers. Boys stammer more often
than girls; usually reduced in adulthood.
Palilalia: Repetition of last uttered word, without any apparent purpose; seen in learning
disabled, pervasive developmental disorders and in Tourette's. 
Logoclonia: Repetition of last syllable of a word, seen in Parkinson's.
The correct answer is: Inability to vocalize. – Aphonia, Disorders of articulation – Dysarthria,
The normal flow of speech is interrupted by pauses or by the repetition of fragments of the
word. – Stammering, Repetition of last uttered word, without any apparent purpose – Palilalia,
Repetition of last syllable of a word – Logoclonia

Question 24 HiY Assessment EMI075
Not answered Aphasia 
Identify the correct terms that represent each of the following speech disorder
Marked out of 4.00

Flag question
Broca's aphasia
Pure word dumbness
The ability to speak is grossly Syntactical aphasia
impaired, but comprehension is intact Wernicke's aphasia
Pure word deafness
Coprophasia
Amnestic aphasia
Pure word blindness
Wernicke's aphasia
Nominal aphasia
Pure word dumbness
Speech is fluid and spontaneous, but Pure word deafness
there is loss of ability to comprehend
Coprophasia
the meaning of words
Syntactical aphasia
Broca's aphasia
Nominal aphasia
Amnestic aphasia
Wernicke's aphasia
Pure word blindness
Pure word deafness
Patient can speak, read & write Amnestic aphasia
fluently, but comprehension is
Nominal aphasia
impaired only for spoken language
Broca's aphasia
Pure word dumbness
Pure word blindness
Coprophasia
Syntactical aphasia

1584
The patient can speak well and Wernicke's aphasia
comprehend what is spoken; he can
Nominal aphasia
also write spontaneously and to
dictation, but reading comprehension Pure word blindness
is impaired. Broca's aphasia
Coprophasia
Pure word dumbness
Pure word deafness
Amnestic aphasia
Check Syntactical aphasia

In Broca's aphasia the speech is nonfluent; it often appears laboured with many interruptions
and pauses. Function words (prepositions, conjunctions) are the most affected though a good
degree of meaningful nouns and verbs are still produced. Abnormal order of words along with
agrammatism is noted. Speech is telegraphic. Harrison textbook of Medicine quotes the
following example: "I see...the dotor, dotor sent me...Bosson. Go to hospital. Dotor...kept me
beside. Two, tee days, doctor send me home." 
In Wernicke's aphasia, the comprehension is impaired for both spoken and written language.
Language output is fluent but is highly paraphasic, sometimes with string of neologisms and
circumlocutions. Hence, it is also termed as "jargon aphasia." The speech contains large
numbers of function words (e.g., prepositions, conjunctions) but few substantive nouns or
verbs that refer to specific actions. The output is, therefore, voluminous but uninformative.
Sometimes this is compared to the speech disturbances seen in schizophrenia. 
Pure word deafness: Patient can speak read & write fluently, but comprehension is impaired
only for spoken language. Bilateral (or left sided with disrupted connections to non­dominant
circuit) damage to the superior temporal pole is suspected. 
Pure word blindness: Here the patient can speak well, comprehend what is spoken and write
(both spontaneously and upon dictation), but displays an impairment in reading
comprehension.
The correct answer is: The ability to speak is grossly impaired, but comprehension is intact –
Broca's aphasia, Speech is fluid and spontaneous, but there is loss of ability to comprehend
the meaning of words – Wernicke's aphasia, Patient can speak, read & write fluently, but
comprehension is impaired only for spoken language – Pure word deafness, The patient can
speak well and comprehend what is spoken; he can also write spontaneously and to dictation,
but reading comprehension is impaired. – Pure word blindness

Question 25 HiY Assessment EMI076
Not answered Pioneers in psychopathology
Match each concept below with the pioneer from the list shown:
Marked out of 5.00

Flag question
Fish
Sims
Kurt Schneider
Founder of psychopathology
Cameron
Jaspers
Goldstein
Bleuler
Freud
Carl Schneider
Jung

1585
Concrete thinking
Kurt Schneider
Sims
Jaspers
Carl Schneider
Fish
Bleuler
Goldstein
Freud
Bleuler
Jung
Kurt Schneider
Cameron
Fish
First rank symptoms
Cameron
Jaspers
Freud
Goldstein
Sims
Freud
Carl Schneider
Kurt Schneider
Jung
Sims
Loosening of association
Jung
Jaspers
Bleuler
Goldstein
Cameron
Fish
Fish
Sims
Carl Schneider
Cameron
Overinclusive thinking
Freud
Kurt Schneider
Bleuler
Carl Schneider
Jung
Check
Goldstein
Jaspers

Karl Jasper is widely considered a major figure in philosophy and psychiatry. He is the
founder of psychopathology. It is important to know about Jaspers and his method of enquiry
into psychiatry for one to understand psychopathology. He introduced phenomenology, which
was a long regarded as a method of philosophic enquiry, to psychiatry. 
Concrete thinking is associated with Goldstein. He claimed that both in schizophrenia and in
organic brain diseases, a loss of the abstract attitude takes place so that the thinking becomes
concrete i.e. the patient is unable to free himself from the superficial concrete aspects of
thinking. 
Kurt Schneider, a German psychiatrist and a pupil of Karl Jaspers, pointed out certain
symptoms as being characteristic of schizophrenia and therefore exhibiting a "first­rank" status
in the hierarchy of potentially diagnostic symptoms. The "first­rank" symptoms (FRS) have
played an extremely important role in the recent diagnostic systems: in the International
Statistical Classification of Diseases, Tenth Revision (ICD­10) as well as in Diagnostic and
Statistical Manual of Mental Disorder, (DSM­ for versions 3 and 4), the presence of one FRS is
symptomatically sufficient for the schizophrenia diagnosis (not for DSM­5) but FRS are not
essential to diagnose schizophrenia. 
Bleuler proposed four fundamental symptoms of schizophrenia which includes ambivalence,
autism, affect disturbances and loosening of associations.

1586
Cameron proposed four characteristic formal thought disorders which include over­inclusive
thinking.
The correct answer is: Founder of psychopathology – Jaspers, Concrete thinking – Goldstein,
First rank symptoms – Kurt Schneider, Loosening of association – Bleuler, Overinclusive
thinking – Cameron

Question 26 HiY Assessment EMI077
Not answered Dementia
Choose salient features for each item below from the list shown:
Marked out of 9.00

Flag question
Stepwise cognitive deterioration
Seizures
A salient feature of Lewy body History of TIAs
dementia Personality change
Myoclonus
Striking insight loss
Weight­loss
Visual hallucinations
Weight­loss
Auditory hallucinations
Myoclonus
A salient feature of frontotemporal
Emotional blunting
Auditory hallucinations
dementia History of TIAs
Emotional blunting
Personality change
Stepwise cognitive deterioration
Seizures
Emotional blunting
Striking insight loss
Personality change
A salient features of vascular
Visual hallucinations
History of TIAs
dementia Striking insight loss
Auditory hallucinations
Visual hallucinations
Myoclonus
Seizures
Seizures
Weight­loss
Striking insight loss
Stepwise cognitive deterioration
Visual hallucinations
Myoclonus
A salient feature of CJD Emotional blunting
Personality change
Weight­loss
Stepwise cognitive deterioration
History of TIAs
Auditory hallucinations
Check

Dementia with Lewy Bodies is characterised by spontaneous motor features of Parkinsonism,
along with a fluctuating cognition, recurrent visual hallucinations of well formed and detailed
imageries. The cognitive decline is severe enough to affect normal social and occupational
functioning. 

1587
Frontotemporal dementia: Insidious onset and gradual progression, Early loss of personal
and social awareness, Early emotional blunting, Early loss of insight. Behavioural features
include early signs of disinhibition, a decline in personal hygiene & grooming, mental rigidity,
inflexibility, hyperorality, stereotyped and perseverative behaviour. Speech disorder includes
Reduction and stereotypy of speech, echolalia, and perseveration. Affective symptoms
include Anxiety, depression, and frequent mood changes, emotional indifference. Physical
signs include Incontinence, primitive reflexes, akinesia, rigidity and tremor. 
Vascular dementia: Onset may usually follow a cerebrovascular event and is more acute. The
course is usually stepwise, with periods of intervening stability. Focal neurological signs &
symptoms or neurological evidence of cerebrovascular disease (CVD) judged etiologically
related to the disturbance.
Creutzfeldt­Jakob disease: The clinical picture is one of rapidly deteriorating dementia,
myoclonus, cerebellar and extrapyramidal signs leading to death within a year. Patients may
present with non­specific symptoms such as lethargy, depression and fatigue. Within weeks,
more fulminant symptoms develop, including progressive cortical­pattern dementia,
myoclonus and pyramidal and extrapyramidal signs. Myoclonus becomes prominent as the
disease progresses. Patients may develop cortical blindness.
The correct answer is: A salient feature of Lewy body dementia – Visual hallucinations, A
salient feature of frontotemporal dementia – Personality change, A salient features of vascular
dementia – Stepwise cognitive deterioration, A salient feature of CJD
– Myoclonus

Question 27 HiY Assessment EMI078
Not answered Psychosomatic disorders
Identify the diagnosis for each of the following clinical situations:
Marked out of 4.00

Flag question A 30­year­old woman presents with
hyperpigmented buccal mucosa. She
Celiac disease
has also lost weight of late, which she Binge eating disorder
considers on a positive note. She Lymphoma (Hodgkin's)
reports some nausea in the Autoimmune lupus
mornings, abdominal pains, feeling Irritable bowel syndrome
dizzy on and off and feeling confused Addison's disease
at times. Hypothyroidism
Cushing's disease
A 55­year­old woman presents with
Autoimmune lupus
Hyperthyroidism
Addison's disease
weight loss, foul­smelling bulky
stools, abdominal pain, bloating, Lymphoma (Hodgkin's)
mouth ulcers and vomiting. Pathology Irritable bowel syndrome
shows villous atrophy on jejunal Cushing's disease
biopsy. Hypothyroidism
Hyperthyroidism
Celiac disease
Cushing's disease
A 24­year­old lady has weight loss
Binge eating disorder
Irritable bowel syndrome
and irregular periods. She also has
tremors and frequent stools of normal Lymphoma (Hodgkin's)
consistency. She is not comfortable Hypothyroidism
with the heating even in the coldest of Addison's disease
days. Hyperthyroidism
Autoimmune lupus
Binge eating disorder
Celiac disease

1588
A 27­year­old woman has weight
Irritable bowel syndrome
gain and hirsutism. She is also
Addison's disease
clinically depressed and has a history
of hypertension. Binge eating disorder
Lymphoma (Hodgkin's)
Hypothyroidism
Hyperthyroidism
Autoimmune lupus
Celiac disease
Check Cushing's disease

Addison's disease: Clinical features include weakness, abdominal pain, anorexia, weight
loss, constipation, myalgia and depression. Signs include hyperpigmentation (palmar
creases, buccal mucosa), vitiligo and postural hypotension. 
Coeliac disease: Clinical presentations include abdominal pain, steatorrhoea, bloating,
nausea/vomiting, aphthous ulcers, angular stomatitis, weight loss and fatigue. Jejunal biopsy
shows villous atrophy, reversing on gluten free diet. 
Hyperthyroidism: Clinical symptoms include weight loss despite increased appetite, heat
intolerance, sweating, diarrhoea, tremor, irritability, emotional lability, psychosis,
oligomenorrhoea, and itch. Signs include atrial fibrillation, warm peripheries, fine tremor,
goitre, palmar erythema, hair thinning and lid lag. 
Cushing's syndrome: Clinical features include weight gain, menstrual irregularity,
amenorrhea, and hirsutism in females, impotence in males, depression, muscle weakness
and fractures. Signs include tissue wasting, myopathy, thin skin, purple abdominal striae,
osteoporosis, hirsutism, hypertension, hyperglycemia, predisposition to infection and bad
wound healing.
The correct answer is: A 30­year­old woman presents with hyperpigmented buccal mucosa.
She has also lost weight of late, which she considers on a positive note. She reports some
nausea in the mornings, abdominal pains, feeling dizzy on and off and feeling confused at
times. – Addison's disease, A 55­year­old woman presents with weight loss, foul­smelling
bulky stools, abdominal pain, bloating, mouth ulcers and vomiting. Pathology shows villous
atrophy on jejunal biopsy. – Celiac disease, A 24­year­old lady has weight loss and irregular
periods. She also has tremors and frequent stools of normal consistency. She is not
comfortable with the heating even in the coldest of days. – Hyperthyroidism, A 27­year­old
woman has weight gain and hirsutism. She is also clinically depressed and has a history of
hypertension. – Cushing's disease

Question 28 HiY Assessment EMI079
Not answered Terms related to memory 
For each of the description below chose one most appropriate term from the above list
Marked out of 4.00

Flag question
Recapitulation
Déjà vu
Reintegrating events using a variety Recollection
of components Confabulation
Delusional elaboration
Jamais vu
Retrospective falsification
Recognition
Registration

1589
Modifying memory in terms of Registration
attitudes and emotions
Recollection
Déjà vu
Recapitulation
Confabulation
Delusional elaboration
Recognition
Retrospective falsification
Confabulation
Absence of the normal feeling of
Jamais vu
Recapitulation
familiarity that accompanies memory Delusional elaboration
when visiting a place for the second Recognition
time Déjà vu
Retrospective falsification
Recollection
Registration
Retrospective falsification
Jamais vu
Recapitulation
Feeling of familiarity that
Recollection
accompanies return of stored
Recognition
material to consciousness
Delusional elaboration
Déjà vu
Registration
Jamais vu
Check
Confabulation

Recollection is the reintegration of a complete event from a variety of different components
and is a function of recall. For example, to remember the sequence of events on your wedding
day requires recalling a number of separate items of information by exploring many different
avenues. 
Retrospective falsification involves applying 'emotional colour' to the recall of events. 
Jamais vu refers to a pathology of familiarity that is the phenomenological antagonism of déjà
vu. It is a form of paramnesia characterised by an erroneous belief of having never before
seen or experienced something that one has in reality encountered before. 
Recognition is the feeling of familiarity, which accompanies the return of stored material to
consciousness. It is therefore not strictly part of the process of memory but is intimately
connected
The correct answer is: Reintegrating events using a variety of components – Recollection,
Modifying memory in terms of attitudes and emotions – Retrospective falsification, Absence of
the normal feeling of familiarity that accompanies memory when visiting a place for the
second time – Jamais vu, Feeling of familiarity that accompanies return of stored material to
consciousness – Recognition

Question 29 HiY Assessment EMI080
Not answered Psychopathology of memory
Choose the appropriate terms associated with each of the following descriptions:
Marked out of 5.00

Flag question

1590
Falsification of memory occurring in Eidetic images
clear consciousness seen in Pseudologiafantastica
alcoholic subjects
Repression
Ganser's syndrome
Possession states
Lethologica
Confabulation
Dissociative trance
Pseudologiafantastica
Fugue
Repression
Fluent possible lying seen in
Paramnesia
Dissociative trance
dissocial personality disorders Lethologica
Confabulation
Fugue
Ganser's syndrome
Possession states
Ganser's syndrome
Eidetic images
Repression
Wandering away from surroundings Paramnesia
Fugue
and subsequent amnesia for the
Pseudologiafantastica
episode
Dissociative trance
Paramnesia
Lethologica
Possession states
Eidetic images
Eidetic images
Repression
Hysterical dissociative disorder Confabulation
Pseudologiafantastica
presenting with approximate answers
Possession states
seen in prisoners
Dissociative trance
Fugue
Confabulation
Ganser's syndrome
Paramnesia
Lethologica
Dissociative trance
Paramnesia
Pseudologiafantastica
Occurs as part of a dissociation Eidetic images
reaction with narrow consciousness
Lethologica
and loss of personal identity
Confabulation
Possession states
Ganser's syndrome
Fugue
Repression
Check

Confabulation is a falsification (or temporal confusion) of memory occurring in clear
consciousness associated with organic states. Suggestibility is a prominent feature of
confabulation. It is often described in Korsakoff syndrome. 
In pseudologia fantastica, there is a fluent but plausible (as opposed to fantastic) lying. It is
usually associated with dissocial or histrionic personality disorders, and also reported in
Munchausen syndrome. 
In a dissociative fugue, wandering away from surroundings and subsequent amnesia for the
episode can be seen. There are marked memory loss and loss of identity, but the patient can
carry out complicated patterns of behaviour and maintains self­care. 

1591
Ganser's syndrome, considered as a hysterical dissociative disorder, includes approximate
answers, clouding of consciousness with disorientation, psychogenic, physical symptoms ­
analgesia & hyperaesthesia, and pseudohallucinations the latter is not always present).
Possession states can occur as a part of dissociation or in normal religious experiences, or
under hypnosis. Consciousness is altered in dissociative states. There is the loss of personal
identity and replaced with another identity.
The correct answer is: Falsification of memory occurring in clear consciousness seen in
alcoholic subjects – Confabulation, Fluent possible lying seen in dissocial personality
disorders – Pseudologiafantastica, Wandering away from surroundings and subsequent
amnesia for the episode – Fugue, Hysterical dissociative disorder presenting with
approximate answers seen in prisoners – Ganser's syndrome, Occurs as part of a dissociation
reaction with narrow consciousness and loss of personal identity – Possession states

Question 30 HiY Assessment EMI081
Not answered Motor Symptoms 
From the above list chose the most appropriate terms describing each of the following clinical
Marked out of 6.00
observations
Flag question

Catalepsy
Waxy flexibility
A patient moves his arm on slightest Mitgehen
pressure regardless of any instruction
Mannerisms
given
Ambitendency
Stereotypy
Cataplexy
Gegenhalten
Mitgehen
Automatic obedience
Ambitendency
Despite being asked to comply with, Mannerisms
a patient resists with same pressure
Catalepsy
an attempt to move his arm.
Stereotypy
Waxy flexibility
Gegenhalten
Cataplexy
Mitgehen
When asked to put out his hand, a Automatic obedience
Mannerisms
patient extends, withdraws and then
Cataplexy
extends his hand repeatedly for 23
Ambitendency
times before allowing it to rest on the
examiner's hand. Waxy flexibility
Automatic obedience
Catalepsy
Stereotypy
Waxy flexibility
Gegenhalten
Mitgehen
On hearing a sudden noise, a patient Automatic obedience
feels that his knees are buckling. He
Mannerisms
falls and hurts himself.
Cataplexy
Gegenhalten
Catalepsy
Ambitendency
A patient allows his arms to be Stereotypy
manipulated freely without much
resistance and maintains the posture

1592
in which the examiner leaves him for
Mannerisms
few minutes. (TWO)
Cataplexy
Gegenhalten
Automatic obedience
Ambitendency
Waxy flexibility
Stereotypy
Catalepsy
Check
Mitgehen

Mitgehen or "Angle poise lamp" sign: The patient yields to slightest of pressures, without much
resistance, similar to an angle poise lamp that bends easily. This happens even if the patient
is instructed to resist any manipulation. 
Gegenhalten is a milder form of resistance/negativism. Patients with negativism resist or
oppose all passive movements attempted by the examiner. A mild form of such resistance is
called Gegenhalten or opposition. 
In ambitendency, the patient makes a series of tentative, opposing alternate movements that
do not reach the intended goal. This becomes evident when the patient is asked to carry out a
motor act. Ambivalence is the inability to make a decision ­ dilemma of the volitional faculty. 
Cataplexy refers to an abrupt loss of muscle tone leading to collapse. It occurs following
emotional stress. 
In catalepsy, the patient's limbs can be passively moved to any posture which will then be
held for a prolonged period of time. It is also called as waxy flexibility (flexibilitas cerea) Here
the patient shows wax like plastic 'mouldable' quality. His limbs can be moved by the
examiner to occupy certain postures, which are then maintained, even if these are
uncomfortable and bizarre
The correct answer is: A patient moves his arm on slightest pressure regardless of any
instruction given – Mitgehen, Despite being asked to comply with, a patient resists with same
pressure an attempt to move his arm. – Gegenhalten, When asked to put out his hand, a
patient extends, withdraws and then extends his hand repeatedly for 23 times before allowing
it to rest on the examiner's hand. – Ambitendency, On hearing a sudden noise, a patient feels
that his knees are buckling. He falls and hurts himself. – Cataplexy, A patient allows his arms
to be manipulated freely without much resistance and maintains the posture in which the
examiner leaves him for few minutes. (TWO) – Catalepsy, Waxy flexibility

Question 31 HiY Assessment EMI082
Not answered Disorders of perception

Marked out of 5.00 Choose the most appropriate term(s) to describe the following bodily sensations:

Flag question
Reflex hallucination
Thermic hallucination
A 53­year­old schizophrenic tells 'My Haptic hallucinations
feet on fire.' Hygric hallucinations
Affect illusions
Kinaesthetic hallucination
Pareidolic illusion
Hypnagogic hallucination
Complete illusion
Hypnapompic hallucination

1593
A 47­year­old woman claims 'A dead
Kinaesthetic hallucination
hand touched me.'
Hypnapompic hallucination
Thermic hallucination
Hypnagogic hallucination
Reflex hallucination
Haptic hallucinations
Affect illusions
Hygric hallucinations
Thermic hallucination
A 23­year­old man with psychosis
was assessed due to decline in his
Complete illusion
Reflex hallucination
mental state. During assessment, he Pareidolic illusion
Complete illusion
tells 'all my blood has dropped into Affect illusions
my legs, and I can feel a water level Hygric hallucinations
in my chest.' Pareidolic illusion
Hypnagogic hallucination
Kinaesthetic hallucination
Hypnapompic hallucination
Hypnapompic hallucination
Thermic hallucination
A 27­year­old man with paranoid
schizophrenia says 'I thought my life
Haptic hallucinations
Hypnagogic hallucination
was outside my feet and made them Haptic hallucinations
vibrate.' Hygric hallucinations
Kinaesthetic hallucination
Affect illusions
Reflex hallucination
Pareidolic illusion
Pareidolic illusion
Affect illusions
A 23­year­old drug addict tells 'I can Complete illusion
Hypnagogic hallucination
feel insects crawling under my skin.' Hygric hallucinations
Hypnapompic hallucination
Complete illusion
Reflex hallucination
Kinaesthetic hallucination
Check
Haptic hallucinations
Thermic hallucination

Superficial hallucination affecting the skin may be thermic, e.g., 'My feet is on fire'; haptic
hallucination may be experienced as touch, e.g., 'A dead hand touched me' or hygric
hallucinations (a perception of fluid), e.g., 'all my blood has dropped into my legs, and I can
feel a water level in my chest'. 
Kinaesthetic hallucinations are those of joint or muscle sense. The patient feels that his limbs
are being contorted, or muscles pressed. 
One particularly unpleasant form of haptic hallucination is called formication, the sensation of
little animals or insects crawling under the skin. Formication has been reported in alcohol
withdrawal and cocaine intoxication. (Ref: Symptoms in the mind; Andrew Sims; pg 90­91)
The correct answer is: A 53­year­old schizophrenic tells 'My feet on fire.' – Thermic
hallucination, A 47­year­old woman claims 'A dead hand touched me.' – Haptic hallucinations,
A 23­year­old man with psychosis was assessed due to decline in his mental state. During
assessment, he tells 'all my blood has dropped into my legs, and I can feel a water level in my
chest.' – Hygric hallucinations, A 27­year­old man with paranoid schizophrenia says 'I thought
my life was outside my feet and made them vibrate.' – Kinaesthetic hallucination, A 23­year­
old drug addict tells 'I can feel insects crawling under my skin.' – Haptic hallucinations

1594
Question 32 HiY Assessment EMI083
Not answered Thought & speech problems
Identify the correct psychopathological term for each of the following descriptions:
Marked out of 4.00

Flag question
Vorbeireden
Paragrammatism
A 27­year­old prisoner with
Logorrhoea
schizophrenia gives approximate
Concrete thinking
answers to simple questions
Neologism
Overinclusive thinking
Verschmelzung
Logoclonia
Concrete thinking
Witzelsucht
Neologism
A 47­year­old man sustained a head Logoclonia
injury make inappropriate comments
Verschmelzung
and cracks jokes
Paragrammatism
Overinclusive thinking
Vorbeireden
Logorrhoea
Overinclusive thinking
Witzelsucht
Logoclonia
A 69­year­old man with Parkinson's
Paragrammatism
disease gets stuck on a particular
Witzelsucht
word of a sentence and repeats it
Vorbeireden
Concrete thinking
Logorrhoea
Neologism
Verschmelzung
Verschmelzung
Logoclonia
Vorbeireden
Logorrhoea
A 23­year­old manic woman has
excessive talking that was incoherent
Neologism
and has no logic Concrete thinking
Paragrammatism
Witzelsucht
Overinclusive thinking

Check

Vorbeireden is talking past the point, described as formal thought disorder, different from the
flight of ideas, which is a disorder of thought stream. It is not exclusively associated with
Ganser's syndrome. It is also seen in acute schizophrenia and hebephrenic schizophrenia.
Vorbeireden ('talking past the point') is often used interchangeably with vorbeigehen ('going
past the point'), although the latter was originally defined as part of the 'Ganser syndrome',
whereby some criminals would give incorrect answers ('approximate answers') to simple
questions that none the less suggested that the correct answer was known (e.g. saying dogs
have five legs.

1595
Witzelsucht (from the German witzeln, meaning to joke or wisecrack, and sucht, meaning
addiction) is characterized by a tendency to make puns, tell inappropriate jokes or pointless
stories in socially inappropriate situations. Ironically, however, the person is insensitive to
humor produced by themselves or others around them. This disorder is most commonly seen
in patients with frontal lobe damage, particularly right frontal lobe tumors or trauma. 
Logoclonia is a symptom of Parkinson's disease and Alzheimer's dementia where the patient
gets stuck on a particular word of a sentence and repeats it. 
Logorrhoea is excess speech or verbal diarrhoea, which is a symptom of mania. It involves
excessive talking or mumbling monotonously and often incoherently with words that avoid any
logic or reason.
(Excerpts from Ordered thoughts on thought disorder. Psychiatric Bulletin. (n.d.). Retrieved
from http://pb.rcpsych.org/content/29/12/462)
The correct answer is: A 27­year­old prisoner with schizophrenia gives approximate answers
to simple questions – Vorbeireden, A 47­year­old man sustained a head injury make
inappropriate comments and cracks jokes – Witzelsucht, A 69­year­old man with Parkinson's
disease gets stuck on a particular word of a sentence and repeats it – Logoclonia, A 23­year­
old manic woman has excessive talking that was incoherent and has no logic – Logorrhoea

Question 33 HiY Assessment EMI084
Not answered Defence mechanisms ­ categories
Identify 2 or more defences for each of the following categories
Marked out of 11.00

Flag question
Humour
Regression
Obsession
Mature defenses
Intellectualisation
Deindividuation
Denial

Intellectualisation
Denial
Regression
Neurotic defenses 
Obsession
Deindividuation
Humour

Denial
Regression
Intellectualisation
Humour
Deindividuation
Narcissistic defenses 
Obsession

1596
Kleinian defenses 
Intellectualisation
Obsession
Denial
Deindividuation
Humour
Regression

Deindividuation
Humour
Intellectualisation
Immature defenses 
Denial
Obsession
Regression

Check

Mature defences include altruism, anticipation, humour, sublimation and suppression
(mnemonic ­ SASHA).
Neurotic defences include displacement, dissociation, isolation, reaction formation,
rationalization, repression, intellectualization and identification with an aggressor. 
Narcissistic defences include denial and projection. 
Kleinian defences include splitting (with idealization & denigration), projective identification,
omnipotence, grandiosity, denial, and introjection. 
Immature defences include acting out, passive aggression, somatisation and regression
The correct answer is: Mature defenses – Humour, Neurotic defenses 
– Intellectualisation, Narcissistic defenses 
– Denial, Kleinian defenses 
– Denial, Immature defenses 
– Regression

Question 34 HiY Assessment EMI085
Not answered Problem Workers
There are three people in an office with problems at work, and all of these are the result of
Marked out of 3.00
long­standing behaviour patterns. Identify the type of personality disorders for each of the
Flag question following situations

Anxious avoidant personality disorder
Borderline personality disorder
The manager is rigid about Paranoid personality disorder
timetables and expects all their
Antisocial personality disorder
colleagues to be like them
Histrionic personality disorder
Schizoid personality disorder
Anankastic personality disorder
Schizotypal personality disorder
Dependent personality disorder
Narcissistic personality disorder

1597
A secretary who is nervous and Borderline personality disorder
keeps to herself, but wants to have
Paranoid personality disorder
friends and a boyfriend
Schizotypal personality disorder
Histrionic personality disorder
Antisocial personality disorder
Dependent personality disorder
Anxious avoidant personality disorder
Anankastic personality disorder
A woman who comes to work for a
Narcissistic personality disorder
Histrionic personality disorder
long weekend is annoyed that her Schizoid personality disorder
Dependent personality disorder
office plants have died because Borderline personality disorder
nobody watered them. Schizoid personality disorder
Antisocial personality disorder
Schizotypal personality disorder
Narcissistic personality disorder
Check
Paranoid personality disorder
Anxious avoidant personality disorder

Anankastic personality disorder­ frequently perfectionists, often lack clarity in seeing other
perspectives or ways of doing things, Rigid attention to detail may prevent them from
completing tasks; may not be able to delegate tasks; workaholics. 
Avoidant personality disorder ­ Fears being judged negatively by other people, Feelings of
discomfort in group/social settings; May come across as being socially withdrawn and have
low self­esteem. 
Paranoid personality disorder: Patients often suspect that others are exploiting, harming, or
deceiving them without sufficient basis. This personality disorder is characterized by the
presence of a pervasive distrust and suspiciousness that others' motives are malevolent.
The correct answer is: The manager is rigid about timetables and expects all their colleagues
to be like them – Anankastic personality disorder, A secretary who is nervous and keeps to
herself, but wants to have friends and a boyfriend – Anxious avoidant personality disorder, A
woman who comes to work for a long weekend is annoyed that her office plants have died
because nobody watered them. – Paranoid personality disorder

Question 35 HiY Assessment EMI086
Not answered Neurological symptoms
Find one term each to describe the following presentation
Marked out of 3.00

Flag question
A 65­year­old gentleman was unable
Finger agnosia
to identify familiar objects such as key Right ­ left disorientation
or coins by touch with eyes closed. Prosopagnosia
He has difficulty in describing their Dressing apraxia
shape and texture. However, he Visuospatial agnosia
identifies them when eyes are open. Anosognosia
Constructional apraxia
Astereognosis
Agraphaesthesia
A 67­year­old woman is unable to
recognize the number scratched on
her skin without seeing the act of
scratching

1598
Anosognosia
Prosopagnosia
Right ­ left disorientation
Dressing apraxia
Visuospatial agnosia
Finger agnosia
Constructional apraxia
Agraphaesthesia
Visuospatial agnosia
Astereognosis
Finger agnosia
A man has left side hemiplegia after a Agraphaesthesia
stroke. He is unaware of his deficit. Constructional apraxia
Prosopagnosia
Anosognosia
Astereognosis
Dressing apraxia
Right ­ left disorientation
Check

Agraphaesthesia is defined as inability to recognize what number or alphabet is scratched
on one's skin without seeing. Astereognosis is defined as inability to recognize objects by
palpation, and without visual inspection. Anosognosia refers to inability to identify one's
neurological deficits.
The correct answer is: A 65­year­old gentleman was unable to identify familiar objects such as
key or coins by touch with eyes closed. He has difficulty in describing their shape and texture.
However, he identifies them when eyes are open. – Astereognosis, A 67­year­old woman is
unable to recognize the number scratched on her skin without seeing the act of scratching –
Agraphaesthesia, A man has left side hemiplegia after a stroke. He is unaware of his deficit. –
Anosognosia

Question 36 HiY Assessment EMI087
Not answered Validity of rating scales
Find one term each to describe the following descriptions of psychometric validity of scales
Marked out of 5.00
measuring depression
Flag question

Content
Criterion
A scale 'X' adds some more
Concurrent
information to the other scales
Face
measuring depression
Predictive
Incremental

Incremental
Criterion
On inspection, the items of a scale 'Y' Predictive
appears to assess depression Content
Face
Concurrent

1599
Incremental
A scale 'Z' includes an exhaustive Criterion
coverage of all of the expected Face
symptoms of depression Content
Concurrent
Predictive

Face
Scores from a new scale for Content
depression has a high degree of Criterion
correlation with the scores from Predictive
Hamilton depression scale Concurrent
Incremental

Concurrent
Higher scores on the Hamilton
depression scale predict the risk of Content
future self­harm in a cohort of Face
untreated patients. Incremental
Predictive
Criterion

Check

Face validity refers to the subjective measure of deciding whether the test measures the
construct of interest on its face value. e.g., Hamilton depression scale clearly has a face
validity for measuring depression; but not for measuring obsessions. 
Content validity refers to whether the contents i.e. each subscale, items or elements of the test
are in line with the general objectives or specifications the test was originally designed to
measure. It assesses the degree of coverage of all domains in relation to the measured
condition. 
Criterion validity refers to the performance against an external criterion such as another
instrument (concurrent) or future diagnostic possibility (predictive). 
Predictive validity refers to the ability of a test to predict future group differences according to
current group differences in the score. e.g. high aggression score in childhood could predict
high criminal incidents in adult life. 
Incremental validity refers to the ability of a measure to predict or explain variance over and
above other measures.
The correct answer is: A scale 'X' adds some more information to the other scales measuring
depression – Incremental, On inspection, the items of a scale 'Y' appears to assess
depression – Face, A scale 'Z' includes an exhaustive coverage of all of the expected
symptoms of depression – Content, Scores from a new scale for depression has a high
degree of correlation with the scores from Hamilton depression scale – Criterion, Higher
scores on the Hamilton depression scale predict the risk of future self­harm in a cohort of
untreated patients. – Predictive

Question 37 HiY Assessment EMI088
Not answered

1600
Marked out of 5.00 Schizophrenia & subtypes
Flag question
For each of the description below choose one most appropriate diagnostic term using ICD­10
nomenclature:

Schizotypal disorder
Simple schizophrenia
Prominent delusions and auditory Catatonic schizophrenia
hallucinations Residual Schizophrenia
Post schizophrenic depression
Paranoid schizophrenia
Hebephrenic schizophrenia
Schizoid personality disorder
Residual Schizophrenia
Schizophreniform psychosis
Post schizophrenic depression
Delusions and hallucinations are
Schizophreniform psychosis
present at a lesser intensity that the
Simple schizophrenia
acute phase
Hebephrenic schizophrenia
Schizotypal disorder
Schizoid personality disorder
Catatonic schizophrenia
Paranoid schizophrenia
Paranoid schizophrenia
Hebephrenic schizophrenia
Marked social withdrawal, loss of
Schizophreniform psychosis
initiative & drive, with poorly formed
Residual Schizophrenia
delusions
Schizoid personality disorder
Post schizophrenic depression
Catatonic schizophrenia
Simple schizophrenia
Schizoid personality disorder
Schizotypal disorder
Paranoid schizophrenia
Poor social functioning, affective Hebephrenic schizophrenia
blunting and marked thought disorder Schizotypal disorder
Simple schizophrenia
Residual Schizophrenia
Post schizophrenic depression
Catatonic schizophrenia
Paranoid schizophrenia
Schizophreniform psychosis
Post schizophrenic depression
Marked acute onset psychomotor Schizophreniform psychosis
disturbance Residual Schizophrenia
Catatonic schizophrenia
Schizotypal disorder
Simple schizophrenia
Schizoid personality disorder
Check
Hebephrenic schizophrenia

Paranoid schizophrenia is a common subtype characterized by prominent delusions (often
persecutory in content) and hallucinations (usually auditory). Thought disorders, affective
abnormality and negative symptoms may be present but not prominent. 
Residual schizophrenia consists of a long term but not necessarily irreversible negative
symptoms. Delusions and hallucinations are often of low intensity or reduced for at least one
year, often following a more florid acute phase. 

1601
Simple schizophrenia is characterized by an early onset (usually in the second decade), very
insidious and progressive course, and presence of characteristic negative symptoms like
marked social withdrawal, loss of initiative and drive and shallow emotional response.
Delusions and hallucinations are usually absent; if present they are short lasting and poorly
formed. 
Hebephrenic schizophrenia is also known as disorganized schizophrenia. It is characterized
by marked thought disorder (e.g. severe loosening of associations), inappropriate and blunted
affect (e.g. senseless giggling) and abnormal mannerisms (e.g. prolonged mirror gazing).
Social and occupational functioning is markedly impaired, often with poor self­care, hygiene,
extreme asocial and disorganized behaviour. 
Catatonic schizophrenia is a somewhat less common subtype characterized by psychomotor
symptoms ranging from violent excitement through posturing, negativism, waxy flexibility,
perseveration to stupor. The onset is usually acute with an episodic course and usually a very
good degree of recovery.
The correct answer is: Prominent delusions and auditory hallucinations – Paranoid
schizophrenia, Delusions and hallucinations are present at a lesser intensity that the acute
phase – Residual Schizophrenia, Marked social withdrawal, loss of initiative & drive, with
poorly formed delusions – Simple schizophrenia, Poor social functioning, affective blunting
and marked thought disorder – Hebephrenic schizophrenia, Marked acute onset psychomotor
disturbance – Catatonic schizophrenia

Question 38 HiY Assessment EMI089
Not answered Coping mechanisms
For each of the following examples, please select the most likely descriptions of defence
Marked out of 4.00
mechanisms from the list above. Please select only one option, but an option may be used
Flag question more than once, if required.

A young man brings his 19­year­old
Passive­aggression
girlfriend who vacantly stares at the Acting out
wall and doesn't respond to Dissociation
conversations. She has been in this Reaction formation
state since they had a serious H Projection
argument one day ago. Intellectualisation
Denial
Anticipation
Passive­aggression
Isolation
Projective identification
After his father's sudden death, Tom Projective identification
H Projection
took over his family business though
Dissociation
he used to detest it for a long time
Anticipation
Reaction formation
Intellectualisation
Acting out
Dissociation
A 45­year­old woman who was Isolation
Projective identification
bullied at work comes late to work Denial
Denial
consistently instead of raising the
Anticipation
issue of bullying with the concerned
managers
H Projection
Intellectualisation
Acting out
Isolation
Reaction formation
Passive­aggression

1602
A 45­year­old man holds long­ H Projection
standing suspiciousness towards the
Reaction formation
police. He becomes concerned and
acts in an unusual manner on seeing Denial
two policemen patrolling streets. The Dissociation
police then become suspicious of him Isolation
and arrest him as a precautionary Anticipation
measure. Passive­aggression
Intellectualisation
Acting out
Projective identification

Check

Dissociation is the defense mechanism used here. Dissociation is the temporarily but drastic
modification of one's sense of personal identity in order to avoid emotional unpleasantness.
Fugue states and conversion reactions are manifestations of dissociation.
Reaction formation is the transformation of unacceptable thoughts and feelings to their
opposite. For example, a man struggling to accept his homosexuality may engage in a
number of heterosexual relationships. 
The passive­aggressive behavior involves a regular pattern of passive resistance to
anticipated work requirements displayed by means of a negativistic attitude in response to
demands and requirements related to one's performance.
Projective identification is a Kleinian defence where an aspect of self is projected onto
someone else. The projector influences the recipient to identify with what has been projected
and projector herself now believes that the aspect originated from the recipient who is
reacting. This may result in the recipient behaving in a manner similar to the projector. 
Ref: Kaplan & Sadock's Synopsis of Psychiatry: Behavioral Sciences/Clinical Psychiatry, 10th
Edition. Lippincott Williams & Wilkins 2007
The correct answer is: A young man brings his 19­year­old girlfriend who vacantly stares at
the wall and doesn't respond to conversations. She has been in this state since they had a
serious argument one day ago. – Dissociation, After his father's sudden death, Tom took over
his family business though he used to detest it for a long time – Reaction formation, A 45­year­
old woman who was bullied at work comes late to work consistently instead of raising the
issue of bullying with the concerned managers – Passive­aggression, A 45­year­old man
holds long­standing suspiciousness towards the police. He becomes concerned and acts in
an unusual manner on seeing two policemen patrolling streets. The police then become
suspicious of him and arrest him as a precautionary measure. – Projective identification

Question 39 HiY Assessment EMI090
Not answered Psychopathological processes
Identify the name of the psychopathological process involved, for each of the following
Marked out of 5.00
descriptions;
Flag question

1603
Abrupt interruption of thoughts
Derailment
Cotard's syndrome
Thought blocking
Substitution
Capgras' syndrome
Thought insertion
Asyndesis
Moria
Substitution
Couvade syndrome
Moria
Thought echo
Asyndesis
Deviation of train of thought without
blocking Capgras' syndrome
Thought insertion
Couvade syndrome
Cotard's syndrome
Thought echo
Thought echo
Thought blocking
Couvade syndrome
Lack of adequate logical connection Derailment
Cotard's syndrome
between successive thoughts Moria
Thought blocking
Asyndesis
Substitution
Derailment
Couvade syndrome
Capgras' syndrome
Thought insertion
A woman believes her intestines Thought insertion
Asyndesis
have stopped working and are rotting
Derailment
away
Thought blocking
Moria
Substitution
Thought echo
Asyndesis
Capgras' syndrome
Derailment
Cotard's syndrome
Thought insertion
A conversion symptom is seen in Moria
partners of expectant mothers during Cotard's syndrome
their pregnancy Couvade syndrome
Thought blocking
Thought echo
Capgras' syndrome
Substitution
Check

Thought blocking is a feature of schizophrenic thought disorder where the patient experiences
a sudden break in the train of thought. In derailment, a normally flowing train of ideas
suddenly changes. The determining tendency is preserved but is misdirected. Lack of
adequate logical connection between successive thoughts is termed as asyndesis 
Cotard's syndrome is a presentation of psychotic depressive illness (a combination of
depressed mood with nihilistic delusions and hypochondriacal delusions). 

1604
Couvade syndrome is a conversion symptom seen in partners of expectant mothers during
their pregnancy. It is not a delusional phenomenon (as the affected individual does not
believe they are pregnant though they develop unexplained somatic features of pregnancy).
The correct answer is: Abrupt interruption of thoughts – Thought blocking, Deviation of train of
thought without blocking – Derailment, Lack of adequate logical connection between
successive thoughts – Asyndesis, A woman believes her intestines have stopped working and
are rotting away – Cotard's syndrome, A conversion symptom is seen in partners of expectant
mothers during their pregnancy – Couvade syndrome

Question 40 HiY Assessment EMI091
Not answered Features of personality disorders
Identify the type of personality disorder for each of the following traits
Marked out of 4.00

Flag question
Paranoid personality disorder
Narcissistic personality disorder
Excessively emotional with Histrionic personality disorder
temperamental instability Schizoid personality disorder
Anankastic personality disorder
Schizotypal personality disorder
Dissocial personality disorder
Obsessive personality disorder
Schizoid personality disorder
Borderline personality disorder
Dissocial personality disorder
Dependent personality disorder
Borderline personality disorder
Increased self­referential tendency
Paranoid personality disorder
Anankastic personality disorder
Obsessive personality disorder
Histrionic personality disorder
Narcissistic personality disorder
Dependent personality disorder
Dependent personality disorder
Schizoid personality disorder
Schizotypal personality disorder
Borderline personality disorder
Excessive sense of entitlement
Narcissistic personality disorder
Obsessive personality disorder
Schizotypal personality disorder
Dissocial personality disorder
Paranoid personality disorder
Anankastic personality disorder
Anankastic personality disorder
Histrionic personality disorder
Histrionic personality disorder
Obsessive personality disorder
Chronic feelings of emptiness Narcissistic personality disorder
Schizotypal personality disorder
Paranoid personality disorder
Dependent personality disorder
Dissocial personality disorder
Check Schizoid personality disorder
Borderline personality disorder

Borderline personality disorder is characterized by emotional instability and lack of impulse
control. Chronic feelings of emptiness or boredom with an inability to stay alone are seen in
borderline personality disorder.

1605
In paranoid personality disorder, a tendency to hold an excessively self­referential stance is
noted. 
Narcissistic personality disorder is marked by an excessive sense of entitlement, e.g.,
unreasonable expectations of especially favorable treatment or automatic compliance of
others to his/her wishes.
The correct answer is: Excessively emotional with temperamental instability – Borderline
personality disorder, Increased self­referential tendency – Paranoid personality disorder,
Excessive sense of entitlement – Narcissistic personality disorder, Chronic feelings of
emptiness – Borderline personality disorder

Question 41 HiY Assessment EMI092
Not answered Diagnosing substance use disorders
For each of the following clinical presentations, which one of the listed is the most likely
Marked out of 3.00
diagnosis?
Flag question

A 25­year­old lady who is on a
controlled drug prescription went on Cocaine withdrawal
a short holiday for three days and Cannabis withdrawal
forgot to take her prescribed Opiate withdrawal
medication. When she returned
Cocaine intoxication
home, she developed diarrhoea,
running nose and goose flesh. On
Opiate intoxication
examination, she had dilated pupils Amphetamine intoxication
and tachycardia. Cannabis intoxication
Alcohol withdrawal
A 30­year­old gentleman presented Opiate intoxication
Benzodiazepine withdrawal
to the A&E with paranoid delusions Cocaine intoxication
LSD intoxication
and prominent visual hallucinations Benzodiazepine withdrawal
of very rapid onset following
Opiate withdrawal
ingesting of a street drug. On
examination, he had tachycardia,
Alcohol withdrawal
hypertension and pupillary dilatation. Amphetamine intoxication
Cannabis withdrawal
Cannabis intoxication
LSD intoxication
Cocaine withdrawal
Cannabis withdrawal
LSD intoxication
Cocaine withdrawal
A 24­year­old woman attends the Cocaine intoxication
A&E in an agitated state. She Benzodiazepine withdrawal
complains of being anxious and has Alcohol withdrawal
insomnia. On examination, she has
Opiate withdrawal
tachycardia and tremor.
Opiate intoxication
Amphetamine intoxication
Cannabis intoxication

Check

Opiate withdrawal effects: Morphine and heroin withdrawal syndrome begins 6 to 8 hours
after the last dose, peaks in 2 days and reduces in a week usually. The clinical features
include dysphoric mood, nausea or vomiting, muscle aches, lacrimation or rhinorrhea,

1606
pupillary dilation, piloerection (goose flesh), or sweating, diarrhea, yawning, fever and
insomnia. 
Amphetamine Intoxication effects: Tachycardia or bradycardia ­ arrhythmias may be fatal,
pupillary dilation, elevated or lowered blood pressure, perspiration or chills, nausea or
vomiting, psychomotor agitation or retardation, muscular weakness, respiratory depression,
chest pain, or cardiac arrhythmias. Psychological effects such as euphoria, changes in
sociability, anxiety, tension, stereotyped behaviours, impaired judgment, etc. The hallmark of
amphetamine­induced psychosis is the paranoia. Sensitisation is suggested to be the
biological mechanism behind stimulant­induced psychosis. 
Benzodiazepine withdrawal: Autonomic hyperactivity, increased tremor, insomnia, nausea or
vomiting, transient visual, tactile, or auditory hallucinations or illusions, psychomotor agitation,
anxiety (most prominent), grand mal seizures and kinaesthetic hallucinations reported in a
few. 

Excerpt from Amphetamine Related Psychiatric Disorders: Clinical Presentation. (n.d.).
Retrieved from http://emedicine.medscape.com/article/289973­clinical.
The correct answer is: A 25­year­old lady who is on a controlled drug prescription went on a
short holiday for three days and forgot to take her prescribed medication. When she returned
home, she developed diarrhoea, running nose and goose flesh. On examination, she had
dilated pupils and tachycardia. – Opiate withdrawal, A 30­year­old gentleman presented to
the A&E with paranoid delusions and prominent visual hallucinations of very rapid onset
following ingesting of a street drug. On examination, he had tachycardia, hypertension and
pupillary dilatation. – Amphetamine intoxication, A 24­year­old woman attends the A&E in an
agitated state. She complains of being anxious and has insomnia. On examination, she has
tachycardia and tremor. – Benzodiazepine withdrawal

Question 42 HiY Assessment EMI093
Not answered ICD diagnostic features 
For each of the following ICD­10 diagnosis given below choose the most common clinical
Marked out of 6.00
features from the given list
Flag question

Affective changes
Auditory hallucinations
Loss of consciousness
Mania (TWO)
Bizarre delusion
Visual hallucinations
Systematized delusion
Transient delusion
Persecutory delusion
Visual hallucinations
Persecutory delusion
Affective changes
Delirium (TWO)
Auditory hallucinations
Bizarre delusion
Systematized delusion
Loss of consciousness
Transient delusion

Delusional disorder (ONE)

1607
Systematized delusion
Loss of consciousness
Bizarre delusion
Persecutory delusion
Visual hallucinations
Transient delusion
Auditory hallucinations
Affective changes
Check

Mania: Persecutory delusions are common in psychotic mania next only to grandiose
delusions. Though visual hallucinations of a religious nature are commoner in mania than
schizophrenia, these are still uncommon as a diagnostic feature. Delusions accompanying
mania are often short­lived (but not as transient as in delirium) and are poorly systematized.
Affective changes are often seen in mania. 
Delirium: Visual hallucinations clearly outnumber auditory hallucinations in delirium.
Transient delusions are often seen in delirium. 
Delusional disorder: Most delusional disorders often have a single delusion and usually these
are well systematised, and are often persecutory in theme. In delusional disorders, usually
there are no hallucinations (though olfactory and tactile hallucinations may be present as
secondary phenomenon); psychosocial functioning may be at a higher level than expected for
schizophrenia.
The correct answer is: Mania (TWO) – Persecutory delusion, Affective changes, Delirium
(TWO) – Transient delusion, Visual hallucinations, Delusional disorder (ONE) – Systematized
delusion

Question 43 HiY Assessment EMI094
Not answered Culture­bound syndromes
Identify the culture­bound syndromes using the descriptions given below:
Marked out of 3.00

Flag question
Piblokto
Amok
A West­African student complains of Pa­Leng
being too tired to do his university
Brain Fag
work and is struggling to concentrate.
Latah
Dhat
Windigo
Koro
Brain Fag
A young Indonesian gentleman has Pa­Leng
an excessive fear of dying as a result Latah
of his genital retracting into his
Dhat
abdomen.
Amok
Piblokto
Windigo
Koro

A Chinese gentleman displays an
excessive fear of coldness.

1608
Pa­Leng
Amok
Koro
Piblokto
Windigo
Dhat
Latah
Brain Fag
Check

Case 1 exemplifies Brain Fag, a condition seen in Nigerian and East African students wherein
a syndrome equivalent to depression presents in young adults with symptoms of poor
concentration, tightness around head, blurred vision, anxiety/depression, extreme tiredness
and recurrent episodes of sleep. 
Case 2 is typical of an anxiety seen in Indonesian/Southeast Asian males that the penis will
recede into the body. This is called Koro; the female for equivalent presents with the anxiety
that the vulva and breasts will recede into the body. 
Pa­Feng (phobic fear of wind) and pa­leng (phobic fear of cold) are seen in China. Patients
fear an excess of yin (negative energy) from exposure to wind and cold. Afflicted individuals
bundle up in warm clothing, eat symbolically "hot" food, and avoid wind or drafts. Symptoms of
both often co­occur. 
Timothy Hall: Index of Culture­Bound Syndromes By Culture. (n.d.). Retrieved from
http://mccajor.net/cbs_cul.html
The correct answer is: A West­African student complains of being too tired to do his university
work and is struggling to concentrate. – Brain Fag, A young Indonesian gentleman has an
excessive fear of dying as a result of his genital retracting into his abdomen. – Koro, A
Chinese gentleman displays an excessive fear of coldness. – Pa­Leng

Question 44 HiY Assessment EMI095
Not answered Diagnosing culture­bound syndromes
What is the most appropriate diagnosis in the following cases?
Marked out of 3.00

Flag question
Taijin Kyofusho
A 25­year­old South East Asian man Susto
who was depressed for a while who Windigo
suddenly becomes indiscriminately Latah
violent. Brain Fag
Frigophobia
Zar
Dhat
Brain Fag
Amok
Frigophobia
A 27­year­old Native American man Koro
Amok
who is convinced that he has become Windigo
a cannibal Taijin Kyofusho
Koro
Dhat
Latah
Zar
Susto

1609
Taijin Kyofusho
A 29­year­old South American man Dhat
who has become acutely anxious as Zar
he has "lost his soul" Latah
Windigo
Susto
Koro
Amok
Check Brain Fag
Frigophobia

Amok is a condition seen in Malaysia &Indonesia. It is a dissociative episode characterized by
a period of brooding followed by an outburst of violent, aggressive, destructive, or homicidal
behavior. 
Windigo is seen among Algonkian Indians living in the Northeast US and Eastern Canada. It
is a syndrome of obsessions regarding cannibalism, supposedly related to a long dead
practice of consuming human flesh in famine situations. 
Susto is an illness attributed to a frightening event that causes the soul to leave the body,
leading to symptoms of unhappiness and sickness. It is seen in Latin America. Symptoms are
extremely variable and may occur months or years after the supposedly precipitating event. 
Timothy Hall, Index of Culture­Bound Syndromes By Culture. (n.d.). Retrieved from
http://mccajor.net/cbs_cul.html
The correct answer is: A 25­year­old South East Asian man who was depressed for a while
who suddenly becomes indiscriminately violent. – Amok, A 27­year­old Native American man
who is convinced that he has become a cannibal – Windigo, A 29­year­old South American
man who has become acutely anxious as he has "lost his soul" – Susto

Finish review

1610

Potrebbero piacerti anche